You are on page 1of 677

F IF T H ED IT IO N

Complete Companion for FIFTH EDITION


JEE Main 2020 Complete Companion for
MATHEMATICS
 JEE Main 2020

Companion for
MATHEMATICS
Dinesh Khattar I Rohan Sinha

Complete


JEE Main 2020


The Complete Resource Book for JEE Main series has been designed to be an independent
resource to enable faster and effective learning. This series includes three separate books on Y
Physics, Chemistry and Mathematics where the core objective of each book is to provide ‘effective U L L D
F VISE
preparation via modular and graded content’. Developed by highly experienced and qualified RE
faculties, these books would act as trusted content for aspirants who are aiming to clear the JEE
(Joint Entrance Examinations) and other key engineering entrance examinations.
HIGHLIGHTS
H I G H LI G H TS  Coverage of key topics along with

MATHEMATICS
solved examples
 Complete coverage of JEE Main curriculum with emphasis on important concepts
 Includes practice problems with
 Provides varieties of solved example after each concept for better understanding complete solutions
 Key points highlighted within text along with features like – ‘Concepts at a glance’,  Chapter-wise Previous 18 years’
‘info boxes’, and ‘concept notes’ to aid in quick last-minute revision AIEEE/JEE Main questions

Cover Image: Senoldo.shutterstock.com


 Includes ample number of solved examples along with NCERT Exemplar  Fully-solved JEE Main 2019
questions (April & Jan)
 Updated Practice Exercises as per recent patterns of JEE Main for result-oriented preparation
 Includes online tests based on JEE
Main pattern
FREE
Online
Mock Tests

MRP Inclusive
of all Taxes `550.00

in.pearson.com 5e
Dinesh Khattar
Khattar
Sinha

Rohan Sinha

Size: 203x254mm Spine: 25mm ISBN: 9789353435097 Title Sub Title Edition Authors / Editors Name With CD Red Band Territory line URL Price mQuest
About Pearson
Pearson is the world’s learning company, with presence across 70 countries
worldwide. Our unique insights and world-class expertise comes from a long
history of working closely with renowned teachers, authors and thought leaders, as
a result of which, we have emerged as the preferred choice for millions of teachers
and learners across the world.
We believe learning opens up opportunities, creates fulfilling careers and hence
better lives. We hence collaborate with the best of minds to deliver you class-
leading products, spread across the Higher Education and K12 spectrum.
Superior learning experience and improved outcomes are at the heart of everything
we do. This product is the result of one such effort.
Your feedback plays a critical role in the evolution of our products and you can
contact us - reachus@pearson.com. We look forward to it.
This page is intentionally left blank.
FIFTH EDITION
Complete Companion for

JEE Main 2020


MATHEMATICS


Dinesh Khattar
Rohan Sinha
This page is intentionally left blank.
FIFTH EDITION
Complete Companion for

JEE Main 2020


MATHEMATICS


Dinesh Khattar
Rohan Sinha
Copyright © 2019 Pearson India Education Services Pvt. Ltd

Published by Pearson India Education Services Pvt. Ltd, CIN: U72200TN2005PTC057128.

No part of this eBook may be used or reproduced in any manner whatsoever without the publisher’s
prior written consent.

This eBook may or may not include all assets that were part of the print version. The publisher
reserves the right to remove any material in this eBook at any time.

ISBN 978-93-534-3509-7
eISBN:

Head Office: 15th Floor, Tower-B, World Trade Tower, Plot No. 1, Block-C, Sector-16,
Noida 201 301,Uttar Pradesh, India.
Registered Office: 4th Floor, Software Block, Elnet Software City, TS-140, Block 2 & 9,
Rajiv Gandhi Salai, Taramani, Chennai 600 113, Tamil Nadu, India.
Fax: 080-30461003, Phone: 080-30461060
www.in.pearson.com, Email: companysecretary.india@pearson.com
Contents

Prefacexi Chapter 3 Quadratic Equations and


Mathematics Trend Analysis (2011 to 2019) xiii Expressions3.1
Quadratic Equation 3.1
Chapter 1  Set Theory 1.1 Common Roots 3.7
Set 1.1 Symmetric Function of the Roots 3.9
Representation of a Set 1.1 Graph of a Quadratic Expression 3.9
Types of Sets 1.1 Greatest and Least Values of a Quadratic
Operations on Sets 1.2  Expression 3.9
Algebra of Sets 1.4 Nature of Roots of a Quadratic Equation with
Cartesian Product of Two Sets 1.4   Respect to One or Two Real Numbers 3.10
Relations 1.9 Relation Between Roots and Coefficients
  of a Polynomial Equation 3.11
Types of Relations on a Set 1.10
Formation of a Polynomial Equation From
Equivalence Relation 1.10
  Given Roots 3.11
Congruence Modulo m1.10
Sign of a Polynomial Expression 3.11
NCERT Exemplars 1.14
Rational Algebraic Expression 3.16
Practice Exercises 1.17
NCERT Exemplars 3.19
Practice Exercises 3.23
Chapter 2  Complex Numbers 2.1
Imaginary Numbers 2.1 Chapter 4 Permutations and Combinations
Integral Powers of i 2.1 4.1
Complex Numbers 2.2 Factorial Notation 4.1
Conjugate of a Complex Number 2.3 Fundamental Principles of Counting 4.1
Modulus of a Complex Number 2.4 Permutation 4.4
Square Roots of a Complex Number 2.6 Combination 4.7
Argand Plane and Geometrical Representation Key Results on Combination 4.8
  of Complex Numbers 2.7 Derangement 4.13
Polar Form of a Complex Number 2.7 Exponent of prime p in n!4.14
Particular Cases of Polar Form 2.9 Number of Divisors 4.15
Eulerian Representation of a Complex Number 2.10 NCERT Exemplars 4.16
Logarithm of a Complex Number 2.10 Practice Exercises 4.19
Vectorial Representation of a Complex Number 2.10
Roots of a Complex Number 2.12 Chapter 5  Mathematical Induction 5.1
Geometry of Complex Numbers 2.15 Mathematical Induction 5.1
Practice Exercises 2.21 NCERT Exemplars 5.4
Practice Exercises 5.5
viii  Contents

Chapter 6  Binomial Theorem Chapter 10 Coordinates and Straight


Binomial Expression 6.1 Lines10.1
Binomial Theorem 6.1 Distance Formula 10.1
Special Cases 6.1 Section Formulae 10.3
Pascal’s Triangle 6.2 Area of a Triangle 10.5
Middle Term in the Binomial Expansion  6.9 Condition for Collinearity of Three Points 10.5
NCERT Exemplars 6.14 Stair Method for Finding the Area 10.6
Practice Exercises 6.16 Area of a Quadrilateral 10.8
Area of a Polygon 10.8
Chapter 7  Sequence and Series 7.1 Stair Method 10.8
Sequence 7.1 Locus 10.9
Series 7.1 Translation of Axes 10.9
Progressions 7.1 Rotation of Axes 10.10
Arithmetic Progression (A.P.) 7.1 Reflection (Image) of a Point 10.10
Sum of n Terms of an A.P. 7.3 General Equation of a Straight Line 10.12
Properties of A.P. 7.4 Slope of a Line 10.12
Arithmetic Mean (A.M.) 7.7 Intercept of a Line on the Axes 10.12
Geometric Progression (G.P.) 7.9 Equation of a Straight Line in Various forms 10.13
Geometric Mean (G.M.) 7.14 Reduction of the General Equation to Different
Some Special Sequences 7.16   Standard Forms 10.17

Arithmetico-Geometric Progression (A.G.P.) 7.18 Angle Between Two Intersecting Lines 10.17

Method for Finding Sum of A.G. Series 7.18 Condition for Two Lines to be Coincident,
  Parallel, Perpendicular or Intersecting 10.18
NCERT Exemplars 7.21
Equation of a Line Parallel to a Given Line 10.18
Practice Exercises 7.24
Equation of a Line Perpendicular to a Given Line 10.19
Chapter 8  Limits 8.1 Point of Intersection of Two Given Lines 10.19
Limit of A Function 8.1 Concurrent Lines 10.19
Indeterminate Forms 8.2 Position of Two Points Relative to a Line 10.19
Algebra of Limits 8.3 Length of Perpendicular from a Point on a Line 10.20
Evaluation of Limits 8.3 Distance between Two Parallel Lines 10.21
Algebraic Limits 8.3 Equations of Straight Lines Passing through
Limit of an Algebraic Function when x → ∞ 8.4   a Given Point and Making a Given
Trigonometric Limits 8.6   Angle with a Given Line 10.21
Exponential and Logarithmic Limits 8.8 Reflection on the Surface 10.21
Evaluation of Limits using L’Hospital’s Rule 8.12 Image of a Point with Respect to a Line 10.22
Practice Exercises 8.16 Equations of the Bisectors of the Angles
  Between Two Lines 10.23

Chapter 9  Differential Equations 9.1 Equations of Lines Passing through the Point
  of Intersection of Two Given Lines 10.24
Differential Equation 9.1 Standard Points of a Triangle 10.24
Linear and Non-linear Differential Equations 9.3 Orthocentre 10.26
Initial Value Problems 9.7 Coordinates of Nine Point Circle 10.26
Homogeneous Differential Equations 9.10 NCERT Exemplars 10.28
Solution by Inspection 9.13 Practice Exercises 10.33
NCERT Exemplars 9.17
Practice Exercises 9.26
Contents  ix

Chapter 11  Circles 11.1 Centre of Conic 12.3

Circle 11.1 Parabola 12.3

Standard Equation of a Circle 11.1 Some Terms Related to Parabola 12.3

General Equation of a Circle 11.1 Intersection of a Line and a Parabola 12.5

Conditions for an Equation to Represent a Circle11.1 Equation of a Chord 12.5

Equation of a Circle in some Special Cases 11.2 Point of Intersection of tangents 12.5

Equation of A Circle In Diameter Form 11.4 Position of a Point with Respect to a Parabola 12.6

Intercepts Made by a Circle on the Axes 11.5 Number of Tangents Drawn from a Point
  to a Parabola 12.6
Parametric Equations of a Circle 11.5
Equation of the Pair of Tangents 12.6
Position of a Point with Respect to a Circle 11.6
Equations of Normal in Different Forms 12.6
Circle Through Three Points 11.7
Point of Intersection of Normals 12.6
Intersection of a Line and a Circle 11.8
Co-normal Points 12.7
Length of Intercept Made by a Circle on a Line 11.8
Chord of Contact 12.7
The Least and Greatest Distance of a Point
  from a Circle 11.9 Chord with a Given Mid Point 12.7

Contact of Two Circles 11.10 Ellipse 12.14

Tangent to a Circle at a Given Point 11.11 Position of a Point with Respect to an Ellipse 12.16

Equation of the Tangent in Slope form 11.11 Equation of Normal in Different Forms 12.18

Condition of Tangency 11.11 Equation of the Pair of Tangents 12.18

Tangents From a Point Outside the Circle 11.11 Chord with a Given Mid Point 12.19

Length of the Tangent From a Point to a Circle 11.12 Optical Property of Parabola 12.25

Normal to the Circle at a Given Point 11.12 Equation of a Hyperbola in Standard Form 12.26

Pair of Tangents 11.13 Some Terms and Properties Related to a


 Hyperbola 12.26
Common Tangents to Two Circles 11.13
Conjugate Hyperbola 12.27
Power of a Point with Respect to a Circle 11.15
Position of a Point with Respect to a
Director Circle 11.15  Hyperbola 12.28
Equation of Chord of Contact 11.16 Equation of the Pair of Tangents 12.29
Equation of Chord if its Mid Point is Known 11.16 Chord with a Given Mid Point 12.29
Common Chord of Two Circles 11.17 Chord of Contact 12.29
Diameter of a Circle 11.17 NCERT Exemplars 12.35
Angle of Intersection of Two Circles 11.18 Practice Exercises 12.38
Orthogonal Intersection of Two Circles 11.18
Family of Circles 11.19 Chapter 13 Vector Algebra 13.1
Image of the Circle by the Line Mirror 11.20 Scalars and Vectors 13.1
Practice Exercises 11.21 Representation of Vectors 13.1
Types of Vectors 13.2
Chapter 12 Conic Sections (Parabola, Equal Vectors  13.2
Ellipse and Hyperbola) 12.1 Fixed Vectors  13.2
Conic Section 12.1 Free Vectors  13.2
Important Terms 12.1 Angle between Two Vectors 13.2
Section of a Right Circular Cone by Different Addition (Sum or Resultant) of Two Vectors 13.3
 Planes 12.1 Position Vector  13.4
Equation of Conic 12.2 Component of a Vector 13.8
General Equation 12.2 Linear Combination 13.8
x  Contents

Linearly Dependent and Independent Table for Trigonometric Ratios of Allied Angles 15.5
  System of Vectors 13.8 Trigonometric Ratios in Terms of Each Other 15.6
Collinearity of Three Points 13.8 Addition and Subtraction Formulae 15.6
Coplanarity of Four Points 13.9 Transformation  Formulae 15.7
Some Results on Linearly Dependent and Product into Sum or Difference 15.7
  Independent Vectors 13.9
Sum and Difference into Product 15.7
Product of Two Vectors 13.10
Trigonometric Ratios of Multiple Angles 15.7
Scalar Product of Two Vectors 13.10
Trigonometric Ratios of Submultiple Angles 15.8
Some Useful Identities 13.10
Greatest and Least Values of the Expression 15.9
Work Done by a Force 13.10
Conditional Identities 15.9
Vector Product of Two Vectors 13.14
Graphs of Trigonometric Functions 15.9
Moment of a Force about a Point 13.15
Practice Exercises 15.11
Scalar Triple Product 13.18
NCERT Exemplars 13.21 Chapter 16  Trigonometric Equations 16.1
Practice Exercises 13.24
Trigonometric Equation 16.1
Solution or Root of a Trigonometric Equation 16.1
Chapter 14 Measures of Central
Method for finding Principal Value (Solution) 16.1
Tendency and Dispersion 14.1
Solution of an Equation of the Form 16.5
Measures of Central Tendency 14.1 Solutions of Basic Trigonometric Inequalities 16.6
Arithmetic Mean 14.1 NCERT Exemplars 16.9
Geometric Mean 14.2 Practice Exercises 16.15
Harmonic Mean 14.2
Median 14.7 Chapter 17  Heights and Distances 17.1
Quartiles, Deciles and Percentiles 14.7
Some Terminology Related to
Mode 14.8   Height and Distance 17.1
Symmetric Distribution 14.9 North East 17.1
Practice Exercises 14.13 Bearings of a Point 17.1
Some Properties Related to Triangle 17.2
Chapter 15 Trigonometric Ratios Some Properties Related to Circle 17.4
and Identities 15.1 Some Important Results 17.7
Angle 15.1 Practice Exercises 17.12
Measurement of Angles 15.1
Relation between Different Systems of Chapter 18  Mathematical Reasoning 18.1
  Measurement of Angles 15.2 Mathematical Reasoning 18.1
Relation between Sides and Interior Angles of Statement 18.1
  a Regular Polygon 15.2
The Connecting Word ‘OR’ 18.2
Fundamental Trigonometric Identities 15.2
Quantifiers 18.2
Signs of Trigonometric Ratios in
  Different Quadrants 15.2 Implications 18.3
Increase and Decrease of Trigonometric NCERT Exemplars 18.5
  Functions  15.3 Practice Exercises 18.8
Domain and Range of Trigonometric Ratios 15.3
Trigonometric Ratios of Standard Angles 15.4
Trigonometric Ratios for Some Special Angles 15.4
Trigonometric Ratios of Allied Angles 15.5
Preface
About the Series
Complete Companion for JEE Main series is a must have resource for students preparing for Joint Entrance Examination.
There are three subjective books—Physics, Chemistry, and Mathematics; the core objective of this series is to strengthen
the fundamental concepts and prepare students for various engineering entrance examinations. It provides class-tested
course material and numerical applications that will supplement any ready material available as student resource.
To ensure high level of accuracy and practicality, this series has been authored by highly qualified and experienced
faculties for all three titles.
About the Book
Complete Companion for JEE Main 2020 Mathematics, Volume 1 particularly developed for class 11th students, so that
they can start their preparation from the early days. This title is designed as per the latest JEE Main syllabus, where the
important topics are covered in 18 chapters. It has been structured in user friendly approach such that each chapter begins
with topic-wise theory, followed by sufficient solved examples and then practice questions along with previous years'
questions.
The chapter-end exercises are structured in line with JEE questions with ample of questions on single choice correct
question (SCQ) for extensive practice. Previous 18 years’ questions from JEE Main and AIEEE are also added in every
chapter. Hints and Solutions at the end of every chapter will help the students to evaluate their concepts and numerical
applications. Because of its comprehensive and in-depth approach, it will be especially helpful for those students who
prefers self-study than going for any classroom teaching.
Series Features
• Complete coverage of topics along with ample number of solved examples
• Large variety of practice problems with complete solutions
• Chapter-wise Previous 18 years’ AIEEE/JEE Main questions
• Fully solved JEE Main 2019 (Jan/Apr) questions added in opening section of the book
• Includes 5 Mock Tests papers based on JEE Main pattern in the book
• Free Online Mock Tests as per the recent JEE Main pattern
It would have been difficult to prepare this book without aid and support from a number of different quarters. I shall be
grateful to the readers for their regular feedback. I am deeply indebted to my parents without whose encouragement this
dream could not have been translated into reality. The cherubic smiles of my daughters, Nikita and Nishita, have inspired
me to treat my work as worship.
Anuj Agarwal from IIT-Delhi, Ankit Katial from National Institute of Technology (Kurukshetra) and Raudrashish
Chakraborty from Kirori Mal College, University of Delhi, with whom I have had fruitful discussions, deserve special
mention.
I earnestly hope that the book will help the students grasp the subject well and respond with a commendable score in
the JEE Main examination. There are a plethora of options available to students for Mathematics, however, ever grateful to
them and to the readers for their candid feedback.
Despite of our best efforts, some errors may have crept into the book. Constructive comments and suggestions to
further improve the book are welcome and shall be acknowledged gratefully.
Best of luck!

Dinesh Khattar
This page is intentionally left blank.
Mathematics Trend Analysis
(2011 to 2019)

S. No. Chapters 2011 2012 2013 2014 2015 2016 2017 2018 2019
Volume 1
1 Sets 1 0 0 1 1 0 0 1 2
2 Complex Numbers and
7 2 10 3 0 5 3 1 4
Quadratic Equations
3 Permutations and
0 3 2 2 2 2 1 1 2
Combinations
4 Principles of Mathematical
0 0 0 0 1 0 0 0 0
Induction
5 Binomial Theorem 0 0 2 1 1 2 1 1 3
6 Sequences and Series 2 1 3 3 4 3 0 2 1
7 Straight Iines 1 0 2 2 0 1 0 3 5
8 Conic Sections 5 3 2 6 0 6 0 7 4
9 Limits and Derivatives 2 4 2 8 11 5 4 2 3
10 Mathematical Reasoning 0 0 1 1 1 1 1 1 1
Volume 2
11 Relations and Functions 3 2 1 0 1 1 1 1 2
12 Inverse Trigonometric
1 0 2 0 2 0 1 1 1
Functions
13 Matrices 2 2 2 1 2 3 1 1 3
14 Determinants 2 1 2 1 2 3 1 1 2
15 Continuity and
2 1 2 1 8 4 2 2 3
Differentiability
16 Applications of Derivatives 2 6 9 3 3 4 2 1 2
17 Integrals 2 2 2 3 3 4 1 2 3
18 Applications of Intergrals 1 1 2 2 3 3 1 1 2
19 Differential Equation 2 1 2 1 3 3 1 1 4
20 Vector Algebra 3 2 3 3 3 2 1 1 3
21 Three-dimensional
0 3 7 3 6 4 3 1 3
Geometry
22 Probability 3 3 5 1 0 4 2 2 3
This page is intentionally left blank.
CHAPTER
Set Theory
1
LEARNING OBJECTIVES
After reading this chapter, you will be able to:
  Learn the definition of set and how is it denoted  Understand the operations on sets and Identify some key
 Know how the sets are represented and what are its results
types  Establish a relation between the two sets and study about
its types

SET
I M P O R TA N T P O I N T S
A set is a well-defined collection of objects such that given
an object, it is possible to determine whether that object ■ In roster form, every element of the set is listed only
belongs to the given collection or not. once.
For example, the collection of all students of Delhi ■ The order in which the elements are listed is immaterial

University is a set, whereas, collection of all good books For example, each of the following sets denotes the same
on mathematics is not a set, since a mathematics book con- set {1, 2, 3}, {3, 2, 1}, {1, 3, 2}.
sidered good by one person might be considered bad or
­average by another. Set-builder Form
Notations In this form, we write a variable (say x) representing any
member of the set followed by a property satisfied by each
The sets are usually denoted by capital letters A, B, C, etc. member of the set.
and the members or elements of the set are denoted by For example, the set A of all prime numbers less than
­lower-case letters a, b, c etc. If x is a member of the set A, 10 in the set-builder form is written as
we write x ∈ A (read as ‘x belongs to A’) and if x is not a
member of the set A, we write x ∉ A (read as ‘x does not A = {x | x is a prime number less than 10}
belong to A’). If x and y both belong to A, we write x, y ∈ A. The symbol ‘|’ stands for the words ‘such that’. Sometimes,
we use the symbol ‘:’ in place of the symbol ‘|’.
REPRESENTATION OF A SET
TYPES OF SETS
Usually, sets are represented in the following two ways:
1. Roster form or tabular form Empty Set or Null Set
2. Set builder form or rule method A set which has no element is called the null set or empty
set. It is denoted by the symbol f.
Roster Form For example, each of the following is a null set:
In this form, we list all the members of the set within braces 1. The set of all real numbers whose square is –1.
(curly brackets) and separate these by commas. 2. The set of all rational numbers whose square is 2.
For example, the set A of all odd natural numbers less 3. The set of all those integers that are both even and odd.
than 10 in the roster form is written as:
A set consisting of atleast one element is called a ­non-empty
A = {1, 3, 5, 7, 9} set.
1.2  Chapter 1

Singleton Set For example, let A = {3, 4}, then the subsets of A are f,
{3}, {4}, {3, 4}. Here, n(A) = 2 and number of subsets of
A set having only one element is called singleton set.
For example, {0} is a singleton set, whose only A = 22 = 4.
­member is 0. Also, {3} ⊂ {3, 4} and {2, 3} ⊄ {3, 4}

Finite and Infinite Set Power Set

A set which has finite number of elements is called a finite The set of all subsets of a given set A is called the power set
set. Otherwise, it is called an infinite set. of A and is denoted by P(A).
For example, the set of all days in a week is a finite set For example, if A = {1, 2, 3}, then
whereas, the set of all integers, denoted by {…, – 2, – 1, 0, P(A) = [f, {1}, {2}, {3}, {1, 2} {1, 3}, {2, 3}, {1, 2, 3}]
1, 2, …} or {x | x is an integer}, is an infinite set.
An empty set f which has no element, is a finite set. Clearly, if A has n elements, then its power set P(A) ­contains
The number of distinct elements in a finite set A is exactly 2n elements.
called the cardinal number of the set A and it is denoted
by n (A). QUICK TIPS

Equal Sets Number of elements in P {P [P(f)]} is 4


or Cardinal Number of P{P[P(f)]} = 4
Two sets A and B are said to be equal, written as A = B, if
Since, P (f) = {f}
every element of A is in B and every element of B is in A.
Also, P [P (f)] = {f, {f}}
Equivalent Sets and P {P[P (f)]} = {f, {f}, [{f}], [f, {f}]}
Hence, n [P {P[P (f)]}] = 4
Two finite sets A and B are said to be equivalent, if n(A) =
n(B).
Euler–Venn Diagrams

ERROR CHECK To express the relationship among sets, we represent them


pictorially by means of diagrams, known as Euler–Venn
Equal sets are equivalent but equivalent sets need not be Diagrams or simply Venn diagrams.
equal. In Venn diagrams, the universal set U is represented
For example, the sets A = {4, 5, 3, 2} and B = {1, 6, by the rectangular region and its subsets are represented by
8, 9} are equivalent but are not equal. closed bounded circles inside this rectangular region.

OPERATIONS ON SETS
Subset
Let A and B be two sets. If every element of A is an element Union of Two Sets
of B, then A is called a subset of B and we write A ⊆ B or The union of two sets A and B, written as A ∪ B (read as
B ⊇ A (read as ‘A is contained in B’ or B contains A’). B is ‘A union B’), is the set consisting of all the elements which
called superset of A. are either in A or in B or in both. Thus,

A ∪ B = {x: x ∈ A or x ∈ B}


I M P O R TA N T P O I N T S
Clearly, x ∈ A ∪ B
■ If A ⊆ B and A ≠ B, we write A ⊂ B or B ⊃ A (read as : A ⇒ x ∈ A
is a proper subset of B or B is a proper superset of A). or x ∈ B,
■ Every set is a subset and a superset of itself. and x ∉ A ∪ B
■ If A is not a subset of B, we write A ⊄ B.
⇒ x ∉ A
■ The empty set is the subset of every set.
and x ∉ B.
■ If A is a set with n (A) = m, then the number of subsets

of A are 2m and the number of proper subsets of A are For example, if A = {a, b, c d} and B = {c, d, e, f }, then
2m–1. A ∪ B = {a, b, c, d, e, f }.
Set Theory  1.3

Similarly,
U
B – A = {x: x ∈ B and x ∉ A}
A∪B For example,
if A = {1, 2, 3, 4, 5} and B = {1, 3, 5, 7, 9},
A B
then A – B = {2, 4} and B – A = {7, 9}
FIGURE 1.1
U U
Intersection of Two Sets
The intersection of two sets A and B, written as A ∩ B (read A–B B–A
as ‘A intersection B’) is the set consisting of all the com-
mon elements of A and B. Thus,
A B A B
A ∩ B = {x: x ∈ A and x ∈ B}
FIGURE 1.4  (a–b)
Clearly, x ∈ A ∩ B
⇒ x ∈ A and x ∈ B,
Info Box!
and x ∉ A ∩ B ■ A–B≠B–A
⇒ x ∉ A or x ∉ B. ■ The sets A – B, B – A and A ∩ B are disjoint sets
■ A – B ⊆ A and B – A ⊆ B
For example, if A = {a, b, c, d} and B = {c, d, e, f }, then ■ A – ϕ = A and A – A = ϕ
A ∩ B = {c, d}.

U
Symmetric Difference of Two Sets
The symmetric difference of two sets A and B, denoted by
A∩B A D B, is defined as
A D B = (A – B) ∪ (B – A).
A B For example, if A = {1, 2, 3, 4, 5} and B = {1, 3, 5, 7, 9} then A
D B = (A – B) ∪ (B – A) = {2, 4} ∪ {7, 9} = {2, 4, 7, 9}.
FIGURE 1.2

Disjoint Sets
Two sets A and B are said to be disjoint, if A ∩ B = f, i.e.,
A and B have no element in common.
For example, if A = {1, 2, 5} and B = {2, 4, 6}, then
B
A ∩ B = f, so A and B are disjoint sets.
A
U
FIGURE 1.5

Complement of a Set
A B
If U is a universal set and A is a subset of U, then the com-
plement of A is the set which contains those elements of
FIGURE 1.3
U, which are not contained in A and is denoted by A′ or Ac.
Difference of Two Sets Thus,
If A and B are two sets, then their difference A – B is defined as A′ = {x: x ∈ U and x ∉ A}
A – B = {x: x ∈ A and x ∉ B} For example,
1.4  Chapter 1

if U = {1, 2, 3, 4, …} and A = {2, 4, 6, 8, …}, (b) (A ∩ B)′ = A′ ∪ B′


(c) A – (B ∪ C) = (A – B) ∩ (A – C)
then, A′ = {1, 3, 5, 7, …}
(d) A – (B ∩ C) = (A – B) ∪ (A – C)

Key Results on Operations on Sets


U
1. A ⊆ A ∪ B, B ⊆ A ∪ B, A ∩ B ⊆ A, A ∩ B ⊆ B
2. A – B = A ∩ B′
A 3. (A – B) ∪ B = A ∪ B
4. (A – B) ∩ B = f
A′ 5. A ⊆ B ⇔ B′ ⊆ A′
6. A – B = B′ – A′
FIGURE 1.6 7. (A ∪ B) ∩ (A ∪ B′) = A
8. A ∪ B = (A – B) ∪ (B – A) ∪ (A ∩ B)
9. A – (A – B) = A ∩ B
Info Box! 10. A – B = B – A ⇔ A = B
■ U′ = f 11. A ∪ B = A ∩ B ⇔ A = B
■ f′ = U 12. A ∩ (B D C) = (A ∩ B) D (A ∩ C)
■ A ∪ A′ = U
■ A ∩ A′ = f Some Results about Cardinal Number
If A, B and C are finite sets and U be the finite universal
set, then
ALGEBRA OF SETS 1. n (A′) = n (U) – n (A)
1. Idempotent Laws: For any set A, we have 2. n (A ∪ B) = n (A) + n (B) – n (A ∩ B)
(a) A ∪ A = A 3. n (A ∪ B) = n (A) + n(B),
(b) A ∩ A = A where A and B are disjoint non-empty sets
2. Identity Laws: For any set A, we have: 4. n (A ∩ B′) = n (A) – n (A ∩ B)
(a) A ∪ f = A 5. n (A′ ∩ B′) = n (A ∪ B) ‘ = n (U) – n (A ∪ B)
(b) A ∩ f = f 6. n (A′ ∪ B′) = n (A ∩ B) ‘ = n (U) – n (A ∩ B)
(c) A ∪ U = U 7. n (A – B) = n (A) – n (A ∩ B)
(d) A ∩ U = A 8. n (A ∩ B) = n (A ∪ B) – n (A ∩ B′) – n (A′ ∩ B)
3. Commutative Laws: For any two sets A and B, we 9. n (A ∪ B ∪ C) = n (A) + n (B) + n (C) – n (A ∩ B)
have – n (B ∩ C) – n (C ∩ A) + n (A ∩ B ∩ C)
(a) A ∪ B = B ∪ A 10. If A1, A2, A3, … An are disjoint sets, then
(b) A ∩ B = B ∩ A n (A1 ∪ A2 ∪ A3 ∪ … ∪ An) = n (A1) + n (A2) + n (A3)
4. Associative Laws: For any three sets A, B and C, we + … + n (An)
have
11. n (A D B) = number of elements which belong to
(a) A ∪ (B ∪ C) = (A ∪ B) ∪ C
exactly one of A or B
(b) A ∩ (B ∩ C) = (A ∩ B) ∩ C
5. Distributive Laws: For any three sets A, B and C, we
have CARTESIAN PRODUCT OF TWO SETS
(a) A ∪ (B ∩ C) = (A ∪ B) ∩ (A ∪ C) If A and B are any two non-empty sets, then cartesian
(b) A ∩ (B ∪ C) = (A ∩ B) ∪ (A ∩ C) ­product of A and B is defined as
6. For any two sets A and B, we have
(a) P(A) ∩ P(B) = P(A ∩ B) A × B = [(a, b) : a ∈ A and b ∈ B]
(b) P(A) ∪ P(B) ⊆ P(A ∪ B), where P(A) is the power
set of A.
7. If A is any set, we have (A′)′ = A. ERROR CHECK
8. Demorgan’s Laws: For any three sets A, B and C, we
have A×B≠B×A
(a) (A ∪ B)′ = A′ ∩ B′
Set Theory  1.5

⇒ 50 = 35 + 24 – n (A ∩ B)
QUICK TIPS
⇒ n(A ∩ B) = 59 – 50 = 9.
■ If A = f or B = f, then we define A × B = f.
2. Let A = {2, 3, 4} and X = {0, 1, 2, 3, 4}, then which of
■ If A has n elements and B has m elements then A × B has
the following statements is correct?
mn elements.
■ If A1, A2, …, Ap are p non-empty sets, then their cartesian
(A) {0} ∈ A′ in X
p (B) f ∈ A′ with respect to. X
product, is defined as ∏ Ai = [(a1, a2, a3, …, ap); ai ∈ Ai (C) {0} ⊂ A′ with respect to X
i =1
for all i] (D) 0 ⊂ A′ with respect to X.
Solution: (C)
Key Results on Cartesian Product We have, A′ in X = The set of elements in X which are
not in A = {0, 1}
If A, B, C are three sets, then {0} ∈ A′ in X is false, because {0} is not an e­ lement
1. A × (B ∪ C) = (A × B) ∪ (A × C) of A′ in X.
2. A × (B ∩ C) = (A × B) ∩ (A × C) f ⊂ A′ in X is false, because f is not an element
3. A × (B – C) = (A × B) – (A × C) of A′ in X
4. (A × B) ∩ (S × T) = (A ∩ S) × (B ∩ T), {0} ⊂ A′ in X is correct, because the only element
where S and T are two sets. of {0} namely 0 also belongs to A′ in X.
5. If A ⊆ B, then (A × C) ⊆ (B × C) 0 ⊂ A′ in X is false, because 0 is not a set.
6. If A ⊆ B, then (A × B) ∩ (B × A) = A2 3. If X = {8n – 7 n – 1/n ∈ N} and Y = {49 (n – 1)/n ∈ N},
7. If A ⊆ B and C ⊆ D then A × C ⊆ B × D then
8. If A ⊆ B, then A × A ⊆ (A × B) ∩ (B × A)
(A) X ⊂ Y (B)  Y⊂X
9. If A and B are two non-empty sets having n elements
(C) X = Y (D)  None of these
in common, then A × B and B × A have n2 elements in
common. Solution: (A)
10. A × B = B × A if and only if A = B We have, 8n – 7n – 1
11. A × (B′ ∪ C ′)′ = (A × B) ∩ (A × C) = (7 + 1)n – 7n – 1 = (nC272 + nC373 + … + nCn7n)
12. A × (B′ ∩ C ′)′ = (A × B) ∪ (A × C)
= 49(nC2 + nC37 + … + nCn7n – 2) for n ≥ 2

SOLVED EXAMPLES For n = 1, 8n – 7n – 1 = 0


Thus, 8n – 7 n – 1 is a multiple of 49 for n ≥ 2 and
1. If n (U) = 60, n (A) = 35, n (B) = 24 and n (A ∪ B)′ = 0 for n = 1. Hence, X consists of all positive integral
10 then n (A ∩ B) is multiples of 49 of the form 49 Kn. where Kn = nC2 +
n
C37 + … + nCn7n – 2 together with zero. Also, Y con-
(A) 9 (B) 8
sists of all positive integral multiples of 49 including
(C)  6 (D)  None of these
zero. Therefore, X ⊂ Y.
Solution: (A)
4. The set (A ∪ B ∪ C) ∩ (A ∩ B′ ∩ C′)′ ∩ C′ is equal to
We have,
(A) A ∩ B (B)  A ∩ C ′
n(A ∪ B) = n (U) – n(A ∪ B)′ = 60 – 10 = 50 (C) B ∩ C ′ (D)  B′ ∩ C ′
Now, n (A ∪ B) = n (A) + n(B) – n(A ∩ B) Solution: (C)

U (A ∪ B ∪ C) ∩ (A ∩ B′ ∩ C ′)′ ∩ C ′
(A ∪ B)′ =
(A ∪ B ∪ C) ∩ (A′ ∪ B ∪ C) ∩ C ′
A B
= [(A ∩ A′) ∪ (B ∪ C)] ∩ C ′
= (f ∪ B ∪ C) ∩ C ′ = (B ∪ C) ∩ C ′
= (B ∩ C ′) ∪ (C ∩ C ′)
A∩B
= (B ∩ C ′) ∪ f = B ∩ C ′
1.6  Chapter 1

5. If A, B and C are non-empty subsets of a set, then = {x : (x ∈ R and x ≥ 2) or (x ∈ R and x ≤ 0)}


(A – B) ∪ (B – A) equals
= {x : x ∈ R and x ≥ 2} ∪ {x : x ∈ R and x ≤ 0}
(A) (A ∩ B) ∪ (A ∪ B) (B) (A ∪ B) – (A ∩ B)
(C) A – (A ∩ B) (D) (A ∪ B) – B Similarly,
B′ = {x : x ∈ R and x ≤ 1} ∪ {x : x ∈ R and x > 3}
Solution: (B)
\ A ∪ B = {x : x ∈ R and 0 < x ≤ 3},
(A – B) ∪ (B – A) = (A ∪ B) – (A ∩ B)
A ∩ B = {x : x ∈ R and 1 < x < 2}
A – B = {x : x ∈ R and 0 < x ≤ 1}

A B
9. If Y ∪ {1, 2} = {1, 2, 3, 5, 9}, then
(A)  The smallest set of Y is {3, 5, 9}
(B)  The smallest set of Y is {2, 3, 5, 9}
(C)  The largest set of Y is {1, 2, 3, 4, 9}
A−B A∩B B−A
(D)  The largest set of Y is {2, 3, 4, 9}
6. Let A and B two non-empty subsets of a set X such that Solution:  (A and C)
A is not a subset of B then Since the set on the right hand side has 5 elements,
(A) A is subset of the complement of B \ smallest set of Y has three elements and largest
(C) B is a subset of A set of Y has five elements,
(C) A and B are disjoint \ smallest set of Y is {3, 5, 9}
(D) A and the complement of B are non-disjoint and largest set of Y is {1, 2, 3, 4, 9}.
Solution: (D) 10. If A has 3 elements and B has 6 elements, then the
Since A ⊄ B, $ x ∈ A such that x ∉ B minimum number of elements in the set A ∪ B is
(A) 6 (B) 3
Then x ∈ B ′. (C) f (D)  None of these
\ A ∩ B ′ ≠ f Solution: (A)
7. Two finite sets have m and n elements, then total number Clearly the number of elements in A ∪ B will be mini-
of subsets of the first set is 56 more that the total number mum when A ⊂ B. Hence the minimum number of ele-
of subsets of the second. The values of m and n are, ments in A ∪ B is the same as the number of elements
(A)  7, 6 (B)  6, 3 (D)  5, 1 (D)  8, 7 in B, that is, 6.
11. Suppose A1, A2, … A30 are thirty sets, each with five
Solution: (B)
elements and B1, B2, …, Bn are n sets each with three
Since the two finite sets have m and n elements, so
30 n
number of subsets of these sets will be 2m and 2n
respectively. According to the question
elements. Let ∪ Ai = ∪ Bj =S
i =1 j =1

2m – 2n = 56 If each element of S belongs to exactly ten of the Ai′s


putting m = 6, n = 3, we get and exactly nine of the Bj′s then n =
26 – 23 = 56  or  64 – 8 = 56 (A) 45 (B) 35
(C)  40 (D)  None of these
8. Let U = R (the set of all real numbers) If A = {x : x ∈
R, 0 < x < 2}, B = {x : x ∈ R, 1 < x ≤ 3}, then Solution: (A)
(A) A ∪ B = {x : x ∈ R and 0 < x ≤ 3} Given Ai′s are thirty sets with five elements each, so
(B) A ∩ B = {x : x ∈ R and 1 < x < 2} 30
(C) A – B = {x : x ∈ R and 0 < x ≤ 1} ∑ n ( Ai ) = 5 × 30 = 150 (1)
(D)  All of these i =1

Solution: (D) If there are m distinct elements in S and each element


of S belongs to exactly 10 of the Ai′s, we have
We have
30
A′ = R – A = {x : ∈ R and x ∉ A} ∑ n ( Ai ) = 10 m (2)
i =1
Set Theory  1.7

\ From Eq. (1) and (2), we get ∵ n (A ∪ B) = n(A) + n(B) – n(A ∩ B)
10 m = 150 65 = 40 + n (B) – 10
\ m = 15 (3) n (B) = 65 – 40 + 10 = 35
30 30
Similarly ∑ n ( B j ) = 3n and ∑ n ( B j ) = 9 m Number of people who like only tennis
j =1 j =1
= n(B) – n (A ∩ B) = 35 – 10 = 25
\ 3 n = 9 m \ Number of people who like tennis only and not
9m cricket = 25.
⇒ n= = 3 m = 3 × 15 = 45 [from (3)]
3
15. In a group of 1000 people, there are 750 people who
Hence, n = 45.
can speak Hindi and 400 who can speak English. Then
12. If A = {1, 3, 5, 7, 9, 11, 13, 15, 17}, B = {2, 4, …, 18} number of persons who can speak Hindi only is
and N is the universal set, then A′ ∪ ((A ∪ B) ∩ B′) is (A) 300 (B) 400
(A) A (B)  N (C)  600 (D)  None of these
(C) B (D)  None of these
Solution: (C)
Solution: (B) Here
We have, n(H ∪ E) = 1000, n (H) = 750,
(A ∪ B) ∩ B′ = A
n (E) = 400
[(A ∪ B) ∩ B′] ∪ A′ = A ∪ A′ = N.
Using n(H ∪ E) = n(H) + n(E) – n(H ∩ E)
13. If X and Y are two sets and X′ denotes the complement 1000 = 750 + 400 – n (H ∩ E)
of X, then X ∩ (X ∪ Y)′ equals
⇒ n (H ∩ E) = 1150 – 1000 = 150.
(A) X (B)  Y
(C) f (D)  None of these U

Solution: (C)
H E
X ∩ (X ∪ Y)′ = X ∩ (X ′ ∩ Y ′)
[∵ By De-Morgan’s Law (A ∪ B)′ = (A′ ∩ B′)] 600 150 250

= (X ∩ X ′) ∩ Y ′ = f ∩ Y ′ = f


14. In a group of 65 people, 40 like cricket, 10 like both
cricket and tennis. The number of persons liking tennis
Number of people who can speak Hindi only
only and not cricket is
(A) 21 (B) 25 = n(H ∩ E′) = n(H) – n(H ∩ E)
(C)  15 (D)  None of these = 750 – 150 = 600.
Solution: (B)
16. If f : R → R, defined by f (x) = x2 + 1, then the values of
Let A be the set of people who like cricket and B the f –1(17) and f –1(–3) respectively are
set of people who like tennis.
Then n(A ∪ B) = 65 (A) f, {4, –4} (B)  {3, –3}, f
(C) f, {3, –3} (D)  {4, –4}, f
n (A) = 40, n(A ∩ B) = 10
Solution: (D)
U Let y = x2 + 1. Then for y = 17,
A B
we have x = ± 4 and for y = –3, x becomes ­imaginary
that is, there is no value of x.
30 10 25
Hence, f (17) = {–4, 4}
–1
and f (–3) = f
1.8  Chapter 1

17. In a statistical investigation of 1,003 families of Using


Kolkata, it was found that 63 families had neither a
n(A ∪ B ∪ C) = n (A) + n(B) + n(C) – n(A ∩ B)
radio nor a TV, 794 families had a radio and 187 had a
TV. The number of families in that group having both – n(B ∩ C) – n(A ∩ C) + n(A ∩ B ∩ C)
a radio and a TV is
Substituting the above values, we have
(A) 36 (B) 41
(C)  32 (D)  None of these 92 = 42 + 51 + 68 – 30 – 28 – 36 + n (A ∩ B ∩ C)

Solution: (B) ⇒ n (A ∩ B ∩ C) = 92 – 161 + 94


Let R be the set of families having a radio and T, the ⇒ n (A ∩ B ∩ C) = 92 – 67 = 25
set of families having a TV, then
n (R ∪ T) = The no. of families having at least one Hence, 25% of the people read all the three papers.
of radio and TV 19. Out of 800 boys in a school, 224 played cricket, 240
played hockey and 336 played basketball. Of the total,
= 1003 – 63 = 940
64 played both basketball and hockey; 80 played
n (R) = 794 and n(T) = 187 cricket and basketball and 40 played cricket and
hockey; 24 played all the three games. The number of
Let x families had both a radio and a TV i.e.,
boys who did not play any game is
∪ (A) 160 (B) 240 (C) 216 (D) 128
R T Solution: (A)
794 – x

187 – x

x n (C) = 224, n (H) = 240, n (B) = 336


n (H ∩ B) = 64, n (B ∩ C) = 80
n (H ∩ C) = 40, n (C ∩ H ∩ B) = 24
n (R ∩ T ) = x n (C c ∩ H  c ∩ B c) = n [(C ∪ H ∪ B)c]
The number of families who have only radio = 794 – x = n (U) – n (C ∪ H ∪ B)
and the number of families who have only TV = 187 – x
= 800 – [n (C) + n (H) + n (B) – n (H ∩ C)
From Venn diagram,
– n (H ∩ B) – n (C ∩ B) + n (C ∩ H ∩ B)]
794 – x + x + 187 – x = 940
= 800 – [224 + 240 + 336 – 64 – 80 – 40 + 24]
⇒ 981 – x = 940 or x = 981 – 940 = 41
= 800 – [824 – 184] = 984 – 824 = 160.
Hence, the required no. of families having both a radio
and a TV = 41. 20. In a certain town 25% families own a phone and 15%
own a car, 65% families own neither a phone nor a car.
18. In a city, three daily newspapers A, B, C are published.
2000 families own both a car and a phone. Consider
42% of the people in that city read A, 51% read B and
the following statements in this regard:
68% read C. 30% read A and B; 28% read B and C;
36% read A and C; 8% do not read any of the three 1. 10% families own both a car and a phone.
newspapers. The percentage of persons who read all 2. 35% families own either a car or a phone.
the three papers is 3.  40,000 families live in the town.
Which of the above statements are correct?
(A) 25% (B)  18%
(A)  1 and 2 (B)  1 and 3
(C) 20% (D)  None of these
(C)  2 and 3 (D)  1, 2 and 3
Solution: (A)
Solution: (C)
Let the no. of persons in the city be 100.
Then we have n(P) = 25%, n(C) = 15%,
n (A) = 42, n (B) = 51, n (C) = 68;
n (P ′ ∩ C ′) = 65%, n (P ∩ C) = 2000
n (A ∩ B) = 30, n(B ∩ C) = 28, n(A ∩ C) = 36
n (A ∪ B ∪ C) = 100 – 8 = 92 Since, n (P ′ ∩ C ′) = 65%
Set Theory  1.9

\ n(P ∪ C)′ = 65% Now, if x ∈ N, then the solution set is {4}.


\ n(P ∪ C) = 35% Also, if x ∈ I, then the solution set is {0, 4}.
Further, since there is no root of the form a + ib,
Now, n (P ∪ C) = n (P) + n (C) – n (P ∩ C)
where a, b are real and b ≠ 0,
\ 35 = 25 + 15 – n (P ∩ C) \ if x ∈ S = {a + ib : b ≠ 0, a, b ∈ R} then the
\ n (P ∩ C) = 40 – 35 = 5 solution set is f.
Thus, n(P ∩ C) = 5%
But, n(P ∩ C) = 2000 RELATIONS
\ 5% of the total = 2000 Let A, B be any two non-empty sets, then every subset of
A × B defines a relation from A to B and every relation from
2000 × 100 A to B is a subset of A × B.
\ total no. of families = = 40000
5 If R is a relation from A to B and if (a, b) ∈ R, then we
\ n(P ∪ C) = 35%, write a R b and say that ‘a is related to b’ and if (a, b) ∉ R,
then we write a R b and say that a is not related to b.
Total no. of families = 40,000 and n(P ∩ C) = 5%.
21. If P, Q and R are subsets of a set A, then R × (P ′ ∪ Q′)′ Key Results on Relations
equals 1. Every subset of A × A is said to be a relation on A.
(A) (R × P) ∩ (R × Q) (B) (R × Q) ∩ (R × P) 2. If A has m elements and B has n elements, then A × B
(C) (R × P) ∪ (R × Q) (D)  None of these has mn elements and total number of different relations
Solution: (A) from A to B is 2mn.
3. Let R be a relation from A to B, i.e. R ⊆ A × B, then
R × (P′ ∪ Q′)′ = R × [(P ′)′ ∩ (Q ′)′] Domain of R = {a : a ∈ A, (a, b) ∈ R for some b ∈ B}
= R × (P ∩ Q) Range of R = {b : b ∈ B, (a, b) ∈ R for some a ∈ A}
= (R × P) ∩ (R × Q) For example, let A = {1, 3, 4, 5, 7}, B = {2, 4, 6, 8} and
R be the relation ‘is one less than’ from A to B, then
22. If sets A and B are defined as R = [(1, 2), (3, 4), (5, 6), (7, 8)].
A = {(x, y) : y = ex, x ∈ R} Here, domain of R = {1, 3, 5, 7} and range of R = {2,
B = {(x, y) : y = x, x ∈ R} then 4, 6, 8}.
(A) B ⊂ A (B)  A⊂B Info Box!
(C) A ∩ B = f (D)  A∪B=A
Info Box!
Solution: (C) Domain of a relation from A to B is a subset of A and its
x x 2 x3 range is a subset of B
Since y=e =1+x+ + + …
2 ! 3!
\ ex > x ∀ x ∈ R so that the two curves given by y = ex Identity Relation
and y = x do not intersect in any point Hence, there is
no common point, so that A ∩ B = f. R is an identity relation if (a, b) ∈ R if a = b, a ∈ A, b ∈ A.
In other words, every element of A is related to only itself.
23. The solution of 3x2 – 12x = 0 when
(A) x ∈ N is {4} Universal Relation
(B) x ∈ I is {0, 4} Let A be any set and R be the set A × A, then R is called the
(C) x ∈ S = {a + ib : b ≠ 0, a, b ∈ R} is f universal relation in A.
(D)  All of these
Void Relation
Solution: (D)
f is called void relation in a set.
We have,
3x2 – 12x = 0 QUICK TIPS
⇒ 3x (x – 4) = 0 The void and the universal relations on a set A are respec-
⇒ x = 0, 4 tively the smallest and the largest relations on A
1.10  Chapter 1

Inverse Relation 1. Since x – x = 0 = 0 . m ⇒ x – x is divisible by m


⇒ (x, x) ∈ R ⇒ R is reflexive.
Let R ⊆ A × B be a relation from A to B. Then R–1 ⊆ B × A
2. Let (x, y) ∈ R ⇒ x – y is divisible by m
is defined by
⇒  x – y = mq, for some q ∈ I
R–1 = [(b, a): (a, b) ∈ R] ⇒  y – x = m (–q)
Thus, ⇒  y – x is divisible by m
⇒ (y, x) ∈ R
(a, b) ∈ R ⇔ (b, a) ∈ R– 1, ∀ a ∈ A, b ∈ B Thus, (x, y) ∈ R ⇒ (y, x) ∈ R ⇒ R is symmetric.
3. Let (x, y) ∈ R and (y, z) ∈ R
I M P O R TA N T P O I N T S ⇒  x – y is divisible by m and y – z is divisible by m
⇒  x ­– y = mq and y – z = mq′ for some q, q′ ∈ I
■ dom (R–1) = range (R) and range (R–1) = dom (R) ⇒ (x – y) + (y – z) = m (q + q′)
–1 –1
■ (R ) = R. For example, if R = [(1, 2), (3, 4), (5, 6)] ⇒  x – z = m (q + q′), q + q′ ∈ I
then R–1 = [(2, 1), (4, 3), (6, 5)] ⇒ (x, z) ∈ R
and –1 –1
(R ) = [(1, 2), (3, 4), (5, 6)] = R. Thus, (x, y) ∈ R and (y, z) ∈ R ⇒ (x, z) ∈ R, so R is
dom (R) = (1, 3, 5), range (R) = (2, 4, 6) transitive.
and dom (R–1) = (2, 4, 6), range (R–1) = (1, 3, 5) Hence the relation R is reflexive, symmetric and transitive
So, dom (R–1) = range (R) and range (R–1) and therefore it is an equivalence relation.
= dom (R)
ERROR CHECK
Every identity relation is reflexive but every reflexive relation
TYPES OF RELATIONS ON A SET need not be an identity relation. Also, identity relation is
Let A be a non-empty set, then a relation R on A is said to reflexive, symmetric and transitive.
be:
1. Reflexive: If a R a, ∀ a ∈ A, i.e., if CONGRUENCE MODULO M
(a, a) ∈ R, ∀ a ∈ A
Let m be a positive integer and x, y ∈ I, then x is said to be
2. Symmetric: If a R b ⇒ b R a, ∀ a, b ∈ A, i.e., if
congruent to y modulo m, written as x ≡ y (mod m), if x – y
(a, b) ∈ R ⇒ (b, a) ∈ R, ∀ a, b ∈ A
is divisible by m.
3. Anti-symmetric: If a R b and b R a ⇒ a = b, ∀ a, b ∈ A
For example, 155 ≡ 7 (mod 4) as
4. Transitive: If a R b and b R c ⇒ a R c, ∀ a, b, c ∈ A
i.e., (a, b) ∈ R and (b, c) ∈ R ⇒ (a, c) ∈ R, ∀ a, b, c ∈ A 155 − 7 148
= = 37 (integer)
4 4
EQUIVALENCE RELATION 27 − 5 22
But 27 ≡/ 5 (mod 4) as = (Not an integer)
A relation R on a non-empty set A is called an equivalence 4 4
relation if and only if it is
1. reflexive I M P O R TA N T P O I N T S
2. symmetric and
3. transitive ■ The universal relation on a non-void set is reflexive
■ The identity and the universal relations on a non-void set
That is, R satisfies following properties:
are symmetric and transitive
1. a R a, ∀ a ∈ A ■ The identity relation on a set is an anti-symmetric relation

2. a R b ⇒ b R a, ∀ a, b ∈ A ■ The relation R on a set A is symmetric if and only if R = R


–1

3. a R b, b R c ⇒ a R c, ∀ a, b, c ∈ A ■ If R and S are two equalvalence relations on a set A, then

R ∩ S is also an equivalence relation on A.


For example, let I be the set of all integers, m be a positive ■ The union of two equivalence equivalence relations on a
integer. Then the relation, R on I is defined by, set is not necessarily an equivalence realtion on the set.
■ The inverse of an equivalence relation is an equivalence
R = [(x, y): x, y ∈ I, x – y is divisible by m]
relation.
Consider any x, y, z ∈ I.
Set Theory  1.11

Hence, 2 R ⎛⎜ − ⎞⎟ and ⎛⎜ −
1 1⎞
ERROR CHECK
⎝ 6⎠ ⎝ ⎟ R (– 2)
6⎠
A reflexive relation on a set is not necessarily symmetric.
But 2 R – 2 since 1 + 2 (– 2) = – 3 >/ 0
26. Let R be a relation defined as a R b if | a | ≤ b. Then,
Equivalence Classes of an Equivalence Relation relation R is
Let R be an equivalence relation in A (≠ f), Let a ∈ A. Then (A) Reflexive (B) Symmetric
the equivalence class of a, denoted by [a] or { a }is defined (C)  Transitive (D)  None of these
as the set of all those points of A which are related to a Solution: (C)
under the relation R. Thus, [a] = {x ∈ A : x Ra} R is not reflexive, if – a is any negative real number,
It is easy to see that
then | – a | > – a so that – a R – a. R is not symmetric
1. b ∈ [a] ⇒ a ∈[b]
consider the real numbers a = – 2 and b = 3. Then a R
2. Two equivalence classes are either disjoint or identical
b since |– 2 | < 3. But b R a since | 3 | > – 2.
R is transitive: let a, b, c be three real numbers such
SOLVED EXAMPLES that
| a | ≤ b and | b | ≤ c.
24. Let R be a relation defined as a R b if | a – b | > 0. Then,
the relation R is But | a | ≤ b ⇒ b ≥ 0, and so | b | ≤ c ⇒ b ≤ c.
It follows | a | ≤ c.
(A) Reflexive (B) Symmetric Thus a R b and b R c ⇒ a R c.
(C)  Transitive (D)  None of these
27. N is the set of natural numbers. The relation R is
Solution: (B) defined on N × N as follows (a, b) R (c, d) ⇔ a + d = b
R is not reflexive since | a – a | = 0 and so | a – a | w 0. + c. Then, R is
Thus a R a for any real number a. (A)  Reflexive only (B)  Symmetric only
R is symmetric since if | a – b | > 0, then (C)  Transitive only (D)  an equivalence relation

| b – a | = | a – b | > 0. Solution: (D)
We have, (a, b) R (a, b) for all (a, b) ∈ N × N since a +
Thus a R b ⇒ b R a b = b + a.
R is not transitive. For example, Hence, R is reflexive.
consider the numbers 3, 7, 3. Then we have 3 R 7 R is symmetric: we have
since | 3 – 7 | = 4 > 0 and 7 R 3 since | 7 – 3 | = 4 > 0.
(a, b) R (c, d) ⇒ a + d = b + c ⇒ d + a = c + b
But 3 R 3 since | 3 – 3 | = 0 so that | 3 – 3 | >/ 0.
⇒ c + b = d + a ⇒ (c, d) R (a, b)
25. Let R be a relation defined as a R b if 1 + ab > 0. Then, R is transitive: let
the relation R is
(A) Reflexive (B) Symmetric (a, b) R (c, d) and (c, d) R (e, f ).
(C)  Transitive (D)  None of these Then by definition of R, we have
Solution: (A) and (B) a + d = b + c and c + f = d + e,
Here relation R is reflexive since 1 + a × a > 0 ∀ real
numbers a. It is symmetric since 1 + ab > 0 ⇒ 1 + ⇒ a + d + c + f = b + c + d + e
ba > 0. However R is not transitive: consider three real or a + f = b + e.
1
numbers 2, – and –2. We have Hence, (a, b) R (e, f )
6
Thus, (a, b) R (c, d) and (c, d) R (e, f )
⎛ 1⎞ 2
1 + 2 × ⎜ − ⎟ = > 0
⎝ 6⎠ 3 ⇒ (a, b) R (e, f )
28. Let S = {1, 2, 3, 4, 5} and let A = S × S. Define the
⎛ 1⎞ 4 relation R on A as follows (a, b) R (c, d) if and only if
and 1 + ⎜− ⎟⎠ (– 2) = > 0
⎝ 6 3 ad = cb. Then, R is
1.12  Chapter 1

(A)  reflexive only Solution: (C)


(B)  symmetric only Let the universal set be
(C)  transitive only U = {x1, x2, x3 …xn}
(D)  equivalence relation
We know every set is a subset of itself. Therefore,
Solution: (D) inclusion of a subset is reflexive Now the elements of
Given that S = {1, 2, 3, 4, 5} and A = S × S the set {x1} are included in the set {x1, x2} but con-
A relation R on A is defined as follows: “(a, b) R verse is not true i.e.,
(c, d)” if and only if ad = cb {x1} ⊂ {x1, x2} but {x1, x2} ⊄ {x1}
(A) R is reflexive, since ab = ba Hence, the inclusion of a subset is not symmetric.
⇒  ba = ab, therefore, Thus, the inclusion of a subset is not an equiva-
(a, b) R (b, a) ∀ a, b ∈ S lence relation.
(B) R is symmetric, 32. Let R be the relation on the set R of all real numbers
since (a, b) R (c, d) defined by a R b if | a – b | ≤ 1. Then R is
⇒  ad = cb ⇒ cd = da
(A) reflexive (B) symmetric
⇒ (c, d) R (a, b) ∀ a, b ∈ S
(C) transitive (D) anti-symmetric
(C) R is transitive (a, b) R (c, d) and (c, d) R (e, f )
⇒  ad = cb and cf = ed ⇒ adcf = cb ed Solution:  (A and B)
⇒  cd (af ) = cd (be) ⇒ af = eb | a – a | = 0 < 1 so a R a ∀ a ∈ R.
⇒ (a, b) R (e, f ) ∀ a, b, c, d, e, f ∈ S \ R is reflexive
29. The relation ‘less than’ in the set of natural numbers is a R b ⇒ | a – b | ≤ 1 ⇒ | b – a | ≤ 1 ⇒ b R a.
(A)  only symmetric \ R is symmetric
(B)  only transitive 2 R 3/2, 3/2 R 2 but 2 ≠ 3/2 so R is not anti-symmetric.
(C)  only reflexive Finally, 1 R2, 2 R 3 but 1 R 3 as
(D)  equivalence relation | 1 – 3 | = 2 > 1.
Solution: (B) 33. If R is a relation ‘<’ from A = {1, 2, 3, 4} to B = {1, 3,
The relation ‘less than’ is only transitive because 5} i.e., (a, b) ∈ R if a < b, then R o R–1 is
(A)  [(1, 2), (1, 5), (2, 3) (2, 5) (3, 5), (4, 5)]
x < y, y < z ⇒ x < z, x, y, z ∈ N
(B)  [(3, 1), (5, 1), (5, 2), (5, 3), (5, 4)]
\ x R y, y R z ⇒ x R z (C)  [(3, 3), (3, 5), (5, 3), (5, 5)]
30. If R and R′ are symmetric relations (not disjoint) on a (D)  [(3, 3), (3, 4), (4, 5)]
set A, then the relation R ∩ R′ is Solution: (C)
(A) reflexive (B) symmetric Here
(C)  transitive (D)  None of these
R = [(1, 3), (1, 5), (2, 3), (2, 5), (3, 5), (4, 5)]
Solution: (B)
\ R–1 = [(3, 1), (5, 1), (3, 2), (5, 2), (5, 3), (5, 4)]
Since R ∩ R′ are not disjoint, there is at least one
ordered pair, say, (a, b) in R ∩ R′. Hence,
But (a, b) ∈ R ∩ R′ ⇒ (a, b) ∈ R and (a, b) ∈ R′ R 0 R–1 = [(3, 3), (3, 5), (5, 3), (5, 5)]
since R and R′ are symmetric relations, we get
34. A relation R on the set of complex numbers is defined
(b, a) ∈ R and (b, a) ∈ R′ z − z2
by, z1 R z2 ⇔ 1 is real, then the relation R is
and consequently (b, a) ∈ R ∩ R′ z1 + z2
similarly if any other ordered pair (c, d) ∈ R ∩ R′, (A) equivalence (B) only reflexive
then we must also have, (d, c) ∈ R ∩ R′ (C)  only transitive (D)  None of these
Hence, R ∩ R′ is symmetric
31. With reference to a universal set, the inclusion of a Solution: (A)
subset in another, is relation which is z − z1
Since 1 = 0, which is real ∀ z1 ∈ C, therefore
(A) symmetric only (B) equivalence z1 + z1
(C)  reflexive only (D)  None of these R is reflexive.
Set Theory  1.13

z1 − z2 ⇒ (a1+ a2) (b1 – b2) – (a1 – a2) (b1 + b2) = 0


For z1, z2 ∈ C, z1 R z2  ⇒  is real
z1 + z2 ⇒ 2a2b1 – 2b2a1 = 0

⎛ z − z2 ⎞ ⎛ z − z1 ⎞ a1 b1 a a
⇒ – ⎜ 1 is real  ⇒  ⎜ 2 is real ⇒ =  or  1 = 2
⎝ 1 + ⎟ ⎝ z2 + z1 ⎟⎠ a2 b2 b1 b2
z z 2⎠ 
⇒ z2 R z1. a2 a3
and =
For b2 b3

z1, z2, z3 ∈ C, Similarly,
let z1 = a1 + ib1, z2 = a2 + ib2 and z3 = a3 + ib3. a2 a3
z2 R z3  ⇒  =
b2 b3

z1 − z2 Therefore,
Now, z1 R z2  ⇒  is real
z1 + z2 z1 R z2 and z2 R z3
( a1 − a2 ) + i (b1 − b2 ) a1 a2
⇒ is real ⇒ =
( a1 + a2 ) + i (b1 + b2 ) b1 b2

( a − a2 ) + i (b1 − b2 ) ( a1 + a2 ) − i (b1 + b2 ) a2 a3
⇒ 1 × and =
( a1 + a2 ) + i (b1 + b2 ) ( a1 + a2 ) − i (b1 + b2 ) b2 b3

is real a1 a3
⇒ =
b1 b3
( a1 − a2 ) ( a1 + a2 ) + (b1 − b2 ) (b1 + b2 ) 
+ i [(b − b2 ) ( a1 + a2 ) − (b1 + b2 ) ( a1 − a2 )] ⇒ z1 R z3
⇒  1 \ R is transitive.
( a1 + a2 ) 2 + (b1 + b2 ) 2
Hence R is an equivalence relation.
is real
1.14  Chapter 1

NCERT EXEMPLARS
1. Suppose, A1, A2, ………, A30 are thirty sets each hav- 7. In a town of 840 persons, 450 persons read Hindi, 300
ing 5 elements and B1, B2, Bn are n sets each with 3 read English and 200 read both. Then, the number of
30 n
persons who read neither, is
elements, let ∪
= Ai ∪=
B j S and each element of S (A) 210 (B) 290 (C) 180 (D) 260
=i 1 =j 1

belongs to exactly 10 of the Ai’s and exactly 9 of the 8. If X = {8n − 7n − 1 n ∈ N } and


Bj’s. Then, n is equal to
(A) 15 (B) 3 (C) 45 (D) 35 Y = {49n − 49 n ∈ N } . Then,
2. Two finite sets have m and n elements. The number (A)  X ⊂ Y (B) 
Y⊂X
of subsets of the first set is 112 more than that of the
second set. The values of m and n are, respectively
(C)  X ∩Y = φ
X = Y (D) 
(A)  4, 7 (B)  7, 4 (C)  4, 4 (D)  7, 7
9. A survey shows that 63% of the people watch a news
3. The set (A ∩ B ′) ′∪ (A ∩ B) is equal to channel whereas 76% watch another channel. If x% of
the people watch both channel, then
(A) A ′∪ B ∪ C A ′∪ B
(B) 
(A) x = 35 (B)  x = 63
(C) A ′∪ C (D) 
A ′∩ B
(C)  39 ″ x ″ 63 (D) 
x = 39
4. Let F1 be the set of parallelograms, F2 the set of rect-
angles, F3 the set of rhombuses, F4 the set of squares 10. If sets A and B are defined as
and F5 the set of trapeziums in a plane. Then F1 may  
1
be equal to A = ( x , y ) y = , 0 ≠ x ∈ R  ,
(A)  F2 ∩ F3  x 
B = {( x, y ) y = − x, x ∈ R ,} .
(B)  F3 ∩ F4

(C)  F2 ∪ F5 (A)  A ∩ B = A (B) 


A∩ B = B
(D)  F2 ∪ F3 ∪ F4 ∪ F1 (C)  A ∩ B = φ (D) 
A∪ B = A
5. Let S = set of points inside the square, T = set of points
11. If A and B are two sets, then A ∩ ( A ∪ B ) equals to
inside the triangle and C = set of points inside the cir-
cle. If the triangle and circle intersect each other and (A) A (B) 
B
are contained in a square. Then, A∩ B
(C) ϕ (D) 
(A)  S ∩ T ∩ C = φ
12. If A = {1, 3, 5, 7, 11, 13, 15, 17}, B = {2, 4, ….., 18}
NCERT EXEMPLARS

(B)  S ∪ T ∪ C = C and N the set of natural numbers is the universal set,


(C)  S ∪ T ∪ C = S
then ( A ’∪ ( A ∪ B ) ∩ B ’) is
(D)  S ∪ T = S ∩ C
(A) ϕ (B) N (C) A (D) B
6. If R be the set of points inside a rectangle of sides a
and b (a, b > 1) with two sides along the positive direc- 13. If {x|x is a positive multiple of 3 less than 100} and P =
tion of X-axis and Y-axis. Then, {x|x is a prime number less than 20}. Then, n(S) + n(P)
(A)  R = {( x, y ) : 0 ≤ x ≤ a, 0 ≤ y ≤ b} is equal to
(A) 34 (B) 31 (C) 33 (D) 41
(B)  R = {( x, y ) : 0 ≤ x < a, 0 ≤ y ≤ b}
14. If X and Y are two sets and X’ denotes the complement
(C)  R = {( x, y ) : 0 ≤ x ≤ a, 0 < y < b}
of X, then X ∩ ( X ∪ Y ) ’ is equal to
(D)  R = {( x, y ) : 0 < x < a, 0 < y < b} (A) X (B) Y (C) ϕ (D) 
X ∩Y
Set Theory  1.15

ANSWER K EYS
1. (C) 2. (A) 3. (B) 4. (D) 5. (C) 6. (D) 7. (B) 8. (A) 9. (C) 10. (C)
11. (A) 12. (B) 13. (D) 14. (C)

HINTS AND EXPLANATIONS

1. If elements are not repeated, then number of elements in


So, F1 is either of F1, F2, F3 and F4.

A1 ∪ A2 ∪ A3 ∪,.... ∪ A30 is 30 × 5. ∴ F1 = F2 ∪ F3 ∪ F4 ∪ F1


But each element is used 10 times, so 5. The given sets can be represented in Venn diagram as shown
below
30 × 5
S = = 15
10 S
T

If elements in B1, B2,…….., Bn are not repeated, then total
numberof elements is 3n but each element is repeated 9
times, so
C
3n 3n
S= ⇒ 15 =
9 9

2. Since, number of subsets of a set containing m elements is
112 more than the subsets of the set containing n elements. It is clear from the diagram that, S ∪ T ∪ C = S .

6. Since, R be the set of points inside the rectangle.
∵ 2m − 2n = 112

∴ R = {( x, y ) : 0 < x < a and 0 < y < b}

(
⇒ 2n. 2m− n − 1 = 24 .7)

HINTS AND EXPLANATIONS


⇒ 2n = 24 and 2m− n − 1 = 7
b
⇒ n = 4 and 2m−1 = 8

⇒ 2 m − n = 23 ⇒ m − n = 3
0, 0 a
⇒ m − 4 = 3⇒ m = 4 + 3 7. Let H be the set of persons who read Hindi and E be the set
of persons who read English.
∴ m=7

Then,
3. We know that, n (∪) = 840, n ( H ) = 450, n ( E ) = 300, n ( H ∩ E ) = 200
Number of persons who read neither = n ( H ′ ∩ F ′ )
( A ∩ B ) ′ = ( A′ ∪ B′ ) and ( A′)′ = A


= n(H ∪ E )′

∴ = ( A ∩ B′ ) ′ ∪ ( B ∩ C )

= n (U ) − n ( H ∪ E )


=  A′ ∪ ( B′ ) ′  ∪ ( B ∩ C )
  = 840 − n ( H ) + n ( E ) − n ( H ∩ E )
= 840 – (450 + 300 – 200)
= ( A′ ∪ B ) ∪ ( B ∩ C ) = A′ ∪ B
= 840 – 550 = 290
4. Every rectangle, rhombus, square in a plane is a parallelo- { } {
8. X = 8n − 7n − 1 n ∈ N = 0, 49, 490,.....
}
gram but every trapezium is not a parallelogram.
1.16  Chapter 1

Y = {49n − 49 n ∈ N } = {0, 49, 98,147,.....} 12. A′ ∪ ( A ∪ B ) ∩ B 


 
Clearly, every elements of X is in Y but every element of Y is
not in X.
∵ A ∩ B ( B ∪ C ) = ( A ∩ B ) ∪ ( A ∩ C )
∴ X ⊂Y = A′ ∪ ( A ∩ B ′ ) ∪ ( B ∩ B ′ ) 
9. Let A be the set of percentage of those people who watch a
news channel and B be the set of percentage of those people = A′ ∪ ( A ∩ B ′ ) ∪ φ  = A′ ∪ ( A ∩ B′ )
who watch another channel.
n ( A) = 63, n ( B ) = 76, and n ( a ∩ B ) = x = ( A′ ∪ A) ∩ ( A′ ∪ B′ )
∵ n ( A ∪ B ) ≤ 100

= A′ ∪ B ′ = N ∩ ( A′ ∪ B ′ )
⇒ n ( A) + n ( B ) − n ( A ∩ B ) ≤ 100
= A′ ∪ B′ = ( A ∩ B ) ′  ∵ A ∩ B = φ 
⇒ 63 + 76 − x ≤ 100 ⇒ 139 − x ≤ 100
=φ=N
⇒ 139 − 100 ≤ x ⇒ x ≤ 63
13.  S = {x|x is a positive multiple of 3 less than 100}
∵ x ( A ∩ b) ≤ n ( A) ⇒ x ≤ 63 ∴ n(S) = 33
and P = {x|x is a prime number less than 20}
∴ 39 ≤ x ≤ 63
∴ n(P) = 8
10. Let x ∈ R n(S) + n(P) = 38 + 8 = 41
14.
We know that, − x ≠ 1
X ∩ ( X ∪ Y ) ′ = X ∩ ( X ′ ∩ Y ′ )
x
∵ ( A ∪ B ) ′ = A′ ∩ B′ 

A∩ B =φ
= ( X ∩ X ′) ∩ ( X ∩Y ′)

A ∩ ( A ∪ B) = A = φ ∩ ( X ∩Y ′) = φ

11. 
 ∵ φ ∩ A = φ 
HINTS AND EXPLANATIONS

U
A B

A ∩ ( A ∪ B)
Set Theory  1.17

PRACTICE EXERCISES

Single Option Correct Type

1. Let F1 be the set of all parallelograms, F2 the set of rect- (A) reflexive


angles, F3 the set of rhombuses, F4 the set of squares (B) symmetric
and F5 the set of trapeziums in a plane then F1is equal to (C) transitive
(A) F2 ∩ F3 (B)  F2 ∪ F3 ∪ F4 ∪ F1 (D)  an equivalence relagtion
(C) F3 ∩ F4 (D)  None of these 9. If A = { x : x2 = 1} and B { x : x4 = 1}, then A D B is
equal to:
2. (i) Let R be the relation on the set R of all real numbers
1 (A) {i, – i} (B)  {–1, 1}
defined by setting a R b if | a – b | ≤ . Then R is (C)  {–1, 1, i,–i} (D)  None of these
2
(A)  Reflexive and symmetric but not transitive 10. Which of the following is a singleton set?
(B)  Symmetric and transitive but not reflexive (A) {x : | x | < 1, x ∈ Z}
(C)  Transitive but neither reflexive nor symmetric (B) {x : | x | = 5, x ∈Z}
(D)  None of these (C) {x : x2 = 1, x ∈Z}
3. n/m means that n is a factor of m, then the relation ‘/’ is (D) {x : x2 + x + 1 = 0, x ∈R}
(A)  reflexive and symmetric. 11. Consider the following relations:
(B)  transitive and reflexive. (1) A– B = A – (A ∩ B)
(C)  reflexive, transitive and symmetric. (2) A = (A ∩ B) ∪ (A– B)
(D)  reflexive, transitive and not symmetric. (3) A – (B ∪ C) = (A – B) ∪ (A – C)
4. Let A = {x: x ∈ R, | x | < 1} Which of these is/are correct?
(A) 1 and 3 (B)  2 only
B = {x: x ∈ R, | x – 1 | ≥ 1}
(C)  2 and 3 (D)  1 and 2
and  A ∪ B = R – D, then the set D is
(A) {x: 1 < x ≤ 2} (B)  {x: 1 ≤ x < 2} 12. Let R be a reflexive relation on a finite set A having n
elements, and let there be m ordered pairs in R. Then
(C) {x: 1 ≤ x ≤ 2} (D)  None of these
(A) m ≥ n (B)  m≤n
5. Consider the set A of all determinants of order 3 with (C) m = n (D)  None of these
entries 0 or 1 only. Let B be subset of A consisting of 13. If two sets A and B are having 99 elements in common
all determinants with value 1. Let C be the subset A of then the number of elements common to each of the
consisting of all determinants with value –1. Then sets A × B and B × A are

PRACTICE EXERCISES
(A) C is empty. (A) 299 (B) 992 (C) 100 (D) 18
(B) B has as many elements as C.
(C) A = B ∪ C. 14. The relation R defined on the set A = [1, 2, 3, 4, 5] by
(D) B has twice as many elements as C. R = [(x, y): |x2 – y2| < 16] is given by,
(A)  [(1, 1), (2, 1) (3, 1), (4, 1), (2, 3)]
6. Let A and B be two sets then (A ∪ B)′ ∪ (A′ ∩ B) is (B)  [(2, 2), (3, 2) (4, 2), (2, 4)]
equal to (C)  [(3, 3), (3, 4) (5, 4), (4, 3), (3, 1)]
(A) B′ (B)  B (C)  A (D)  A′ (D)  None of these
7. If A is the set of even natural numbers less than 8 and 15. Let L denotes the set of all straight lines in a plane. Let
B is the set of prime numbers less then 7, then the a relation R be defined by a R b ⇔ a ^ b, a, b ∈ L.
number of relations from A to B is Then R is
(A) 29 (b) 92 (C) 32 (D) 29 – 1 (A) reflexive
8. For real numbers x and y, define a relation R, x R y if (B) symmetric
and only if x – y + 2 is an irrational number. Then the (C) transitive
relation R is (D)  None of these
1.18  Chapter 1

16. Let R = [(2,3), (3,4)] be a relation defined on the set (A)  [8], [6] (B)  [6], [14]
of natural numbers. The minimum number of ordered (C)  [6], [13] (D)  [8], [14], [16]
pairs required to be added in R so that enlarged rela-
tion be comes an equivalence relation is 18. Let X = {1, 2, 3, 4, 5}. The number of different ordered
pairs (Y, Z) that can formed such that Y ⊆ X, Z ⊆ X and
(A) 3 (B) 5 (C) 7 (D) 9
Y ∩ Z is empty, is
17. The solution of 8x ≡ 6 (mod 14) is (A) 52 (B) 35 (C) 25 (D) 53

Previous Years’ Questions

19. Let R = {(1, 3), (4, 2), (2, 4), (2, 3), (3, 1)} be a relation 24. If A, B and C are three sets such that A ∩ B = A ∩ C
on the set A = {1, 2, 3, 4}. The relation R is [2004] and A ∪ B = A ∪ C, then [2009]
(A) a function (B) reflexive (A) A = B (B)  A=C
(C) not symmetric (D) transitive (C) B = C (D)  A∩B=f
20. Let R = {(3, 3), (6, 6), (9, 9), (12, 12), (6, 12), (3, 9), 25. Let S be a non-empty subset of R. Consider the follow-
(3, 12), (3, 6)} be a relation on the set A = {3, 6, 9, 12} ing statement: [2010]
be a relation the set A = {3, 6, 9, 12}. The relation is P: There is a rational number x ∈ S such that x > 0.
 [2005] Which of the following statements is the negation of
(A)  reflexive and transitive only the statement P ?
(B)  reflexive only (A) There is no rational number x ∈ S such that x ≤ 0
(C)  an equivalence relation (B)  Every rational number x ∈ S satisfies x ≤ 0
(D)  reflexive and symmetric only (C) x ∈ S and x ≤ 0 ⇒ x is not rational
(D)  There is a rational number x ∈ S such that x ≤ 0
21. Let W denote the words in the English dictionary.
Define the relation R by: [2006] 26. Let R be the set of all real numbers. [2011]
R = {(x, y) ∈ W × W | the words x and y have at least Statement 1: A = {(x, y) ∈R × R : y − x is an integer}
one letter in common}. Then R is is an equivalence relation on R.
(A)  not reflexive, symmetric and transitive Statement 2: B = {(x, y) ∈ R × R : x = α y for some
(B)  reflexive, symmetric and not transitive rational number α} is an equivalence relation on R.
(C)  reflexive, symmetric and transitive (A) Statement 1 is true, Statement 2 is true; Statement 2
(D)  reflexive, not symmetric and transitive is not a correct explanation for Statement 1
(B)  Statement 1 is true, Statement 2 is false.
22. The set S = {1, 2, 3, …, 12) is to be partitioned into (C)  Statement 1 is false, Statement 2 is true.
three sets A, B, C of equal size. Thus, A ∪ B ∪ C = S, (D) Statement 1 is true, Statement 2 is true; Statement 2
A ∩ B = B ∩ C = A ∩ C = f. The number of ways to is a correct explanation for Statement 1
PRACTICE EXERCISES

partition S is [2007]


12 ! 12 ! 27. Let A and B be two sets containing 2 elements and 4
(A)  (B)  elements respectively. The number of subsets of A × B,
3!( 4 !)3 3!(3!) 4
each having at least three elements is
12 ! 12 ! (A) 220 (B) 219
(C)  3
(D) 
( 4 !) (3!) 4 (C) 211 (D) 256
23. Let R be the real line. Consider the following subsets 28. Two sets A and B are as under: [2018]
of the plane R × R.
A = {(a, b) ∈ R × R: |a – 5| < 1 and |b – 5| < 1};
S = {(x, y) : y = x + 1 and 0 < x < 2}, T = {(x, y) : x - y
B = {(a, b) ∈ R × R: 4(a – 6)2 + 9(b – 5)2 ≤ 36}.
is an integer}. Which one of the following is true?
 [2008] Then:
(A) neither S nor T is an equivalence relation on R (A) B ⊂ A
(B) both S and T are equivalence relations on R (B) A ⊂ B
(C) S is an equivalence relation on R but T is not (C) A ∩ B = ϕ (an empty set)
(D) T is an equivalence relation on R but S is not (D) neither A ⊂ B nor B ⊂ A
Set Theory  1.19

(A)  is an empty set


29. Let S = {x ∈ R: x ≥ 0 and 2| x – 3| + x ( x – 6) (B)  contains exactly one element
+ 6 = 0}. Then S [2018] (C)  contains exactly two elements
(D)  contains exactly four elements

ANSWER K EYS
Single Option Correct Type
1. (B) 2. (A) 3. (D) 4. (B) 5. (B) 6. (D) 7. (A) 8. (A) 9. (A) 10. (A)
11. (D) 12. (A) 13. (B) 14. (D) 15. (B) 16. (C) 17. (C) 18. (B)

Previous Years’ Questions


19. (C) 20. (A) 21. (B) 22. (C) 23. (D) 24. (C) 25. (B) 26. (B) 27. (B) 28. (B)
29. (C)

PRACTICE EXERCISES
1.20  Chapter 1

HINTS AND EXPLANATIONS

Single Option Correct Type


1. Since every rectangle, rhombus and square is a parallelo- 6. (A ∪ B)′ ∪ (A′ ∩ B) = (A′ ∩ B′) ∪ (A′ ∩ B)
gram so, F1 = F2 ∪ F3 ∪ F4∪ F1 A′ ∩ (B′ ∪ B)

The correct option is (B)
1 A′ ∩ U = A′

2. R is reflexive since | a – a | = 0 < for all a ∈ R.
2
The correct option is (D)
1 1 7. A = {2,4,6}, B = {2,3,5}
R is symmetric since | a – b | < ⇒|b–a|<
2 2
Number of relations from A to B = 23×3 = 29

R is not transitive: we take three numbers

The correct option is (A)
3 1 1
, , , then 8. Clearly x R x as x – x + 2 = 2 is an irrational number.
4 3 8
Thus, R is reflexive. Also ( 2 , 1) ∈ R as 2 –1 + 2 = 2
3 1 5 1 1 1 5 1
− = < and − = < 2 – 1 is an irrational number but (1, 2 ) ∉ R as 1– 2 +
4 3 12 2 3 8 24 2
2 = 1 is a rational number. So, R is not symmetric,
3 1 5 1
1R 2 and 2 2 R 2

But − = >
4 8 8 2
Since, 1 R 2 and 2 R 2 but 1 is not related to 2 . So R is
not transitive.
3 1 1 1 3 1

Thus R and R but R
The correct option is (A)
4 3 3 8 4 8
9. A = {–1, 1}, B = {–1, 1, –i, i}
The correct option is (A)
A – B = f, B–A= {–i, i}
3. ‘/’ is reflexive since every natural number is a factor of itself,
that is n/n for n ∈ N. ‘/’ is transitive: if n is a factor of m (A – B) ∪ (B – A) ={–i, i}
and m is a factor of p, then n is surely a factor of p. Thus The correct option is (A)
‘n/m’ and ‘m/p’ ⇒ ‘n/p’. However ‘/’ is not symmetric: for 10. | x | < 1 ⇒ –1 < x < 1  (x = 0 integer satisfies it)
­example, 2 is factor of 4 but 4 is not a factor of 2.
The correct option is (A)
HINTS AND EXPLANATIONS

The correct option is (D)


11. A – B = A – (A ∩ B) is correct
4. We have A = (A ∩ B) ∪ (A - B) is correct

A = {x : x ∈ R, – 1 < x < 1}
A B
and  B = {x : x ∈ R, x – 1 ≤ – 1 or x – 1 ≥ 1}


= {x : x ∈ R, x ≤ 0 or x ≥ 2}
\ A ∪ B = R – D,

where D = {x : x ∈ R, 1 ≤ x < 2}


The correct option is (B)
5. We know that the interchange of two adjacent rows (or A–B A–(A ∩ B)
­columns) changes the value of a determinant only in sign and
(3) is false.
not in magnitude. Hence corresponding to every element D
of B there is an element D′ in C obtained by interchanging \ (1) and (2) are true.
two adjacent rows (or columns) in D. It follows that The correct option is (D)
n (B) ≤ n (C)
12. The set consists of n elements and for relation to be reflexive
it must have at least n ordered pairs. It has m ordered pairs

that is, the number of elements in B is less than or equal to
therefore m ≥ n.
the number of elements in C.
The correct option is (A)
Similarly,  n (C) ≤ n (B)

13. n [(A × B) ∩ (B × A)]
Hence,  n (B) = n (C)

= n [(A ∩ B) × (B ∩ A)] = n (A ∩ B) ⋅ n (B ∩ A)

That is, B has as many elements as C.
= n (A ∩ B) . n (A ∩ B) = (99)(99) = 992

The correct option is (B)
The correct option is (B)
Set Theory  1.21

14. Here, R = {(x, y) : |x2 – y2| < 16} and A = { 1, 2, 3, 4, 5} R = [(2,2), (3,3), (4,4) (2,3),(3, 2), (3, 4), (4,3), (2,4),(4,2)]

\ R = [(1,2), (1,3), (1,4); (2,1), (2,2), (2,3), (2,4); (3,1),
The correct option is (C)
(3,2) (3,3), (3,4); (4,1), (4,2), (4,3); (4,4), (4,5); (5,4), (5,5)]
17. Since 8x ≡ 6 (mod 14) i.e, 8x –6 = 14 k for k ∈ I.
The correct option is (D)
The values 6 and 13 satisfy this equation, while 8, 14, and 16
15. Here a ^ b ⇒ b ^ a. Hence, R is symmetric. do not.
The correct option is (B) The correct option is (C)

16. To make it reflexive, we need to add (2, 2), (3, 3), (4,4). 18. Every element has 3 options. Either set Y or set Z or none, so
To make symmetric, it requires (3, 2), (4, 3) to be added. number of ordered pairs = 35.
To make transitive, (2, 4) and (4, 2) must be added, so, the The correct option is (B)
relation.

Previous Years’ Questions


19. (2, 3) belongs to R but (3, 2) is not. 26. x − y is an integer
Hence R is not symmetric. x – x = 0 is an integer ⇒ A is Reflexive
The correct option is (C) x − y is an integer ⇒ y − x is an integer ⇒ A is symmetric
20. R is Reflexive and transitive only so not an equivalence x − y, y − z are integers
relation. As sum of two integers is an integer.
e.g. (3, 3), (6, 6), (9, 9), (12, 12) [Reflexive] ⇒ (x − y) + (y − z) = x − z is an integer
(3, 6), (6, 12), (3, 12) [Transitive]. ⇒ A is transitive. Hence statement 1 is true.
x
The correct option is (A) Also = 1 is a rational number ⇒ B is reflexive
x
21. Clearly (x, x) ∈ R ∀ x ∈ W. So, R is reflexive. x y
= α is rational ⇒ need not be rational
Let (x, y) ∈ R, then (y, x) ∈ R as x and y have at least one x x
letter in common. So, R is symmetric.
0 1
But R is not transitive for example
i.e., is rational ⇒ is not rational
1 0
Particularly if x = DELHI, y = DWARKA and z = PARK,
then
Hence B is not symmetric

HINTS AND EXPLANATIONS


(x, y) ∈ R and (y, z) ∈ R but (x, z) ∉ R. ⇒ B is not an equivalence relation.

The correct option is (B)
The correct option is (B)
12! 27. n(A) = 4, n(B) = 2
22. The required number of ways is 12C4 × 8C4 × 4C4 =
( 4!)3 n(A × B) = 8
The correct option is (C)
therefore symbol Number of
23. Given T = {(x, y) : x - y ∈ I}
subsets having at least 3
As 0 ∈ I, T is a reflexive relation.
elements
If x - y ∈ I ⇒ y - x ∈ I
= 28 − (1 + 8C1 + 8C2) = 219
\ T is symmetrical also
The correct option is (B)
If x - y = I1 and y - z = I2
Then x - z = (x - y) + (y - z) = I1 + I2 ∈ I 28. The figure is represented as
\ T is also transitive. A B
Hence T is an equivalence relation.
Clearly x ≠ x + 1 ⇒ (x, x) ∉ S
\ S is not reflexive.
The correct option is (D)
24. The correct option is (C)
25. P: there is a rational number x ∈ S such that x > 0

The region A fully falls in the region B, hence A is the subset
~P: Every rational number x ∈ S satisfies x ≤ 0
of B
The correct option is (B)
Put a = x and b = y

1.22  Chapter 1

2 2
B represents an ellipse ( x − 6) + ( y − 5) ≤ 1 ⇒ 2(3 − x ) + x ( x − 6) + 6 = 0

9 4
A represent a rectangle lines ⇒ 6 − 2 x + x − 6 x + 6 = 0

−1 < x − 5 < 1 ⇒ x − 8 x + 12 = 0

4< x<6 ⇒ x − 6 x − 2 x + 12


−1 < y − 5 < 1

x [ x − 6] − 2[ x − 6] = 0
4< y<6

( x − 6)( x − 2) = 0
Even through x and y ≠ 4, 6. We will check whether the
boundary lies in the ellipse B ∴ x = 22 = 4

  (i) (x, y) = (4, 4)
x = 62 = 36
 (ii) (x, y) = (4, 6)
For Case 1
(iii) (x, y) = (6, 4)
x x Condition (x) Result
  (iv) (x, y) = (6, 6)
1 4 2 [0, 9) Accepted
( x − 6 )2 ( y − 5)2 2 36 6 [0, 9) Rejected
x y + Result
9 4
Case II: 9 ≤ x < ∞ or x ∈[9, ∞)
  (i) 4 4 25 Inside
<1 The equation reduces to
36
 (ii) 4 6 25 Inside ⇒ 2( x − 3) + x ( x − 6) + 6 = 0

<1
36 ⇒ 2 x − 6 + x − 6 x + 6

(iii) 6 4 1 Inside ⇒ x − 4 x = 0

<1
4 ⇒ ( x − 0)( x − 4) = 0

  (iv) 6 6 1 Inside ∴ x = 0

<1
4
x = 16
Hence check also prove that A lies wholly in B.
x x Condition (x) Result
29. We know that S = { x ∈ R : x ≥ 0}
1 0 0 [9, ∞) Rejected
2 x − 3 + x ( x − 6) + 6 = 0
HINTS AND EXPLANATIONS

2 16 4 [9, ∞) Accepted
Case I: 0 ≤ x < 9 or x ∈[0, 9)
Hence, S contains exactly two solutions.
The equation reduces to
CHAPTER
Complex Numbers
2
LEARNING OBJECTIVES
After reading the chapter, you will be able to:
 Understand the concept behind complex numbers and  Know how to represent complex numbers geometrically
the power of i and in argand plane along with eulerian and vectorial
 Learn what are complex numbers its equality concept, representation
and algebra  Learn about the roots and geometry of complex numbers
 Grasp the conjugate of a complex number, its properties
and modulus of a complex numbers

IMAGINARY NUMBERS
QUICK TIPS
Square root of a negative number is called an imaginary
number. ■ For any n ∈ N
⎡ 1, when n is even⎤
1.  i2n = (i2)n = (–1)n = ⎢ ⎥
Illustration 1 ⎣ − 1, when n is odd ⎦

−1, −4 , −7 , −18 , and so on are all imaginary ⎡ i, when n is even⎤


2.  i2n+1 = (i2n) i = (–1)ni = ⎢ ⎥.
numbers. ⎣ − i, when n is odd ⎦
■ The sum of four consecutive powers of i is zero. For
−1 is denoted by the Greek letter i (pronounced as
example,
iota), where i is a number such that i2 = –1. Thus,
i10 + i11 + i13 + i14 = 0
−2 = 2i , −3 = 3i , −4 = 2i. ■ Also, for any n ∈ N, the value of i–n is found out by writing
1
this as n and solving i n.
i
Info Box! ■ Thus, any integral power of i can be expressed in terms of

If a < 0, then a = |a| i. ±1 or ± i.


The symbol ‘i’ was first introduced by the famous
mathematician, Leonhard Euler (1707–1783) in
­ For any two real numbers a and b, a × b = ab is true
1748, possibly because ‘i’ is the first letter of the only when at least one of a and b is either zero or positive. If
Latin word ‘imaginarius’. both a and b are positive real numbers, then the calculation
− a × − b = ( − a) ( − b ) = ab is wrong.
The correct calculation is
INTEGRAL POWERS OF i
− a × − b = ( − 1 a )( − 1 b ) 
We have
i = −1 , i2 = –1. = (i a ) (i b )
Therefore,
= i2( a × b ) = (–1) ( ab )
i3 = i2 × i = (–1) × i = –i,
i4 = i2 × i2 = (–1) × (–1) = 1 = – ab 
2.2  Chapter 2

Thus, the calculation −2 × −3 = ( −2) × ( −3) ⎡ i (1 − i ) ⎤


= (1 + i) ⎢ ⎥
= 6 is wrong. ⎣ 1− i ⎦
= (1 + i) i = –1 + i
The correct result is −2 × −3 = (i 2 ) (i 3 )
4. The least positive integer n for which
= i2( 2 × 3) n
⎛1+ i⎞ 2 −1 1 + x 2
=– 6 ⎜⎝ 1 − i ⎟⎠ = π sin 2x
, where x > 0, is

(A) 1 (B) 2
SOLVED EXAMPLES (C)  4 (D)  None of these

⎛1+ i ⎞ ⎛1− i ⎞
8 8 Solution: (A)
1. The value of ⎜ + is equal to
⎝ 2 ⎟⎠ ⎜⎝ 2 ⎟⎠ 1 + x2
–1
For sin 2 x to be defined,
(A) 4 (B) 6 (C) 8 (D) 2
1+ x 2
Solution: (D) –1 ≤ ≤ 1
2x
8 8
⎛1+ i ⎞ ⎛1− i ⎞ 1 + x2
We have, ⎜ + or ≤ 1
⎝ 2 ⎟⎠ ⎜⎝ 2 ⎟⎠ 2x
π π⎤ ⎡ π
8
π⎤
8 or 1 + x2 ≤ 2x

= ⎢cos + i sin ⎥ + ⎢cos − i sin ⎥ or (1 – x)2 ≤ 0 or x = 1
⎣ 4 4⎦ ⎣ 4 4⎦ 
= cos 2p + i sin 2p + cos 2p – i sin 2p Now,
n n
⎛1+ i ⎞ ⎛ (1 + i ) 2 ⎞ n
= 2 cos 2p = 2 (1) = 2  [By De-Moivre’s theorem] ⎜⎝ 1 − i ⎟⎠ = 1 ⇒ ⎜ 2 ⎟ = 1 ⇒ i = 1
⎝ ⎠
2. i − −i is equal to
1 COMPLEX NUMBERS
(A) i 2 (B)  (C) 0 (D) – i 2
i 2 An expression of the form x + iy, where x and y are real
Solution:  (A, D)
numbers and i = −1 , is called a complex number. It is
1 usually denoted by z, i.e.,
We have, i=0+i⋅1= (0 + 2i)
2 z = x + iy
1 1
(1 + i2 + 2 ⋅ 1 ⋅ i) = (1 + i)2
= x is called the real part and y the imaginary part of z and
2 2
1 may be denoted by Re (z) and Im (z) respectively.
\ i =± (1 + i) If y = 0, z is called purely real and if x = 0, z is called
2 purely imaginary.
1 The set of complex numbers is denoted by C.
\ −i = ± (1 – i)
2 If x = 0 and y = 0, the complex number reduces to
Hence, 0 + i ⋅ 0 = 0, which is called the zero complex number.
1
i − −i = ± [(1 + i) – (1 – i)] = ± 2i
2 I M P O R TA N T P O I N T S
13
3. The value of the sum ∑ (i n + i n +1 ), where i = −1, ■ We observe that the system of complex numbers includes
equals n =1 the system of real numbers, i.e., R ⊂ C.
■ Every real number is a complex number.
(A) i (B)  i – 1 (C)  –i (D) 0
■ 0 is both purely real and purely imaginary number.

Solution: (B) ■ A complex number is an imaginary number if and only if its


13 13 imaginary part is non-zero. Here, real part may or may not be
∑ (i n + i n +1 ) = ∑ i n (1 + i) zero. 4 + 3i is an imaginary number but not purely imaginary.
n =1 n =1  ■ All purely imaginary numbers except zero are imaginary
⎡ i (1 − i13 ) ⎤ numbers but an imaginary number may or may not be
= (1 + i) ⎢ ⎥ purely imaginary.
⎣ 1− i ⎦
Complex Numbers  2.3

EQUALITY OF COMPLEX NUMBERS x 1x 2 + y 1y 2 + i ( x 2 y 1 − x 1y 2 )


= 
Two complex numbers are said to be equal if and only if x 22 + y 22
their real parts and imaginary parts are separately equal. x1 x2 + y1 y2 i ( x2 y1 − x1 y2 )
= +
i.e., a + ib = c + id x22 + y22 x22 + y22

⇔ a = c and b = d. Multiplicative Inverse of a Non-zero
i.e., z1 = z2 Complex Number
⇔ Re (z1) = Re (z2) and Im (z1) = Im (z2) Multiplicative inverse of a non-zero complex number z = a
+ ib is defined as
ERROR CHECK 1 1 1 a − ib a − ib
z–1 = = = × = 2
z a + ib a + ib a − ib a + b2 
Inequality relation does not hold good in case of complex
numbers having non-zero imaginary parts. For example, the a b
= 2 2
−i 2
statement 8 + 5i > 4 + 2i makes no sense. a +b a + b2 
Re (z ) [ − Im (z )]
Algebra of Complex Numbers i.e., z– 1 = +i
| z |2 | z |2

Addition
For two complex numbers z1 = a1 + ib1 and z2 = a2 + ib2, SOLVED EXAMPLES
their sum is defined as
5. The number of integral solutions of the equation
z = z1 + z2 = (a1 + a2) + i (b1 + b2) (1 – i)x = 2x are
Subtraction (A) 1 (B) 2
(C)  0 (D)  None of these
For two complex numbers z1 = a1 + ib1 and z2 = a2 + ib2, the
subtraction of z2 from z1 is defined as Solution: (C)
Let k be an integral solution of the given equation.
z1 – z2 = z1 + (– z2) = (a1 – a2) + i (b1 – b2)
( 2)
k
Multiplication Then, (1 – i)k = 2k ⇒ | (1 – i)k | = 2k ⇒ = 2k,
which is possible only if k = 0.
Multiplication of two complex numbers z1 = a + ib and
z2 = c + id is defined as 6. Let z1 and z2 be two non real complex cube roots of
unity and |z – z1|2 + |z – z2|2 = l be the equation of a
z1z2 = (ac – bd) + i(ad + bc)
circle with z1, z2 as ends of a diameter, then the value
of l is
QUICK TIPS (A) 4 (B) 3 (C) 2 (D)  2

The product of complex numbers can be easily computed if Solution: (B)


we actually carry out the multiplication as given below: We have,
(a + ib) (c + id) = ac + iad + ibc + i2bd
|z – w |2 + |z – w2|2 = l
= ac + i (ad + bc) – bd  (∵ i2 = − 1)
= (ac – bd) + i (ad + bc) ⇒ l = |w – w2|2 = |w2 + w4 – 2w3|
= |w2 + w – 2| = |– 1 – 2| = 3
Division
Division of two complex numbers CONJUGATE OF A COMPLEX NUMBER
z1 = x1 + iy1 and z2 = x2 + iy2, Conjugate of a complex number z = a + ib is defined as
where x2 + iy2 ≠ 0, is defined as z = a – ib.
z1 x + iy 1 ( x + iy1 ) ( x2 − iy2 )
= 1 = 1 For example, 4 + 5i = 4 – 5i and 4 − 5i = 4 + 5i.
z2 x 2 + iy 2 ( x2 + iy2 ) ( x2 − iy2 )

2.4  Chapter 2

where
I M P O R TA N T P O I N T S ac + bd
A= 
c2 + d 2
Geometrically, the conjugate of z is the reflection or point and
image of z in the real axis.
bc − ad
B= .
Y c2 + d 2
Imaginary axis

P(z)
QUICK TIPS
a + ib
To put the complex number in the form A + iB we
θ c + id
O X
–θ should multiply the numerator and the denominator by the
conjugate of the denominator.

Q(z )
MODULUS OF A COMPLEX NUMBER
Properties of Conjugate Modulus of a complex number z = a + ib, denoted as mod
(z) or |z|, is defined as
1. ( z ) = z
2. z = z if and only if z is purely real |z| = a2 + b 2 , where a = Re (z), b = Im (z).
3. z = – z if and only if z is purely imaginary
4. z + z = 2 Re (z) and z – z = 2i Im (z) Sometimes, |z| is called absolute value of z. Note that |z| ≥ 0.

5. z1 + z2 = z1 + z2 For example, if z = 3 + 2i, then |z| = 32 + 2 2 = 13.


6. z1 − z2 = z1 − z2
7. z1 z2 = z1 ⋅ z2 Properties of Modulus
1. |z| ≥ 0 and |z| = 0 if and only if z = 0, i.e., x = 0, y = 0
⎛z ⎞ z
8. ⎜ 1 ⎟ = 1 , z2 ≠ 0 2. |z| = | z | = |– z| = | − z | .
⎝ z2 ⎠ z2
3. z z = |z|2
9. If z = f (z1), then z = f (z1 )
4. –|z| ≤ Re (z) ≤ |z| and – |z| ≤ Im (z) ≤ |z|
( )
10. z n = ( z ) n 5. |zn| = |z|n
11. z1 z2 + z1 z2 = 2 Re ( z1 z2) = 2 Re (z1 z2 ) 6. |z1z2| = |z1| |z2|

a + ib z1 |z |
Method of Writing the Complex Number 7. = 1
c + id z2 | z2 |
in the form A + iB
8. |z1 ± z2| ≤ |z1| + |z2|
We have,
a + ib (a + ib ) ( c − id ) 9. |z1 – z2| ≥ |z1| – |z2|
=
c + id (c + id ) (c − id ) 10. |z1 + z2|2 + |z1 – z2|2 = 2 (|z1|2 + |z2|2)

[Multiplying the Nu. and the Dn. by 11. |z1 + z2|2 = |z1|2 + |z2|2 + 2 Re (z1 z )
2
the conjugate of the Dn.] 12. |z1 – z2|2 = |z1|2 + |z2|2 – 2 Re (z1 z )
2
( ac + bd ) + i (bc − ad )
= 2 2
13. |z1 + z2| = |z1| + |z2| 2
c2 + d 2 
ac + bd bc − ad z1 ⎛z ⎞
= 2 +i 2 ⇒
is purely imaginary or Re ⎜ 1 ⎟ = 0
c + d2 c + d2  z2 ⎝ z2 ⎠
= A + iB,
Complex Numbers  2.5

z +1 1
I M P O R TA N T P O I N T S ⇒ =
z − 1 iy
Geometrically |z| represents the distance of point P from 2 z 1 + iy
the origin. i.e., |z| = OP
⇒ =   (by componendo and dividendo)
2 1 − iy
Y
1 + iy 1 + y2
⇒ z= ⇒ |z| = = 1
P(z) 1 − iy 1 + y2

9. If |z – i| < 1, then |z + 12 – 6i|


(A) <14 (B)  <16
(C) >14 (D)  =14
X Solution: (A)
O M
Given, |z – i| < 1
Now, |z + 12 – 6i| = | (z – i) + (12 – 5i)|
QUICK TIPS ≤ |z – i| + |12 – 5i|
Most of the complex equations are solved using the property
zz = |z|2. ( |z1 + z2| ≤ |z1| + |z2|)
< 1 + 13 = 14
Hence |z + 12 – 6i| < 14.
SOLVED EXAMPLES
10. The maximum value of |z| when z satisfies the condi-
7. The solution of the equation |z| – z = 1 + 2i is 2
tion z + = 2 is
3 3 z
(A)  – 2i (B)  + 2i
2 2 3 +1
(A)  3 – 1 (B) 
3
(C)  2 – i (D)  None of these 2+ 3
(C)  3 (D) 
2
Solution: (A) Solution: (B)
We have, |z| – z = 1 + 2i 2 2 2 2
We have, |z| = z +
− ≤ z+ + .
z z z | z|
⇒ x 2 + y 2 – (x + iy) = 1 + 2i, 2
⇒ |z| ≤ 2 + ⇒ |z|2 ≤ 2|z| + 2
where z = x + iy |z |
⇒ |z|2 – 2|z| + 1 ≤ 1 + 2 ⇒ (|z| – 1)2 ≤ 3
⇒ x 2 + y 2 – x = 1 and y = – 2
⇒ – 3 ≤ |z| – 1 ≤ 3 ⇒ 1 – 3 ≤ |z| ≤ 1 + 3 
[Comparing real and imaginary parts]
That is, the maximum value of |z| is 1 + 3.
3
⇒ x= and y = – 2.
2 11. If |z| = Max. {|z – 1|, |z + 1|}, then
3 1
\ The solution of the given equation is – 2i. (A) |z + z | = (B)  z+ z =1
2 2
z −1
8. If is purely imaginary, then (C) |z + z | = 1 (D)  None of these
z +1
(A) |z| > 1 (B)  |z| < 1 Solution: (C)
(C) |z| = 1 (D)  None of these We have, |z| = |z – 1|
Solution: (C) ⇒ |z|2 = |z – 1|2 ⇒ z z = (z – 1) ( z – 1)
z −1
Let = iy, where y is real ⇒ z z = z z – z – z + 1 ⇒ z + z = 1
z +1
2.6  Chapter 2

Also, |z| = |z + 1| ⇒ |z|2 = |z + 1|2 From Eq. (2), we can determine the sign of xy. If xy > 0,
then x and y will have same sign. Thus,
⇒ z z = (z + 1) ( z + 1) = z z + z + z + 1
⇒ z + z = –1, ⎡ ⎛ a2 + b 2 + a ⎞ ⎛ a2 + b 2 − a ⎞ ⎤
a + ib = ± ⎢ ⎜ ⎟ +i ⎜ ⎟ ⎥⎥
\ |z + z | = 1 ⎢ ⎜⎝ 2 ⎟⎠ ⎜⎝ 2 ⎟⎠
⎢⎣ ⎥⎦
z −2 If xy < 0, then
12. If (z ≠ –2) is purely imaginary then |z| is equal to
z +2
⎡ ⎛ 2 ⎛ a2 + b2 − a ⎞ ⎤
(A) 1 (B) 2 (C) 3 (D) 4 a + b2 + a ⎞
a + ib = ± ⎢ ⎜ ⎟ −i ⎜ ⎟ ⎥⎥
⎢ ⎜ 2 ⎟⎠ ⎜⎝ 2 ⎟⎠
Solution: (B) ⎢⎣ ⎝ ⎥⎦
Let z = x + iy
z−2 x + iy − 2 ( x − 2) + iy QUICK TIPS
Then, = =
z+2 x + iy + 2 ( x + 2) + iy  Square roots of z = a + ib are:
[( x − 2) + iy ][( x + 2) − iy ] ⎡ |z| + a |z| − a ⎤
= ± ⎢ +i
( x + 2) 2 + y 2 ⎥ for b > 0 and
 ⎣ 2 2 ⎦
2 2
( x + y − 4) + i ( 4 y )
= ⎡ |z| + a |z| − a ⎤
( x + 2) 2 + y 2 ± ⎢ −i ⎥ for b < 0
 ⎣ 2 2 ⎦
z −2
Since is purely imaginary, ⎧ ⎫
z +2 a + tb + a − tb = ± 2⎨ a2 + b2 + a ⎬
⎩ ⎭
\ x2 + y2 – 4 = 0 where b > 0

⇒ x2 + y2 = 4 ⇒ |z|2 = 4 ⇒ |z| = 2.


⇒ z+ z =± 2 { z +a }
where lm(z) > 0
⎧ ⎫
Also, a + tb − a − tb = ± 2⎨ a2 + b2 − a ⎬ t
SQUARE ROOTS OF A COMPLEX NUMBER ⎩ ⎭
where b > 0
Let z = a + ib and let the square root of z be the complex
number x + iy. Then ⇒ z− z =± 2 { }
z − a t
where b > 0
a + ib = x + iy
or (a + ib) = (x + iy)2 = (x2 – y2) + (2xy) i
SOLVED EXAMPLES
Equating real and imaginary part, we get
a = x2 – y2(1) 13. If 3 a − ib = x – iy, then 3 a + ib =
and b = 2xy (2) (A) x + iy (B)  x – iy
(C) y + ix (D)  y – ix
Now, x2 + y2 = ( x 2 − y 2 )2 + 4 x 2 y 2
 Solution: (A)
2 2
= a + b (3) We have, 3
a − ib = x – iy
Solving the equations (1) and (3), we get
⇒ a – ib = (x – iy)3 = x3 – 3x2 ⋅ iy + 3x (iy)2 – (iy)3
⎛ a2 + b 2 + a ⎞ = (x3 – 3xy2) – i (3x2y – y3)
x = ± ⎜ ⎟
⎜⎝ 2 ⎟⎠
 \ a + ib = (x3 – 3xy2) + i (3x2y – y3)
= x3 + 3x2 ⋅ (iy) + 3x (iy)2 + (iy)3
⎛ a2 + b2 − a ⎞
and y=± ⎜ ⎟ = (x + iy)3
⎜⎝ 2 ⎟⎠

\ 3 a + ib = x + iy.
Complex Numbers  2.7

14. The complex number z satisfying the equations |z – i| = 1


or = – i z2 =
|z + 1| = 1 is i
(A) 0 (B) 1 + i Now, z = i ⇒ |z| = |i| = 1
(C) –1 + i (D)  1–i and z2 = – i ⇒ |z2| = | –i|
Solution:  (A, C) ⇒ |z|2 = 1 ⇒ |z| = 1
Let z = x + iy. Then, Thus, in both cases |z| = 1.
|(x + iy) – i| = |(x + iy) + 1| = 1
ARGAND PLANE AND GEOMETRICAL
or x 2 + ( y − 1)2 = ( x + 1) + y 2 = 1 REPRESENTATION OF COMPLEX NUMBERS
\ x2 + y2 – 2y + 1 = x2 + y2 + 2x + 1 Let O be the origin and OX and OY be the x-axis and y-axis
respectively. Then, any complex number z = x + iy = (x, y)
i.e., x = – y (1)
may be represented by a unique point P whose coordinates
and x2 + y2 – 2y + 1 = 1 (2) are (x, y).
From Eq. (1) and (2), x2 + x2 + 2x = 0; or x (x + 1) = 0 The representation of complex numbers as points in a
plane forms an Argand diagram.
\ x = 0, –1; The plane on which complex numbers are repre-
\ y = 0, 1 sented is known as the complex plane or Argand’s plane
or Gaussian plane. The x-axis is called the real axis and
\ z = x + iy = 0, –1 + i. y-axis the imaginary axis.
15. The complex number z satisfying the equations The complex number z = x + iy is known as the affix
|z| – 4 = |z – i| – |z + 5i| = 0, is of the point (x, y) which it represents.

(A)  3 – i (B) 2 3 – 2i POLAR FORM OF A COMPLEX NUMBER


(C)  – 2 3 – 2i (D) 0
Let O be the origin and OX and OY be the x-axis and y-axis
Solution:  (B, C) respectively. Let z = x + iy be a complex number repre-
We have two equations sented by the point P(x, y).
Draw PM ^ OX. Then,
|z| – 4 = 0 and |z – i| – |z + 5i| = 0
OM = x and PM = y. Join OP
Putting z = x + iy, these equations become
Let OP = r and ∠XOP = q.
|x + iy| = 4 i.e., x2 + y2 = 16 (1)
Then
and |x + iy – i| = |x + iy + 5i|
z = x + iy = r (cos q + i sin q)
or x2 + (y – 1)2 = x2 + (y + 5)2
i.e. y = –2 (2) This form of z is called polar or trigonometric form.
Comparing real and imaginary parts, we get
Putting y = –2 in (1), x2 + 4 = 16  or  x = ±2.
Hence, the complex numbers z satisfying the given x = r cos q(1)
equations are and y = r sin q(2)
z1 = 2 – 2i, and z2 = – 2 – 2i. Squaring Eq. (1) and (2) and adding, we get
r2 = x2 + y2 or r = x 2 + y 2 = |z|
16. If i z3 + z2 – z + i = 0, then
(A) |z| < 1 (B)  |z| > 1 Thus, r is known and is equal to the modulus of the ­complex
(C) |z| = 1 (D)  |z| = 0 number z.
Substituting the value of r in Eq. (1) and (2), we get
Solution: (C)
x y
cos q = and sin q = (3)
Given, iz3 + z2 – z + i = 0 2
x +y 2
x +y2
2

⇒ i z2 (z – i) – (z – i) = 0 y
Dividing Eq. (2) by (1), we get tan q = .
⇒ (z – i) (i z – 1) = 0 ⇒ z = i2 x
2.8  Chapter 2

The form z = r (cos q + i sinq ) = reiq of the complex


3. arg ⎛⎜ ⎞⎟ = 2 arg z
z
number z is called exponential form. ⎝z ⎠
Any value of q satisfying (3) is known as amplitude
or argument of z and written as q = arg (z) or q = amp z. 4. arg (zn) = n arg z

⎛z ⎞ ⎛z ⎞
I M P O R TA N T P O I N T S 5. If arg ⎜ 2 ⎟ = q, then arg ⎜ 1 ⎟ = 2kp – q where k ∈ I
⎝ z1 ⎠ ⎝ z2 ⎠
The unique value of q such that – p < q ≤ p for which x =
r cos q and y = r sin q, is known as the principal value of
6. arg z = – arg z
the argument.
The general value of the argument is (2np + q),
7. arg ( z z ) = arg z ( ) = arg (positive real number) = 0
2

where n is an integer and q is the principal value of arg (z).


While reducing a complex number to polar form, we
always take the principal value. SOLVED EXAMPLES
The complex number z = r (cos q + i sin q) can also
be written as rcisq.
17. The inequality |z – 4 | < |z – 2| represents the region
r ( c os θ + i s in θ )
given by,
(A)  Re (z) > 0
(B)  Re (z) < 0
rcis θ (C)  Re (z) > 3
(D)  None of these
Solution: (C)

QUICK TIPS Given |z – 4 |2 < |z – 2|2


⇒ |(x – 4) + iy |2 < | (x – 2) + iy |2
■ If x > 0, y > 0 (i.e., z is in first quadrant), then
⎛ y⎞ ⇒ (x – 4)2 + y2 < (x – 2)2 + y2
arg z = q = tan– 1 ⎜ ⎟ .
⎝ x⎠ ⇒ – 4x < – 12 ⇒ 4x > 12; x > 3
■ If x < 0, y > 0 (i.e., z is in second quadrant), then
⇒ Re (z) > 3.
⎛ y⎞
arg z = q = p – tan–1 ⎜ ⎟ .
⎝||
x⎠
⎛π⎞ ⎛π⎞
■ If x < 0, y < 0 (i.e., z is in third quadrant), then 18. If zr = cos ⎜ r ⎟ + i sin ⎜⎝ r ⎟⎠ , r = 1, 2, 3, …, then
⎝3 ⎠ 3
⎛ y⎞
arg z = q = – p + tan–1 ⎜ ⎟ . z1 z2 z3 … ∞ =
⎝ x⎠
(A) i (B)  –i
(C) 1 (D) –1
■ If x > 0, y < 0 (i.e., z is in fourth quadrant), then
⎛||
y⎞ Solution: (A)
arg z = q = – tan–1 ⎜ ⎟ .
⎝ x⎠ ⎛π⎞ ⎛π⎞
Since zr = cos ⎜ r ⎟ + i sin ⎜⎝ r ⎟⎠ ,
■ Argument of the complex number 0 is not defined. ⎝3 ⎠ 3
⎧0, if x > 0
■ arg (x + i0) = ⎨ r = 1, 2, 3, …
⎩π , if x < 0 we have, z1 · z2 · z3 … ∞
⎧π /2, if y > 0
■ arg (0 + iy) = ⎨ . ⎛ π π⎞ ⎛ π π⎞
⎩3π /2, if y < 0 = ⎜ cos + i sin ⎟ ⎜ cos 2 + i sin 2 ⎟
⎝ 3 3⎠ ⎝ 3 3 ⎠
⎛ π π⎞
Properties of Argument ⎜⎝ cos 3 + i sin 3 ⎟⎠ ...∞ 
3 3
1. arg (z1z2) = arg (z1) + arg (z2)
⎛π π π ⎞ ⎛π π π ⎞
⎛z ⎞ =cos ⎜ + 2 + 3 + ...⎟ + i sin ⎜ + 2 + 3 + ...⎟
2. arg ⎜ 1 ⎟ = arg z1 – arg z2 ⎝3 3 3 ⎠ ⎝3 3 3 ⎠
⎝z ⎠ 2
Complex Numbers  2.9

⎛ π ⎞ ⎛ π ⎞ 22. Let zk (k = 0, 1, 2, …, 6) be the roots of the equation


⎜ 3 ⎟ ⎜ ⎟ 6
= + i sin ⎜ 3 ⎟
cos ⎜
1⎟
⎜1− ⎟
1
⎜1− ⎟
(z + 1)7 + z7 = 0, then ∑ Re (z k ) is equal to
k =0
⎝ 3⎠ ⎝ 3⎠ 
3 7 7
π π (A) 0 (B)  (C) – (D) 
=
cos + i sin = 0 + i ⋅ 1 = i 2 2 2
2 2 Solution: (C)
10
⎛ 2π k 2π k ⎞ zk = xk + iyk,
19. The value of ∑ ⎜⎝ sin 11
− i cos
11 ⎟⎠
is Let
k =1
we have (zk + 1)7 + z k7 = 0
(A) 1 (B) – 1 (C) i (D) –i
Solution: (C) ⇒ (zk + 1)7 = – zk7 ⇒ |zk + 1|7 = |zk|7
We have, ⇒ |zk + 1| = |zk| ⇒ |xk + iyk + 1|2 = |xk + iyk|2
10
⎛ 2π k 2π k ⎞
∑ ⎜⎝ sin 11
− i cos
11 ⎟⎠
⇒ (xk + 1)2 + y k2 = xk2 + yk2 
k =1  1
10 ⇒ 2xk + 1 = 0  or  xk = –
⎛ 2 2π k 2π k ⎞ 2
= ∑ ⎜ −i sin − i cos
k =1
⎝ 11 11 ⎟⎠ 6 6
7

10
⎛ 2π k 2π k ⎞ 10 i 2π k
Thus, ∑ Re ( zk ) = ∑ x k = – 2
.
= –i ∑ ⎜⎝ cos 11
+ i sin
11 ⎟⎠
=–i ∑ e 11 k =0 k =0
23. If arg (z) < 0, then arg (–z) – arg (z) =
k =1 k =1 
⎡ 10 i 2π k ⎤ (A) p (B)  –p
= – i ⎢ ∑ e 11 − 1⎥ π π
⎢ k =0 ⎥ (C) – (D) 
⎣ ⎦ 2 2
= – i (sum of 11th roots of unity – 1) Solution: (A)
= – i (0 – 1) = i. As –q = arg (z) < 0,
we take z = r [cos (– q ) + i sin (– q )]
1− i 3
20. The argument of is
1+ i 3 = r (cos q – i sinq )
π 2π 4π 2π
(A)  (B)  (C)  (D) – (–z)
3 3 3 3
r
Solution: (D) π –θ
2
1− i 3 (1 − i 3 ) −2 − 2 3 i O –θ
= =
1+ i 3 4 4  r
1 3
= − − i . (Z)
2 2
⎛ 1 3 ⎞ 2π ⇒ – z = r (– cos q + i sinq )
\ arg ⎜ − − i⎟ = – (p – tan–1 3) = –
⎝ 2 2 ⎠ 3  = r [cos (p – q ) + i sin (p – q )]
21. arg bi (b > 0) is \ arg (–z) = p – q
π π Thus,
(A) p (B)  (C) – (D) 0
2 2 arg (–z) – arg (z) = p – q + (q ) = p
Solution: (B)
Since b > 0, bi represents a point on the positive side PARTICULAR CASES OF POLAR FORM
of the imaginary axis on which the argument of every
π 1. 1 = 1 + i0 = cos 0 + i sin 0
point is . 2. –1 = – 1 + i0 = cos p + i sin p
2
2.10  Chapter 2

π π π πi
3. i = 0 + i1 = cos + i sin i (12 n +1) 2 nπ i
2 2 = 2⋅e 6 = 2⋅e ⋅e 6 
2npi
⎛ π⎞ ⎛ π⎞ i/6
= 2 ⋅ ep   ( e = 1)
4. –i = 0 + i (– 1) = cos ⎜ − ⎟ + i sin ⎜ − ⎟
⎝ 2⎠ ⎝ 2⎠ π
\ arg z = .
⎡ ⎛ π⎞ ⎛ π⎞⎤ 6
5. 1 – i = 2 ⎢cos ⎜ − ⎟ + i sin ⎜ − ⎟ ⎥
⎣ ⎝ 4 ⎠ ⎝ 4⎠⎦

⎡ ⎛ 3π ⎞ ⎛ 3π ⎞ ⎤
LOGARITHM OF A COMPLEX NUMBER
6. –1 – i = 2 ⎢cos ⎜ − ⎟ + i sin ⎜ − ⎟ ⎥
⎣ ⎝ 4 ⎠ ⎝ 4 ⎠⎦ log (x + iy) = loge (reiq) = loge eiq = loge r + iq

⎛ y⎞
= loge ( x 2 + y 2 ) + i tan −1 ⎜ ⎟
EULERIAN REPRESENTATION OF A COMPLEX ⎝ x⎠

NUMBER loge (z) = loge |z| + i arg (z)
Since eiq = cos q + i sin q, thus any non zero complex num-
ber z = x + iy = r (cos q + i sin q ) can be represented in
Eulerian form as ERROR CHECK
z = reiq = r (cos q + i sin q ), Since the argument of a complex number is not unique, the
log of a complex number cannot be unique. In general,
where |z| = r and q = arg (z). loge (z) = loge |z| + i [2kp + arg (z)], k ∈ I

Info Box!
e iθ + e − iθ e iθ − e − iθ Info Box!
cos q = and sin q =
2 2i iπ
iπ ⎛ iπ ⎞
log i = log e 2 = , log (log i) = log ⎜ ⎟
2 ⎝ 2⎠
⎛π⎞ iπ
log i + log ⎜ ⎟ =
= + log (p/2).
⎝ 2⎠ 2
SOLVED EXAMPLE
( 3 + i ) 4 n +1
24. For any integer n, the argument of z = is
(1 − i 3 ) 4 n
π π VECTORIAL REPRESENTATION OF A COMPLEX
(A)  (B) 
6 3 NUMBER
π 2π If P is the point (a, b) on the argand plane corresponding to
(C)  (D) 
2 3 the complex number z = a + ib.
(E)  All of the above Then 
OP i=ˆ aiˆ + bjˆ ,
Solution: (A)
We have, 
\ OP = a 2 + b 2 = |z|
( 3 + i ) 4 n +1
z= and
(1 − i 3 ) 4 n   ⎛ b⎞
arg(z) = direction of the vector OP = tan–1 ⎜ ⎟ .
⎛ iπ ⎞
4 n +1 ⎝ a⎠
⎜ 2e 6 ⎟ i ( 4 n +1)
π
⎜⎝ ⎟⎠ 2 4 n +1e 6 De’Moivre’s Theorem
= 4n
= π
⎛ −i π ⎞ − i 4n If n is any integer, then
⎜ 2e 3 ⎟ 24n e 3

⎜⎝ ⎟⎠ (cos q + i sin q)n = cos nq + i sin nq
Complex Numbers  2.11

QUICK TIPS z 2n − 1
26. If z = cos q + i sin q, then
=
z 2n + 1
■ If n is any rational number, then cos nq + i sin nq is one of (A) i cot nq (B)  i tan nq
the values of (cos q + i sinq)n.
–n
(C) tan nq (D)  cot nq
■ (cos q + i sinq ) = cos (–n)q + i sin (–n)q (n is an integer)
= cos nq – i sin n q
n
■ (cos q – isinq ) = [cos (–q ) + i sin (–q )]
n Solution: (B)
= cos (–nq ) + isin (–nq ) We have,
= cos nq – isin nq z 2n − 1 (cos θ + i sin θ )2 n − 1
1 =
■ = (cos q + isin q )–1 = cos q – i sin q (cos θ + i sin θ )2 n + 1 
z 2n + 1
cos θ + i sinθ
n cos 2nθ + i sin 2nθ − 1
■ The theorem cannot be applied to (cos q + isinf) i.e., q =
must be same with cos and sin both. cos 2nθ + i sin 2nθ + 1 
n
■ The theorem is not directly applicable to (sin q + i cosq ) , (Using De Moivre’s Theorem)
rather n (1 − 2 sin 2 nθ ) + 2i sin nθ cos nθ − 1
⎡ ⎛π ⎞ ⎛π ⎞⎤ =
(sin q + icosq )n = ⎢cos ⎜ − θ ⎟ + i sin ⎜ − θ ⎟ ⎥ (2 cos2 nθ − 1) + 2i sin nθ cos nθ + 1 
⎣ ⎝2 ⎠ ⎝2 ⎠⎦
⎛π ⎞ ⎛π ⎞
i sin nθ cos nθ + i 2 sin 2 nθ
= cos n ⎜ − θ ⎟ + i sin ⎜ − θ ⎟ = 2
 ( i2 = –1)
⎝2 ⎠ ⎝2 ⎠ cos nθ + i sin nθ cos nθ
■ (cosq1 + i sinq1) (cos q2 + isin q2) … (cosqn + isin qn) i sin nθ (cos nθ + i sin nθ )
= = i tan nq.
= cos (q1 + q2 + … + qn) + isin (q1 + q2 + … + qn) cos nθ (cos nθ + i sin nθ )
27. If a = cos a + i sin a, b = cos b + i sin b,
a b c
c = cos g + i sin g and + + = – 1, then
SOLVED EXAMPLES b c a
cos (b – g ) + cos (g – a) + cos (a – b ) =
25. If z = cos q + i sin q, then (A) 0 (B) 1
1 (C)  –1 (D)  None of these
(A) z n + n = 2 cos nq
z Solution: (C)
1 We have,
(B) z n + n = 2n cos nq 1 1
z = cos a – i sin a, = cos b – i sin b
a b
1
(C)  z n − = 2i sin nq a
zn Now = (cos a + i sina) (cos b – i sinb )
b
1 a
(D) z n − = (2i)n sin nq or = cos (a – b ) + i sin (a – b )
zn b
Solution:  (A, C) b
Similarly, = cos (b – g ) + i sin (b – g )
We have, c
1 1 c
= = cos q – i sin q. and = cos (g – a) + i sin (g – a)
z cos θ + i sin θ a
\ zn = (cos q + i sinq)n = cos nq + i sin nq, a b c
Putting these values in + + = –1,
1 b c a
and = (cos q – i sinq)n = cos nq – i sin nq we get
zn
[cos (a – b ) + cos (b – g ) + cos (g – a)]
1
Hence,  z + n
= 2 cos nq + i [sin (a – b ) + sin (b – g ) + sin (g – a)]
zn
= –1 = – 1 + 0 i
n 1
and  z − = 2i sin nq. Comparing real part on both sides, we get
zn cos (a – b ) + cos (b – g ) + cos (g – a) = –1
2.12  Chapter 2

28. If n is a positive integer, then ( 3 + i)n + ( 3 – i)n is 2 cos θ ± 4 cos 2 θ − 4


equal to ⇒ z= = cos θ ± cos 2 θ − 1
nπ nπ 2 
(A) 2n cos (B) 2n + 1 cos
6 6 = cos θ ± − sin 2 θ = cos θ ± i 2 sin 2 θ 
n–1 nπ
(C) 2 cos (D)  None of these
6 = cosθ ± isinθ.
Solution: (B) When z = cosθ + isinθ
Let 3 = r cos q and 1 = r sin q
z2 + z–2= cos2θ + isin2θ + (cos2θ – isin2θ)
so that
1 π = 2cos2θ
r2 = 4 and tan q = ⇒ r = 2, q =
3 6  and when z = cosθ – isinθ,
n
⎛ π π⎞ 2 –2
z + z = cos2θ – isin2θ + cos2θ + isin2θ
\ ( 3 + i)n = 2n ⎜ cos + i sin ⎟
⎝ 6 6⎠  2cos2θ
=
⎧ ⎛ nπ ⎞ ⎛ nπ ⎞ ⎫
or ( 3 + i)n = 2n ⎨cos ⎜ ⎟ + i sin ⎜ ⎟ ⎬ (1)
⎩ ⎝ 6 ⎠ ⎝ 6 ⎠⎭
ROOTS OF A COMPLEX NUMBER
Similarly,
⎧ ⎛ nπ ⎞ ⎛ nπ ⎞ ⎫ If z = r (cos q + i sinq ) and n is a positive integer, then
( 3 – i)n = 2n ⎨cos ⎜ ⎟ − i sin ⎜ ⎟ ⎬ (2)
⎩ ⎝ 6 ⎠ ⎝ 6 ⎠⎭ 1 1
⎡ ⎛ 2k π + θ ⎞ ⎛ 2k π + θ ⎞ ⎤
zn = rn ⎢ cos ⎜⎝ ⎟ + i sin ⎜ ,
Adding Eq. (1) and (2), we obtain
⎣ n ⎠ ⎝ n ⎟⎠ ⎥⎦
⎛ nπ ⎞
( 3 + i)n + ( 3 – i)n = 2 ⋅ 2n cos ⎜ ⎟ where k = 0, 1, 2, 3, … (n – 1).
⎝ 6 ⎠

n+1 ⎛ nπ ⎞
=2 cos ⎜ ⎟ Cube Roots of Unity
⎝ 6⎠

29. If (sin q1 + i cos q1) (sin q2 + i cos q2) … (sin qn + i cos Let z = 11/3 or z3 – 1 = 0
qn) = a + ib, then a2 + b2 = ⇒ (z – 1) (z2 + z + 1) = 0
(A) 4 (B) 2 −1 + i 3 −1 − i 3
(C)  1 (D)  None of these i.e., z = 1, , 
2 2
Solution: (C)
−1 + i 3
Given expression Put w= ,
2
n
⎛ ⎛π ⎞ ⎛π ⎞⎞
= ∏ ⎜⎝ cos ⎜⎝ 2 − θ r ⎟⎠ + i sin ⎜⎝ 2 − θ r ⎟⎠ ⎟⎠ then w2 =
−1 − i 3
.
r =1
 2
n n
⎛π ⎞ ⎛π ⎞
= cos ∑ ⎜ − θ r ⎟ + i sin ∑ ⎜ − θ r ⎟ Thus cube roots of unity are 1, w, w2.

r =1 2
⎠ ⎝
r =1 2


= cos a + i sin a, Properties of Cube Roots of Unity
n
⎛π ⎞ 1. 1 + w + w2 = 0
where a = ∑ ⎜ − θr ⎟
r =1
⎝ 2 ⎠ 2. w3 = 1

= a + ib 3. w3n = 1, w3n + 1 = w, w3n + 2 = w2 2π i

\ a2 + b2 = cos2 a + sin2 a = 1. 4. ω = w2 and ( ω )2 = w, w ω = w3, w = e 3 ,


2π i

2
2 2
30. If z – 2zcosθ + 1 = 0, then z + z is equal to–2 w = e 3

(A) 2cos2θ (B) 2sin2θ (C)  2 cosθ (D)  2 sinθ 5. If a + bw + cw2 = 0, then a = b = c provided a, b, c are
real.
Solution: (A) 6. If these roots are marked on the argand plane, then
We have, these are vertices of an equilateral triangle with
z2 – 2zcosθ + 1 = 0 ­circumcentre at origin, as shown in the Fig. 3.1.
Complex Numbers  2.13

Imaginary = [(1 + w) (1 + w) … to n factors]


axis
–1 , 3
[(1 + w2) (1 + w2) … to n factors]
2 2 = (1 + w)n (1 + w2)n = [(1 + w) (1 + w2)]n
= (1 + w + w2 + w3)n = (0 + 1)n = 1

3 ( 1 + w + w2 = 0, w3 = 1).
Real axis
O 2π (1, 0)
3
32. If 1, w, w2 are the three cube roots of unity, then (1 – w
+ w2) (1 – w2 + w4) (1 – w4 + w8) … to 2n factors =
–1 , 3 (A) 2n (B)  22n
2 2 (C) 2 4n
(D)  None of these
FIGURE 2.1
Solution: (B)
Fourth Roots of Unity We have,

The four, fourth roots of unity are given by the solution set (1 – w + w2) (1 – w2 + w4) (1 – w4 + w8)
of the equation x4 – 1 = 0 (1 – w8 + w16) … to 2n factors
⇒ (x2 – 1) (x2 + 1) = 0 ⇒ x = ± 1, ± i
= (1 – w + w2) (1 – w2 + w) (1 – w + w2)
Fourth roots of unity are vertices of a square which lies on
coordinate axes. (1 – w2 + w) … to 2n factors.

[ w4 = w, w8 = w2, w16 = w and so on]


Some Useful Relations
1. x2 + y2 = (x + iy) (x – iy) = (– 2w) (– 2w2) (– 2w) (– 2w2) … to 2n factors
2. x3 + y3 = (x + y) (x + yw) (x + yw 2) = (22 w3) (22 w3) … to n factors
3. x3 – y3 = (x – y) (x – yw) (x – yw 2)
4. x2 + xy + y2 = (x – yw) (x – yw 2), in particular, x2 + x + [ (– 2w) (– 2w2) = 22 w3 = 22]
1 = (x – w) (x – w 2) = (22)n = 22n.
5. x2 – xy + y2 = (x + yw) (x + yw2), in particular, x2 – x +
1 = (x + w) (x + w2)
6. x2 + y2 + z2 – xy – xz – yz = (x + yw + zw2) (x + yw2 + zw) 33. − 1 − − 1 − − 1 − ... to ∞ =
7. x3 + y3 + z3 – 3xyz = (x + y + z) (x + w y + w2z) (x + w2y
+ w z) (A) 1 (B) –1 w2
(C) w (D) 

Solution:  (C, D)
SOLVED EXAMPLES
Let x= − 1 − − 1 − − 1 − ... to ∞

2
31. If 1, w, w be the three cube roots of unity, then (1 + w) Then x= 2
− 1 − x  or x = – 1 – x
(1 + w2) (1 + w4) (1 + w8) … to 2n factors =
2
(A) 1 (B) –1 or x + x + 1 = 0
(C)  0 (D)  None of these − 1 ± 1 − 4 ⋅1⋅1 −1 ± − 3
\ x= =
Solution: (A) 2 ⋅1 2 
We have, − 1 ± 3i
2 4 8
= = w or w2.
(1 + w) (1 + w ) (1 + w ) (1 + w ) … to 2n factors 2
= (1 + w) (1 + w2) (1 + w3 ⋅ w) (1 + w6 ⋅ w2) ... to 2n 6 6
factors ⎛ 3 + i⎞ ⎛ i − 3⎞
34. ⎜ ⎟ +⎜ ⎟ =
= (1 + w) (1 + w2) (1 + w) (1 + w2) … to 2n factors ⎝ 2 ⎠ ⎝ 2 ⎠
( w3 = w6 = … = 1) (A) –2 (B) 2 (C) –1 (D) 1
2.14  Chapter 2

Solution: (A) nth Roots of Unity


We have,
Since 1 = cos 0 + i sin 0, therefore,
3+i i 3+i 2 ⎛ −1 + 3 i ⎞ (1)1/n = (cos 0 + i sin 0)1/n

2
=
2i
=–i ⎜ ⎟ = –iw
⎝ 2 ⎠ 2π r + 0 2π r + 0
= cos + i sin ; r = 0, 1, 2, …, (n – 1)
⎛ −1 − 3 i ⎞ n n
i− 3 i2 − i 3
and = =–i ⎜ ⎟ = –iw
2
2π r 2π r
2 2i ⎝ 2 ⎠ = cos + i sin ; r = 0, 1, 2, …, (n – 1)
n n
6 6 2 rπ
⎛ 3 + i⎞ ⎛ i − 3⎞ i
6 2 6 =e n ; r = 0, 1, 2, …, (n – 1)
Hence, ⎜ 2 ⎟ + ⎜ 2 ⎟ = (–iw) + (–iw )
⎝ ⎠ ⎝ ⎠ (i2p/n)
= 1, e , e(i4p/n), …, e[i2(n – 1)p/n]
= i6(w6 + w12)
= 1, a, a2, a3, …, a n – 1,
= –1 (1 + 1) = –2.
where a = e(i2p/n)
35. The common roots of the equations z3 + 2 z2 + 2 z + 1 = 0
and z1985 + z100 + 1 = 0 are Properties of nth Roots of Unity
(A) –1, w (B)  –1, w2 1. 1 + a + a2 + … + a n – 1 = 0
2
(C) w, w (D)  None of these
2. 1 ⋅ a ⋅ a2 ⋅ … a n–1 = (–1)n–1
Solution: (C)
3. The n, nth roots of unity lie on the unit circle |z| = 1 and
We have, z3 + 2 z2 + 2 z + 1 = 0 form the vertices of a regular polygon of n sides.
⇒ (z + 1) (z2 + z + 1) = 0 4. nth roots of unity form a G.P. with common ratio
Its roots are – 1, w and w2. The root z = – 1 does not e(i2p/n).
satisfy the equation z1985 + z100 + 1 = 0 but z = w and
z  =  w2 satisfy it. Hence, w and w2 are the common
roots. SOLVED EXAMPLES

36. (i + 3 )100 + (i – 3 )100 + 2100 = 37. If r is non-real and r = 5 1, then the value of
(A) 1 (B) – 1 (1 + r + r 2 + r −2 − r −1 is equal to )
(C)  0 (D)  None of these (A) 2 (B) 4
(C)  8 (D)  None of these
Solution: (C)
Solution: (B)
−1 + 3 i 2 2ω
We have, i+ 3 = ⋅ = |1 + r + r2 + r–2 – r–1| = |1 + r + r2 + r3 – r4|
2 i i 
[ r5 = 1 ⇒ r3 ⋅ r2 = 1 or r–2 = r3
−1 − 3 i 2 2ω 2
and i– 3 = ⋅ =
2 i i  and r4 ⋅ r = 1  or  r–1 = r4]
\ (i + 3 )100 + (i – 3 )100 + 2100 = |1 + r + r2 + r3 + r4 – 2r4|
100 100 1 − r5
⎛ 2ω ⎞ ⎛ 2ω 2 ⎞ 100 − 2r 4 = 0 − 2r 4 ( r5 = 1)
= ⎜ +⎜ +2 1− r
⎝ i ⎟⎠ ⎝ i ⎠

 = 2|r|4 = 2(1) = 2 ( |r| = 1 as r5 = 1)
100
2
= (w100 + w200) + 2100 2|1+ r + r + r −2 − r −1 |
= 22 = 4
2

\
i100
= 2100 (w + w2) + 2100 38. The values of (16)1/4 are
(A) ±2, ±2 i (B)  ±4, ±4 i
= –2100 + 2100 = 0. (C) ±1, ±i (D)  None of these
Complex Numbers  2.15

Solution: (A) 4. Equation of a Straight Line


We have  (i) Parametric form: Equation of a straight line join-
ing the points having affixes z1 and z2 is z = t z1 +
(16)1/4 = (24)1/4 = 2 (1)1/4 (1 – t)z2, where t ∈ R
= 2 (cos 0 + i sin 0)1/4 (ii) Non-parametric form: Equation of a straight
⎧ 1 1 ⎫ line joining the points having affixes z1 and z2 is
= 2 ⎨cos (2 k π + 0) + i sin (2 k π + 0) ⎬ ,
⎩ 4 4 ⎭ z z 1
 k = 0, 1, 2, 3 z1 z1 1 = 0
= 2 × 1, 2 × i, 2 × –1, 2 × –i = ±2, ±2i z2 z2 1

GEOMETRY OF COMPLEX NUMBERS


⇒  z ( z 1 − z 2 ) − z ( z 1 − z 2 ) + z 1z 2 − z 2 z 1 = 0
1. Distance Formula: The distance between two points
P(z1) and Q(z2) is given by PQ = |z2 – z1| = |affix of QUICK TIPS
Q – affix of P| (see Fig. 3.2)
■ Three points z1, z2 and z3 are collinear if,
Q(z2)
z1 z1 1
z2 z2 1 = 0
z3 z3 1
■ If three points A(z1), B(z2), C(z3) are collinear then slope
of AB = slope of BC = slope of AC
z1 − z2 z2 − z3 z1 − z3
⇒ = =
P(z1) z1 − z2 z2 − z3 z1 − z3
FIGURE 2.2
(iii) General equation of a straight line: The general
2. Section Formula: If R(z) divides the line segment
equation of a straight line is of the form az + az
joining P(z1) and Q(z2) in the ratio m1 : m2(m1, m2 > 0)
+ b, where a is complex number and b is real
then
m z + m2z1 number.
 (i)  For internal division, z = 1 2  (iv) Slope of a line: The complex slope of the line
m1 + m 2
a coeff. of z
m1z 2 − m 2 z 1 az + az + b = 0 is – =– and real
(ii)  For external division, z = a coeff. of z
m1 − m 2
Re(a)
3. Equation of the Perpendicular Bisector: If P(z1) and slope of the line az + az + b is – = –i
Q(z2) are two fixed points and R(z) is moving point (a + a ) Im( a)
(see Fig. 3.3) such that it is always at equal distance .
(a − a )
from P(z1) and Q(z2) then locus of R(z) is perpendicu-  (v) Length of perpendicular: The length of perpen-
lar bisector of PQ dicular from a point z1 to the line az + az + b = 0
i.e., PR = QR or |z – z1| = |z – z2|
az 1 + az 1 + b az 1 + az 1 + b
⇒ |z – z1|2 = |z – z2|2
is given by or .
a+a 2a
P(z1)
5. Equation of a circle
  (i) The equation of a circle whose centre is at point
having affix z0 and radius r is |z – z0 | = r.
R(z)
(ii) If the centre of the circle is at origin and radius r,
then its equation is |z| = r (see Fig. 3.4).
P(z)

Q(z2) r

FIGURE 2.3 C(z0)



After solving,

z ( z1 − z2 ) + z ( z1 − z2 ) = |z1|2 – |z2|2
FIGURE 2.4
2.16  Chapter 2

(iii) |z – z0 | < r represents interior of a circle |z – z0| = r


QUICK TIPS
whereas |z – z0| > r represents exterior of the circle
|z – z0| = r. ■ ||z1| – |z2|| ≤ |z1 + z2| ≤ |z1| + |z2|
  Thus |z1| + |z2| is the greatest possible value of |z1 + z2|

A C = AB eiθ  or (z3 – z1) = (z2 – z1)eiq and ||z1| – |z2|| is the least possible value of |z1 + z2|.
z3 − z1 1
or = eiq ■ If z + = a, the greatest and least values of |z| are
z2 − z1 z
a + a2 + 4 − a + a2 + 4
 (iv) If A, B and C are three points in argand plane such respectively and .
2 2
that AC = AB and ∠CAB = q then use the rota- ■ The area of the triangle whose vertices are z, iz and z + iz

tion about A to find eiq, but if AC ≠ AB use coni 1


is |z|2.
method. 2
  (v)  If four points z1, z2, z3 and z4 are con-cyclic then ■ The area of the triangle with vertices z, wz and z + wz is

3
(z 4 − z 1 ) (z 2 − z 3 ) |z|2.
= real 4
(z 4 − z 2 ) (z 1 − z 3 ) ■ If z1, z2, z3 be the vertices of an equilateral triangle and z0

be the circumcentre, then z12 + z22 + z32 = 3z02 .


⎛ (z − z 3 ) (z 4 − z 1 ) ⎞
  or arg ⎜ 2
= ±p, 0 ■ If z1, z2, z3 be the vertices of a triangle, then the triangle is
⎝ ( z 1 − z 3 ) ( z 4 − z 2 ) ⎟⎠ equilateral iff (z1 – z2)2 + (z2 – z3)2 + (z3 – z1)2 = 0

or z12 + z22 + z32 = z1z2 + z2z3 + z3z1

QUICK TIPS 1 1 1
or + + =0
z1 − z2 z2 − z3 z3 − z1
If z is a variable point and z1, z2 are two fixed points in the
argand plane, then
■ The equation |z – z|2 + |z – z2|2 = k (where k is a real
1. |z – z1| = |z – z2| ⇒ Locus of z is the perpendicular 1
bisector of the line segment joining z1 and z2. number) will represent a circle with centre at (z1 + z2)
2
2. |z – z1| + |z – z2| = constant (≠ |z1 – z2|)
1 1
⇒ Locus of z is an ellipse and radius 2k − | z1 − z2 |2 provided k ≥ |z – z |2.
2 2 1 2
3. |z – z1| + |z – z2| = |z1 – z2|
⇒ Locus of z is the line segment joining z1 and z2 ■ The one and only one case in which |z1| + |z2| + … +
|zn| = |z1 + z2 + … + zn| is that the numbers z1, z2, … zn
4. |z – z1| – |z – z2| = |z – z2|
have the same amplitude.
⇒ Locus of z is a straight line joining z1 and z2 but z
■ If three points z1, z2, z3 are connected by relation az1 +
does not lie between z1 and z2.
bz2 + cz3 = 0 where a + b + c = 0, then the three points
5. |z – z1| – |z – z2| = constant (≠ |z1 – z2|) are collinear.
⇒ Locus of z is a hyperbola. z
■ If z is a complex number, then e is periodic.

6. |z – z1|2 + |z – z2|2 = |z1 – z2|2 ⇒ Locus of z is a ■ If three complex numbers are in A.P., then they lie on a
circle with z1 and z2 as the extremities of diameter. straight line in the complex plane.
7. |z – z1| = k |z – z2|, (k ≠ 1) ⇒ Locus of z is a circle.
⎛ z − z1 ⎞
8. arg ⎜ = a (fixed) ⇒ Locus of z is a segment of
⎝ z − z2 ⎟⎠ SOLVED EXAMPLES
circle.
⎛ z − z1 ⎞ π 39. The centre of a square ABCD is at z = 0. If A is z1, then
9. arg ⎜ = ± ⇒ Locus of z is a circle with z1 and
⎝ z − z2 ⎟⎠ 2 the centroid of triangle ABC is
z2 as the vertices of diameter.
z1 ⎛ π π⎞
⎛ z − z1 ⎞
10. arg ⎜ = 0 or p ⇒ Locus of z is a straight line
(A) 
3 ⎜⎝ cos 2 + i sin 2 ⎟⎠
⎝ z − z ⎟⎠
2
­passing through z1 and z2. z1
(B)  (cos p + i sin p)
3
Complex Numbers  2.17

π π 41. If z = x + iy and ‘a’ is a real number such that |z – ai| =


(C) z1 ⎛⎜ cos + i sin ⎞⎟ |z + ai|, then locus of z is
⎝ 2 2⎠
(A) x-axis (B)  y-axis
(D)  None of these
(C) x = y (B)  x2 + y2 = 1

Solution: (A)
π Solution: (A)
Since A is z1 and ∠AOB =
2 We have, |z – ai| = |z + ai|
⎛ π
\ B is z1 cos + i sin π ⎞
⎜⎝ ⇒ |x + i (y – a)|2 = |x + i (y + a)|2
2 2 ⎟⎠
⇒ x2 + (y – a)2 = x2 + (y + a)2
B A(z1)
⇒ 4ay = 0; y = 0, which is x-axis.
42. The locus represented by |z – 1| = |z + i| is
O (0, 0)
(A)  a circle of radius 1
(B)  an ellipse with foci at 1 and –i
(C)  a line through the origin
C D (D)  a circle on the join of 1 and –i as diameter
Also, c is z1(cos p + i sin p) Solution: (C)
\ Centroid of DABC is We have, |z – 1| = |z + i|

z1 ⎛ π π ⎞ ⇒ |(x – 1) + iy| = |x + i (y + 1)|


1 + cos + i sin + cos π + i sin π ⎟
3 ⎜⎝ 2 2 ⎠
 ⇒ (x – 1)2 + y2 = x2 + (y + 1)2
z1 z1
= (1 + 0 + i – 1 + 0) = i ⇒ x + y = 0, which is a line through the origin.
3 3
z1 ⎛ π π⎞ 43. The centre of a regular polygon of n sides is located at
= ⎜ cos + i sin ⎟ the point z = 0, and one of its vertex z1 is known. If z2
3⎝ 2 2⎠ 
be the vertex adjacent to z1, then z2 is equal to
40. If z1 and z2 (≠ 0) are two complex numbers such that ⎛ 2π 2π ⎞
(A) z1 ⎜ cos ± i sin ⎟
z1 − z2 ⎝ n n⎠
= 1, then
z1 + z2 ⎛ π π⎞
(B)  z1 ⎜ cos ± i sin ⎟
(A) z2 = ikz1, k ∈ R (B) z2 = kz1, k ∈ R ⎝ n n⎠

(C) z2 = z1 (D)  None of these ⎛ π π⎞
(C) z1 ⎜ cos ± i sin ⎟
Solution: (A) ⎝ 2n 2n ⎠
We have, (D)  None of these

z1 − z 2 z /z − 1 Solution: (A)
=1⇒ 1 2 = 1
z1 + z 2 z 1/z 2 + 1 Let A be the vertex with affix z1. There are two possi-

z1 z bilities and can be obtained by rotating z1 through
⇒ −1 = 1 +1 either in clockwise or in anti-clockwise direction. n
z2 z2
 12π
z1 ±
⇒ lies on the perpendicular bisector of the z2 = z1e n   ( |z2| = |z1|)
z2
­segment joining A (–1 + 0i) and B (1 + 0i). O
z1
\ = ai for some a ∈ R
z2
z2 1 −i
⇒ = =
z1 ai a 
C(z2) B(z2)
\ z2 = i kz1 for some k ∈ R A(z1)
2.18  Chapter 2

44. The locus of the complex number z in the Argand Solution: (B)


1− iz
= 1, is z −1 π
plane if We have, arg =
z −i z +1 3
(A) a circle (B)  x-axis x + iy − 1 π
(C) y-axis (D)  None of these ⇒ arg = (Putting z = x + iy)
x + iy + 1 3
y y π
Solution: (B) ⇒ tan −1 − tan −1 =
x −1 x +1 3
Let z = x + iy
1− iz ⎛ z1 ⎞
Given, = 1 ⎜⎝∵ Arg z = Arg z1 − Arg z2 ⎟⎠
z −i  2
y y
1− i ( x + iy ) −
π

x + iy − i
= 1 ⇒ tan −1 x − 1 x2 + 1 =
y 3
1+ 2
1 + y − ix x −1
⇒ = 1
x + i ( y − 1) 2y π
⇒ 2 2
= tan = 3
x + y −1 3
(1 + y )2 + x 2
⇒ = 1 2
⇒ x2 + y2 – y – 1 = 0, which is a circle.
x 2 + ( y − 1)2 3
⇒ (1 + y)2 + x2 = x2 + (y – 1)2 n
⎛ z −i ⎞
47. If w = ⎜ , n integral, then w lies on the unit
⇒ 1 + y2 + 2 y + x2 = x2 + y2 – 2 y + 1 ⎝ 1 + iz ⎟⎠
circle for
⇒ 4 y = 0
(A)  only even n (B)  only odd n
⇒ y = 0, which is the equation of x-axis. (C)  only positive n (D) all n
45. The equation |z – 1|2 + |z + 1|2 = 4 represents on the Solution: (D)
Argand plane n n
⎛ z −i ⎞ ⎛ z −i ⎞
(A)  a straight line We have, w= ⎜ = ⎜
⎝ 1 + iz ⎟⎠ ⎝ i ( z − i ) ⎟⎠
(B)  an ellipse 
n
(C)  a circle with centre origin and radius 2 ⎛ 1⎞
= ⎜ ⎟ = (–i)n
(D)  a circle with centre origin and radius unity ⎝ i⎠
Solution: (D) \ |w | = |(–i)n| = |–i|n = 1 for all n.
We have, |z – 1|2 + |z + 1|2 = 4 (1) \ w lies on unit circle for all n.
2 2 2 2
⇒ (x – 1) + y + (x + 1) + y = 4 48. The equation z z + a z + a z + b = 0, b ∈ R ­represents
(Putting z = x + iy) a circle (not point circle) if
⇒ 2 (x2 + y2 + 1) = 4 (A) |a|2 > b (B)  |a|2 < b
(C) |a| > b (D)  |a| < b
\ x2 + y2 = 1 or |z|2 = 1
Solution: (A)
⇒ |z| = 1 (since |z| cannot be –ve)
We have, z z + a z + a z + b = 0
Thus, the Eq. (1) represents all points z on the circle
⇒ z z + a z + a z + a a = a a – b
with centre origin and radius unity.
⇒ (z + a) ( z + a ) = a a – b
46. The locus of the point z satisfying the condition
⇒ |z + a|2 = |a|2 – b
z −1 π
arg = is This represents a circle (not point circle) if |a|2 > b.
z +1 3
(A)  a straight line (B)  circle 49. If z4 = (z – 1)4, then the roots are represented in the
(C)  a parabola (D)  None of these argand plane by the points that are
Complex Numbers  2.19

(A) collinear 53. If the area of the triangle on the argand plane formed
(B) concyclic by the complex numbers –z, iz, z – iz is 600 square
(C)  vertices of a parallelogram units, then |z| is equal to
(D)  None of these (A) 10 (B) 20
Solution: (A) (C)  30 (D)  None of these
We have, z4 = (z – 1)4 Solution: (B)
2 nπ i
⎛ z − 1⎞ 1/4 Area of the triangle on the argand plane formed by the
⇒ ⎜⎝ z ⎟⎠ = 1 = e 4 ,  n = 0, 1, 2, 3
3
complex numbers – z, iz, z – iz is |z|2.
Since for all these values of z, 2
3 2
z −1 \ |z| = 600 ⇒ |z| = 20
= 1 so they lie on the line bisecting perpendic- 2
z
ularly the join of z = 1 and z = 0. 54. If |z + z | + |z – z | = 8, then z lies on
(A)  a circle
50. The equation z2 + z 2 – 2|z|2 + z + z = 0 represents a (B)  a straight line
(A) straight line (B) circle (C)  a square
(C) hyperbola (D) parabola (D)  None of these
Solution: (D) Solution: (C)
We have, 2
z +z 2 2
– 2|z| + z + z = 0 We have, |z + z | + |z – z | = 8

⇒ (x + iy) + (x – iy) – 2(x2 + y2) + x + iy + x – iy = 0


2 2 ⇒ 2|x| + 2|y| = 8 or |x| + |y| = 4
(Putting z = x + iy)
⇒ 2x2 + 2 (iy)2 – 2x2 – 2y2 + 2x = 0 ⎛ z + 2i ⎞
55. If Im ⎜ = 0, then z lies on the curve
1 ⎝ z + 2 ⎟⎠
⇒ – 4 y2 + 2x = 0  or  y2 = x,
2 (A) x2 + y2 + 2x + 2y = 0
which is a parabola. (B) x2 + y2 – 2x = 0
(C) x + y + 2 = 0
51. Let z1 and z2 be two non real complex cube roots of (D)  None of these
unity and |z – z1|2 + |z – z2|2 = λ be the equation of a
circle with z1, z2 as ends of a diameter, then the value Solution: (C)
of λ is Let z = x + iy
(A) 4 (B) 3 (C) 2 (D)  2 z + 2i x + iy + 2i x + ( y + 2) i
Then, = =
Solution: (B) z+2 x + iy + 2 ( x + 2) + iy 
We have, [ x + ( y + 2) i ] [( x + 2) − iy ]
=
|z – ω |2 + |z – ω2|2 = λ ( x + 2) 2 + y 2 
⇒ λ = |ω – ω2|2 = |ω2 + ω4 – 2ω3|
( x 2 + y 2 + 2 x + 2 y ) + i ( 2 x + 2 y + 4)
2 =
|ω + ω – 2| = |– 1 – 2| = 3
= ( x + 2) 2 + y 2 
52. The region in the Argand diagram defined by |z – 3| + ⎛ z + 2i ⎞
|z + 3| < 6 is the interior of the ellipse with major axis Since Im ⎜ =0⇒x+y+2=0
⎝ z + 2 ⎟⎠
along
which represents a straight line.
(A)  real axis (B)  imaginary axis
(C) y = x (D)  y=–x 56. The cube roots of unity
Solution: (A) (A)  lie on the circle |z| = 1
The equation |z – (3 + 0i)| + |z – (–3 + 0i)| < 6 rep- (B)  are collinear
resents the interior of ellipse with foci at (3, 0) and (C)  form an equilateral triangle
(–3, 0). So, major axis is along real axis. (D)  None of these
2.20  Chapter 2

Solution:  (A, C) (A)  (0, 8) (B)  [0, 8]


Clearly, cube roots of unity 1, w, w2 satisfy |z| = 1. (C)  [1, 9] (D)  [5, 9]
2 2
2 ⎛ 3⎞ ⎛ 3⎞ Solution: (C)
Also, |1 – w | = ⎜ ⎟ + ⎜ ⎟ = 3
⎝ 2⎠ ⎝ 2 ⎠ Given |z – 1| + |z + 3| ≤ 8
\ z lies inside or on the ellipse whose foci are (1, 0)
⇒ |1 – w | = 3
and (– 3, 0) and vertices are (– 5, 0) and (3, 0).
|w – w2| = | 3 i| = 3
Y
⎛ 1 i 3⎞
and |1 – w2| = 1 − ⎜ − − 
⎝ 2 2 ⎟⎠ (4, 0)
X
(–5, 0) (–3, 0) O (1, 0) (3, 0)
3 i 3
= + = 3 .
2 2
Therefore, 1, w, w2 form an equilateral triangle. Now, |z – 4| is distance of z from (4, 0). Minimum
57. If |z – 1| + |z + 3| ≤ 8, then the range of values of |z – 4| ­distance is 1 and maximum is 9.
is
Complex Numbers  2.21

PRACTICE EXERCISES

Single Option Correct Type

1. If a, b, c, p, q, r are three complex numbers such that 8. If ( 3 + i)100 = 299 (a + ib), then b =
p q r a b c
+ + = 1 + i and + + = 0, then the value (A)  3 (B)  2
a b c p q r
(C)  1 (D)  None of these
p2 q2 r 2
of 2 + 2 + 2 is
a b c 9. The real value of a for which the expression
1 − i sin α
(A) 2i (B) 
i is purely real is
1 + 2i sin α
(C) –2i (D)  None of these π π
(A) (2n + 1) (B) (n + 1)
2 2
2. The complex numbers sin x + i cos 2x and cos x –
(C) np (D)  None of these
i sin 2x are conjugate to each other, for
(A) x = np (B) 
x=0 10. The locus of z which satisfies the inequality
log0.3 |z – 1| > log0.3 |z – i| is given by,
⎛ 1⎞
(C) x = ⎜ n + ⎟ p (D)  no value of x (A) x + y > 0 (B)  x – y < 0
⎝ 2⎠
(C) x + y < 0 (D)  x – y > 0
3. The number of solutions of the equation z2 + |z|2 = 0,
where z ∈ C is 11. If centre of a regular hexagon is at origin and one
of the vertices on argand diagram is 1 + 2i then its
(A) one (B) two ­perimeter is
(C) three (D) infinitely many (A) 2 5 (B) 6 2 (C) 4 5 (D) 6 5
4. If w is the nth root of unity, then 12. If z1, z2, z3 are three complex numbers, then z1 Im
(1 + w + w2 + … + wn –1) is ( z2 z3 ) + z2 Im ( z 3 z 1 ) + z3 Im ( z 1z 2 ) is equal to
(A) 2 (B) 0 (C) 1 (D) –1 (A) 1 (B) –1
(C)  0 (D)  None of these
5. The complex number which satisfies the equation 4
2z 1 z − z2
z+ 2 |z + 1| + i = 0 is 13. If is purely imaginary number, then 1
z1 + z 2

PRACTICE EXERCISES
3z 2
(A)  2 – i (B)  –2 – i is equal to
(C) 2 + i (D)  –2 + i
3 2 4
6. z1, z2 are two non-real complex numbers such that (A)  (B) 1 (C)  (D) 
2 3 9
z1 z 2
+ = 1. Then z1, z2 and the origin 14. If x6 = (4 – 3i)5, then the product of all of its roots is
z 2 z1
(where q = – tan–1 (3/4))
(A)  are collinear
(A) 55 (cos 5q + i sin 5q)
(B)  form right angled triangle
(C)  form right angle isosceles triangle (B) –55 (cos 5q + i sin 5q)
(D)  form an equilateral triangle (C) 55 (cos 5q – i sin 5q)
(D) –55 (cos 5q – i sin 5q)
⎡ a − ib ⎤
7. tan ⎢i log is equal to
⎣ a + ib ⎥⎦ 15. |z1 + z2| = |z1| + |z2| is possible if
1
(A) z2 = z1 (B) z2 =
2ab a2 − b 2 z1
(A)  2 (B) 
a + b2 2ab (C) arg z1 = arg z2 (D) |z1| = |z2|
2ab 16. If z = x + iy, x, y real, then |x| + |y| ≤ k |z|, where k is
(C)  2 2
(D) 
ab
a −b equal to
2.22  Chapter 2

26. The integral solution of the equation (1 – i)n = 2n is


(A) 1 (B)  2
(A) n = 0 (B)  n=1
(C)  3 (D)  None of these (C) n = – 1 (D)  None of these
17. If (1 + i) (1 + 2i) (1 + 3i) … (1 + ni) = a + ib then 2 × 5 27. The greatest value of the moduli of complex numbres
× 10 … (1 + n2) =
(A) a – ib (B)  a2 – b2
4
z satisfying the equation z− = 2 is
2
(C) a + b 2
(D)  None of these z
18. Let z1 = a + ib, z2 = p + iq be two unimodular complex (A)  5 (B)  5 –1
numbers such that Im ( z 1z 2 ) = 1. (C)  5 + 1 (D)  None of these
If w1 = a + ip, w2 = b + iq, then
28. The locus of the complex number z in an argand plane
(A)  Re (w1 w2) = 1 (B)  Im (w1 w2) = 1 satisfying the equation
(C)  Re (w1 w2) = 0 (D)  Im (ω1 ω 2 ) = 1
π
Arg (z + i) – Arg (z – i) = is
2
3 a b (A)  boundary of a circle (B)  interior of a circle
19. If a + ib = x + iy, then + =
x y (C)  exterior of a circle (D)  None of these
2 2
(A)  4 (x + y ) (B)  4 (x2 – y2)
2 2 z2
(C)  2 (x – y ) (D)  None of these 29. is always real, then
z −1
20. If z = a + ib where a > 0, b > 0, then (A) z lies only on a circle
1 1 (B) z lies only on the real axis
(A) |z| ≥ (a – b) (B) |z| ≥ (a + b)
2 2 (C) z lies either on the real axis or on a circle
1 (D)  None of these
(C) |z| < (a + b) (D)  None of these
2 z − 2z 2
30. z1 and z2 are two complex numbers such that 1
z1 − z 3 2 − z 1z 2
21. The complex numbers z1, z2 and z3 satisfying = is unimodular whereas z2 is not unimodular. Then |z1| =
z2 − z3
1 − 3i (A) 1 (B) 2 (C) 3 (D) 4
are the vertices of a triangle which is
2
31. If for the complex numbers z1 and z2, |z1 + z2| = |z1 – z2|,
(A)  of area zero (B)  right angled isosceles then amp z1 ~ amp z2 =
(C)  equilateral (D)  obtuse angled isosceles π
(A) p (B) 
2
PRACTICE EXERCISES

22. If (1 + x + x2)n = a0 + a1 x + a2 x2 + … + a2n x2n, then π


(C)  (D)  None of these
a0 + a3 + a6 + … = 4
(A) 3n (B)  3n – 1 32. The locus of the complex number z in an argand plane
n–2
(C) 3 (D)  None of these satisfying the inequality
23. If 1, a1, a2, …, an – 1 are the n nth roots of unity, then ⎛ | z − 1| + 4 ⎞ ⎛ 2⎞
(1 – a1) (1 – a2) (1 – a3) … (1 – an – 1) = log1/ 2 ⎜ >1 ⎜⎝ where | z − 1| ≠ 3 ⎟⎠ is
⎝ 3 | z − 1| − 2 ⎟⎠
(A) n + 1 (B)  n
(C) n – 1 (D)  None of these (A)  a circle
(B)  interior of a circle
24. The closest distance of the origin from a curve given
as = 0 (a is a complex number) is (C)  exterior of a circle
(D)  None of these
|a| Re a Im a
(A) 1 (B)  (C)  (D) 
2 |a| |a| 33. The equation z3 + iz – 1 = 0 has
(A)  three real roots
25. The roots of the equation z4 + 1 = 0 are
(B)  one real root
(A) (± 1 ± i) (B)  (± 2 ± 2i)
(C)  no real roots
1
(C)  (± 1 ± i) (D)  None of these (D)  no real or complex roots
2
Complex Numbers  2.23

34. If all the roots of z3 + az2 + bz + c = 0 are of unit 42. The locus of the points representing the complex num-
­modulus, then bers which satisfy |z| – 2 = 0, |z – i| – |z + 5i| = 0 is:
(A) |a| ≤ 3 (B)  |b| > 3 (A)  a circle with centre at origin
(C) |c| ≤ 3 (D)  None of these (B)  a straight line passing through origin
35. Let z1 and z2 be two complex numbers such that (C)  the single point (0, –2)
z1 z2 (D)  None of these
+ = 1, then
z2 z1
43. Let the affix of 2 – 4i be P. Then OP is rotated about
(A) z1, z2 are collinear O through an angle of 180° and is stretched 5/2 times.
(B) z1, z2 and the origin from a right angled triangle The complex number corresponding to the new posi-
(C) z1, z2 and the origin form an equilateral triangle tion of P is
(D)  None of these (A)  5 – 10i (B) 5 + 10i
(C) –5 + 10i (D)  None of these
36. If S (n) = i n + i –n, where i = −1 and n is a positive
integer, then the total number of distinct values of S (n)
44. If P, P′ represent the complex number z1 and its addi-
is
tive inverse respectively then the complex equation of
(A) 1 (B) 2 (C) 3 (D) 4 the circle with PP′ as a diameter is

1 1 z ⎛z ⎞
37. If z1 ≠ –z2 and |z1 + z2| = + , then (A)  = 1 (B)  zz + z 1z 1 = 0
z1 z 2 z 1 ⎜⎝ z ⎟⎠
(A)  at least one of z1, z2 is unimodular (C)  zz1 + zz1 (D)  None of these
(B) z1 × z2 is unimodular
(C) both z1, z2 are unimodular
45. If a, b, c, p, q, r are three non-zero complex numbers
(D)  None of these
p q r a b c
such that + + = 1 + i and + + = 0, then
38. If z = x + iy satisfies amp (z – 1) = amp (z + 3i) then the a b c p q r
value of (x – 1) : y is equal to p 2
q 2
r 2

(A)  2 : 1 (B)  – 1 : 3 value of 2 + 2 + 2 is


a b c
(C)  1 : 3 (D)  None of these
(A) 0 (B) –1
39. Let z be a complex number with modulus 2 and argu- (C) 2i (D) –2i

PRACTICE EXERCISES
ment , then z is equal to
3 z1 − z2
46. If z1, z2 are two complex numbers such that =1
(A) –1 + i 3 (B)  1 – i 3 z1 + z2
1 i 3 and tz1 = kz2 where k ∈  , then the angle between
(C) − + (D)  None of these (z1 – z2) and (z1 + z2) is
2 2
⎛ 2k ⎞ ⎛ 2k ⎞
(A) tan–1 ⎜ 2 ⎟ (B) tan–1 ⎜
⎛ | z |2 − | z | +1⎞ ⎝ k + 1⎠ ⎝ 1 − k 2 ⎟⎠
40. If log 3 ⎜ ⎟ < 2, then the locus of z is
⎝ 2 + |z | ⎠ (C)  –2 tan–1(k) (D)  2 tan–1(k)
(A) |z| < 5 (B)  |z| = 5
(C) |z| > 5 (D)  None of these 24 2
⎛ 1⎞
47. 1 + x = 3x , then ∑ ⎜ x n − n ⎟ is equal to
2
41. If z1 and z2 are complex numbers, such that z1 + z2 is a n =1
⎝ x ⎠
real number, then (A) 48 (B)  – 48
(A) z1 = – z2 (C) ±48(w – w2) (D) 1 ± 48w
(B) z2 = z 1 48. For all complex numbers z1, z2 satisfying |z1| = 12 and
(C) z1 and z2 are any two complex numbers |z2 – 3 – 4i| = 5, the minimum value of |z1 – z2| is
(D) z1 = z1 , z2 = z 2 (A) 0 (B) 2 (C) 7 (D) 17
2.24  Chapter 2

49. For any two complex numbers z1 and z2 with |z1| ≠ |z2| (A) d2 – abd – c2 = 0 (B)  d2 – abd + b2c = 0
2 2 (C) d2 + abd + c2 = 0 (D)  None of these
2 z 1 + i 3z 2 + 3z 1 + i 2 z 2 is
57. If |z – i| ≤ 2 and z0 = 5 + 3i, the maximum value of |iz
(A)  less than 5 |z1|2 + |z2|2

+ z0| is
(B)  greater than 10 |z1z2|
(A) 7 (B) 9
(C)  equal to 2|z1|2 + 3 |z2|2 (C)  13 (D)  None of these
(D) zero
50. If the roots of (z – 1)25 = 2w2(z + 1)25 where w is a ( )
58. The solutions of the equation z z − 2i = 2 (2 + i) are
complex cube root of unity are plotted in the argand
(A) 3 + i, 3 – i (B) 1 + 3i, 1 – 3i
plane, they lie on (C) 1 + 3i, 1 – i (D)  1 – 3i, 1 + i
(A)  a straight line (B)  a circle
(C)  an ellipse (D)  None of these z2
59. If z ≠ 1 and is real, then the point represented by
z −1
51. Let A0A1A2A3A4A5 be a regular hexagon inscribed in a the complex number z lies
circle of unit radius. Then the product of the lengths of
(A)  either on the real axis or on a circle passing
the line segments A0A1, A0A2 and A0A4 is
through the origin.
3 3 3 (B)  on a circle with centre at the origin.
(A)  (B)  3 3 (C) 3 (D) 
4 2 (C) either on the real axis or on a circle not passing
52. If z1 and z2 are the two complex roots of equal magni- through the origin.
π (D)  on the imaginary axis.
tude and their arguments differ by , of the quadratic
2 60. Two circles in complex plane are
equation ax2 + bx + c = 0 (a ≠ 0) then a (in terms of b C1: |z – i| = 2
and c) is
C2: |z – 1 – 2i| = 4. Then
b 2 b2 (A) C1 and C2 touch each other.
(A)  (B) 
2c c (B) C1 and C2 intersect at two distinct points.
b (C) C1 lies within C2.
(C)  (D)  None of these
2c (D) C2 lies within C1.
61. If z1z2 ∈ C, z21 + z22 ∈ R, z1(z21 – 3z22) = 2 and z2(3z21
–1 ⎡1 ⎤
53. sin ⎢ ( z − 1) ⎥ , where z is non-real, can be the angle – z22) = 11, then the value of z21 + z22 is
⎣i ⎦
(A) 2 (B) 3 (C) 4 (D) 5
PRACTICE EXERCISES

of a triangle if
(A)  Re (z) = 1, Im (z) = 2
(B)  Re (z) = 1, –1 ≤ Im (z) ≤ 1 62. If 1 − C 2 = nc – 1 and z = eiq,
(C)  Re (z) + Im (z) = 0
(D)  Re (z) = Im (z) c ⎛ n⎞
then (1 + nz ) ⎜1 + =
5 2 2n ⎝ z ⎟⎠
⎛ 1⎞
54. If x2 – x + 1 = 0 then the value of ∑ ⎜ x n + n ⎟ is
n =1
⎝ x ⎠ (A) 1 + c cosq (B)  1 – c cosq
(C) 1 + 2c cosq (D)  1 – 2c cosq
(A) 8 (B) 10
(C)  12 (D)  None of these 63. Let ‘a’ be a complex number such that | a | < 1 and z1,
z2,..., zn be the vetices of a polygon such that zk = 1 +
1+ i
55. The triangle formed by the points 1, and i as a + a2 + … + ak, then the vertices of the polygon lie
­vertices in the Argand diagram is 2 within the circle

(A) scalene (B) equilateral 1 1
(A) | z | = (B)  |z – a | =
(C) isosceles (D) right-angled |1 − a | |1 − a |
56. If the quadratic equation z2 + (a + ib)z + c + id = 0, 1 1
where a, b, c, d are non-zero real numbers, has a real (C)  z − = (D)  None of these
root, then 1− a |1 − a |
Complex Numbers  2.25

64. All the roots of the equation a1z3 + a2z2 + a3z + a4 = 3, π


where |ai| ≤ 1, i = 1, 2, 3, 4 lie outside the circle with 71. If |z – i| = 1 and arg (z) = q, θ ∈ ⎛⎜ 0, ⎞⎟ , then the value
2 ⎝ 2⎠
centre origin and radius of cot θ − is equal to
1 2 z
(A)  (B)  (A) 0 (B) i (C) – i (D) 1
3 3
(C)  1 (D)  None of these 4 + 3i
72. The reflection of the complex number in the
4 4
65. If z = (z – 1) , then the roots are represented in the straight line iz = is 1 + 2i
argand plane by the points that are (A) 2 + i (B)  2–i
(A) collinear (C) 1 + 2i (D)  1 – 2i
(B) concyclic
(C)  vertices of a parallelogram 73. If i = −1 , then
334 365
(D)  None of these ⎛ 1 i 3⎞ ⎛ 1 i 3⎞
4 + 5 ⎜ − + +3 ⎜− + is equal to
66. If a, b, c are real, a2 + b2 + c2 = 1 and b + ic = (1 + a)z, ⎝ 2 2 ⎟⎠ ⎝ 2 2 ⎟⎠
1 + iz
then = (A)  1 – i 3 (B)  – 1 + i 3
1 − iz
(C)  i 3 (D)  – 3i
a − ib a + ib
(A)  (B) 
1+ c 1+ c 74. Let b z + b z = c, b ≠ 0, be a line in the complex plane,
where b is the complex conjugate of b. If a point z1 is
a + ib a − ib the reflection of a point z2 through the line, then z 1 b +
(C)  (D) 
1− c 1− c z2 b =
67. If z1, z2 are two complex numbers and wk, k = 0, 1, …, (A) 4c (B)  2c
n −1
(C) c (D)  None of these
n – 1 are the nth roots of unity, then ∑ | z1 + z2 w k |2 75. Let z1 and z2 be roots of the equation z2 + pz + q = 0,
k =0
where the coefficients p and q may be complex
(A) < n (|z1|2 + |z2|2) (B)  = n (|z1|2 + |z2|2) ­numbers. Let A and B represent z1 and z2 in the com-
> n (|z1|2 + |z2|2)
(C)  (D)  can’t say plex plane. If ∠AOB = a ≠ 0 and OA = OB, where O is
α
the origin, then p2 = k cos2 , where k =
68. The equation |z – z1|2 + |z – z2|2 = k, k ∈ R represents a 2
circle if (A) q (B)  2q
(C) 4q (D)  None of these
1 1

PRACTICE EXERCISES
(A) k ≥ | z 1 − z 2 |2 (B)  k ≤ | z1 − z2 |2
2 2 76. If z1, z2, z3 are complex numbers such that |z1| = |z2| =
1 1 1 1 1
(C) k ≥ | z 1 + z 2 |2 (D)  k ≤ | z 1 + z 2 |2 |z3| = + + = 1, then |z1 + z2 + z3| is
2 2 z1 z2 z3
69. f (z) when divided by z – i gives remainder i; when (A)  equal to 1 (B)  less than 1
divided by z + i gives remainder i + 1. When f (z) is (C)  greater than 3 (D)  equal to 3
divided by z2 + 1, the remainder is 77. If |z| ≤ 1, |w | ≤ 1, then |z – w |2
i ⎛ 1⎞ i ⎛ 1⎞ (A) ≤ (|z| – |w |)2 – (Arg z – Arg w)2
(A)  z + ⎜ i − ⎟ (B)  z − ⎜ i + ⎟ (B) ≤ (|z| – |w |)2 + (Arg z – Arg w)2
2 ⎝ 2⎠ 2 ⎝ 2⎠
(C) ≤ (|z| – |w |)2 + 2 (Arg z – Arg w)2
i ⎛ 1⎞ −i ⎛ 1⎞ (D)  None of these
(C)  z + ⎜ i + ⎟ (D)  z + ⎜ i + ⎟
2 ⎝ 2⎠ 2 ⎝ 2⎠
78. Suppose, z1, z2, z3 are the vertices of an equilateral tri-
2 3 angle inscribed in the circle |z| = 2. If z1 = 1 + i 3 then
70. The value of the expression (w – 1) (w – w ) (w – w )
z2 and z3 are equal to
… (w – wn–1), where w is the nth root of unity, is
(A)  nwn–1 (B)  nwn (A)  – 2, 1 – i 3 (B)  2, 1 – i 3
n
(C) (n – 1) w (D) (n – 1) wn–1 (C)  – 2, 1 + i 3 (D)  None of these
2.26  Chapter 2

3n
79. If k = , where n is an even positive integer, then (A) z 12 + z 22 + z 1 z 2 = 0 (B)  z 12 + z 22 = z1 z2
2
k
(C) z 22 − z 12 = z1 z2 (D) z 12 − z 22 = z1 z2
∑ (−3)r −1 ⋅ 3n C2r −1 =
r =1
87. If |z – 25i| ≤ 15, then |maximum amp (z) – minimum
(A) 0 (B) 1 amp (z)| is equal to
(C)  – 1 (D)  None of these
⎛ 3⎞ ⎛ 3⎞
(A) sin–1 ⎜ ⎟ – cos–1 ⎜ ⎟
80. If a and b are real numbers between 0 and 1 such that ⎝ 5⎠ ⎝ 5⎠
the points z1 = a + i, z2 = 1 + bi and z3 = 0 form an
π ⎛ 3⎞
equilateral triangle, then a and b are (B)  + cos −1 ⎜ ⎟
2 ⎝ 5⎠
(A) 2 + 3 , 2 – 3 (B)  2 – 3,2– 3
p – 2 cos–1 ⎛⎜ ⎞⎟
3
(C) 
(C)  2 – 3 , 2 + 3 (D)  None of these ⎝ 5⎠
81. Let z1 and z2 be complex numbers such that z1 ≠ z2 and ⎛ 3⎞
(D) cos–1 ⎜ ⎟
⎝ 5⎠
|z1| = |z2|. If z1 has positive real part and z2 has negative
z + z2 88. If z2 + (p + iq) z + r + is = 0 where p, q, r, s are n­ on-zero,
imaginary part, then 1 may be
z − z2 has real roots, then
(A)  0 1 (B)  real and positive
(A) pqs = s2 + q2r (B)  pqr = r2 + p2s
(C)  real and negative (D)  purely imaginary 2 2
(C) prs = q + r p (D)  qrs = p2 + s2q
82. If the complex numbers z1, z2, z3 are the vertices A,
B, C respectively of an isosceles right angled triangle 89. If z1 and z2 are any two complex numbers, then
with right angle at C, then
z 1 + z 12 − z 22 + z 1 − z 12 − z 22 is equal to
(z1 – z2)2 = k (z1 – z3) (z3 – z2), where k =
(A) 1 (B) 2 (A) |z1 + z2| (B)  |z1|
(C)  4 (D)  None of these (C) |z2| (D)  None of these
83. If the origin and the two points represented by com- z
plex numbers A and B form vertices of an equilateral 90. If z = x + iy lies in IIIrd quadrant, then also lies in
the IIIrd quadrant if z
A B
triangle, then + =
B A (A) y > x > 0 (B)  y < x < 0
(A) 1 (B) – 1 (C) x < y < 0 (D)  x > y > 0
(C)  2 (D)  None of these
91. If in an argand plane points z1, z2, z3 are the vertices of
PRACTICE EXERCISES

84. If 2 2 x 4 = ( 3 – 1) + i ( 3 + 1), then an isosceles triangle right angled at z2, then


1 1 (A)  z12 + 2 z22 + z32 = 2 z2 (z1 + z3)
x = cos (2np + k) + i sin (2np + k);
4 4 (B) z 12 + z 22 + z 32 = 2 z2 (z1 + z3)
n = 0, 1, 2, 3, where k =
π 5π (C) z 12 + z 22 + 2 z 32 = 2 z2 (z1 + z3)
(A)  (B) 
12 12 (D) 2 z12 + z22 + z32 = 2 z2 (z1 + z3)
7π 92. In the Argand diagram, if O, P and Q represent respec-
(C)  (D)  None of these
12 tively the origin and the complex numbers z and z + iz,
p then the ∠OPQ is
⎡ 10 ⎛ 2q π ⎞ ⎤
32
2q π π π π 2π
85. ∑ ∑
(3 p + 2) ⎢ ⎜ sin
⎢⎣ q =1 ⎝ 11
− i cos ⎥ =
11 ⎟⎠ ⎥
(A)  (B) 
4 3
(C) 
2
(D) 
3
p =1 ⎦
93. If z satisfies |z + 1| < |z – 2|, and w = 3z + 2 + i, then
(A)  8 (1 – i) (B)  16 (1 – i)
(C)  48 (1 – i) (D)  None of these (A) |w + 1| < |w – 8| (B)  |w + 1| < |w – 7|
(C) w + ω > 7 (D)  |w + 5| < |w – 4|
86. The three vertices of a triangle are represented by
the complex numbers 0, z1 and z2. If the triangle is 94. If P (x) and Q (x) are two polynomials such that f (x) =
­equilateral, then P (x3) + x Q (x3) is divisible by x2 + x + 1, then
Complex Numbers  2.27

(A) 
P (x) is divisible by (x – 1) but Q (x) is not divisi- 102. If z1, z2 are two complex numbers and c > 0 such that
ble by x – 1 |z1 + z2|2 ≤ (1 + c) |z1|2 + k |z2|2, then k =
(B) 
Q (x) is divisible by (x – 1) but P (x) is not divisi-
(A)  1 – c (B) 
c – 1 (C)  1 + c–1 (D)  1 – c–1
ble by x – 1
(C) Both P (x) and Q (x) are divisible by x – 1
(D) f (x) is divisible by x – 1 103. If |z – 4 + 3i| ≤ 1 and m and n are the least and greatest
x4 + x2 + 4
values of |z| and k is the least value of on
⎛ 8π ⎞ ⎛ 8π ⎞ the interval (0, ∞), then k is equal to x
95. If a = cos ⎜ ⎟ + i sin ⎜ ⎟ , then
⎝ 11 ⎠ ⎝ 11 ⎠
(A)  m (B)  n
Re (a + a2 + a3 + a4 + a5) is equal to (C)  m + n (D)  None of these
1 1
(A)  (B)  –
2 2 104. If n > 1, then the roots of zn = (z + 1)n lie on a
(C)  0 (D)  None of these (A) circle
(B)  straight line
96. If A, B, C are the angles of a triangle and eiA, eiB, eiC are
(C) parabola
in A.P., then the triangle must be
(D)  None of these
(A)  right angle (B)  isosceles triangle
(C)  equilateral (D)  None of these 105. Let z be a complex number satisfying z2 + z + 1
= 0. If n is not a multiple of 3, then the value of zn
m
2 mi cot −1 p ⎛ pi + 1⎞ + z2n =
97. e ⋅⎜ =
⎝ pi − 1⎟⎠ (A) 2 (B) –2
(A) 0 (B) 1 (C) 0 (D) –1
(C)  – 1 (D)  None of these
106. If 1, a1, a2, a3 and a4 be the roots of x5 – 1 = 0, then
98. If z1 and z1 represent adjacent vertices of a regular ω −α ω −α ω −α ω −α
Im ( z 1 ) 2 1 ⋅ 2 2 ⋅ 2 3 ⋅ 2 4 =
polygon of n sides and if = 2 – 1, then n ω − α1 ω − α 2 ω − α 3 ω − α 4
is equal to Re ( z 1 )
(A) 1 (B)  w
(A) 4 (B) 8
(C)  16 (D)  None of these w2
(C)  (D)  None of these

99. If z1, z2, z3 are non-zero, non-collinear complex num- 107. If z1 and z2 both satisfy the relation z + z = 2|z – 1|

PRACTICE EXERCISES
2 1 1 π
bers such that = + , then the points z1, z2, z3 and arg (z1 – z2) = , then the imaginary part of
z1 z 2 z 3 4
lie (z1 + z2) is
(A)  in the interior of a circle
(A) 0 (B) 1
(B)  on a circle passing through origin
(C)  in the exterior of a circle (C)  2 (D)  None of these
(D)  None of these
1 1 1 1
100. If |z – 25 i| ≤ 15, then the least positive value of arg 108. If + + + =, where a, b, c are
z is a+ω b +ω c +ω d +ω
4
4 real and w is a non-real cube root of unity, then
(A)  p – tan– 1 (B) tan– 1 3
3 (A)  a + b + c + d = – 2abcd
–1 4 1 1 1 1
(C) –p + tan (D)  None of these (B)  + + + =2
3 1+ a 1+ b 1+ c 1+ d
101. If |z – 4 + 3i| ≤ 2, then the least and the greatest ­values 1 1 1 1 2
of |z| are (C)  2
+ 2
+ 2
+ 2
=– 2
a+ω b +ω c +ω d +ω ω
(A)  3, 7 (B)  4, 7 (D) abc + bcd + abd + acd = 4
(C)  3, 9 (D)  None of these
2.28  Chapter 2

109. If z1 + z2 + z3 = A, z1 + z2 w + z3 w2 = B and z1 + z2 OA  ·  OB  = 1. If O and B represent the complex


w2 + z3 w = C, where 1, w, w2 are the three cube roots ­numbers o and z, then A represents
of unity, then |A|2 + |B|2 + |C|2 = 1 1
(A)  (B) 
(A)  3 (|z1|2 + |z2|2 + |z3|2) z z
(B)  2 (|z1|2 + |z2|2 + |z3|2) (C) z (D)  z2

(C) (|z1|2 + |z2|2 + |z3|2) 114. ABCD is a rhombus. Its diagonals AC and BD inter-
(D)  None of these sect at the point M and satisfy BD = 2AC. If the points
D and M represent the complex numbers 1 + i and
1 2  –  i, respectively, then A represents the complex
110. If a, b are the roots of z + = 2(cos q + sin q ) Then,
z number
(A) |a – i| > |b – i| (B) |a – i| < |b – i|
(C) |a – i| = |i – b | (D) |a – i| = |b – i| i i i 3
(A) 3 − or 3 + (B)  3 + or 1 + i
2 2 2 2
111. If at least one value of the complex number z = x + iy
(C)  3 – i or 1 – 3i (D)  None of these
satisfies the condition z + 2 = a2 – 3a + 2 and the
115. The locus represented by the complex equation
inequality z + i 2 < a2 , then
⎛π ⎞
|z – 2 – i| = | z | sin ⎜ − arg z ⎟ is the part of
(A) a > 2 (B)  a=2 ⎝4 ⎠
a < 2
(C)  (D)  None of these (A)  a pair of straight lines
112. If a is the nth root of unity, then 1 + 2a + 3a2 + … to (B)  a circle
n terms is equal to (C)  a parabola
n n (D)  a rectangular hyperbola
(A) − (B)  −
(1 − α ) 2 1−α 116. If z1, z2, z3 are three points lying on the circle |z| = 2,
2n 2n then the minimum value of |z1 + z2|2 + |z2 + z3|2 +
(C) − (D)  − |z3 + z1|2 is equal to
1−α (1 − α ) 2
(A) 6 (B) 12
113. Let O, A, B be three collinear points such that (C) 15 (D) 24

Previous Year’s Questions

117. If ω is an imaginary cube root of unity, then (1 + ω (A) x = 4n, where n is any positive integer
PRACTICE EXERCISES

– ω 2)7 equals: [2002] x = 2n, where n is any positive integer


(B) 
(A) 128 ω (B)  –128 ω x = 4n + 1, where n is any positive integer
(C) 
(C) 128 ω2 (D) –128 ω2 (D) x = 2n + 1, where n is any positive integer
118. Let z1 and z2 be two roots of the equation z2 + az + b = 0,
121. Let z, w be complex numbers such that z + iw = 0
z being complex. Further, assume that the origin, z1
and arg zw = π. Then arg z equals [2004]
and z2 form an equilateral triangle, then
 [2003] π 5π
(A) (B) 
(A) a2 = b (B)  a2 = 2b 4 4
(C) a2 = 3b (D)  a2 = 4b
3π π
119. If z and ω are two non-zero complex numbers such (C) (D) 
π 4 2
that | zω | = 1, and Arg (z) − Arg (ω) = , then Z ω
2
is equal to [2003] ⎛ x y⎞
1 ⎜⎝ p + q ⎟⎠
(A) 1 (B) –1
122. If z = x − iy and z3 = p + iq , then is equal to
(C) i (D)  −i ( p2 + q2 )
 [2004]
x
⎛1+ i⎞ (A) 1 (B) –2
1 20. If ⎜ = 1 , then [2003]
⎝ 1 − i ⎟⎠ (C) 2 (D) –1
Complex Numbers  2.29

123. If | z2 –1| = | z |2+ 1, then z lies on [2004]


*131. If Z − 4 = 2 , then the maximum value of | Z | is
(A)  the real axis Z
(B)  an ellipse equal to [2009]
(C)  a circle
(D)  the imaginary axis. (A)  3 + 1 (B)  5+1
(C) 2 (D) 2 + 2
124. If the cubes roots of unity are 1, ω, ω2 then the roots
of the equation [2005] 132. The number of complex numbers z such that | z− 1|
3
(x − 1) + 8 = 0, are = |z + 1| = |z−i| equals [2010]
(A)  −1, −1 + 2ω, −1 −ω2 (A) 1 (B) 2
(B)  −1, −1, −1 (C)  ∞ (D) 0
(C)  −1, 1 − 2ω, 1 − 2ω2
133. Let α, β be real numbers and z a complex number. If
(D)  −1, 1 + 2ω, 1 + 2ω2
z2 + α z+ β = 0 has two distinct roots on the line
125. If z1 and z2 are two non-zero complex numbers such Re(z) = 1, then it is necessary that [2011]
that | z1 + z2 | = | z1 | + | z2 | then argz1− argz2 is equal to
(A)  β ∈(−1, 0) (B)  | β | = 1
 [2005]
π (C)  β ∈(1,∞) (D)  β ∈(0, 1)
(A)  (B)  −π
2 134. If ω (≠ 1) is a cube root of unity, and (1 + ω)7 = A +
π Bω. Then (A, B) equals [2011]
(C) 0 (D)  −
2 (A)  (1, 1) (B)  (1, 0)
z (C)  (–1, 1) (D)  (0, 1)
126. If w = and | w | = 1, then z lies on [2005]
1
z− i z2
3 135. If z ≠ 1 and is real, then the point which is rep-
z −1
(A)  an ellipse (B)  a circle
resented by the complex number z lies [2012]
(C)  a straight line (D)  a parabola
(A) either on the real axis or on a circle passing

10
2k π 2k π ⎞ through the origin
127. The value of ∑ ⎜ sin + i cos ⎟ is (B)  on a circle with centre at the origin
⎝ 11 11 ⎠
 k =1 [2006] (C) either on the real axis or on a circle not passing
(A)  i (B) 1 through the origin
(C)  –1 (D)  −i (D)  on the imaginary axis
2
128. If z + z + 1 = 0, where z is a complex number, then 136. If z is a complex number of unit modulus and argu-
the value of

PRACTICE EXERCISES
⎛ 1+ z ⎞
ment θ , then ⎜ equals [2013]
⎛ 1⎞ ⎛
2
1⎞ ⎛
2
1⎞ ⎛
2
1⎞
2 ⎝ 1 + z ⎟⎠
⎜ z + ⎟ + ⎜ z 2 + 2 ⎟ + ⎜ z 3 + 3 ⎟ + ... + ⎜ z 6 + 6 ⎟ π
⎝ z⎠ ⎝ z ⎠ ⎝ z ⎠ ⎝ z ⎠ (A)  − θ (B)  θ
is [2006] 2
(A) 18 (B) 54 (C)  π − θ (D)  −θ
(C) 6 (D) 12
137. If z is a complex number such that z ≥ 2 , then the
129. If | z + 4 | ≤ 3, then the maximum value of |z + 1| is
1
 [2007] minimum value of z +  [2014]
(A) 4 (B) 10 2
(C) 6 (D) 0 5
(A)  is equal to
1 2
130. The conjugate of a complex number is Then the
complex number is i − 1 [2008] (B)  lies in the interval (1, 2)

−1 1 5
(A)  (B)  (C)  is strictly greater than
i −1 i +1 2
−1 1
(C)  (D)  3 5
i +1 i +1 (D)  is strictly greater than but less than
2 2
2.30  Chapter 2

138. A complex number z is said to be unimodular if ⎛ 1 ⎞ π


| z | = 1. Suppose z1 and z2 are complex numbers such (A) sin −1 ⎜ ⎟ (B) 
⎝ 3⎠ 3
z − 2 z2
that 1 is unimodular and z2 is not unimodular. ⎛ 3⎞
2 − z1 z2 π
sin −1 ⎜ ⎟
(C) (D) 
Then the point z1 lies on a [2015] 6 ⎝ 4 ⎠
(A)  straight line parallel to y-axis.
5 5
(B)  circle of radius 2.    
140. Let z =  3 + i  +  3 − i  . If R(z) and I(z)
(C)  circle of radius 2.  2 2  2 2
   
(D)  straight line parallel to x-axis.
respectively denote the real and imaginary parts of z,
then: [2019]
2 + 3i sin θ
139. A value of θ for which is purely imagi- (A) R(z) = – 3
1 − 2i sin θ
(B) R(I) < 0 and I(z) > 0
nary, is [2016]
(C) I(z) = 0
(D) R(z) > 0 and I(z) > 0

ANSWER K EYS
Single Option Correct Type
  1. (A) 2. (D) 3. (D) 4.  (B) 5. (B) 6.  (D) 7. (C) 8. (A) 9. (C)  10.  (D)
  11. (D) 12.  (C) 13. (B) 14.  (B) 15. (C) 16.  (B) 17. (C) 18.  (D) 19. (B)  20.  (B)
  21. (C) 22.  (B) 23. (B) 24. (B) 25.  (C) 26. (A) 27.  (C) 28. (A) 29.  (C)   30. (B)
  31. (B) 32. (C) 33.  (C) 34. (A) 35.  (C) 36. (C) 37.  (B) 38. (C)  39. (A) 40.  (A)
  41.  (B) 42. (C) 43.  (C) 44. (A) 45. (C) 46. (C) 47.  (B) 48. (B)  49.  (B) 50.  (B)
  51. (C) 52. (A) 53.  (B)   54. (A) 55.  (C) 56. (B) 57.  (A)  58. (C) 59. (A) 60. (C)
  61.  (D) 62. (A) 63.  (C) 64.  (B) 65. (A) 66.  (B) 67. (B)  68.  (A) 69. (C) 70.  (A)
  71 (B) 72. (D) 73. (C) 74.  (C) 75. (C) 76.  (A) 77. (B)  78.  (A) 79. (A) 80.  (B)
  81. (D) 82.  (B) 83. (A) 84.  (B) 85. (C) 86.  (B) 87. (C)  88. (A) 89.  (D) 90. (C)
  91.  (B) 92. (C) 93.  (A) 94. (D) 95. (B) 96.  (C) 97. (B) 98.  (B) 99. (B) 100.  (B)
101. (A) 102.  (C) 103. (B) 104.  (B) 105. (D) 106.  (B) 107. (C) 108. (B) 109. (A) 110. (D)
PRACTICE EXERCISES

111.  (A) 112. (B) 113.  (A) 114. (A) 115.  (C) 116. (B)

Previous Years’ Questions


1 17. (D) 118. (C) 119.  (D) or (C) 120. (A) 121. (C) 122. (B) 123. (D) 124. (C) 125. (C)
126. (C) 127. (D) 128. (D) 129. (C) 130. (C) 131. (B) 132. (A) 133. (C) 134. (A)
135. (A) 136. (B) 137. (B) 138. (B) 139. (A) 140. (C)
Complex Numbers  2.31

HINTS AND EXPLANATIONS

Single Option Correct Type


p q r
1. We have, + + =1+i \ x2 + 4x + 4 = 0
a b c
⇒ (x + 2)2 = 0
2
⎛ p q r⎞ or x = –2
⇒ ⎜ + + ⎟ = 1 – 1 + 2i = 2i

⎝ a b c⎠ \ z = –2 – i.
The correct option is (B)
p2 q2 r2 ⎛ qr rp pq ⎞

+ + + 2⎜ + + = 2i z
a 2
b 2
c 2⎝ bc ca ab ⎠⎟ 6. If 1 = z, the given equation becomes
z2
p 2
q2 r2 2 pqr ⎛ a b c ⎞ z2 – z + 1 = 0 i.e., z = – w and – w2

+ + + + + = 2i z
a 2
b 2
c 2
abc ⎜⎝ p q r ⎟⎠ or, 1 = – w ⇒ z1 = – z2w
z2
p 2
q 2
r2 OB = |z2 – 0| = |z2|

+= 2i +
a2 b2 c2 OA = |z1 – 0| = |– z2w | = |z2| |–w | = |z2|

The correct option is (A) and AB = |z2 – z1| = |z2 + z2w |
= |z2| |1 + w | = |z2| |–w2| = |z2|
2. sin x + i cos 2x and cos x – i sin 2x are conjugate of each
Thus z1, z2 and origin form an equilateral triangle.
other
The correct option is (D)
if sin x + i cos 2x = cos x − i sin 2 x

7. Let a = r cos q and b = r sin q
⇒ sin x + i cos 2x = cos x + i sin 2x
b
\ tan q =
⇒ sin x = cos x and cos 2x = sin 2x
a
⇒ tan x = 1 and tan 2x = 1,
a − ib r (cos θ − i sin θ )
Now = = (cos q – i sinq)2

which is not possible for any value of x. a + ib r (cos θ + i sin θ )


The correct option is (D)
= cos 2q – i sin 2q = e– 2iq
a − ib
= i log e– 2iq = i (– 2 iq) = 2q

HINTS AND EXPLANATIONS


3. Let z = x + iy, then \ i log

a + ib b
z2 + |z|2 = 0 ⇒ (x + iy)2 + |x + iy|2 = 0
2
⎡ a − ib ⎤ 2 tan θ a
\ tan ⎢i log
= tan 2q = =
⇒ x2 – y2 + 2ixy + x2 + y2 = 0
⎣ a + ib ⎥⎦ 1 − tan 2 θ b2
1− 2
⇒ 2x2 + 2ixy = 0
2ab a

= 2
⇒ 2x2 = 0 and 2xy = 0
a −b 2

⇒ x = 0 and xy = 0

The correct option is (C)

Clearly y can be any real number. Hence, we will get 8. Since ( 3 + i)100 = 299 (a + ib)
infinitely many solutions.
\ ( 3 – i)100 = 299 (a – ib)


The correct option is (D)
\ ( 3 + i)100 – ( 3 – i)100 = 299 (2ib) = 2100 (ib)

n
4. As w is the nth root of unity so, w – 1 = 0 ⇒ i100 [1 – 3 i]100 – i100 [–1 +
3 i]100 = 2100 (ib)
⇒ (w – 1) (1 + w + w2 +, …, + wn – 1) = 0

or (– 2w)100 – (2w2)100 = 2100 (ib)

Hence, 1 + w + w2 +, …, + wn – 1 = 0

or w – w2 = (ib) or
3 i = ib
or w – 1 = 0 i.e., w = 1

\ b = 3


The correct option is (B)

The correct option is (A)
5. Since z + 2 |z + 1| + i = 0
9. We have,
\ x + i (y + 1) + 2  |x + iy + 1| = 0
\ y + 1 = 0 ( |x + iy + 1| is real) 1 − i sin α (1 − i sin α ) (1 − 2i sin α )
=
\ y = – 1 1 + 2i sin α (1 + 2i sin α ) (1 − 2i sin α )
\ x + 2 |x – i + 1| = 0 1 − 3i sin α − 2 sin 2 α
⇒ x2 = 2 [(x + 1)2 + 1] = 2 (x2 + 2x + 2)
=
1 − 4i 2 sin 2 α
2.32  Chapter 2

(1 − 2 sin 2 α ) − 3i sin α x = 55/6 (cos 5q + i sin 5q)1/6



=
,
1 + 4 2 sin α ⎡ ⎛ 2kπ + 5θ ⎞ ⎛ 2kπ + 5θ ⎞ ⎤
which is purely real if sin a = 0, i.e., if a = np, where n is an
= 55/6 ⎢cos ⎜⎝
⎟⎠ + i sin ⎜⎝ ⎟⎠ ⎥
⎣ 6 6 ⎦
integer.

The correct option is (C) x1x2 … x6 = 55 [cos (5p + 5q) + i sin (5p + 5q)]


= 55 (– cos 5q – i sin 5q)
10. By the given condition, |z – 1| < |z – i|

= – 55(cos 5q + i sin 5q)
⇒ |x – 1 + iy| < |x + i (y – 1)|

The correct option is (B)
⇒ (x – 1)2 + y2 < x2 + (y – 1)2
⇒ –2x < – 2y ⇒ x > y ⇒ x – y > 0 15. Given |z1 + z2| = |z1| + |z2|
The correct option is (D) Squaring, |z1 + z2|2 = (|z1| + |z2|)2

11. Perimeter = 6 × distance of vertex 1 + 2i ⇒ |z1|2 + |z2|2 + 2Re |z1| |z2| = |z1|2 + |z2|2 + 2 |z1| |z2|

 from the centre (0, 0). ⇒ 2Re |z1| | z2 | = 2 |z1| |z2|

= 6 [(1 – 0) + i (2 – 0)] = 6 5 ⇒ |z1| | z2 | cos (q1 – q2) = |z1| |z2|

The correct option is (D)
⇒ cos (q1 – q2) = 1

12. Let z1 = x1 + iy1 ⇒ q1 – q2 = 0  or  q1 = q2

z2 = x2 + iy2 or arg z1 = arg z2

z3 = x3 + iy3
The correct option is (C)
z2 z3 = (x2 – iy2) (x3 + iy3)
= (x2x3 + y2y3) + i (x2y3 – x3y2) 16. For every a ∈ R, |a| = a2
⇒ Im ( z2 z3 ) = x2y3 – x3y2 \ |a|2 = a2

\ z1 Im ( z2 z3 ) = (x1 + iy1) (x2y3 – x3y2)
Now (|x| – |y|)2 ≥ 0

Similarly
⇒ |x|2 + |y|2 – 2 |x| |y| ≥ 0

z2 Im ( z3 z1 ) = (x2 + iy2) (x3y1 – x1y3)
⇒ 2 |x| |y| ≤ |x|2 + |y|2

z3 Im ( z1z2 ) = (x3 + iy3) (x1y2 – x2y1)
Adding we get the result = 0 ⇒ |x|2 + |y|2 + 2 |x| |y| ≤ 2 |x|2 + 2 |y|2

The correct option is (C) ⇒ (|x| + |y|)2 ≤ 2 (x2 + y2)

HINTS AND EXPLANATIONS

⇒ (|x| + |y|)2 ≤ 2 |z|2



2 z1
13. ∵ is purely an imaginary number, \ |x| + |y| ≤ 2 |z|.

3 z2

The correct option is (B)
2z z 3
\ let 1 = ib  or  1 =
ib (1)
3 z2 z2 2 17. We have, |1 + i|2 × |1 + 2i|2 × |1 + 3i|2 … |1 + ni|2
4 4 = |a + ib |2
z1 3
4 −1 ib − 1 ⇒ (1 + 1) × (1 + 4) × (1 + 9) … (1 + n2) = a2 + b2
z1 − z2 z2
\
= = 2 ⇒ 2 × 5 × 10 … (1 + n2) = a 2 + b 2.
z1 + z2 z1 3
+1 ib + 1 The correct option is (C)
z2 2
4 18. |z1| = 1, |z2| = 1
4 ⎛ 9b 2 ⎞
3
ib − 1 ⎜ + 1⎟ ⇒ a2 + b2 = 1
(1)
2 ⎜⎝ 4 ⎟⎠

= = =1 p2 + q2 = 1
(2)
4 4
3 ⎛ 9b 2 ⎞ Im ( z1 z2 ) = Im (a + ib) (p – iq) = bp – aq

ib + 1
2 ⎜ + 1⎟
⎜⎝ 4 ⎟⎠ \ bp – aq = 1
(3)

The correct option is (B) ω1ω 2 = (a + ip) (b – iq) = (ab + pq) + i (bp – aq)

= (ab + pq) + i (1)
14. x6 = (4 – 3i)5
⎛ 4 3i ⎞
\ Im ω1ω 2 = 1
( )
x6 = 56 ⎜ − ⎟ = 55 (cos q + i sin q)5

The correct option is (D)
⎝5 5⎠
3
⎛ 3⎞ 19. We have, a + ib = x + iy
where  q = –tan–1 ⎜ ⎟ = 55 (cos 5q + i sin 5q)

⎝ 4⎠ a + ib = (x + iy)3 = (x3 – 3xy2) + i (3x2y – y3)

Complex Numbers  2.33

\ a = x3 – 3xy2 and b = 3x2y – y3 25. We have, z4 + 1 = 0 ⇒ z4 = –1


a b ⇒ z = (cos p + i sin p)1/4
\ = x2 – 3y2 and = 3x2 – y2
x y 1 1
⇒ z = cos (2kp + p) + i sin (2kp + p), k = 0, 1, 2, 3.
a b 4 4
\ + = x2 – 3y2 + 3x2 – y2 = 4 (x2 – y2)
x y π π 3π 3π
⇒ z = cos
+ i sin , cos + i sin ,
The correct option is (B) 4 4 4 4
20. As (a – b)2 ≥ 0, a2 + b2 ≥ 2ab(1)
5π 5π 7π 7π

cos + i sin , cos + i sin
But |z| =
a 2 + b 2 ; so from Eq. (1), |z|2 ≥ 2ab 4 4 4 4
\ |z|2 + a2 + b2 ≥ a2 + b2 + 2ab
=
1
(1 + i),
1
(– 1 + i),
1
(– 1 – i),
1
(1 – i).
or |z|2 + |z|2 ≥ (a + b)2;
2 2 2 2
\ 2 |z|2 ≥ (a + b)2
1
Hence, the four roots of z4 + 1 = 0 are (±1 ± i).
2
\
2 |z| ≥ a + b as |z| is positive. The correct option is (C)
1 26. Put 1 = r cos q and – 1 = r sin q
\ |z| ≥
(a + b) π
2 ⇒ r = 2 and q = –

The correct option is (B) 4
Then given equation takes the form
z1 − z3 1 − i 3 z3 n
21. = ⎡ ⎛ π⎞ ⎛ π⎞⎤
z2 − z3 2 ( 2 )n ⎢cos ⎜ − ⎟ + i sin ⎜ − ⎟ ⎥ = 2n

π ⎣ ⎝ 4 ⎠ ⎝ 4⎠⎦
⎛ −π ⎞ ⎛ −π ⎞ 3

= cos ⎜ + sin ⎜ = e– i p/3 nπ nπ ⎤
⎝ 3 ⎟⎠ ⎝ 3 ⎟⎠ ⎡
or 2n/2 ⎢cos
− i sin = 2n
⎣ 4 4 ⎥⎦
z1 − z3
\
= e − i π /3 = 1 z1 z2
Equating real and imaginary parts, we get
z2 − z3 nπ nπ
π
cos = 2n/2 and – sin =0
and angle between z1 – z3 and z2 – z3 is . 4 4
3 These are satisfied only for n = 0.

\ triangle is equilateral.
The correct option is (C) Hence, n = 0 is the only solution.

22. Putting x = 1, w, w2 respectively,
The correct option is (A)
3n = a0 + a1 + a2 + … + a2n 4 4

HINTS AND EXPLANATIONS


27. We have, 2 = z − ≥ |z| –
(1 + w + w2)n = a0 + a1 w + a2 w2 + … + a2n w2n z z
(1 + w2 + w4)n = a0 + a1 w2 + a2 w4 + … + a2n w4n 4
⇒ |z| –
≤2
Adding these, 3n + (1 + w + w2)n + (1 + w2 + w4)n z
= 3a0 + a1 (1 + w + w2) + a2 (1 + w2 + w4) ⇒ |z|2 – 2 |z| – 4 ≤ 0 or (|z| – 1)2 – 5 ≤ 0

+ a3 (1 + w3 + w6) + … ⇒ (|z| – 1)2 ≤ 5 or |z| – 1 ≤ ⇒ |z| ≤ + 1

\ 3n + 0n + 0n = 3a0 + 3a3 + 3a6 + …

Hence the greatest value of |z| is 5 + 1.
\ 3n – 1 = a0 + a3 + a6 + …

The correct option is (C)
The correct option is (B)
π
23. Let n 1 = x; \ xn = 1; \ xn – 1 = 0 28. We have, Arg (z + i) – Arg (z – i) =
2
\ xn – 1 = (x – 1) (x – a1) (x – a2) … (x – an – 1)
⎛ z + i⎞ π
\ (x – a1) (x – a2) (x – a3) … (x – an – 1) ⇒ Arg ⎜
=
⎝ z − i ⎟⎠ 2
xn − 1 1 − xn
=
= = 1 + x + x2 +, …, + xn – 1. ⎛ z + i⎞
x −1 1− x \ Re ⎜
=0
⎝ z − i ⎟⎠
Putting x = 1, we get
(1 – a1) (1 – a2) (1 – a3) … (1 – an – 1) = n. ⎛ z + i⎞ ⎛ z + i⎞
⎜⎝ ⎟ +⎜ ⎟
The correct option is (B)

z − i⎠ ⎝ z − i⎠ = 0 ⇒ ⎛ z + i⎞ + ⎛ z + i ⎞ = 0
⎜⎝ ⎟
24. The closest distance = length of the perpendicular from the 2 z − i ⎠ ⎜⎝ z − i ⎟⎠
origin on the line a z + a z + a a = 0 ⇒ (z + i) ( z + i) + (z – i) ( z – i) = 0

a( 0 ) + a | 0 | + a a | a |2 | a| ⇒ z z + i (z + z ) – 1 + z z – i (z + z ) – 1 = 0

=
= =
2| a | 2| a | 2 ⇒ 2 (z z ) = 2


The correct option is (B) ⇒ z z = 1 or |z|2 = 1

2.34  Chapter 2

⇒ |z| = 1
π

The equation represents a circle centred at origin and radius \ |amp z1 – amp z2| =
.
2
1 unit. The correct option is (B)

The correct option is (A) 32. We have,
29. Let z = x + iy
⎛ | z − 1| + 4 ⎞ ⎛ 1⎞
z2 log1/ 2 ⎜ > 1 = log1/ 2 ⎜ ⎟

Since is real = k (say), where k ∈ R ⎝ 3 | z − 1| − 2 ⎟⎠ ⎝ 2⎠
z −1
| z − 1| + 4 1
( x + iy ) 2 ⇒
< <1
\
=k 3 | z − 1| − 2 2
x + iy − 1
( loga x is a decreasing function if a < 1)

⇒ x2 + 2i xy – y2 = k (x – 1) + i ky

⇒ |z – 1| + 4 < 3 |z – 1| – 2

⇒ x2 – y2 = k (x – 1)
(1)
⇒ 2 |z – 1| > 6 ⇒ |z – 1| > 3

2xy = ky
(2)
Eq. (2) gives either y = 0 or k = 2x

which is an exterior of a circle.
If k = 2x, then x2 – y2 = 2x2 – 2x

The correct option is (C)
⇒ x2 + y2 – 2x = 0, which is a circle.

33. Suppose x is a real root
Thus, lies either on the real axis y = 0 or on a circle.

Then x3 + ix – 1 = 0 ⇒ x3 – 1 = 0 and x = 0


The correct option is (C)

There is no real number satisfying these two equations.
z − 2 z2
The correct option is (C)
30. Clearly 1 =1
2 − z1z2
34. Let a, b, g  be the roots
⎛ z − 2 z2 ⎞ ⎛ z1 − 2 z2 ⎞ \ a + b + g  = –a
⇒ ⎜ 1
=1
⎝ 2 − z1z2 ⎟⎠ ⎜⎝ 2 − z1z2 ⎟⎠ \ |–a| = |a + b + g | ≤ |a| + |b | + |g | = 1 + 1 + 1
\ |a| ≤ 3
⇒ z1z1 − 2 z1z2 − 2 z1z2 + 4 z2 z2

The correct option is (A)
= 4 − 2 z1z2 − 2 z1z2 + z1z2 z1z2

z1 z2
⇒ |z1|2 + 4 |z2|2 = 4 + |z1|2 |z2|2
35. We have, + =1
z2 z1
HINTS AND EXPLANATIONS

⇒ |z1|2 [1 – |z2|2] = 4 [1 – |z2|2]



⇒ z12 + z22 = z1 z2

⇒ |z1|2 = 4
( |z2| ≠ 1)
⇒ |z1| = 2
⇒ z12 + z22 + z32 = z1 z2 + z2 z3 + z3 z1,


The correct option is (B) where z3 = 0.

⇒ z1, z2 and the origin form an equilateral triangle.

31. Let z1 = x1 + iy1 and z2 = x2 + iy2

The correct option is (C)
Now z1 + z2 = (x1 + x2) + i (y1 + y2)
and z1 – z2 = (x1 – x2) + i (y1 – y2) 1 i 2n + 1
36. We have, S (n) = in + i–n = in + =
As |z1 + z2| = |z1 – z2|, we get i n in
( −1) n + 1
(x1 + x2)2 + (y1 + y2)2 = (x1 – x2)2 + (y1 – y2)2
= , n = 1, 2, 3, 4, …
in
or, x1 x2 + y1 y2 = 0 (1)
y1 \ values of S (n) are 0, – 2, 2, 0, – 2, 2, …

y
Now amp z1 – amp z2 = tan– 1 x – tan– 1 2 \ Total number of distinct values of S (n) is 3.

1 x 2
The correct option is (C)
y1 y2

x1 x2 z1 + z2

= tan– 1 37. We have, |z1 + z2| =
1 1
+ =
y y
1+ 1 ⋅ 2 z1 z2 z1 z2
x1 x2
⎛ 1 ⎞
x2 y1 − y2 x1 ⇒ |z1 + z2| ⎜1 −
=0

= tan– 1 ⎝ | z1 z2 | ⎟⎠
x1 x2 + y1 y2
⇒ |z1 z2| = 1.( z1 ≠ –z2)


= tan–1 ∞, by (1)

The correct option is (B)
Complex Numbers  2.35

38. We have, amp (z – 1) = amp (z + 3i) 44. Midpoint of P and P′ is centre of circle C such that
y y+3 z + ( − z1 )
⇒ tan–1
= tan– 1 1 =0
x −1 x 2
⇒ xy = (x – 1) (y + 3)
\ Centre of circle lies at origin.
⇒ 3 (x – 1) = y
Now, the equation of circle with centre at origin and radius
|z1| or |–z1| is
\ (x – 1) : y = 1 : 3.

|z – 0| = |z1|

The correct option is (C)
⇒ |z|2 = |z1|2 P(–z1)
⇒ z · z = z1 · z1

39. Given: |z| = 2 and arg (z) = .
3 z z ⎛z ⎞
2π \
= 1 = ⎜ 1⎟ P(z1)
\ If z = r (cos q + i sinq), then r = 2 and q =
z1 z ⎝ z⎠
3

The correct option is (A)
⎛ 2π 2π ⎞ ⎛ 1 3⎞
\ z = 2 ⎜ cos
+ i sin ⎟⎠ = 2 ⎜ − + i ⎟ 45. We have
⎝ 3 3 ⎝ 2 2 ⎠ 2
⎛ p q r⎞
(1 + i)2 = ⎜ + + ⎟


= (–1 + i 3) ⎝ a b c⎠

The correct option is (A) p2 q2 r2 ⎛ qr rp pq ⎞
⇒ 1 – 1 + 2i =
+ + + 2⎜ + +
a 2
b 2
c2 ⎝ bc ca ab ⎟⎠
⎛ | z |2 − | z | +1⎞
40. log 3 ⎜ 2 + | z| ⎟ < 2 p2 q2 r2 2abc ⎛ a b c ⎞
⎝ ⎠ ⇒ 2i =
+ + + + +
a 2
b 2
c 2
pqr ⎜⎝ p q r ⎟⎠
| z |2 − | z | +1 2 2

< ( 3 )2 p q r2 2abc
2 + | z|
= 2
+ 2
+ 2
+ (0)
a b c pqr
⇒ |z|2 – |z| + 1 < 6 + 3 |z|
2 2 2
p q r
⇒ |z|2 – 4 |z| – 5 < 0 ⇒ (|z| – 5) (|z| + 1) < 0

2
+ = 2i 2
+
a b c2
⇒ |z| – 5 < 0, since |z| + 1 > 0 ⇒ |z| < 5


The correct option is (C)

The correct option is (A)

HINTS AND EXPLANATIONS


41. Let z1 = x1 + iy1 46. Interpretating according to Coni’s Theorem. Let the angle
between the lines joining
\
z1 = x1 – iy1 z1, z2 and z1, –z2 be a
Now z2 = z1 ⇒ z1 + z2 = z1 + z1
z −z
\ 1 2 = cos a + t sin a

= x1 + iy1 + x1 – iy1 z1 + z2
Using Componendo and Dividendo, we have

= 2x1, which is real.
Hence, result holds if z2 = z1 .
2z 1 + cos α + i sin α
1 =

The correct option is (B) −2 z2 cos α − 1 + i sin α

42. |z – i| = |z + 5i| represents equation of perpendicular bisector ⎛α⎞ α α


2 cos 2 ⎜ ⎟ + i 2 sin cos
of points (0, 1) and (0, –5) i.e., y = –2, now |z| = 2 is x2 + z ⎝ 2⎠ 2 2
⇒ 1 =

y2 = 4 z2 ⎛ α ⎞ α α
−2 sin 2 ⎜ ⎟ + i sin cos
⇒ x2 + 4 = 4 ⇒ x = 0 ⎝ 2⎠ 2 2
\ z represents a single point (0, –2). z1 α

= i cot
The correct option is (C) z2 2
α
43. If a complex number z is rotated through an angle 180º, then ⇒ iz1 = –cot
z2
2
it’s new position is –z. So, 2 – 4i is –2 + 4i and stretching it 5/2 ⎡ α ⎤
But iz1 = kz2 

times means modulus 5/2 times of previous complex number ⎢ where, k = − cot 2 (say)⎦⎥

α
Now, k = –cot

5 2

i.e., (–2 + 4i) = –5 + 10i α 2k
2 ⇒ cot = –k ⇒ tan a = 2


The correct option is (C) 2 k −1
2.36  Chapter 2

Since A · M > G · M for |z1| ≠ |z2|



2k
The correct option is (B)
⇒ tan a = –

1− k2 50. If z is a root of (z – 1)25 = 2w2(z + 1)25, then
⎛ 2k ⎞ ⎛ z − 1⎞
25
z −1
25
⇒ a = tan–1 ⎜⎝ − 1 − k 2 ⎟⎠ = –2 tan–1 k
⎜⎝ ⎟ = 2w2 ⇒ = 2 |w2| = 2
z + 1⎠ z +1
z1 − z2 z −1
Now,
= cos a + i sin a ⇒
= 21/25
z1 + z2 z +1
where a is the angle between (z1 – z2) and (z1 + z2).

As 21/25 ≠ 1, we get z lies on a circle.
⇒ a = –2tan–1k is the angle between (z1 – z2) and (z1 + z2).

The correct option is (B)

The correct option is (C)
1 3
51. We have (A0  A1)2 = + = 1
47. x2 – 3x + 1 = 0 4 4
⇒ A0A1 = 1
3 ± 3−4 3 i π π
⇒ x =
= ± = cos + i sin ⎛ 3⎞ 2 2
2 2 2 6 6 ⎛ 3⎞ 9+3
A0A1 = ⎜ ⎟ + ⎜ ⎟ =
=3
2 ⎝ 2⎠ ⎝ 2 ⎠ 4
⎛ n 1⎞ ⎛ 1 ⎞ nπ
⎜⎝ − n ⎟⎠
x = ⎜ x 2 n + 2 n − 2⎟ = –2 + 2 cos
x ⎝ x ⎠ 3 Imaginary
2
axis
24 24
⎛ 1⎞ ⎛ nπ ⎞

∑ ⎜⎝ x n − x n ⎟⎠ = ∑ ⎜⎝ −2 + 2 cos 3⎠
⎟ –1 , 3 1, 3
n =1 n =1 A2 A1
2 2 2 2
⎡ π 2π 24π ⎤
= –48 + 2 ⎢cos + cos + ... + cos
⎣ 3 3 3 ⎥⎦
⎡ ⎛ 2π 23π ⎞ ⎛ 24π ⎞ ⎤ A3 A0 Real
+2 ⎢cos ⎜ + ⎟⎠ .sin ⎜⎝ ⎟
⎣ ⎝ 3 6 6 ⎠ ⎥⎦ (–1, 0) (1, 0) axis
= – 48
π
sin
6
= 0 – 48 = – 48 –1 3 –1 3
A4 A5

The correct option is (B) 2 2 2 2
48. C1(0, 0) is centre of bigger circle and C2 (3, 4) is centre of
HINTS AND EXPLANATIONS

smaller circle ⇒ A0A2 = 3



C1B = r1 = 12 (radius of bigger circle) Similarly, A0A4 = 3

C2A = r2 = 5 (radius of smaller circle) Thus, (A0A1) (A0A2) (A0A4) = 3

C1C2 = 5, minimum value of |z1 – z2| = AB
The correct option is (C)
b
52. z1 + z2 = – (1)
a
c
z1z2 = (2)
a
C1 z2 = iz1(3)
From Eq. (1) and (2)
C2
−b
A B z1(1 + i) =

a
−b b2 −b 2
⇒ z1 =
(1 – i) ⇒ z12 = (–2i) = i
Now, C1C2 + C2A + AB = C1B
2a 4a 2
2a 2
⇒ AB = 12 – 5 – 5 = 2

From Eq. (2) and (3)

The correct option is (B) c −c −b 2 c b2
2 2 z12 = = i⇒ 2 i= i⇒a=
49. 2 z1 + i 3 z2 + 3 z1 + i 2 z2 ai a 2a a 2c
The correct option is (A)
= ( 2z + i 1 3 z2)( 2 z1 − i 3 z2 ) z −1
+ ( 3z +i 2 z )( 2z )
53. should be real
1 2 3 z1 − i 2 i
x + iy − 1
= 5 (|z1|2 + |z2|2) > 5 · 2 2 2
= 10 |z1 z2| i.e., = y – i (x – 1) is real
z1 z2 i
Complex Numbers  2.37

⇒ x – 1 = 0 i.e., x = 1
z −1
( )
58. z z − 2i = zz + 2iz = 2 (2 + i) gives
\ sin–1 = sin–1 y 2 2
i x + y – 2y = 4 and 2x = 2,

\ –1 ≤ y ≤ 1
on equating the real and imaginary parts.
The correct option is (B) \ y2 – 2y – 3 = 0 giving y = 3, –1

54. Since, x2 – x + 1 = 0 (given) The solutions are 1 + 3i and 1 – i.

\ Solving for x, we have x = –w and x = –w2
The correct option is (C)
Case I:  x = w
z2 z2
⎛ 5
1 ⎞
2 59. =
\ S = ∑ ⎜ ω n + n ⎟
z −1 z −1
n =1
⎝ ω ⎠
⇒ zzz − z 2 = zz z − z 2
( )
5 2
⇒ S = ∑ ω n + ω 2 n
⇒ | z |2 ( z − z ) − ( z − z )( z + z ) = 0
n =1 ⇒ ( z − z )[| z |2 − ( z + z )] = 0
⇒ S = (–1)2 + (–1)2 + 22 + (–1)2 + (–1)2(\ S = 8) Either z = z ⇒ real axis
Case II:  x = w2 or |z|2 = z + z ⇒ zz − z − z = 0
2
5 ⎡
⎛ 1 ⎞ ⎤
n represents a circle passing through origin.
\ S = ∑ ⎢ω 2 n + ⎜ 2 ⎟ ⎥

⎝ω ⎠ ⎥ The correct option is (A)
n =1 ⎢
⎣ ⎦
60. C1: |z – i| = 2 ⇒ x2 + (y – 1)2 = 22
( )
5 2
⇒ S = ∑ ω 2 n + ω n
=8 C2: |z – 1 – 2i| = 4 ⇒ (x – 1)2 +(y – 2)2 = 42
n =1 C1C2 = 2
The correct option is (A) r1 = 2, r2 = 4
55. Points A, B and C are at equal distance from origin O and as ⇒ C1C2 < |r1 – r2|
∠ BOA = 45º clearly BC = AB. So, triangle is isoceles. ⇒ one circle lies inside the other. Now point (2, 2) lies
Imaginary outside circle C1 and inside circle C2.
axis ⇒ C1 lies inside C2
The correct option is (C)
C (0, 1) 1, 1
B
2 2
61. We have, z1(z21 – 3z22) = 2 (1)

HINTS AND EXPLANATIONS


2 2
and, z2(3z 1 – z 2) = 11
(2)

Multiplying (2) by i and adding it to (1), we get
45°
Real axis z31 – 3z22z1 + i(3z21z2 – z32) = 2 + 11i

O A (1, 0)
⇒ (z1 + iz2)3 = 2 + 11i(3)

The correct option is (C)
Again, multiplying (2) by i and subtracting it from (1), we
56. Let a be a real root of the given equation. get
Then, a2 + (a + ib) a + c + id = 0 z31 – 3z22z1 – i(3z21z2 – z32) = 2 – 11i

⇒ a2 + aa + c = 0 and ba + d = 0 ⇒ (z1 – iz2)3 = 2 – 11i(4)

2
Multiplying (3) and (4), we get
⎛ d⎞ ⎛ b⎞
⇒ ⎜ − ⎟ + a ⎜ − ⎟ + c = 0

⎝ b⎠ ⎝ d⎠ (z21 + z22)3 = 4 + 121 ⇒ z21 + z22 = 5.

d2 ad
The correct option is (D)

− +c=0
b2 b
or d2 – abd + b2c = 0
62. We have, 1 − c 2 = nc – 1
The correct option is (B) ⇒ 1 – c2 = n2c2 + 1 – 2nc

57. |iz + z0| = |iz – i2 + z0 – 1| c 1
= |i (z – i) + 5 + 3i – 1| ⇒
= (1)
2n 1 + n2
= |i (z – i) + (4 + 3i)|
c ⎛ n⎞
\ |iz + z0| ≤ |i (z – i)| + |4 + 3i|
Now, (1 + nz ) ⎜1 + ⎟
2n ⎝ z⎠
≤ 1.2 + 5 = 7
\ Maximum value of |iz + z0| is 7 1 ⎧ 2 ⎛ 1⎞ ⎫
= ⎨1 + n + n⎜ z + ⎟ ⎬
The correct option is (A) 1 + n2 ⎩ ⎝ z⎠⎭
2.38  Chapter 2


= (1 + a)2 – c2 – b2 = 1 + a2 + 2a – (1 – a2)
=
1
1 + n2
(
1 + n2 + n( 2 cos θ ) ) = 2a2 + 2a = 2a (1 + a)
and, Im(z0) = 2b (1 + a).
⎛ 2n ⎞
= 1+ ⎜ ⎟ cos θ = 1 + ccosq (Using (1)) Thus, z0 = 2 (1 + a) (a + ib)
⎝ 1 + n2 ⎠
Also, denominator of (1) = 1 + a2 + c2 + 2a + 2c + 2ac + b2

The correct option is (A) = 2 + 2a + 2c + 2ac
1 − ak + 1
63. We have, zk = 1 + a + a2 + … ak = = 2 (1 + a) (1 + c)
1− a
1 + iz a + ib
1 −ak + 1 Therefore, =
⇒ zk –
= 1 − iz 1+ c
1− a 1− a
The correct option is (B)
1 | a |k + 1 1 67. We have,
⇒ zk − = < ( |a| < 1)
( )

1− a |1 − a | |1 − a | |z1 + z2 wk|2 = ( z1 + z2 w k ) z1 + z2 w k
Therefore, vertices z1, z2, …, zn of the polygon lie within the
circle. = ( z1 + z2 w k ) ( z1 + z2 w − k )
1 1
z − = ⎡⎣ w k = ei ( 2π k / n) ⇒ w k = e − i ( 2π k / n) = w − k ⎤
1− a |1 − a | ⎦⎥

The correct option is (C)
= | z1 |2 + | z2 |2 + z1 z2 w k + z1z2 w − k
64. We have,
Therefore, we have,
|3| = |a1z3 + a2z2 + a3z + a4|
n −1 n −1
⇒ 3 ≤ |a1z3| + |a2z2| + |a3z| + |a4|
∑ | z1 + z2 w k |2 = n (| z1 |2 + | z2 |2 ) + z1z2 ∑ wk
k =0 k =0
⇒ 3 ≤ |a1| |z3| + |a2| |z2| + |a3| |z| + |a4|

n −1
⇒ 3 ≤ |z|3 + |z|2 + |z| + 1 ( |ai| ≤ 1)

2 3 2
+ z1z2 ∑ w −k
⇒ 3 < 1 + |z| + |z| + |z| < 1 + |z| + |z| + … ∞
k =0

1 1 ⎡ n −1 n −1 ⎤
⇒ 3 <
⇒ 1 – |z| <
1−| z| 3 = n (| z1 |2 + | z2 |2 ) ⎢ ∑ w k = 0 = ∑ w − k ⎥ .
2 ⎣k =0 k =0 ⎦
\ |z| >

3 The correct option is (B)
HINTS AND EXPLANATIONS


The correct option is (B) 68. We have, |z – z1|2 + |z – z2|2 = k
4 4
65. We have, z = (z – 1)
⇒ 2 | z |2 + | z1 |2 + | z2 |2 − 2 Re( z z1 ) − 2 Re( z z2 ) = k
2 nπ i
⎛ z − 1⎞
⇒ ⎜
= 11/4 = e 4 , n = 0, 1, 2, 3. ⇒ 2 | z |2 − 2 Re { z ( z1 + z2 )} = k − (| z1 |2 + | z2 |2 )
⎝ z ⎠⎟

Since for all these values of z, 1
⇒ | z |2 − Re { z ( z1 + z2 )} = ( k − | z1 |2 − | z2 |2 )
z −1 2
= 1 so they lie on the line bisecting perpendicularly
z z1 + z2
2
1 1
the join of z = 1 and z = 0. ⇒ z− − | z1 + z2 |2 = ( k − | z1 |2 − | z2 |2 )
2 4 2
The correct option is (A)
2
z1 + z2
66. We have,
b + ic
⇒ z−
2
=
1
2
k−
1
4
{
| z1 |2 + | z2 |2 − 2 Re( z1 z2 )}
1+ i
1 + iz 1 + a = 1 + a − c + ib
= 1 1
1 − iz b + ic 1 + a + c − ib = k − | z1 − z2 |2
1− i 2 4
1+ a
z1 + z2 = 1 ( 2k − | z − z |2 )
2
[1 + a − c + ib] [1 + a + c + ib] ⇒ z− 1 2
= 2 4
[1 + a + c − ib] [1 + a + c + ib] z + z2
which will represent a real circle having centre at 1
z0 2
= (1)
(1 + a + c) 2 + b 2 1 1
and radius = 2k − | z1 − z2 |2 , provided k ≥ | z1 − z2 |2
Now,  R (z0) = (1 + a – c) (1 + a + c) – b2 2 2
The correct option is (A)
Complex Numbers  2.39

69. Let a z + b be the remainder when f (z) is divided by z2 + 1. iz = z


Then, we have ⇒ i(x + iy) – (x – iy) = 0 [Putting z = x + iy]
f (z) = (z2 + 1) g(z) + Az + B ⇒ i(x + y) – (x + y) = 0
Given: f (z) when divided by z – i gives remainder i ⇒ (i – 1) (x + y) = 0
⇒ f (i) = i which represents the line y = –x
⇒ (i2 + 1) g(i) + Ai + B = i Hence, reflection of the point (2, –1) in the line y = –x gives
⇒ Ai + B = i (1) the point (1, –2) which is equivalent to 1 – 2i in the argand
plane.
Also, f (z) when divided by z + i gives remainder i + 1
The correct option is (D)
⇒ f (– i) = i + 1
⇒ (i2 + 1) g(– i) – B = i + 1 1 3
73. We know that w = − + i
⇒ –Ai + B = i + 1 (2) 2 2
334 365
Solving equations (1) and (2), we get ⎛ 1 i 3⎞ ⎛ 1 i 3⎞
Thus, 4 + 5 ⎜ − + + 3 ⎜− +
A = i/2 and B = i + 1/2 ⎝ 2 2 ⎟⎠ ⎝ 2 2 ⎟⎠
⎛ i⎞ ⎛ 1⎞
\ remainder is ⎜ ⎟ z + ⎜ i + ⎟ = 4 + 5w334 + 3w365
⎝ 2⎠ ⎝ 2⎠
= 4 + 5 (w3)111 w + 3 (w3)123 w2
The correct option is (C)
= 4 + 5 (1)111 w + 3 (1)123 w2
70. We have,
= 4 + 5w + 3w2 = 1 + 2w + 3 (1 + w + w2)
xn – 1 = (x – 1) (x – w) (x – w2) … (x – wn–1)
= 1 + 2w + 3 (0) = 1 + 2w = 1 – 1 + 3 i = 3 i.
xn − 1
⇒ = (x – 1) (x – w2) … (x – wn–1) The correct option is (C)
x−w
74. The given line is
Putting x = w on both sides, we have
b z + b z = c(1)
xn − 1 ⎛ 0 ⎞
(w – 1) (w – w2) … (w – wn–1) = lim ⎜ ⎟ Let A (z1) be a reflection of
x→w x − w ⎝ 0 ⎠
B (z2) in the line (1).
nx n −1 Let P (z) be any point on the line
= lim = nwn–1
x→w 1 (1). We have,
The correct option is (A) AP = BP
71. We have, ⇒ |AP|2 = |BP|2

HINTS AND EXPLANATIONS


z – i = eia ⇒ z = i + eia
⇒ |z – z1|2 = |z – z2|2
= cos a + i (1 + sin a)
⇒ (z – z1) ( z – z1 ) = (z – z2) ( z – z2 )
⎛ 1 + sin α ⎞
\ q = arg (z) = tan −1 ⇒ ( z2 – z1 ) z + (z2 – z1) z + z1 z – z2 z2 = 0 (2)
⎜⎝ cos α ⎟⎠
Since (1) and (2) represent the same line, we get
1 + sin α b b c
⇒ tan q = = = = k (say)
cos α z2 − z1 z2 − z1 z1 z1 − z2 z2
2 cos α 2
\ cot θ − = −
z 1 + sin α cos α + i (1 + sin α ) ⇒ k ( z2 – z1 ) = b , k (z2 – z1) = b, k (z1 z1 – z2 z2 ) = c
cos α 2 [cos α − i (1 + sin α )] Now, z1 b + z2 b
= −
1 + sin α cos 2 α + (1 + sin α ) 2 = z1 {k (z2 – z1)} + z2 {k ( z2 – z1 )}
cos α 2 [cos α − i (1 + sin α )] = k { z1 z2 – z1 z1 + z2 z2 – z2 z1 }
= −
1 + sin α 2 (1 + sin α )
= i. = k (z2 z2 – z1 z1 ) = c.
The correct option is (B) The correct option is (C)
72. We have, 75. We have, z1 + z2 = – p and z1 z2 = q
4 + 3i ( 4 + 3i ) (1 − 2i ) We know that
z1 = =
1 + 2i (1 + 2i ) (1 − 2i ) z1 | z1 |
= (cos a + i sin a)
10 − 5i z2 | z2 |
= =2–i
5 Since |z1| = |z2|  ( OA = OB)
which represents the point whose coordinates are (2, –1) z1 cos α + i sin α
we get =
Also, we have, z2 1
2.40  Chapter 2

Applying Componendo and Dividendo, we get 2


≤ (r1 – r2)2 + 4 × 1 × 1 ⎛⎜ α − β ⎞⎟
z1 + z2 cos α + i sin α + 1 ⎝ 2 ⎠
=
z1 − z2 cos α + i sin α − 1 or, |z – w|2 ≤ (|z| – |w|)2 + (Arg z – Arg w)2.
α α α
2 cos 2 + 2i sin cos The correct option is (B)
= 2 2 2
α α α 78. A(z1), B(z2), C(z3) lie on |z| = 2 whose centre is at O(0, 0)
−2 sin 2 + 2i sin cos and radius 2.
2 2 2 π
z1 = 1 + 3 i hence |z| = 2 and Arg (z1) =
α ⎡ α α⎤ 3
2 cos
cos + i sin
2 ⎢⎣ 2 2 ⎥⎦ α
= = –i cot
α ⎡ α α⎤ 2
2i sin ⎢cos + i sin
2 ⎣ 2 2 ⎥⎦
p α
⇒ = i cot
z1 − z2 2
Squaring we obtain
p2 α
= –cot2
( z1 + z2 ) 2 − 4 z1z2 2
p2 α In turn |z2| = |z3| = 2 and Arg (z2) = Arg (z1) + 120º = 180º
⇒ 2
= –cot2
p − 4q 2 \ z2 = –2
2 2 2 α α Further, Arg (z3) = Arg (z2) + 120º = 300º
⇒ p = – p cot + 4q cot2
2 2
2 ⎛ 2 α⎞ 2 α ⎡ ⎛ π⎞ ⎛ π⎞⎤
⇒ p ⎜1 + cot ⎟ = 4q cot Hence,  z3 = 2 ⎢cos ⎜ 2π − ⎟ + i sin ⎜ 2π − ⎟ ⎥
⎝ 2⎠ 2 ⎝ 3 ⎠ ⎝ 3⎠⎦

α
⇒ p2 cosec2 = 4q cot2 ⎡ π π⎤ ⎛ 1 i 3⎞
2 = 2 ⎢cos − i sin ⎥ = 2 ⎜ −
⇒ p2 = 4q cos2
α ⎣ 3 3⎦ ⎝2 2 ⎟⎠
2 = 1 – 3 i
\ k = 4q
The correct option is (C) Thus, z2 = –2 and z3 = 1 – i 3
76. Since |z1| = |z2| = |z3| = 1, The correct option is (A)
HINTS AND EXPLANATIONS

we get, z1 z1 = z2 z2 = z3 z3 = 1. 79. We know that,


(1 + x)3n = 1 + 3nC1 x + 3nC2 x2 + … + 3nC3n x3n(1)
1 1 1 (1 – x)3n = 1 – 3nC1 x + 3nC2 x2 + … + (– 1)3n 3nC3n x3n(2)
Now, 1 = + + = | z1 + z2 + z3 |
z1 z2 z3 Subtracting (2) from (1) gives
(1 + x)3n – (1 – x)3n = 2 [3nC1 x + 3nC3 x3 + 3nC5 x5 + …]
= z1 + z2 + z3 = |z1 + z2 + z3|.
= 2x [3nC1 + 3nC3 x2 + 3nC5 x4 + …]
The correct option is (A)
Putting x = i 3 , we get
77. Let z = r1 (cos a + i sin a)
(1 + i 3 )3n – (1 – i 3 )3n
and, w = r2 (cos b + i sin b) ⇒ Arg z = a, Arg w = b
|z| = r1, |w| = r2. = 2i 3 [3nC1 – 3 × 3nC3 + 32 × 3nC5 …]
Given: |z| ≤ 1, |w | ≤ 1 ⇒ r1 ≤ 1, r2 ≤ 1 Therefore, 3nC1 – 3 3nC3 + 32 × 3nC5 …
1
Now, consider |z – w |2 = |(r1 cos a – r2 cos b) = [(1 + i 3 )3n – (1 – i 3 )3n]
+ i (r1 sin a – r2 sin b )2| 2i 3
⎡⎛
1 i 3 ⎞ ⎛ 1 i 3 ⎞ ⎥⎤
3n
= (r1 cos a – r2 cos b)2 + (r1 sin a – r2 sin b)2 1
= × 23n ⎢⎜ + − −
2i 3 ⎢⎝ 2 2 ⎟⎠ ⎜⎝ 2 2 ⎟⎠ ⎥
= r12 (cos2a + sin2a) + r22 (cos2b + sin2b) ⎣ ⎦
–2 r1 r2 (cos a cos b + sin a sin b) 23n −1
= [(cos np + i sin np) – (cos np – i sin np)]
i 3
= r12 + r22 – 2 r1 r2 cos (a – b)
= (r1 – r2)2 + 2 r1 r2 [1 – cos (a – b)] 23n −1
= 2i sin np = 0 as n is an integer.
⎛ α − β⎞ i 3
= (r1 – r2)2 + 4 r1 r2 sin2 ⎝⎜ 2 ⎠⎟ The correct option is (A)
Complex Numbers  2.41

80. We know that the triangle with vertices z1, z2, z3 is an ⇒ (z1 – z2)2 = 2 [z1 – z3] [z3 – z2]
­equilateral if The correct option is (B)
z12 + z22 + z32 = z1 z2 + z2 z3 + z3 z1
83. Let A = z1, B = z2 and C = z3,
\ The triangle with vertices z1 = a + i, z2 = 1 + bi and
where A, B, C are vertices of equilateral triangle.
z3 = 0 will be equilateral if
Given that third point C is origin, so z3 = 0.
(a + i)2 + (1 + bi)2 + 0 = (a + i) (1 + bi) + 0 + 0
Let z2 – z3 = a, z3 – z1 = b, z1 – z2 = g
⇒ a2 – 1 + 2ai + 1 – b2 + 2bi = (a – b) + i (1 + ab)
or, z2 = a, – z1 = b, z1 – z2 = g
⇒ a2 – b2 = a – b (1)
\ a + b + g = z2 – z1 + z1 – z2 = 0
and 2 (a + b) = 1 + ab (2)
[Equating real and imaginary parts] or, α + β + γ = 0 (1)
(1) ⇒ (a – b) [a + b – 1] = 0 Since the triangle is equilateral triangle,
⇒ a = b or a = 1 – b. \ BC = CA = AB
Subsitituting the value of a – b in (2), we get or, |(z2 – 0)| = |0 – z1| = |z1 – z2|
2 (a + a) = 1 + a2 ⇒ a2 – 4a + 1 = 0 or, |a| = |b | = |g |
4 ± 16 − 4 or, |a|2 = |b |2 = |g |2
⇒ a = =2± 3
2 or, α α =β β =γ γ = k (say)
Since 0 < a < 1 and 0 < b < 1,
\ a = b = 2 – 3 . k k k
\ α = , β = , γ =
Substituting a + b = 1 in (2), we get α β γ
2 = 1 + a (1 – a) ⇒ a2 – a + 1 = 0 Substituting values of α , β and γ in (1), we get
which gives imaginary values of a and b.
k k k
Hence, a = b = 2 – 3 + + =0
α β γ
The correct option is (B)
81. Let z1 = x1 + iy1 and z2 = x2 + iy2 1 1 1
or, + + =0
where, x1 ≠ x2, y1 ≠ y2 and x12 + y12 = x22 + y22 α β γ
z1 + z2 ( x + x2 ) + i ( y1 + y2 ) 1 1 1
Now, = 1 or, + + =0
z1 − z2 ( x1 − x2 ) + i ( y1 − y2 ) z2 − z1 z1 − z2

HINTS AND EXPLANATIONS


=
[( x1 + x2 ) + i ( y1 + y2 )] [( x1 − x2 ) − i ( y1 − y2 )] or,
z1 ( z1 − z2 ) − z2 ( z1 − z2 ) + z1 z2
=0
2
( x1 − x2 ) + ( y1 − y2 ) 2 z1 z2 ( z1 − z2 )

[( x12 − x22 ) + ( y12 − y22 )] + i [ x1 y1 − y1x2 + y2 x1 or, z12 – z1 z2 – z1 z2 + z22 + z1 z2 = 0


− y2 x2 − x1 y1 + x1 y2 − x2 y1 + x2 y2 ] or, z12 + z22 = z1 z2
=
2 2
( x1 − x2 ) + ( y1 − y2 ) z12 z2
or, + 2 =1
2 i ( x1 y2 − y1x2 ) z1 z2 z1 z2
=
( x1 − x2 ) 2 + ( y1 − y2 ) 2 z1 z2
x y or, + =1
= a purely imaginary or 0 if 1 = 1 . z2 z1
x2 y2
y A B
x
If 1 = 1 then x1 + iy1 = k (x2 + iy2) or, + = 1.
x2 y B A
2
The correct option is (A)
If k = 1, z1 = z2, which is not true and if k ≠ 1, |z1| ≠ |z2|.
z1 + z2 84. Given 2 2 x4 = ( 3 – 1) + i ( 3 + 1)
\ is purely imaginary.
z1 − z2 ⎛ 3 − 1⎞ ⎛ 3 + 1⎞
⇒ x4 = ⎜ ⎟ + i⎜ ⎟
The correct option is (D) ⎝ 2 2 ⎠ ⎝ 2 2 ⎠
82. Since |CA| = |CB| and ∠ACB = 90º 2 2
⎛ 3 − 1⎞ ⎛ 3 + 1⎞
\ (z2 – z3) = ± i (z1 – z3) ⇒ |x4|2 = ⎜ ⎟ +⎜ ⎟ =1
⎝ 2 2 ⎠ ⎝ 2 2 ⎠
⇒ (z2 – z3)2 = – (z1 – z3)2
\ |x4| = 1
⇒ z22 + z32 − 2 z2 z3 = − z12 − z32 − 2 z1 z3 ⎡⎛ 3 + 1⎞ ⎛ 3 − 1⎞ ⎤
and,  arg (x4) = tan– 1 ⎢⎜ ⎟ ⎜ ⎟⎥
z12 + z22 − 2 z1 z2 = 2 ( z1 z3 − z1 z2 − z2 z3 + z22 ) ⎢⎣⎝ 2 2 ⎠ ⎝ 2 2 ⎠ ⎥⎦
2.42  Chapter 2

⎛ 3 + 1⎞ 5π
= tan– 1 ⎜ ⇒ z12 + z22 = z1 z2.
⎟ = 75º =
⎝ 3 − 1⎠ 12 The correct option is (B)
⎡ ⎛ 5π ⎞ ⎛ 5π ⎞ ⎤ 87. We have, max. amp (z) = amp (z2),
\ x4 = 1 ⎢cos ⎜ 2nπ + ⎟ + i sin ⎜ 2nπ + ⎟ ⎥ min. amp (z) = amp (z1).
⎣ ⎝ 12 ⎠ ⎝ 12 ⎠ ⎦
n
Using (cos q + i sin q) = cos nq + i sin nq, we have
1⎛ 5π ⎞ 1⎛ 5π ⎞
x = cos ⎜ 2nπ + ⎟ + i sin ⎜ 2nπ + ⎟ ;
4⎝ 12 ⎠ 4⎝ 12 ⎠
n = 0, 1, 2, 3
The correct option is (B)

10
⎛ 2qπ 2qπ ⎞
85. ∑ ⎜⎝ sin 11
− i cos ⎟
11 ⎠ ⎛ 15 ⎞ ⎛ 3⎞
q =1
Now, amp (z1) = q1 = cos–1 ⎜ ⎟ = cos–1 ⎜ ⎟
⎝ 25 ⎠ ⎝ 5⎠
⎧ 2π 4π ⎫ π
= ⎨sin + sin + ... + 10 terms⎬ and,  amp (z2) = + q2
⎩ 11 11 ⎭ 2
⎧ 2π 4π ⎫ π ⎛ 15 ⎞
i ⎨cos + cos + ... + 10 terms⎬ = + sin–1 ⎜ ⎟
2 ⎝ 25 ⎠
⎩ 11 11 ⎭
⎛ 2π 9π ⎞ 10π ⎛ 2π 9π ⎞ 10π π ⎛ 3⎞
sin ⎜ + sin cos ⎜ + ⎟ sin = + sin–1 ⎜ ⎟
⎝ 11 11 ⎟⎠ 11 ⎝ 11 11 ⎠ 11 2 ⎝ 5⎠
= −i π 3 3
π π \ |max. amp (z) – min. amp (z)| = + sin −1 − cos −1
sin sin 2 5 5
11 11
π π 3 3
= 0 – i (– 1) = i (1) = + − − cos −1 − cos −1
p 2 2 5 5
32 ⎡ 10 ⎛ 2qπ 2qπ ⎞ ⎤ ⎛ 3⎞
\ S = ∑ (3 p + 2) ⎢∑ ⎜⎝ sin 11 − i cos 11 ⎟⎠ ⎥ = p – 2 cos–1 ⎜ ⎟
⎝ 5⎠
p =1 ⎢⎣ q =1 ⎥⎦
32 32 32
The correct option is (C)
= ∑ (3 p + 2) i p =3 ∑ pi p + 2 ∑ i p 88. Putting z = x + iy, we get
p =1 p =1 p =1 (x + iy)2 + (p + iq) (x + iy) + r + is = 0
HINTS AND EXPLANATIONS

= 3A + 2B(2) ⇒ (x2 – y2 + px – q y + r) + i (2xy + py + qx + s) = 0


Now,  A = i + 2i2 + 3i3 + … + 32i32 ⇒ x2 – y2 + px – q y + r = 0 (1)
⇒ Ai = i2 + 2i3 + … + 31i32 + 32i33 and, 2xy + py + qx + s = 0 (2)
⇒ A (1 – i) = i + i2 + … + i32 – 32i33 If the roots are real, then y = 0
i 32 − 1 \ (1) gives x2 + px + r = 0 (3)
=
– 32i = – 32i [∵ i32 = 1]
i −1 and (2) gives pqx + s = 0 (4)
−32i 32i (1 + i ) s
\ A = = = 16 (1 – i)(3) From (4), x = –
1− i 2 q
s 2 ps
and,  B = i + i2 + … + i32 = 0 (4) Putting in (3), we get 2 − +r =0
q q
Hence,  S = 3 × 16 (1 – i) = 48 (1 – i).
or, r2 – pqs + rq2 = 0 ⇒ pqs = s2 + rq2.
The correct option is (C)
The correct option is (A)
86. Since the triangle is equilateral
\ |z1 – 0| = |z2 – z1| = |0 – z2| 89. We know that |z1 + z2|2 + |z1 – z2|2
= 2 [|z1|2 + |z2|2](1)
⇒ |z1|2 = |z2 – z1|2 = |z2|2
2
⇒ z1 z1 = (z2 – z1) × ( z2 – z1 ) = z2 z2
Now, ⎡ z1 + z12 − z22 + z1 − z12 − z22 ⎤
z z z −z ⎢⎣ ⎥⎦
Now,  z1 z = z2 z2 ⇒ 1 = 2 = 1 2 .
1 z2 z1 z2 − z1 2 2
(z − z ) z = z1 + z12 − z22 + z1 − z12 − z22 + 2 | z12 − ( z12 − z22 )|
Also,  z2 z2 = (z2 – z1) ( z2 – z1 ) = (z2 – z1) 1 2 2
z1
\ z1 z2 = (z2 – z1) (z1 – z2) = z2 z1 – z12 − z22 + z1 z2 = 2 |z1|2 + 2 | z12 − z22 | + 2 | z22 | [By (1)]
Complex Numbers  2.43

z1 − z2 z −z z −z z −z
= 2 |z1|2 + 2 |z2|2 + 2 | z12 − z22 | ⇒ = – 1 2 ⇒ 1 2 = 2 3 (2)
z3 − z2 z3 − z2 z1 − z2 z3 − z2
= |z1 + z2|2 + |z1 – z2|2 + 2 |z1 + z2| |z1 – z2| (1) × (2) ⇒ (z1 – z2)2 = – (z2 – z3)2
= (|z1 + z2| + |z1 – z2|)2
⇒ z12 + z22 + z32 = 2 z2 (z1 + z3)
Taking square root of both sides, we get
The correct option is (B)
z1 + z12 − z22 + z1 − z12 − z22 = |z1 + z2| + |z1 – z2|. 92. Let z = r (cos q + i sin q), then
z + iz = r (cos q + i sin q) + ir (cos q + i sin q)
The correct option is (D)
= r [(cos q – sin q) + i (sin q + cos q)]
90. Since z = x + iy lies in the third quadrant
⎡ ⎛ π⎞ ⎛ π⎞⎤
\ x < 0, y < 0. Again z = x – iy = 2 r ⎢cos ⎜ θ + ⎟ + i sin ⎜ θ + ⎟ ⎥
⎣ ⎝ 4⎠ ⎝ 4⎠⎦
z x − iy ( x − iy )( x − iy ) x 2 − y 2 − 2ixy
\ = = =
z x + iy 2
x +y 2
x2 + y2

x2 − y2 2 xy
= − i = A + iB
x2 + y2 x2 + y2

x2 − y2 2 xy
where,  A = and B = –
x2 + y2 x2 + y2
Since x < 0, y < 0
− 2 xy In DOPQ,
\ <0 π
x2 + y2 PQ2 = r2 + ( 2r ) 2 − 2r ( 2r ) cos
4
\ B < 0 = r2 + 2r2 – 2r2 = r2
z \ PQ = r,
Now, lies in the IIIrd quadrant if A < 0 π
z \ ∠OPQ =
2
i.e., if x2 – y2 < 0 or x2 < y2 i.e., if x < y < 0.
The correct option is (C)
The correct option is (C)

HINTS AND EXPLANATIONS


91. Let BA = BC 93. Given |z + 1| < |z – 2|
⇒ |z1 – z2|2 = |z3 – z2|2 and,  w = 3z + 2 + i
\ ω = 3 z + 2 – i
⇒ (z1 – z2) ( z1 – z2 ) = (z3 – z2) ( z3 – z2 )(1) \ w + ω = 3 (z + z ) + 4 (i)
Again, Now, |z + 1|2 < |z – 2|2
\ ∠ABC = 90º ⇒ (z + 1) ( z + 1) < (z – 2) ( z − 2)
BA ⇒ (z + 1) ( z + 1) < (z – 2) ( z – 2)
\ arg = 90º
BC ⇒ z z + z + z + 1 < z z – 2z – 2 z + 4
z1 − z2 z −z ⇒ 3z + 3 z < 3 ⇒ z + z < 1 (ii)
⇒ arg = 90º ⇒ real part of 1 2 = 0
z3 − z2 z3 − z2 From (i) and (ii), we get
w + ω < 3 × 1 + 4 = 7 ⇒ w + ω < 7
1 ⎡ z1 − z2 z1 − z2 ⎤
⇒ ⎢ + ⎥ =0 Clearly, |w + 1| < |w – 8| gives
2 ⎣ z3 − z2 z3 − z2 ⎦ |w + 1|2 < |w – 8|2

⇒ (w + 1) ( ω + 1) < (w – 8) ( ω – 8)
⇒ w ω + w + ω + 1 < w ω – 8 ω – 8w + 64
⇒ 9 (w + ω ) < 63 ⇒ w + ω < 7
The correct option is (A)
94. We have, x2 + x + 1 = (x – w) (x – w2)
Since f (x) is divisible by x2 + x + 1, f (w) = 0, f (w2) = 0
\ P(w3) + w Q (w3) = 0 or P(1) + w Q(1) = 0 (1)
and,  P(w6) + w2 Q (w6) = 0 or P(1) + w2 Q(1) = 0 (2)

2.44  Chapter 2

From (1) and (2) we obtain 98. Since z1 and z1 are the adjacent vertices of a regular
P(1) = 0 and Q(1) = 0. 2π
­polygon of n sides, we have, ∠ z1 0 z1 =
\ Both P(x) and Q(x) are divisible by x – 1 n
and, |z1| = | z1 |
Since f (x) = P(x) + x Q(x), we get f (x) is divisible by x – 1.
Thus, z1 = z1 e2pi/n
The correct option is (D)
Let z1 = r (cos q + i sin q) = reiq

⎛ 8π ⎞ ⎛ 8π ⎞ i
⇒ z1 = re– iq
95. We have, a = cos ⎜ ⎟ + isin ⎜ ⎟ = e 11
⎝ 11 ⎠ ⎝ 11 ⎠ Since z1 = z1 e2pi/n
Re (a + a2 + a3 + a4 + a5) ⇒ reiq = re– iq e2pi/n = re2pi/n – iq
2 3 4 5 2 3 4 5
α +α +α +α +α +α +α +α +α +α 2π π
= ⇒ q = – q or q =
2 n n
⎡ π π⎤
−1 + (1 + α + α 2 + α 3 + α 4 + α 5 + α + α 2 + α 3 + α 4 + α 5 ) Therefore, z1 = r (cos q + i sin q) = r ⎢cos + i sin ⎥
= ⎣ n n⎦
2
−1 + 0 ⎛π⎞
= (sum of 11, 11th roots of unity) r sin ⎜ ⎟
2 Im ( z1 ) ⎝ n⎠
1 Now, = 2 –1⇒ = 2 –1
= – Re ( z1 ) ⎛π⎞
2 r cos ⎜ ⎟
⎝ n⎠
The correct option is (B) π π
⇒ tan = 2 – 1 = tan ⇒n=8
96. We have, eiA, eiB, eiC are in A. P. n 8
The correct option is (B)
⇒ 2eiB = eiA + eiC
99. We have,
⇒ 2 (cos B + i sin B)
2 1 1 z +z
= (cos A + cos C) + i (sin A + sin C) = + = 3 2
z1 z2 z3 z2 z3
⇒ 2 cos B = cos A + cos C
2 z2 z3
and,  2 sin B = sin A + sin C ⇒ z1 = .
A+C A−C z2 + z3
⇒ 2 cos B = 2 cos cos (1)
2 2 ⎛ z − z4 ⎞ ⎛ z1 − z3 ⎞
Now, ⎜ 2
and, 2 sin B = 2 sin
A+C
cos
A−C
(2) ⎝ z1 − z4 ⎟⎠ ⎜⎝ z2 − z3 ⎟⎠
2 2
Dividing (1) by (2), we get ⎛ ⎞ ⎛ 2 z2 z3 ⎞
− z3
HINTS AND EXPLANATIONS

⎜ z −z ⎟ ⎜z +z ⎟
⎛ A+C⎞ B 3B = ⎜ 2 4
⎟ ⎜ 2 3

cot B = cot ⎜ = tan ⇒ cos =0
⎝ 2 ⎟⎠ 2 2 ⎜ 2 z2 z3 − z ⎟ ⎜ z2 − z3 ⎟
⎜⎝ z + z 4⎟
⎠ ⎜⎝ ⎟⎠
3B π π 2π 2 3
⇒ =  or B = ⇒A+C= .
2 2 3 3 z2 z3 ( z2 − z3 )
Putting this value in (1), we get =  [taking z4 = 0]
2 z2 z3 ( z2 + z3 )( z2 − z3 )
π π ⎛ A−C⎞ ⎛ A−C⎞ z2 + z3
2cos = 2 cos cos ⎜ ⇒ cos ⎜ =1
3 3 ⎝ 2 ⎟⎠ ⎝ 2 ⎟⎠
1
⎛ A−C⎞ π = (a real number).
⇒ ⎜ = 0 or A = C ⇒ A = B = C = . 2
⎝ 2 ⎟⎠ 3 Hence, points z1, z2, z3 and origin are concyclic and
The correct option is (C) ­therefore, z1, z2, z3 lie on a circle passing through the origin.
97. Let cot–1 p = q, then p = cot q The correct option is (B)
m m 100. Since |z – 25i| ≤ 15, therefore, distance between z and 25i is
⎛ pi + 1⎞ ⎛ i cot θ + 1⎞
= e 2 miθ ⋅ ⎜
−1

\ e 2 mi cot p
⋅⎜ less than or equal to 15.
⎝ pi − 1⎟⎠ ⎝ i cot θ − 1⎟⎠
m m
⎛ i (cot θ − i ) ⎞ ⎛ cot θ − i ⎞
= e 2 miθ ⋅ ⎜ = e 2 miθ ⋅ ⎜
⎝ i (cot θ + i ) ⎟⎠ ⎝ cot θ + i ⎟⎠
m m
⎛ cos θ − i sin θ ⎞ ⎛ e −i θ ⎞
= e 2 miθ ⋅ ⎜ = e 2 miθ ⋅ ⎜ i θ ⎟
⎝ cos θ + i sin θ ⎟⎠ ⎝ e ⎠
= e2miq (e–2iq)m = e2miq × e–2miq = e0 = 1
The correct option is (B)
Complex Numbers  2.45

Thus, point z will lie in the interior and boundary of the 103. We have, 1 ≥ |z – (4 – 3i)|
circle whose centre is (0, 25) and radius is 15. ⎧| z | − | 4 − 3i | ⎧| z | − 5
Let OP be tangent to the circle at point P. ≥ ⎨ ⇒1≥ ⎨
⎩| 4 − 3 i | − | z | ⎩5 − | z |
Let ∠POX = q. Then, ∠OCP = q
⇒ |z| ≤ 6 and |z| ≥ 4
Now, OC = 25, CP = 15
⇒ 4 ≤ |z| ≤ 6 ⇒ m = 4, n = 6
\ OP = 20.
OP 20 4 x4 + x2 + 4 4
Now, tan q = = = Let y = = x3 + x +
CP 15 3 x x
⎛ 4⎞ 1 1 1 1
\ Least positive value of arg z = q = tan–1 ⎜ ⎟ 3
=x +x+ + + +
⎝ 3⎠ x x x x
The correct option is (B)
1 1 1 1
101. Let z = x + iy 3
Since x ∈ (0, ∞), \ x , x, , , , are all positive
x x x x
Given, |z – 4 + 3i| ≤ 2. numbers whose product is 1.
\ |x + iy – 4 + 3i| ≤ 2 \ their sum y will be least when
or, ( x − 4) 2 + ( y + 3) 2 ≤ 2 1
x3 = x = ⇒x=1
x
or, (x – 4)2 + (y + 3)2 ≤ 22
\ least value of y = 6
Thus, z lies in the interior or on the boundary of the circle
\ k = 6
whose centre is (4, –3) and radius is 2.
Hence, k = n
Least value of |z| = OA = OC – AC = 5 – 2 = 3.
The correct option is (B)
104. We have, zn = (z + 1)n
n
⎛ z + 1⎞
⇒ ⎜ = 1 = cos 0 + i sin 0
⎝ z ⎟⎠
z +1
⇒ = (cos 2p r + i sin 2p r)1/n
z
2π r 2π r
= cos + i sin
n n
where, r = 0, 1, 2, …, n – 1.
2π r 2π r

HINTS AND EXPLANATIONS


1
⇒ 1 + = cos + i sin
Greatest value of |z| = OB = OC + CB = 5 + 2 = 7. z n n
Thus, 3 ≤ |z| ≤ 7. 1 2 rπ πr πr
⇒ 1 + = 1 – 2 sin + i × 2 sin × cos
The correct option is (A) z n n n
1 rπ rπ rπ
102. Since Re (z1 z ) ≤ | z1z2 | ⇒ = – 2 sin2 + 2i × sin cos
2 z n n n
\ |z1| + |z2| + 2 Re (z1 z ) ≤ |z1|2 + |z2|2 + 2 |z1 z |
2 2
1
2 2 ⇒ z =
⇒ |z1 + z2|2 ≤ |z1|2 + |z2|2 + 2 |z1| |z2|(1) ⎛ rπ ⎞ ⎡ rπ rπ ⎤
i ⎜ 2 sin ⎟ cos + i sin
Also, Since A.M. ≥ G.M. ⎝ n ⎠ ⎢⎣ n n ⎥⎦
2
⎛ 1 ⎞ 1 ⎡ rπ rπ ⎤
( c | z1 |) 2 + ⎜ |z | 1
=
⎝ c 2 ⎟⎠ ⎧ 1 ⎫2

⎛ rπ ⎞ ⎢cos n − i sin n ⎥
\ ≥ ⎨c ⋅| z1 |2 ⋅ | z2 |2 ⎬ i ⎜ 2 sin ⎣ ⎦
⎝ ⎟
2 ⎩ c ⎭ n⎠
(∵ c > 0)
1 i rπ 1 1
⇒ c |z1|2 + |z2|2 ≥ 2 |z1| |z2| ⇒ x + iy = − cot − ⇒x=–
c 2 n 2 2
1 1
\ |z1|2 + |z1|2 + 2 |z1| |z2| ≤ |z1|2 + |z2|2 + c |z1|2 + |z2|2 Hence, all the points lie on the line x = –
c 2
⇒ |z1|2 + |z2|2 + 2 |z1| |z2| ≤ (1 + c) |z1|2 The correct option is (B)
+ (1 + c–1) (|z2|2)(2) 105. We have, z2 + z + 1 = 0 (1)
From (1) and (2), we get ⇒ (z – 1) (z2 + z + 1) = 0,
|z1 + z2|2 ≤ (1 + c) |z1|2 + (1 + c–1) |z2|2 \ z3 = 1.
If n is not a multiple of 3, then we can write
\ k = 1 + c–1
n = 3m + r, where m ∈ I and r = 1 or 2,
The correct option is (C)
2.46  Chapter 2

then 2n = 6m + 2r \ y1 – y2 = x1 – x2,(4)
If r = 1, then 2r = 2 \ From (3) and (4) we get
\ zn + z2n = (z3)m × zr + (z3)2n × z2r = zr + z2r y1 + y2 = 2
= z + z2 = –1 [Using (1)] \ Im (z1 + z2) = 2.
If r = 2, then 2r = 4 The correct option is (C)
\ 2n = 3 (m + 1) + 1 1 1 1 1 2
108. Since, + + + =
\ zn + z2n = (z3)m × zr + (z3)m + 1 × z1 = z2 + z = –1 a+ω b+ω c+ω d +ω ω
Hence, zn + z2n = – 1 \ w is the root of the equation
The correct option is (D) 1 1 1 1 2
+ + + =
106. Since 1, a1, a2, a3, a4 are the roots of the equation x5 – 1 = 0. a+ x b+ x c+ x d + x x
\ (x5 – 1) = (x – 1) (x – a1) (x – a2) (x – a3) (x – a4) ⇒ 2x4 + (S a) x3 + 0 × x2 – (S abc) x – 2 abcd = 0
x5 − 1 Let a, b, g  be the other roots, then
⇒ = (x – a1) (x – a2) (x – a3) (x – a4)(1)
x −1 Σa
Putting x = w in (1), we get w + a + b + g = – (1)
2
ω5 − 1 Sab = 0
= (w – a1) (w – a2) (w – a3) (w – a4)
ω −1 ab + aw + bw + g w + bg + g a = 0 (2)
ω2 −1 Σ abc
⇒ = (w – a1) (w – a2) (w – a3) (w – a4)(2) Sabg = (3)
ω −1 2
and putting x = w2 in (1), we get abg w = –abcd(4)
ω10 − 1 Since complex roots occurs in conjugate pairs
2 = (w2 – a1) (w2 – a2) (w2 – a3) (w2 – a4)
ω −1 \ g = ω = w2.
ω −1 \ From (2),
⇒ = (w2 – a1) (w2 – a2) (w2 – a3) (w2 – a4)(3)
ω2 −1 ab + w (a + b) + w × w2 + w2 (a + b) = 0
Dividing (2) by (3), we get ⇒ ab + (w + w2) (a + b) + w3 = 0
ω −α ω −α ω −α ω −α (ω 2 − 1) 2 ⇒ ab + (– 1) (a + b) + 1 = 0
2 1 ⋅ 2 2 ⋅ 2 3 ⋅ 2 4 = ⇒ ab – a – b + 1 = 0
ω − α1 ω − α 2 ω − α 3 ω − α 4 (ω − 1) 2
⇒ a (b – 1) – (b – 1) = 0 ⇒ (a – 1) (b – 1) = 0
ω 4 + 1 − 2ω 2 ω + 1 − 2ω 2
HINTS AND EXPLANATIONS

= = \ either a = 1 or b = 1
ω 2 + 1 − 2ω ω 2 + 1 − 2ω
Hence one root is unity
− ω 2 − 2ω 2 − 3ω 2 1 1 1 1
= = =w \ + + + =2
− ω − 2ω − 3ω a +1 b +1 c +1 d +1
The correct option is (B) The correct option is (B)
107. Let z = x + iy 109. We have, |A|2 + |B|2 + |C|2 = A A + B B + C C (1)
We have, z + z = 2 |z – 1| But  A A = (z1 + z2 + z3) ( z1 + z2 + z3 )
z+z
⇒ = |z – 1| = z1z1 + z2 z2 + z3 z3 + z1 ( z2 + z3 )
2
⇒ x = |x + iy – 1| ⇒ x = |(x – 1) + iy| + z2 ( z3 + z1 ) + z3 ( z1 + z2 )
⇒ x2 = (x – 1)2 + y2 ⇒ 2x = 1 + y2. = | z1 |2 + | z2 |2 + | z3 |2 + z1 ( z2 + z3 )
If z1 = x1 + iy1 and z2 = x2 + iy2 + z2 ( z3 + z1 ) + z3 ( z1 + z2 )

then, 2x1 = 1 + y12 (1) B B = (z1 + z2 w + z3 w2) ( z1 + z2ω + z3ω 2 )


and, 2x2 = 1 + y22 (2) = (z1 + z2 w + z3 w2) ( z1 + z2ω 2 + z3ω )
Subtracting (2) from (1), we get [∵ ω = w2 and (ω 2 ) = w]
2 (x1 – x2) = y12 – y22 = z1z1 + z2 z2ω 3 + z3 z3ω 3 + z1 (z2 w + z3 w2)
⇒ 2 (x1 – x2) = (y1 + y2) (y1 – y2)(3) + z2 (z3 w4 + z1 w2) + z3 (z1 w + z2 w2)
π = | z1 |2 + | z2 |2 + | z3 |2 + z1 ( z2ω + z3ω 2 )
But given arg (z1 – z2) =
4
+ z2 ( z3ω + z1ω 2 ) + z3 ( z1ω + z2ω 2 ) (2)
⎛ y − y ⎞ π y1 − y2
i.e., tan– 1 ⎜ 1 2
⎟ = ⇒ =1 Similarly, C C = | z1 |2 + | z2 |2 + | z3 |2 + z1 ( z2ω 2 + z3ω )
⎝x −x ⎠1 2 4 x −x 1 2
Complex Numbers  2.47

+ z2 ( z3ω 2 + z1ω ) + z3 ( z1ω 2 + z2ω ) (3) Since z + 2 = a2 – 3a + 2 represents a circle with centre

Adding (1), (2) and (3), we get at A − ( )


2 , 0 and radius a 2 − 3a + 2 and z + 2i
2 2 2 2
A A + B B + C C = 3 [|z1| + |z2| + |z3| ] < a represents the interior of the circle with centre at
+ z1 [z2 (1 + w + w2) + z3 (1 + w2 + w)] (
B 0, 2 ) and radius a. Therefore, there will be a com-
+ z2 [z3 (1 + w + w2) + z1 (1 + w + w2)] plex number satisfying the given condition and the given
+ z3 [z1 (1 + w + w2) + z2 (1 + w2 + w)] inequality if the distance AB is less than the sum or differ-
ence of the radii of the two circles, i.e., if
= 3 [|z1|2 + |z2|2 + |z3|2][∵ 1 + w + w2 = 0]
\ From (1) and (2), we conclude
(− ) + (0 + 2 )
2 2
2 −0 < a 2 − 3a + 2 ± a
2 2 2 2 2 2
|A| + |B| + |C| = 3 [|z1| + |z2| + |z3| ].
The correct option is (A) ⇒ 2 ± a < a 2 − 3a + 2
⇒ 4 + a2 ± 4a < a2 – 3a + 2
1
110. z + = 2 (cos q + i sin q ) = 2eiq ⇒ –a < – 2  or  7a < – 2
z −2
\ z2 – 2eiq z + 1 = 0 ⇒ a > 2  or  a <
7
But,  a > 0,
⇒ z = eiθ ± e 2iθ − 1
\ a > 2
e2iq – 1 = cos 2q + i sin 2q – 1 The correct option is (A)
= 2 sinq (– sin q + i cos q)
112. Let Sn = 1 + 2a + 3a2 + … + nan–1
⎡ ⎛π ⎞ ⎛π ⎞⎤
= 2 sin θ ⎢cos ⎜ + θ ⎟ + i sin ⎜ + θ ⎟ ⎥ \ a Sn = a + 2a2 + … + (n – 1)an–1 + nan
⎣ ⎝ 2 ⎠ ⎝ 2 ⎠⎦
Subtracting, we get,
\ Let a = eiθ + e 2iθ − 1 Sn (1 – a) = 1 + a + a2 + … + an–1 – nan
⎛π θ⎞ 1 (1 − α n )
= cos θ + sin θ cos ⎜ + ⎟ = − nα n
⎝ 4 2⎠ 1−α
⎡ ⎛π θ⎞⎤ 1 − αn nα n −n
+ i ⎢sin θ + 2 sin θ sin ⎜ + ⎟ ⎥ \ Sn = − = .
⎣ ⎝ 4 2⎠ ⎦ (1 − α ) 2 1−α 1−α

HINTS AND EXPLANATIONS


2
⎧ ⎛π θ⎞⎫ The correct option is (B)
\ |a – i|2 = ⎨cos θ + 2 sin θ cos ⎜ + ⎟ ⎬
⎩ ⎝ 4 2⎠ ⎭ 113. Let A represent z1
2
⎧ ⎛π θ⎞ ⎫ z1 z2
+ ⎨sin θ + 2 sin θ sin ⎜ + ⎟ − 1⎬
⎩ ⎝ 4 2⎠ ⎭ 0 A B

⎛π θ ⎞
= 2 + 2 2 sin θ cos ⎜ + − θ ⎟ Since OA · OB = 1, \ |z1 – 0| × |z – 0| = 1
⎝4 2 ⎠ 1
⇒ |z1| =
⎛π θ⎞ |z|
− 2 2 sin θ sin ⎜ + ⎟ = 2
⎝ 4 2⎠
⎛ z − 0⎞ ⎛z ⎞
Similarly, |b – i|2 = 2 Also, arg ⎜ 1 = 0 ⇒ arg ⎜ 1 ⎟ = 0
⎝ z − 0 ⎟⎠ ⎝ z⎠
\ |a – i| = |b – i|
⇒ arg z1 = arg z
The correct option is (D)
If q is the argument of z, then
111. If z = x + iy is a complex number satisfying the given con- z = |z| eiq
ditions, then
1 iθ 1 z 1
a2 – 3a + 2 = z + 2 \ z1 = e = | z | eiθ = =
|z| |z|2 zz z
= z+i 2+ 2−i 2 1
\ A is
z
≤ z+i 2 + 2 |1 − i |
The correct option is (A)
2
<a +2 114. Let ABCD be the rhombus and M be the point of intersec-
⇒ –3a < 0 ⇒ a > 0 tion of the diagonals AC and BD
2.48  Chapter 2

Similarly, taking clockwise rotation we get another possible


position of A as
z3 − z1 |z −z |
= 3 1 e − iθ
z2 − z1 | z2 − z1 |
⎛1 − i⎞ 3
⇒ z3 = ⎜ (1 − 2i ) + (1 + i ) ⇒ z3 = 1 − i
⎝ 2 ⎟⎠ 2
i 3
So, A represents the complex numbers 3 − or 1 − i
2 2
The correct option is (A)
Let point D be z1 = 1 + i and point M be z2 = 2 – i 115. Let z = x + iy = r (cosq + i sin q), then the equation is
Also, let point A be z3
⎛ 1 1 ⎞
Then, z2 – z1 = 1 – 2i and |z2 – z1| = 5 = MD |(x – 2) + i (y – 1)| = r ⎜ cos θ − sin θ ⎟
As given, ⎝ 2 2 ⎠
1 1 5 1
AC = BD ⇒ AM = DM ⇒ AM = = ( r cos θ − r sin θ )
2 2 2 2
1
⇒ AD = |z3 – z1| = DM 2 + AM 2 or, ( x − 2) 2 + ( y − 1) 2 = ( x − y)
2
2
⎛ 5⎞ which is the part of a parabola with focus (2, 1) and ­directrix
( 5) 5
2
= +⎜ ⎟ = x – y = 0.
⎝ 2 ⎠ 2
The correct option is (C)
Therefore, in D AMD,
116. |z1 + z2|2 + |z2 + z3|2 + |z3 + z1|2
5 2 5/2 1
cos q = = and sin q = =
5/2 5 5 / 2 5 = 2 (| z1 |2 + | z2 |2 + | z3 |2 ) + ( z1z2 + z1z2
z −z + z2 z3 + z2 z3 + z3 z1 + z1z3 )
Now, by rotation of complex numbers we know that 3 1
z2 − z1
|z −z |
= 3 1 eiθ (anticlockwise rotation) = 24 + ( z1z2 + z1z2 + z2 z3 + z2 z3 + z3 z1 + z3 z1 ) (1)
| z2 − z1 | Also,
z − (1 + i ) 5/2 | z1 + z2 + z3 |2 ≥ 0
⇒ 3 = (cos θ + i sin θ )
HINTS AND EXPLANATIONS

1 − 2i 5
⇒ z1z2 + z1z2 + z2 z3 + z2 z3 + z3 z1 + z3 z1 ≥ – 12
z − (1 + i ) i
⇒ 3 = 1 + (using values of cosq and sinq) \ |z1 + z2|2 + |z2 + z3|2 + |z3 + z1|2 ≥ 12
1 − 2i 2
The correct option is (B)
2+i i
⇒ z3 = (1 − 2i ) + (1 + i ) ⇒ z3 = 3 −
2 2

Previous Year’s Questions


117. Key Idea : If w is a cube root of unity, then 1 + ω + ω2 = 0
⎛ z⎞ π
and ω3 = 1 119. arg( z ) − arg(ω ) = arg ⎜ ⎟ =
⎝ω⎠ 2
(l + ω + w2) = 0 and ω3 = 1
(1 + ω – ω2)7 = (−ω2 − ω2)7 (∵1 + ω + ω2 = 0) ⇒ | zω | = 1
2 7
= (− 2ω ) ⇒ z ω = −i or + i .
= –27⋅ω14 The correct option is (D) or (C)
= –128(ω3)4ω2
= − 128 ω2(∵ ω3= 1) 1 + i (1 + i ) 2
120. = =i
The correct option is (D) 1− i 2
x
118. Given z12 + z22 − z1z2 = 0 ⎛1+ i⎞
⇒ ⎜ = ix
⇒ (z1 + z2)2 – 3z1z2 = 0 ⎝ 1 − i ⎟⎠
⇒ a2 = 3b. ⇒ x = 4 n .
The correct option is (C) The correct option is (A)
Complex Numbers  2.49

Z ⎛ z⎞ 128. The given equation z2 + z + 1 = 0


121. Here ω = ⇒ arg ⎜ z ⎟ = π ⇒ z = ω or ω2.
i ⎝ i⎠
1 1
So, z+ = ω + ω2 = −1, z2 + 2
⇒ 2arg( z ) − arg(i ) = π z z
π 1
⇒ 2 arg( z ) −=π = ω2 + ω = −1, z3 + 3 = ω3 + ω3 = 2,
2 z
3π 1 1 1
⇒ arg( z ) = . z4 + 4 = –1, z5 + 5 = –1 and z6 + 6 = 2.
4 z z z
The correct option is (C) ∴ The given sum = 1 + 1 + 4 + 1 + 1 + 4 = 12
The correct option is (D)
122. z = (p + iq)3 = p(p2− 3q2)− iq(q2− 3p2) 129. From the Argand diagram, maximum value of | z + 1| is 6.
x y Alternative:
+
x 2 2 y 2 2 p q | z + 1| = | z + 4 − 3| ≤ | z + 4| + |−3| = 6.
⇒ = p − 3q and = q − 3 p ⇒ 2 = −2 .
p q ( p + q2 )
The correct option is (B)

( )
2
123. Since | z 2 − 1 |2 = | z |2 +1 , we have


( z − 1)( z
2 2
)
− 1 = | z |4 + 2 | z |2 +1
(–7, 0) (–4, 0) (–1, 0)
2 2
⇒ z + z + 2 zz = 0 ⇒ z + z = 0

⇒ R (z) = 0 ⇒ z lies on the imaginary axis.


The correct option is (D)
124. Given equation (x − 1)3 + 8 = 0 implies that (x − 1)
The correct option is (C)
= (−2) (1)1/3
130. Put −i in place of i
⇒ x − 1 = −2 or −2ω or −2ω2 −1
Or n = −1 or 1 − 2ω or 1 − 2ω2. Hence, the complex conjugate is
i +1
The correct option is (C) The correct option is (C)

HINTS AND EXPLANATIONS


125. | z1 + z2| = | z1 | + | z2| ⇒ z1 and z2 are collinear and are to the ⎛ 4⎞ 4 4 4
same side of origin; hence argz1− argz2 = 0. 131. One can write Z = ⎜ Z − ⎟ + ⇒ Z = Z − +
⎝ Z⎠ Z Z Z
The correct option is (C)
4 4 4
⇒ Z ≤ Z− + ⇒ Z ≤ 2+
z z Z Z Z
126. As given w = ⇒| w |= = 1 ⇒ distance of z
1 1
z− i z− i 2
⇒ Z −2 Z −4≤0
3 3
⎛ 1⎞
from origin and point ⎜ 0, ⎟ is same. ( ( ))(
∴ Z − 5 + 1 Z − 1 − 5 ≤ 0 ( ))
⎝ 3⎠
⇒1 − 5 ≤ Z ≤ 5 + 1
Hence z lies on the bisector of the line joining points (0, 0)
and (0, 1/3). The correct option is (B)
Hence z lies on a straight line. 132. Let z = x + iy
The correct option is (C) |z− 1| = |z + 1| ⇒ Re z = 0 ⇒ x = 0
127. Given sum |z−1| = |z−i| ⇒ x = y
10
|z + 1| = |z−i| ⇒ y = −x
⎛ 2kπ 2kπ ⎞

10 ⎜
sin
⎝ 211
+ i cos ⎟ Only (0, 0) will satisfy all conditions.
k =1 ⎛ kπ kπ ⎠⎞
211 ⇒ Number of complex number z = 1
∑1⎜⎝0 11
sin
2kπ
+ i cos
10 11 ⎟
2⎠kπ The correct option is (A)
= ∑
k =1
sin
11
+ i ∑ cos
10
k =1 2kπ k =1 10
211
kπ 133. Suppose roots are 1 + pi, 1 + qi
= ∑ sin + i ∑ cos
k =1 11 k =111 Sum of roots 1 + pi + 1 + qi = −α which is real
⇒ roots of 1 + pi,1 − pi
=0 + i (−1) = −i
Product of roots = β = 1 + p2 ∈(1, ∞)
The correct option is (D)
2.50  Chapter 2

p ≠ 0 since roots are distinct.


The correct option is (C) = ( 2 − z1z2 ) ( 2 − z1z2 )

134. 1 + ω = −ω2 2 2
⇒ z1 − 2 z2 − z1 − 2 z2 z1 + 4 z2
(1 + ω )7 = ( −ω 2 ) = −ω14 = −ω 2

= 1 + ω = A + Bω ⇒ ( A, B ) = (1,1) 2 2
= 4 − 2 z1z2 − 2 z1z2 + z1 z2

The correct option is (A)
2 2 2 2
135. Let z = x + iy (∴ x ≠ 1 as z ≠ 1 ) ⇒ z1 + 4 z2 − z1 z2 − 4 = 0

z 2 = ( x 2 − y 2 ) + i( 2 xy )

z2
is real ⇒ its imaginary part = 0
⇒ z1
2
(1 − z ) − 4 (1 − z ) = 0
2
2
2
2

z −1
⇒ z1 = 2( as z2 ≠ 1)

⇒ 2 xy( x − 1) − y( x 2 − y 2 ) = 0
The correct option is (B)
⇒ y( x 2 + y 2 − 2 x ) = 0 139. We have,
⇒ y = 0; x 2 + y 2 − 2 x = 0 2 + 3i sin θ
z =
1 − 2i sin θ
∴z lies either on real axis or on a circle through origin.
The correct option is (A) ( 2 + 3i sin θ )(1 + 2i sin θ )
⇒ z =
136. Given z = 1 ⇒ zz = 1 1 + 4 sin 2 θ
1+ z 1+ z ( 2 − 6 sin 2 θ ) + 7i sin θ
∴ = =z. =
1+ z 1+ 1 1 + 4 sin 2 θ
z For z to be purely imaginary, we have Re (z) = 0
The correct option is (B)
1
137. z ≥2 ⇒ 2 − 6 sin 2 θ = 0 ⇒ sin θ = ±
3
1 1 1 3
z+ ≥ z − ≥2− ≥ . ⎛ 1 ⎞
2 2 2 2 ⇒ θ = ± sin −1 ⎜ ⎟
⎛ 1 ⎞ 3 ⎝ 3⎠
Hence, minimum distance between z and ⎜⎝ − , 0⎟⎠ is The correct option is (A)
2 2
The correct option is (B)
HINTS AND EXPLANATIONS

z1 − 2 z2 3 i
138. Given that =1
140. Let a = +
2 − z1z2 2 2
() ()
5 5
then z = a5 + a = 2 Re a
⇒ ( z1 − 2 z2 ) ( z1 − 2 z2 )
So, lm (z) 0
CHAPTER Quadratic Equations
3 and Expressions

LEARNING OBJECTIVES
After reading this chapter, you will be able to:
  Know about quadratic equations and their roots  Learn how to represent quadratic equation in a graph
and what are rational algebraic expressions

QUADRATIC EQUATION
QUICK TIPS
An algebraic expression of the form: ax2 + bx + c, where
a (≠0), b, c ∈ R is called a real quadratic expression. For the quadratic equation ax2 + bx + c = 0
An equation of the form: ax2 + bx + c = 0, where ■ One root will be reciprocal of the other if a = c.

a (≠0), b, c ∈ R is called a real quadratic equation. ■ One root is zero if c = 0.

The numbers a, b, c are called the coefficients of the ■ Roots are equal in magnitude but opposite in sign if b = 0.

quadratic equation and the expression b 2 − 4 ac is called ■ Both roots are zero if b = c = 0.
its discriminant. Discriminant of a quadratic equation is ■ Roots are positive if D > 0, a and c are of same sign and b
usually denoted by D or D. is of opposite sign.
■ Roots are of opposite sign if a and c are of opposite
Roots of the Quadratic Equation sign.
A root of the quadratic equation ■ Roots are negative if D > 0 and a, b, c are of the same

sign.
ax2 + bx + c = 0 (1) ■ Roots are rational ⇔ D is a perfect square
2
is a number a (real or complex) such that aa + ba + c = 0. ■ Roots are irrational ⇔ D is positive but not a perfect

The roots of the quadratic Eq. (1) are given by, square.
c
■ If a + b + c = 0, then 1, are the roots of the equation
−b ± b 2 − 4 ac a
x= ax2 + bx + c = 0
2a c
and if a – b + c = 0, then the roots are –1 and – .
a
Nature of Roots of the 2
■ If ax + bx + c = 0 is satisfied by more than two values, it
Quadratic Equation is an identity and a = b = c = 0 and vice-versa.
1. If D < 0, then roots a, b are imaginary 2
■ If ax + bx + c = 0, where a, b, c ∈ R, has one root p + iq,

2. If D > 0, then roots α, β are real and distinct then the other root will be p – iq. Hence, the imaginary
3. If D = 0, then roots α, β are real and equal roots occur in conjugate pair.
3.2  Chapter 3

1 1
■ If ax2 + bx + c = 0, where a, b, c are rational, has one root Solving Eq. (1) and (2), we get α = − and β = .
5 5
p + q then the other root will be p − q . Hence, irra-
But these values do not satisfy Eq. (3). Hence the
tional roots occur in conjugate pair if the coefficients are
rational.
given equation cannot be an identify for any values
of α and β.
■ The quadratic equation whose roots are reciprocals of the

roots of ax2 + bx + c = 0 is cx2 + bx + a = 0 (i.e., the coef- 3. If x2 + lx + m is an integer for every integer x, then
ficients are written in reverse order). (A) l is always an integer but m need not be an integer
■ If a = 1, b, c ∈ Z and the roots are rational numbers, then
(B) m is always in integer but l need not be an integer
these roots must be integers. (C) l and m, both are always integer
2
■ The condition that the roots of the equation ax + bx + c =
2
(D)  None of these
0 may be in the ration m : n is mnb = ac(m + n).
2 3
■ If sum of roots of ax + bx + c = 0 is equal to the sum of
Solution: (C)
c b a
their reciprocals, then ab2, bc2, ca2 are in A.P. or , ,
Let g(x) = x2 + lx + m, then g(x) is an integer for every
are in H.P. b a c integer x.
Therefore, g(0) = m ⇒ m is an integer
Also,
SOLVED EXAMPLES g(1) = 1 + l + m ⇒ l is an integer ( m is integer)

1. If a + b + c = 0 and a, b, c are rational, then the roots of 4. The equation 125x + 45x = 227x has
the equation (b + c – a) x2 + (c + a – b) x + (a + b– c) = 0 (A)  no solution
are (B)  one solution
(A) rational (B) irrational (C)  two solutions
(C) imaginary (D) equal (C)  more than two solutions
Solution: (A) Solution: (B)
We have, The given equation can be written as
D = (c + a – b)2 – 4 (b + c – a) (a + b – c) (5/3)3x + (5/3)x = 2
= (a + b + c – 2b)2 – 4 (a + b + c – 2a) Putting (5/3)x = t, the equation becomes
(a + b + c – 2c)
t3 + t – 2 = 0
2 2
= (– 2b) – 4 (­­– 2a) (– 2c) = 4 (b – 4ac) ⇒ t3 – 1 + (t – 1) = 0
= 4 [(– a – c)2 – 4ac] = 4 (a – c)2 ⇒ (t – 1) (t2 + t + 1) + (t – 1) = 0
= [2 (a – c)]2 = perfect square ⇒ (t – 1) (t2 + t + 2) = 0
∴ Roots are rational ⇒ t = 1
2. The number of values of the pair (α, β ) for which the or 2
t + t + 2 = 0
equation α (x + 1)2 + β (x2 – 3x – 2) + x + 1 = 0, ∀ x ∈ R is 2
But t + t + 2 = 0 does not have real solutions.
(A) 1 (B) 0
(C)  infinite (D)  None of these Therefore, t = 1 ⇒ (5/3)x = 1 ⇒ x = 0.
Solution: (B) 5. For a > 0, the roots of the equation
The equation logax a + logx a2 + log a x a3 = 0, are given by:
2

(α + β)x2 + (2α – 3β + 1)x + (α + 2β + 1) = 0 (A) a–4/3 (B) 


a–3/4 (C) 
a1/2 (D) 
a–1
is an identify in x if Solution: (A)
α + β = 0 (1) We have,
2α – 3β + 1 = 0 (2) log a a 2 log a a 3 log a a
+ + =0
log a a + log a x log a x 2 log a a + log a x
and α – 2β + 1 = 0 (3)
Quadratic Equations and Expressions  3.3

1 2 3 So, D = 4 (a2 + 1)2 – 12 (a2 – 3a + 2) > 0


⇒ + + = 0  (let loga x = t)
1+ t t 2 + t a 2 − 3a + 2
2 2 2 and product of roots = <0
2t + t + 2t + 6t + 4 + 3t + 3t 3
⇒ =0
t (1 + t ) ( 2 + t ) ⇒ a2 – 3a + 2 = (a – 1) (a – 2) < 0
2
⇒ 6t + 11t + 4 = 0 ∴ 1 < a < 2
2
⇒ 6t + 8t + 3t + 4 = 0 Clearly for these values of a, D > 0.
⇒ (2t + 1) (3t + 4) = 0 Hence, 1 < a < 2.
1 4 9. The number of real solutions of the equation 271/x +
⇒ t=– ,–
2 3 121/x = 2 × 81/x is
1 4 (A) one (B) two (C) infinite (D) zero
⇒ loga x = – , –
2 3 Solution: (D)
–1/2 –4/3
∴ x=a ,a The given equation can be written as
6. The number of solutions of the equation sin (ex) = 5x + ⎛ 3⎞
3/ x
⎛ 3⎞
1/ x

5– x is ⎜⎝ ⎟⎠ +⎜ ⎟ = 2.
2 ⎝ 2⎠
(A) 0 (B) 1 (C) 2 (D) infinite 1/ x
⎛ 3⎞
Put ⎜ ⎟ = t, then the equation becomes
Solution: (A) ⎝ 2⎠
Put 5x = y. Then the given equation becomes
t3 + t – 2 = 0  ⇒ (t – 1) (t2 + t + 2) = 0.
2
1 ⎛ 1 ⎞
sin (ex) = y + = ⎜ y− x
⎟ + 2  (∵ 5 > 0) But t2 + t + 2 = 0 has no real roots,
y ⎝ y⎠ ∴ t = 1
1/ x
⇒ sin (ex) ≥ 2. ⎛ 3⎞ 1
⇒ ⎜⎝ ⎟⎠ = 1  ⇒  = 0
Which is not possible for any real value of x. 2 x
Hence, the given equation has no real solution. which is not possible for any value of x.
2/3 1/3 3 2
7. If x = 2 + 2 + 2 then the value of x – 6x + 6x is
12 x
(A) 3 (B) 2 10. For all real values of x,
(C)  1 (D)  None of these 4x2 + 9

Solution: (B) ≤2 (C) 
(A) ≤1 (B)  >1  (D) 
>2
We have, Solution: (A)
x – 2 = 22/3 + 21/3 12 x
Let = y,
Cube both sides, we get 4x2 + 9
(x – 2)3 = 22 + 2 + 3 ⋅ 22/3 ⋅ 21/3 (x – 2) Now, 4yx2 – 12x + 9y = 0
= 6 + 6 (x – 2) As x is real,
or 3 2
x – 6x + 12x – 8 = –6 + 6x.  D = 144 – 4 ⋅ 4y ⋅ 9y ≥ 0  ⇒  1 – y2 ≥ 0
∴ x3 – 6x2 + 6x = 2. ⇒ y2 ≤ 1;
8. The values of a, for which the quadratic equation ∴ |y| ≤ 1.
3x2 + 2 (a2 + 1) x + (a2 – 3a + 2) = 0 possesses roots of
opposite sign, are 12 x
Hence, ≤ 1.
(A) 1 < a < 2 (B)  a ∈ (2, ∞) 4x2 + 9
(C) 1 < a < 3 (D)  None of these 11. If x2 – 3x + 2 be one of the factors of the expression
Solution: (A) x4 – px2 + q, then
Roots are of opposite sign if (a) roots are real and (A) p = 4, q = 5 (B)  p = 5, q = 4
­distinct, (b) product is negative. (C) p = –5, q = –4 (D)  None of these
3.4  Chapter 3

Solution: (B) ⇒ rp – rq = pq – pr
Since x2 – 3x + 2 is one of the factors of the expression ⇒ 2rp = q (p + r)
x4 – px2 + q, therefore, on dividing the expression by
factor, remainder = 0 i.e., on dividing x4– px2 + q by 2 p+r 1 1
∴ = = + .
x2 – 3x + 2, the remainder q pr p r 
(15 – 3p) x + (2p + q – 14) = 0 p a b
14. If c ≠ 0 and the equation = + has two
On comparing both sides, we get 2x x+c x−c
equal roots, then p can be
15 – 3p = 0  or  p = 5
(A) ( a − b ) 2 (B) 
( a + b )2
and 2p + q – 14 = 0  or  q = 4. (C) a + b (D)  a–b
12. If the roots of x2 – bx + c = 0 are two consecutive Solution:  (A, B)
­integers, then b2 – 4c is We can write the given equation as
(A) 1 (B) 0 p ( a + b) x + c ( b − a)
(C)  2 (D)  None of these =
2x x 2 − c2 
Solution: (A)
or p (x2 – c2) = 2 (a + b) x2 – 2c (a – b) x
The roots of the equation are given by,
or (2a + 2b – p) x2 – 2c (a – b) x + pc2 = 0
2
b ± b − 4c For this equation to have equal roots
x =
2 
c2(a – b)2 – pc2 (2a + 2b – p) = 0
2
b + b − 4c ⇒ (a – b)2 – 2p (a + b) + p2 = 0 ( c2 ≠ 0)
If α=
2 
p – (a + b)]2 = (a + b)2 – (a – b)2 = 4ab
⇒ [ 
2
b − b − 4c
and β= ⇒ p – (a + b) = ± 2 ab 
2 
Then, α – β = 1 ⇒ p = a + b ± 2 ab = ( a ± b ) 2 

15. If (7 − 4 3 ) x − 4 x + 3 + (7 + 4 3 ) x2 − 4 x +3
2

⇒ b 2 − 4c = 1 = 14, then
the value of x is given by
⇒ b2 – 4c = 1.
(A)  2, 2 ± 2 (B) 2 ± 3,3
2
13. If p (q – r) x + q (r – p) x + r (p – q) = 0 has equal (C) 3 ± 2 , 2 (D)  None of these
2
roots, then = Solution: (A)
q
Since (7 + 4 3 ) (7 − 4 3 ) = 1,
1 1
(A) p + (B)  +r ∴ the given equation becomes
r p
1
1 1 y+ = 14
(C) p + r (D)  + y
p r 2
− 4x + 3
where y = (7 − 4 3 ) x
Solution: (D) 
Since p (q – r) + q (r – p) + r (p – q) = 0 ⇒ 2
y – 14y + 1 = 0 ⇒ y = 7 ± 4 3 
∴ one root is 1
Now y= 7±4 3
r ( p − q)
∴ other root = . ⇒ x2 – 4x + 3 = –1
p (q − r) 
⇒ x = 2, 2
Since roots are equal
rp − rq Also, y= 7−4 3
∴ = 1
pq − pr ⇒ 2
x – 4x + 3 = 1 ⇒ x = 2 ± 2 
Quadratic Equations and Expressions  3.5

Sum and Product of the Roots 17. If the ratio of the roots of lx2 + nx + n = 0 is p : q, then
If α and β are roots of ax2 + bx + c = 0, then q p l
(A)  + + =0
−b Coefficient of x p q n
Sum of roots = α + β = =–
a Coefficient of x 2  p q n
(B)  + + =0
c Constant term q p l
Product of roots = αβ = =
a Coefficient of x 2 
q p l
(C)  + + =1
Formation of Equation with Given Roots p q n

If α and β are roots of p q n


(D)  + + =1
2 q p l
f (x) = ax + bx + c = 0,
then f (x) = (x – α) (x – β ) = 0 Solution: (B)
= x2 – (α + β )x + αβ = 0 Let the roots be α and β.
−n n
i.e., x2 – (sum of the roots) x + (product of the roots) = 0. Then α+β= ; αβ = ;
l l
α p
ERROR CHECK and =
β q
A quadratic equation with all odd integer coefficients cannot p q n α β
have rational roots. Now, + + = + + αβ
q p l β α
n n
− +
α + β + αβ l l = 0
SOLVED EXAMPLES = =
αβ n
16. If r be the ratio of the roots of the equation l
( r + 1) 2 18. In a quadratic equation with leading coefficient 1, a
ax2 + bx + c = 0, then = student reads the coefficient 16 of x wrongly as 19 and
r
a2 b2 obtain the roots as – 15 and – 4. The correct roots are
(A)  (B) 
bc ca (A)  6, 10 (B)  –6, –10
c 2 (C)  –7, –9 (D)  None of these
(C)  (D)  None of these
ab Solution: (B)
Solution: (B) Since coefficient of x = 16,
∴ sum of roots = –16
Given equation is ax2 + bx + c = 0 (1)
Since constant term = (–15) (–4) = 60,
Let the root of equation (1) be α and rα, then
∴ correct answer is –6, –10.
−b
α + rα = (2) 1 1 1
a 19. If the roots of the equation + = are
x+a x+b c
c
and rα2 = (3) equal in magnitude but opposite in sign, then their
a product is
From Eq. (2),
1 −1 2
b (A)  (a2 + b2)
(B)  (a + b2)
α=– (4) 2 2
a ( r + 1) −1 × ⎡⎣(ω 33 )3ω 2 + ω 214 ⎤⎦

−1
Putting the value of α in Eq. (3), we get (C)  ab (D)  ab
2
rb 2 c
2 2
= Solution: (B)
a ( r + 1) a
We have, ((x + b) + (x + a)c = (x + a) (x + b)
b2 ( r + 1) 2
or, = ⇒ x2 + bx + ax – 2cx + ab – bc – ca = 0
ac r 
3.6  Chapter 3

Now, let roots be α and β, then 22. If α, β are the roots of x2 – 2px + q = 0 and γ, δ are
roots of x2 – 2rx + s = 0 and α, β, γ, δ are in A.P., then
α + β = 0, αβ = ab – bc – ac
(A) p – q = r2 – s2 (B)  s – q = r2 – p2
α + β = 0 ⇒ b + a = 2c (C) r – s = p2 – q2 (D)  None of these
and αβ = ab – (b + a) c Solution: (B)
( a + b) 2 We have, α + β = 2p;
⇒ αβ = ab –
2  αβ = q, γ + δ = 2r and γ δ = s
1
⇒ 2
αβ = (– a – b ) 2
   α, β, γ, δ are in A.P.
2
1 ∴ β – α = δ – γ ⇒ (β – α)2 = (δ – γ )2
∴ αβ = – (a2 + b2)
2 ⇒ (β + α)2 – 4βα = (δ + γ )2 – 4δγ
20. If sin θ and cos θ are the roots of the equation ax2 + ⇒ 4p2 – 4q = 4r2 – 4s;
bx + c = 0, then
or s – q = r2 – p2
(A) (a – c)2 = b2 – c2 (B) (a – c)2 = b2 + c2
(C) (a + c)2 = b2 – c2 (D)  (a + c)2 = b2 + c2 23. The rational values of a and b in ax2 + bx + 1 = 0 if
1
Solution: (D) is a root, are
Since sin θ and cos θ are the roots of the equation 4+ 3
ax2 + bx + c = 0 (A) a = 13, b = – 8 (B)  a = – 13, b = 8
b c (C) a = 13, b = 8 (D)  a = – 13, b = ­– 8
∴ sin θ + cos θ = − and sin θ cos θ =
a a Solution: (A)
Now (sin θ + cos θ )2 = 1 + 2 sin θ cos θ 1 4− 3 4− 3
One root = × =
b 2
2c a + 2c 4+ 3 4− 3 13 
∴ =1 +=
a 2 a a  4+ 3
∴ other root =
⇒ b = a (a + 2c) = a2 + 2ac
2 13 
∴ The quadratic equation is
⇒ b2 + c2 = a2 + 2ac + c2 = (a + c)2
⎛ 4 + 3 4 − 3⎞ 4+ 3 4− 3
Hence, (a + c)2 = b2 + c2 x 2 − ⎜ + ⎟ x+ ⋅ = 0
⎝ 13 13 ⎠ 13 13
21. In copying a quadratic equation of the form x2 + px +
q = 0, a student wrote the coefficient of x incorrectly or 13x2 – 8x + 1 = 0
and the roots were found to be 3 and 10; another stu- This equation must be identical with ax2 + bx + 1 = 0;
dent wrote the same equation but he wrote the ­constant ∴ a = 13  and  b = –8.
term incorrectly and thus he found the roots to be
4 and 7. The roots of the correct equation are 24. If a and b are rational and α, β be the roots of x2 +
(A)  5, 6 (B)  4, 6 2ax + b = 0, then the equation with rational coefficients
(C)  4, 5 (D)  None of these one of whose roots is α + β + α 2 + β 2 is
Solution: (A) (A) x2 + 4ax – 2b = 0 (B)  x2 + 4ax + 2b = 0
In case of the first student, product of the roots = 3 × (C) x2 – 4ax + 2b = 0 (D)  x2 – 4ax – 2b = 0
10 = q. So the correct value of q is 30. Solution: (B)
In case of the second student, sum of the roots
Since α, β are roots of x2 + 2ax + b = 0
= 4 + 7 = –p.
So the correct value of p is –11. α + β = –2a and αβ = b
2
∴ The correct equation is x – 11x + 30 = 0 Let y = α + β + α 2 + β2 
or (x – 5) (x – 6) = 0; ⇒ (y + 2a)2 =α2 + β 2 = (α + β)2 – 2αβ = 4a2 – 2b
∴ x = 5, 6. ⇒ y2 + 4ay + 2b = 0
∴ Roots of the correct equation are 5, 6. So, the required equation is x2 + 4ax + 2b = 0.
Quadratic Equations and Expressions  3.7

25. If c, d are the roots of the equation (x – a) (x – b) – k = 0, − b − i 4 ac − b 2


then the roots of the equation (x – c) (x – d) + k = 0 are and β= 
2a
(A) c, d (B)  a, c (C)  b, d (D)  a, b
∴ α = β .
Solution: (D)
We have, (x – a) (x – b) – k = 0
COMMON ROOTS
⇒ x2 – (a + b) x + ab – k = 0 (1)
One Root Common
Since the roots of Eq. (1) are c and d
If α is a common root of the equations
∴ c + d = a + b,(2)
a1x2 + b1x + c1 = 0(1)
and cd = ab – k(3)
and a2x2 + b2x + c2 = 0(2)
Now (x – c) (x – d) + k = 0
then we have
⇒ x2 – (c + d) x + cd + k = 0
a1α 2 + b1α + c1 = 0
2
⇒ x – (a + b) x + ab = 0
and a2α 2 + b2α + c2 = 0
 [Putting the values of a + b and
 ab from Eqs (2) and (3)] α2 α
These give =
⇒ (x – a) (x – b) = 0 ⇒ x = a, b. b1c2 − b2 c1 c1a2 − c2 a1

26. If the roots of the equations x2 – bx + c = 0 and x2 – cx+ 1
= ( a1b2 − a2 b1 ≠ 0).
b = 0 differ by the same quantity then b + c is equal to a1b2 − a2 b1
(A) 4 (B) 1 (C) 0 (D) –4
Thus, the required condition for one common root is
Solution: (D) ( a1b2 − a2 b1 ) (b1c2 − b2 c1 ) = (c1a2 − c2 a1 ) 2 and the value
We know that if α, β are roots of the equation c a − c2 a1 b c − b2 c1
of the common root is α = 1 2 or 1 2 .
Ax2 + Bx + C = 0, a1b2 − a2 b1 c1a2 − c2 a1

B 2 − 4 AC Both Roots Common


then α–β=
A  If the Eq. (1) and (2) have both roots common, then these
Equating the value of α – β from both the given equations will be identical. Thus the required condition for
­equations, we get both roots common is
a1 b c
= 1 = 1
b 2 − 4c = c 2 − 4b a2 b2 c2

⇒ b2 – 4c = c2 – 4b
QUICK TIPS
⇒ b2 – c2 = –4 (b – c)
■ To find the common root of two equations, make the coef-
⇒ (b – c) (b + c + 4) = 0 ficient of second degree terms in two equations equal and
⇒ b + c = –4  (∵ b ≠ c) subtract. The value of x so obtained is the required com-
mon root.
27. If α, β are non-real roots of ax2 + bx + c = 0, (a, b, ■ If two quadratic equations with real coefficients have an
c ∈ R), then imaginary root common, then both roots will be common and
(A) αβ = 1 (B)  α=β the two equations will be identical. The required condition is
α= β
(C)  αβ = 1 (D)  a1 b c
= 1 = 1
a2 b2 c2
Solution: (D)
■ If two quadratic equations have an irrational root common,
Here b2 – 4ac < 0 then both roots will be common and the two equations will
be identical. The required condition is
− b + i 4 ac − b 2 a1 b c
∴ α= = 1 = 1
2a  a2 b2 c2
3.8  Chapter 3

30. If a, b, c ∈ R and the equations ax2 + bx + c = 0 and


If α is a repeated root of the quadratic equation
x3 + 3x2 + 3x + 2 = 0 have two roots in common, then

f(x) = ax2 + bx + c = 0,
(A) a = b ≠ c (B)  a=b=–c
then α is also a root of the equation f  ′(x) = 0.
(C) a = b = c (D)  None of these
■ If α is repeated common root of two quadratic equations

f(x) = 0 and ϕ(x) = 0, then α is also a common root of the Solution: (C)


equations f  ′(x) = 0 and ϕ  ′(x) = 0. We have, x3 + 3x2 + 3x + 2 = 0
⇒ (x + 1)3 + 1 = 0

SOLVED EXAMPLES ⇒ (x + 1 + 1) [(x + 1)2 – (x + 1) + 1] = 0


⇒ (x + 2) (x2 + x + 1) = 0
2
28. The value of k so that the equations x – x – 12 = 0 and
− 1 ± 3i
kx2 + 10x + 3 = 0 may have one root in common, is ⇒ x = –2,
2 
43 43
(A)  (B) 3 (C) – 3 (D)  ⇒ x = –2, ω, ω 2
16 18
Solution:  (B, D) Since a, b, c ∈ R, ax2 + bx + c = 0 cannot have one real
Let α be the common root and one imaginary root. Therefore, two common roots
of ax2 + bx + c = 0 and x3 + 3x2 + 3x + 2 = 0 are ω, ω2.
Then, α2 – α – 12 = 0  and  kα2 + 10α + 3 = 0
b
Solving the two equations, we get Thus, − = ω + ω2 = –1
a
α2 α 1 ⇒ a = b
= = c
117 − 12k − 3 10 + k  and = ω ⋅ ω2 = 1 ⇒ c = a
a
⇒ (– 12k – 3)2 = 117 (10 + k) ⇒ a = b = c
⇒ 9 (4k + 1)2 = 117 (10 + k) 31. If the equations k (6x2 + 3) + rx + 2x2 – 1 = 0 and
⇒ (4k + 1) = 13 (10 + k) 2
6k  (2x2 + 1) + px + 4x2 – 2 = 0 have both the roots
common, then the value of 2r – p is
⇒ 16k2 + 8k + 1 = 130 + 13k
(A) 0 (B) 1
⇒ 16k2 – 5k – 129 = 0 (C)  –1 (D)  None of these
⇒ 16k2 – 48k + 43k – 129 = 0 Solution: (A)
− 43 The two equations can be written as
∴ k = 3  or  k =
16  x2 (6k + 2) + rx + (3k – 1) = 0 (1)
2 2
29. If the equations ax + bx + c = 0 and x + 2x + 3 = 0 and x2 (12k + 4) + px + (6k – 2) = 0 (2)
have a common root, then a : b : c =
Divide by 2, we get
(A)  2 : 4 : 5 (B)  1 : 3 : 4
(C)  1 : 2 : 3 (D)  None of these p
x2 (6k + 2) + x + (3k – 1) = 0 (3)
2
Solution: (C) Comparing Eq. (1) and (3), we get
For the equation x2 + 2x + 3 = 0,
p
Discriminant = (2)2 – 4 ⋅ 1 ⋅ 3 < 0. r =
2
∴ roots of x2 + 2x + 3 = 0 are imaginary. Since the ∴ 2r – p = 0
equations x2 + 2x + 3 = 0 and ax2 + bx + c = 0 are given
to have a common root, therefore both roots will be 32. If the equations x2 – ax + b = 0 and x2 + bx – a = 0 have
common. Hence both the equations are identical. a common root, then
a b c (A) a + b = 1
∴ = = (B) a = b
1 2 3
(C) a – b = 2
i.e. a : b : c = 1 : 2 : 3. (D) a + b = 0 or a – b = 1
Quadratic Equations and Expressions  3.9

Solution: (D) 2
= ⎛ p2 − ⎞
1
Let α be a common root of the given equations. ⎜⎝ 2⎟
+2+ 2 ≥ 2+ 2 
2p ⎠
Then α2 – aα + b = 0  and  α2 + bα – a = 0 Therefore, minimum value of α4 + β4 is 2 + 2
⇒ (a + b) α – (a + b) = 0
⇒ (a + b) (α – 1) = 0 GRAPH OF A QUADRATIC EXPRESSION
⇒ a + b = 0 or α = 1 We have, y or f (x) = ax2 + bx + c where a, b, c ∈ R, a ≠ 0.

If α = 1, 1. The shape of the curve y = f (x) is a parabola


2. The axis of the parabola is y-axis (incase b = 0) or
then 1 – a + b = 0 ⇒ a – b = 1. parallel to y-axis.
3. If a > 0, then the parabola opens upwards.
SYMMETRIC FUNCTION OF THE ROOTS 4. If a < 0, then the parabola opens downwards
5. For D > 0, parabola cuts x-axis in two distinct points
A function of α and β is said to be a symmetric function if it
remains unchanged when α and β are interchanged. a > 0, D > 0
For example, α 2 + β 2 + 2αβ is a symmetric ­function x-axis
of α and β whereas α 2 – β 2 + 3αβ is not a symmetric
­function of α and β.
x-axis
a > 0, D > 0
QUICK TIPS FIGURE 3.1 (A) FIGURE 3.1 (B)

In order to find the value of a symmetric function of α and 6. For D = 0, parabola touches x-axis in one point.
β, express the given function in terms of α + β and αβ. The
a > 0, D = 0
following results may be useful. x-axis
2 2 2
■ α + β = (α + β ) – 2αβ
3 3 3
■ α + β = (α + β ) – 3αβ (α + β )
4 4 3 3 2 2 x-axis
■ α + β = (α + β ) (α + β ) – αβ (α + β ) a > 0, D = 0
5 5 3 3 2 2 2 2
■ α + β = (α + β ) (α + β ) – α β (α + β )
FIGURE 3.2 (A) FIGURE 3.2 (B)
2
■ |α – β | = (α + β ) − 4αβ 7. For D < 0, parabola does not cut x-axis.
2 2
■ α – β = (α + β ) (α – β )
a < 0, D < 0
3 3 2
■ α – β = (α – β ) [(α + β ) – αβ] x-axis
4 4 2 2
■ α – β = (α + β ) (α – β ) (α + β )

x-axis
a > 0, D < 0
SOLVED EXAMPLE FIGURE 3.3 (A) FIGURE 3.3 (B)

1
33. If α and β be the roots of the equation x2 + px –
= 0,
2 p2 GREATEST AND LEAST VALUES OF A
where p ∈ R, then the minimum value of α4 + β 4 is QUADRATIC EXPRESSION
(A)  2 (B)  2+ 2 1. If a > 0, then the quadratic expression y = ax2 + bx + c
(C) 2 − 2 (D) 2 has no greatest value but it has least value
4 ac − b 2 b
Solution: (B)  at x = –
4a 2a
α 4 + β 4 = (α 2 + β 2) – 2α2β 2
2. If a < 0, then the quadratic expression y = ax2 + bx + c
2 2 2
= [(α + β) – 2αβ] – 2(αβ ) has no least value but it has greatest value
⎛ 1 ⎞ 1
2
1 4 ac − b 2 b
= ⎜ p2 + 2 ⎟ − = p4 + +2  at x = −
⎝ p ⎠ 2p 4
2 p4 4a 2a

3.10  Chapter 3

Sign of Quadratic Expression 2. If both roots of f (x) = 0 lie between k1 and k2, then
α+β
We have, y or f (x) = ax2 + bx + c where a, b, c ∈ R, a ≠ 0. D ≥ 0, a f (k1) > 0, a f (k2) > 0 and k1 < < k2
2
1. If a > 0 and D < 0, then f (x) > 0 for all x ∈ R i.e., f (x)
is positive for all real values of x. k1 α β k2
2. If a < 0 and D < 0, then f (x) < 0 for all x ∈ R i.e., f (x) 3. If k1 and k2 lie between the roots of f (x) = 0, then D ≥ 0,
is negative for all real values of x. a f (k1) < 0 and a f (k2) < 0.
3. If a > 0 and D = 0, then f (x) ≥ 0 for all x ∈ R i.e., f (x) is
positive for all real values of x except at vertex, where α k1 k2 β
f (x) = 0.
4. If a < 0 and D = 0, then f (x) ≤ 0 for all x ∈ R i.e. f (x) is
negative for all real values of x except at vertex, where QUICK TIPS
f (x) = 0.
1. Let f(x) = 0 be a polynomial equation. Let p and q be two
5. If a > 0 and D > 0, let f (x) = 0 have two real roots α and
real numbers, p < q.
β(α < β ), then f (x) > 0 for all x ∈ (–∞, α) ∪ (β, ∞) and
(a) If f(p) · f(q) < 0, then the equation f(x) = 0 has odd
f (x) < 0 for all x ∈ (α, β ).
number of real roots between p and q.
6. If a < 0 and D > 0, let f (x) = 0 have two real roots α and
(b) If f(p) · f(q) > 0, then the equation f(x) = 0 has
β(α < β ). Then f (x) < 0 for all x ∈ (–∞, α) ∪ (β, ∞) and
either no real root or even number of real roots
f (x) > 0 for all x ∈ (α, β ). between p and q.
(c) If f(p) = f(q), then the equation f  ′(x) = 0 has at
NATURE OF ROOTS OF A QUADRATIC least one real root between p and q (This is due to
Rolle’s Theorem)
EQUATION WITH RESPECT TO ONE OR TWO
2. (a) If the coefficients of the polynomial equation f(x) =
REAL NUMBERS 0 have p changes of signs, then the equation f(x) =
Let f (x) = ax2 + bx + c, where a, b, c ∈ R, a ≠ 0. Let α, 0 will have atmost p, positive roots.
β(α  < β ) be the roots of the corresponding quadratic (b) If the coefficients of the polynomial equation f(–x) =
­equation. Let k, k1, k2 ∈ R and k1 < k2. 0 have q changes of signs, then the equation f(x) =
0 will have atmost q, negative roots.
Nature of Roots with Respect to (c) The polynomial equation f(x) = 0 will have atmost
One Real Number p + q real roots where p and q are the changes of
signs of coefficients in f(x) and f(–x). (This is due to
1. If both the roots of f (x) = 0 are greater than k, then Descarte’s Rule of signs)
b For example, consider
D ≥ 0, a f (k) > 0 and k < –
2a f(x) = 2x5 – 6x4 + 7x3 – 8x2 + 5x + 3
+ – + – + +
k α β
Then, f(–x) = –2x5 – 6x4 – 7x3 – 8x2 – 5x + 3
2. If both the roots of f (x) = 0 are less than k, then D ≥ 0, – – – – – +
b Clearly, f(x) has 4 changes of signs and f(–x) has
a f (k) > 0 and k > –
2a only one change of sign, Therefore, the equation
f(x) = 2x5 – 6x4 + 7x3 – 8x2 + 5x + 3 = 0 has
α β k atmost four positive roots and one negative root.
3. If one root is less than k and other is greater than k, Also, the equation has atmost (4 + 1) = 5 real
roots.
then D > 0 and a f (k) < 0
3. (a) A polynomial equation f(x) = 0 has exactly one root
α k β equal to α if f(α) = 0 and f  ′(α) ≠ 0.
(b) A polynomial equation f(x) = 0 has exactly two roots
Roots with Respect to Two Real Numbers equal to α if f(α) = 0, f  ′(α) = 0 and f ′′ (α) ≠ 0.
(c)  In general, a polynomial equation f(x) = 0 has
1. If exactly one root of f (x) = 0 lies in the interval (k1, k2),
exactly n roots equal to α if
then D > 0 and f (k1) · f (k2) < 0
f(α) = f  ′(α) = f ′′ (α) = … = f n–1(α) = 0
k1 α k2 β and     f n(α) ≠ 0
Quadratic Equations and Expressions  3.11

RELATION BETWEEN ROOTS AND FORMATION OF A POLYNOMIAL EQUATION


COEFFICIENTS OF A POLYNOMIAL EQUATION FROM GIVEN ROOTS
Let f (x) = a0xn + a1xn – 1 + a2xn – 2 + … + an – 1x + an = 0, a0, If α1, α2, α3, … αn are the roots of a polynomial equation
a1, a2, …, an ∈ R, a0 ≠ 0 be a polynomial equation of degree of degree n, then the equation is
n, having n roots α1, α2, … αn. Then,
xn – σ1xn – 1 + σ2xn – 2 – σ3xn – 3 + … + (–1)nσn = 0
1. Sum of all roots
where σr = Σα1α2 … αr.
σ1 = α1 + α2 + … + αn
a a Particular Cases
= Σα1 = – 1 = (–1)1 1
a0 a0
Quadratic Equation
2. Sum of the product of two roots
σ2 = α1α2 + α1α3 + … If α, β are the roots of a quadratic equation, then the
a a ­equation is x2 – (α + β)x + αβ = 0.
= Σα1α2 = 2 = (–1)2 2
a0 a0 Cubic Equation
3. Sum of the product of three roots
If α, β, γ are the roots of a cubic equation, then the equation is
σ3 = α1α2α3 + α2α3α4 + …
a a x3 – σ1x2 + σ2x – σ3 = 0
= Σα1α2α3 = – 3 = (–1)3 3 and so on.
a0 a0 or x3 – (α + β + γ )x2 + (αβ + αγ + βγ )x – αβγ = 0
a
In general, σr = Σα1α2 … αr = (–1)r r . Biquadratic Equation
a0
If α, β, γ, δ are the roots of a biquadratic equation, then the
Particular Cases equation is
Quadratic Equation x4 – σ1x3 + σ2x2 – σ3x + σ4 = 0

If α, β are roots of the quadratic equation ax2 + bx + c = 0, or


then x4 – (α + β + γ + δ )x3 + (αβ + αγ + αδ + βγ + βδ + γ δ )x2
b c
α + β = – and αβ =
a a – (αβγ + αβδ + αγδ + βγδ)x + αβγδ = 0
Cubic Equation SIGN OF A POLYNOMIAL EXPRESSION
If α, β, γ are roots of the cubic equation ax3 + bx2 + cx + d
Step 1: Factorize the given polynomial expression as
= 0, then
b f (x) = ( x − α1 ) k ( x − α 2 ) k ( x − α 3 ) k …
1 2 3

σ1 = α + β + γ  = –
a ( x − α n −1 ) k ( x − α n ) k
n −1 n

c
σ2 = αβ + αγ + βγ =  where k1, k2, … kn ∈ N and
a
d α1, α2, α3, … αn ∈ R (α1 < α2 < α3 … < αn)
σ3 = αβγ = –
a Step 2: Plot the points α1, α2, α3, …, αn on the real line.
Biquadratic Equation Step 3: Mark plus sign in the interval of the right of the
largest of these numbers i.e., on the right of αn.
If α, β, γ, δ are roots of the biquadratic equation ax4 + bx3 +
cx2 + dx + e = 0, then –∞ α1 α2 α 3 .... αn ∞
b
σ1 = α + β + γ  + δ = – Step 4: If kn is even, put ‘+’ sign of the left of αn and if kn
a is odd, put ‘–’ sign on the left of αn.
c
σ2 = αβ + αγ + αδ + βγ + βδ + γδ = Step 5: Consider the next interval and put a sign in it using
a the above rule. Thus, consider all the intervals.
d Step 6: The solution of f (x) > 0 is the union of all the inter-
σ3 = αβγ + αβδ + αγδ + βγδ = –
a vals in which there is a ‘+’ sign and the solution of
e
σ4 = αβγδ = f (x) < 0 is the union of all the intervals in which
a
there is a ‘–’ sign.
3.12  Chapter 3

For example, consider the polynomial expression 36. The equation


58
⎛ 3⎞ x + 3 − 4 x − 1 + x + 8 − 6 x − 1 = 1 has
40
f (x) = (x + 2) (x + 1) 31 ⎜⎝ x − ⎟ (x – 4)37
2⎠
(A)  no solution
(B)  one solution
+ – + – +
(C)  two solutions
–2 –1 3/2 4 (D)  more than two solutions
⎛ 3⎞ Solution: (D)
Thus, f (x) > 0 if x ∈ (–∞, –2) ∪ ⎜ − 1, ⎟ ∪ (4, ∞)
⎝ 2⎠ Put x − 1 = t ⇒ x – 1 = t2 or x = t2 + 1,
⎛3 ⎞ The given equation reduces to
and f (x) < 0 if x ∈ (–2, –1) ∪ ⎜ , 4⎟ .
⎝2 ⎠
t 2 + 1 + 3 − 4 t + t 2 + 1 + 8 − 6t = 1
where t ≥ 0.
SOLVED EXAMPLES ⇒ |t – 2| + |t – 3| = 1,
⎛ 1⎞ where t ≥ 0. This equation will be satisfied if 2 ≤ t ≤ 3.
34. If the expression ⎜ ax − 1 + ⎟ is non-negative for all
⎝ x⎠ Therefore, 2 ≤ x − 1 ≤ 3 or 5 ≤ x ≤ 10.
positive real x, then the minimum value of a must be
1 ∴ The given equation is satisfied for all values of x
(A) 0 (B)  lying in [5, 10].
2
1 37. If the roots of the equation
(C)  (D)  None of these
4 x2 – 2ax + a2 + a – 3 = 0
Solution: (C) are real and less than 3, then
We have, (A) a < 2 (B)  2≤a≤3
1 ax 2 − x + 1 (C) 3 < a ≤ 4 (D)  a > 4
ax – 1 + ≥0⇒ ≥ 0
x x Solution: (A)
⇒ 2
ax – x + 1 ≥ 0 as x > 0 We can write the given equation as

It will hold if a > 0 and D ≤ 0 (x – a)2 = 3 – a


1 This shows that a ≤ 3 and x = a ± 3 − a
⇒ a > 0 and 1 – 4a ≤ 0 ⇒ a ≥
4 Both the roots of the given equation will be less than
1 3 if the larger of the two roots is less than 3, that is, if
Therefore, the minimum value of a is .
4
a + 3 − a < 3
2
35. If α, β are the roots of the equation x – 3x + a = 0, a
⇒ 3 − a – (3 – a) < 0
∈ R and α < 1 < β, then a belong to
⎛ 9⎞ ⇒ 3 − a (1 − 3 − a ) < 0
(A)  ⎜ − ∞, ⎟ (B) (–∞, 2)
⎝ 4⎠ ⇒ a < 3 and 1 − 3 − a < 0
⎛9 ⎞
(C) (2, ∞) (D)  ⎜⎝ , ∞⎟⎠ But 3 − a > 1 ⇒ 3 – a > 1 or a < 2
4
Solution:  (B, C) Thus, a < 3 and a < 2 ⇒ a < 2
Since 1 lies between the roots of the given equation,
therefore, D > 0 and f (1) < 0 ⇒ 9 – 4a > 0 and 1 – 3 + 38. If f (x) = x2 + 2bx + 2c2 and g(x) = – x2 – 2cx + b2 such
a<0 that min f (x) > max g(x), then the relation between
9 b and c is:
⇒ a< and a < 2
4 (A) |c| < |b| 2 (B) 0 < c < b 2
∴ a < 2 (C) |c| < |b| 2 (D)  |c| > |b| 2
Quadratic Equations and Expressions  3.13

Solution: (D) ⇒ x2 – 2ax – a2 = 0
2 2
D 4b − 8c ⇒ x = (1 + 2 ) a, (1 − 2 ) a
min f (x) = – =–
4a 4     x ≥ a and a ≤ 0
2 2
= –(b – 2c ) (upward parabola)
∴ x = (1 − 2 ) a.
D 4c 2 + 4b 2
max g(x) = – = 41. If f (x) = x – [x], x (≠0) ∈ R, where [x] is the greatest
4a 4 
2 2
integer less than or equal to x, then the number of solu-
=b +c  (downward parabola)
⎛ 1⎞
tions of f (x) + f ⎜ ⎟ = 1 are
Now 2c2 – b2 > b2 + c2 ⎝ x⎠
⇒ c2 > 2b2 ⇒ |c| > 2 |b| (A) 0 (B) 1 (C) infinite   (D) 2
39. For the equation |x2| + |x| – 6 = 0, the roots are Solution: (C)
(A)  real and equal ⎛ 1⎞
We have, f (x) + f ⎜ ⎟ = 1
(B)  real with sum 0 ⎝ x⎠
(C)  real with sum 1 1 ⎡1⎤
(D)  real with product 0 ⇒ x – [x] + − = 1
x ⎢⎣ x ⎥⎦
Solution: (B) 1 ⎡1⎤
⇒ x + − 1 = [x] + ⎢ ⎥
For, x < 0, |x| = – x x ⎣x⎦ 
∴ equation is 2
x +1− x
⇒ = (integer) k (say)
x2 – x – 6 = 0 ⇒ x = – 2, 3 x
  x < 0 ⇒ x2 – (k + 1) x + 1 = 0
∴ x = –2 is the solution Since x is real, so (k + 1)2 – 4 ≥ 0
For, x ≥ 0, |x| = x, ⇒ k2 + 2k – 3 ≥ 0  ⇒ (k + 3) (k – 1) ≥ 0
∴ equation is ⇒ k ≤ – 3  or  k ≥ 1
2
x + x – 6 = 0 ⇒ x = 2, –3 Therefore, number of solutions is infinite.
  x ≥ 0
42. If (log5x)2 + log5x < 2, then x belongs to the interval
∴ x = 2 is the solution.
⎛ 1 ⎞ ⎛1 1 ⎞
Hence, x = 2, –2 are the solutions and their sum is zero. (A)  ⎜ , 5⎟ (B) 
⎜⎝ 5 , ⎟
⎝ 25 ⎠ 5⎠
40. If a ≤ 0, then the root of the equation (C) (1, ∞) (D)  None of these
x2 – 2a | x – a | – 3a2 = 0 is
Solution: (A)
(A) (1 − 2 )a (B)  ( − 1 + 6 )a We have, (log5 x)2 + log5 x < 2
(C) (1 + 2 )a (D) 
− (1 + 6 )a Put log5 x = a then a2 + a < 2
Solution:  (A, B) ⇒ a2 + a – 2 < 0
If x – a < 0, |x – a| = –(x – a) ⇒ (a + 2) (a – 1) < 0
∴ equation becomes x2 + 2a (x – a) – 3a2 = 0
⇒ –2 < a < 1 or –2 < log5 x < 1
⇒ x2 + 2ax – 5a2 = 0
∴ 5–2 < x < 5
⇒ x = – (1 + 6 ) a, ( − 1 + 6 ) a 1
  x < a ≤ 0 i.e., < x < 5
25
∴ x = ( − 1 + 6 ) a 43. The greatest negative integer satisfying x2 – 4x –
77 < 0 and x2 > 4 is
If x – a ≥ 0, |x – a| = x – a
(A) –4 (B) –7
∴ the equation becomes x2 – 2a (x – a) – 3a2 = 0 (C)  –6 (D)  None of these
3.14  Chapter 3

Solution: (C) 46. The solution set of the inequality log ⎛ π ⎞ (x2 – 3x


sin ⎜ ⎟
+ 2) ≥ 2 is ⎝ 3⎠
We have, x2 – 4x – 77 < 0 and x2 – 4 > 0
⎛1 ⎞ ⎛ 5⎞
⇒ (x + 7) (x – 11) < 0 and (x – 2) (x + 2) > 0 (A)  ⎜ , 2⎟ (B) 
⎜⎝1, ⎟⎠
⎝2 ⎠ 2
⇒ –7 < x < 11 and x < – 2 or x > 2
⎡1 ⎞ ⎛ 5⎤
∴ – 7 < x < –2 (C)  ⎢ , 1⎟ ∪ ⎜ 2, ⎥ (D)  None of these
⎣2 ⎠ ⎝ 2⎦
44. The solution of the inequation 4–x + 0.5 – 7.2–x < 4,
Solution: (C)
x ∈ R, is
(A) (–2, ∞) (B)  (2, ∞) We have, log ⎛π⎞
(x2 – 3x + 2) ≥ 2
sin ⎜ ⎟
⎝ 3⎠
⎛ 7⎞ 3
(C)  ⎜ 2, ⎟ (D)  None of these ⇒ x2 – 3x + 2 ≤
⎝ 2⎠
4
Solution: (A) 5
⇒ ≤ 0 x2 – 3x +
The given inequation is 4
⇒ 4x2 – 12x + 5 ≤ 0
4–x + 0.5 – 7.2–x < 4, x ∈ R
Let 2–x = t ⇒ 4x2 – 10x – 2x + 5 ≤ 0
∴ 2t2 – 7t < 4 ⇒ (2x – 5) (2x – 1) ≤ 0
⇒ 2t2 – 7t – 4 < 0 1 5
⇒ ≤ x ≤ (1)
2 2
⇒ (2t + 1) (t – 4) < 0 2
Also, x – 3x + 2 > 0
1
⇒ – < t < 4 ⇒ (x – 1) (x – 2) > 0
2
but 2–x > 0 ⇒ x < 1  or  x > 2 (2)
so 0 < t < 4
From Eqs (1) and (2), we get
⇒ 0 < 2–x < 4
⎡1 ⎞ ⎛ 5⎤
⇒ –2 < x < ∞ or x ∈ (–2, ∞) x ∈ ⎢ , 1⎟ ∪ ⎜ 2,
⎣ 2 ⎠ ⎝ 2 ⎥⎦
x x
⎛ 1⎞ ⎛ 1⎞ 47. The values of a which make the expression x2 – ax +
45. The real values of x for which 372 ⎜ ⎟ ⎜ ⎟ > 1,
are
⎝ 3⎠ ⎝ 3⎠ 1 – 2a2 always positive for real values of x, are
(A) x ∈ [0, 64] (B)  x ∈ (0, 64) 2 2 2 2
(A) − < a < ≤a≤
(B) –
(C) x ∈ [0, 64) (D)  None of these 3 3 3 3
2
Solution: (C) (C) a < 1 (D)  0<a<
3
The given inequation is valid only when Solution: (A)
x ≥ 0 (1) Since the coefficient of x2 is 1 which is positive, there-
The given inequation can be written in the form fore the given expression is positive for all real values
of x if D < 0.
372 − x − x
> 1
⇒ (–a)2 – 4 (1 – 2a2) < 0
⇒ 72 – x – x > 0 (∵ 3 > 1)
⇒ 9a2 – 4 < 0
⇒ x+ x – 72 < 0
⇒ (3a + 2) (3a – 2) < 0
⇒ ( x + 9) ( x – 8) < 0 2 2
⇒ – < a < .
But x + 9 > 0 for all x ≥ 0 3 3

∴ x –8<0⇒ x < 8 48. If the roots of the equation x2 – 2ax + a2 + a – 3 = 0 are


real and less than 3, then
∴ 0 ≤ x < 64  [from (1)].
Quadratic Equations and Expressions  3.15

(A) a < 2 (B)  2≤a≤3 1


⇒ t+ ≥ 2 2 (where t = 2x)
(B) 3 ≤ a ≤ 4 (D)  a > 4 t
Solution: (A) ⇒ t2 – 2 2 t + 1 ≥ 0
Since the roots of the given equation are real ⇒ [t − ( 2 − 1)] [t − ( 2 + 1)] ≥ 0
2
∴ B – 4AC ≥ 0
⇒ t≤ 2 − 1  or t ≥ 2 +1 
2 2
⇒ 4a – 4 (a + a – 3) ≥ 0 but t > 0
⇒ –a + 3 ≥ 0  or  a ≤ 3 (1) ⇒ 0 < 2x ≤ 2 − 1  or 2x ≥ 2 +1 
Since the root is less than 3, so f (3) > 0 ⇒ –∞ < x ≤ log2 ( 2 − 1) 
2 2
⇒ 3 – 2a (3) + a + a – 3 > 0 or x ≥ log2 ( 2 + 1) 
⇒ a2 – 5a + 6 > 0 or (a – 2) (a – 3) > 0  (but not acceptable as x < 0)

⇒ a < 2  or  a > 3 (2) ⎡1 ⎞


∴ x ∈ (–∞, log2 ( 2 − 1) ] ∪ ⎢ , ∞⎟
From Eq. (1) and (2), we have a < 2. ⎣2 ⎠ 
2
51. If a < b, then the solution of x + (a + b) x + ab < 0, is
49. The value of k for which the number 3 lies between the given by
roots of the equation x2 + (1 – 2k) x + (k2 – k – 2) = 0
(A) x < –b or x < –a (B)  a<x<c
is given by,
(B) x < a or x > b (D) –b < x < –a
(A) 2 < k < 5 (B)  k < 2
(C) 2 < k < 3 (D)  k > 5 Solution: (D)

Solution: (A) x2 + (a + b) x + ab < 0
Let f (x) = x2 + (1 – 2k) x + k2 – k – 2 ⇒ (x + a) (x + b) < 0
The number 3 lies between the roots of the given ⇒ x + a < 0, x + b > 0
­equation, if f (3) < 0. or x + a > 0, x + b < 0
2
Now, f (3) = 9 + (1 – 2k) 3 + k – k – 2 ⇒ x < –a, x > –b
2
= 10 – 7k + k = k – 7k + 10 2 or x > ­–a, x < –b
⇒ –b < x < –a
Hence f (3) < 0
or –a < x < –b
⇒ k2 – 7k + 10 < 0
Since a < b
⇒ (k – 2) (k –­5) < 0 ⇒ 2 < k < 5.
∴ –a > –b
50. Solution of 2x + 2|x| ≥ 2 2 is Hence –b < x < –a.
(A) (–∞, log2 ( 2 + 1) 52. The conditions that the equation ax2 + bx + c = 0 has
both the roots positive is that
(B)  ⎣⎡log 2 ( )
2 + 1 , ∞)
(A) a and b are of the same sign
⎛1 ⎞ (B) a, b and c are of the same sign
(C)  ⎜ , log 2 ( 2 − 1)⎟ (C) a and c are of the same sign opposite to that of b
⎝2 ⎠
(D) b and c are of the same sign opposite to that of a
⎡1 ⎞
(D) (–∞, log2 ( 2 − 1) ] ∪ ⎢ , ∞⎟ Solution: (C)
⎣2 ⎠
Solution: (D) Since both the roots are positive
We have, 2x + 2x ≥ 2 2 (x ≥ 0) −b c
∴ >0, >0
a a 
1
⇒ 2x ≥ 2 ⇒x≥ b c
2 ⇒ <0, >0
a a 
and 2x + 2–x ≥ 2 2 (x < 0) ∴ a and c have same sign opposite to that of b.
3.16  Chapter 3

53. The smallest value of x2 – 3x + 3 in the interval To Find the Values of a Rational Expression in
⎛ 3⎞ x, Where x is Real
⎜⎝ − 3, ⎟⎠ is
2
(A) –20 (B) –15 QUICK TIPS
3

(C) 5 (D)  ■ Put the given rational expression equal to y and form the
4 quadratic equation in x.
Solution: (D) ■ Find the discriminant D of the quadratic equation obtained
2
⎛ 3⎞ 9 in step 1.
We have, x2 – 3x + 3 = ⎜ x − ⎟ + 3 −
⎝ 2 ⎠ 4 ■ Since x is real, therefore, put D ≥ 0. We get an inequation

2
in y.
⎛ 3⎞ 3 ■ Solve the above inequation for y. The values of y so
= ⎜x − ⎟⎠ +
⎝ 2 4 obtained determine the set of values attained by the given
3 rational expression.
∴ smallest value = , which lies in the interval
4
⎛ 3 ⎞
⎜⎝ − 3, ⎟⎠ .
2
QUICK TIPS
RATIONAL ALGEBRAIC EXPRESSION The general quadratic expression ax2 + 2hxy + by2 + 2gx +
2fy + c in x and y may be resolved into two linear rational
P (x ) factors if
An expression of the form where P(x) and Q(x) are
Q (x ) abc + 2fgh – af 2 – bg2 – ch2 = 0
polynomials and Q(x) ≠ 0, is known as a rational algebraic a h g
expression.
or h b f =0
Sign Scheme for a Rational Algebraic Expression g f c
in x ■ If sum of coefficients of a polynomial equation a0 + a1x +
a2x2 + … + an xn = 0 is zero, then x = 1 is always atleast
Step 1: Factorise the numerator and denominator of the one root of equation e.g., if a(b – c) x2 + b(c – a) x + c (a
given rational expression into linear factors. Make – b) = 0, then as Σa (b – c) = 0, x = 1 is atleast one root
the coefficient of x positive in all factors. of this equation.
Step 2: Find the real values of x by equating all the factors 2 2
■ Least value of the expression (x – y) + (y – z) + (z – x)
2

to zero. is 0.
Step 3: If n distinct real values of x are obtained then the n
■ Sum of real roots of the equation an |x| + an – 1 |x|
n–1
+
entire line will be divided into (n + 1) parts. … + a0 = 0 is 0, e.g. if |x| = 2 satisfies the equation, then
Step 4: Plot all these points on the number line in order. x = 2 and x = –2 are real roots, their sum is 0.
2 1
Step 5: Start with ‘+’ sign from extreme right and change ■ Length of latus rectum of parabola y = ax + bx + c is .
a
the sign alternatively in other parts.
– + – + – +
–∞ ∞
SOLVED EXAMPLES

ERROR CHECK 54. The sum of the real roots of the equation

If the rational expression in x occurs under modulus sign, |x – 2|2 + |x – 2| – 2 = 0 is


then first of all remove the modulus sign and then proceed. (A) 2 (B) 6 (C) 4 (D) 8
In order to remove the modulus sign, the following
results may be useful: Solution: (C)
■ |x| = k ⇔ x = ±k
Put |x – 2| = t.
■ |x| < k ⇔ = – k < x < k

■ |x| > k ⇔ = x < – k or x > k.


The given equation becomes
t2 + t – 2 = 0 or (t + 2) (t – 1) = 0
Quadratic Equations and Expressions  3.17

Since t + 2 = |x – 2| + 2 > 0 57. If x is real, then the maximum value of 3 – 6x – 8x2 is


∴ we get t – 1 = 0 17 33
(A)  (B) 
⇒ |x – 2| = 1 ⇒ x – 2 = ±1 8 8
21
⇒ x = 3, 1. (C)  (D)  None of these
8
Thus, the sum of roots is 4.
Solution: (B)
x 2 + 2 x − 11 Let y = 3 – 6x – 8x2
55. If x is real, the expression takes all real
x −3
values except those which lie between a and b, then then 8x2 + 6x + y – 3 = 0.
a and b are Since x is real,
(A)  –12, –4 (B)  –12, 2 (C)  4, 12 (D)  –4, 4
∴ 62 – 4 ⋅ 8 (y – 3) ≥ 0,
Solution: (C)
or 36 – 32y + 96 ≥ 0
x 2 + 2 x − 11
Let y = or 32y ≤ 132
x −3 
⇒ xy – 3y = x2 + 2x – 11 132
∴ y≤
32 
⇒ x2 + (2 – y) x + (3y – 11) = 0 (D ≥ 0)
33
2 or y≤
⇒ (2 – y) – 4 (3y – 11) ≥ 0 8 
⇒ 4 + y2 – 4y – 12y + 44 ≥ 0 33
Hence, maximum value of y = .
⇒ y2 – 16y + 48 ≥ 0 8
1 − x + x2
⇒ y2 – 12y – 4y + 48 ≥ 0 58. For all real x, the maximum value of is
1 + x + x2
⇒ (y – 4) (y – 12) ≥ 0 1
(A) 0 (B)  (C) 1 (D) 3
⇒ y ≤ 4  or  y ≥ 12 3
Solution: (B)
x 2 − 3x + 4 1 − x + x2
56. For real values of x, the expression 2 lies Let z=
between x + 3x + 4 1 + x + x2 
1 1 ⇒ z + zx + zx2 = 1 – x + x2
(A) − and 7 (B)  and 7
7 7 ⇒ zx2 – x2 + zx + x + z – 1 = 0
1
(C)  and 3 (D)  None of these ⇒ x2 (z – 1) + x (z + 1) + (z – 1) = 0
3
Solution: (B) For real x,
x 2 − 3x + 4 B2 – 4AC ≥ 0
Let y= 2
x + 3x + 4  ⇒ (z + 1)2 – 4 (z – 1) (z – 1) ≥ 0
⇒ yx2 + 3xy + 4y = x2 – 3x + 4
⇒ z2 + 2z + 1 – 4z2 + 8z – 4 ≥ 0
2
⇒ (y – 1) x + 3 (y + 1) x + 4 (y – 1) = 0
⇒ – 3z2 + 10z – 3 ≥ 0
Since x is real,
⇒ – 3z2 + 9z + z – 3 ≥ 0
∴ discriminant ≥ 0
⇒ – 3z (z – 3) + 1 (z – 3) ≥ 0
⇒ 9 (y + 1)2 – 16 (y – 1)2 ≥ 0
⇒ (z – 3) (– 3z + 1) ≥ 0
⇒ 9 (y2 + 2y + 1) – 16 (y2 – 2y + 1) ≥ 0 1
2 2 ⇒ ≤ z ≤ 3
⇒ –7y + 50y – 7 ≥ 0 ⇒ 7y – 50y + 7 ≤ 0 3
⎛ 1⎞ 1 1
⇒ (y – 7) ⎜ y − ⎟ ≤ 0 ⇒ ≤ y ≤ 7. ∴ minimum value of z = .
⎝ 7⎠ 7 3
3.18  Chapter 3

x2 − 2x + 4 Let 3x + 1 = y
59. Given that, for all real x, the expression 2
x + 2x + 4 Then y ∈ R for all x ∈ R.
1
lies between and 3. The values between which the
3 9 ⋅ 32 x + 6 ⋅ 3x + 4 32 x + 2 + 2 ⋅ 3x + 1 + 4
∴ =
9 ⋅ 32 x + 6 ⋅ 3x + 4 9 ⋅ 32 x − 6 ⋅ 3x + 4 32 x + 2 − 2 ⋅ 3x + 1 + 4 
expression lies are
9 ⋅ 32 x − 6 ⋅ 3x + 4 y2 + 2 y + 4
(A)  0 and 2 (B)  –1 and 1 =
y2 − 2 y + 4 
1
(C)  –2 and 0 (D)  and 3. From Eq. (1),
3
1 y2 + 2 y + 4
Solution: (D) < < 3
3 y2 − 2 y + 4
1 x2 − 2x + 4
Given < < 3 for all x ∈ R. 1 9 ⋅ 32 x + 6 ⋅ 3x + 4
3 x2 + 2x + 4 ∴ < < 3.
3 9 ⋅ 32 x − 6 ⋅ 3x + 4
1 x2 + 2x + 4
⇒ < < 3 for all x ∈ R.(1)
3 x2 − 2x + 4
Quadratic Equations and Expressions  3.19

NCERT EXEMPLARS
1. sin x + i cos 2x and cos x – i sin 2x are conjugate to 9. If x, y ∈R, then x + iy is a non-real complex number, if
each other for (A) x = 0 (B)  y = 0
(A) x = nπ (B)  x =  n + 1  π (C) x ≠ 0 (D)  y≠0
 2 2
10. If a + ib = c + id, then
(C) x = 0 (D)  No value of x
1 − i sin α (A) a2 + c2 = 0
2. The real value of a for which expression is (B) b2 + c2 = 0
1 + 2i sin α
(C) b2 + d2 = 0
purely real is
(D) a2 + b2 = c2 + d2
π π
(A)  ( n + 1) (B)  ( 2n + 1)
2 2 11. The complex number z which satisfies the condition
(C) nπ (D)  None of these i+z
= 1 lies on
z i−z
3. If z = x + iy lies in the third quadrant, then also lies
z (A) circle x2 + y2 = 1 (B)  the X-axis
in the third quadrant, if (C) the Y-axis (D)  the line x + y = 1
(A) x > y > 0 (B)  x < y < 0 12. If z is a complex number, then
(C) y < x < 0 (D)  y > x > 0
2
(A)  z 2 > z (B) 
z2 = z
4. The value of (z + 3) ( z + 3) is equivalent to
(A) |z + 3|2 (B)  |z – 3| (C)  z 2 < z
2
(D)  z 2 ≥ z
2

(C) z2 + 3 (D)  None of these


x
1+ i 13. z1 + z2 = z1 + z2 is possible, if
5. If  1 − i  = 1, then
1
z2 =
(A)  z2 + z1 (B) 
(A) x = 2n + 1 (B)  x = 4n z1
(C) x = 2n (D)  x = 4n + 1
(C)  arg (z1) = arg (z2) (D)  z1 = z2
6. A real value of x satisfies the equation
 3 − 4ix  1 + i cos θ
 3 + 4ix  = α − i β (α , β ∈ R ) , if α + β is equal to
2 2
14. The real value of θ for which the expression
  1 − 2i cos θ
is a real number is
(A) 1 (B) – 1
(C) 2 (D) – 2 π n π
nπ + ( −1)

NCERT EXEMPLARS
(A) nπ + (B) 
4 4
7. Which of the following is correct for any two complex
π
number z1 and z2? (C)  2nπ ± (D)  None of these
2
(A)  z1 z2 = z1 z2
15. The value of arg(x), when x < 0 is
(B)  arg (z1z2) = arg (z1) · arg (z2) π
(A)  0 (B) 
(C)  z1 + z2 = z1 + z2 2
(D)  z1 + z2 ≥ z1 − z2 (C)  π (D)  None of these

8. The point represented by the complex number (2 – i) is 7− z


16. If ƒ ( z ) = , where z = 1 + 2i, then ƒ ( z ) is equal to
π 1 − z2
rotated about origin through an angle in the clock-
2 z
wise direction, the new position of point is (A)  z
(B) 
2
(A)  1 + 2i (B)  – 1 – 2i
(C)  2 z (D)  None of these
(C)  2 + i (D)  – 1 + 2i
3.20  Chapter 3

ANSWER K EYS
  1. (D) 2. (C) 3. (B) 4.  (A) 5. (B) 6.  (A) 7. (A) 8. (B) 9. (D)  10.  (D)
  11. (B) 12.  (B) 13. (C) 14.  (C) 15. (C) 16.  (A)

HINTS AND EXPLANATIONS

1. Let z = sin x + I cos2x


x2 − y2 −2 xy
and z = sin x − i cos 2 x  (i) ∴
< 0 and <0
x2 + y2 x2 + y2

Given that, z = cos x − i sin 2 x  (ii)
x2 – y2 < 0 and – 2 xy < 0
∴ sin x – I cos2x = cos x – i sin2x

⇒ x2< y2 and xy > 0
⇒ sin x = cos x and cos2x = sin2x

So, x < y < 0
⇒ tan x = 1 and tan 2x = 1

4. Given that, (z + 3) ( z + 3)
π
⇒ tan x = tan π and tan 2 x = tan
Let z = x + iy
4 4
π π
( )
⇒ ( z + 3) z + 3 = ( x + iy + 3) ( x + 3 − iy )
⇒ x = nπ +
and 2 x = nπ +
4 4
= (x + 3) – (iy)2 = (x + 3)2 + y2
2

⇒ 2x – x = 0 ⇒ x = 0

= |x + 3 + iy|2 = |z + 3|2

1 − i sin α 1 + i x
2. Given expression, Z= 5. Given, that   =1
1 + 2i sin α 1 − i 
(1 − i sin α ) (1 − 2i sin α )  (1 + i ) (1 + i ) x 1 + 2i + i 2 x
=

(1 + 2i sin α ) (1 − 2i sin α ) ⇒ 
 =1⇒  =1
 (1 − i ) (1 + i )   1 − i 2 
1 − i sin α − 2i sin α + 2i 2 sin 2 α
HINTS AND EXPLANAT I O N S


=  2i x  2i x
1 − 4i 2 sin 2 α ⇒ 
 =1⇒  =1
1 + 1 2
1 − 3i sin α − 2i sin 2 α
=

1 + 4 sin 2 α ⇒ ix = 1 ⇒ ix = (i4n)
[∴ i4n = 1, n ∈ R]
2 ⇒ x = 4n

= 1 − 2 sin α − 3i sin α

1 + 4 sin 2 α 1 + 4 sin 2 α
6. Given equation,  3 − 4ix  = α − iβ (α , β ∈ R )

It is given that z is a purely real.  3 + 4ix 
 
−3 sin α

=0 ⇒  3 − 4ix  = α − iβ

1 + 4 sin 2 α  3 + 4ix 
 
⇒ – 3 sin α = 0 ⇒ sin α = 0
( 3 − 4ix ) ( 3 − 4ix ) = 9 + 16i 2 x 2 − 24ix
α = nπ Now, (α − iβ ) =

3. Given that, z = x + iy lies in third quadrant. ( 3 + 4ix ) ( 3 − 4ix ) 9 − 16i 2 x 2
x <0 and y < 0. 9 − 16 x 2 − 24ix
⇒ α − iβ =

z x − iy ( x − iy ) ( x − iy ) x 2 − y 2 − 2ixy 9 + 16 x 2

Now, = = =
z x + iy ( x + iy ) ( x − iy ) x2 + y2 9 − 16 x 2 i 24 x
⇒ α − iβ =
2  − (i)
9 + 16 x 2
9 + 16 x
2 2

z x −y 2ixy 9 − 16 x 2
i 24 x 
= − ∴ α + iβ =
− (ii)
z x2 + y2 x2 + y2 9 + 16 x 2 9 + 16 x 2
z

Since, also lies in third quadrant. 2 2
So, (α − iβ ) (α + β ) =  9 − 16 x  −  i 24 x 
2
z
 9 + 16 x 2   9 + 16 x 2 
 
Quadratic Equations and Expressions  3.21

12. If z is a complex number, then z = x + iy


2 2 81 + 256 x 4 − 288 x 2 + 576 x 2
∴ α + β =

( 9 + 16 x )
2
2
z = x + iy and z 2 = x + iy 2

⇒ z 2 = x 2 + y 2 
(i)
81 + 256 x 4 + 288 x 2
=

( 9 + 16 x ) x 2 2
and z 2 = x 2 − y 2 + i 2 xy

z 2 = x 2 − y 2 + i 2 xy
( 9 + 16 x )
2
2

=
=1 ⇒ z 2 =
(x 2
)
− y 2 ( 2 xy )
2

( 9 + 16 x )
2
2

⇒ z 2 = x 4 + y 4 − 2 x 2 y 2 + 4 x 2 y 2

7. Let z1 = r1 ( cosθ1 + i sin θ1 )
(x )
2

|z1| = r1  (i) ⇒ z 2 = x 4 + y 4 − 2 x 2 y 2 =
2
+ y2


and z2 = r2 ( cos 2 + i sin θ 2 ) 2 2 2
⇒ z = x + y  (ii)


z2 = r2  (ii)
From Eqs. (i) and (ii).
2
z = z2

Now, z1z2 = r1r2

 θ1 cosθ 2 + i sin θ1 cosθ 2 + i cos θ1 sin θ 2 + i 2 sin θ1 sin θ 2 


cos 13. Given that, z1 + z2 = z1 + z2
 
= r1r2 cos (θ1 + 2 ) + i sin ( 1 +θ 2 ) 
⇒ r ( cos + i sin θ ) + r ( cos + i sin )

  1 1 1 2 2 2
⇒ z1z2 = r1r2
= r1 ( cosθ1 + i sin θ1 ) + r2 ( cosθ 2 + i sin θ 2 )
∴ z1z2 = z1 z2  [using Eqs. (i) and (ii)]
⇒ ( r1 + cosθ1 + r2 cosθ ) + i ( r1 sin θ1 + r2 sin θ 2 ) = r1 + r2
8. Given that, z = 2 – i
π ⇒ r12 cos 2 θ1 + r22 cos 2 θ 2 + 2r1r2 cosθ1 + r12 sin 2 θ1 + r22 sin 2 θ 2
It is rotated about origin through an angle in the clock-
wise direction 2 2 + 2r1r2 sin θ1 sin θ 2 = r1 + r2
∴ New position = ze −iπ / 2 = ( 2 − i ) e −iπ / 2

HINTS AND EXPLANAT I O N S



⇒ r12 + r22 + 2r1r2 cos (θ1 − θ 2 )  = r1 + r2
 
  −π   −π  
= ( 2 − i ) cos 
2  + i sin  2   = ( 2 − i ) [0 − i ]
On squaring both sides, we get
    
= – 2i – 1 = –1 – 2i r12 + r22 + 2r1r2 cos (θ1 − θ 2 ) = r12 + r22 + 2r1r2
9. Given that, x, y ∈ R
⇒ 2r1r2 1 − cos (θ1 − θ 2 )  = 0

The, x + iy is non-real complex number if and only if y ≠ 0.  
10. Given that, a + ib = c + id ⇒ 1 − cos (θ1 − θ 2 ) = 1

⇒ |a + ib| = |c + id|
⇒ cos (θ1 − θ 2 ) = 1


a2 + b2 = c2 + d 2
⇒ cos (θ1 − θ 2 ) = cos 0°


On squaring both sides, we get

a2 + b2 = c2 + d2 ⇒ θ1 − θ 2 = 0°

i+z ⇒ θ1 = θ 2

11. Given that, =1
i−z

Let z = x + iy ∴ arg (z1) = arg (z2)

x + i ( y + 1) x 2 + ( y + 1)
2
14. Given expression

=1⇒ =1
− x − i ( y − 1) x + ( y − 1)
2 2
1 + i cosθ (1 + i cosθ ) (1 + 2i cosθ )
= =
1 − 2i cosθ (1 − 2i cosθ ) (1 + 2i cosθ )
⇒ x 2 + ( y + 1)2 = x 2 + ( y − x )2

⇒ 4y = 0 ⇒ y = 0
1 + i cosθ + 2i cosθ + 2i 2 cos 2 θ

=

So, z lies on X-axis (real axis). 1 − 4i 2 cos 2 θ
3.22  Chapter 3

16. Let z = 1 + 2i

1 + 3i cosθ − 2 cos 2 θ
=
1 + 4 cos 2 θ ⇒
z = 1+ 4 = 5
3 cosθ 7− z 7 −1
For real value of θ ,
=0
1 + 4 cos 2 θ Now, f ( z ) =
=
1 − (1 + 2i )
2 2
1− z
⇒ 3 cosθ = 0

6 − 2i 6 − 2i
π
= =
⇒ cosθ = cos
2
1 − 1 − 4i − 4i 4 − 4i
2
π
=
(3 − i) (2 + 2i)
⇒ θ = 2nπ ±

2 (2 − 2i) (2 + 2i)
15. Let z = x + 0i and x < 0 6 − 2i + 6i − 2i 2 6 + 4i + 2

= =
4 − 4i 2 4+4
z =
(−1)
2
( )
+ 02 = 1
8 + 4i 1

Since, the point (x, 0) represent z = x + 0i lies on the negative
= = 1+ i
8 2
side of real axis.
1
∴ Principal arg (z) = π

f ( z) = 1 + i
2
π
2 1 4 +1 5 z

∴ f ( z) = 1 + = = =
4 4 2 2

X'π Z
X
HINTS AND EXPLANAT I O N S


2


Quadratic Equations and Expressions  3.23

PRACTICE EXERCISES

Single Option Correct Type

3x

1. The roots of the equation 2 x + 2 ⋅ 3 x − 1 = 9 are given by 9. Let a, b, c be positive real numbers, such that bx2 +
⎛ 2⎞ ( ( a + c) 2 + 4b 2 ) x + (a + c) ≥ 0, ∀ x ∈ R, then a, b, c
(A) log2, ⎜ ⎟ – 2 (B)  3, – 3
⎝ 3⎠ are in:
log 3 (A)  G.P. (B)  A.P.
(C)  – 2, 1 – (D)  1 – log23, 2
log 2 (C)  H.P. (D)  None of these
10. If the ratio of the roots of x2 + bx + c = 0 and x2 + qx +
2. If a, b, c are positive real numbers, then the number of r = 0 be the same, then
real roots of the equation ax2 + b |x| + c = 0 is
(A) r2c = b2q (B)  r2b = c2q
(A) 0 (B) 2
(C)  4 (D)  None of these (C) rb2 = cq2 (D)  rc2 = bq2
π
11. If 0 ≤ x < , then the solution of the equation
3. If x2 – x + 1 = 0, then value of x3n is 2
2 2

(A) 0 (B) –1 (C) 1 (D) –1, 1 16sin x


+ 16cos x
= 10 is given by x equal to
4. The number of negative integral solutions of π π π π
(A)  , (B) ,
6 3 3 2
x2 · 2x + 1 + 2|x – 3| + 2 = x2 · 2(|x – 3| + 4) + 2x – 1 is
π π
(A) 4 (B) 2 (C) 1 (D) 0 (C)  , (D)  None of these
2
6 2
5. If α and β (α < β), are the roots of the equation x +
bx + c = 0, where c < 0 < b, then 12. If one of the roots of the equation x2 – (p + 1) x + p2 +
(A) 0 < α < β p – 8 = 0 is greater than 2 and the other root is smaller
(B) α < 0 < β < |α | than 2, then p is such that
(C) α < β < 0 11
(A) − < p < 3 (B)  –2 < p < 3
(D) α < 0 < |α | < β 3
(C) 2 < p < 3 (D)  None of these

PRACTICE EXERCISES
6. If α and β are the roots of x2 + px + q = 0 and α4 and
β 4 are the roots of x2 – rx + s = 0, then the equation 13. If ‘x’ satisfies |x2 – 3x + 2| + |x – 1| = x – 3, then
x2 – 4qx + 2q2 – r = 0 has always
(A) x ∈ ϕ (B)  x ∈ [1,2]
(A)  two real roots
(C) x ∈ [3, ∞) (D)  x ∈ (–∞, ∞)
(B) ­two positive roots
(C)  two negative roots 14. The number of solutions (s) of the equation
(D) ­one positive and one negative root
3 x 2 + 6 x + 7 + 5 x 2 + 10 x + 14 ≤ 4 − 2 x − x 2 is
7. If a, b, c, d and p are distinct real numbers such that
(a2 + b2 + c2) p2 – 2 (ab + bc + cd) p + (b2 + c2 + d 2) ≤ 0 (A) one (B) two (C) four (D) infinite
then a, b, c and d
15. If (a2 – 1) x2 + (a – 1) x + a2 – 4a + 3 = 0 is an identity
(A)  are in A.P. (B)  are in G.P. in x, then the value of a is
(C)  are in H.P. (D) ­satisfy ab = cd
(A) 1 (B) 3 (C) –1 (D) –3
8. Let S denotes the set of all values of x for which the
equation 2x2 – 2 (2a + 1)x + a (a + 1) = 0 has one root 16. Both the roots of the equation (x – b) (x – c) + (x – a)
less than a and other root greater than a, then S equals (x – c) + (x – a) (x – b) = 0 are always
(A)  (0, 1) (B)  (–1, 0) (A) positive (B) negative
(C)  (0, 1/2) (D)  None of these (C)  real (D)  None of these
3.24  Chapter 3

17. If α, β are the roots of the equation x2 + px + q = 0 then 25. If α, β are the roots of the equation 375x2 – 25x – 2 = 0
α n

β
is a root of the equation and Sn = α n + β n, then Lt
n →∞
∑ Sr is
r =1
(A) px2 + (2q – p2) x + p = 0
7 1
(B) qx2 + (p2 – 2q) x + q = 0 (A)  (B) 
12 12
(C) qx2 + (2q – p2) x + q = 0
35
(D)  None of these (C)  (D)  None of these
12
18. If a, b, c ∈ R and quadratic equation x2 + (a + b) x + 26. The solution set of (x)2 + (x + 1)2 = 25, where (x) is the
c = 0 has no real roots then least integer greater than or equal to x, is
(A) c (a + b + c) > 0 (A)  (2, 4) (B)  (–5, 4] ∪ (2, 3]
(B) c + c (a + b + c) > 0 (C)  [–4, –3) ∪ [3, 4) (D)  None of these
(C) c + c (a + b – c) > 0
( 3− x )
(D) c (a + b – c) > 0 27. Number of solutions of log2 (9 – 2x) = 10log 10
is
(A) 1
19. If ax2 + bx + c = 0, a ≠ 0, a, b, c ∈ R has distinct real (B) 2
roots in (1, 2) then a and 5a + 2b + c have (C) 3
(A)  same sign (B)  opposite sign (D)  None of these
(C)  not determined (D)  None of these
28. If ax2 + bx + 6 = 0 does not have two distinct real roots
2
20. If a < 0 the positive root of the equation x – 2a |x – a| a ∈ R, b ∈ R, then the least value of 3a + b is
– 3a2 = 0 is (A) 4 (B) –1 (C) 1 (D) –2
(A) a ( −1 − 6 ) a ( −1 + 6 )
(B)  29. If α, β be the roots of x2 + px – q = 0 and γ, δ be the
(C) a (1 − 2 ) (D)  None of these (α − γ )(α − δ )
roots of x2 + px + r = 0, q + r ≠ 0, then =
(β − γ )(β − δ )
21. If px2 + qx + r = 0 has no real roots and p, q, r are real (A) 1 (B) q (C)  r (D)  q+r
such that p + r > 0, then
(A) p – q + r ≤ 0 (B)  p + r ≥ q x+2 1
30. Number of integral solutions of > is
(C) p + r = q (D)  None of these x2 + 1 2
22. Given lx2 – mx + 5 = 0 does not have two distinct real (A) 0 (B) 1 (C) 2 (D) 3
roots, the minimum value of 5l + m is 31. If the ratio of the roots of λx2 + μx + v = 0 is equal to the
PRACTICE EXERCISES

(A) 5 (B) –5 (C) 1 (D) –1 ratio of the roots of x2 + x + 1 = 0, then λ, μ, v are in


(A)  A.P. (B)  G.P.
23. The set of possible values of λ for which x2 – (C)  H.P. (D)  None of these
(λ2 – 5λ + 5)x + (2λ2 – 3λ – 4) = 0 has roots whose sum
and product are both less than 1 is 32. If c < a < b < d, then roots of the equation bx2 + (1 – b
(c + d)) x + bcd – a = 0
⎛ 5⎞ (A)  are real and one lies between c and a
(A)  ⎜ −1, ⎟ (B)  (1, 4)
⎝ 2⎠ (B)  real and distinct in which one lies between a and b
⎡ 5⎤ ⎛ 5⎞ (C)  real and distinct in which one lies between c and d
(C)  ⎢1, (D) 
⎜⎝1, ⎟⎠ (D)  roots are not real
⎣ 2 ⎥⎦ 2
33. If the roots of the equation ax2 + bx + c = 0 are of the
2 2
24. If 1 lies between the roots of 3x – 3sin θ – 2cos θ = 0 α α +1
then form and , then the value of (a + b + c)2 is
α −1 α
−1 1 −1 (A) b2 – 2ac (B)  b2 – 4ac
(A)  < sin θ < (B)  < sin θ < 0
2 2 2 (C) 4b2 – ac (D) 2b2 – ac
1 34. If α, β be roots of x2 + px + 1 = 0 and γ, δ be the roots
(C)  < sin θ < 1 (D)  None of these
2 of x2 + qx + 1 = 0, then (α – γ ) (β – γ ) (α + δ ) (β + δ ) =
Quadratic Equations and Expressions  3.25

(A) p2 + q2 (B)  p2 – q2 (A) {0} (B) {1}


(C) q2 – p2 (D)  None of these (C) ϕ (D)  None of these

35. If a and b are odd integers then [x]2 + a [x] + b = 0 45. The equation esin x – e–sin x – 4 = 0 has
(where [·] denotes greatest integer function) has (A)  infinite number of real roots
(A)  finite number of roots (B)  no real roots
(B)  infinite number of roots (C)  exactly one real root
(C)  no roots (D)  exactly four real roots
(D)  None of these 46. Suppose the cube x3 – px + q has three distinct real
2
36. If log9 (x – 5x + 6) > log3 (x – 4), x belongs to roots where p > 0 and q > 0. Then which one of the
(A) (–∞, 4) following holds?
p
(B) (4, ∞) (A) The cubic has minima at and maxima at
(C) (–∞, –4) ∪ (4, ∞) 3
p
(D)  no real value of x –
3
p
37. Let a, b, c be real numbers, a ≠ 0. If α is a root of (B) The cubic has minima at – and maxima at
a2x2 + bx + c = 0, β is a root of a2x2 – bx – c = 0 and 3
p
0 < α < β, then the equation a2x2 + 2bx + 2c = 0 has a
root γ  that always satisfies 3
α +β β p p
(A) γ  = (B)  γ  = α + (C) The cubic has minima at both and –
2 2 3 3
(C) γ  = α (D)  α<γ<β
p p
38. Number of solutions of the equation x2 – 2 – 2[x] = 0 (D)  The cubic has maxima at both and –
3 3
([⋅] denotes greatest integer function) is
(A) 1 (B) 2 47. The quadratic equations x2 – 6x + a = 0 and x2 – cx
(C)  3 (D)  None of these + 6 = 0 have one root in common. The other roots of
2 2

39. The number of real roots of the equation 2sin x − 2cos x = the first and second equations are integers in the ratio
1 is 4 : 3. Then the common root is
(A) 2 (B) 1 (A) 1 (B) 4 (C) 3 (D) 2
(C)  infinite (D)  None of these 48. If the roots of the equation bx2 + cx + a = 0 be
40. If the absolute value of the difference of roots of the ­imaginary, then for all real values of x, the expression
equation x2 + px + 1 = 0 exceeds, 3 p , then 3b2x2 + 6bcx + 2c2 is

PRACTICE EXERCISES
(A) p < –1 or p > 4 (B)  p > 4 (A)  greater than 4ab (B)  less than 4ab
(C) –1 < p < 4 (D)  0 ≤ p < 4 (C)  greater than –4ab (D)  less than –4ab
2
41. If the roots of x + ax + b = 0 are c and d, then roots of
49. If x, y ∈ [0, 10], then the number of solutions (x, y) of
x2 + (2c + a) x + c2 + ac + b = 0 are
the inequation 3sec x − 1 9 y 2 − 6 y + 2 ≤ 1 is
2

(A) 1, d – c (B)  0, d – c
(C) 1, c – d (D)  None of these (A) 2 (B) 4 (C) 6 (D) infinite
42. If the equation x2 + 2 (k + 1)x + 9k – 5 = 0 has only 50. The equation (x – n)m + (x – n2)m + (x – n3)m + … +
negative roots, then (x – nm)m = 0 (m is odd positive integer), has
(A) k ≤ 0 (B)  k ≥ 0 (C)  k ≥ 6 (D)  k ≤ 6 (A)  all real roots
(B)  one real and (n – 1) imaginary roots
43. If the product of the roots of the equation x2 – 3kx +
(C)  one real and (m – 1) imaginary roots
2e2 ln k – 1 = 0 is 7, then for real roots the value of k is
(D)  no real root
equal to
(A) 1 (B) 2 (C) 3 (D) 4 51. If x2 – (a + b + c)x + (ab + bc + ca) = 0 has imaginary
44. The solution set of roots, where a, b, c ∈ R+, then a , b , c
x
⎛ 3⎞ 2
(A)  can be the sides of a triangle
⎜⎝ ⎟⎠ = x – x – 9 is (B)  cannot be the sides of a triangle
5
3.26  Chapter 3

(C)  nothing can be said


x +1 ( x + 1) 2
(D)  None of these 60. The solution set of + |x + 1| = is
x x
52. If x1, x2, x3, …, xn are the roots of the equation xn + ax +
b = 0, then the value of (x1 – x2)(x1 – x3)(x1 – x4) … (A) {x | x ≥ 0}
(x1 – xn) is equal to (B) {x | x > 0} ∪ {–1}
(C)  {– 1, 1}
(A) nx1n − 1 + a (B) 
n (x1)n – 1 (D) {x | x ≥ 1 or x ≤ –1}
nx1n − 1 + b
(C) nx1 + b (D)  61. If α, β are the roots of the equation ax2 + bx + c = 0,
(a ≠ 0) and α + δ, β + δ are the roots of Ax2 + Bx +
53. If the roots of the equation x2 – 2ax + a2 + a – 3 = 0 are C = 0, (A ≠ 0) for some constant δ, then
real and less than 3, then
b 2 − 4 ac B 2 − 4 AC
(A) a < 2 (B)  2≤a≤3 (A)  =
(B) 3 ≤ a ≤ 4 (D)  a > 4 a2 A2
2 2
b − 2ac B − 2 AC
(B)  =
54. The value of k for which the number 3 lies between a2 A2
the roots of the equation x2 + (1 – 2k) x + (k2 –
b 2 − 8ac B 2 − 8 AC
k – 2) = 0 is given by (C)  =
a2 A2
(A) 2 < k < 5 (B) 
k<2 (D)  None of these
(C) 2 < k < 3 (D) 
k>5 62. Let a, b, c be real, if ax2 + bx + c = 0 has two real roots
55. The number of solutions of |[x] – 2x| = 4, where [x] is c b
the greatest integer ≤ x, is α and β, where α < – 1 and β > 1 then 1 + + is
a a
(A) 2 (B) 4 (C) 1 (D) infinite (A) < 0 (B)  >0
(C) ≤ 0 (D)  None of these.
56. If a < 0, the positive root of the equation x2 – 2a
|x – a| – 3a2 = 0 is 63. If a, b, c are in G.P., then the equations ax2 + 2bx + c =
(A) a ( −1 − 6 ) (B)  a ( −1 + 6 ) d e
0 and dx2 + 2ex + f = 0 have a common root if , ,
(C) a (1 − 2 ) (D)  None of these f a b
are in
57. Let S denote the set of all values of S for which the c
equation 2x2 – 2 (2a + 1)x + a (a + 1) = 0 has one root (A) H.P. (B) G.P.
less than a and other root greater than a, then S equals (C)  A.P. (D)  None of these
PRACTICE EXERCISES

(A)  (0, 1) (B)  (–1, 0) 64. If the equations x2 + abx + c = 0 and x2 + acx + b = 0
⎛ 1⎞ have a common root, then their other roots satisfy the
(C)  ⎜ 0, ⎟ (D)  None of these
⎝ 2⎠ equation
(A) x2 + a (b + c) x + a2bc = 0
58. Solution of 2x + 2| x | ≥ 2 2 is (B) x2 – a (b + c) x + a2bc = 0
(A) (–∞, log2 ( 2 + 1) (C) x2 – a (b + c) x – a2bc = 0
(B)  (0, 8) (D)  None of these
⎛1 ⎞
(C)  ⎜ , log 2 ( 2 − 1)⎟ 65. If (ax2 + bx + c) y + a'x2 + b'x + c' = 0, then the condi-
⎝2 ⎠
tion that x may be a rational function of y is
⎡1 ⎞ (A) (ac' – a' c)2 = (ab' – a' b) (bc' – b'c)
(D) (–∞, log2 ( 2 − 1) ] ∪ ⎢ , ∞⎟
⎣2 ⎠ (B) (ab' – a ' b)2 = (ac' – a ' c) (bc' – b'c)
(C) (bc' – b ' c)2 = (ab' – a ' b) (ac' – a'c)
59. If f (x) = x2 + 2bx + 2c2 and g(x) = – x2 – 2cx + b2 such
(D)  None of these
that min. f (x) > max. g(x), then the relation between
b and c is 66. If n and r are positive integers such that 0 < r < n, then
(A) |c| < |b| 2 (B) 0 < c < b 2 the roots of the quadratic equation nCr–1 x2 + 2 ⋅ nCr x +
n
(C) |c| < |b| 2 (D) |c| > |b| 2 Cr+1 = 0 are
Quadratic Equations and Expressions  3.27

(A)  real and distinct 74. If a, b are the roots of the equation x2 + px + 1 = 0 and
(B) rational c, d are the roots of the equation x2 + qx + 1 = 0, then
(C)  rational but not integer (a – c) (b – c) (a + d) (b + d) =
(D) imaginary (A) p2 – q2 (B)  q2 – p2
67. If the equations x2 – px + q = 0 and x2 – ax + b = 0 (C) p2 + q2 (D)  2 (p2 – q2)
have a common root and the other root of the second 75. If q ≠ 0 and the equation x3 + px2 + q = 0 has a root of
equation is the reciprocal of the other root of the first, multiplicity 2, then p and q are connected by
then (q – b)2 = (A) p2 + 2q = 0
(A) aq (p – b)2 (B)  bq (p – a)2 (B) p2 – 2q = 0
2
(C) bq (p – b) (D)  None of these (C) 4p3 + 27q + 1 = 0
(D) 4p3 + 27q = 0
68. If the two equations ax2 + bx + c = 0 and 2x2 – 3x +
4 = 0 have a common root, then 76. If the sum of the roots of the quadratic equation ax2 +
(A) 6a = 4b = –3c (B) 3a = –4b = 3c bx + c = 0 is equal to the sum of the squares of their
(C) 6a = –4b = 3c (D)  None of these a b c
reciprocals, then , and are in
c a b
69. If a, b, c are rational and ax2 + bx + c = 0 and 3x2 + (A)  arithmetic progression
x – 5 = 0 have a common root, then 3a + b + 2c = (B)  geometric progression
(A) 0 (B) 1 (C)  harmonic progression
(C)  2 (D)  None of these (D)  arithmetico-geometric progression
70. If ax2 + 2bx + c = 0 and a1x2 + 2b1x + c1 = 0 have a 77. If both the roots of the quadratic equation x2 – 2kx +
a b c k2 + k – 5 = 0 are less than 5, then k lies in the interval
common root and , , are in A.P., then a1, b1,
c are in a1 b1 c1 (A) (–∞, 4) (B)  [4, 5]
1
(A)  A.P. (B)  G.P. (C) (5, 6] (D) (6, ∞)
(C)  H.P. (D)  None of these 78. If for real number a, the equation (a– 2) (x– [x])2 +
71. If x is real, then the minimum value of 2  (x  – [x]) + a2 = 0 (where [x] denotes the greatest
integer ≤ x) has no integral solution and has exactly
( a + x ) (b + x ) one solution in (2, 3), then a lies in the interval
(x > – c), for a > c, b > c is
(c + x ) (A)  (–1, 2) (B)  (0, 1)
(A) ( a − b + c − b ) 2 (C)  (–1, 0) (D)  (2, 3)
79. Let a, b, c be distinct positive numbers such that

PRACTICE EXERCISES
(B) ( a − c + b − c ) 2 each of the quadratics ax2 + bx + c, bx2 + cx + a
(C) ( a − c − b − c ) 2 and cx2 + ax  + b is non-negative for all x ∈ R. If
(D)  None of these a2 + b2 + c2
R= , then
ab + bc + ca
72. If the ratio of the roots of a1x2 + b1x + c1 = 0 be equal
a (A) 1 ≤ R < 4 (B)  1 < R ≤ 4
to the ratio of the roots of a2x2 + b2x + c2 = 0, then 1 , (C) 1 ≤ R ≤ 4 (D)  1 < R < 4
b1 c1 a2
, are in 80. The set of values of a for which the equation (x2 + x
b2 c2 + 2)2 – (a – 3) (x2 +x + 2) (x2 + x + 1) + (a – 4) (x2 +
(A)  A.P. (B)  G.P. x + 1)2 = 0 has at least one real root is
(C)  H.P. (D)  None of these
⎛ 19 ⎞ ⎡ 19 ⎤
73. If α, β be the roots of the equation x2 – px + q = 0 and (A) ⎜ 5, ⎟ ⎢5, 3 ⎥
(B) 
⎝ 3⎠ ⎣ ⎦

α > 0, β > 0, then the value of α1/4 + β1/4 is
⎡ 19 ⎞ ⎛ 19 ⎤
( ) , where k is equal to
k
⎢ , 3 ⎟⎠ (D) 
(C) 5 ⎜⎝ 5, ⎥
p + 6 q + 4 q1/ 4 p+2 q ⎣ 3⎦

1 1 1 81. If all real values of x obtained from the equation


(A) 1 (B)  (C)  (D)  4x – (a – 3)2x + a – 4 = 0 are non-positive, then a
2 3 4
belongs to
3.28  Chapter 3

(A)  [4, 5] (B)  (4, 5] ⎛ 5 − 3 5 ⎤ ⎡5 + 3 5 ⎞


(C)  [ 4, 5) (D)  (4, 5) (C) ⎜ −1, ⎥∪⎢ , 6⎟
⎝ 2 ⎦ ⎣ 2 ⎠
82. Let f (x) = x2 + ax + b be a quadratic polynomial,
(D)  None of these
where a and b are integers. If for a given integer n,
f (n) f (n + 1) = f (m) for some integer m, then the value 88. Let α1, β be the roots of the equation x2 – ax + p = 0
of m is and γ, δ be the roots of the equation x2 – bx + q = 0.
(A)  n (n + 1) + an + b (B)  n (n + 1) + a + bn If α, β, √, δ are in increasing G.P., then the value of
(C)  n (n + 1) + a + b (D)  None of these q+ p
is equal to
x 2 + nx − 2 q− p
83. If for any real x, we have –1 ≤ 2 ≤ 2 , then n b2 − a2 b2 + a2
belongs to x − 3x + 4 (A)  2 (B) 
b + a2 b2 − a2
(A) [− 40 + 6, −1]
b+a b−a
(C)  (D) 
(B) [− 40 + 6, 40 − 6] b−a b+a
(C) [−1, 40 − 6] 1
89. If tn denotes the nth term of an A.P. and tp = and tq
(D)  None of these
1
q
84. If b > a, then the equation (x – a) (x – b) –1 = 0 has = , then which of the following is necessarily a root
p
(A)  both roots in (–∞, a) of the equation (p + 2q – 3r) x2 + (q + 2r – 3p) x + (r
(B)  one root in (–∞, a) and other in (b, ∞) + 2p – 3q) = 0
(C)  both roots in (b, ∞)
(A)  tp (B)  tq
(D)  both roots in [a, b]
(C)  tpq (D)  tp + q
85. The quadratic equation
90. If the roots of the equation 4x2 + 4ax + b = 0 are real
( x + b)( x + c) ( x + c)( x + a) ( x + a)( x + b) and differ at most by a, then b lies in
+ + =1
(b − a)(c − a) (c − b)( a − b) ( a − c)(b − c) ⎛ a2 2 ⎞
⎛ a2 ⎞
has (A) ⎜ 0, ⎟ (B)  ⎜ 2 ,a ⎟
(A)  two real and distinct roots ⎝ 2⎠ ⎝ ⎠
(B)  imaginary roots (C) [0, a2] (D) 
(0, a2)
(C)  equal roots
91. The roots of the equation ax2 + bx + c = 0, where
(D)  infinite roots
a  ∈  R+, are two consecutive odd positive integers,
86. The equation ax4 – 2x2 – (a – 1) = 0 will have real and then
PRACTICE EXERCISES

unequal roots if (A) |b| ≤ 4a (B)  |b| ≥ 4a


(A)  a < 0, a ≠ 1 (B)  a > 0, a ≠1 (C) |b| = 2 a (D)  None of these
(C) 0 < a < 1 (D)  None of these
92. If a, b, c, d are real numbers, then the number of real
2 2
87. If the equation x + [a – 5a + b + 4] x + b = 0 has roots of the equation (x2 + ax – 3b) (x2 – cx + b) (x2 –
roots –5 and 1, where [a] denotes the greatest integer dx + 2b) = 0 are
less than or equal to a, then the set of values of a is (A) 3 (B) 4
⎛ 5−3 5 5 + 3 5⎞ ⎛ 5 + 3 5⎞ (C)  6 (D)  at least 2
(A) ⎜ , (B)  ⎜ 0,
⎝ 2 2 ⎟⎠ ⎝ 2 ⎟⎠

Previous Year’s Questions

93. If α ≠ β with a2 = 5α − 3 and β2 = 5β − 3, then the 94. The number of real roots of 32 x − 7 x + 7 = 9 is
2

equation having α/β and β/α as its roots, is [2002]  [2002]
(A) 3x2 + 19x + 3 = 0 (B)  3x2− 19x + 3 = 0 (A) Zero (B) 2
2
(C) 3x − 19x− 3 = 0 (D)  x2− 16x + 1 = 0 (C) 1 (D) 4
Quadratic Equations and Expressions  3.29

95. If the sum of the roots of the quadratic equation 104. All the values of m for which both roots of the equa-
ax2 + bx + c = 0 is equal to the sum of the squares of tions x2− 2mx + m2− 1 = 0 are greater than –2 but less
a b c than 4, lie in the interval [2006]
their reciprocals, then , and , are in [2003]
c a b (A)  –2 < m < 0 (B)  m > 3
(A)  arithmetic progression. (C)  –1 < m < 3 (D)  1 < m < 4
(B)  geometric progression.
(C)  harmonic progression. 3 x 2 + 9 x + 17
(D) arithmetic-geometric-progression. 105. If x is real, the maximum value of
is
3x 2 + 9 x + 7
2
96. The number of real solutions of the equation x − 3 [2006]
| x | + 2 = 0 is [2003] (A) 1/4 (B) 41
(A) 2 (B) 4 (C) 1 (D) 3 (C) 1 (D) 17/7
97. The value of ‘a’ for which one root of the quadratic
106. If the difference between the roots of the equation
equation (a2−5a + 3) x2 + (3a – 1) x + 2 = 0 is twice
x2 + ax + 1 = 0 is less than 5 , then the set of possi-
as large as the other, is [2003]
ble values of a is [2007]
2 2
(A)  (B)  − (A) (−3, 3) (B)  (−3, ∞)
3 3
(C) (3, ∞) (D)  (−∞, −3)
1 1
(C)  (D)  −
3 3 107. The quadratic equations x2− 6x + a = 0 and x2− cx +
6 = 0 have one root in common. The other roots of
98. If (1 − p) is a root of quadratic equation x2 + px +
the first and second equations are integers in the ratio
(1 − p) = 0, then its roots are [2004]
4 : 3. Then the common root is [2008]
(A)  0, 1 (B)  –1, 2
(A) 1 (B) 4
(C) 0, –1 (D)  –1, 1
(C) 3 (D) 2
99. If one root of the equation x2 + px + 12 = 0 is 4, while
108. If the roots of the equation bx2 + cx + a = 0 be imagi-
the equation x2 + px + q = 0 has equal roots, then the
nary, then for all real values of x, the expression 3b2x2
value of ‘q’ is [2004]
+ 6bcx + 2c2 is [2009]
49
(A)  (B)  4 (A)  greater than 4ab
4
(B)  less than 4ab
(C) 3 (D) 12 (C)  greater than − 4ab
100. If 2a + 3b + 6c = 0, then at least one root of the equa- (D)  less than − 4ab
tion ax2 + bx + c = 0 lies in the interval [2004]

PRACTICE EXERCISES
109. If a and β are the roots of the equation x2−x + 1 = 0,
(A)  (0, 1) (B)  (1, 2) then the value of α2009 + β2009 = [2010]
(C)  (2, 3) (D)  (1, 3) (A)  −1 (B)  1
101. The values of α for which the sum of the squares of (C)  2 (D)  −2
the roots of the equation x2− (a − 2)x − a − 1 = 0 110. The equation esin x –e–sin x – 4 = 0, for x real, has
assume the least value is [2005]  [2012]
(A) 1 (B) 0 (A)  infinite number of roots
(C) 3 (D) 2 (B)  no roots
102. If roots of the equation x2− bx + c = 0 be two (C)  exactly one root
­consectutive integers, then b2− 4c equals [2005] (D)  exactly four roots
(A)  −2 (B)  3 111. The real number k for which the equation,
(C) 2 (D) 1 2 x 3 + 3 x + k = 0 has two distinct real roots in [0, 1]
2
103. If both the roots of the quadratic equation x − 2kx +  [2013]
k2 + k− 5 = 0 are less than 5, then k lies in the interval (A)  lies between 2 and 3
 [2005] (B)  lies between −1 and 0
(A) (5, 6] (B) (6, ∞) (C)  does not exist
(C) (−∞, 4) (D)  [4, 5] (D)  lies between 1 and 2
3.30  Chapter 3

115. If, for a positive integer n, the quadratic equation,


112. If the equations x2 + 2x + 3 = 0 and x(x + 1) + (x + 1) (x + 2) + … + (x + n – 1)(x + n) =
2
ax + bx + c = 0, a, b, c ∈ R have a common root, then 10n has two consecutive integral solutions, then n is
a : b : c is [2013] equal to [2017]
(A)  3 : 2 : 1 (B)  1 : 3 : 2 (A) 9 (B) 10 (C) 11 (D) 12
(C)  3 : 1 : 2 (D)  1 : 2 : 3
116. If α, β ∈ C are the distinct roots, of the equation
113. If a ∈ R and the equation −3 ( x − [ x ]) + 2 ( x − [ x ]) + a = 0 x2 – x + 1 = 0, then α101 + β107 is equal to
2 2 [2018]
(A) –1 (B) 0 (C) 1 (D) 2
3 ( x − [ x ]) + 2 ( x − [ x ]) + a 2 = 0 (where [ x ] denotes the greatest integer ≤ x )
2

117. The value of λ such that sum of the squares of the


has no integral solution, then all possible values of a
roots of the quadratic equation, x2 + (3 – λ) x + 2 =λ
lie in the interval [2014]
has the least value is: [2019]
(A) − ( 1, 0) ∪ (0,1) ( , 2)
(B) 1
(A) 2 (B) 1
( 2, −1) (D) 
(C) − ( −∞, −2) ∪ ( 2, ∞ ) 4 15
(C)  (D) 
9 8
114. Let α and β be the roots of equation x 2 − 6 x − 2 = 0.
a − 2a8
If an = α n − β n, for n ≥ 1,then the value of 10
2a9
is equal to [2015]
(A) − 6 (B)  3
(C) − 3 (D)  6

ANSWER K EYS
Single Option Correct Type
1. (C) 2. (A) 3. (C) 4. (D) 5. (B) 6. (A) 7. (B) 8. (D) 9. (B) 10. (C)
11. (A) 12. (B) 13. (A) 14. (A) 15. (A) 16. (C) 17. (C) 18. (B)
19. (A) 20. (C) 21. (B) 22. (D) 23. (D) 24. (C) 25. (B) 26. (B) 27. (A) 28. (D)
29. (A) 30. (D) 31. (B) 32. (C) 33. (B) 34. (C) 35. (C) 36. (B) 37. (D) 38. (A)
PRACTICE EXERCISES

39. (C) 40. (B) 41. (B) 42. (C) 43. (B) 44. (C) 45. (B) 46. (A) 47. (D) 48. (C)
49. (B) 50. (C) 51. (A) 52. (A) 53. (A) 54. (A) 55. (B) 56. (C) 57. (D) 58. (D)
59. (D) 60. (B) 61. (A) 62. (A) 63. (C) 64. (B) 65. (A) 66. (A) 67. (B) 68. (C)
69. (A) 70. (B) 71. (B) 72. (B) 73. (D) 74. (B) 75. (D) 76. (C) 77. (A) 78. (C)
79. (D) 80. (D) 81. (B) 82. (A) 83. (C) 84. (B) 85. (D) 86. (C) 87. (C) 88. (B)
89. (C) 90. (C) 91. (B) 92. (D)

Previous Years’ Questions


  93. (A) 94. (B) 95. (C) 96. (B) 97. (A) 98. (C) 99. (A) 100. (A) 101. (A) 102. (C)
103. (C) 104. (C) 105. (C) 106. (A) 107. (D) 108. (C) 109. (B) 110. (B) 111. (C) 112. (D)
113. (A) 114. (B) 115. (C) 116. (C) 117. (A)
Quadratic Equations and Expressions  3.31

HINTS AND EXPLANATIONS

Single Option Correct Type


1. We have, 2x + 2 ⋅ 33x /(x – 1) = 9 = 32 7. We have, (a2 + b2 + c2) p2 – 2 (ab + bc + cd) p
3x

⇒ (x + 2) log 2 + log 3 = 2 log 3 + (b2 + c2 + d2) ≤ 0
x −1
⇒ (ap – b)2 + (bp – c)2 + (cp – d)2 ≤ 0
⎛ 3x ⎞
⇒ (x + 2) log 2 + ⎜ − 2⎟ log 3 = 0 ⇒ (ap – b)2 + (bp – c)2 + (cp – d)2 = 0
⎝ x −1 ⎠
(a, b, c, d, p ∈ R)
⎛ 1 ⎞ ⇒ ap – b = 0, bp – c = 0, cp – d = 0
⇒ (x + 2) ⎜ log 2 +
log 3⎟ = 0
⎝ x −1 ⎠
b c d
log 3 ⇒
= = =p
⇒ x = –2 or x = 1 –
. a b c
log 2 ⇒ a, b, c, d are in G.P.
The correct option is (C) The correct option is (B)
2. Since a, b, c are all +ve 8. The required a satisfies the inequality
∴ ax2 + b | x | + c > 0 for all real x 2a2 – 2(2a + 1)a + a(a + 1) < 0
∴ ax2 + b | x | + c ≠ 0 for any real x ⇒ a(a + 1) > 0 ⇒ a ∈ (–∞, –1) ∪ (0, ∞)
∴ no real solution is possible. The correct option is (D)
The correct option is (A) 9. (a + c)2 + 4b2 – 4b (a + c) ≤ 0
3. Since x2 – x + 1 = 0 ⇒ (a – 2b + c)2 ≤ 0
∴ (x – 1) (x2 – x + 1) = 0 ⇒ a – 2b + c = 0
⇒ x3 – 1 = 0 ⇒ 2b = a + c
⇒ x3 = 1, ⇒ a, b, c are in A.P.
∴ x3n = 1 The correct option is (B)
The correct option is (C) 10. Let α, β be the roots of equation x2 + bx + c = 0 and α ′, β ′
4. The given equation can be written as be the roots of the x2 + qx + r = 0. Then,

HINTS AND EXPLANAT I O N S


⎡ 1⎤ α + β = – b; aβ = c, α ′ + β ′ = – q, α ′ β ′ = r.
2 x +1 ⎢ x 2 − ⎥ = 2| x − 3 | ⋅ 4 [4 x 2 − 1]
⎣ 4⎦ α α′ α+β α ′ + β′
It is given that = ⇒ =
⎡ 2 1⎤ β β′ α −β α ′ − β′

= 16 · 2|x – 3| ⎢⎣ x − 4 ⎥⎦ (α + β ) 2
(α ′ + β ′ ) 2
⇒ =

⇒ 2x – 3 = 2|x – 3| (α − β ) 2 (α ′ − β ′ ) 2
1


[ x2 = does not give negative integral value] b2 q2
4 ⇒
2
= 2
⇒ b2r = q2c.
⇒ x – 3 = ± (x – 3) b − 4c q − 4r

The correct option is (C)
⇒ either x – 3 = x – 3 or x – 3 = – x + 3
16
11 Let 16sin x = y, then 16cos x = 161−sin x =
2 2 2

⇒ 2x = 6 or x = 3
y
∴ Given equation does not give any negative integral The given equation becomes
solution. 16
y+ = 10 ⇒ y2 – 10y + 16 = 0 or y = 2, 8
The correct option is (D) y
16sin x = 2 ⇒ 24 sin x = 2(1)
2 2

5. We have α + β = – b, aβ = c Now,
2
As c < 0, b > 0, we get ⇒ 4 sin x = 1
1 π ⎛ π⎞
α<0<β ∴ sin x =± ⇒x=  ⎜⎝∵0 ≤ x < ⎟⎠
2 6 2
Also, β = – b – α < – α = |α | sin x 2

and 16 =8
Thus, α < 0 < β < |α |.
= 23
2

The correct option is (B) ⇒ 24 sin


x

6. The discriminant = 16q2 – 4 (2q2 – r) 3 π ⎛ π⎞


⇒ sin x = ± ⇒x=  ⎜⎝∵ 0 ≤ x < ⎟⎠
= 8q2 + 4r ⇒ D = 8q2 + 4 (α4 + β4) > 0 2 3 2
∴ roots are real.
The correct option is (A)
The correct option is (A)
3.32  Chapter 3

12. The given condition is fulfilled if and only if 18. Since f (x) has no real roots, f (x) has same sign for every x
f (2) = 4 – 2 (p + 1) + p2 + p – 8 < 0 ∴ f (0) · f (1) > 0
⇒ (p – 3) (p + 2) < 0 ⇒ –2 < p < 3 The correct option is (B)
The correct option is (B)
19. Let x1 and x2 be two roots of ax2 + bx + c = 0
2
13. We have, |x – 3x + 2| + |x – 1| = x – 3 1 < x1 < 2 and 1 < x2 < 2
⇒ x ≥ 3 ⇒ x2 – 3x + 2 + x – 1 = x – 3 ⎛ b c⎞
Now a (5a + 2b + c) = a2 ⎜⎝ 5 + 2 a + a ⎟⎠
⇒ x2 – 3x + 4 = 0


2 = a2(5 + 2(–1) (x1 + x2) + x1x2)
⎛ 3⎞ 9
⇒ ⎜ x − ⎟ + 4 − = 0

= a2[(x1 – 2) (x2 – 2) + 1] > 0
⎝ 2⎠ 4
⇒ No solution.

Hence a and 5a + 2b + c are of same sign

The correct option is (A)
The correct option is (A)

14. We have, 20. Since a < 0, in case of positive root of the equation x > a
∴ The equation is x2 – 2a (x – a) – 3a2 = 0
3 x 2 + 6 x + 7 = 3 ( x + 1) 2 + 4 ≥ 2 ⇒ x2 – 2ax – a2 = 0
2a ± 4 a 2 + 4 a 2 2a ± 2 2a

and 5 x 2 + 10 x + 14 = 5( x + 1) 2 + 9 ≥ 3 Thus, the roots are =
2 2
∴ L.H.S. ≥ 5 = a(1 + 2 ) or a(1 − 2 )
R.H.S. = 4 – 2x – x2 = 5 – (x + 1)2 ≤ 5 ∴ the only positive root possible is a(1 − 2 ) .
∴ the equation holds only when The correct option is (C)
L.H.S. = R.H.S. = 5 21. Let α + iβ, α – iβ be the roots
∴ x = – 1. r
Then, α 2 + β 2 = >0
The correct option is (A) p
15. Equating the coefficients of similar powers of x, we get ∴ p, r must be of the same sign.
a 2 − 1 = 0 ⇒ a = ±1 ⎫ Since p + r > 0
⎪ ∴ p, r are both positive.
a − 1 = 0 ⇒ a = 1 ⎬
2 ⎪ If q < r, p – q + r > 0
a − 4 a + 3 = 0 ⇒ a = 1, 3⎭
HINTS AND EXPLANAT I O N S

If q > 0, (p + r)2 – (p – r)2 = 4pr ≥ q2


∴ common value of a = 1.

( roots are non-real)

The correct option is (A) ∴ (p + r)2 ≤ q2 + (p – r)2 ≥ q2

16. The given equation can be written as ∴ p + r ≥ q
3x2 – 2x (a + b + c) + bc + ca + ab = 0 The correct option is (B)
Discriminant = 4 (a + b + c)2 – 12 (bc + ca + ab) 22. Let f (x) = lx2 – mx + 5
= 4 (a2 + b2 + c2 – bc – ca – ab) Since lx2 – mx + 5 = 0 does not have two distinct real roots,
= 2 [(b – c)2 + (c – a)2 + (a – b)2] ≥ 0 therefore either f (x) ≥ 0 ∀ x ∈ R, or f (x) ≤ 0 ∀ x ∈ R
Hence, the roots are real. But f (0) = 5 > 0
The correct option is (C) ∴ f (x) ≥ 0 ∀ x ∈ R
∴ f (–5) ≥ 0 ⇒ 25l + 5m + 5 ≥ 0 ⇒ 5l + m ≥ –1
17. Sinceα, β are roots of the equation x2 + px + q = 0
Hence, minimum value of 5l + m is –1.
∴ α + β = – p and aβ = q
The correct option is (D)
2
⎛α⎞ α 23. Let α, β be the roots of the given equation,
Now q ⎜ ⎟ + ( 2q − p 2 ) + q

⎝ β⎠ β then α + β = λ2 – 5λ + 5 < 1
1 1 ⇒ λ2 – 5λ + 4 < 0
=
[q (α + β)2 – p2ab] = (qp2 – p2q) = 0
β 2 β2 aβ = 2λ2 – 3λ – 4 < 1
α ⇒ 2λ2 – 3λ – 5 < 0

Thus, is a root of the equation
β ∴ (λ – 4) (λ – 1) < 0 or 1 < λ < 4
qx2 + (2q – p2) x + q = 0
5
and (2λ – 5) (λ + 1) < 0 or –1 < λ <


The correct option is (C) 2
Quadratic Equations and Expressions  3.33

⎛ 5⎞ ∴ x = 2 + k, –5 + k, where 0 < k < 1


∴ Required set = ⎜ −1, ⎟ ∩ (1, 4) = ⎛⎜1, ⎞⎟ .
5
∴ x > 2, x > –5
⎝ 2⎠ ⎝ 2⎠
The correct option is (D) ∴ Solution set is (–5, –4] ∪ (2, 3]
24. Since coefficient of x2 > 0 and 1 lies between the roots of The correct option is (B)
3x2 – 3sin θ – 2cos2 θ = 0 27. log2 (9 – 2x) = 3 – x
∴ f (1) < 0 ⇒ 9 – 2x = 23–x
8
⇒ 3 – 3sin θ – 2cos2θ < 0
⇒ 9 – 2x = x
2
⇒ 1 + 2(1 – cos2 θ) – 3sin θ < 0
⇒ 9 · 2x – 22x = 8
⇒ 2sin2 θ – 3sin θ + 1 < 0
⇒ 22x – 9 · 2x + 8 = 0
⇒ (2sin θ – 1) (sin θ – 1) < 0
⇒ (2x – 8) (2x – 1) = 0
1 ⇒ x = 3  or  x = 0

< sin θ < 1
2 But x = 3 is not the solution of original equation,
∴ x = 0.
QUICK TIPS The correct option is (A)
If one root is less than k and other is greatter than k, then 28. Since ax2 + bx + 6 = 0 does not have two distinct real roots.
D > 0 and af (k) < 0, where ∴ b2 – 24a ≤ 0
f(x) = ax2 + bx + c, a, b, c ∈ R, a ≠ 0 Let 3a + b = y
∴ 3a = y – b

The correct option is (C) ∴ b2 – 8(y – b) ≤ 0
i.e., b2 + 8b – 8y ≤ 0
n
25. ∑ Sr = (α + β ) + (α 2 + β 2) + … + (α n + β n) Since b is real
r =1 ∴ 64 + 32y ≥ 0 ⇒ y ≥ –2

= (α + α 2 + … + α n) + (β + β 2 + … + β n)
n ∴ Min. value of y i.e., 3a + b = –2.
Lt
n→∞
∑ Sr = (α + α 2 + … + ∞) + (β + β 2 + … + ∞) The correct option is (D)
r =1
α β 29. Here α + β = –p ⇒ α + β = γ + δ

= +
1− α 1− β γ + δ = –p

HINTS AND EXPLANAT I O N S


α − αβ + β − αβ Now (α – γ) (α – δ) = α 2 – α(γ + δ) + γ δ = α 2 – α (α + β ) + r

=
1 − (α + β ) + αβ = –αβ + r = –(–q) + r = q + r
α + β − 2αβ By symmetry (β – γ ) (β – δ ) = q + r

= ∴ Ratio is 1.
1 − (α + β ) + αβ
The correct option is (A)
25 4
+
375 375 29 1 x+2 1
= = = 30. − >0
25 2 348 12 2
1− − x +1 2
375 375
− x2 − 1 + 2x + 4
The correct option is (B) ⇒
>0
26. If x = n ∈ Z, (x)2 + (x + 1)2 = 25 2( x 2 + 1)
⇒ n2 + (n + 1)2 = 25 3 + 2x − x2

>0
⇒ 2n2 + 2n – 24 = 0 2( x 2 + 1)
⇒ n2 + n – 12 = 0
Since denominator is positive
⇒ n = 3, – 4 ∴ 3 + 2x – x2 > 0

∴ x = 3, – 4 ⇒ –1 < x < 3

If x = n + k, n ∈ Z, 0 < k < 1, then ⇒ x = 0, 1, 2

(x)2 + (x + 1)2 = 25
The correct option is (D)
⇒ (n + 1)2 + (n + 2)2 = 25
31. Let α, β be the roots of λx2 + μx + v = 0
⇒ 2n2 + 6n – 20 = 0
μ v
⇒ n2 + 3n – 10 = 0 ∴ α + β = – , aβ =
λ λ
⇒ n = 2, –5
3.34  Chapter 3

μ2 loge ( x 2 − 5 x + 6) loge ( x − 4)
36. >
(α + β ) 2 2 μ2
= λ = loge 9 loge 3


αβ v λv 2 2
⇒ x – 5x + 6 > x – 8x + 16
λ 10
⇒ 3x – 10 > 0 ⇒ x >  (1)
α β μ2 3

+ +2=  (1) 2
Also, x – 5x + 6 > 0 ⇒ x > 3 or x < 2  (2)
β α λv
and  x – 4 > 0 ⇒ x > 4  (3)
Let γ, δ be the roots of x2 + x + 1 = 0

Common solution from Eqs (1), (2) and (3) is x > 4
∴ γ + δ = –1, γ δ = 1

The correct option is (B)
(γ + δ ) 2 γ δ

= 1 ⇒ + + 2 = 1 (2) 37. Let f (x) = a2x2 + 2bx + 2c = 0
γδ δ γ
Given: a2α 2 + bα + c = 0
α γ and a2β 2 – bβ – c = 0

Since = ,
β δ Now,  f (α) = a2α 2 + 2bα + 2c
∴ from Eq. (1) and (2)
= (a2α 2 + bα + c) + (bα + c)
μ2 = bα + c = – a2α 2
= 1 ⇒ μ2 = λv
λv ⎫
∴ λ, µ, v are in G.P. f (β ) = a2β 2 + 2bβ + 2c ⎬

The correct option is (B)
= 3bβ + 3c = 3 (bβ + c)
32. Given equation can be written as
= 3a2β 2
bx2 + x – bcx – bdx + bcd – a = 0
Since 0 < α < β
⇒ bx (x – c) – bd (x – c) + x – a = 0
∴ α, β are real
⇒ b (x – c) (x – d) + (x – a) = 0
∴ f (α) < 0, f (β) > 0
Let f (x) = b (x – c) (x – d) + (x – a)
∴ f (γ ) = 0 where α < γ < β
f (c) = c – a < 0; f (d) = d – a > 0
Hence, one root of the given equation lies between c and d. QUICK TIPS
The correct option is (C)
Let f(x) = 0 be a polynomial equation. Let p and q be two real
α α +1 b α α +1 c
HINTS AND EXPLANAT I O N S

33. We have, + = − and ⋅ = numbers, p < q. If f(p) ⋅ f(q) < 0, then the equation f(x) = 0
α −1 α a α −1 α a has atleast one real roots between p and q.
2α 2 − 1 b c+a

= − and α =
α2 − α a c−a The correct option is (D)
2
⇒ (c + a) + 4ac = –2b(c + a) 38. Let us see the graph of y = x2 – 2 and y = [x]
⇒ (c + a)2 + 2b(c + a) + b2 = b2 – 4ac Y
⇒ (a + b + c)2 = b2 – 4ac
The correct option is (B)
34. Here α + β = p; aβ = 1 ⇒ aβ = γ δ
γ + δ = q; γ δ = 1
X
Now, (α – γ ) (β – γ) (α + δ ) (β + δ ) O
= [aβ – γ (α + β ) + γ 2] [αβ + δ(α + β ) + δ 2]
= [1 + γ p + γ 2] [1 – pδ + δ 2]
= [(γ 2 + 1) + γ p] [(δ 2 + 1) – pδ]
= (–qγ + γ p) (–qδ – pδ )
= γ δ (q2 – p2) = 1 (q2 – p2) If [x] = –1

The correct option is (C) We have x2 – 2 + 2 = 0 ⇒ x = 0 not possible

35. Let y = [x] [x] = 0 ⇒ x = ± 2 not possible

∴ The given equation y2 + ay + b = 0 must have integral [x] = 1 ⇒ x = ± 4 = ± 2 not possible

roots which is not possible as a and b are odd integers. [x] = 2 ⇒ x = ± 6

∴ Discriminant can’t be perfect square. ⇒ x = 6 is the only solution.

The correct option is (C)
The correct option is (A)
Quadratic Equations and Expressions  3.35

2 ⇒ k = ±2
39. Let 2sin 2
x = Z ⇒ 2cos 2
x =
Z ⇒ k = 2 (Since ln k is defined for k > 0)

Therefore, the given equation becomes The correct option is (B)
2 44. x – x2 – 9 = –(x2 – x + 9)

Z– – 1 = 0 ⇒ Z = 2 or Z = –1
Z ⎡⎛ 35 ⎤
2
1⎞
= – ⎢⎜ x − ⎟ + ⎥ < 0 for all x ∈ R
2 2

⇒ 2sin x = 2 or 2sin x = –1 (not possible)




⎢⎣ ⎝ 2 ⎠ 4 ⎥⎦
π
⇒ sin2 x = 1 ⇒ x = (2n + 1) , n ∈ 1
∴ no. solution i.e., solution set = ϕ

2

The correct option is (C) ⎡ ⎛ 3⎞ x ⎤

 ⎢∵ ⎜ ⎟ > 0 for all x ∈ R ⎥
40. Given: |α – β | > 3p ⎢⎣ ⎝ 5 ⎠ ⎥⎦
If α, β are the roots of x2 + px + 1 = 0, then The correct option is (C)
α + β = –p, aβ = 1 45. Let esin x = t
∴ (α – β)2 > 3p ⇒ t2 – 4t – 1 = 0
⇒ (α + β)2 – 4aβ > 3p
4 ± 16 + 4
⇒ (–p)2 – 4 · 1 > 3p ⇒ t =

2
⇒ p2 – 3p – 4 > 0
⇒ t = esin x = 2 ± 5

⇒ (p – 4) (p + 1) > 0
⇒ p > 4, p > –1 or p < 4, p < –1 ⇒ esin x = 2 –
5 , esin x = 2 + 5
sin x
⇒ p > 4 or p < –1
e =2– 5 < 0,
But p is not –ve ( If p is –ve, then 3 p is not real) ⇒ sin x = ln(2 + 5 ) > 1
⇒ p > 4 So both rejected.
The correct option is (B) Hence no solution
41. If f (x) = x2 + ax + b The correct option is (B)
f  (x + c) = x2 + (2c + a) x + c2 + ac + b 46. Let f (x) = x3 – px + q
∴ Roots of the given equation are 0 and d – c. Now for maxima/minima
 (since roots of x2 + ax + b = 0 are c and d.) f ′(x) = 0
The correct option is (B) ⇒ 3x2 – p = 0

HINTS AND EXPLANAT I O N S


42. Let f (x) = x2 + 2 (k + 1) x + 9k – 5. Let α, β be the roots of
f (x) = 0. The equation f (x) = 0 will have both negative roots
if and only if –(p/3)
  (i) Disc. ≥ 0
–(p/3)
 (ii)  α + β < 0 and
(iii)  f (0) > 0
Now, discriminant ≥ 0 p
⇒ 4 (k + 1)2 – 36k + 20 ≥ 0  ⇒  k2 – 7k + 6 ≥ 0 ⇒ x2 =

3
⇒ (k – 1) (k – 6) ≥ 0 p
⇒ k ≤ 1 or k ≥ 6  (1) ∴ x = ±

3
(α + β) < 0 The correct option is (A)
⇒ –2 (k + 1) < 0 47. Let α and 4β be roots of x2 – 6x + a = 0 and α, 3β be the roots
⇒ k + 1 > 0 ⇒ k > –1 (2) of x2 – cx + 6 = 0, then
and aβ > 0 α + 4β = 6 and 4αβ = a
⇒ 9k – 5 > 0 α + 3β = c and 3αβ = 6.
5 We get αβ = 2 ⇒ a = 8
⇒ k >  (3)
9 So the first equation is x2 – 6x + 8 = 0 ⇒ x = 2, 4
From Eqs (1), (2) and (3) we get k ≥ 6.
If α = 2 and 4β = 4 then 3β = 3
The correct option is (C)
If α = 4 and 4β = 2, then 3β = 3/2 (non-integer)
43. Product of roots = 2 e2 ln k – 1 = 7 (given)
2 ∴ common root is x = 2.
⇒ 2e ln k − 1 = 7
The correct option is (D)
⇒ 2k2 – 1 = 7
3.36  Chapter 3

48. bx2 + cx + a = 0 53. Since the roots of the given equation are real
Roots are imaginary ⇒ c2 – 4ab < 0 ∴ B2 – 4AC ≥ 0 ⇒ 4a2 – 4 (a2 + a – 3) ≥ 0
⇒ c2 < 4ab ⇒ –c2 > –4ab ⇒ –a + 3 ≥ 0 or a ≤ 3.  (1)
Given expression has minimum value Since the root is less than 3, so f (3) > 0
4(3b 2 )( 2c 2 ) − 36b 2c 2 ⇒ 32 – 2a (3) + a2 + a – 3 > 0
=

4 + 3b 2 ⇒ a2 – 5a + 6 > 0 or (a – 2) (a – 3) > 0
12b 2c 2 ⇒ a < 2 or a > 3 (2)
=–
= –c2 > –4ab. From (1) and (2), we have a < 2.
12b 2

The correct option is (C) The correct option is (A)
54. Let f (x) = x2 + (1 – 2k) x + k2 – k – 2
2
x −1
49. We have, 3sec 9 y2 − 6 y + 2 ≤ 1 The number 3 lies between the roots of the given equation, if
2 2 2 f (3) < 0
⇒ 3sec
x
y2 − y+ ≤1 Now, f (3) = 9 + (1 – 2k) 3 + k2 – k – 2
3 9
= 10 – 7k + k2 = k2 – 7k + 10
2
2 ⎛ 1⎞ 1 Hence, f (3) < 0 ⇒ k2 – 7k + 10 < 0
⇒ 3sec
x
⎜⎝ y − ⎟⎠ + ≤ 1
3 9 ⇒ (k – 2) (k – 5) < 0 ⇒ 2 < k < 5.
2
⎛ 1⎞ 1 1
Now, sec2x ≥ 1 ⇒ 3sec2x ≥ 3 and
⎜⎝ y − ⎟⎠ + ≥ , so we The correct option is (A)
3 9 3
55. If x = n ∈ I, |n – 2n| = 4
1
must have sec2x = 1 and y – = 0. ∴ n = ± 4,
3
1 If x = n + k, n ∈ I, 0 < k < 1 then
⇒ x = 0, π, 2π, 3π and y = . |n – 2 (n + k)| = 4
3
∴ There are 4 solutions. ∴ |–n – 2k| = 4.
The correct option is (B) 1
It is possible if k = .
50. Let 2
f (x) = (x – n)m + (x – n2)m + (x – n3)m + … + (x – nm)m. The correct option is (B)
⇒ f ′(x) = m(x – n)m–1 + m(x – n2)m–1 + … + m(x – nm)m–1 56. Since a < 0, in case of positive root of the equation x > a

Since m is odd, (m – 1) is even. Therefore, f ′(x) = 0 has no ∴ The equation is x2 – 2a (x – a) – 3a2 = 0
HINTS AND EXPLANAT I O N S

real root. ⇒ x2 – 2ax – a2 = 0


⇒ f (x) = 0 has one real and (m – 1) imaginary roots. 2a ± 4 a 2 + 4 a 2 2a ± 2 2a

Thus, the roots are =
The correct option is (C) 2 2
51. Since the given equation has imaginary roots = a(1 + 2 ) or a(1 − 2 )
⇒ D < 0 or (a + b + c)2 – 4(ab + bc + ca) < 0 ∴ the only positive root possible is a(1 − 2 ) .
⇒ (a2 + b2 + c2 – 2ab – 2bc + 2ac) < 4ac The correct option is (C)
⇒ (a – b + c)2 < 4ac 57. The required a satisfies the inequality
⇒ − 2 ac < a – b + c
2a2 – 2(2a + 1)a + a(a + 1) < 0

( a + c + 2 ac ) > b ⇒ a(a + 1) > 0 ⇒ a ∈ (–∞, –1) ∪ (0, ∞)

( a + c ) > b or The correct option is (D)


2

a + c > b.
58. We have, 2x + 2x ≥ 2 2 (x ≥ 0)

Similarly, b + c > a and a + b > c.
1
Therefore, a , b , c can be the sides of a triangle. ⇒ 2x ≥
2 ⇒x≥
2
The correct option is (A) and, 2x + 2–x ≥ 2 2 (x < 0)

52. We have, 1
⇒ t + ≥ 2 2 (where t = 2x)

xn + ax + b = (x – x1) (x – x2) … (x – xn) t
x n + ax + b ⇒ t2 – 2 2 t + 1 ≥ 0

⇒ (x – x2) (x – x3) … (x – xn) =
x − x1
⇒ (t − ( 2 − 1)) (t − ( 2 + 1)) ≥ 0

x n + ax + b
⇒ (x1 – x2) (x1 – x3) … (x1 – xn) = lim ⇒ t ≤
2 − 1 or t ≥ 2 + 1 but t > 0
x→ x x − x1
1

= nx1n –1 + a. ⇒ 0 < 2 ≤
x
2 − 1 or 2x ≥ 2 +1
The correct option is (A)
Quadratic Equations and Expressions  3.37

⇒ –∞ < x ≤ log2 ( 2 − 1)
Also, since α < – 1, β > 1

∴ aβ < – 1

or, x ≥ log2 ( 2 + 1) 
(but not acceptable as x < 0) c c

< – 1 or +1<0
⎡1 ⎞ a a
∴ x ∈ (–∞, log2 ( 2 − 1) ] ∪ ⎢ , ∞⎟

⎣2 ⎠ Let f (x) = ax2 + bx + c


The correct option is (D) As f (1) f (–1) > 0,

D 4b 2 − 8c 2 ∴ (a + b + c) (a – b + c) > 0

59. min. f (x) = – =–
4a 4 or, (a + c)2 – b2 > 0 or (a + c)2 > b2


= –(b2 – 2c2) (upward parabola) ⎛ c⎞
2
⎛ b⎞
2
or, ⎜1 + ⎟ > ⎜ ⎟
D 4c 2 + 4b 2 ⎝ a⎠ ⎝ a⎠
max. g(x) = –
=
4a 4 ⎛ c⎞ b ⎡ ⎛c ⎞ ⎤

= b2 + c2 (downward parabola) ⇒ ⎜1 + ⎟ < −
 ⎢∵ ⎜⎝ a + 1⎟⎠ < 0 ⎥
⎝ a⎠ a ⎣ ⎦
Now, 2c2 – b2 > b2 + c2

c b

⇒ c2 > 2b2 ⇒ |c| > 2 |b| or, 1 +
+ <0.
a a

The correct option is (D)

The correct option is (A)
| x + 1| | x + 1 |2
60. + | x + 1| = 63. Roots of ax2 + 2bx + c = 0 are given by
|x| |x|
⎧ 1 | x + 1 |⎫ − 2b ± 4b 2 − 4 ac −b
⇒ | x + 1 | ⎨ +1− x=
=
⎬ =0 2a a
⎩| x | |x| ⎭
∴ |x + 1| = 0 or 1 + |x| – |x + 1| = 0. (Since b2 = ac as a, b, c, are in G.P.)

|x + 1| = 0 ⇒ x = –1. This is root of dx2 + 2ex + f = 0

2
If x < – 1, 1 + |x| – |x + 1| = 0 ⎛ − b⎞ ⎛ − b⎞
∴ d ⎜
+ 2e ⎜ + f =0
⎝ a ⎟⎠ ⎝ a ⎟⎠
⇒ 1 – x + x + 1 = 0 ⇒ 2 = 0 (absurd)
If –1 ≤ x < 0, 1 + |x| – |x + 1| = 0 ⇒ db2 – 2eba + a f = 0

⇒ 1 – x – (x + 1) = 0 ⇒ x = 0 (not possible) ⇒ dac – 2eba + a f = 0
(∵ b2 = ac)
If x ≥ 0, 1 + x – (x + 1) = 0 ⇒ 0 = 0 ⇒ 2eb = dc + a f

2e dc + af dc + af

HINTS AND EXPLANAT I O N S


⇒ x can have any value in the interval ⇒
= = (∵ b2 = ac)
2
∴ x = –1, x > 0.( x ≠ 0) b b ac
The correct option is (B) d f

= +
61. As α, β are roots of ax2 + bx + c = 0, we have a c
d e f
α + β = –b/a, aβ = c/a ⇒
, , are in A.P.
a b c
Now, (α – β )2 = (α + β )2 – 4aβ
2
The correct option is (C)
⎛ b⎞ 4c b 2 − 4 ac

= ⎜− ⎟ − = (1) 64. Let α, β be the roots of x2 + abx + c = 0 and α, γ  be the roots
⎝ a⎠ a a2
of x2 + acx + b = 0, a being the common root.
Now, as α + δ, β + δ are the roots of Ax2 + Bx + C = 0,

∴ α + β = – ab(1)
we have  α + δ + β + δ = –B/A, (α + δ ) (β + δ ) = C/A.

αβ = c(2)
Now, (α – β )2 = [(α + δ ) – (β + δ )]2


α + γ  = – ac(3)
= (α + δ + β + δ )2 – 4 (α + δ ) (β + δ )
αγ  = b (4)
B2 4C B 2 − 4 AC

= − = (2) From (1) – (3),
A2 A A2 β – γ = a (c – b)

From (1) and (2), we get From (2)– (4),
b 2 − 4 ac B 2 − 4 AC α (β – γ) = c – b
2 = .
a A2 α (β − γ ) c−b
The correct option is (A) ∴
= ;
β −γ a ( c − b)
62. Given equation is ax2 + bx + c = 0 1
Since α, β are the roots of the given equation or α = .

a
−b c
∴ From (2) and (4),
∴ α + β = , aβ = .
a a
3.38  Chapter 3

β 68. The roots of 2x2 – 3x + 4 = 0 are imaginary, because disc. =


= c, i.e., β = ac
a (–3)2 – 4 · 2 · 4 < 0. Hence, the common root must be
γ ­imaginary. But imaginary roots occur in pair. Hence both the
and,  = b,
a roots will be common, i.e., two equations will be identical.
i.e., γ = ab. So their coefficients will be proportional
∴ The quadratic equation whose roots are β, γ is a b c
i.e., = = ,
x2 – (β + γ ) x + bγ = 0 2 −3 4
or, x2 – (ac + ab) x + ac · ab = 0; ∴ 6a = –4b = 3c.
or, x2 – a (b + c) x + a2bc = 0. The correct option is (C)
The correct option is (B) 69. We have, 3x2 + x – 5 = 0.
65. The given equation can be written as Its discriminant = 1 – 4 · 3 (– 5) = 61, which is positive but
(ay + a′) x2 + (by + b′) x + (cy + c′) = 0. not a perfect square. Hence, both the roots of 3x2 + x – 5 = 0
The condition that x may be a rational function of y is, must be irrational as the irrational roots occur in conjugate
pair. But one root of ax2 + bx + c = 0 and 3x2 + x – 5 = 0 is
(by + b′)2 – 4 (ay + a′) (cy + c′) is a perfect square;
common. Hence, both the roots of ax2 + bx + c = 0 must also
that is, (b2 – 4ac) y2 + (2bb′ – 4ac′ – 4a′c) y + b′2 – 4a′c′ is a be irrational, that is, both the roots of the given equations are
perfect square. common. Thus, both the equations are the same.
The corresponding quadratic equation has discriminant = 0 a b c
∴ = = = k (say)
that is, 4 (bb′ – 2ac′ – 2a′c)2 – 4 (b2 – 4ac) (b′2 – 4a’c’) = 0; 3 1 −5
or, (ac′ + a′c)2 – 4aa′cc′ = abb′c + a′bb′c – a′c′b2 – acb′ 2 ⇒ a = 3k; b = k, c = –5k.

or, (ac′ – a′c)2 = (ab′ – a′b) (bc′ – b′c). ∴ 3a + b + 2c = 9k + k – 10k = 10k – 10k = 0.
The correct option is (A) The correct option is (A)
66. The discriminant of the given equation is
70. Let α be the common root.
D = 4 [(nCr)2 – nCr – 1 nCr + 1]
Then,  aα2 + 2bα + c = 0
= 4 (a – b),
and,  a1α2 + 2b1α + c1 = 0
where a = (nCr)2, b = nCr – 1 · nCr + 1
By cross-multiplication, we get
n
a Cr ⋅ n Cr α2 α 1
Now, 
= n n = =
b Cr − 1 ⋅ Cr + 1 2(bc1 − b1c) ca1 − ac1 2 ( ab1 − ba1 )
HINTS AND EXPLANAT I O N S

n! n! ( r − 1)! ( n − r + 1)! ⇒ (ca1 – ac1)2 = 4 (bc1 – b1c) (ab1 – ba1) 


(1)

= ⋅
r ! ( n − r )! r ! ( n − r )! n! a b c

, , are in A.P.,
( r + 1)! ⋅ ( n − r − 1)! a1 b1 c1


n! b a c b

− = − = k (say)
r +1 n − r +1 ⎛ 1⎞ ⎛ 1 ⎞ b1 a1 c1 b1

= ⋅ = ⎜1 + ⎟ ⎜1 + >1
r n−r ⎝ r⎠ ⎝ n − r ⎟⎠ ⇒ ab1 – a1b = – ka1b1 and bc1 – b1c = – kb1c1.

∴ a > b ⇒ D > 0
b a c b ca − ac1
Also, 2k = k + k =
− + − = 1
⇒ roots of given equation are real and distinct.
b1 a1 c1 b1 a1c1

The correct option is (A) or, ca1 – ac1 = 2ka1c1.

2 1
67. Let α and β be the roots of x – px + q = 0 and α and be ∴ From (1), 4 k 2 a12c12 = 4 (–ka1b1) (–kb1c1)

the roots of x2 – ax + b = 0. β
or, a1c1 = b12. Hence a1, b1, c1 are in G.P.

Then,  α + β = p and aβ = q.

The correct option is (B)
1 α
Also,  α + = a and = b. ( a + x ) (b + x )
β β 71. Let y =
2 2 (c + x )
⎛ α⎞ ⎛ 1⎞
Now, (q – b)2 = ⎜ αβ − ⎟ = α 2 ⎜ β − ⎟
⇒ x2 + (a + b) x + ab = cy + xy

⎝ β⎠ ⎝ β⎠
⇒ x2 + (a + b – y) x + ab – cy = 0.

2
α ⎡ ⎛ 1⎞⎤ For real x, B2 – 4AC ≥ 0


⋅ βα ⎢(α + β ) − ⎜ α + ⎟ ⎥
=
β ⎣ ⎝ β ⎠⎦ ⇒ (a + b – y)2 – 4ab + 4cy ≥ 0


2
= bq (p – a) . ⇒ (a + b)2 + y2 – 2 (a + b) y – 4ab + 4cy ≥ 0


The correct option is (B) ⇒ (a – b)2 + y2 – 2 (a + b – 2c) y ≥ 0

Quadratic Equations and Expressions  3.39

⇒ y2 – 2 (a + b – 2c) y + (a – b)2 ≥ 0
1/ 4
∴ α1/4 + β1/4 = ⎡ p + 6 q + 4 q1/ 4 p + 2 q ⎤
⇒ [ y − ( ( a − c) − (b − c)) ]
2 ⎢⎣ ⎥⎦
The correct option is (D)
× [ y − ( ( a − c) + (b − c)) 2 ] ≥ 0
74. Since a and b are the roots of the equation
∴ y ≤ ( ( a − c) − (b − c)) 2 x2 + px + 1 = 0
or, y ≥ ( ( a − c) + (b − c)) 2 .
∴ a + b = –p(1)

Hence, the minimum value of y is and, ab = 1 (2)
Also, since c and d are the roots of the equation
( ( a − c) + (b − c)) 2
x2 + qx + 1 = 0
The correct option is (B)
∴ c + d = –q(3)
72. Let the ratio of the roots be k. Then, the roots of
and, cd = 1 (4)
a1x2 + b1x + c1 = 0 are α, kα
Now, (a – c) (b – c) (a + d) (b + d)
and the roots of a2x2 + b2x + c2 = 0 are β, kβ.
= (ab – bc – ac + c2) (ab + db + ad + d2)
−b1
∴ α + kα = (1) = [ab – c (b + a) + c2] ⋅ [ab + d (a + b) + d2]
a1
c1 = (1 + cp + c2) (1 – pd + d2)
α ⋅ kα = (2)
a1 [Putting the values of a + b and ab]
−b2 = 1 + cp + c2 – pd – cdp2 – c2pd + d2 + cpd2 + c2d2
β + kβ =
(3)
= 1 + (c2 + d2) + c2d2 – cdp2 + p (c – d) + cpd (d – c)
a2
c = 1 + [(c + d)2 – 2cd] + c2d2 – cdp2 + p (c – d) + cpd (d – c)
β ⋅ kβ = 2 .(4)
= 1 + (q2 – 2) + 1 – p2 + p (c – d) + p (d – c)
a2
[Putting the values of c + d and cd]

Dividing (1) by (3), we get
= 2 – 2 + q2 – p2 = q2 – p2.
α (1 + k ) ba α ba
= 1 2 , or = 1 2 .(5) The correct option is (B)
β (1 + k ) a1b2 β a1b2
75. Let f (x) = x3 + px2 + q = 0 (1)

Dividing (2) by (4), we get
Since f (x) = 0 has a root of multiplicity 2
kα 2 ca ∴ f (x) = 0 and f ′ (x) = 3x2 + 2px = 0 have a common root.
2 = 1 2 ;
kβ a1c2 The roots of 3x2 + 2px = 0 are x = 0 and x = –2p/3.

HINTS AND EXPLANAT I O N S


⎛α⎞
2
c1a2 But x = 0 is not a root of f (x) = 0 ( q ≠ 0)

or ⎜ ⎟ = ∴ common root of f (x) = 0 and f ′ (x) = 0 is x = –2p/3.
⎝ β⎠ a1c2
2 ∴ (–2p/3)3 + p (–2p/3)2 + q = 0
⎛ba ⎞ ca
or, ⎜ 1 2 ⎟ = 1 2 
(Using (5)) ⇒ 4p3 + 27q = 0.
⎝ a1b2 ⎠ a1c2
The correct option is (D)
2
⎛b ⎞ c1a2 a12 ca a c 76. Given equation is
⇒ ⎜ 1 ⎟
= × 2= 11 = 1⋅ 1
⎝b ⎠ a c c a a ax2 + bx + c = 0
2 1 2 a 2 2 2 2 c2
a1 b1 c1 Let α, β be the roots of this equation.
∴ , , are in G.P.
a2 b2 c2 b c
then, α + β = –
and aβ =
The correct option is (B) a a
73. Since α, β are the roots of the equation 1 1 a2 + β 2
Also,  α + β = 2 + 2 =

x2 – px + q = 0 α β α 2β 2
∴ α + β = p and aβ = q. (α + β ) 2 − 2αβ

=
Now, (α1/4 + β1/4)4 = [(α1/4 + β1/4)2]2
(αβ ) 2
2

= [α1/2 + β1/2 + 2 (aβ)1/4]2 ⎛ α + β⎞ 2
⇒ α + β = ⎜ –
2 ⎝ αβ ⎟⎠ αβ


= ⎡ α + β + 2 αβ + 2(αβ )1/ 4 ⎤
⎢⎣ ⎥⎦ 2 2
⎛ b⎞ ⎛ − b /a ⎞ 2 b ⎛ b⎞ 2a
⇒ ⎜ − ⎟ = ⎜ – ⇒– = ⎜ ⎟ –
⎝ c /a ⎟⎠
2

= ⎡ p + 2 q + 2 ( q)1/ 4 ⎤ ⎝ a⎠ c /a a ⎝ c⎠ c
⎣⎢ ⎦⎥ 2
2a ⎛ b⎞ b 2a b ⎡b c⎤
⇒ = ⎜ ⎟ + ⇒ = +
c ⎢⎣ c a ⎥⎦
1/ 4

=p+6 q + 4q p+2 q c ⎝ c⎠ a c
3.40  Chapter 3

2a b c c a b 81. Given equation can be written as


⇒ = + ⇒ , , are in A.P.
b c a a b c (2x)2 – ( a– 4)2x – 2x + a– 4 = 0
a b c ⇒ (2x – 1) (2x – a + 4) = 0
⇒ , , are in H.P.
c a b ⇒ 2x = 1, 2x = a – 4
The correct option is (C) Since x ≤ 0 and 2x = a– 4 ( x is non-positive)
77. Discreminent equals –4(k – 5) ≥ 0 ⇒ k ≤ 5. The quadratic ∴ 0 < a – 4 ≤ 1 i.e., 4 < a ≤ 5 i.e., a ∈ (4, 5]
equation at x = 5 must be positive and sum of the roots must The correct option is (B)
be less than 10. These conditions imply k2 – 9k + 20 > 0. So,
k < 4. 82. Let f (x) = (x – α) (x – β)(1)
The correct option is (A) Now, f (n) f (n + 1) = (n – α) ( n– β) ( n + 1– α) ( n + 1 – β)
78. Let y = x – [x] = (n – α) ( n + 1– β) (n– β) (n + 1 – α)
∴ the given equation becomes = {n (n + 1) – n (α + β) – α + αβ} {n (n + 1)
f (y) = (a – 2) y2 + 2y + a2 = 0 (1) – n (α + β) –β + αβ}
Since x is not an integer, = {n (n + 1) + na + b–α } {n (n + 1) + na + b – β}
∴ y = x – [x] ≠ 0 = (m – α) (m – β), where m = n (n + 1) + an + b
Then,  a ≠ 0 [ of (1)] The correct option is (A)
When 2 < x < 3, [x] = 2 83. We have,
⇒ 0 < x – [x] <1 i.e. 0 < y < 1
x 2 + nx − 2
Since given equation has exactly one solution in the interval −2 ≤0
(2, 3) x 2 − 3x + 4
∴ (1) has exactly one solution in the interval (0,1) x 2 − ( n + 6) x + 10
This is possible if f (0) f (1) < 0 ⇒ ≥0
x 2 − 3x + 4
[ otherwise, the equation (1) has either no or two
 solutions in (0,1)] ⇒ x2 – (n + 6) x + 10 ≥ 0
⇒ a2 (a – 2 + 2 + a2) < 0 ⇒ a (a + 1) < 0 [ x2 – 3x + 4 >0 ∀ x ∈ R, as its D < 0 and a > 0]
⇒ –1 < a < 0 i.e., a ∈ (–1, 0) The above inequality will be true for all real x if its discrim-
inant ≤ 0
The correct option is (C)
i.e., (n + 6)2 – 40 ≤ 0
79. Given, b2 ≤ 4 ac, c2 ≤ 4ab and a2 ≤ 4ac
HINTS AND EXPLANAT I O N S

Equality cannot hold simultaneously ⇒ –( 40 – 6) ≤ n ≤ ( 40 – 6) (1)


[ a, b, c are different] Also, we have,
∴ a2 + b2 + c2 < 4 (ab + bc + ca) ⇒ R < 4 x 2 + nx − 2 2 x 2 + ( n − 3) x + 2
Also, a2 + b2 + c2 – ab – bc – ca 2
+ 1≥ 0 ⇒ ≥0
x − 3x + 4 x 2 − 3x + 4
1
⎡⎣(b − c) 2 + (c − a) 2 + ( a − b) 2 ⎤⎦ > 0 = ⇒ R > 1 ⇒ 2x2 + (n – 3) x + 2 ≥ 0
2
The correct option is (D) The above inequality will be true for all real x if its discrim-
inant ≤ 0
80. The given equation can be written as
⇒ (n – 3) 2 – 16 ≤ 0
(z + 1)2 – (a – 3) z (z + 1) + (a – 4) z2 = 0
⇒ 1 ≤ n ≤ 7  (2)
[Putting x2 + x + 1 = z]
Drawing the number line for inequalities (1), (2) and taking
⇒ (1 + 3 – a + a – 4) z2 + (2 + 3 – a) z + 1 = 0
their intersection we get
1
⇒ (5 – a) z + 1 = 0  or  z = n ∈ [ −1, 40 − 6]
a−5
1 The correct option is (C)
⇒ x2 + x + 1 – =0
a−5
a−6 84. Let f (x) = (x – a) ( x – b) –1
⇒ x2 + x + =0
a−5 ⇒ f (a) = –1 and f (b) = –1.
whose roots will be real if discriminant ≥ 0 Also, the coefficient of x2 = 1 > 0.
4( a − 6 ) 3a − 19 Hence, a and b both lie between the roots of the equation
⇒ 1 – ≥0⇒ ≤0
a−5 a−5 f (x) = 0
19 ∴ The equation (x – a) (x – b) – 1 = 0 has one root in
∴ 5 < a ≤
3 (–∞, a) and other in (b, ∞)[ b > a]
The correct option is (D) The correct option is (B)
Quadratic Equations and Expressions  3.41

85. The given quadratic equation is satisfied by x = –a, x = –b The correct option is (B)
and x = –c, Hence, the quadratic equation has three roots, 89. The sum of the coefficients of the equation = 0
which is only possible if it is an identity hence it has infinite ∴ x = 1 is a root of the equation. Let a be the first term
roots. and d be the common difference of given A.P.
The correct option is (D) 1
86. Putting x2 = y, the given equation in x reduces to tp = a + (p – 1)d = (1)
q
ay2 – 2y – (a – 1) = 0  (1) 1
and, tq = a + (q –1) d = (2)
The given biquadratic equation will have four real and dis- p
tinct roots, if the quadratic equation (1) has two distinct and 1
positive roots. For that, we must have Solving (1) and (2), a = d =
pq
D > 0 ⇒ a2 – a + 1 > 0, which is true ∀ a ∈ R
∴ tpq = a + (pq – 1)d = 1
Product of roots > 0 ⇒ 0 < a < 1
∴ tpq is the root of the given equation.
sum of roots > 0 ⇒ a > 0
The correct option is (C)
Hence, the acceptable values of a are 0 < a < 1.
The correct option is (C) 90. Let α, β be the roots of the equation 4x2 + 4ax + b = 0, then
we have,
87. Since –5 and 1 are the roots. b
α + β = –a and aβ =
Product of roots = –5 × 1 = b ⇒ b = –5 4
According to the given condition,
and, sum of roots = –5 + 1= –[ a2 – 5a + b + 4]
|α – β | ≤ a
⇒ [a2– 5a – 1] = 4 ⇒ 4 ≤ a2 – 5a – 1< 5
⇒ (α + β)2 – 4 α β ≤ a2
⇒ a2 – 5a – 5 ≥ 0 and a2 – 5a – 6 < 0
⇒ a2 – b2 ≤ a2
5 − 45 5 + 45
⇒ a ≤  or a ≥ and – 1 < a < 6 ⇒ b ≥ 0
2 2
Also, we have for real roots
5−3 5 5+3 5 (4a)2 – 16 b ≥ 0 i.e., b ≤ a2
⇒ – 1 < a ≤  or  ≤a<6
2 2
Therefore, 0 ≤ b ≤ a2
⎛ 5 − 3 5 ⎤ ⎡5 + 3 5 ⎞ The correct option is (C)
⇒ a ∈ ⎜ −1, ⎥∪⎢ , 6⎟
⎝ 2 ⎥⎦ ⎢⎣ 2 ⎠ 91. Let the roots be α and α + 2, where α is an odd positive
The correct option is (C) integer. Then, aa2 + bα + c = 0  (1)

HINTS AND EXPLANAT I O N S


88. We have, and a (α + 2)2 + b (α + 2) + c = 0 ⇒ aa2 + bα + c + (4 aα
α + β = α, aβ = p + 4a + 2b) = 0
and,  γ + δ = b, γδ = q ⇒ 2a (1 + α) + b = 0 [using (1)]
If r (r > 1) be the common ratio of the increasing G.P. α, β, ⇒ b = – 2a (1 + α)
γ, δ then
⇒ b2 = 4a2 (1 + α)2 ⇒ b2 ≥ 4a2 (1 + 1)2
β = ar, γ = ar2 and δ = ar3 
[ α ≥ 1as α is odd positive integer]
The above equations then reduce to
⇒ b2 ≥ 16 a2 or |b| ≥ 4a
α (1 + r) = a, a2r = p
and,  ar2 (1 + r) = b, a2 r5 = q The correct option is (B)
b q 92. The discriminants of the given quadratic equations are,
⇒ r2 = and r4 =
a p D1 = a2 + 12b, D2 = c2– 4b and D3 = d2 – 8b
q ⎛ b⎞
2
∴ D1+ D2 + D3 = a2 + c2 + d2 ≥ 0
⇒ = ⎜ ⎟
p ⎝ a⎠ ⇒ At least one of D1, D2, D3 is non-negative. Hence, the
q+ p b2 + a2 equation has at least two real roots.
Hence, we have = 2 .
q− p b − a2 The correct option is (D)

Previous Year’s Questions


93. Key Idea : The equation having α and β as its roots, is The above two equations imply that α and β are the roots of
x2− (α + β )x + aβ = 0. the equation
Since a2 = 5α − 3 ⇒ α2− 5α + 3 = 0 x2−5x + 3 = 0.
and β 2 = 5β − 3 ⇒ β 2− 5β + 3 = 0 α + β = 5 and αβ = 3
3.42  Chapter 3

α β α 2 + β 2 (α + β ) 2 − 2αβ So, (3a − 1) 2 1


Now + = = =
β α αβ αβ 2
9( a − 5a + 3) 2 2
a − 5a + 3
25 − 6 19 2
= = ⇒a= .
3 3 3
α β The correct option is (A)
and . = 1
β α 98. (1 − p)2 + p (1 − p) + (1 − p) = 0(since (1 − p) is a root of
α β the equation x2 + px + (1 − p) = 0)
Thus, the equation having and as its roots is given by
β α ⇒ (1− p)(1− p + p + 1) = 0
⎛α β⎞ α β ⇒ 2(1− p) = 0 ⇒ (1 − p) = 0 ⇒ p = 1
x2 − ⎜ + ⎟ x + ⋅ = 0 Now, the sum of roots is α + β = -p and the product αβ =
⎝ β α⎠ β α
1 − p = 0 (where β = 1 − p = 0)
19
⇒ x2 − x +1= 0 ⇒ α + 0 = –1 ⇒ α = –1 ⇒ Roots are 0, –1
3
The correct option is (C)
⇒ 3 x 2 − 19 x + 1 = 0 99. Since 4 is one of the root of x2 + px + 12 = 0, we have
The correct option is (A)
16 + 4p + 12 = 0 ⇒ p = −7
94. Key Idea : If the discriminant of ax2 + bx + c = 0 is positive,
And the equation x2 + px + q = 0 has equal roots
then this equation has two real roots.
49
We have 32 x − 7 x + 7 = 32 ⇒ D = 49 − 4q = 0 ⇒ q =
2
.
4
⇒ 2x2 − 7x + 7 = 2 The correct option is (A)
⇒ 2x2 − 7x + 5 = 0 ax 3 bx 2
100. Let f  ′(x) = ax2 + bx + c, then f ( x ) = + + cx + d
3 2
D = b 2 − 4 ac
1
= ( −7) 2 − 4 × 2 × 5 ⇒ f ( x ) = ( 2ax 3 + 3bx 2 + 6cx + 6 d ), Now f (1) = f (0) = d,
∴ 6
= 49 − 40 then according to Rolle’s theorem
=9 2
⇒ f  ′(x) = ax + bx + c = 0 has at least one root in (0, 1)
Since the discriminant is positive, the equation has two real The correct option is (A)
roots.
101. x2− (a − 2)x − a − 1 = 0
The correct option is (B)
HINTS AND EXPLANAT I O N S

Sum of roots, α + β = a − 2
95. Let α, β be the roots
Product, αβ = − (a + 1)
1 1
α+β = 2 + 2 α2 + β2 = (α + β)2− 2αβ
α β
= a2− 2a + 6 = (a − 1)2 + 5

α+β =
(α + β )2 − 2αβ ⇒ a = 1
(αβ ) 2 The correct option is (A)

2
102. Let α, α + 1 be the roots of the equation, then
⎛ b ⎞ b − 2ac α + (α + 1) = b
⎜⎝ − ⎟⎠ =
a c2 α(α + 1) = c
⇒ 2a 2c = b(c 2 + ba) ∴  b2− 4c = (2α + 1)2 − 4α (α + 1) = 1
a b c The correct option is (D)
⇒ , , are in HP
c a b −b
The correct option is (C) 103. = 2k < 10 ⇒ k < 5
a
96. x2 – 3 | x | + 2 = 0 c
⇒ (| x | –1) (| x | – 2) = 0, as | x |2 = x2 or, = k 2 + k − 5 < 25 ⇒ ( k + 6)( k − 5) < 0
a
⇒ x = ±1, ± 2 . ⇒k <5
The correct option is (B)
⇒ k ∈ ( −∞, 4)
97. β = 2α The correct option is (C)
1 − 3a 104. Equation x2− 2mx + m2− 1 = 0
3α = 2
a − 5a + 3 ⇒ (x − m)2− 1 = 0
2 ⇒ (x − m + 1) (x − m − 1) = 0
2α 2 = 2 ⇒ x = m − 1, m + 1
a − 5a + 3
Quadratic Equations and Expressions  3.43

According to question ⎛π⎞


α 2009 + β 2009 = 2 cos 2009 ⎜ ⎟
− 2 < m − 1, m + 1 < 4 ⎝ 3⎠

⇒ m > − 1 and m < 3
⎡ 2π ⎤ ⎛ 2π ⎞
⇒ − 1 < m < 3. = 2 cos ⎢668π + π + = 2 cos ⎜ π + ⎟
⎣ 3 ⎥⎦ ⎝ 3⎠
The correct option is (C)
2π ⎛ 1⎞
= −2 cos
= −2 ⎜ − ⎟ = 1
3 x 2 + 9 x + 17 3 ⎝ 2⎠
105. Let y = , then
3x 2 + 9 x + 7 The correct option is (B)
3x2 (y − 1) + 9x (y − 1) + 7y − 17 = 0 110. esin x − e − sin x = 4 ⇒ esin x = t
Now, D ≥ 0 (∵ x is real) implies that
1
81(y − 1)2 − 4x3 (y − 1) (7y − 17) ≥ 0 t− −4
t
⇒ (y − 1) (y − 41) ≤ 0 ⇒ 1 ≤ y ≤ 41 4 ± 16 + 4
t 2 − 4t − 1 − 0 ⇒ t =
The correct option is (C) 2
106. Given equation x2 + ax + 1 = 0 4±2 5
⇒t = ⇒t = 2± 5
The sum and product of the roots 2
α + β = −a 1
esin x = 2 ± 5 −1 ≤ sin x ≤ 1 ≤ esin x ≤ e
αβ = 1 e
sin x
e = 2 + 5 not possible
Now, | α − β | = (α + β ) 2 − 4αβ sin x
e = 2 − 5 not possible
⇒ |α − β | = a2 − 4 ∴ Hence no solution
⇒ a2 − 4 < 5 The correct option is (B)
⇒ a − 4 < 5 ⇒ a2− 9 < 0
2 111. If 2 x 3 + 3 x + k = 0 has 2 distinct real roots in [0, 1], then
f  ′ (x) will change sign.
⇒ a ∈ (−3, 3)
But f  ′(x) = 6x2 + 3 > 0
The correct option is (A)
So, no value of k exists.
107. Let α and 4β be roots of x2− 6x + a = 0 and α, 3β be the The correct option is (C)
roots of x2− cx + 6 = 0, then 112. Both the roots of the equation x 2 + 2 x + 3 = 0 are
α + 4β = 6 and 4αβ = a imaginary.

HINTS AND EXPLANAT I O N S


α + 3β = c and 3αβ = 6 Since, a, b, c ∈ R.
We get αβ = 2 ⇒ a = 8 So, if one root is common then both roots are common
So the first equation is x2− 6x + 8 = 0 ⇒ x = 2, 4 a b c
Now, if α = 2 and 4β = 4 then 3β = 3 Hence, = =
1 2 3
If α = 4 and 4β = 2, then 3β = 3/2 (non-integer)
a : b : c = 1 : 2 : 3.
∴ common root is x = 2. The correct option is (D)
The correct option is (D) 113. a 2 = 3t 2 − 2t
108. Given equation bx2 + cx + a = 0 has imaginary roots For non-integral solution
⇒ c2− 4ab < 0 ⇒ c2< 4ab ⇒ –c2 > −4ab 2
0 < a < 1
Since 3b2 > 0, the expression 3b2x2 + 6bcx + 2c2 has
­minimum value. a ∈ ( −1, 0) ∪ (0,1).
[Note: It is assumed that a real solution of given equation
Minimum value exists.]
4(3b 2 )( 2c 2 ) − 36b 2c 2 12b 2c 2 The correct option is (A)
= 2
=− = − c 2 > −4 ab.
4(3b ) 12b 2 114. x 2 = 6 x + 2 ⇒ α 2 = 6α + 2
The correct option is (C) α 10 = 6α 9 + 2α 8  (1)

1± 1− 4 and β10 = 6β 9 + 2β 8  (2)
109. x2 − x + 1 = 0 ⇒x= Subtract (2) from (1)
2
1 ± 3i ⇒ a10 = 6 a9 + 2a8
x=
2 a10 − 2a8
⇒ = 3.
π π 2a9
1 3
α= +i , β = cos − i sin The correct option is (B)
2 2 3 3
3.44  Chapter 3

115. Rearranging equation,  α + β = 1


we get nx2 +(1 + 3 + 5 + … + (2n + 1)}x + {1.2 + 2.3 + … αβ=1
(n – 1)n} = 10n 1 + ω + ω2 = 0 [cube roots fully]
( n − 1)n( n + 1)  ω3 = 1
⇒ nx2 + n2x + = 10n
3  –ω – ω2 = 1
⎛ n2 − 31⎞ α = –ω
⇒ x2 + nx + ⎜ ⎟ =0
⎝ 3 ⎠  β = –ω2
Given difference of roots = 1 To calculate
⇒ | α – β | = 1 α101 + β107

⇒ D = 1 ⇒ (–ω)101 + (–ω2)107
4
⇒ −1 × ⎡⎣(ω 33 )3ω 2 + ω 214 ⎤⎦
⇒ n2 – (n2 – 31) = 1
3
So, n = 11 ⇒ –[ω2 + (ω3)71 ⋅ ω]
Hence, the correct option is (C) ⇒ –[ω + ω2] = –1 × –1 = 1
116. Given x2 – x + 1 = 0 117. Sum of square of roots is (α + β)2 – 2αβ = λ2 – 4λ + 5
Roots are α and β which is minimum for λ = 2.
HINTS AND EXPLANAT I O N S
CHAPTER Permutations and
4 Combinations
LEARNING OBJECTIVES
After reading this chapter, you will be able to:
 Know about factorial notation and fundamental princi-  Grasp the knowledge of some key results on combina-
ples of counting tion and some useful results for geometrical problems
 Learn to define permutation and combination, and how   Be acquainted with exponent of prime p in n!
are they denoted

FACTORIAL NOTATION
ERROR CHECK
We often come across products of the form 1 ⋅ 2, 1 ⋅ 2 ⋅ 3,
1 ⋅ 2 ⋅ 3 ⋅ 4, …  The factorial is defined only for whole numbers.
Instead of writing all the factors of such a product We do not define the factorial of proper fractions or
in full, it is convenient to use a special notation. We write ­negative integers.
1! = 1,  (2n)! ≠ 2(n)!

 (m + n)! ≠ m! + n!
2! = 1 ⋅ 2,
 (mn)! ≠ m! n!
3! = 1 ⋅ 2 ⋅ 3,
……………….....
n! = 1 ⋅ 2 ⋅ 3 … n.
“n!” denotes the product of the first n natural numbers. We FUNDAMENTAL PRINCIPLES OF COUNTING
read ‘n!’ as ‘n factorial’. n! is also written as ‘ |−−
n ’ and read
as ‘factorial n’. It is easy to see that
1! = 1, 2! = 1 ⋅ 2 = 2, 3! = 1 ⋅ 2 ⋅ 3 = 6, 4! = 1 ⋅ 2 ⋅ Multiplication Principle
3 ⋅ 4. = 24, and so on. If an operation can be performed in ‘m’ different ways;
­following which a second operation can be performed in
I M P O R TA N T P O I N T S ‘n’ different ways, then the two operations in succession can
be performed in m × n different ways.
 We know that

n! = n (n – 1) (n – 2) (n – 3) … 3 · 2 · 1 Illustration
= n (n – 1)! = n (n – 1) (n – 2)!
= n (n – 1) (n – 2) (n – 3)! and so on. Anu wishes to buy a birthday card for the brother Manu and
Thus, if m, n ∈ N and m > n, then m! can be expressed in
send it by post. Five different types of cards are available at
terms of n! the card-shop, and four different types of postage stamps
For example are available at the post-office. In how many ways can she
8! = 8 · 7 · 6! choose the card and the stamp?
10! = 10 · 9 · 8 · 7 · 6 · 5!
Solution
Also, m! = n! if and only if m = n
 Putting n = 1 in n! = n (n – 1)!, we have She can choose the card in five ways. For each choice of
1! = 1 · 0! the card she has four choices for the stamp. Therefore, there
\ 0! = 1 are 5 × 4 ways, i.e., 20 ways of choosing the card and the
n
 (2n)! = 2 · n! [1 · 3 · 5 … (2n – 1)] stamp.
4.2  Chapter 4

Addition Principle (C) a1 + a2 + … + ak + 1


(D)  None of these
If an operation can be performed in ‘m’ different ways
and another operation, which is independent of the first Solution: (A)
operation, can be performed in ‘n’ different ways. Then Total number of wrong answers
either of the two operations can be performed in (m + n)
= 1 · (a1 – a2) + 2 ⋅ (a2 – a3) +, …,
ways.
+ (k – 1) (ak – 1 – ak) + kak
= a1 + a2 + a3 +, …, + ak
Info Box!
The above two principles can be extended for any
finite number of operations. 3x 2 + 9x + 17
2. If x is real, the maximum value of is
3x 2 + 9x + 7
17 1
(A)  (B)  (C) 41 (D) 1
7 4
Illustration Solution: (C)
Suppose there are 5 gates in a stadium, 2 on one side and 3 The word BAC cannot be spelt if the m selected
on the other. Sohan has to go out of the stadium. He can go ­coupons do not contain atleast one of A, B and C.
out from any one of the 5 gates. Thus, the number of ways Number of ways of selecting m coupons which are
in which he can go out is 5. Hence, the work of going out A or B = 2m.
through the gates on one side will be done in 2 ways and the This also includes the case when all the m coupons
work of going out through the gates on other side will be are A or all are B.
done in 3 ways. The work of going out will be done when Number of ways of selecting m coupons which are
the Sohan goes out from side I or side II. Thus, the work of B or C = 2m.
going out can be done in (2 + 3) = 5 ways. This also includes the case when all the m coupons
are B or all are C.
Number of ways of selecting m coupons which are
C or A = 2m.
This also includes the case when all the m coupons
are C or all are A.
Number of ways of selecting m coupons when all
E are A = 1m.
D I
A eI Number of ways of selecting m coupons when all
Side B
C Sid are B = 1m.
I
Number of ways of selecting m coupons when all
FIGURE 4.1
are C = 1m.
\ Required number = 2m + 2m + 2m – (1m + 1m + 1m)
Info Box! = 3 ⋅ 2m – 3 ⋅ 1m = 3 (2m – 1)
Addition theorem of counting is also true for more 3. There are 4 candidates for the post of a lecturer in
than two works. Mathematics and one is to be selected by votes of
5 men. The number of ways in which the votes can be
given is
(A) 1048 (B) 1024
SOLVED EXAMPLES
(C)  1072 (D)  None of these
1. In a certain test, ai students gave wrong answers to at Solution: (B)
least i questions where i = 1, 2, 3, …, k. No student Each man can vote for any one of the 4 candidates and
gave more than k wrong answers. The total number of this can be done in 4 ways.
wrong answers given is Similar is the case with every other man.
(A) a1 + a2 + … + ak ( Repetition is allowed)
(B) a1 + a2 + … + ak – 1 Hence 5 men can vote in 45 i.e., 1024 ways.
Permutations and Combinations  4.3

4. There are 10 lamps in a hall. Each one of them can Solution: (B)


be switched on independently. The number of ways in Total number of coins = 2 + 2 + 3 + 1 = 8.
which the hall can be illuminated is 2 coins are 10 paise, 2 are 20 paise, 3 are 25 paise
(A) 102 (B) 
18 and 1 is of 50 paise.
10
(C) 2 (D) 1023 \ Required no. of ways
8!
Solution: (D) =
2 ! × 2 ! × 3! × 1! 
Each bulb has two choices, either switched on or off
\ Required number of ways 8 × 7 × 6 × 5 × 4 × 3 × 2 ×1
=
2 ×1× 2 ×1× 3 × 2 ×1×1
= 210 – 1 = 1024 – 1 = 1023 
= 8 × 7 × 6 × 5 = 1680
(Since in one way when all are switched off, the hall
will not be illuminated.) 8. The number of ways in which the letters of the word
BALLOON can be arranged so that two L’s do not
5. A telegraph has 5 arms and each arm is capable of 4 come together, is
distinct positions, including the position of rest. The (A) 700 (B) 800
total number of signals that can be made is (C)  900 (D)  None of these
(A) 473 (B) 1023 Solution: (C)
(C)  1173 (D)  None of these There are in all seven letters in the word BALLOON in
which L occurs 2 times and O occurs 2 times.
Solution: (B) \ The number of arrangements of the seven letters
Each arm can be set in 4 ways. 7!
\ Five arms can be set in 4 × 4 × 4 × 4 × 4 ways. of the word = = 1260.
2! x 2!
But this includes the way when all the arms are in
If two L’s always come together, taking them as
the position of rest, when no signal is sent.
one letter, we have to arrange 6 letters in which O
Hence, required number of signals
occurs 2 times.
= 45 – 1 = 1024 – 1 = 1023 \ The no. of arrangements in which the two L’s
come together
6. There are stalls for 10 animals in a ship. The number 6!
= = 6 × 5 × 4 × 3 = 360.
of ways the shipload can be made if there are cows, 2!
calves and horses to be transported, animals of each Hence, the required no. of ways in which the two L’s
kind being not less than 10, is do not come together = 1260 – 360 = 900.
(A) 59049 (B) 49049
9. A Letter lock contains 5 rings each marked with four
(C)  69049 (D)  None of these different letters. The number of all possible unsuccess-
Solution: (A) ful attempts to open the lock is
Each stall can be filled in 3 ways as there are three (A) 625 (B) 1024 (C) 624 (C) 1023
types of animals (animals of each category being not Solution: (B)
less than 10).
Shipload, i.e., filling up of 10 stalls, can be made in
1 1 1 1 1
3 × 3 × … up to 10 times = 310 = 59049
2 2 2 2 2

7. The number of ways in which two 10-paise, two 3 3 3 3 3


20-paise, three 25-paise and one 50-paise coins can be 4 4 4 4 4
distributed among 8 children so that each child gets
only one coin, is Number of options on 1st Ring = 4
(A) 1720 (B) 1680 Number of options on 2nd Ring = 4
(C)  1570 (D)  None of these Number of options on 3rd Ring = 4
Number of options on 4th Ring = 4
4.4  Chapter 4

Number of options on 5th Ring = 4 The grand children wish to occupy the 4 seats on either
\ Total number of options/arrangements side of the table = 4! ways
= 4 × 4 × 4 × 4 × 4 = 1024 = 24 ways
10. We have (n + 1) white balls and (n + 1) black balls. In and grand father can occupy a seat in (5 – 1) ways =
each set the balls are numbered from 1 to (n + 1). If 4 ways
these balls are to be arranged in a row so that two con- (Since 4 gaps between 5 sons and daughters)
secutive balls are of different colours, then the number and the remaining seats can be occupied in 5! ways
of these arrangements is
(A) (2n + 2)! (B)  2 × (2n + 2) = 120 ways (5 seats for sons and daughters)
(C) 2 × (n + 1)! (D)  2 [(n + 1)!]2
Hence, required number of ways
Solution: (D)
Between (n + 1) white balls there are (n + 2) gaps in =
24 × 4 × 120 = 11520
which (n + 1) black balls are to arranged. Number of
required arrangements
= (n + 1)! (n + 1)! = [(n + 1)!]2
PERMUTATION
Now between (n + 1) black balls (n + 1) white balls are
to be filled in number of ways = (n + 1)! (n + 1)! Each of the different arrangements which can be made by
taking some or all of given number of things or objects at a
\ Required number of ways = 2[(n + 1)!]2 time is called a permutation.

11. Number of points having position vectoriˆ aiˆ + bjˆ + ckˆ,


where a, b, c ∈ {1, 2, 3, 4, 5} such that 2a + 3b + 5c is I M P O R TA N T P O I N T S
divisible by 4 is
(A) 140 (B) 70 Permutation of things means arrangement of things. The
(C)  100 (D)  None of these word arrangement is used if order of things is taken into
account. Thus, if order of different things changes, then their
Solution: (B) arrangement also changes.

4m = 2a + 3b + 5c = 2a + (4 – 1)b + (4 + 1)c
4m = 4k + 2a + (–1)b + (1)c
Notations
\ a = 1, b = even, c = any number
Let r and n be positive integers such that 1 ≤ r ≤ n. Then,
a ≠ 1, b = odd, c = any number the number of permutations of n different things, taken r at
\ Required number = 1 × 2 × 5 × 4 × 3 × 5 = 70 a time, is denoted by the symbol nPr or P (n, r).
12. A family consists of grandfather, 5 sons and daughters
and 8 grandchildren. They are to be seated in a row for QUICK TIPS
dinner. The grandchildren wish to occupy the 4 seats
at each end and the grandfather refuses to have a grand n!
child on either side of him. The number of ways in 1. nPr = = n(n – 1) (n – 2) … [n – (r – 1)],
(n − r )!
which the family can be made to sit is 0 ≤ r ≤ n.
(A) 11360 (B) 11520 2. Number of permutations of n different things taken all at
(C)  21530 (D)  None of these a time is: nPn = n!.
Solution: (B) 3. The number of permutations of n things, taken all at a
time, out of which p are alike and are of one type, q are
The total number of seats alike and are of second type and rest are all different is
= 1 grand father + 5 sons and daughters n!
.
p! q!
+ 8 grand children
= 14
Permutations and Combinations  4.5

4. The number of permutations of n different things taken  All those numbers whose last three digit number is ­divisible
r at a time when each thing may be repeated any number by 8 are divisible by 8.
of times is nr.  All those numbers the sum of whose digits is divisible by
5. Permutations under Restrictions 9 are themselves divisible by 9.
(a) Number of permutations of n different things, taken  All those numbers whose last two digits are divisible by 25
r at a time, when a particular thing is to be always are themselves divisible by 25.
included in each arrangement, is
r ⋅ n – 1Pr – 1
(b) Number of permutations of n different things, taken SOLVED EXAMPLES
r at a time, when s particular things are to be always
included in each arrangement, is 13. The number of positive terms in the sequence
s! [r – (s – 1)] ⋅ n – sPr – s 195 n+3
P3
xn = − , n ∈ N is
(c) Number of permutations of n different things, taken n
4 Pn n +1
Pn + 1
r at a time, when a particular thing is never taken in
each arrangement, is (A) 2 (B) 3
n–1
Pr (C)  4 (D)  None of these
(d) Number of permutations of n different things, taken Solution: (C)
all at a time, when m specified things always come We have,
together, is m! × (n – m + 1)!. 195 n+3
p3
(e) Number of permutations of n different things, taken xn = n
− n +1
4 ⋅ pn pn + 1
all at a time, when m specified things never come 
together, is n! – m!× (n – m + 1)!. 195 ( n + 3) ( n + 2) ( n + 1)
6. Circular Permutations = −
4⋅ n! ( n + 1)! 
(a) Number of circular arrangements (permutations) of
195 ( n + 3) ( n + 2)
n different things is: (n – 1)!. = −
(b) Number of circular arrangements (permutations) of 4 ⋅ n! n! 
2
n different things when clockwise and anti-clockwise 195 − 4 n − 20 n − 24
arrangements are not different, i.e., when observa- =
4 ⋅ n! 
1
tion can be made from both sides is: (n − 1)! . 171 − 4 n2 − 20 n
2 =
(c) Number of circular permutations of n different things, 4 ⋅ n! 
taken r at a time, when clockwise, and anti-clockwise xn is positive.
n
P
orders are taken as different, is = r . 171 − 4 n2 − 20 n
r \ > 0
(d) Number of circular permutations of n different things,
4 ⋅ n!
taken r at a time, when clockwise and anti-clockwise ⇒ 4n2 + 20n – 171 < 0
n
P
orders are not different, is = r . which is true for n = 1, 2, 3, 4.
2r
Hence, the given sequence has 4 positive terms.
14. The number of ways in which the letters of the word
QUICK TIPS “STRANGE” can be arranged so that the vowels may
appear in the odd places, is
Numbers Divisible by 2, 3, 4, 5, 8, 9, 25 (A) 1440
 All those numbers having their last digit as an even number
(B) 1470
(i.e., 0, 2, 4, 6 or 8) are divisible by 2. (C) 1370
 All those numbers the sum of whose digits is divisible by
(D)  None of these
3 are themselves divisible by 3.
 All those numbers whose last two digits are divisible by 4
Solution: (A)
are themselves divisible by 4. There are 5 consonants and 2 vowels in the word
 All those numbers whose last digit is either 0 or 5 are STRANGE. Out of 7 places for the 7 letters, 4 places
divisible by 5. are odd and 3 places are even.
4.6  Chapter 4

Two vowels can be arranged in 4 odd places in P Each of the digits 3, 4, 5, 6 occurs in 3!
(4, 2) ways = 12 ways and then 5 consonants can be = 3 × 2 = 6 times in unit’s place.
arranged in the remaining 5 places in P(5, 5) ways
\ Sum of the digits in the unit’s place of all the
= 5 × 4 × 5 × 3 × 2 × 1 = 120 ways. numbers
Hence, the required number of ways = (3 + 4 + 5 + 6) × 6 = 18 × 6 = 108
= P (4, 2) × P (5, 5) = 12 × 120 = 1440 17. Three boys and three girls are to be seated around a
table, in a circle. Among them, the boy X does not want
15. How many numbers greater than 0 and less than a any girl neighbour and the girl Y does not want any boy
million can be formed with the digits from 0 to 9? neighbour. The number of such arrangements possible is
(A) 25927 (B) 27925 (A) 4 (B) 6
(C) 22759 (C) 72925 (C)  8 (D)  None of these
Solution: (A) Solution: (A)
Any number between 1 and 1000000 must be of less As shown in figure, 1, 2 and X are the three boys and
than seven digits. Therefore, it must be of the form 3, 4 and Y are three girls, Boy X will have neighbours
as boys 1 and 2 and the girl Y will have neighbors as
a1 a2 a3 a4 a5 a6
girls 3 and 4.
where a1, a2, a3, a4, a5, a6 ∈ {0, 1, 2, …, 9} 1 and 2 can be arranged in P (2, 2) ways
According to question, sum of the digits = 18 X
Thus, a1 + a2 + a3 + a4 + a5 + a6 = 18
1 2
where 0 ≤ ai ≤ 9, i = 1, 2, 3, …, 9.
Required number
= coefficient of x18 in (1 + x + x2 + … + x9)6
6 3 4
⎛ 1 − x 10 ⎞
18
= coefficient of x in ⎜ ⎟
⎝ 1− x ⎠ Y

= coeff. of x18 in [(1 – x10)6 (1 – x)– 6] = 2! = 2 × 1 = 2 ways.
18 6 10 –6
= coeff. of x in [(1 – C1 x ) (1 – x) ] Also, 3 and 4 can be arranged in P (2, 2) ways
(leaving terms containing powers of x greater than 18) = 2! = 2 × 1 = 2 ways.
18
= coeff. of x in (1 – x) –6 6
– C1. Hence, required number of permutations

coeff. of x8 in (1 – x)– 6 = 2 × 2 = 4

= 6 + 18 – 1C18 – 6 ⋅ 6 + 8 – 1C18 = 23C5 – 6 ⋅ 13C8 18. The number of divisors of 9600 including 1 and 9600 are
(A) 60 (B) 58 (C) 48 (D) 46
23 ⋅ 22 ⋅ 21⋅ 20 ⋅19 13 ⋅12 ⋅11⋅10 ⋅ 9
= −6
120 120  Solution: (C)
= 33649 – 7722 = 25927  9600 = 27 × 3 × 52
16. The sum of the digits in the unit place of all the num- \ No. of divisors = (7 + 1) × (1 + 1) × (2 + 1)
bers formed with the help of 3, 4, 5, 6 taken all at a
time is = 8 × 2 × 3 = 48
(A) 432 (B) 108 (C) 36 (D) 18
QUICK TIPS
Solution: (B)
The total number of numbers that can be formed with If N = p1a ⋅ p2a … pka , where p1, p2, …, pk are different primes
1 2 k

the digits 3, 4, 5, 6 taken all at a time and a1, a2,…, ak are natural numbers, then the total number
of divisors of N including, and N is (a1 + 1) (a2 + 1) …
= P (4, 4) = 4! = 24. (ak + 1).
Permutations and Combinations  4.7

19. There are six teachers. Out of them two are primary Solution: (B)
teachers, two are middle teachers and two secondary Four lines intersect each other in 4C2 = 6 points and
teachers. They are to stand in a row, so as the primary 4 circles intersect in 4P2 = 12 points. Each line cuts 4
teachers, middle teachers and secondary teachers are circles into 8.
always in a set. The number of ways in which they can \ 4 lines cut four circles into 32 points.
do so, is \ Required number = 6 + 12 + 32 = 50.
(A) 34 (B) 48
22. There are n seats round a table numbered 1, 2, 3, …, n.
(C)  52 (D)  None of these
The number of ways in which m(≤ n) persons can take
Solution: (B) seats is
There are 2 primary teachers. They can stand in a row (A) nPm (B)  n
Cm × (m – 1)!
in P (2, 2) = 2! 1 n n –1
(C)  . Pm (D)  Pm
= 2 × 1 ways = 2 ways 2
There are 2 middle teachers. They can stand in a row in Solution: (A)
P (2, 2) = 2! = 2 × 1 ways = 2 ways. Since the seats are numbered,
There are 2 secondary teachers. They can stand in \ the arrangement is not circular.
a row in P (2, 2) = 2! \ the required number of ways
= 2 × 1 ways = 2 ways = the number of arrangements of n things
These three sets can be arranged in themselves in taken m at a time
= 3! = 3 × 2 × 1 = 6 ways = nPm
Hence, the required number of ways COMBINATION
= 2 × 2 × 2 × 6 = 48
Each of the different groups or selections which can be
20. A teaparty is arranged for 16 people along two sides of made by taking some or all of a number of things (irrespec-
a large table with 8 chairs on each side. Four men want tive of order) is called a combination.
to sit on one particular side and two on the other side.
The number of ways in which they can be seated is I M P O R TA N T P O I N T S
6 ! 8! 10 ! 8! 8! 10 !
(A)  (B)  Combination of things means selection of things. Obviously,
4! 6! 4! 6!
in selection of things order of things has no importance.
8! 8! 6 ! Thus, with the change of order of things selection of things
(C)  (D)  None of these does not change.
6! 4!
Solution: (B)
Notations
There are 8 chairs on each side of the table. Let the
sides be represented by A and B. Let four persons sit The number of combinations of n different things taken r at
on side A, then number of ways of arranging 4 persons a time is denoted by nCr or C(n, r). Thus,
on 8 chairs on side A = 8P4 and then two persons sit on n n! n
Pr
side B. The number of ways of arranging 2 persons on Cr = = (0 ≤ r ≤ n)
r ! ( n − r )! r!
8 chairs on side B = 8P2 and the remaining 10 persons
can be arranged in remaining 10 chairs in 10! ways. n ( n − 1) ( n − 2) … ( n − r + 1)
=
Hence the total number of ways in which the r ( r − 1) ( r − 2) … 3 ⋅ 2 ⋅1 
­persons can be arranged If r > n,
n
8! 8! 10 ! then Cr = 0.
= 8P4 × 8P2 × 10! =
4! 6!
21. The maximum number of points into which 4 circles Info Box!
and 4 straight lines intersect is Selecting things without any order in called
(A) 26 (B) 50 combination and arrangement of things in some
(C) 56 (D) 72 order is called permutation.
4.8  Chapter 4

KEY RESULTS ON COMBINATION (e) Number of selections of one or more things out of
n identical things = n.
1. nCr = nCn – r , 0 ≤ r ≤ n (f ) If out of (p + q + r + t) things, p are alike of one
2. nC0 = nCn = 1, nC1 = n kind, q are alike of second kind, r are alike of third
3. If nCx = nCy then either x = y or x + y = n. kind and t are different, then the total number of
4. nCr + nCr–1 = n + 1Cr , 1 ≤ r ≤ n selections is
(p + 1) (q + 1) (r + 1) 2t – 1
5. r · nCr = n · n–1Cr–1
(g) The number of ways of selecting some or all out of
6. n · n–1Cr–1 = = (n – r + 1) nCr–1
n
p + q + r items where p are alike of one kind, q are
C n − r +1 alike of second kind and rest are alike of third kind
7. n r = ,1≤r≤n
C r −1 r is [(p + 1) (q + 1) (r + 1)] – 1.
8. If n is even then the greatest value of nCr is nCn/2.
Division into Groups
9. If n is odd then the greatest value of nCr is
n
C n +1  or  n C n −1 1. (a) Number of ways of dividing m + n different things
in two groups containing m and n things respec-
2 2
tively (m ≠ n) is
r decreasing numbers starting with n
10. nCr = m+n ( m + n)!
r increasing numbers starrting with 1 Cm =
m ! n!
n ( n − 1) ( n − 2) ....( n − r + 1)
= (b) Number of ways of dividing m + n + p differ-
1⋅ 2 ⋅ 3.....r
ent things in three groups containing m, n and p
11. nPr = r! nCr = n (n – 1) (n – 2) … (n – r + 1).
( m + n + p)!
12. nC0 + nC1 + nC2 + … + nCn = 2n. things respectively (m ≠ n ≠ p) is ,
m ! n! p !
13. nC0 + nC2 + nC4 + … = nC1 + nC3 + nC5 + … = 2n – 1. if the order of the groups is not important and
14. Number of combinations of n different things taken ( m + n + p )!
× 3! , if the order of the groups is
r at a time m ! n! p !
(a) when p particular things are always included = important.
n–p
Cr–p. (c) Number of ways of dividing 2m different things in
(b) when p particular things are never included = two groups, each containing m things and the order
n–p
Cr. ( 2n)!
of the groups is not important, is , of the
(c) when p particular things are not together in any 2 ! ( n !) 2
selection = nCr – n–pCr–p. ( m + n + p)!
groups is not important and × 3!, if
15. (a) Number of selections of r consecutive things out m ! n! p !
of n things in a row = n – r + 1. the order of the groups is important.
(b) Number of selections of r consecutive things out
of n things along a circle (d) Number of ways of dividing 2m different things
in two groups, each containing m things and the
⎧n, when r < n ( 2n)!
⎨ order of the groups is important, is .
⎩1, when r = n ( n !) 2
16. (a) Number of selections of zero or more things out of (e) Number of ways of dividing 3m different things
n different things in three groups, each containing m things and the
n (3m)!
C0 + nC1 + nC2 + … + nCn = 2n order of the groups is not important, is .
3! ( m !)3
(b) 
Number of combinations of n different things
(f ) Number of ways of dividing 3m diferent things
selecting at least one of them is
in three groups, each containing m things and the
n
C1 + nC2 + … + nCn = 2n – 1 (3m)!
order of the groups is important, is .
(c) Number of selections of r things (r ≤ n) out of n ( m !)3
identical things is 1. 2. (a) Number of ways of dividing n identical things into
(d) Number of selections of zero or more things out of r groups, if blank groups are allowed is
n identical things = n + 1. n+r–1
Cr – 1
Permutations and Combinations  4.9

(b) Number of ways of dividing n identical things Solution: (A)


into r groups, if blank groups are not allowed is We have,
n–1
Cr–1. 5
(c) Number of ways of dividing n identical things into
47
C4 + ∑ 52 − j
C3
r groups such that no group contains less than m j =1

things and more than k (m < k) things is coefficient 47


= C4 + 51C3 + 50C3 + 49C3 + 48C3+ 47C3
of xn in the expansion of
= (47C4 + 47C3) + 48C3 + 49C3 + 50C3 + 51C3
(xm + xm + 1 + … + xk)r
= 48C4 + 48C3 + 49C3 + 50C3 + 51C3
3. The number of ways of selecting r things out of n
things of which p are alike and are of one kind, q are = 49C4 + 49C3 + 50C3 + 51C3 = 50C4 + 50C3 + 51C3
alike and are of second, s are alike and are of third kind
= 51C4 + 51C3 = 52C4
and so on, is
= coefficient of xr in
24. 15C8 + 15C9 – 15C6 – 15C7 =
[(1 + x + x2 + … + xp) (1 + x + x2 + … + xq) × (1 + x +
(A) 8 (B) 0
x2 + … + xs) …] (C)  6 (D)  None of these
4. The number of ways of selecting r things out of n
things of which p are a like and are of one kind, q are Solution: (B)
alike and are of second kind and rest (n – p – q) things We have,
are all different is 15
C8 + 15C9 – 15C6 – 15C7
= coefficient of xr in
[(1 + x + x2 + … + xp) (1 + x + x2 + … + xq) (15C8 + 15C9) – (15C6 + 15C7)
=
× (1 + x)n–p–q]
= 16C9 – 16C7( nCr + nCr + 1 = n + 1Cr + 1)
5. The number of ways of selecting r things out of n
things of which p are alike and are of one kind, q are = 16C9 – 16C9( nCr = nCn – r)
alike and are of second kind, s are alike and are of third = 0
kind when each thing is taken at least once
= coefficient of xr – 3 in ⎛ n⎞ ⎛ n ⎞ ⎛ n ⎞
25. For 2 ≤ r ≤ n, ⎜ ⎟ + 2 ⎜ + =
[(1 + x + x2 + … + xp–1) (1 + x + x2 + … + xq–1) × ⎝ r⎠ ⎝ r − 1⎟⎠ ⎜⎝ r − 2⎟⎠
(1 + x + x2 + … + xs–1) …]
⎛ n + 1⎞ ⎛ n + 1⎞
6. The number of ways in which r identical things can be (A)  ⎜ (B) 
2⎜
distributed among n persons when each person can get ⎝ r − 1⎟⎠ ⎝ r + 1⎟⎠
zero or more things ⎛ n + 2⎞ ⎛ n + 2⎞
= coefficient of xr in (1 + x + x2 + … + xr)n (C) 2 ⎜ ⎟ (D) 
⎜⎝ r ⎟⎠
⎝ r ⎠
= coefficient of xr in (1 – x)–n = n + r – 1Cr
Solution: (D)
7. The number of non-negative integral solutions of the
equation x1 + x2 + … + xr = n is n + r – 1Cr.
n
Cr + 2nCr–1 + nCr–2 = (nCr + nCr–1) + (nCr–1 + nCr–2)
8. The number of terms in the expansion of
r n+r–1
= n+1Cr + n+1Cr–1
(a1 + a2 + a3 + … + an) is Cr
⎛ n + 2⎞
= n+2Cr = ⎜
SOLVED EXAMPLES ⎝ r ⎟⎠

26. A gentleman invites 13 guests to a dinner and places
5
23. 47
C4 + ∑ 52− j C3 = 8 of them at one table and remaining 5 at the other,
the tables being round. The number of ways he can
j =1
arrange the guests is
(A) 52C4
11!
(B) 51C4 (A)  (B) 
9!
40
(C) 52C3
12 ! 13!
(D)  None of these (C)  (D) 
40 40
4.10  Chapter 4

Solution: (D) 29. How many non-negative integers solutions does the


The number of ways in which 13 guests may be divided equation 3x + y + z + w = 30 have?
13!
into groups of 8 and 5 = 13C5 = . (A) 2151 (B) 1521
5! 8! (C) 1215 (C) 1512
Now, corresponding to one such group, the 8
guests may be seated at one round table in (8 – 1)! i.e., Solution: (C)
7! ways and the five guests at the other table in (5 – 1)! Let w be a non-negative integer such that
i.e., 4! ways.
3x + y + z + w = 30
But each way of arranging the first group of 8 per-
sons can be associated with each way of arranging the Let a = x – 1, b = y – 1, c = z – 1, d = w,
second group of 5, therefore, the two processes can be then
performed together in 7! × 4! ways. 3a + b + c + d = 25, where a, b, c, d ≥ 0 (1)
Hence required number of arrangements
Clearly, 0 ≤ a ≤ 8. If a = k, then
13! 13! 13!
= × 7! × 4! = × 7! × 4! = b + c + d = 25 – 3. (2)
5! 8! 5 ⋅ 4 ! 8 ⋅ 7! 40
Number of non-negative integral solutions of Eq. (2)
27. If there are 12 persons in a party, and if each of them
shakes hands with each other, then number of hand- = n + r – 1Cr
shakes happen in the party is
= 3 + 25 – 3k – 1C25 – 3k
(A) 66 (B) 48
(C)  72 (D)  None of these = 27 – 3kC25 – 3k = 27 – 3kC2
Solution: (A)
(27 − 3k ) (26 − 3k )
Total number of handshakes =
2 
=T
 he number of ways of selecting 2 persons
from among 12 persons = 3 (3k2 – 53k – 234)
2
12 x 11
= 12C2 = = 66 \ Required number
2x 1
8
28. The number of ways in which a committee of 5 can be 3
chosen from 10 candidates so as to exclude the young- = ∑
2k =0
(3k 2 − 53k + 234)
est if it includes the oldest, is 
(A) 196 (B) 178 3 ⎛ 8 × 9 × 17 8×9 ⎞
(C)  202 (D)  None of these = ⎜ 3 − 53 + 234 × 9⎟ = 1215
2⎝ 6 2 ⎠
Solution: (A)
There are two different ways of forming the committee 30. The number of different ways in which 8 persons can
 (i) oldest may be included stand in a row so that between two particular persons
(ii) oldest may be excluded A and B there are always two persons, is
 (i) If oldest is included, then youngest has to be (A)  60 (5!)
excluded and we are to select 4 candidates out (B) 4! × 5!
of 8. This can be done in (C)  15 (4!) × 5!
8 8! 8x 7x 6x 5 (D)  None of these
C4 = = = 70 ways
4! 4! 4x 3x 2
Solution: (A)
(ii) If oldest is excluded, then we are to select 5 candi-
dates from 9 which can be done in The number of selections of 4 persons including A, B
9 9! 9x 8x 7x 6 considering these four as a group, the number = 6C2 of
C5 = = = 126 ways
5! 4 ! 4x 3x 2x 1 arrangement with the other four = 5!.
 Hence, the total number of ways in which But in each group of four, number of arrange-
­committee can be formed ments  = 2! × 2! therefore, the required number of
= 126 + 70 = 196 ways = 6C2 × 5! × 2! × 2!.
Permutations and Combinations  4.11

31. There are 10 points in a plane of which no three points containing 6 questions. He is not permitted to attempt
are collinear but 4 points are concyclic. The number of more than 5 questions from each group. The number
different circles that can be drawn through atleast 3 of of ways in which he can choose the 7 questions is
these points is (A) 780 (B) 640
(A) 110 (B) 112 (C) 116 (D) 117 (C)  820 (D)  None of these
Solution: (D) Solution: (A)
Since a unique circle can be drawn through three A candidate can attempt 5 questions from group I and
points, therefore a selection of three points results in 2 from group II or 4 from group I and 3 from group II
a circle. So, the maximum number of circles using 10 or 3 from group I and 4 from group III or 2 from group
points is 10C3. Now, out of these 10 points 4 are con- I and 5 from group II. This can be done in
cyclic, hence 4C3 circles are actually single circle. 6
C5 × 6C2 + 6C4 × 6C3 + 6C3 × 6C4 + 6C2 × 6C5
\ Required number of circles = 10C3 – 4C3 + 1 = 117. = 6 × 15 + 15 × 20 + 20 × 15 + 15 × 6
32. The number of ways in which a committee of 3 ladies = 90 + 300 + 300 + 90 = 780
and 4 gentlemen can be appointed from a meeting con-
35. A boy has 3 Library tickets and 8 books of his inter-
sisting of 8 ladies and 7 gentlemen, if Mr X refuses to
est in the library. Out of these 8, he does not want to
serve in a committee if Mr Y is a member is
­borrow Chemistry part II, unless Chemistry part I is
(A) 1960 (B) 1540 also borrowed. The number of ways in which he can
(C)  3240 (D)  None of these choose the three books to be borrowed is
Solution: (B) (A) 41 (B) 32
3 ladies out of 8 can be selected in 8C3 ways and (C)  51 (D)  None of these
4 ­gentlemen out of 7 in 7C4 ways. Solution: (A)
Now each way of selecting 3 ladies is associated
The following are the different possibilities in which
with each way of selecting 4 gentlemen.
three books can be borrowed:
Hence, the required number of ways
  (i) When Chemistry part II is selected, then
= 8C3 × 7C4 = 56 × 35 = 1960 Chemistry part I is also borrowed and the third
book is selected from the remaining 6 books.
We now find the number of committees of 3 ladies and
(ii) When Chemistry part II is not selected, in this case
4 gentlemen in which both Mrs X and Mr Y are mem-
he has to select the three books from the remain-
bers. In this case, we can select 2 other ladies from the
ing 7 books.
remaining 7 in 7C2 ways and 3 other gentlemen from
First choice can be made in 6C1 = 6 ways.
the remaining 6 in 6C3 ways.
Second choice can be made in
\ The number of ways in which both Mrs X and
Mr Y are always included = 7C2 × 8C3 = 21 × 20 = 420. 7 7×6×5
C3 = = 35 ways
Hence, the required number of committes in which 1× 2 × 3
Mrs X and Mr Y do not serve together = 1960 – 420 = Total number of ways in which he can choose the
1540. three books to be borrowed = 6 + 35 = 41.
33. The number of ways in which a team of eleven players 36. The number of words that can be formed from the
can be selected from 22 players including 2 of them ­letters a, b, c, d, e, f, taken 3 at a time, each word
and excluding 4 of them is containing at least one vowel is
(A) 16C11 (B)  16
C5 (C)  16
C9 (D)  20
C9 (A) 96
(B) 84
Solution: (C)
(C) 106
Out of 22 players, 2 are to be included and 4 are to (D)  None of these
be excluded. We have to select a team of 11 players.
So the remaining 9 players are to be selected from the Solution: (A)
remaining 16 players. This can be done in 16C9 ways. The total number of words
34. A candidate is required to answer 7 questions out of = (2C1 × 4C2 + 2C2 × 4C1) 3!
12 questions which are divided into two groups each
= (12 + 4) × 6 = 96.
4.12  Chapter 4

37. In an examination a candidate has to pass in each of 40. The number of ways in which a mixed doubles game
the papers to be successful. If the total number of in tennis can be arranged from 5 married couples, if no
ways to fail is 63, how many papers are there in the husband and wife play in the same game, is
examination? (A) 46 (B) 54
(A) 6 (B) 8 (C)  60 (D)  None of these
(C)  14 (D)  None of these
Solution: (C)
Solution: (A) Let the sides of the game be A and B. Given 5 married
Let the number of papers be n. couples, i.e., 5 husbands and 5 wives. Now, 2 husbands
\ Total number of ways to fail or pass for two sides A and B can be selected out of 5 = 5C2 =
n 10 ways.
C0 + nC1 + nC2 + … + nCn = 2n
After choosing the two husbands their wives are
But there is only one way to pass, i.e., when he fails to be excluded (since no husband and wife play in
in none. the same game). So we are to choose 2 wives out of
\ Total number of ways to fail = 2n – 1 remaining 5 – 2 = 3 wives, i.e., 3C2 = 3 ways.
\ From question, 2n – 1 = 63; Again two wives can interchange their sides A and
\ 2n = 64 = 26 B in 2! = 2 ways.
Therefore, the required number of ways = 10 × 3 ×
\ n = 6. 2 = 60.
38. The total number of selections from 4 boys and 3 girls 41. The number of seven letter words that can be formed
if each selection has to contain at least one boy is by using the letters of the word SUCCESS so that the
(A) 106 (B) 120 two C are together but no two S are together, is
(C)  240 (D)  None of these (A) 24 (B) 36
Solution: (B) (C)  54 (D)  None of these
Number of selections of at least one boy from 4 boys Solution: (A)
= 4C1 + 4C2 + 4C3 + 4C4 = 24 – 1 Considering CC as single object, U, CC, E can be
Number of selections of any number of girls from arranged in 3! ways
3 girls × U × CC × E ×
3 3 3 3 3
= C0 + C1 + C2 + C3 = 2 Now the three S are to be placed in the four available
places.
\ Required number of selections of at least one boy Hence required number of ways = 3! · 4C3 = 24.
from 4 boys and 3 girls = (24 – 1)23 = 15 × 8 = 120.
42. Four boys picked up 30 mangoes. The number of ways in
39. A boat is to be manned by eight men of whom 2 can which they can divide them if all mangoes be identical, is
only row on bow side and 1 can only row on stroke
(A) 5456 (B) 3456
side; the number of ways in which the crew can be
(C)  5462 (D)  None of these
arranged is
(A) 4360 (B) 5760 Solution: (A)
(C)  5930 (D)  None of these Clearly, 30 mangoes can be distributed among 4 boys
such that each boy can receive any number of mangoes.
Solution: (B)
Hence, total number of ways = 30 + 4 – 1C4 – 1 = 33C3
First we have to select 2 men for bow side and 3 for
stroke side. 33.32.31
= = 5456
\ The number of selections of the crew for two sides 1.2.3
= 5C2 × 3C3
QUICK TIPS
For each selection, there are 4 persons each on both
sides who can be arranged in 4! × 4! ways. Number of ways of dividing n idential things into r groups, if
\ Required number of arrangements blank groups are allowed is
5x 4 n + r – 1
Cr – 1
= 5C2 × 3C3 × 4! × 4! = x 1 x 24 x 24 = 5760
2
Permutations and Combinations  4.13

43. The number of ways of choosing 10 balls from infinite Solution: (B)


white, red, blue and green balls is Number of ways in which at least one question can be
(A) 70 (B) 84 (C) 286 (D) 86 selected out of 3 are
3
Solution: (C) C1 + 3C2 + 3C3 = 23 – 1 = 7
⎧⎪ Coefficient of x ⎫⎪ 10
Number of ways in which at least one question can be
Required ways = ⎨ ⎬
2 4 selected out of 4 are
⎩⎪in (1 + x + x + ...) ⎭⎪
4
4 C1 + 4C2 + 4C3 + 4C4 = 24 – 1 = 15
⎛ 1 ⎞
10
=
Coefficient of x in ⎜ \ Total number of ways = 7 × 15 = 105
⎝ 1 − x ⎟⎠

46. Given 5 different green dyes, 4 different blue dyes and
Coefficient of x10 in (1 – x)–4
=
3 different red dyes, the number of combinations of
= Coefficient of x10 in dyes that can be chosen by taking atleast one green and
⎛ 5.4 2 4.5.6 3 ⎞ one blue dye is
⎜⎝1 + 4 x + 2 ! x + 3! x + …⎟⎠ (A) 248 (B) 120
(C) 3720 (D) 465
= Coefficient of x10 in (1 + 4C1x + 5C2x2 + 6C3x3
Solution: (C)
+ 7C4x4 + 8C5x5 + 9C6x6 + … 13C10x10)
Number of ways of selecting at least one green dye out
13.12.11 of 5 different green dyes
\ Required number of ways = 13C10 = = 286
3.2.1 5
C1 + 5C2 + 5C3 + 5C4 + 5C5 = 25 – 1
44. In a class tournament where the participants were to
play one game with another, two class players fell ill, Also, at least one blue dye can be selected out of 4
having played 3 games each. If the total number of blue dyes in
games played is 84, the number of participants at the 4
C1 + 4C2 + 4C3 + 4C4 = 24 – 1
beginning was
(A) 22 (B) 15 Again, 3 different red dyes can be selected in
(C)  17 (D)  None of these 3
C0 + 3C1 + 3C2 + 3C3 = 23 ways
Solution: (B) \ Required ways = (25 – 1) (24 – 1) (23) = 3720
Suppose the two players did not play at all so that the
47. Out of 18 points in a plane no three are in the same
remaining (n – 2) players played n – 2C2 matches. Since
straight line except five points which are collinear. The
these two players played 3 matches each, hence the
number of straight lines that can be formed joining
total number of matches is
them is
n–1
C2 + 3 + 3 = 84  (given) (A) 143 (B) 144
( n − 2)( n − 3) (C)  153 (D)  None of these
or = 78 or n2 – 5n + 6 = 156
1.2 Solution: (B)
or n2 – 5n – 150 = 0 or (n – 15) (n + 10) = 0 The number of st. lines
\ n = 15 (n ≠ –10) = 18C2 – (5C2 – 1) = 144
45. An examination paper, which is divided into two
groups consisting of 3 and 4 questions respectively DERANGEMENT
carries the note: It is not necessary to answer all the
questions. One question atleast should be answered Rearrangement of objects such that no one goes to its
from each group. The number of ways can an exam- ­original place is called derangement.
inee select the questions is If ‘n’ things are arranged in a row, the number of
ways in which they can be deranged so that none of them
(A) 22
occupies its original place is
(B) 105
n
(C) 3P3 × 4P4 ⎛ 1 1 1 1⎞ 1
n ! ⎜1 − + − + … + ( −1) n ⎟ = n ! ∑ ( −1) r
(D) 3C3 × 4C4 ⎝ 1! 2 ! 3! n !⎠ r =0 r !
4.14  Chapter 4

For example, if 4 letters are taken out of 4 different


­envelopes, then the number of ways in which they can be SOLVED EXAMPLES
reinserted in the envelopes so that no letter goes in to its
original envelope 48. The number of diagonals in a polygon of n sides is
⎛ 1 1 1⎞ n ( n − 3) n ( n − 1)
= 4 ! ⎜1 − 1 + − + ⎟ (A)  (B) 
⎝ 2 ! 3! 4 !⎠  2 2
( n − 1) ( n − 2)
⎛ 1 1 1⎞ (C)  (D)  None of these
= 24 ⎜1 − 1 + − + ⎟ = 9 2
⎝ 2 6 24 ⎠
Solution: (A)
Some Useful Results for The number of diagonals + number of sides = number
Geometrical Problems of selections of two vertices from n vertices
1. If n distinct points are given in the plane such that no \ the number of diagonals. = nC2 – n
three of which are collinear, then the number of line n ( n − 1) n 2 − n − 2n n ( n − 3)
segments formed = nC2. = −n = = .
2 2 2
If m of these points are collinear (m ≥ 3), then the
­number of line segments is (nC2 – mC2) + 1. 49. If each of 10 points on a straight line be joined to each
of 10 points on a parallel line then the total number of
2. The number of diagonals in an n-sided closed
triangles that can be formed with the given points as
­polygon = nC2 – n.
vertices, is
3. If n distinct points are given in the plane such that no
(A) 860 (B) 900
three of which are collinear, then the number of trian-
(C)  920 (D)  None of these
gles formed = nC3.
If m of these points are collinear (m ≥ 3), then the num- Solution: (B)
ber of triangles formed = nC3 – mC3. A triangle is formed for each selection of 2 points
4. If n distinct points are given on the circumference of a from one line and 1 point from the other line.
circle, then \ The number of triangles
(a) Number of st. lines = nC2
= 10C2 × 10C1 + 10C1 × 10C2
(b) Number of triangles = nC3
(c) Number of quadrilaterals = nC4 and so on 10 × 9 10 × 9
= × 10 + 10 × = 900
5. Number of Rectangles and Squares: 2 2
(a) Number of rectangles of any size in a square of
n EXPONENT OF PRIME p IN n!
size n × n is ∑ r 3 and number of squares of any Exponent of a prime p in n! is denoted by Ep (n!) and is
r =1
n given by,
size is ∑ r 2. ⎛ n⎞ ⎛ n ⎞ ⎛ n ⎞ ⎛ n⎞
r =1 Ep (n!) = ⎜ ⎟ + ⎜ 2 ⎟ + ⎜ 3 ⎟ + .... + ⎜ k ⎟ ,
(b) Number of rectangles of any size in a rectangle of ⎝ p⎠ ⎝ p ⎠ ⎝ p ⎠ ⎝p ⎠
np
size n × p (n < p) is (n + 1) (p + 1) and number where pk < n < pk + 1
4 n
of squares of any size is ∑ ( n + 1 − r ) ( p + 1 − r ) . ⎛ n⎞
r =1
and  ⎜⎝ p ⎟⎠ denotes the greatest integer less than or
6. If m parallel lines in a plane are intersected by a family n
equal to .
of other n parallel lines. Then, total number of parallel- p
ograms so formed is For example, exponent of 3 in (100)! is
mC × nC
⎛ 100 ⎞ ⎛ 100 ⎞ ⎛ 100 ⎞ ⎛ 100 ⎞
E3 (100!) = ⎜ + + +
2 2

mn ( m − 1) ( n − 1) ⎝ 3 ⎟⎠ ⎜⎝ 32 ⎟⎠ ⎜⎝ 33 ⎟⎠ ⎜⎝ 34 ⎟⎠

i.e., 
4  = 33 + 11 + 3 + 1 = 48
Permutations and Combinations  4.15

NUMBER OF DIVISORS 5. The number of ways in which N can be resolved as a


a a a a product of two factors is
Let N = p11 . p 22 . p33 ... p k k . where p1, p2, p3, … pk are dif-
ferent primes and a1, a2, a3, …, ak are natural numbers then: ⎧1
⎪⎪ 2 (α1 + 1)(α 2 + 1)… (α k + 1), If N is not a perfect square
1. The total number of divisors of N including 1 and N is ⎨
= (a1 + 1) (a2 + 1) (a3 + 1) … (ak + 1). ⎪ 1 [(α + 1)(α + 1)… (α + 1) + 1], If N is a perfect square
⎪⎩ 2 1 2 k
2. The total number of divisors of N excluding 1 and N is
= (a1 + 1) (a2 + 1) (a3 + 1) … (ak + 1) – 2. 6. The number of ways in which a composite number N
3. The total number of divisors of N excluding 1 or N is can be resolved into two factors which are relatively
= (a1 + 1) (a2 + 1) (a3 + 1) … (ak + 1) – 1. prime (or co-prime) to each other is equal to 2n–1,
4. The susm of these divisors is where n is the number of different factors in N.
= ( p10 + p11 + p12 + ... + p1a )
a2
( p20 + p12 + p22 + ... + p2 )
a
( p k0 + p1k + p k2 + ... + p k k )
4.16  Chapter 4

NCERT EXEMPLARS
1. If n C12 = n C8 , then n is equal to 10. The number of ways in which a team of eleven players
can be selected from 22 player always including 2 of
(A) 20 (B) 12 (C) 6 (D) 30 them and excluding 4 of them is
2. The number of possible outcomes when a coin is (A) 16C11 (B)  16
C5
16 20
tossed 6 times is (C) C9 (D)  C9
(A) 36 (B) 64 (C) 12 (D) 32 11. The number of 5-digit telephone numbers having
3. The number of different four-digit numbers that can be atleast one of their digits repeated is
formed with the digits 2, 3, 4, 7 and using each digit (A) 90000 (B) 10000
only once is (C) 30240 (D) 69760
(A) 120 (B) 96 (C) 24 (D) 100 12. The number of ways in which we can choose a com-
4. The sum of the digits in unit place of all the numbers mittee from four men and six women, so that the com-
formed with the help of 3, 4, 5 and 6 taken all at a time mittee includes atleast two men and exactly twice as
is many women as men is
(A) 432 (B) 108 (C) 36 (D) 18 (A) 94 (B) 126
(C)  128 (D)  None of these
5. The total number of words formed by 2 vowels and 3
consonants taken from 4 vowels and 5 consonants is 13. The total number of 9-digit numbers which have all
(A) 60 (B) 120 (C) 7200 (D) 720 different digits is
(A) 10! (B) 9!
6. If a five-digit number divisible by 3 is to be formed (C)  9 × 9! (D)  10 × 10!
using the numbers 0, 1, 2, 3, 4 and 5 without repeti-
tions, then the total number of ways this can be done is 14. The number of words which can be formed out of the
(A) 216 (B) 600 (C) 240 (D) 3125 letters of the word ‘ARTICLE’, so that vowels occupy
the even place is
7. Everybody in a room shakes hands with everybody (A) 1440 (B) 144
else. If the total number of hand shakes is 66, then the
(C)  7! (D) 
4
C 4 × 3 C3
total number of persons in the room is
(A) 11 (B) 12 (C) 13 (D) 14 15. Given 5 different green dyes, four different blue dyes
and three different red dyes, the number of combina-
8. The number of triangles that are formed by choosing tions of dyes which can be chosen taking atleast one
the vertices from a set of 12 points, seven of which lie green and one blue dye is
on the same line is
(A) 3600 (B) 3720
NCERT EXEMPLARS

(A) 105 (B) 15 (C) 175 (D) 185


(C) 3800 (D) 3600
9. The number of parallelograms that can be formed
from a set of four parallel lines intersecting another set
of three parallel lines is
(A) 6 (B) 18 (C) 12 (D) 9

ANSWER K EYS
  1. (A) 2.  (B) 3. (C) 4.  (B) 5. (C) 6.  (A) 7. (B) 8.  (D) 9. (B) 10. (C)
11. (D) 12.  (A) 13. (C) 14.  (B) 15. (B)
Permutations and Combinations  4.17

HINTS AND EXPLANATIONS


n
1. Given that, C12 = n C8 , 7. Let the total number of person in the room is n.
∵ nC = n C  We know that, two person form 1 hand shaken.
⇒ nCn−12 = n C8  r n− r 
∴ Required number of hand shakes
⇒ n − 12 = 8
n! n ( n − 1)
⇒ n = 12 + 8 = 20 aα = nC2 = =
2!( n − 2 )! 2
2. Number of outcomes when tossing a coin 1 times = 2 (head n ( n − 1)
or tail)
According to the question, = 66
2
∴ Total possible outcomes when a coin tossed 6 times = 26
∵ 2n for n time tossed coin  ⇒ n ( n − 1) = 132
= 64  
⇒ n2 − n − 132 = 0
3 Given, digits 2, 3, 4 and 7, we have to form four-digit num-
bers using these digits. ⇒ ( n − 12) ( n + 11) = 0
∴ Required number of ways = 4 P4 = 4! = 4 × 3 × 2! = 24 ⇒ n = 12, − 11 [inadmissible]
4. If we fixed 3 at units place.
Total possible number is 3!, i.e., 6. ∴ n = 12

Sum of the digits in unit place of all these numbers = 3! × 3 8. Total number of triangles formed from 12 points taking 3 at
Similarly, if we fixed 4, 5 and 6 at units place, in each case time = 12C3
total possible numbers are 3!. But out of 12 points 7 are collinear. So, these 7 points con-
Required sum of unit digits of all such numbers stitute a straight line mean no triangle is formed by joining
= (3 + 4 + 5 + 6) × 3! these 7 points.
= 18 × 3! = 18 × 6 =108 ∴
Required number of triangles = 12C3 −7 C3
5. Given that, number of vowels = 4 = 220 − 35 = 185a
and total number of consonants = 5 9. To form parallelogram we required a pair of line from a set
Total number of words formed by 2 vowels and 3 consonants of 4 lines another pair of line from another set of 3 lines.
4! 5! ∴ Required number of parallelograms
= 4C2 × 3C2

HINTS AND EXPLANATIONS



= 4 C2 ×5 C3 = ×
2! 2! 3! 2! = 6 × 3 = 18
4 × 3! 5 × 4 × 3 × 2! 4 × 5 × 4 × 3

= × = 10. Total number of players = 22
2! 2! 3! × 2! 4
We have to select a team of 11 players. Selection of 11
= 5 × 4 × 3 = 60 players when 2 of them ia always included and 4 are never
Choose what order they appear in 5!, i.e., 120. included.
So, total number of words = 60 × 120 = 7200 Total number of player = 22 – 2 – 4 = 16
6. We know that, a number is divisible by 3, when sum of digits ∴ Required number of selections = 16C9
in the number must be divisible by 3. 11. If all the digits repeated, then number of 5 digit telephone
So, if we consider the digits 0, 1, 2, 4, 5, then (0 + 1 + 2 + 4 + numbers can be formed in 105 ways and if no digit repeated,
5) = 12 then 5-digit telephone numbers can be formed in 10P5 ways.
We see that, sum is divisible by 3. Therefore, five-digit num-
10!
bers using the digit ∴ Required number of ways 105 −10 P5 = 100000 −

5!
0, 1, 2, 4, 5 = 4 × 4 × 3 × 2 × 1 = 96
= 100000 – 10 × 9 × 8 × 7 × 6
4 4 3 2 1 = 100000 – 30240 = 69760

and if we consider the digit 1, 2, 3, 4, 5, then (1 + 2 + 4 + 5 12. ∴ Number of men = 4
= 15) and number of women = 6

This sum is also divisible by 3. It is given committee includes two men and exactly twice as

So, five-digit number can be formed using the digit 1, 2, 3, 4, many
5 in 5! ways. women as men Thus, possible selection is given in following

Total number of ways = 96 + 5! = 96 + 120 = 216 table
4.18  Chapter 4

Men Women
Since, it is given that vowels occupy even place, therefore
the arrangement of vowel, consonant can be understand with
2 4 the help of following diagram.
3 6 1 2 3 4 5 6 7
Now, vowels can be placed at 2, 4 and 6th position.

Required number of committee formed Therefore, number of arrangement = 3P3 = 3! = 6 ways
= 4 C2 × 6C4 + 4C3 × 6C6 And consonants can be placed at 1, 3, 5 and 7th position.
= 6 × 15 + 4 × 1 = 94 ∴ Total number of words = 6 × 24 = 144
13. (C) We have to form 9-digit numbers with the digit 0, 1, 2, 15. (B) Possible number of choosing green dyes = 25
3, 4, 5, 6, 7, 8, 9, 0 cannot be placed at the first place from Possible number of choosing blue dyes = 24
left. So, first place from left can be filled in 9 way. Possible number of choosing red dyes = 23
Since, repetition is not allowed, so remaining 8 places can be If at least one blue and one green dyes are selected.
filled in 9! ways.
∴ Required number of ways = 9 × 9! ( )( )
Then, total number of selection = 25 − 1 24 − 1 × 23 = 3720

14. (B) Total number of letters in the word article is 7, out of
which A, E, I are vowels and R, T, C, L are consonants.
HINTS AND EXPLANATIONS
Permutations and Combinations  4.19

PRACTICE EXERCISES

Single Option Correct Type

1. Let y be an element of the set A = {1, 2, 3, 5, 6, 10, 15, 9. If a represents the number of permutations of (x + 2)
30} and x1, x2, x3 be integers such that x1x2x3 = y, then things taken together, b represents the number of per-
the number of positive integral solutions of x1x2x3 = y is mutations of 11 things taken together out of x things,
(A) 64 (B) 27 and c represents the number of permutations of (x – 11)
(C)  81 (D)  None of these things taken together so that a = 182bc, then x =
(A) 15 (B) 12 (C) 10 (D) 18
6! 3!
× × 4!
4! × 2! 2! × 1!

2. If m = number of distinct rational numbers ∈


10. How many different nine digit numbers can be formed
(0, 1) such that p, q ∈ {1, 2, 3, 4, 5} and n = number from the number 22 33 55 8 88 by rearranging its dig-
of mappings from {1, 2, 3} onto {1, 2}, then m – n is its so that the odd digits occupy even positions?
(A) 1 (B) –1 (A) 16 (B) 36 (C) 60 (D) 180
(C)  0 (D)  None of these
11. For a game in which two partners play against two
3. The letters of the word RANDOM are written in all other partners, six persons are available. If every pos-
possible orders and these words are written out as in a sible pair must play with every other possible pair,
dictionary then the rank of the word RANDOM is then the total number of games played is
(A) 614 (B) 615 (C) 613 (D) 616 (A) 90 (B) 45 (C) 30 (D) 60
4. If eight persons are to address a meeting then the num- 12. A five digit number divisible by 3 is to be formed using
ber of ways in which a specified speaker is to speak the numerals 0, 1, 2, 3, 4 and 5 without repetition. The
before another specified speaker, is total number of ways this can be done is
(A) 40320 (B) 2520 (A) 216 (B) 600 (C) 240 (D) 3125
(C)  20160 (D)  None of these
13. A box contains two white balls, three black balls and
5. The number of permutations of letters a, b, c, d, e, f, g four red balls. The number of ways in which three balls
so that neither the pattern beg nor cad appears is can be drawn from the box if atleast one black ball is to
6! × 3! 7! be included in the draw, is
(A)  = 1080 (B) 
( 2!) 2
2 ! 3! 3! (A) 32 (B) 64
(C)  128 (D)  None of these

PRACTICE EXERCISES
(C)  4806 (D)  None of these
6. The number of ways of selecting 10 balls from the 14. The sum of all the numbers that can be formed with
unlimited number of red, green, white and yellow the digits 2, 3, 4, 5 taken all at a time is
balls, if selection must include 2 red and 3 yellow (A) 66666 (B) 84844
balls, is (C)  93324 (D)  None of these
(A) 36 (B) 56 15. If the number of ways in which n different things can
(C)  112 (D)  None of these be distributed among n persons so that at least one per-
7. Let A = {1, 2, 3, 4} and B = {1, 2}. Then, the number son does not get any thing is 232. Then n is equal to
of onto functions from A to B is: (A) 3 (B) 4
(A) 8 (B) 14 (C)  5 (D)  None of these
(C)  12 (D)  None of these n

8. Given five line segments of lengths 2, 3, 4, 5, 6 units. 16. m


Cr + 1 + = ∑ k Cr =
k =m
Then the number of triangles that can be formed by n +1
joining these lines is (A) nCr + 1 (B)  Cr + 1
n
(A) 5C3 – 3 (B)  5C3 – 1 (C)  Cr (D)  None of these
5 5
(C)  C3 (D)  C3 – 2 17. Two straight lines intersect at a point O. Points A1,
A2, …, An are taken on one line and points B1, B2, …,
4.20  Chapter 4

Bn on the other. If the point O is not to be used, the for 6 of the animals, then the number of ways of cag-
number of triangles that can be drawn using these ing the animals is
points as vertices, is (A) 304800 (B) 504800
(A) n (n – 1) (B)  n (n – 1)2 (C)  604800 (D)  None of these
2
(C) n (n – 1) (D)  n2 (n – 1)2
28. If n is even and
18. If the letters of the word MOTHER are written in all n
C0 < nC1 < nC2 < … < nCr > nCr+1 > … > nCn
possible orders and these words are written out as in a then r =
dictionary, then the rank of the word MOTHER is
n n −1
(A) 240 (B) 261 (C) 308 (D) 309 (A)  (B) 
2 2
19. The number of divisors a number 38808 can have, n−2 n+2
excluding 1 and the number itself is (C)  (C) 
2 2
(A) 70 (B) 72
(C)  71 (D)  None of these 29. In a network of railways, a small island has 15 stations.
The number of different types of tickets to be printed
20. The number of positive integral solutions of 15 < x1 + for each class, if every station must have tickets for
x2 + x3 ≤ 20, is equal to other station, is
(A) 785 (B) 685 (A) 230 (B) 210
(C)  1150 (D)  None of these (C)  340 (D)  None of these
21. The number of different 7 digit numbers that can be 30. The number of ordered pairs (m, n), m, n ∈ {1, 2,…,
written using only the three digits 1, 2 and 3 with the 50} such that 6n + 9m is a multiple of 5 is
condition that the digit 2 occurs twice in each number is (A) 6250 (B) 1250
(A) 7P225 (B)  7
C2 25 (C)  1875 (D)  None of these
(C) 7C252 (D)  None of these 31. A set contains (2n + 1) elements. The number of sub-
22. The tensdigit of 1! + 2! + 3! + … + 49! is sets of the set which contain at most n elements is
(A) 1 (B) 2 (C) 3 (D) 4 (A) 2n (B)  2n+1
2n–1
(C) 2 (D)  22n
23. Let S be the set of all functions from the set A to the set
32. There are n concurrent lines and another line parallel
A. If n (A) = k then n (S ) is
to one of them. The number of different triangles that
(A) k! (B)  kk (C) 2k – 1 (d) 2k will be formed by the (n + 1) lines, is
24. There are three coplanar parallel lines. If any p points
PRACTICE EXERCISES

( n −1) n ( n − 1) ( n − 2)
are taken on each of the lines, the maximum umber of (A)  (B) 
2 2
triangles with vertices at these points is
(A) 3pC3 (B)  p2 (p – 1) n( n +1) ( n + 1) ( n + 2)
(C)  (D) 
(C) p2 (4p – 1) (D)  p2 (4p – 3) 2 2

25. The number of ways in which thirty five apples can 33. An n-digit number is a positive number with exactly
be distributed among 3 boys so that each can have any n digits. Nine hundred distinct n-digit numbers are to
number of apples, is be formed using only the three digits 2, 5 and 7. The
smallest value of n for which this is possible is
(A) 1332 (B) 666
(C)  333 (D)  None of these (A) 6 (B) 7 (C) 8 (D) 9
34. If all permutations of the letters of the word AGAIN are
26. The number of non-negative solutions of x1 + x2 + x3
arranged as in dictionary, the forty ninth word is
+, …, + xn ≤ n (where n is positive integer) is
(A) NAAGI (B)  NAGAI
(A) 2nCn – 1 (B)  2n–1Cn – 1 (C)  NAAIG (D) NAIAG
(C) 2n+1Cn – 1 (D)  2n–1
Cn–1 – 1
35. The number of ways of choosing n objects out of
27. Eleven animals of a circus have to be placed in eleven (3n + 1) objects of which n are identical and (2n + 1)
cages one in each cage. If 4 of the cages are too small are distinct, is
Permutations and Combinations  4.21

(A) 22n (B)  22n+1 m


∑ n + r C k , then
2n
(C) 2 – 1 (D)  None of these 45. If S =
r=0
36. In a group of boys, two boys are brothers and in this
group 6 more boys are there. In how many ways they (A) S + nCk+1 = n+mCk+1
can sit if the brothers are not to sit along with each (B) S + nCk+1 = n+m+1Ck+1
other (C) S + nCk = n+mCk
(A) 4820 (B) 1410
(D)  None of these
(C)  2830 (D)  None of these
46. The number of ways of dividing 15 men and 15
37. If 20% of three subsets (i.e., subsets containing exactly women into 15 couples, each consisting of a man and
three elements) of the set A = {a1, a2,…, an} contain a woman, is
a1, then the value of n is (A) 1240 (B) 1840
(A) 15 (B) 16 (C) 17 (C) 18 (C) 1820 (D) 2005
38. The number of two digit numbers which are of the 47. Suman writes letters to his five friends. The number of
form xy with y < x are given by ways can be letters be placed in the envelopes so that
(A) 45 (B) 55 atleast two of them are in the wrong envelopes are
(C)  17 (D)  None of these (A) 119 (B) 120
(C)  125 (D)  None of these
39. A crocodile is known to have not more than 68 teeth.
The total number of crocodiles with different set of 48. Assuming the balls to be identical except for differ-
teeth is ence in colours, the number of ways in which one or
more balls can be selected from 10 white, 9 green and
(A) 68 (B) 68! (C) 1617 (D) 6868
7 black balls is
40. For x ∈ R, let [x] denotes the greatest integer ≤ x, then (A) 880 (B) 629 (C) 630 (D) 879
the value of
49. The number of 4-digit numbers with distinct digits is
 1  1 1   1 2 
 3  +  − 3 − 100  +  − 3 − 100 
− (A) 504 (B) 4536
      (C) 4634 (D) 5040
 1 99 
+, ..., +  − − is 50. In a shop there are five types of ice-creams available.
 3 100  A child buys six ice-creams.
(A) –100 (B) – 123 (C) –135 (D) –153 Statement 1: The number of different ways the child
41. The total number of ways in which a beggar can be can buy the six ice-creams is 10C5.

PRACTICE EXERCISES
given at least one rupee from four 25 p. coins, three Statement 2: The number of different ways the child
50 p. coins and 2 one rupee coins is can buy the six ice-creams is equal to the number of
different ways of arranging 6 A’s and 4 B’s in a row.
(A) 54 (B) 53 (C) 51 (D) 48
(A) Statement 1 is false, Statement 2 is true
42. A student is allowed to select atmost n books from a (B) Statement 1 is true, Statement 2 is true, Statement 2
collection of (2n + 1) books. If the total number of is a correct explanation for Statement 1
ways in which he can select books is 63, then n = (C) Statement 1 is true, Statement 2 is true; Statement 2
(A) 4 (B) 3 (C) 7 (D) 8 is not a correct explanation for Statement 1
(D)  Statement 1 is true, Statement 2 is false
43. In a certain test there are n questions. In this test 2k
­students gave wrong answers to at least (n – k) ques- 51. In an examination a candidate has to pass in each of the
tions, where k = 0, 1, 2,…, n. If the total number of papers to be successful. If the total number of ways to
wrong answers is 4095, then value of n is fail is 63, how many papers are there in the examination?
(A) 11 (B) 12 (C) 13 (D) 15 (A) 6 (B) 8
(C)  14 (D)  None of these
44. The number of permutations of the letters a, b, c, d
such that b does not follow a, c does not follow b, and
d does not follow c, is
52. ∑ ∑ 10
C j j C i is equal to
0 ≤ i ≤ j ≤ 10
(A) 12 (B) 14 (C) 13 (D) 11
(A) 310 (B) 310 – 1 (C)  210 (D) 210 – 1
4.22  Chapter 4

53. The number of ways of choosing n objects out of (A) 38664 (B) 48664


(3n + 1) objects of which n are identical and (2n + 1) (C)  58664 (D)  None of these
are distinct, is 63. The number of four digit numbers that can be formed
(A) 22n (B)  22n+1 from the digits 0, 1, 2, 3, 4, 5 with at least one digit
(C) 22n – 1 (D)  None of these repeated is
2n (A) 420 (B) 560
54. Cr (0 ≤ r ≤ 2n) is greatest when r is equal to
(C)  780 (D)  None of these
n n +1
(A)  (B)  64. The number of odd numbers lying between 40000 and
2 2
70000 that can be made from the digits 0, 1, 2, 4, 5, 7
(C) r = n (D)  None of these
if digits can be repeated in the same number is
55. The number of even numbers greater than 100 that (A) 864 (B) 932
can be formed by the digits 0, 1, 2, 3 (no digit being (C)  766 (D)  None of these
repeated) is
65. A table has provision for 7 seats, 4 being on one side
(A) 20 (B) 30 facing the window and 3 being on the opposite side.
(C)  40 (D)  None of these The number of ways in which 7 people can be seated
56. The number of positive numbers less than 1000 and at the table if 2 people, X and Y, must sit on the same
divisible by 5 (no digit being repeated) is side, is
(A) 150 (B) 154 (A) 3260 (B) 2160
(C)  166 (D)  None of these (C)  3350 (D)  None of these
57. In a certain city, all telephone numbers have six digits, 66. There are four oranges, five apples and six mangoes in
the first two digits always being 41 or 42 or 46 or 62 a fruit basket. The number of ways in which a person
or 64. The number of telephone numbers having all six can make a selection of fruits among the fruits in the
digits distinct is basket, is
(A) 8400 (B) 7200 (A) 210 (B) 330
(C)  9200 (D)  None of these (C)  209 (D)  None of these

58. The total number of ways of selecting five letters from 67. The number of zeros at the end of 100! is
the letters of the word INDEPENDENT, is (A) 36 (B) 18
(A) 4200 (B) 3320 (C)  24 (D)  None of these
(C)  3840 (D)  None of these 68. The largest integer n such that 33! is divisible by 2n is
59. The sum of five digit numbers which can be formed (A) 30 (B) 31
PRACTICE EXERCISES

with the digits 3, 4, 5, 6, 7 using each digit only once (C)  32 (D)  None of these
in each arrangement, is 69. The number of non-negative integral solutions of x1 +
(A) 5666600 (B) 6666600 x2 + x3 + 4x4 = 20 is
(C)  7666600 (D)  None of these (A) 436 (B) 536
60. The sum of all the numbers that can be formed by (C)  602 (D)  None of these
­writing all the digits 3, 2, 3, 4 only once is 70. The product of r consecutive positive integers is
(A) 39996 (B) 49996 divisible by
(C)  57776 (D)  None of these (A)  r! (B)  (r – 1)!
61. The sum of all numbers greater than 10000 formed by (C) (r + 1)! (D)  None of these
using the digits 1, 3, 5, 7, 9, no digit being repeated in 71. The number of ordered triplets of positive integers
any number, is which are solutions of the equation x + y + z = 100 is
(A) 4666600 (B) 5666600 (A) 5081 (B) 6005
(C)  6666600 (D)  None of these (C)  4851 (D)  None of these
62. The sum of all numbers greater than 1000 formed by
72. The number of words that can be formed, with the
using the digits 0, 1, 2, 3, no digit being repeated in
letters of the work ‘Pataliputra’ without changing the
any number, is
relative order of the vowels and consonants, is
Permutations and Combinations  4.23

(A) 3600 (B) 4200 81. The number of integers between 1 and 1000000 that
(C)  3680 (D)  None of these have the sum of the digits 18, is
73. On a new year day every student of a class sends a (A) 25927 (B) 25827
card to every other student. The postman delivers 600 (C)  24927 (D)  None of these
cards. The number of students in the class are 82. The number of non-negative integral solutions to the
(A) 42 (B) 34 system of equations x + y + z + u + t =20 and x + y +
(C)  25 (D)  None of these z = 5 is
74. For any positive integers m, n (with n ≥ m), let (A) 336 (B) 346
(C)  246 (D)  None of these
and( ) = C , then ( ) + ( ) + ( ) + ... + ( ) =
n
m
n
m
n
m
n −1
m
n− 2
m
m
m
83. The number of positive integral solutions of the
(A) ( ) ( )
n+1 n +1 inequality 3x + y + z ≤ 30, is
m (B)  m +1
(A) 1115 (B) 1215
(C) ( ) n
m +1 (D)  None of these (C)  1315 (D)  None of these

84. In a city no person has identical set of teeth and


75. The number of 7 digit numbers the sum of whose there is no person without a tooth. Also, no person
­digits is even, is has more than 32 teeth. If we disregard the shape and
(A) 35 × 105 (B) 45 × 105 size of tooth and consider only the positioning of the
5
(C) 50 × 10 (D)  None of these teeth, then the maximum population of the city is
76. The number of ways of choosing m coupons out of an (A) 232 (B)  232 – 1
32
unlimited number of coupons bearing the letters A, B (C) 2 + 1 (D)  None of these
and C so that they cannot be used to spell the word
BAC, is 85. Eleven scientists are working on a secret project.
They wish to lock up the documents in a cabinet such
(A)  3 (2m – 1) (B)  3 (2m – 1 – 1)
m that cabinet can be opened if six or more scientists are
(C)  3 (2 + 1) (D)  None of these
present. Then, the smallest number of locks needed
77. Six X ’s have to be placed in squares of is
the figure given below, such that each (A) 460 (B) 461
row contains at least one X. The number (C)  462 (D)  None of these
of different ways in which this can be
done is 86. The number of numbers greater than 106 that can be
(A) 26 (B) 28 formed using the digits of the number 2334203, if all

PRACTICE EXERCISES
(C)  18 (D)  None of these the digits of the given number must be used, is
(A) 360
78. The number of ways in which 16 identical things can
(B) 420
be distributed among 4 persons if each person gets at
(C) 260
least 3 things, is
(D)  None of these
(A) 33 (B) 35
(C)  38 (D)  None of these 87. If ‘n’is an integer between 0 and 21, then the mini-
mum value of n! (21 – n)! is
79. The number of ways in which 30 marks can be alloted
to 8 questions if each question carries at least 2 marks, is (A)  9! 2! (B)  10! 11!
(C) 20! (D) 21!
(A) 115280 (B) 117280
(C)  116280 (D)  None of these 88. In how many ways can 20 oranges be given to four
80. In an examination the maximum marks for each of children if each child should get at least one orange?
the three papers are 50 each. Maximum marks for the (A) 869 (B) 969
fourth paper are 100. The number of ways in which (C)  973 (D)  None of these
the candidate can score 60% marks in aggregate is
89. The total number of 5-digit numbers of different dig-
(A) 110256 (B) 110456 its in which the digit in the middle is the largest is
(C)  110556 (D)  None of these
4.24  Chapter 4

9 91. An n-digit number is a positive number with exactly


( )
9
(A) ∑ ∑
n n −1
P4 − P3 (B) n P4 n digits. Nine hundred distinct n-digit numbers are to
n=4 n=4 be formed using only the three digits 2, 5 and 7. The
smallest value of n for which this is possible is
9
(C) ∑ n −1
P3 (D)  None of these (A) 5 (B) 6 (C) 7 (D) 8
n=4
92. If a, b, c are three natural numbers in A.P. such that
a + b + c = 21, then the possible number of values of
90. A train is going from Delhi to Indore, stops at nine a, b, c is
intermediate stations. Six persons enter the train (A) 13 (B) 14 (C) 15 (D) 16
during the journey with six different tickets. The num-
ber of different sets of tickets possessed by them is 93. The number of ways in which a mixed doubles game
can be arranged from amongst n couples such that no
50 54
(A)  C6 (B)  C6 husband and wife play in the same game, is
45
(C)  C6 (D)  None of these n n 1n 1
(A)  P4 (B)  C4 (C)  P4 (D)  nC4
2 2

Previous Year’s Questions

94. A student is to answer 10 out of 13 questions in an 99. At an election, a voter may vote for any number of
examination such that he must choose at least 4 from candidates, not greater than the number to be elected.
the first five questions. The number of choices avail- There are 10 candidates and 4 are of be elected. If a
able to him is [2003] voter votes for at least one candidate, then the number
(A) 140 (B) 196 of ways in which he can vote is [2006]
(C) 280 (D) 346 (A) 5040 (B) 6210
(C) 385 (D) 1110
95. If nCr denotes the number of combinations of n things
taken r at a time, then the expression nCr + 1 + nCr-1+ 100. How many different words can be formed by jum-
2 × nCr equals [2003] bling the letters in the word MISSISSIPPI in which
(A) n+2
Cr (B)  n + 2Cr + 1 no two S are adjacent? [2008]
n+1
(C)  Cr (D)  n + 1Cr + 1 (A) 8 ⋅ 6C4 ⋅ 7C4 (B) 6 ⋅ 7 ⋅ 8C4
(C) 6 ⋅ 8 ⋅ 7C4 (D) 7 ⋅ 6C4 ⋅ 8C4
PRACTICE EXERCISES

96. How many ways are there to arrange the letters in


the word GARDEN with the vowels in alphabetical 101. From 6 different novels and 3 different dictionar-
order? [2004] ies, 4 novels and 1 dictionary are to be selected and
(A) 120 (B) 480 arranged in a row on the shelf so that the dictionary
(C) 360 (D) 240 is always in the middle. Then the number of such
arrangements is [2009]
97. The number of ways of distributing 8 identical balls (A)  less than 500
in 3 distinct boxes so that none of the boxes is empty (B)  at least 500 but less than 750
is [2004]
8
(C)  at least 750 but less than 1000
(A)  5 (B)  C3 (D)  at least 1000
(C) 38 (D) 21
102. There are two urns. Urn I has 3 distinct red balls and
98. If the letters of word SACHIN are arranged in all Urn II has 9 distinct blue balls. From each urn two
possible ways and these words are written out as in balls are taken out at random and then transferred to
dictionary, then the word SACHIN appears at serial the other. The number of ways in which this can be
number [2005] done is [2010]
(A) 601 (B) 600 (A) 36 (B) 66
(C) 603 (D) 602 (C) 108 (D) 3
Permutations and Combinations  4.25

103. Statement-1: The number of ways of distributing 10 (A) 192 (B) 120


identical balls in 4 distinct boxes such that no box is (C) 72 (D) 216
empty is 9C3 [2011]
107. If all the words (with or without meaning) having five
Statement-2: The number of ways of choosing any 3 letters, formed using the letters of the word SMALL
places from 9 different places is 9C3. and arranged as in a dictionary; then the position of
(A)  Statement-1 is true, Statement-2 is true; the word SMALL is: [2016]
th th
Statement-2 is not a correct explanation for (A) 58 (B)  46
Statement-1 (C) 59th (D)  52nd
(B)  Statement-1 is true, Statement-2 is false. 108. A man X has 7 friends, 4 of them are ladies and 3
(C)  Statement-1 is false, Statement-2 is true. are men. His wife Y also has 7 friends, 3 of them are
(D)  Statement-1 is true, Statement-2 is true; ladies and 4 are men. Assume X and Y have no com-
Statement-2 is a correct explanation for mon friends. Then the total number of ways in which
Statement-1 X and Y together can throw a party inviting 3 ladies
104. Assuming the balls to be identical except for differ- and 3 men, so that 3 friends of each of X and Y are in
ence in colors, the number of ways in which one or this party, is [2017]
more balls can be selected from 10 white, 9 green and (A) 468 (B) 469
7 black balls is [2012] (C) 484 (D) 485
(A) 880 (B) 629
109. From 6 different novels and 3 different dictionar-
(C) 630 (D) 879
ies, 4 novels and 1 dictionary are to be selected and
105. Let Tn be the number of all possible triangles formed arranged in a row on a shelf so that the dictionary is
by joining vertices of an n-sided regular polygon. If always in the middle. The number of such arrange-
Tn+1 − Tn = 10, then the value of n is [2013] ments is [2018]
(A) 5 (B) 10 (A)  at least 1000
(C) 8 (D) 7 (B)  less than 500
(C)  at least 500 but less than 750
106. The number of integers greater than 6,000 that can
(D)  at least 750 but less than 1000
be formed, using the digits 3, 5, 6, 7 and 8, without
repetition, is: [2015]

PRACTICE EXERCISES
ANSWER K EYS
Single Option Correct Type
  1. (A) 2.  (D) 3. (A) 4.  (C) 5. (C) 6.  (B) 7. (B) 8.  (A) 9. (B) 10. (C)
11. (B) 12.  (A) 13. (B) 14.  (C) 15. (B) 16. (B) 17. (C) 18. (D) 19.  (A) 20. (B)
21.  (B) 22. (A) 23.  (B) 24. (D) 25.  (B) 26. (A) 27.  (C) 28. (A) 29.  (B) 30. (B)
31.  (D) 32. (B) 33. (B) 34. (A) 35.  (A) 36. (D) 37.  (A) 38. (A) 39.  (C) 40. (C)
41. (A) 42.  (B) 43.  (B) 44. (D) 45.  (B) 46. (A) 47.  (B) 48. (D) 49.  (B) 50. (A)
51.  (A) 52. (A) 53.  (A) 54.  (C) 55. (A) 56.  (B) 57. (A) 58.  (B) 59. (B) 60. (A)
61.  (C) 62. (A) 63. (C) 64. (A) 65. (B) 66. (C) 67.  (C) 68. (B) 69.  (B) 70. (A)
71. (C) 72. (A) 73. (C) 74. (B) 75. (B) 76.  (A) 77. (A) 78. (B) 79. (C) 80. (C)
81. (A) 82. (A) 83.  (B) 84. (B) 85.  (C) 86. (A) 87.  (B) 88. (B) 89.  (A) 90. (C)
91.  (C) 92. (A) 93.  (C)

Previous Years’ Questions


  94. (B) 95. (B) 96. (C) 97. (D) 98. (A) 99. (C) 100. (D) 101. (D) 102. (C) 103. (D)
104. (D) 105. (A) 106. (A) 107. (A) 108. (D) 109. (A)
4.26  Chapter 4

HINTS AND EXPLANATIONS

Single Option Correct Type


1. Number of solutions of the given equation is the same as the The correct option is (C)
number of solutions of the equation 6. Number of 10 balls selections
x1x2x3x4 = 30 = 2 × 3 × 5 = coefficient of x10 in (x2 + x3 + …) (1 + x + x2 +…) (1 + x +
Here, x4 is there because if x1x2x3 = 15, then x4 = 2 and if x2 +…) (x3 + x4 +…)
x1x2x3 = 5, then x4 = 6 etc. = coefficient of x5 in (1 + x + x2 +…)4
x4 is in fact a dummy variable.
= coefficient of x5 in (1 – x)–4
Each of 2, 3 and 5 will be a factor of exactly one of x1, x2, x3, (4 + 5 – 1)
= C5 = 8C5 = 56 ways.
x4 in 4 ways. The correct option is (B)
∴ Required number = 43 = 64
7. Since each element of A can be associated with elements of
The correct option is (A) 8 in two ways, therefore the total number of functions from
2. Now, n = 23 – 2 = 6 A to B is 2 × 2 × 2 × 2 = 16. Out of these functions, the func-
⎛ 2 1⎞ tions which are not onto are f (x) = 1, ∀ x ∈ A and f (x) = 2 ∀
Also, m = 4 + 3 + 2 + 1 – 1 = 9 ⎜ as = ⎟
⎝ 4 2⎠ x ∈ A. Thus, the number of onto functions = 16 – 2 = 14.
∴ m – n = 3
The correct option is (B)
The correct option is (D)
8. We know that in any trinagle the sum of two sides is always
3. A D M N O R (in order)
greater than the third side.
Number of words beginning with
∴ The triangle will not be formed if we select segments of
A _ _ _ _ _ = 5! lengths (2, 3, 5), (2, 3, 6) or (2, 4, 6).
D _ _ _ _ _ = 5! Hence number of triangles formed = 5C3 – 3.
M _ _ _ _ _ = 5! The correct option is (A)
N _ _ _ _ _ = 5!
9. a = x+2Px+2, b = xP11, C = x–11Px–11
O _ _ _ _ _ = 5! x!
R A D _ _ _ = 3! ⇒ a = (x + 2)!, b = , c = (x – 11)!
( x − 11)!
R A M _ _ _ = 3! Now, a = 182 bc
HINTS AND EXPLANATIONS

x!
R A N D M O = 1 ⇒ (x + 2)! = 182 (x – 11)!
( x − 11)!
R A N D O M = 1
⇒ (x + 2) (x + 1)x! = 182x!
∴ Rank of word RANDOM = 614 ⇒ (x + 2) (x + 1) = 182 = 14 × 13
The correct option is (A) ⇒ x + 1 = 13
∴ x = 12
4. Let A, B be the corresponding speakers. Without any restric-
The correct option is (B)
tion the eight persons can be arranged among themselves in
8! ways, but the number of ways in which A speaks before 10. The four digits 3, 3, 5, 5, can be arranged at four even places
B and the number of ways in which B speaks before A make 4!
in = 6 ways and the remaining digits viz., 2, 2, 8, 8, 8
up 8!. Also number of ways in which A speaks before B 2! 2! 5!
is exactly same as the number of ways in which B speaks can be arranged at the five odd places in = 10 ways.
2! 3!
before A. Thus, the number of possible arrangements is (6) (10) = 60.
1
∴ the required number of ways = (8!) = 20160. The correct option is (C)
2
The correct option is (C) 11. For one game four persons are required. This can be done
5. Total number of permutations = 7! in 6C4 = 15 ways. Once a set of four persons are selected,
4
Let A be the property that ‘beg’ occurs. C2
number of games possible will be = 3 games.
B be the property that cad occurs. 2
Number of permutations with A = 5! ∴ Total number of possible games = 3 × 15 = 45.
= that of with B and n (A ∩ B) = 3! The correct option is (B)
∴ n(A ∪ B) = 5! + 5! – 3! = 234 12. We know that a number is divisible by 3 if the sum of its
∴ Required number = 7! – 234 = 4806 digits is divisible by 3.
Permutations and Combinations  4.27


Now the sum of the digits 1, 2, 3, 4 and 5 is 15, which is 17. No. of triangles = 2nC3 – nC3 – nC3
divisible by 3. 2n ( 2n − 1) ( 2n − 2) 2n ( n − 1) ( n − 2)
∴ All the five digit numbers formed by the digits 1, 2, 3, 4,
=

6 6
5 are divisible by 3 and their number = 5! = 120. 1
= n (n – 1) (3n) = n2 (n – 1)

When we include 0, the four other digits whose sum is divis- 3
ible by 3 are 1, 2, 4 and 5. The correct option is (C)
∴ The number of numbers in this case
18. E H M O R T (alphabetical order)
= 4 × 4! = 4 × 24 = 96. Number of words beginning with

Hence the required number of numbers E _ _ _ _ _ = 5!
= 120 + 96 = 216 H _ _ _ _ _ = 5!

The correct option is (A) M E _ _ _ _ = 4!
13. The number of ways of selecting 3 balls out of total 9 M H _ _ _ _ = 4!
(2 white, 3 black, 4 red balls) is 9C3 M O E _ _ _ = 3!
9×8×7 M O H _ _ _ = 3!
i.e., = 84
6 M O R _ _ _ = 3!
The number of ways of selecting 3 balls out of non-black six M O T E _ _ = 2!
balls is 6C3
M O T H E R = 1!
6×8×4
i.e., = 20
3 × 2 ×1
∴ Rank of word MOTHER = 309
Therefore, the number of ways of selecting 3 balls out of
9 balls so as to include atleast one black ball = 84 – 20 = 64. The correct option is (D)
The correct option is (B) 19. Factorizing the given number, we have
38808 = 23 ⋅ 32 ⋅ 72 ⋅ 11
14. The total number of numbers that can be formed with the
digits 2, 3, 4, 5 taken all at a time = 4P4 = 4! = 24. Consider Therefore the total number of divisors
the digit in the unit’s place in all these numbers. Each of the = (3 +1) (2 + 1) (1 + 1) – 1 = 71.
digits 2, 3, 4, 5 occurs in 3! = 6 times in the unit’s place But this includes the division by the number itself.
∴ total for the digits in the unit’s place Hence, the required number of divisors
= (2 + 3 + 4 + 5) 6 = 84 = 71 – 1 = 70
Since each of the digits 2, 3, 4, 5 occurs 6 times in any one

HINTS AND EXPLANATIONS


of the remaining places QUICK TIPS
∴ the required total
Let N = p1a ⋅ p2a ⋅ p3a … pka where p1, p2, p3 …pk are dif-
1 2 3 k

= 84 (1 + 10 + 102 + 103) = 84 (1111) = 93324.


ferent primes and a1, a2, …, ak are natural numbers then
The correct option is (C) total ­number of divisors of N excluding 1 and N is (a1 + 1)
15. Total number of ways = nn (a2 + 1) … (ak + 1) – 2

Number of ways so that each person gets at least one thing
(here exactly one thing) = n! The correct option is (A)
Given, nn – n! = 232 20. We have, 15 < x1 + x2 + x3 ≤ 20
11 – 1! = 0, 22 – 2! = 2, 33 – 3! = 21, 44– 4! = 232 ⇒ x1 + x2 + x3 = 16 + r, r = 0, 1, 2, 3, 4.
∴ n = 4 Now, number of positive integral solutions of x1 + x2 + x3 =
16 + r is 16 + r – 1C3 – 1 = 15 + r C2
The correct option is (B)
n Thus, total number of solutions
16. mCr + 1 + ∑ k
Cr

4
= ∑ 15+ r C2 15
C2 + 16C2 + 17C2 + 18C2 + 19C2
k=m
m m m+1 n–1 n r=0
= Cr + 1 + Cr +
Cr + … + Cr + Cr 20 15
m+1 m+1 n–1 C3 – C3 = 685
=
Cr + 1 + Cr + … + Cr + nCr
m+2
=
Cr + 1 + m + 2Cr + … + n – 1Cr + nCr QUICK TIPS
   
= nCr + 1 + nCr = n + 1Cr + 1
The total number of positive integral solutions of the equation
x1 + x2 + … + xr = n is n – 1Cr – 1

The correct option is (B)

The correct option is (B)
4.28  Chapter 4

21. Other than 2, remaining five places are to be filled by by = (2n–1Cn + 2n–1Cn–1) – nCn

1 and 3 2n
=
Cn – nCn
∴ number of ways for five places
= 2 × 2 × 2 × 2 × 2 = 25 ∴
W = 2nCn – 1

For 2, selecting 2 places out of 7 = 7C2
The correct option is (A)

∴ Required no. of ways = 7C2 ⋅ 25. 27. 6 large animals can be caged in 7 large cages in 7P6 = 7!
The correct option is (B) ways. 5 small animals can be caged in remaining 5 cages
(4 small + 1 large) in 5! ways. Hence, the number of ways is
22. We know that 1! + 2! + 3! + 4! = 33.
7! × 5! = 5040 × 120 = 604800
Also, 5! = 120, 6! = 720, 7! = 5040, 8! = 40320 and
The correct option is (C)
9! = 362880.
Thus, tens digit of 1! + 2! + … + 9! is 1. 28. By the given condition, it is clear that nCr is the greatest
Also, note that n! is divisible by 100 for all n ≥ 10. Therefore, among nC0, nC1, …, nCn.
the tens digit of 1! + 2! + … + 49! is 1. Since n is even,
n
The correct option is (A) ∴ nCr is the greatest for r =
n 2
23. Each element of the set A can be given the image in the set A ∴ r =
2
in k ways.
The correct option is (A)
∴ the required number of functions, i.e.,
29. For each pair of stations, two different types of tickets are
n (S) = k × k × … (k times) = kk. required, Now, the number of selections of 2 stations from
The correct option is (B) 15 stations = 15C2.
24. pC × pC × pC + pC × 2 pC × 3C ∴ Required number of types of tickets
1 1 1 2 1 1
15!
2
= p (4p – 3) = 2 15C2 = 2 = 15 × 14 = 210
2!13!
The correct option is (D) The correct option is (B)
37!
25. Number of ways = 37C2 = = 666 ways 30. All the numbers of the form 6n will end with 6 and 9m will end
35! × 2!
For the distribution equation. with 9, if m is odd and will end with 1, if m is even. Therefore,
6n + 9m will end with 5 if n is any number and m is odd.
x1 + x2 + x3 + … + xr = n
Thus, number of ordered pairs = 50 × 25 = 1250.
The number of ways in which n things can be distributed
HINTS AND EXPLANATIONS

among r in such a way each can receive none, one or more or The correct option is (B)
all of n items are n+r–1Cr–1 31. Number of subsets
The correct option is (B) = 2n+1C0 + 2n+1C1 + … + 2n+1Cn = N (say)

26. In general, we know that ∴ N = 22n+1 – N

For the distribution equation ⇒ 2N = 22n+1

x1 + x2 + x3 + … + xn ≤ n
⇒ N = 22n
Let required ways = W
The correct option is (D)
⎧ No. of ways of ⎫ ⎧ No. of ways of ⎫ 32. The number of triangles = number of selections of 2 lines
⎪ ⎪
⇒ W = ⎪⎨ distributing ⎪⎬ + ⎨ distributing ⎬
from the (n – 1) lines which are cut by the last line
⎪ 1 item ⎪ ⎪ 2 items ⎪ ( n − 1)! ( n − 1) ( n − 2)
⎩ ⎭ ⎩ ⎭ = n – 1C2 = =
2!( n − 3)! 2
⎧ No. of Ways of ⎫
⎪ ⎪ The correct option is (B)
+ … + ⎨ distributing ⎬

⎪ ⎪ 33. Distinct n digit numbers which can be formed using digits 2,
⎩ n items ⎭ 5 and 7 are 3n. We have to find n so that
= 1+n–1Cn–1 + 2+n–1Cn–1 + … + n+n–1Cn–1
3n ≥ 900  ⇒ 3n–2 ≥ 100
=

n
Cn–1 + n+1Cn–1 + … + 2n–1Cn – 1 ⇒ n – 2 ≥ 5  or  n ≥ 7
The correct option is (B)
= (nCn–1
+ nCn) + n+1Cn–1 + … + 2n–1Cn–1 – nCn
So, the least value of n is 7.
+ n+1Cn–1) + … + 2n–1Cn–1} – nCn
= {(n+1Cn 34. Starting with the letter A and arranging the other four letters,
----------------------------------
there are 4! = 24 words. These are the first 24 words. Then,

---------------------------------- starting with G and arranging A, A, I and N in different ways,
Permutations and Combinations  4.29

4! r 67
there are = 12 words, the 37th word start with I and ⇒ –1 ≤ –
≤−
2! 1! 1! 100 100
there are 12 words starting with I. Total goes upto 80 words.
1 1 r 1 67
Now, 49th word is NAAGI. ⇒ − − 1 ≤ − −
≤− −
3 3 100 3 100

The correct option is (A)
⎛ 1 r ⎞
35. If we choose k (0 ≤ k ≤ n) identical objects, then we must ∴ ⎜ − −
= –2 for 67 ≤ r ≤ 100
⎝ 3 100 ⎟⎠
choose (n – k) distinct objects. This can be done in 2n+1Cn–k
ways. Thus, the required number of ways 100
⎛ 1 r ⎞
n

Hence, ∑ ⎜⎝ − 3 − 100 ⎟⎠ = 67 (– 1) + 2(– 34) = – 135
∑ 2 n +1 2n+1 2n+1 2n+1 r=0
=
Cn − k = Cn + Cn–1 + … + C0
k=0
The correct option is (C)
2n
=2
41. The following four cases are there:

The correct option is (A)
(A) At least one one rupee coin and any number of other
36. 6 boys can sit in 6! ways coins = 2 × 4 × 5 = 40 ways.
× B1 × B2 × B3 × B4 × B5 × B6 × (B) At least two 50 p. coins and any number of 25 p. coins
Now, two brothers can sit in any of the 7 cross marked (×) = 2 × 5 = 10 ways.
places (C) One 50 p. coin and at least two 25 p. coins = 1 × 3 =
Therefore, required number of ways = 6! × 7C2 × 2! 3 ways.
= 30240 (D) Four 25 p. coins in one way only ∴ total = 54.
The correct option is (D) The correct option is (A)
37. The number of subsets of A containing exactly three ele- 42. The number of ways in which at most n books can be selected
ments is nC3 whereas the number of three subsets of A that out of a collection of (2n + 1) books is
contain a1 is n–1C2. We are given, 2n + 1
C1 + 2n + 1C2 + … 2n + 1Cn
n–1 20 n
C2 = ( C3) 1 2n + 1
100 ⇒
[( C1 + 2n + 1C1) + (2n + 1C2 + 2n + 1C2) + …
2
( n − 1) ( n −2) 1 n ( n − 1) ( n −2)

= … + (2n + 1Cn + 2n + 1Cn)] = 63

2 5 6
⇒ n = 15
⇒ (2n + 1C1 + 2n + 1C2n) + (2n + 1C2 + 2n + 1C2n – 1) + …

HINTS AND EXPLANATIONS




The correct option is (A)
… + (2n + 1Cn + 2n + 1Cn + 1) = 126

38. When zero is in the place of y then there are nine numbers
2n + 1
and when 1 is in the place of y then there are 8 numbers and ⇒
C1 + 2n + 1C2 + … + 2n + 1C2n = 126
9
⇒ 22n + 1 – 2 = 126

so on. Hence, required number is ∑ n = 45.

The correct option is (A) n =1 ⇒ 22n + 1 = 128 = 27


⇒ 2n + 1 = 7
39. The required number is equal to the number of all the subsets ⇒ n = 3

of a 68 elements set i.e.,

The correct option is (B)
68C + 68C + 68C + … + 68C = 268 = 1617
0 1 2 68
43. The number of students answering at least r questions incor-

The correct option is (C)
rectly is 2n–r.
r 2 ∴ The number of students answering exactly r (1 ≤ r ≤ n – 1)
40. For 0 ≤ r ≤ 66, 0 ≤ <
100 3 questions incorrectly is 2n–r – 2n–(r + 1).
2 r Also, the number of students answering all questions
⇒ −
<− ≤0 wrongly is 20 = 1.
3 100
1 2 1 r 1 Thus, the total number of wrong answers is
⇒ − − < − −
≤− 1 (2n–1 – 2n–2) + 2 (2n–2 – 2n–3) + 3 (2n–3 – 2n–4) + …

3 3 3 100 3
+ (n – 1) (21 – 20) + n (20)

⎛ 1 r ⎞
∴ ⎜ − −
= – 1 for 0 ≤ r ≤ 66 = 2n–1 + 2n–2 + … + 20 = 2n –1
⎝ 3 100 ⎟⎠
67 r Now, 2n – 1 = 4095 ⇒ 2n = 4096 = 212 ⇒ n = 12

Also, for 67 ≤ r ≤ 100,
≤ ≤1
The correct option is (B)
100 100
4.30  Chapter 4

44. d = 10 + 20 + (60 – 20 + 5) + (60 – 20 + 5 – 1)



c = 10 + 20 + 45 + 44 = 119.

a ⎯⎯
→ 2 ways.
QUICK TIPS
d
a If r (0 ≤ r ≤ n) objects occurpy the places assigned to them
and none of the remaining objects occupies its original places,
b a d c ⎯⎯
→ 3 ways.
then the number of such ways is
d a c
⎡ 1 1 1 1 ⎤
= nCr · (n – r)! ⎢1 − + − + … + ( −1)n − r
(n − r )! ⎥⎦
b c a
⎣ 1! 2 ! 3 !
a
c ⎯⎯
→ 3 ways. The correct option is (B)
48. (10 + 1)(9 + 1)(7 + 1) – 1 = 11.10.8 – 1 = 879
d ⎯⎯
→ 3 ways. The correct option is (D)
49. Number of ways to fill thousand’s place = 9
∴ Total number of ways = 11.
Number of ways to fill hundred’s place = 9

The correct option is (D) Number of ways to fill ten’s place = 8
m Number of ways to fill unit place = 7
45. S = ∑ n + r Ck ∴ 9 × 9 × 8 × 7 = 4536
r=0
The correct option is (B)
= Ck + n+1Ck + n+2Ck + … + n+mCk
n
50. x1 + x2 + x3 + x4 + x5 = 6
n
= Ck +1 + nCk + n+1Ck + n+2Ck + … + n+mCk – nCk+1 5 + 6 – 1
C5 – 1 = 10C4
n+1 n+1 n+2 n+m n
=
Ck+1 + Ck + Ck + … + Ck – Ck+1 The correct option is (A)
n+m+1 n

S= Ck+1 – Ck+1 51. Let the number of papers be n.

The correct option is (B) ∴ Total number of ways to fail or pass
n
C0 + nC1 + nC2 + … + nCn = 2n.
46. The number of ways of choosing first couple is (15C1)
(15C1) = 152. The number of ways of choosing 2nd couple is But there is only one way to pass, i.e., when he fails in none.
HINTS AND EXPLANATIONS

(14C1) (14C1) = 142, and so on. Thus, the number of ways of ∴ Total number of ways to fail = 2n – 1.
choosing the couples is ∴ From question, 2n – 1 = 63;
152 + 142 + 132 + … + 22 + 12 ∴ 2n = 64 = 26

15 × (15 + 1) [2 (15) + 1] ∴ n = 6.
= = 1240
6 The correct option is (A)

52. ∑ ∑ 10
C j jCi
QUICK TIPS 0 ≤ i ≤ j ≤ 10

= 10C0 × 0C0 + 10C1 (1C0 + 1C1) + 10C2 (2C0 + 2C1 + 2C2)



Number of ways of dividing n identical things into r groups, if
blank groups are allowed is n + r – 1Cr – 1. + 10C3 (3C0 + 3C1 + 3C2 + 3C3) + …

+ 10C10(10C0 + 10C1 + … + 10C10)

The correct option is (A)
= 10C0 · 20 + 10C1 · 21 + 10C2 · 22 + … + 10C10 · 210

47. Required number of ways

⎡ 1 1 1 = (1 + 2)10 = 310.
5
( −1) r ⎤
= ∑ C5 − r r ! ⎢1 − + − + … +
5
⎥ The correct option is (A)
r=2 ⎢⎣ 1! 2! 3! r ! ⎥⎦
53. If we choose k (0 ≤ k ≤ n) identical objects, then we must
5
5! ⎡ 1 1 ( −1) r ⎤ choose (n – k) distinct objects. This can be done in 2n+1Cn–k
= ∑
⎢ − +… + ⎥
r = 2 (5 − r )! ⎢
⎣ 2! 3! r ! ⎥⎦ ways. Thus, the required number of ways
n

=

5! ⎛ 1 ⎞ 5! ⎛ 1 1 ⎞ 5! ⎛ 1 1 1 ⎞
⎜ ⎟+ ⎜ − ⎟+ ⎜ − + ⎟
=
∑ 2n+1Cn − k = 2n+1Cn + 2n+1Cn–1 + … + 2n+1C0
3! ⎝ 2!⎠ 2! ⎝ 2! 3!⎠ 1! ⎝ 2! 3! 4!⎠ k=0

= 22n.

5! ⎛ 1 1 1 1 ⎞
+ ⎜ − + − ⎟
The correct option is (A)
0! ⎝ 2! 3! 4! 5!⎠
Permutations and Combinations  4.31

2n The number of numbers of three digits divisible by 5 = num-



Cr 2n − r + 1
54. We have, 2n
= (1) ber of all the numbers of three digits divisible by 5 = number
Cr −1 r of numbers of three digits divisible by 5 and having 0 in
2n − r + 1 hundred’s place
For 2nCr to be greatest
≥1
r = 2P1 × 9P2 – 8P1 × 1 = 2 × 9 × 8 – 8 = 136.

⇒ 2n – r + 1 ≥ r ⇒ 2r ≤ 2n + 1
∴ Required number of numbers

1 = 1 + 17 + 136 = 154.

⇒ r ≤ n + (2)

2
The correct option is (B)
2n
Cr +1 2n − ( r + 1) + 1 2n − r 57. Suppose, the first two digits are 41, remaining four digits are

From (1), = = .
2n
Cr r +1 r +1 to be chosen from 0, 2, 3, 5, 6, 7, 8, 9 so as to make all the
2n − r digits distinct.
For 2nCr to be greatest
≤1 ∴ Remaining four can be chosen in P (8, 4) ways.
r +1
We have 8 letters and they are to be arranged at 4 places.
⇒ 2n – r ≤ r + 1 ⇒ 2r ≥ 2n – 1

1
First place can be arranged in 8 different ways.
⇒ r ≥ n − 
(3)
Second place can be arranged in 7 different ways.
2
1 1
Third place can be arranged in 6 different ways.

From (2) and (3), we get n – ≤r≤n+
2 2
Fourth place can be arranged in 5 different ways.
⇒ r = n (since r is a positive integer)

Hence, 2nCr is greatest when r = n.

Number of such telephone numbers

The correct option is (C) = 8 × 7 × 6 × 5 = 1680.


Now, considering the all given first two digits (41 or 42 or 46
55. The numbers are of three or four digits. or 64), total number of the telephone numbers
To find the number of even numbers of three digits— = 1680 × 5 = 8400.

The unit’s place must be filled by 0 or 2.

The correct option is (A)
∴ The number of even numbers of three digits (having or not
having 0 in hundred’s place) = 2P1 × 3P2. 58. There are 11 letters in the word INDEPENDENT.
But the number of even numbers of three digits having 0 in 3N, 3E, 2D, I, P, T (6 types)
hundred’s place = 2P1 (∵ unit’s place is naturally filled by 2 Different possibilities of choosing 5 letters are
and ten’s place by one of 1 and 3). (A) All different

HINTS AND EXPLANATIONS


∴ The number of even numbers of three digits Number of ways = 6C5 × 5! = 6 × 120 = 720.
= 2P1 × 3P2 – 2P1.(1) (B) 2 alike, 3 different
Similarly, the number of even numbers of four digits 5!
Number of ways = 3C1 × 5C3 × = 3 × 10 × 60 = 1800
= 2P1 × 3P3 – 2P2.(2) 2!
(C) 3 alike, 2 different
Adding (1) and (2), we get the number of even numbers 5!
greater than 100 Number of ways = 2C1 × 5C2 ×
3!
= (2P1 × 3P2 – 2P1) + (2P1 × 3P3 – 2P2) = 2 × 10 × 20 = 400
= 2 × 3 × 2 – 2 + 2 × 3 × 2 × 1 – 2 × 1 = 20. (D) 2 alike, 2 alike, 1 different
The correct option is (A) 5!
= 3C2 × 4C1 × = 3 × 4 × 30 = 360
2! 2!
56. Here, the available digits are 0, 1, 2, 3, 4, 5, 6, 7, 8, 9. 5!
(E) 3 alike, 2 alike = 2C1 × 2C1 ×
The numbers can be of one, two or three digits and in each of 3! 2!
them unit’s place must have 0 or 5 as they must be divisible = 2 × 2 × 10 = 40
by 5. Hence, total number of ways
The number of numbers of one digit = 1 = 720 + 1800 + 400 + 360 + 40 = 3320.
( 5 is the only number). The correct option is (B)
The number of numbers of two digits divisible by 5 = num- 59. The number of numbers of five digits, having 3 in unit’s
ber of all the numbers of two digits divisible by 5 – number place = 4P4 (number of arrangements of the remaining four
of numbers of two digits divisible by 5 and having 0 in ten’s digits)
place = 2P1 × 9P1 – 1, = 4! = 24.
( unit’s place can be filled by either 0 or 5 in first category Similar is the case for other digits, i.e., each digit occurs 24
and only by 5 in the second category) times in the unit’s place.
= 2 × 9 – 1 = 17.
4.32  Chapter 4

Hence, the sum of the digits in unit’s place (


thousand’s place can be filled by one of 2, 3 and the
= 24 × 3 + 24 × 4 + 24 × 5 + 24 × 6 + 24 × 7 remaining two places can be filled by the remaining two
= 24 (3 + 4 + 5 + 6 + 7) = 24 × 25 = 600 digits).
Similarly, the sum of the digits in each of the other places = Similarly, the number of numbers having 2 or 3 in unit’s
600. place
∴ The required sum = 2P1 × 2P2 in each case.
= 600 × 10000 + 600 × 1000 + 600 × 100 + 600 × 10 + Thus, the sum of the digits in unit’s place for all the numbers
600 × 1 = 3! × 0 + 2P1 × 2P2 × 1 + 2P1 × 2P2 × 2 + 2P1 × 2P2 × 3
= 600 (10000 + 1000 + 100 + 10 + 1) = 4 + 8 + 12 = 24.
= 600 × 11111 = 6666600. Similarly, the sum of the digits in ten’s and hundred’s places
The correct option is (B) = 24 each.
3! Now, the thousand’s place can have only 1 or 2 or 3.
60. The number of numbers having 2 in unit’s place = Number of numbers having 1 in thousand’s place = 3!, (for
2!
(because the other three places are to be filled by 3, 3, 4 of the other three places will be filled by 0, 2, 3).
which two are identical). Similarly, the number of numbers having 2 or 3 in ­thousands’s
The number of numbers having 3 in unit’s place = 3!, place is 3! in each case.
(because the other three places are to be filled by 2, 3, 4). ∴ Sum of the digits in the thousands’s place for all the
3! numbers = 3! × 1 + 3! × 2 + 3! × 3 = 6 + 12 + 18 = 36.
The number of numbers having 4 in unit’s place = , ∴ Required sum of all the numbers
2!
(because the other three places are to be filled by 3, 2, 3 of = 36 × 1000 + 24 × 100 + 24 × 10 + 24 × 1
which two are identical) = 36000 + 2400 + 240 + 24 = 38664.
∴ Sum of the digits in unit’s place The correct option is (A)
3! 3! 63. The thousand’s place cannot be filled by O. So the number of
= × 2 + 3! × 3 + × 4 = 6 + 18 + 12 = 36.
2! 2! ways to fill the thousands’s place = 5.
Similarly, the sum of the digits in other places is 36 each. The remaining three places can be filled by six digits in 63
∴ Required sum = 36 × 1000 + 36 × 100 + 36 × 10 + 36 × 1 ways, as digits can be repeated.
= 36 (1000 + 100 + 10 +1) = 36 × 1111 = 39996. ∴ The number of four digit numbers
The correct option is (A) = 5 × 63 = 1080. (1)
Now, the number of numbers of four digits that do not con-
HINTS AND EXPLANATIONS

61. All the numbers are of five digits. tain any repeated digit = 5P1 × 5P3 { thousand’s place is to
The number of numbers having 1 in unit’s place = 4! = 24 be filled by one of 1, 2, 3, 4, 5 and the remaining three places
( the other four places are to be filled by 3, 5, 7, 9). are to be filled by three of the remaining five digits including
Similarly, the number of numbers having 3, 5, 7, or 9 in 0}. But (1) contains numbers which contain no repeated dig-
unit’s place is 24 in each case. its as well as those which contain at least one repeated digit.
∴ The sum of the digits in unit’s place for all the numbers ∴ The number of four digit numbers which contain at least
= 24 × 1 + 24 × 3 + 24 × 5 + 24 × 7 + 24 × 9 one repeated digit = 1080 – 5P1 × 5P3
= 24 (1 + 3 + 5 + 7 + 9) = 24 × 25 = 600. = 1080 – 5 × 5 × 4 × 3 = 780.
Similarly, sum of the digits in other places = 600 in every The correct option is (C)
case. 64. The numbers are of five digits having 4 or 5 in ten thousand’s
∴ The required sum of all the numbers that can be made place and 1 or 5 in unit’s place and the remaining digits are
any of the given six digits.
= 600 × 10000 + 600 × 1000 + 600 × 100 + 600 × 10 +
600 × 1 ∴ Number of ways to fill ten thousand’s place = 2P1 = 2
= 600 (10000 + 1000 + 100 + 10 +1) = 600 × 11111 Number of ways to fill unit’s place = 2P1 = 2
= 6666600. Number of ways to fill other three places = 63.
The correct option is (C) ∴ Required number of numbers = 2 × 2 × 63 = 864.
62. All the numbers are of four digits and they do not have 0 in The correct option is (A)
thousand’s place. 65. When X and Y sit on the side facing the window—
The number of numbers having 0 in unit’s place = 3! First, we have to select 2 persons for the side facing the
( the other three places are to be filled by 1, 2 or 3). ­window and 3 for the opposite side from the remaining 5.
The number of selections for this is equal to 5C2 × 3C3.
The number of numbers having 1 in unit’s place = 2P1 × 2P2
For each of the selections, the number of arrangements
Permutations and Combinations  4.33

= 4! × 3!.
69. Number of non-negative integral solutions of the given
∴ The number of ways to sit when X, Y are on the side
equation
facing the window = 5C2 × 3C3 × 4! × 3!. = coefficient of x20 in

When X and Y sit on the other side (1 – x)–1 (1 – x)–1 (1 – x)–1 (1 – x4)– 1.

As before, the number of ways to sit = coefficient of x20 in (1 – x)–3 (1 – x4)–1

= 5C1 × 4C4 × 4! × 3!
= coefficient of x20 in (1 + 3C1x + 4C2 x2 + 5C3 x3 + 6C4 x4+ …
∴ Required number of ways
+ 10C8x8 … + 14C12 x12 + …+ 18C16 x16 + … + 22C20 x20 + …)
5 3 5 4

= C2 × C3 × 4! × 3! + C1 × C4 × 4! × 3!
× (1 + x4 + x8 + x12 + x16 + x20 + …)
5×4 = 1 + 6C4 + 10C8 + 14C12 + 18C16 + 22C20

= × 1 × 24 × 6 + 5 × 1 × 24 × 6 = 2160.
2
= 1+ 6C2 + 10C2 + 14C2 + 18C2 + 22C2

The correct option is (B)
⎛ 6.5 ⎞ ⎛ 10.9 ⎞ ⎛ 14.13 ⎞ ⎛ 18.17 ⎞ ⎛ 22.21⎞
= 1+ ⎜ ⎟ +⎜ + + +
66. In selection of fruits, fruits of the same type are treated as
identical. ⎝ 1.2 ⎠ ⎝ 1.2 ⎟⎠ ⎜⎝ 1.2 ⎟⎠ ⎜⎝ 1.2 ⎟⎠ ⎜⎝ 1.2 ⎟⎠
Number of selections of any number of oranges from = 1 + 15 + 45 + 91 + 153 + 231 = 536.
4 oranges = 4 + 1. The correct option is (B)
Number of selections of any number of apples from 5 apples
= 5 + 1. 70. Let r consecutive positive integers be
Number of selections of any number of mangoes from m, m + 1, m + 2, … (m + r – 1), where m ∈ N
6 mangoes = 6 + 1. ∴ Product = m (m + 1) (m + 2) … (m + r – 1)
∴ Number of selections of any number of fruits from the ( m − 1)! m ( m + 1) ( m + 2)…( m + r − 1)
=
basket = (4 + 1) (5 + 1) (6 + 1) = 5 × 6 × 7 = 210. ( m − 1)!
But in one of these selections, there is one which contains ( m + r − 1)! ( m + r − 1)!
no fruits. Excluding this selection, the required number of = = r !.
( m − 1)! r !( m − 1)!
selections = 210 – 1 = 209.
The correct option is (C) = r!. m + r – 1Cr
m+r–1
67. In terms of prime factors 100! can be written as 2a ⋅ 3b ⋅ 5c ⋅ 7d which is divisible by r!( Cr is a natural number)
… The correct option is (A)
Now,  E2 (100!) 71. The number of triplets of positive integers which are solu-

HINTS AND EXPLANATIONS


⎡100 ⎤ ⎡100 ⎤ ⎡100 ⎤ ⎡100 ⎤ ⎡100 ⎤ ⎡100 ⎤ tions of x + y + z = 100
= ⎢
⎥+⎢ 2 ⎥+⎢ 3 ⎥+⎢ 4 ⎥+⎢ 5 ⎥+⎢ 6 ⎥
⎣ 2 ⎦ ⎣2 ⎦ ⎣2 ⎦ ⎣2 ⎦ ⎣2 ⎦ ⎣2 ⎦ = Coefficient of x100 in (x + x2 + x3 + …)3

= 50 + 25 + 12 + 6 + 3 + 1 = 97
= Coefficient of x100 in x3 (1 – x)–3

⎡100 ⎤ ⎡100 ⎤ = Coefficient of x100 in
and,  E5 (100!) = ⎢
⎥ + ⎢ 2 ⎥ = 20 + 4 = 24
⎣ 5 ⎦ ⎣5 ⎦ ⎡ ( n + 1) ( n + 2) n ⎤
x 3 ⎢1 + 3 x + 6 x 2 + … + x + …⎥
100! = 297 ⋅ 3b ⋅ 524 ⋅ 7d …
⎣ 2 ⎦

= 273 ⋅ 3b ⋅ (2 × 5)24 ⋅ 7d …
(97 + 1) (97 + 2)

= 273 ⋅ 3b ⋅ (10)24 ⋅ 7d … = = 49 × 99 = 4851.
2

Hence, number of zeros at the end of 100! is 24. The correct option is (C)

The correct option is (C)
72. There are eleven letters in the word ‘Pataliputra’ and there
  68. In terms of prime factors 33! can be written as 2a ⋅ 3b5c ⋅ 7d are two p’s, two r’s, three a’s and four other different letters.

Number of consonants = 6, number of vowels = 5.
Now, E2 (33!)
Since relative order of the vowels and consonants remains
⎡ 33 ⎤ ⎡ 33 ⎤ ⎡ 33 ⎤ ⎡ 33 ⎤ ⎡ 33 ⎤ unchanged, therefore, vowels will occupy only vowel’s
= ⎢ ⎥ + ⎢ 2 ⎥ + ⎢ 3 ⎥ + ⎢ 4 ⎥ + ⎢ 5 ⎥
⎣ 2 ⎦ ⎣2 ⎦ ⎣2 ⎦ ⎣2 ⎦ ⎣2 ⎦ place and consonants will occupy only consonants’s place.
= 16 + 8 + 4 + 2 + 1 = 31. Now, 6 consonants can be arranged among themselves in
6!
Hence, the exponent of 2 in 33! is 31. ways [since there are two p’s and two t’s] and five
2! 2!
∴ Largest value of n is 31. 5!
vowels can be arranged among themselves in ways,
The correct option is (B) since a occurs thrice. 3!
4.34  Chapter 4

6! 5! 76. The word BAC cannot be spelt if the m selected coupons do


∴ Required number = . = 3600. not contain atleast one of A, B and C.
2! 2! 3!
The correct option is (A) Number of ways of selecting m coupons which are A or
B = 2m.
73. Let n be the number of students. This also includes the case when all the m coupons are A or
Now, number of ways in which two students can be selected all are B.
out of n students is nC2. Number of ways of selecting m coupons which are B or
∴ number of pairs of students = nC2. C = 2m.
But for each pair of students, number of cards sent is 2 This also includes the case when all the m coupons are B or
(since if there are two students A and B, A will send a card all are C.
to B and B will send a card to A). Number of ways of selecting m coupons which are C or
∴ For nC2 pairs, number of cards sent = 2 ⋅ nC2. A = 2m.
According to the question, 2 ⋅ nC2 = 600 This also includes the case when all the m coupons are C or
n( n − 1) all are A.
or, 2 ⋅ = 600
2! Number of ways of selecting m coupons when all are
or, n2 – n – 600 = 0 A = 1m.
or, (n – 25) (n + 24) = 0 Number of ways of selecting m coupons when all are
∴ n = 25, –24 B = 1m.
But n ≠ –24 Number of ways of selecting m coupons when all are
∴ n = 25. C = 1m.
The correct option is (C) ∴ Required number = 2m + 2m + 2m – (1m + 1m + 1m)
= 3 ⋅ 2m – 3 ⋅ 1m = 3 (2m – 1).
⎛ m⎞ ⎛ m + 1⎞ ⎛ m + 2⎞ ⎛ n⎞
74. + + +… + ⎜ ⎟ The correct option is (A)
⎝⎜ m⎠⎟ ⎜⎝ m⎠⎟ ⎜⎝ m⎠⎟ ⎝ m⎠
77. First Method: Number of cross marks in
⎛ m + 1⎞ ⎛ m + 1⎞ ⎛ m + 2⎞ ⎛ n⎞ 1st row 2nd row 3rd row Number of ways
= ⎜ ⎟ +⎜ ⎟ +⎜ ⎟ +… + ⎜ ⎟
⎝ m + 1⎠ ⎝ m⎠ ⎝ m⎠ ⎝ m⎠ (Number of selections of squares)
⎛ m + 2⎞ ⎛ m + 2⎞ ⎛ n⎞ 1 3 2 2C1 × 4C3 × 2C2 = 8
= ⎜ + +…+ ⎜ ⎟
⎝ m + 1⎟⎠ ⎜⎝ m⎟⎠ ⎝ m⎠ 1 4 1 2C1 × 4C4 × 2C1 = 4
2 2 2 2C2 × 4C2 × 2C2 = 6
HINTS AND EXPLANATIONS

⎛ m + 3⎞ ⎛ m + 3⎞ ⎛ n⎞
= ⎜ ⎟ +⎜ ⎟ + ... + ⎜ ⎟ 2 3 1 2C2 × 4C3 × 2C1 = 8
⎝ m + 1⎠ ⎝ m⎠ ⎝ m⎠
∴ Required number = 8 + 4 + 6 + 8 = 26
  
Second Method:
⎛ n ⎞ ⎛ n⎞ ⎛ n + 1⎞ Number of ways of putting six cross marks in 8 squares
= ⎜ + = ⎜
⎝ m + 1⎠⎟ ⎜⎝ m⎠⎟ ⎝ m + 1⎠⎟ when there is no restriction = 8C6 = 28.
⎛ n ⎞ ⎛ n − 1⎞ ⎛ n − 2⎞ ⎛ m⎞ ⎛ n + 1⎞ Number of ways when the six cross marks are put in first
Thus, ⎜ ⎟ + ⎜ + +…+ ⎜ ⎟ = ⎜ . and second rows = 6C6 = 1.
⎝ m⎠ ⎝ m⎟⎠ ⎜⎝ m⎟⎠ ⎝ m⎠ ⎝ m + 1⎟⎠
Number of ways when the six cross marks are put in second
The correct option is (B) and third rows = 6C6 = 1.
75. A number of the seven digits will be of the form. ∴ Required number = 28 – 2 = 26.
ax1 x2 x3 x4 x5 0, ax1 x2 x3 x4 x5 1, ax1 x2 x3 x4 x5 2, The correct option is (A)
ax1 x2 x3 x4 x5 3, … ax1 x2 x3 x4 x5 9 78. Required number = coefficient of x16 in (x3 + x4 + … + x7)4
where a ∈ {1, 2, 3, … 9}
= coefficient of x16 in x12 (1 + x + … + x4)
and x1, x2, x3, x4, x5 ∈ {0, 1, 2, 3, …9}. 4
⎛ 1 − x5 ⎞
Since sum of the digits should be even, therefore, 4
= coefficient of x in ⎜ ⎟
if a + x1 + x2 + x3 + x4 + x5 is an even number, then the digit ⎝ 1− x ⎠
at units place must be 0, 2, 4, 6, 8 and if a + x1 + x2 + x3 + = coefficient of x4 in [(1 – x5)4 (1 – x)– 4]
x4 + x5 is an odd number, then the digit at units place will be
= coefficient of x4 in [(1 – 4x5 + 6x10 – …) (1 – x)– 4]
1, 3, 5, 7, 9.
∴ Required number = 9 × 10 × 10 × 10 × 10 × 10 × 5 = coefficient of x4 in (1 – x)–4
4⋅5⋅6⋅7
= 9 × 105 × 5 = 45 × 105. = 7C4 = = 35
The correct option is (B) 4!
Permutations and Combinations  4.35

The correct option is (B) = 33649 – 7722 = 25927


79. Required number = coefficient of x30 in (x2 + x3 + … + x16)8 The correct option is (A)

= coefficient of x30 in x16 (1 + x + … + x14)8 82. Given, x + y + z + u + t = 20 (1)
15 ⎞ 8 x + y + z = 5 (2)
⎛1 − x
= coefficient of x14 in ⎜ ⎟ Given, system of equations is equivalent to
⎝ 1− x ⎠
x + y + z = 5 (3)
= coefficient of x14 in (1– x)– 8 = 21C14 = = 116280. and,  u + t = 15 (4)
The correct option is (C)
Number of non-negative integral solutions of equation (3)
80. The candidate must score 150 marks.
= n + r – 1Cr = 3 + 5 – 1C5 = 7C5,
∴ Required number
Number of non-negative integral solutions of equation (4)
= coefficient of x150 in (1 + x + … + x50)3 (1 + x + … + x100)
3 = n + r – 1Cr = 2 + 15 – 1C15 = 16C15
150 ⎛ 1 − x 51 ⎞ 1 − x101
= coefficient of x in ⎜ ⎟ Required number = 7C5 × 16C15 = 336.
⎝ 1− x ⎠ 1− x ∴
The correct option is (A)
= coefficient of x150 in (1 – x51)3 (1 – x101) (1 – x)–4
83. Let w be a non-negative integer such that

= coefficient of x150 in (1 – 3x51 + 3x102 – x153) 3x + y + z + w = 30
 (1 – x101) (1 – x)–4 Let a = x – 1, b = y – 1, c = z – 1, d = w, then

[leaving terms containing powers of x greater than 150] 3a + b + c + d = 25, where a, b, c, d ≥ 0 (1)
= coefficient of x150 in (1 – x)–4 – 3. coefficient of x99 in Clearly, 0 ≤ a ≤ 8. If a = k, then
(1 – x)–4 + 3 coefficient of x48 in (1 – x)–4 coefficient of x49 b + c + d = 25 – 3k(2)
in (1 – x)–4 Number of non-negative integral solutions of equation (2)
153
= C150 – 3 ⋅ 102C99 + 3 ⋅ 51C48 – 52C49. = n + r – 1Cr = 3 + 25 – 3k – 1C25 – 3k
153 ⋅ 152 ⋅ 151 102 ⋅ 101 ⋅ 100 ( 27 − 3k ) ( 26 − 3k )
= − 3⋅ = 27 – 3kC25 – 3k = 27 – 3kC2 =
6 6 2
3
51 ⋅ 50 ⋅ 49 52 ⋅ 51 ⋅ 50 = (3k2 – 53k – 234)
+ 3 · − 2
6 6 3 8
∴ Required number = ∑ (3k 2 − 53k + 234)

HINTS AND EXPLANATIONS


= 110556 2k=0
The correct option is (C)
81. Any number between 1 and 1000000 must be of less than 3 ⎡ 8 × 9 × 17 8×9 ⎤
= 3⋅ − 53 + 234 × 9⎥ = 1215.
seven digits. Therefore, it must be of the form 2 ⎢⎣ 6 2 ⎦
a1 a2 a3 a4 a5 a6 The correct option is (B)
where a1, a2, a3, a4, a5, a6 ∈ {0, 1, 2, …, 9} 84. In the mouth, there are 32 places for the teeth. For each
According to question, sum of the digits = 18 place, two choices are possible, i.e., either tooth or not.
Thus, a1 + a2 + a3 + a4 + a5 + a6 = 18 Choices for 1 teeth = 2
where 0 ≤ ai ≤ 9, i = 1, 2, 3, …, 9. Similarly choices for 2 teeth = 2 × 2 = 22

Required number Choices for 3 teeth = 2 × 2 × 2 = 23

= coefficient of x18 in (1 + x + x2 + … + x9)6 ……………………………………..
6 ……………………………………..
⎛ 1 − x10 ⎞
18
= coefficient of x in ⎜ ⎟ Hence, choices for 32 teeth = 232
⎝ 1− x ⎠

But this includes one case where there is no teeth.
= coefficient of x18 in [(1 – x10)6 (1 – x)–6]
∴ Required number of ways = 232 – 1.
= coefficient of x18 in [(1 – 6C1 x10) (1 – x)–6]
The correct option is (B)
[leaving terms containing powers of x greater than 18] 85. For any group of five scientists there must be at least one
= coefficient of x18 in (1 – x)– 6 – 6C1. coefficient of x8 in lock they can not open. Moreover, for any two different
(1 – x)– 6 groups of five scientists, there must be two locks they can-
6 + 18 – 1
= C18 – 6 ⋅ 6 + 8 – 1C18 = 23C5 – 6 ⋅ 13C8 not open, because if both groups cannot open the same lock,
there is a group of six scientists among these two groups
23 ⋅ 22 ⋅ 21 ⋅ 20 ⋅ 19 13 ⋅ 12 ⋅ 11 ⋅ 10 ⋅ 9 who will not be able to open the cabinet. Thus, at least
= − 6⋅
120 120
4.36  Chapter 4

11 × 10 × 9 × 8 × 7 = (4P4 – 3P3) + (5P4 – 4P3) + (6P4 –5P3) + … + (9P4 – 8P3)


11C6 = = 11 × 3 × 2 × 7
1× 2 ×3× 4 × 5 9
= 462 locks are needed. = ∑ ( n P4 − n−1P3 )
n=4
The correct option is (C)
86. The total number of permutations of the digits of given The correct option is (A)
number is
90. Let D denotes Delhi and I denotes Indore.
7!
= 420
2!3! S1 S2 S3 S4 S5 S6 S7 S8 S9
D I
But when 0 occupies the first position, the number becomes
a six-digit number which are not acceptable, since such a For S1, 9 different tickets are available, one for each of the
number will become less then 106. remaining 9 stations. Similarly, for S2, 8 different tickets are
The total number of such six-digit numbers is available and so on.
6! Therefore, total number of 9 different tickets
= = 60
2!3! = 9 + 8 + 7 + 6 + 5 + 4 +3 +2 +1 = 45
Hence, the required number is 420 – 60 = 360. Thus, the six different tickets must be any six of these 45
The correct option is (A) and there are clearly as many different sets of 6 tickets as
they are combinations of 45 things taken 6 at a time.
n!( 21 − n)! 1 21!
87. n! (21 – n)! = 21! = 21! 21 = 21 Hence, the required number = 45C6.
21! Cn Cn
The correct option is (C)
For minimum value, 21Cn is maximum.
91. Using 2, 5 and 7 with repetition each place of n digit num-
Maximum value of 21 Cn = 21C 21 − 1 = 21C10 ber can be chosen in 3 ways. Hence, total number of n- digit
2
numbers = 3 × 3 ×3 … n times = 3n.
21! 21!
∴ Minimum value = = × 10!11! = 10!11! According to given condition 3n ≥ 900 ⇒ 3n–2 ≥ 100.
21
C10 21!
∴ n – 2 ≥ 5 ⇒ n ≥ 7.
The correct option is (B) The correct option is (C)
88. The maximum number of oranges that a child can get =
92. Let b = a + d and c = a + 2d.
20 – 3 = 17.
Given: a + a + d + a + 2d = 21 ⇒ a + d = 7
Thus, the problem is equivalent to finding the number of
HINTS AND EXPLANATIONS

integral solutions to the equation ∴ a + c = 14 and b = 7


x1 + x2 + x3 + x4 = 20 Now, the number of positive integral solutions of a + c =
where 1 ≤ x1, x2, x3, x4 ≤ 17, and x1, x2, x3, x4 denote the 14 is equal to coefficient of x14 in (x + x2 + x3 +….)2 =
number of oranges given to the four children. Coefficient of x12 in (1 + x + x2…)2 = 13 C12 = 13.
Hence, the required number of ways is The correct option is (A)
= coefficient of x20 in (x + x2 + x3 + … + x17)4 93. A mixed doubles game involves two males and two females.

= coefficient of x16 in (1 + x + x2 + … + x16)4 Two males can be chosen from n males in nC2 ways. Having
chosen two males, now 2 females are to be chosen from
= coefficient of x16 in (1 – x17)4 (1 – x)–4
(n –2) females leaving the wives of the two already chosen

= coefficient of x16 in (1 – x)–4 males. This can be done in n –2C2 ways.
16 + 4 –1 Hence, the required number of ways
= C16
19 = n C2. n –2 C2.2
= C3 = 969.
The correct option is (B) [for every choice of 2 males and 2 females,
 they can be paired in 2 ways].
89. The smallest number, which can occur in the middle is 4.
The number of numbers with 4 in the middle = 4P4 – 3P3 n( n − 1) ( n − 2)( n − 3)
= . .2
(∴ The other four places are to be filled by 0, 1, 2 and 3, and
2 2
a number can not begin with 0)
n( n − 1)( n − 2)( n − 3) 1 n
Similarly, the number of numbers with 5 in the middle = = = . P4
5 2 2
P4 – 4P3, etc.
The correct option is (C)
∴ The required number of numbers
Permutations and Combinations  4.37

Previous Year’s Questions


94. Number of choices is equal to 5C4 × 8C6 + 5C5 × 8C5 103. ( n −1) C( r −1) = (10 −1) C( 4 −1) = 9 C3
= 140 + 56. Statement 1 is correct
The correct option is (B) Statement 2 is also correct
95. n
Cr +1 + nCr −1 + nCr + nCr From 9 we can select 3 in 9C3ways. It is correct explanation.
The correct option is (D)
n +1 n +1
= Cr +1 + Cr
104. Number of ways of selecting one or more balls from 10
= n+ 2
Cr +1 . white, 9 green, and 7 black balls

The correct option is (B) = (10 + 1)(9 + 1) (7 + 1) – 1 = 11*10* 8 – 1 = 879.
96. Since in half the arrangement A will be before E and other The correct option is (D)
half E will be before A. 105. Since n +1C3 − n C3 = 10 ⇒ n C2 = 10 ⇒ n = 5 .
6!
Therefore total number of ways = = 360. The correct option is (A)
2
The correct option is (C) 1 06. Four digit numbers, which start from 6, 7, 8 = 3 × 4 × 3 × 2 = 72
97. Number of balls = 8 Five digit numbers = 5! = 120
And, number of boxes = 3
∴ Total number of integers = 192.
Hence number of ways = 7C2 = 21.
The correct option is (A)
The correct option is (D)
107. Total number of words which can be formed using all the
98. Alphabetical order for the name SACHIN is
letters of the word ‘SMALL’
A, C, H, I, N, S
5!
No. of words starting with A = 5! = = 60
2!
No. of words starting with C = 5!
No. of words starting with H = 5! Now, 60th word is → SMLLA
No. of words starting with I = 5! 59th word is → SMLAL
No. of words starting with N = 5! 58th word is → SMALL
SACHIN − 1 The correct option is (A)
So the sum is 601. 108. X(4 L 3 G) Y(3 L 4 G)
The correct option is (A) 3 L 0 G 0 L 3 G

HINTS AND EXPLANATIONS


99. Total number of ways = 10C1 + 10C2 + 10C3 + 10C4 2 L 1 G 1 L 2 G
= 10 + 45 + 120 + 210 1 L 2 G 2 L 1 G
= 385 0 L 3 G 3 L 0 G
The correct option is (C) Required number of ways
100. Other than S, seven letters M, I, I, I, P, P, I can be arranged = 4 C3 . 4C3 + ( 4 C2 . 3C1 ) 2 + ( 4 C1 . 3C2 ) 2 + (3 C3 ) 2
7!
in = 7 . 5 . 3. = 16 + 324 + 144 + 1
2! 4 !
Now four S can be placed in 8 spaces in 8C4 ways. = 485
Hence, desired number of ways = 7 ⋅ 5 ⋅ 3 ⋅ 8C4 = 7 ⋅ 6C4 ⋅ Hence, the correct option is (D)
8 109. Refer to the position where N denotes Novel and D denotes
C4.
The correct option is (D) Dictionary
101. Since, 4 novels can be selected from 6 novels in 6C4 ways N N D N N
and 1 dictionary can be selected from 3 dictionaries in 3C1
ways. As the dictionary selected is fixed in the middle, the Number of ways of selection 4 novel and one dictionary
remaining 4 novels can be arranged in 4!ways. form 6 different novel and three different dictionary
∴  The required number of ways of arrangement ⇒ 6 C4 × 3C1
= 6C4 ×3 C1 × 4! = 1080 ∴ Total number of arrangement are
6
The correct option is (D) ⇒ C4 × 3C1 × 4!  [Position of dictionary is fixed]
3 9 6! 3!
102. Total number of ways = C2 × C2 ⇒ × × 4!
9×8 4! × 2! 2! × 1!
= 3× = 3 × 36 = 108
2 6! × 3!
⇒ = 1080
The correct option is (C) ( 2!) 2
This page is intentionally left blank.
CHAPTER
Mathematical Induction
5
LEARNING OBJECTIVES
After reading this chapter, you will be able to:
  Define mathematical induction   Understand the principle of mathematical induction
  Learn how to denote mathematical induction

MATHEMATICAL INDUCTION Thus, in order to prove a statement p(n) to be true for all
natural numbers, we have the following working rule:
In drawing mathematical or scientific conclusions, there are
two basic processes of reasoning that are commonly used. WORKING RULE
These are deduction and induction. Deduction is the p­ rocess
of reasoning from general to particular and ­induction is the ■ Prove that p(1) is true; i.e., p(n) is true for n = 1.
process of reasoning from particular to general. In this unit, ■ Assume p(k) to be true; i.e., p(n) is true for n – k.
we plan to study induction. Induction begins by observa-
■ Prove that p(k + 1) is also true; i.e., p(n) is also true for n =
tions and from observations we arrive at some tentative k + 1.
conclusions, called conjectures. A conjecture may be true
or false. The principle of mathematical induction helps us
in proving some of these conjectures which are true. Info Box!
It is important to note that for the proof by mathe-
A Notation for Statements
matical induction both the conditions (i) and (ii) as
Consider the statements: stated above must be fulfilled. The result obtained
may be fallacious if only one of these conditions is
1. n(n + 1) is divisible by 2. satisfied. Even if we prove a certain statement for a
2. 23n – 1 is divisible by 7. large number of values of n, say n = 1, 2, …, 100,
3. 2n > n, etc. we cannot say that the statement is true for all val-
ues of n unless we also establish the condition (ii).
All these statements are concerned with n, which takes
values 1, 2, 3, … Such statements are usually denoted by
p(n). By giving particular values to n, we get particular
SOLVED EXAMPLES
statement.
For example, if the statement 32n – 1 is divisible by
1. If n ∈ N, then 72n + 33n –3 ⋅ 3n – 1 is always divisible by
8 is denoted by p(n), then p(4) is the statement 32.4 – 1 is
divisible by 8. (A) 25 (B) 35
(C)  45 (D)  None of these
The Principle of Mathematical Induction Solution: (A)
This principle states: If p(n) is a statement involving natural Putting n = 1 is 72n + 23n–3.3n–1, we get
number n, then 72.1 + 23.1–3 ⋅ 31–1 = 7220.30 = 49 + 1 = 50 (1)
1. if p(1) is true, and Also, for n = 2
2. if p(k + 1) is true whenever p(k) is true, then p(n) is 72.2 + 23.2–3 ⋅ 32–1 = 2401 + 24 = 2425 (2)
true for all natural numbers n.
From Eq. (1) and (2), it is always divisible by 25.
5.2  Chapter 5

2. For every natural number n, (n2 – 1) is divisible by p (3) : 23 < 3!, which is not true
(A) 4 (B) 6 p (4) : 24 < 4!, which is true.
(C)  10 (D)  None of these Let p(k) be true if k ≥ 4, i.e., 2k < k !, k ≥ 4
⇒ 2.2k < 2 (k!) ⇒ 2k + 1 < k(k!) ( k ≥ 4 >2)
Solution: (B)
⇒ 2k + 1 (k + 1)! ⇒ p(k + 1) is true.
We have, n, (n2 – 1) = (n – 1) n(n + 1). Hence, we conclude that p(n) is not true for n = 2,3 but
It is product of three consecutive natural numbers, so hold true for n ≥ 4.
by Lagrange’s theorem it is divisible by 3! i.e., 6.
8. x(xn–1 – nan – 1) + an(n – 1) is divisible by (x – a)2 for
3. For a positive integer n, (A) n > 1 (B) n > 2
1 1 1 1 (C) all n ∈ N (D)  None of these
Let a(n) = 1 + + + + … + n .Then
2 3 4 2 −1
Solution: (C)
(A) a (100) ≤ 100 (B)  a (100) > 100
Check the options. The condition is satisfied for all
(C) a (200) ≤ 100 (D)  a (200) > 100
n ∈ N.
Solution: (A)
n 9. For natural number n, 2n(n – 1)! < nn, if
It can be proved by induction that > a( n) ≤ n .
2 (A) n < 2 (B) n > 2
200 (C) n ≥ 2 (D)  for no n
∴ < a( 200) ⇒ a(200) > 100 and a(100) ≤ 100.
2
Solution: (B)
4. Let p(n) denote the statement that n2 + n is odd. It is Check the options. The condition is satisfied for n > 3
seen that p(n) ⇒ p(n + 1), Pn is true for all
n
(A) n > 1 (B) n ⎛ n + 1⎞
10. If n is a natural number then ⎜ ≥ n ! is true
(C) n > 2 (D)  None of these ⎝ 2 ⎟⎠
when
Solution: (D) (A) n > 1 (B) n ≥ 1
Since the square of any odd number is always odd an (C) n > 2 (D) n ≥ 2
sum of two odd numbers is always even, so for no ‘n’
Solution: (B)
this statement is true.
Check the options. The condition is true for n ≥ 1.
5. If n ∈ N, then 10n + 3 (4n + 2) + 5 is divisible by
11. Statement 1: For every natural number n ≥ 2,
(A) 7 (B) 5 (C) 9 (D) 17
1 1 1
Solution: (C) + +… + > n
1 2 n
For n = 102 + 3(44) + 5 = 100 + 768 + 5 = 873, which
is divisible by 9. Statement  2: For every natural number n ≥ 2,
n( n +1) < n + 1.
6. The value of the natural numbers n such that the
inequality 2n > 2n + 1 is valid, for (A)  Statement 1 is false, Statement 2 is true
(A) n ≥ 3 (B) n < 3 (B) Statement 1 is true, Statement 2 is true, Statement 2
(C) no n (D)  any n is a correct explanation for Statement 1
(C) Statement 1 is true, Statement 2 is true; Statement 2
Solution: (A) is not a correct explanation for Statement 1
Check through options, the condition 2n > 2n + 1 is (D)  Statement 1 is true, Statement 2 is false
valid for n ≥ 3.
Solution: (C)
7. Let p(n) be statement 2n < n!, where n is a natural num- 1 1 1
ber, then p(n) is true for: P(n) = + +…+
1 2 n
(A) all n (B)  all n > 2
1 1
(C) all n > 3 (D)  None of these P(2) = + > 2
1 2 
Solution: (C)
1 1 1
Let  p(n) : 2n < n! Let us assume that P(k) = + +… + > k
Then, p (1) : 2! < 1!, which is not true 1 2 k
is true.
 p (2) : 22 < 2!, which is not true
Mathematical Induction  5.3

1 1 1 1 Statement 1 is correct.
∴ P(k + 1) = + +…+ + > k +1
1 2 k k +1 P(2) = 2 × 3 < 3
has to be true. If P(k) = k ( k + 1) < (k + 1) is true
1 k ( k + 1) + 1
L.H.S. > k + = Now  P(k + 1) = ( k + 1)( k + 2) < k + 2 has to be
k +1 k +1
true.
Since k ( k + 1) > k (∀ k ≥ 0) Since (k + 1) < k + 2
k ( k + 1) + 1 k +1 ∴ ( k + 1)( k + 2) < (k + 2)
∴ > = k +1 
k +1 k +1 Hence, Statement  2 is not a correct explanation of
Let P(n) = n( n + 1) < n + 1 Statement 1.
5.4  Chapter 5

NCERT EXEMPLARS
1. If 10n + 3.4n + 2 + k is divisible by 9, for all n Î N, then 3. If xn – 1 is divisible by x – k, then the least positive
the least positive integral value of k is integral value of k is
(A) 5 (B) 3 (C) 7 (D) 1 (A) 1 (B) 2 (C) 3 (D) 4
2. For all n Î N, 3.52n+1 + 23n+1 is divisible by
(A) 19 (B) 17 (C) 23 (D) 25

ANSWER K EYS
1. (A) 2.  (B), (C) 3. (A)

HINTS AND EXPLANATIONS

1. Let P(n) : 10n + 3.4n+2 + k is divisible by 9, for all n Î N. 2. Given that, 3.52n+1 + 23n+1
For n = 1, the given statement is also true 101+3.41+2+k is For n = 1,
divisible by 9. 3.52(1)+1 + 23(1) + 1
∴ = 10 + 3.64 + k = 10 + 192 + k = 3.53 + 24
= 202 + k = 3 × 125 + 16 = 375 + 16 = 391
If (202 + k) id divisible by 9, then the least value of k must be Now, 391 = 17 × 23
5. Which id divisible by both 17 and 23.
202 + 5 = 207 is divisible by 9 3. Let P(n) : xn –1 is divisible by (x – k).
207 For n = 1, x1 – 1 is divisible by (x – k).
⇒ = 23
9 Since, if x – 1 is divis ible by x – k. Then, the least possible

Hence, the least value of k is 5. integral value of k is 1.
NCERT EXEMPLARS
Mathematical Induction  5.5

PRACTICE EXERCISES

Single Option Correct Type

1. For each natural number n, 3n > n3 for (C)  an odd positive integer
(A) n > 2 (B) n ≥ 3 (D)  None of these
(C) n ≥ 4 (D) n < 4 n7 n5 2n3 n
6. For every positive integer n, + + − is
n5 n3 7 7 5 3 105
2. For n ∈ N , + + n is (A)  an integer
5 3 15
(B)  a rational number
(A)  an integer (B)  a natural number (C)  an odd integer
(C)  a positive fraction (D)  None of these (D)  a negative real number
⎡ cos θ sin θ ⎤ n 7. 10n + 3.4n+2 + k is divisible by 9 for n ∈ N . Then, the
3. If A = ⎢ ⎥ , then for n ∈ N, A is equal to
⎣ − sin θ cos θ ⎦ least positive integral value of k is
(A) 1 (B) 3 (C) 5 (D) 7
⎡ cos n θ sin n θ ⎤ ⎡ cos nθ sin nθ ⎤
(A)  ⎢ ⎥ (B) 
⎢ − sin nθ cos nθ ⎥ 8. The sum of the cubes of three consecutive natural
n n
⎢⎣ − sin θ cos θ ⎥⎦ ⎣ ⎦
numbers is divisible by
⎡cos nθ − sin nθ ⎤ ⎡ n cos θ n sin θ ⎤ (A) 2 (B) 4 (C) 6 (D) 9
(C)  ⎢ (D) 
⎣ sin nθ cos nθ ⎥⎦ ⎢ − n sin θ
⎣ n cos θ ⎥⎦ 1 1 1
9. For all n ∈ N, 1 + + +… +
1 1 1 2 3 4
4. + + +… to n terms = (A) > n (B) < n
1.4 4.7 7.10
1 1 (C) ≤ n (D)  ≥ n
(A)  (B) 
5n − 1 3n − 1 10. The statement p(n): 1 × 1! + 2 × 2! + 3 × 3! + …
n n + n × n! = (n + 1)! – 1 is
(C)  (D) 
3n + 1 5n − 1 (A)  true for all n > 1
(B)  not true for any n
5. x2n–1 + y2n–1 is divisible by x + y if n is (C)  true for all n ∈ N
(A)  a positive integer (D)  None of these

PRACTICE EXERCISES
(B)  an even positive integer

ANSWER K EYS
Single Option Correct Type
1. (C) 2.  (B) 3. (B) 4.  (C) 5. (A) 6.  (A) 7. (C) 8.  (D) 9. (D) 10.  (C)
This page is intentionally left blank.
CHAPTER
Binomial Theorem
6
LEARNING OBJECTIVES
After reading this chapter, you will be able to:
 Learn the definition of binomial theorem and binomial  
Know how to calculate middle term in the binomial
expression expansion
 Be familiar with pascal’s triangle and methods to find
general term, independent term and greatest term in the
expansion of (1 + x)n

BINOMIAL EXPRESSION 3.
Replacing x by 1 and y by – x, we get
(1 – x)n = nC0 – nC1 x + nC2 x2 – ... + (– 1)n nCn xn.
An algebraic expression consisting of only two terms is called
a binomial expression. For example, expressions such as 4.
Adding (1) and (2), we get
4 (x + y)n + (x – y)n = 2 (xn + nC2 xn – 2 y2 +
x + a, 4x + 3y, 2x – n
C4 xn – 4 y4 + ...)
y
are all binomial expressions. = 2 (sum of terms at odd places).
The last term is nCn yn or nCn – 1 xyn – 1 according as n is
BINOMIAL THEOREM
even or odd respectively.
This theorem gives a formula by which any power of a 5. Subtracting Eq. (2) from (1), we get
binomial expression can be expanded. It was first given by (x + y)n – (x – y)n = 2 (nC1 xn – 1 y + nC3 xn – 3 y3 + ...)
Sir Isaac Newton. = 2 (sum of terms at even places)
Binomial Theorem for Positive Integral Index The last term is nCn – 1 xyn – 1 or nCn yn according as n is
even or odd respectively.
If x and y are real numbers, then for all n ∈ N,
(x + y)n = nC0 xn y0 + nC1 xn – 1 y1 + nC2 xn – 2 y2 QUICK TIPS
+ ...+ nCn – 1 x1 yn – 1 + nCn x0 yn  (1)
n  The coefficient of (r + 1)th term in the expansion of
i.e., (x + y)n = ∑ n Cr x n−r y r (1 + x)n is nCr.
r =0
 The coefficient of x in the expansion of (1 + x) is C .
r n n
Here nC0, nC1, nC2, ..., nCn are called binomial coefficients. r

For the sake of convenience, we may denote nCr by Cr ⋅ nCr


 n I M P O R TA N T P O I N T S
may also be denoted as  .
 r 
 The positive integer n is called the index of the binomial.
SPECIAL CASES
 Number of terms in the expansion of (x + y)n is n + 1,

1.
Replacing y by – y in (1), we get i.e., one more than the index n.
(x – y)n = nC0 xn y0 – nC1 xn – 1 y1 + nC2 xn – 2 y2  In the expansion of (x + y)n, the power of x goes on

... + (– 1)n nCn x0 yn(2) decreasing by 1 and that of y goes on increasing by 1 so


2.
Replacing x by 1 and y by x, we get that the sum of powers of x and y in any term is n.
(1 + x)n = nC0 + nC1 x + nC2 x2 + ... + nCn xn
6.2  Chapter 6

PASCAL’S TRIANGLE
 The binomial coefficients of the terms equidistant from
the beginning and the end are equal. The coefficients nC0, nC1, nC2, ... , nCn or simply C0, C1, C2
 If n is odd, then number of terms in (x + a)n +
..., Cn are called binomial coefficients and they can be eval-
n 1 uated with the help of Pascal’s triangle as below:
(x – a)n and (x + a)n – (x – a)n are equal to .
2
 If n is even, then the number of terms in the expansion of Exponent of Binomial     Coefficients of successive
 n + 2     terms in Binomial Theorem
(x + a)n + (x – a)n are  and that in the expan-
 2  n = 0
  n = 1
sion of (x + a)n – (x – a)n are  n 
 2  n = 2
 xn + yn is divisible by x + y if n is odd as xn + yn = (x + y) n = 3
(xn – 1 – xn – 2y + xn – 3y2 – ... + yn–1) n = 4
 xn – yn is divisible by x – y if n is even as xn – yn = (x – y)

(xn – 1 + xn–2y + xn–3y2 + ... + yn – 1)


n = 5
 The coefficient of xn in the expansion of (x + 1) (x + 2) ...
–1 n = 6
n ( n 1) I. Each row starts and ends in 1.
(x + n) = .
2 II. The coefficients of an expansion are obtained from
 The coefficient of xn – 1 in the expansion of (x – 1) (x – 2) the coefficients of the previous expansion.
n ( n 1) III. Each coeffcient is the sum of the two coefficients
... (x – n) = – .
2 which lie on either side of it in the previous row.
General Term in the Expansion of (x + y)n
Info Box! In the binomial expansion of (x + y)n, the (r + 1)th term
from the beginning is usually called the general term and it
Students are advised to remember the follow-

is denoted by Tr + 1, i.e.,
ing values:
n ( n −1) Tr + 1 = nCr xn – r yr
n
C0 = 1, nC1 = n, nC2 = ,
2!
SOLVED EXAMPLES
n ( n - 1) ( n - 2 )
n
C3 = ,
3! 1. The sum of the coefficients in the expansion of
n( n − 1) ( n − 2) ... ( n − r + 1) (1 + 5x – 7x3)3165 is
n
Cr =
r! (A) 1 (B) 23165
and nCn = 1. (C) 2
3164
(D) –1
Also, nCr = nCn–r for 1 ≤ r ≤ n.
Solution (D)
i.e., nC0 = nCn, nC1 = nCn – 1, nC2 = nCn – 2, ...
Putting x = 1 in (1 + 5x – 7x3)3165, the required sum of
i.e., coefficients of terms equidistant from the
coefficients = (1 + 5 – 7)3165 = (– 1)3165 = –1.
beginning and end are equal.
12
n
 2 
2. The 8th term of 3 x + 2  , when expanded in
n 2
 ∑ ( Ci ) = 2nCn
i=0  3x 
n
n ascending power of x, is
 ∑ i ( Ci ) = n ⋅ 2n – 1
i=0
228096 228096
n
n
(A) 3 (B) 
 ∑ i (i − 1) ⋅ Ci = n(n – 1)2 x x9
n–2
i=2
n 328179

n
∑ i(i − 1) ... (i − k + 1) ⋅ Ci = n(n – 1) ... (C) 9 (D)  none of these
i=k x
(n – k + 1)⋅ 2n – k Solution (A)
12
n
 2 
∑ i ( Ci ) = n ⋅ 2n – 1Cn – 1
n 2

When 3 x + 2  is expanded, the power of x goes
i =1  3x 
on decreasing as the terms proceed. Hence, it is
Binomial Theorem  6.3

n
 2 
12  1
expanded in descending powers of x. So  2 + 3 x  , 5. The term independent of x in (1 + x)m 1 +  is
 3x   x
(A) m + nCm (B)  m+n
Cn
when expanded, will be in ascending powers of x.

(C) m + nCm – n (D)  none of these
12 12−7
 2   2 
Now, t8 in  2 + 3 x  = 12C7 
 3 x 2  ⋅ (3x)7 Solution (B)
 3x 
We have,
n n
12 !  2 
5  1   x + 1
= 7 ! 5! ⋅  2  ⋅ (3x)7 (1 + x) 1 +  = (1 + x)m
m 
 x 
 3x   x
12´11´10 ´ 9´8 25 × 32 (1+ x ) m+ n
= ⋅ = = x– n (1 + x)m + n
5´ 4 ´3´ 2 x3 xn
∴ Required term independent of x = coefficient of
228096
= x0 in
x3 x– n (1 + x)m + n = coefficient of xn in (1 + x)m + n
3. If A is the sum of the odd terms and B the sum of even
= m+n
Cn
terms in the expansion of (x + a)n, then A2 – B2 =
(A) (x2 + a2)n (B)  (x2 – a2)n 6. The coefficient of x53 in the expansion
(C) 2 (x – a )
2 2 n
(D)  none of these 100

100
Cm ( x − 3)100−m ⋅ 2m is
Solution (B) m=0

We have, (A) 100C47 (B) 100


C53
(x + a)n = nC0 xn + nC1 xn – 1 a1 + nC2 xn – 2 a2 (C) – 100C53 (D) –100C100
 + nC3 xn – 3 a3 + ... + nCn xn Solution (C)
= (nC0 xn + nC2 xn – 2 a2 + ...) 100
+ (nC1 xn – 1 a1 + nC3 xn – 3 a3 + ...) We have, ∑ 100
Cm ( x − 3)100−m ⋅ 2m
= A + B m=0

(x – a)n = nC0 xn – nC1 xn – 1 a1 + nC2 xn – 2 a2 = (x – 3) + 100C1 (x – 3)99 ⋅ 21


100

– nC3 xn – 3 a3 + ... + nCn (– 1)n an + 100C2 (x – 3)98 ⋅ 22 + ... + 100C100 2100


= (nC0 xn + nC2 xn – 2 a2 + ...) = [(x – 3) + 2]100 = (x – 1)100 = (1 – x)100
– (nC1 xn – 1 a1 + nC3 xn – 3 a3 + ...) ∴  coefficient of x54 = 100C53 (– 1)53 = – 100C53
= A – B
7. The coefficient of xm in (1 + x)m + (1 + x)m + 1 +, ..., + (1
∴ A2 – B2 = (A + B) (A – B) = (x + a)n (x – a)n
+ x)n, m ≤ n is
= (x2 – a2)n
10 (A) nCm (B)  n
Cm + 1
1 
4. The 7th term in  + y  , when expanded in descend- (C) n + 1Cm + 1 (D)  none of these
2

y 
Solution (C)
ing power of y, is
The coefficient of xm in
210 y2
(a) 2 (b)  (1 + x)m + (1 + x)m + 1 + (1 + x)m + 2 + ... + (1 + x)n
y 210 = mCm + m + 1Cm + m + 2Cm + ... + nCm
(c) 210 y  2
(d)  none of these = m + 1Cm + 1 + m + 1Cm + m + 2Cm + ... + nCm
Solution (C) (  mCm = m + 1Cm + 1 = 1)
10 = m + 2Cm + 1 + m + 2Cm + ... + nCm
1 
When  + y  is expanded, the powers of y go on (  nCr + nCr + 1 = n + 1Cr + 1)
2

y  = m + 3Cm + 1 + ... + nCm = n + 1Cm + 1.


increasing as the terms proceed. Hence it is expanded
10 8. The coefficient of x3 in the expansion of (1 – x + x2)6 is
 2 1
in ascending powers of y. So  y +  , when (A) 50 (B)  – 50
 y (C) 68 (D)  none of these
expanded, will be in descending powers of y.
6
Solution (B)
 1  10 ´ 9´8´ 7
 (1 – x + x2)6 = [1 – x (1 – x)]6
Hence, t7 = 10C6 (y2)4  y  = y2
  4 ´3´ 2´1 = 6C0 – 6C1 x (1 – x) + 6C2 x2 (1 – x)2
= 210 y2
6.4  Chapter 6

– 6C3 x3 (1 – x)3 + ... to 7 terms 11. The coefficient of x5 in the expansion of


= 6C0 – 6C1 x (1 – x) + 6C2 x2 (1 – 2x + x2) (1 + x2)5 (1 + x)4 is
– 6C3 x3 (1 – 3x + 3x2 – x3) + ... to 7 terms (A) 40 (B)  50
∴ Coefficient of x3 = – 2 ⋅ 6C2 – 6C3, (collecting coef- (C) – 50 (D)  60
ficients of x3 from each term)
Solution (D)
6! 6!
=–2 - = – 50 We have, (1 + x2)5 (1 + x)4
2 ! 4 ! 3! 3! = (1 + 5C1 x2 + 5C2 x4 + ...) (1 + 4C1 x
log x 5
9. The value of x in the expression ( x + x 10 ) , if the + 4C2 x2 + 4C3 x3 + 4C4 x4)
third term in the expansion is 10,00,000, is = (1 + 5x2 + 10x4 + ...) (1 + 4x + 6x2 + 4x3 + x4)
The term giving x5 in the above product is
(A) 10– 1 (B) 101
(5x2) (4x3) + (10x4) (4x) = (20 + 40) x5 = 60x5
(C) 10 – 5/2
(D) 105/2
Hence, the coefficient is 60.
Solution (B, C)
12. If (1 + x – 2x2)6 = 1 + a1 x + a2 x2 + ... + a12 x12, then
Put log10 x = z
Then, given expression = (x + xz)5. a2 + a4 + a6 + ... + a12 =
Now, T3 = 5C2 x3 (xz)2 = 10x3 + 2z = 106 (A) 21 (B)  11
∴  x3 + 2z = 105. (C) 31 (D)  none of these
Taking log, we get Solution (C)
(3 + 2z) log10 x = 5 log10 10
Given
⇒ (3 + 2z) z = 5 or 2z2 + 3z – 5 = 0
5 (1 + x – 2x2)6 = 1 + a1x + a2x2 + ... + a12x12
⇒ (z – 1) (2z + 5) = 0 ⇒ z = 1, – Putting x = 1, we get
2
5 0 = 1 + a1 + a2 + ... + a12...(1)
∴ log10 x = 1 or –   ∴  x = 101 or 10–5/2. Putting x = – 1, we get
2
n n n 64 = 1 – a1 + a2 – ... + a12...(2)
C C C
10. The value of 1 + 3 + 5 + ... is Adding Eq. (1) and (2), we get
2 4 6 64 = 2 (1 + a2 + a4 + ...)
2n 1 2n + 1 ∴ a2 + a4 + a6 + ... + a12 = 31
(A) (B) 
n n
13. If 7103 is divided by 25, then the remainder is
2n − 1 2n + 1 (A) 20 (B)  16
(C) (D) 
n +1 n +1 (C) 18 (D)  15
Solution (C) Solution (C)
The rth term of the given expression is We have, 7103 = 7 (49)51 = 7 (50 – 1)51
n
C2 r - 1 = 7 (5051 – 51C1 5050 + 51C2 5049 – ... – 1)
Tr =
2r = 7 (5051 – 51C1 5050 + 51C2 5049 – ...) – 7 + 18 – 18
1 n 1 n +1 = 7 (5051 – 51C1 5050 + 51C2 5049 – ...) – 25 + 18
Since ⋅ Cr = ⋅ C r +1 = k + 18 (say)  Q  k is divisible by 25,
r +1 n +1
∴  remainder is 18.
n
C2 r - 1 1 n +1
∴ Tr = = ⋅ C2 r 14. The sum of rational terms in the expansion of
2r n +1
n n
( 2 + 31/5 )10 is
n
C1 C C (A) 31 (B)  41
∴  + 3 + 5 + ...
2 4 6 (C) 51 (D)  none of these
1 n +1
  = ( C2 + n + 1C4 + ...)
n +1 Solution (B)
1 (r + 1)th term in the given expansion is given by
= ( 2n + 1 − 1 − n + 1C0 )
n +1 tr + 1 = 10Cr 102-r 5r , where r = 0, 1, 2, ..., 10
2 3
2n − 1 For rational terms
= r = a multiple of 5 = 0, 5, 10 ...(1)
n +1
Binomial Theorem  6.5

10 – r = a multiple of 2 = 0, 2, 4, 6, 8, 10 ...(2) For denominator, 31 = 3, 32 = 9, 33 = 27, 34 = 81, 35 = 243


From Eq. (1) and (2) possible values of r are : 0 and 10 6
C1 = 6, 6C2 = 15, 6C3 = 20
∴  sum of rational terms 6
C4 = 6C2 = 15, 6C5 = 6C1 = 6, 6C6 = 1
= t1 + t11 = 10C0 ( 2 )10 (31/5)0 + 10C10 ( 2 )0 (31/5)10 ∴  denominator = 36 + 6C1 35 ⋅ 21 + 6C2 34 ⋅ 22
= 25 + 32 = 32 + 9 = 41 + 6C3 33 ⋅ 23 + 6C4 32 ⋅ 24 + 6C5 3 ⋅ 25 + 6C6 26
15. In the expansion of (x + a)n if the sum of odd terms be This is clearly the expansion of
P and the sum of even terms be Q, then 4PQ = (3 + 2)6 = 56 = (25)3
(A) (x + a)n – (x – a)n (B) (x + a)n + (x – a)n
Numerator (25)3
(C) (x + a) – (x – a)
2n 2n
(D)  none of these ∴  = ( 25)3 = 1
Denominator
Solution (c)
We have, 18. Larger of 9950 + 10050 and 10150 is
(x + a)n = xn + nC1 xn – 1 a + nC2 xn – 2 a2 (A) 10150 (B) 9950 + 10050
+ nC3 xn – 3 a3 + ... (C) both are equal (D)  none of these
= (xn + nC2 xn – 2 a2 + ...) + (nC1 xn – 1 a
Solution (A)
+ nC3 xn – 3 a3 + ...)
=P+Q We have,
∴ (x – a)n = P – Q, as the terms are alternatively pos- 10150 = (100 + 1)50
50 × 49
itive and negative. = 10050 + 50 ⋅ 10049 + ⋅ 10048 + ...
1× 2
∴ 4PQ = (P + Q)2 – (P – Q)2 = (x + a)2n – (x – a)2n and 9950 = (100 – 1)50
16. If C0, C1, C2, ..., Cn are the coefficients of the expansion = 10050 – 50 ⋅ 10049 + 50 × 49 ⋅ 10048 – ...
n C 1× 2
of (1 + x)n, then the value of ∑ k is Subtracting, we get
0 k +1
10150 – 9950 = 2 (50 ⋅ 10049 + × 10047 + ...)
2n 1
(A) 0 (B)  50 × 49 × 48
n = 10050 + 2 ⋅ ⋅ 10047 + ... > 10050
n +1
2 −1 1× 2 × 3
(C) (D)  none of these
n +1 Hence, 10150 > 9950 + 10050.

Solution (C) 19. For all n ∈ N, 24n – 15n – 1 is divisible by


n
Cr 1 (A) 225 (B)  125
Here, tr + 1 = = ⋅ nCr
r +1 r +1 (C) 325 (D)  none of these
1
= ⋅ n + 1Cr + 1 Solution (A)
n +1 n
Ck
Putting r = 0, 1, 2, ... n and adding, we get ∑ 0 k +1
We have, 24n = (24)n = (16)n = (1 + 15)n
∴ 24n = 1 + nC1 ⋅ 15 + nC2 152 + nC3 153 + ...
1 ⇒ 24n – 1 – 15n = 152 (nC2 + nC3 ⋅ 15 + ...)
= (n + 1C1 + n + 1C2 + n + 1C3 + ... + n + 1Cn + 1)
n +1 = 225 K, where K is an integer.
1 2n+1 −1 Hence, 24n – 15n – 1 is divisible by 225.
= (2n + 1 – n + 1C0) =
n +1 n +1 20. When 599 is divided by 13, the remainder is
17. The value of (A) 8 (B)  9
(183 + 73 + 3 ⋅18 ⋅ 7 ⋅ 25) (C) 10 (D)  none of these
6 is
3 + 6 ⋅ 243 ⋅ 2 + 15 ⋅ 81⋅ 4 + 20 ⋅ 27 ⋅ 8 + 15 ⋅ 9 ⋅16 + 6 ⋅ 3 ⋅ 32 + 64 Solution (A)
(A) 0 (B)  1 We have,
(C) 2 (D)  none of these 599 = 53 ⋅ 596 = (125) (625)24
Solution (B) = [13 × 9 + 8] (1 + 48 × 13)24
The numerator is of the form = (13 × 9 + 8) [1 + 24C1 × (48 × 13)
a3 + b3 + 3ab (a + b) = (a + b)3 + 24C2 (48 × 13)2 +...+ (48 × 13)24]
where a = 18 and b = 7 = 8 + terms containing powers of 13.
Hence remainder = 8.
∴  Numerator = (18 + 7)3 = 253.
6.6  Chapter 6

(32)
21. The last digit of the number (32)32 is 24. When 32(32) is divided by 7, the remainder is
(A) 4 (B)  6 (A) 4 (B)  6
(C) 8 (D)  none of these (C) 8 (D)  none of these
Solution (B) Solution (A)
(32) = (2 + 3 × 10)
32 32
(32)32 = (25)32 = 2160 = (3 – 1)160
= 232 + 10k, where k ∈ N = 160C0 3160 – 160C1 ⋅ 3159
Therefore, last digits in (32)32 = last digit in (2)32 + ... + 160C159 ⋅ 3 + 160C160 ⋅ 30
But 21 = 2, 22 = 4, 23 = 8, 24 = 16, 25 = 32 = 3k + 1, where k ∈ N
∴ 232 = (25)6 ⋅ 22 = (32)6 ⋅ 4 = (2 + 30)6 ⋅ 4 ( 32 )
Now, 32( 32 ) = (32)3k + 1 = (25)3k + 1 = 215k + 5
= (26 + 10r) 4, r ∈ N
= 23(5k + 1) ⋅ 22 = (23)5k + 1 ⋅ 4
Last digit in 232 = last digit in (2)6 ⋅ 4 = last digit in
= 4(7 + 1)5k + 1
4×4=6
∴  Last digit in (32)32 = 6. = 4[5k + 1C0 75k + 1 + 5k + 1C1 75k
+ ... + 5k + 1C5k7 + 5k + 1C5k + 1 ⋅ 70]
22. If (1 + x)n = C0 + C1 x + C2 x2 + ... + Cn xn, then = 4(7n + 1), where n ∈ N
C1 C C = 28n + 4.
2C0 + 22 ⋅ + 23 ⋅ 2 + ... + 2n + 1 n = ( 32 )
2 3 n +1 Therefore, when 32( 32 ) is divided by 7, the remain-
3n+1 −1 3n 1 der is 4.
(A) (B) 
n +1 n 25. The number of non zero terms in the expansion of
n+ 2
3 −1 (1 + 3 2 x )9 + (1 − 3 2 x )9 is
(C) (D)  none of these
n+2 (A) 9 (B)  0
(C) 5 (D)  10
Solution (A)
We have, Solution (C)
Cr n
1 In the expansion of
tr + 1 = 2r + 1 = 2r + 1 ⋅ ⋅ n + 1Cr + 1 (1 + 3 2 x)9 + (1 – 3 2 x)9
r +1 n +1
Putting r = 0, 1, 2, ..., n and adding, we get the required 2nd, 4th, 6th, 8th and 10th terms get cancelled.
sum ∴  Number of non-zero terms in
1 2 [9C0 + 9C2 (3 2x )2 + ... + 9C8 (3 2x)8] is 5.
= (2 ⋅ n + 1C1 + 22 ⋅ n + 1C2 + ... + 2n + 1 ⋅ n + 1Cn + 1)
n +1 26. The expression [x + (x3 – 1)1/2]5 + [x – (x3 – 1)1/2]5 is a
1 3n+1 −1 polynomial of degree
= [(1 + 2)n + 1 – n + 1C0] = .
n +1 n +1 (A) 5 (B)  6
(C) 7 (D)  8
23. For integer n > 1, the digit at units place in the number
Solution (c)
100 n
2
∑ r ! + 2 is [x + (x3 – 1)1/2]5 + [x – (x3 – 1)1/2]5
r=0
= 2 [5C0 x5 + 5C2 x3 (x3 – 1) + 5C4 x (x3 – 1)2]
(A) 0 (B)  1 = 2 [x5 + 10x3 (x3 – 1) + 5x (x3 – 1)2]
(C) 2 (D)  3 = 5x7 + 10x6 + x5 – 10x4 – 10x3 + 5x
Solution (A) which is a polynomial of degree 7.
Since the digit at units place in each of 5!, 6!, ..., 100! 27. The value of x, for which the 6th term in the expansion
is 0 and 0! + 1! + 2! + 3! + 4! = 34. 7
 
100  log2 ( 9x−1 +7 ) 1 
of  2 + 1  is 84, is equal to
therefore the digit at units place in ∑ r ! is 4.
r=0

 2 5
log 2 ( 3x−1 +1) 

n

Now, 22 = 24k, k ∈ N (2n is a multiple


n
of 4 form n > 1) (A) 4 (B)  3
∴  The digit at units place in 22 = 24k = (16)k is 6. (C) 2 (D)  1
100
Solution (C,D)
Thus, the digit at units place in ∑ r! + 2
r=0
2n
is 0.
The given expression
Binomial Theorem  6.7

 x−1 
7 ∴  Number of rational terms = 5.
1 ∴  Number of irrational terms = 46 – 5 = 41.
=  9 + 7 + x−1 
1/ 5 
 ( 3 + 1)  31. In the expansion of (1 + x + x3 + x4)10, the coefficient of
Given, T6 = 84 x4 is
5
 1  (A) 40C4 (B)  10
C4
⇒ C5 ( 9
x −1
+ 7) 7−5  
 (3x−1 + 1)1/ 5  = 84
7
(C) 210 (D)  310

1 Solution (D)
⇒ 7C5 (9x – 1 + 7) ⋅ = 84
x−1  (1 + x + x3 + x4)10 = [(1 + x) (1 + x3)]10
(3 + 1)
= (1 + x)10 (1 + x3)10
⇒ 9x – 1 + 7 = 4 (3x – 1 + 1) = (1 + 10C1 x + 10C2 x2 + 10C3 x3 + 10C4 x4 ...)
⇒ 32x – 12 ⋅ 3x + 27 = 0 × (1 + 10C1 x3 + 10C2 x6 ...)
⇒ (3x – 3) (3x – 9) = 0 ∴  Coefficient of x4 = (10C1) (10C1) + 10C4
⇒ 3x = 3, 9 ⇒ x = 1, 2 10 9 8 7
28. When 337 is divided by 80, the remainder is = 100 + = 100 + 210 = 310
12 3 4
(A) 3 (B)  4
(C) 6 (D)  none of these 32. If A = 2nC0 · 2nC1 + 2nC1 2n – 1C1 + 2nC2 2n – 2C1 + ..., then A
is
Solution (A)
(A) 0 (B)  2n
We have, 337 = 34.9 ⋅ 3 = 3(81)9 = 3(80 + 1)9 (C) n 2 2n
(D) 1
= 3(9C0 ⋅ 809 + 9C1808 + ... + 9C9)
Thus, when 337 is divided by 80, the m remainder is 3. Solution (C)
n
 A = coeff. of x in [2nC0(1 + x)2n
a 
29. If the second term in the expansion 13 a +  is + 2nC1 (1 + x)2n – 1 + ...)]
n
 a−1  = coeff. of x in (1 + (1 + x))2n
C3
14 a5/2, then the value of is = coeff. of x in (2 + x)2n
n
C2  x
2n

(A) 8 (B)  12 = coeff. of x in 22n 1 +  = n · 22n


(C) 4 (D)  none of these
 2 
Solution (C) 33. The greatest integer which divides the number
101100 – 1 is
Given : T2 = 14 a5/2
 a 
1 (A) 100 (B)  1000
⇒ C1 (an
)
1/13 n – 1
⋅  −1/ 2  = 14 a5/2 (C) 10000 (D)  100000
a 
Solution (C)
⇒ n ⋅ a(n – 1)/13 ⋅ a3/2 = 14 a5/2
By Binomial theorem
⇒ n ⋅ a(n – 1)/13 = 14 a ⇒ n ⋅ a(n – 14)/13 = 14
 n ( n −1) 2 
⇒ n = 14 (1 + x)n = 1 + nx + ⋅ x ... + x n 
14  2 
n
C3 C3
∴ = = 4
n 14
C2 or (1 + x)n – 1 = nx + n ( n −1) x2 ... + xn
C2 2
30. The number of irrational terms in the expansion of If x = n, (1 + n) – 1 = n + n ( n −1) n2 ... nn
n 2
(41/5 + 71/10)45 is 2
(A) 40 (B)  5  n ( n − 1) 
(C) 41 (D)  none of these (1 + n)n – 1 = n2 1 + ... + nn−2 
 2 
Solution (C) Put n = 100,
 100 (100 −1) 
Total number of terms in the expansion of (1 + 100)100 – 1 = (100)2 1 + ... + 100 98 
(41/5 + 71/10)45 is 45 + 1,  i.e., 46.  2 
The general term in the expansion is  100 × 99 
r (101)100 – 1 = (100)2 1 + ... + 100 98 
Tr + 1 = 45Cr ⋅ 455−r ⋅ 10  2 
4 7
Clearly (101)100 – 1 is divisible by
Tr + 1 is rational if r = 0, 10, 20, 30, 40.
(100)2 = 10000
6.8  Chapter 6

2n 5
 2 1 1
34. If x  occurs in the expansion of  x +  , its coeffi-
p = 2–5 =  
 x  2
cient is
2n 2n ⇒  n = 10
(A) C 4 n- p (B)  C 2 n- p  1 
10
3 3 Therefore, 5th term in  3 2 −  is
2n
 2 
(C) C 4 n - p (D)  none of these 4
10 − 4  
3
T5 = T4 + 1 = 10C4 ( 3 2 ) − 1 
Solution (A)
 2 
Let tr + 1 contains x p. r = 10C4 ( 4) 1 = 10C4 = 10C6
1
Then, tr + 1 = 2nCr (x2)2n – r   = 2nCr x4n – 3r 4
 x  37. If (1 – x + x ) = a0 + a1 x + a2 x2 + ... + a2n x2n, then
2 n

4n − p a0 + a2 + a4 + ... + a2n is equal to


∴ 4n – 3r = p; or r =
3 1 1- 3n
2n (A) 3n2 + (B) 
Hence, the coefficient of x = C 4n − p
p
2 2
3
3n -1 3n + 1
35. Given positive integers r > 1, n > 2 and the coeffi- (C) (D) 
cients of (3r)th term and (r + 2)th term in the binomial 2 2
Solution (D)
expansion of (1 + x)2n are equal, then r =
Putting x = – 1, 1 successively in the given equation
n n
(A) , n even (B)  and adding, we shall get the result.
2 2
(C) n (D) 1 Method for Finding the Independent Term or
Solution (A) Constant Term
We have, t3r = 2nC3r – 1 x3r – 1 Step I  rite down the general term in the expansion of
W
and tr + 2 = 2nCr + 1 xr + 1 (x + a)n i.e., (r + 1) th term
Given, 2n
C3r – 1 = 2nCr + 1 ⇒ tr+1 = nCrxn–rar
⇒ 3r – 1 = r + 1; or (3r – 1) + (r + 1) = 2n Step II Separate the constants and variables. Also group
⇒ 2r = 2 ; or 4r = 2n them separately.
n
⇒ r = 1 (impossible); or r = . Setp II Since, we need to find the term independent of x in
2 the given binomial expansion, equate to zero the
But r is a positive integer greater than 1. So the value index of x and accordingly we will get the value of r
n for which there exists a term independent of x in the
of r is provided n is an even integer (> 2), other-
2 expansion.
wise r has no value.
36. If the last term in the binomial expansion of Greatest Term (Numerically) in the Expansion of
 
n
 1  3
log 8 (1 + x)n
3 2 − 1  is   , then the 5th term is
 3.3 9 
 2  Method 1
(A) 2 ⋅ 10C6 (B) 4 ⋅ 10C4 1. Let Tr (the rth term) be the greatest term.
1 2. Find Tr – 1, Tr , Tr + 1 from the given expansion.
10
(C) C6 (D) 10
C6 Tr T
2 3. Put ≥ 1 and r ≥ 1. This will give an inequal-
Solution (D) Tr+1 Tr-1

n
1   1 
log3 8 ity from where value or values of r can be obtained.
The last term of  3 2 −  =  3  4. Then, find the rth term Tr which is the greatest term.
 2   3⋅ 9 
Method 2
n
 1   1  3
log 8
( n + 1)| x |
⇒  nCn ⋅ −  =   1.
Find the value of k =
 2   3 ⋅ 3 9  1+| x |
 
n/2
  3
log 8
2.
If k is an integer, then Tk and Tk + 1 are equal and both
⇒  (−1) n ⋅  1   =  1  = 3− 3 ⋅ 3 log3 2
5


 2   3 
5/3 are greatest terms.
Binomial Theorem  6.9

3.
If k is not an integer, then T(k) + 1 is the greatest term, Solution (A)
where (k) is the greatest integral part of k. We have, 11 11
 5x   1  
∵ x = 1 
(3 – 5x)11 = 311 1−  = 311 1− 
QUICK TIPS  3  3   5 
| x | ( n + 1)  1 
To find the greatest term in the expansion of (x + y)n, write ∴ m = − < 0
n (| x | + 1)  3 
 y 
(x + y) = x 1+  and then find the greatest term in
n n
 1 
 x  −  (11 + 1)
 
n
 3 
1+ y  . = =3
 x   1 
 − + 1
 3 

The greatest terms in the expansion are T3 and T4


SOLVED EXAMPLES ∴  Greatest term (when r = 2) = 311 | T2 + 1 |
2
 1 11⋅10 1
38. The greatest term (numerically) in the expansion of
=3 11
11
C2 −  = 311 × = 55 × 39
 3 
3 1⋅ 2 9
(2 + 3x)9, when x = , is
2 and greatest term (when r = 3) = 311 | T3 + 1 |
5 311 5 313 3
(A) (B)   1 11⋅10 ⋅ 9
2 2 =3 11 C3 − 
11
= 311 ×−
1
= 55 × 39
 3 
7 313 1⋅ 2 ⋅ 3 27
(C) (D)  none of these
2 From above, we see that the values of both greatest
terms are equal.
Solution (C)
We have,
9 9
MIDDLE TERM IN THE BINOMIAL EXPANSION 
 3x   9  3 
(2 + 3x)9 = 29 1 +  = 29 1 +  ∵ x =  The middle term in the binomial expansion of (x + y)n
 2   4   2 
depends upon the value of n.
x ( n + 1) 1.
If n is even, then there is only one middle term i.e.
( x + 1)  n 
∴ m =  + 1 th term.
 2 
 9 
  (9 + 1)
 4  2.
If n is odd, then there are two middle terms, i.e.,
9 90 12  n + 1  n + 3 
=   + 1 = =6 ≠ Integer 
 2  
 th and 
 2  th terms.
4 13 13
The greatest term in the expansion is T[m] + 1 = T6 + 1 = T7
Hence, the greatest term = 29 ⋅ T7 I M P O R TA N T P O I N T S
6
9
= 29 ⋅ T6 + 1 = 29 ⋅ 9C6   When there are two middle terms in the expansion, their
 4  

binomial coefficients are equal.


6
9 9 × 8 × 7 312 13  Binomial coefficient of the middle term is the greatest
= 29 ⋅ 9C3   = 29 ⋅ ⋅ = 7´3
 4  1× 2 × 3 2 12
2
binomial coefficient.

39. The greatest term (numerically) in the expansion of


1 SOLVED EXAMPLE
(3 – 5x)11 when x = is
5 2n
 1
(A) 55 × 39 (B)  46 × 39 40. The greatest coefficient in the expansion of  x + 
 x
(C) 55 × 3 6
(D)  none of these is
6.10  Chapter 6

1⋅ 3 ⋅ 5...( 2n −1) ⋅ 2n 2n ! Properties of nCr


(A) (B) 2
n! ( n!) If 0 < r < n, n, r ∈ N, then
n! 1. r ⋅ nCr = n ⋅ n – 1Cr – 1
2
 n  
(C)   !
n n +1
(D)  none of these Cr Cr +1
 2   2. =
r +1 n +1
Solution (A, B)
n
Since the middle term has greatest coefficient, 3.
n
Cr = n–1
Cr – 1
∴ greatest coefficient = coefficient of the middle term r
( 2n)! n
Cr n − r +1
= 2nCn =
n! n! 4. n =
Cr 1 r
2n ( 2n −1) ( 2n − 2) ( 2n − 3)...4 ⋅ 3 ⋅ 2 ⋅1
= n
Cr r +1
n! n! 5. n = 6.  nCr – 1 + nCr = n + 1Cr
= Cr +1 n − r
[( 2n −1) ( 2n − 3)...3 ⋅1] [2n ( 2n − 2) ( 2n − 4)...4 ⋅ 2]
n! n! 7. n
Cx = nCy ⇒ x = y or x + y = n
8. Cr = nCn – r.
n
[1⋅ 3 ⋅ 5...( 2n −1)] 2n [n ( n −1) ( n − 2)...2 ⋅1]
= n / 2 if n is even
n! n! 
9. Cr is greatest if r =  n − 1 n + 1
n

1⋅ 3 ⋅ 5...( 2n −1) 2n n ! 1⋅ 3 ⋅ 5...( 2n −1) 2n  or if n is odd


= = .  2 2
n! n! n!

Thus, if n is even, the greatest coefficient is nCn/2 and
pth Term from the End in the Binomial
Expansion  of  (x + y)n if n is odd, the greatest coefficient is nC n 1 or nC n + 1,
2 2
both being equal.
pth term from the end in the expansion of (x + y)n is
(n – p + 2)th term from the beginning. 10. The greatest term in (1 + x)2n has the greatest coeffi-
Properties of Binomial Coefficients n n +1
cient if <x< .
n +1 n
In the binomial expansion of (1 + x)n, the coefficients nC0,
n
C1, nC2, ... nCn are denoted by C0, C1, C2, ... Cn respectively.
n
1. If n is even, then greatest coefficient = Cn / 2 QUICK TIPS
2.
If n is odd, then greatest coefficient is C(n – 1)/2 or n
1. The number of terms in the expansion of (x + y + z)n,
n
C(n + 1)/2.
( n + 1) ( n +2)
3.
C0 + C1 + C2 + ... + Cn = 2n where n is a positive integer, is .
2
4.
C0 + C2 + C4 + ... = C1 + C3 + C5 + ... = 2n – 1
2. The number of terms in the expansion of (x + y + z + w)n,
5.
C0 – C1 + C2 – C3 + C4 – ... + (– 1)n Cn = 0 ( n + 1) ( n + 2) ( n + 3)
( 2n)! where n is a positive integer, is .
6. C02 + C12 + C22 + ... + Cn2 = ( n !) 2 = 2nCn 6
n n n
3. Coefficient of x 1 y 2 z 3 in the expansion of (x + y + z)n is
2 2 2 2
7. C − C + C − C +...
0 1 2 3 n!
  
0, if n is odd n1 ! n2 ! n2 ! where n = n1 + n2 + n3
= 
( −1) n / 2 ⋅ n cn / 2 , if n is even 4. In the expansion of (x1 + x2 + ... + xk)n, the sum of all

the coefficients is obtained by putting all the variables xi
8. C0C1 + C1C2 + C2C3 + ... + Cn – 1 Cn = 2nCn – 1 equal to 1 and it is equal to kn.
9. C0Cr + C1Cr + 1 + ... Cn – rCn = 2nCn – r or 2nCn + r 5. Coefficient of xm in (1 + xr)n (m, r and n ∈ N) is zero, if
10. C1 + 2C2 + 3C3 + ... + n.Cn = n.2n–1 m is not an integral multiple of r, e.g., coefficient of x1000
11. C1– 2C2 + 3C3 – ... = 0 in the expansion of (1 + x3)4000 is 0 as 1000 is not an
integral multiple of  3.
12. Cn + 2C1 + 3C2 + ... + (n + 1)Cn = (n + 2)2n–1
Binomial Theorem  6.11

SOLVED EXAMPLES 43. If (1 + x)n = C0 + C1 x + C2 x2 + ... + Cn xn, then


ΣΣ (Ci + C j ) 2 =
C C C C (−1) n Cn 0 ≤i < j ≤ n
41. The value of 0 − 1 + 2 − 3 + ... +
1.3 2.3 3.3 4.3 ( n + 1) ⋅ 3 (A) (n – 1) ⋅ 2nCn + 22n (B)  n ⋅ 2nCn + 22n
is
(C) (n + 1) ⋅ Cn + 2
2n 2n
(D)  none of these
1 1
(A) (B)  Solution (A)
3( n + 1) n + 1
ΣΣ (Ci + C j )2   i  = 0, 1, 2, ..., (n – 1)
0≤ i ≤ j ≤ n
3
(C) (D)  none of these j  = 1, 2, 3, ..., n
n +1
and  i < j
Solution (A)
= n ( C02 + C12 + ... + Cn2 ) + 2 Σ Σ Ci Cj 0 ≤ i < j ≤ n
We have, (1 + x)n = C0 + C1x + C2x2 + ... + Cnxn
Integrating both sides w.r.t. x form –1 to 0, we get = n ⋅ 2nCn + [(C0 + C1 + ... + Cn)2
0 0 – ( C02 + C12 + ... + Cn2 )]
∫ (1 + x) dx = ∫ (C
n 2 n
0 + C1 x + C2 x + ... + Cn x )dx = n ⋅ 2nCn + (2n)2 – 2nCn = (n – 1) ⋅ 2nCn + 22n
−1 −1

C0 + C1 x + C2 x 2 + ... + Cn x n )dx 44. If (1 + x)n = C0 + C1x + ... + Cnxn, then the value of
n n
∑ ∑ (Cr + Cs ) is equal to
0 0 r=0 s=0
(1 + x ) n + 1  2 n +1 
cos x + C1 x + ... + Cn x 
=   (A) (n + 1)2n + 1 (B) (n + 1)2n
n +1  2 n + 1 
−1 −1 (C) n2n + 1 (C)  none of these
C1 C2 C 1 Solution (A)
⇒  C0 − + +, ..., + (−1) n n =
2 3 n +1 n +1 n n n n n n

1  C1 C2  ∑∑ (Cr + Cs ) = ∑∑ Cr + ∑ ∑C s
∴  The given expression = C0 − + − ... r=0 s=0 r=0 s=0 r=0 s=0
3 2 3 
n  n  n  n 
1 1 1 = ∑∑ 
 Cr  +  C 
 ∑∑ s
= ⋅ =  r = 0  s = 0 
3 n + 1 3( n + 1) s=0 r=0

n n
42. If (1 + x)n = C0 + C1 x + C2 x2 + ... + Cn xn , then = ∑2
s=0
n
+ ∑2
r=0
n

C0C2 + C1C3 + C2C4 + ... + Cn – 2 Cn =


= (n + 1)2n + (n + 1)2n
( 2n)! ( 2n)!
(A) (B)  = (n + 1)2n + 1
( n !) 2 ( n −1)!( n + 1)!
( 2n)! 45. If (1 + x)n = C0 + C1 + ... + Cnxn, then the value of
(C) (D)  none of these
( n − 2)!( n + 2)! ∑ ∑ (Cr + Cs ) is equal to
0≤r<s≤n
Solution (C)
(A) n2n (B) 2n + 1
We have, (C) (n – 1)2n (D)  none of these
(1 + x)n = C0 + C1 x + C2 x2 + C3 x3 + C4 x4 + ...
+ Cn – 2 xn – 2 + Cn – 1 xn – 1 + Cn xn ...(1) Solution (A)
n n
and (x + 1)n = C0 xn + C1 xn – 1 + C2 xn – 2 + C3 xn – 3
+ C4 xn – 4 + ... + Cn – 2 x2 + Cn – 1 x + Cn ...(2)   ∑ ∑ (C
r=0 s=0
r + Cs )
Multiplying Eq. (1) and (2) and equating the coeffi- n
cients of xn – 2, we get = ∑ (C r + Cr ) + 2 ∑ ∑ (C r + Cs )
C0C2 + C1C3 + C2C4 + ... + Cn – 2 Cn r=0 0≤r< s≤n

= the coefficient of xn – 2 in (1 + x)2n  n 



= 2nCn – 2 =
( 2n)! ⇒ (n + 1)2n + 1 = 2  Cr  + 2
r = 0  0≤r< s≤n

(Cr + Cs ) ∑∑
( n − 2)! ( n + 2)!
6.12  Chapter 6

n
⇒ (n + 1)2n + 1 = 2 ⋅ 2 + 2 ∑ ∑ (C r + Cs ) 48. The coefficient of x in the expansion of [ 1 + x 2 – x]–1
0≤r< s≤n in ascending powers of x, when | x | < 1, is
⇒  ∑ ∑ (C r + Cs ) = n ⋅ 2n (A) 0 (B) 
1
0≤r< s≤n 2
1
10 (C) – (D) 1
46. The sum of the series ∑ 20
Cr is 2
r= 0

1 20 1 20 Solution (D)
(A) 219 – ⋅ C10 (B) 219 + ⋅ C10 We have,
2 2
1 ( 1 + x 2 + x)
(C) 219 (D) 220 ( 1 + x – x) =
2 –1 ×
( 1 + x 2 − x) ( 1 + x 2 + x)
Solution (B)
10 (1 + x 2 )1/ 2 + x
 = = (1 + x2)1/2 + x
We have, ∑
r= 0
20
Cr = C0 + C1 + ... + C10
20 20 20
1+ x2 − x2
But 20C0 + 20C1 + ... 20C20 = 220 ∴  coefficient of x in the expansion of ( 1 + x 2 – x)–1
and    20C20 = 20C0, 20C19 = 20C1 = coefficient of x in the exp. of [(1 + x2)1/2 + x]
      20
C18 = 20C2, 20C11 = 20C9 = 1. ( coefficient of x in the exp. of 1 + x 2 is 0)
10
49. If the expansion of (1 + x)50, the sum of coefficients of
∴ ∑
r= 0
20
Cr = (20C0 + 20C1 + ... + 20C20) add powers of x is
– (20C11 + 20C12 + ... + 20C20) (A) 250 (B) 249
= 220 + 20C10 – (20C10 + 20C9 + ... + 20C0) (C) 0 (D)  none of these
⇒ 2 ( C0 + C1 + ... + 20C10) = 220 + 20C10
20 20 Solution (B)
1 The sum of coefficients of odd powers of x
∴   20C0 + 20C1 + ... + 20C10 = 219 +  20C10
2 = 50C1 + 50C3 + ... + 50C49
= 250 – 1 = 249
47. n+1
C2 + 2 [2C2 + 3C2 + 4C2 + ... + nC2] = 50. The value of the sum of the series
n ( n + 1) ( 2n + 1) n ( n +1) 14
C0 ⋅ 15C1 + 14C1 ⋅ 15C2 + 14C2 ⋅ 15C3 + ... + 14C14 ⋅ 15C15 is
(A) (B) 
6 2
(A) 29C12 (B) 
29
C10
n ( n -1) ( 2n -1) (C) 29C14 (D) 
29
C16
(C) (D)  none of these
6 Solution (C)
Solution (A) We have,
We have, (1 + x)14 = 14C0 + 14C1 x + 14C2 x2
  n + 1
C2 + 2 [2C2 + 3C2 + 4C2 + ... + nC2] + ... + 14C14 x14 ...(1)
= C2 + 2 [3C3 + 3C2 + 4C2 + ... + nC2]
n+1
and (x + 1)15 = 15C0 x15 + 15C1 x14 + 15C2 x13
= n + 1C2 + 2 [4C3 + 4C2 + ... + nC2] + 15C3 x12 + ... + 15C15 ...(2)
= n + 1C2 + 2 [5C3 + ... + nC2] Multiplying Eq. (1) and (2) and equating the coeffi-
= n + 1C2 + 2 ⋅ n + 1C3 cient of x14, we get
= n + 1C2 + n + 1C3 + n + 1C3 14
C0 ⋅ 15C1 + 14C1 ⋅ 15C2 + 14C2 ⋅ 15C3 + ... + 14C14 ⋅ 15C15
= n + 2C3 + n + 1C3
= the coefficient of x14 in (1 + x)29 = 29C14
n ( n + 1) ( n + 2) n ( n + 1) ( n −1)
= + 51. If (1 + x)n = C0 + C1x + ... + Cnxn, then the value of
6 6 n n

n ( n + 1) ( 2n + 1) ∑ ∑ Cr Cs is equal to
= r=0s=0
6 (A) 2n (B) 22n
(C) 24n (D)  none of these
Binomial Theorem  6.13

Solution (B)
n n n  n 
⇒ 22n =
2n
Cn + 2 ∑ ∑C r Cs
Cs  =
n
C
∑ ∑
0≤r< s≤n
∑ ∑C C r s =  r  ∑ 2n ⋅ Cr
r=0 s=0 r=0 s=0 r=0
⇒ ∑ ∑C r Cs = 1 ( 22 n − 2 nC )
n
 n 
0≤r< s≤n 2
= 2n  Cr  = 2n ⋅ 2n = 22n.
∑ r = 0  53. 79 + 97 is divisible by
52. If (1 + x)n = C0 + C1x + ... + Cnxn, then the value of (A) 16 (B)  24
(C) 64 (D)  72
∑ ∑ Cr Cs is equal to
0≤r<s≤n
Solution (C)
1 1 We have,
(A) ( 22 n - 2 nCn ) (B)  ( 22 n - 2 nCn )
2 4 79 + 97 = (1 + 8)7 – (1 – 8)9
1 n 2n = (1 + 7C1 ⋅ 81 + 7C2 ⋅ 82 + ... + 7C7 ⋅ 87)
(C) ( 2 - Cn ) (D)  none of these
2 – (1 – 9C1 ⋅ 81 + 9C2 ⋅ 82 – ... 9C9 ⋅ 89)
Solution (A) = 16 × 8 + 64 [(7C2 + ... + 7C7 ⋅ 85)
n n n
– (9C2 – ... – 9C9 ⋅ 87)]
∑ ∑C C
r=0 s=0
r s = ∑C
r=0
2
r + ∑ ∑C C
0≤r< s≤n
r s
= 64 (an integer)
Hence, 79 + 97 is divisible by 64.
6.14  Chapter 6

NCERT EXEMPLARS
1. The total number of terms in the expansion of 5. If the coefficients of 2nd, 3rd and the 4th terms in the
( x + a)100 + ( x − a)100 after simplification is expansion of (1 + x)n are in AP, then the value of n is
(A) 50 (B) 202 (A) 2 (B) 7 (C) 11 (D) 14
(C)  51 (D)  None of these 6. If A and B are coefficient of xn in the expansions of
2. If the integers r > 1, n > 2 and coefficients of (3r)th and A
(1 + x)2n and (1 + x)2n – 1 respectively, then equals to
(r + 2)nd terms in the Binomial expansion of (1 + x)2n B
are equal, then 1 1
(A) n = 2r (B)  n = 3r (A) 1 (B) 2 (C)  (D) 
2 n
(C) n = 2r + 1 (D)  None of these
10
3. The two successive terms in the expansion of (1 + x)24 7
7. If the middle term of  + x sin x 
1
is equal to 7 ,
whose coefficients are in the ratio 1 : 4 are x  8
(A)  3rd and 4th (B)  4th and 5th then the value of x is
(C)  5th and 6th (D)  6th and 7th
π
4. The coefficient of xn in the expansion of (1 + x)2n and (A)  2nπ + π (B) 
nπ +
6 6
(1 + x)2n–1 are in the ratio
π π
nπ + ( −1)
n
(A)  1 : 2 (B)  1 : 3 (C)  nπ + ( −1)
n
(D) 
(C)  3 : 1 (D)  2 : 1 6 3

ANSWER K EYS
1. (C) 2. 
(A) 3. 
(C) 4. 
(D) 5. 
(B) 6.  (B) 7. 
(C)

HINTS AND EXPLANATIONS


2n
1. Here, (x + a)100 + (x – a)100
Given, C3r −1 = 2 n Cr +1
Total number of terms is 102 in the expansion of (x + a)100

∵ nC = nC ⇒ x + y = n
+ (x – a)100  x y 
50 terms of (x + a)100 cancel out 50 terms (x – a)100. 51 terms
NCERT EXEMPLARS

⇒ 3r – 1 + r + 1 = 2n

of (x + a)100
Get added to the 51 terms of (x – a)100. 4r
Alternate Method ⇒ 4 r = 2n ⇒ n =

2
   ( x + a) + ( x − a)
100 100
= 100C0 x100 +100 C1x 99 a + .... +100 C100 a100 ∴ n = 2r

100 100 100 99 100 100

+ C0 x − C1x a + .... + C100 a 3. Let two successive terms in the expansion of (1 + x)24 are
 100 100 100
=  C0 x + C2 x a + ... + C100 a  98 2 100 100  (r + 2)th terms.
  
51 terms ∴ Tr +1 = 24Cr x r

2. Given that, r > 1, n > 2 and the coefficients of (3r)th and and Tr + 2 = 24Cr +1x r +1

(r + 2)th term are equal in the expansion (1 + x)2n.
2n 3r −1
(24)!
Then, T3r = T3r −1+1 = C3r −1x
r !( 24 − r )! 1
⇒ =
(24)! 4
2n
and Tr + 2 = Tr +1+1 = Cr +1x

r +1
( )(
r + 1 ! 24 − r − 1) !
Binomial Theorem  6.15


(r + 1) r !(23 − r )! = 1 ⇒ 6 n − 6 = 6 + n2 − 3n + 2
r !( 24 − r ) ( 23 − r )! 4 2
⇒ n − 9n + 14 = 0
r +1 1 ⇒ n2 − 7n − 2n + 14 = 0
⇒ = ⇒ 4 r + 4 = 24 − r
24 −r 4
⇒ n ( n − 7) − 2 ( n − 7) = 0
⇒ 5r = 20 ⇒ r = 4
∴ T = T and T4 + 2 = T6 ⇒ ( n − 7) ( n − 2) = 0
4 +1 5
∴ n = 2 or n = 7
Hence, 5th and 6th terms.
Since, n = 2 is not possible.
4. ∵ Coefficient of xn in the expansion of (1 + x )2 n = 2 nCn
∴ n=7
And coefficient of xn in the expansion of (1 + x )2 n−1 = 2 n−1Cn
6. Since, the coefficient of xn in the expansion (1 + x)2n is 2nCn.
∴ A = 2nCn
( 2 n) !
2n
Cn Now, the coefficient of xn in the expansion of (1 + x)2n – 1 is
∵ = n! n! 2n – 1
Cn.
(2n − 1)!
2 n−1
Cn
∴ B = 2n – 1Cn
n ! n!( n − 1)!
2n
A Cn 2
(2n)!ni (n − 1)!
Now, = = =2
= n!n!(2n − 1)! B 2 n −1
Cn 1

2n ( 2n − 1)! n!( n − 1)!


= 1 10
n! n ( n − 1)!( 2n − 1) 7. Given expansion is  + x sin x  .
x 
2n 2
= = = 2 :1  1 10 −5
( x sin x )
10 5
n 1 ∴ T6 = T5+1 = C5  

x
5. The expansion of
63 10
⇒ = C5 x –5 x 5 sin 5 x
(1 + x ) is nC0 + nC1x + nC2 x 2 + nC3 x 3 + ...... + nCn x n
n

8
∴ Coefficient of 2nd term = n C1,
63 10.9.8.7.6.5! 5
⇒ = sin x

HINTS AND EXPLANATIONS



Coefficient of 3rd term nC2, 8 5.4.3.2.1.5!

and coefficient of 4th term = nC3. ⇒ 63 = 3 ⋅ 2 ⋅ 7 ⋅ 6 sin 5 x
8
Given that, n C1, nC2 and nC3 are in AP .

n r n
⇒ sin 5 x = 1
∴ 2 C2 = C1 + C3 32
 ( n)!  ( n) ! + ( n)! 5
⇒ 2 = ⇒ sin 5 x =  1 
 ( n − 2) 2! (n − 1)! 3!( n − 3)! 2
2.n ( n − 1) ( n − 2)! n ( n − 1)! n ( n − 1) ( n − 2) ( n − 3)! 1
⇒ = + ⇒ sin x =
( n − 2) ! 2 ! (n − 1)! 3.2.1( n − 3)! 2
∴ x = nπ + ( −1) π / 6
n
n ( n − 1) ( n − 2)
⇒ n ( n − 1) = n +
6
6.16  Chapter 6

PRACTICE EXERCISES

Single Option Correct Type

1. The coefficient of x17 in the expansion of 8. The interval in which x must lie so that the numeri-
(x – 1) (x – 2) (x – 3) ... (x – 18) is cally greatest term in the expansion of (1 – x)21 has the
greatest coefficient is, (x > 0).
171
(A) (B) 
342 5 6  5 6 
2 (A)  ,  (B)   , 
6 5  6 5 

(C)
– 171 (D) 
684
 4 5  4 5
(C)  ,  (D)   , 
24n 5 4  5 4 
2. The fractional part of 15 is 9. If Cr stands for nCr, then the sum of the series
 n  n
2 1 2  !  !
(A) (B)   2   2  ... + (− 1) n ( n + 1) Cn2 ],
15 15 [C02 − 2C12 + 3C22 −
n!
4
(C) (D)  none of these
... + (− 1) n ( n + 1) Cn2 ], where n is an even positive integer, is
15
(A) 0 (B)  (–1)n/2 (n + 1)
 2 
2003 (C) (–1) (n + 2)
n/2
(D)  (– 1)n n
3. If {x} denotes the fractional part of x, then   is
 17  2n
(A) 2/17 (B)  4/17  10. If (1 + 2x + x2)n = ∑ ar x r , then ar =
r =0
(C) 8/17 (D)  16/17
(A) (nCr)2 (B)  n
Cr ⋅ nCr + 1
(C) Cr (D) 
2n 2n
Cr + 1
4. The sum of the coefficients of all the integral powers of
 n
1 + 4 x + 1  1 − 4 x + 1  
n
x in the expansion of (1 + 2 x ) is
80
1
11. If 
  −   
1 80 1 80 4 x + 1  2   2  
(A) (3 + 1) (B)  (3 - 1) 
 
2 2 = a0 + a1x + ... + a5x , then n equals
5

(C) (380 + 1) (D)  (380 – 1) (A) 11 (B)  9


(C) 10 (D)  none of these
PRACTICE EXERCISES

5. If [x] denotes the greatest integer less than or equal to


m 10 20 
 p
x, then [(6 6 + 14) 2 n+1 ]
12. The sum ∑    , (where   = 0 if p < q) is
(A) is an even integer (B)  is an odd integer  i m − i  q 
i = 0
(C) depends on n (D)  none of these maximum when m is
6. The two consecutive terms in the expansion of (A) 5 (B)  10
(3x + 2)74, whose coefficients are equal, are (C) 15 (D)  20
(A) 20th and 21st (B)  30th and 31st 13. The number of distinct terms in the expansion of
(C) 40th and 41st (D)  none of these n
 3 
n  x + 1 + 1  ; x ∈ R+ and n ∈ N is
 x 1  T  x 3 
7. If in the expansion of 2 + x  , T = 7 and the sum
3

 4  2 (A) 2n (B)  3n
of the coefficients of 2nd and 3rd terms is 36, then the (C) 2n + 1 (C)  3n + 1
value of x is
1 14. The number of terms with integral coefficients in the
1
(A) - (B)  - expansion of (171/3 + 351/2x)600 is
3 2
(A) 100 (B) 50
1 1
(C) (D)  (C) 150 (D) 101
3 2
Binomial Theorem  6.17

5 5 24. The interval in which x (> 0) must be so that the great-


 3 i   3 i 
  est term in the expansion of (1 + x)2n has the greatest
15. If z =  2 + 2  +  2 – 2  , then
    coefficient is
(A) Re (z) = 0 (B)  Im (z) = 0  n − 1 n   n n + 1
(A)  ,  
(B)  n + 1 , n 

(C) Re (z) > 0, Im (z) > 0 (D)  Re (z) > 0, Im (z) < 0  n n − 1
16. The greatest value of the term independent of x in the  n n + 2 
expansion of (x sinα + x–1 cosα)10, α ∈ R, is (C)  ,  (D)  none of these
n + 2 n 
10 !
(A) 10 ! (B)  25. If n is positive integer and k is a positive integer not
2 5 ( 5!) 2
exceeding n, then
1 10 ! n  
2
(C) (D)  none of these 3  Ck 
25 (5!) 2 ∑k =1
k 
k −1

 C  , where Ck = Ck, is
n

17. If coefficient of xn in (1 + x)101 (1 – x + x2)100 is non- 2


zero, then n can not be of the form (A) n( n + 1) ( n + 2) (B)  n( n + 1) ( n + 2)
12 12
(A) 3t + 1 (B)  3t
(C) 3t + 2 (D)  4t + 1 n( n + 1) 2 ( n + 2)
(C) (D)  none of these
6
18. The sum of the last ten coefficients in the expansion of 6
(1 + x)19 when expanded in ascending powers of x is  1 
 1/12 

26. If the fourth term in the expansion of  x log x +1 + x
(A) 218 (B)  219  

(C) 218 – 19C10 (D)  1  (219 – 1) is equal to 200 and x > 1, then x is equal to
2 (A) 10 2 (B)  10
19. The number of integral terms in the expansion of (C) 104 (D)  none of these
( 2 5 + 6 7 )642 is 27. The coefficient of λnµn in the expansion of [(1 + λ) (1 + µ)
(A) 105 (B)  107 (λ + µ)]n is
n n
(C) 321 (D)  108
(A) ∑ Cr2 (B) Cr2+ 2 ∑
20. The number of positive terms in the sequence r=0 r=0

( n + 3) n n
195 P3

xn =  n
Pn
− ( n + 1)
Pn+1
is (C) ∑C 2
r +3 (D) Cr3 ∑

PRACTICE EXERCISES
r=0 r=0


(A)
14 (B) 
11 n−3
 1

(C)
12 (D) 
13 28. If there is a term containing x2r in  x + 2  , then
 x 
21. The digit at unit’s place in the number 171995 + 111995 (A) n –2r is a positive integral multiple of 3
–71995 is (B) n – 2r is even
(A) 0 (B)  1 (C) n – 2r is odd
(C) 2 (D)  3 (D) none of these
22. The positive integer which is just greater than 29. If Pn denotes the product of the binomial coefficients
(1 + 0.0001)1000 is Pn+1
in the expansion of (1 + x)n, then equals
(A) 3 (B)  4 Pn
(C) 5 (D)  2
( n + 1) n nn
23. The coefficient of xn in the polynomial (x + nC0 ) (x + 3 (A)  (B) 
n! n!
n
C1) (x + 5 nC2)... (x + (2n + 1) nCn) is
(A) n.2n (B)  n.2n+1 ( n + 1) n ( n + 1) n+1
(C)  (C) 
(C) (n +1).2 (D) n.2n–1
n
( n +1)! ( n +1)!
6.18  Chapter 6

30. The coefficient of the term independent of x in the 38. The value of the sum of the series 3nC0 – 8nC1 +
 x +1 x − 1 
10 13n C2 – 18nC3 + ... upto (n + 1) terms is
expansion of  2 / 3 − 
 is (A) 0 (B)  3n
 x − x1/ 3 + 1 x − x1/ 2 
(B) 5 n
(D)  none of these
(A) 210 (B)  105
(C) 70 (D)  112 4
1 10 2 n 10 2 2 n 39. The value of 2(nC0) + 3  (nC1) +  (nC2) + 5  (nC3)...is
31. The value of n − n C2 + n C2 2 3 4
81 81 81
3 2n 2n (1 − n) − 1 2n ( n + 3) − 1
10 10 (A) (B) 
− n 2 nC3 + ... + n is n +1 n +1
81 81
(A) 2 (B)  0 2n − 1 2n + 2
(C) (D) 
(C) 1/2 (D)  1 n +1 n −1
32. If n is an even integer and a, b, c are distinct, the num-
ber of distinct terms in the expansion of (a + b + c)n 40. Which of the following expansions will have term
+ (a + b – c)n is containing x3 ?
3 25  3 1 24
 −1  x 5 + 2 x− 5 
 5
2
 n
2
 n + 1 (A)  x 5
+ 2 x 
 (B)  

(A)   (B)  
 2   2    
23 22
 3 − 
1  3 1
 5 5  x 5 + 2 x− 5 
2 2
 n + 2   n + 3  (C)  x − 2 x  (D)  
(C)    
 2 
 (D)    
 2 

33. Coefficient of t 24 in (1 + t 2)12 (1 + t 12) (1 + t 24) is 41. The coefficient of x7 in the expansion of (1 – x – x2 +
x3)6 is
(A) 12C6 + 3 (B)  12C6 + 1
(C) C6 (D) 
12 12
C6 + 2 (A) 132 (B)  144
(C) –132 (D)  –144
34. (mC0 + mC1 – mC2 – mC3) + (mC4 + mC5 – mC6 – mC7) + ...
If n is a positive integer, then ( 3 + 1) − ( 3 − 1)
2n 2n
= 0 if and only if for some positive integer k, m = 42. is
(A) 4k (B)  4k + 1 (A) an irrational number
(C) 4k – 1 (D)  4k + 2 (B) an odd positive integer
(C) an even positive integer
35. If the sum of the coefficients in the expansions of (D) a rational number other than positive integers
(1 + 2x)m and (2 + x)n are respectively 6561 and 243,
PRACTICE EXERCISES

then the position of the point (m, n) with respect to the 43. If ai(i = 0, 1, 2, ..., 16) be real constants such that for
circle x2 + y2 – 4x – 6y – 32 = 0 every real value of x, (1 + x + x2)8 = a0 + a1x2 + a2x2 ...
+ a16x16, then a5 is equal to
(A) is inside the circle
(B) is outside the circle (A) 502 (B)  504
(C) is on the circle (C) 506 (D)  508
n
(D) can not be fixed
44. Statement-1:  ∑ (r + 1) n
Cr = ( n + 2)2n − 1
36. Let n(> 1) be a positive integer. Then largest integer m r=0

such that (nm + 1) divides 1 + n + n2 + ... + n255 is n

(A) 128 (B)  63 Statement-2: ∑ (r + 1)


r=0
n
Cr x r = (1 + x ) n + nx(1 + x ) n − 1
(C) 64 (D)  32
n

37. The coefficient of xn in the expansion( r(2x∑


=0 2
+ 1)+nC3)r xn r –= (1 + x ) n + nx(1 + x ) n − 1
(2x + 3)n–1 (5 – 2x) + (2x + 3)n–2 (5 –r 2x) + ... + (– 1)n (A) Statement-1 is false, Statement-2 is true
(5 – 2x) is
n
(B) Statement-1 is true, Statement-2 is true, Statement-2
1 n is a correct explanation for Statement-1
(A) 2 (B)  (n + 1)2n (C) Statement-1 is true, Statement-2 is true; Statement-2
8
is not a correct explanation for Statement-1
(C) (n + 1)2n–3 (D)  – (n + 1)2n–2 (D) Statement-1 is true, Statement-2 is false
Binomial Theorem  6.19

 1 52. Given positive integers r > 1, n > 2 and the coeffi-


45. In a binomial distribution B n, p =  , if the prob- cients of (3r)th term and (r + 2)th term in the binomial
 4
expansion of (1 + x)2n are equal, then r =
ability of at least one success is greater than equal to
9
, then n is greater than (A) n , n even (B) 
n
10 2 2
1 1
(A)   4 3 (B)  (C) n (D)  1
log10 - log10 log10 + log10 3
4

53. Let n be a positive integer such that


9 4
(C) ( D)  (1 + x + x2)n = a0 + a1x + a2x2 + ... + a2nx2n, then ar =
log10 4 - log10 3 log10 4 - log10 3
(A) an–r, 0 ≤ r ≤ 2n (B)  a2n, 0 ≤ r ≤ 2n
46. The remainder left out when 82n – (62)2n+1 is divided by (C) a2n – r, 0 ≤ r ≤ 2n (D)  none of these
9 is 54. If Cr stands for nCr, then the sum of the series
(A) 0 (B)  2
 n  n
(C) 7 (D)  8 2  !  !
 2   2 
[C02 − 2C12 + 3C22 −
47. If C0, C1, C2, ..., Cn denote the binomial coefficients in n!
the expansion of (1 + x)n, then
... + (− 1) n ( n + 1) Cn2 ], where n is an even positive
n
r n 1 + r log e 10 integer, is
∑ (−1) ⋅ Cr ⋅ =
r=0(1 + log e 10 n ) r (A) 0 (B)  (–1)n/2 (n + 1)
(A) 2 (B)  1 (C) (–1) (n + 2)
n/2
(D)  (– 1)n n
(C) 0 (D)  none of these 55. The sum of the series
48. If C0, C1, C2, ..., Cn are the coefficients of the expansion 1 1.4.1 1.4.7 1
1+ 2 + + + ... is
3 1.2.34 1.2.3 36
Ck n
of (1 + x)n, then the value of ∑
is 1
0 k +1 3  3 3
(A)  
(B) 
2 n -1 2  2 
(A) 0 (B) 
n 1
2n+1 −1 1  1 3
(C) (D)  none of these  
(C) (D) 
n +1 3  3 

49. The greatest coefficient in the expansion of 56. The digit at unit’s place in the number 171995 + 111995

PRACTICE EXERCISES
(x + y + z + w)15 is –71995 is
(A) 0 (B)  1
15! 15!
(A) (B)  (C) 2 (D)  3
3!( 4 !)3 (3!)3 4 !
57. The coefficient of xn in the polynomial (x + nC0 ) (x + 3
(C) (D)  none of these
n
C1) (x + 5 nC2)... (x + (2n + 1) nCn) is
(A) n.2n (B)  n.2n+1
10
50. The sum of the series ∑ 20 Cr is (C) (n +1).2n (D) n.2n–1
r= 0
58. Let n(> 1) be a positive integer. Then, largest integer m
(A) 219 – 1 ⋅ 20C10 (B) 219 + 1 ⋅ 20C10 such that (nm + 1) divides 1 + n + n2 + ... + n255 is
2 2
(C) 219 (D)  220 (A) 128 (B)  63
(C) 64 (D)  32
51. n+1
C2 + 2 [2C2 + 3C2 + 4C2 + ... + nC2] =
59. The coefficient of λnµn in the expansion of
n ( n + 1) ( 2n + 1)
(A) (B)  n ( n +1) [(1 + λ) (1 + µ) (λ + µ)]n is
6 2 (A) n Cr2 (B) 
n
Cr2+2
n ( n -1) ( 2n -1)
(C) (D)  none of these n 3
6 (C) n Cr2+3 (D) Cr
6.20  Chapter 6

60. The sum to (n + 1) terms of the series (A) 1000C50 (B)  1001
C50
C C C C (C) C50
1002
(D)  none of these
0 − 1 + 2 − 3 +... is
2 3 4 5 69. The sum of the series
 r r r 
1 1 ∑ (−1) r ⋅ nCr  1 + 3 + 7 + 15 + ... to m terms
n
(A) (B)  2 r 2 r 3 r 4 r 
n ( n + 1) n+2 r =0
 2 2 2 
1 is 1 1
(C) (D)  none of these 1 1
n +1 mn
2mn
(A) 2 (B) 
61. Let R = (5 5 + 11)2n + 1 and f = R – [R] where [ ] m
2 1 n
2 1
denotes the greatest integer function. Then R f =
1
(A) 22n + 1 (B)  W24n + 1 1
(C) 2m (D)  none of these
(C) 4
2n + 1
(D)  none of these
2n 1
62. Let n and k be positive integers such that n ≥ k ( k +1) 70. If (1 + x)n = C0 + C1 x + C2 x2 + ... + Cn xn, then for n
2
. The number of solutions (x1, x2, ..., xk), x1 ≥ 1, x2 ≥ 2, even, C02 − C12 + C22 − ... + (−1) n Cn2 is equal to
..., xk ≥ k, all integers, satisfying x1 + x2 + ... + xk = n, is
(A)
0 (B) 
( 1) n / 2 nCn 2
(A) mCk – 1 (B)  m
Ck
(C) Ck + 1
m
(D)  none of these (C) n Cn 2 (D)  none of these
1
where m = (2n – k2 + k – 2) n
n
Ck
2 71. ∑ =
n
k =0 ( k + 1) ( k + 2)
63. ∑ n C sin rx cos ( n − r ) x =
r
r =0 2n+1 − n − 3 2n+ 2 − n − 3
(A) 2 sin (n – 1) x
n–1
(B) 2 sin nx
n (A) (B) 
( n + 1) ( n + 2) ( n + 1) ( n + 2)
(C) 2n – 1 sin nx (D)  none of these
2n+ 2 − n + 3
64. nCn + n + 1Cn + n + 2Cn + ... + n + kCn = (C) (D)  none of these
( n + 1) ( n + 2)
(A) n + k – 1Cn + 1 (B)  n+k
Cn + 1
(C) n+k+1
Cn + 1 (D)  none of these 72. For all n ∈ N, the integer just above ( 3 + 1)2n is
65. If Sn = 1 + q + q + q + ... + q and
2 3 n divisible by
2 n
(A) 2n + 1 (B)  2n + 1
     
S’n = 1 +  q + 1 +  q + 1 + ... +  q + 1 , q ≠ 1, then (C) 2 + 1 (D)  none of these
n + 1

PRACTICE EXERCISES

 2   2   2 
73. If C0, C1, C2, ..., Cn be the coefficients in the expansion
n+1
C1 + n + 1C2 ⋅ S1 + n + 1C3 ⋅ S2 + ... + n + 1Cn + 1 ⋅ Sn =
of (1 + x)n, then
(A) 2n – 1 ⋅ S’n (B)  2n ⋅ S’n
(C) 2 ⋅ S’n
n+1
(D)  none of these 22 ⋅ C0 23 ⋅ C1 2 n+ 2 ⋅ Cn
+ + ... + is equal to
1⋅ 2 2⋅3 ( n + 1) ( n + 2)
66. If (1 + x)15 = C0 + C1 x + C2 x2 + ... + C15 x15, then the
value of C2 + 2 C3 + 3 C4 + ... + 14 C15 is 3n+1 − 2n − 5 3n+ 2 − 2n − 5
(A) 219923 (B) 16789 (A) (B) 
( n + 1) ( n + 2) ( n + 1) ( n + 2)
(C) 219982 (D)  none of these
n+ 2
67. If a0, a1, a2, ..., a2n be the coefficients in the expansion (C) 3 + 2n − 5 (D)  none of these
of (1 + x + x2)n in ascending powers of x, then ( n + 1) ( n + 2)
a02 − a12 + a22 − a32 + ... − a22n−1 + a22n = 74. mCr + mCr – 1 ⋅ nC1 + mCr – 2 ⋅ nC2 + ... + mC1 ⋅ nCr – 1 + nCr =
(A) a2n (B)  an (A) m + nCr – 1 (B)  m+n
Cr
(C) a0 (D)  none of these (C) Cr + 1
m+n
(D)  none of these

68. The coefficient of x50 in the expression 75. If a, b, c and d are any four consecutive coefficients of
(1 + x)1000 + 2x (1 + x)999 + 3x2 (1 + x)998 + ... + 1001 x1000 a+b b+c c+d
any binomial expansion, then , , are
is in a b c
Binomial Theorem  6.21

(A) A.P. (B)  G.P. 1 1


(C) H.P. (D)  none of these (A) (B) 
3 5
76. The last two digits of the number 3400 are 1
(C) (D)  none of these
(A) 38 (B)  27 6
(C) 01 (D)  none of these 82. The coefficient of xn in the expansion of (x + C0)
(x –3C1) (x + 5C2)..... up to (n + 1) terms, where Cr =
C0 C1 C C
77. The sum − + 2 − 3 + ... to (n + 1) terms is n
Cr is equal to
1.2 2.3 3.4 4.5
(A) 0 (B)  1
1 2n (C) –1 (D)  none of these
(A) (B) 
( n + 2) ( n + 2)
83. The number of irrational terms in the expansion of
2 n −1
(C) (D)  none of these
( )
100
( n + 2) 8 5 + 6 2 is

78. The sum of the series (A) 96 (B)  97


(1.2) C2 + (2.3) C3 +......+ (n – 1.n) Cn is (C) 98 (D)  none of these
(A) n (n –1)2n –1 (B)  n (n –1)2n –2 84. Let n be an odd natural number greater than 1. Then,
(C) n (n –1)2 n
(D)  none of these the number of zeros at the end of the sum 99n + 1 is
(A) 2 (B)  3
79. If n is an even positive integer and k = 3n , then (C) 4 (D)  none of these
k 2
∑ (−3) r −1 3nC2 r −1 = n 1
r =1 85. ∑ =
(A) 1 (B)  –1 r=0 ( 2r )!( 2n − 2r )!
(C) 0 (D)  none of these
22n 22 n-1

(A) (B) 
80. The coefficient of x301 in the expansion of ( 2n)! ( 2n)!
(1 + x)500 + x (1 + x)499 + x2 (1 + x)498 +.... .+ x500 is
22 n + 1
(A) 501C301 (B) 500C301
(C) (D)  none of these
( 2n)!
(C) 501C300 (D) none of these
86. The coefficient of xn in polynomial
( 6)
2n
(x + 2n + 1C0) (x + 2n +1C1)(x + 2n +1C2)....(x + 2n + 1Cn) is
81. The fractional part of , n ∈ N is equal to

PRACTICE EXERCISES
5 (A) 22n + 1 (B)  22n
(C) 2 2n – 1
(D)  none of these

Previous Year’s Questions

87. The coefficient of x5 in (1 + 2x + 3x2 + ...)−3/2 is: 90. The coefficient of the middle term in the binomial
(A)
21 (B)  25[2002] expansion in powers of x of (1 + αx)4 and of (1–αx)6 is
the same if α equals [2004]
(C) 26 (D)  none of these
88. If | x | < 1, then the coefficient of xn in expansion of (1 5 3
(A) −
(B) 
+ x + x2 + x3 + ...)2 is : [2002] 3 5
(A) n (B)  n−1 10
3
(C) n + 2 (D)  n + 1
(C) (D) 
10 3
89. The number of integral terms in the expansion of
( 3 8 5) 256 is [2003] 91. The coefficient of xn in expansion of (1 + x) (1 − x)n is
(A)
32 (B)  33 (A) (n – 1) (B)  (– 1)n (1 − n) [2004]
(C)
34 (D)  35 (C) (–1) (n– 1) (D) (–1) n
n–1 2 n–1
6.22  Chapter 6

92. If the coefficients of rth, (r + 1)th and (r + 2)th terms in 98. In the binomial expansion of (a - b)n, n ≥ 5, the sum
the binomial expansion of (1 + y)m are in A. P., then m
a
and r satisfy the equation [2005] of 5th and 6th terms is zero, then
equals [2007]
b
(A) m2 − m(4r − 1) + 4r2 − 2 = 0 5 6
(B) m2 − m(4r + 1) + 4r2 + 2 = 0 (A) (B)
(C) m2 − m(4r + 1) + 4r2 − 2 = 0 n 4 n 5
(D) m2 − m(4r − 1) + 4r2 + 2 = 0
(C) n 5 (D) n 4
6 6 5
93. The value of 50C4 56 r
C3 is [2005]
r 1
99. The sum of the series [2007]
(A) 55C4 (B) 
55
C3 20
C0 − C1 + C2 − C3 + … − … + C10 is
20 20 20 20

(C) 56C3 (D) 


56
C4
(A) − 20C10 (B) 1 20
C10
2
11

94. If the coefficient of x7 in ax 2 1 equals the (C) 0 (D)  2 0


C10
bx
11
1 1
coefficient of x−7 in ax 1 , then a and b sat- 100. In a binomial distribution B n, p , if the proba-
bx2 4
bility of at least one success is greater than or equal to
isfy the relation [2005]
9
(A) a − b = 1 (B) a + b = 1 , then n is greater than  [2008]
10
(B) a = 1 (D) ab = 1
b (A) 1 (B) 1
4 3 4 3
log10 log10 log10 + log10
95. If x is so small that x3 and higher powers of x may be
3 (C) 9 (D) 4
3/ 2 1 4
log10 3
log10 4
log10 3
log10
(1 x) 1 x
neglected, then 2 may be approxi-
101. The remainder left out when 82n − (62)2n +1 is divided
mated as (1 x )1/ 2 [2005]
by 9 is [2008]
3 2
(A) 1 − x (B) 3x + 3 x 2 (A) 0 (B) 2
8 8 (C) 7 (D) 8
PRACTICE EXERCISES

3 2 x 3 2 102. The coefficient of x7 in the expansion of the expression


(C) x (D) x (1 – x − x2 + x3)6 is [2011]
8 2 8
(A) –132 (B) –144
96. If the expansion in powers of x of the function (c)132 (D) 144
1 103. If n is a natural number, then ( 3 1) 2 n ( 3 1) 2 n is
is a0 + a1x + a2x2 + a3x3 + … , then an is
(1 ax )(1 bx ) (1) an irrational number [2012]
(2) an odd positive integer
bn an an bn (3) an even positive integer
(A) (B)  [2006]
b a b a (4) a rational number other than positive integers

a n+1 b n+1 104. If x = -1 and x = 2 are extreme points of


b n+1 a n+1
(C) (D) f ( x ) α log x β x 2 x , then [2013]
b a b a

1 1
97. For natural numbers m, n if (1 - y)m (1 + y)n = 1 + a1y (A) α 6, (B) α 6,
2 2
+ a2y2 + … , and a1 = a2 = 10, then (m, n) is
(A) (20, 45) (B)  (35, 20) [2006] 1 1
(C) α 2, (D) α 2,
(C) (45, 35) (D)  (35, 45) 2 2
Binomial Theorem  6.23

105. If the coefficients of x3 and x4 in the expansion of 108. The value of (21C1 – 10C1) + (21C2 – 10C2) + (21C3 – 10C3)
(1 ax bx 2 )(1 2 x )18, in powers of x, are both zero, + (21C4 – 10C4) + … + (21C10 – 10C10) is [2017]
then (a, b) is equal to [2014] (A) 221 - 210 (B) 
220 - 29
(C) 2 - 2 (D) 
20 10
221 - 211
251
(A) 16, (B) 14, 251 109. The sum of the coefficients of all odd degree terms in
3 3
the expansion of [2018]

(C) 14, 272 (D) 16, 272


3 3
(x + ) +(x − )
5 5
x3 − 1 x 3 − 1 , ( x > 1)

n
106. If X = 4 3n 1: n N and Y 9( n 1) : n N , (A) –1 (B) 0 (C) 1 (D) 2
where N is the set of natural numbers, then the set 25

XY is equal to [2014] 110. If ∑{


r =0
50
Cr .50 − r C25− r } =K( 50
C25), then K is equal to :
(A) N (B) Y -X
 [2019]
(C) X (D) Y
(A) (25)2 (B)  224
107. The sum of the coefficients of integral powers of x in
(C) 2 –1 (D) 
25
225
( )
50
the binomial expansion of 1 2 x is: [2015]
111. The positive value of l for which the co-efficient of x2

(A) 1 (350 ) (B) 1 (350


10
1) in the expression x2  x + λ   [2019]
2 2  x2 

(A)  4 (B) 
(C) 1 (250 + 1) (D) 1 (350 + 1) 5
2 2

(C)  2 2 (D) 
3

ANSWER K EYS
Single Option Correct Type

PRACTICE EXERCISES
1. (C) 2.  (B) 3.  (C) 4. (A) 5. (A) 6.  (A) 7.  (A) 8. (B) 9. (C) 10. 
(C)
11.  (A) 12.  (C) 13.  (C) 14. 
(D) 15. 
(B) 16.  (C) 17.  (C) 18. 
(A) 19. 
(D) 20. 
(B)
21.  (B) 22.  (D) 23.  (C) 24. 
(B) 25. 
(B) 26.  (B) 27.  (D) 28. 
(A) 29. 
(A) 30. 
(A)
31.  (D) 32.  (C) 33.  (D) 34. 
(C) 35. 
(A) 36.  (A) 37.  (A) 38. 
(A) 39. 
(B) 40. 
(A)
41.  (D) 42.  (A) 43.  (B) 44. 
(B) 45. 
(A) 46.  (B) 47.  (C) 48. 
(C) 49. 
(A) 50. 
(B)
51.  (A) 52.  (A) 53.  (C) 54. 
(C) 55. 
(B) 56.  (B) 57.  (C) 58. 
(A) 59. 
(D) 60. 
(D)
61.  (C) 62.  (A) 63.  (C) 64. 
(C) 65. 
(B) 66.  (A) 67.  (B) 68. 
(C) 69. 
(B) 70. 
(B)
71.  (B) 72. (A) 73.  (B) 74. 
(B) 75. 
(C) 76.  (C) 77.  (A) 78. 
(B) 79. 
(C) 80. 
(A)
81.  (B) 82. 
(A) 83.  (B) 84. 
(A) 85. 
(B) 86.  (B)

Previous Years’ Questions


  87. (D) 88. (D) 89. (B) 90.  (C) 91. (B) 92. (C) 93. (D) 94. (D) 95.  (C) 96. (D)
  97. (D) 98. (D) 99. (B) 100.  (A) 101. (B) 102. (B) 103. (A) 104. (C) 105.  (D) 106. (D)
107. (D) 108. (C) 109. (D) 110. (D) 111. (A)
6.24  Chapter 6

HINTS AND EXPLANATIONS

Single Option Correct Type


1. Coefficient of x17 = – (1 + 2 + 3 + ... + 18) 5. Let l + f = (6 6 14) 2 n 1

18 2 n +1
=– (1 + 18) Assuming, f′ = (6 6 − 14)
...(1)
2
2 n +1

= – 9 × 19 = – 171 Now, I + f – f′ = (6 6
14) 2 n 1
– (6 6 − 14)

The correct option is (C)
⇒ I + f – f′ = 2 [ 2 n 1 C1 (6 6 ) 2 n 141
4n n
2 16 (1 + 15) n
2. We have, = =
2n 1
C3 ( 6 6 ) 2 n 2
(14)3 ...]
15 15 15
⇒ I + f – f′ = 2 (Integer) = even ...(2)
1 + nC115 + nC2 152 + ... + nCn 15n

= Now, 0 ≤ f < 1
15 Also, 0 ≤ f – f′ < 1
= 1 + 15 k , where k ∈ N ∴ 0 ≤ f – f′ < 0 ⇒ f – f′ = 0
15 Substituting respective values in (2), we get
1 I = even integer
= +k
15 The correct option is (A)
24n 1 6. Given, T4 = 200
∴  Fractional part of is .
15 15 3
 1 

The correct option is (B) ⇒ 6
C3  log x +1  ( x1/12 )3 = 200
 x 10 
 1 
3. 22003 = (24)500. 23 3 1  3
+  2 (log x +1) + 4 
⇒ 22003 = 8 (16)500 ⇒ 20. x 2(log10 x+1) 4
= 200 ⇒ x  
= 10
⇒ 22003 = 8 (17 – 1)500
3 1 1
⇒ 22003 = 8[(17)500 – 500C1(17)499 + ... ⇒ + = logx 10 =
2 (log10 x + 1) 4 log10 x
HINTS AND EXPLANATIONS

– 500C499 (17) + 1]
3 1 1
22003 8 ⇒ + = where y = log10x
⇒ = 8k , 2 ( y + 1) 4 y
17 17
⇒ y = – 4 or y = 1

where k = (17)499 – 500C1(17)498 + ... + 500C499
⇒ log10 x = – 4 or log10 x = 1

such that k is an integer
⇒ x = 10–4 or 10
 22003 

 8
∴   = ⇒ x = 10  (∴ x > 1)
 17 
 17 The correct option is (B)
The correct option is (C)
T3
4. The coefficients of the integral powers of x are 7. Since, = 7 (given)
T2
80
C0, 80C2 ⋅ 22, 80C4 ⋅ 24, ..., 80C80 ⋅ 280
Now, (1 + 2)80 = 80C0 + 80C1 ⋅ 2 + 80C2 ⋅ 22
n
C2 ( 2 x ) n 2 ( 4 x ) 2
⇒ =7
+ ... + 80C80 ⋅ 280  ...(1) n
C1 ( 2 x ) n 1 . ( 4 x )
and  (1 – 2)80 = 80C0 – 80C12 + 80C2 ⋅ 22
 n − 1 1
– ... + 80C80 ⋅ 280 ...(2) ⇒   . x 3 = 7 ...(1)
 2  (2 )
Adding Eq. (1) and (2), we get
380 + 1 = 2(80C0 + 80C2 ⋅ 22 + 80C4 ⋅ 24
Also, nC2 + nC1 = 36
+ ... + 80C80 ⋅ 280) ⇒ n( n − 1) + n = 36
2
∴ 80
C0 + 80C2 ⋅ 22 + 80C4 ⋅ 24 + ... + 80C80 ⋅ 280 = 1 (380 1) ⇒ n2 + n – 72 = 0
2

The correct option is (A) ⇒ n = 8, – 9
Binomial Theorem  6.25

n = – 9 is not possible as in Eq. (1), n – 1 should be positive. 11. Given:


Substituting n = 8 in Eq. (1), we get 

n n
1 1 + 4 x + 1  1 − 4 x + 1 
1 =2–1
  −   

23x = 4 x + 1  2   2  

2 
1
= a0 + a1x + a2x2 + ... + a5x5
⇒ 3x = – 1 ⇒  x = −
3 1
n 
4 x + 1  
The correct option is (A) −  −  

Now,   2 2  
8. If n is odd, then numerically greatest coefficient in the 
n n
Cn Cn  n−1
 n C  1 
1
expansion of (1 – x)n is 1
or . 1 4x + 1
2 2
=  1  2 
4 x + 1  2

Therefore in (1 – x)21, the numerically greatest coefficient is
21
C10 or 21C11. So, the numerically greatest term 3 
1
n−3  4 x + 1 

=  21
C11x11 or 21C10x10 and + C3  
n   + ...

 3   2  

|  21
C10x10 | > | 21C9.x9 |  
21! 21!
⇒ > x and
10 ! 11! 9 ! 12 ! =n
C1 1 n
C3
1
(4 x 1) n
C5
1 (4x + 1)2
2n 2n 2n
21! 21! r −1

x> (Q x > 0) 1
11! 10 ! 9 ! 12 ! + ... + nCr
( 4 x + 1) 2
+ ...
2n
6 5 5 6
⇒ x and x >   ⇒  x ∈  ,  The expansion contains a term x5 if r −1 = 5  or  r = 11.

5 6  6 5 
2

The correct option is (B)
The correct option is (A)
2 2 2
9. C − 2C + 3C − 4C + ... + (− 1) ( n + 1)C 2 n 2 m 10  20 
0
2
1
2 2
2
2 n
3
2
n
12. ∑  i  m − i
= [C0 − C1 + C2 − C3 + ... + (− 1) Cn ] i=0

= Coefficient of xm in (1 + x)10 (1 + x)20 = 30Cm



− [C12 − 2C22 + 3C32 − ... + (− 1) n nCn2 ]
m = 15 (for maximum value)

HINTS AND EXPLANATIONS


n! n
−1 n The correct option is (C)
= (− 1) n / 2 − (− 1) 2 n
Cn
 n   n  2  3 1   3
n
1 
n
 !  !
13.  x + 1 + 3  = 1 +  x + 3 
2
 2   2  
 x    x 
n !  n
= (− 1)
n/ 2
1 + 
n n 
n
 1   3 1
2 = n C0 + nC1  x 3 +  + ... + Cn  x + 3 
n
 !  ! 3
    x   x 
2 2

All the terms are distinct with powers (x3)0, (x3), (x3)2, ... (x3)
 n   n  n
, (x3)–n,... (x3)–1. Hence, (2n+1) terms.
 
∴  2  2 !  2 ! [C02 − 2C12 + 3C22 − ...
The correct option is (C)
n 2 n/ 2
n ! + (− 1) ( n + 1) C n ] = (− 1) ( n + 2 )
[C02 − 2C12 + 3C22 − ... 14. (d).  t = 600 600−r r
r
r+1
Cr 17 3
35 2 x
+ 1) Cn2 ] = (− 1) n / 2 ( n + 2)
+ (− 1) n ( n

The correct option is (C) r and r
As 0 ≤ r ≤ 600 and
200 − are integers  ⇒  r should
2n 2 3
10. We have, (1 + 2x + x2)n = ∑a x r
r
be a multiple of 6
2n
r =0 ∴  r = 0, 6, 12,... 600
⇒ (1 + x)2n = ∑a x
r =0
r
r
The correct option is (D)

 5 3
 3  i 4 
2n  3   3  i 2   
2n
r  

5
⇒ 
2n
Cr x r = ∑ a x ⇒ ar = 2nCr. 2
15. z =    + 5
C 2   + C 4  2  16 
r =0 r=0
r  2   2  4   


The correct option is (C)
= Purely real number
6.26  Chapter 6


Hence, Im (z) = 0
Thus, the units place digit is 1.

The correct option is (B)
The correct option is (B)

16. Term independent of x = 10C5 (sinα)5 (cosα)5 22. (1 + 0.0001)1000


1 1000 × 999 −8 1000

= 10
C5 sin5 2α
= 1 + 1000 × 10–4 + 10 + C3 10−12 + ...
25 2
1 10 ! 1 1 1 1 10

Hence, the greatest value = <1 + +
+ + ... = =
25 (5!) 2 10 100 1000 1 9
1

The correct option is (C) 10

So, the integer just greater than the given expression must be 2.
17. (1 + x)101 (1 – x + x2)100 = (1 + x) (1 + x3)100

The correct option is (D)
=  (1 + x) (C0 + C1x3 + C2x6 + ... + C100x300)
Clearly in this expression xλ will be present if λ = 3t, or 23. Given polynomial is
λ = 3t + 1 (x + nC0) (x + 3. nC1) (x + 5. nC2) ...(x + (2n + 1). nCn)
So, λ can not be of the form 3t + 2. =  xn+1 + xn [nC0 + 3. nC1 + 5. nC2 +
The correct option is (C) ... + (2n + 1). nCn] + xn–1 (...) + ...
18. Sum of last 10 coefficients, ∴  Coefficient of xn in the expression is
n n n
C10 + 19C11 + ... + 19C19 = S (say)
∑ ∑ ∑
19 n
( 2r + 1) Cr = 2r n Cr + n
Cr
Also, 19C0 + 19C1 + ... + 19C9 = 19C19 + 19C18 + ... + 19C10 r=0 r=0 r=0

= S  ∴  (nCn = nCn–r) n
n
19 =  2
∑r r
n−1
Cr −1 + 2n
∴ 2 S = ∑
n=0
19
Cn = 219 ⇒ S = 218
r=0


The correct option is (A) =  2n

r=0
n−1
Cr −1 + 2n

19. (2.51/2 + 71/6)642 has a general term of the form


= 2n.2n–1 + 2n = (n + 1) 2n
642
Cr (2.51/2)642–r (71/6)r

The correct option is (C)
=  642Cr 2642–r. 5321–r/2. 7r/6
and will be rational if only r is a multiple of 2 and 6. 24. Greatest coefficient in the expansion of (1 + x)2n is 2nCn. We
are given 2nCn xn is the greatest term.
HINTS AND EXPLANATIONS

∴ r must be the LCM of 2 and 6 which is 6.


∴ 2nCn–1 xn–1 < 2nCn xn
∴ r takes the values 0, 6, 12, 18,..., 642.
and 2nCn+1 xn+1 < 2nCn xn
There are 108 values.
2n 2n
The correct option is (D) Cn−1 Cn
⇒  2n
<x< 2n
195 ( n + 3) ( n + 2) ( n + 1) Cn Cn+1
20. xn = − >0
n! ( n + 1)! ( 2n)! n! n! ( 2n)! ( n + 1)! ( n − 1)!
⇒  <x<
⇒  195 > (n + 2) (n + 3). ( n − 1)! ( n + 1)! ( 2n)! n! n! ( 2n)!
Hence, n ≤ 11
⇒ 
n n +1
∴  n can take the values 1, 2, 3,..., 11. <x<
n +1 n
∴  Number of positive terms = 11

The correct option is (B)
The correct option is (B)
n
Ck C n − k +1
21. We have 25. We know that = n k =
  171995 + 111995 – 71995
Ck 1 Ck 1 k
2
=  (7 + 10)1995 + (1 + 10)1995 – 71995 n C  n
 n − k + 1
2

= [71995 + 1995C1. 71994. 101 + 1995C2. 71993. 102 + ∴ ∑ k 3  k  = ∑k 3  



k =1
 Ck −1  k = 1  k
... + 1995C1995. 101995] + [1995C0 + 1995C1. 101
n
+ 1995C2. 102 + ... + 1995C1995. 101995] – 71995
= [1995C1. 71994. 101 + ... + 101995]

= ∑ k (n − k + 1)
k =1
2

+ [1995C1. 101 + ... + 1995C1995. 101995] + 1 Put n – k + 1 = p ⇒ k = n – p + 1.



=  a multiple of 10 + 1.
When k = 1, p = n and when k = n, p = 1.
Binomial Theorem  6.27

n−3
n n
 1 
∴ Series = ∑ p =1
( n − p + 1) p 2 = ∑
p =1
( np 2 − p3 + p 2 ) 28. General Term in  x +
 
x 2 
is

k
n n  1 
tk+1 = n–3Ckxn–3–k 

= ∑ (n + 1) p − ∑ p
p =1
2

p =1
3
 x 2 

= (n + 1) (1 + 2 + 32 + ... + n2)
2 2 ⇒ tk+1 = n–3Ckxn–3(k+1)

– (13 + 23 + 33 + ... + n3) There is a term containing x2r, if
n – 3(k+1) = 2r
( n +1)n ( n + 1) ( 2n + 1) n2 ( n + 1) 2

= − ⇒ n – 2r = 3(k + 1), k ∈ N
6 4
∴ n – 2r is a positive integral multiple of 3.
n( n + 1) 2  2n + 1 n  The correct option is (A)

=  − 
2  3 2
29. Given: Pn = nC0 nC1 nC2... nCn
2
= n( n + 1) ( n + 2)
n 1 n 1 n 1
12 P C0 C1 C2 ...n 1 Cn 1
Now, n 1 =
n
The correct option is (B) Pn C0 nC1 nC2 ...n Cn
26. 6C3 a6–3 b3 = 200
 n+1 C   n+1 C 
66  1 
3
⇒ 
Pn 1
=
n +1
C0  n 1   n 2 

 log x +1  x 3/ 2 = 200 Pn  C0   C1 
3136  x 
 n+1 C 
⇒ 20

1
x1/ 4 = 200
... n n  n+1Cn+1
 Cn 
( x log x +1 )
3/ 2

1n n
= 10 = x
log
x10
Since,  n + 1
Cr
Cr + 1 =
⇒ x
3
+
1 1 r
2(log x +1) 4
P  n + 1  n + 1  n + 1
⇒  n 1 = 1  ...
 1   2   n 
3 1 1 1

+ = log x = 10 Pn
2(log x + 1) 4 log10 x

let log10 x = y Pn 1 ( n 1) n
∴  =

HINTS AND EXPLANATIONS


Pn n!
3 1 1

+ =
The correct option is (A)
2( y + 1) 4 y

y2 + 3y − 4 x +1 x −1

=0 ⇒ y = 1, −4 30. 2/3

( y + 4)( y − 1) x − x + 1 x − x1/ 2
1/ 3

⇒ log10 x = 1
and log10 x = – 4 ( x1/ 3 )3 + 13 x −1

= 2/3 1/ 3
− 1 / 2 1/ 2
⇒ x = 10 x = 10–4 x − x + 1 x ( x −1)
Not possible
x = 10 ( x1/ 3+ 1) ( x 2 / 3 − x1/ 3 + 1) x1/ 2 + 1

= −
x 2 / 3 − x1/ 3 + 1 x1/ 2
27. General term in (1 + λ)n (1 + µ)n (λ + µ)n is
= x1/3 + 1 – 1 – x–1/2 = x1/3 – x–1/2
10
tp, q, r = (nCpλp) (nCqµq) (nCrλn–rµr)  x +1 x − 1 
 −  = (x1/3 – x–1/2)10
⇒ tp, q, r = nCp nCq nCr λp+n–r µq+r ⇒   2 / 3
 x − x + 1 x − x1/ 2 
1/ 3

The term contains coefficient of λnµn if


p + n − r = n  and q + r = n Tr + 1 for (x1/3 – x–1/2)10 is 10Cr(x1/3)10–r (– 1)r (x –1/2)r
⇒ p = r  and q = n – r For term independent of x,
Now, tr, (n–r), r contains coefficient of λnµn 10 r r
 = 0 ⇒  20 – 2r – 3r = 0  ⇒  r  = 4
⇒ Coefficient of λnµn = nCr nCn–r nCr 3 2
∴ Coefficient of λnµn = (nCr)3

Hence, required coefficient = 10C4(– 1)4 = 210.
The correct option is (D)

The correct option is (A)
6.28  Chapter 6

1 35. Sum of coefficients in (1 + 2x)m = 3m = 6561 = 38


31. Given expression = ((1 10) 2 n ) = 1 ⇒ m = 8 Sum of coefficient on (2 + x)n = 3n = 243 = 35
(81) n ⇒n=5

The correct option is (D) Since S1 <  0, so the point lies inside the circle.
32. Let n = 2m, m ∈ N The correct option is (A)
∴ (a + b + c)n + (a + b – c)n = [(a + b) + c]2m 36. We have, S = 1 + n + n2 + ... + n255
+ [(a + b) – c]2m 1( n256 1) 128 n128 − 1
= 2[(a + b)2m + 2mC2(a + b)2m–2 c2 + ⇒  S = = ( n + 1)
n 1 n −1
... + 2mC2mc2m]
∴  S = (n128 + 1) (1 + n + n2 + ... + n127)
Therefore, the number of distinct terms in the expansion
Thus, the largest value of m for which 1 + n + n2
= (2m + 1) + (2m – 1) +
+ ... + n255 is divisible by nm + 1 is 128.
m + 1  

... + 3 + 1 =   . (2m + 1 + 1) The correct option is (A)
 2  37. The expansion is a G.P. with (n + 1) terms of the form
n   n + 2 
2 2
a n+1 − b n+1

= (m + 1) =  + 1 = 
2
 an + an–1b + an–2b2 + ... + bn =
 2   2  a−b

The correct option is (C) ( 2 x + 3) n + 1 − ( 2 x − 5) n+1

=  ,
33. Coefficient of t24 in (1 + t2)12 (1 + t12) (1 + t24) 8
=  coefficient of t24 in (1 + 12C6 t12 + 12C12 t24)
where a = 2x + 3 and b = 2x – 5
 (1 + t12 + t24) 1
∴  Coefficient of xn = [(n + 1). 2n(3) – (n + 1). 2n(– 5)]
=  coefficient of t24 in (12C6 + 2)t24 = 12C6 + 2 = (n + 1) ⋅ 2 n 8
The correct option is (D) The correct option is (B)
34. Consider 38. Let S denotes the sum of the series. General term of the
(cosθ – i sinθ)m = mC0 cosmθ – mC1cosm–1θ i sinθ series is given by,
+ ... + mCm (– i sinθ)m  ...(1) Tr = (– 1)r (3 + 5r) nCr, where r  =  0, 1, 2,..., n
(cosθ + i sinθ)m = mC0cosmθ + mC1 cosm–1θ i sinθ n

+ ... + mCm (i sin θ)m  ...(2) ∴  S = ∑ (− 1) r


(3 + 5r ) nCr
HINTS AND EXPLANATIONS

r=0
Adding (1) and (2), we get
2cos mθ = 2[mC0cosmθ – mC2cosm–2θ sin2θ...] ...(3) n n

Subtracting (1) from (2), we get ⇒  S = 3 ∑ (− 1)


r=0
r n
∑ (− 1)
Cr + 5
r=0
r
r nCr
2 i sin mθ = 2i [mC1cosm–1θsinθ – mC3cosm–3
⇒  S = 3(C0 – C1 + C2 – C3 + C4 ...)
θsin3θ...] ...(4)
Adding (3) and (4), we get
+ 5(– C1 + 2C2 – 3C3 + 4C4...)
cosmθ + sinmθ = [mC0 cosmθ + mC1 cosm–1θ sinθ ∴  S  =  0 + 0 = 0
– mC2cosm–2θ sin2θ – mC3cosm–3θ sin3θ...]
The correct option is (A)
 π
⇒  2 sin mθ +  = [mC0cosmθ + mC1cosm–1θ sinθ 39. (1 + x)n = nC0 + nC1x + nC2x2 + nC3x3 ... nCnxn
 4 On integrating between the limits 0 and, we get
– mC2cosm–2θ sin2θ – mC3cosm–3θ sin3θ ...]

p (1 + x ) n+1 − 1 n n
C1 x 2
n
C2 x 3 n
C3 x 4
Putting θ =  , we get
= C x+ + + ...
4 n +1 0
2 3 4

( m +1)π   1
Multiplying with x and differentiating, we get
2 sin   = [(mC0 + mC1 – mC2
 4  2m/ 2 d   (1 + x ) n+1 − 1
– C3) + ( C4 + C5 – C6 – C7) + ...
m m m m m  x  

dx   n +1 
+ (mCm–3 + mCm–2 – mCm–1 – mCm)]
 n 3 n 4 n 5 
Hence, m + 1 = 4k, for given quantity to be 0.

=
d  n C0 x 2 + C1 x + C2 x + C3 x ...
⇒  m = 4k – 1, where k ∈ N dx  2 3 5 
 
The correct option is (C)
Binomial Theorem  6.29

(1 + x ) n+1 + x( n + 1) (1+ x ) n − 1 n n
⇒ 
n +1
44. ∑ (r + 1)
r=0
n
Cr = ∑r n
Cr + nCr
r=0
n 2 n 3
3 C1 x 4 C2 x n
n n
= 2 n C0 x +

2
+
3
...
= ∑
r=0
r
r
n −1
Cr − 1 + ∑
r=0
n
Cr

put x = 1, we get

= n 2n – 1 + 2n = 2n – 1(n + 2)
2n+1 + ( n + 1)2n− 1 3 4
Statement-1 is true
= 2n C0 + nC1 + nC2 + ...
n +1 2 3
∑ (r + 1) n
Cr x r = ∑r n
Cr x r + ∑ n
Cr x r
n
2 ( n + 3) − 1
= n n
n +1 = n

r=0
n −1
Cr − 1 x r + ∑
r=0
n
Cr x r

The correct option is (B)

= nx(1 + x)n – 1 + (1 + x)n
40. For option (a)

Substituting x = 1
General term
= 25Cr (x–1/5)25–r (2x3/5)r ∑ (r + 1) C n
r = n 2n – 1 + 2n
There is a term containing x3 if
Hence Statement-2 is also true and is a correct explanation
− 25 + r 3r of Statement-1.
+ =3
The correct option is (B)
5 5
4r
⇒ − 5 + =3 9
5 45. 1 – qn ≥
10
∴ r = 10 i.e. an integer
n
Hence, T11 will be the term containing x3 and it will be  3 1
25
C10210x3. ⇒   ≤   ⇒  n ≥ log 3 10
 4  10
4
Similarly, try all the other options, and in none you will have
the value of r as an integer, Hence, no other binomial will
1
have the term of x3. ⇒ n ≥ 4
The correct option is (A)
log10 log10 3

HINTS AND EXPLANATIONS



The correct option is (A)
41. We have (1 – x – x2 + x3)6 = (1 – x)6 (1 – x2)6
46. 82n – (62)2n+1 = (1 + 63)n – (63 – 1)2n+1
Coefficient of x7 in
= (1 + 63)n + (1 – 63)2n+1
(1 – x – x2 + x3)6 = 6C1 ⋅ 6C3 – 6C3 ⋅ 6C2 + 6C5 ⋅ 6C1
= (1 + nc163 + nc2 (63)2 + .... + (63)n)
= 6 × 20 – 20 × 15 + 6 × 6 = –144
+ (1 – (2n+1)c163 + (2n+1)c2 (63)2 + .... + (–1)(63)(2n+1))
The correct option is (D)
= 2 + 63(n c1 + nc2 (63) + .... + (63)n–1
– (2n+1)c1 + (2n+1)c2 (63) + .... – (63)(2n))
( 3 + 1) − ( 3 − 1)
2n 2n
42. ∴  Remainder is 2.
2n − 1 2n − 3
= 2[ C1 ( 3 ) + 2 n C3 ( 3 )
2n The correct option is (B)

2n − 5 47. Let loge10 = x.
+ 2n
C5 ( 3 ) + ....]
r n
n 1 + r log e 10

= which is an irrational number Then, ∑ (−1) ⋅ Cr ⋅

r=0 (1 + log e 10 n ) r

The correct option is (A)
n 1 + rx
= ∑ (−1) r nCr ⋅
r
3 8
r=0 (1 + nx)
1 − x 
43. (1 + x + x2)8 =   = (1 – x3)8(1 – x)–8  1 
r
 1 − x  n n n rx
= ∑ (−1) r ⋅ nCr   + ∑ (−1) r ⋅ ⋅ n−1Cr −1
= (1 – 8C1x3 + 8C2x6 – ....)(1 + 8C1x1 + 9C2x2 + 10C3x3 + ...) r=0 1 + nx  r=0 r (1 + nx ) r
a5 = coefficient of x5 = 12C5 – 8C19C2 = 792 – 288 = 504 n  1 
r

= ∑ (−1) ⋅ Cr  
r n
The correct option is (B) 1 + nx 
r=0
6.30  Chapter 6

r −1
 1 
nx = n ( n + 1)( 2n + 1)
n

− ⋅ ∑ (−1) r −1 ⋅ n−1
Cr −1  
1 + nx r = 0 1 + nx  6
n n−1

The correct option is (A)
 1   
= 1 −  −
nx 1 − 1 
 1 + nx  1 + nx  1 + nx  52. We have, t3r = 2nC3r – 1 x3r – 1
and, tr + 2 = 2nCr + 1 xr + 1.
n n
 nx   nx  Given, 2nC3r – 1 = 2nCr + 1
=   −   = 0.
1 + nx  1 + nx  ⇒ 3r – 1 = r + 1; or (3r – 1) + (r + 1) = 2n

The correct option is (C) ⇒ 2r = 2 ; or 4r = 2n
n
n
Cr ⇒ r = 1 (impossible); or r = .
48. Here, tr + 1 = = 1 ⋅ nCr 2
r 1 r +1
But r is a positive integer greater than 1. So, the value of

= 1 ⋅ n + 1Cr
+1 r is n , provided n is an even integer (> 2), otherwise r has
n +1 2
Putting r = 0, 1, 2, ... n and adding we get, ∑ Ck
n
no value.
0 k +1

The correct option is (A)
= 1 {n + 1C1 + n + 1C2 + n + 1C3 + ... + n + 1Cn + 1}

n +1 53. We have,
n +1 2n

= 1 {2n + 1 – n + 1C } = 2 − 1 (1 + x + x2)n = ∑ ar x r...(1)

0 r=0
n +1 n +1
1

The correct option is (C)
Replacing x by , we get
x
49. The greatest coefficient is  
n
2n a
1 + 1 + 1  = ∑ rr
n!  x x 2  r=0 x

= r  [Here, n = 15, q = 3, r = 3, k = 4]
( q!) k −r [( q + 1)!]
Multiplying both sides by x2n, we get

The correct option is (A) 2 n− r
2n
(1 + x + x2)n = ∑ ar x
...(2)
r=0
10
50. We have, ∑ 20 Cr = 20C0 + 20C1 + ... + 20C10
From (1) and (2), we have
r= 0
2n 2n
2 n− r
∑ ar x = ∑ ar x
r

But C0 + 20C1 + ... 20C20 = 220
20
HINTS AND EXPLANATIONS

r=0 r=0
and, ∵ 20C20 = 20C0 , 20C19 = 20C1


On equating the coefficient of x2n – r on both sides, we get

20
C18 = 20C2 ...  and  20C11 = 20C9
10
a2n – r = ar for 0 ≤ r ≤ 2n.

∴ ∑ 20 Cr = (20C0 + 20C1 + ... + 20C20)
The correct option is (C)
r= 0

– (20C11 + 20C12 + ... + 20C20) 54. C02 − 2C12 + 3C22 − 4C32+ ...
= 220 + 20C10 – (20C10 + 20C9 + ... + 20C0)
⇒ 2 [20C0 + 20C1 + ... + 20C10] = 220 + 20C10 + (− 1) n ( n + 1)Cn2

∴ 20C0 + 20C1 + ... + 20C10 = 219 + 1  20C10


2 2 2 2 n 2
= [C0 − C1 + C2 − C3 + ... + (− 1) Cn ]
2
The correct option is (B) − [C12 − 2C22 + 3C32 − ... + (− 1) n nCn2 ]
n
51. We have, n! −1 n
= (− 1) n / 2 − (− 1) 2 n
Cn
n + 1C2 + 2 [2C2 + 3C2 + 4C2 + ... + nC2]  n   n  2
 !  ! 2
 2   2 
= n + 1C2 + 2 [3C3 + 3C2 + 4C2 + ... + nC2]
n!  n
= n + 1C2 + 2 [4C3 + 4C2 + ... + nC2] = (− 1) n / 2 1 + 
n n  2 
= n + 1C2 + 2 [5C3 + ... + nC2]  !  !
2 2
= n + 1C2 + 2 ⋅ n + 1C3
= n + 1C2 + n + 1C3 + n + 1C3  n  n
∴  2 !  ! [C02 − 2C12 + 3C22 − ...
= n + 2C3 + n + 1C3  2   2 

= n ( n + 1)( n + 2) + n ( n + 1)( n − 1)
+ (− 1) n ( n + 1) Cn2 ] = (− 1) n / 2 ( n + 2)
6 6
The correct option is (C)
Binomial Theorem  6.31

1 1.4 1 1.4.7 1 59. General term in (1 + λ)n (1 + µ)n (λ + µ)n is


55. Let S = 1 ... tp, q, r = (nCpλp) (nCqµq) (nCrλn–rµr)
32 1.2 34 1.2.3 36
⇒ tp, q, r = nCp nCq nCr λp+n–r µq+r
1.4 1.4.7
 1   1  3 3  1 
2 3
 1  term contains coefficient of λnµn if
⇒ S = 1 +     +   +3 3 3   + ...
 3   3  1.2  3  1. 2 . 3  3  p + n − r = n  and q + r = n
1  1  ⇒ p = r  and q = n – r
1 1 1 +   3 Now, tr, (n–r), r contains coefficient of λnµn
 3  1 
⇒ S = 1 +     + 3  
 3   3  2!  3 ⇒ Coefficient of λnµn = nCr nCn–r nCr
1 1   1  ∴ Coefficient of λnµn = (nCr)3
1 +  2 +   3 The correct option is (D)
3  3  3  1 
+   + ...
3!  3
60. Since (1 – x)n = C0 – C1 x + C2 x2 – C3 x3 + ...
which is an equivalent of ∴ x (1 – x)n = C0 x – C1 x2 + C2 x3 – C3 x4 + ...
1 1 1
− −
 1 3  2 3  3 3
1
⇒ ∫ x (1 − x ) n dx
S = 1 −  ⇒ S =    =  
 3  3   2  0
1
The correct option is (B) = ∫ (C0 x − C1 x 2 + C2 x 3 − C3 x 4 + ...) dx
56. We have, 0
C C C
171995 + 111995 – 71995 = 0 − 1 + 2 −... up to (n + 1) terms
2 3 4
=  (7 + 10)1995 + (1 + 10)1995 – 71995
For L.H.S. put 1 – x = t, ∴ dx = – dt
= [71995 + 1995C1. 71994. 101 + 1995C2. 71993. 102 +
1
... + 1995C1995. 101995] + [1995C0 + 1995C1. 101 ∴ L.H.S. =
( n 1)( n 2)
+ 1995C2. 102 + ...

The correct option is (D)
+ 1995C1995. 101995] – 71995
= [1995C1. 71994. 101 + ... + 101995] 61. Here, f = R – [R] is the fraction part of R. Thus if I is the
+ [1995C1. 101 + ... + 1995C1995. 101995] + 1 integral part of R, then
=  a multiple of 10 + 1. R = I + f = (5 5 + 11)2n + 1, and 0 < f < 1.
Thus, the unit’s place digit is 1. Now, ∵ 5 5 – 11 = 18034 < 1,

The correct option is (B) ∴ if f ‘ = (5 5 – 12)2n + 1, then 0 < f ‘ < 1

HINTS AND EXPLANATIONS


57. Given polynomial is
Now, I + f – f ‘ = (5 5 + 11)2n + 1 – (5 5 – 11)2n + 1
(x + nC0) (x + 3. nC1) (x + 5. nC2) ...(x + (2n + 1). nCn)
= 2 [2n + 1C1 (5 5)2n × 11
=  xn+1 + xn {nC0 + 3. nC1 + 5. nC2 +
+ 2n + 1C3 (5 5)2n – 2 × 113 + ...] ...(1)
... + (2n + 1). nCn} + xn–1 (...) + ... = An even integer.
∴  Coefficient of xn in the expression is ⇒ f – f ‘ must also be an integer.
n
∑ ( 2r + 1) nCr = ∑ 2r nCr + ∑ n Cr
n n
⇒ f – f ‘ = 0, ∵ 0 < f < 1, 0 < f ‘ < 1
r=0 r=0 r=0 ⇒ f = f ‘

=  2 ∑ r
n n n−1 ∴ R f = R f ‘ = (5 5 + 11)2n + 1 (5 5 – 11)2n + 1
Cr −1 + 2n
r=0 r = (125 – 121)2n + 1 = 42n + 1.
n

=  2n ∑ n−1
Cr −1 + 2n The correct option is (C)
r=0

= 2n.2n + 2n = (n + 1) 2n
–1 62. The number of solutions of x1 + x2 + ... + xk = n

The correct option is (C) = coefficient of tn in (t + t2 + t3 + ...) (t2 + t3 + ...)
...(tk + tk + 1 + ...)
58. We have, S = 1 + n + n2 + ... + n255
= coefficient of tn in t1 + 2 + ... + k (1 + t + t2 + ...)k
1( n256 − 1) n128 − 1 But, 1 + 2 + ... + k = 1 k (k + 1) = r (say)
⇒  S = = ( n128 + 1)
n −1 n −1 2
∴  S = (n128 + 1) (1 + n + n2 + ... + n127) and, 1 + t + t2 + ... = 1
Thus, the largest value of m for which 1 + n + n2 1- t
Thus, number of required solutions
+ ... + n255 is divisible by nm + 1 is 128.
= coefficient of tn – r in (1 – t)– k
The correct option is (A)
6.32  Chapter 6


= coefficient of tn – r in (1 + kC1 t + k + 1C2 t2 n +1
 q + 1
1 − 

+ k + 2C3 t3 + ...)  2  2n+1 − ( q + 1) n+1

= k + n – r – 1Cn – r = = ...(2)
q +1 (1 − q) ⋅ 2n

= k + n – r – 1Ck – 1 = mCk – 1 1−
2

where, m = k + n – r – 1 Now, n + 1C1 + n + 1C2 ⋅ S1 + n + 1C3 ⋅ S2 + ...
= k + n – 1 – 1 k (k + 1)
+ n + 1Cn + 1 ⋅ Sn
2 1 − q  1 − q 2  1 − q3 
= n + 1C1   + n + 1C2   + n + 1C  

= 1 [2k + 2n – 2 – k2 – k] = 1 (2n – k2 + k – 2). 1 − q   1 − q   1 − q 
3 

2 2 1 − q n+1 

The correct option is (A) + ... + n + 1Cn + 1 

 1 − q 
n 1
n
63. We have, ∑ Cr sin r x cos ( n − r ) x
= [n + 1C1 (1 – q) + n + 1C2 (1 – q2)
r =0 1- q

+ n + 1C3 (1 – q3) + ... + n + 1Cn + 1 (1 – qn + 1)]
= 1 [(nC0 sin 0x cos nx + nCn sin nx cos 0x)
2 1
+ (nC1 sin x cos (n – 1) x + nCn – 1 sin (n – 1) x ⋅ cos x)
= [(n + 1C1 + n + 1C2 + ... + n + 1Cn + 1)
1- q
+ (nC2 sin 2x cos (n – 2) x + nCn – 2 sin (n – 2) x ⋅ cos 2x) – (n + 1C1 ⋅ q + n + 1C2 ⋅ q2 + ... + n + 1Cn + 1 ⋅ qn + 1)]
+ ... + (nCn sin nx cos 0x + nC0 sin 0x cos nx)]
1

= [(2n + 1 – n + 1C0) – {(n + 1C0 + n + 1C1 q + n + 1C2 q2
= 1 [nC0 sin nx + nC1 sin nx + ... + nCn sin nx] 1- q
2
+ ... + n + 1Cn + 1 qn +1 – n + 1C0}]
n
= 1 [nC0 + nC1 + ... + nCn] sin nx = 2 sin nx 1
2 2
= [(2n + 1 – 1) – {(1 + q)n + 1 – 1}]
n
n
1- q
∴ ∑ C r sin r x cos ( n − r ) x = 2n – 1 sin nx.
r =0 1

= [2n + 1 – (1 + q)n + 1] = 2n S′n.

The correct option is (C) 1- q

64. We have,
The correct option is (B)
nCn + n + 1Cn + n + 2Cn + ... + n + kCn 66. We have, (1 + x)15 = C0 + C1 x + C2 x2 + ... + C15 x15
= coeff. of xn in (1 + x)n + (1 + x)n + 1 + ... 15
⇒ (1 + x ) = C0 + C1 + C2 x + C3 x2 + ... + C15 x14
HINTS AND EXPLANATIONS

+ (1 + x)n + k.
x x
Now, (1 + x)n + (1 + x)n + 1 + (1 + x)n + 2 + ... Differentiating both sides w.r.t. x, we get
+ (1 + x)n + k 14 15
x ⋅ 15 (1 + x ) − 1⋅ (1 + x )
 (1 + x ) k +1 − 1 x2
= (1 + x)n  
 x  C
  = – 0 + C2 + 2 C3 x + 3 C4 x2 + ... + 14C15 x13
x2
= 1 (1 + x)n + k + 1 – 1 (1 + x)n
Putting x = 1 on both sides, we get
x x 15 ⋅ 214 – 215 = – C0 + C2 + 2 C3 + 3 C4 + ... + 14 C15
Equating the coefficient of xn, we get ⇒ 214 (15 – 2) + 1 = C2 + 2 C3 + 3 C4 + ... + 14 C15
nC0 + n + 1Cn + n + 2Cn + ... + n + kCn ∴ The given series = 214 ⋅ 13 + 1 = 219923.
= n + k + 1Cn + 1 – 0 = n + k + 1Cn + 1 The correct option is (A)
 1  67. (1 + x + x2)n = a0 + a1 x + a2 x2 + a3 x3 + a4 x4 +
There is no term containing x nin (1 + x ) n  .
 x  ... + a2n – 1 x2n – 1 + a2n x2n ...(1)

The correct option is (C)
Replacing x by 1 in (1), we get

65. We have, x
1 − q n+1  1 − q n+1 
(1 + x + x2)n = a0 x2n + a1 x2n – 1 + a2 x2n – 2 + ...
Sn = 1 + q + q2 + ... + qn =   =   ...(1)
+ a2n – 1 x + a2n
 1 − q   1 − q 

Again, replacing x by – x in (1), we get
2 n
(1 – x + x2)n = a0 – a1 x + a2 x2 – a3 x3 + ...
 q + 1  q + 1  q + 1
and, S′n = 1 + 
+  + ... +  
– a2n – 1 x2n – 1 + a2n x2n ...(2)
 2   2   2 

Multiplying (1) and (2), we get
Binomial Theorem  6.33


(1 + x2 + x4)n = (a0 x2n + a1 x2n – 1 + a2 x2n – 2 + r
n 3 n  3

... + a2n – 1 x + a2n) × (a0 – a1 x + a2 x2 + t2 = ∑ (−1) r ⋅ nCr  2  = ∑ (−1) r ⋅ nCr  
r =0 
2  r =0  4 

... – a2n – 1 x2n – 1 + a2n x2n) ...(3)
n
 3 
[Note that (1 – x + x2) (1 + x + x2) = (1 + x2)2 – x2
= 1 −  = 1 = 1

= 1 + x2 + x4]  4  4n 22n

Finally, replace x by x2 in (1), we get r r
n 7 n 7

(1 + x2 + x4)n = a0 + a1 x2 + ... + an x2n + ... + a2n x4n ...(4) t3 = ∑ (−1) r ⋅ nCr  3  = ∑ (−1) r ⋅ nCr  
r =0 
2  r =0  8 

Now, equating the coefficients of x2n on the right hand sides
n
of (3) and (4), we get  7
= 1 −  = 1 = 1
a02 − a12 + a22 − a32 + ... − a22n−1 + a22n = an.  8 8n 23n

The correct option is (B) ... ... ... ... ... ... ... ... ... ... ... ... ... ... ...
... ... ... ... ... ... ... ... ... ... ... ... ... ... ...
68. Let S = (1 + x)100 + 2x (1 + x)999 + 3x2 (1 + x)998 + ...
∴ Required sum
+ 1000 x999 (1 + x) + 1001 ⋅ x1000

= 1 1 1

This is an A.G.S. of common ratio r = x + + +... to m terms
2 n 2 2 n 23 n
1+ x
 x    1 m 
∴   S = x (1 + x)999 + 2x2 (1 + x)998 + ... 1 −    1

1+ x  1   2n   1 - mn

= = n 2 .
1001 2n  1 − 1  2 -1
+ 1000 ⋅ x1000 + 1001x
 2 
n
1+ x  

On subtracting, we get
The correct option is (B)
 x  70. We have,
1 −  S = (1 + x)1000 + x (1 + x)999
 1 + x  (1 – x)n = nC0 – nC1 x + nC2 x2 – ... + (– 1)n nCn xn,
1001 and, (x + 1)n = nC0 xn + nC1 xn – 1 + nC2 xn – 2 + ... + nCn.
+ x2 (1 + x)998 + ... + x1000 – 1001x

1+ x The given series is the coefficient of xn in the product of
R.H.S. of the above two.
⇒ S = [(1 + x)1001 + x (1 + x)1000 + x2 (1 + x)999
∴ Sum of the series = coefficient of xn in (1 – x)n ⋅ (x + 1)n
+ ... + x (1 + x)] – 1001 x1001
1000

= coefficient of xn in (1 – x2)n

HINTS AND EXPLANATIONS


 1001

  x  
1001 = coefficient of xn in
(1 + x ) 1 −   
 1 + x   [nC0 + nC1 (– x2) + nC2 (– x2)2 + ... + nCn (– x2)n]

=  
– 1001 x1001
x Since n is even, let n = 2m. Then,
1− sum = coefficient of x2m in
1+ x
[2mC0 + 2mC1 (– x2) + 2mC2 (– x2)2 + ... + 2mC2m (– x2)2m]
  x 1001 
= (1 + x)1002 1 − 
  – 1001 x1001 = 2mCm (– 1)m = nCn/2 (– 1)n/2.
 
 1 + x   The correct option is (B)

= (1 + x)1002 – x1001 (1 + x) – 1001 x1001 71. We have,

= (1 + x)1002 – x1002 – 1002 x1001 ...(1) Cr n
 1 n 

Now, the coefficient of x50 on the R.H.S. of (1) tr + 1 = = 1 ⋅  Cr 
( r + 1)( r + 2) r + 2  r + 1 

= 1002C50.
 1 n+1 

The correct option is (C) = 1  ·  
Cr +1 

r + 2  n +1 
69. We have, 1  1 n+1 

= ⋅ ⋅ Cr +1 
n 1
r
n 1
r

n +1  r + 2 
t1 = 1 ∑ (−1) r ⋅ nCr   = ∑ (−1) r ⋅ nCr  
r =0  2  r =0  2  1 1 ⋅ n + 2Cr

= ⋅ +2
 1
n
n +1 n + 2
= 1 −  = 1

Putting r = 0, 1, 2, ..., n and adding, we get the required sum
 2  2n
1
 n r n r n
= [n + 2C2 + n + 2C3 + ... + n + 2Cn + 2]
∵ ∑ (−1) ⋅ Cr x = (1 − x )  ( n + 1)( n + 2)
 r = 0 
6.34  Chapter 6

1 1

= [2n + 2 – (n + 2C0 + n + 2C1)]
= {22 ⋅ n + 2C2 + 23 ⋅ n + 2C3 + ...
( n + 1)( n + 2) ( n + 1)( n + 2)
2n + 2 - n - 3 + 2n + 2 ⋅ n + 2Cn + 2}


= .
(n + 1)(n + 2) 1

= {(1 + 2)n + 2 – n + 2C0 – 2 ⋅ n + 2C1}

The correct option is (B) ( n + 1)( n + 2)
3n+ 2 − 2 ( n + 2) − 1 3n + 2 - 2n - 5
72. Let ( 3 + 1)2n = p + f, where p is the integral part and 0 < f
= = .
( n + 1)( n + 2) (n + 1)(n + 2)
< 1.

The correct option is (B)
∴ integer just above ( 3 + 1)2n = p + 1
Now, ( 3 + 1)2n = {( 3 + 1)2}n = (4 + 2 3 )n 74. The given series
= 2n (2 + 3 )n = mCr ⋅ nC0 + mCr – 1 ⋅ nC1 + ... + mC0 ⋅ nCr,
Thus, p + f = 2n (2 + 3 )n Now, (1 + x)m = mC0 + mC1 ⋅ x + mC2 x2 + ...
Also, 0 < 3 – 1 < 1 + mCr xr + ... + mCm xm,
∴ 0 < ( 3 – 1)2n < 1 and, (1 + x)n = nC0 + nC1 x + nC2 x2 + ... + nCr xr + ... + nCn
xn
Let f1 = ( 3 – 1)2n = (4 – 2 3 )n = 2n (2 – 3 )n,
The given series is the coefficient of xr in the product of
then 0 < f1 < 1.
R.H.S. of above two.
Now, p + f = 2n (2 + 3 )n
∴ Sum of the series = coefficient of xr in (1 + x)m ⋅
= 2n [2n + nC1 2n – 1 3 + nC2 2n – 2 ( 3 )2 (1 + x)n
+ ... + nCn ( 3 )n] ...(1) = coefficient of xr in (1 + x)m + n
f1 = 2n (2 – 3 )n = m + nCr.
= 2n [2n – nC1 2n – 1 3 + nC2 2n – 2 ( 3 )2 The correct option is (B)
+ ... + (– 1)n ⋅ nCn ( 3 )n] ...(2)
75. Let the expansion be that of (1 + x)n.
(1) + (2) ⇒
Let a, b, c, d be the (r + 1)th, (r + 2)th, (r + 3)th and (r + 4)
p + f + f1 = 2n ⋅ 2 [⋅ 2n + nC2 2n – 2 ( 3 )2 + ...]
th coefficients.
= an even integer  ...(A)
∴ a = nCr , b = nCr + 1, c = nCr + 2, d = nCr + 3.
∴ f + f1 = even number – p = an integer ...(B) n n
a = Cr C
HINTS AND EXPLANATIONS

Also, 0 < f < 1, 0 < f1 < 1


Now, n
= n+1 r
a+b C + n
C Cr +1
∴ 0 < f + f1 < 2  ...(C) r r +1

n! ( r + 1)! ( n − r )!
From (B) and (C), f + f1 = 1 ...(D)
= × = r +1
From (A), p + 1 = 2n + 1, an integer. r ! ( n − r )! ( n + 1)! n +1
Hence, integer just above ( 3 + 1)2n i.e., (p + 1) is divisible b = ( r + 1) + 1 = r + 2 ,

Similarly,
by 2n + 1. b+c n +1 n +1
The correct option is (A)
c = ( r + 2) + 1 = r + 3 .
73. We have, c+d n +1 n +1
2r + 2 nCr r +2 a c = r + 1 r + 3 = 2r + 4 = 2 ( r + 2)
tr + 1 = = 2 ⋅
1 nCr ∴ + +
( r + 1)( r + 2) r + 2 r −1 a+b c+d n +1 n +1 n +1 n +1

= 2b .
r +2
= 2

1 n + 1Cr
r + 2 n +1
+1
b+c

2r + 2  1 n+1  ⇒ a , b , c are in A.P.



= ⋅ Cr +1  a+b b+c c+d

n +1  r + 2 
r +2 a + b , b + c , c + d are in H.P.

= 2 ⋅ 1 n+ 2Cr + 2
a b c
n +1 n + 2

The correct option is (C)
 1 n 1 n+1 
∵ C = Cr +1 
 r + 1 r n + 1  76. 3400 = (34)100 = (81)100 = (1 + 80)100 = 1 + 100 C1 (80) + 100
C2
(80) 2 +....+ 100C100 (80)100

Putting r = 0, 1, 2, ..., n and adding we get,
= 1 + 8000 + last two digits in each term is 00

The given expression
Binomial Theorem  6.35

∴ Last two digits = 01 6m


1  π π
The correct option is (C)
= imaginary part of 26m cos + i sin 
3 
 3 3 
77. Consider (1 + x)n = C0 + C1x + C2x2 + C3x3 +...... 1

= × 26 m × 0 = 0 ( ∵ sin 2m π = 0)
....(1) 3
Integrating equation (1) w.r. to x, between limits 0 and x, we
The correct option is (C)
get
x 2 n x
80. The given series
∫0 (C0 + C1 x + C2 x + ......)dx = ∫0 (1 + x ) dx
 x  x 
2
 x  
500
    
x x 2 3
(1 + x ) − 1 n +1
S = (1 + x)500 1 + 1 + x + 1 + x  + + 1 + x  
....
⇒ C0x + C1 + C2 + ...... = ...(2)      
2 3 n +1 
501

Integrating equation (2), taking limits from – 1 to 0, we get  x 
1 −  
  1 + x 
x2 x3 0 (1 + x )
n +1
−1 = (1 + x) × 500
= (1 + x)501 – x501
∫−1 C0 x + C1 + C2 + ... dx = ∫−1
0
dx . x
 2 3  n + 1 1−
1+ x

...(3)
Hence, the coefficient of x301 in S = 501C301.
0 0
 C x 2 C x3 C x 4   (1 + x ) n + 2 x  The correct option is (A)
⇒  0 + 1 + 2 + .... =  −

 2 2.3 3.4  −1  ( n + 1)( n + 2) n + 1 −1 81. We have,
C C C  ( 6 ) 2n = 6n = (1 + 5)n
⇒ –  0 − 1 + 2 − ....
1.2 2.3 3.4  = C0 + C1.5 + C2.52 + C3.53 +...+ Cn.5n [where Cr = nCr]
1 1 1 2n

= − =− Thus, we have ( 6 )

( n + 1)( n + 2) n + 1 n+2 5

∴  C0 C1 C 1 = + C1 + C2 .5 + C3 .52 + ..... + Cn .5n−1  = 1 + integer


1
− + 2 + ..... = 5   5
1.2 2.3 3.4 n + 2

The correct option is (A) whose fractional part is 1 .

5
78. We have,
The correct option is (B)
(1 + x)n = C0 + C1x + C2x2 +...+ Cnxn ...(1) 82. We have,

HINTS AND EXPLANATIONS


Differentiating equation (1) w.r.t. x, we get (x + C0) (x –3C1) (x + 5C2)..... up to (n + 1) terms
n (1 + x)n –1 = C1 + 2C2x + 3C3x2 + ... + nCnxn–1...(2) = x n + 1 + [C0 –3C1 + 5C2–.....(n + 1) terms]xn+....
Differentiating equation (2) w. r. t. x, we get Thus, coefficient of xn = C0 – 3C1 + 5C2–....(n + 1) terms
n(n – 1) (1 + x)n–2 = (1.2)C2 + (2.3) C3x +.... n
....+ n(n –1)Cnxn–2...(3)
= ∑ Cr (−1) r ( 2r + 1)
r=0
Putting x = 1 in equation (3), we have
We have,
(1.2) C2 + (2.3)C3 +.......+ (n – 1)n Cn = n(n– 1)2n –2. n

(1 – x2)n = ∑ Cr (−1) r x 2 r
The correct option is (B) r=0
n
79. Let n = 2m, then k = 3m
i.e., x (1 – x2)n = ∑ Cr (−1) r x 2 r + 1 [multiplying by x]
k 3m r=0
r −1
∴ ∑ (−3)
3n
C2 r −1 = ∑ (−3) r −1 3m
C2 r −1 n
r =1 r =1
i.e., (1 – x )n– 2nx2 (1 –x2)n–1 = ∑ Cr (−1) r ( 2r + 1) x 2 r
2

r=0

= (–3)0 6mC1 + (–3)1 6m C3 + (–3)2 6mC5+....
[differentiating w.r.t.x]
= 6mC1– ( 3 ) 2 6 m C3 + ( 3 ) 4 6 m C5−.....

Putting x = 1, we have,
1  6m  n
( 3) ( 3)
3 6m 5 6m

=  3 C1− C3 + C5−....  ∑ Cr (−1) r ( 2r + 1) = 0.
3   r=0

The correct option is (A)
1

= imaginary part of (1 + 3 i )6 m
( 5)
100 −r r
3 83. tr +1 = 100
Cr 8
( 6 2 ) . As 2 and 5 are coprime, tr+1
6m
1 3 
1  will be rational if 100– r is a multiple of 8 and r is a multiple

= imaginary part of 26m  + i

3 
 2 2 of 6. Also, 0 ≤ r ≤ 100
∴ r = 0, 6, 12,....96 ...(1)
6.36  Chapter 6

⇒ 100 – r = 4, 10, 16,.....,100 Now,


But 100 – r is to be a multiple of 8, so  (1+ 1)2n = 2nC0 + 2nC1 + 2nC2 + .....+2nC2n
100 – r = 0, 8 16, 24,..., 96 ...(2) and, (1 – 1)2n = 2nC0 – 2nC1 + 2nC2 – .....+2nC2n
The common terms in (1) and (2) are 16, 40, 64 and 88 On adding, we get
∴ r = 84, 60, 36, 12 give rational terms. 22n = 2(2nC0 + 2nC2 + 2nC4 + .....+2nC2n )
∴ The number of irrational terms = 101 – 4 = 97 ⇒ 22n–1 = 2nC0 + 2nC2 + .....+2nC2n
The correct option is (B) 22 n-1
∴S= .
84. 1 + 99n =1 + (100 – 1)n = 1+ {nC0100n –nC1.100n–1 +....– (2n)!
nCn} because n is odd
The correct option is (B)
= 100{nC0.100n–1 –n C1.100n–2+... – n Cn –2.100 + nCn–1} 86. (x + 2n + 1C0) (x + 2n +1C1)(x + 2n +1C2)....(x + 2n + 1Cn)
= 100 × integer whose unit’s place is different from 0. = xn +1 + xn (2n + 1C0 + 2n +1 C1 + 2n +1C2 +....
[ ∵ nCn–1 = n, has odd digit at unit’s place] +2n + 1Cn) +....
∴ There are two zeros at the end of the sum 99n + 1. ∴ Coeffcient of xn (say)
The correct option is (A) S = 2n + 1C0 + 2n + 1C1 + 2n + 1C2 + ..... + 2n + 1Cn ...(1)
n 1 ⇒ S = 2n + 1C2n + 1 + 2n + 1C2n + 2n + 1C2n – 1 + ... 2n + 1Cn +1
85. Let S = ∑ ( ∵ nCr = nCn – r)...(2)
r=0 ( 2r )!( 2n − 2r )!
On adding (1) and (2), we get
1 n ( 2n)! 1 n 2n

= ∑ = ∑ C2 r 2S = 22n + 1  ∴  S = 22n
( 2n)! r = 0 ( 2r )!( 2n − 2r )! ( 2n)! r = 0
The correct option is (B)
1 2n

=
( 2n)!
( C0 + 2 nC2 + 2nC4 + .... +2n C2n )

Previous Year’s Questions


87. ∴ (1 2x 3x 2 ...) 3/ 2
[(1 x ) 2 ] 3/ 2
The correct option is (C)
(1 x ) 3 91. Coefficient of xn in the expansion of (1 + x)(1 − x)n = (1 + x)
(nC0 − nC1x + …….. + (−1)n –1 nCn – 1 xn – 1 + (−1)n nCn xn) is (−1)
Now, coefficient of x in (1 + 2x + 3x2 + ...)-3/2 = coefficient of
5
HINTS AND EXPLANATIONS

n n
Cn + (−1)n –1 nCn – 1 = (−1)n (1− n).
x5 in (1 − x)3 = 0
The correct option is (B)

The correct option is (D)
92. Given that m Cr 1, m Cr , m Cr 1 are in A.P .
88. ∴ (1 x x2 x3 ...) 2 [(1 x ) 1 ]2
2
⇒ 2 mC = mC + mCr
r r 1 1
(1 x )
m m

Now, coefficient of xn in (1 + x + x2 + ...)2 = coefficient of xn Cr 1 Cr +1
2 m m
in (1 − x)3 = 0 Cr Cr
n 2 1 n +1
= C2 1 = C1 r m r
= +
= n +1 m r 1 r 1

The correct option is (D)
⇒ m2 − m (4r + 1) + 4r2 − 2 = 0.
256 256 r 1/8 r
89. General term = Cr ( 3) [(5) ]
The correct option is (C)
For integral terms, r should be 8k And then k assumes values
from 0 to 32.Hence, (B) is the correct answer. 6
93. The expression 50
C4 + 56 r
C3
The correct option is (B) r 1
90. Coefficient of Middle term in (1 + αx)4 = t3 = 4C2⋅ α2 Coefficient
of Middle term in (1 − αx)6 = t4 = 6C3 (− α)3 = 50C4 + 55
C3 + 54C3 + 53C3 + 52C3 + 51C3 + 50C3

Given that 4C2α2 = −6C3.α3
⇒ −6 = 20 α = ( 50C4 + 50C3 ) + 51C3 + 52C3 + 53C3 + 54C3 + 55C3

−3
⇒ α = = 51
C4 + 51C3 + 52C3 + 54C3 + 55C3
10
Binomial Theorem  6.37

So, n − m = 10 and (m − n)2 - (m + n) = 20 ⇒ m + n = 80


= C + C = C .
55 55 56
∴ m = 35, n = 45
4 3 4


The correct option is (D) The correct option is (D)

94. Tr+1 in the expansion ∴ 98 Since the sum of 5th and 6th terms is zero, we have
1
11
11−r 1
r
a n − 5 +1
ax 2 + = 11Cr (ax 2 ) n
C4 an‑4(−b)4 + nC5 an−5 (− b)5 = 0
bx bx b 5


The correct option is (D)
= 11Cr (a)11−r (b)−r (x)22−2r−r
⇒ 22 − 3r = 7 ⇒ r = 5 99. We have
∴ coefficient of x7 = 11C5 (a)6 (b)−5 ……(1)

(1 + x)20 = 20C0 + 20C1x + … + 20C10x10 + … + 20C20x20

Also, Tr+1 in the expansion
Put x = 1,
1
11
1
r

0 = 20C0 − 20C1 + … − 20C9 + 20C10 − 20C11 + … + 20C20
ax − = 11Cr ( ax 2 )11−r
bx 2 bx
0 = 2 (20C0 − 20C1 + … − 20C9) + 20C10
= 11Cr a 11 − r (−1)r × (b) −r (x) −2r (x)11−r 1
⇒ 20C0 − 20C1 + … + 20C10 = 20
C10.
Now 11 − 3r = −7 ⇒ 3r = 18 ⇒ r = 6 2

The correct option is (B)
∴ coefficient of x-7 = 11C6 a5 × 1 × (b)−6
⇒ 11C5 (a)6 (b) −5 = 11C6 a5 (b) −6 9
100. 1 qn
⇒ ab = 1 10
The correct option is (D) n
3 1
3 3 3 1 4 10
95. (1− x )1/ 2 1+ x + −1 x 2 −1 − 3 x
2 2 2 2 n log 3 10
4
2
1 1
− 3(2) x n
2 log10 4 − log10 3

3 3
The correct option is (A)

HINTS AND EXPLANATIONS


= (1− x )1/ 2 − x 2 = − x 2 (because the higher powers of

8 8 101. We can write 82n − (62)2n + 1
x are neglected) = (1 + 63)n-(63 − 1)2n + 1

The correct option is (C) = (1 + 63)n + (1 − 63)2n + 1
96. We have = (1 + nc163 + nc2(63)2 + .... + (63)n) + (1 − (2n + 1)c1 63
(1 − ax)−1 (1 − bx)−1 = (1 + ax + a2x2 + ......) (1 + bx + (2n + 1)c2 (63)2 + …+ (−1) (63)(2n + 1))
+ b2x2 + ....) = 2 + 63(nc1 + nc2(63) + .... + (63)n−1 − (2n + 1) c1 + (2n +
∴ Coefficient of xn = bn + abn−1 + a2bn−2 + .... + an−1b 1)
c2(63) + .... (63)(2n))
b n+1 − a n+1 ∴ Reminder is 2
+ an =
b−a
The correct option is (B)

b n+1 − a n+1 102. 1− x − x 2 (1− x ) = (1− x )6 (1− x 2 )6


6
an
b−a
6 6 6 2 6

The correct option is (D) C0 C1 x C2 C3 x 3 6
C4 x 4
=

97. We have −6 C5 x 5 +6 C6 x 6
(1 − y)m(1 + y)n = [l − m C1y + m C2y2 −....][1 + n C1y
6
C 6
C x2 6
C2 x 4 6
C3 x 6 ....
+ n C2y2 + ...] 0 1


Coefficient of
m( m −1) n( n −1)
= 1 + ( n − m) +
+ − mn y 2 + ....
x 7 =6 C16C3 −6 C36C2 +6 C56C1
2 2
m 2 + n2 − m − n − 2mn = 120 − 300 + 36 = −144
∴ a1 = n − m = 10 and a2 = = 10
2
The correct option is (B)
6.38  Chapter 6

2 n
= 9(3n−2 +n Cn−1 3n−1... +n C2 )
n 2
103. ( 3 + 1) − ( 3 −1)
2n
= ( 3 +1 )
Set X has natural numbers which are multiples of 9
2 n (not all)

( − 3 −1 ) = (4 + 2 3) n − (4 − 2 3) n

Set Y has all multiples of 9 X Y Y

(2 + 3 )
n
n n
=2 − (2 − 3)
The correct option is (D)

. (−2 x1/ 2 )
50−r r
n n n n−1 n n− 2
107. t r +1 =50 Cr . (1)
C0 2 + C1 2 3 + C2 2 3 + ....
= 2n =50 Cr . 2r . x r / 2 (−1) r
n n− n n−1 n n− 2
C0 2 C1 2 3 C2 2 3 .... r an even integer.
Sum of coefficient

= 2n+1 n C1 2n−1 3 +n C3 2n−33 3 + .... = 2n+1 3
(some integer) r =0
2r
2
( 2
)
25 50 C . 22 r = 1 (1 + 2)50 + (1− 2)50 = 1 (350 + 1)
S

which is irrational
The correct option is (D)

The correct option is (A) 1
108. 21C1 + 21C2 + …….. + 21C10­ = (21C0 + 21C1 + … + 21C21 – 1 =
2
α 220 – 1
104. f ( x ) = + 2 β x + 1 2 β x 2 + x + α = 0 has roots –1
x (10C1 + 10C2 + ……. + 10C10) = 210 – 1

and 2
\ Required sum
= (220 – 1) – (210 – 1)

The correct option is (C)
= 220 – 210

105. 1(1− 2 x )18 + ax(1− 2 x )18 + bx 2 (1− 2 x )18
Hence, the correct option is (C)
Coefficient of
109. ( x + x 3 − 1)5 + ( x − x 3 − 1)5
x 3 : (−2)318 C3 + a(−2) 218 C2 + b(−2)18 C1 = 0
( x 3 − 1)1 2 = t
4 (17 16) 17 ⇒ (x + t)5 + (x – t)5

− 2a . + b = 0 ….(i)
(3 2) 2 Root needs to be removed



Coefficient of Hence, we need t 0 , t 2 , t 4

x 4 : (−2)418 C4 + a(−2)318 C3 + b(−2) 218 C2 = 0 ⇒ [5C0 x5t0 + 5C2x3t2 + 5C4 xt4 + other term]

HINTS AND EXPLANATIONS

16
+ [5C0x5t0 + 5C2x3t2 + 5C4xt4 + other term]
(4 20) 2a b 0 (ii)
Remove other terms
3
⇒ 2[5C0x5 + 5C2x3t2 + 5C4 xt4]


From equation (i) and (ii), we get ⇒ 2[x5 + 10x3 (x3 – 1) + 5x(x3 – 1)2]

17 8 16 17 ⇒ 2[x5 + 10x6 – 10x3 + 5x(x6 – 2x3 + 1)]

4 − 20 + 2a − =0
3 3 2 ⇒ 2[x5 + 10x6 – 10x3 + 5x7 – 10x4 + 5x]



Taking coefficient of odd degree term
17 8 60 2a( 19)
4 + =0 ⇒ 2[5x7 + x5 – 10x3 + 5x]

3 6
Sum of coefficient is
4 76 6 ⇒ 2[5 + 1 – 10 + 5] = 2 × 1 = 2

a=
3 2 19 25
50! (50 − r )!
110. ∑ r ! 50 − r ! .
a 16 r =0 ( ) (25 − r )!25!
25
2 16 16 272 50!.25!
b 80 ∑
r =0 (
3 3 r ! 25 − r )!.25!.25!

25

The correct option is (D) 50
C25 ∑ 25 Cr =50 C25 .225
r =0
106. Set X contains elements of the form
n n 111. Coefficient of x2 is 10C2.l2
4 − 3n −1 = (1 + 3) − 3n −1 So, 45 l2 = 720
= 3n +n Cn−1 3n−1.....n C2 32 \l=±4

CHAPTER
Sequence and Series
7
LEARNING OBJECTIVES
After reading this chapter, you will be able to:
 Learn the definition of sequence and the way they are  
Grasp in detail the knowledge on arithmetic mean,
denoted geometric mean and its properties
 Understand the concepts of series and arithmetic,  
Be familiar with arithmetico-geometric progression
geometric and harmonic progression (A.G.P.) and the method of finding the sum of A.G.P.

SEQUENCE Note that sequence (1) is a finite sequence whereas others


are infinite sequences.
A succession of numbers a1, a2, …, an formed according to
some definite rule is called a sequence.
A sequence is a function whose domain is the set N SERIES
of natural numbers and range a subset of real numbers or
complex numbers. A series is obtained by adding or subtracting the terms of
A sequence whose range is a subset of real numbers a sequence.
is called a real sequence. Since we shall be dealing with A series is finite or infinite according as the number
real sequences only, we shall use the term sequence to of terms in the corresponding sequence is finite or infinite.
denote a real sequence.
Notation PROGRESSIONS

The different terms of a sequence are usually denoted by If the terms of a sequence follow certain pattern, then the
a1, a2, a3, … or by t1, t2, t3, … The subscript (always a sequence is called a progression. Following are the three
natural number) denotes the position of the term in the special types of progressions:
sequence. The term at the nth place of a sequence, i.e., tn is 1. Arithmetic Progression (A.P.)
called the general term of the sequence. 2. Geometric Progression (G.P.)
3. Harmonic Progression (H.P.)
Info Box!
A sequence is said to be finite or infinite ARITHMETIC PROGRESSION (A.P.)
according as it has finite or infinite number
of terms. A sequence whose terms increase or decrease by a fixed
number is called an arithmetic progression. The fixed num-
ber is called the common difference of the A.P.
Illustrations In an A.P., the first term is usually denoted by a, the
1. 1, 4, 7, 10, … 19. In this sequence each term is obtained common difference by d and the nth term by tn. Obviously
by adding 3 to the previous term. d = tn – tn – 1
2. 2, – 4, 8, – 16, … In this sequence each term is obtained
by multiplying the preceding term by – 2. Thus, an A.P. can be written as
3. 2, 3, 5, 7, 11, 13, … This is the sequence of prime a, a + d, a + 2d, …, a + (n – 1) d, …
numbers.
7.2  Chapter 7

For example, (A) 21 (B) 28


(C)  14 (D)  None of these
1. 1, 3, 5, 7, 9, …
Since, 2nd term – Ist term = 3rd term – 2nd term Solution: (C)
= 4th term – 3rd term By inspection, first common term to both the series is
= … = 2, 23. Second common term = 51.
the sequence 1, 3, 5, 7, … are in A.P. whose first term Third = 79 and so on. These numbers form an A.P.
is 1 and common difference is 2. 23, 51, 79, …
2. 5, 3, 1, – 1, – 3, – 5, – 7, … are in A.P. whose first term
is 5 and common difference is – 2. Since, T15 = 23 + 14 (28) = 23 + 392 = 415 > 407

The nth term of an Arithmetic Progression and T14 = 23 + 13 (28) = 387 < 407

If a is the first term and d is the common difference of an \ number of common terms = 14.
A.P., then its nth term tn is given by 2. The number of numbers lying between 100 and 500
tn = a + (n – 1) d that are divisible by 7 but not by 21 is
(A) 57 (B) 19
(C)  38 (D)  None of these
QUICK TIPS
Solution: (C)
To find whether the series is an A.P. The numbers between 100 and 500 that are divisible
Step I:  Obtain an (the nth term of the sequence). by 7 are 105, 112, 119, 126, 133, 140, 147, …, 483,
Step II:  Replace n by n – 1 in an to get an – 1. 490, 497.
Step III:  Calculate an – an – 1. Let such numbers be n.
If an – an–1 is independent of n, the given sequence is an A.P.
Then, 497 = 105 + (n – 1) × 7; or n = 57.
otherwise it is not an A.P. The numbers between 100 and 500 that are divisi-
\ tn = An + B represents the nth term of an A.P. with common
ble by 21 are 105, 126, 147, …, 483.
difference A. Let such numbers be m.
Then 483 = 105 + (m – 1) × 21; or m = 19.
Hence, the required number = n – m = 57 – 19 = 38.

I M P O R TA N T P O I N T S 3. In the series 3, 7, 11, 15, … and 2, 5, 8, … each con-


tinued to 100 terms, the number of terms that are
■ If an A.P. has n terms, then the nth term is called the last ­identical is
term of A.P. and it is denoted by l. That is
(A) 21 (B) 27
l = a + (n – 1) d
(C)  25 (D)  None of these
■ Three numbers a, b, c are in A.P. if and only if

b – a = c – b, i.e., if and only if a + c = 2b. Solution: (C)


■ If a is the first term and d the common difference of an Let the nth term of the first series = the mth term of the
A.P. having m terms, then nth term from the end is (m – n second series.
+ 1)th term from the beginning. Thus,
\ 3 + (n – 1) × 4 = 2 + (m – 1) × 3,
nth term from the end = a + (m – n) d.
■ Any three numbers in an A.P. can be taken as
n m
or 4n = 3m or = = k (say)
a – d, a, a + d. Any four numbers in an A.P. can be taken 3 4
as a – 3d, a – d, a + d, a + 3d. Similarly 5 numbers in A.P. \ n = 3k and m = 4k
can be taken as a – 2d, a – d, a, a + d, a + 2d.
As each series is continued to 100 terms,

n = 3k ≤ 100 and m = 4k ≤ 100


SOLVED EXAMPLES
\ Possible values of k are 1, 2, 3, …, 25 and corre-
1. The number of terms common to two A.P.s 3, 7, 11, …, sponding to each value of k we get one identical term.
407 and 2, 9, 16, …, 709 is Hence there are 25 identical terms.
Sequence and Series  7.3

a Solution: (A)
4. If 51 + x + 51 – x, and 25x + 25–x are three consecutive
2 Let d be the common difference of the A.P., then
terms of an A.P., then the values of a are given by
4 = abc = (b – d)b (b + d) = b(b2 – d2)
(A) a ≥ 12 (B)  a > 12
(C) a < 12 (D)  a ≤ 12 ⇒ b3 = 4 + bd2 ≥ 4 ( b > 0, d2 ≥ 0)

Solution: (A) ⇒ b ≥ 22/3


a
Since 51 + x + 51 – x, , 25x + 25–x are in A.P., we have
2 Thus, the minimum possible value of b is 22/3.
a 1+x
2 =5 + 51 – x + 25x + 25–x 7. There are four numbers of which the first three are in
2 1
G.P. whose common ratio is and the last three are
Now put 5x = t so that t > 0, we then have 2
in A.P. If the last number is two less than the first, then
5 1 ⎛ 1⎞ ⎛ 1⎞ the four members are
 a = 5t + + t 2 + = ⎜t2 + 2 ⎟ + 5 ⎜t + ⎟
t t 2 ⎝ t ⎠ ⎝ t⎠ 1 1 1
(A)  3, 1, , – (B)  2, 1, , 0
2 ⎡⎛ 2 ⎤ 3 3 2
1⎞
a = ⎛⎜ t − ⎞⎟ + 2 + 5
1
or ⎢ ⎜ t − ⎟ + 2⎥ 1 1
⎝ t⎠ ⎢⎣⎝ t⎠ ⎥⎦ (C)  4, 1, , – (D)  None of these
4 2
2 2
= ⎛⎜ t − 1⎞⎟ + 5 ⎛ t − 1 ⎞ + 12 ≥ 12. Solution: (B)
⎝ t⎠ ⎜⎝ ⎟ a a
t⎠ The numbers can be taken as a, , , a – 2.
2 4
Thus, values of a are given by the inequality a ≥ 12.
a a
5. If 1, logy x, logz y, –15logx z are in A.P., then By question, , , a – 2 are in A.P.
2 4
(A) x = y–3 (B)  y = z–2
3
(C) x = z (D)  None of these a a
\ 2· = + a – 2; or a = 2.
Solution: (C) 4 2
Let d be the common difference of the A.P. 1
Hence, the numbers are 2, 1, , 0.
1 + d 2
Then, logy x = 1 + d ⇒ x = y
SUM OF n TERMS OF AN A.P.
logz y = 1 + 2d ⇒ y = z1 + 2d
The sum of n terms of an A.P. with first term ‘a’ and com-
and –15logx z = 1 + 3d ⇒ z = x –(1 + 3d)/15 mon difference ‘d’ is given by
n
Sn = [2a + (n – 1) d]
\ x = y1 + d = z(1 + 2d) (1 + d) 2

= x–(1 + d)(1 + 2d)(1 + 3d)/15 QUICK TIPS


⇒ (1 + d)(1 + 2d)(1 + 3d) = –15 ■ If Sn is the sum of n terms of an A.P. whose first term is ‘a’
and last term is l, then
⇒ 6d3 + 11d2 + 6d + 16 = 0 n
Sn = (a + l)
2
⇒ (d + 2)(6d2 – d + 8) = 0 ⇒ d = –2 ■ If common difference d, number of terms n and the last
term l, are given then
\ x = y1 + d = y–1, y = z1 + 2d = z–3 and x = (z–3)–1 = z3. n
Sn = [2l – (n – 1) d]
2
6. If three positive real numbers a, b, c are in A.P. such
■ tn = Sn – Sn – 1.
that abc = 4, then the minimum possible value of b is
■ The sum of an A.P. consisting of odd number of terms = n
(A) 22/3 (B)  21/3 (middle term), where n is number of terms.
5/3
(C) 2 (D)  None of these
7.4  Chapter 7

PROPERTIES OF A.P. n

1. If a1, a2, a3, …, an are in A.P., then


\ Sn = ∑ tn
n =1
(a)  a1 + k, a2 + k, …, an + k are also in A.P.
(b)  a1 – k, a2 – k, …, an – k are also in A.P. n
4 n2 + 1 1 n
⎛ 1 1 ⎞
(c)  ka1, ka2, …, kan are also in A.P. = ∑ 16
+
32
∑ ⎜⎝ 2n − 1 − 2n + 1⎟⎠
n =1 n =1
a a a
(d)  1 , 2 , …, n , k ≠ 0 are also in A.P. 1 n( n + 1) (2 n + 1) 1 1
k k k = + n+
2. If a1, a2, a3, … and b1, b2, b3, … are two A.P.s, then 4 6 16 32
(a)  a1 + b1, a2 + b2, a3 + b3, … are also in A.P. ⎛ 1 1 1 1 1 ⎞
(b)  a1 – b1, a2 – b2, a3 – b3, … are also in A.P. ⎜⎝1 − 3 + 3 − 5 + ... + 2n − 1 − 2n + 1⎟⎠
(c)  a1b1, a2b2, a3b3, … are also in A.P. 
a a a n 1 ⎛ 1 ⎞
(d)  1 , 2 , 3 , … may not be in A.P. = ( 4 n2 + 6 n + 5) + ⎜ 1−
b1 b2 b3 48 32 ⎝ 2n + 1⎟⎠
3. If a1, a2, a3, …, an are in A.P., then n( 4 n2 + 6 n + 5) n
(a)  a1 + an = a2 + an – 1 = a3 + an – 2 = … = +
48 16( 2n + 1)
ar − k + ar + k
(b)  ar =
, 0 ≤ k ≤ n – r. \ f (n) = n(4n2 + 6n + 5)
2
In other words, in an A.P., the sum of two terms equi- 9. The maximum sum of the series
distant from the beginning and end is a constant and is 1 2
equal to the sum of first term and last term 20 + 19 + 18 + 18 + … is
3 3
4. If nth term of a sequence is a linear expression is n (A) 310 (B) 290
then the sequence is an A.P. (C)  320 (D)  None of these
5. If the sum of first n terms of a sequence is a quadratic
expression in n, then the sequence is an A.P. Solution: (A)
6. If a1, a2, a3, a4, . . . , an are in A.P., then terms taken The given series is arithmetic whose first term = 20,
at regular intervals from this A.P. are also in A.P. e.g., common difference = – .
a1, a4, a7, a10, . . . also form an A.P. As the common difference is negative, the terms
will become negative after some stage. So the sum is
maximum if only positive terms are added.
SOLVED EXAMPLES Now
⎛ 2⎞
14 24 34 n4 tn = 20 + (n – 1) ⎜ − ⎟ ≥ 0 if 60 – 2 (n – 1) ≥ 0;
8. If + + + ... + = ⎝ 3⎠
1.3 3.5 5.7 (2n − 1) (2n + 1)
or 62 ≥ 2n or 31 ≥ n
1 n
f ( n) + , then f (n) is equal to \ The first 31 terms are non-negative.
48 16( 2n + 1)
31 ⎡ ⎛ 2⎞ ⎤
(A) n(4n2 + 3n + 2) (B)  n(4n2 + 6n + 5) \ Maximum sum = S31 = ⎢ 2 × 20 + (31 − 1) ⎜⎝ − 3 ⎟⎠ ⎥
(C) n(4n2 + 5n + 6) (D)  None of these 2 ⎣ ⎦
Solution: (B) 31
= (40 – 20) = 310
2
n4
We have, tn = 10. The sum to n terms of the sequence
(2 n − 1) (2 n + 1)
log a, log ar, log ar2, … is
n2 1 1 n
= + + (A)  log a2 rn – 1 (B)  n log a2 rn – 1
4 16 16( 4 n2 − 1) 2
3n
4n2 + 1 1 ⎛ 1 1 ⎞ (C)  log a2 rn – 1 (D)  None of these
= + − 2
16 32 ⎜⎝ 2 n − 1 2 n + 1⎟⎠
Sequence and Series  7.5

Solution: (A) \ First 24 terms are positive.


The given sequence can be expressed as \ Sum of the positive terms
log a, (log a + log r), (log a + 2 log r) … 24 ⎛ −3 ⎞
= S24 = ⎜ 2 × 10 + 23 × ⎟
2 ⎝ 7⎠
which is clearly an A.P. whose first term is log a and
common difference is log r. ⎛ 69 ⎞ 852
= 12 ⎜ 20 − ⎟ =
The nth term = log a + (n – 1) log r ⎝ 7⎠ 7
n 13. The minimum number of terms from the beginning of
Since sum to n terms = (a1 + an) 2 1
2 the series 20 + 22 + 25 + …, so that the sum may
n 3 3
\ Sn = [log a + log a + (n – 1) log r] exceed 1568, is
2 (A) 25 (B) 27
n n (C) 28 (D) 29
 = [2 log a + (n – 1) log r] = log a2 rn – 1
2 2
Solution: (D)
11. Let Sn denotes the sum of n terms of an A.P. whose first 2 8
It is in A.P. for which a = 20, d = 2 =
term is a. If the common difference d = Sn – k Sn – 1 + 3 3
Sn – 2 then k = Now, Sn > 1568
(A) 1 (B) 2 n ⎡ 8⎤
⇒ 40 + ( n − 1) ⎥ > 1568
(C)  3 (D)  None of these 2 ⎢⎣ 3⎦
Solution: (B) n 112 + 8n
⇒ × > 1568
2 3
We have, an = Sn – Sn – 1(1)
6
⇒ n2 + 14n > × 1568 = 1176
and an – 1 = Sn – 1 – Sn – 2(2) 8
⇒ n2 + 14n – 1176 > 0,
\ d = an – an – 1
or (n + 42) (n – 28) > 0
= (Sn – Sn – 1) – (Sn – 1 – Sn – 2) As n is positive, n – 28 > 0 i.e., n > 28
\ Minimum value of n = 29.
 [From (1) and (2)]
14. If the first, second and last terms of an A.P. are a, b and
= Sn – 2Sn – 1 + Sn – 2.
2a respectively, then its sum is
12. The sum of positive terms of the series ab ab
(A)  (B) 
4 1 2(b − a) b−a
10 + 9 + 9 + … is
7 7 3ab
(C)  (D)  None of these
352 437 2 ( b − a)
(A)  (B) 
7 7 Solution: (C)
852 Here a1 = a and a2 = b
(C)  (D)  None of these \ Common difference d = a2 – a1 = b – a
7
Let n be the number of terms in the series
Solution: (C)
3 \ an = 2a = a + (n – 1) d
Here, a = 10, d = – .
7 or (n – 1) d = a or (n – 1) (b – a) = a
tn = 10 +(n – 1) ⎛⎜ − ⎞⎟
3 a
Then, \ n–1=
⎝ 7⎠ b−a
a a+b−a b
tn is positive if 10 + (n – 1) ⎛⎜ − ⎞⎟ ≥ 0;
3 or n= +1= =
⎝ 7⎠ b−a b−a b−a
1 n b 3ab
or 70 – 3 (n – 1) ≥ 0 or 73 ≥ 3n; or 24 ≥ n \ Sum = (a1 + an) = (a + 2a) = .
3 2 2(b − a) 2 ( b − a)
7.6  Chapter 7

15. If S1 is the sum of an arithmetic series of ‘n’ odd num- 17. If a is the first term, d the common difference and Sk
ber of terms and S2, the sum of the terms of the series S
the sum to k terms of an A.P., then for k x to be inde-
S1 Sx
in odd places, then = pendent of x
S2 (A) a = 2d (B)  a=d
2n n
(A)  (B)  (C) 2a = d (D)  None of these
n +1 n +1
Solution: (C)
n +1 n +1
(C)  (D)  kx
2n n
Skx [ 2a + (kx − 1) d ]
Solution: (A) We have, = 2
Sx x
Let the odd number of terms of an arithmetic series be 2
[ 2a + ( x − 1) d ]
a, a + d, a + 2d, a + 3d, a + 4d, ….., a + (n – 1) d k [( 2a − d ) + kxd ]
=
Then, ( 2a − d ) + xd
n Skx
S1 = {2a + (n – 1) d} For to be independent of x, 2a – d = 0 or 2a = d.
2 Sx
n +1
S2 = a + (a + 2d) + (a + 4d) + … to terms
2 18. The sum of n terms of m A.P.s are S1, S2, S3, …, Sm. If
n +1 ⎡ ⎛ n +1 ⎞ ⎤ the first term and common difference are 1, 2, 3, …, m
= ⎢ 2a + ⎜ − 1⎟ × 2d ⎥ respectively, then S1 + S2 + S3 + … + Sm =
2×2 ⎣ ⎝ 2 ⎠ ⎦
1
n +1 (A)  mn (m + 1) (n + 1)
= (2a + (n – 1) d) 4
4
1
(B)  mn (m + 1) (n + 1)
S1 2n 2
\ =
S2 n +1 (C) mn (m + 1) (n + 1)
16. A club consists of members whose ages are in A.P., the (D)  None of these
common difference being 3 months. If the youngest Solution: (A)
member of the club is just 7 years old and the sum
of the ages of all the members is 250 years, then the We have, S1 = (n/2) [2 . 1 + (n – 1)  . 1]
number of members in the club are
(A) 15 (B) 25 S2 = (n/2) [2 . 2 + (n – 1)  . 2]
(C) 20 (D) 30
Sm = (n/2) [2 . m + (n – 1)  . m]
Solution: (B)
n \ S1 + S2 + … + Sm
Sn =  [2a + (n – 1) d]
2 n ( n −1)
1 = n (1 + 2 + 3 … + m) + × (1 + 2 + … + m)
Here a = 1st term = 7 years, d = 3 months = year, 2
4
Sn = 250 m ( m + 1) ⎛ n2 − n ⎞
= ⎜ n+
2 ⎝ 2 ⎟⎠
n ⎡ 1⎤
\ 250 = ⎢ 2 × 7 + ( n − 1) × 4 ⎥
2 ⎣ ⎦ m ( m + 1) n ( n + 1) 1
= ⋅ = mn (m + 1) (n + 1).
n ⎛ n + 55 ⎞ 2 2 4
⇒ 250 = ⎜ ⎟ 19. If the first, second and the last terms of an A.P. are
2⎝ 4 ⎠
a, b, c respectively, then the sum is
⇒ 2000 = n2 + 55n
( a + b) ( a + c − 2b)
⇒ n2 + 55n – 2000 = 0 (A) 
2 ( b − a)
⇒ (n – 25) (n + 80) = 0 ⇒ n = 25. ( b + c ) ( a + b − 2c )
(B) 
\ Number of members in the club = 25. 2 ( b − a)
Sequence and Series  7.7

( a + c ) ( b + c − 2a) Solution: (C)


(C) 
2 ( b − a) The first two digit number which when divided by 4
leaves remainder 1 is 4 . 3 + 1 = 13 and last is 4  .  24
(D)  None of these
+ 1 = 97.
Thus, we have to find the sum
Solution: (C)
13 + 17 + 21 + … + 97
We have, first term = a, \ T1 = a
which is an A.P.
Second term = b, \ T2 = b
\ 97 = 13 + (n – 1) . 4 ⇒ n = 22.
Then common difference d = T2 – T1 = b – a 17 × 18 × 35
6
18
+ 14 × 17 × + 49 × 17 = 140 m
2

Also, last term = c. and Sn = [a + l] = 11 × [13 + 97]


c −a+d
⇒ c = a + (n – 1) d ⇒ n = .
d = 11 × 110 = 1210.
(b + c − 2a)
⇒ n= (∵ d = b – a)
(b − a) 22. If there are (2n + 1) terms in A.P., then the ratio of the
12
sum of odd terms and the sum of even terms is
n
∑ a4 k +1 = 416
k =0

\ Sum of n terms Sn = (a + l) = 2 [2a + ( n − 1)4d ] = 416 (A) n : (n + 1) (B)  (n + 1) : n


(C) (n – 1) : n (D)  None of these

20. Four different integers form an increasing A.P. If one Solution: (B)
of these numbers is equal to the sum of the squares of Let the A.P. containing (2n + 1) terms be
the other three numbers, then the numbers are a, a + d, a + 2d, a + 3d, a + 4d, a + 5d, …, a + 2nd.
(A)  – 2, – 1, 0, 1 (B)  0, 1, 2, 3
(C)  – 1, 0, 1, 2 (D)  None of these The sum of odd terms of this A.P.

Solution: (C) = a + (a + 2d) + (a + 4d) + … to (n + 1) terms


( 2n + 1)
S1n = n( n + 1)

Let the numbers be a – d, a, a + d, a + 2d 6

= [2a + (n + 1 – 1) × 2d] = (n + 1) (a + nd)


where a, d ∈ Z and d > 0
Given: (a – d)2 + a2 + (a + d)2 = a + 2d The sum of even terms of this A.P.
2 2
⇒ 2d – 2d + 3a – a = 0 = (a + d) + (a + 3d) + (a + 5d) + … to n terms
n
n1 =

\ d=
13 =
2

[2 (a + d) + (n – 1) × 2d] = n (a + nd)
[2a + 48d ] = 416
2 S2 n = ∑ ( 2n1 ) 2 = 4∑ n12
1

Since d is positive integer, Hence, the required ratio = .

\ 1 + 2a – 6a2 > 0
a12 + a22 + … + a17
2
= 140 n
17 17
∑ Tn = ∑ ( n + 7)2
ARITHMETIC MEAN (A.M.)
n =1 n =1

⇒ <a<
17 17 17
= ∑ n2 + 14∑ n + 49∑1
n =1 n =1 n =1
Single Arithmetic Mean

A number ‘A’ is said to be the single A.M. between two
Since a is an integer,
given numbers a and b provided a, A, b are in A.P.
\ a = 0, For example, since 2, 4, 6 are in A.P., therefore, 4 is
the single A.M. between 2 and 6.
n( n + 1)( 2n + 1) 14 n( n + 1) ⎫
+ + 49n ⎬ n = 17
6 2 ⎭

then d = [1 ± 1] = 1 or 0. Since d > 0,


n-Arithmetic Means
\ d = 1.
Hence, the numbers are – 1, 0, 1, 2. The numbers A1, A2, …, An are said to be the n arithmetic
means between two given numbers a and b provided
21. The sum of all two digit numbers which when divided
a, A1, A2, …, An , b are in A.P.
by 4, yield unity as remainder, is
(A) 1100 (B) 1200 For example, since 2, 4, 6, 8, 10, 12 are in A.P., therefore,
(C)  1210 (D)  None of these 4, 6, 8, 10 are the four arithmetic means between 2 and 12.
7.8  Chapter 7

Inserting Single A.M. between Two given


SOLVED EXAMPLES
Numbers
1
Let a and b be two given numbers and A be the A.M. 23. Between two numbers whose sum is 2 , an even
between them. Then, a, A, b are in A.P. Thus, 6
= 4 n1( n1 + 1)
( 2n1 + 1)
6
number of arithmetic means are inserted. If the sum
A – a = b – A or 2A = a + b, or A = . of these means exceeds their number by unity, then the
number of means are
Inserting n-Arithmetic Means between Two given (A) 12 (B) 10
Numbers (C)  8 (D)  None of these

Let A1, A2, …, An be the n arithmetic means between two Solution: (A)
given numbers a and b. Then a, A1, A2, …, An, b are in A.P. Let 2n arithmetic means be A1, A2, A3, …, A2n between
Now, b = (n + 2)th term of A.P. a and b.
a+b
= a + (n + 2 – 1) d = a + (n + 1) d Then, A1 + A2 + A3 + … + A2n = × 2n
2
13
( 2n + 1) ( 2n + 1)
S = n( n + 1) + 4 n1( n1 + 1) 1
6 6

or d= , where d is common difference of A.P. 13n


A = 20 × 21 ×
41 21
+ 4 × 10 × 11 × = 4410 = 6 × 2n =
2 6
6 6

and A1 = a + d = a + ,
A1 + A2 + A3 + … + A2n = 2n + 1;
Given:
13n
81 41
B = 40 × 41 × + 4 × 20 × 21 × = 22140 + 11480
6 6

A2 = a + 2d = a + 2 , \ 2n + 1 = ; or 12n + 6 = 13n;
6
\ n = 6.
  
\ The number of means = 2n = 2 × 6 = 12.
⎛ b − a⎞
An = a + nd = a + n ⎜
⎝ n + 1 ⎟⎠ 24. If a, b, c are in A.P. and p is the A.M. between a and b

and q is the A.M. between b and c, then
(A) a is the A.M. between p and q
(B) b is the A.M. between p and q
REMEMBER
(C) c is the A.M. between p and q
The sum of n arithmetic means between two given numbers (D)  None of these
is n times the single A.M. between them, i.e. if a and b are Solution: (B)
two given numbers and A1, A2, …, An are n arithmetic means
between them, then  a, b, c are in A.P.,
a + b⎞
A1 + A2 + … + An = n ⎛⎜
⎝ 2 ⎠⎟
\ 2b = a + c (1)
 p is the A.M. between a and b
a+b
QUICK TIPS \ p= (2)
2
■ Sum to n terms of the series of the form  q is the A.M. between b and c
1 1 1 b+c
+ + ... + is \ q= (3)
t1t2 ... t k t2t3 ... t k +1 t n t n +1 ... t n + k −1 2
1 ⎛ 1 1 ⎞ Adding Eq. (2) and (3)
Sn = −
(k − 1)(t2 − t1) ⎝ t1t2 ... t k −1 t n +1 t n +2 ... t n + k −1 ⎟⎠
⎜ a+b b+c a + c + 2b
p + q = + =
2 2 2
■ Sum to n terms of the series of the form
2b + 2b
(t1t2 … tk) + (t2t3 … tk+1) + … + (tn tn+1 … tn+k–1) is = = 2b [Using (1)]
2
1
Sn = (tn tn+1 … tn+k – t0 t1 t2 … tk) p+q
(k + 1)(t2 − t1) \ 2b = p + q or b =
2
Hence, b is the A.M. between p and q.
Sequence and Series  7.9

GEOMETRIC PROGRESSION (G.P.) Solution: (C)


Let A be the first term and R, the common ratio of G.P.
A sequence (finite or infinite) of non-zero numbers in which
Then a = ax = ARx – 1, b = ay = ARy – 1
every term, except the first one, bears a constant ratio with
its preceding term, is called a geometric progression. and c = az = ARz – 1
The constant ratio, also called the common ratio of
the G.P., is usually denoted by r. \ (y – z) log a + (z – x) log b + (x – y) log c
For example, in the sequence, 1, 2, 4, 8, …,
2 4 8 = log ay – z + log bz – x + log cx – y
= = = … = 2, which is a constant.
1 2 4
= log (ay – z . bz – x . cx –y)
Thus, the sequence is a G.P. whose first term is 1 and the
common ratio is 2.
= log [(ARx – 1) y – z × (ARy – 1)z – x . (ARz – 1)x – y]
nth Term of a G.P.
= log [Ay – z + z – x + x – y . R(x – 1) (y – z) + (y – 1) (z – x) + (z – 1)
If a is the first term and r is the common ratio of a G.P., then (x – y)
]
its nth term tn is given by
= log (A0 × R0) = log 1 = 0.
tn = arn – 1
26. If p, q, r are in A.P. and x, y, z are in G.P., then
I M P O R TA N T P O I N T S xq – r . yr – p . zp – q =
(A) 1 (B) 2
■ If a is the first term and r is the common ratio of a G.P., (C)  – 1 (D)  None of these
then the G.P. can be written as
Solution: (A)
a, ar, ar2, …, arn – 1, … (a ≠ 0)
Let d be the common difference of A.P. and R (≠ 0), the
■ If a is the first term and r is the common ratio of a finite
common ratio of G.P., then
G.P. consisting of m terms, then the nth term from the end
is given by arm – n. q = p + d, r = p + 2d
■ The nth term from the end of a G.P. with last term l and
l and y = xR, z = xR2
common ratio r is ( n −1) .
r
■ Three numbers in G.P. can be taken as
a
, a, ar;
so that q – r = – d, r – p = 2d, p – q = – d
r
Four numbers in G.P. can be taken as \ xq – r  . yr – p . zp – q = x– d . (xR)2d . (xR2)– d
a a
, , ar, ar3; = (x– d . x2d . x– d) (R2d × R– 2d )
r3 r
Five numbers in G.P. can be taken as = (x– d + 2d – d ) . (R2d – 2d )
a a
, , a, ar, ar2 = x0 . R0 = 1 × 1 = 1.
r2 r
■ Three numbers a, b, c are in G.P. if and only if 27. If, in a G.P., the (p + q)th term is a and the (p – q)th
b c term is b, then pth term is
= i.e. if and only if b2 = ac.
a b (A) (ab)1/3 (B)  (ab)1/2
1/4
(C) (ab) (D)  None of these
Solution: (B)
SOLVED EXAMPLES Let the G.P. be x, xy, xy2, …
Then tp + q = xy p + q – 1 = a (1)
25. If a, b, c are respectively the xth, yth and zth terms of a
p–q–1
G.P., then and tp – q = xy  = b (2)
( y – z) log a + (z – x) log b + (x – y) log c = Dividing Eqs (1) by (2),
(A) 1 (B) – 1 a
(C)  0 (D)  None of these y2q = ;
b
7.10  Chapter 7

1 t n+1
⎛ a ⎞ 2q or tn = 2 .  ,( common ratio = r)
\ y= ⎜ ⎟ 1− r
⎝ b⎠
t 1− r
\ = n +1 ; or = r
From Eq. (1), tn 2
p + q −1
⎛ b⎞ 1
x = a ⎜ ⎟
2q or 1 – r = 2r ; \ r = .
⎝ a⎠ 3

30. The three numbers a, b, c between 2 and 18 are such
p + q −1
p −1 that their sum is 25; the numbers 2, a, b are consec-
⎛ b⎞ 2q
⎛ a⎞
\ tp = xy p – 1 = a  . ⎜ ⎟ 2q
utive terms of an A.P. and the numbers b, c, 18 are
⎝ a⎠ ⎜⎝ b ⎟⎠
consecutive terms of a G.P. The three numbers are
p + q −1 p −1 p + q −1 p −1 (A)  3, 8, 14 (B)  2, 9, 14
= 1−
2q
+
2q 2q

2q (C)  5, 8, 12 (D)  None of these
a ×b
1 1 Solution: (C)
= a2b2 = ab
We have, a + b + c = 25 (1)
28. In a set of four numbers the first three are in G.P. and
 2, a, b are in A.P., \ 2a = 2 + b (2)
the last three are in A.P. with a common difference 6.
If the first number is same as the fourth, the four num-
 b, c 18 are in G.P., \ c2 = 18b(3)
bers are
(A)  3, 9, 15, 21 (B)  1, 7, 13, 19 From Eq. (1) and (2),
(C)  8, – 4, 2, 8 (D)  None of these 3b + 2c = 48; or 3b = 48 – 2c
Solution: (C) \ From Eq. (3),
Let the last three numbers in A.P. be b, b + 6, b + 12
and the first number be a. c2 = 6 (48 – 2c) = 288 – 12c
Hence the four numbers are a, b, b + 6, b + 12
or c2 + 12c – 288 = 0;
Given, a = b + 12 (1)
or c2 + 24c – 12c – 288 = 0
and a, b, b + 6 are in G.P. i.e., b2 = a (b + 6)
or (c + 24) (c – 12) = 0;
or b2 = (b + 12) (b + 6) ( a = b + 12)

or 18b = – 72 \ b = –4, \ c = 12, as c ≠ – 24.


\ From Eq. (3), b = 8 and from (2), a = 5.
From Eq. (1),
31. Let an be the nth term of the G.P. of positive numbers.
a = – 4 + 12 = 8. 100 100

Hence, the four numbers are 8, –4, 2 and 8. Let ∑ a2n = a and ∑ a2n−1 = b, such that a ≠ b, then
n =1 n =1
29. If the first term of an infinite G.P. is 1 and each term is the common ratio is
twice the sum of the succeeding terms, then the com- α β
(A)  (B) 
mon ratio is β α
1 2
(A)  (B)  α β
3 3 (C)  (D) 
β α
3
(C)  (D)  None of these
4 Solution: (A)
Solution: (A) Let a be the first term and r, the common ratio of the
given G.P. Then
Let the G.P. be 1, r, r2, r3, … 100
Given: tn = 2 (tn + 1 + tn + 2 + tn + 3 + … to ∞) a= ∑ a2n ⇒ a = a2 + a4 + … + a200
n =1
Sequence and Series  7.11

⇒ a = ar + ar3 + … + ar199 Sum of an Infinite G.P. when | r | < 1


2 4 198
⇒ a = ar (1 + r + r + … + r )(1) The sum of an infinite G.P. with first term a and common
100 ratio r is
and b= ∑ a2n−1 ⇒ b = a1 + a3 + … + a199
S∞ =
a
; when | r | < 1, i.e., – 1 < r < 1.
n =1 1− r
⇒ b = a + ar2 + … + ar198
⇒ b = a (1 + r2 + … + r198)(2) ERROR CHECK
From Eqs (1) and (2), we get
If r ≥ 1, then S∞ does not exist.
α
= r.
β
SOLVED EXAMPLES
⎛ 1 1 1⎞
32. If a, b, c are in G.P., then a2 b2 c2 ⎜ 3 + 3 + 3 ⎟ =
⎝a b c ⎠
33. The sum Sn to n terms of the series
(A) a + b + c (B)  ab + ac + bc 1 3 7 15
(C) a3 + b3 + c3 (D)  None of these + + + + ... is equal to
2 4 8 16
Solution: (C) (A) 2n – n – 1 (B)  1 – 2– n
–n
Since a, b, c are in G.P. (C) 2 + n – 1 (D)  2n – 1
b c Solution: (C)
\ = ⇒ b2 = ac (1)
a b We have,
⎛ 1⎞ ⎛ 1⎞ ⎛ 1⎞ ⎛ 1⎞
⎛ 1 1 1⎞ Sn = ⎜1 − ⎟ + ⎜ 1 − ⎟ + ⎜ 1 − 3 ⎟ + ... + ⎜1 − n ⎟
Now, a2 b2 c2 ⎜ + + ⎟ ⎝ 2⎠ ⎝ 4⎠ ⎝ 2 ⎠ ⎝ 2 ⎠
⎝a b
3 3
c3 ⎠
⎛1 1 1⎞
2 2 2 2 2 2
= n−⎜ + + ... + n ⎟
= b c +a c +a b ⎝ 2 22 2 ⎠
a b c
1 ⎡ ⎛ 1⎞
n⎤
⎢1 − ⎥
ac ⋅ c 2 (b 2 )2 a2 ⋅ ac = 2 ⎢⎣ ⎜⎝ 2 ⎟⎠ ⎥⎦ = n – 1 + 2–n
= + +  [Using (1)] n−
a b c 1
1−
= a3 + b3 + c3. 2
35. The minimum number of terms of the series
1 + 3 + 9 + 27 + …
Sum of n terms of a G.P.
so that the sum may exceed 1000, is
The sum of first n terms of a G.P. with first term a and com- (A) 7 (B) 5
mon ratio r(≠ 1) is given by, (C)  3 (D)  None of these

a ( r n − 1) a(1 − r n ) Solution: (A)


Sn = =
r −1 1− r Let, the sum of n terms exceeds 1000.
1(1 − 3n ) 3n − 1
Then > 1000; or > 1000
1− 3 2
QUICK TIPS
or 3n > 2001; but 36 = 729 and 37 = 2187;
■ When r = 1,
Sn = a + a + a + … upto n terms = na. \ n > 6; but n is a positive integer;
If l is the last term of the G.P., then
\ n = 7, 8, 9, …

lr − a
Sn = r≠1 \ The minimum number of terms = 7.
r −1
7.12  Chapter 7

Solution: (A)
x x2 x4
36. The sum of series + + + ... to n n
1− x2 1− x4 1 − x8 n + 2 ⎛ 1⎞ 2( n + 1) − n ⎛ 1 ⎞
We have, tn = ⋅⎜ ⎟ = ⎜ ⎟
infinite terms, if | x | < 1 is n ( n + 1) ⎝ 2 ⎠ n( n + 1) ⎝ 2 ⎠
1 x n −1 n
(A)  (B)  1 ⎛ 1⎞ 1 ⎛ 1⎞
1− x 1− x = ⎜ ⎟ − ⎜ ⎟
n ⎝ 2⎠ n + 1 ⎝ 2⎠
1 x n
(C)  (D) 
1+ x 1+ x \ Sn = ∑ tn
n =1

Solution: (B)
⎡1 ⎛ 1 ⎞ 1 ⎛ 1 ⎞ 1 ⎤ ⎡ 1 ⎛ 1 ⎞ 1 1 ⎛ 1 ⎞ 2 ⎤
2n − 1
1+ x 2n − 1
−1 = ⎢ ⎜ ⎟ − ⎜ ⎟ ⎥+⎢ ⎜ ⎟ − ⎜ ⎟ ⎥
⎢⎣1 ⎝ 2 ⎠ 2 ⎝ 2 ⎠ ⎥⎦ ⎢⎣ 2 ⎝ 2 ⎠ 3 ⎝ 2 ⎠ ⎥⎦
x
We have, tn = n −1
= 2n − 1 n −1

1− x2 (1 + x ) (1 − x 2 )
⎡1 ⎛ 1⎞ n −1 1 ⎛ 1⎞ ⎤
n

1 1  + ... + ⎢ ⎜ ⎟ − ⎜ ⎟ ⎥
= − ⎢⎣ n ⎝ 2 ⎠ n + 1 ⎝ 2 ⎠ ⎥⎦
2n − 1 n
1− x 1− x2
1
n = 1−
Therefore, Sn = ∑ tn ( n + 1)2 n
n =1
⎡⎛ 1 1 ⎞ ⎛ 1 1 ⎞ 38. If (1 – y) (1 + 2x + 4x2 + 8x3 + 16x4 + 32x5) = 1 – y6,
 ⎢⎜ 1 − x −
= 2⎟
+ − (y ≠ 1), then a value of y/x is
⎜ 2
1 − x 4 ⎟⎠
⎣⎝ 1− x ⎠ ⎝ 1 − x
1
1 ⎞⎤ (A)  (B)  2
⎛ 1 2
+ ... + − n ⎟⎥
⎜ n −1
⎝1 − x2 1 − x 2 ⎠ ⎥⎦ 1
 (C)  (D)  4
4
1 1
= − Solution: (B)
1 − x 1 − x2
n

We can rewrite the given expression as


\ The sum to infinite terms 6
(1 – y) [1 − ( 2 x ) ] = 1 – y6,
1 1 − 2x
=
lim S n = −1
n→∞ 1− x one of the possible values of y is clearly 2x. Therefore,
⎛ n
⎞ one of the possible values of y/x is 2.
⎜⎝∵ nlim x 2 = 0, as | x | < 1⎟
→∞ ⎠
 ⎛1 1 1 ⎞
log 5 ⎜ + + + ...∞⎟
⎝ 4 8 16 ⎠
x 39. The value of (0.2) is
=
1− x (A) 1 (B) 2
1
(C)  (D)  4
37. The sum to n terms of the series 2
2 3
3 ⋅ 1 + 4 ⎛⎜ 1 ⎞⎟ + 5 ⎛⎜ 1 ⎞⎟ + ... is Solution: (D)
1.2 2 2.3 ⎝ 2 ⎠ 3.4 ⎝ 2 ⎠ 1
1 1 1 4 1
1 1 We have, + + + ...∞ = = = 2–1.
(A) 1 − 1−
(B)  4 8 16 1 2
1−
( n + 1)2 n ( n + 1)2n −1 2
1 ⎛1 1 1 ⎞
log 5 ⎜ + + + ...∞⎟ log 5 2−1
(C) 1 − (D)  None of these ⎝ 4 8 16 ⎠ ⎛ 1⎞
( n − 1)2 n − 1 \ (0.2) = ⎜ ⎟
⎝ 5⎠
Sequence and Series  7.13

−1
log5 2 ⎛ A − 1⎞ ⎛ B − 1⎞
= (5−1 )1/ 2 \ a log r = log ⎜ and b log r = log ⎜
⎝ A ⎟⎠ ⎝ B ⎟⎠
2
= 52 log 5
2
= 5log5 ( 2 ) = 22 = 4 ⎛ A − 1⎞
log ⎜
a ⎝ A ⎟⎠
40. i – 2 – 3i + 4 … to 100 terms = \ =
b ⎛ B − 1⎞
log ⎜
(A)  50 (1 – i) (B) 25i ⎝ B ⎟⎠
(C)  25 (1 + i) (D)  100 (1 – i)
⎛ A − 1⎞
=
log B − 1 ⎜
Solution: (A) ⎝ A ⎟⎠
B
S = i + 2i2 + 3i3 +, …, + 100i100
43. Sum of the series : (1 + x) + (1 + x + x2) +

S . i = i2 + 2i3 +, …, + 99i100 + 100i101 (1 + x + x2 + x3) + … upto n terms is

1 ⎡ x 2 (1 − x n ) ⎤
\ S(1 – i) = (i + i2 + i3 +, …, + 100 terms) – 100i101 (A)  ⎢n − ⎥
1− x ⎣ 1− x ⎦
i (1 − i100 ) i (1 − 1)
= − 100 i = − 100 i = – 100 i 1 ⎡ x 3 (1 − x n ) ⎤
1− i 1− i (B)  ⎢ n − ⎥
1− x ⎣ 1− x ⎦
− 100i − 100 i (1 + i )
\ S= = = 50 (1 – i) 1 ⎡ x(1 − x n ) ⎤
1− i 2 (C)  ⎢n − ⎥
1− x ⎣ 1− x ⎦
41. The largest value of the positive integer k for which (D)  None of these
nk + 1 divides 1 + n + n2 + … + n127 is divisible by
Solution: (A)
(A) 8 (B) 16
(C) 32 (D) 64 We have,
(1 + x) + (1 + x + x2) + (1 + x + x2 + x3)
Solution: (D)
 + … upto n terms
We have, 1 − x 2 1 − x3 1 − x 4
n128 − 1 = + + + … to n terms
1 + n + n2 + … + n127 = 1− x 1− x 1− x
n −1
1
= [(1 + 1 + 1 + … n terms)
( n64 − 1) ( n64 + 1) 1− x
=
n −1  – (x2 + x3 + x4 + … to n terms)]
= (1 + n + n + … + n63) (n64 + 1)
2 1 ⎡ x 2 (1 − x n ) ⎤
= ⎢n − ⎥
1 − x ⎢⎣ 1 − x ⎥⎦
\ k = 64 which is divisible by 8, 16, 32 and 64.

42. If A = 1+ ra + r2a + r3a + … ∞ and


Properties of G.P.
a
If B = 1 + rb + r2b + r3b + … ∞, then is equal to 1. If a1, a2, a3, … are in G.P., then
b (a) a1k, a2k, a3k, … are also in G.P.
(A) logB A (B)  log1 – B (1 – A) a a a
(b) 1 , 2 , 3 , … are also in G.P.
⎛ A − 1⎞ k k k
(C) log B − 1 ⎜ (D)  None of these
⎝ A ⎟⎠ 1 1 1
B (c) , , , … are also in G.P.
a1 a2 a3
Solution:(C)
(d) ak1, ak2, a3k … are also in G.P.
1 1 1 A −1 2. If a1, a2, a3, … and b1, b2, b3, … are two G.P.s, then
A= ⇒ 1 – ra = ⇒ ra = 1 – =
1− r a A A A (a) a1b1, a2b2, a3b3, … are also in G.P.
a a a
1 1 1 B −1 (b) 1 , 2 , 3 … are also in G.P.
B = ⇒ 1 – rb = ⇒ rb = 1 – = b1 b2 b3
1− r b B B B
(c) a1 ± b1, a2 ± b2, a3 ± b3, … may not be in G.P.
7.14  Chapter 7

3. If a1, a2, a3, …, an are in G.P., then or (r – 4) (r2 + 5r + 21) = 0;


(a) a1 an = a2 an – 1 = a3 an – 2 = . . .
−5 ± 25 − 84
(b) ar =
ar − k ar + k , 0 ≤ k ≤ n – r. \ r = 4,
2
4. If a1, a2, a3, …, is a G.P. of positive terms, then log a1,
\ Real value of r is 4. Putting this value in (2),
log a2, log a3, …, is also an A.P. and vice-versa.
5. If a1, a2, a3, …, an–2, an–1, an are in G.P., then a1an = 3
a=× 4 = 3.
(a2an–1) = (a3an–2) = … = aran–(r–1) 4
In a G.P., the product of two terms equidistant from \ The three numbers are, 3, 3 × 4, 3 × 42,
the beginning and end is a constant and is equal to the i.e., 3, 12, 48.
product of first term and last term.
6. If first term of a G.P. of n terms is a and last term is l,
GEOMETRIC MEAN (G.M.)
then the product of all terms of the G.P. is (al)n/2.
7. If there be n quantities in G.P. whose common ratio is Single Geometric Mean
r and Sm denotes the sum of the first m terms, then the
r A number G is said to be the single geometric mean between
sum of their product taken two by two is Sn Sn–1. two given numbers a and b if a, G, b are in G.P.
r +1
For example, since 2, 4, 8 are in G.P., therefore 4 is
8. If a x , a x , a x , ..., a x are in G.P., then x1, x2, x3, …, xn
1 2 3 n

the G.M. between 2 and 8.


will be in A.P.
9. Product of a G.P. n-Geometric Means
 Case I: If number of terms is odd, then Product of
terms = (middle term)No. of terms The numbers G1, G2, …, Gn are said to be the n geometric
Case II: If number of terms is even, then Product of means between two given positive numbers a and b if a, G1,
terms G2, …, Gn , b are in G.P.
=(Geometric mean of middle terms)No. of For example, since 1, 2, 4, 8, 16 are in G.P., therefore
terms 2, 4, 8 are three geometric means between 1 and 16.

Inserting Single G.M. between Two given


REMEMBER Numbers
Equal non zero numbers are in G.P. Let a and b be two given positive numbers and G be the
G.M. between them. Then a, G, b are in G.P. Thus
G b
= or G2 = ab, or G = ab (∵ G > 0)
SOLVED EXAMPLE a G

44. If the sum of three numbers in G.P. is 63 and the prod- Inserting n-Geometric Means between Two given
3 Numbers
uct of the first and the second term is of the third
term, then the numbers are 4
Let G1, G2, G3, …, Gn be the n geometric means between
(A)  3, 12, 48 (B)  4, 12, 36 two given numbers a and b. Then, a, G1, G2, G3, …, Gn, b
(C)  2, 10, 50 (D)  None of these are in G.P.
Solution: (A) Now, b = (n + 2)th term of G.P.
Let the three numbers be a, ar, ar2 = arn + 1, where r is the common ratio
Given a + ar + ar2 = 63 (1) 1
n+1 b ⎛ b ⎞ n +1
3 3 or r = or r = ⎜ ⎟
and a . ar =  . ar2 or a = r (2) a ⎝ a⎠
4 4
1
3 3 . 3 . 2 ⎛ b ⎞ n +1
Putting in (1), r+ r   r + r   r = 63 and G1 = ar = a ⎜ ⎟
4 4 4 ⎝ a⎠

or r3 + r2 + r – 84 = 0
Sequence and Series  7.15

2
i i i
⎛ b ⎞ n +1 234 234 26
2
G2 = ar = a ⎜ ⎟ For example, 0. 234 = = =
⎝ a⎠ 103 − 1 999 111

i
   ■ If R = 0.X Y and x denotes the number of digits in X and
2 y denotes the number of digits in Y, then
⎛ b ⎞ n +1
n
Gn = ar = a ⎜ ⎟ R= XY − X
⎝ a⎠
10 x + y − 10 x
i i
For example, if R = 0.4362, then
QUICK TIPS
4362 − 43 4319
R= =
■ The product of n geometric means between two given 104 − 102 9900
numbers is nth power of the single G.M. between them i.e.
if a and b are two given numbers and G1, G2, …, Gn are n
geometric means between them, then

G1G2G3 … Gn = ( ab)n. SOLVED EXAMPLES


■ If A and G are respectively arithmetic and geometric means
45. If two geometric means g1 and g2 and one arithmetic
between two positive numbers a and b then
mean A be inserted between two numbers, then
(A) A > G
g2 g2
1 + 2 =
(B)  the quadratic equation having a, b as its roots is g2 g1
x2 – 2Ax + G2 = 0 (A) 4A (B) 
3A
(C) 2A (D) 
A
(C)  the two positive numbers are A ± A2 − G 2 .
Solution: (C)
■ If number of terms of any A.P./G.P. is odd, then A.M./G.M.
of first and last terms is middle term of series. Let the two numbers be a and b.
■ If number of terms of any A.P./G.P. is even, then A.M./G.M. a+b
of middle two terms is A.M./G.M./H.M. of first and last \ A= or 2A = a + b (1)
2
terms respectively.
th th Again, a, g1, g2, b are in G.P.
■ If p , q and rth terms of a G.P. are in G.P., then p, q, r are
in A.P. g1 g b
\ = 2 =
■ If a, b, c are in A.P. as well as in G.P. then a = b = c. a g1 g2
a b c

■ If a, b, c are in A.P., then x , x , x will be in G.P. (x ≠ ±1) 2
g1 g g
■ Value of recurring decimal Now = 2 ⇒ 1 = a
a g1 g2
i b
If R = 0.bbb … = 0. b, then R = g2 g2

10′ − 1 b
and = ⇒ 2 = b
5
g1 g2 g1
i 5
For example, 0.5 = =
10′ − 1 9 g12 g2
\ a+b= + 2 (2)
g2 g1
i i ab
If R = 0.ab ab ab … = 0. ab, then R =

102 − 1 \ From Eqs (1) and (2), we get
i i 37 37 g12 g2
For example, 0. 37 = = . 2A = + 2
102 − 1 99 g2 g1

i i iabc
■ If R = 0.abc abc abc … , 0. abc then R = 3
and so 46. Let a = 1 . 2 . 3 . 4 . 5. Then
on. 10 −1
(A) 55 ≥ a (B)  35 ≥ 5!
5
(C) 5 ≥ 6a (D)  None of these
7.16  Chapter 7

Solution: (A, B) Solution: (B)


Since A.M. > G.M. The required sum
1+ 2 + 3 + 4 + 5 = (1 + 2 + 3 + … + 199) – (3 + 6 + 9 + … + 198)
5
⇒ ≥ 1⋅ 2 ⋅ 3 ⋅ 4 ⋅ 5
5  – (5 + 10 + 15 + … + 195) + (15 + 30 + 45 +
⇒ 3 ≥ 5 5
a ⇒ 3 ≥ a = 5!  … + 195)
199 66 39
Also, 5 ≥ 1 . 2 . 3 . 4 . 5 = a.
5 = (1 + 199) – (3 + 198) – (5 + 195)
2 2 2
13
47. If a, b, c are positive then the minimum value of + (15 + 195)
2
alog b – log c + blog c – log a + clog a – log b is
(A) 3 (B) 1 = 199 × 100 – 33 × 201 – 39 × 100 + 13 × 105
(C) 9 (D) 16 = 10732
Solution: (A) 49. Sum to 20 terms of the series 1.32 + 2.52 + 3.72 + … is
Since A.M. ≥ G.M. (A) 178090 (B) 168090
alog b − log c + blog c − log a + c log a − log b (C)  188090 (D)  None of these
\
3 Solution :(C)
We have,
alog b − log c ⋅ blog c − log a ⋅ c log a − log b (1)
3

tn = [nth term of 1, 2, 3, …] × 
Let x = alog b – log c . blog c – log a . clog a – log b [nth term of 3, 5, 7, …]2
⇒ log x = (log b – log c) log a + (log c – log a) log b = n (2n + 1)2 = 4n3 + 4n2 + n.
 + (log a – log b) log c
\ Sn = S tn = 4 S n3 + 4 S n2 + S n
= 0 ⇒ x = 1. 2
⎡ n ( n + 1) ⎤ n ( n + 1) ( 2n + 1) n ( n + 1)
\ From (1), = 4 .  ⎢ ⎥ + 4⋅ +
⎣ 2 ⎦ 6 2
alog b – log c + blog c – log a + clog a – log b ≥ 3.
2
= n2 (n + 1)2 + n (n + 1) (2n + 1)
3 1
+ n (n + 1);
SOME SPECIAL SEQUENCES 2
2
\ S20 = 202 . 212 + × 20 . 21 . 41 
1. The sum of first n natural numbers 3
1
n ( n +1) +  . 20 . 21 = 188090
Sn = 1 + 2 + 3 + … + n =
. 2
2
50. Number of terms in the sequence 1, 3, 6, 10, 15, …,
2. The sum of squares of first n natural numbers
5050 is
n ( n + 1) ( 2n + 1) (A) 50 (B) 75
Sn2 = 12 + 22 + 32 + … + n2 = .

6 (C) 100 (D) 125
3. The sum of cubes of the first n natural numbers
Solution: (C)
2
3 3 ⎡ n ( n + 1) ⎤
3 3 3
Sn = 1 + 2 + 3 + … + n = ⎢
⎥ . Let S = 1 + 3 + 6 + 10 + 15 +, …, + tn(1)
⎣ 2 ⎦

SOLVED EXAMPLES then  S = 1 + 3 + 6 + 10 +, …, + tn – 1 + tn (2)


(1) – (2) ⇒ 0 = (1 + 2 + 3 + 4 + … to n terms) – tn
48. The sum of all natural numbers less than 200, that are n ( n +1)
divisible neither by 3 nor by 5, is ⇒ tn =
2
(A) 10730 (B) 10732
n ( n +1)
(C)  15375 (D)  None of these Given, 5050 = ⇒ n2 + n – 10100 = 0
2
Sequence and Series  7.17

−1 ± 1 + 40400 QUICK TIPS


⇒ n=
2
If nth term of a sequence is
−1 ± 40401 Tn = an3 + bn2 + cn + d,
=
2 then the sum of n terms is given by,
−1 ± 201 Sn = STn = aSn3 + bSn2 + cSn + Sd,
= = – 101,100
2 which can be evaluated using the above results.

\ n = 100. ( n is a positive integer)


n i j
51. ∑∑∑ 1 = SOLVED EXAMPLES
i =1 j =1 k =1

n ( n + 1) ( 2n + 1) n ( n +1) 9 13 17
(A)  (B)  53. The sum of the series 2 + 3 + 4 + ... to
6 2 infinite terms, is 5 .2.1 5 .3.2 5 .4.3
2 2 1
⎛ n ( n + 1) ⎞ n ( n + 1) ( n + 2) (A)  (B) 
(C)  ⎜ (D)  5 5
⎝ 2 ⎟⎠ 6
(C)  1 (D)  None of these
Solution: (D)
Solution: (B)
We have,
The general term of the series is
n i j n i
4r + 1
∑∑∑ 1 = ∑∑ j tr = r , where r ≥ z
i =1 j =1 k =1 i =1 j =1 5 ⋅ r ( r − 1)
n
i (i + 1) 1 ⎡ n n ⎤ 5r − ( r − 1) 1 1
∑ 2 = 2 ⎢∑ i 2 + ∑ i ⎥
= = r
5 ⋅ r ( r − 1)
=
5 r −1
( r − 1)
− r
5 ⋅r
i =1 ⎣ i =1 i =1 ⎦ ∞
\ ∑ tr =
1 ⎡ n ( n + 1) ( 2n + 1) n ( n + 1) ⎤ r=2
= ⎢ +
2 ⎣ 6 2 ⎥⎦
⎛ 1 1 ⎞ ⎛ 1 1 ⎞ ⎛ 1 1 ⎞
 = ⎜ 1 − 2 ⎟ +⎜ 2 − 3 ⎟ +⎜ 3 − 4 ⎟
n ( n + 1) ( n + 2) ⎝ 5 .1 5 .2 ⎠ ⎝ 5 .2 5 .3 ⎠ ⎝ 5 .3 5 .4 ⎠
=
6  + … to infinity
1
52. The sum of the products of the 2n numbers ± 1, ± 2, = ( terms tend to zero as n → ∞)
5
± 3, …, ± 2n taking two at a time is
n ( n +1) n ( n + 1) ( 2n + 1) 54. For any odd integer n ≥ 1,
(A) – (B) 
2 6 n3 – (n – 1)3 + … + (–1)n–113 =
n ( n + 1) ( 2n + 1) 1
(C) – (D)  None of these (A)  (n – 1)2 (2n – 1)
6 2
1
Solution: (C) (B)  (n – 1)2 (2n – 1)
4
We have, (1 – 1 + 2 – 2 + 3 – 3 + … + n – n)2 1
(C)  (n + 1)2 (2n – 1)
2
12 + 12 + 22 + 22 + … + n2 + n2 + 2S,
=
1
(D)  (n + 1)2 (2n – 1)
where S is the required sum 4
⇒ 0 = 2 (12 + 22 + … + n2) + 2S Solution: (D)
n ( n + 1) ( 2n + 1) Since n is an odd integer (–1)n–1 = 1 and n – 1, n – 3,
⇒ S = – (12 + 22 + … + n2) = – n – 5 etc., are even integers. We have
6
7.18  Chapter 7

n3 – (n – 1)3 + (n – 2)3 – (n – 3)3 + … + (–1)n–113 In other words, when | r | < 1 the sum to infinity of an

a dr
= n3 + (n – 1)3 + (n – 2)3 + … + 13 arithmetico-geometric series is S∞ = + .
1 − r (1 − r ) 2
 – 2[(n – 1)3 + (n – 3)3 + … + 23]
METHOD FOR FINDING SUM OF A.G. SERIES
= n3 + (n – 1)3 + (n – 2)3 + … + 13
⎡⎛ n − 1⎞ 3 ⎛ n − 3 ⎞ 3 ⎤ Method of Differences
3
 – 2 × 2 ⎝ 2 ⎟⎠ + ⎜⎝ 2 ⎟⎠ + ... + 1 ⎥
3 ⎢⎜
Suppose a1, a2, a3, … is a sequence such that the sequence
⎢⎣ ⎥⎦
a2 – a1, a3 – a2, … is either an A.P. or a G.P. The nth term
[ n –1, n – 3 are even integers] ‘a’n of this sequence is obtained as follows:
2 2
⎡ n( n + 1) ⎤ ⎡ 1 ⎛ n − 1⎞ ⎛ n − 1 ⎞ ⎤ S = a1 + a2 + a3 + … + an–1 + an
= ⎢ ⎥ − 16 ⎢ ⎜ ⎟⎜ + 1⎟ ⎥
⎣ 2 ⎦ ⎣2 ⎝ 2 ⎠ ⎝ 2 ⎠⎦ S = a1 + a2 + … + an–1 + an
2 2
1 2 ( n − 1) ( n + 1) ⇒ an = a1 + [a2 – a1) + (a3 – a2) + … + (an – an–1)]
= n (n + 1)2 – 16
4 16 × 4 Since the terms within the brackets are either in an A.P. or
1 1 in a G.P., we can find the value of an, the nth term, we can
= (n + 1)2 [n2 – (n – 1)2] = (n + 1)2(2n – 1).
4 4 now find the sum of the n terms of the sequence as
n

ARITHMETICO-GEOMETRIC PROGRESSION
S= ∑ ak
k =1
(A.G.P.)
If a1, a2, a3, …, an, … is an A.P. and b1, b2, …, bn, … is G.P. SOLVED EXAMPLES
then the sequence a1b1, a2b2, a3b3, …, anbn, … is said to be
an arithmetico-geometric sequence. 55. Find the sum to n terms of the series:
Thus, the general form of an arithmetico geometric
(A) 2 + 5 + 10 + 17 + …
sequence is a, (a + d) r, (a + 2d) r2, (a + 3d) r3, …
(B) 3 + 5 + 9 + 17 + …
nth term of A.G.P. Solution
From the symmetry we obtain that the n term of this th (A) Here, the difference in consecutive terms are 3, 5,
sequence is [a + (n – 1)d] rn–1. 7, … which are in A.P.
Also, let a, (a + d) r, (a + 2d) r2, (a + 3d) r3, … be an Let Sn = 2 + 5 + 10 + 17 + … + tn–1 + tn
arithmetico-geometric sequence. Shifting every term one place to the right
Then, a + (a + d) r + (a + 2d) r2 + (a + 3d) r3 + … is Sn = 2 + 5 + 10 + … + tn–1 + tn
an arithmetico-geometric series. Subtracting, we get 0 = (2 + 3 + 5 + 7 + … to n
terms) – tn
Sum of A.G.P. ⇒ tn = 2 + [3 + 5 + 7 + … to (n – 1) terms]
n −1
1. Sum to n terms: The sum of n terms of an arithmeti- =2+ [2 × 3 + (n – 2) × 2]
co-geometric sequence a, (a + d) r, (a + 2d) r2, (a + 3d) 2
r3, … is given by, n −1
=2+ (2n + 2) = 2 + (n – 1) (n + 1)
2
⎧ a (1 − r n −1 ) [a + ( n − 1)d ]r n = 2 + (n2 – 1) = n2 + 1.
⎪ + dr − , when r ≠ 1
Sn = ⎪⎨1 − r (1 − r )2 1− r Putting n = 1, 2, 3, … , n and adding, we get
n
n( )( )
⎪n Sn = ∑ k 2 + n = n + 1 2n + 1 + n
⎪⎩ 2 [2a + ( n − 1)d ], when r = 1 k =1 6
2. Sum to infinite terms: Let | r | < 1. Then rn, rn–1 → 0 n n
= (2n2 + 3n + 1 + 6) = (2n2 + 3n + 7)
as n → ∞ and it can also be shown that n . rn → 0 as 6 6
a dr (B) Here the differences of consecutive terms are 2, 4,
n → ∞. So, we obtain that Sn → + , as 8, … which are in G.P.
1 − r (1 − r )2
Let Sn = 3 + 5 + 9 + 17 + … + tn–1 + tn
n → ∞.
Sequence and Series  7.19

Shifting every term one place to the right 57. The sum of first n terms of the series
Sn = 3 + 5 + 9 + … + tn–1 + tn 1 . 1! + 2 . 2! + 3 . 3! + 4 . 4! + … is
Subtracting, we get 0 = (3 + 2 + 4 + 8 + … to n terms) (A) (n + 1)! – 1 (B)  n! – 1
– tn (C) (n – 1)! – 1 (D)  None of these
⇒ tn = 3 + [2 + 4 + 8 + … to (n – 1) terms]
n−1
Solution: (A)
= 3 + 2 ( 2 − 1) = 3 + 2n – 2
2 −1 Let Sn = 1 . 1! + 2 . 2! + 3 . 3! + 4 . 4!
= 1 + 2n  + … + n × n!
Putting n = 1, 2, 3, … , n, we get
t1 = 1 + 21 ⇒ Sn = (2 – 1) 1! + (3 – 1) 2! + (4 – 1) 3!
t2 = 1 + 22
 + (5 – 1) 4! + … + [(n + 1) – 1] n!
t3 = 1 + 23
 = (2 . 1! – 1!) + (3 . 2! – 2!) + (4 . 3! – 3!)
tn = 1 + 2n
Adding column-wise, we get  + (5 . 4! – 4!) + … + [(n + 1) n! – n!]
Sn = n + (2 + 22 + 23 + … + 2n)
n = (2! – 1!) + (3! – 2!) + (4! – 3!) + (5! – 4!)
= n + 2 ( 2 − 1) = 2n+1 + n – 2

2 −1  + … + [(n + 1)! – n!]
56. Sum to infinity of the series = (n + 1)! – 1! = (n + 1)! – 1.
2 5 2 11
− + − +… is
3 6 3 24 58. If a, b, c are digits, then the rational number repre-
sented by 0 . cababab … is
4 1
(A)  (B)  99c + ab 99c + 10 a + b
9 3 (A)  (B) 
990 99
2
(C)  (D)  None of these 99c + 10 a + b
9 (C)  (D)  None of these
990
Solution: (C)
Solution: (C)
2 5 2 11
Let S= − + − + ... to ∞(1) Let R = 0 . cababab…
3 6 3 24
1 ⇒ 102R = ca . bababa …
Multiplying both sides by – , the common ratio of
G.P. 2
and 104R = caba . baba …
1 2 5 8
– S=– + − + ... to ∞(2) ⇒ (104 – 102) R = caba – ca
2 6 12 24
caba − ca 1000c + 100a + 10b + a − 10c − a
Subtracting Eqs (2) from (1), we have ⇒ R = =
9900 9900
3 2 3 3 3
S= − + − + ... to ∞ 99c + 10 a + b
2 3 6 12 24 =
990
2 ⎛1 1 1 ⎞
= − − + + ... to ∞⎟
3 ⎜⎝ 2 4 8
59. The sum of first n terms of the series
⎠ 2
1 12 + 2.22 + 32 + 2.42 + 52 + 5.62 + … is n ( n +1)
2 2
2 2 1 1 when n is even. When n is odd the sum is
= − = − =
3 ⎛ 1⎞ 3 3 3 n2 ( n + 1)
1− ⎜ − ⎟ (A)  (B)  n ( n +1) 2
⎝ 2⎠ 2 2
1 2 2 2
\ S= × = ⎡ n ( n + 1) ⎤ n ( n +1)
3 3 9 (C)  ⎢ ⎥ (D) 
⎣ 2 ⎦ 2
7.20  Chapter 7

Solution: (A) Solution: (D)


When n is odd, last term will be n2, \ then the sum is
Let S = 1 + 2 . 2 + 3 . 22 + 4 . 23 + 5 . 24 
2 2 2 2 2 2
1 + 2.2 + 3 + 2.4 + 5 + 2.6 + … + 2 (n – 1) + n 2 2
 + … + 100 . 299
( n − 1) n2 ⎡ n ( n + 1) 2 ⎤ \ 2S = 1 . 2 + 2 . 22 + 3 . 23 + … + 99 . 299 
= + n2 ⎢ Replacing n by n − 1 in ⎥
2 2  + 100 . 2100
⎣ ⎦
Substracting, we get
3 2 2 n3 + n2 n2 ( n + 1)
= n − n + 2n = = – S = 1 + 1 . 2 + 1 . 22 + … + 1 . 299 –100 . 2100
2 2 2
= (1 + 2 + 22 + … + 299) – 100 . 2100
60. The sum of the series
1( 2100 − 1)
1 + 2 . 2 + 3 . 22 + 4 . 23 + 5 . 24 + … + 100 . 299 is = – 100 · 2100 = 2100 – 1 – 100 . 2100
(A) 99 . 2100 + 1 (B)  100 . 2100 2 −1
(C) 99 . 2 (D) 
100 . 2100 + 1
99  \ S = 100 . 2100 – 2100 + 1 = 99 . 2100 + 1.
Sequence and Series  7.21

NCERT EXEMPLARS
1. If the sum of n terms of an AP is given by Sn = 3n + 2n2, (A) 4 (B) 6
then the common difference of the AP is (C) 8 (D) 10
(A) 3 (B) 2 7. The minimum value of x x + 41− x , x ∈ R is
(C) 6 (D) 4
(A) 2 (B) 4
2. If the third term of GP is 4, then the product of its first (C) 1 (D) 0
5 terms is
8. Let Sn denote the sum of the cubes of the first n natural
(A) 43 (B)  2
numbers and Sn denote the sum of the first n natural
(C) 6 (D) 4 n
S
numbers, then ∑ r equals to
3. If 9 times the 9th term of an AP is equal to 13 times the r =1 S4
13th term, then the 22nd term of the AP is n ( n + 1) ( n + 2 ) n ( n + 1)
(A) 0 (B) 22 (A)  (B) 
6 2
(C) 198 (D) 220
n + 3n + 2
2

4. If x, 2y and 3z are in AP where the distinct numbers x, (C)  (D)  None of these
2
y and z are in GP, then the common ratio of the GP is
1
(A)  3 (B)  9. If tn denotes the nth term of the series 2 + 3 + 6 + 11 +
3
1 18 + …., then t50 is
(C)  2 (D)  (A) 492 – 1 (B)  492
2 2
(C) 50 + 1 (D)  492 + 2
5. If in an AP, Sn = q n2 and Sm = qm2, where Sr denotes the
sum of r terms of the AP, then Sq equals to 10. The lengths of three unequal edges of a rectangular
a3 solid block are in GP. If the volume of the block is
(A)  (B)  mnq 216 cm3 and the total surface area is 252 cm2, then the
2
(C)  q3 (D) 
(m + n)q2 length of the longest edge is
(A)  12 cm (B)  6 cm
6. Let Sn denote the sum of the first n terms of an AP, if (C)  18 cm (D)  3 cm
S2n = 3Sn, then S3n : Sn is equal to

ANSWER K EYS
1. (D) 2.  (C) 3. (A) 4.  (B) 5. (C) 6.  (B) 7.  (B) 8.  (A) 9.  (D) 10.  (A)

NCERT EXEMPLARS
HINTS AND EXPLANATIONS

1. Given, Sn = 3n + 2n2, = a5 r10 = (ar2)5 = (4)5 [using Eq. (i)]


First term of the AP, 3. Let the first term be a and common difference be d.
T1 = 3 ×1 + 2 (1) = 3 + 2 = 5
2
∴ According to the question, 9.T9 = 13.T13
and T2 = S2 − S1 ⇒ 9 ( a + 8d ) = 13 ( a + 12d )
= 3 × 2 + 2 × ( 2 )  − 3 × 1 + 2 × (1) 
2 2
⇒ 9a + 72d = 13a + 156 d
   
= 14 – 5 = 9 ⇒ ( 9a − 13a ) = 156 d − 72d
∴ Common difference (d) = T2 – T1 = 9 – 5 = 4 ⇒ − 4 a = 84 d
2. It is given that, T3 = 4 ⇒ a = −21d
Let a and r the first term and common ratio, respectively. ⇒ a + 21d = 0
Then, ar2 = 4  (i) ∴ 22nd term i.e., T22 = [ a + 21d ]  (i)
2 3 4
Product of first 5 terms = a · ar · ar · ar · ar T22 = 0  [using Eq. (i)]
7.22  Chapter 7

4. Given, x, 2y and 3z are in AP. 2a


Then, x + 3z ⇒ d=  (iv)
2y = n +1
2
3n 2a
x + 3z  2a + ( 3n − 1) d  6 a + ( 9n − 3)
⇒ y= S3 n n +1
4
Now, = 2 =
Sn n 2a
 2a + ( n − 1) d  2a + ( n −11)
⇒ 4 y = x + 3z 2 n +1
and x, y, z are in GP. 4y = x + 3z 6 an + 6 a + 18an − 6 a
y z
=
Then, = =λ 2an + 2a + 2an − 2a
x y 24 an S

= = 3n = 6

y = xλ and z = λ y = λ 2 x 4 an an

On substituting these values in Eq. (i), we get
7. We know that, AM ≥ GM
4 ( xλ ) = x + 3 λ x
2
( )

4 x + 41− x
⇒ 4λ x = x + 3λ 2 x
⇒ ≥ 4 x ·41− x
2
⇒ 3λ 2 = x + 3λ 2 x

⇒ 4 x + 41− x ≥ 2 4
⇒ 4λ = 1 + 3λ 2

⇒ 4 x + 41− x ≥ 2.2
⇒ 3λ − 4λ + 1 = 0
2

⇒ 4 x + 41− x ≥ 4
⇒ ( 3λ − 1) ( λ − 1) = 0
n
Sr Sr S2 S3 S
1 8. ∑ = + + + ...... + n
∴ λ = ,λ = 1
r =1 Sr S1 S2 S3 Sn
3

Let Tn be the nth term of the above series.
5. Given, Sn = qn2 and Sm = qm2
 n ( n + 1) 
2


=
S1 q=
, S= 4 q, S3 9=
q and S4 16 q  
2 Sn  2 

Now, T1 = q ∴ Tn =
=
n ( n + 1)
Sn

T2 = S2 − S1 = 4 q − q = 3q 2

T3 = S3 − S2 = 9q − 4 q = 5q n ( n + 1) 1 2

= =  n + n 
2 2

T4 = S4 − S3 = 16 q − 9q = 7q
1
∴ Sum of the above series = ∑ Tn =
 ∑ n2 + ∑ n 
HINTS AND EXPLANATIONS


So, the series is q, 3q, 5q, 7q, ……..
2

Here, a = q and d = 3q – q = 2q
1  n ( n + 1) ( 2n + 1) n ( n + 1)  1 n ( n + 1)  ( 2n + 1) 
= 
+ = .  + 1
q 2 6 2  2 2  3

Sq =  2 × q + ( q − 1) 2q  
2
1  2n + 1 + 3  1
q q
= ×  2q + 2q 2 − 2q  = × 2q 2 = q3
= n ( n + 1)   = 4 × 3 n ( n + 1) ( 2n + 4 )
2 2 4  3 
6. Let first term be a and common difference be d. 1 1

= n ( n + 1) ( 2n + 4 ) = n ( n + 1) ( n + 2 )
n 12 6
Then, Sn =  2a + ( n − 1) d   (i)
2 9. Let Sn be sum of the series 2 + 3 + 6 + 11 + 18 …….+ t50.
2n ∴ Sn = 2 + 3 + 6 + 11 + 18 + ....... + t50 . 

S2 n =  2a + ( 2n − 1) d  (i)
2  and Sn = 0 + 2 + 3 + 6 + 11 + 18 + .... + t 49 + t50 (ii)


S2 n = n  2a + ( 2n − 1) d   (ii)
On subtracting Eq. (ii) from Eq. (i), we get
3n
0 = 2 + 1 + 3 + 6 + 7 + …… + t50

S3n =  2a + ( 3n − 1) d   (iii)
2 ⇒
t50 = 2 + 1 + 3 + 5 + 7 + …… upto 49 terms

According to the question, S2n = 3Sn ∴ t50 = 2 + [1 + 3 + 5 + 7 + ……. Utpo 49 terms]

n
⇒ n  2a + ( 2n − 1) d  = 3  2a + ( n − 1) d  49
= 2 + [ 2 × 1 + 48 × 2]

2 2
⇒ 4 a + ( 4 n − 2 ) d = 6 a + ( 3n − 3) d 49
= 2 + [ 2 + 96 ]

⇒ − 2a + ( 4 n − 2 − 3n + 3) d = 0 2
= 2 + [ 49 + 49 × 48]

⇒ − 2a + ( n + 1) d = 0
= 2 + 49 × 49 = 2 + ( 49 )
2

Sequence and Series  7.23

10. Let the length, breadth and height of rectangular solid block 1+ r2 + r 252
a ⇒
=
is , a and ar, respectively. r 2 × 36
r
a 126 21
∴ Volume =
× a × ar = 216 cm3 ⇒ r + r2 + r =
r ⇒ 1+ r2 + r = r
r 36 6

⇒ 6 + 6 r 2 + 6 r = 21r ⇒ 6 r 2 − 15r + 6 = 0
⇒ a3 = 216 ⇒ a3 = 63
⇒ 2r 2 − 5r + 2 = 0 ⇒ ( 2r − 1) ( r − 2 ) = 0

∴ a=6

1
∴ r = ,2

 a2  2
Surface area = 2  . + a 2 r + a 2  = 252
1 a 6×2
 r  For r = : Length = = = 12
2 r 1
1 
Breadth = a = 6
⇒ 2a 2  + r + 1 = 252
1
r  Height = ar = 6 × = 3

2
a 6
 1+ r2 + r 
For r = 2 : Length= = =3
⇒ 2 × 36 
 = 252 r 2
 r 
Breadth = a = 6

Height = ar = 6 × 2 = 12

HINTS AND EXPLANATIONS


7.24  Chapter 7

PRACTICE EXERCISES

Single Option Correct Type

1. If a, b, c are positive numbers in A.P. such that their (A) 5 (B) 25


product is 64, then the minimum value of b (C)  125 (D)  None of these
(A) = 2 (B)  =4 ∞ ∞
(C) = 1 (D)  Does not exist 8. If ∑ x n−1 = a and ∑ y n−1 = b where | x |, | y | < 1,
n =1 n =1
2. If three successive terms of a G.P. with common ratio ∞
r(r > 1) form the sides of a DABC and [r] denotes then ∑ ( xy)n−1 =
greatest integer function, then [r] + [–r] = n =1

(A) 0 (B) 1 a + b −1
(A) ab (B) 
(C)  –1 (D)  None of these ab
1 ab
(C)  (D) 
21 1− ab a + b −1
3. If ∑ aj = 693, where a1, a2, …, a21, are in A.P., then
j =1 9. Let p, q, r ∈ R+ and 27pqr ≥ (p + q + r)3 and 3p + 4q
10 + 5r = 12 then p3 + q4 + r5 is equal to
∑ a2i +1 is (A) 3 (B) 6
i=0
(C)  2 (D)  None of these
(A) 361 (B) 396
(C) 363 (D) data insufficient 10. The sum of the series

4. Number of increasing geometrical progression(s) with 1 2 3


2 4 + + + … to n terms
first term unity, such that any three consecutive terms, 1 + 1 + 1 1 + 2 + 2 1 + 3 + 34
2 4 2

on doubling the middle become an A.P. is is


(A) 0 (B) 1 n( n2 + 1) n( n + 1)
(A)  2 (B) 
(C) 2 (D) infinity n + n +1 2( n2 + n + 1)
5. If a1, a2, a3 (with a1 > 0) are in G.P. with common ratio n( n2 − 1)
r, then the value of r for which the inequality 9a1 + 5a3 (C)  (D)  None of these
PRACTICE EXERCISES

2( n2 + n + 1)
> 14a2 holds, cannot be in the interval
11. a, b, c are three distinct real numbers, which are in G.P.
⎡ 9⎤ and a + b + c = xb. Then
(A)  ⎢1, (– ∞, 0)
(B) 
⎣ 2 ⎥⎦ (A) x < –1 or x > 3 (B)  –1 < x < 3
⎡ 9⎤ (C) –1 < x < 2 (D)  0 < x < 1
(C)  ⎢⎡ , 1⎤⎥ (D) 
5
⎢1, 5 ⎥
⎣9 ⎦ ⎣ ⎦ 12. The sum of the first hundred terms of an A.P. is x and
6. Let Sn (1 ≤ n ≤ 9) denotes the sum of n terms of series the sum of the hundred terms starting from the third
1 + 22 + 333 + … + 999999999, then for 2 ≤ n ≤ 9 term is y. Then the common difference is
1
(A) Sn – Sn–1 = (10n – n2 + n) y−x y−x
(A)  (B) 
9 2 50
1 y−x y−x
(B) Sn = (10 – n2 + 2n – 2)
n
(C)  (D) 
9 100 200
(C) 9(Sn – Sn–1) = n(10n – 1)
(D)  None of these
∞ ∞
1 1
7. If log 5 x + log5 x + log5 x + … upto 7 terms = 35,
1/3 1/4 13. If λ = ∑ i4 , then ∑ (2i − 1)4 is
i =1 i =1
then x is equal to
Sequence and Series  7.25

14 λ ⎛ 1⎞
2
(A)  λ (B)  (A) n (n – 1) (B)  n ⎜1 − ⎟
15 2 ⎝ n⎠
16 15 2
(C)  λ (D)  λ ⎛ n − 1⎞
15 16 (C) n2 (D) 
⎜⎝ n ⎟⎠
14. The sum of all possible products of the first n natural
21. a1, a2, a3, … are in A.P. with common difference not a
numbers taken two at a time is
multiple of 3. Then, maximum number of consecutive
1 1 terms so that all the terms are prime numbers is
(A)  [Sn2 – Sn] (B) [(Sn)2 – Sn]
2 2 (A) 2 (B) 3
1 1 (C) 5 (D) infinite
(C)  [Sn2 – Σ(n + 1)] (D)  [(Sn)2 – Sn2]
2 2 22. The coefficient of x49 in the product (x – 1) (x – 3) …
(x – 99) is
15. The minimum value of 8sin x/8 + 8cos x/8 is (A)  – 992 (B)  1
1 3+ 2 (C)  – 2500 (D)  None of these
3− 2 / 2
(A) 2 2 2
(B)  23. If x, y, z are three real numbers of the same sign then
x y z
1 3− 2 the value of + + lies in the interval
3+ 2 / 2 y z x
(C) 2 2 2
(D) 
(A) [2, ∞) (B)  [3, ∞)
(C) (3, ∞) (D)  (–∞, 3)
16. If log 2 a + log 2 a + log2 a + log2 a + … upto 20
1/2 1/4 1/6 18
/

terms is 840, then a is equal to 24. In a G.P. of alternating positive and negative terms,
(A) 2 (B) 1 any term is the A.M. of the next two terms. Then the
(C)  4 (D)  2 common ratio is
(A) –1 (B) –3
17. Sum to n terms of the series
−1
1 1
(C) 
–2 (D) 
+ is 2
(1 + x ) (1 + 2 x ) (1 + 2 x ) (1 + 3 x )
25. If A = 1 + ra + r2a + r3a + .... as and B = 1+ rb + r2b +
nx n a
(A)  (B)  r3b + …. as, then is equal to
(1 + x ) (1 + nx ) (1 + x ) [1 + ( n + 1) x ] b

PRACTICE EXERCISES
x (1− A)
(C)  (D)  None of these (A) log BA (B)  log1− B
(1 + x ) (1 + ( n − 1) x )
⎛ A −1 ⎞
18. If a, b, c are distinct positive real numbers and a2 + b2 (C) log B −1 ⎜ (D)  None of these
B ⎝
A ⎟⎠
+ c2 = 1, then ab + bc + ca is
(A)  less than 1 (B)  equal to 1
(C)  greater than 1 (D)  any real number 26. If the sum of n terms of an A.P. is cn (n – 1), where
c ≠ 0, then sum of the squares of these terms is
19. The value of (n – 2)2 + (n – 4)2 + (n – 6)2 + … to n
terms is (A) c2n2(n + 1)2
n n 2
(A)  (n2 + 2) (B)  (n2 + 3) (B)  c2 n (n – 1) (2n – 1)
3 2 3
n n 2 2
(C)  (n2 – 2) (D)  (n – 3) (C)  2c n (n + 1) (2n + 1)
3 2 3
20. The sum to infinity of the series
(D)  None of these
2
1 + 2 ⎛⎜1 − 1 ⎞⎟ + 3 ⎛⎜1 − 1 ⎞⎟ + … where n ∈ N, is given
⎝ n⎠ ⎝ n⎠ 27. If in an A.P., Sn = p.n2 and Sm = p.m2 where Sr denotes
by the sum of r terms of the A.P., then Sp is equal to
7.26  Chapter 7

1 34. Suppose a, b, c are in A.P. and a2, b2, c2 are in G.P. If


(A)  p3 (B)  mnp 3
2 a < b < c and a + b + c = , then the value of a is
2
(C) p3 (D) 
(m + n) p2
1 1
(A)  (B) 
28. If p, q, r are positive and are in A.P., the roots of qua- 2 2 2 3
dratic equation px2 + qx + r = 0 are all real for 1 1
1 1
(C)  − (D)  −
r p 2 3 2 2
(A)  − 7 ≥ 4 3 (B)  −7 ≥ 4 3
p r
35. If a1, a2, …, an are in A.P. with common difference
(C) all p and r (D) no p and r d ≠ 0, then sum of the series sin d [sec a1 sec a2 + sec
a2 sec a3 + … + sec an–1 sec an] is
29. The sum to n terms of the series (A) tan an – tan a1 (B) cot an – cot a1
1 5 19 65 (C) sec an – sec a1 (D) cosec an – cosec a1
+ + + + … is
3 9 27 81 36. The first and last term of an A.P. are a and l respec-
n n n n
tively. If S is the sum of all the terms of the A.P. and the
(3 − 2 ) 2 (3 − 2 ) l 2 − a2
(A) n – (B)  n – common difference is , then k is equal to
2n 3n k − ( l + a)
(C) 2n – 1 (D) 
3n – 1 (A) S (B)  2S
(C) 3S (D)  None of these
30. Sum to n terms of the series 1 + 1! + 2 ! + 3! + … is
5! 6 ! 7 ! 8! 37. If a, b, c, d are in G.P., then
(a2 + b2 + c2) (b2 + c2 + d2) =
2 1 1⎛ 1 n! ⎞
(A)  − (B)  − (A) (ab + ac + bc)2 (B) (ac + cd + ad)2
5! ( n + 1)! 4 ⎜⎝ 4 ! ( n + 4)!⎟⎠ (C) (ab + bc + cd) 2
(D)  None of these
1⎛ 1 3! ⎞ 38. If one geometric mean G and two arithmetic means
(C)  ⎜ − (D)  None of these p and q be inserted between two numbers, then G2 is
4 ⎝ 3! ( n + 2)!⎟⎠
equal to
31. If a, b, c, d and p are distinct real numbers such that (A) (3p – q) (3q – p) (B) (2p – q) (2q – p)
(a2 + b2 + c2) p2 – 2p (ab + bc + cd) + (b2 + c2 + d2) (C) (4p – q) (4q – p) (D)  None of these
PRACTICE EXERCISES

≤ 0 then a, b, c, d are in 39. The product of n positive integers is 1, then their sum
(A) A.P. (B) G.P. is a positive integer, that is
(C) H.P. (D)  ab = cd
(A)  equal to 1 (B)  equal to n + n2
32. If a + b + c = 3 and a > 0, b > 0, c > 0, then the greatest (C)  divisible by n (D)  never less than n
value of a2 b3 c2 is 40. A man saves ` 200 in each of the first three months of
10 4 his service. In each of the subsequent months his sav-
3 ⋅2 39 ⋅ 24
(A)  7
(B)  ing increases by ` 40 more than the saving of immedi-
7 77 ately previous months. His total saving from the start
8 4 of service will be ` 11040 after
(C)  3 ⋅ 2 (D)  None of these
(A)  21 months (B)  18 months
77
(C)  19 months (D)  20 months

a b aα − b 41. Statement-1: The sum of the series 1 + (1 + 2 + 4) +


1 (4 + 6 + 9) + (9 + 12 + 16) + …+ (361 + 380 + 400) is
33. If b c bα − c = 0 and α ≠ , then
2 8000.
2 1 0 n

(A) a, b, c are in A.P. (B)  a, b, c are in G.P.


Statement-2:
∑ (k 3 − (k − 1)3 ) = n3 , for any natural
number n. k =1
(C) a, b, c are in H.P. (D)  None of these
Sequence and Series  7.27

(A)  Statement-1 is false, Statement-2 is true. 48. The sum of the products of the 2n numbers ±1, ±2, ±3.
(B)  Statement-1 is true, statement-2 is true; state- …, ±n taking two at a time is
ment-2 is a correct explanation for Statement-1
n ( n + 1) n ( n + 1)
(C)  Statement-1 is true, statement-2 is true; (A)  (B) 

statement-2 is not a correct explanation for 2 2
Statement-1. n ( n + 1) ( 2n + 1) n ( n + 1) ( 2n + 1)
(D)  Statement-1 is true, statement-2 is false. (C)  (D) –
6 6
42. If 100 times the 100th term of an AP with non-zero
49. If a is the first term, d the common difference and Sk
common difference equals the 50 times its 50th term, S
then the 150th term of this AP is the sum to k terms of an A.P., then for kx to be inde-
pendent of x Sx
(A) –150
(B)  150 times its 50th term (A) a = 2d (B)  a=d
(C) 150 (C) 2a = d (D)  None of these
(D) zero
50. Given that α, γ are roots of the equation Ax2 – 4x + 1 = 0
43. If the sum of first n terms of two A.P.’s are in the ratio and β, δ are roots of the equation Bx2 – 6x + 1 = 0. If
3n + 8 : 7n + 15, then the ratio of their 12th terms is α, β, γ and δ are in H.P., then
(A)  8 : 7 (B)  7 : 16 (A) A = 5 (B)  A=–3
(C)  74 : 169 (D)  13 : 47 (C) B = 8 (D)  B=–8

1 3 7 15 51. If three positive numbers a, b, c are in H.P., then


44. The sum of n terms of the series + + + + ... an + cn
is 2 4 8 16
(A) > 2bn (B)  = 2bn
n
1 (C) < 2b (D)  > bn
(A)  2n − n − (B)  1 – 2–n
2 52. a, b, c are three distinct real numbers, which are in G.P.
1
(C) n + 2–n – 1 (D)  ( 2n − 1) and a + b + c = xb. Then,
2
(A) x < –1 or x > 3 (B)  –1 < x < 3
45. The first two terms of a geometric progression add up (C) –1 < x < 2 (D)  0 < x < 1
to 12. The sum of the third and the fourth terms is 48.
If the terms of the geometric progression are alter- 3
1
nately positive and negative, then the first term is 53. If a1, a2, a3, a4 are in H.P., then
a
∑ ar ar +1 is a
(A) –4 (B) –12 root of 1a4 r =1

PRACTICE EXERCISES
(C) 12 (D) 4
(A) x2 + 2x + 15 = 0 (B)  x2 + 2x – 15 = 0
46. The
sum to the infinity of the series (C) x2 – 6x – 8 = 0 (D)  x2 – 9x + 20 = 0
2 6 10 14
1 + + 2 + 2 + 4 + ... is 54. Let the harmonic mean and the geometric mean of two
3 3 3 3 positive numbers be in the ratio 4 : 5. The two numbers
(A) 2 (B) 3 are in the ratio
(C) 4 (D) 6 (A)  1 : 1 (B)  2 : 1 (C)  3 : 1 (D)  4 : 1
47. The sum of positive terms of the series
55. If an and bn be two sequences given by
4 1 1/2n 1/2n n n

10 + 9 + 9 + … is an = ( x ) + ( y) and bn = ( x )1/2 − ( y )1/2 for all


7 7
n ∈ N. Then, a1a2a3 … an is equal to
352 437
(A)  (B)  x+ y
7 7 (A) x – y (B) 
bn
852
(C)  (D)  None of these x− y xy
7 (C)  (D) 
bn bn
7.28  Chapter 7

56. For any odd integer n ≥ 1, 62. If a1 = 0 and a1, a2, a3, …, an are real numbers such
n3 – (n – 1)3 + … + (–1)n–113 = that | ai | = | ai – 1 + 1 | for all i then the A.M. of the
numbers a1, a2, …, an has value x where
1
(A)  (n – 1)2 (2n – 1) 1 1
2 (A) x ≤ – (B)  x≥–
2 2
1
(B)  (n – 1)2 (2n – 1) 1
4 (C) x < – (D)  None of these
2
1
(C)  (n + 1)2 (2n – 1) 63. If a1, a2, a3, …, an are in H.P., then
2
a1 a2
1 , …,
(D)  (n + 1)2 (2n – 1) a2 + a3 + ... + an a1 + a3 + ... + an
4
an
are in
57. For a positive integer n, let a (n) = a1 + a2 + ... + an −1
1 1 1 1 (A)  A.P. (B)  G.P.
1+ + + + ... + n . Then (C)  H.P. (D)  None of these
2 3 4 (2 ) − 1
64. The consecutive numbers of a three digit number form
(A) a (100) ≤ 100 (B)  a (100) > 100
a G.P. If we subtract 792 from this number, we get a
(C) a (200) ≤ 100 (D)  a (200) > 100
number consisting of the same digits written in the
reverse order and if we increase the second digit of the
58. Let α, β, γ be the roots of the equation required number by 2, the resulting number forms an
3x3 – x2 – 3x + 1 = 0. If α, β, γ are in H.P. then A.P. The number is
| α – γ | = (A) 139 (B) 193
1 2 (C)  931 (D)  None of these
(A)  (B) 
3 3
65. The largest term of the sequence
4
(C)  (D)  None of these 1 4 9 16
3 , , , , … is
503 524 581 692
59. Suppose a, b > 0 and x1, x2, x3 (x1 > x2 > x3) are roots
16 4
x−a x−b b a (A)  (B) 
of + = + and x1 – x2 – x3 = c, 692 524
b a x − a x − b
PRACTICE EXERCISES

then a, b, c are in 49
(C)  (D)  None of these
(A) A.P. (B)  G.P. 1529
(C)  H.P. (D)  None of these
66. The coefficient of x99 and x98 in the polynomial
60. The coefficient of xn in the product
(x – 1) (x – 2) (x – 3) … (x – 100) are
(1 – x) (1 – 2x) (1 – 22 ⋅ x) (1 – 23 ⋅ x) … (1 – 2n ⋅ x) is
equal to (A)  – 5050 and 12482075
n ( n −1) (B)  – 4050 and 12582075
n+1
(A)  (1 – 2 ) 2 2 ⋅ (C)  – 5050 and 12582075
n ( n −1) (D)  None of these
(B) (2n + 1 – 1) ⋅2 2
67. The three successive terms of a G.P. will form the
n ( n −1)
n sides of a triangle if the common ratio r satisfies the
(C)  (1 – 2 ) 2 2 ⋅ inequality
(D)  None of these
3 −1 3 +1
61. If 0.272727…, x and 0.727272… are in H.P., then x (A)  <r<
2 2
must be
(A) rational (B) integer (B)  5 − 1 < r < 5 +1
(C)  irrational (D)  None of these 2 2
Sequence and Series  7.29

75. If the (m + 1) th, (n + 1) th and (r +1)th terms of an A.P.


2 −1 2 +1
(C)  <r< are in G.P. and m, n, r are in H.P., then the ratio of the
2 2 first term of the A.P. to its common difference is
(D)  None of these
n n

(A)  (B) 
68. If the sides of a right angled triangle are in G.P., then 3 3
the cosine of the greater acute angle is n n
(C)  (D) 

1 1 2 2
(A)  (B) 
1+ 5 1− 5 76. Let there be n numbers in G.P. whose common ratio
is r and Sm denotes the sum of their first m terms. The
(C)  1 + 5 (D)  None of these
2 sum of their products taken two at a time is k Sn Sn–1
where k =
69. Sum to n terms of the series 2 + 5 + 14 + 41 + … is
r −1 r −1
n 1 n 3 (A)  (B) 
(A)  + (3n − 1) (B)  + (3n − 1) r r +1
2 4 2 4
r
n 1 (C)  (D)  None of these
(C)  + (3n − 1) (D)  None of these r +1
2 2
70. If the pth, qth and rth terms of both an A.P. and a G.P. 77. If a, b, c, d are distinct integers in A.P. such that d = a2
be respectively a, b and c, then + b2 + c2, then a + b + c + d =
(A) ac ⋅ cb ⋅ ba = ac ⋅ bc ⋅ ab (A) 2 (B) 1
(B) ab – 1 ⋅ bc + 1 ⋅ ca – 1 = ac – 1 ⋅ ba – 1 ⋅ cb + 1 (C)  0 (D)  None of these
(C) ab ⋅ bc ⋅ ca = ac ⋅ ba ⋅ cb 1 1 1
(D)  None of these 78. If Hn = 1 + + + ..... + , then the value of
2 3 n
71. If, in a G.P. of 3n terms, S1 denotes the sum of the first 3 5 2n − 1
1 + + + .... + is
n terms, S2 the sum of the second block of n terms and 2 3 n
S3 the sum of the last n terms, then S1, S2, S3 are in (A) n – Hn (B)  2n – Hn
(A)  A.P. (B)  G.P. (C) (n – 1) – Hn (D)  n – 2Hn
(C)  H.P. (D)  None of these
79. If am be the mth term of an A.P., then
72. In a geometric series, the first term is a and common
ratio is r. If Sn denotes the sum of n terms and Un a21 –a22 + a23 – a24 +….+ a22n–1– a22n =
n n −1 2 n
= ∑ Sn, then rSn + (1 – r) un = ( a − a22n )

PRACTICE EXERCISES
(A)  (B)  (a2 − a2 )
n =1
2n − 1 1 2n − 1 2 n 1
(A) na (B)  (n – 1) a n
(C)  ( a 2 − a22n ) (D)  None of these
(C) (n + 1) a (D)  None of these 2n − 1 1
73. In a Dabc, if cot A, cot B, cot C are in A.P. then a2, b2,
c2 are in 1
for n ≥ 1 and a3 = a1, then (a2001)2001=
80. If an + 1 =
(A)  A.P. (B)  G.P. 1− an
(C)  H.P. (D)  A.G. P. (A) 1 (B) –1
(C)  0 (D)  None of these
1 1 1 4
74. If 4
+ 4
+ 4
+ .... up to ∞ = π , then the value of
1 2 3 90 81. If a, b, c are positive numbers in G.P. and log
1 1 1 ⎛ 5c ⎞ ⎛ 3b ⎞ ⎛ a⎞
+ + + …. up to ∞ is
14 34 54 ⎜⎝ a ⎟⎠ , log ⎜⎝ 5c ⎟⎠ and log ⎜⎝ 3b ⎟⎠ are in A.P. then a, b, c

π4 π4
(A)  (B)  (A)  form the sides of an equilateral triangle
45 96 (B)  form the sides of an isosceles triangle
π4 (C)  form the sides of a right angled triangle
(C)  (D)  None of these (D)  can not form the sides of a triangle
124
7.30  Chapter 7

82. If a, b, c are in G.P. and log a – log 2b, log 2b – log 3c If p, q, r are in A.P. then x, y, z are in
and log 3c – log a are in A.P., then a, b, c are the sides (A)  A.P. (B)  G.P.
of a triangle which is (C)  H. P (D)  None of these
(A)  right angled 87. If | a | < 1 and | b | < 1, then the sum of the series
(B)  acute angled 1 + (1 + a) b + (1 + a + a2) b2 + (1 + a + a2 + a3) b3 +
(C)  obtuse angled … ∞ is equal to
(D)  None of these
1 1
(A)  (B) 
83. In a sequence of 4n + 1 terms, the first 2n +1 terms (1 − b)(1 − ab) (1 − a)(1 − ab)
are in A.P. having common difference 2 and the last
1
1 (C)  (D)  None of these
2n + 1 terms are in G.P. having common ratio . If the (1 − a)(1 − b)
2
middle term of the A.P. is equal to the middle term of 88. If < an > and < bn > be two sequences given by an =
the G.P. then the middle term of the sequence is −n −n −n −n

x 2 + y 2 and bn = x 2 − y 2 ∀ n ∈ N, then the


n.2n + 1 n.2n + 1 value of a1 a2 a3 ….an is
(A)  (B) 
2n + 1 2n − 1 x+ y x−y
(A)  (B) 
n bn bn
(C)  n.2 (D)  None of these
2n − 1 x2 + y2 x2 − y2
(C)  (D) 
bn bn
84. If S1, S2 and S3 denote the sums up to n > 1 terms of
three sequences in A.P. whose first terms are unity and 89. The sixth term of an A.P. is equal to 2. The value of
common differences are in H.P. then n = the common difference of the A.P. which makes the
product a1 a4 a5 greatest, is
2S3 S1 + S1S2 + S2 S3 8 2
(A)  (A)  (B) 
S1 − 2S2 + S3 5 3
2S3 S1 − S1S2 − S2 S3 3 3
(B)  (C)  (D) 
S1 + 2S2 + S3 5 4
90. If the natural numbers are written as
2S3 S1 − S1S2 − S2 S3
(C)  1
S1 − 2S2 + S3
2 3
(D)  None of these
4 5 6
PRACTICE EXERCISES

85. Sum to n terms of the series 13 + 3.23 + 33 + 3.43 + 53


7 8 9 10
+….., where n is even, is
......................
n2 ( n2 − 3n + 1) n2 ( n2 + 3n + 1)
(A)  (B)  ......................
2 2
2 2
Then, the sum of the terms of the nth row is
(C)  n ( n + 3n + 1) (D)  None of these n ( n2 − 1) n ( n2 + 1)
4 (A)  (B) 
2 4
86. Let a be a fixed real number such that
n ( n2 + 1)
a− x a− y a− z (C)  (D)  None of these
= = 2
px qy rz

Previous Year’s Questions

91. If 1, log3 (31− x + 2) , log3 (4 ⋅ 3x− l) are in AP, then 92. The value of 21/4⋅ 41/8⋅ 81/16 . . . ∞ is: [2002]
x equals: [2002] (A) 1 (B) 2
(A) log3 4 (B)  l − log3 4 (C) 3/2 (D) 4
(C) 1 − log, 3 (D)  log4 3
Sequence and Series  7.31

93. Fifth term of a GP is 2, then the product of its 9 terms 3n( n + 1) n2 ( n + 1)


is: [2002] (A)  (B) 
2 2
(A) 256 (B) 512 2
(C)  1024 (D)  None of these n( n +1) 2 ⎡ n( n + 1) ⎤
(C)  (D)  ⎢ ⎥
4 ⎣ 2 ⎦
94. Let Tn denote the number of triangles which can be
formed using the vertices of a regular polygon of n ∞ ∞ ∞
sides. If Tn+1 ⋅ = Tn = 21, then n equals : [2002] 100. If x = ∑ an , y= ∑ bn , z = ∑ cn where a, b, c are
n= 0 n= 0 n= 0
(A) 5 (B) 7
in A.P. and |a| < 1, |b|< 1, |c|< 1, then x, y, z are in
(C) 6 (D) 4
 [2005]
1 1 1 (A) G.P.
95. The sum of the series − + − ……… upto (B) A.P.
1.2 2.3 3.4
∞ is equal to [2003] (C) Arithmetic − Geometric Progression
(A)  2 loge 2 (B)  log2 2 − 1 (D) H.P.

loge ⎛ 4 ⎞
(C) loge 2 (D)  a1 + a2 + ...a p p 2
⎜⎝ ⎟⎠ 1 01. Let a1, a2, a3, … be terms of an A.P. If = , p ≠ q,
e a1 + a2 ... + aq q 2
a1 + a2 + ...a p p 2
a
96. If f : R → R satisfies f (x + y) = f (x) + f (y), for all x, y = 2 , p ≠ q, then 6 equals [2006]
a1 + a2 ... + aq q a21
n
∈ R and f (1) = 7, then ∑ f (r ) is [2003]
(A) 
41 7
(B) 
r =1 11 2

7n 7( n + 1)
(A)  (B)  2 11
2 2 (C)  (D) 
7 41
7n( n + 1) 102. If a1, a2, … , an are in H.P., then the expression a1 a2 +
(C) 7n( n + 1) (D) 
2 a2 a3 + … + an-1 an is equal to [2006]
n n (A)  n(a1− an) (B) (n − 1) (a1−an)
1 r tn
97. If Sn = ∑ nC and t n = ∑ nC , then
Sn
is equal to (C)  na1an (D) (n − 1)a1an
r =0 r r =0 r
 [2004] 103. In a geometric progression consisting of positive
1 1 terms, each term equals the sum of the next two terms.

PRACTICE EXERCISES
(A)  n (B)  n − 1
2 2 Then the common ratio of this progression equals

[2007]
2n − 1 1 1
n − 1 (D) 
(C)  (A)  (1 − 5 ) (B)  5
2 2 2

98. Let Tr be the r th term of an A.P. whose first term is 1
a and common difference is d. If for some positive (C)  5 (D)  ( 5 − 1)
2
1 1
integers m, n, m ≠ n, Tm = and Tn = , then a − d, 104. If p and q are positive real numbers such that p2 + q2
equals n m [2004] = 1, then the maximum value of (p + q) is [2007]
(A) 0 (B) 1 (A) 2 (B) 1/2
1 1 1 1
(C)  (D)  + (C)  (D)  2
mn m n 2
105. The first two terms of a geometric progression add
99. The sum of the first n terms of the series 12 + 2 ⋅ 22 up to 12. The sum of the third and the fourth terms
n( n +1) 2 is 48. If the terms of the geometric progression are
+ 32 + 2 ⋅ 42 + 52 + 2 ⋅ 62 + ... is when n is
2 alternately positive and negative, then the first term is
even. When n is odd the sum is [2004]  [2008]
7.32  Chapter 7

(A)  −4 (B)  −12


112. Let α and β be the roots of equation
(C) 12 (D) 4
px 2 + qx + r − 0, p ≠ 0 . If p, q, r are in A.P. and
106. The sum to the infinity of the series
1 1
2 6 10 14 + = 4 , then the value of α − β is [2014]
1 + + 2 + 3 + 4 + ....... is [2009] α β
3 3 3 3
(A) 2 (B) 3 61 2 17
(A)  (B) 
(C) 4 (D) 6 9 9
107. A person is to count 4500 currency notes. Let an 34 2 13
denote the number of notes he counts in the nth min- (C)  (D) 
9 9
ute. If a1 = a2 = ...... = a10 = 150 and a10, a11 . . . are in
A.P. with common difference −2, then the time taken 113. Three positive numbers form an increasing G.P. If
by him to count all notes is [2010] the middle term in this G.P. is doubled, the new num-
(A)  34 minutes (B)  125 minutes bers are in A.P. Then the common ratio of the G.P. is
(C)  135 minutes (D)  24 minutes  [2014]
(A)  2 + 3 (B)  3 + 2
108. A man saves Rs. 200 in each of the first three months
of his service. In each of the subsequent months his (C) 2 − 3 (D)  2+ 3
saving increases by Rs. 40 more than the saving of
immediate preceding month. His total saving from 114. If (10)9 + 2(11)1 (10)8 + 3(11) 2 (10)7 + ..... + 10(11)9,
the start of service will be Rs. 11040 after [2011] = k(10)9 then k is equal to [2014]
(A)  19 months (B)  20 months
(C)  21 months (D)  18 months 121 441
(A)  (B) 
109. Statement 1: The sum of the series 1 + (1 + 2 + 4) + 10 100
(4 + 6 + 9) + (9 + 12 + 16) + ...... + (361 + 380 + 400) (C) 100 (D) 110
is 8000. n
115. The sum of first 9 terms of the series
Statement 2: ∑ ( k 3 = ( k − 1)3 ) = n3 for any natural 3 3 3 3 3 3
k =1 1 1 +2 1 +2 +3
number n. [2012] + + + ...... is: [2015]
1 1+ 3 1+ 3 + 5
(A)  Statement 1 is false, statement 2 is true (A) 96 (B) 142
(B) Statement 1 is true, statement 2 is true; state-
ment 2 is a correct explanation for statement 1 (C) 192 (D) 71
PRACTICE EXERCISES

(C) Statement 1 is true, statement 2 is true; statement 116. If the 2nd, 5th and 9th terms of a non-constant A.P. are
2 is not a correct explanation for statement 1 in G.P., then the common ratio of this G.P. is
(D)  Statement 1 is true, statement 2 is false  [2016]
7 8
110. If 100 times the 100th term of an Arithmetic (A)  (B) 
4 5
Progression with non zero common difference equals
the 50 times its 50th term, then the 150th term of this 4
(C)  (D)  1
A.P. is [2012] 3
(A) –150
(B)  150 times its 50th term 117. If the sum of the first terms of the series
2 2 2 2
(C) 150 ⎛ 3⎞ ⎛ 2⎞ ⎛ 1⎞ 2 ⎛ 4⎞ 16
(D) zero ⎜⎝1 ⎟⎠ + ⎜⎝ 2 ⎟⎠ + ⎜⎝ 3 ⎟⎠ + 4 + ⎜⎝ 4 ⎟⎠ +…, is
5 5 5 5 5
111. The sum of first 20 terms of the sequence 0.7, 0.77, m, then m is equal to [2016]
0.777, . . . is [2013] (A) 99 (B) 102
7
(
(A)  99 − 10 −20
9
) (B) 
7
81
(179 + 10 −20 ) (C) 101 (D) 100
118. For any three positive real numbers a, b and c,
9(25a2 + b2) + 25(c2 – 3ac) = 15b(3a + c). Then
7
(
(C)  99 + 10 −20
9
) (D) 
7
81
(
179 − 10 −20 ) [2017]
Sequence and Series  7.33

(A) b, c and a are in A.P. (A) 66 (B) 68


(B) 
a, b and c are in A.P. (C) 34 (D) 33
(C) 
a, b and c are in G.P.
120. Let A be the sum of the first 20 terms and B be the
(D) b, c and a are in G.P.
12 sum of the first 40 terms of the series 12 + 2 · 22 + 32
119. Let a1, a2, a3, …, a49 be in A.P. such that ∑ a4 k +1 = 416 + 2 · 42 + 52 + 2 · 62 + …. If B – 2A = 100λ, then λ is
k =0 equal to [2018]
and a9 + a43 = 66. If a12 + a22 +…+ a17
2
= 140 m , then (A) 232 (B) 248
m is equal to [2018] (C) 464 (D) 496

ANSWER K EYS
Single Option Correct Type
  1. (B)  2.  (C) 3. (C) 4.  (B) 5. (D) 6.  (C) 7.  (A) 8.  (D) 9.  (A) 10. (B)
11.  (A) 12.  (D) 13.  (D) 14.  (D) 15.  (B) 16.  (C) 17.  (B) 18.  (A) 19.  (A)  20.  (C)
21.  (B) 22.  (C) 23.  (B) 24.  (C) 25.  (C) 26.  (B) 27.  (C) 28.  (B)  29.  (B) 30.  (B)
31.  (B) 32.  (A) 33.  (B) 34.  (D) 35.  (A) 36.  (B) 37.  (C) 38.  (B) 39.  (D) 40.  (A)
41.  (B) 42.  (D) 43.  (B) 44.  (C) 45.  (B) 46.  (B) 47.  (C) 48.  (D) 49.  (C) 50.  (C)
51.  (A) 52.  (A) 53.  (B) 54.  (D) 55.  (C) 56.  (D) 57.  (D) 58.  (C) 59.  (C) 60.  (A)
61.  (A) 62.  (B) 63.  (C) 64.  (C) 65.  (C) 66.  (C) 67.  (B) 68.  (A) 69.  (B) 70.  (C)
71.  (B) 72.  (A) 73.  (A) 74.  (B) 75.  (D) 76.  (C) 77.  (A) 78.  (B) 79.  (C) 80.  (B)
81.  (D) 82.  (C) 83.  (B) 84.  (C) 85.  (B) 86.  (C) 87.  (A) 88.  (B) 89.  (A) 90.  (C)

Previous Years’ Questions


  91.  (B) 92.  (B) 93.  (B) 94.  (B) 95.  (D) 96.  (D) 97.  (A) 98.  (A) 99.  (D) 100.  (D)
101.  (D) 102.  (D) 103.  (D) 104.  (D) 105.  (B) 106.  (B) 107.  (A) 108.  (C) 109.  (B) 110.  (D)
111.  (B) 112.  (D) 113.  (D) 114.  (C) 115.  (A) 116.  (C) 117.  (C) 118.  (A) 119.  (C) 120.  (B)

PRACTICE EXERCISES
7.34  Chapter 7

HINTS AND EXPLANATIONS

Single Option Correct Type


⎡ 9⎤
1. Given a + c = 2b ⇒
r ∉ ⎢1, ⎥
a+b+c ⎣ 5⎦
Also, ≥ 3 abc = 3 64 = 4
The correct option is (D)
3
3b 1 2 3
⇒ ≥4 6.
Sn = (9) + (99) + (999) + …
3 9 9 9
⇒ b≥4 1 1

= [10 + 2.102 + 3.103 + …] – [1 + 2 + 3 + …]
∴ Minimum b=4 9 9
The correct option is (B) 1 1 n( n + 1)

S− =
2. Let sides of triangle be a, ar, ar2. 9 9 2
Since r > 1, ∴ ar2 is greatest side
S = 10 + 2.102 + 3.103 + … + n 10n
∴ a + ar > ar2 ⇒ r2 – r – 1 < 0 ⇒ 10S = 102 + 2.103 + … + (n – 1)10n + n.10n+1



1− 5 1+ 5 –9S = (10 + 102 + 103 + … + 10n) – n.10n+1

<r< ⇒ 1 < r < 1+ 5 n +1
2 S = n 10 n +1 − 10 − 1
2 2

∴ (r) = 1.
9 81

Also, – 1 + 5 < – r < –1


n +1

Sn = n 10 n +1 − 10 − 1 − 1 n( n + 1)
2 81 9.81 9 2
∴ (– r) = –2
n +1
∴ (r) + (–r) = 1 – 2 = –1
⇒ 9.Sn = (9n − 1)10 + 1 − n( n + 1)

81 81 2

The correct option is (C)
10 n
21 ∴ 9(Sn – Sn–1) =
[10(9n – 1) – (9n – 10)]– n
21 81
3. ∑ aj = 693 = (a1 + a21)
j =1 2 = n (10n – 1)
The correct option is (C)
HINTS AND EXPLANATIONS


a1 + a21 = 66
693

Now, a11 = A.M. = = 33 7. log 5
x + log5 x + log5 x + … upto 7 terms
1/3 1/4

21

Also, a2 + a20 = a3 + a19 = … = a9 + a13 = a10 + a12 log x log x log x

= + + + … upto 7 terms
10
log 5 log 51/3 log 51/4

∑ a2i +1 = 5 × (a1 + a21) + a11
log x
i=0

= [2 + 3 + 4 + … + 8]

= 5 × 66 + 33 = 363 log 5
The correct option is (C) log x ⎡ 7 ⎤ log x

= ⎢ ( 2 + 8) ⎥ = (35) = 35 (given)
4. Let a, ar, ar2 be any three consecutive terms, then according log 5 ⎣ 2 ⎦ log 5
to the given condition, a, 2ar, ar2 are in A.P.
i.e. 4ar = a + ar2 log x

=1

or r=2± 3 log 5
Since, given G.P. is increasing, r = 2 + . 3 Hence, there is ⇒ log x = log 5

only one such G.P. ⇒
x=5
The correct option is (B)
The correct option is (A)
5. Given a1 > 0 and a1, a2, a3 are in G.P. ∞
1
such that a2 = a1r and a3 = a1r2 8. Given a = ∑ x n −1 = 1 + x + x2 + … =
n =1 1− x
∴ 9a1 + 5a3 > 14a2
⇒ 9a1 + 5a1r2 > 14a1r a −1

x=
and since a1 > 0, we get 9 + 5r2 > 14r a
2 ∞
⇒ 5r – 14r + 9 > 0 ⇒ (5r – 9) (r – 1) > 0 b −1
Similarly y =

b
∴ ∑ ( xy)n−1 = 1 + (xy) + (xy)2 + …
n =1
Sequence and Series  7.35

1 Divide by b,

1 ⎛ a − 1⎞ ⎛ b − 1⎞ a c
1− ⎜
+1+ =x
⎝ a ⎟⎠ ⎜⎝ b ⎟⎠ b b

= 1 − xy =
1

+ 1 + r = x, r is the common ratio of the G.P.
r
2
ab ab ⇒ r + r (1 – x) + 1 = 0. Since r is real, therefore discrimi-


= = nent > 0
ab − ( ab − ( a + b) + 1) a + b −1
⇒ (1 – x)2 – 4 > 0 ⇒ x2 – 2x + 1 – 4 > 0


The correct option is (D) ⇒ x2 – 2x – 3 > 0 ⇒ (x + 1) (x – 3) > 0

⇒ x < –1 or x > 3

9. 27pqr ≥ (p + q + r)3

The correct option is (A)
p +q +r
⇒ (pqr)1/3 ≥ ⇒p=q=r 100
3 12. Given: [2a + 99d] = x
2
Also, 3p + 4q + 5r = 12
100
and
[2(a + 2d) + 99d] = y

p=q=r=1 2

The correct option is (A) On subtraction, 200d = y – x

y−x
⇒d=

10. Let Tn be the nth term of the series 200
1 2 3
The correct option is (D)
2 4 + + +…
1 + 1 + 1 1 + 22 + 24 1 + 32 + 34 1 1 1 1
13. Given: … + + + =λ
n n 14 24 34 44
Then, Tn =
=
1 + n2 + n4 (1 + n2 ) 2 − n2 ∞
1 1 1 1

∑ ( 2i − 1)4 =
14
+
34
+
54
+ ...
n i =1

=
( n2 + n + 1) ( n2 − n + 1)
= 1 + 1 + 1 + 1 + ... ∞ − ⎡ 1 + 1 + ... ∞ ⎤

14 24 34 4 4 ⎢ 24 4 4 ⎥
1⎛ 1 1 ⎞ ⎣ ⎦

= −
2 ⎜⎝ n2 − n + 1 n2 + n + 1⎟⎠ 1 ⎡1 1 1 ⎤
= λ−
4 ⎢ 4
+ 4 + 4 + ... ∞ ⎥
2 ⎣1 2 3 ⎦
1⎡ 1 1 ⎤
=

HINTS AND EXPLANATIONS



⎢ − ⎥
2 ⎣1 + ( n − 1)n 1 + n( n + 1) ⎦ 1 15
=λ–
λ= λ
16 16

Now

The correct option is (D)
n
1 ⎡1 1 ⎤ 1⎡ 1 1 ⎤ 14. Let A = 1.2 + 1.3 + … + 2.3 + 2.4 + … + (n – 1) · n

∑ Tr = − + −
2 ⎢⎣1 1 + 1.2 ⎥⎦ 2 ⎢⎣1 + 1.2 1 + 2.3 ⎥⎦
r =1 Now, (1 + 2 + 3 + … + n)2 – (12 + 22 + 32 + … + n2) = 2A
1⎡ 1 1 ⎤ 1 ⎡ n2 ( n + 1) 2 n( n + 1) ( 2n + 1) ⎤

+ − +… ⇒ A = −
2 ⎢⎣1 + 2.3 1 + 3.4 ⎥⎦
⎢ ⎥
2 ⎢⎣ 4 6 ⎥⎦
1⎡ 1 1 ⎤ 1

+ ⎢ − ⎥
[(Sn)2 – Sn2]
=
2 ⎣1 + n − 1 n 1 + n( n + 1) ⎦
( ) 2

The correct option is (D)
1⎡ 1 ⎤

= ⎢1 − ⎥ 15. A.M. ≥ G.M.
2 ⎣ 1 + n( n + 1) ⎦
⎛ x π⎞
23 sin x/8 + 23 cos x/8 3 2 cos ⎜ − ⎟
⎝ 8 4⎠
n( n + 1) ⇒
≥ 2

= . 2
2( n2 + n + 1)
⎛ x π⎞
3 2 cos ⎜ − ⎟
⎝ 8 4⎠
1 3
Now maximum of 2 = 23 2 .1
= 23/ 2
Trick: Checking for n = 1, 2. S1 = and S2 = which are
given by (b). 3 7 x x ⎛ 3 ⎞
sin cos ⎜ +1⎟

The correct option is (B) So, A.M. ≥ 23/
2
⇒ 8 8 +8 8 ≥ 2⎝ 2 ⎠


The correct option is (B)
11. a + b + c = xb
7.36  Chapter 7

16. log 2 a + log 2 a + log2 a + log 2 a + … 2


⎛ 1⎞
1/2 1/4 1/6 18
/

∴ ⎜1 − ⎟ S = ⎛ 1⎞ ⎛ 1⎞

= 2 log2 a + 4 log2 a + 6 log2 a + … + 40 log2 a

⎝ n⎠ ⎜⎝1 − n ⎟⎠ + 2 ⎜⎝1 − n ⎟⎠ + … (2)

= log2 a [2 + 4 + 6 + … + 40]
(1) and (2)
20 2

= (2 + 40) log2 a S ⎛ 1⎞ ⎛ 1⎞ + … ∞
2 ⇒
=1+ ⎜⎝1 − n ⎟⎠ + ⎜⎝1 − n ⎟⎠
n
= 420 log2 a = 840 (Given)
⇒ log2 a = 2 ⇒ a = 4 1
The correct option is (C)
= ⎛ 1⎞ = n
1 − ⎜1 − ⎟
⎝ n⎠
17. If tr denotes the rth term of the series, then

S = n2
x
x tr =
The correct option is (C)
(1 + rx ) (1 + ( r + 1) x )
21. In this type of A.P. it can be easily shown that exactly one out
1 1 of any 3 consecutive terms will be multiple of 3. So at most

= −
1 + rx 1 + ( r + 1) x 3 consecutive terms can be prime numbers.
n
The correct option is (B)
n
⎡ 1 1 ⎤
⇒ x ∑ t r =
∑ ⎢1 + rx − 1 + ( r + 1) x ⎥ 22. The coefficient of x49 = – [1 + 3 + 5 + … + 99]
r =1 r =1 ⎣ ⎦
= – 2500
1 1 The correct option is (C)

= −
1 + x 1 + ( n + 1) x
23. A.M. ≥ G.M.
nx 1/3

= ⎛ x y z⎞ ⎛ ⎞
(1 + x ) (1 + ( n + 1) x ) ⇒ ⎜ + + ⎟ 3 ≥ x⋅ y⋅z

⎝ y z x⎠ ⎜⎝ y z x ⎟⎠
n
n

∑ tr =
(1 + x ) [1 + ( n + 1) x ] ⎛ x y z⎞
r =1 ⇒ ⎜ + + ⎟ ≥3

⎝ y z x⎠

The correct option is (B)

The correct option is (B)
18. Since a and b are unequal,
24. Let the G.P. be a – ar + ar2 – ar3 + … with common ratio
HINTS AND EXPLANATIONS

2 2
a + b > a 2b 2 [A.M. > G.M. for unequal numbers]
=–r
2 By the given condition


a2 + b2 > 2ab 2
Similarly,
b2 + c2 > 2bc and c2 + a2 > 2ca
a = ar − ar ⇒ 2a = ar2 – ar
Hence, 2 (a2 + b2 + c2) > 2 (ab + bc + ca)
2

ab + bc + ca < 1 ⇒ 2 = r – r
2


The correct option is (A) ⇒
r2 – r – 2 = 0
n ⇒ (r – 2) (r + 1) = 0

19. The given series = ∑ ( n − 2r ) 2 ⇒
r = 2, –1
r =1
∴ Common ratio = –2 or 1

n

= ∑ (n 2
− 4 nr + 4 r ) 2
Hence, common ratio = –2
( common ratio is –ve)
1

The correct option is (C)
n ( n + 1)
2 n

= n · n – 4n · + 4 · (n + 1) (2n + 1)
2 6 1 1 1 A −1
25. A = ⇒ 1 − ra = ⇒ ra = 1 − =
n 2 1 − ra A 4 A

= (n + 2)
3

The correct option is (A) 1 1 1 B −1
B= ⇒ 1 − rb = ⇒ rb = 1 − =
1 − rb B B B
2
20. Let S = 1 + 2 ⎛⎜1 − 1 ⎞⎟ + 3 ⎛⎜1 − 1 ⎞⎟ +…(1) ⎛ A −1 ⎞ ⎛ B −1 ⎞

∴ a log r = log ⎜
⎝ n⎠ ⎝ n⎠ ⎟ and b log r = log ⎜ B ⎟
⎝ A ⎠ ⎝ ⎠
Sequence and Series  7.37

28. Since p, q, r are in A.P. so 2q = p + r . The roots of the equa-


⎛ A −1 ⎞
log ⎜ ⎟ tion px2 + qx + r = 0 are real if and only if
a ⎝ A ⎠ ⎛ A −1 ⎞
∴ = = log B −1 ⎜ .
A ⎟⎠
p + r⎞
2
b ⎛ B −1 ⎞ B ⎝
q2 – 4pr ≥ 0 ⇒ ⎛⎜ - 4pr ≥ 0
log ⎜
⎝ B ⎠
⎟ ⎝ 2 ⎟⎠


The correct option is (C)
p2 p

p2 + r2 - 14pr ≥ 0 ⇒ - 14 +1≥0
2 r
26. If tr be the rth term of the A.P., then r
tr = Sr – Sr–1
2
⎛p ⎞ p
= cr (r – 1) – c (r – 1) (r – 2) ⇒ ⎜
− 7⎟ - 48 ≥ 0 ⇒ −7 ≥ 4 3

⎝ r ⎠ r

= c (r – 1) (r – r + 2) = 2c (r – 1)

The correct option is (B)

We have, t12 + t 22 + … + tn 2

1 5 19 65


= 4c2[02 + 12 + 22 + … + (n – 1)2] 29. + + + + … to n terms
3 9 27 81
( n − 1) n (2 n − 1)

= 4c2 ⎛ 2⎞ ⎛ 4⎞ ⎛ 8 ⎞ ⎛ 16 ⎞
6 = ⎜1 − ⎟ + ⎜1 − ⎟ + ⎜1 −
+ 1− +…
⎝ ⎠ ⎝ 3⎠ ⎝ 9 27 ⎟⎠ ⎜⎝ 81⎟⎠

2 2
= c n (n – 1) (2n – 1)
2 ⎡ 2 ⎛ 2⎞ ⎤
2
3 =n–
⎢1 + + ⎜ ⎟ + ... to n terms ⎥

The correct option is (B) 3 ⎢⎣ 3 ⎝ 3 ⎠ ⎥⎦
n
27. Sn =
n
[2a + (n – 1) d] = pn2(1) ⎛ 2⎞
1− ⎜ ⎟ 2
2 2
=n– ·
⎝ 3⎠ =n– (3n – 2n)
3 1− 2 3n
m
Sm =
[2a + (m – 1) d] = pm2 3
2

The correct option is (B)
2a + ( n − 1) d n
⇒ =

HINTS AND EXPLANATIONS


2a + ( m − 1) d m ( r − 1)! r!
30. We have, tr = and tr+1 =
⇒ 2am + (n – 1) md = 2an + n (m – 1) d
( r + 4)! ( r + 5)!

⇒ 2a (m – n) + d (mn – m – nm + n) = 0

r! r!
⇒ 2a (m – n) + d (n – m) = 0

Now, rtr – (r + 5)tr+1 = – =0
( r + 4)! ( r + 4)!
⇒ 2a – d = 0

∴ d = 2a

rtr – (r + 1)tr+1 = 4tr+1

∴ (1) gives, 2a + (n – 1) 2a = 2pn


n −1 n −1

4 ∑ t r +1 = ∑ [rtr − ( r + 1) tr +1]
⇒ 2an = 2pn ⇒ a = p ∴ d = 2p
r =1 r =1
p ⇒ 4(t2 + t3 + … + tn) = 1t1 - ntn
Now,
Sp = · [2a + (p – 1) d]
2
⇒ 4 (t1 + t2 + … + tn) = 5t1 - ntn = 5 ⎜
⎛ 0 !⎞ n( n − 1)!

⎝ 5!⎟⎠ ( n + 4)!
p

= · [2p + (p – 1) 2p]
2
1 n!

=
p
· [2p + 2p2 – 2p]

= −
4 ! ( n + 4)!
2
p . 2 3 1⎡1 n! ⎤

= n 2p = p ⇒ t1 + t2 + … + tn =

2 4 ⎣ 4 ! ( n + 4)! ⎥⎦


The correct option is (C)

The correct option is (B)
7.38  Chapter 7

31. We have, (a2 + b2 + c2)p2 – 2p(ab + bc + cd) + 2 2


(b2 + c2 + d2) ≤ 0
4
⎛1 ⎞ ⎛1 ⎞
⇒ ⎛⎜ 1 ⎞⎟ = ⎜ − d ⎟ ⎜ + d ⎟
⇒ (ap – b)2 + (bp – c)2 + (cp – d)2 ≤ 0 ⎝ 2⎠ ⎝2 ⎠ ⎝2 ⎠
Therefore, 1 2 1 1 1
ap − b = 0 cp − d = 0 ⇒
= ⎛⎜ 1 − d 2 ⎞⎟ ⇒ − d 2 = ± ⇒ d = ± .
bp − c = 0 16 ⎝ 4 ⎠ 4 4 2


b = ap ⇒ c = bp ⇒ d = cp 1

d= ( d > 0)
b c d 2

= = =p 1 1
a b c ∴
a=b–d= −
2 2
∴ a, b, c, d are in G.P.


The correct option is (B) The correct option is (D)
35. As a1, a2, a3, … an–1, an are in A.P.,
32. Taking A.M. and G.M. of 7 numbers
d = a2 - a1 = a3 - a2 = … = an - an–1
a a b b b c c sin d [sec a1 sec a2 + sec a2 sec a3 + .. + sec an–1 sec an]
, , , , , , , we get
2 2 3 3 3 2 2
sin (a2 − a1 ) sin (a3 − a2 ) sin (an − an −1 )
1 =
+ + .. +
a b c cos a1 cos a2 cos a2 cos a3 cos an −1 cos an
2 ⋅ + 3⋅ + 2 ⋅ ⎡⎛ a ⎞ 2 ⎛ b ⎞ 3 ⎛ c ⎞ 2 ⎤ 7
2 3 2 ≥ ⎢

⎜ ⎟ ⎜ ⎟ ⎜ ⎟ ⎥ = (tan a2 – tan a1) + (tan a3 – tan a2)

7 ⎢⎝ 2 ⎠ ⎝ 3 ⎠ ⎝ 2 ⎠ ⎥
⎣ ⎦
 + … + (tan an – tan an–1)
1 = tan an – tan a1

3 ⎛ a2b 3c 2 ⎞ 7 37 a2b 3c 2
The correct option is (A)

≥ ⎜ ⎟ ⇒ 7 ≥
7 ⎝ 22 3322 ⎠ 7 22 ⋅ 33 ⋅ 22 n 2S
36. We have, S = (a + l) ⇒ = n (1)
2 a+l
310 ⋅ 2 4
⇒ a2 b3 c2 ≤
l−a
77
Also, l = a + (n – 1) d ⇒ d =
n −1
310 ⋅ 2 4
∴ greatest value of a2 b3 c2 =
. l −a
77
= 2S [Using (1)]

The correct option is (A)
HINTS AND EXPLANATIONS

−1
a+l
33. Expanding along R3, we get
b aα − b a aα − b
l 2 − a2
=

2 − +0=0 2S − ( l + a)
c bα − c b bα − c
∴ k = 2S.

⇒ 2 (b2α - bc - acα + bc) - (abα - ac - abα + b2) = 0


The correct option is (B)
⇒ 2α (b2 - ac) - (b2 - ac) = 0

37. As a, b, c, d are in G.P., therefore
or (b2 - ac) (2α - 1) = 0

b c d

or b2 - ac = 0 [∵ (2α – 1) ≠ 0]
= = = r (say)
a b c

b2 = ac ⇒ b = ar, c = br = ar ⋅ r = ar2,
∴ a, b, c are in G.P.
d = cr = ar2 ⋅ r = ar3. (a2 + b2 + c2) (b2 + c2 + d 2)

The correct option is (B)

= (a2 + a2 r2 + a2 r4) (a2 r2 + a2 r4 + a2 r6)
34. Let a = b – d and c = b + d,
= a4 r2 (1 + r2 + r4) (1 + r2 + r4)
3 1
= (a2 r + a2 r3 + a2 r5)2
then a + b + c =
⇒β= .

2 2 = (a ⋅ ar + ar ⋅ ar2 + ar2 ⋅ ar3)2


1 1 1 = (ab + bc + cd)2.
−d, , +d
Therefore, the number are
The correct option is (C)
2 2 2
(d > 0 as a < b < c)
38. Let the two numbers be a and b,then

Now a2, b2, c2 are in G.P. ⇒ (b2)2 = a2c2


G=
ab or G2 = ab
Sequence and Series  7.39

Also, p and q are two A.M.s between a and b.


The correct option is (B)
∴ a, p, q, b are in A.P.
1 3 7 15
∴ p – a = q – p and q – p = b – q 44. Sn = + + + + … upto n terms
2 4 8 16
∴ a = 2p – q and b = 2q – p
∴ G2 = ab = (2p – q) (2q – p). ⇒ Sn = ⎛⎜1 − ⎞⎟ + ⎛⎜1 − ⎞⎟ + ⎛⎜1 − ⎞⎟ + ...
1 1 1

The correct option is (B) ⎝ 2⎠ ⎝ 4⎠ ⎝ 8⎠
39. Given x1 · x2 …xn = 1 1⎛ 1⎞
Since A.M. ≥ G.M. ⎛1 1 1 ⎞ ⎜⎝1 − n ⎟⎠

= n − ⎜ + + + …⎟ = n − 2 2
⎝2 4 8 ⎠ 1
⎛ x + x 2 + ... + x n ⎞ 1−
∴ ⎜ 1
1/2 1/n
⎟⎠ ≥ (x1 · x2 …xn) = (1) = 1 2
⎝ n 1 –n
⇒ x1 + x2 + … + xn ≥ n.

= n −1+ n = n + 2 – 1
2

Hence x1 + x2 + … + xn can never be less than n.

The correct option is (C)

The correct option is (D)
45. Let a, ar, ar2, …
40. Total savings = 200 + 200 + 200 + 240 + 280 + … to n
a + ar = 12 (1)
months = 11040
ar2 + ar3 = 48 (2)
n−2
⇒ 400 +
( 400 + ( n − 3) ⋅ 40) = 11040 dividing Eq. (2) by (1), we have
2
⇒ (n – 2)(140 + 20n) = 10640 ar 2 (1 + r )

=4
⇒ 20n2 + 100n – 280 = 10640 a( r + 1)
⇒ n2 + 5n – 546 = 0 ⇒
r2 = 4 if r ≠ –1
⇒ (n – 21)(n + 26) = 0 ∴
r = –2
⇒ n = 21 as n ≠ –26
Also, a = –12 [using (1)].
The correct option is (A)
The correct option is (B)
(( n − 1)3 − n3 )
41. Tn = (n – 1)2 + (n – 1)n + n2 =
( n − 1) − n 2 6 10 14
46. Let S = 1+ + + + + ... (1)
= n3 – (n – 1)3 3 32 33 34

T1 = 13 – 03 1 1 2 6 10

HINTS AND EXPLANATIONS



T2 = 23 – 13
S = + + 3 + 4 + ... (2)
3 3 32

3 3

From (1) and (2),
T20 = 203 – 193

S20 = 203 – 03 = 8000 ⎛ 1⎞
S ⎜1 − ⎟ = 1 + +
1 4 4 4
+ + + ...
The correct option is (B) ⎝ 3⎠ 3 32 33 34
42. 100(a + 99d) = 50(a + 49d) 2 4 4 ⎛ 1 1 ⎞

S = + ⎜ 1 + + 2 + ...⎟
2a + 198d = a + 49d 3 3 32 ⎝ 3 3 ⎠
a + 149d = 0
4 4 ⎛ 1 ⎞
T150 = a + 149d = 0 2 +

S = 3 32 ⎜ 1⎟
The correct option is (D) 3 ⎜⎝ 1 − ⎟⎠
3
n 4 4 3 4 2 6
sn 3n + 8 ( 2a + ( n − 1)d ) 3n + 8
= + = + =
43. = ⇒ 2 = 3 32 2 3 3 2
sn′ 7n + 15 n 7n + 15
( 2a′ + ( n − 1)d ′ ) 2
2 6

S =
3 3
⎛ n − 1⎞
a+⎜
⎝ 2 ⎟⎠
d ⇒
S=3
3n + 8 a + 11d 3( 23) + 8

= = =
⎛ n − 1⎞ 7n + 15 a′ + 11d ′ 7( 23) + 15
The correct option is (B)
a′ + ⎜ d′
⎝ 2 ⎟⎠ 3
47. Here, a = 10, d = – .
77 7 7

= =
176 16
7.40  Chapter 7

⎛ 3⎞ ⇒
1

Then, tn = 10 +(n – 1) . ⎜ − ⎟ + 2d = 3 (2)
⎝ ⎠7 α
1

Solving (1) and (2), we get = 1 and d = 1.
⎛ 3⎞ α
tn is positive if 10 + (n – 1) ⎜ − ⎟ ≥ 0;

⎝ ⎠ 7 1 1 1 1

= 1, = 2, = 3 and = 4.
1 α β γ δ
or, 70 – 3 (n – 1) ≥ 0 or 73 ≥ 3n; or 24
≥n
3 1 1
∴ First 24 terms are positive.

Since, = A ⇒ A = 3. Also, = B ⇒ B = 8.
αγ βδ
∴ Sum of the positive terms


The correct option is (C)
24 ⎡ −3 ⎤

= S24 = ⎢ 2 × 10 + 23 × 7 ⎥
2 ⎣ ⎦ 51. For the positive numbers a, c we have harmonic mean H = b
{ a, b, c are in H.P.} and geometric mean G = ac .
= 12 ⎡ 20 −
69 ⎤ = 852 .

But G > H; ∴ ac > b. (1)
⎢ 7 ⎥⎦ 7
⎣ For the positive numbers an, cn, we have
The correct option is (C) geometric mean =
an c n
48. We have,
n n
(1 – 1 + 2 – 2 + 3 – 3 + … + n – n)2 arithmetic mean = a + c ;

2
= 1 + 12 + 22 + 22 + … + n2 + n2 + 25, 2
n n
where S is the required sum.  A.M. > G.M., ∴ a + c
> an c n (2)
⇒ o = 2(12 + 22 + … + n2) + 2S 2
n ( n + 1)( 2n + 1)
From (1) and (2), we get
⇒ S = – (12 + 22 + … + n2) = –
6
The correct option is (D) an + c n > ( ac)n > bn, ∴ an + cn > 2bn.
2
kx The correct option is (A)
Skx [ 2a + ( kx − 1) d ] 52. a + b + c = xb
49. We have, = 2
x Divide by b,
Sx
[ 2a + ( x − 1) d ]
HINTS AND EXPLANATIONS

2 a c
+ 1 + =x
k [( 2a − d ) + kxd ] b b

=
( 2a − d ) + xd ⇒ 1 + 1 + r = x, r is the common ratio of the G.P.

S kx r

For to be independent of x, 2a – d = 0 or 2a = d. ⇒ r2 + r (1 – x) + 1 = 0. Since r is real, therefore, discrim-

Sx inent > 0

The correct option is (C) ⇒ (1 – x)2 – 4 > 0 ⇒ x2 – 2x + 1 – 4 > 0

⇒ x2 – 2x – 3 > 0 ⇒ (x + 1) (x – 3) > 0

50. α, β, γ and are in H.P.
⇒ x < –1 or x > 3

1 1 1 1

, , , are in A.P.
The correct option is (A)
α β γ δ

Let d be the common difference of the A.P. 3
Since, a, γ are roots of Ax2 – 4x + 1 = 0
53. ∑ ar ar +1 = a1a2 + a2a3 + a3a4 = 3a1a4
1
α +γ 4/A 1 1 1 1 1 1

= = 4 or + =4 Since a1, a2, a3, a4 are in H.P. , , , are in A.P.
αγ 1/ A α γ
a1 a2 a3 a4
1 1
1 1 1

− = d ⇒ a1 – a2 = da1a2(1)

+ + 2d = 4 or + d = 2 (1) a2 a1
α α α Similarly, a2 – a3 = d a2a3(2)

Also, β, δ are roots of Bx2 – 6x + 1 = 0

a3 – a4 = d a3a4 (3)
β +δ 1 1 6/B 1 1
On adding (1), (2) and (3), we get

= + = = 6 or + d + + 3d = 6
βδ β δ 1/ B α α a1 – a4 = d[a1a2 + a2a3 + a3a4]

Sequence and Series  7.41

a − a4 a1b1 b0 x −y
∴ a1a2 + a2a3 + a3a4 = 1

== =
d bn bn bn
1 1 a −a
The correct option is (C)

= + 3d ⇒ 1 4 = 3a1a4
a4 a1 d
56. Since n is an odd integer (–1)n–1 = 1 and n – 1,
⇒ a1a2 + a2a3 + a3a4 = 3a1a4

n – 3, n – 5 etc., are even integers. We have,
∴ Given expression = 3. It is a root of x2 + 2x – 15 = 0

n3 – (n – 1)3 + (n – 2)3 – (n – 3)3 + … + (–1)n–113

The correct option is (B)
= n3 + (n – 1)3 + (n – 2)3 + … + 13

2ab – 2[(n – 1)3 + (n – 3)3 + … + 23]
54. Harmonic mean of a, b is H =
a+b = n3 + (n – 1)3 + (n – 2)3 + … + 13
Geometric mean G =
ab ⎡⎛ n − 1⎞ 3 ⎛ n − 3 ⎞ 3 ⎤
3
H 4 4


– 2 × 2 ⎢⎜⎝ 2 ⎟⎠ + ⎜⎝ 2 ⎟⎠ + ... + 1 ⎥
3


Given: = , so 2 ab = ⎢⎣ ⎥⎦
G 5 a+b 5

[ n –1, n – 3 are even integers]
a+b 5 n( n + 1) ⎤
2
⎡ 1 ⎛ n − 1⎞ ⎛ n − 1 ⎞ ⎤
2

or, = = ⎡
⎥ − 16 ⎢ ⎜ + 1⎟ ⎥

2 ab 4 ⎢ ⎟⎜
⎣ 2 ⎦ ⎣2 ⎝ 2 ⎠ ⎝ 2 ⎠⎦

By componendo and dividendo
1 2 ( n − 1)2 ( n + 1)2
( a + b )2 9 a+ b 3 =
n (n + 1)2 – 16
= or = 4 16 × 4
( a − b )2 1 a− b 1
1 1
=
(n + 1)2 [n2 – (n – 1)2] = (n + 1)2(2n – 1).
3+1 2 a 4 4

Again, by componendo and dividendo =
2 b 3 −1
The correct option is (D)
a a 4 57. We have,
= 2 or =
b b 1 1 1 1 1
a (n) = 1 +
+ + + ... + n

The correct option is (D) 2 3 4 (2 ) − 1
⎛ 1 1⎞ ⎛ 1 1⎞ ⎛ 1 1⎞
n n

55. an = x1/2 + y1/2 and bn = x1/2 − y1/2


n n

= 1+ ⎜ + ⎟ + ⎜ +… + ⎟ + ⎜ +… + ⎟
⎝ 2 3⎠ ⎝ 4 7⎠ ⎝ 8 15 ⎠

HINTS AND EXPLANATIONS


(
Now, anbn = x1/2 + y1/2

n n

) (x1/2n
− y1/2
n

)

⎛ 1 1 ⎞
+… + ⎜ n −1 + … + n −1 ⎟
⎝2 2 ⎠
n −1
< 1 + 2 + 4 + 8 + ... + 2
( ) − (y )
2 2

1/2n 1/2n
⇒ anbn = x
2 4 8 2n −1
n −1 n −1 1+… +1
⇒ anbn = x 1/2
− y 1/2 = bn–1(1)
=1+ = n.
( n + 1) times

Now, a1a2a3 … an Thus, a (100) < 100

⎛ a a a ...a ⎞ 1 ⎛ 1 1⎞ ⎛ 1 1⎞
= ⎜ 1 2 3 n ⎟ bn Next, a (n) = 1 + + ⎜ + ⎟ + ⎜ + … + ⎟ + …


⎝ bn ⎠ 2 ⎝ 3 4 ⎠ ⎝ 5 8⎠
1 1
(a1a2 a3 ...an −1 ) (anbn )
+ + ... +

= 2 n −1
+1 n
2 −1
bn
n −1
a a a ...a b
> 1+ 1 + 2 + 4 +… + 2 − 1

= 1 2 3 n −1 n −1  {using (1)} 2 4 8 2n 2n
bn 1 1 1 1
+ + +… + 1
( a1a2 a3 ...an − 2 ) ( an −1bn −1 )
=1+ 2 2 2 2 – n

=    2
bn n times

⎛ 1⎞ n
a a a ...a b
= ⎜1 − n ⎟ +

= 1 2 3 n −1 n − 2 ⎝ 2 ⎠ 2
bn
Thus, a (200) > ⎛ 1 −

1 ⎞ 200 > 100.

……………… ⎜⎝ 100 ⎟
+
⎠ 2 2

………………
7.42  Chapter 7


The correct option is (D) ⎛ 1⎞ ⎛ 1⎞

 ⎜⎝ x − 2 ⎟⎠ … ⎜⎝ x − n ⎟⎠
1 2 2
1 1
58. Let α = ,β= and γ  = . n ( n +1)
a−d a a+d ⎛ 1⎞ ⎛ 1⎞ ⎛ 1⎞
= (– 1)n ⋅ 2
⋅ (x – 1) ⎜ x − ⎟ ⎜ x − 2 ⎟ … ⎜ x − n ⎟
2
1 1 1 1 ⎝ 2⎠ ⎝ 2 ⎠ ⎝ 2 ⎠

Then, ⋅ ⋅ =–
a−d a a+d 3 ∴ coefficient of xn

n ( n +1)
1 1 1 1 1 1 ⎛ 1 1 1⎞

and, ⋅ + ⋅ + ⋅ =–1
= (– 1)n 2 2 ⋅ ⎜ −1 − − 2 ... n ⎟
a−d a a+d a a−d a+d ⎝ 2 2 2 ⎠
⇒ (a – d) a (a + d) = – 3
⎛ 1 ⎞
n ( n +1) ⎜⎝1 − n+1 ⎟⎠ n ( n − 1)
(a + d ) + (a − d ) + a = (–
1)2n + 1 ⋅ 2 2 ⋅ 2 ⋅ = (1 – 2n + 1) ⋅ 2 2 .

and, =–1
(a − d ) a (a + d ) ⎛ 1⎞
⎜⎝1 − ⎟⎠
⇒ 3a = 3 or a = 1 and (1 – d) (1 + d) = – 3
2

or, 1 – d2 = – 3 or d2 = 4 ⇒ d = ± 2.
The correct option is (A)
1 61. Let R = 0.272727…

When d = 2, α = – 1, β = 1, γ  = .
3 2
⇒ 10 R = 27.2727…
1

When d = – 2, α = , β = 1, γ  = – 1. and, 104R = 2727.2727…
3
4 ⇒ (104 – 102) R = 2727 – 27
Therefore, | α – γ | =

3 2700 3

R= =.

The correct option is (C) 9900 11
x−a x−b b a 8
59. We have, + = + Similarly, 0.727272… =
.
b a x−a x−b 11
x−a b a x−b
Since 0.272727…, x and 0.727272… are in H.P.

− = −
b x−a x−b a
3 8

, x, are in H.P.
( x − a) 2 − b 2 a 2 − ( x − b) 2 11 11

=
b ( x − a) ( x − b) a 3 8
2⋅ ⋅
(x − a − b ) (x − a + b ) ( x − b − a) ( x − b + a) ⇒ x = 11 11 = 48
HINTS AND EXPLANATIONS



=– 3 8 121
b ( x − a) (x − b ) a +
11 11
⇒ (x – a – b) [a (x – b) (x – a + b) + b (x – b + a)

∴ x is rational.

(x – a)] = 0

The correct option is (A)
⇒ x (x – a – b) [(a + b) x – (a2 + b2)] = 0

2 2 62. We have, | ai | = | ai – 1 + 1 |
⇒ x = 0, x = a + b or x = a + b .

a+b ⇒ ai2 = ai2−1 + 2 ai −1 + 1

a2 + b 2 2ab

Since =a+b– < a + b (∵ a, b > 0) Putting i = 1, 2, 3, …, n + 1, we get

a+b a+b
2 2

a12 = 0
we take x1 = a + b, x2 = a + b and x3 = 0.

a+b
a22 = a12 + 2a1 + 1
2 2
Since x1 – x2 – x3 = c, we get a + b – a + b = c

a32 = a22 + 2a2 + 1
a+b
2ab
  

= c ⇒ a, b, c are in H.P.
a+b

The correct option is (C)
an2 = an−1
2 + 2an – 1 + 1

60. The given product 2



an+1 = an2 + 2an + 1
= (– 1)n (x – 1) (2x – 1) (22 x – 1) (23 x – 1) … (2n x – 1)
n +1 n n

= (– 1) ⋅ 2
n 1+2+…+n ⎛ 1⎞
⋅ (x – 1) ⎜ x − ⎟
On adding, we get ∑ ai2 = ∑ ai2 + 2∑ ai + n
⎝ 2⎠ i =1 i =1 i =1
Sequence and Series  7.43

n 65. The general term of the given sequence is


⇒ 2
∑ ai = – n + an+1
2 ≥–n
n2
i =1
Tn =
500 + 3n3
a + a + ... + an 1 1
⇒ 1 2
≥– ⇒x≥– .
n 2 2 d Tn n (1000 − 3n3 )

then, =

The correct option is (B) dn (500 + 3n3 )2

For maximum or minimum of Tn
63. Given: a1, a2, a3, …, an are in H.P.
d Tn
1 1 1 1 =0

, , , …, are in A.P.
dn
a1 a2 a3 an
1
a + a + a + ... + an a1 + a2 + ... + an ⎛ 1000 ⎞ 3
⇒ 1 2 3 ∴
n= ⎜
⎝ 3 ⎟⎠
, , ….
a1 a2
1
a1 + a2 + ... + an
Now, 6 < ⎛ 1000 ⎞
3
 , are in A.P. <7
an ⎜⎝ 3 ⎟⎠

a1 + a2 + a3 + ... + an a + a + ... + an
Hence, T7 is largest term. So largest term in the given

– 1, 1 2 – 1, …,
a1 a2
sequence is 49 .
a1 + a2 + ... + an 1529

 – 1, are in A.P.
The correct option is (C)
an
a2 + a3 + ... + an a1 + a3 + ... + an 66. Consider the equation

,, …,
a1 a2 (x – 1) (x – 2) (x – 3) … (x – 100) = 0 (1)
Its root are 1, 2, 3, …, 100.
a1 + a2 + ... + an −1

 are in A.P. (1) is a polynomial equation in x of degree 100. Coefficient
an
of x100 = 1.
a1 a2 Now, sum of the roots of equation (1); taken one at a time

, , …,
a2 + a3 + ... + an a1 + a3 + ... + an 99
i.e., 1 + 2 + 3 + … + 100 = (– 1)1 coeff. of x

an 100
coeff. of x
 are in H.P.
a1 + a2 + ... + an −1 = – coeff. of x99

HINTS AND EXPLANATIONS





The correct option is (C) 100 × 101
∴ coeff. of x99 = – (1 + 2 + 3 + … + 100) = –
64. Let the three digits be a, ar, ar . 2 2

Then, according to the hypothesis,
= – 5050

Sum of the product of the roots 1, 2, 3, …, 100 taken two at
100a + 10ar + ar2 + 792 = 100ar2 + 10ar + a

a time
⇒ a (r2 – 1) = 8
(1) 98
2
and, a, ar + 2, ar are in A.P.
= (– 1)2 coeff. of x = coeff. of x98
coeff. of x 100

then, 2 (ar + 2) = a + ar2

∴ coeff. of x98 = sum of the product of 1, 2, 3, …, 100 taken
⇒ a (r2 – 2r + 1) = 4 (2) two at a time

Dividing (1) by (2),
= 1
[(1 + 2 + 3 + … + 100)2 – (12 + 22 + … + 1002)]
a ( r 2 − 1) 8 2

then, =
a ( r 2 − 2 r + 1) 4 1 ⎡ 2 100 × 101 × 201 ⎤
=
⎢(5050) − ⎥
( r + 1)( r − 1) 2 ⎣ 6 ⎦

=2
( r − 1)2 1
=
[(5050)2 – 338350)] = 12582075.
r +1 2

=2
r −1
The correct option is (C)
∴ r = 3 ∴ from (1), a = 1.
67. Let the sides of the triangle be a, ar, ar2.

Thus, digits are 1, 3, 9 and so the required number is 931. If r = 1, then the three terms of G.P. will be a, a, a and hence

The correct option is (C) an equilateral triangle will be formed.
7.44  Chapter 7

Thus when r = 1, triangle will be formed


(1) n −1 3
If r > 1, then greatest side will be ar2 and in this case triangle

= 2 + 3 (1 − 3 ) = 2 – (1 – 3n – 1)
1− 3 2
will be formed if
1 1

a + ar > ar2 ⇒ r2 – r – 1 < 0
= + ⋅ 3n;
2 2
1− 5

< r < 1+ 5 1 1
2 2 ∴ Sn = Σ tn =
Σ1+ Σ 3n
2 2

r < 1 + 5 [∵r > 1]  (2) 1 1
2

= ⋅n+ (3 + 32 + 33 + … + 3n)
2 2
If r < 1, then greatest side will be a and triangle will be

formed if n 1 3 (1 − 3n ) n 3 n

=
+ ⋅ = + (3 − 1) .

ar + ar2 > a ⇒ r2 + r – 1 > 0 2 2 1− 3 2 4

The correct option is (B)
−1 − 5

r< or r > −1 + 5
2 2 70. Let the A.P. be x, x + y, x + 2y …
Then, a = x – (p – 1) y,(1)
5 −1

< r < 1. [∵ 0 < r < 1]  (3) b = x + (q – 1) y (2)
2

From (1), (2) and (3), possible values of r are given by c = x + (r – 1) y (3)
∴ b – c = (q – r) y (4)
5 −1 5 +1 . c – a = (r – p) y
<r< (5)
2 2 a – b = ( p – q) y (6)

The correct option is (B) 2
Let the G.P. be u, uv, uv , …
68. Let the sides of the right angled triangle be a, ar, ar2 out of Then, a = uvp – 1(7)
which ar2 is the hypotenuse, then r > 1.
b = uvq – 1,(8)
A

c = uvr – 1,(9)
Now, log (ab – c ⋅ bc – a ⋅ ca – b)


= (b – c) log a + (c – a) log b + (a – b) log c
= (q – r) y log (uvp – 1) + (r – p) y log (uvq – 1)
ar 2
HINTS AND EXPLANATIONS

ar

 + ( p – q) y log (uvr – 1), using (4), (5), (6),
 (7), (8), (9).

= y [(q – r) {log u + (p – 1) log v} + (r – p) {log u
 + (q – 1) log v} + (p – q) {log u + (r – 1) log v}
B a C
= y [log u (q – r + r – p + p – q) + log v {(q – r) (p – 1)
2 4
Now, a r = a + a r
2 2 2
 + (r – p) (q – 1) + (p – q) (r – 1)}]
= y [log u × 0 + log v × 0] = 0.
or, r4 – r2 – 1 = 0 ∴ r2 = 1 ± 5

2 Then, log [ab – c ⋅ bc – a ⋅ ca – b] = 0

2 2 1+ 5 . ∴ ab – c ⋅ bc – a ⋅ ca – b = 1;

∵ r > 1 ∴ r > 1 ∴ r =

2 ∴ ab bc ca = ac ba cb.

Angle C is the greater acute angle The correct option is (C)
a 1 1 71. Let the 3n terms of G.P. are a, ar, ar2, …,
Now, cos C =
= = . arn – 1, arn, arn + 1, arn + 2, … ar2n – 1, ar2n,

ar 2 r2 1+ 5
ar2n + 1, ar2n + 2, …, ar2n – 1


The correct option is (A) n

Then, S1 = a + ar + ar2 + … + arn – 1 = a (1 − r )
69. Here, the series is 1− r
2 + (2 + 3) + (2 + 3 + 9) + (2 + 3 + 9 + 27) + …

S2 = arn + arn + 1 + arn + 2 + … + ar3n – 1
the difference of the consecutive terms being 3, 9, 27, …
n n
∴ tn = 2 + 3 + 9 + 27 … to n terms
= ar (1 − r )
= 2 + [3 + 9 + 27 + … to (n – 1) terms] 1− r
Sequence and Series  7.45


S3 = ar2n + ar2n + 1 + ar2n + 2 + … + ar3n – 1
The correct option is (A)
n n

= ar (1 − r ) 1 1 1
1− r 74. Let S =
4
+ 4
+ + ….up to ∞
n 2
1 3 54
(1 − r )
Now, (S2)2 = a2 r2n
π4 1 1 1 1 1
(1 − r ) 2
Given: = + + + + + …..
90 14 2 4 2 4 34 4 4
a (1 − r n ) n

= ⋅ ar2n (1 − r ) = S1S3 1 1 1
1− r
=S+ + + + ...
1− r 24 44 64
Hence, S , S , S are in G.P. 1 ⎡1
1 2 3 1 1 ⎤

= S+ + + + ....⎥

The correct option is (B) 24 ⎢⎣14 24 34 ⎦

= S+
1 π4
n
×
72. Let r > 1. Then, Sn = a ( r − 1) 24 90
r −1
4
un = S1 + S2 + S3 + … + Sn π4 ⎛ 1⎞ π
∴ S =
⎜ 1− ⎟ =
90 ⎝ 16 ⎠ 96
2 2 n
= a ( r − 1) + a ( r − 1) + a ( r − 1) + ... + a ( r − 1)
The correct option is (B)
r −1 r −1 r −1 r −1
a 75. Let a and d respectively be the first term and common differ-

= [(r + r2 + r3 + … + rn) – n] ence of the A.P.
r −1
2mr
a ⎡ r ( r n − 1) ⎤ Given: n =
(1)

= ⎢ − n⎥ m+r
r − 1 ⎢⎣ r − 1 ⎥⎦ and, (a + nd)2 = (a + md) (a + rd)

∴ r Sn + (1 – r) un
2
⎛a ⎞
+ n⎟ = ⎛⎜ + m ⎞⎟ ⎛⎜ + r ⎞⎟
a a
n
a ( r − 1) n
ar ( r − 1) an (1 − r ) ⇒ ⎜

= r⋅ + (1 − r ) − ⎝ d ⎠ ⎝d ⎠⎝d ⎠
r −1 (1 − r ) 2 r −1
⇒ (x + n)2 = (x + m) (x + r)
⎡ a ⎤

⎢ Putting d = x ⎥
n n ⎣ ⎦

= – ar ( r − 1) + ar ( r − 1) + an = na ⇒ x2 + 2nx + n2 = x2 + (m + r) x + mr

HINTS AND EXPLANATIONS


1− r 1− r

The correct option is (A) n( m + r )
⇒ (m + r – 2n) x = n2 – mr = n 2 −

2
2 2  [Using (1)]
73. We have, cos A = b + c − a
n

= ( 2n − m − r )
2bc 2
and, sin A = ka (k is a content)
n
∴x= −
, which is the required ratio.
2
2 2 2
∴ cot A = b + c − a and similarly, we have
The correct option is (D)
2abck 76. Let the n numbers in G.P. be
a, ar, ar2,…, arn–1
2 2 a2 + b2 − c2
2
cot B = a + c − b , cot C =

Thus, we have,
2abck 2abck ⎛1− rn ⎞

Sn = a ⎜ ⎟
⎝ 1− r ⎠
b 2 + c 2 − a2 a2 + c 2 − b 2 a2 + b 2 − c 2 Also,


Given: , , are
2abck 2abck 2abck
S2n = [a + ar + ar2 +….+ arn–1]2
in A.P.
⎛1− rn ⎞
⇒ b2 + c2 –a2, a2 + c2 – b2, a2 + b2 –c2 are in A.P.


⇒ a2 ⎜ ⎟ = a2 + (ar)2 + (ar2)2 +…..+(arn–1)2 + 2S

(Multiplying each term by 2abck) ⎝ 1− r ⎠
⇒ –2a2, – 2b2, – 2c2 are in A.P.
where S denotes the sum of the product of the terms of the

G.P. taken two at a time.
(Subtracting a2 + b2 + c2 from each term)

⇒ a2, b2, c2 are in A.P. (dividing each term by –2]

7.46  Chapter 7

2
where d is the common difference of the A.P.
⎛1− rn ⎞
2
⎛ 1 − r 2n ⎞
Since, a2n = a1 +(2n–1) d

⇒ a ⎜
⎟ = a2 ⎜ ⎟ + 2S
⎝ 1− r ⎠ ⎝ 1 − r2 ⎠
a2 n − a1
⎡ (1 − r n )2 1 − r 2 n ⎤ ⇒
d=
a2 2n − 1

S= ⎢ 2
− ⎥
2 ⎢⎣ (1 − r ) 1 − r 2 ⎥⎦
Thus, we have from (1), required expression

a2 ⎛ 1 − r n ⎞ ⎡1 − r n 1 + r n ⎤ a − a2 n

= ⎜ 1− r ⎟ ⎢ 1− r − 1 + r ⎥

= 1
2n − 1
[
× n 2a1 + a2 n − a1 ]
2 ⎝ ⎠ ⎢⎣ ⎥⎦
n ( a1 − a2 n )( a1 + a2 n ) n ( a12 − a22n )
a (1 − r n )(1 + r ) − (1 + r n )(1 − r )
= =

= Sn × × 2n − 1 2n − 1
2 (1 + r )(1 − r )

The correct option is (C)
a 2( r − r n )

= Sn × ×
2 (1 + r )(1 − r )
1
80. We have, an + 1 =
r a(1 − r n −1 ) ⎛ r ⎞ 1− an
= Sn × × =⎜
⎝ 1+ r ⎟⎠ n n–1
S S
1+ r 1− r 1 1
1 =
r ∴ a2 =
and a3 = 1 − a2 1

k= 1 − a1 1−
r +1 1 − a1

The correct option is (C)
1 − a1 1 − a1
77. Let the common difference of the given A.P. be t. Then,
= =
1 − a1 − 1 −a1
d = a2 + b2 + c2 ⇒ a + 3t = a2 + (a + t)2 + (a + 2t)2

⇒ 5t2 + 3 (2a –1) t + 3a2– a = 0 (1) 1 − a1

Since a3 = a1, ∴ = a1
 t is real ⇒ D ≥ 0 −a1
⇒ 9 (2a – 1)2–4 (5) (3a2– a) ≥ 0 ⇒
a21 – a1 + 1 = 0 ⇒ a1 = – w or – w2
⇒ 24 a2 + 16a – 9 ≤ 0 1 1 − a3
1 =
1 Now, a5 = = 1 − a3
⇒ − – 70 < a < −1 + 70
1 − a4 1−
1 − a3
3 12 3 12
1 − a1
HINTS AND EXPLANATIONS

⇒ a = –1, 0
[  a is integer] = = a1 = a3 and so on
3 −a1
When a = 0, from (1), t = 0, . Rejecting both these values

since t must be non zero 5 ∴
a1 = a3 = a5 ………a2001
4 2001
= (–w)2001 or (–w2)2001

When, a = – 1, from (1), t = 1, ⇒t=1 Thus, (a 2001)

5

= –1
∴ a + b + c + d = – 1 + 0 + 1 + 2 = 2


The correct option is (A)
The correct option is (B)

3 5 ⎛ 2n − 1⎞ 81. Given: b2 = ac(1)


78. 1 + + +… + ⎜
2 3 ⎝ n ⎟⎠ ⎛ 3b ⎞ ⎛ 5c ⎞ ⎛ a⎞
and, 2 log ⎜ ⎟ = log ⎜ ⎟ + log ⎜ ⎟

⎝ 5c ⎠ ⎝ a⎠ ⎝ 3b ⎠
⎛ 1⎞ ⎛ 1⎞
= (2–1) + ⎜ 2 − ⎟ + ⎜ 2 − ⎟ + ..... + ⎜ 2 − ⎟

⎛ 1⎞
⎝ 2⎠ ⎝ 3⎠ ⎝ n⎠ ⎛ 5c a ⎞ ⎛ 5c ⎞ ⎛ 3b ⎞
= log ⎜
. = log ⎜ ⎟ = – log
⎛ 1 1 1⎞ ⎝ a 3b ⎟⎠ ⎝ 3b ⎠ ⎜⎝ 5c ⎟⎠
= 2n – ⎜ 1 + + + ..... + ⎟ = 2n – Hn

⎝ 2 3 n⎠
⎛ 3b ⎞ 5

The correct option is (B)
i.e., 3 log ⎜ ⎟ = 0 or b = c (2)
⎝ 5c ⎠ 3
79. We have, a21 – a22 + a23 – a24 + ……+ a22n–1 – a22n

From (1) and (2), we have,
= (a1– a2) [a1 + a2 + a3 +….+ a2n]
2
[ a1 – a2 = a3 – a4 = ……..a2n–1 – a2n]
a = b = 25c

2n c 9

=–d×
2
[ 2a1 + ( 2n − 1)d ] (1)
Now, we have,
Sequence and Series  7.47

5c 8c 25c
Since d1, d2 and d3 are given to be in H.P, therefore,
b+c=
+c= < =a 1 1 1 1
3 3 9
− = −
d2 d1 d3 d2

and hence, a, b, c cannot form the sides of a triangle.

The correct option is (D) 1 1 1 1

− = −
S2 − n S1 − n S3 − n S2 − n
82. Given: b2 = ac  ( a, b, c are in G.P.)
 [Using results (1), (2), (3)]
and, 2(log 2b – log 3c) = log a – log 2b + log 3c – log a S1 − S2 S2 − S3

=
 ( given terms are in A.P) ( S2 − n)( S1 − n) ( S2 − n)( S3 − n)
2
⎛ 2b ⎞ ⎛ 3c ⎞ S1 − S 2 = S 2 − S 3 ⇒ n = 2S3S1 − S1S2 − S2 S3
⇒ log ⎜
= log ⎜ ⎟ ⇒ b = 3c ⇒
3c ⎟⎠

⎝ ⎝ 2b ⎠ 2 S1 − n S3 − n S1 − 2S2 + S3
2
The correct option is (C)
Now, a = b = 3b = 9c

c 2 4 85. We have,
∴ a is the largest side
Sn = 13 + 3.23 + 33 + 3.43 + 53 +…
Let n = 2m. Then,
9c 2 81 S2m = (13 + 33 + 53 …..to m terms)
b + c −2
a 2 + c2 − c2
2
Now, cos A =
= 4 16 = negative  + 3 (23 + 43 + 63 + …..to m terms)
2bc 3
2× c ×c = {1 + 2 + 3 + 4 + ……+ (2m–1)3 + (2m)3}
3 3 3 3
2
 – { 23 + 43 +…..+ (2m)3} +3{ 23 + 43 + 63 +….+ (2m)3}
∴ A > 90º, ∴ triangle is obtuse.
2
⎡ 2m( 2m + 1) ⎤ 3 3 3 3

The correct option is (C) = ⎢
⎥ + 8 × 2{1 + 2 + 3 + .... + m }
⎣ 2 ⎦
83. The middle term of the 4n + 1 terms is the (2n + 1)
2 2
th term. Let it be m. = m2 (2 m + 1)2 + 16 m ( m + 1)

The middle term of (2n + 1) terms is the (n + 1) th term. 4
Thus, the middle term of the A.P. is n2 ( n2 + 3n + 1) n
=
 [Put m = ]
= m – (n + 1– 1)2 = m –2n 2 2
and the middle term of the G.P is
The correct option is (B)

HINTS AND EXPLANATIONS


n + 1−1 86. Since p, q, r are in A.P. we have
=m ⎛ ⎞
1 m


⎜⎝ 2 ⎟⎠ = p – q = q – r = k (say) (1)
2n

According to the given condition, we have a−x a− y a−z
Given: = =
px qy rz
m

m – 2n =
2n ⎛a ⎞ ⎛a ⎞ ⎛a ⎞ ⎛a ⎞
⎜⎝ x − 1⎟⎠ − ⎜⎝ y − 1⎟⎠ ⎜⎝ y − 1⎟⎠ − ⎜⎝ z − 1⎟⎠
n.2n + 1 ⇒ =

m= p −q q−r
2n − 1
 (by componendo–dividendo)

The correct option is (B)

84. Let the common difference of the three A.P.s be d1, d2 and d3 a a a a
⇒ − = −  [Using equation (1)]
Then, we have x y y z
n
S1 = [ 2.1 + ( n − 1)d1 ] 1 1 2 1 1
2 1 1
⇒ − = − ⇒ = +
x y y z y x z
2( S1 − n)

d1 = (1) ∴ x, y, z are in H.P.
n( n − 1)
The correct option is (C)
Similarly, d2 = 2(S 2 − n )(2)
87. We have,
n( n − 1) S = 1 + (1 + a)b + (1 + a + a2) b2 + (1 + a + a2 + a3) b3 +…∞

and, d3 = (S 3 − n )(3)
2
= (1 + b + b2 +….∞) + a (b + b2 + b3 +…..∞)
n( n − 1)
+ a2 (b2 + b3 + ……∞) +….∞
7.48  Chapter 7

1 ab a 2b 2 1 34 ± 14 2 8
= + + + ....∞ = ⇒ d= = ,
1− b 1− b 1− b (1 − b)(1 − ab) 30 3 5

The correct option is (A) d 2y 2
Now, = 34 – 30 d > 0 for d =
88. We have, dx 2 3
8
( ) − (y )  < 0 for d =
− n −1 2 − n −1 2
2 2
bn = x 5
8
Hence, y is maximum for d =
= (x − y ) (x
2 − n −1 2 − n −1 2 − n −1
+y 2 − n −1
) 5

The correct option is (A)
= bn + 1 an + 1
90. Let tn denotes the nth term of the sequence
b
an = n −1(Putting n in place of n – 1) 1, 2, 4, 7
i.e.,
bn Let S = 1 + 2 + 4 + 7 + ….+ tn
b0 b1 b2 bn−1 x−y Again, S = 1 + 2 + 4 + …..+ tn–1 + tn
∴ a1, a2… an = . . .... =
b1 b2 b3 bn bn On subtracting, we get
0 = 1 + [1 + 2 + 3 +……(n – 1) terms] – tn

The correct option is (B) n ( n − 1) n2 − n + 2
⇒ tn = 1 + =
89. Given: a1 + 5d = 2 2 2
Let y = a1 a4 a5 = a1 (a1 + 3d) (a1 + 4d) which denotes the first term of the nth row which contains
= (2 – 5d) (2 – 2d) (2 – d) n terms in A.P., having common difference 1.
(Putting a1 = 2 – 5d) Hence, we have,
= 2 (4 – 16d + 17d 2 – 5d3) n
S=
⎡ 2t n + ( n − 1)1⎤⎦
The value of d at which y attains maxima is given by 2⎣
dy n n ( n2 + 1)
= 0 (by calculus) = ⎡⎣ n2 − n + 2 + n − 1⎤⎦ =
dx 2 2
The correct option is (C)
⇒ – 16 + 34 d – 15 d2 = 0
HINTS AND EXPLANATIONS

Previous Year’s Questions

91. ∵1,log3 31− x + 2 ,log3 ( 4 ⋅ 3x − 1) are in AP The correct option is (B)

∴ 2 log3 (31− x + 2)1/ 2 = log3 3 + log3 ( 4 ⋅ 3x − 1) 92. The product 21/4 ⋅ 41/4 ⋅ 81/16 …



⇒ log3 (31− x + 2)1/ 2 = log3 3( 4 ⋅ 3x − 1) = 21/ 4 ⋅ 22 / 8 ⋅ 23/16...

1⎡ 2 3 ⎤
Let 3x = t, then ⎢1+ 2 ⋅ 22 + ...⎥
= 24 ⎣ ⎦
3
+ 2 = 12t − 3 ⎡ 1 ⎤
t ⎢
1⎢ 1

+ 2 ⎥
⇒ 12t2− 5t − 3 = 0 4 ⎢ 1 ⎛ 1⎞ 2 ⎥
⎢1− 1− ⎥
⇒ (3t + l)(4t− 3) = 0 2 ⎜⎝ 2 ⎟⎠ ⎥⎦
= 2 ⎢⎣
1
1 3 [ 2 + 2]
⇒ t=− , = 24
3 4
=2
x 3
⇒ 3 = (neglecting the negative value) The correct option is (B)
4
93. Since fifth term of a GP = 2
⎛ 3⎞
⇒ log3 ⎜ ⎟ = x ∴ ar4 = 2
⎝ 4⎠
where a and r are the first term and the common ratio
⇒ x = log3 3 − log3 4 respectively of a GP.
Now required product
⇒ x = 1 − log3 4
Sequence and Series  7.49

= a × ar × ar2 × ar3 × ar4 × ar5 × ar6 × ar7 × ar8 n


r+n−r n
n
= a9r36 = (ar4)9 = 29 = 512 2t n = ∑ n
Cr
= ∑ nC
r=0 r=0 r
The correct option is (B)
94. Key Idea : The number of triangles those can be formed n n 1 n t n
using n points = nC3
⇒ tn = ∑ = sn ⇒ Sn = 2
2 r = 0 nCr 2
n
∵Tn = nC3
The correct option is (A)
∴ Tn +1 − Tn = 21 1
98. Given that Tm = = a + ( m − 1)d (1)
⇒ n +1C3 − nC3 = 21 n

1
⇒ nC2 + nC3 − nC3 = 21 And Tn = = a + ( n − 1)d (2)
m
(∵ nC2 + nC3 = n −1
C3 ) 1 1
From (1) and (2) we get a = ,d=
mn mn
Hence, a − d = 0
⇒ nC2 = 21
The correct option is (A)
n( n − 1)
⇒ = 21 ⇒ n2 − n − 42 = 0 99. If n is odd then (n − 1) is even and so the sum of odd terms
2
⇒ ( n − 7)( n + 6) = 0 ( n − 1)n2 n2 ( n + 1)
= + n2 = .
2 2
⇒n=7 (∵ n ≠ −6)
The correct option is (D)
The correct option is (B)

1 1
95.
1

1
+
1
− ..........
100. x= ∑ an = 1 − a ⇒a = 1−
x
n= 0
1.2 2.3 3.4

1 1
= 1−
1 1 1 1 1
− + + − − .........
y= ∑ bn = 1 − b ⇒ b = 1 − y
2 2 3 3 4 n= 0

1 1
⎛1 1 1 ⎞
= 1 − 2 ⎜ − + − ........⎟ (Rearranging the terms) z= ∑ cn = 1 − c ⇒ c = 1 − z
⎝2 3 4 ⎠ n= 0

a, b, c are in A.P.
⎛ 1 1 1 ⎞
= 2 ⎜1 − + − + ........⎟ − 1 2b = a + c
⎝ 2 3 4 ⎠

HINTS AND EXPLANATIONS


⎛ 1⎞ 1 1
= 2 log 2 − log e 2 ⎜1 − ⎟ = 1 − + 1 −
⎝ y⎠ x y

⎛ 4⎞
= log ⎜ ⎟ . 2 1 1
⎝ e⎠ = +

y x z
The correct option is (D) ⇒ x, y, z are in H. P
96. f (1) = 7 The correct option is (D)
f (1 + 1) = f (1) + f (1) 101. Given condition implies that
⇒ f (2) = 2 ×7 p
[2a + ( p − 1)d ] p 2 2a + ( p − 1)d p
Also, f (3) = 3 ×7 2 1 = 2 ⇒ 1 =
n q
[2a1 + ( q − 1)d ] q 2a1 + ( q − 1)d q
∴ ∑ f ( r ) = 7(1 + 2 + ......... + n) 2
r =1
⎛ p − 1⎞
a1 + ⎜ d
n( n + 1)
=7 ⎝ 2 ⎟⎠ p
2 =
⎛ q − 1⎞ q
a1 + ⎜ d
The correct option is (D) ⎝ 2 ⎟⎠

n
r
97. Given t n = ∑ nC For
a6
, p = 11, q = 41,
r=0 r a21

(∵ C )
n n
n−r n−r a6 11
Also, t n = ∑ nC = ∑ n
Cr
n
r = nCn − r =
a21 41
r=0 n−r r=0
Adding above two equalities we write The correct option is (D)
7.50  Chapter 7

1 1 1 1 1 1 2 4 4⎛ 1 1 ⎞
102. Given that − = − = .... = − = d ( say ) S = + 2 ⎜1 + + 2 + .... ⎟
a2 a1 a3 a2 an an −1 3 3 3 ⎝ 3 3 ⎠
a1 − a2 a −a a −a ⎛ ⎞
Then a1a2 = , a2 a3 = 2 3 ,...., an−1an = n−1 n 2 4 4⎜ 1 ⎟ 4 4 3 4 2 6 2 6
d d d ⇒ S= + 2⎜ ⎟= + 2 = + = ⇒ S =
3 1
3 3 ⎜1− ⎟ 3 3 2 3 3 2 3 3
⎜ ⎟
a1 − an ⎝ 3⎠
∴ a1a2 + a2 a3 + .... + an−1an =
d ⇒S =3
1 1 The correct option is (B)
Also, = + ( n − 1)d
an a1

107. Till 10th minute number of counted notes = 1500
a −a
⇒ 1 n = ( n − 1)a1an n
d 3000 = [2 × 148 + ( n − 1)( −2)] = n[148 − n + 1]
2
The correct option is (D)
n2−149n + 3000 = 0
103. Given that arn-1 = arn + arn + 1 n = 125, 24
⇒ 1 = r + r2 n = 125 is not possible.
5 −1
∴r = Total time = 24 + 10 = 34 minutes.
2
The correct option is (A)
The correct option is (D)
108. 1 2 3 4 5 6 ……
104. Using the condition A.M. ≥ G.M., we write
200 200 200 240 280 ……. ……..
p2 + q2
≥ pq Sum = 11040
2
120 + 80 + 160 + 40 + 200 + 240 + … = 11040
1
⇒ pq ≤ n
2
2
[2a + ( n − 1)d ] + 80 + 40 = 11040
(p + q)2 = p2 + q2 + 2pq
n
⇒ p+q≤ 2
2
[240 + ( n − 1)40] = 10920

The correct option is (D)
HINTS AND EXPLANATIONS

n [6 + n − 1] = 546
105. Let a, ar, ar2, ar3 be the first four terms of a G.P., then n (n + 5) = 546
a + ar = 12    (1) n = 21
ar2 + ar3 = 48  (2) The correct option is (C)
dividing (2) by (1), we have
109. Statement 1 has 20 terms whose sum is 8000
ar 2 (1 + r )
=4 And statement 2 is true and supporting statement 1.
a( r + 1) th 2 2 2
∵ k bracket is (k – 1) + k(k – 1) + k = 3k – 3k + 1.
⇒ r2 = 4  if r ≠ −1
The correct option is (B)
∴ r = −2
also, a = −12  (using (1))
The correct option is (B) 110. 100(T100 ) = 50(T50 )

⇒ 2[a + 99d ] = a + 49d


2 6 10 10
1 06. Let S = 1 + + 2 + 3 + 4 + ......  (1) ⇒ a + 149d = 0
3 3 3 3
⇒ T150 = 0

1 1 2 6 10 The correct option is (D)
S = + 2 + 3 + 4 + ....  (2)
3 3 3 3 3
111. tr = .7 + .77 + . . .r times
Subtracting (2) from (1), we get

(
= 7 10 −1 + 10 −2 + 10 −3 + ..... + 10 − r )
⎛ 1⎞ 1 4 4 4
S ⎜1 − ⎟ = 1 + + 2 + 3 + 4 + ....
= (1 − 10 )
⎝ 3⎠ 7 −r
3 3 3 3
9
Sequence and Series  7.51

Now, The correct option is (C)


20
7⎛ 20 ⎞
S20 = ∑ t r = ⎜⎜ 20 − ∑10 − r ⎟⎟
r =1 9 ⎝ r =1 ⎠ 115. tr =
∑ r 3 = r 2 ( r + 1)2 = 1 (r + 1)2
∑ (2r − 1) 4r 2 4
7⎛
9⎝
1
(
= ⎜ 20 − 1 − 10 −20 ⎟ =
9


) 7
81
(
179 + 10 −20 )

1 9
The correct option is (B) Now, S9 = ∑ ( r + 1)2 , let t = r + 1
4 r =1
1 1
112. + =4
α β 1 ⎛ 10 2 ⎞
2q = p + r
= ∑ t − 1⎟⎠ = 96 .
4 ⎜⎝ t =1

⇒ −2(α + β ) = 1 + αβ The correct option is (A)
⎛ 1 1⎞ 1 116. Let ‘a’ be the first term and d be the common difference
⇒ −2 ⎜ + ⎟ = +1 2nd term = a + d, 5th term = a + 4d,
⎝ α β ⎠ αβ

9th term = 4 + 8d
1 a + 4 d a + 8d 4 d 4
⇒ = −9 ∴ Common ratio = = = =
αβ a+d a + 4 d 3d 3
Equation having roots α , β is 9 x 2 + 4 x − 1 = 0 The correct option is (C)
−4 ± 16 + 36
α,β =
2×9 82 122 16 2
117. Given series is S = 2
+ 2
+ + …10 terms
2 13 5 5 52
α −β = .
9 42
= ( 22 + 32 + 4 2 + …10 terms)
The correct option is (D) 52
16 ⎛ 11.12.23 ⎞ 16
113. Let numbers be a, ar , ar 2 = ⎜ − 1⎟ = × 505
25 ⎝ 6 ⎠ 25
Now, 2( 2ar ) = a + ar 2 [ a ≠ 0 ] ∴ m = 101
⇒ 4r = 1 + r 2 The correct option is (C)

⇒ r 2 − 4r + 1 = 0 118. 9(25a2 + b2) + 25(c2 – 3ac) = 15b(3a + c)

HINTS AND EXPLANATIONS



⇒ (15a)2 + (3b)2 + (5c)2 – 45ab – 15bc – 75ac = 0
⇒r = 2± 3
⇒ (15a – 3b)2 + (3b – 5c)2 + (15a – 5c)2 = 0
r = 2 + 3 (Positive value) It is possible when 15a – 3b = 0 and 3b – 5c = 0 and 15a –
The correct option is (D) 5c = 0
9 1 8 9 15a = 3b = 5c
114.
S = 10 + 2 ⋅ 11 ⋅ 10 + ... + 10 ⋅ 11
a b c
11 = =
⋅ S = 111 ⋅ 108 + ... + 9 ⋅ 119 + 1110 1 5 3
10 \ b, c, a are in A.P.
1 Hence, the correct option is (A)
⇒ S = 109 + 111 ⋅ 108 + 112 ⋅ 107 + ... + 119 − 1110
10 119. Let the first term be ‘a’ and common difference be ‘d’
⎛ ⎛ 11 ⎞10 ⎞ a9 + a43 = 66
⎜ ⎜ ⎟ −1 ⎟ (a + 8d) + (a + 42d) = 66
1
⇒ S = 109 ⎜ ⎝ ⎠
10 ⎟ − 1110
10 ⎜ 11 ⎟ 2a + 50d = 66
⎜ −1 ⎟
⎜ 10 ⎟ a + 25d = 33 (i)
⎝ ⎠
12
1 10 10
⇒ S = 11 − 10 − 11 10 ∑ a4 k +1 = 416
10 k =0

11 a1 + a5 + a9 + … + a49 = 416 (A)


  S = 10 Number of terms of the above series
9
  S = 100 ⋅ 10 1 + (n – 1)4 = 49
(n – 1)4 = 48
  ⇒ k = 100 .
7.52  Chapter 7

n = 13 S = 12 + 2 ⋅ 22 + 32 + 2 ⋅ 42 + 52 + 2 ⋅ 62 + …
Common difference of series A is 4d S = S1 + S2
a1 = a S1 = 12 + 22 + 32 + 42 + …
n S2 = 22 + 42 + 62+ …
[2a + ( n − 1)4 d ] = 416
2 Sum of n terms of series S1
13 ( 2n + 1)
[2a + 48d ] = 416 S1n = n( n + 1)
2 6
13[a + 24d] = 416 Sum of n1 terms of series S2 where
a + 24d = 32 (ii) n
n1 =
Solving equation (i) and (ii), we get 2
d = 1 and a = 8 S2 n = ∑ ( 2n1 ) 2 = 4∑ n12
1

In the series a12 + a22 + … + a17


2
= 140 n
( 2n1 + 1)
The nth term is (8 + (n – 1) × 1)2 = 4 n1 ( n1 + 1)

6
Tn = (n + 7)2
( 2n + 1) ( 2n + 1)
17 17 S = n( n + 1) + 4 n1( n1 + 1) 1
∑ Tn = ∑ ( n + 7) 2 6 6
n =1 n =1 A = S where n = 20 and n1 = 10
17 17 17 Solving, we get,
= ∑ n2 + 14∑ n + 49∑1
n =1 n =1 n =1 41 21
A = 20 × 21 × + 4 × 10 × 11 × = 4410
n( n + 1)( 2n + 1) 14 n( n + 1) ⎫ 6 6
⇒ + + 49n ⎬ n = 17 B=S
6 2 ⎭
Solving, we get
17 × 18 × 35 18
⇒ + 14 × 17 × + 49 × 17 = 140 m 81 41
6 2 B = 40 × 41 × + 4 × 20 × 21 × = 22140 + 11480
6 6
⇒ 1785 + 2142 + 833 = 140m
= 33620
⇒ 140m = 4760
B – 2A ⇒ 33620 – 2 × 4410
m = 34
= 100λ
120. We are required to find the sum of 20 terms and 40 terms,
HINTS AND EXPLANATIONS

therefore the number of terms are even ⇒ 24800 = 100λ


λ = 248
CHAPTER
Limits
8
LEARNING OBJECTIVES
After reading this chapter, you will be able to:
 Understand the limit of a function and the method to  Learn about algebra of limits, evaluation of limits, meth-
calculate right and left hand limit of a function ods of factorization and rationalization, trigonometric
limits, exponential and logarithmic limits
 Know how to evaluate limits using L’hospital’s rule

LIMIT OF A FUNCTION Method for Finding Right Hand Limit


Let a function f be defined at every point in the To evaluate lim f ( x )
x → a+
­neighbourhood of a (an open interval about a) except pos-
sibly at a. If as x approaches closer and closer to a, but not 1. Put x = a + h in f (x) to get
equal to a, then the value of the function f (x) approaches a 2. Take the limit as h → 0.
real number l. The number l is referred to as the limit of f (x)
as x tends to a and we write it as Left Hand Limit
lim f ( x ) = l We say that left hand limit of f (x) as x tends to ‘a’ exists
x→a
and is equal to l2 if as x approaches ‘a’ through values less
Note that f (x) approaches l means the absolute difference than ‘a’, the values of f (x) approach a definite unique real
between f (x) and l, i.e. |f (x) – l| can be made as small as number l2 and we write
we please.
When the values of f (x) do not approach a single lim f ( x ) = l2
x→ a−

finite value as x approaches a, we say that the limit Does
not exist. or f (a – 0) = l2

ERROR CHECK Method for Finding Left Hand Limit


To evaluate lim f ( x ).
A number is said to be a limiting value only if it is finite and x→ a−
real, otherwise we say that the limit does not exist. 1. Put x = a – h in f (x) to get lim f ( a + h).
h→ 0
2. Take the limit as h → 0.
Right Hand Limit
We say that right hand limit of f (x) as x tends to ‘a’ exists ERROR CHECK
and is equal to l1 if as x approaches ‘a’ through values
greater than ‘a’, the values of f (x) approach a definite For finding lim f( x ), we study the behaviour of the function f
x →a
unique real number l1 and we write in the neighbourhood of ‘a’ and not at ‘a’. Thus, the function
f may or may not be defined at x = a.
lim f ( x ) = l1 or f (a + 0) = l1
x → a+
8.2  Chapter 8

SOLVED EXAMPLES and x – ai > 0  for  i = 1, 2, …, m – 1

x 2 − 9 x + 20 x − ai
1. lim = \ Ai = = –1, for i = m, m + 1, …, n
x→5 x − [ x] − ( x − ai )
(A) 1 x − ai
and Ai = = 1, for i = 1, 2, …, m – 1
(B) 0 x − ai
(C)  Does not exist Similarly, if x is in the right neighbourhood of ai
(D)  Cannot be determined Then x – ai < 0 for i = m + 1, …., n and x – ai > 0
Solution: (C) for i = 1, 2, …, m
x − ai
x 2 − 9 x + 20 ( x − 4)( x − 5) \ Ai = = – 1 for i = m + 1, … n
lim = lim − ( x − ai )
x→5 x − [ x] x→5 x − [ x]
x − ai
( x − 4)( x − 5) and Ai = = 1 for i = 1, 2, …, m
LHL = lim x − ai
x → 5− x − [ x] 
(5 − h − 4)(5 − h − 5) Now, lim ( A1 A2 ... An ) = (–1)n – m + 1
x → am−
= lim (h > 0)
h→0 (5 − h) − [5 − h]
and lim ( A1 A2 ... An ) = (–1)n – m
x → am+
(1 − h)( − h) (1 − h)( − h)
= lim = lim Hence, lim ( A1 A2 ... An ) Does not exist.
h→0 5−h−4 h→0 (1 − h)  x → am
\ LHL = 0
( x − 4)( x − 5) INDETERMINATE FORMS
Also, RHL = lim
x→5 x − [ x]  If a unique value cannot be assigned to f (a), then f (x) is
(5 + h − 4)(5 + h − 5) said to be inderminate at x = a.
= lim 0
h→0 (5 + h) − [5 + h]  Most general of all indeterminate forms is , others
being 0
(1 + h)( h) (1 + h)h 1 1 0−0 0
= lim = lim 1. ∞ – ∞ = − = = which is indeterminate
h→0 5+h−5 h→0 h  0 0 0 0
\ RHL = 1 and hence is (∞ – ∞)
∞ 1/ 0 0
As LHL ≠ RHL 2. = = which is indeterminate and hence is
∞ 1/ 0 0
\ Limit does not exist. ⎛ ∞⎞
x − ai ⎜⎝ ⎟⎠
2. If Ai = , i = 1, 2, …, n and if a1 < a2 < a3 < … < an. ∞
| x − ai | 1 0
3. 0 × ∞ = 0. = which is indeterminate and hence is
Then lim ( A1 A2 ... An ) , 1 ≤ m ≤ n 0 0
x → am (0 × ∞)
(A)  is equal to (–1)m 4. 1∞

(B)  is equal to (–1)m + 1 Let y = 1∞
(C)  is equal to (–1)m – 1 ⇒ log y = ∞ log 1 = ∞ × 0 which is indeterminate

(D)  Does not exist and hence is 1∞
Solution: (D) 5. 00
x − ai Let y = 00
We have, Ai = , i = 1, 2, …, n
| x − ai | ⇒ log y = 0 ⋅ log 0 = 0 × ∞ which is indeterminate
and a1 < a2 < … an – 1 < an. and hence is 0º
6. ∞º
Let x be in the left neighbourhood of am. Let y = ∞º
Then,
⇒ log y = 0 ⋅ log ∞ = 0 × ∞ which is indeterminate
x – ai < 0 for i = m, m + 1, … n and hence is ∞º.
Limits  8.3

ALGEBRA OF LIMITS Solution: (B)


If lim f ( x ) = l and lim g ( x ) = m, then following results Since, 0 ≤ (rx) < 1 for r = 1, 2, 3, …, n
x→a x→a n n
are true: ⇒ 0 ≤ ∑ (rx) < ∑ (1)
r =1 r =1
1. lim [ f ( x ) + g ( x )] = lim f ( x ) + lim g ( x ) = l + m. 
x→a x→a x→a n

2. lim [ f ( x ) − g ( x )] = lim f ( x ) – lim g ( x ) = l – m.


⇒ 0 ≤ ∑ (rx) < n
x→a x→a x→a r =1

3. lim k ⋅ f ( x ) = k ⋅ lim f ( x ) = kl, Dividing throughout by n2, we have


x→a x→a n
where k is a constant. ∑ (rx)
0 r =1 1
4. lim [ f ( x ) ⋅ g ( x )] = lim f ( x ) ⋅ lim g ( x ) = lm. ≤ <
x→a x→a x→a 2 2 n
n n
lim f ( x ) n

5. lim ⎢
⎡ f ( x) ⎤ x → a
⎥ = =
l
(provided m ≠ 0). ∑ (rx)
x → a ⎣ g ( x) ⎦ lim g ( x ) m r =1 1
x→a ⇒ lim 0 ≤ lim 2
< lim
n→∞ n→∞ n n→∞ n
⎛ ⎞
6. lim ( fog ) ( x ) = lim f [ g ( x )] = f ⎜ lim g ( x )⎟ = n
⎝ x→a ⎠
x→a
f (m).
x→a
∑ (rx)
r =1
⇒ 0 ≤ lim < 0

In particular, n→∞ n2
⎛ ⎞ ( x ) + (2 x ) + ... + ( nx )
(a) lim log g ( x ) = log ⎜ lim g ( x )⎟ = log m. \ 0 ≤ lim < 0
x→a ⎝ x→a ⎠ n→∞ n2
lim g ( x)
(b) lim e g ( x ) = e x→a
= em. According to Sandwich Theorem or Squeeze Principle
x→a
( x ) + ( 2 x ) + ... + ( nx )
⎡ ⎤
n
lim =0
7. lim[ f ( x )] = ⎢ lim f ( x ) ⎥ = ln, for all n ∈ N.
n
n→∞ n2
x→a ⎣x→a ⎦
8. Sandwich Theorem (or Squeeze Principle).
If f, g and h are functions such that f (x) ≤ g (x) ≤ h (x) ERROR CHECK
for all x in some neighbourhood of the point a (except
possibly at x = a) and if lim f ( x ) = l = lim h ( x ) , The converse of the above result may not be true,
x→a x→a i.e., lim f( x ) = |l| = |l| ⇒ lim f( x ) = l
then lim g ( x ) = l. x →a x →a
x→a

9. If lim f ( x ) = l, then lim f ( x ) = |l |.


x→a x→a
EVALUATION OF LIMITS
I M P O R TA N T P O I N T S The problems on limits can be divided into the following
categories:
Sandwich theorem helps in calculating the limits, when limits
cannot be calculated using the usual method Limits

SOLVED EXAMPLE Algebraic Trigonometric Exponential and


limits limits logarithmic limits
( x ) + ( 2 x ) + (3 x ) + ... + ( nx )
3. lim , where
n→∞ n2
{x} = x – [x] denotes the fractional part of x, is
ALGEBRAIC LIMITS
(A) 1 (B) 0
1 The following methods are useful for evaluating limits of
(C)  (D)  None of these algebraic functions:
2
8.4  Chapter 8

Method of Factorization Solution: (A)


⎡ x −3 ⎤
If f (x) and g (x) are polynomials and g (a) ≠ 0, then we have lim ⎢log a ⎥
x →3 ⎣ x + 6 − 3⎦
lim f ( x )
f ( x) x→a f ( a)
lim = = . ⎡ ( x − 3) ( x + 6 + 3) ⎤
x→a g ( x) lim g ( x ) g ( a) = lim ⎢log a ⎥
x→a x →3
⎣ ( x − 3) ⎦
Now, if f (a) = 0 = g (a), then (x – a) is a factor of both f (x)
and g (x). We cancel this common factor (x – a) from both = lim log a ( x + 6 + 3) = loga 6
x →3
the numerator and denominator and again put x = a in the
given expression. If we get a meaningful number then that Standard Formula
number is the limit of the given expression, otherwise we x n − an
repeat this process till we get a meaningful number. lim = nan – 1,
x→a x−a
where n ∈ Q, the set of rational numbers.
SOLVED EXAMPLE
3
x2 − 2 3 x + 1 SOLVED EXAMPLE
4. lim is equal to
x →1 ( x − 1) 2 x r − 1r
n
6. lim ∑ =
r =1 x − 1
1 1 x →1
(A)  (B)  n( n + 1)
9 6 (A)  0 (B) 
1 2
(C)  (D)  None of these (C)  1 (D)  None of these
3
Solution: (A) Solution: (B)
n
x r − 1r
3
x2 − 2 3 x + 1 y2 − 2 y + 1 We have, lim ∑
= lim r =1 x − 1
lim x →1
x →1 ( x − 1) 2 y →1 ( y 3 − 1) 2 
 n
n( n + 1)
3
[Putting x = y; as x → 1, y → 1]
= ∑ r ⋅1 r−1
=1+2+3+…+n=
2
.
r =1
( y − 1) 2
=
lim LIMIT OF AN ALGEBRAIC FUNCTION
y →1 ( y − 1) 2 ( y 2 + y + 1) 2
 WHEN X → ∞
1 1 f ( x)
=
lim 2 = . In order to find the limit of a function of the type as
y →1 ( y + y + 1) 2 9 g ( x)
x → ∞, where f (x) and g (x) are algebraic functions of x, it
Method of Rationalization is convenient to divide all the terms of f (x) and g (x) by the
This method is useful where radical signs (i.e., expressions highest power of x in numerator and denominator both and
of the form a ± b ) are involved either in the numera- use the following standard limits:
tor or in the denominator or both. The numerator or (and) 1
1. lim =0
the denominator (as required) is (are) rationalised and limit x →∞ x
taken after cancelling out the common factors. 1
2. lim p = 0, if p > 0.
x →∞ x
SOLVED EXAMPLE
QUICK TIPS
⎛ x −3 ⎞
5. The value of lim ⎜ log a ⎟ is ∞, if a > 1
x →3 ⎝ x + 6 − 3⎠ ⎧
⎪ 1, if a = 1

(A) loga 6 ■ lim a n = ⎨
n→∞ ⎪ 0, if − 1 < a < 1
(B) loga 3 ⎪⎩does not exist, if a ≤ − 1
(C) loga 2
(D)  None of these
Limits  8.5

1 n 3
p
a0 x + a1x p −1
+ … … + ap − 1 x + ap \ S= ∑ [r − 2nr 2 + (n2 − 2n − 1)r + n2 ] 
2 r =1
■ lim
q q −1
x→∞ b0 x + b1x + … … + bq − 1 x + bq 2
1 ⎡ ⎧ n( n + 1) ⎫ ⎧1 ⎫
⎧ a0 ⇒ S= ⎢ ⎨ 2 ⎬ − 2n ⎨ 6 n ( n + 1)( 2n + 1)⎬
⎪b , if p = q 2 ⎣ ⎩ ⎭ ⎩ ⎭
= ⎪⎪0,
0
⎧1 ⎫ ⎤
⎨ if p < q + ( n2 − 2n − 1) ⎨ n( n + 1)⎬ + n2 ( n) ⎥
⎪∞, if p > q  ⎩2 ⎭ ⎦

⎪⎩ Solving and rearranging, we have
1 4
S = ( n − 11n3 − 19n2 + 6 n) 
24
Some Useful Summations
S 1 ⎛ n4 − 11n3 − 19n2 + 6 n ⎞
n ( n + 1) ∴ lim 4 = lim ⎜ ⎟
1. S n = 1 + 2 + 3 + … + n = n→∞ n n → ∞ 24 ⎝ n4 ⎠
2
n ( n + 1) ( 2n + 1)
1 ⎛ 11 19 6 ⎞
= lim ⎜1 − − 2 + 3 ⎟ 
2. S n2 = 12 + 22 + 32 + … + n2 = 24 n → ∞ ⎝ n n n ⎠
6
n −1 ⎧
⎪⎫
2 n−r
⎡ n ( n + 1) ⎤ 1 ⎪ 1
3. S n3 = 13 + 23 + 33 + … + n3 = ⎢ ⎥
∴ lim
n→∞ n 4
∑ ⎨⎪(r + 1) ∑ k ⎬⎪ =
24

⎣ 2 ⎦ r =0 ⎩ k =1 ⎭
n–1 2 n–1 a (1 − r n )
4. S ar = a + ar + ar + … + ar = ; n ⎛ r 3 − 1⎞
­provided r < 1. 1− r 8. lim
n→∞
∏ ⎜⎝ r 3 + 1⎟⎠
r=3
1 6
SOLVED EXAMPLES (A)  (B) 
3 7
2
7. The value of (C) − (D)  None of these
3
1 ⎡ ⎛ n ⎞ ⎛ n −1 ⎞ ⎛n−2 ⎞ ⎤
Solution: (B)
lim ⎢1
n → ∞ n4 ⎢ ⎝
⎜ ∑ k ⎟ + 2 ⎜ ∑ k ⎟ + 3 ⎜ ∑ k ⎟ + ... + n ⋅ 1⎥
⎥⎦
⎣ k =1 ⎠ ⎝ k =1 ⎠ ⎝ k =1 ⎠ (n – 2)th factor of the series is
will be n − 1 n2 + n + 1
tn = ⋅
1 1 1 1 n + 1 n2 − n + 1
(A)  (B)  (C)  (D) 
24 12 6 3 Therefore, required limit = lim t3t 4 t5 ... t n − 2 t n − 1t n
n→∞
Solution: (A)
⎡⎛ 2 3 4 n − 3 n − 2 n − 1⎞
The (r + 1)th term of the series is =
lim ⎢⎜ ⋅ ⋅ ... ⋅ ⋅
n−r n→∞ ⎝ 4 5 6
⎣ n −1 n n + 1⎟⎠

tr + 1 = ( r + 1) ∑ k
⎛ 13 ⎞ 21 31 n + n +1 ⎤
2
k =1
 ⋅ ⎜ ⎟ ⋅ ⋅ ... 2 ⎥
⇒ tr + 1 = (r + 1)[1 + 2 + 3 + … (n – r) terms] ⎝ 7 ⎠ 13 21 n − n + 1⎦

1 2 ⋅ 3 n2 + n + 1 6
⇒ tr + 1 = ( r + 1) ( n − r )( n − r + 1) =
lim ⋅ = .
2 n → ∞ n( n + 1) 7 7

1
⇒ tr + 1 = ( r + 1)( n − rn + n − rn + r 2 − r )
2 9. If [x] denotes the integral part of x, then
2 
1 1 ⎛ n 2 ⎞
⇒ tr + 1 = ( r + 1)( r 2 − (1 + 2n)r + n2 ) ⎜ ∑ [k x ]⎟ =lim
2 n→∞ n3 ⎝ ⎠
 k =1
1 3 x
⇒ tr + 1 = ( r − 2nr 2 + ( n2 − 2n − 1)r + n2 ) (A) 0 (B) 
2  2
n −1
x x
Now, S= ∑ tr + 1 (C)  (D) 
3 6
r=0

8.6  Chapter 8

Solution: (C) Solution: (C)
1 ⎛ ⎞ 5r + 2 r
n n
L = lim 3
n→∞ n ⎝
⎜ ∑ [k x ]⎟
2
Required limit = lim
n→∞
∑ 10 r
k =1 ⎠ r =1


Since k2x – 1 ≤ (k2x) < k2x ⎧⎪⎛ 1 ⎞ r ⎛ 1 ⎞ r ⎫⎪
n
= lim ∑ ⎨⎜ ⎟ + ⎜ ⎟ ⎬
⎝ ⎠ ⎝ 5⎠ ⎪
⎩ 2
n n n n→∞
r =1⎪ ⎭
⇒ ∑ (k 2 x − 1) ≤ ∑ (k 2 x) < ∑ k2x
k =1 k =1 k =1 ⎧
 ⎛ 1⎞ ⎫
n n
⎛ 1⎞
⎛ n ⎞ n n ⎛ n ⎞ ⎪ 1− ⎜ ⎟ 1− ⎜ ⎟ ⎪
⎪1 ⎝ 2⎠ 1 ⎝ 5⎠ ⎪
⇒ x ⎜ ∑ k 2 ⎟ − ∑ (1) ≤ ∑ [k 2 x ] < x ⎜ ∑ k 2 ⎟ = lim ⎨ ⋅ + ⎬
⎝ k =1 ⎠ k =1 ⎝ k =1 ⎠ n→∞ 2
⎪ 1 5 1 ⎪
k =1
 1− 1−
⎪ 2 5 ⎪
xn( n + 1)( 2n + 1) ⎩ ⎭
⇒ −n
6 1 5
n
= 1+ =
xn ( n + 1)( 2n + 1) 4 4
≤ ∑ [k 2 x ] <
k =1 6 2x − 3 2 x 2 + 5x
11. lim f ( x ), where < f (x) < , is
Dividing throughout by n3, we have x →∞ x x2
xn ( n + 1)( 2n + 1) 1 (A) 1 (B) 2 (C) –1 (D) –2
3

6n n2 
Solution: (B)
n
[k 2 x ] xn ( n + 1)( 2n + 1)
≤ ∑ n3
<
6 n3 lim
2x − 3 ⎛ 3⎞
= lim ⎜ 2 − ⎟ = 2
k =1
x →∞ x x →∞ ⎝ x⎠
x⎛ 1⎞ ⎛ 1⎞ 1 n
[k 2 x ] 2 x 2 + 5x ⎛ 5⎞
⇒ ⎜1 +
6⎝
⎟ ⎜2 +
n⎠ ⎝ n
⎟⎠ − 2 ≤
n
∑ n3 and lim
x →∞ 2
= lim ⎜ 2 + ⎟ = 2,
x →∞ ⎝ x⎠
k =1 x
x⎛ 1⎞ ⎛ 1⎞ \ Using Sandwitch theorem, lim f ( x ) = 2.
< ⎜1 + ⎟ ⎜2 + ⎟ x →∞
6⎝ n⎠ ⎝ n⎠
Taking limits as n → ∞, we get TRIGONOMETRIC LIMITS
⎡x ⎛ 1⎞ ⎛ 1⎞ 1⎤ For finding the limits of trigonometric functions, we use
lim ⎢ ⎜1 + ⎟ ⎜2 + ⎟⎠ − 2 ⎥ ≤ L
n→∞ ⎣6 ⎝ n⎠ ⎝ n n ⎦ trigonometric transformations and simplify. The following
results are quite useful:
x⎛ 1⎞ ⎛ 1⎞
< lim⎜⎝1 + ⎟⎠ ⎜⎝ 2 + ⎟ sin x
6 n→∞ n n⎠ 1. (a) lim =1
x →0 x
1
Since, as n → ∞, we have → 0 (b) lim cos x = 1
n x →0

x x tan x
⇒ ≤L< (c) lim =1
3 3
x →0 x
sin −1 x
According to Squeeze Principle or Sandwich Theorem, (d) lim =1
x→0 x
we have
x tan −1 x
L = . (e) lim =1
3 x→0 x
sin x 0 π
⎪⎧ 7 29 133 5n + 2n ⎪⎫ (f) lim = .
10. lim ⎨ + 2 + 3 + ... + ⎬ is equal to x →0 x 180
n → ∞ ⎪10
⎩ 10 10 10 n ⎪⎭
2. lim f ( x ) = lim f ( a + h) , where a ≠ 0, on taking
3 5 1 x→a h→ 0
(A)  (B) 2 (C)  (D) 
4 4 2 x = a + h.
Limits  8.7

Some Useful Expansions x


sin 2

1
= lim ⋅ 2 ⋅ 1 + cos x + cos x 
x3 x5 x7 x→0 2 x x
1. sin x = x − + − + ... ∞; x ∈  cos ⋅ cos x
3! 5! 7 ! 2 2
x 2 x 4 x6 1 3 3
2. cos x= 1 − + − + ... ∞; x ∈  = ⋅1⋅ =
2! 4 ! 6 ! 2 1 2
x3 2 17 7 62 9 ⎛ x x x x⎞
3. tan x = x + + x3 + x + x + ... ∞ ; 13. lim ⎜ cos cos cos … cos n ⎟ =
3 15 315 2835 n→∞ ⎝ 2 4 8 2 ⎠
π
|x| < x sin x
2 (A)  (B) 
x2 5 4 61 6 π sin x x
4. sec x = 1 + + x + x + ... ∞; 0 < |x| < (C)  0 (D)  None of these
2 24 720 2
1 x 7 3 31 5 Solution: (B)
5. cosec x = + + x + x + ... ∞; 0 < |x| < p
x 6 360 15120 x x x x
3
Required Limit = lim cos cos 2 cos 3 ... cos n
1 x x 2 5 n→∞ 2 2 2 2
6. cot x = − − − x − ... ∞; 0 < |x| < p
x 3 45 945 1 ⎡ x x ⎛ x x ⎞⎤
= lim ⎢ cos ... cos n − 1 ⎜ 2 sin n cos n ⎟ ⎥
1 x 3 1.3 x 5 1. 3 . 5 x 7 x 2 ⎝ 2 ⎠⎦
2 sin n ⎣
n→∞ 2 2
7. sin– 1 x = x + + + + ... ∞; |x| < 1
2 3 2.4 5 2 . 4 . 6 . 7 2
π ⎡ x ⎞⎤
8. cos– 1 x = − sin −1 x = lim
1 x ⎛ x
cos ... ⎜ 2 cos n − 1 sin n − 1 ⎟ ⎥
2 ⎢ ⎝
x
2 sin n ⎣
n→∞ 2 2 2 2 ⎠⎦
π ⎪⎧ 1 x 3 1.3 x 5 1.3.5 x 7 ⎪⎫ 2
=
− ⎨x + + ⋅ + ⋅ + ... ∞⎬; |x| < 1
2 ⎪⎩ 2 3 2.4 5 2.4.6 7 ⎪⎭ …………………………………………......................
…………………………………………......................
9. tan–1x =
⎧ x3 x5 x7 1 ⎛ x x⎞ sin x
x − + − + ... ∞; | x |<1 = lim ⎜ 2 cos sin ⎟ = lim
⎪ n→∞ n x ⎝ 2 2⎠ n→∞ n ⎛ x⎞
⎪ 3 5 7 2 sin n 2 sin ⎜ n ⎟
⎨ 2 ⎝2 ⎠
⎪± π − 1 + 1 − 1 + 1 − ... ∞; ⎧ + if x ≥ 1
⎪⎩ 2 x 3x 3 5 x 5 7 x 7 ⎨ ⎧ ⎛ x⎞ ⎫
⎩ − if x ≤ − 1
sin x ⎪⎪ ⎜⎝ n ⎟⎠ ⎪⎪ sin x
2
= lim ⎨ ⎬ = .
SOLVED EXAMPLES x n→∞⎪ ⎛ x ⎞ ⎪ x
sin
⎪⎩ ⎜⎝ 2n ⎟⎠ ⎪⎭
1 − cos3 x
12. lim =
x → 0 x sin x cos x ⎛ 1 θ 1 θ 1 θ⎞
14. lim ⎜ tan θ + tan + 2 tan 2 + … + n tan n ⎟ =
3 1 n→∞ ⎝ 2 2 2 2 2 2 ⎠
(A) 
2 2 1 1
(A)  (B)  − 2 cot 2θ
(C)  1 (D)  None of these θ θ
(C)  2 cot 2q (D)  None of these
Solution: (A)
1 − cos3 x Solution: (B)
lim
x → 0 x sin x cos x sin 2θ
tan 2q =
(1 − cos x ) (1 + cos3 x + cos x ) cos 2θ 
= lim
x→0 x sin x cos x  1 2 sin θ cos θ
=
2 x 2 cot 2θ cos 2 θ − sin 2 θ 
2 sin (1 + cos x + cos x )
= lim 2
x→0 x x cos 2 θ − sin 2 θ
2 x sin cos cos x ⇒ cot 2q =
2 2  2 sin θ cos θ 
8.8  Chapter 8

cos 2 θ sin 2 θ 4 4 2 3
⇒ 2cot 2q = −  (A)  (B) – (C)  (D) 
sin θ cos θ sin θ cos θ 3 3 3 4
⇒ 2cot 2q = cot q – tan q Solution: (A)
⇒ tan q = cot q – 2 cot 2q(1) ⎡ ⎛π ⎞ ⎛π ⎞⎤
2 ⎢ 3 sin ⎜ + h⎟ − cos ⎜ + h⎟ ⎥
Now, tan q = cot q – 2 cot 2q ⎝ 6 ⎠ ⎝ 6 ⎠⎦
lim ⎣
1 θ 1 θ h→ 0 3h ( 3 cos h − sin h)
⇒ tan = cot − cot θ
2 2 2 2  ⎡ ⎛1 3 ⎞ ⎛ 3 1 ⎞⎤
1 θ 1 θ 1 2 ⎢ 3 ⎜ cos h + sin h⎟ − ⎜ cos h − sin h⎟ ⎥
⇒ tan 2 = cot − cot θ ⎢ ⎝2 2 ⎠ ⎝ 2 2 ⎠ ⎥⎦
2 2
2 22 2 2  = lim ⎣
h→ 0 3h( 3 cos h − sin h)
1 θ 1 θ 1 θ
⇒ n
tan n
= n cot n − n − 1 cot n − 1 2 [ 2 sin h]
2 2 2 2 2 2  = lim
1 θ
h→ 0 3 h ( 3 cos h − sin h)
⇒ S = − 2 cot 2θ + n cot n 
2 2  sin h
4⋅
Therefore, h
= lim
θ 1 θ θ⎞ h→ 0 3 ( 3 cos h − sin h)
⎛ 1 1 
lim ⎜ tan θ + tan + 2 tan 2 + ... + n tan n ⎟
n→∞⎝ 2 2 2 2 2 2 ⎠ 4 4
= =
⎛ 1 θ⎞ 3 ( 3 − 0) 3
= lim S = lim ⎜ − 2 cot 2θ + n cot n ⎟
n→∞ n→∞⎝ 2 2 ⎠ n
x − sin x n

⎛ θ ⎞ 17. If lim is non-zero finite, then n may be


x → 0 x − sin n x
1⎜ n ⎟
= − 2 cot 2θ + lim ⎜ 2 ⎟ equal to
n→∞θ θ
⎜ tan ⎟ (A) 1 (B) 2
⎝ 2n ⎠  (C)  3 (D)  None of these
1
= − 2 cot 2θ + Solution: (A)
θ
tan ([ − π 2 ]x ) − x 2 tan ([ − π 2 ]) x − sin x
15. lim equals, where [ ] Clearly, for n = 1, lim = 1.
x→0 x − sin x
x→0 sin 2 x
denotes the greatest integer function ⎛ cot 4 x ⎞
18. lim ⎜ cosec3 x ⋅ cot x − 2 cot 3 x ⋅ cosec x + is
(A) 0 (B) 1 x →0 ⎝ sec x ⎟⎠
(C)  tan 10 – 10 (D)  ∞
equal to
Solution: (C) (A) 1 (B) – 1
Since, [–p2] = [–9.87] = –10, therefore (C)  0 (D)  None of these
tan[ − π 2 ]x 2 − x 2 tan[ − π 2 ]
lim Solution: (A)
x→0 sin 2 x
tan ( − 10 x 2 ) − x 2 tan ( − 10) x2 ⎡ cot 4 x ⎤
=
lim lim ⎢cosec3 x ⋅ cot x − 2 cot 3 x ⋅ cosec x + ⎥
x→0 x2 sin 2 x  x →0
⎣ sec x ⎦
2
tan (10 x ) x tan 10 2 ⎛ cos x 2 cos3 x cos5 x ⎞
= – lim + lim = lim ⎜ 4 − + ⎟
x→0 x2 x→0 x2  x → 0 ⎝ sin x sin 4 x sin 4 x ⎠

tan (10 x 2 ) cos x (1 − cos 2 x ) 2
= lim ( − 10) + tan 10 = lim = lim cos x = 1
x→0 10 x 2  x →0 sin 4 x x →0
= tan 10 – 10
⎡ ⎛π ⎞ ⎛π ⎞⎤ EXPONENTIAL AND LOGARITHMIC LIMITS
2 ⎢ 3 sin ⎜ + h⎟ − cos ⎜ + h⎟ ⎥
⎝6 ⎠ ⎝6 ⎠⎦
16. lim ⎣ is equal to For finding the limits of exponential and logarithmic func-
h→ 0 3h ( 3 cos h − sin h) tions, the following results are useful:
Limits  8.9

ex − 1 ⎛ ⎞
1. lim =1 x 2 x 4 x6 x2
x→0 x ⎜1 − 2 ! + 4 ! − 6 ! + ...⎟ − 1 + 2
⎝ ⎠ 
ax − 1 = lim
2. lim = loge a, a > 0 x→0 x 4
x→0 x
⎛1 ⎞
ax − bx ⎛ a⎞ = lim ⎜ + terms containing x and its powers⎟
3. lim = loge ⎜ ⎟ ; a, b > 0 x → 0 ⎝ 4! ⎠
x→0 x ⎝ b⎠ 
n
(1 + x ) − 1 1 1
4. lim =n = =
x→0 x 4! 24 
n
⎛ 1⎞
5. lim (1 + x )1/ x = lim ⎜1 + ⎟ = e
x→0 n →∞ ⎝ n⎠ ERROR CHECK
1/ h a
6. lim (1 + a h) =e If [⋅] denotes the greatest integer function, then
h→ 0
log x lim [ − x ] = [0] = 0
7. lim = 0, (m > 0) x →0
x →∞ xm Is the above statement true?
log a (1 + x ) No. If fact, lim [ − x ] = lim [0 − x ] = lim − 1 = –1
8. lim = loga e, (a > 0, a ≠ 1) x →0 x →0 x→ 0
x→0 x
Thus, limit must be applied only after removing [⋅] sign.
x
⎛ a⎞ a
9. lim ⎜1 + ⎟⎠ = e
x →∞ ⎝ x
f ( x) SOLVED EXAMPLES
⎛ 1 ⎞
10. lim ⎜1 + = e, where f (x) → ∞ as x → ∞.
x →∞ ⎝ f ( x ) ⎟⎠ x sin{x}
19. lim , where {x} denotes the fractional part of x,
x →1 x − 1
11. lim (1 + f ( x )1/ f ( x ) = e. is equal to
x→a
(A) –1 (B) 0
Some Useful Expansions (C)  1 (D)  Does not exist

x x 2 x3 Solution: (D)
1. ex = 1 + + + + ... to ∞ x sin{x}
1! 2 ! 3! LHL = lim
x −1 
x → 1−
x x 2 x3
2. e–x = 1 – + − + ... to ∞ Let x = 1 – h, as x → 1, h → 0
1! 2 ! 3!
(1 − h) sin{1 − h}
x 2 x3 ⇒ LHL = lim
3. loge (1 + x) = x – + − ... to ∞, – 1 < x ≤ 1 h→0 h 
2 3 (1 − h) sin (1 − h)
⇒ LHL = lim
x 2 x3 h→0 h
4. loge (1 ­– x) = – x – − − ... to ∞, – 1 ≤ x < 1 
2 3 (1 − h)
\ LHL = lim sin (1) = ∞
( x log a) 2 h→0 h
5. ax = ex log a = 1 + x log a + + ... to ∞
2! x sin ( x )
n ( n − 1) 2 Now, RHL = lim
x → 1+ x − 1
6. (1 + x)n = 1 + nx + x + ... to ∞, – 1 < x < 1, 
2! Let x = 1 + h, as x → 1, h → 0
n being any negative integer or fraction.
(1 + h) sin (1 + h)
The expansion formulae mentioned above can be used with ⇒ RHL = lim
h
h→0

advantage in simplification and evaluation of limits.
(1 + h) sin h
⇒ RHL = lim = lim (1 + h)
x2 h→0 h h→0

cos x − 1 +
For example, lim 2 \ RHL = (1 + 0) = 1
x→0 x4  Since LHL ≠ RHL,
\ the limit of the function Does not exist at x = 1.
8.10  Chapter 8

⎛ b⎞ ⎡ x ⎤ where [x] denotes the greatest integer less than or


lim ⎜ ⎟ ⎥ where a > 0, b > 0 and [x] denotes
20.
x →0 ⎝ x⎠ ⎢
⎣a⎦
+ equal to x, then lim f ( x ) is equal to
x →0
greatest integer less than or equal to x is 1
(A) – (B)  1
1 b 2
(A)  (B)  b (C)  (D) 0 π
a a (C)  (D)  Does not exist
Solution: (D) 4

⎛ b⎞ ⎡ x ⎤ ⎛ b ⎞ ⎡0 + h⎤ Solution: (D)
lim ⎜ ⎟ ⎥ = lim ⎜ tan −1 ([ − h] − h)
x →0 ⎝ x⎠ ⎢
+
⎣ ⎦
a h → 0 ⎝ 0 + h ⎟⎠ ⎢⎣ a ⎥⎦ LHL = lim f (0 − h) = lim
 h→ 0 h→ 0 [ − h] + 2 h 
⎛ b⎞ ⎡ h⎤ −1
tan ( −1 − h)
= lim ⎜ ⎟ ⎢ ⎥ = 0 = lim
h → 0 ⎝ h⎠ ⎣ a ⎦
h→ 0 ( 2h − 1) 
⎧ sin[ x ]
, [ x] ≠ 0 tan −1 (1 + h)
⎪ = lim
21. If f (x) = ⎨ [ x ] , where [x] denotes the h→ 0 (1 − 2h)

⎪0 , [ x ] = 0
⎩ π /4 π
greatest integer ≤ x, then lim f ( x ) equals = . =
x →0 1 4
tan −1 ([h] + h)
(A) 0 (B) –1 RHL = lim f (0 + h) = lim
(C)  1 (D)  None of these h→ 0 h→ 0 [h] − 2h 
tan −1 ( h) 1
Solution: (A) = lim =–
h→ 0 −2h 2
sin[ − h] sin( −1)
lim f (0 − h) = lim = lim = sin 1. Since LHL ≠ RHL
h→ 0 h → 0 [ − h] h→ 0 ( −1)
\ lim f ( x ) Does not exist.
sin[h] x →0
lim f (0 + h) = lim
h→ 0 h→0 [ h] 
QUICK TIPS
= 1 [∵ h → 0 ⇒ (h) → 0]
\ lim f ( x ) does not exist. ■ If lim f( x ) = A > 0 and lim g( x ) = B, then
x →0 x →a x →a

lim [f( x )]g ( x ) = AB


22. Let f (x) = x – [x], where [x] denotes the greatest ­integer x →a

≤ x and g (x) = lim


[ f ( x)] − 1
2n
, then g (x) =
■ If lim f( x ) = 1 and lim g( x ) = ∞, then
x →a x →a

[ ]
n→∞ f ( x ) 2 n + 1
lim [f( x )]g( x ) = e
lim
x→a
g( x )[ f ( x )−1]

x →a
(A) 0 (B) 1
(C)  –1 (D)  None of these
Solution: (C) SOLVED EXAMPLES
As 0 ≤ x – [x] < 1 ∀ x ∈ R, 0 ≤ f (x) < 1.
729 x − 243x − 81x + 9 x + 3x − 1
lim [ f ( x ) ]
2n
\ = 0 24. If lim = k(log 3)3,
n→∞ x→0 x3
Thus, for x ∈ R, g (x) = lim
[ f ( x)]2n − 1 then k =
(A) 4 (B) 5
[ ]
n→∞ f ( x ) 2 n + 1
 (C)  6 (D)  None of these
0 −1
= = –1 Solution: (C)
0 +1
Required limit
⎧ tan −1 ([ x ] + x ) 243x (3x − 1) − 9 x (32 x − 1) + (3x − 1)
⎪ , [ x] ≠ 0 = lim
23. If f (x) = ⎨ [ x ] − 2 x x→0 x3 
⎪0 , [ x] = 0 (3 − 1) {( 243) − ( 27) x − 9 x + 1}
x x
⎩ = lim
x→0 x3 
Limits  8.11

(3x − 1) {( 243) x − ( 27) x − 9 x + 1} ⎛ 1 + tan x ⎞


1/sin x
= lim  27. lim ⎜ is equal to
x→0 x 3 ⎟
x → 0 ⎝ 1 + sin x ⎠
(3 − 1) (9 − 1) ( 27 x − 1)
x x
(A) 0 (B) 1
= lim
x→0 x x x  (C)  –1 (D)  None of these
= log 3 ⋅ log 9 ⋅ log 27 Solution: (B)
= log 3 ⋅ 2log 3 ⋅ 3log 3 1 + tan x
Let f (x) =
= 6(log 3)3 = k(log 3)3(given) 1 + sin x 
\ k = 6 1
and g (x) = .
sin x
2
25. If a and b be the roots of ax + bx + c = 0, then Clearly f (x) → 1 and g (x) → ∞ as x → 0.
lim (1 + ax 2 + bx + c)1 ( x −α ) is 1 /sin x 1 ⎛ 1+ tan x ⎞
x→α ⎛ 1 + tan x ⎞ lim ⎜ − 1⎟
sin x ⎝ 1+ sin x ⎠
\ = e
x →0

(A)  log |a (a – b )| (B) 


e a (a – b) lim ⎜ ⎟
x → 0 ⎝ 1 + sin x ⎠

(C) ea (b – a)
Solution: (B)
(D)  None of these

{Using lim [ f ( x)]
x→a
g( x)
=e
lim g ( x ) [ f ( x ) −1]
x→a
}
1
lim ⎡(1+ ax 2 + bx + c ) −1⎤⎦ 1 ⎛ tan x − sin x ⎞ 1− cos x
( x −α ) ⎣
lim (1 + ax 2 + bx + c)1 ( x −α ) = e lim
x →α
lim ⎜ ⎟
sin x ⎝ 1+ sin x ⎠ cos x (1+ sin x )
= = e = e0 = 1
x →0 x →0

x →α e
g (x) lim g ( x )[ f ( x ) −1]
[Using lim [f (x)] = e x→a
tan
πx
x→a ⎛ x⎞ 2a
 provided f (x) → 1 and g (x) → ∞ as x → a] 28. lim ⎜ 2 − ⎟ is equal to
x→a ⎝ a⎠
( ax 2 + bx + c ) a ( x −α )( x − β )
lim lim
= e
x →α
( x −α )
= e
x →α
( x −α ) (A) ep/2 (B) 
e2/p (C) 
e–2/p (D) 
e–p/2

[∵ a, b are roots of ax2 + bx + c = 0] Solution: (B)
πx ⎛ π x⎞ ⎛ x ⎞
= ea (a – b)
tan ⋅ 2 − −1
⎛ x⎞ 2a lim tan ⎜
⎝ 2 a ⎟⎠ ⎜⎝ a ⎟⎠
lim ⎜ 2 − ⎟ = e
x→a

1 x→a ⎝ a⎠
⎛ sin x ⎞ x − a
26. lim ⎜ ⎟
x → a ⎝ sin a ⎠
, a ≠ np, n is an integer, equals

{Using lim [ f ( x)]x→a
g( x)
=e
lim g ( x ) [ f ( x ) −1]
x→a

(A) ecot a (B) 
etan a (C) 
esin a (D) 
ecos a ⎫
as f ( x ) → 1 and g ( x ) → ∞ as x → a⎬
Solution: (A) ⎭
⎛ x⎞ ⎛ π x⎞ (1− x a )
1 1 lim ⎜1− ⎟⎠ tan ⎜⎝
x→a ⎝
⎟ lim
a 2a ⎠ cot (π x 2 a )
= e = e
x→a

⎛ sin x ⎞ x −a ⎛ sin x − sin a ⎞ x −a 


lim ⎜ ⎟ = lim ⎜1 + ⎟ −1 / a
x → a ⎝ sin a ⎠ x→a ⎝ sin a ⎠ lim
x→a
⎛ π x⎞ π 2 2 ⎛ π x⎞
− cosec 2 ⎜ ⎟ ⋅ lim sin ⎜
⎝ 2 a ⎟⎠
sin x − sin a
= e ⎝ 2a ⎠ 2a
= e π
= e2/p
x→a

⎡ sin a ⎤ ( x − a )sin a
⎢ ⎧ ⎛ sin x − sin a ⎞ ⎫ sin x − sin a ⎥ a
= lim ⎢ ⎨1 + ⎜ ⎟⎬ ⎥ 29. lim (cos x + a sin bx ) x is equal to
x→a ⎩ ⎝ sin a ⎠ ⎭ x →0
⎢⎣ ⎥⎦
 (A) e − a b (B) 
2

e ab (C)  e−b a
e a b (D) 
2 2 2

sin x − sin a
= lim Solution: (C)
e ( x − a) sin a 
x→a
a a
lim (cos x + a sin bx −1)
2 ⎛ x + a⎞ ⎛ x − a⎞ 1 lim (cos x + a sin bx ) x = e x→ 0
x
= lim cos ⎜ sin ⎜ ⋅ x →0
x−a
e x→a ⎝ 2 ⎟⎠ ⎝ 2 ⎟⎠ sin a ⎡ Using lim f ( x ) ϕ ( x ) = e lim ϕ ( x )[ f ( x ) −1]



[ ] x→a

⎛ x + a⎞ ⎡ ⎛ x − a⎞ ⎛ x − a⎞ ⎤ 1  x→a
= lim cos ⎜ sin
e x→a
⎝ 2 ⎟⎠ ⎢⎣ ⎜⎝ 2 ⎟⎠ ⎜⎝ 2 ⎟⎠ ⎥⎦ sin a as f (x) → 1 and f (x) → ∞ as x → a]

cos a a ( − sin x + ab cos bx )
lim 2

= e sin a =e cot a = e x →0
1 = ea b 
8.12  Chapter 8

sin x ⎛ ⎛ x ⎞⎞
2

⎛ sin x ⎞ x − sin x ⎜ sin ⎜⎝ 2 n ⎟⎠ ⎟ x 2


30. The value of lim ⎜ ⎟ is −2 lim ⎜ ⎟ ⋅ 2 ⋅n
x →0 ⎝ x ⎠ ⎜ x ⎟ 4n
n→∞

⎜⎝ 2 n ⎟⎠
(A) 1 (B) –1 =e
(C)  0 (D)  None of these x2
−2 × lim
Solution: (B) = e n→∞
4n = e0 = 1
sin x sin x ⎛ sin x ⎞
−1⎟
⎛ sin x ⎞ x − sin x lim ⎜
EVALUATION OF LIMITS USING L’HOSPITAL’S
= e x − sin x ⎝ x ⎠
x →0
lim ⎜ ⎟
x →0 ⎝ x ⎠ RULE
⎡ Using lim f ( x ) g ( x ) = e lim g ( x )[ f ( x ) −1]

⎣⎢ x→a
[ ] x→a Besides the methods given above to evaluate limits, there
is yet another method for finding limits, usually known as
sin x L’Hospital’s Rule as given below for indeterminate forms:
as f ( x) = → 1 and g ( x )
x ⎛ 0⎞
sin x 1. ⎜ ⎟ form: If lim f ( x ) = 0 and lim g ( x ) = 0, then
sin x ⎝ 0⎠ x→a x→a
= = x → ∞ as x → 0⎤⎦ f ( x) f ′ ( x)
x − sin x sin x lim = lim , provided the limit on the
1− x → a g ( x) x → a g ′ ( x)
x
lim −
sin x R.H.S. exists.
= e x→ 0
x = e– 1 Here, f ′ is derivative of f.
1− cos( x +1)
⎛ ∞⎞
⎛ x 4 + x 2 + x + 1⎞ ( x +1) 2 2. ⎜ ⎟ form: If lim f ( x ) = ∞ and lim g ( x ) = ∞,
31. lim ⎜ is equal to ⎝ ∞⎠ x→a x→a
⎟ f ′ ( x)
x →−1 ⎝ x2 − x + 1 ⎠ f ( x)
then lim = lim , provided the limit on
1/ 2 1/ 2 x → a g ( x) x → a g ′ ( x)
⎛ 2⎞ ⎛ 3⎞
(A) 1 (B)  ⎜ ⎟ (C) ⎜ ⎟ (D) e1/2 the R.H.S. exists.
⎝ 3⎠ ⎝ 2⎠
Solution: (B) Note that sometimes we have to repeat the process if the
1− cos( x +1)
0 ∞
form is or again.
⎛ x 4 + x 2 + x + 1⎞ ( x +1) 2 0 ∞
lim ⎜ ⎟
x →−1 ⎝ x2 − x + 1 ⎠
 QUICK TIPS
⎛ x +1⎞
2 sin 2 ⎜
⎝ 2 ⎟⎠
⎛ x4 + x2 + x + 1⎞ ( x +1)
2 f( x )
L’Hospital’s Rule is applicable only when becomes of
= lim ⎜

⎟ g( x )
x →−1 ⎝ x2 − x + 1 ⎠ 0 ∞
 the form or .
2 0 ∞
⎛ ⎛ x +1⎞ ⎞
sin 0 ∞
1 ⎜ ⎜⎝ 2 ⎟⎠ ⎟ ■ If the form is not or , simplify the given expression till
⎜ ⎟ 1/ 2 0 ∞
⎛ x4 + x2 + x + 1⎞ 2 ⎜⎜ ⎛⎜ x +1⎞⎟ ⎟⎟ ⎛ 2⎞
= lim ⎜ = ⎜ ⎟ 0 ∞
⎟ ⎝ ⎝ 2 ⎠ ⎠ ⎝ 3⎠ it reduces to the form or and then use L’Hospital’s
x →−1 ⎝ x2 − x + 1 ⎠  rule. 0 ∞
■ For applying L’Hospital’s rule differentiate the numerator
n and denominator separately.
⎛ x⎞
32. lim ⎜ cos ⎟ is equal to
n→∞ ⎝ n⎠
(A) e1 (B)  e–1 ERROR CHECK
(C)  1 (D)  None of these
L’ Hospital’s rule cannot be applied in every problem.
Solution: (C) 3 x + sin 2 x ⎛ ∞⎞
⎛ x ⎞ Consider the example, lim ⎜ form ⎟⎠ .
⎛ x⎞
n
lim n ⎜ cos −1⎟ x →0 3x − sin 2 x ⎝ ∞
lim ⎜ cos ⎟ = e ⎝ n ⎠ 
n→∞

n→∞ ⎝ n⎠ Here, if we apply L’ Hospital’s rule, we get
lim
⎛ x⎞
− n⋅2 sin 2 ⎜ ⎟ 3 x + sin 2 x = 3 + 2 cos 2 x
⎝ 2n⎠ lim lim
= e
n→∞
 x →∞ 3 x − sin 2 x x →∞ 3 − 2 cos 2 x
Limits  8.13

e x − e − x + 2 sin x ⎛0 ⎞
Now, both the numerator and denominator are undefined = lim  ⎜⎝ form⎟⎠
x →0 24 x 0
because lim cos 2 x Does not exist.
x →∞
We can find the above limit as: e x + e − x + 2 cos x 4 1
= lim = = .
⎛ sin 2 x ⎞ x →0 24 24 6
3 + 2⎜
3 x + sin 2 x = ⎝ 2 x ⎟⎠ = 3 + 2(0)
lim lim 35. If f (2), g (x) be differentiable functions and f (1) =
x →∞ 3 x − sin 2 x x →∞ ⎛ sin 2 x ⎞ 3 − 2(0)
3 − 2⎜ f (1) g ( x ) − f ( x ) g (1) − f (1) + g (1)
⎝ 2 x ⎟⎠ g (1) = 2 then lim
x →1 g( x) − f ( x)
sin 2 x is equal to
= 1, since lim =0
x →∞ 2x (A) 0 (B) 1
(C)  2 (D)  None of these

SOLVED EXAMPLES Solution: (C)


f (1) g ( x ) − f ( x ) g (1) − f (1) + g (1) ⎛0 ⎞
lim  ⎜⎝ form⎟⎠
33. If a is a repeated root of ax2 + bx + c = 0, then x →1 g( x) − f ( x) 0
tan ( ax 2 + bx + c) f (1) g’( x ) − f’( x ) g (1)
lim is =
lim

x →α ( x − α )2 x →1 g’( x ) − f’( x ) 
(A) a (B) 
b (C) 
c (D) 0 g’( x ) − f’( x )
=
2 lim
Solution: (A) x →1 g’( x ) − f’( x )

tan ( ax 2 + bx + c) = 2.
lim 2
x →α (x − α)
36. Let f (x) be a twice differentiable function and f  ″ (0) = 5,
⎛0 2 ⎞ 3 f ( x ) − 4 f (3 x ) + f ( 9 x )
 ⎜⎝ form as aα + bα + c = 0⎟⎠ then lim is equal to
0 x →0 x2
( 2ax + b) sec 2 ( ax 2 + bx + c) (A) 30 (B) 120
= lim (C)  40 (D)  None of these
x →α 2(x − α)
⎛0 ⎞ Solution: (B)
form as α being a repeated root of ax 2 + bx + c = 0,
⎜0 ⎟ 3 f ( x ) − 4 f (3 x ) + f ( 9 x )
⎜ ⎟ ⎛0 ⎞
⎝ 2aα + b = 0 ⎠ lim 2
 ⎜⎝ form⎟⎠
x →0 x 0
2a sec 2 ( ax 2 + bx + c) + ( 2ax + b) 2 ×
3 f’( x ) − 12 f’(3 x ) + 9 f’(9 x ) ⎛0 ⎞
= lim  ⎜⎝ form⎟⎠
2 sec 2 ( ax 2 + bx + c) tan( ax 2 + bx + c) x →0 2x 0
= lim
x →α 2  3 f" ( x ) − 36 f" (3 x ) + 81 f" (9 x )
= lim
2a x →0 2 
= = a.
2 3 f" (0) − 36 f" (0) + 81 f" (0)
x −x = = 24 f  ″(0)
e +e + 2 cos x − 4 2
34. lim is equal to
x →0 x4 = 24 ⋅ 5 = 120.
1 1
(A) 0 (B) 1 (C)  (D) – 3− f ( x)
6 6 37. If f (9) = 9 and f ′ (9) = 1, then lim is equal
Solution: (C) x →9 3− x
to
e x + e − x + 2 cos x − 4 ⎛0 ⎞
lim  ⎜⎝ form⎟⎠ (A) 0 (B) 1
x →0 x4 0 (C)  –1 (D)  None of these
e x − e − x − 2 sin x ⎛0 ⎞ Solution: (B)
= lim 3
 ⎜⎝ form⎟⎠
x →0 4x 0
3− f ( x) ⎛0 ⎞
e x + e − x − 2 cos x ⎛0 ⎞ lim  ⎜⎝ form⎟⎠
= lim  ⎜⎝ form⎟⎠ x →9 3− x 0
x →0 12 x 2 0
8.14  Chapter 8

1 Solution: (B)
0− ⋅ f ′( x )
= lim
2 f ( x)
 ⎛ ln cos x ⎞ ln [1 + (cos x − 1) ]
x →9 1 lim ⎜ ⎟ = lim
0− x →0 ⎝ 4
1 + x 2 − 1⎠ x →0 4 1 + x 2 − 1 
2 x
cos x − 1
 [Using L’Hospital’s Rule] = 4 lim
x →0 x2 

( )
x 3 ⎡
= lim ⋅ f’( x ) = × f ′ (9) = 1. x2 ⎤
⎢∵ ln [1 + (cos x − 1) ] ~ (cos x − 1) and
4
x →9 f ( x) 3 1 + x2 − 1 ~ ⎥
⎣ 4⎦
sin 2 x + a sin x ⎡
38. If lim be finite, then the value of a and x2/ 2 x2 ⎤
x →0 x3 = –4 lim  ⎢∵(1 − cos x ) ~ ⎥
the limit are given by
x →0 x2 ⎣ 2⎦
= –2
(A)  – 2, 1 (B)  – 2, –1
(C)  2, 1 (D)  2, –1 2 − x2
x
41. lim is equal to
Solution: (B) x x − 22
x→2

sin 2 x + a sin x log 2 − 1 log 2 + 1


Let k = lim (A)  (B) 
x →0 x3 log 2 + 1 log 2 − 1

(C) 1 (D) –1
2 cos 2 x + a cos x
= lim Solution: (A)
x →0 3x 2

2x − x2 ⎛0 ⎞
 [Using L’Hospital’s Rule] lim x
x → 2 x − 22
⎜⎝ form⎟⎠
 0
We require 2 cos2x + a cos x = 0 for x = 0 as denomi-
x
nator is zero. 2 log 2 − 2 x 4 log 2 − 4 log 2 − 1
= lim x = = .
\ a = –2. x → 2 x (1 + log x ) 4 (1 + log 2) log 2 + 1
2 cos 2 x − 2 cos x ⎛ 0⎞
Hence, k = lim  ⎜⎝ ⎟⎠ sin x − (sin x )sin x
x →0 3x 2 0 42. lim equals
x→
π 1 − sin x + ln sin x
−4 sin 2 x + 2 sin x ⎛ 0⎞ 2
= lim  ⎜⎝ ⎟⎠ (A) 1 (B) 2 (C) 3 (D) 4
x →0 6x 0
−8 cos 2 x + 2 cos x −8 + 2 Solution: (B)
= lim = = –1. Let sin x = h, then as x → p/2, h → 1
x →0 6 6
39. lim x x is equal to \ given limit
x →0
h − hh 1 − hh − hh ln h
(A) 0 (B) 1 =
lim = lim
h → 1 1 − h + ln h h →1 −1 + 1 / h
(C)  – 1 (D)  None of these 
 (Using L’ Hospital Rule)
Solution: (B)
Let y = lim x x − h − 2h ln h − hh − 1 − hh (ln h) 2
h h
= lim
x →0  h →1 − 1 / h2
log x ⎛∞ ⎞ 
⇒ log y = lim x log x = lim  ⎜⎝ form⎟⎠ −1 − 1
x →0 x →0 1 x ∞ = = 2
−1
1/ x
= lim = – lim x = 0
x → 0 −1/ x 2 x →0
π a−x
0 43. The value of lim a 2 − x 2 cot is
⇒ y = e = 1. x→a 2 a+ x

⎛ ln cos x ⎞ 2a 2a
(A)  (B) 

40. lim ⎜ ⎟ is equal to π π
x →0 ⎝ 4
1 + x 2 − 1⎠ 4a 4a
(C)  (D)  –
(A) 2 (B) –2 (C) 1 (D) –1 π π
Limits  8.15

Solution: (C) −2 x
π a−x 2 a2 − x 2
2 2
lim a − x cot  (0 × ∞ form) = lim 
x→a 2 a+ x x→a π a−x π 2a
− sec 2 × ×
2 a + x 2 2( a + x ) a 2 − x 2
a2 − x 2 ⎛0 ⎞
= lim  ⎜⎝ form⎟⎠ 4a
x→a π a−x 0 =
tan π 
2 a+ x
8.16  Chapter 8

PRACTICE EXERCISES

Single Option Correct Type

⎛ ⎡100 x ⎤ ⎡ 99 sin x ⎤⎞ x5
1. The value of lim ⎜ ⎢ ⎥ + ⎢ x ⎥⎟ , where [⋅] 8. The value of lim
is
x → 0 ⎝ ⎣ sin x ⎦ ⎣ ⎦⎠ 5x x →∞
(A) 1 (B) –1
represents greatest integer function, is (C)  0 (D)  None of these
(A) 199 (B) 198 1
(C)  0 (D)  None of these 9. lim (cos x + sin x ) x is equal to
x →0
2. If f (x) = sin x, x ≠ np,
= 2, x = np (A) e (B) e2 (C) 
e–1 (D) 1
where n ∈Z and 2 2 − (cos x + sin x ) 3

g(x) = x2 + 1, x ≠ 2, 10. The value of lim is


x→
π 1 − sin 2 x
= 3, x = 2. 4
then lim g [ f ( x ) ] is 3 2 1
x →0 (A)  (B)  (C)  (D) 2
(A) 1 (B) 0 2 3 2
(C)  3 (D)  Does not exist
ln(1 + 2h) − 2 ln(1 + h)
11. The value of lim is
⎛ ⎞ h→ 0 h2
3. The value of lim ⎜ x + x + x − x ⎟ is (A) 1 (B) – 1
x →∞ ⎝ ⎠
1 (C)  0 (D)  None of these
(A)  1
2 ⎛ 1 e1/ n e 2 / n e( n −1)/ n ⎞
(C)  0 (D)  None of these 12. The value of lim ⎜ + + + ... + is
n→∞ ⎝ n n n n ⎟⎠
⎡ x +1 π ⎤ (A) 1 (B) 0 (C) e– 1 (D)  e + 1
4. The value of lim x ⎢ tan −1 − is
x →∞ ⎣ x + 2 4 ⎥⎦
x sin ( x − [ x ])
1 1 13. lim , where [⋅] denotes the greatest
(A)  (B) – (C) 1 (D) – 1 x →1 x −1
2 2 ­integer function, is equal to
PRACTICE EXERCISES

( )
(A) 1 (B) –1
5. lim cos π n2 + n , n ∈ Z is equal to (C) ∞ (D)  Does not exist
n→∞

(A) 0 (B) 1 2 sin x − sin 2 x


(C)  – 1 (D)  None of these 14. If f (x) = ∫ x3
dx , x ≠ 0, then lim f’( x ) is
x →0

nk sin 2 ( n !) (A) 0 (B) ∞ (C) –1 (D) 1


6. lim 0 < k < 1, is equal to
n →∞ n+2 ⎡x⎤
(A) ∞ (B)  1 ⎢⎣ 2 ⎥⎦
(C)  0 (D)  None of these 15. lim (where [⋅] denotes the greatest integer
x → π / 2 ln(sin x )

1 − cos 2( x − 1) function)
7. lim (A)  Does not exist (B)  equals 1
x →1 x −1 (C)  equals 0 (D)  equals –1
(A)  exists and it equals 2
16. lim lim (1 + cos 2 m n !π x ) is equal to
(B)  exists and it equals – 2 m→∞ n→∞
(C)  Does not exist because (x – 1) → 0 (A) 2 (B) 1
(D) Does not exist because left hand limit is not equal (C)  0 (D)  None of these
to right hand limit
Limits  8.17

⎡ sin([ x − 3]) ⎤ (A) 1 (B) –1 (C) 0 (D) ∞


17. lim ⎢ ⎥ , where [⋅] represents greatest
x → 0 ⎣ [ x − 3] ⎦
2 x + 23 − x − 6
­
integer function, is 26. lim is equal to
x→2 2 − x / 2 − 21− x
(A) 0 (B) 1
(C)  Does not exist (D)  sin 1 (A) 8 (B) 4
(C)  2 (D)  None of these
18. The values of constants a and b so that
⎛ x2 + 1 ⎞ 8 ⎛ x2 x4 x2 x2 ⎞
lim ⎜ − ax − b⎟ = 0 are 27. lim8 ⎜ 1 − cos − cos + cos cos ⎟ is equal
x →0 x ⎝ 2 4 2 4⎠
x →∞ ⎝ x + 1 ⎠
to
(A) a= 1, b = –1 (B)  a = –1, b = 1 1 1 1 1
(C) a= 0, b = 0 (D)  a = 2, b = –1 (A)  (B) – (C)  (D) –
16 16 32 32
⎛ 1 1 1 1 ⎞ 28. lim [log n −1 ( n) ⋅ log n ( n + 1) ⋅ log n +1 ( n + 2)… log n k
19. lim ⎜ + + + ... + is equal to −1 ( n )]
n( n + 1) ⎟⎠
k

n→∞ ⎝ 1 ⋅ 2 2 ⋅ 3 3⋅ 4 n→∞
is equal to
(A) 1 (B) –1 (A) ∞ (B)  n
(C)  0 (D)  None of these (C) k (D)  None of these
(log x ) 2 ⎡ 1 1 1 1 ⎤
20. lim , n > 0 is equal to 29. lim ⎢ + + + ... + is equal
x →∞ xn n→∞ ⎣1 ⋅ 3 3 ⋅ 5 5 ⋅ 7 ( 2n + 1) ( 2n + 3) ⎥⎦
(A) 1 (B) 0 (C) – 1 (D) ∞ to
1
21. If the rth term, tr, of a series is given by (A)  1 (B) 
n 2
r 1
tr = 4
r + r2 + 1
, then lim
n→∞
∑ tr is (C) –
2
(D)  None of these
r =1 nx
1 ⎡11/ x + 21/ x + 31/ x + ... + n1/ x ⎤
(A)  1 (B)  30. The value of lim ⎢ ⎥ is
2 x →∞
⎣ n ⎦
1
(C)  (D)  None of these (A) n! (B)  n (C) (n – 1)!   (D)  0
3
22. lim( −1)[ x ] , where [x] denotes the greatest integer less 31. lim (1 + x) (1 + x2) (1 + x4) … (1 + x2n), | x | < 1, is
n→∞
x→n
than or equal to x, is equal to equal to
1 1
(A)  (B)  (C)  1 – x (D) 
x–1

PRACTICE EXERCISES
(A) (–1)n (B)  (–1)n – 1 x −1 x −1
(C)  0 (D)  Does not exist
xn
y3 32. lim = 0, (n integer), for
· 23. lim 3 2
as (x, y) → (1, 0) along the line y = x →∞ex
x →1 x − y −1
y →0 (A)  no value of n
x – 1 is given by (B)  all values of n
(A)  1 (B)  ∞ (C)  only negative values of n
(C)  0 (D)  None of these (D)  only positive values of n
1 − 2 + 3 − 4 + 5 − 6 + ... − 2n x n + x n −1 + x n − 2 + ... + x 2 + x − n
24. lim is equal to 33. The value of lim is
n →∞
n2 + 1 + 4 n2 − 1 x →1 x −1
n ( n +1)
1 1 (A)  (B) 0
(A)  (B)  – 2
3 3
(C) 
1 (D)  n
1
(C) – (D)  None of these 12 + 22 + 32 + ... + r 2 n
5 34. If tr = 3 3
1 + 2 + 3 + ... + r 3 3
and Sn= ∑ ( −1)r ⋅ tr, then
⎡ x 4 sin (1/ x ) + x 2 ⎤ r =1
25. The value of lim ⎢ ⎥ is lim Sn is given by
x→−∞
⎣ 1 + | x |3 ⎦ n→∞
8.18  Chapter 8

2 1 (A) 0 (B) 1
2 1
(A)  3 (B) – (C)  (D) – (C)  – 1 (D)  None of these
3 3 3
(1 + a3 ) + 8e1 x
n Sn + 1 − Sn
35. If lim = 2, then 44. If Sn= ∑ ai and lim an = a, then lim
n→∞ n→∞ n
is
x → 0 1 + (1 − b3 ) e1 x i =1
∑i
(A) a = 1, b = (–3)1/3 (B)  a = 1, b = 31/3 equal to i =1
(C) a = –1, b = –(3)1/3 (D)  None of these (A) 0 (B)  a
1 − cos 2θ (C)  2a (D)  None of these
36. If a = min {x2 + 4x + 5, x∈R} and b = lim ,
n θ →0 θ2
then the value of ∑a r
⋅b n− r
is 45. The value of lim ⎡ n2 − n3 + n⎤ is

3
⎥⎦
r=0 n→∞ ⎣
2n +1 − 1 1 1 2 2
(A)  n
(B) 2n + 1 – 1 (A)  (B) – (C)  (D) –
4⋅2 3 3 3 3
2n +1 − 1
(C)  (D)  None of these 4 3
3 ⋅ 2n n5 + 2 − n2 + 1
46. The value of lim is
n→∞ 5
1⋅ 2 + 2 ⋅ 3 + 3 ⋅ 4 + ... + n ( n + 1) n 4 + 2 − 2 n3 + 1
37. lim is equal to
n→∞n3 (A) 1 (B) 0 (C) –1 (D) ∞
(A) 1 (B) –1 (cos x − 1)(cos x − e x )
1 47. The integer n for which lim is
(C)  (D)  None of these x→0 xn
3 a finite non-zero number, is
2 2 (A) 1 (B) 2 (C) 3 (D) 4
log (1 + x + x ) + log (1 − x + x )
38. lim is equal to
x →0 sec x − cos x
2 x +3 3 x +5 5 x
(A) 1 (B) –1 (C) 0 (D) ∞ 48. The value of lim is
x →∞ 3x − 2 + 3 2 x − 3
ln x − 1 2
39. lim is equal to (A)  (B) 
3
| x−e|
x →e 3
1 1 1
(A)  (B)  – (C)  (D)  None of these
e e 3
(C) e (D)  Does not exist
PRACTICE EXERCISES

40. If x1 = 3 and xn + 1= 2 + x n , n≥ 1, then lim xn is x 3


z 2 − ( z − x)2
49. lim is equal to
( )
n→∞ x→0 3 4
equal to 8 xz − 4 x 2 + 3 8 xz
(A) –1 (B) 2 (C)  5 (D) 3
z 1
3x +1 − 5 x +1 (A)  (B) 
41. The value of lim is 211/ 3 223/ 3 ⋅ z
x →∞ 3x − 5 x
1 (C) 221/3 z (D)  None of these
(A)  5 (B) 
5 50. In a circle of radius r, an isosceles triangle ABC is
(C)  –5 (D)  None of these
inscribed with AB = AC. If the D ABC has perimeter P =
1⎛
n −1 ⎞
42. lim ⎜1 + e1/ n + e 2 / n + ... + e n
⎟ is equal to 2 ⎡ 2 hr − h2 + 2 hr ⎤ and area A = h 2 hr − h2 ,
n→∞ n ⎣⎢ ⎦⎥
⎝ ⎠ A
where h is the altitude from A to BC, then lim 3 is
(A) e (B)  –e h→ 0 P +

equal to
(C) e– 1 (D)  1 – e 1
(A) 128 r (B) 
x + sin x 128 r
43. lim = 1
x →∞ x − cos x (C)  (D)  None of these
64 r
Limits  8.19

⎛ 1 − cos{2( x − 2)} ⎞ ⎛ ⎡100 x ⎤ ⎡ 99 sin x ⎤⎞


51. lim ⎜ ⎟ 58. The value of xlim
→0 ⎜
⎢ ⎥+
⎝ ⎣ sin x ⎦ ⎢⎣ x ⎥⎦⎟⎠
, where [⋅]
x→2⎝ x−2 ⎠
1 represents greatest integer function, is
(A) equals (B)  Does not exist
2 (A) 199 (B) 198
1 (C)  0 (D)  None of these
(C) equals (D) equals − 2 5
2 x
59. The value of lim x is
x →∞ 5
⎛ x x x x⎞ (A) 1 (B) –1
52. lim ⎜ cos cos cos ... cos n ⎟ =
n→∞ ⎝ 2 4 8 2 ⎠ (C)  0 (D)  None of these
x sin x 60. If the rth term, tr, of a series is given by
(A)  (B)  r n
sin x x tr= 4
r + r2 + 1
, then lim
n→∞
∑ t r is
(C)  0 (D)  None of these r =1
1
53. The value of (A)  1 (B) 
2
1
1 ⎡ ⎛ n ⎞ ⎛ n −1 ⎞ ⎛n−2 ⎞ ⎤ (C)  (D)  None of these
lim 4
⎢1 ⎜∑ ⎟ k + 2 ⎜∑ ⎟ k + 3 ⎜ ∑ k ⎟ + ... + n ⋅ 1⎥ 3
n→∞ n ⎢⎣ ⎝ k = 1 ⎠ ⎝ k =1 ⎠ ⎝ k =1 ⎠ ⎥⎦ nx
⎡11/x + 21/x + 31/x + ... + n1/x ⎤
will be 61. The value of lim ⎢ ⎥ is
x →∞
⎣ n ⎦
1 1 1 1
(A)  (B)  (C)  (D) 
24 12 6 3 (A) n! (B)  n (C) (n – 1)!   (D)  0

54. If [x] denotes the integral part of x, then 62. lim (1 + x )(1 + x 2 )(1 + x 4 )… (1 + x 2 n ), | x | < 1, is equal to
n→∞

1 ⎛ n ⎞ 1 1
lim 3 ⎜ ∑ [k 2 x ]⎟ = (A)  (B) 
n→∞ n ⎝ ⎠ x −1 1− x
k =1
(C)  1 – x (D) x–1
x x x
(A) 0 (B)  (C)  (D)  xn
2 3 6 63. lim = 0, (n integer), for
ex
x →∞

⎛ 1 θ 1 θ 1 θ⎞ (A)  no value of n
55. lim ⎜ tan θ + tan + 2 tan 2 + … + n tan n ⎟ = (B)  all values of n
n→∞ ⎝ 2 2 2 2 2 2 ⎠
(C)  only negative values of n

PRACTICE EXERCISES
1 1
(A)  (B)  − 2 cot 2θ (D)  only positive values of n
θ θ 1 − cos 2θ
64. If a = min {x2 + 4x + 5, x ∈R} and b = lim
(C)  2 cot 2q (D)  None of these n θ →0 θ2
43n − 2 − 9 n + 1
then the value of ∑ a ⋅b r n−r
is
n +1 r=0
56. lim = 2 −1
82 n − 1 − 9 n − 1
n→ 0 (A)  n
(B) 2n + 1 – 1
4⋅2
1 n +1
(A)  (B)  81 2 −1
2 (C)  (D)  None of these
(C)  Does not exist (D)  None of these 3 ⋅ 2n

x − ai log (1 + x + x 2 ) + log (1 − x + x 2 )
57. If Ai = , i = 1, 2, …, n and if a1 < a2 < a3 … < an. 65. lim is equal to
| x − ai | x →0 sec x − cos x
Then, lim ( A1 A2 ... An ), 1 ≤ m ≤ n (A) 1 (B) –1 (C) 0 (D) ∞
x → am

(A)  is equal to (–1)m 4


n5 + 2 − 3 n2 + 1

(B)  is equal to (–1)m +1 66. The value of lim is
n→∞ 5
(C)  is equal to (–1)m – 1 n4 + 2 − 2 n3 + 1

(D)  Does not exist (A) 1 (B) 0 (C) –1 (D) ∞
8.20  Chapter 8

x 3
z 2 − ( z − x)2 ( 2m + x )1/m − ( 2n + x )1/n
lim 74. lim is equal to

( 8xz − 4 x )
67. x →0 4 is equal to x→ 0 x
2
3
+ 3 8 xz
1 1 1 1
(A)  m
− n
(B)  m + n
z 1 m2 n2 m2 n2
(A)  11/3
(B)  1 1
2 223/3 ⋅ z (C)  − (D)  None of these
m −1
21/3 m2 n2n −1
(C) 2 z (D)  None of these
68. In a circle of radius r, an isosceles triangle ABC is (cos θ ) x − (sin θ ) x − cos 2θ
75. lim =
inscribed with AB= AC. If the DABC has perimeter P = x→ 4 x−4
(A) cos4 q lncos q – sin4 q ln sin q
2 ⎡ 2hr − h2 + 2hr ⎤ and area A = h 2hr − h2 , (B) cos4 q lncos q + sin4 q ln sin q
⎣⎢ ⎦⎥
A (C) cos4 q ln sin q – sin4 q lncos q
whereh is the altitude from A to BC, then lim 3 is
equal to h→ 0 P +
(D)  None of these
1 1/ x
(A) 128r (B)  ⎛ x − 1 + cos x ⎞
128r 76. lim ⎜ ⎟⎠ =
1 x→ 0 ⎝ x
(C)  (D)  None of these
64r (A) e1/2 (B)  e–1/2
1/4
⎛ π⎞ (C) e (D)  None of these
cos ⎜ x + ⎟
⎝ 6⎠ ⎡ ⎤
x
69. lim = e
x →π /3 (1 −
2 cos x ) 2/3 77. lim ⎢ ⎥ =
(A) 1 (B) –1 ⎣( ) ⎦
x →∞ ⎢ 1 + 1/ x x ⎥

(C)  0 (D)  None of these e–1 (C) 


(A) e (B)  e1/2 (D) 
e–1/2
ln ( 2 − cos 2 x ) ⎡ a sin x ⎤ ⎡ b tan x ⎤
70. lim is equal to 78. lim ⎢ + , where a, b are integers and
ln 2 (sin 3 x + 1)
x →0
x→ 0 ⎣ x ⎥⎦ ⎢⎣ x ⎥⎦
2 2 [ ] denotes integral part, is equal to
(A)  (B)  –
9 9 (A) a+ b (B)  a+b–1
(C)  0 (D)  None of these (C) a– b (D)  a – b – 1
1 − cos (cx 2 + bx + a) [ x ] + [2 x ] + [3 x ] + ... + [nx ]
71. lim , where a is a root of 79. lim =
x → 1/α (1 − xα ) 2 n→∞ 1 + 2 + 3 + ... + n
2
ax  + bx+ c = 0, is equal to
PRACTICE EXERCISES

(A) x (B)  2x
b 2 − 4 ac b 2 − 4 ac (C)  0 (D)  None of these
(A)  (B) 2
2α 2
2
α ( )
80. lim n2 x1/n − x1/n + 1 , x > 0 is equal to
n→∞
4 ac − b
(C)  (D)  None of these (A) 0 (B)  ex
2α 2 (C) lnx (D)  None of these
1 − cos x 1/ x 1/ x
72. lim = ⎡ f ( x) ⎤
81. If lim ⎢⎡1 + x +
f ( x) ⎤
x→ 0 x = e3, then lim ⎢1 + =
x→ 0 ⎣ x ⎦ ⎥ x → 0 ⎣ x ⎥⎦
1 1
(A)  (B)  – (A) e (B)  e2
2 2 3
(C)  Does not exist (D)  None of these (C) e (D)  None of these

x + 7 − 3 2x − 3 x x
73. lim = 82. If y = x + , then lim is equal to
x→ 2 3 x + 6 − 2 3x − 5 3 x x →∞ y
x+
17 34 x
(A)  (B)  x+
23 23 ... ∞
(A) 1 (B) –1
(C)  1 (D)  None of these (C)  0 (D)  None of these
Limits  8.21

(A) e (B)  e2
cos x − (cos x )cos x 4
83. lim = (C) e (D)  None of these
x → 0 1 − cos x + ln (cos x ) y x
x −y 1− k
(A) 0 (B) 1 93. If lim x = , then k =
(C)  2 (D)  None of these x→ y x − y y 1+ k
(A) log y (B)  ey
(tan x ) tan x − tan x
84. The value of lim is (C) y (D)  None of these
x → π /4 ln (tan x ) − tan x + 1
⎛ 3 1⎞
(A) –2 (B) 1 n r −r +
⎜ r ⎟ is equal to
(C)  0 (D)  None of these 94. lim ∑ cot −1 ⎜ ⎟
n→∞ 2
r =1 ⎜ ⎟
( )
cos 2 x
85. lim 11/cos
2
x
+ 21/cos
2
x
+ ... + n1/cos
2
x
= ⎝ ⎠
x → π /2 (A) 0 (B)  p
n ( n + 1) π
(A) n (B)  (C)  (D)  None of these
2 2
(C) n! (D)  None of these
⎛ ⎡ n sin x ⎤ ⎡ n tan x ⎤⎞
95. The value of lim ⎜ ⎢ + , where [·]
x ⎥⎦ ⎢⎣ x ⎥⎦⎟⎠
n
⎛ 3⎞ n→∞ ⎝ ⎣
86. lim
n→∞
∑ cot −1 ⎜⎝ r 2 + 4 ⎟⎠ =
r =1 denotes the greatest integer function, is
(A) 0 (B) tan–12 (A) n (B)  2n + 1
π (C) 2n – 1 (D)  None of these
(C)  (D)  None of these
4
⎡ x2 ⎤
87. The value of lim [sin x + cos x], where [·] denotes 96. lim ⎢ ⎥ , where [·] denotes the greatest
x→ 5π / 4 x → 0 sin x tan x
⎣ ⎦
the greatest integer function, is ­integer function, is
(A) 2 (B) –2 (C) 1 (D) –1 (A) 0 (B) 1
88. lim lim (C)  2 (D)  Does not exist
m→∞ n→∞

⎛ 1 + n 1n + 2n + n 2n + 3n + n 3n + 4 n + … + n ( m − 1) n + m n ⎞ cos 2 (1 − cos 2 (1 − cos 2 (1 ... cos 2 θ ))


97. lim =
⎜ ⎟ θ→ 0 ⎛ π ( θ + 4 − 2) ⎞
⎜⎝ m2 ⎟⎠ sin ⎜ ⎟
⎝ θ ⎠
1
(A) 0 (B) 1 (C) –1 (D) 
2 (A) 1 (B) 0 (C)  2 (D) – 2
⎡ n 1⎤
89. The value of lim ⎢ ∑ r ⎥ , where [·] denotes the

PRACTICE EXERCISES
n→∞ ⎢ tan x − sin{tan −1 (tan x )}
⎣ r = 1 2 ⎥⎦ 98. Let f (x) = , then lim f ( x ) =
greatest integer, is tan x + cos 2 (tan x ) x→
π
1 2
(A) 0 (B) 1 (C) –1 (D)  (A) 1
2
[cos x ] (B) –1
90. The value of lim | x | , where [·] denotes the
x →∞ (C) 0
greatest integer, is (D)  Does not exist
(A) 0 (B) 1 ⎧ ⎡ 1/2 1/2 −1 −1

(C)  –1 (D)  Does not exist ⎪ ⎢⎛ a + x ⎞ 2( ax )1/4
  99. lim ⎨ ⎜ 1/2 1/4 ⎟ − 3/4 1/4 1/2 1/2 1/4 3/4 ⎥
_ n ≥ 2, then x → a ⎢⎝ a − x ⎠ x −a x +a x −a ⎥
91. If a1 = 1 and an= n (1 + an – 1), ∨ ⎪⎣ ⎦

⎛ 1⎞ ⎛ 1⎞ ⎛ 1⎞ ⎫⎪
lim ⎜1 + ⎟ ⎜1 + ⎟ ... ⎜1 + ⎟ = 4

−2log ⎬ =
a
n→∞ ⎝ a1 ⎠ ⎝ a2 ⎠ ⎝ an ⎠
⎪⎭
(A) 0 (B)  e
(A)  a3/4 (B)  a
(C) e2 (D)  Does not exist
(C)  a2 (D)  None of these
n
⎡ ⎛ 1⎞ ⎛ 1⎞ ⎛ 1 ⎞⎤ (log (1 + x ) − log 2) (3.4 x −1 − 3 x )
92. lim n − n ⎢( n + 1) ⎜ n +
2

⎝ ⎟⎠ ⎜⎝ n + 2 ⎟⎠ ... ⎜⎝ n + n −1 ⎟⎠ ⎥ 100. lim =


n→∞ ⎣ 2 2 2 ⎦ x →1 {(7 + x )1/3 − (1 + 3 x )1/2 } sin π x
8.22  Chapter 8

9 4 3 4 c ⎛ 1 1⎞ c ⎛ 1 1⎞
(A)  log (B)  log (A)  − (B)  2β ⎜⎝ α − β ⎟⎠
π e π e 2α ⎜⎝ α β ⎟⎠
9 2
(C)  log (D)  None of these
π e c ⎛ 1 1⎞
(C)  − (D)  None of these
(1 − x ) (1 − x 2 ) ... (1 − x 2 n ) αβ ⎜⎝ α β ⎟⎠
101. lim =
x →1 [(1 − x ) (1 − x 2 ) ... (1 − x n )]2 104. Given a real valued function f  such that
( 2n)!
(A) n! (B)  ⎧ tan 2 {x}
n! ⎪ 2 , x>0
( 2n)! ⎪ x − [ x]
2
(C)  2 (D)  None of these ⎨
( n !) f (x) = ⎪ 1 , x=0
k
kπ ⎪ {x} cot {x} , x < 0

102. If ∑ cos −1 α r = 2
for any k ≥ 1 2
r =1 ⎛ ⎞
The value of cot ⎝ xlim
→0
f ( x )⎟ is
–1 ⎜

k
(1 + x 2 )1/3 − (1 − 2 x )1/4
and q  = ∑ (α r )r , then xlim
→θ x + x2
is
(A) 0 (B) 1
r =1

(C)  –1 (D)  None of these
1 1
(A) 1 (B) –1 (C)  (D) –
2 2 x a sin b x
105. If lim , a, b, c∈ R – {0} exists and has
sin ( x c )
x →0
103. If a and b are the roots of the quadratic equation
­non-zero value, then
1 − cos( cx 2 + bx + a) (A)  a, b, c are in A.P.
ax2 + bx+ c = 0, then lim =
1 2(1 − α x ) 2 (B)  a, c, b are in A.P.
x→
α (C)  a, c, b are in G.P.
(D)  None of these

Previous Year’s Questions


⎡ ⎛ x⎞ ⎤
1 − cos 2 x ⎢1 − tan ⎜⎝ 2 ⎟⎠ ⎥ [1 − sin x ]
106. lim is [2002] 1 10. lim ⎣ ⎦ is [2003]
x →∞ 2x x →π / 2 ⎡ ⎛ x⎞ ⎤ 3
⎢1 + tan ⎜⎝ 2 ⎟⎠ ⎥ [π − 2xx ]
PRACTICE EXERCISES

(A) l (B) − 1 ⎣ ⎦
(C)  Zero (D)  Does not exist 1
(A)  (B)  0
x 8
⎛ x 2 + 5x + 3⎞
107. lim ⎜ 2 ⎟ is equal to [2002] 1
x →∞ ⎝ x + x + 2 ⎠ (C)  (D)  ∞
32
(A) e4 e2
(B)  log(3 + x ) − log(3 − x )
111. If lim = k , the value of k is
(C) e3 (D) 
e x
 x →0 [2003]
2 1
⎛ x − 3⎞ (A)  0 (B)  −
108. For x ∈ R, lim ⎜ ⎟ is equal to [2002] 3
x →∞ ⎝ x + 2 ⎠
2 2
(A) e (B)  e−1 (C)  (D)  −
(B) e− 5
(D)  e5 3 3
112. Let f (a) = g(a) = k and their nth derivatives f  n(a),
xf ( 2) − 2 f ( x )
109. Let f (2) = 4 and f  ′ (2) = 4. Then lim is gn(a) exist and are not equal for some n. Further if
x→2 x−2
f ( a) g ( x ) − f ( a) − g ( a) f ( x ) + g ( a)
given by [2002] lim = 4 , then the
x→a g( x) − f ( x)
(A) 2 (B) − 2
value of k is [2003]
(C) − 4 (D) 3
Limits  8.23

(A) 4 (B) 2 118. Let f (x) be a forth degree polynomial having extreme
(C) 1 (D) 0
⎡ f ( x) ⎤
values at x = 1 and x = 2. If lim ⎢1 + 2 ⎥ = 3 , then
x →0 ⎣ x ⎦
2x
⎛ a b⎞ f (2) is equal to [2015]
113. If lim ⎜1 + + 2 ⎟ = e 2 , then the values of a and b,
x →∞ ⎝ x x ⎠ (A) -4 (B)  0
are  [2004] (C)  4 (D)  -8
(A) a ∈ R, b ∈ R (B)  a = 1, b ∈ R
119. The value of lim
(1 − cos 2 x ) (3 + cos x ) is equal to
(C) a ∈ R, b = R a = 1 and b = 2
(D)  x →0 x tan 4 x
 [2015]
114. Let α and β be the distinct roots of ax2 + bx + c = 0, (A) 3 (B) 2
1 − cos( ax 2 + bx + c) 1
then lim is equal to [2005] (C)  (D)  4
2
x →α ( x − α )2 1
2 120. Let p = lim (1 + tan 2 x ) 2 x then log p is equal to
a x →0 +
(A)  (α − β ) 2 (B) 0
2  [2016]
1
(A)  (B)  2
a2 1 4
(C) − (α − β ) 2 (D) (α − β ) 2 1
2 2 (C)  1 (D) 
2
115. Let f : R → R be a positive increasing function such cot x − cos x
121. lim equals [2017]
f (3 x ) f (2 x) π (π − 2π )3
x→
that lim = 1 . Then, lim = [2010] 2
x →∞ f ( x ) x →∞ f ( x )
1 1 1 1
(A)  (B)  (C)  (D) 
2 3 16 6 4 24
(A)  (B) 
3 2
122. For each t ∈ R, let [t] be the greatest integer less than
(C) 3 (D) 1
or equal to t. Then [2018]
⎛ 1 − cos{2( x − 2)} ⎞
1 16. Limit of ⎜ ⎟ as x tends to 2 [2011] ⎛⎡1⎤ ⎡2⎤ ⎡15 ⎤ ⎞
⎝ x−2 ⎠ lim x ⎜ ⎢ ⎥ + ⎢ ⎥ +…+ ⎢ ⎥ ⎟
x →0 + ⎝⎣ ⎦ ⎣ ⎦
x x ⎣ x ⎦⎠
(A) equals 2 (B)  equals – 2 (A)  is equal to 0.
(B)  is equal to 15.

PRACTICE EXERCISES
1
(C) equals (D)  does not exist (C)  is equal to 120.
2
(D)  does not exist (in R).

117. The value of lim


(1 − cos 2 x ) (3 + cos x ) is equal to
x →0 x tan 4 x
 [2013]
1
(A)  (B)  1
2 1
(C) 2 (D)  −
4
8.24  Chapter 8

ANSWER K EYS
Single Option Correct Type
  1. (B) 2. (A) 3.  (A) 4. (B) 5.  (A) 6. (C) 7.  (D) 8. (C) 9.  (A) 10. (A)
  11.  (B) 12. (C) 13.  (D) 14. (D) 15. (C) 16.  (a, b) 17.  (C) 18. (A) 19.  (A) 20. (B)
  21.  (B) 22. (D) 23. (C) 24. (B) 25.  (B) 26. (A) 27.  (C) 28. (C) 29.  (B) 30. (A)
  31.  (B) 32. (B) 33.  (A) 34. (B) 35.  (A) 36. (B) 37.  (C) 38. (A) 39. (D) 40. (B)
  41.  (A) 42. (C) 43.  (B) 44. (A) 45.  (A) 46. (B) 47. (C) 48. (A) 49.  (B) 50. (B)
  51. (B) 52.  (B) 53. (A) 54. (C) 55.  (B) 56. (C) 57.  (D) 58.  (B) 59. (C) 60.  (B)
  61. (A) 62.  (B) 63. (B) 64. (B) 65.  (A) 66. (B) 67.  (B) 68. (B) 69. (C) 70. (A)
  71.  (A) 72. (C) 73. (B) 74.  (C) 75. (A) 76.  (B) 77. (C) 78.  (B) 79. (A) 80.  (C)
  81. (B) 82.  (A) 83. (C) 84. (A) 85.  (A) 86. (B) 87. (B) 88. (D) 89. (A) 90. (B)
  91.  (B) 92. (B) 93.  (A) 94. (C) 95. (C) 96. (A) 97. (C) 98. (A) 99.  (C) 100. (A)
101.  (C) 102. (C) 103. (A) 104. (D) 105. (D)

Previous Years’ Questions


106. (D) 107. (A) 108. (C) 109. (C) 110. (C) 111. (C) 112. (A) 113. (B) 114. (A) 115. (D)
116. (D) 117. (C) 118. (A) 119.  (B) 120.  (D) 121.  (A) 122. (C)
PRACTICE EXERCISES
Limits  8.25

HINTS AND EXPLANATIONS

Single Option Correct Type

sin x x
1. We know that lim → 1– and lim → 1+
The correct option is (B)
x→0 x x → 0 sin x

⎡ x ⎤ ⎡ 1 ⎤
⎡ sin x ⎤ ⎛ 1⎞ 2

∴ lim ⎢100 ⎥ + lim 99 = 100 + 98 = 198. 5. lim cos ⎡π n2 + n ⎤ = lim cos ⎢ nπ ⎜1 + ⎟ ⎥
x→0⎣ sin x ⎦ x → 0 ⎢⎣ x ⎥⎦ n→∞ ⎣⎢ ⎦⎥ n→∞ ⎢ ⎝ n⎠ ⎥
⎣ ⎦
The correct option is (B)
⎡ ⎛ 1 1 ⎞⎤
2. g [ f (x)] = [ f (x)]2 + 1, f (x) ≠ 2
= lim cos ⎢ nπ ⎜1 + − 2 + ...⎟ ⎥
3, f (x) = 2
n→∞ ⎣ ⎝ 2 n 8n ⎠⎦

2
∴ g [ f (x)] = sin x + 1, x ≠ nπ ⎛ π π ⎞
= lim cos ⎜ nπ + −
+ ...⎟
3, x = nπ n→∞ ⎝ 2 8n ⎠
RHL = lim g [ f (0 + h)] = lim(sin 2 h + 1) = 1. ⎛ π ⎞
h→ 0 h→ 0 = – lim sin ⎜ nπ −
+ ...⎟
n→∞ ⎝ 8n ⎠
LHL = lim g [ f (0 − h)] = lim (sin 2 h + 1) = 1.

h→ 0 h→ 0 π
= – lim ( −1) n −1 sin ⎛⎜ − ...⎞⎟


lim g [ f ( x )] = 1 n→∞ ⎝ 8n ⎠
x→0
⎛ π ⎞

The correct option is (A) = 0
⎜⎝∵ − ... → 0 as n → ∞⎟⎠
8n

The correct option is (A)
⎡ ⎤
3. lim ⎢ x + x + x − x ⎥ nk sin 2 ( n!) nk sin 2 ( n!)
x→∞ ⎣ ⎦ 6. lim = lim
n→∞ n+2 n→∞ ⎛ 2⎞
n ⎜1 + ⎟
x+ x+ x −x ⎝ n⎠
= lim

x→∞
x+ x+ x + x sin 2 ( n!)

= lim
n → ∞ 1− k ⎛ 2⎞

HINTS AND EXPLANATIONS


x+ x n ⎜1 + ⎟
= lim
⎝ n⎠
x→∞
x+ x+ x + x a finite quantity

=
1/ 2 ∞
⎛ 1 ⎞
x ⎜1 + ⎟ [∵ sin2 (n !) always lies between 0 and 1. Also,

⎝ x⎠
= lim

 since 1 – k > 0, \  n1 – k→ ∞ as n → ∞]
x→∞ ⎡⎛ 1 1 ⎞
1/ 2 ⎤
x ⎢⎜1 + 1+ ⎟ + 1⎥
= 0.
⎢⎝ x x⎠ ⎥
⎣ ⎦
The correct option is (C)
1 1
=
=
1+1 2 1 − cos 2( x − 1) 2 sin 2 ( x − 1)
7. lim = lim

The correct option is (A) x →1 x −1 x →1 x −1
2 sin( x − 1)
⎡ x +1 π ⎤
= lim
4. lim x ⎢ tan −1 − x →1 ( x − 1)
x →∞ ⎣ x + 2 4 ⎥⎦
⎛ x +1 ⎞ 2 sin( x − 1) 2 sin(1 − h − 1)
−1 LHL = lim
= lim −
⎡ −1 x + 1 −1 ⎤ −1 ⎜ x + 2 ⎟
= lim x ⎢ tan
− tan 1⎥ = lim x tan ⎜ ⎟ x →1 −
( x − 1) h → 0 (1 − h − 1)
x →∞ ⎣ x+2 ⎦ x →∞ x + 1
⎜1+ ⎟
⎝ x + 2⎠ 2 − sin h sin h

= lim = – 2 lim
⎛ 1 ⎞ h→ 0 −h h→ 0 h
tan −1 ⎜
−1 ⎞−1 ⎛ ⎝ 2 x + 3 ⎟⎠ 1
=– 2⋅1=– 2
= lim x tan ⎜
= – lim ⋅
x →∞ ⎝ 2 x + 3 ⎟⎠ x →∞ ⎛ 1 ⎞ ⎛ 3⎞
⎜⎝ ⎟ ⎜⎝ 2 + ⎟⎠ 2 sin( x − 1) 2 sin(1 + h − 1)
2 x + 3⎠ x
1 1 RHL = lim
= lim
= –1 ×
=– x →1 +
( x − 1) h→ 0 (1 + h − 1)
2 2
8.26  Chapter 8

2 sin h sin h ⎛ (1 + h) 2 ⎞

= lim = 2 hlim = 2⋅1= 2 ln ⎜ ⎟
h→0 h →0 h
= – lim
⎝ 1 + 2h ⎠
Since LHL ≠ RHL,
h→ 0 h2
⎛ h2 ⎞
1 − cos 2( x − 1) ln ⎜1 + ⎟
∴ lim
does not exist. ⎝ 1 + 2h ⎠
x →1 x −1 = – lim

h→ 0 ⎛ h2 ⎞

The correct option is (D) ⎜ 1 + 2h ⎟ (1 + 2h)
⎝ ⎠
x5 x5 x5
8. lim = lim = lim , ⎛
x →∞ 5 x x →∞ e x log 5 x →∞ e kx h2 ⎞
ln ⎜1 + ⎟
where k = log 5
⎝ 1 + 2h ⎠ 1
= – lim
⋅ = –1
h→ 0 ⎛ h ⎞ 2 1 + 2h
x5
= lim
⎜ 1 + 2h ⎟
x →∞ ⎛ k x 2 2
k x 3 3
k 4 x 4 x 5k 5 k 6 x 6 ⎞ ⎝ ⎠
⎜1 + kx + 2! + 3! + 4! + 5! + 6! + ...⎟ ⎡ log(1 + x ) ⎤
⎝ ⎠
 ⎢ Using xlim = 1⎥
⎣ →0 x ⎦
1
The correct option is (B)
= lim

x →∞ ⎡⎛ 1 1 k 1 k3 1 k4 1⎞
2
⎡ 1 e1 n e 2 n
⎢⎜ 5 + k ⋅ 4 + ⋅ + ⋅ + ⋅ e( n −1) n ⎤
⎢⎣⎝ x x 2! x 3 3! x 2 4! x ⎟⎠ 12. lim ⎢ +
n→∞ ⎢ n n
+
n
+ ... +
n ⎥⎦


k5 ⎛ k6 ⎞⎤
+ + ⎜ x + ...⎟ ⎥ ⎡1 + e1 n + (e1 n ) 2 + ... + (e1 n ) n −1 ⎤
5! ⎝ 6! ⎠ ⎥⎦ = lim ⎢

n→∞ ⎢
⎣ n ⎥⎦
1
=
=0
∞ 1 ⋅ ⎡⎣(e1 n ) n − 1⎤⎦ 1

The correct option is (C) = lim
= (e – 1) lim 1 n
n→∞ n ( e1 n − 1) n→∞ ⎛ e − 1⎞
1 1 ⎜ 1n ⎟
lim (cos x + sin x −1) ⎝ ⎠
9. lim (cos x + sin x ) x = e x→0
x
x→0 ( − sin x + cos x )
= (e – 1) × 1 = (e – 1).

lim

= e x→0
1
The correct option is (C)

 (Using L’Hospital’s Rule) (1 + h)sin (1 + h − [1 + h])
13. RHL = lim
= e1 = e
HINTS AND EXPLANATIONS


h→ 0 1+ h −1

The correct option is (A) (1 + h)sin (1 + h − 1)

= lim
3 h
2 2 − (cos x + sin x ) ⎛0 ⎞ h→ 0
10. lim  ⎜⎝ form⎟⎠
π 1 − sin 2 x 0 sin h
x→
4

= lim(1+ h) =1
2
h→ 0 h
−3 (cos x + sin x ) ( − sin x + cos x )
= lim
(1 − h)sin (1 − h − [1 − h])
x→
π −2 cos 2 x LHL = lim

4 h→ 0 1− h −1

 (Using L’Hospital’s Rule)
(1 − h)sin (1 − h)
−3 (cos x + sin x )(cos x − sin 2 x ) 2
= lim = –∞
= lim
h→ 0 −h
x→
π −2 cos 2 x Since LHL ≠ RHL,

4
−3 (cos x + sin x )cos 2 x x sin ( x − [ x ])
= lim
∴ lim
does not exist.
π −2 cos 2 x x →1 x −1
x→
4
The correct option is (D)
3 (cos x + sin x ) 3 ⎛ 1 1 ⎞ 3
= lim
= ⋅⎜ + ⎟ = 2 sin x − sin 2 x
x→
π
4
2 2 ⎝ 2 2⎠ 2 14. f (x) = ∫
x3
dx


The correct option is (A) d 2 sin x − sin 2 x 2 sin x − sin 2 x
dx ∫
⇒ f ′ (x) =
dx =
ln(1 + 2h) − 2 ln(1 + h) x3 x3
11. lim
h→ 0 h2 2 sin x − sin 2 x ⎛0 ⎞

lim f ′( x ) = lim  ⎜⎝ form⎟⎠
ln(1 + h) 2 − ln(1 + 2h) x →0 x→0 x3 0
= – lim

h→ 0 h2 2 cos x − 2 cos 2 x ⎛0 ⎞

= lim 2
 ⎜⎝ form⎟⎠
x→0 3x 0
Limits  8.27

−2 sin x + 4 sin 2 x ⎛0 ⎞ ⎡⎛ 1 1 ⎞ ⎛ 1 1 ⎞ ⎛ 1 1 ⎞ ⎛1 1 ⎞⎤

= lim  ⎜⎝ form⎟⎠ = nlim
⎢⎜ − 2 ⎟⎠ + ⎜⎝ 2 − 3 ⎟⎠ + ⎜⎝ 3 − 4 ⎟⎠ + ... + ⎜⎝ n − n + 1⎟⎠ ⎥
x→0 6x 0 →∞ ⎣ ⎝ 1 ⎦
−2 cos x + 8 cos 2 x

= lim ⎡ 1 ⎤
x→0 6 = lim ⎢1 −
= 1 – 0 = 1.
6
n→∞ ⎣ n + 1⎥⎦

= =1
The correct option is (A)
6

The correct option is (D)
(log x ) 2 ⎛∞ ⎞
20. lim  ⎜⎝ ∞ form⎟⎠
π x →∞ x n
15.  < 1,
4 1
2 log x ⋅
⎛π⎞ x = lim 2 log x  ⎛∞ ⎞
∴ ⎜ ⎟ = 0 lim form⎟
⎝ 4⎠ x →∞ n x n −1 x →∞ n x n ⎝⎜ ∞ ⎠
⎛ x⎞ 2
⎜⎝ ⎟⎠ = lim 2 n = 0
2 x →∞ n x

lim =0
x → π / 2 ln(sin x )
The correct option is (B)

The correct option is (C)
r r
16. We know that |cosθ | ≤ 1 for all θ. 21. tr = =
r4 + r2 + 1 ( r 2 + 1) 2 − r 2
So, if |cos n! px| < 1,
lim lim (1 + cos 2 m n!π x ) = (1 + 0) = 1 1⎛ 1 1 ⎞
=
⎜ − ⎟
m →∞ n→∞
2 ⎝ r 2 − r + 1 r 2 + r + 1⎠

and if |cosn! px| = 1,
1⎡ 1 1 ⎤
lim lim (1 + cos 2 m n!π x ) = lim lim (1 + 12 m ) =
⎢ − ⎥
m →∞ n→∞ m →∞ n→∞ 2 ⎣ r ( r − 1) + 1 ( r + 1) r + 1⎦

= lim lim (1 + 1) = 2 n n
1
m →∞ n→∞ ∴
∑ tr = ∑ 2 [ f ( r ) − f ( r + 1)] ,

The correct option is (A) and (B) r =1 r =1

1 1
where f (r) =
= [ f (1) − f ( n + 1)]
⎡ sin([ x − 3]) ⎤ ⎡ sin( − 4) ⎤ r ( r − 1) + 1 2
17. LHL = lim ⎢ ⎥ = ⎢ ⎥

HINTS AND EXPLANATIONS


x → 0 ⎣ [ x − 3] ⎦

⎣ −4 ⎦
1⎡ 1 ⎤ 1
⎡ sin 4 ⎤ 3π
= ⎢1 − ⎥ → as n → ∞
= ⎢   π < 4 < 2 ⎣ ( n + 1)n + 1⎦ 2

⎥ = –1
⎣ 4 ⎦ 2

The correct option is (B)
⎡ sin[ x − 3] ⎤ ⎡ sin( − 3) ⎤
RHL = lim ⎢
⎥ = ⎢ ⎥
x → 0 ⎣ [ x − 3] ⎦
+
⎣ −3 ⎦ 22. LHL = lim( −1)[ n − h] = lim( −1) n −1 = (–1)n – 1
h→ 0 h→ 0
⎡ sin 3 ⎤ π

= ⎢ ⎥ = 0   < 3 < π.
RHL = lim( −1)[ n + h] = lim( −1) n = (– 1)n
⎣ 3 ⎦ 2 h→ 0 h→ 0


The correct option is (C)
Since LHL ≠ RHL

⎛ x2 + 1 ⎞ ∴
lim( −1)[ x ] does not exist.
x→n
18. We have, lim ⎜ − ax − b⎟ = 0
x →∞ ⎝ x + 1 ⎠
The correct option is (D)
2
( x + 1) − ( ax + b)( x + 1) 23. Since y = x – 1,
⇒ lim
=0
x →∞ x +1 \ x = y + 1.
x 2 (1 − a) − ( a + b) x − b + 1 As (x, y) → (1, 0) along the line y = x – 1, x = y + 1 holds
⇒ lim
=0
x →∞ x +1 throughout.
⇒ 1 – a = 0 and a + b = 0
y3 y3

lim = lim
⇒ a = 1 and b = –1.
x →1 3
x − y −1 2 y→0 ( y + 1) − y 2 − 1
3
y→0

The correct option is (A)
y3 y2 0
⎡ 1 1 1 1 ⎤
lim 3 2
= lim 2
= =0
19. lim ⎢ + + + ... + ⎥
y→0 y + 2 y + 3y y→0 y + 2y + 3 3
n→∞ ⎣1 ⋅ 2 2 ⋅ 3 3⋅ 4 n( n + 1) ⎦

The correct option is (C)
8.28  Chapter 8

1 − 2 + 3 − 4 + 5 − 6 + ... − 2n = xlim
( 2 x / 2 + 2) ( 2 x − 2) = (2 + 2) ⋅ (4 – 2) = 8
24. lim →2
n→∞
n2 + 1 + 4 n2 − 1

The correct option is (A)
[1 + 3 + 5 + 7 + ... + ( 2n − 1)] − ( 2 + 4 + 6 + ... + 2n)
= lim

n→∞
n 1+
1 1
+n 4− 2 8 ⎡ x2 x2 x2 x2 ⎤
n2 n
27. lim 8 ⎢1 − cos − cos + cos cos ⎥
x→0 x ⎢
⎣ 2 4 2 4 ⎥⎦
n n
[2 ⋅1 + ( n − 1) ⋅ 2] − [2 ⋅ 2 + ( n − 1) ⋅ 2] 8 ⎡⎛ x2 ⎞ x4 ⎛ x2 ⎞ ⎤
= lim 2 2 = lim
⎢ ⎜ 1 − cos − cos 1 − cos ⎥
2 ⎟⎠ 4 ⎜⎝ 2 ⎟⎠ ⎥⎦
x → 0 x8 ⎢⎝
n→∞ ⎛ 1 1⎞ ⎣
n ⎜ 1+ 2 + 4 − 2 ⎟
⎝ n n ⎠ 8 ⎛ x2 ⎞ ⎛ x2 ⎞
n n = lim 8 ⎜1 − cos ⎟ ⎜1 − cos ⎟

x→0 x ⎝ 2 ⎠⎝ 4⎠
⋅ 2n − 2 ( n + 1)
= lim 2 2
⎛ 8 x2 x2
n→∞ 1 1⎞ = lim
⋅ 2 sin 2 ⋅ 2 sin 2
n ⎜ 1+ 2 + 4 − 2 ⎟ x→0 x 8
4 8
⎝ n n ⎠
2 2
⎛ x2 ⎞ ⎛ 2⎞
n2 − n2 − n sin 2 sin x 2
= lim
32 ⎜ ⎟ ⎛x ⎞ ⎜
2 ⎟ ⎛ x2 ⎞

n→∞ 1 1⎞ = lim 8 ⎜ 24 ⎟ ⎜ ⎟ ⎜ 28 ⎟ ⎜ ⎟

n ⎜ 1+ 2 + 4 − 2 ⎟ x→0 x
⎜ x ⎟ ⎝ 4⎠ ⎜ x ⎟ ⎝ 8⎠
⎝ n n ⎠ ⎝ 4 ⎠ ⎜⎝ ⎟
8 ⎠
−n 1
= lim
=

n→∞ ⎛ 1 1⎞ 32
n ⎜ 1+ 2 + 4 − 2 ⎟
⎝ n n ⎠
The correct option is (C)
−1 −1 −1
= lim
= =
n→∞ 1 1 1+ 2 3 28. lim ⎡⎣log n −1( n) ⋅ log n ( n + 1) ⋅ log n +1( n + 2)...log n k
−1 ( n
k
)⎤⎦
1+
+ 4− 2 n→ ∞
n2 n
⎡ log n log( n + 1) log( n + 2) log( nk ) ⎤

The correct option is (B) = lim ⎢
⋅ ⋅ ... ⎥
n→∞ ⎢ log( n − 1) log n log( n + 1) log( nk − 1) ⎥⎦

⎛ x 4 sin (1 / x ) + x 2 ⎞
25. lim ⎜ ⎟ ⎛ log m ⎞
x→−∞ ⎝ 1 + | x |3 ⎠
 ⎜⎝ Using log n m = log n ⎟⎠
HINTS AND EXPLANATIONS

1 log nk log n ⎛∞ ⎞
− y 4 sin + y 2 = lim
= k lim  form⎟
= lim
y n→∞ log( n − 1) n→∞ log( n − 1) ⎝⎜ ∞ ⎠
y→∞ 1+ y 3
(Putting x = – y; as x→ – ∞, y→ ∞) 1/ n
= k lim
 (Using L’Hospital’s Rule)
⎛ 1⎞ n→∞ 1 / n −1
sin
⎜ y⎟ 1 ⎛ 1⎞
−⎜ ⎟+ = k lim ⎜1 − ⎟ = k

1 n→∞ ⎝ n⎠
⎜ ⎟ y
⎝ y ⎠ −1 + 0
The correct option is (C)
= lim = = –1
y→∞ 1 1+ 0
1+ 3
y ⎡ 1 1 1 1 ⎤
29. lim ⎢ + + + ... + ⎥

The correct option is (B) n→∞ ⎣1 ⋅ 3 3⋅5 5⋅ 7 ( 2n + 1)( 2n + 3) ⎦
2 x + 23 − x − 6 ⎡ 1 ⎛ 1 1⎞ 1 ⎛ 1 1⎞ 1⎛ 1 1 ⎞⎤
26. lim = lim ⎢ ⎜ − ⎟ + ⎜ − ⎟ ... + ⎜
− ⎟
x→2 2 − x / 2 − 21 − x n→∞ ⎣ 2 ⎝ 1 3 ⎠ 2 ⎝ 3 5⎠ 2 ⎝ 2n + 1 2n + 3 ⎠ ⎥⎦
( 22 x + 23 − 6 ⋅ 2 x ) / 2 x
= lim
1⎛ 1 ⎞ 1 1
x→2 1 2 = lim
⎜1 − ⎟ = (1 – 0) =
x/2
− x n→∞ 2 ⎝ 2n + 3 ⎠ 2 2
2 2
( 2 x − 4 ) ( 2 x − 2)
The correct option is (B)
22 x − 6 ⋅ 2 x + 8
= lim
= lim
x→2 2x / 2 − 2 x→2 ( 2 x / 2 − 2) nx
⎛ 1x 1 1 1 ⎞
x/2 x/2 x 1 + 2 x + 3 x + ... + n x
(2 + 2) ( 2 − 2) ( 2 − 2) 30. lim ⎜ ⎟
= lim
x →∞ ⎜ n ⎟⎠
x→2 (2 x/2
− 2) ⎝
Limits  8.29

n
 (Using L’Hospital’s Rule)
⎛ y y y y⎞ y
= lim 1 + 2 + 3 + ... + n
= n + (n – 1) + … + 2 + 1 =

n ( n + 1)

y→0 ⎝

n ⎠ 2

The correct option is (A)
n ⎛ 1y + 2 y + 3 y + ... + n y ⎞
−1⎟
lim
y ⎝⎜ n ⎠ 12 + 22 + 32 + ... + r 2
= e
y→0
34. tr =
13 + 23 + 33 + ... + r 3
⎛ 1y + 2 y + 3 y + ... + n y − n ⎞ 2
lim
y→0 ⎜
⎝ y ⎠⎟ r ( r + 1)( 2r + 1) ⎛ 2 ⎞
= e
=
⋅⎜
6 ⎝ r ( r + 1) ⎟⎠
⎡ (1y −1) ( 2 y −1) ( 3 y −1) ( n y −1) ⎤
lim + + + ...+
y→0 ⎢
⎣ y y y y ⎦
⎥ 2⎛1 1 ⎞
=
e =
⎜ + ⎟
3 ⎝ r r + 1⎠
= e (log 1 + log 2 + log 3 + … + log n)

log (1 ⋅ 2 ⋅ 3 … n) 2 ⎡ ⎛ 1 ⎞ ⎛ 1 1⎞ ⎛ 1 1 ⎞ ⎛1 1 ⎞⎤
=e = n! ∴ Sn =
− ⎜1+ ⎟ + ⎜ + ⎟ − ⎜ + ⎟ + … ± ⎜ +
3 ⎢⎣ ⎝ 2 ⎠ ⎝ 2 3 ⎠ ⎝ 3 4 ⎠ ⎝ n n + 1⎟⎠ ⎥⎦

The correct option is (A)
2⎛ 1 ⎞
31. lim (1 + x) (1 + x2) (1 + x4) … (1 + x2n)
⎜ −1 ± = ⎟
n→∞ 3⎝ n + 1⎠
(1 − x )(1 + x )(1 + x 2 )(1 + x 4 )...(1 + x 2 n ) 2
= lim
∴ lim Sn = –

n→∞ 1− x n→∞ 3

The correct option is (B)
(1 − x 2 )(1 + x 2 )(1 + x 4 )...(1 + x 2 n )
= lim
(1 + a3 ) + 8e1 x ⎛∞ ⎞
n→∞ 1− x 35. We have 2 = lim ⎜⎝ ∞ form⎟⎠ (1)

. . . x → 0 1 + (1 − b3 ) e1 x


. . .

. . . 0 + 8e1 x ( −1 x 2 )
⇒ 2 = lim

x → 0 0 + (1 − b3 ) e1 x ( −1 x2 )
1 − x 4n + 2 1
= lim
= for |x| < 1  (Using L’Hospital’s Rule)
n→∞ 1− x 1− x
⇒ 1 – b3 = 4

The correct option is (B)
⇒ b3 = – 3
32. Case I:  n is a positive integer ⇒ b = (–3)1/3
xn nx n −1

HINTS AND EXPLANATIONS


(1 + a3 ) + 8e1 x
lim x
= lim x

∴ From Eq. (1), 2 = lim
x →∞ e x →∞ e x→0 1 + 4e1 x
3
n ( n − 1) x n − 2 n! ⇒ 1 + a = 2 i.e., a = 1

= lim x
= … = lim x Hence a = 1 and b = (–3)1/3.
x →∞ e x →∞ e

 (Using L’Hospital’s Rule repeatedly) The correct option is (A)


=0 36. x2 + 4x + 5 = (x + 2)2 + 1 ≥ 1. So, a = 1.
Case II:  n is a negative integer. 1 − cos 2θ 2 sin 2 θ
Also, b = lim
2
= lim = 2.
x n
x −m θ →0 θ θ →0 θ2
lim = lim n
e x x →∞ e x
x →∞ ∴
∑ ar ⋅ bn − r = bn + abn – 1 + a2 bn – 2 + … + an
(Putting n = – m, where m is a positive integer) r=0
1 1 ⎡ ⎛ a ⎞ n +1 ⎤ ⎡ ⎛ 1 ⎞ n +1 ⎤
= lim m x = = 0. b n ⎢1 − ⎜ ⎟ ⎥ 2n ⎢1 − ⎜ ⎟ ⎥
x →∞ x e ∞
⎢⎣ ⎝ b ⎠ ⎥⎦ ⎢⎣ ⎝ 2 ⎠ ⎥⎦
Case III:  n = 0 = =
a 1
xn 1 1 1− 1−
lim x = lim x = = 0. b 2
x →∞ e x →∞ e ∞
2n +1 ( 2n +1 − 1)
x n

= = (2n + 1 – 1)
Hence, lim x = 0 for all values of n. 2n +1
x →∞ e

The correct option is (B)
The correct option is (B)
1 ⋅ 2 + 2 ⋅ 3 + 3 ⋅ 4 + ... + n ( n + 1)
x n + x n −1 + x n − 2 + ... + x 2 + x − n ⎛0 ⎞ 37. lim
33. lim  ⎜⎝ form⎟⎠ n→∞ n3
x →1 x −1 0
Σ n ( n + 1) Σ n 2 + Σn
nx n −1 + ( n − 1) x n − 2 + ... + 2 x + 1 = lim
= lim
= lim
n→∞ n3 n→∞ n3
x →1 1
8.30  Chapter 8

1 ⎡ n ( n + 1)( 2n + 1) n ( n + 1) ⎤ x3 = 2 + x2 = 2 + 5

= lim
+
n→∞ n3 ⎢⎣ 6 2 ⎥⎦ ∴ x1 > x2 > x3

⎡1 ⎛ 1⎞ ⎛ 1⎞ 1 ⎛ 1 1 ⎞ ⎤ It can be easily shown by mathematical induction that the

= lim ⎢ ⎜1 + ⎟ ⎜ 2 + ⎟ + ⋅ ⎜ + 2 ⎟ ⎥
sequence x1, x2, …xn, … is a monotonically decreasing
n→∞ ⎣ 6 ⎝ n⎠ ⎝ n⎠ 2 ⎝ n n ⎠ ⎦
sequence bounded below by 2. So it is convergent.
1 1
=
×1×2= . Let lim xn = x. Then

6 3 n→∞

The correct option is (C)
xn + 1 =
2 + xn
log(1 + x + x 2 ) + log(1 − x + x 2 ) ⇒ lim xn + 1 = 2 + lim xn
38. lim n→∞ n→∞
x→0 sec x − cos x
log ⎡⎣(1 + x ) − x ⎦
2 2 2⎤ ⇒ x = 2 + x

= lim
⇒ x2 – x – 2 = 0

x → 0 (1 − cos 2 x ) cos x
⇒ (x – 2)(x + 1) = 0

log(1 + x 2 + x 4 ) ⎛0 ⎞ ⇒ x = 2
( xn > 0 ∀n, ∴ x > 0)
= lim
⎜⎝ form⎟⎠
x→0 sin x tan x  0
The correct option is (B)
log[1 + x 2 (1 + x 2 )] 2 1
= lim ⋅ x (1 + x 2 ) ⋅ 3x +1 − 5 x +1 3 ⋅ 3x − 5 ⋅ 5 x
x→0 x 2 (1 + x 2 ) sin x tan x 2 41. lim = lim
⋅ ⋅x x →∞ x
3 −5 x x →∞ 3x − 5 x
x x
⎡ log(1 + x ) ⎤ x
= 1. ⎢as lim = 1⎥ ⎛ 3⎞
⎣ x→0 x ⎦ 3⋅ ⎜ ⎟ − 5
⎝ 5⎠ −5

= lim =
The correct option is (A) x →∞ x
−1
⎛ 3⎞
⎛ h⎞ ⎜⎝ ⎟⎠ − 1
loge e ⎜1 − ⎟ − 1 5
ln(e − h) − 1 ⎝ e⎠
39. LHL = lim = lim ⎛ ⎞
h→ 0 | e − h − e | h→ 0 | −h | = 5. a n = 0, if − 1 < a < 1⎟

⎜⎝∵ nlim
→∞ ⎠
⎛ h⎞
The correct option is (A)
log e + log ⎜1 − ⎟ − 1
⎝ e⎠

= lim
1⎛
h→ 0 h n −1 ⎞
1/ n 2/ n
2 42. lim ⎜1 + e + e + ... + e n

h h n→∞ n
⎝ ⎠
− − 2 − ...
HINTS AND EXPLANATIONS

1

= lim e 2e =–
( )
n
h→ 0 | e + h − e | e 1/ n
1 1− e 1− e
ln (e + h) − 1 = lim ⋅
= lim
n→∞ n 1 − e1/ n n→∞ ⎛ 1 1 1 ⎞
RHL = lim
n ⎜1 − 1 − − ⋅ 2 ...⎟
h→ 0 | e + h − e | ⎝ n 2! n ⎠
⎛ h⎞ 1− e 1− e
log e ⎜1 + ⎟ − 1 = lim
= =e–1
⎝ e⎠ n→∞ 1 1 −1

= lim −1 − ⋅ ...
h→ 0 |h| 2! n

The correct option is (C)
⎛ h⎞
log e + log ⎜1 + ⎟ − 1
⎝ e⎠

= lim sin x
h→ 0 h 1+
x + sin x x 1+ 0
h h2 43. lim = lim = = 1.
− 2 + ... x →∞ x − cos x x →∞ cos x 1− 0
1 1−
= lim e 2e = x
h→ 0 h e
⎡ sin x ⎛1⎞ ⎤
Since LHL ≠ RHL ⎢∵ lim = lim y sin ⎜ ⎟ = O × (a finite quantity) ⎥
ln x − 1
 ⎢ x →∞ x y →0 ⎝ y⎠ ⎥
∴ lim does not exist. ⎢ ⎥
x→e | x − e | cos x
⎢ = 0. Similarly lim =0 ⎥
The correct option is (D) ⎣ x →∞ x ⎦
40. We have
The correct option is (B)
n
x1 = 3, xn + 1 = 2 + xn 44. Sn= ∑ ai , lim an = a
n→∞
x2 =
2 + x1 = 2+3 = 5 i =1
Sn + 1 – Sn = an + 1
Limits  8.31

an + 1
(Dividing the numerator and denominator by the highest
lim 2a power x1/2)
So, n → ∞
n( n + 1) = nlim
→ ∞ n( n + 1)
=0
2
2 =

3

The correct option is (A)
The correct option is (A)
⎡⎛ 1⎞
13 ⎤ x 3
z 2 − ( z − x )2
45. lim ⎡ n2 − n3 + n⎤ = lim n ⎢⎜ −1 + ⎟ + 11 3 ⎥
3
⎢ ⎥⎦ 49. lim
n→∞ ⎣ n→∞ ⎢ ⎝ n ⎠ ⎥⎦ x→0 3
⎣ ( 8 xz − 4 x 2 + 3 8 xz ) 4
⎛1 ⎞ 3
⎜⎝ − 1⎟⎠ + 1 x 2 xz − x 2
n = lim


= lim n ⋅ 23 13
x→0 ( 3 x 3
8z − 4 x + 3 8z 3
x )4
n→∞ ⎛1 ⎞ ⎛1 ⎞
⎜⎝ − 1⎟⎠ + 1 − ⎜ − 1⎟
⎝n ⎠ x4 3 3
2z − x
n = lim

( )
x→0 4
43 3
⎛ a3 + b3 ⎞ x 8z − 4 x + 3 8z

 ⎜ Using a + b = 2 ⎟
⎝ a − ab + b 2 ⎠ 3
2z
=
(2 )
4
1 1 1 3
8z
= lim
= =
n→∞ ⎛ 1 ⎞
23
⎛1 ⎞
13
1+1+1 3 1
⎜⎝ − 1⎟⎠ + 1 − ⎜ − 1⎟ =
n ⎝n ⎠ 2 ⋅z 23 3


The correct option is (A)
The correct option is (B)
4
n5 + 2 − n2 + 1
3 A h 2 hr − h2
50. lim = lim
( 2 hr − h )
46. lim 3 3
n→∞ 5 2 h→0 +
P h→0 +

n4 + 2 − n3 + 1 8 2
+ 2 hr
2 1
n5 4 4 1 +
− n2 3 3 1 + 2 h ⋅ h 2r − h
n 5
n
= lim
= lim

( )
h→0 3
n→∞ 4 5 2 1 8⋅ h ⋅ h 2r − h + 2r
n 5 1 + 4 − n3 2 2 1 + 3
n n
2r − h

= lim
n5 4 4 2 n2 3
( )
1 h→0 3
1+ 5 − 3 2 3 1+ 2 8 2r − h + 2r

HINTS AND EXPLANATIONS


32
n
= lim 4 5
n n n
n→∞ n 2 n3 2 2 1 2r 1
5 1+ − 1+ 3
= =
( )
32 3
n n 4 n3 2 n 8 2 2r 128r

(Dividing the numerator and denominator by the highest
The correct option is (B)
power n3/2)
⎛ 1 − cos 2( x − 2) ⎞
1 4 2 1 1 51. lim ⎜ ⎟
1+ 5 − 5 6 3 1+ 2 x → 2⎝ x−2 ⎠
n14
n n n 0−0
= lim
= = 0.
n→∞ 1 5 2 2 1 0 −1 2 | sin( x − 2) |
1+ 4 − 1+ 3 = lim

n7 10 n n x→2 x−2
which doesn’t exist as LHL = − 2 whereas

The correct option is (B)
47. Minimum power in numerator on x is 3. So n = 3. RHL =
2
The correct option is (C)
The correct option is (B)
3 5 x x x x
2 x +3 x +5 x 52. Required limit= lim cos cos 2 cos 3 ... cos n
48. lim n→∞ 2 2 2 2
x→∞ 3x − 2 + 3 2 x − 3
2 x +33 x +55 x ⎧ x 1 x ⎛ x x ⎞⎫
= lim
cos ... cos n − 1 ⎜ 2 sin n cos n ⎟ ⎬
=
lim n→∞ x ⎨⎩ 2 2 ⎝ 2 2 ⎠⎭
x→∞ 2 3 3 2 sin n
x 3− + x 3 2− 2
x x
1 ⎧ x ⎛ x x ⎞⎫
3 5 = lim
cos ... ⎜ 2 cos n − 1 sin n − 1 ⎟ ⎬
2+ + n→∞ 2 x ⎨⎩ 2 ⎝ 2 2 ⎠⎭
x1/ 6 x 3/10 2 sin n
=
lim 2
x→∞ 2 1 3
…………………………………………………………..
3 − + 1/ 6 3 2 −
x x x
…………………………………………………………..
8.32  Chapter 8

1⎛ x x⎞ n n n
= nlim
→∞ n x ⎜⎝
2 cos sin ⎟
2 2 ⎠

∑ ( k 2 x − 1) ≤ ∑ [k 2 x] < ∑ k 2 xa
2 sin n k =1 k =1 k =1
2
⎛ n ⎞ n n ⎛ n ⎞
sin x ⇒ x ⎜ ∑ k 2 ⎟ − ∑ (1) ≤
∑ [k 2 x ] < x ⎜ ∑ k 2 ⎟
= lim

n→∞ n ⎛ x⎞ ⎝k =1 ⎠ k =1 k =1 ⎝k =1 ⎠
2 sin ⎜ n ⎟
⎝2 ⎠ n
xn( n + 1)( 2n + 1) xn ( n + 1)( 2n + 1)

− n ≤ ∑ [k 2 x ] <
⎧ ⎛ x⎞ ⎫ 6 6
k =1
sin x ⎪⎪ ⎜⎝ n ⎟⎠ ⎪⎪ sin x
2 Dividing throughout by n3, we have

=
lim ⎨ ⎬ =
x n → ∞
⎪ sin ⎛⎜ ⎞⎟ ⎪
x x
xn ( n + 1)( 2n + 1) 1 n
[k 2 x ] xn ( n + 1)( 2n + 1)
⎪⎩ ⎝ 2n ⎠ ⎪⎭
6 n3
− 2 ≤
n
∑ n3
<
6 n3
k =1
The correct option is (B)
x⎛ 1⎞ ⎛ 1⎞ 1 n
[k 2 x ] x ⎛ 1⎞ ⎛ 1⎞
⎜⎝1 + ⎟⎠ ⎜⎝ 2 + ⎟⎠ − 2 ≤ ∑ 3 < ⎜⎝1 + ⎟⎠ ⎜⎝ 2 + ⎟⎠
53. The (r + 1)th term of the series is ⇒

n−r 6 n n n k =1 n 6 n n

tr + 1 = ( r + 1) ∑ k
Taking limits as n→ ∞, we get

k =1
⇒ tr + 1 = (r + 1){1 + 2 + 3 + … (n – r) terms}
⎧x ⎛ 1⎞ ⎛ 1⎞ 1 ⎫ x⎛ 1⎞ ⎛ 1⎞
lim ⎨ ⎜1 + ⎟ ⎜2 + ⎟⎠ − 2 ⎬ ≤ L < nlim ⎜1 + ⎟ ⎜2 + ⎟
1 n → ∞ ⎩6 ⎝ n⎠ ⎝ n n ⎭ → ∞ 6⎝ n⎠ ⎝ n⎠
⇒ tr + 1 = ( r + 1) ( n − r )( n − r + 1)

2 1
1 Since, as n→ ∞, we have → 0

⇒ tr + 1 = ( r + 1)( n2 − rn + n − rn + r 2 − r )
n
2 x x

≤L<
1 3 3
⇒ tr + 1 =
( r + 1)( r 2 − (1 + 2n)r + n2 )
According to Squeeze Principle or Sandwich Theorem, we
2 have
1 x
⇒ tr + 1 = ( r 3 − 2nr 2 + ( n2 − 2n − 1)r + n2 )
L= .

2 3
n −1
The correct option is (C)

Now, S= ∑ tr + 1 sin 2θ
r=0 55. tan 2θ =
1 n cos 2θ
∴ S =
∑ {r 3 − 2nr 2 + ( n2 − 2n − 1)r + n2}
2 r =1 1 2 sin θ cos θ
⇒ =
HINTS AND EXPLANATIONS


2 cot 2θ cos 2 θ − sin 2 θ
1 ⎡ ⎧ n( n + 1) ⎫ ⎧1 ⎫
⇒ S =
⎢ ⎨ 2 ⎬ − 2n ⎨ 6 n ( n + 1)( 2n + 1)⎬ cos 2 θ − sin 2 θ
2 ⎣ ⎩ ⎭ ⎩ ⎭ ⇒ cot 2θ =

2 sin θ cos θ
⎧1 ⎫ ⎤

+ ( n2 − 2n − 1) ⎨ n( n + 1)⎬ + n2 ( n)⎥ cos 2 θ sin 2 θ
⎩2 ⎭ ⎦ ⇒ 2cot 2θ =

sin θ cos θ sin θ cos θ

Solving and rearranging, we have,
⇒ 2cot 2θ = cot θ – tan θ

1 4
S=
{n − 11n3 − 19n2 + 6 n} ⇒ tan θ = cot θ – 2 cot 2θ(1)

24
S 1 ⎛ n4 − 11n3 − 19n2 + 6 n ⎞ Now, tan θ = cot θ – 2 cot 2θ

∴ lim 4 = lim
⎜ ⎟ 1 θ 1 θ
n→∞ n n → ∞ 24 ⎝ n4 ⎠ ⇒
tan = cot − cot θ
2 2 2 2
1 ⎛ 11 19 6 ⎞

= lim ⎜1 − − 2 + 3 ⎟ 1 θ 1 θ 1
24 n → ∞ ⎝ n n n ⎠ ⇒ 2 tan 2 = 2 cot − cot θ

2 2 2 2 2
n −1⎧ n−r ⎫⎪
………
1 ⎪ 1

lim
n→∞ n 4 ∑ ⎨( r + 1) ∑ k ⎬ = 24
………
r = 0⎪
⎩ ⎪
k =1 ⎭


The correct option is (A) 1 θ 1 θ 1 θ

tan = cot − cot
2n 2n 2n 2n 2n − 1 2n − 1
1 ⎛ n 2 ⎞ 1 θ
54. Let L = lim ⎜ ∑ [k x ]⎟ ⇒ S = − 2 cot 2θ +
cot n
n → ∞ n3 ⎝
k =1 ⎠ 2n 2
⎛ 1 θ 1 θ 1 θ⎞

Since k2x – 1 ≤ [k2x] < k2x
= lim ⎜ tan θ + tan + 2 tan 2 +… + n tan n ⎟
n→∞ ⎝ 2 2 2 2 2 2 ⎠
Limits  8.33

θ⎞
lim S = lim ⎛⎜ − 2 cot 2θ + Hence, xlim
1 ( A1 A2 ... An ) does not exist.

cot n ⎟ →a
n→∞ n→ ∞⎝ 2n 2 ⎠
m


The correct option is (D)
⎡ θ ⎤ sin x
1 ⎢ 2n ⎥ 58. We know that lim → 1–

= − 2 cot 2θ + lim ⎢ ⎥ x→0 x
n→∞θ ⎢ θ
tan n ⎥
lim
x
→ 1+
⎢⎣ 2 ⎥⎦ x → 0 sin x
1 ⎡ x ⎤

= − 2 cot 2θ + ⎡ sin x ⎤
θ ∴ lim ⎢100 ⎥ + lim 99 = 100 + 98 = 198.
x→0⎣ sin x ⎦ x → 0 ⎢⎣ x ⎥⎦

The correct option is (B)

The correct option is (B)
4 3n − 2 − 9 n + 1
56. lim 2n − 1 n −1 x5
n→∞ 8 −9 59. lim
x →∞ 5 x
4 − 2 ⋅ 64 n − 9 ⋅ 9n
= lim
x5
n→ + ∞ 8−1 ⋅ 64 n − 9−1 ⋅ 9n = lim
x →∞ e x log 5
⎛ 9⎞
n
4− 2 − 9 ⎜ ⎟ x5
⎝ 64 ⎠ = lim
, where k = log 5
= lim
n x →∞ e kx
n→ + ∞ ⎛ 9⎞
8−1 − 9−1 ⎜ ⎟ x5
⎝ 64 ⎠ = lim

x →∞ ⎛ k 2 x 2 k 3 x 3 k 4 x 4 x 5k 5 k 6 x 6 ⎞
4− 2 − 0
1 ⎜1 + kx + 2! + 3! + 4! + 5! + 6! + ...⎟
= −1
= ⎝ ⎠
8 −0 2
n 1
−2 ⎛ 64 ⎞ = lim

4 ⎜ ⎟ −9 x →∞ ⎡ ⎛ 1 1 k2 1 k3 1 k4 1⎞
⎝ 9⎠ ⎢⎜ 5 + k ⋅ 4 + ⋅ + ⋅ + ⋅
= lim
2! x 3 3! x 2 4! x ⎟⎠
n→− ∞ ⎛ 64 ⎞
n ⎢⎣⎝ x x
8−1 ⎜ ⎟ − 9 −1
⎝ 9⎠ k5 ⎛ k6 ⎞⎤
+ + ⎜ x + ...⎟ ⎥
0−9 5! ⎝ 6! ⎠ ⎥⎦
=
= 81
0 − 9 −1 1
=
=0

HINTS AND EXPLANATIONS



Hence, limit does not exist. ∞

The correct option is (C)

The correct option is (C)
r r
x − ai 60. tr = 4 2
=
57. We have, Ai = , i = 1, 2, …, n r + r +1 ( r + 1) 2 − r 2
2

| x − ai |
1⎡ 1 1 ⎤
and, a1 < a2 < … an – 1 < an =
− 2
2 ⎣ r − r + 1 r + r + 1⎥⎦
⎢ 2
Let x be in the left neighbourhood of am
Then,  x – ai < 0 for i = m, m + 1, … n 1⎡ 1 1 ⎤
=
⎢ − ⎥
and  x – ai > 0 for i = 1, 2, …, m – 1 2 ⎣ r ( r − 1) + 1 ( r + 1) r + 1⎦
x − ai n n
∴ Ai = = –1, for i = m, m + 1, …, n 1
− ( x − ai ) ∴
∑ tr = ∑ 2 [ f ( r ) − f ( r + 1)] ,
x − ai r =1 r =1
and,  Ai = = 1, for i = 1, 2, …, m – 1 1
x − ai where,  f (r) =

r ( r − 1) + 1
Similarly, if x is in the right neighbourhood of ai
1
Then,  x – ai< 0 for i = m + 1, …, n
= [ f (1) − f ( n + 1)]
2
and  x – ai> 0 for i = 1, 2, …, m
x − ai 1⎡ 1 ⎤ 1
∴ Ai = = –1 for i = m + 1, …n
= ⎢1 − ⎥ → as n→ ∞
− ( x − ai ) 2 ⎣ ( n + 1)n + 1⎦ 2
x − ai
The correct option is (B)
and,  Ai = = 1 for i = 1, 2, …, m
x − ai nx
⎡11/x + 21/x + 31/x + ... + n1/x ⎤
Now, 
lim ( A1 A2 ... An ) = (–1)
n–m+1 61. lim ⎢ ⎥
x → am− x →∞ ⎢
⎣ n ⎥⎦

and,  lim ( A1 A2 ... An ) = (–1)n – m
x → am+
8.34  Chapter 8

n
⎡ ⎛ a ⎞ n +1 ⎤ ⎡ ⎛ 1 ⎞ n +1 ⎤
⎡1y + 2 y + 3 y + ... + n y ⎤ y b n ⎢1 − ⎜ ⎟ ⎥ 2n ⎢1 − ⎜ ⎟ ⎥
= lim ⎢

y→0 ⎢ n
⎥ ⎢⎣ ⎝ b ⎠ ⎥⎦ ⎢⎣ ⎝ 2 ⎠ ⎥⎦
⎣ ⎥⎦
= =
a 1
lim
n ⎡1 + 2 + 3 + ... + n
y y y

−1⎥
y
1− 1−
y ⎢⎣ n ⎦ b 2
= e
y→0

⎡1y + 2 y + 3 y + ... + n y − n ⎤ 2n +1 ( 2n +1 − 1)
lim
y→0 ⎢
y

= = (2n + 1 – 1)
=
e ⎣ ⎦
2n +1
⎡ (1y −1) ( 2 y −1) ( 3 y −1) ( n y −1) ⎤
lim
y→0 ⎢
+ + + ...+ ⎥
The correct option is (B)
⎣ y y y y ⎦
=
e
= e (log 1 + log 2 + log 3 + … + log n) log(1 + x + x 2 ) + log(1 − x + x 2 )
log (1 ⋅ 2 ⋅ 3 …. n) 65. lim
=e = n! x→0 sec x − cos x
The correct option is (A)
log ⎡⎣(1 + x 2 ) 2 − x 2 ⎤⎦
62. lim (1 + x) (1 + x2) (1 + x4) … (1 + x2n) = lim

n→∞ x→0 (1 − cos 2 x ) cos x
(1 − x )(1 + x )(1 + x 2 )(1 + x 4 )...(1 + x 2 n )
= lim

n→∞ 1− x log(1 + x 2 + x 4 ) ⎛0 ⎞
= lim
⎜⎝ form⎟⎠
2
(1 − x )(1 + x )(1 + x )...(1 + x ) 2 4 2n x→0 sin x tan x  0
= lim

n→∞ 1− x log(1 + x 2 (1 + x 2 )) 1

. . . = lim
⋅ x 2 (1 + x 2 ) ⋅
x→0 x 2 (1 + x 2 ) sin x tan x 2

. . . ⋅ ⋅x
x x

. . .
⎛ log(1 + x ) ⎞
1 − x 4n + 2 1 = 1. ⎜ as lim
= 1⎟
= lim
= for |x| < 1 ⎝ x→0 x ⎠
n→∞ 1− x 1− x

The correct option is (A)
The correct option is (B)
63. Case I:  n is a positive integer 4 3
n5 + 2 − n2 + 1
x n
nx n −1 66. lim
n→∞ 5 2
lim = lim n4 + 2 − n3 + 1
x →∞ ex x →∞ ex
n ( n − 1) x n − 2 n! 2 1
n5 4 4 1 +
− n2 3 3 1 + 2
HINTS AND EXPLANATIONS


= lim = … = lim n 5
n
x →∞ ex x →∞ e x = lim

n→∞ 4 5 2 3 2 1
 [Using L’Hospital’s rule repeatedly] n 5 1+ 4 − n 2 1+ 3
n n
=0
Case II:  n is a negative integer. n5 4 4 2 n2 3 1
n −m 32
1+ 5 − 3 2 3 1+ 2
x x n
= lim 4 5
n n n
lim = lim
x →∞ e x x →∞ e x n→∞ n
5 1+
2 n3 2 2 1
32
− 1+ 3
[Putting n = – m, where m is a positive integer] n n 4 n3 2 n
1 1
[Dividing the numerator and denominator by the highest
= lim m x = =0
x →∞ x e ∞ power n3/2]
Case III:  n = 0
1 4 2 1 1
xn 1 1 1+ 5 − 5 6 3 1+ 2
lim x = lim x = =0 n14
n n n 0−0
x →∞ e x →∞ e ∞ = lim
= = 0.
n→∞ 1 5 2 2 1 0 −1
xn 1 + − 1 +
Hence, lim x = 0 for all values of n. n7 10 n4 n3
x →∞ e
The correct option is (B)
The correct option is (B)
64. x2 + 4x + 5 = (x + 2)2 + 1 ≥ 1.
So, a = 1 x 3
z 2 − ( z − x )2
67. lim
1 − cos 2θ 2 sin 2 θ x→0 3
( 8 xz − 4 x 2 + 3 8 xz ) 4
Also, b = lim = lim =2
θ →0 θ 2 θ →0 θ 2
3
n x 2 xz − x 2
∑ ar ⋅ bn − r = b + ab + a b + … + a = lim
n n–1 2 n–2 n


r=0
x→0 ( 3 x 3
8z − 4 x + 3 8z 3
x )4
Limits  8.35

x4 3 3
2z − x
3
2z 1 − cos z ( ay 2 + by + c) 2
= lim
= 4
= lim
⋅ lim
x→0
x 4 3 ⎡⎣ 3 8 z − 4 x + 3 8 z ⎤⎦
4
⎡2 3 8z ⎤ z2 y →α y 2 ( y − α )2
z→ 0
⎣ ⎦
[putting cx2 + bx+ a = z and x = 1/y]

1
=
1 a2 ( y − α )2 ( y − β )2
223 3 ⋅ z =
lim
2 y→ α y 2 ( y − α )2

The correct option is (B)
[If α, β are roots of ax2 + bx+ c = 0 then ax2 + bx+ c = a

A h 2 hr − h2 (x – α) (x – β)]
68. lim = lim
h→0 +
P 3 h→0 + 3 a 2 (α − β ) 2 a2 ⎡
8 ⎡ 2hr − h2 + 2hr ⎤ =
= (α + β ) 2 − 4αβ ⎤⎦
⎣⎢ ⎦⎥ 2α 2
2α 2 ⎣

h ⋅ h 2r − h a ⎛ b 2 4c ⎞
2
b 2 − 4 ac

= lim =
⎜ − =
h→0 3 2α 2 ⎝ a 2 a ⎟⎠ 2α 2
8⋅ h ⋅ h ⎡⎣ 2r − h + 2r ⎤⎦

The correct option is (A)
2r − h

= lim 3 1 − cos x
h→0
8 ⎡⎣ 2r − h + 2r ⎤⎦ 72. lim
x→ 0 x
2r 1 1 − cos x 1

= = = lim

( )
3
128r x→ 0 x
8 2 2r 1 + cos x

The correct option is (B)
2 sin x ( 2) ⋅ 1

69. lim
(
cos x + π
6 = lim ) 2
cos π ( +z ) = lim

x→ 0 2(x )
2 1 + cos x
x →π /3 (1 − 2 cos x ) 2/3
( )
z→ 0 ⎡ 2/3
1 − 2 cos π + z ⎤
Now, we have,
⎣ 3 ⎦
[putting x –π/3 = z] LHL = lim

−1 sin 2

x

1 ( )
x→ 0 2 x 2

1 + cos x
− sin z
= lim
−1
(1 − cos z + )
z→ 0 2/3 1 1
3 sin z
=− ⋅ =

HINTS AND EXPLANATIONS


2 2 2

⎛ z⎞ ⎛ z⎞
−2 sin ⎜ ⎟ cos ⎜ ⎟ RHL = lim

1 sin 2
x

1 ( )
⎝ 2⎠ ⎝ 2⎠ x→ 0 2 x +

= lim
2/3 2/3 2 1 + cos x
z→ 0 ⎡ ⎛ z⎞⎤ ⎡ ⎛ z⎞ ⎛ z⎞⎤ 1 1 1
⎢ 2 sin ⎜⎝ 2 ⎟⎠ ⎥ ⎢sin ⎜⎝ 2 ⎟⎠ + 3 cos ⎜⎝ 2 ⎟⎠ ⎥
= ⋅ =
⎣ ⎦ ⎣ ⎦ 2 2 2
1/3
Hence, limit does not exist.
⎡ ⎛ z⎞⎤ ⎛ z⎞
−21/3 ⎢sin ⎜ ⎟ ⎥ cos ⎜ ⎟
The correct option is (C)
⎣ ⎝ 2⎠ ⎦ ⎝ 2⎠ −21/3 ⋅ 0 ⋅1
= lim
= =0
( )
2/3 2/3
z→ 0 ⎡
⎛ z⎞ ⎛ z⎞⎤ 3 x + 7 − 3 2x − 3
⎢sin ⎜⎝ 2 ⎟⎠ + 3 cos ⎜⎝ 2 ⎟⎠ ⎥ 73. lim
x→ 2 3 x + 6 − 2 3 3x − 5
⎣ ⎦

The correct option is (C) ( x + 7) − 9( 2 x − 3) ⎡ a2 − b2 ⎤
= lim
× ⎢ using a − b = ⎥
ln ( 2 − cos 2 x ) ln {1 + (1 − cos 2 x )} x→ 2 x + 7 + 3 2x − 3 ⎢⎣ a + b ⎥⎦
70. lim 2
= lim 2
x→ 0 ln (sin 3 x + 1) x→ 0 ln (1 + sin 3 x ) ( x + 6) 2/3 + 2( x + 6)1/3 (3 x − 5)1/3 + 4(3 x − 5) 2/3
2
1 − cos 2 x 2x 2 ( x + 6) − 8 (3 x − 5)

= lim = lim =
x→ 0 (sin 3 x ) 2 x→ 0 (3 x ) 2 9
⎡ a3 − b3 ⎤

The correct option is (A)
 ⎢ using a − b = 2 ⎥
⎢⎣ a + ab + b 2 ⎥⎦
1 − cos (cx 2 + bx + a) −17 ( x − 2)
71. lim = lim

x → 1/α (1 − xα ) 2 x→ z x + 7 + 3 2x − 3
1 − cos (cx 2 + bx + a) (cx 2 + bx + a) 2 ( x + 6) 2/3 + 2( x + 6)1/3 (3 x − 5)1/3 + 4(3 x − 5) 2/3
= lim
⋅ ×

x → 1/α (cx 2 + bx + a) 2 (1 − xα ) 2 −23 ( x − 2)
8.36  Chapter 8


Now, we have,
82/3 + 2.81/3 + 4
−17 17 12 34 1
=
⋅ = ⋅ = ln e − ln (1 + y )
9+3 1 − 23 6 23 23 1 ⎡ e ⎤ y
lim ln ⎢ ⎥ = lim

The correct option is (B) y→ 0 y ⎢⎣ (1 + y )1/ y ⎥⎦ y→ 0 y
( 2m + x )1/m − ( 2n + x )1/n y − ln (1 + y )
74. lim = lim
x→ 0 x y→ 0 y2
( 2m + x )1/m − 2 ( 2n + x )1/n − 2 ⎛ y 2 y3 y 4 ⎞
= lim
− lim y−⎜y− + − + ...⎟
x→ 0 x x→ 0 x ⎝ 2 3 4 ⎠
a−2 b−2
= lim
= lim m
− lim n y→ 0 y2
a → 2 a − 2m b → 2 b − 2n


 [Putting 2m+ x = am and 2n+ x = bn] 1 y y2 1

= lim − + − ... =
y→ 0 2 3 4 2
1 1
=
− Hence, the required limit is e1/2.

m 2m − 1 n 2n − 1

The correct option is (C)
The correct option is (C)

(cos θ ) x − (sin θ ) x − cos 2θ ⎡ a sin x ⎤ ⎡ b tan x ⎤


78. lim ⎢ +
75. lim
x→ 4 x−4 x→ 0 ⎣ x ⎥⎦ ⎢⎣ x ⎥⎦

(cos θ ) y + 4 − (sin θ ) y + 4 − (cos 4 θ − sin 4 θ ) ⎡ sin x sin x


= lim
⎢ x → 0, x → 1 but x < 1
as
y→ 0 y ⎣
[Putting x – 4 = y and cos 2θ = cos4 θ – sin4 q]
tan x tan x ⎤

 while → 1 but > 1⎥
⎡ (cos θ ) − 1⎤ ⎡ (sin θ ) − 1⎤
y y x x ⎦
= lim cos 4 θ ⎢
4
⎥ − sin θ ⎢ ⎥ = (a – 1) + b

y→ 0 ⎢⎣ y ⎥⎦ ⎢⎣ y ⎥⎦
=a+b–1

= cos4 q lncos θ – sin4 q lnsin θ


The correct option is (B)

The correct option is (A)
[ x ] + [2 x ] + [3 x ] + ... + [nx ]
1/ x 1 ⎛ x − 1 + cos x ⎞ 79. Let f (x) =
⎛ x − 1 + cos x ⎞ ln ⎜
x ⎝ x
⎟⎠ 1 + 2 + 3 + ... + n
76. lim ⎜ ⎟⎠ = lim e
x→ 0 ⎝ x x→ 0
Now, we have,
x + 2 x + 3 x + ... + nx
HINTS AND EXPLANATIONS

1 ⎛ 1 − cos x ⎞ f (x) ≤
=x

Now,  lim ln ⎜1 − ⎟⎠ 1 + 2 + 3 + ... + n
x→ 0 x ⎝ x
( x − 1) + ( 2 x − 1) + (3 x − 1) + ... + ( nx − 1)
⎛ cos x − 1⎞ and,  f (x) >

ln ⎜1 + ⎟⎠ 1 + 2 + 3 + ... + n
cos x − 1 ⎝ x
= lim
⋅ x Σn − n 2
x→ 0 x2 cos x − 1 =
=x−
Σn n +1
x

 [∵ x − 1 ≤ [ x ] < x ∀x ∈ R]
−2 sin 2 ( x/2) ln (1 + z ) −1
= lim
⋅ lim = Thus, we have,
x→ 0 4( x/2) 2 z→ 0 z 2
2
⎡ cos x − 1 x– < f (x) ≤ x
n +1

 ⎢ Putting z = , we can see that
⎣ x Now, we have,
cos x − 1 ⎛ cos x − 1⎞ −1 ⎤ 2

 lim = lim x ⎜ ⎟ = 0 ⋅ = 0⎥ lim x − = x and lim x = x
x→ 0 x n→ 0 ⎝ x2 ⎠ 2 ⎦ n→∞ n +1 n→∞

Hence, by Sandwich Theorem, we have


Hence, the required limit is e–1/2.


The correct option is (B) lim f ( x ) = x
n→∞
x 1/ y
The correct option is (A)
⎡ e ⎤ ⎡ e ⎤
77. lim ⎢
( )
⎥ = lim ⎢ ⎥ 1 ⎛ 1
1 1 ⎞
⎣( ) ⎦ y → 0 ⎢⎣ (1 + y ) ⎥⎦
x
x →∞ ⎢ 1 + 1/ x ⎥ 1/ y −
80. lim n2 x1/n − x n + 1 = lim n2 ⋅ x n + 1 ⎜⎝ x n n + 1 − 1⎟⎠
n→∞ n→∞
1 ⎡
( )
e ⎤
ln ⎢ ⎥ 1 1
y ⎢⎣ (1 + y )1/ y ⎥⎦

= lim e
= lim x n + 1 x n ( n + 1) − 1 n2
y→ 0 n→∞
Limits  8.37

1 83. We have,
1
n +1 x n ( n + 1)
−1 n2
cos x − (cos x )cos x

= nlim x ⋅ ⋅ lim
→∞ 1
n ( n + 1) x → 0 1 − cos x + ln (cos x )
n ( n + 1)


= 1 · lnx · 1 = lnx ⎡ 1 − (cos x )cos x −1 ⎤
= lim cos x ⎢


The correct option is (C) x→ 0 ⎢⎣1 − cos x + ln (cos x ) ⎥⎦

81. We have, 1 − (1 + t )t
= lim
 [Putting cos x – 1 = t]
1/ x t→ 0 ln (1 + t ) − t
⎡ f ( x) ⎤ 3
lim ⎢1 + x + =e
x→ 0 ⎣ x ⎥⎦ t 3 (t − 1) t 4 (t − 1) (t − 2)
t2 + + + ...
ln [1 + x + g ( x )]
= lim
2! 3! =2
⎡ f ( x) ⎤
⇒ lim e x = e3 ⎢ Putting x = g ( x )⎥
t→ 0 t 2
t 3
t 4
x→ 0 ⎣ ⎦ − + − ...
2 3 4
ln [1 + x + g ( x )]
⇒ lim
=3
The correct option is (C)
x→ 0 x
Since, the denominator approaches zero, the numerator 84. We have,
should also approach zero for a finite limit to exist. (tan x ) tan x − tan x
lim
Thus, we have, x → π /4 ln (tan x ) − tan x + 1
lim g ( x ) = 0
x→ 0 tt − t ⎛ 0⎞
= lim
⎜ ⎟ [Putting tan x = t]

Now, using L’Hospital’s rule, the above equation reduces to t → 1 ln t − t + 1 ⎝ 0 ⎠

1 + g ′( x )
lim =3 t t (1 + ln t ) − 1 ⎛ 0 ⎞
x→ 0 1 + x + g( x) = lim

1 ⎜⎝ ⎟⎠
t→ 1
−1 0
i.e., 1 + g′(0) = 3  ⇒  g′(0) = 2

t

Hence, we have,
⎛ 1⎞
⎡ f ( x) ⎤
1/ x ln [1 + g ( x )] t t (1 + ln t ) 2 + t t ⎜ ⎟
lim ⎢1 + = lim e x ⎝t⎠ 1+1
x→ 0 ⎣ x ⎥⎦ x→ 0 = lim

t→ 1 −1
=
−1
= –2
g ′( x )
t2
= lim e
1 + g( x)
= e g ′( 0 )

HINTS AND EXPLANATIONS


x→ 0
The correct option is (A)
= e2


The correct option is (B) 85. We have,

( )
2 2 2 cos 2 x
lim 11/cos x
+ 21/cos x
+ ... + n1/cos x
x → π /2
x x
82. Let y = x + =x+
( ) ⎡ 1 ⎤
1/n
x y = lim 1y + 2 y + ... + n y
x+  ⎢ Putting cos 2 x = y ⎥
x
y→∞ ⎣ ⎦
x+ 1/ y
... ∞ ⎡⎛ 1 ⎞ y ⎛ 2 ⎞ y ⎛ n⎞ ⎤
y
= lim n ⎢⎜ ⎟ + ⎜ ⎟ + ... + ⎜ ⎟ ⎥

i.e.,  y2 – xy–
x =0 y →∞ ⎢ ⎝ n ⎠ ⎝ n⎠ ⎝ n⎠ ⎥
⎣ ⎦
x ± x2 + 4 x = n (0 + 0 + … + 1) = n

i.e.,  y =

2
The correct option is (A)

We can see y is a positive quantity for positive x, therefore
n
⎛ 3⎞
y=

x + x2 + 4 x 86. lim
n→∞
∑ cot −1 ⎜⎝ r 2 + 4 ⎟⎠
2 r =1

Hence, the required limit is n ⎡ ( r − 1/2) ( r + 1/2) + 1⎤
x 2x
= lim

n→∞
∑ cot −1 ⎢⎢ ( r + 1/2) − ( r − 1/2) ⎥⎥
= lim = lim
r =1 ⎣ ⎦
x →∞ y x →∞
x + x2 + 4 x ⎛ n
1⎞ ⎛ 1⎞
= lim ∑ cot −1 ⎜ r − ⎟ − cot −1 ⎜ r + ⎟

n→∞ ⎝ 2 ⎠ ⎝ 2⎠
2 2 r =1
= lim
= =1
x →∞
1 + 1 + 4x −3/2 1+1 ⎡ −1 ⎛ ab + 1⎞ −1 −1 ⎤

 ⎢∵cot ⎜
⎝ b − a ⎟⎠ = cot a − cot b ⎥

The correct option is (A) ⎣ ⎦
8.38  Chapter 8

89. We know,

= lim ⎢cot −1 ⎛⎜ ⎞⎟ − cot −1 ⎛⎜ ⎞⎟ + cot −1 ⎛⎜ ⎞⎟ − cot −1 ⎛⎜ ⎞⎟ +
1 3 3 5
1⎛ 1⎞
n→∞ ⎣ ⎝ 2⎠ ⎝ 2⎠ ⎝ 2⎠ ⎝ 2⎠ n
1⎜⎝1 − n ⎟⎠
∑ r = 2 2 i.e., sum of n terms of G.P.
⎛ 1⎞ ⎛ 1⎞ ⎤

 ... + cot −1 ⎜ n − ⎟ − cot −1 ⎜ n + ⎟ r =12 1
⎝ 2⎠ ⎝ 2 ⎠ ⎥⎦ 1−
2
⎡ 1 ⎤ which tends to one as n → ∞ but always remains less than
= lim ⎢cot −1 ⎛⎜ ⎞⎟ − cot −1 ⎛⎜ n + ⎞⎟ ⎥
1
one.
n→∞ ⎣ ⎝ 2⎠ ⎝ 2⎠ ⎦
⎡ n 1⎤
⎛ 1⎞ Thus, ⎢ ∑ r ⎥ → 0 as n → ∞

= cot–1 ⎜⎝ 2 ⎟⎠ – 0 = tan–1 2
⎢⎣ r = 1 2 ⎥⎦

The correct option is (B) ⎡ n 1⎤

lim ⎢ ∑ r ⎥ = 0
⎛ π⎞ n→∞ ⎢
87. Here, sin x + cos x = 2 sin ⎜ x + ⎟ ⎣ r = 1 2 ⎥⎦
⎝ 4⎠

The correct option is (A)
5π ⎛ π⎞
For x →
+ h, 2 sin ⎜ x + ⎟ → − 2 , 90. Here, 0 < cos x < 1; if 0 – h < x < 0 + h
4 ⎝ 4⎠
∴ [cos x] = 0

But greater than – 2
Hence, lim | x | [cos x ]

x →0
⎡ ⎛ π⎞⎤
∴ ⎢ 2 sin ⎜ x + ⎟ ⎥ = –2
(1)
⎣ ⎝ 4⎠⎦ = lim | x |0 = lim 1 = 1

x →0 x→0
5π ⎛ π⎞
The correct option is (B)
Also, for x →
− h, 2 sin ⎜ x + ⎟ → − 2 , but greater
4 ⎝ 4⎠
than – 2 an
91. We have, an – 1 + 1 = (1)
⎡ ⎛ π⎞⎤ n
∴ ⎢ 2 sin ⎜ x + ⎟ ⎥ = –2
(2)
⎣ ⎝ 4⎠⎦ ⎛ a + 1⎞ ⎛ a + 1⎞ ⎛ a + 1⎞

From (1) and (2), we get ∴
lim ⎜ 1 ⎟ ⎜ 2 ⎟ ... ⎜ n ⎟
n→∞ ⎝ a1 ⎠ ⎝ a2 ⎠ ⎝ an ⎠
lim [sin x + cos x ]
x → 5π /4 ⎛a ⎞ ⎛a ⎞ ⎛a ⎞ ⎛ a ⎞ 1
= lim ⎜ 2 ⎟ ⎜ 3 ⎟ ⎜ 4 ⎟ ... ⎜ n +1 ⎟ ⋅

= lim − 2 = –2
n→∞ ⎝ 2 ⎠ ⎝ 3 ⎠ ⎝ 4 ⎠ ⎝ n + 1⎠ a ⋅ a ... a
1 2 n
x→ 5π /4
an +1 1 + an
HINTS AND EXPLANATIONS


The correct option is (B) = lim
= lim  [using (1)]
88. lim lim n→∞ ( n + 1)! n→∞ n!
m →∞ n→∞
⎛1 a ⎞
⎧⎪1 + n 1n + 2n + n 2n + 3n + ... + n ( m − 1) n + m n ⎫⎪ = lim ⎜ + n ⎟

n→∞ ⎝ n! n! ⎠
⎨ ⎬
⎪⎩ m2 ⎪⎭ ⎡1 1 a ⎤
= lim ⎢ +
+ n −1 ⎥  [using (1)]
= lim
n→∞ ⎣ n! ( n − 1)! ( n − 1)! ⎦
m→∞
⎡1 1 1 1 1 a⎤
⎧ n n n ⎫ = lim ⎢ +
+ + ... + + + 1 ⎥
⎪ ⎛ 1⎞ ⎛ 2⎞ ⎛ ( m − 1) ⎞ n→∞ ⎣ n! ( n − 1)! ( n − 2)! 2! 1! 1! ⎦
1 + 2 n ⎜ ⎟ + 1 + 3n ⎜ ⎟ + 1 + ... + m n ⎜ ⎟ + 1⎪
⎪ ⎝ 2⎠ ⎝ 3⎠ ⎝ m ⎠ ⎪
⎨ lim ⎬ ⎡1 1 1 1 1 1⎤
⎪ n→∞ m 2
⎪ = lim ⎢ +
+ + ... + + + ⎥
n→∞ ⎣ n! ( n − 1)! ( n − 2)! ( 2)! 1! 1⎦
⎪ ⎪
⎩ ⎭ [  a1 = 1; given]

1 + 2 + 3 + ... + m
= lim ⎡ 1 1 1 ⎤
m→∞ m2 = e ⎢as, e = 1 + + + + ... ∞ ⎥

⎣ 1! 2! 3! ⎦
⎛ ⎛ 1⎞ n
⎛ 2⎞
n
→ → ∞
The correct option is (B)

⎜ ⎜ ⎟ 0 as n ; ⎜⎝ ⎟⎠ → 0 as n → ∞; ... ;
⎝ ⎝ 2⎠ 3 n
⎡ ⎛ 1⎞ ⎛ 1 ⎞⎤
92. lim n − n ⎢( n + 1) ⎜ n + ⎟ ... ⎜ n + n −1 ⎟ ⎥
2

⎛ m − 1⎞
n ⎞
n→∞ ⎣ ⎝ 2 ⎠ ⎝ 2 ⎠⎦

 ⎜⎝ ⎟⎠ → 0 as n → ∞⎟
m ⎠ n
⎡ ⎛ 1⎞ ⎛ 1 ⎞⎤
m ( m + 1)
1⎛ 1⎞ 1 ⎢ ( n + 1) ⎜⎝ n + ⎟ ... ⎜ n + n −1 ⎟⎠ ⎥
2⎠ ⎝
= lim
= lim ⎜1 + ⎟ = = lim ⎢

2 ⎥
2m 2
m →∞ m →∞ 2 ⎝ m⎠ 2 n→∞ ⎢ nn ⎥

The correct option is (D) ⎢ ⎥
⎣ ⎦
Limits  8.39

n n 95. We know n ≤ [x] < n + 1  ⇒ [x] = n


⎛ 1⎞ ⎛ 1 ⎞
n + n −1
n + 1⎞ ⎜ n + 2 ⎟
n
⎛ ⎜ 2 ⎟ n sin x
= lim ⎜
⎟ ⋅⎜ ⎟ ... ⎜ ⎟ Here,
→ n as x → 0 but less than n
n→∞ ⎝ n ⎠ n n x
⎜ ⎟ ⎜ ⎟
⎝ ⎠ ⎝ ⎠
n tan x
n n n Also,
→ n as x → 0 but more than n
= lim ⎛⎜1 + ⎞⎟ ⋅ ⎛⎜1 + ⎞⎟ ... ⎛⎜1 +
1 1 1 ⎞
(1∞ form) x
n→∞ ⎝ n −1 ⎟
n⎠ ⎝ 2n ⎠ ⎝ 2 n ⎠
2n 2n −1. n ⎡ n sin x ⎤
n Thus,  n – 1 ≤ ⎢
⎥ < n as x → 0
⎛ 1⎞ ⎛ 1⎞ 2 ⎛ 1 ⎞ 2n − 1 ⎣ x ⎦
= lim ⎜1 +
⎟⎠ ⋅ ⎜⎝1 + ⎟⎠ ... ⎜⎝1 + n −1 ⎟⎠
n→∞ ⎝ n 2n 2 n
⎡ n sin x ⎤
⇒ ⎢
⎥ =n–1
= e1 · e1/2 · e1/4 … e1/2n–1
⎣ x ⎦
⎧⎪ ⎛ 1⎞
an ⎫⎪
 … ⎨ using; lim ⎜1 + ⎟ = e a ⎬ ⎡ n tan x ⎤
n→∞ ⎝ n⎠
⎩⎪ ⎭⎪ Again,  n ≤ ⎢
⎥ < n + 1 as x → 0
1 ⎣ x ⎦
1
1−
= e(1 + 1/2 + 1/4 + …) = e 2 = e2
⎡ n tan x ⎤
⇒ ⎢
⎥ =n

The correct option is (B) ⎣ x ⎦
x y − yx ⎛0 ⎞ ⎛ ⎡ n sin x ⎤ ⎡ n tan x ⎤⎞
93. lim ⎜⎝ form⎟⎠ Thus, 
lim ⎜ ⎢ +
xx − y y 
x→ y 0 x →∞⎝ ⎣ x ⎥⎦ ⎢⎣ x ⎥⎦⎟⎠
yx y −1 − y x log y = (n – 1) + (n) = (2n – 1)
= lim x
 (applying L-Hospital’s rule)
x → y x (1 + x log x ) − 0 The correct option is (C)
96. We know,
yx y −1 − y x log y
= lim
x
x→ y x x (1 + x log x ) → 1 , as x → 0, but less than 1
sin x
y ⋅ y y −1 − y y log y
=
x
y y (log y + 1)
Also, → 1 , as x → 0 but less than 1
tan x
1 − log y
=
x2
1 + log y
Thus, → 1 as x → 0, but less than 1.
sin x tan x

HINTS AND EXPLANATIONS


The correct option is (A)
⎡ x2 ⎤ x2
n ⎛ 3 1⎞
Hence, lim ⎢ ⎥ = 0 as 0 ≤ < 1 as x → 0
94. lim ∑ cot −1 ⎜ r − r + r ⎟ x → 0 ⎢ sin x tan x ⎥
⎣ ⎦ sin x tan x
n→∞
r =1 ⎜⎝ ⎟⎠
2
The correct option is (A)
n
⎛ 2r ⎞
= lim ∑ tan −1 ⎜
4⎟ cos 2 (1 − cos 2 (1 − cos 2 (1 ... cos 2 θ ))
n→∞
r =1
⎝ 1 − r 2
+ r ⎠ 97. lim
θ→ 0 ⎛ π ( θ + 4 − 2⎞

n
2r ⎞ sin ⎜ ⎟
= lim ∑ tan −1 ⎜
2 2 ⎟ ⎝ θ ⎠
n→∞
r =1 ⎝ 1 − ( r − r ) ( r + r ) ⎠
cos 2 (sin 2 (sin 2 ... (sin 2 θ ))
⎛ (r 2 + r) − (r 2 − r) ⎞
n = lim

= lim ∑ tan −1 ⎜
θ→ 0 ⎛ π ( θ + 4 − 2⎞
⎟ sin ⎜ ⎟
n→∞
r =1 ⎝ 1 − (r 2 + r) (r 2 − r) ⎠ ⎝ θ ⎠
n
= lim

n→∞
∑ ⎡⎣tan −1( r 2 + r ) − tan −1( r 2 − r )⎤⎦ = lim

cos 2 (sin 2 (sin 2 ... (sin 2 θ ))
=
cos 2 0
= 2
r =1 θ→ 0 ⎛ ⎞ π
θ sin
= lim [tan–1 2 – tan–1 0) + (tan–1 6 – tan–1 2) + (tan–1 12 –
sin ⎜ π lim ⎟
( )
4
n→∞ ⎜⎝ θ → 0 θ θ + 4 + 2 ⎟⎠
tan–1 6) + … + {tan–1 (n2 + n) – tan–1 (n2 – n)}]

The correct option is (C)
= lim {tan–1 (n2 + n) – tan–1 (0)}

n→∞
π tan x − sin{tan −1(tan x )}
= tan–1 (∞) – tan–1 (0) =
98. lim f ( x ) = lim
2 x→
π
− x→
π
− tan x + cos 2 (tan x )
2 2
n ⎛ 3 1⎞ π
−r+

lim
n→∞
∑ cot −1 ⎜⎜ r r⎟ =
⎟⎠ 2
= lim
tan x − sin x
r =1 2 ⎝ π tan x + cos 2 (tan x )
x→ −
2

The correct option is (C)
8.40  Chapter 8

[ tan–1(tan x) = x, where x < π/2]


π t

sin π t ⎡ t 3t 2 3 ⎤
1−
sin x ⎢1 + 24 − 1 − 8 + terms containing t , t , etc.⎥
tan x ⎣ ⎦

= lim
π cos 2 (tan x ) 3 9 4
x→
2 1+ = ·(3 log 4 − 3) = log
tan x π π e
The correct option is (A)
1− 0

= =1
1+ 0 101. (1 − x ) (1 − x 2 ) ... (1 − x 2 n )
lim
x →1 [(1 − x ) (1 − x 2 ) ... (1 − x n )]2
⎡ cos 2 (tan x ) finite quantity ⎤

 ⎢∵ lim = = 0⎥
π
⎢ x→ − tan x ∞ ⎥ (1 − x )(1 − x 2 )…(1 − x n )(1 − x n +1 )(1 − x n + 2 )…(1 − x 2 n )
⎣ 2 ⎦ lim
x →1 [(1 − x )(1 − x 2 )…(1 − x n )]2
sin x
1+ (1 − x n +1 )(1 − x n + 2 )...(1 − x 2 n )

Similarly,  lim f ( x ) = lim tan x = lim
π π cos 2 (tan x )
x →1 [(1 − x )(1 − x 2 ) (1 − x 3 ) ... (1 − x n )]
x→ + x→ +
2 2 1+
tan x (1 − x n +1 )(1 − x n + 2 )...(1 − x 2 n )
–1 = lim
[ tan (tan x) = x – π, if x > π/2]
x →1 (1 − x )(1 − x ) ... (1 − x )
1+ 0 (1 − x ) (1 − x ) ... (1 − x )

= =1  ×
1+ 0 (1 − x ) (1 − x 2 ) ... (1 − x n )

lim f ( x ) = 1
π
x→ 1 1 1 ( 2n)!
2 = (n + 1) (n + 2) … 2n · · ... =

The correct option is (A) 1 2 n ( n!) 2
The correct option is (C)
99. Put c = a1/4 and z = x1/4, we get the function whose limit is
π
required, as 102. Since maximum value of cos–1x =
8 k
2
⎧⎡ 2 −1 ⎫ kπ
2 −1
⎪ ⎢⎛ c + z ⎞ 2cz ⎤ 1
log c4 ⎪ ∑ cos −1 α r = is possible if and only if each
⎨ ⎜ ⎟ − ⎥ − 2 2
4

⎬ r =1 2
⎪ ⎢⎣⎝ c − z ⎠ z − cz + c z − c ⎥
3 2 2 3
⎦ ⎪ π
⎩ ⎭ cos–1 a r = ⇒  a r = 0
2 
8
HINTS AND EXPLANATIONS

⎧⎡ c2 + z 2 ⎤
−1 ⎫ k
⎪ 2cz ⎪
∑ (α r )
r
= ⎨⎢
− ⎥ − c ⎬ ∴ θ = =0
⎪⎩ ⎢⎣ c − z ( z − c) ( z 2 + c 2 ) ⎥⎦ ⎪⎭ r =1
8
= (c – z – c) = z = x
8 2 (1 + x 2 )1/3 − (1 − 2 x )1/4
∴ lim

Hence, required limit as x → a = a2 x→θ x + x2

The correct option is (C) (1 + x 2 )1/3 − (1 − 2 x )1/4
= lim
(log (1 + x ) − log 2) (3.4 x −1 − 3 x )
x→ 0 x + x2
100. lim
x →1 {(7 + x )1/3 − (1 + 3 x )1/2} sin π x ⎛ 1 2 4 ⎞ ⎛ x 2 ⎞
⎜⎝1 + x + O ( x )⎟⎠ − ⎜⎝1 − + O ( x )⎟⎠
3 2
= lim
[log ( 2 + t ) − log 2] [3.4t − 3(t + 1) x→ 0 x (1 + x )
= lim 1/3 1/2
t →0 {(8 + t ) − ( 4 + 3t ) } sin π (t + 1)
1 1
 [By putting x = 1 + t] + x + O ( x2 )
1
= lim 2 3 =
⎛ t⎞ 1+ x 2
log ⎜1 + ⎟ (3 ( 4t − 1) − 3t ) x→0
⎝ 2⎠
= − lim [O (x2) means terms containing x2, x3, x4, …]
t →0 ⎡
3t ⎞ ⎤
1/3 1/2
⎛ t⎞ ⎛ The correct option is (C)
⎢ 2 ⎜1 + ⎟ − 2 ⎜1 + ⎟ ⎥ sin π t
⎢⎣ ⎝ 8⎠ ⎝ 4 ⎠ ⎥⎦
103.  ax2 + bx+ c = 0 has roots α and β, therefore
⎛ t⎞ a b 1 1
log ⎜1 + 2 + + c = 0 i.e., cx2 + bx+ a = 0 has roots and
⎟ α β
2 ⎠ ⎡ 3 ( 4 − 1) ⎤ x
t
1 ⎝ x
= − lim ⋅ ⋅⎢ − 3⎥ 1⎞ ⎛ 1⎞
t →0 π t ⎢⎣ t ⎛ b a⎞ ⎛
⎦⎥ ⇒ c ⎜ x 2 + + ⎟ = c⎜x − ⎟ ⎜x − ⎟
2 ⎝ cx c ⎠ ⎝ α⎠ ⎝ β⎠
Limits  8.41

⎡1 − cos (cx 2 + bx + a) ⎤
Now, lim1 ⎢ ⎥ c ⎛ 1 1⎞
x→ ⎢⎣ 2 (1 − α x )2 ⎥⎦ 1· ⎜ − ⎟
α 2 ⎝ α β⎠ c ⎛ 1 1⎞
= = −
⎧ 2 ⎛ cx 2 + bx + a ⎞ ⎫
1/ 2 −α 2α ⎜⎝ α β ⎟⎠

= lim ⎨
⎪ sin ⎜ ⎟⎪ The correct option is (A)
1
⎝ 2 ⎠⎬
x→ ⎪ ⎪
α ⎩ (1 − α x ) 2
⎭ 104. lim f ( x ) = lim {− h} cot {− h}
x → 0− h→0
⎛ cx + bx + a ⎞ 2
sin ⎜ ⎟ = lim (1 − h) cot (1 − h) = cot 1
= lim ⎝ 2 ⎠ h→0
1
x→
α 1 − αx tan 2 {h}
lim f ( x ) = lim
⎛c ⎛ 1⎞ ⎛ 1⎞⎞ x → 0+ h→0 h2 − [h]2
sin ⎜ ⎜ x − ⎟ ⎜ x − ⎟ ⎟
⎝2 ⎝ α⎠ ⎝ β⎠⎠ tan 2 h2
= lim = lim =1
1 ⎛ 1⎞ h2
x→
α −α ⎜ x − ⎟ h→0
⎝ α⎠
∴ lim f ( x ) does not exist,
x→0
⎛c ⎛ 1⎞ ⎛ 1⎞⎞ c⎛ 1⎞
sin ⎜ ⎜ x − ⎟ ⎜⎝ x − β ⎟⎠ ⎟ x− ⎟ The correct option is (D)
⎝2 ⎝ α⎠ ⎠ 2 ⎜⎝ β⎠
= lim · lim
1 c⎛ 1⎞ ⎛ 1⎞ 1 −α b
x→ x→
x a sin b x ⎛ sin x ⎞ ⎛ x ⎞
c
α ⎜ x − ⎟⎠ ⎜⎝ x − ⎟ α
= lim x a + b − c ⎜
2⎝ α β⎠ 105. lim ⎟
x→0 sin ( x ) c
x→0 ⎝ x ⎠ ⎝ sin ( x c ) ⎟⎠

The above limit is non-zero if a + b – c = 0


The correct option is (D)

Previous Year’s Questions


x
⎛ x 2 + 5 x + 3⎞
106. Key Idea: Limit of a function exists only, if
Now, the limit lim ⎜ 2 ⎟
x →∞ ⎝ x + x + 2 ⎠
LHL = RHL.

HINTS AND EXPLANATIONS


x
1 − cos 2 x 1 − 1 + 2 sin 2 x ⎛ 4x + 1 ⎞

Now, lim = lim = lim ⎜1 + 2 ⎟
x→0 2x x→0 2x x →∞ ⎝ x + x + 2⎠
( 4 x +1) x
2 sin x sin x
 = lim = lim ⎡ 1
( 4 x +1) ⎤ x 2 + x +2
⎛ 4x + 1 ⎞
= lim ⎢⎜1 + 2
x→0 2x x→0 x x2 + x + 2 ⎥

x →∞ ⎢⎝ x + x + 2⎠ ⎥
sin x ⎢⎣ ⎥⎦
Let f ( x ) =

x ⎛ 1⎞
⎜⎝ 4 + x ⎟⎠
sin( 0 − h) lim
LHL = lim x →∞ 1 2
1+ − 2
h→ 0 0−h = e x x

Now,
sin h
= lim = −1 = e4
h→ 0 − h
The correct option is (A)
sin( 0 + h)
RHL = lim x
0+h h→ 0 ⎛ x − 3⎞
and 108. The limit lim ⎜ ⎟
sin h x →∞ ⎝ x + 2 ⎠
= lim =1
h→ 0 h
x
∵ LHL ≠ RHL ⎡ 5 ⎤
= lim ⎢1 −
x →∞ ⎣ x + 2 ⎥⎦
sin x
∴ lim does not exist. ⎛ −5 x ⎞
x x→0
⎡ ⎛ 5 ⎞ ⎤ ⎜⎝ x + 2 ⎟⎠
1/ ⎜ ⎟

The correct option is (D) ⎢⎛ ⎛ −5 ⎞ ⎞ ⎝ x + 2 ⎠ ⎥
= lim ⎢⎜1 + ⎜ ⎟ ⎟ ⎥
107. Key Idea : lim (1 + λ x )1/ x = e λ
x →∞ ⎝
⎢ ⎝ x + 2 ⎠⎠ ⎥
x →∞ ⎣ ⎦
8.42  Chapter 8

111. By applying L’Hopital Rule, the given limit equals


⎛ 5 ⎞
lim ⎜ − 1+ 2 / x ⎠⎟
x →∞ ⎝
=e 1 1
+
= e −5 lim 3 + x 3 − x =2.
Alternative Method: x→0 1 3

⎛ x − 3⎞
x
The correct option is (C)
lim ⎜ ⎟
x →∞ ⎝ x + 2 ⎠
112. Applying L. Hospital’s Rule
x
⎛ 3⎞
⎜⎝1 − x ⎟⎠ f ( a) g ′( a) − g ( a) f ′( a)
lim =4
= lim x
x→ 2a g ′( a) − f ′ ( a)
x →∞ ⎛ 2⎞
⎜⎝1 + x ⎟⎠ k ( g ′( a) − ff ′( a))
=4
( g ′( a) − f ′( a))
e −3
== e −5 k = 4.
e2
The correct option is (A)

The correct option is (C)
2x
113. The limit lim ⎛⎜1 + + ⎞⎟
a b
xf ( 2) − 2 f ( x ) =
1 09. The limit lim x →∞ ⎝ x x2 ⎠
x→2 x−2
⎛ 1 ⎞
⎜ a b ⎟ × 2 x × ⎛⎜ + 2 ⎞⎟
a b
xf ( 2) − 2 f ( x ) + 2 f ( 2) − 2 f ( x )
= lim ⎛ a b ⎞⎜ + 2 ⎟
⎝x x ⎠
x→2 x−2 lim ⎜1 + + 2 ⎟ ⎝ x x ⎠ = e2a
x →∞ ⎝ x x ⎠

f ( 2)( x − 2) − 2{ f ( x ) − f ( 2)}
= lim ⇒ a = 1, b ∈ R
x→2 x−2 The correct option is (B)
f ( x ) − f ( 2)}
= f ( 2) − 2 lim 114. Let L =
x→2 x−2

= f ( 2) − 2 f ′( 2) = 4 − 2 × 4 = −4 ⎛ ( x − α )( x − β ) ⎞
2 sin 2 ⎜ a ⎟⎠
1 − cos a( x − α )( x − β ) ⎝ 2
Alternative Solution:
lim = lim
( x − α )2 ( x − α )2
HINTS AND EXPLANATIONS

x →α x →α
⎧ xf ( 2) − 2 f ( x ) ⎫
lim ⎨ ⎬
x→2 ⎩ x−2 ⎭ ⎛ ( x − α )( x − β ) ⎞
sin 2 ⎜ a ⎟⎠
2 ⎝ 2 a2 ( x − α )2 ( x − β )2
= lim{ f ( 2) − 2 f ′( x )} ( by L′ Hopital’s Rule) = lim × ×
2 2 2 2
x→2 x →α (x − α) a (x − α) (x − β) 4
=f (2) − 2f  ′(2) 4
=4 − 2 × 4 = − 4
a 2 (α − β ) 2
The correct option is (C) Then, the limit L =
.
2
110. The limit, by applying L’Hopital rule,
The correct option is (A)

⎛ π x⎞ 115. f (x) is a positive increasing function


tan ⎜ − ⎟ (1 − sin x )
⎝ 4 2⎠ ⇒ 0 < f (x) < f (2x) < f (3x)
lim
x →π / 2 ⎛ π x⎞
4 ⎜ − ⎟ (π − 2 x ) 2 f ( 2 x ) f (3 x )
⎝ 4 2⎠ ⇒ 0 <1< <
f ( x) f ( x)
( − cos x )
= lim
x →π / 2 8( −2)(π − 2 x ) f (2 x) f (3 x )
⇒ lim 1 ≤ lim ≤ lim
− sin x x →∞ x →∞ f ( x ) x →∞ f ( x )
= lim
x →π / 2 16( −2)
By sandwich theorem.
1 f (2 x)
= . ⇒ lim =1
32 x →∞ f ( x)

The correct option is (C)
The correct option is (D)
Limits  8.43

2
116. lim 2 sin ( x − 2)
2
1 ⎛ tan x ⎞
lim ⎜ ⎟
2⎝
x → 0+ x ⎠
x→2 x−2 120. We have p = e = e
2 | sin( x − 2) |
lim 1
x→2 x−2 ∴ log p =
2
R.H.L. = 2 , L.H.L. = − 2 The correct option is (D)

Limit does not exist. cot x − cos x
121. lim
π (π − 2 x )3

The correct option is (D) x→
2
π
(1 − cos 2 x ) Put, –x=t
117. lim (3 + cos x ) 2
x→0 x(tan 4 x ) t
tan t − sin t sin t . 2 sin 2
2 lim = lim 2t
⎛ sin x ⎞ 1 ⎛ 4 x ⎞
= lim ⎜ ⎟ ⋅ 4 ⎜ tan 4 x ⎟ ( 3 + cos x ) 8t 3 8t 3
t →0 t →0
x →0 ⎝ x ⎠ ⎝ ⎠
1
1 =
= 2 × 1 × × 1 × ( 3 + 1) = 2. 16
4
Hence, the correct option is (A)

The correct option is (C) 122. x = [x] + {x} [Integral + FRACTIONAL PART]
⎛ x 2 + f ( x) ⎞ ⎛ 1 ⎧1 ⎫ 2 ⎧2 ⎫ ⎞
118. Given that lim ⎜ ⎟ =3. lim x ⎜ − ⎨ ⎬ + − ⎨ ⎬ + …⎟
x→0 ⎝ x2 ⎠ x →0 ⎝ x ⎩ ⎭
x +
x ⎩ ⎭
x ⎠
Since limit exits, the expression x 2 + f ( x ) = ax 4 + bx 3 + 3 x 2 ⎛1 2 3 ⎞ ⎛ ⎧1⎫ ⎧2⎫ ⎧3⎫ ⎞
lim x ⎜ + + …⎟ − x ⎜ ⎨ ⎬ + ⎨ ⎬ + ⎨ ⎬ …⎟ ,
⇒ f ( x ) = ax 4 + bx 3 + 2 x 2 x→0 ⎝x x x ⎠
+
⎝⎩x⎭ ⎩x⎭ ⎩x⎭ ⎠
⇒ f ′( x ) = 4 ax 3 + 3bx 2 + 4 x
⎡ n( n + 1) ⎤
Also, f ′( x ) = 0 at x = 1, 2
⇒ lim x ⎢ 2 ⎥ − 0
x→0+
⎢ ⎥
⎣ x ⎦
1 n( n + 1) 15 × 16
⇒ a = , b = −2 ⇒ lim = = 120
2 2

HINTS AND EXPLANATIONS


2 x →0+

x4
⇒ f ( x) = − 2 x3 + 2 x 2
2
⇒ f ( x ) = 8 − 16 + 8 = 0.

The correct option is (A)
119. The value of the limit

2 sin 2 x × (3 + cos x ) 2 × 4
lim = =2.
x→0 ⎛ tan 4 x ⎞ 4
x×⎜ × 4 x
⎝ rx ⎟⎠


The correct option is (B)
This page is intentionally left blank.
CHAPTER
Differential Equations
9
LEARNING OBJECTIVES
After reading this chapter, you will be able to:
 Understand the concept of differential equation, the order  
Learn the formation of a differential equation, and
of differential equation and degree of a differential equation solution of first order and first degree differential equation
 Be familiar with linear and non-linear differential  
Know about initial value problems and solution by
equation and solutions of differential equation inspection

DIFFERENTIAL EQUATION Solution: (C)


We have,
An equation involving an independent variable, a ­dependent
variable and the derivatives of the dependent variable, is y = c1 cos (2x + c2) – (c3 + c4) a x + c + c6 sin (x – c7)
5

called a differential equation.


= c1 cos (2x + c2) – c8. ac . ax + c6 sin (x – c7)
5

Illustrations
where c3 + c4 = c8
dy
1. x + y = x3
dx ⇒ y = c1 cos (2x + c2) – c9 ax + c6 sin (x – c7),
d3 y d2 y dy where c9 = c8. ac
2. 3 + 2 2 + 6 + 7y = 0
5

dx dx dx
⎛ d2 y⎞ ⎛ dy ⎞
3 Since the above relation contains five arbitrary con-
3. ⎜ 2 ⎟ + 4 ⎜ ⎟ + 3 y = 0 stants, so the order of the differential equation satis-
⎝ dx ⎠ ⎝ dx ⎠
fying it is 5.
3/ 2
⎡ ⎛ dy ⎞ 2 ⎤ d3 y 2. The order of the differential equation whose general
4. ⎢1 + ⎜ ⎟ ⎥ = solution is given by y = (c1 + c2) cos (x + c3) – c4 e x + c 5

⎢⎣ ⎝ dx ⎠ ⎥⎦
3
dx where c1, c2, c3, c4, c5 are arbitrary constants, is
are some examples of differential equations. (A) 5 (B) 4 (C) 3 (D) 2
Order of a Differential Equation Solution: (C)
The order of highest derivative appearing in a differential We can write y = A cos (x + B) – cex
equation is called the order of the differential equation. where A = c1 + c2, B = c3 and C = c4 e c .
5

SOLVED EXAMPLES dy
= – A sin (x + B) – Cex
dx
1. The order of the differential equation whose general
solution is given by d2 y
= – A cos (x + B) – Cex
dx 2
y = c1 cos (2x + c2) – (c3 + c4) a x + c + c6 sin (x – c7) is
5

d2 y
⇒ 2
+ y = – 2cex
(A) 3 (B) 4 (C) 5 (D) 2 dx
9.2  Chapter 9

Substituting the value of m in (1),


d3 y dy d2 y
⇒ + = – 2cex = +y dy a
dx 3 dx dx 2 we get y= x +
dx dy
d3 y d2 y dy
⇒ 3
− + –y=0 dx
dx dx 2 dx 2
⎛ dy ⎞ dy
which is a differential equation of order 3. ⇒ x⎜ ⎟ − y + a = 0,
⎝ dx ⎠ dx
Degree of a Differential Equation which is a differential equation of degree 2.
The power of the highest order derivative appearing in a 5. The degree of the differential equation
differential equation, after it is made free from radicals and 3/ 5
⎛ d4 y⎞ d3 y d2 y dy
fraction, is called the degree of the differential equation. ⎜ 4⎟ −5 +6 −8 + 5 = 0 is
3 2
In the above illustrations, differential equation (1) ⎝ dx ⎠ dx dx dx
is of first order and first degree, differential equation (2) (A) 2 (B) 3 (C) 4 (D) 5
is of order 3 and degree 1, differential equation (3) is of
order 2 and degree 1 and differential equation (4) is of Solution: (B)
order 3 and degree 2, as after making it free from fractional We have,
3
⎡ ⎛ dy ⎞ 2 ⎤ ⎛ d4 y⎞
3/ 5
d3 y d2 y dy
exponent by squaring, it can be re-written as ⎢1 + ⎜ ⎟ ⎥ ⎜ 4⎟ = 5 −6 +8 −5
2 ⎢⎣ ⎝ dx ⎠ ⎥⎦ ⎝ dx ⎠ dx 3
dx 2 dx
⎛ d3 y ⎞
= ⎜ 3⎟ 3 5
⎝ dx ⎠ ⎛ d4 y⎞ ⎛ d3 y d2 y dy ⎞
⇒ ⎜ 4 ⎟ = ⎜ 5 3 − 6 2 + 8 dx − 5⎟
⎝ dx ⎠ ⎝ dx dx ⎠

SOLVED EXAMPLES
which is a differential equation of order 4 and degree
3. The degree of differential equation 3.

2 3
6. The order of the differential equation satisfying
⎛ dy ⎞ 1 ⎛ dy ⎞ 1 ⎛ dy ⎞
x = 1 + ⎜ ⎟ + ⎜ ⎟ + ⎜ ⎟ + ………… is
1 − x 4 + 1 − y 4 = a (x2 – y2). is
⎝ dx ⎠ 2 ! ⎝ dx ⎠ 3! ⎝ dx ⎠
(A) 1 (B) 2
(A)  Three (B)  One (C)  3 (D)  None of these
(C)  Not defined (D)  None of these
Solution: (A)
Solution: (B)
Put x2 = sin a, y2 = sin b.
⎛ dy ⎞
dy ⎜ ⎟ \ Given equation reduces to
x = e ⎝ dx ⎠ ⇒
= ln x
dx cos a + cos b = a (sin a – sin b)
\ order = 1 and degree = 1 ⎛ α + β⎞ ⎛ α − β⎞
⇒ 2 cos ⎜ ⎟ cos ⎜
⎝ 2 ⎠ ⎝ 2 ⎟⎠
4. The degree of the differential equation of all tangent
lines to the parabola y2 = 4ax is ⎛ α + β⎞ ⎛ α − β⎞
 =
2a cos ⎜ ⎟ ⋅ sin ⎜
(A) 1 (B) 2 ⎝ 2 ⎠ ⎝ 2 ⎟⎠
(C)  3 (D)  None of these
⎛ α − β⎞
Solution: (B) ⇒ cot ⎜ = a ⇒ a – b = 2 cot– 1 a
⎝ 2 ⎟⎠
The equation of any tangent to the parabola
a ⇒ sin– 1x2 – sin– 1y2 = 2 cot– 1 a
2
y = 4ax is y = mx + ,(1)
m Differentiating with respect to x, we get
where m is any arbitrary constant.
dy 1 1 dy
Differentiating with respect to x, we get =m ⋅ 2x − ⋅ 2y =0
dx 1− x 4
1− y 4 dx
Differential Equations  9.3

dy x 1 − y4 ERROR CHECK
⇒ = ,
dx y 1 − x4
A linear differential equation is always of the first degree
which is a differential equation of first order and first but every differential equation of the first degree need not
degree. be linear.

7. The degree of the differential equation


Illustration 5
4
d y 3
⎛ dy ⎞ ⎛ d y⎞4
3 + x ⎜ ⎟ = 4 log ⎜ 4 ⎟ is
dx ⎝ dx ⎠ ⎝ dx ⎠ d3 y d2 y dy
3
+2 2
+ + 3y2 = 0
dx dx dx
(A) 1 (B) 3
(C)  4 (D)  None of these is a differential equation of the first degree but not linear.

Solution: (D) Solution of a Differential Equation


Since, the given differential equation is not a polyno-
Any relation between the dependent and independent varia-
mial in differential coefficients, so its degree is not
bles (not involving the derivatives) which, when substituted
defined.
in the differential equation, reduces it to an identity is called
LINEAR AND NON-LINEAR DIFFERENTIAL a solution of the differential equation.
Consider the differential equation:
EQUATIONS
dy
= cos x(1)
A differential equation in which the dependent variable and dx
its differential coefficients occur only in the first degree and Then, y = sin x is a solution of (1). We notice that
are not multiplied together is called a linear differential
y = sin x + 1, y = sin x – 3
equation.
are also solutions of (1).
Illustration 1 The solution y = sinx + c, where c is an arbitrary con-
stant, is called the general solution of (1), since every solu-
dy y dy
+ 2 = 3x2 and 3 x log x + 4 y = 2log x tion of (1) can be obtained from y = sin x + c, for a suitable
dx x dx choice of c.
are linear differential equations of the first order.
General Solution
Illustration 2
The solution of a differential equation which contains a
d2 y dy number of arbitrary constants equal to the order of the dif-
3 2 + 8 + 6 y = ex is a linear differential equation of
dx dx ferential equation is called the general solution. Thus, the
the second order. general solution of a differential equation of the nth order
has n arbitrary constants.
Illustration 3
dy Particular Solution
2y + 2 = 5x
dx A solution obtained by giving particular values to arbi-
is a non-linear differential equation because y and dy/dx are trary constants in the general solution is called a particular
multiplied together. solution.

Illustration 4 SOLVED EXAMPLE


2 8. A solution of the differential equation
d2 y ⎛ dy ⎞
+ 2⎜ ⎟ + 5y = 0 2
dx 2 ⎝ dx ⎠ ⎛ dy ⎞ dy
⎜⎝ ⎟⎠ − x + y = 0 is
dx dx
dy
is a non-linear differential equation because occurs in (A) y = 2 (B)  y = 2x
the second degree. dx (C) y = 2x – 4 (D)  y = 2x2 – 4
9.4  Chapter 9

Solution: (C) 10. The differential equation of family of parabolas with


Direct substitution of y = 2x – 4 in the equation shows foci at the origin and axis along the x-axis is
that it is a solution of the given differential equation. 2
⎛ dy ⎞ dy
(A)  y ⎜ ⎟ + 2 x − y=0
⎝ dx ⎠ dx
FORMATION OF A DIFFERENTIAL EQUATION 2
⎛ dy ⎞ dy
Let f (x, y, c1, c2, … cn) = 0 be the solution of a d­ ifferential (B)  x ⎜ ⎟ + 2 y − y=0
⎝ dx ⎠ dx
equation, where c1, c2, …, cn are n arbitrary constants. If we 2
eliminate these n constants, we obtain the differential equa- ⎛ dy ⎞ dy
(C)  y ⎜ ⎟ + 2 x + y=0
tion of the nth order satisfied by the given s­ olution v­ alues. ⎝ dx ⎠ dx
Any equation taken together with n relations obtained by dif- (D)  None of these
ferentiating it n times helps us to eliminate the n constants.
Solution: (A)
Working Rule for Formation of Differential Let the directrix be x = – 2a and latus rectum be 4a.
Equations Then, the equation of the parabola is
(distance from focus = distance from directrix),
■ Write the given equation. x2 + y2 = (2a + x)2 or y2 = 4a (a + x)(1)
■ Differentiate the given equation with respect to indepen-

dent variable x as many times as the number of arbitrary Differentiating with respect to x, we get
constants. dy 1 dy
■ Eliminate the arbitrary constants with the help of the given y = 2a or a = y ⋅
dx 2 dx
equation and the equations obtained by differentiation to
get the required differential equation. Putting this value of a in (1), the differential equation
is

SOLVED EXAMPLES dy ⎛ y dy ⎞
y2 = 2 y ⎜ + x⎟
dx ⎝ 2 dx ⎠

9. The differential equation of all circles passing through 2
⎛ dy ⎞ ⎛ dy ⎞
the origin and having their centres on the x-axis is or y ⎜ ⎟ + 2 x ⎜ ⎟ − y = 0
⎝ dx ⎠ ⎝ dx ⎠
dy dy
(A) y2 = x2 + 2xy (B)  y2 = x2 – 2xy
dx dx 11. The differential equation that represents all parabolas
dy each of which has a latus rectum 4a and whose axes
(C) x2 = y2 + xy (D)  None of these are parallel to x-axis, is
dx
3
Solution: (A) d2 y
⎛ dy ⎞
(A) a 2 + ⎜ ⎟ = 0
The equation of circles passing through the origin and dx ⎝ dx ⎠
having their centres on the x-axis is 3
d2 y ⎛ dy ⎞
2 2 (B) 2a +⎜ ⎟ =0
x + y + 2gx = 0, (1) dx 2 ⎝ dx ⎠
where g is an arbitrary constant. d2 y
3
⎛ dy ⎞
Differentiating with respect to x, we get (C) 2a −⎜ ⎟ =0
dx 2 ⎝ dx ⎠
dy ⎛ dy ⎞ (D)  None of these
x + y + g = 0. i.e., g = – ⎜ x + y ⎟
dx ⎝ dx ⎠
Putting this value of g in (1), we get Solution: (B)
⎛ dy ⎞ Equation of the family of such parabolas is
x2 + y2 – 2x ⎜ x + y ⎟ = 0,
⎝ dx ⎠ (y – k)2 = 4a (x – h)(1)
dy where h and k are arbitrary constants.
i.e., y2 = x2 + 2xy
dx Differentiating with respect to x, we get
Differential Equations  9.5

Solution: (A)
dy
(y – k) = 2a (2) The equation of the general circle is given by
dx
x2 + y2 + 2gx + 2fy + c = 0 (1)
Differentiating again,
2 Differentiating with respect to x, we get
d2 y ⎛ dy ⎞
(y – k) + ⎜ ⎟ = 0 (3) 2x + 2yy′ + 2g + 2fy′ = 0 (2)
dx 2 ⎝ dx ⎠
Differentiating again, we get
Putting value of y – k from (2) in (3), we get
3 1 + y′2 + yy″ + fy″ = 0(3)
d2 y
⎛ dy ⎞
2a + ⎜ ⎟ = 0, Differentiating again, we have
dx 2 ⎝ dx ⎠
2y′ y′′ + yy′′′ + y′ y′′ + fy′′′ = 0 (4)
which is the required differential equation.
Eliminating f from (3) and (4), we get
12. The differential equation satisfied by ax2 + by2 = 1 is
(A) xy y″ + xy′ 2 + yy′ = 0 y′′′ (1 + yy′′ + y′2) – y′′ (yy′′′ + 3y′ y′′) = 0
(B) xy y″ + xy′ 2 – yy′ = 0
(C) xy y″ – xy′ 2 + yy′ = 0 ⇒ y′′′ (1 + y′2) – 3y′ y′′2 = 0,
(D)  None of these which is the required differential equation.
Solution: (B) 14. The equation of the family of curves which intersect
The given solution is the hyperbola xy = 2 orthogonally is
ax2 + by2 = 1 (1) x3 x2
(A) y = + c y=
(B)  +c
Differentiating with respect to x, we get 6 4
2ax + 2byy′ = 0 (2) x3 x2
(C) y = − + c y= −
(D)  +c
Differentiating again, we get 6 4

2a + 2b (y′2 + yy″) = 0 (3) Solution: (A)


From (2), we have dy
Let m1 = for required family of curves at (x, y).
dx
a − yy ′
= ⋅ dy
b x Let m2 = for the hyperbola xy = 2.
dx
Also from (3), we have dy −2
Then m2 = = 2⋅
a dx x
= – ( y′ 2 + yy″)
b Since the required family of curves is orthogonal to
− yy ′ the hyperbola,
⇒ = – (y′2 + yy″)
x \ m1 × m2 = – 1
⇒ yy′ = xy′2 + xyy″
dy ⎛ −2 ⎞ dy x 2 x2
⇒ ×⎜ 2⎟ =–1⇒ = ⋅ ⇒ dy = dx.
⇒ 2
x yy″ + xy′ – yy′ = 0 dx ⎝ x ⎠ dx 2 2

which is the required differential equation. x3


Integrating, we get y = + c, which is the required
13. The differential equation of the family of general family. 6
­circles is
(A) y′′′ (1 + y′ 2) – 3y′ y″ 2 = 0 15. The order of the differential equation, of which
(B) y′′′ (1 + y′ 2) + 3y′ y″ 2 = 0 xy = cex + be– x + x2 is a solution, is
(C) y′′′ (1 + y′ 2) – 3y″ y′ 2 = 0 (A) 1 (B) 2
(D)  None of these (C)  3 (D)  None of these
9.6  Chapter 9

Solution: (B) Solution: (B)


We have, The general equation of all ellipses centred at the
xy = cex + be– x + x2(1) x2 y2
­origin is +
= 1.
a2 b2
Differentiating with respect to x, we get
Differentiating with respect to x, we get
dy
x + y = cex – be– x + 2x (2) 2x 2y x y
dx 2
+ 2
⋅ y1 = 0 i.e., 2
+ y1 = 0 (1)
a b a b2
Differentiating again,
Differentiating again, we get
d2 y dy
x 2 +2 = cex + be– x + 2 = xy – x2 + 2 1 1
dx dx ( y12 + yy2) = 0
2
+ (2)
a b2
 [Using (1)] From (1) and (2), we have
Hence, the required differential equation is
2
d y dy
1
b2
( yy 1 )
− xy12 − xyy2 = 0
x 2 +2 − xy + x 2 − 2 = 0
dx dx ⇒ xyy2 + xy12 − yy1 = 0,
The order of this differential equation is 2.
which is the required differential equation.
16. The differential equation of all parabolas whose axes Solution of First Order and First Degree
are parallel to the axis of y, is Differential Equations
d3 y d3 y The following methods may be used to solve first order and
(A)  3
= 1 (B)  3
=–1 first degree differential equations.
dx dx
3
d y Variable Separable Differential Equations
(C)  3
= 0 (D)  None of these
dx A differential equation of the form
Solution: (C) dy
f  (x) + g (y) = 0 (1)
Such parabolas are given by dx
(x – h)2 = 4a (y – k), or f (x) dx + g (y) dy = 0

where h, k a are three arbitrary constants. is said to have separated variables.


dy Integrating Equation (1), we obtain
Differentiating with respect to x, (x – h) = 2a ⋅
dx dy
2 2
∫ f ( x) dx + ∫ g ( y) dx dx = c,
d y d y 1
Differentiating again, 1 = 2a 2
i.e., 2
= ⋅ where c is an arbitrary constant.
dx dx 2a
Hence ∫ f ( x ) dx + ∫ g ( y ) dy = c is the solution of
d3 y Equation (1).
Differentiating once again, = 0.
dx 3
This is the required differential equation. QUICK TIPS
17. The differential equation of all ellipses centred at the ■ There is no need to add arbitrary constants of integration
origin is on both sides, since they can be combined to give just
(A) xyy2 – x y12 + yy1 = 0 one arbitrary constant. Moreover, the general solution of
a differential equation of order one must have only one
(B) xyy2 + x y12 – yy1 = 0 arbitrary constant.
■ The constant of integration can be taken as c or log c or
(C) xyy2 + x y12 + yy1 = 0
tan–1c and so on depending on the nature of the problem.
(D)  None of these
Differential Equations  9.7

INITIAL VALUE PROBLEMS


SOLVED EXAMPLES
In many cases instead of finding the general solution, we
dy
find a particular solution satisfying a given initial condi-
19. Solve: (x – y)2 dx = 1.
tion, say, the condition that at some point x0, the solution
y (x) has the value y0. This is expressed as y (x0) = y0. Solution:
A first-order differential equation together with an We have,
initial condition is called an initial value problem.
dy dy 1
( x − y)2 =1⇒ = (1)
SOLVED EXAMPLE dx dx ( x − y)2
dy dz
⎛ dy ⎞ Put x – y = z ⇒ 1− =
18. Find the particular solution of sin ⎜ ⎟ = a, given that dx dx
⎝ dx ⎠
when x = 0, y = 1. dy dz
⇒ = 1− (2)
Solution: dx dx
⎛ dy ⎞ dy From (1) and (2), we get
We have, sin ⎜ ⎟ = a or = sin–1a
⎝ dx ⎠ dx dz 1
1– =
or –1
dy = sin a dx dx ( x − y)2
Integrating both sides, we get dz 1
⇒ 1− = 2
dx z
∫ dy = ∫ sin −1 a dx

dz 1
⇒ y = sin −1 a∫ dx ⇒ = 1− 2
dx z
= sin–1a . x + c
dz z2 − 1
–1 ⇒ =
⇒ y = x sin a + c dx z2
When x = 0, y = 1 \ 1 = 0 + c or c = 1 z2
\ The required particular solution is ⇒ dz = dx
z2 − 1
y −1
y = x sin–1a + 1 or = sin–1a Integrating both sides, we get
x
z2
or
⎛ y − 1⎞
sin ⎜ = a
∫ z 2 − 1 dz = ∫ dx
⎝ x ⎟⎠
⎛ 1 ⎞
Equations Reducible to Variable Separable
⇒ ∫ ⎜⎝1 + z 2 − 1⎟⎠ dz = x + c

Form
1 z −1
⇒ z+ log = x + c
Sometimes in a given differential equation, the variables are 2 z +1
not separable. But, some suitable substitution reduces it to a
form in which the variables are separable. For example, the 1 x − y −1
⇒ ( x − y ) + log = x + c.
dy 2 x − y +1
differential equations of the type = f (ax + by + c) can
dx
be reduced to variable separable form by substituting ax + 20. The general solution of the differential equation
by + c = t. The reduced variable separable form is: dy
+ y g′ (x) = g (x). g′ (x),
dx
dt where g (x) is a given function of x, is
= dx
b f (t ) + a (A) g (x) + log [1 + y + g (x)] = c
(B) g (x) + log [1 + y – g (x)] = c
Integrating both sides to obtain the solution of this (C) g (x) – log [1 + y – g (x)] = c
­differential equation. (D)  None of these
9.8  Chapter 9

Solution: (B) 22. The particular solution of cos y dx + (1 + 2e– x)


We have, π
sin y dy = 0, when x = 0, y = is
dy 4
= (g (x) – y) . g′ (x) (A) ex – 2 = 3 2 cos y
dx
Put g (x) – y = V (B)  ex + 2 =
2 cos y
x
dy dV (C) e + 2 = 3 2 cos y
⇒ g′ (x) – =
dx dx (D)  None of these
dV Solution: (C)
Hence g′ (x) – = V · g ′(x)
dx We have,
dV
⇒ = (1 – V) g′ (x) cos y dx + (1 + 2e–x) sin y dy = 0
dx
dx sin y
dV ⇒ + dy = 0
⇒ = g′(x) dx 1 + 2e −x cos y
1−V
dV ex − sin y
⇒ ∫1−V = ∫ g ′ ( x ) dx ⇒ ∫ e x + 2 dx − ∫ cos y
dy = log c

⇒ – log (1 – V) = g (x) – c ⇒ log (e x + 2) − log cos y = log c


⇒ g (x) + log (1 – V) = c ⎛ ex + 2⎞
⇒ log ⎜ ⎟ = log c
\ g (x) + log [1 + y – g (x)] = c ⎝ cos y ⎠
dy ⇒ ex + 2 = c cos y,
21. The solution of the equation log = 9x – 6y + 6,
given that y = 1 when x = 0, is dx 1
where 1 + 2 = c ⋅ i.e. c = 3 2
(A) 3e6y = 2e9x– 6 + 6e6 2

(B) 3e6y = 2e9x + 6 – 6e6
\ The solution is ex + 2 = 3 2 cos y
(C) 3e6y = 2e9x + 6 + e6

(D)  None of these
23. The equation of the curve which passes through the
Solution: (C) point (2a, a) and for which the sum of the cartesian
We have, sub tangent and the abscissa is equal to the constant
dy a, is
log = 9x – 6y + 6 (A) y (x – a) = a2 (B)  y (x + a) = a2
dx 2
(C) x (y – a) = a (D)  x (y + a) = a2
dy
⇒ = e9x – 6y + 6 = e9x + 6 · e– 6y Solution: (A)
dx
We have,
⇒ e6y dy = e9x + 6 dx Cartesian subtangent + abscissa = constant
e6 y e9 x + 6 ⇒
y
+ x = a
Integrating, we get = +c dy / dx
6 9

Putting x = 0, y = 1, we get dx
⇒ y + x = a
dy
e6 e6 e6
= + c i.e. c =
6 9 18 dy dx
⇒ =
y a−x
e6 y e 9 x + 6 e6
\ The solution is = + Integrating, we get
6 9 18
log y + log (x – a) = log c
⇒ 3e6y = 2e9x + 6 + e6
Differential Equations  9.9

\ y (x – a) = c Solution: (A)
As the curve passes through the point (2a, a), we have We have,
c = a2. y2 2x
= 2. 2 ⋅
Hence, the required curve is y (x – a) = a2. y1 x +1

24. Solution of the differential equation x dy – y dx = 0 Integrating with respect to. x, we get
represents
log y1 = 2 log (x2 + 1) + log c ⇒ y1 = c (x2 + 1)2
(A)  parabola whose vertex is at origin
(B)  circle whose centre is at origin Since y1 (0) = 4, \ c = 4
(C)  a rectangular hyperbola \ y1 = 4 (x2 + 1)2 ⇒ dy = 4 (x4 + 1 + 2x2) dx
(D)  straight line passing through origin
⎛ x5 2 x 2 ⎞
Solution: (D) Integrating again, we get y = 4 ⎜ + + x⎟ + k .
⎝ 5 3 ⎠
We have, Putting x = 0, y = – 4, we get k = – 4.
dy dx \ The required equation of curve is
x dy – y dx = 0 ⇒ − =0
y x ⎛ x5 2 x3 ⎞
Integrating, we get log y – log x = log c y = 4 ⎜ + + x − 1⎟
⎝ 5 3 ⎠

y y
⇒ log = log c ⇒ = c ⇒ y = cx 27. If 2 f (x) = f ‘ (x) and f (0) = 3, then f (2) equals
x x
which is a straight line through the origin. (A) 4e3 (B) 3e4 (C) 2e3 (D) 3e2
Solution: (B)
dy
25. The solution of the differential equation 2x − y=3 We have,
represents dx
f’ ( x )
(A) Circles (B) Straight lines 2 f (x) = f ′ (x) ⇒ = 2.
f ( x)
(C) Ellipse (D) Parabola
Integrating, we get
Solution: (D)
log f (x) = 2x + c1.
We have,
dy 2 dx ⇒ f ( x ) = e 2 x + c = e c ⋅ e 2 x = ce2x,
1 1

2x =y+3⇒ dy = ⋅
dx y+3 x where c = ec 1

Integrating, we get Putting x = 0, f (0) = 3, we get c = 3.


2 log (y + 3) = log x + log c = log cx \ f (x) = 3e2x ⇒ f (2) = 3e4
⇒ log ( y + 3)2 = log cx dy ax + g
=
28. The solution of represents a circle when
⇒ ( y + 3)2 = cx dx by + f
which is a parabola. (A) a = b (B)  a=–b
(C) a = – 2b (D)  a = 2b
26. The equation of the curve satisfying the differential
equation y2 (x2 + 1) = 4xy1, passing through the point Solution: (B)
(0, – 4) and having slope of tangent at x = 0 as 4 is We have,
⎛ x5 2 x3 ⎞ dy ax + g
(A) y = 4 ⎜ +x+ − 1⎟ = ⇒ (by + f ) dy = (ax + g) dx
⎝ 5 3 ⎠ dx by + f
⎛ x5 2 x3 ⎞ Integrating, we get
(B) y = 4 ⎜ −x+ − 1⎟
⎝ 5 3 ⎠ by 2 ax 2
+ fy = + gx + c′
⎛ x5 2 x3 ⎞ 2 2
(C) y = 4 ⎜ +x+ + 1⎟ ⇒ ax2 – by2 + 2gx – 2fy + c = 0.
⎝ 5 3 ⎠
(D)  None of these This represents a circle if a = – b.
9.10  Chapter 9

HOMOGENEOUS DIFFERENTIAL EQUATIONS


Homogeneous Function dy dv
Put y = vx so that
■ = v+x .
A function f  (x, y) in x and y is called a homogeneous func- dx dx
tion of degree n, if the degree of each term is n. Putting these values in the given equation, we get

Illustration 1 dv dv dx
v+x = F  (v) ⇒ =
2 2
f  (x, y) = x + y + xy is a homogeneous function of degree dx [F(v ) − v ] x
2, since each term is of degree 2.
dv
Illustration 2 ⇒ ∫ F(v ) − v
= log | x | + c.

g (x, y) = x3 + 4x2y + 7xy2 is a homogeneous function of


y
degree 3, since each term is of degree 3. Replace v by
■ to obtain the required solution.
x
Illustration 3
h (x, y) = x2 + 4x2y + 7xy2 is not a homogeneous function,
since first term is of degree 2, while each one of second and Type I: Consider a differential equation of the form:
third terms is of degree 3.
dy ax + by + c a b
Homogeneous Differential Equations = , where ≠ (1)
dx Ax + By + C A B
dy f ( x, y )
A differential equation of the form = , where
dx g ( x, y ) This is clearly non-homogeneous. In order to make it
f (x, y) as well as g (x, y) is a homogeneous function of homogeneous, we proceed as follows:
same degree in x and y is called a homogenous differential We substitute x = X + h and y = Y + k in Equation (1),
equation. where h, k are constants to be determined suitably.
dx dy
We have = 1 and = 1, so that
REMEMBER dX dY

y dy dy d Y d X dY
A function of is always a homogenous function. = ⋅ ⋅ =
x dx d Y d X dx dX
Now Equation (1) becomes
Illustration 4
dY a X + bY + ( ah + bk + c )
dy x2 + y2 = (2)
= is a homogenous differential equation, as dX A X + BY + (Ah + B k + C )
dx xy
(x2 + y2) as well as xy is a homogenous function of degree 2. Choose h and k so that
ah + bk + c = 0,
Illustration 5
Ah + Bk + C = 0.
dy ⎛ y⎞
= x sin ⎜⎝ ⎟⎠ is a homogeneous differential equation, These equations give
dx x
⎛ y⎞ y bC − Bc Ac − aC
as x sin ⎜ ⎟ being a function of , is a homogenous h = ,k= (3)
⎝ x⎠ x a B − Ab a B − Ab
function.
Now Equation (2) becomes
Equations
WorkingReducible to the Homogeneous
Rule for Solving a Homogeneous dY a X + bY
Differential Equation = ,
dX AX + BY
dy f( x , y )
Let = be the given homogeneous differential which being a homogeneous equation can be solved by
dx g( x, y ) means of the substitution Y = vX.
equation.
Differential Equations  9.11

Type II: Consider a differential equation of the form Linear Differential Equations
dy ax + by + c A differential equation of the form
= ,
dx Ax + B y + C
dy
a b + Py = Q,(1)
where = dx
A B
= k (say) where P and Q are functions of x (or constants), is called a
linear differential equation of the first order.
Since a B – A b = 0, the above method fails in view of
Equation (3). dy
Working Rule For Solving + Py = Q
dy k (A x + B y ) + c
dx
We have = (4)
dx Ax + B y + C ■ Find integrating factor (I.F.) = e ∫
Pdx
.
Substitute Ax + By = z so that
■ The solution of the differential equation is
dy dz
A+B =
dx dx y (I.F.) = ∫ Q (I.F) dx + c,
Now Equation (4) becomes where c is constant of integration.

dz kz + c
= B. + A,
dx z+C
QUICK TIPS
which is an equation with variables separable.
■ Sometimes a first order differential equation which is not
expressible as Equation (1) becomes a linear equation of
SOLVED EXAMPLE the form

⎛ f ( y /x ) ⎞ dx
+ Ry = S,
29. Solution of the equation x dy = ⎜ y + x dx is
f’ ( y /x ) ⎟⎠
dy

where R and S are functions of y alone (or constants). The
⎛ x⎞ ⎛ y⎞ integrating factor in this case is given by,
(A)  f ⎜ ⎟ = cy (B)  f ⎜ ⎟ = cx
⎝ y⎠ ⎝ x⎠
I.F. = e ∫
R dy

⎛ y⎞
(C)  f ⎜ ⎟ = cxy
(D)  None of these The solution of this equation is
⎝ x⎠
Solution: (B) x . (I.F.) = ∫ [S × (I.F.)] dy + C ,
We have, where C is the constant of integration.
⎛ xf ( y /x ) ⎞ ■ The fact e
logt
= t will be frequently used in the solution of
x dy = ⎜ y + dx
⎝ f ′ ( y /x ) ⎟⎠ linear equations.

dy y f ( y /x )
⇒ = + which is homogeneous. Equations Reducible to the Linear Form
dx x f ′ ( y /x )
dy dV Consider a differential equation of the form:
Put y = Vx so that =V+x ,
dx dx dy
we obtain + Py = Q y n,(1)
dx
dV f (V ) f (V ) dx
V+x =V+ ⇒ dV =
dx f ′ (V ) f ′ (V ) x where P and Q are functions of x. This equation can be
reduced to the linear form as follows:
Integrating, we get
Dividing both sides of Equation (1) by y n, we get
log f (V) = log x + log c
⎛ y⎞ dy
⇒ log f (V) = log cx ⇒ f ⎜ ⎟ = cx. y– n + Py − n + 1 = Q(2)
⎝ x⎠ dx
9.12  Chapter 9

Dividing by y2ex, we get


–n+1 –n dy dz
Put y = z ⇒ (– n + 1) y =
dx dx 1 dy 1
− = x2e– x
Substituting in Equation (2), we obtain
y 2 dx y
dz 1 −1 dy dV
+ (1 − n) Pz = (1 – n) Q, Put = V so that 2 =
dx y y dx dx
which is now a linear equation with z as the dependent dV
variable. We thus have + V = –x2e– x, which is linear
dx

I. F. = e ∫
1dx
SOLVED EXAMPLES \ = ex

dy Hence, the solution is


30. The general solution of the differential equation
dx c
∫x
2 2 −x
= y tan x – y sec x is V × ex = – e ⋅ e x dx +
3
(A) tan x = (c + sec x) y
(B) sec y = (c + tan y) x 1 x x3 c
(C) sec x = (c + tan x) y or e =– + or x3y + 3ex = cy
y 3 3
(D)  None of these
Solution: (C) dy
We have 32. The solution of x3 + 4x2 tan y = ex sec y satisfying
y (1) = 0, is dx  
dy
= y tan x – y2 sec x x
(A) tan y = (x – 2)e log x
dx
(B) sin y = ex (x – 1) x–4
1 dy 1
⇒ − tan x = – sec x (C) tan y = (x – 1) ex x–3
y 2 dx y
(D) sin y = ex (x – 1) x–3
1 −1 dy dV Solution: (B)
Putting =V⇒ 2 = ,
y y dx dx We have,
dV dy 4 ex
we obtain + tan x . V = sec x, which is linear. cos y + sin y = 3
dx dx x x

I.F. = e ∫
tan x dx
= elog sec x = sec x. dy dt
Let sin y = t ⇒ cos y =
dx dx
Hence, the solution is

V sec x = ∫ sec 2 x dx + c or
1
sec x = tan x + c dt 4 ex
\ + t= 3
y dx x x
or sec x = y (c + tan x) 4
∫ x dx
I.F. = e = e4 log x = x4
31. The general solution of the differential equation y (x2y
+ ex) dx – ex dy = 0 is \ The solution is
(A) x3y – 3ex = cy (B)  x3y + 3ex = cy ex
t x4 = ∫x ⋅ dx = xex – ex + c
3 4
y
(C) y x – 3e = cx (D)  y3x + 3ey = cx
x3
Solution: (B)
We have sin y x4 = xex – ex + c

y (x2y + ex) dx – ex dy = 0  x = 1, y = 0 \ c = 0
dy
⇒ ex = x2y2 + yex \ sin y = ex (x – 1) x–4
dx
Differential Equations  9.13

dy ⇒ y eg(x) = g(x) eg(x) – eg(x) + c


33. Solution of the equation = ex – y (ex – ey) is
dx or y = [g(x) – 1] + ce–g(x)
(A) ey = e x − 1 + ce − e (B) 
ey = e x − 1 + ce e
x x

(C) ex = e y − 1 + ce − e
y

(D)  None of these SOLUTION BY INSPECTION


Solution: (A) The following derivatives must be remembered as they are
We have, very useful in solving some differential equations directly.
1. d (x + y) = dx + dy
dy 2. d (xy) = ydx + xdy
= ex – y (ex – ey)
dx ⎛ x⎞ y dx − x dy
3. d ⎜ ⎟ =
dy ⎝ y⎠ y2
⇒ ey dx + ex . ey = e2x
⎛ y⎞ x dy − y dx
4. d ⎜ ⎟ =
dy dV ⎝ x⎠ x2
Putting ey = V so that e y = , we get
dx dx y dx + x dy
5. d (log xy) =
dV xy
+ ex . V = e2x, which is linear in V.
dx ⎛ y⎞ x dy − y dx
6. d ⎜ log ⎟ =
I.F. = e ∫
e dx
= ee
x x
⎝ x⎠ xy
7. d (xm yn) = xm – 1 yn – 1 (my dx + nx dy)
So, the solution is
⎛1 ⎞ x dx + y dy
V  . e e = ∫ e e ⋅ e dx + c 8. d ⎜ log ( x 2 + y 2 )⎟ =
x x 2x


⎝2 ⎠ x2 + y2
∫e
x

⇒ e y ⋅ ee = z
⋅ z dz + c ⎛1 x + y⎞ x dy − y dx
9. d ⎜ log ⎟ =
(Putting e = z ⇒ e dx = dz) x x
⎝2 x − y⎠ x2 − y2
⇒ e y⋅ e e = (z – 1) ez + c = (e x − 1) e e + c
x x
⎛ y⎞ x dy − y dx
10. d ⎜ tan −1 ⎟ =
⎝ x⎠ x2 + y2
⇒ ey = ex – 1 + ce–ex
d [ f ( x, y ) ]
1− n
f ′ ( x, y )
11. =
34. If g(x) is a differential function, then the solution of the
differential equation
1− n [ f ( x, y)]n
dy + [y g′(x) – g(x) g′(x)]dx = 0 is
(A) y = [g(x) – 1] + ce–g(x)
12. d ( x2 + y2 = ) x dx + y dy
x2 + y2
(B) yeg(x) = [g(x) + 1]eg(x) + c
(C) y = [g(x) – 1]eg(x) + c
SOLVED EXAMPLES
(D)  None of these.
Solution: (A) xdy − ydx
35. Solution of the equation xdx + ydy + = 0 is
We have, x2 + y2
dy + [yg′(x) – g(x) g′(x)] dx = 0
⎛ c + x2 + y2 ⎞
dy (A) y = x tan ⎜ ⎟
⇒ + g ′( x ) ⋅ y = g(x) . g′(x) ⎝ 2 ⎠
dx
Thus is a linear differential equation ⎛ c + x2 + y2 ⎞
(B) x = y tan ⎜ ⎟
⎝ 2 ⎠
with I.F. = e ∫
g ′ ( x ) dx
= eg(x)
⎛ c − x2 − y2 ⎞
So, the solution is (C) y = x tan ⎜ ⎟
⎝ 2 ⎠
(D)  None of these
y eg(x) = ∫ g ( x ) ⋅ g ′( x ) e
g( x)
dx

9.14  Chapter 9

Solution: (C)
⎛ dy ⎞ ⎛ dy ⎞
We have, ⇒ ⎜⎝ − 1⎟ ⎜ x + y ⎟ = 0
dx ⎠ ⎝ dx ⎠
x dy − y dx
x dx + y dy + = 0 dy dy
x2 + y2 ⇒ = 1 or x = – y
dx dx
1 ⎛ y⎞ dy
⇒ d (x2 + y2) + d tan– 1 ⎜⎝ x ⎟⎠ = 0 The solution of = 1 is y = x + c
2 dx
y dy dy dx
1 2 c and solution of x = – y i.e. + = 0 is
Integrating, (x + y2) + tan– 1 x = dx y x
2 2
2 2 –1 y log (xy) = log c i.e. xy = c
⇒ x + y + 2 tan =c
x Hence, general solution is (x – y + c) (xy – c) = 0.
⎛ c − x2 − y2 ⎞
\ y = x tan ⎜ ⎟ is the required solution. 38. Solution of equation (xy4 + y) dx – xdy = 0 is
⎝ 2 ⎠ (A) 4x4y3 + 3x3 = cy3 (B)  3x3y4 + 4y3 = cx3
4 3 3 3
36. The solution of the equation (C) 3x y + 4x = cy (D)  None of these

dy ⎛ y2 ⎞ Solution: (C)

y sin x = cos x ⎜ sin x − ⎟ , given y = 1 when We have,
dx ⎝ 2⎠
π
x= is xy4 dx + y dx – x dy = 0
2
y dx − x dy
(A) y2 = sin x (B)  y2 = 2 sin x ⇒ x dx + = 0
2
(C) x = sin y (D)  x2 = 2 sin y y4
2
Solution: (A) 3 ⎛ x ⎞ y dx − x dy
⇒ x dx + ⎜ ⎟ ⋅ = 0
Put y2 sin x = V. ⎝ y⎠ y2
dy dV
Then 2y sin x + y2 cos x = ⎛ x⎞
2
⎛ x⎞
dx dx ⇒ x3 dx + ⎜ ⎟ d ⎜ ⎟ = 0
⎝ y⎠ ⎝ y⎠
1 dV
So, the given equation becomes = sin x cos x
2 dx Integrating, we get
⇒ dV = 2 sin x cos x dx 3
x4 1 ⎛ x ⎞
+ = c′
Integrating, we get 4 3 ⎜⎝ y ⎟⎠

V = sin2x + c. i.e., y2 sin x = sin2x + c or 3x4y3 + 4x3 = c y3,
π
Putting y = 1, x = , we get 1 = 1 + c i.e. c = 0 which is the required solution.
2
So, the solution is y2 sin x = sin2x i.e. y2 = sin x.
39. The solution of the differential equation
2
⎛ dy ⎞ dy dy y
37. Solution of the equation x ⎜ ⎟ + ( y − x ) − y = 0 x 2 − xy = 1 + cos is
⎝ dx ⎠ dx dx x
is
(A) (x – y + c) (xy – c) = 0 y c
(A) cos =1+
(B) (x + y + c) (xy – c) = 0 x x
(C) (x – y + c) (2xy – c) = 0 y
(D) ( y – x + c) (xy – c) = 0 (B) x2 = (c + x2) tan
x
Solution: (A) y 1
(C) tan =c–
We have, 2x 2x2
2
⎛ dy ⎞ dy y 1
x + (y – x) ⎜ ⎟ – y =0 (D) tan = c +
⎝ dx ⎠ dx x x

Differential Equations  9.15

Solution: (C) 41. Solution of the differential equation


We have
y (xy + 2x2y2)dx + x(xy – x2y2)dy = 0 is
dy y y
x2 dx – xy = 1 + cos = 2cos2 2 x ⋅ 1
x (A) 2 log| x | − log | y | − =c
xy
⎛ y ⎞ ⎛ 2 dy ⎞ 1
⇒ sec2 ⎜⎝ 2 x ⎟⎠ ⋅ ⎜⎝ x dx − xy ⎟⎠ = 2 (B) 2 log| x | + log | y | − =c
xy
dy 1
x −y (C) 2 log| x | − log | y | + =c
1 ⎛ y⎞ dx 1
⇒ sec 2 ⎜ ⎟ ⋅ = 3 xy
2 ⎝ 2x ⎠ x 2
x (D)  None of these
d ⎛ y⎞ 1 Solution: (A)
⇒ ⎜⎝ tan ⎟⎠ = 3 ⋅
dx 2x x We have,
Integrating, we get (xy2 + 2x2y3) dx + (x2y – x3y2)dy = 0
y 1
tan =c − 2 ⇒ xy( y dx + x dy) + x2y2(2y dx – x dy) = 0
2x 2x
which is the required solution. ⎛2
d ( xy ) 1 ⎞
3 ⇒ + ⎜ dx − dy ⎟ = 0 (Dividing by x3y3)
40. Solution of the equation xdy – [ y + xy (1 + log x)] 2 2
x y ⎝x y ⎠
dx = 0 is
On integrating, we get
−x 2 2 x3 ⎛ 2 ⎞
(A)  = ⎜⎝ + log x ⎟⎠ + c 1
y2 3 3 – + 2 log | x | − log| y | = c
xy
x2 2 x3 ⎛ 2 ⎞ 42. The general solution of the differential equation
(B)  = ⎜⎝ + log x ⎟⎠ + c
y2 3 3 y (x2y + ex) dx – exdy = 0 is
−x 2 x3 ⎛ 2 (A) x3y – 3ex = 3cy (B)  x3y + 3ex = 3cy

(C)  = ⎜ + log x ⎟⎠ + c
3 x
(C) xy – 3e = 3 cx (D)  None of these
y2 3 ⎝3
(D)  None of these Solution: (B)
Solution: (A) We have,
We have, y(x2y + ex)dx – exdy = 0
x dy – y dx = xy3 (1 + log x) dx
⇒ x2y2dx + yex dx – ex dy = 0
⎛ ydx − xdy ⎞
⇒ – ⎜ ⎟ = xy (1 + log x) dx ye x dx − e x dy
⎝ y2 ⎠ ⇒ x2dx + = 0
y2
⎛ x⎞
⇒ – d ⎜ ⎟ = xy (1 + log x) dx ⎛ ex ⎞
⎝ y⎠ ⇒ x2dx + d ⎜ ⎟ = 0
⎝ y⎠
x ⎛ x⎞
⇒ − d = x2 (1 + log x) dx
y ⎜⎝ y ⎟⎠ On integrating, we get

Integrating, we get x3 e x
+ = c or x3y + 3ex = 3cy
2 3 y
⎛ x⎞
−⎜ ⎟ dy
⎝ y⎠ x3 x3 1 Equations of the Form + Pf (y) = Q f (y),
= (1 + log x) −∫ ⋅ dx dx
2 3 3 x
where P and Q are functions of x alone or constants
x2 x 3
x 3
c
⇒ − = (1 + log x ) − + Step I: Divide both sides by f (y) to get rid of f (y) from the
2 y2 3 9 2 R.H.S., such that
x2 2 x3 ⎛ 2 ⎞ 1 dy ϕ( y)
⇒ − 2
= ⎜⎝ + log x ⎟⎠ + c ⋅ + P⋅ =Q
y 3 3 f ( y ) dx f ( y)
9.16  Chapter 9

ϕ( y) dv
Step II: Put =v or + kPv = kQ,
f ( y) dx
dv d ⎡ ϕ( y) ⎤ 1 dy
Step III: If = ⎢ ⎥ =k· , where k is which is a linear differential equation with v as the depend-
dx dx ⎣ f ( y ) ⎦ f ( y ) dx
ent variable.
some constant, the equation reduces to the form.
1 dv
+ Pv = Q
k dx
Differential Equations  9.17

NCERT EXEMPLARS
1. The degree of the differential equation (A) cos x (B) 
tan x
2
 d 2 y   dy 
2
 dy  (C) sec x (D) 
sin x
 2    = x sin   is
+
 dx   dx  dx  8. The solution of differential equation tan y sec2 xdc +
(A) 1 (B) 2 tan x sec2 ydy = 0 is
(C) 3 (D) not defined (A) tan x + tan y = k (B) tan x – tan y = k
2. The degree of the differential equation (C)  tan x
−3/ 2
= k (D) tan x . tan y = k
  dy  2  2 tan y
d y is 3
1 +    = 9. The family y = Ax + A of curves is represented by
  dx   dx 2 differential equation of degree

(A)  3
4 (B)  (A) 1 (B) 2
2 (C) 3 (D) 4
(C) not defined (D) 2
10. The integrating factor of xdy − y = x 4 − 3x is
dx
3. The order and degree of the differential equation (A) x (B)  log x
1/ 4
d 2 y  dy  1
+ + x1/5 = 0 respectively, are (C)  (D)  –x
dx 2  dx  x
(A)  2 and 4 (B)  2 and 2 dy
11. The solution of − y = 1, y(0) = 1 is given by
(C)  2 and 3 (D)  3 and 3 dx
(A) xy = –ex (B)  xy = –e–x
4. If y = y = e ( A cos x + B sin x ) , then y is a solution of
−x

(C) xy = – 1 (D)  y = 2ex – 1
d2 y dy
(A)  +2 =0
dx 2
dx 12. The number of solutions of dy = y + 1 , when y(1) = 2 is
d2 y dy dx x − 1
(B)  dx 2 − 2 dx + 2 y = 0 (A) none (B) one
(C) two (D) infinite
d2 y dy
(C)  2 + 2 + 2 y = 0
dx dx 13. Which of the following is a second order differential
d2 y equation ?
(D)  2 + 2 y = 0
dx (A) (y’)2 + x = y2 (B)  y’ y” + y = sin x
(C) y”’ + (y”)2 + y = 0 (D)  y’ = y2

NCERT EXEMPLARS
5. The differential equation for
y = A cos α x + B sin α x + B sin α x,where A and B are 14. The integrating factor of differential equation
arbitrary constants is (1 − x 2 ) dy
dx
− xy = 1 is
2
(A) d y2 − α 2 y = 0
2
(B)  d y2 + α 2 y = 0 (A) – x (B)  x
dx dx 1+ x2
1
log (1 − x 2 )
2 2
d y
(C)  2 + α y = 0 (D)  d y2 − α y = 0 (C)  1 − x 2 (D) 
dx dx 2
–1 –1
6. The solution of differential equation xdy – ydx – 0 15. tan x + tan y = C is general solution of the differen-
represents tial equation

(A) dy = 1 + y 2
(A)  a rectangular hyperbola 2

(B)  parabola whose vertex is at origin dx 1 + x


(C)  straight line passing through origin dy 1+ x2
(D)  a circle whose center is at origin (B)  =
dx 1 + y
2

7. The integrating factor of differential equation ( ) ( )


(C)  1 + x 2 dy + 1 + y 2 dx = 0
dy
cos x
dx
+ y sin x = 1 is ( ) ( )
(D)  1 + x 2 dx + 1 + y 2 dy = 0
9.18  Chapter 9

dy sin x
16. The differential equation y + x = C represents (A)  = C (B) sin x sin y = C
dx sin y
(A)  family of hyperbolas (C) sin x + sin y = C (D)  cos x cos y = C
(B)  family of parabolas dy
(C)  family of ellipses 25. The solution of x = + y = e x is
dx
(D)  family of circles x
(A) y = e + k (B)  y = xex + Cx
17. The general solution of e 2 cos ydx − e x sin ydy = 0 is x x
(C) y = xex + k (D)  ey k
(A) ex cos y = k ex sin y = k
(B)  x= +
y y
(C) ex = k cos y ex = k sin y
(D) 
26. The differential equation of the family of curves x2 + y2 –
18. The degree of differential equation 2ay = 0, where a is arbitrary constant, is
3
d 2 y  dy 
+ + 6 y 5 = 0 is (A)  x 2 − y 2 dy = 2 xy
( )
dx 2  dx  dx
(A) 1 (B) 2 (B)  2 x 2 + y 2 dy = xy
( )
(C) 3 (D) 5 dx
2 dy
(C) 2 x − y
2
( )
= xy
19. The solution of dy + y = e − x , y ( 0 ) = 0 is dx
dx (D)  x 2 + y 2 dy = 2 xy
(A) y = ex(x – 1) (B)  y = xe–x
( )
dx
–x
(C) y = xe + 1 (D)  y = (x + 1)e–x
27. The family Y = Ax + A3 of curves will correspond to a
20. The integrating factor of differential equation differential equation of order
dy (A) 3 (B) 2
+ y tan x − sec x = 0 is (C) 1 (D) not defined
dx
28. The general solution of dy 2
(A) cos x (B)  sec x = 2 xe x − y is
(C) ecos x (D)  esec x dx
2 2
(A) e x − y = C (B)  e− y + e x = C
21. The solution of differential equation dy = 1 + y 2 is
2
2 2
(C) e y + e x + C (D)  ex +y = C
dx 1 + x
(A) y = tan–1 x (B)  y – x = k(1 + xy) 29. The cuve for which the slope of the tangent at any
(C) x = tan–1 y (D) tan(xy) = k point is equal to the ratio of the abscissa to the ordinate
of the point is
NCERT EXEMPLARS

22. The integrating factor of differential equation (A)  an ellipse


dy 1 + y is (B) parabola
+y=
dx x (C) circle
x ex (D)  rectangular hyperbola
(A)  x (B) 
e x x 2

30. The general solution of differential equation dy = e 2


+ xy
x x
(C) xe (D) 
e dx
is
23. y = ae mx = be − mx satisfies whichof the following differ- 2 2

ential equation ? (A) y = Ce − x / 2 (B)  y = Ce x / 2


(C) y = ( x + C ) e x / 2 (D)  y = ( C − x ) e x / 2
2 2

(A) dy + my = 0 (B)  dy − my = 0d
dx dx
2 2 31. The solution of equation (2y – 1) dx – (2x + 3) dy = 0 is
(C) d y2 − m 2 y = 0 (D)  d y2 + m 2 y = 0
dx dx (A)  2 x − 1 = k (B)  2 y + 1 = k
2y + 3 2x − 3
24. The solution of differential equation cos x sin ydx + sin (C)  2 x + 3 (D)  2 x − 1 = k
=k
x cos ydy = 0 is 2 y −1 2 y −1
Differential Equations  9.19

32. The differential equation for which y = a cos x + b sin 37. Which of the following is the general solution of
x is a solution, is d2 y dy
− 2 + y = 0?
2
dx 2 dx
(A) d y2 + y = 0
dx (A) y ( Ax + B ) e x
= (B)  y = ( Ax + B ) e − x
2
(B)  y2 − y = 0
d (C) y = Ae x + Be − x (D) 
y = Acosx + Bsinx
dx
38. The general solution of dy + y tan x = sec x is
2
(C) d y2 + ( a + b ) y = 0
dx dx
2
(A) y sec x = tan x + C (B)  y tan x = sec x + C
(D)  y2 + ( a − b ) y = 0
d
dx (C) tan x = y tan x + C (D)  x sec x = tan y + C

33. The solution of dy + y = e − x , y ( 0 ) = 0 is 39. The solution of differential equation dy + y = sin x is


dx dx x
(A) y = e − x ( x − 1) (B)  y = xex (A) x(y + cos x) = sin x + C
(B) x(y – cos x) = sin x + C
(C) y = xe–x + 1 (D)  y = xe–x (C) xy cos x = sin x + C
(D) x(y + cos x) = cos x + C
34. The order and degree of differential equation
2
 d3 y 
4 40. The general solution of differential equation
d2 y  dy 
 3  − 3 2 + 2   = y are
 dx  dx  dx 
4
( )
e x + 1 ydy = ( y + 1) e x dx is

(A)  1, 4 (B)  3, 4 (A) (y + 1) = k (ex + 1)


(C)  2, 4 (D)  3, 2 (B) y + 1 = ex + 1 + k
(C) y = log {k(y + 1) (ex + 1)}
 x 
35. The order and degree of differential equation (D) y = log  e + 1  + k
  dy  2  d 2 y  y +1 
1 +    = 2 are
  dx   dx 41. The solution of differential equation dy = e x − y + x 2e − y is
dx
(A) 2, 3 (B)  2, 3 3
2 (A) y = e x − y − x 2 e − y + C (B)  e y − e x = x + C
(C)  2, 1 (D)  3, 4 3
3 3
(C) e x + e 4 = x + C (D)  e x − e y = x + C
36. The differential equation of family of curves 3 3
y 2 = 4 a ( x + a ) is
42. The solution of differential equation

NCERT EXEMPLARS
(A) y 2 = 4 dy  x + dy  dy 2 xy
+ =
1 is
dx  dx  dx 1 + x 2 ( )
2
1+ x2
(B)  2 y dy = 4 a
dx (2
)
(A) y 1 + x = C + tan x
−1

2 2
d y  dy  y
(C) y + =0 (B)  = C + tan �1 x
dx 2  dx  1+ x 2

dy  dy 
2 (
(C) y log 1 + x = C + tan x
2
)−1

(D) 2 x + y  − y = 0
dx  dx  (D) y 1 + x( 2
) = C + sin −1
x
9.20  Chapter 9

ANSWER K EYS
  1. (D) 2. (D) 3.  (A) 4. (C) 5.  (B) 6. (C) 7.  (C) 8. (D) 9.  (A) 10. (C)
  11.  (B) 12. (B) 13.  (B) 14. (C) 15. (C) 16. (D) 17.  (A) 18. (A) 19.  (B) 20. (B)
  21.  (B) 22. (B) 23. (C) 24. (B) 25. (A) 26. (A) 27.  (A) 28. (C) 29.  (D) 30. (C)
  31.  (C) 32. (C) 33. (D) 34. (D) 35. (C) 36. (D) 37.  (A) 38. (A) 39.  (A) 40. (C)
  41.  (B) 42. (A)

HINTS AND EXPLANATIONS

1. The degree of above differential equation is not defined


Again, differentiating both sides w.r.t. x, we get
 dy  d 2 y − dy − x
because when we expand sin   we get an infinite series = + e ( − cos x − B sin x ) − e − x ( − A sin x + B cos x )
 dx  dx 2 dx
2

in the increasing powers of


dy
. Therefore its degree is not ⇒ d y = − dy − y −  dy + y 
dx 2  dx 
dx dx  
2
defined. ⇒ d y = − dy − y − dy − y
dx 2 dx dx
3/ 2 2
  ⇒ d y = −2 dy − 2 y
3 2
2. Given that 1 +  dy   = d y2
dx 2 dx
  dx   dx
2
On squaring both sides, we get ⇒ d y + 2 dy + 2 y = 0
  dy  2   d 2 y 
3 2 dx 2 dx

1 +    =  2  5. Given, y = Acos α + B sin a
  dx    dx  dy
⇒ = −α A sin α x + α B cos α x

So, the degree of differential equation is 2. dx
1/ 4
3. Given that, d 3 y  dy 
Again, differentiating both sides w.r.t. x, we get
+ = − x1/5
dx 2  dx 
HINTS AND EXPLANATIONS


d2y
1/ 4 = − Aα 2 cos α x − α 2 B sin α x
d y  dy  2
dx 2
⇒ 2 +   = − x1/5
dx  dx  d2y
⇒ = −α 2 ( A cos α x + B sin α x )
 dy 
1/ 4
 d2y  dx 2

⇒ ⇒   = −  x1/5 + 2  d2y
 dx   dx  ⇒ = −a 2 y
dx 2

On squaring both sides, we get d2y
1/ 2 2
⇒ + a2 y − 0
 dy   1/5 d 2 y  dx 2

 dx  = −  x + dx 2 
    6. Given that, xdy − ydx = 0

Again, on squaring both sides, we have
⇒ xdy = ydx
4
dy dx

dy  1/5 d 2 y  ⇒ =
=x + 2  y x
dx  dx  On integrating both sides, we get

order = 2, degree = 4 log y = log x + log C
⇒ log y = log Cx
4. Given that, y = e − x ( A cos x + B sin x ) ,
⇒ y = Cx.

On differentiating both sides w.r.t., x we get Which is a straight line passing through origin.
dy
dy 7. Given, that, cos x + y sin x = 1

= −e − x ( A cos x + B sin x ) + e − x ( − A sin x + B cos x ) dx
dx
dy
dy ⇒ + y tan x = sec x

= − y + e − x ( − A sin x + B cos x ) dx
dx Here, P = tan x and Q = sec x
Differential Equations  9.21


IF = e ∫ = e∫ 1+ y 
⇒ log 
Pdx tan xdx
= e log sec x
∴ = sec x

=x
 2 
8. Given that, tan y sec 2 xdc + tan x sec 2 ydy = 0 ⇒ 1 + y = ex

2
⇒ tan sec 2 xdx = − tan x sec 2 ydy ⇒ 1 + y = 2ex

sec 2 x − sec 2 y  ⇒ y = 2ex – 1

⇒ dx = dy (i)
tan x tan y

On integrating both sides, we have 12. Given that, dy y + 1
+
sec 2 x sec 2 y dx x − 1


∫ tan x dx = − ∫ tan y dy dy dx

=

Put tan x = t in LHS integral, we get y +1 x −1

sec 2 xdx = dt ⇒ sec 2 xdx = dt
On integrating both sides, we get

and tan y = u in RHS integral, we get
log(y + 1) = log(x – 1) – logC
sec2ydy = du

C(y + 1) = (x – 1)

On substituting these values in Eq. (i), we get x −1
dt du ⇒ C =

y +1


∫ t = −∫ u
When x = 1 and y = 2, then C = 0

log t = – log u + log k
So, the required solution is x – 1 = 0.
⇒ log(t.u) = log k

⇒ log(tan x tan y) = log k Hence, only one solution exist.

⇒ tan x tan y = k 13. The second order differential equation is y’y” + y = sin x.

9. Given that, y = Ax + A3 14. (C) Given that, (1 − x ) dy


dx
− xy = 1
2

dy
⇒ = A dt
dx
⇒ 1 − x 2 = t ⇒ −2 xdx = dt ⇒ xdx = −

[we can differential above equation only once because it has 2
only one arbitrary constant]
which is a linear differential equation.
x
∴ Degree = 1

− ∫ 1− x 2 dx
∴ IF = e
10. Given that, dy dt
x= − y = x 4 − 3x
Put 1 − x 2 = t ⇒ −2 xdx = dt ⇒ xdx = −

HINTS AND EXPLANATIONS


dx 2
dy y 1 dt 1 1
( )
⇒ − = x 3 − 3 If = e 2 ∫ t = e 2

Now,
log t log 1− x 2

dx x = e2 = 1− x2

Here, P = − 1 , Q = x 3 − 3
15. Given that, tan −1 x + tan −1 y = C
x
1

On differentiating w.r.t. x, we get
− ∫ dx
∴ IF = e ∫ Pdx =e
x
= e − log x t 1 dy

+ . =0
1 1 + x 2 1 + y 2 dx
=
x ⇒
1 dy 1
. =−
11. Given that, dy − y = 1 1 + y 2 dx 1+ x2
dx
dy ( ) (
⇒ 1 + x 2 dy + 1 + y 2 dx = 0
)
⇒ = 1 + y
dx 16. Given that, dy
y + x =C
dy dx
⇒ = dx
1+ y ⇒ dy
y= =C −x
On integrating both sides, we get dx
log (1 + y) = x + C(i)
⇒ yd y = (C – x)dx
When x = 0 and y = 1, then
On integrating both sides, we get
log 2 = 0 + c y2 x2

= Cx − + K
⇒ C = log 2 2 2
The required solution is x2 y2

+ = Cx + K
Log ( 1 + y) = x + log 2 2 2
9.22  Chapter 9

x2 y2  y−x 

+ − Cx = K ⇒
tan −1  =C
2 2  1 + xy 

Which represent family of circles. y−x

⇒ = tan C
1 + xy
17. Given that, e x cos ydx − e x sin ydy = 0
⇒ y – x = tan c(1 + xy)

e cos ydx = e sin ydy
x x

⇒ y – x = K (1 + xy)
dx
where k = tan C

= tan y
22. Given that,   dy + y = 1 + y
dy

dx = tan ydy dx x

On integrating both sides, we get ⇒ dy 1 +y
= −y

X = log sec y + C dx x
dy 1 + y − xy

x – C = log sec y ⇒ =
dx x

sec y = ex –C
dy 1 y (1 − x )

sec y = exe–C ⇒ = +
1 ex dx x x

= C
cos y e dy  1 − x  1
⇒ − =

ex cos y = eC dx  x  x

ex cos y = K [where , K = eC] − (1 − x ) 1
Here, P= ,Q =
2 3 x x
18. d y2 +  dy  + 6 y 5 = 0 −∫
1− x x −1
∫ =∫ x
Pdx dx dx
dx  dx  IF = e =e x

We know that, the degree of a differential equation is expo-


nent heightest of order derivative.  1
∫ 1− x dx
=e
∴ Degree = 1

= e∫
x − log x


19. Given that, dy + y = e − x 1
log 
dx
= e x ·e x

Here, P = 1, Q = e–x 1

= ex ·
IF = e ∫ = e ∫ = e − x
Pdx dx
x
HINTS AND EXPLANATIONS

The general solution is


y.e x = ∫ e − x e x dx + C 23. Given that, y = ae mx + be − mx
On differentiating both sides w.r.t.x, we get
⇒ y.ex = x + C
dy
When x = 0 and y = 0, then  (i) = mae mx − bme − mx
dx
0 = 0 + C  ⇒ C = 0 Again, differentiating both sides w.r.t.x, we get
Eq. (i) becomes y.ex = x d2y
= m 2 aa mx + bm 2e − mx
⇒ y = xe–x dx 2
20. Given that, dy + y tan x − sec x = 0 d2y
dx

dx 2
= m 2 ae mn + be − mn( )
Here, P = tan x, Q = sec x d2y

= m2 y
IF = e ∫ = e ∫ dx 2
Pdx tan xdx



=e (log sec c)

d2y
− m2 y = 0

= sec x dx 2
dy 1 + y 2 24. Given differential equation is
21. Given that, =
dx 1 + x 2 cos x sin ydx + sin x cos ydy = 0
dy dx ⇒ cosxsinydx = –sin x cos ydy

⇒ =
1+ y 2
1+ x2 ⇒ cos x dx = − cos y dy


On integrating both sides, we get sin x sin y

tan −1 y = tan −1 x + C ⇒ cot x dx = – cot ydy


⇒ tan −1 y − tan −1 x = C
On integratinb oth sides, we get
Differential Equations  9.23


log sin x = – log sin y + log C
2x dx = dt
⇒ logsin x sin y = log C


≡e dy ≡e dt
y t

⇒ sin x. siny = C


⇒ ey = et + C
dy ⇒
2

25. Given that, e y = ex + C


x=
+ y = ex
dx
29. Slope of tangent to the curve = dy
dy y e x dx
⇒ + =
dx x x and ratio of abscissa to the ordinate = x


Which is a linear differential equation. y
1
IF = e ∫ x = e ( According to the question, dy = x
dx log x )
∴ =x
 dx  dx y

∫ x 
The general solution is y. x =  . x dx ]

Yd y = xd x

On integrating both sides, we get
y. x = e x dx ∫x y2 x2

⇒ y.x = e + k
= +C
ex k 2 2

y= + 2
x x ⇒
y x2
− = C ⇒ y 2 − x 2 = 2C
2 2
26. Given equation of curve is

Which is an equation of rectangular hyperbola.
x 2 + y 2 − 2ay = 0
x2 + y2 30. Given that, dy 2

⇒ = 2a = e x / 2 + xy
y dx

On differentiating both sides w.r.t. x, we get ⇒ dy 2
− xy = e x / 2
 dy  dy dx
y  2x + 2 y  − x2 + y2
 dx 
(
dx
)
Here,
2

=0 P = − x, Q = e x /2

y2 − x2 / 2

IF = e ∫
e 2
− e x / 2 dx + C
⇒ 2 xy + 2 y 2 dy − x 2 + y 2 dy = 0
( )
The general solution is
dx dx

2 2 2

dy
y.e − x / 2 = e − x / 2 − e x / 2 dx + C
(
⇒ 2y − x − y
2 2 2
)
= −2 xy

HINTS AND EXPLANATIONS


dx
=∫
1dx +C

y.e − x2 /2
dy
(
⇒ y2 − x2
dx
)
= −2 xy
⇒ y.e − x
2
/2
= x +C

dy
(
⇒ y2 − x2
dx
)
= 2 xy
⇒ y = xe x
2
/2
= x +C


2 2
27. Given family of curves is y = Ax + A3 y = xe
x /2
+ Ce + x /2


dy ⇒ y = ( x + C )ex
2
/2
=A
dx
31. Given that, (2y – 1) dx – (2x + 3)d y = 0
Replacing A by dy in Eq. (i), we get

dx
⇒ (2y – 1)dx = (2x + 3)dy
dy  dy 
3
dx dy

y=x + ⇒ =
dx  dx  2x + 3 2 y −1

On integrating both sides, we get

Order = 1
1 1
dy 2 2
log ( 2 x + 3) = log ( 2 y − 1) + log C
28. Given that, = 2 xe x − y = 2 xe x ·e − y 2 2
dx
1
⇒ log. ( 2 x + 3) − log ( 2 y − 1)  = log C

dy
2 
2

⇒ e y = 2 xe x dx
dx
1  2x + 3 

⇒ e y dy = 2 xe x dx
2
⇒ log   = log C
2  2 y −1 

On integrating both sides, we get 1/ 2
 2x + 3 
∫ ∫
2

e y dy = 2 xe x dx
⇒  =C

 2 y −1 
Put x2 = t in RHS integral, we get

9.24  Chapter 9

⇒ 2x + 3 = C 2
d2y dy
37. Given that,
−2 + y =0
2 y −1 dx 2 dx
⇒ x + 3 = k , where K = C2

2 Where D = d

2 y −1 dx
32. Given that, y = a cos x + b sin x
D y − 2 Dy + y = 0,
2

On differentiating both sides w.r.t. x, we get


The auxiliary equation is m 2 − 2m + 1 = 0
dy
( m − 1)
2
= − a sin x + b cos x
= 0 ⇒ m = 1,1
dx

Since, the roots are real and equal.
Again, differentiating w.r.t.x, we get
d2y

∴ CF = ( Ax + B ) e x ⇒ y = ( Ax + B ) e x
= − a sin x + b cos x
dx 2
[since, if root of Auxiliary equation are real and equal say
d2y (m), then CF = (C1 x + C2 ) e mx ]
⇒ = −y
dx 2 38. Given differential equation is
2
d y dy
⇒ + y=0 + y + tan x = sec x
dx 2 dx
dy Which is a linear differential equation
33. Given that, + y = e− x
dx Here, P = tan x, Q = sec x,
IF = e ∫
tan xdx

Which is a linear differential equation. ∴ =e
log sec x
= sec x
Here, P = 1 and Q = e–x

The general solution is
IF = e ∫ = e − x
dx


y.sec x = ∫ sec x.sec x + C

The general solution is

⇒ y.sec x = ∫ sec 2 xdx + C



y.e x = e − x .e x dx + C

⇒ y.sec x = tan x + C

ye x
= ∫ dx + C
⇒ yex = x + C(i) 39. Given differential equation is

When x = 0 and y = 0 then, 0 = 0 + C  ⇒ C = 0
dy y
+ = sin x
Eq. (i) becomes y.ex  ⇒ y = xe–x dx x

Which is linear differential equation.
HINTS AND EXPLANATIONS

2 4
 d3 y  d2 y  dy  1 and Q = sin x
 3  − 3 2 + 2  = y
4
34. Given that,
Here, P=
 dx  dx  
dx x
∴ Order = 3 1
IF = e ∫ x e log x + C 
dx

∴ (i)
and degree = 2
 2
 2

Take I = ∫ x sin x dx
35. Given that, 1 +  dy   = d y2
  dx   dx
− x cos x − ∫ − cos x dx
∴ Order = 2 and degree = 1

= – x cos x + sin x
36. Given that, y2 = 4a(x + a)(i)
Put the value of I in Eq. (i), we get

On differentiating both sides w.r.t. x, we get
xy = – x cos x + sin x + C

dy dy
⇒ x(y + cos x) = sin x + C
2y = 4a ⇒ 2 y = 4a
dx dx
dy 1 dy 40. Given differential equation
⇒ y = 2a ⇒ a = y
2 dx  (ii)
dx
(e x
)
+ 1 ydy = ( y + 1) e x dx

On putting the value of a from Eq. (ii) in Eq. (i) we get
dy  1 dy 

dy e (1 + y )
x

= x ⇒
dx ex +1 y
= x
( )

y = 2y  x + y 
2

dx  2 dx 
dx (
e +1 y )
dy e (1 + y )
x
dy  dy 
2
dx e y y

⇒ y 2 = 2 xy + y 2   ⇒ = +
dx  dx  dy e x (1 + y ) e x (1 + y )
2
dy  dy  dx y y

⇒ 2x + y   − y = 0 ⇒ = +
dx  dx  dy 1 + y (1 + y ) e x
Differential Equations  9.25

dx y  1
On integrating both sides, we get

= 1+
dy 1 + y  e x 

∫ ∫(
e y dy = e x + x 2 dx )
dx y  ex +1  x 3
⇒ =  
⇒ e y = ex + +C
dy 1 + y  e x  3
 y   ex  ⇒ x3

⇒   dy =  x  dx e y − ex = + C
1+ y   e +1 3
On integrating both sides, we get dy 2 xy 1
42. Given that, + =
ex dx 1 + x 2 ( )
2
y 1+ x2

∫ 1 + y dy = ∫ 1 + e x dx
1 + y −1 ex Here, P = 2 x and Q =
1
⇒ ∫ 1 + y dy = ∫ 1 + e x dx 1 + x2 1 + x2 ( )
2

1 ex
Which is a linear differential equation.
⇒ ∫ 1 dy − ∫ 1 + y ∫ 1 + e x dx
dy =


2x
∫ 2 dx
IF = e 1+ x
x
⇒ y – log |(1 + y) = log| (1 + e ) + log k
Put 1 + x 2 = t ⇒ 2 xdx = dt
x
⇒ y = log (1 + y) + log (1 + e ) + log (k) dt
log(1+ x 2 )
⇒ y = log {k (1 + y) (1 + e )}
x
∴ IF = e ∫ t = e log t = e = 1+ x2
41. Given that,
The general solution is
1
dy
= e x − y + x 2e − y
( ) (
y. 1 + x 2 = ∫ 1 + x 2 ) +C
( )
2
dx 1+ x2
dy
⇒ = e x e − y + x 2e − y 1
dx ⇒
(
y 1+ x2 ) =∫
1+ x2
dx + C
⇒ dy e x + x 2

dx
=
ey

( )
y 1 + x 2 = tan −1 x + C
⇒ ( )
e y dy = e x + x 2 dx

HINTS AND EXPLANATIONS


9.26  Chapter 9

PRACTICE EXERCISES

Single Option Correct Type

d (ϕ ( x ))
y − y2
dy dx
1. Solution of equation = is (A) y = (f (x) – 1) + c e–f (x)
dx ϕ( x) (B)  yf (x) = (f (x))2 + c
(C) yef (x) = f (x) ef (x) + c
ϕ( x) + c ϕ( x)
(A) y = (B)  y = +c (D) (y – f (x)) = (f (x)) e–f (x)
x x
ϕ( x) 7. Solution of the differential equation
(C) y = (D)  y = f (x) + x + c 2 2 2 3
x+c ⎛ dy ⎞ ( x y ) ⎛ dy ⎞ ( xy )3 ⎛ dy ⎞
x = 1 + ⎜ xy ⎟ + ⎜ ⎟ + ⎜ ⎟ +…
⎝ dx ⎠ 2 ! ⎝ dx ⎠ 3! ⎝ dx ⎠
2. The family passing through (0, 0) and satisfying the is
y dn y (A) y = ln (x) + c (B)  y = (ln x)2 + c
differential equation 2 = 1 (where yn= ) is
y1 dx n (C) y = ± xy = xy+ k
(ln x ) 2 + c (D) 
(A) y = k (B)  y = kx
8. The orthogonal trajectories of the family of semi-­
(C) y = k (ex+ 1) (D)  y = k (ex– 1) cubical parabola is given by
3. The solution of differential equation (A) x2 + 3y2 = c2 (B) 3x2 + y2 = c2
2 2
dy (C) x+ 3y = c (D) 3y2 + 2x2 = c2
sec2 y + 2x tan y = x3 is
dx 9. Solution of the differential equation
1 2 dy ⎛π⎞
(x – 1) + ce − x = 2sin x cos x – y2 cos x satisfying y ⎜ ⎟
2

(A) tan y = 2y sin x
2 dx ⎝ 2⎠
1 = 1 is given by
(B) tan y = (x2 – 1) + ce x
2


2 (A) y2 = sin x (B)  y = sin2 x
1 (C) y2 = cos x + 1 (D)  y2 sin x = 4 cos2 x
(C) tan y = (x2 + 1) + ce − x
2

2 10. The solution of the differential equation


1 y dx – x dy+ (log x) dx = 0 is
(D) tan y = (x2 + 1) + ce x
2


PRACTICE EXERCISES

2 (A) y = log x + cx (B)  y = 1 + log x + c


4. Solution of the differential equation ydx + (x + x2y) 1
(C) y + cx = log (D)  None of these
dy = 0 is x
1
(A) log y = Cx (B) – + log y = C 11. Solution of
xy
(x2 sin3 y – y2 cos x) dx + (x3 cos y sin2 y – 2y sin x)
1 1
(C)  + log y = C =C
(D) – dy = 0 is
xy xy
x 3 sin 3 y
dy x2 + x + 1 (A)  =c
5. The family of curves represented by = 2 3
dx y + y +1 (B)  x3 sin3 y = y2 sin x + c
dy y2 + y + 1
and the family represented by + 2 =0 x 3 sin 3 y
dx x + x +1 (C)  = y2 sin x + c
(A)  Touch each other 3
(B)  Are orthogonal (D)  None of these
(C)  Are one and the same 12. The equation of the curve for which the square of
(D)  None of these the ordinate is twice the rectangle contained by the
6. If f (x) is a differentiable function then the solution of abscissa and the x-intercept of the normal and passing
dy+ (y f ′(x) – f (x) f ′(x)) dx = 0 is through (2, 1) is
Differential Equations  9.27

(A) x2 + y2 – x = 0 (B) 4x2 + 2y2 – 9y = 0


2 2 ( x + y)2 + 2
(C) 2x + 4y – 9x = 0 (D)  4x2 + 2y2 – 9x = 0 (C) log = 2(x – y)
2
13. The solution of the differential equation
dy (D)  None of these
x+ y 4
dx = x2 + 2y2 + y is
18. Solution of
dy x2
y−x
dx (y + x xy (x + y) + y) dx + (y xy (x + y) – x) dy = 0 is

y 1 y 1 y
(A)  + 2 = c (B)  − 2 =c
4 x + y2 x x + y2 (A) x2 + y2 = 2 tan–1 x +c
x 1 y
(C)  − 2 = c (D)  None of these
y x + y2 (B)  x2 + y2 = 4 tan–1
x +c
14. The solution of the differential equation y
(x – y)(2dy – dx) = 3dx – 5dy is (C) x2 + y2 = tan–1 x +c
(A) 2x – y = log(x – y + z) + c (D)  None of these
(B) 2x + y = log(x – y + z) + c
(C) 2y – x = log(x – y + z) + c 19. A particle starts at the origin and moves along the
(D)  None of these x-axis in such a way that its velocity at the point
dx
(x, 0) is given by the formula = cos2 π x. Then the
x dx + y dy 1 − ( x2 + y2 ) dt
15. Solution of = is particle never reaches the point on
x dy − y dx x2 + y2
1 3
2 2 (A) x = (B)  x =
(A) sin–1 x + y = c 4 4
y 1
(C) x = (D)  x=1
(B) tan–1 x = c 2

y 20. The equation of the curve for which the cartesian


(C) sin x 2 + y 2 = tan–1 = c
–1 ­subtangent varies as the reciprocal of the square of the
x
abscissa, is
(D)  None of these 3 2

(A) x = ce y /3k (B)  x = ce y /3k


16. Which of the following does not represent the 3

(C) y = ce x /3k (D)  None of these


2
⎛ dy ⎞

PRACTICE EXERCISES
­orthogonal trajectory of the system of curves ⎜ ⎟
⎝ dx ⎠ dy y ( x log y − y )
a 21. Solution of = is
= dx x ( y log x − x )
x (A) xy = cyx (B)  xy = c
(A) 9a(y + c)2 = 4x3 (C) (xy)x = c (D)  None of these
−2 3/2

(B)  y + c = x dy ax + 3
3 a 22. If the solution of = represents a circle,
dx 2y + f
2 3/2 then the value of a is
(C) y + c = x
3 a (A) 2 (B) –2 (C) 3 (D) –4
(D)  All are orthogonal trajectories

⎛ x + y − 1 ⎞ dy ⎛ x + y + 1 ⎞ x dy ⎛ y ⎞
17. Solution of ⎜ = , given that 23. Solution of = ⎜ 2 − 1⎟ dx is
⎝ x + y − 2 ⎟⎠ dx ⎜⎝ x + y + 2 ⎟⎠
2 2 2
x +y ⎝x +y ⎠
y = 1 when x = 1, is
y y
( x − y)2 − 2 (A) x– tan–1 x = c (B) tan–1 x = c
(A) log = 2(x + y)
2
y
( x − y)2 + 2 (C) x + tan–1 x = c (D)  None of these
(B) log
= 2(x – y)
2
9.28  Chapter 9

24. If y = y(x) and 31. The equation of the curve which passes through the
point (2a, a) and for which the sum of the cartesian
2 + sin x ⎛ dy ⎞ ⎛π⎞ sub tangent and the abscissa is equal to the constant a,
⎜⎝ ⎟⎠ = – cos x, y(0) = 1, then y ⎜⎝ ⎟⎠ = is
y + 1 dx 2
1 2 1 (A) y (x – a) = a2 (B)  y (x + a) = a2
(A)  (B)  (C) – (D) 1 (C) x (y – a) = a 2
(D)  x (y + a) = a2
3 3 3
25. The equation of the curve passing through the point 32. Solution of differential equation
2ay ⎛ d g( x) ⎞ 2
(1, 1) and having slope is t⎜ −t
x ( y − a) dt ⎝ dx ⎟⎠
= is
(A) ya ⋅ x2a = e y dx g( x)
(B)  ya ⋅ x2a = e y – 1
(C) y2a ⋅ xa = e y g( x) + c g( x)
(A) t = (B)  +c
(D)  None of these x x
g( x)
26. The solution of the differential equation (C) t = (D)  t = g(x) + x + c
x+c
(x cos x – sin x + yx2) dx + x3dy = 0 is equal to
sin x sin x 33. If the curve y = f (x) passing through the point (1, 2)
(A)  + xy = c (B)  +x=c
x x and satisfies the differential equation xdy + (y – x3y2)
sin x dx = 0, then
(C)  + y = c (D)  None of these 1
x (A) xy = (B)  x3y = 2
2
27. The solution of the differential equation 1
3 (C)  = 2 (D)  None of these
y dx – x dy + 3x2y2e x dx = 0 is xy
x 3 x 3

(A)  + e x = c (B)  – ex = c 34. Let I be the purchase value of an equipment and V(t)
y y be the value after it has been used for t years. The value
y 3 y 3 V(t) depreciates at a rate given by differential equation
(C)  + e x = c (D)  – ex = c dV (t )
x x = –k(T – t), where k > 0 is a constant and T is
dt
the total life in years of the equipment. Then the scrap
28. Solution of the differential equation value V(T) of the equipment is
PRACTICE EXERCISES

[y (1 + x–1) + sin y] dx + (x + log x + x cos y) dy = 0 is


(A) xy+ y log x = c
(B)  xy+ x sin y = c I
(A) e–kT (B) 
T2 –
(C) xy+ y log x + x sin y = c k
(D)  None of these kT 2 k (T − t ) 2
(C) I – I−
(D) 
29. The differential equation of the curve for which the 2 2
normal at every point passes through a fixed point dy
(h, k) is 35. If = y + 3 > 0 and y(0) = 2, then y (ln 2) is equal to
dx
dx dx
(A) y – k = (h – x) (B)  y–k= (x – h) (A) –2 (B) 7 (C) 5 (D) 13
dy dy
dy dy
(C) y – k = (h – x) (D)  y – k = (x – h) 36. The population p(t) at time t of a certain mouse
dx dx dp(t )
species satisfies the differential equation
30. Solution of the differential equation dt
= 0.5 p(t) – 450. If p(0) = 850, then the time at which
[3xy2 + x sin (xy)] dy+ [y3 + y sin (xy)] dx = 0 is the population become zero is
(A) xy3 – cos xy = c (B)  xy3 + cos xy = c (A)  2 ln 18 (B)  ln 9
2
(C) xy – cos xy = c (D)  xy2 + sin xy = c (C)  1/2 ln 18 (D)  ln 18
Differential Equations  9.29

dy 1 − y2 ⎛ y⎞
37. The differential equation = determines a (C)  f ⎜ ⎟ = cxy
(D)  None of these
y ⎝ x⎠
family of circles with dx
(A)  variable radius and fixed centre dy
44. Solution of the equation = ex – y (ex – ey) is
(B)  variable radius and variable centre dx
(C)  fixed radius and variable centre on x-axis (A) ey= e x − 1 + ce − e (B) 
ey = e x − 1 + ce e
x x

(D)  fixed radius and variable centre on y-axis


(C) ex = e y − 1 + ce − e
y

(D)  None of these


dy x+ y
38. The solution of the differential equation = ⎛ dy ⎞ dy
2

satisfying the condition y(1) = 1 is dx x 45. Solution of the equation x ⎜ ⎟ + ( y − x ) −y


⎝ dx ⎠ dx
(A) y = ln x + x (B)  y = x ln x + x2 = 0 is
(C) y = xe(x – 1) (D)  y = x ln x + x (A) (x – y + c) (xy – c) = 0
(B) (x + y + c) (xy – c) = 0
39. The differential equation of the family of circles with (C) (x – y + c) (2xy – c) = 0
fixed radius 5 units and centre on the line y = 2 is (D) ( y – x + c) (xy – c) = 0
(A) (x – 2)y′2 = 25 – (y – 2)2
(B) (y – 2)y′2 = 25 – (y – 2)2 46. The differential equation of the family of general
(C) (y – 2)2y′2 = 25 – (y – 2)2 ­circles is
(D) (x – 2)2y′2 = 25 – (y – 2)2 (A) y′′′ (1 + y′ 2) – 3y′y″ 2 = 0
40. The differential equation which represents the family (B) y′′′ (1 + y′ 2) + 3y′y″ 2 = 0
of curves y = c1e c x, where c1 and c2 are arbitrary con-
2 (C) y′′′ (1 + y′ 2) – 3y″y′ 2 = 0
stants is (D)  None of these
(A) y′ = y2 (B)  y″ = y′y 47. The equation of the family of curves which intersect
(C) y″ = y′ (D)  yy″ = (y′)2 the hyperbola xy = 2 orthogonally is
41. The general solution of the differential equation x3 x2
(A) y = + C (B) 
y= +C
dy 6 4
+ y g′ (x) = g (x) . g′ (x),
dx x3 x2
where g (x) is a given function of x, is (C) y = − +C y= −
(D)  +C
6 4
(A) g (x) + log [1 + y + g (x)] = C
(B) g (x) + log [1 + y – g (x)] = C 48. The solution of the differential equation
(C) g (x) – log [1 + y – g (x)] = C dy y
x 2 − xy = 1 + cos is
(D)  None of these dx x

PRACTICE EXERCISES
xdy − ydx
42. Solution of the equation xdx + ydy + = 0 is y c
x2 + y2 (A) cos =1+
x x
⎛ c + x2 + y2 ⎞ y
(A) y = x tan ⎜ ⎟ (B)  x2 = (c + x2) tan
⎝ 2 ⎠ x
⎛c + x + y 2 2⎞ y 1
(C) tan =c–
(B) x = y tan ⎜ ⎟ 2x 2x2
⎝ 2 ⎠
y 1
⎛ c − x2 − y2 ⎞ (D) tan = c +
(C) y = x tan ⎜ x x

⎝ 2 ⎠
49. Solution of the differential equation
(D)  None of these
⎛ x + y − 1 ⎞ dy ⎛ x + y + 1 ⎞
⎛ f ( y / x) ⎞ ⎜⎝ x + y − 2 ⎟⎠ dx = ⎜⎝ x + y + 2 ⎟⎠ , given that y = 1 when
43. Solution of the equation x dy = ⎜ y + x dx is
⎝ f’ ( y / x ) ⎟⎠
( x + y)k − k
⎛ x⎞ ⎛ y⎞ x = 1, is k(y – x) + log = 0, where k =
(A)  f ⎜ ⎟ = cy (B)  f ⎜ ⎟ = cx k
⎝ y⎠ ⎝ x⎠
(A) 1 (B) 2 (C) 3 (D) 4
9.30  Chapter 9

50. Solution of the equation xdy – [ y + xy3 (1 + log x)] 56. Solution of the differential equation
dx = 0 is [y (1 + x–1) + sin y] dx + (x + log x + x cos y) dy = 0 is
−x 2 2 x3 ⎛ 2 ⎞ (A) xy+ y log x = c
(A)  2 = ⎜⎝ + log x ⎟⎠ + C
y 3 3 (B)  xy+ x sin y = c
(C) xy+ y log x + x sin y = c
x2 2 x3 ⎛ 2 ⎞ (D)  None of these
(B)  = ⎜⎝ + log x ⎟⎠ + C
y2 3 3
57. The equation of the curve, passing through (2, 5) and
−x 2 x3 ⎛ 2 ⎞ having the area of triangle formed by the x-axis, the
(C)  = ⎜⎝ + log x ⎟⎠ + C
y2 3 3 ordinate of a point on the curve and the tangent at the
point 5 square units, is
(D)  None of these
(A) xy = 10 (B)  x2 = 10 y
d (ϕ ( x ))
y − y2 (C) y2 = 10x (D)  xy1/2 = 10
dy dx
51. Solution of equation = is
dx ϕ( x) 58. If the curve y = f (x) passing through the point (1, 2)
ϕ( x) + c ϕ( x) and satisfies the differential equation xdy+ (y + x3y2)
(A) y = (B)  y = +c dx = 0, then
x x 1
ϕ( x) (A) xy = (B)  x3y = 2
(C) y = (D)  y = f (x) + x + c 2
x+c 1
(C)  = 2 (D)  None of these
52. Solution of the differential equation ydx + (x + x2y) xy
dy = 0 is 59. The solution of the differential equation
1
(A) log y = Cx (B) – + log y = C
xy x dx + y dy a2 − x 2 − y 2
= is
1 1 x dy − y dx x2 + y2
(C)  + log y = C (D) – =C
xy xy ⎧ y⎫
(A)  x 2 + y 2 = a cos ⎨c + tan −1 ⎬
53. The equation of the curve for which the square of ⎩ x⎭
the ordinate is twice the rectangle contained by the ⎧ y⎫
abscissa and the x-intercept of the normal and passing (B)  x 2 + y 2 = a sin ⎨c + tan −1 ⎬
⎩ x ⎭
through (2, 1) is
⎧ x ⎫
(A) x2 + y2 – x = 0 (B) 4x2 + 2y2 – 9y = 0 (C)  x 2 + y 2 = a sin ⎨c + tan −1 ⎬
2 2 ⎩ y⎭
(C) 2x + 4y – 9x = 0 (D)  4x2 + 2y2 – 9x = 0
PRACTICE EXERCISES

(D)  None of these


60. The solution of the equation
x dx + y dy 1 − ( x2 + y2 )
54. Solution of = is ⎛ x2 ⎞
x dy − y dx x2 + y2 (2x log y) dx + ⎜ + 3 y 2 ⎟ dy = 0 is
⎝ y ⎠
2 2
(A) sin–1 x + y = c (A) x2 log y + y3 = c (B) y3 log x + x3 = c
y (C) x2 log y – y3 = c (D)  None of these
(B) tan–1 x = c
61. The solution of the equation
2 2 y
(C) sin–1 x + y = tan–1 x +c y + sin x cos 2 ( xy ) ⎛ x ⎞
2
dx + ⎜ 2
+ sin y ⎟ dy = 0 is
(D)  None of these cos ( xy ) ⎝ cos ( xy ) ⎠

55. The solution of the differential equation (x cos x – sin (A)  tan (xy) + cos x – cos y = c
x + yx2) dx + x3dy = 0 is equal to (B)  tan (xy) – cos x – cos y = c
sin x sin x (C)  tan (xy) + cos x + cos y = c
(A)  + xy = c (B)  +x=c (D)  None of these
x x
sin x dy y
(C)  + y = c (D)  None of these 62. The solution of the equation = is
x dx 2 y log y + y − x
Differential Equations  9.31

c c 69. The equation of the curve satisfying the differential


(A) x = y log y + (B) y = x log x + equation
y x
dy
c x − y = | x2 − y2 |
(C) x = –y log y + (D)  None of these dx
y and passing through the point (1, 0) is
dy 1
63. Solution of the equation cos2x – y tan 2x = cos4x, (A) ( 2 x − 1) + =0
dx
x − y2
2

π ⎛π⎞ 3 3 1
when | x | < and y ⎜ ⎟ = , is (B) (3 x − 2) + =0
4 ⎝ 6⎠ 8 x − y2
2

sin 2 x sin 2 x 1
(A) y = y=
(B)  (C) ( 2 x − 3) + =0
(
2 tan x − 1 2
)
2 1 − tan 2 x ( ) x − y2 2

(D)  None of these


sin 2 x
(C) y = (D)  None of these dy
2 (1 + tan 2 x ) 70. Solution of the differential equation 2y sin x =
dx
dy
+ x(x + y) = ⎛π⎞
64. The solution of the equation 2 sin x cos x – y2cos x satisfying y ⎜ ⎟ = 1 is given by
3 3
x (x + y) – 1 is dx ⎝ 2⎠
2
(A) y = sin x (B)  y = sin2x
(A) (x + y)–3 = cex2 + x2 + 1 2
(C) y = cos x + 1 (D)  y2 sin x = 4 cos2x
(B) (x + y)–2 = cex2 – x2 + 1
(C) (x + y)–2 = cex2 + x2 + 1 71. The solution of the equation
(D)  None of these y (2x2y + ex) dx – (ex+ y3) dy = 0, if y(0) = 1, is
dy (A) 6ex– 4x3y – 3y3 – 3y = 0
65. The solution of the equation sin y = cos y (1 – x
cos y) is dx (B) 6ex+ 4x3y – 3y3 – 3y = 0
(A) sec y = (1 + x) + cex (C) 6ex+ 4x3y + 3y3 – 3y = 0

(B) tan y = (1 + x) + cex (D)  None of these
(C) sec y = (1 + x) + ce–x 72. The solution of the equation ye–x/y dx – (xe–x/y + y3)

(D)  None of these dy = 0 is
66. The solution of the differential equation (A) 3e–x/y+ y2 = c (B) 2e–x/y+ y2 = c
–x/y 2
(C) 2e + y = c (D)  None of these
x(y2exy + ex/y)dy = y(ex/y– y2exy) dx is
(A) xy = ln(ey/x+ c) xy = ln(ex/y+ c)

PRACTICE EXERCISES
(B)  73. The differential equation corresponding to
y x 3
(C)  = ln(exy+ c) (D)  = ln(exy+ c)
x y y = ∑ ci e m x ,
i
where ci’s are arbitrary constants and
i =1
67. Solution of the equation m1, m2, m3 are roots of the equation m3 – 7m + 6 = 0, is
x x
x ∫ y(t ) dt = ( x + 1) ∫ ty(t ) dt , x > 0 is (A) y3 – 7y1 + 6y = 0 (B)  y3 + 7y1 + 6y = 0
0 0 (C) y3 – 7y1 – 6y = 0 (D)  None of these
1 1
c − c
(A) y = e x (B) y = 3 ex 74. If y = c1e2x+ c2ex+ c3e–xsatisfies the differential
x3 x
1 1 d3 y d2 y dy
c − c equation +a +b + cy = 0,
(C) y = e x (D)  y = ex 3 2
3 3

dx dx dx
x x
68. The solution of the differential equation
(1 + tan y) (dx– dy) + 2xdy = 0 is a3 + b3 + c 3
then is equal to
(A) x (sin y + cos y) = sin y + ce–y abc
(B) x (sin y – cos y) = sin y + ce–y 1 1 1 1
(C) x (sin y + cos y) = cos y + ce–y − (C) 
(A)  (B)  (D) –
(D)  None of these 4 4 2 2
9.32  Chapter 9

Previous Year’s Questions


75. The order and degree of the differential equation
1 1
⎛ dy ⎞
2/3
d3 y (A) − = C (B) 
− + log y = C
⎜⎝ 1 + 3 ⎟ = 4 are [2002] xy xy
dx ⎠ dx 3
1
⎛ 2⎞ (C)  + log y = C (D)  log y = Cx
(A)  ⎜1, ⎟ (B) 
(3, 1) xy
⎝ 3⎠
82. The differential equation representing the family of
(C)  (3, 3) (D)  (l, 2)
( )
curves y 2 = 2c x + c where c > 0, is a parameter,
2 is of order and degree as follows: [2005]
d y
76. The solution of the equation = e −2 x is (A)  order 1, degree 2 (B)  order 1, degree 1
dx 2
 [2002] (C)  order 1, degree 3 (D)  order 2, degree 2
dy
e −2 x e −2 x 83. If x = y(log y − log x + 1) , then the solution of the
(A)  (B)  + cx + d dx
4 4 equation is [2005]
1 1
(C)  e −2 x + cx 2 + d (D)  e −2 x + c + d ⎛ x⎞ ⎛ y⎞
4 4 (A) y log ⎜ ⎟ = cx (B)  x log ⎜ ⎟ = cy
⎝ y⎠ ⎝ x⎠
77. The differential equation of all non-vertical lines in a
plane is [2002] ⎛ y⎞ ⎛ x⎞
(C) log ⎜ ⎟ = cx (D) log ⎜ ⎟ = cy
d2 y d2x ⎝ x⎠ ⎝ y⎠
(A)  2 = 0 (B)  =0
dx dy 2 84. The differential equation whose solution is Ax2 + By2
dy dx = 1, where A and B are arbitrary constants is of
(C)  = 0 (D)  =0  [2006]
dx dy
(A)  second order and second degree
78. The degree and order of the differential equation of (B)  first order and second degree
the family of all parabolas whose axis is x-axis, are (C)  first order and first degree
respectively [2003] (D)  second order and first degree
(A)  2, 1 (B)  1, 2 (C)  3, 2 (D)  2, 3 85. The differential equation of all circles passing through
79. The solution of the differential equation the origin and having their centres on the x-axis is
2 tan −1 y dy  [2007]
(1 + y ) + ( x − e ) = 0 , is [2003]
PRACTICE EXERCISES

dx dy dy
(A) x 2 = y 2 + xy (B)  x 2 = y 2 + 3 xy
− tan −1
y dx dx
(A) ( x − 2) = k e
2 2 dy 2 2 dy
tan 2 tan −1 −1
(C) y = x + 2 xy (D)  y = x − 2 xy
(B) 2 x e y = e +k dx dx
(C) x e tan y = tan −1 y + k
−1

dy x + y
86. The solution of the differential equation =
(D) x e 2 tan −1 y
=e tan −1 y
+k dx x
satisfying the condition y (1) = 1 is [2008]
80. The differential equation for the family of curves (A)  y = x ln x + x2
y = ln x + x (B) 
(x−1)
x2 + y2− 2ay = 0, where a is an arbitrary constant is y = xe
(C)  (D) y = x ln x + x
 [2004]
2 2 87. The differential equation of the family of circles with
(A) 2(x − y )y′ = xy fixed radius 5 units and centre on the line y = 2 is
(B) 2(x2 + y2)y′ = xy  [2008]
(C) (x2 − y2 )y′ = 2xy (A) (x − 2)y′2 = 25 − (y − 2)2
(D) (x2 + y2 )y′ = 2xy (B) (y − 2)y′2 = 25 − (y − 2)2
81. The solution of the differential equation y dx + (x + (C) (y − 2)2y′2 = 25 − (y − 2)2
x2y) dy = 0 is [2004] (D) (x − 2)2y′2 = 25 − (y − 2)2
Differential Equations  9.33

88. The differential equation which represents the fam- to: [2015]
ily of curves y = c1e c x, where c1 and c2 are arbitrary
2
(A) 0 (B) 2
constants is [2009] (C) 2e (D) e
(A)  y′ = y2 (B)  y′′ = y′y 95. If a curve y = f (x) passes through the point (1, –1) and
(C)  yy′ = y′ (D)  yy′ = (y′)2 satisfies the differential equation, y(1+ xy )dx = x dy.
89. Solution to the differential equation cos x dy = y(sin ⎛ 1⎞
π then f ⎜ − ⎟ is equal to: [2016]
x−y) dx, 0 < x < is [2010] ⎝ 2⎠
2 4 2
(A)  y secx = tan x + c (B)  y tan x = sec x + c (A)  (B) −
(C) tan x = (sec x + c)y (D) sec x = (tan x + c)y 5 5
4 2
90. Let l be the purchase value of an equipment and V(t) (C) − (D) 
5 5
be the value of equipment after it has been used for t
years. The value V(t) depreciates at a rate given by the dy
96. If (2 + sin x) + (y + 1) cos x = 0 and y(0) = 1, then
dV (t ) dx
differential equation = k (T − t ) , where k > 0 is ⎛π⎞
dt y ⎜ ⎟ is equal to [2017]
a constant and T is the total life in years of the equip- ⎝ 2⎠
ment. Then, the scrap value V(T) of the equipment is 2 1
(A) – (B)  –
 [2011] 3 3
kT 2 k (T − t ) 2 4 1
(A) l − (B)  l− (C)  (D) 
2 2 3 3
l
(C)  e–kT (D)  T 2 − 97. Let y = y(x) be the solution of the differential equation
k
dy π 
91. The population p(t) at time t of a certain mouse s­ pecies sin x + y cos = 4x, x ∈ (0, π). If y   = 0, then
dp(t ) dx 2
satisfies the differential equation = 0.5 p(t ) π
⎛ ⎞
dt y ⎜ ⎟ is equal to [2018]
– 450 with initial condition p(0) = 850, then the value ⎝6⎠
of t for which p(t) = 0 is [2012] 4 2 −8 2
(A)  2 ln 18 (B)  ln 9 (A)  π (B)  π
9 3 9 3
1
(C)  ln 18 (D)  ln 18 8 4
2 (C)  − π 2 (D)  − π 2
92. At present, a firm is manufacturing 2000 items. It is 9 9

PRACTICE EXERCISES
estimated that the rate of change of production P with 98. Let f be a differentiable function such that f'(x) = 7
respect to additional number of workers x is given
1
– 3 ( ) , ( x > 0 ) and f(1) ≠ 4. Then lim
dP f x xf   :
by = 100 − 12 x . If the firm employs 25 more x → 01 x
dx 4 x
workers, then the new level of production of items is  [2019]
 [2013] (A) does not exist.
(A) 3000 (B) 3500 (C) 4500 (D) 2500 (B) exists and equals 4.
93. Let the population of rabbits surviving at a 4
(C) exists and equals
time t be governed by the differential equation 7
dp(t ) 1 (D) exists and equals 0.
= p(t ) − 200 . If initially p(0) = 100, then p(t)
dt 2
equals [2014] 99. The curve amongst the family of curves represented
by the differential equation, (x2 – y2) dx + 2xy dy = 0
(A)  400 – 300 e t/2
(B)  300 – 200 e–t/2 which passes through (1, 1) is: [2019]
(C)  600 – 500 e t/2
(D)  400 – 300 e–t/2 (A) a circle with centre on the x-axis.
94. Let y(x) be the solution of the differential equation (B) a circle with centre on the y-axis.
dy (C) an ellipse with major axis along the y-axis.
( x log x ) + y = 2 x log x,( x ≥ 1) . Then y(e) is equal (D) a hyperbola with transverse axis along the x-axis.
dx
9.34  Chapter 9

x 1
6 24
100. If ∫ f ( t ) dt = x 2 + ∫ t 2 f ( t ) dt , then f' (1/2) is: (A)  (B) 
25 25
0 x
[2019] 4 18
(C)  (D) 
5 25

ANSWER K EYS
Single Option Correct Type
1. (C) 2. (D) 3. (A) 4. (B) 5.  (B) 6. (A) 7.  (C) 8. (D) 9.  (A) 10. (D)
11.  (C) 12. (D) 13.  (B) 14. (C) 15. (C) 16. (D) 17.  (D) 18. (B) 19.  (C) 20. (C)
21.  (A) 22. (B) 23.  (C) 24. (A) 25.  (D) 26. (A) 27.  (C) 28. (C) 29.  (A) 30. (A)
31. (A) 32.  (C) 33. (B) 34. (C) 35. (B) 36.  (A) 37. (C) 38. (D) 39.  (C) 40. (D)
41. (B) 42. (C) 43. (B) 44. (A) 45. (A) 46. (A) 47. (A) 48. (C) 49. (B) 50. (A)
51. (C) 52. (B) 53. (D) 54. (C) 55. (A) 56. (C) 57. (A) 58. (B) 59. (B) 60. (A)
61. (B) 62. (A) 63. (B) 64. (C) 65. (A) 66. (B) 67. (A) 68. (A) 69. (C) 70. (A)
71. (B) 72. (B) 73. (A) 74. (B)

Previous Years’ Questions


75. (C) 76. (B) 77. (A) 78. (B) 79. (B) 80. (C) 81. (B) 82. (C) 83. (C) 84. (A)
85. (C) 86. (D) 87. (C) 88. (D) 89. (D) 90. (A) 91. (A) 92. (B) 93. (A) 94. (B)
95. (A) 96. (D) 97. (C) 98. (B) 99. (A) 100. (B)
PRACTICE EXERCISES
Differential Equations  9.35

HINTS AND EXPLANATIONS


Single Option Correct Type
1. Given equation can be re-written as 4. ydx + (x + x2y)dy = 0
dy ϕ ′( x ) y2 dx x dx x

−y =– ⇒ =– – x2⇒ + = –x2,
dx ϕ( x) ϕ( x) dy y dy y
1 dy 1 ϕ ′( x ) 1
⇒ – · + · = (1) dx −1 ⎛ 1 ⎞
y 2 dx y ϕ ( x ) ϕ( x) ⇒ x–2 dy + x ⎜⎝ y ⎟⎠ = –1
1 1 dy du dx dt

Putting = u, we get – 2 =
Put x–1 = t ⇒ –x–2 dy = ,
y y dx dx dy
du ϕ ′( x ) 1 dt ⎛ 1⎞ dt ⎛ 1 ⎞
+u =
get, – + t ⎜ ⎟ = –1 ⇒ − t =1

Now (1) becomes
dx ϕ( x) ϕ( x) dy ⎝ y⎠ dy ⎜⎝ y ⎟⎠
ϕ ′( x )
It is linear in t.
∫ ϕ ( x ) dx 1
I.F. = e
= f (x). ∫ − y dy
Integrating factor = e
= e–log y = y–1

Multiplying both sides by I.F., we have
d ϕ( x) \ Solution is t(y–1) = ∫ ( y −1 ) dy + c

[uf (x)] = 1 ⇒ y =
dx x+c
1 1 1

The correct option is (C) ⇒ · = log y + c ⇒ log y – =C
x y xy
y2
2. = 1 ⇒ d (log y1) = 1
y1
The correct option is (B)
⇒ log y1 = x + c
5. For the family of curves represented by the first differential

y1 = kex equation, the slope of the tangent at any point (x, y) is given

y = kex+ B by
Since it passes through (0, 0), k + B = 0
⎛ dy ⎞ x2 + x + 1
=
Thus, family is y = k (ex– 1).
⎝⎜ dx ⎠⎟ c y2 + y + 1

HINTS AND EXPLANATIONS



The correct option is (D) 1

For the family of curves represented by the second differen-


dy tial equation, the slope of the tangent at any point is given by
3. sec2 y + 2x tan y = x3
dx
⎛ dy ⎞ y2 + y + 1

= –
Let tan y = v ⎝⎜ dx ⎠⎟ c x2 + x + 1
dy dv 2

⇒ sec2 y =
dx dx ⎛ dy ⎞ ⎛ dy ⎞
Since, ⎜ ⎟ × ⎜ ⎟ = –1


The given equation becomes ⎝ dx ⎠ c ⎝ dx ⎠ c
1 2

dv
Hence, the two curves are orthogonal.

+ 2xv = x3
The correct option is (B)
dx
Now, I.F. = e ∫
2 xdx
6. The given equation can be written in the linear form as
2

= ex
follows:

Hence, the solution of the differential equation is
dy
+ yf ′(x) = f (x) f ′(x)
∫x ·e
2
3 x2
v · ex = dx + c
dx
2

Let x = t ⇒ dt = 2x dx The integrating factor of this equation is e ∫
ϕ ′ ( x ) dx
= ef (x).
1 t 1
2∫
te dt + c = et (t – 1) + c
2

⇒ v · ex = d
2
Hence (yef (x)) = f (x) f ′(x) ef (x)
dx
1 x
= e (x2 – 1) + c Integrating, we have yef (x) = ∫ tet dt + c , (where t = f (x))
2 2

⇒ v · ex
2
1 2
= tet– et+ c
(x – 1) + ce − x
2

\ tan y =
2
Hence, y = (f (x) – 1) + c e–f (x)

The correct option is (A)
The correct option is (A)
9.36  Chapter 9

dy dy
7. x = ⇒ ln x = xy \ e ∫ = e∫
yx Pdx −1/ x dx −1

e dx = e(– log x) = e log x = x–1


dx
1 log x 1
⇒ ∫ ydy = ∫ x ln xdx \ solution is yx–1 = ∫ x
⋅ dx + c
x
y2 (ln x ) 2 y 1
∫ x 2 log x dx + c = ∫ e
−t
⇒ = +k
i.e., = ⋅ t dt + c
2 2 x
1
Put log x = t \
dx = dt
⇒ y=± (ln x ) 2 + c x
The correct option is (C) Also, x = et
y e −t e −t
8. Family of semi-cubical parabolas is given by \ = t −1· +c
x −1 −1
ay2 = x3(1)
a is variable parameter
= e–t[–t + 1] + c
\ Differentiating the above semi-cubical equation,
= e–log x(– log x + 1) + c
1
we have = (1 – log x) + c
dy x
2ay = 3x2(2) \ y = (1 – log x) + cx
dx
Eliminating a from (1) and (2) The correct option is (D)
11. The given diff. eqn. can be written as
2 x3 dy dy
3 · y = 3x2⇒ 2x = 3y(3) (sin y dx + x cos y dy) x2 sin2 y
y dx dx
= 2y sin x dy + y2 cos x dx
dy dx

Now for orthogonal trajectories changing → –  in (3) ⇒ d (x sin y) (x sin y)2 = d (y2 sin x)
dx dy
Integrating, we get
dx
\ –2x = 3y ⇒ 3y dy + 2x dx = 0 ( x sin y )3
dy = y2 sin x + c

Integrating we have 3

The correct option is (C)
3y2 x2 c2
⇒ + = ⇒ 3y2 + 2x2 = c2 dx
2 1 2 12. Equation of the normal at (x, y) is Y – y = – (X – x)
dy
The correct option is (D)
HINTS AND EXPLANATIONS

dy
9. The given equation can be written as \ x-intercept = y + x (Putting Y = 0)
dx
dy

2y sin x + y2 cos x = sin 2x
Given: y2 = 2x ⎛⎜ y + x⎞⎟
dx dy

d 2 ⎝ dx ⎠
⇒ (y sin x) = sin 2x
dx dy y2 − 2x2
1 ⇒ = . Put y = vx. Then
On integrating, we get y2 sin x = – cos 2x + c dx 2 xy
2
π 1 dy v2 − 2 dv −( 2 + v 2 )
Putting x = and y = 1, we get c = .
v+x = ⇒x =
2 2 dx 2v dx 2v
2 1 2v dx
Hence, y sin x = (1 – cos 2x)

2
⇒ ∫ v 2 + 2 dv + ∫ x
= log k

⇒ y sin x = sin x or y2 = sin x.


2 2
⇒ log (v2 + 2) + log x = log k
The correct option is (A)
⇒ x (v2 + 2) = k ⇒ y2 + 2x2 – kx = 0
dy
10. Here y – x + log x = 0 9
dx If this passes through (2, 1), i =

2
dy Then, the equation becomes 4x2 + 2y2 – 9x = 0.


i.e., x – y = log x
dx
The correct option is (D)

dy y log x dy

i.e., − = x+ y 4
dx x x 13. Given: dx = x2 + 2y2 + y
dy x 2
y−x
dx
Differential Equations  9.37

16. The family of curves which are orthogonal (i.e., intersect


⎛ x⎞
d⎜ ⎟ at right angles) to a given system of curves is obtained by
d ( x2 + y2 ) ⎝ y⎠ dx dy
⇒ =2 2 substituting – for in the differential equation of the
( x 2 + y 2 )2 ⎛ x⎞ dy dx
⎜⎝ y ⎟⎠ given system.
2
⎛ dy ⎞ a

Integrating, we get The given differential equation is ⎜ ⎟ =
⎝ dx ⎠ x
1 −1 y 1 dy dx

– = +c⇒c= − 2 Replacing by – , we get
x2 + y2 x/ y x x + y2 dx dy
The correct option is (B) 2 2
⎛ dx ⎞ a ⎛ dy ⎞ x dy x
14. The given equation can be written as ⎜⎝ dy ⎟⎠ = ⇒ ⎜ ⎟ = ⇒ =±
x ⎝ dx ⎠ a dx a
(2x – 2y + 5)dy = (x – y + 3)dx
x− y+3 2

dy
= Integrating we get, y + c = ±
x3/2(1)
dx 2( x − y ) + 5 3 a
dy dV 4 3

Put x–y=V⇒1– = ⇒ (y + c)2 =x ⇒ 9a(y + c)2 = 4x3 (2)
dx dx 9a

Therefore, the given equation becomes
From (1) and (2), all of the first three given options represent
required equations.
dV V +3 dV V +2

1– = or =
The correct option is (D)
dx 2V + 5 dx 2V + 5
2V + 5 ⎛ 1 ⎞ ⎛ x + y − 1 ⎞ dy ⎛ x + y + 1⎞
⇒ dx = dV or dx = ⎜ 2 + dV 17. ⎜ = ⎜
V +2 ⎝ V + 2 ⎟⎠ ⎝ x + y − 2 ⎟⎠ dx ⎝ x + y + 2 ⎟⎠
On integrating, we get Put x + y = t

x = 2V + log(V + 2) + c dy dt dt ⎛ t + 1⎞ ⎛ t − 2⎞

1+ = ⇒ –1= ⎜
⇒ x = 2(x – y) + log(x – y + 2) + c dx dx dx ⎝ t + 2 ⎟⎠ ⎜⎝ t − 1 ⎟⎠
Therefore, 2y – x = log(x – y + 2) + c, is the required solution.
The correct option is (C) dt ⎛ t 2 − t − 2⎞

= ⎜ 2 ⎟ +1
15. Put x = r cos q, y = r sin q dx ⎝ t + t − 2⎠
\ dx = – r sin q dq + dr cos q ⎛ ( x + y )2 − 2 ⎞
⎟ =0

HINTS AND EXPLANATIONS


On solving, we get 2(y – x) + log ⎜
dy = r cos q dq + dr sin q ⎝ 2 ⎠
\ We have The correct option is (D)
18. The given differential equation is
r cos θ ( − r sin θ dθ + dr cos θ ) + r sin θ
( r cos θ dθ + dr sin θ ) 1 − r2 [y + x xy (x + y)] dx + (y xy (x + y) – x) dy = 0


=
( r cos θ ) ( r cos θ dθ + dr sin θ ) − r sin θ r 2
⇒ y dx – x dy + xy (x + y) (x dx + y dy) = 0
( − r sin θ dθ + dr ⋅ cos θ )
− r 2 sin θ cos θ dθ + r cos 2 θ dr x dy − y dx
⇒ x dx + y dy =
( x + y ) xy
+ r 2 sin θ cos θ dθ + r sin 2 θ dr 1 − r2
⇒ 2 2
=
r cos θ dθ + r cos θ sin θ dr r 1 ⎛ y⎞
⇒ d (x2 + y2) = 2 d ⎜ tan −1
2 2
+ r sin θ dθ − r sin θ cos θ dr 2 ⎝ x ⎟⎠
r dr 1 − r2 y
⇒ =
r 2 dθ r ⇒ x2 + y2 = 4 tan–1 x + c.
dr
The correct option is (B)

= dq
1 − r2
⇒ sin–1 r = q + c
19. Given:
dx
= cos2 p x. Differentiate with respect to t,
y dt
⇒ sin–1 x 2 + y 2 = tan–1 x + c.
d2x

= –2p sin 2px = –ve

The correct option is (C) dt 2
9.38  Chapter 9

d2x dy ⎛ 2 + sin x ⎞

 =0 24. = – cos x, y(0) = 1
dt 2
dx ⎜⎝ 1 + y ⎟⎠
⇒ 2p sin 2px = 0 ⇒ sin 2px = sin p

dy − cos x
1 ⇒ = dx
⇒ 2px = p ⇒ x =
(1 + y ) 2 + sin x
2

The correct option is (C) Integrating both sides ⇒ ln (1 + y) = – ln (2 + sin x) + c Put

1 x = 0 and y = 1 ⇒ ln (2) = – ln 2 + c ⇒ c = ln 4.
20. Given: Cartesian sub-tangent ∝ π 4 1
square of abscissa Put x = , ln (1 + y) = ln 3 + ln 4
= ln ⇒ y =
2 3 3
y k dy x2
The correct option is (A)

i.e., = 2 or = dx
dy/dx x y k
dy 2ay
x3 25. We have, =
Integrating, log y =
+ log c dx x ( y − a)
3k
3
y−a 2a
or y = ce x /3k
⇒ dy = dx

The correct option is (C) y x
On integrating both sides, we get
dy y ( x log y − y )
21. = a log | y | – y = –2a log | x | + loc c
dx x ( y log x − x )
⇒ ya × x2a = cey
dy Since, the curve passes through (1, 1), therefore
⇒ x (y log x – x) = y (x log y – y)
dx
1
⎛ x ⎞ dy y 1 = ce ⇒ c =

⇒ e
⎜⎝ log x − y ⎟⎠ dx = log y – x
So, the equation of the curve is
y dy x dy ya . x2a = ey – 1
⇒ + log x ⋅ = log y +
x dx y dx The correct option is (D)
d d
⇒ (y log x) = (x log y) 26. We have (x cos x – sin x + yx2) dx + x3dy = 0
dx dx
⇒ y log y = x log y + log c x cos x − sin x
⇒ dx + ydx + xdy = 0
⇒ log xy= log yx+ log c x2
HINTS AND EXPLANATIONS


⇒ xy= c yx ⎛ sin x ⎞
⇒ d ⎜ + d (xy) = d (c)
The correct option is (A) ⎝ x ⎟⎠
dy ax + 3 sin x
22. = ⇒ + xy = c
dx 2y + f x
⇒ (2y + f  )dy = (ax + 3)dx The correct option is (A)
2 y2 ax 2 27. Dividing the given equation by y2, we get
⇒ + fy = + 3x + c
2 2 y dx − x dy d ⎛ x⎞
( )
3
x d x
= – 3x2 e dx ⇒
3

⎜ ⎟ =– e
For the curve to be circle, a = –2 where f 2 + 9 + 4c > 0. y 2
dx ⎝ y ⎠ dx
The correct option is (B) x 3 x 3

23. Given diff. equation can be written as


On integrating, we get, = – ex + c ⇒ + ex = c
y y
x dy − y dx x dy − y dx
The correct option is (A)

= – dx ⇒ = – dx
x2 + y2 ⎡ ⎛ y⎞ 2⎤
x 2 ⎢1 + ⎜ ⎟ ⎥ 28. The given diff. equation can be written as
⎢⎣ ⎝ x ⎠ ⎥⎦
d ( y/ x ) ⎛y ⎞

+ dx = 0 ⇒ (y dx + x dy) + ⎜ dx + log x⎟ dy
2 ⎝x ⎠
⎛ y⎞
1+ ⎜ ⎟  + sin y dx + x cos y dy = 0
⎝ x⎠

Integrating, we get ⇒ d (xy) + d (y log x) + d (x sin y) = 0
y Integrating, we get
tan–1
+x=c
x xy + y log x + x sin y = c.

The correct option is (C) The correct option is (C)
Differential Equations  9.39

29. The equation of the normal at any point (x, y) is given by,
Using (1, 2) in (1), we get
dx
Y–y=– (X – x). 1 1
dy − = − + c ⇒ c = 0.
2 2
This passes through (h, k)
dx dx 1 x2 2
⇒k–y=– (h – x) ⇒ (y – k) = (h – x) . \ − = − ⇒ y = 3 or x3y = 2
dy dy xy 2 x

The correct option is (A)
is the required curve.

The correct option is (B)
30. 3xy2dy + y3dx + sin (xy) (xdy + ydx) = 0
or d (xy3) + sin (xy) d (xy) = 0 V (t ) T
On integrating, we get 34. ∫ dV (t ) = ∫ − k (T − t )dt
xy3 – cos xy = c I t=0

The correct option is (A) T


⎡ (T − t ) 2 ⎤
31. We have ⇒ V(T) – I = k ⎢ ⎥
Cartesian subtangent + abscissa = constant ⎣ 2 ⎦0
y dy dy dx ⎛T2⎞
⇒ +x=a⇒y +x=a⇒ = ⇒ V(T) – I = – k ⎜ ⎟
dy/dx dx y a−x
⎝ 2⎠
Integrating, we get log y + log (x – a) = log c
\ y(x – a) = c kT 2
⇒ V(T) = I −
As the curve passes through the point (2a, a), we have c = a2 2
Hence the required curve is y (x – a) = a2.
The correct option is (C)
The correct option is (A)
35. We have
dt g ′( x ) t2 dy
32. −t =–
=y+3
dx g( x) g( x) dx
1 dt 1 g ′( x ) 1 1
⇒ + = (1) ⇒ dy = dx
t 2 dx t g ( x ) g( x) y+3
1 ⇒ ln|(y + 3)| = x + k, where k is a constant of integration
Let = z

HINTS AND EXPLANATIONS


t ⇒ (y + 3) = c ex
1 dt dz Initially when x = 0, y = 2
\  2
=
t dx dx ⇒c=5

Now by (1), we have Finally the required solution is y + 3 = 5ex
dz g ′( x ) 1 ⇒ y(ln 2) = 5eln 2 – 3 = 10 – 3 = 7
+z =
dx g( x) g( x) The correct option is (B)
g ′( x )
∫ g( x)
dx
I.F. = e
= g(x) dp(t )
1 36. 2 = –dt
Therefore, the solution is z g(x) = x + c ⇒ g(x)
=x+c 900 − p(t )
t
g( x)
–2 ln [900 p(t)] = –t + c
⇒ t=
x+c when t = 0, p(0) = 850


The correct option is (C)
–2 ln (50) = c

33. xdy + (y + x3y2) dx = 0 ⇒ xdy + ydx = –x3y2dx ⎛ 50 ⎞


\ 2 ln ⎜ = –t
x dy + ydx ⎝ 900 − p(t ) ⎟⎠
⇒ = –x dx
x2 y2
900 – p(t) = 50 et/2
d ( xy )
p(t) = 900 – 50 et/2

= –x dx
( xy ) 2 let p(t1) = 0


Integrating, we get 0 = 900 – 500 et / 2 1

1 x 2
\ t1 = 2 ln 18
⇒ – =– + c(1)
xy 2 The correct option is (A)
9.40  Chapter 9

ydy 1 −2 y dy dV dV
37. ∫ 1− y 2
= ∫ dx ⇒ −
2 ∫ 1 − y2
=x+c ⇒
dx
= (1 – V) g′ (x) ⇒
1−V
= g′ (x) dx

dV
⇒ ∫ = ∫ g’( x ) dx
⇒ − 1 − y 2 = x + c ⇒ 1 – y2 = x2 + c2 + 2cx 1−V
⇒ – log (1 – V) = g (x) – C
⇒ x2 + y2 + 2cx + c2 – 1 = 0
⇒ g (x) + log (1 – V) = C
Which is a circle with centre (–c, 0) and radius 1.
\ g (x) + log [1 + y – g (x)] = C
The correct option is (C)
38. y = vx
The correct option is (B)
dy dv x dy − y dx
= v + x 42. We have, x dx + y dy + =0
dx dx x2 + y2
dv
v+x =1+v 1 ⎛ y⎞
dx ⇒ d (x2 + y2) + d tan– 1 ⎜ ⎟ = 0
2 ⎝ x⎠
dx 1 2 y c
⇒ dv =
Integrating, (x + y2) + tan– 1 =
x 2 x 2
\ v = log x + c
y
y ⇒ x2 + y2 + 2 tan– 1 =c
⇒ = log x + c x
x
⎛ c − x2 − y2 ⎞
Since, y(1) = 1, we have \ y = x tan ⎜ ⎟ is the required solution.
y = x log x + x ⎝ 2 ⎠
The correct option is (D)
The correct option is (C)

39. (x – h)2 + (y – 2)2 = 25 (1) ⎛ xf ( y /x ) ⎞


43. We have, x dy = ⎜ y + dx
⇒ 2(x – h) + 2(y – 2)
dy
=0 ⎝ f ′ ( y /x ) ⎟⎠
dx
dy y f ( y /x )
dy ⇒ = + which is homogeneous.
⇒ (x – h) = –(y – 2) dx x f ′ ( y /x )
dx

Substituting in Equation (1), we have dy dV
Put y = Vx so that
=V+x ,
HINTS AND EXPLANATIONS

⎛ dy ⎞
2 dx dx
( y − 2) 2 ⎜ ⎟ + ( y − 2) 2 = 25
we obtain,
⎝ dx ⎠
dV f (V ) f (V ) dx
(y – 2)2y′2 = 25 – (y – 2)2

V+x =V+ ⇒ dV =
dx f’ (V ) f’ (V ) x

The correct option is (C)

Integrating, we get
40. y = c1e c x (1)
2
log f (V) = log x + log c

y′ = c2 c e c x
⎛ y⎞
1
2
⇒ log f (V) = log cx ⇒ f ⎜ ⎟ = cx
⎝ x⎠
y′ = c2 y(2)

The correct option is (B)
y′′ = c2 y′
From Equation (2) dy
44. We have, = ex – y (ex – ey)
y′ dx

c2 =
y
dy
( y ′)2 ⇒ ey dx + ex . ey = e2x.
So, y′′ =
⇒ yy ″ = ( y ′ ) 2
y
dy dV

The correct option is (D) Putting ey = V so that e y
= , we get
dx dx
dy
41. We have, = (g (x) – y). g′ (x) dV
dx + ex × V = e2x, which is linear in V.
dy dV dx
Put g (x) – y = V ⇒ g′ (x) –
=
dx dx
I.F. = e ∫
x
e dx x

dV = ee .

Hence, g′ (x) – = V · g′ (x)
dx
So, the solution is
Differential Equations  9.41

∫e x3
2x
ex
V . e e =
x
⋅ e dx + c
Integrating, we get y =
+ C, which is the required family.
6
∫e
x

⇒ e y ⋅ e e = z
⋅ z dz + c
The correct option is (A)
[Putting ex = z ⇒ ex dx = dz]
y
dy y
48. We have, x2 – xy = 1 + cos = 2cos2 ⋅
⇒ e y ⋅ e e = (z – 1) ez + c = (e x − 1) e e + c
x x

dx x 2x
⇒ ey = ex – 1 + ce–ex ⎛ y ⎞ ⎡ 2 dy ⎤
The correct option is (A) ⇒ sec2 ⎜⎝ 2 x ⎟⎠ ⋅ ⎢⎣ x dx − xy ⎥⎦ = 2

dy
2 x −y
⎛ dy ⎞ dy 1 ⎛ y⎞ dx 1
45. We have, x ⎜ ⎟ + (y – x) –y=0 ⇒ sec 2 ⎜⎝ ⎟⎠ ⋅ 2
= 3
⎝ dx ⎠ dx 2 2x x x

⎛ dy ⎞ ⎛ dy ⎞ d ⎛ y⎞ 1
⇒ ⎜ −1 x + y⎟ = 0 ⇒ ⎜⎝ tan ⎟⎠ = 3 ⋅
⎝ dx ⎟⎠ ⎜⎝ dx ⎠ dx 2x x
dy dy y 1
⇒ = 1 or x =–y
Integrating, we get tan =c − 2 ,
dx dx 2x 2x
dy
which is the required solution.

The solution of = 1 is y = x + c
dx
The correct option is (C)
dy dy dx dy dt

and solution of x = – y i.e., + = 0 is 49. Let x + y = t⇒ 1 += .
dx y x dx dx
log (xy) = log c i.e., xy = c.

Therefore, the given equation becomes
Hence, general solution is (x – y + c) (xy – c) = 0.

⎛ t − 1 ⎞ ⎛ dt ⎞ t +1

The correct option is (A)
⎜ ⎟ ⎜ − 1⎟ =

⎝ t − 2 ⎠ dx ⎠ t+2
46. The equation of the general circle is given by
dt ⎛ t + 1⎞ ⎛ t − 2⎞
x2 + y2 + 2gy + 2fy + c = 0 (1) ⇒ −1 = ⎜
dx ⎝ t + 2 ⎟⎠ ⎜⎝ t − 1 ⎟⎠
Differentiating with respect to x, we get
2x + 2yy′ + 2g + 2fy′ = 0 (2) dt t2 − t − 2

HINTS AND EXPLANATIONS


⇒ = 2 +1
Differentiating again, we get dx t +t −2
1 + y′2 + yy″ + fy″ =0(3)
dt 2t 2 − 4
Differentiating again, we have ⇒ = 2
dx t +t −2
2y′y′′ + yy′′′ + y′ y′′ + fy′′′ = 0 (4)
Eliminating f from (3) and (4), we get t2 + t − 2
⇒ dt = 2dx
y′′′ (1 + yy′′ + y′2) – y′′ (yy′′′ +3y′ y′′) = 0 t2 − 2
⇒ y′′′ (1 + y′2) – 3y′ y′′2 = 0, which is the required differen- ⎛ t ⎞
tial equation. ⇒ ⎜1 + 2 ⎟ dt = 2dx
⎝ t − 2⎠
The correct option is (A)

On integrating, we get
dy 1
47. Let m1 = for required family of curves at (x, y).
t+ log t 2 − 2 = 2x + c
dx 2
dy 1
Let m2 =
for the hyperbola xy = 2. log ( x + y ) 2 − 2 = 2c(1)
⇒ ( x + y ) +
dx 2
Given, y = 1, when x = 1, therefore log 2 = 2c.

dy −2
Then, m2 =
= 2 ⋅
Substituting the value of c in (1), we get
dx x
2(y – x) + log | (x + y)2 –2 | = log 2


Since the required family of curves is orthogonal to the
hyperbola, \ m1 × m2 = – 1 ( x + y )2 − 2
2 2 ⇒ 2 ( y − x ) + log = 0. \ k = 2.
dy ⎛ −2 ⎞ dy x x 2
⇒ ×⎜ ⎟ =–1⇒ = ⋅ ⇒ dy = dx
dx ⎝ x 2 ⎠ dx 2 2
The correct option is (B)
9.42  Chapter 9

50. We have, x dy – y dx = xy3 (1 + log x) dx


\ Solution is t(y–1) = ∫ ( y −1 ) dy + c
⎛ ydx − xdy ⎞
⇒ – ⎜ ⎟ = xy (1 + log x) dx 1 1 1
⎝ y2 ⎠ ⇒ · = log y + c ⇒ log y – =C
x y xy

The correct option is (B)
⎛ x⎞
⇒ – d ⎜ ⎟ = xy (1 + log x) dx dx
⎝ y⎠ 53. Equation of the normal at (x, y) is Y – y = – (X – x)
dy
x ⎛ x⎞ dy
⇒ − d = x2 (1 + log x) dx \ x-intercept = y + x (Putting Y = 0)
y ⎜⎝ y ⎟⎠ dx

Integrating, we get ⎛ dy ⎞
2 Given, y2 = 2x ⎜ y + x⎟

⎛ x⎞ ⎝ dx ⎠
−⎜ ⎟
⎝ y⎠ x3 x3 1 y2 − 2x2

= (1 + log x) −∫ ⋅ dx ⇒
dy
= . Putting y = vx. Then,
2 3 3 x dx 2 xy
x2 x3 x3 C
⇒ − 2
= (1 + log x ) − +
v+x
dy
=
v2 − 2
⇒x
dv −( 2 + v 2 )
=
2y 3 9 2
dx 2v dx 2v
x2 2 x3 ⎛ 2 ⎞
⇒ − = ⎜ + log x⎟⎠ + C 2v dx
y 2
3 ⎝3 ⇒ ∫ v 2 + 2 dv + ∫
x
= log K
The correct option is (A) ⇒ log (v2 + 2) + log x = log K
51. Given equation can be rewritten as ⇒ x (v2 + 2) = K ⇒ y2 + 2x2 – Kx = 0
dy ϕ ′( x ) y2 9
−y =– If this passes through (2, 1), K =

dx ϕ( x) ϕ( x) 2
1 dy 1 ϕ ′( x ) 1 Then, the equation becomes 4x + 2y2 – 9x = 0.
2
⇒ – · + · = (1)
y 2 dx y ϕ ( x ) ϕ( x) The correct option is (D)
1 1 dy du 54. Put x = r cos q, y = r sin q

Putting = u, we get – 2 = .
y y dx dx \ dx = – r sin q dq + dr cos q
du ϕ ′( x ) 1 dy = r cos q dq + dr sin q
HINTS AND EXPLANATIONS


Now, (1) becomes +u =
dx ϕ( x) ϕ( x) \ We have
ϕ ′( x )
∫ ϕ( x) r cos θ ( − r sin θ dθ + dr cos θ ) + r sin θ
I.F. = e
= f (x). Multiplying both sides by I.F., we have
( r cos θ dθ + dr sin θ ) 1 − r2
=
d ϕ( x) ( r cos θ ) ( r cos θ dθ + dr sin θ ) − r sin θ r2
[u f (x)] = 1 ⇒ y =
dx x+c ( − r sin θ dθ + dr cos θ )
The correct option is (C)
− r 2 sin θ cos θ dθ + r cos 2 θ dr
52. ydx + (x + x2y)dy = 0
dx x dx x + r 2 sin θ cos θ dθ + r sin 2 θ dr 1 − r2
⇒ =– – x2⇒ + = –x2, ⇒ =
dy y dy y r 2 cos 2 θ dθ + r cos θ sin θ dr r

It is Bernoulli’s form. Divide by x2


+ r 2 sin 2 θ dθ − r sin θ cos θ dr

dx −1 ⎛ 1 ⎞
x–2 dy + x ⎝⎜ y ⎠⎟ = –1 r dr 1 − r2 dr
⇒ 2
= ⇒ = dq
r dθ r 1 − r2
dx dt
Put x–1 = t, –x–2 dy =
2 2 y
dy ⇒ sin–1 r = q + c ⇒ sin–1 x + y = tan–1 x + c.
dt ⎛ 1⎞ dt ⎛ 1 ⎞
The correct option is (C)

We get, – + t ⎜ ⎟ = –1 ⇒ − t =1
dy ⎝ y⎠ dy ⎜⎝ y ⎟⎠

It is linear in t. 55. We have (x cos x – sin x + yx2) dx + x3dy = 0
1
∫ − y dy x cos x − sin x
Integrating factor = e
= e–log y = y–1 ⇒ dx + ydx + xdy = 0
x2
Differential Equations  9.43

⎛ sin x ⎞ 59. Let x = r cos q, y = r sin q


⇒ d ⎜ + d (xy) = d (c)
⎝ x ⎟⎠ so that x2 + y2 = r2(1)

sin x y
⇒ + xy = c and, tan q =
(2)
x x

The correct option is (A) From (1), we have d(x2 + y2) = d(r2)

⇒ x dx + y dy = rdr (3)
56. The given differential equation can be written as
⎛ y⎞
⎛y ⎞ From (2), we have d ⎜ ⎟ = d(tan q)

⇒ (y dx + x dy) + ⎜ dx + log x⎟ dy ⎝ x⎠
⎝x ⎠ x dy − y dx

i.e., = sec2q dq
 + sin y dx + x cos y dy = 0 x2
⇒ d (xy) + d (y log x) + d (x sin y) = 0 i.e., x dy – y dx = x2 sec2q dq = r2 cos2q sec2q dq(4)

Integrating, we get

Using (3) and (4) in the given equation, we get;
xy + y log x + x sin y = c
The correct option is (C) rdr a2 − r 2 dr
2 = i.e., = dθ
r dθ r2 a − r2
2

dy 1 dx ⎛ r⎞
57. Y – y = (X – x) ⇒ 5 = y 2
dx 2 dy i.e., sin–1 ⎜⎝ a ⎟⎠ = q + c

dy or, r = a sin (q + c)

⇒ 10 = – dx
y2

or, x 2 + y 2 = a sin [c + tan–1(y/x)]
10
⇒ − + x = c
y The correct option is (B)
Since it passes through (2, 5), therefore, c = 0. 60. The given differential equation can be written as
Thus, the equation of curve is xy = 10 x2

(log y) 2x dx + dy + 3y2dy = 0
y
y
⇒ (log y) d(x2)+ x2d(log y) + d(y3) = 0
⇒ d(x2 log y) + d(y3) = 0
P (x, y) ⇒ x2 log y + y3 = c (Integrating both sides)

HINTS AND EXPLANATIONS


The correct option is (A)
θ 61. The given differential equation can be written as;
x
y dx + x dy
2 + sin x dx + sin y dy = 0
cos ( xy )

The correct option is (A)
⇒ sec2(xy) d(xy) + sin x dx + sin y dy = 0
58. xdy + (y + x3y2) dx = 0 ⇒ xdy + ydx = –x3y2dx ⇒ tan(xy) – cos x – cos y = c (Integrating both sides)
The correct option is (B)
x dy + ydx d ( xy )
⇒ = –x dx ⇒ = –x dx 62. The equation can be written as
x2 y2 ( xy ) 2
dx 2 y ln y + y − x x
= = ( 2 ln y + 1) −

Integrating, we get dy y y
1 x2 dx 1

– =– + c(1)
i.e., + · x = (2 ln y + 1)
xy 2 dy y

Using (1, 2) in (1), we get 1
∫ dy
1 1 \ I.F. = e ∫ P dy = e y = e ln y = y
− = − +c ⇒c=0
2 2 \ The solution is

1 x2 2 xy = ∫ ( 2 ln y + 1) ⋅ y dy = y 2ln y + c

\ − = − ⇒ y = 3 or x3y = 2
xy 2 x c
⇒ x = y lny +

is the required curve. y

The correct option is (B)
The correct option is (A)
9.44  Chapter 9

63. The given equation can be written as; Dividing by cos2y, we get

dy dy
− sec 2 x ⋅ tan 2 x ⋅ y = cos 2 x
tan y · sec y · – sec y = –x
dx dx
dy dv
2 tan x Let sec y = v ⇒ sec y tan y
=
∫ tan sec 2 x dx
dx dx
I.F. = e ∫
− tan 2 x sec 2
x dx 2
x −1
= e
So, we get
dt
∫ dv

= e t where t = tan2x – 1 − v = –x, which is a linear differential equation with
dx

= e ln | t | = | t | = |tan 2 x − 1| P = –1, Q = –x
It is given that | x | < p/4 and for this region tan2x < 1 \ I.F. = e ∫
Pdx
= e∫
−1dx
= e–x
\ I.F. = 1 – tan2x
The solution is given by
\ The solution is;
∫ −x e
−x

ve–x = dx = xe − x + e − x + c
y(1 – tan x) =
2
∫ cos
2
(
x 1 − tan x dx 2
)
= e–x(x + 1) + c
∫ (cos ) ∫ cos 2x dx v = (1 + x) + cex
2

= x − sin 2 x dx =
or,
or, sec y = (1 + x) + cex

sin 2 x

= +c
The correct option is (A)
2
66. The given equation is
3 3
Now, when x = p/6, y =
xy2exy dy + xex/ydy = yex/y dx – y3exydx
8
⇒ y2 · exy(xdy + ydx) = ex/y(ydx – xdy)
3 3 ⎛ 1⎞ 1 3
\ ⎜⎝1 − ⎟⎠ = · +c⇒c=0
⎛ ⎞
8 3 2 2 ⇒ exy(xdy + ydx) = ex/y y dx − x dy
⎜ ⎟
sin 2 x ⎝ y2 ⎠
\ y=
2 1 − tan 2 x ( ) ⇒ exy· d(xy) = ex/y(d(x/y))
The correct option is (B) ⇒ d(exy) = d(ex/y) ⇒ exy= ex/y+ c
64. The given equation can be written as or, xy = ln (ex/y+ c)
⎛ dy ⎞ The correct option is (B)
⎜⎝ + 1⎟ + x( x + y ) = x3(x + y)3
dx ⎠ 67. Differentiating the given equation with respect to x, we get
HINTS AND EXPLANATIONS

x x
d ( x + y) xy ( x ) + 1 ∫ y(t ) dt = ( x + 1) xy ( x ) + 1 ∫ t y(t ) dt
⇒ + x ( x + y ) = x 3 ( x + y )3
dx 0 0

d ( x + y) x x
⇒ ( x + y )3 + x ( x + y ) −2 = x 3
dx ⇒ ∫ y(t ) dt = x2y (x) + ∫ t y(t ) dt
0 0
d ( x + y ) dz
Let (x + y)–2 = z so that −2 ( x + y ) −3
=
Differentiating again with respect to x we get,
dx dx
y(x) = x2y′(x) = 2xy(x) + xy(x)

The given equation reduces to
x 2 dy( x )
−1 dz dz ⇒ (1 – 3x) y(x) =
+ xz = x3 i.e., − 2 xz = –2x3 dx
2 dx dx (1 − 3 x ) dx dy ( x )
⇒ =
I.F. = e ∫ −2 x dx = e–x2 x2 y ( x)
On Integrating, we get
\ The solution is,
c
2
y = 3 e −1/x
∫ −2 x · e
− x2
z · e–x = dx = ( x 2 + 1) e − x + c
2
3

x
1 2 The correct option is (A)

or, = cex + x2 + 1
( x + y)2
68. We have,
The correct option is (C)
(1 + tan y) (dx – dy) + 2xdy = 0
dy
65. We have, sin y = cos y (1 – x cos y) ⇒ (1 + tan y) dx = (1 + tan y – 2x) dy
dx
dx 2
dy ⇒ + x = 1, which is linear in x
⇒ sin y – cos y = –x cos2y dy 1 + tan y
dx
Differential Equations  9.45

2 2 cos y dy

Now, ∫ 1 + tan y dy = ∫ sin y + cos y dy
Let
ex
=t⇒
ye x − e x
dx = dt
y y2 dx
⎛ cos y − sin y ⎞

= ∫ ⎜⎝1 + cos y + sin y ⎟⎠ dy ⇒

ye x dx − e x dy
= dt
y2

= y + log (cos y + sin y)
\ (1) becomes, dt + 2x2dx – y dy = 0
2
∫ 1 + tan y dy Integrating, we get,
\ I.F. = e = ey · elog (cos y + sin y)
2 x3 y 2
y
t+ − =c

= (cos y + sin y) e 3 2

The solution is given by,
ex 2 3 y2

x · ey(cos y + sin y) = eysin y + c ⇒ + x − =0
y 3 2
or, x(sin y + cos y) = sin y + ce–y

Put x = 0, y = 1, we get c = 1/2

The correct option is (A)
Hence, the solution is
69. We have, 6ex+ 4x3y – 3y3 – 3y = 0
dy The correct option is (B)

x–y = (x2 – y2)2
dx 72. The given equation can be written as
d ( x2 − y2 ) (y dx – x dy)e–x/y – y3dy = 0
⇒ = dx
2 ( x 2 − y 2 )2 y dx − x dy
⇒ e − x/ y = y dy
−1 y2
⇒ = 2x + c
(x 2
− y2 ) ⇒ d(x/y)e–x/y = y dy
On integrating, we get
Since it passes through (1, 0),
\ –1 = 2 + c ⇒ c = –3 y2
–e–x/y =
+c
Thus, the curve is 2
or, 2e–x/y + y2 = c

1

(2x – 3) + =0
The correct option is (B)
x − y2 2
73. Given, y = c1em1x+ c2em2x+ c3em3x(1)

The correct option is (C)

HINTS AND EXPLANATIONS


⇒ y1 = c1m1em1x+ c2m2em2x+ c3m3em3x
70. The given equation can be written as = m1(y – c2em2x– c3em3x) + c2m2em2x + c3m3em3x
dy  [from (1)]

2y sin x + y 2 cos x = sin 2x
dx m2x m3x
= m1y + c2(m1 – m2)e + c3(m3 – m1)e (2)
d
(
y 2 sin x = sin 2x
dx
)

Now,
y2 = m1y1 + c2m2(m2 – m1)em2x + c3m3(m3 – m1)em3x

On integrating, we get = m1y1 + m2[y1 – m1y – c3(m3 – m1)em3x]
−1 + c3m3 (m3 – m1) em3x  [from (2)]
y2 sin x =
cos 2 x + c
2 = (m1 + m2) y1 – m1m2y + c3(m3 – m1) (m3 – m2)em3x(3)
π −1 Further,
Put x =
, y = 1, we get c = y3 = (m1 + m2)y2 – m1m2y1 + c3m3(m3 – m1) (m3 – m2) em3x
2 2
= (m1 + m2)y2 – m1m2y1 + m3[y2 – (m3 – m1) y1 + m1m2y]

Hence, the solution is
 [from (3)]
1
y2 sin x =
(1 − cos 2 x ) = sin 2 x = (m1 + m2 + m3)y2 – (m1m2+ m1m3 + m2m3) y1 + m1m2m3y
2
= 0 .  y2 – (–7)y1 – 6y ⇒ y3 – 7y1 + 6y = 0
⇒ y2 = sin x The correct option is (A)
The correct option is (A)
74. We have,
x x
71. We have, ye dx – e dy + 2x y dx – y dy = 0 2 2 3 dy
= 2c1e2x+ c2ex– c3e–x
dx
ye x dx − e x dy
⇒ + 2 x 2 dx − y dy = 0 (1) d2y
y2 2 = 4c1e2x+ c2ex– c3e–x
dx
9.46  Chapter 9

d3y –1 + a – b + c = 0
3 = 8c1e2x+ c2ex– c3e–x ⇒ a = –2, b = –1, c = 2
dx

Substituting in the given differential equation, we get a3 + b3 + c 3 1

Thus, =−
8 + 4a + 2b + c = 0
abc 4
1+a+b+c=0

The correct option is (B)

Previous Year’s Questions


75. The given differential equation is Therefore the required solutions is
2/3
⎛ dy ⎞ ⎛ d y⎞ 3
e tan
−1
y
⎜⎝1 + 3 ⎟⎠
= 4⎜ 3 ⎟ = ∫ e tan
−1 −1

xe tan y y
⋅ ⋅ dy + K1
dx ⎝ dx ⎠ 1 + y2

Making the above equation free from radical 2/3, the −1

e 2 tan y
=∫
−1

­equation can be rewritten as xe tan y


dy + K1
2 3 1 + y2
⎛ dy ⎞ ⎛ d3y ⎞
⎜⎝1 + 3 ⎟⎠ = 4 ⎜ 3 ⎟ xe tan
−1
y 1
= e 2 tan
−1
y
+ K1
dx ⎝ dx ⎠
2
This shows that the order and degree of given equation are 3 tan y −1

= e 2 tan y + K
−1

2 xe
and 3 respectively.
The correct option is (B)
The correct option is (C)
d2y 80. x 2 + y 2 − 2ay = 0
76. The equation = e −2 x
dx 2 Differentiating above equation:
−2 x
dy e 2x + 2yy′ − 2ay′ = 0
⇒ = +c
dx −2 x + yy ′
⇒ a= (eliminating a)
e2 x y
⇒y= + cx + d
4 ⇒ (x2 − y2)y′ = 2xy.
The correct option is (B) The correct option is (C)
77. The general equation of all non-vertical lines in a plane is 81. The equation ydx + xdy + x2y dy = 0 implies
ax + by = 1, where b ≠ 0 d ( xy ) 1 1
+ dy = 0 ⇒ − + log = C .
HINTS AND EXPLANATIONS

dy x 2 2
y y xy
∴a + b =0
dx
The correct option is (B)
d2y
⇒b =0 82. y2 = 2c (x + √c)(1)
dx 2 Differentiating (1) we obtain
d2y 2yy′ = 2c ⋅1 or yy′ = c(2)
⇒ =0
dx 2
⇒ y 2 = 2 yy ′ ( x + yy ′ ) [on putting value of c from (2) in
which is the desired differential equation.
(1)]
The correct option is (A)
On simplifying, we get
78. Equation y2 = 4a (x – h)
(y − 2xy′)2 = 4yy′3 (3)
⇒ 2 yy ′ = 4 a
Hence equation (3) is order 1 and degree 3.
⇒ yy ′ = 2a
The correct option is (C)
⇒ yy ′′ + ( y ′ ) 2 = 0
xdy
The correct option is (B) 83. = y(log y − log x + 1)
dx
−1 dx
79. (1 + y 2 ) + ( x − e tan y ) =0 dy y ⎛ ⎛ y⎞ ⎞
dy ⇒ = log ⎜ ⎟ + 1⎟
dx x ⎜⎝ ⎝ x⎠ ⎠
dx −1

⇒ (1 + y 2 ) + x = e tan y
Substituting y = vx
dy
dx 1 e tan y
−1
dy xdv
⇒ + x= =v+
2 dx dx
dy 1 + y 1 + y2
xdv
1
∫ 1+ y 2 dy ⇒v+ = v(log v + 1)
I.F = e ∫ dx
pdy −1

=e = e tan y

Differential Equations  9.47

xdv ∴ v = logx + c
= v log v
dx y
⇒ = log x + c
dv dx x
⇒ = k
v log v x Since, y (1) = 1, we have
Put log v = z y = x log x + x
1 The correct option is (D)
dv = dz
v 87. (x − h)2 + (y − 2)2 = 25 (1)
dz dx dy
⇒ = ⇒ 2(x − h) + 2(y − 2) =0
z x dx
ln z = ln x + ln c dy
⇒ (x − h) = −(y − 2)
z = cx dx
log v = cx Substitution in (1) implies that
⎛ y⎞ 2
log ⎜ ⎟ = cx ⎛ dy ⎞
⎝ x⎠ (y − 2) ⎜⎝ dx ⎟⎠ + (y − 2)2 = 25
2

The correct option is (C)
(y − 2)2y′2 = 25 − (y − 2)2.
84. Given equation Ax2 + By2 = 1 implies (1)
The correct option is (C)
dy
Ax + By = 0, and (2)
dx 88. Given y = c1e c x (1) 2


2
d2y ⎛ dy ⎞ ⇒ y ′ = c2c1e c x .
A + By + B ⎜ ⎟ = 0 (3)
2

dx 2 ⎝ dx ⎠

So, y′ = c2y(2)
From (2) and (3)
⇒ y′′ = c2y′
⎧⎪ d2y ⎛ dy ⎞ ⎫⎪
2
dy From (2)
x ⎨ − By 2 − B ⎜ ⎟ ⎬ + By =0
⎪⎩ dx ⎝ dx ⎠ ⎪ dx y′
⎭ c2 =
2 y
d2y ⎛ dy ⎞ dy
⇒ xy + x⎜ ⎟ − y =0 ( y ′)2
dx 2 ⎝ dx ⎠ dx So, y ′′ = ⇒ yy ′′ = ( y ′ ) 2

HINTS AND EXPLANATIONS



y
The correct option is (A)
The correct option is (D)
85. General equation of all such circles is x2 + y2 + 2gx = 0.
89. cos x dy = y (sin x−y) dx
On differentiating, we get
dy
dy ⇒ = y tan x − y 2 sec x
2x + 2 y+ 2g = 0 dx
dx
Equating the expression for g from above equation in the 1 dy 1
⇒ − tan x = − sec x
general equation, we get y 2 dx y
⎛ dy ⎞ 1
x2 + y2 + ⎜ −2 x − 2 y ⎟ x = 0 Let =t
⎝ dx ⎠ y
1 dy dt
dy ⇒ − =
⇒ y2 = x2 + 2xy y 2 dx dx
dx
The correct option is (C) dy dt
⇒ − − t tan x = − sec x ⇒ + (tan x )t = sec x.
dx dx
86. Substituting, y = vx
I.F = e ∫
tan xdx
dy dv = sec x
⇒ =v+x
dx dx
dv Solution is t (I.F) = ∫ (I.F)sec x dx
⇒ v+x = 1+ v
dx 1
⇒= sec x = tan x + c
dx y
⇒ dv =
x The correct option is (D)
9.48  Chapter 9

dV
90. ⇒ p − 400 = 300et / 2
= − k (T − t ) ⇒ dV = − k (T − t )dt
dt
Integrate

⇒ 400 − p = 300et / 2 (as p < 400)
− k (T − t ) 2 k (T − t ) 2 ⇒ p = 400 − 300et / 2
V= + c ⇒V = +c
( −2) 2 The correct option is (A)
at t = 0 ⇒ V = I dy y 2 x ln x
94. + =
dx x ln x x ln x
kT 2 kT 2 kT 2 1
l=+c⇒c=l− ⇒ c = V (T ) = l − ∫ dx
2 2 2 ∴ I.F. = e x ln x = e ln(ln x ) = ln x

The correct option is (A) The solution is
y ln x = ∫ 2 ln xdx
d ( p(t )) 1
91. = p(t ) − 450
dt 2 ⇒ y ln x = 2( x ln x − x ) + c
d ( p(t )) p(t ) − 900 For x = 1, c = 2
=
dt 2 y ln x = 2( x ln x − x + 1)

d ( p(t )) Put x = e ⇒ y(e) = 2 .
2∫
p(t ) − 900 ∫
= dt
The correct option is (B)
2 ln p(t ) − 900 = t + c 95. We have ydx + xy 2 dx = xdy

t=0 xdy − ydx
⇒ = xdx
⇒ 2 ln 50 = 0 + c y2

⇒ c = 2 ln 50 ⎛ x⎞ ⎛ x2 ⎞
⇒ − d ⎜ ⎟ = d ⎜ ⎟
⎝ y⎠ ⎝ 2⎠
∴ 2 ln p(t ) − 900 = t + 2 ln 50
Integrating, we get

p( t ) = 0 x x2
− = +C
⇒ 2 ln 900 = t + 2 ln 50 y 2
Since, it passes through (1, –1)
HINTS AND EXPLANATIONS

⎛ 900 ⎞
t = 2(ln 900 − ln 50) = 2 ln ⎜ = 2 ln18.
⎝ 50 ⎟⎠
1 1
∴ 1 = + C ⇒ C =
2 2
The correct option is (A)
2x −2 x
p 25 ⇒ x 2 + 1 + =0⇒ y= 2
92. Given ∫ dP = ∫ (100 − 12 )
x dx y x +1
2000 0 ⎛ 1⎞ 4
∴ f ⎜− ⎟ =
12 × 2 ⎝ 2⎠ 5
( P − 2000) = 25 × 100 − ( 25)3/ 2 The correct option is (A)
3
P = 3500. dy
96. (2 + sin x) + (y + 1)cos x = 0
The correct option is (B) dx
dp p − 400 ⎛π⎞
93. = y(0) = 1, y ⎜ ⎟ = ?

dt 2 ⎝ 2⎠
1 cos x
dp 1 dy + dx = 0
= dt y +1 2 + sin x
p − 400 2
ln |y + 1 | + ln (2 + sin x ) = ln C

1n p − 400 = 1 t + c (y + 1) (2 + sin x) = C

2 Put x = 0, y = 1

at t = 0, p = 100
(1 + 1) . 2 = C ⇒ C = 4

ln 300 = c
Now, (y + 1) (2 + sin x ) = 4

p − 400 t dy

1n
300
=
2 (
For, x = 1 − x 2
dx
)
− xy = 1
Differential Equations  9.49

(y + 1) (2 + 1) = 4
2
⇒ y = − 8π

4 9
y+1=

3
4 1 98. Let y = f(x)
y=
–1=
3 3
\ dy + 3 y = 7


Hence, the correct option is (D)
dx 4x
dy 3 dx 3
97. sin x + y cos x = 4 x   x ∈ (0, π) I.F. = e ∫ 4
x
= x4
dx
3 3
dy


dx
+ y cot x = 4 x cosec x
So, y. x 4 = 7 x 4 dx

dy 7

+ Py = Q where P and Q are function f(x) 3
x4
dx
y.
x = 7,
4
+c
7/4
⇒ ye ∫ = ∫ Q ⋅ e∫
Pdx Pdx
dx + C −3
\ f(x) = 4x + c. x 4

cos x
∫ dx
⇒ e ∫
Pdx
= e sin x = e ln sin x = sin x

as lim x.f (1/4) = 4
⇒ yx sin x = ∫ 4 x cosec x sin x dx + C
x→01

x2 99. x2dx = y2dzx – 2x ydy


⇒ y sin x = 4 ⋅
+C
2
 y2  y2
⇒ y sin x = 2x2 + C
⇒ – dx = d 
 ⇒ − x = +c
⎛π ⎞  x  x
Given y ⎜ ⎟ = 0

⎝2⎠
As it passes through (1, 1) so c = 2
\ x2 + y2 – 2x = 0

π π2
⇒ yx sin
= 2⋅ +C
2 4 100. Differentiating both side
π 2
π 2 f (x) = 2x – x2 f(x)
⇒ 0 × 1 =
+C ⇒C = −
( )

HINTS AND EXPLANATIONS


2 2 2 1− x2
2x
π π 2 \ f(x) =
f ’( x ) =
⇒ y sin x = 2 x −

2
x= 2
6
1+ x2 (
x2 +1 )
2
π ⎛π ⎞ π
2 4  1  24
⇒ y sin
= 2⎜ ⎟ −
f(1/2) = f’ =
6 ⎝6⎠ 2 5  2  25

y π2 π2 y −8π 2

= − ⇒ =
2 18 2 2 18
This page is intentionally left blank.
CHAPTER Coordinates
10 and Straight Lines
LEARNING OBJECTIVES
After reading this chapter, you will be able to:
 Be acquainted with distance formula and section formula  Be familiar with general equation of a straight line and
 Know how to calculate area of triangle, area of quadrilat- the method to calculate the slope of a line
eral, area of a polygon  Understand various forms of the equation of a line and
 Learn the working rule to find the locus of a point, trans- distance of a point from a given line
lation of axes, rotation of axes and reflection of a point

DISTANCE FORMULA SOLVED EXAMPLES


The distance between two points P(x1, y1) and Q(x2, y2) is
given by 1. Consider the point A ≡ (0, 1) and B ≡ (2, 0). Let P be
a point on the line 4x + 3y + 9 = 0. Coordinates of the
point P such that |PA – PB| is maximum, are
 12 17 
(A)  − ,  (B) 
84 13
− , 
 5 5  5 5
FIGURE 10.1
 6 17 
(C)  − ,  (D)  none of these
PQ = ( x2 − x1 ) 2 + ( y2 − y1 ) 2  5 5
Solution: (A)
We have, |PA – PB | ≤ AB.
Info Box!
 Distance is always positive. Therefore, we often write
PQ instead of |PQ|.
 Distance MN between two points M(x , 0) and N(x ,
1 2
0) on the x-axis is |x2 – x1|. Similarly, the distance
between two points M(x1, y1) and x(x2, y1), (which lie
on a line parallel to x-axis) is |x2 – x1|.
 Distance AB between two points A(0, y ) and B(0,
1
y2) on the y-axis is |y2 – y1|. Similarly, the distance
between two points A(x1, y1) and B(x1, y2), (which
lie on a line parallel to y-axis) is |y2 – y1|.
 Distance between the origin O(0, 0) and the point

P(x, y) is OP = Thus, for |PA – PB| to be maximum, point A, B and P


x2 + y 2 .
must be collinear. The equation of line AB is
x + 2y = 2
10.2  Chapter 10

Solving it with the given line, we get P ≡  − , 


84 13 5 7
(A) 0 < β < (B)  0<β <
 5 5 3 2
2. A ladder of length ‘a’ rests against the floor and a wall 5 7
of a room. If the ladder begins to slide on the floor, (C) ≤ β ≤ (D)  none of these
3 2
then the locus of its middle point is
Solution: (C)
(A) x2 + y2 = a2 (B) 2(x2 + y2) = a2
Clearly point (0, b) lies on y-axis.
(C) x + y = 2a
2 2 2
(D) 4(x2 + y2) = a2
Drawing the graph of the three straight lines, we see
Solution: (D)  7  5
that Q ≡  0,  and P ≡  0, 
Let AB be the ladder. Let the cross section of the floor  2  3
and wall be taken as the coordinates axes. Let P(x, y)
be the mid-point of AB whose locus is required. Then
the coordinates of A and B are (2x, 0) and (0, 2y)
respectively.
Given, AB = a
⇒ ( 2 x − 0) 2 + ( 0 − 2 y ) 2 = a ⇒ 4 x 2 + 4 y 2 = a 2
or 4(x2 + y2) = a2, which is the required locus.

Therefore, the point (0, b) lies on or inside DABC,


when
5 7
≤β ≤
3 2
6. A straight line L with negative slope passes through
3. The point (2t2 + 2t + 4, t2 + t + 1) lies on the line x + 2y the point (8, 2) and intersects the positive coordinate
= 1 for axes at points P and Q. As L varies the absolute mini-
(A) all real values of t mum value of OP + OQ is (O is origin).
(B) some real values of t (A) 12 (B)  14
(C) 18 (D)  20
(C) t = −4 ± 7
8 Solution: (C)
(D) none of these The equation of the line is
Solution: (D) (y – 2) = m(x – 8), m < 0
The point (2t2 + 2t + 4, t2 + t + 1) lies on the line x + 2y  2 
= 1 if (2t2 + 2t + 4) + 2(t2 + t + 1) = 1 The coordinates of P and Q are P  8 − , 0  and
Q(0, 2 – 8m).  m 
i.e., 4t2 + 4t + 5 = 0
Here, discriminent = 16 – 4 × 4 × 5 = –64 < 0. 2
Therefore, OP + OQ = 8 − + 2 − 8m
\ No real value of t is possible. m
Hence, the given point cannot lie on the line. 2
= 10 + + 8( −m)
4. If the sum of the distances of a point from two perpen-   −m
dicular lines in a plane is 1, then its locus is 2
(A) square (B)  circle ≥ 10 + 2 × 8( −m) = 18
  −m
(C) straight line (D)  two intersecting lines
Thus, absolute minimum value of OP + OQ = 18.
Solution: (A)
7. If the point (2 cosq, 2 sinq) does not fall in that angle
If a and b are the lengths of perpendiculars, then
between the lines y = |x – 2| in which the origin lies
|a| + |b | = 1 (given), whose graph is a square.
then q belongs to
5. The condition to be imposed on b so that (0, b) lies on  π π
 π 3π 
or inside the triangle having sides y + 3x + 2 = 0, 3y – (A)  ,  (B)  − 2 , 2 
2 2   
2x – 5 = 0 and 4y + x – 14 = 0 is
(C) (0, p) (D)  none of these
Coordinates and Straight Lines  10.3

Solution: (B) ⇒ lai3 + mai2 + nai − 3m − n = 0


Clearly the point (2 cosq, 2 sinq) lies on the circle m n 3m + n
∴ ∑a 1 =−
l
, ∑ a1a2 = and a1a2 a3 =
l l
∴ a1a2 a3 + 3∑ a1 − ∑ a1a2 = 0

10. Let ax + by + c = 0 be a variable straight line, where a,


b and c are first, third and seventh terms of an increas-
ing A.P. Then, the variable straight line always passes
through a fixed point which lies on
(A) x2 + y2 = 4 (B)  x2 + y2 = 13
x2 + y2 = 4 (C) y = 2x (D) 
2
2x + 3y = 9
The two lines represented by the equation y = |x – 2| Solution: (B)
are y = x – 2 and y = 2 – x. Let d be the common difference of A.P., then
π π
From the figure, q can be vary from − to . b = a + 2d and c = a + 6d. Clearly, (b – a) × 3 = c – a
2 2 ⇒ 2a – 3b + c = 0
8. On the portion of the straight line x + y = 2 which is
Thus, the straight line ax + by + c = 0 passes through
intercepted between the axes, a square is constructed
the point (2, –3) which also satisfies x2 + y2 = 13.
away from the origin, with this portion as one of its
side. If p denotes the perpendicular distance of a side 11. If a, b, c form a G.P., then twice the sum of the ordi-
of this square from the origin, then the maximum nates of the points of intersection of the line ax + by +
value of p is c = 0 and the curve x + 2y2 = 0 is
b c
(A) 2 (B)  2 2 (A) (B) 
a a
(C) 3 2 (D)  4 2
a a
Solution: (C) (C) (D) 
c b
p = ON = OM + MN = ^ distance from
Solution: (A)
Let a, b, c be in G.P. with common ratio r.
Then, b = ar and c = ar2.
So, the equation of the line is ax + by + c = 0
⇒  ax + ary + ar2 = 0  ⇒  x + ry + r2 = 0
This line cuts the curve x + 2y2 = 0
Eliminating x, we get 2y2 – ry + r2 = 0
If the roots of the quadratic equation are y1 and y2, then
r b c
y1 + y2 = ⇒ 2( y1 + y2 ) = r = = .
2 a b
O to the line AB + AD
12. If a, b, c are in A.P., then the straight line ax + by + c =
2 0 will always pass through a fixed point whose coordi-
= +2 2 = 2+2 2 =3 2
2 nates are
 a3 a 2 − 3  (A) (–1, –2) (B)  (1, 2)
9. If the points Pi  i , i  , i = 1, 2, 3 are collinear (C) (–1, 2) (D)  (1, –2)
 ai − 1 ai − 1 
and a1, a2 and a3 are distinct real numbers, then Solution: (D)
Since a, b, c are in A.P., \ 2b = a + c
(A) a1a2a3 – Sa1a2 – 3Sa1 = 0
or a – 2b + c = 0
(B) a1a2a3 + Sa1a2 – 3Sa1 = 0
⇒ The line ax + by + c = 0 passes through the point
(C) a1a2a3 – Sa1a2 + 3Sa1 = 0
(1, –2).
(D) – a1a2a3 + Sa1a2 – 2Sa1 = 0
Solution: (C)
SECTION FORMULAE
Let the given points lie on the line lx + my + n = 0
1. The coordinates of the point P(x, y) dividing the line seg-
 a3   a2 − 3 
∴ l  i  + m i +n = 0 ment joining the two points A(x1, y1) and B(x2, y2) inter-
 ai − 1   ai − 1  nally in the ratio m1 : m2, are given by
10.4  Chapter 10

SOLVED EXAMPLES

13. If a triangle has its orthocentre at (1, 1) and circumcen-


3 3
tre at  ,  , then the coordinates of the centroid of
FIGURE 10.2 2 4
the triangle are
m1 x2 + m2 x1 m y + m2 y1 4 5
(A)  , −  (B) 
x= ,y= 1 2 4 5
m1 + m2 m1 + m2 3,6
3 6  
 4 5  4 5
(C)  − ,  (D)  − 3 ,− 6 
 3 6  
Info Box! Solution: (B)
 To remember the formula it is helpful to note that Since the centroid divides the line joining the ortho-
m1 is multiplied by the coordinate ‘away from it’ centre and circumcentre in the ratio 2 : 1 internally,
and similarly is m2 and the sum is then divided by therefore, if the centroid is (x, y), then
m1 + m2. Thus the above result can be remem-
bered with the help of the figure given below.

3 3
2 ⋅ + 1 ⋅1 2 ⋅ + 1 ⋅1
4 5
FIGURE 10.3 x= 2 = and y = 4 = .
 The coordinates of the point P(x, y), dividing the 2 +1 3 2 +1 6
line segment joining the two points A(x1, y1) and 4 5
B(x2, y2) externally in the ratio m1:m2, are given by
\ Coordinates of centroid are  ,  .
3 6
m1x2 − m2 x1 m y − m2 y1 14. A rectangle has two opposite vertices at the points (1,
x= ,y= 1 2
m1 − m2 m1 − m2 2) and (5, 5). If the other vertices lie on the line x = 3,
then the coordinates of the other vertices are
(A) (3, –1), (3, –6) (B)  (3, 1), (3, 5)
(C) (3, 2), (3, 6) (D)  (3, 1), (3, 6)
Solution: (D)
FIGURE 10.4 Let A ≡ (1, 2) and C ≡ (5, 5). Since the vertices B and D lie
 The coordinates of the mid-point of the line seg- on the line x = 3, therefore, let B ≡ (3, y1) and D ≡ (3, y2).
ment joining the two points A(x1, y1) and B(x2, Since AC and BD bisect each other, so they have
y2) are given by  x1 + x2 , y1 + y2  . same middle point
 
 2 2 

FIGURE 10.5

QUICK TIPS
The coordinates of any point on a line joining the two points y1 + y2 2 + 5
i.e., =
 λ x + x1 λ y2 + y1  2 2
A and B are given by  2 ,  . Such a point
 λ +1 λ +1  or y1 + y2 = 7 (1)
divides the given line in the ratio λ : 1. If λ is positive, then Also, BD2 = AC2
the point divides internally and if λ is negative, then the ⇒ (y1 – y2)2 = (1 – 5)2 + (2 – 5)2 = 25
point divides externally. or y1 – y2 = ± 5 (2)
Coordinates and Straight Lines  10.5

Solving (1) and (2), we get y1 = 6, y2 = 1 SOLVED EXAMPLE


or y1 = 1, y2 = 6
Thus, the other vertices of the rectangle are (3, 1) 16. The diagonals of a parallelogram PQRS are along the
and (3, 6). lines x + 3y = 4 and 6x – 2y = 7. Then PQRS must be a
15. If the centroid and a vertex of an equilateral triangle (A) rectangle (B)  square
are (2, 3) and (4, 3) respectively, then the other two (C) cyclic quadrilateral (D)  rhombus
vertices of the triangle are
Solution: (D)
(A) (1, 3 ± 3 ) (B)  ( 2, 3 ± 3 ) Since the product of slopes of the diagonals is –1,
(C) (1, 2 ± 3 ) (D)  ( 2, 2 ± 3 ) therefore, the diagonals are at right angles. Hence,
Solution: (A) PQRS is a rhombus.
G being the centroid, divides AD in the ratio 2 : 1.
AREA OF A TRIANGLE
The area of ∆ABC with vertices A(x1, y1), B(x2, y2) and C(x3,
y3) is given by:
1
∆ = [ x1 ( y2 − y3 ) + x2 ( y3 − y1 ) + x3 ( y1 − y2 )]
2
1
= [( x1 y2 + x2 y3 + x3 y1 ) − ( x2 y1 + x3 y2 + x1 y3 )]
Since AG = 2,  \  GD = 1, 2
\ Coordinates of D, using section formula, are x1 y1 1
D(1, 3). 1
3 = x2 y2 1 .
Now AD = 1 + 2 = 3, ∴ tan 60° = ⇒ BD = 3. 2
BD x3 y3 1

∴ B ≡ (1, 3 + 3 ) and C ≡ (1, 3 − 3 ).

Info Box!
The given figure is a

FIGURE 10.8

QUICK TIPS
FIGURE 10.6  Area of a triangle is always taken as positive.
(a) Opposite sides are (a)  Opposite sides are  If area of a triangle is given, then use ± sign.
equal equal
(b) Diagonals are unequal (b)  Diagonals are equal
(c) Diagonals bisect (c) Diagonals bisect CONDITION FOR COLLINEARITY OF THREE
each other each other.
POINTS
The points A(x1, y1), B(x2, y2) and C(x3, y3) will be collinear
(i.e., will lie on a straight line) if the area of the triangle,
assumed to be formed by joining them is zero.
i.e., 1
[( x1 y2 − x2 y1 ) + ( x2 y3 − x3 y2 ) + ( x3 y1 − x1 y3 )] = 0
FIGURE 10.7 2
(a)  All four sides are equal (a) All four sides are equal or [(x1 y2 – x2 y1) + (x2 y3 – x3 y2 ) + (x3 y1 – x1 y3 )] = 0,
(b)  Diagonals are unequal (b) Diagonals are equal
(c) Diagonals bisect each (c)  Diagonals bisect each which can also be written in the form
other at right angles. other at right angles. x1( y2 – y3) + x2( y3 – y1) + x3( y1 – y2) = 0.
10.6  Chapter 10

Sign of an area: An area will be considered to be +ve,  If a1x + b1y + c1 = 0; a2x + b2y + c2 = 0 and a3x + b3y +
if in going round the boundary, it always lies to the left c3 = 0 are the equations of three sides of a triangle, then
i.e., if the order of description of the boundary curve is the area of triangle is given by
anti-clockwise. It will be regarded –ve otherwise. a1 b1 c1
a2 b2 c2
a3 b3 c3
∆=
b2 c2 c2 a2 a2 b2
b3 c3 c3 a3 a3 b3

FIGURE 10.9
2c 2
 Area of the rhombus formed by ax ± by ± c = 0 is .
STAIR METHOD FOR FINDING THE AREA ab

• Write the coordinates of the vertices taken in order in  Area of the parallelogram formed by the lines a1x + b1y +
two columns. At the end, repeat the coordinates of the c1 = 0, a2x + b2y + c2 = 0, a1x + b1y + d1 = 0, a2x + b2y
+ d2 = 0 is
first vertex.
• Mark the arrow-heads as indicated. Each arrow-head (d1 − a1)(d2 − c2 )
shows the product. a1b2 − a2b1
• The sign of the product remains the same for downward
arrows while it changes for an upward arrow.
• Divide the result by 2. SOLVED EXAMPLES

17. Through the point P(a, b ), where ab > 0 the straight
x y
line + = 1 is drawn so as to form with coordinate
a b
axes a triangle of area S. If ab > 0, then the least value
of S is
(A) ab (B)  2ab
(C) 4ab (D)  none of these
Solution: (B)
FIGURE 10.10 The equation of the given line is
x y
1 + = 1 (1)
Thus, ∆ = [( x1 y2 − x2 y1 ) + ( x2 y3 − x3 y2 ) + ( x3 y1 − x1 y3 )] a b
2

QUICK TIPS
 In an equilateral triangle

This line cuts x-axis and y-axis at A(a, 0) and B(0, b)


FIGURE 10.11
respectively.
(i)  having sides a, area is
3 2 Since area of DOAB = S (Given)
a
4 1
p2 ∴ ab = S or ab = 2S (∵ ab > 0) (2)
(ii)  having length of perpendicular as ‘p’, area is . 2
3
Coordinates and Straight Lines  10.7

Since the line (1) passes through the point P(a, b) 20. Let P(2, – 4) and Q(3, 1) be two given points. Let R (x,
α β α aβ y) be a point such that (x – 2) (x – 3) + (y – 1) (y + 4) =
∴ + = 1 or + =1 [Using (2)]
a b a 2S 13
0. If area of DPQR is , then the number of possible
or a2b – 2aS + 2aS = 0. 2
Since a is real, \ 4S2 – 8abS ≥ 0 positions of R are
(A) 2 (B)  3
or 4 S 2 ≥ 8αβ S or S ≥ 2αβ ∵ S = ab > 0 as ab > 0 
1
2 (C) 4 (D)  none of these
 
Hence the least value of S = 2ab. Solution: (A)
18. P(3, 1), Q(6, 5) and R(x, y) are three points such that We have
the angle RPQ is a right angle and the area of DRPQ = (x – 2) (x – 3) + (y – 1) (y + 4) = 0
7, then the number of such points R is  y + 4   y −1 
⇒  ×  = −1
(A) 0 (B)  1  x −2   x −3
(C) 2 (D)  4 π
⇒ RP ⊥ RQ or ∠PRQ =
Solution: (C) 2
Since the angle RPQ is a right angle, \ The point R lies on the circle whose diameter is PQ.
\ slope of RP × slope of PQ = –1
1− y 5 −1
⇒ × = −1 ⇒ 3 x + 4 y = 13 (1)
3− x 6 −3
Also, area of DRPQ = 7
x y 1
1
⇒ | 3 1 1 |=7
2
6 5 1
1 13
⇒ [ x (1 − 5) − y(3 − 6) + 1(15 − 6)] = ±7 Now, area of ∆PQR =
2 2
⇒  – 4x + 3y + 9 = ± 14  ⇒  – 4x + 3y = 5 (2) 1 13
and – 4x + 3y = –23 (3) ⇒ × 26 × (altitude) =
2 2
Solving Eq. (1) and (2) and (1) and (3), we get two
26
different coordinates of the point R. So, there are two ⇒ altitude = = radius
such points R. 2
⇒  there are two possible positions of R.
19. If two vertices of an equilateral triangle have integral
coordinates then the third vertex will have 21. The base of a triangle lies along the line x = a and is of
(A) coordinates which are irrational length a. The area of the triangle is a2, if the vertex lies
(B) atleast one coordinate which is irrational on the line
(C) coordinates which are rational (A) x = 0 (B)  x = –a
(D) coordinates which are integers (C) x = 3a (D)  x = –3a
Solution: (B) Solution: (B, C)
Let the vertices of the equilateral triangle be (x1, y1), Let h be the height of the triangle.
(x2, y2) and (x3, y3). If none of xi and yi (i = 1, 2, 3) are Since, the area of the triangle is a2
irrational, then 1
∴ × a × h = a 2 ⇒ h = 2a
x1 y1 1 2
1
area of ∆ = x2 y2 1 = retional. Since the base lies along the line x = a, the vertex
2 lies on the line parallel to the base at a distance 2a
x3 y3 1
from it. So, the required lines are
But the area of an equilateral triangle
x = a ± 2a i.e., x = –a or x = 3a
3
= = (side) 2 irrational 22. If a, c, b are three terms of a G.P., then the line ax + by
4
Thus, the two statements are contradictory. Therefore, +c=0
both the coordinates of the third vertex cannot be (A) has a fixed direction
rational. (B) always passes through a fixed point
10.8  Chapter 10

(C) forms a triangle with the axes whose area is 24. Two vertices of a triangle are (2, –1) and (3, 2) and
constant third vertex lies on the line x + y = 5. If the area of the
(D) always cuts intercepts on the axes such that their triangle is 4 units then third vertex is
sum is zero. (A) (0, 5) or (4, 1) (B)  (5, 0) or (1, 4)
Solution: (C) (C) (5, 0) or (4, 1) (D)  (0, 5) or (1, 4)
Since a, c, b are in G.P., Solution: (B)
\ c2 = ab(1) Let A ≡ (2, –1) and B ≡ (3, 2).
The area of the triangle Let the third vertex be C (a, b).
1  c  c Then, a + b = 5 (given) (1)
= × − × −
2  a   b  1
2 −1 1
Area of ∆ABC = 3 2 1 = ± 4 (given )
2
α β 1
⇒ b – 3a = 1 (2)
or b – 3a = –15 (3)
Solving (1) and (2), we get, a = 1, b = 4
Solving (1) and (3), we get, a = 5, b = 0
Thus, the third vertex is either (5, 0) or (1, 4).

AREA OF A QUADRILATERAL
The area of a quadrilateral, whose vertices are A(x1, y1),
1 c2 1
= × =  [Using (1)] B(x2, y2), C(x3, y3) and D(x4, y4), is
2 ab 2
= constant 1  x1 y1 x2 y2 x3 y3 x4 y4 
=  + + + 
2  x2 y2 x3 y3 x4 y4 x1 y1 
23. If x1, x2, x3 as well as y1, y2, y3 are in G.P. with the same
common ratio, then the points (x1, y1), (x2, y2) and (x3,
y3) Info Box!
(A) lie on a straight line The rule for writing down the area of a quadrilateral is
(B) lie on an ellipse the same as that of a triangle.
(C) lie on a circle
(D) are vertices of a triangle
Thus, area of quadrilateral with vertices (xr, yr),
Solution: (A) r = 1, 2, 3, 4 is
x2 x3 y2 y3 1
Let = = r and = = r [( x1 y2 − x2 y1 ) + ( x2 y3 − x3 y2 ) + ( x3 y4 − x4 y3 ) + ( x4 y1 − x1 y4 )]
x1 x2 y1 y2 2
⇒  x2 = x1r, x3 = x1r2, y2 = y1r and y3 = y1r2.
We have,
AREA OF A POLYGON
x1 y1 1 x1 y1 1
• The area of a polygon of n sides with vertices A1(x1, y1),
∆ = x2 y2 1 = x1r y1r 1 A2(x2, y2), ..., An(xn, yn) is
x3 y3 1 x1r 2 y1r 2 1
1  x1 y1 x2 y2 x yn −1 xn yn 
x1 y1 1 =  + +  + n −1 + 
2  x2 y2 x3 y3 xn yn x1 y1 
= 0 0 1− r
0 0 1− r

STAIR METHOD
(Applying R3 → R3 – rR2 and R2 → R2 – rR1) = 0

( R2 and R3 are identical) Repeat first coordinates one time in last. For down arrow
Thus, (x1, y1), (x2, y2), (x3, y3) lie on a straight line. use positive sign and for up arrow use negative sign.
Coordinates and Straight Lines  10.9

100
Now, area of Sn < 1 ⇒ an2 < 1 ⇒ <1
2n −1
⇒ 2n > 200 > 27  ⇒  n > 7
∴ Area of polygon = \ n = 8, 9, 10

LOCUS
The locus of a moving point is the path traced by it under
1 certain geometrical condition or conditions.
= {[( x1 y2 + x2 y3 +  + xn y1 ) − ( y1 x2 + y2 x3 +  + yn x1 )]}
2 For example, if a point moves in a plane under the
geometrical condition that its distance from a fixed point O
in the plane is always equal to a constant quantity a, then
SOLVED EXAMPLES the curve traced by the moving point will be a circle with
centre O and radius a. Thus, locus of the point is a circle
25. The area of the region enclosed by 4 |x| + 5 |y| ≤ 20 is with centre O and radius a.
(A) 10 (B)  20
(C) 40 (D)  none of these WORKING RULE TO FIND THE LOCUS OF A POINT
Let the coordinates of the moving point P be (h, k).
Solution: (C)

 Using the given geometrical conditions, find the relation


The four lines enclosing the given region are 4x + 5y
between h and k. This relation must contain only h, k and
= 20, 4x – 5y = 20, – 4x + 5y = 20 and – 4x – 5y = 20. known quantities.
Clearly, the four lines form a rhombus having
 Express the given relation in h and k in the simplest form
diagonals of length 10 and 8. and then put x for h and y for k. The relation, thus obtained,
will be the required equation of the locus of (h, k).

TRANSLATION OF AXES
Sometimes a problem with a given set of axes can be
solved more easily by translation of axes. The translation of
axes involves the shifting of the origin to a new point, the
new axes remaining parallel to the original axes.

1
\ Required area = × 10 × 8 = 40.
2
26. Let S1, S2, ... be squares such that for each n ≥ 1, the
length of a side of Sn equals the length of a diagonal
of Sn+1. If the length of a side of S1 is 10 cm, then for
which of the following values of n is the area of Sn less
than 1 cm2?
(A) 7 (B)  8
(C) 9 (D)  10
FIGURE 10.12
Solution: (B, C, D)
Let a be the side of the square, then diagonal d = a 2.
Given : an = 2an +1 Let OX, OY be the original axes and O′ be the new origin.
Let coodinates of O’ referred to original axes, i.e., OX, OY
an an −1 an − 2 a1 be (h, k).
⇒ an +1 = = = = =
2 ( 2) 2
( 2) 3
( 2 )n Let O′ X′ and O′ Y′ be drawn parallel to and in the same
direction as OX and OY respectively. Let P be any point in
a1 a1 10 the plane having coordinates (x, y) referred to old axes and
∴ an +1 = n
⇒ an = n −1
= n −1
( 2) ( 2) (X, Y) referred to new axes. Then,
2 2
10.10  Chapter 10

x = OM = ON + NM = ON + O′ M′
= h + X = X + h  or  X = x – h
and y = MP = MM′ + M′ P = NO′ + M′ P
= k + Y = Y + k. or  Y = y – k

Thus, the point whose coordinates were (x, y) has now the
coordinates (x – h, y – k).

QUICK TIPS
If origin is shifted to point (h, k) without rotation of axes,
then new equation of curve can be obtained by putting x +
FIGURE 10.14
h in place of x and y + k in place of y

x = h + x′ cosθ – y′ sinθ
and y = β + x′ sinθ + y′ cosθ
ROTATION OF AXES
Rotation of Axes without Changing REFLECTION (IMAGE) OF A POINT
the Origin
Let (x, y) be any point, then its image with respect to
Let OX, OY be the original axes and OX′, OY′ be the new (i) x axis is (x, –y) (ii)  y-axis is (–x, y)
axes obtained by rotating OX and OY through an angle θ (iii) origin is (– x, –y) (iv) line y = x is (y, x)
in the anticlockwise sense. Let P be any point in the plane
having coordinates (x, y) with respect to axes OX and OY QUICK TIPS
and (x′, y′) with respect to axes OX′ and OY′. Then,  If area is a rational number. Then the triangle cannot be
equilateral.
 If two opposite vertices of a rectangle are (x , y ) and (x ,
1 1 2
y2), then its area is |(y2 – y1)(x2 – x1)|.
 If two opposite vertices of a square are A(x , y ) and
1 1
C(x2, y2), then its area is

1 2 1
= AC = [( x2 − x1)2 + ( y2 − y1)2 ]
2 2

SOLVED EXAMPLES
FIGURE 10.13
27. The image of the point (3, –8) under the transforma-
x = x′ cosθ – y′ sinθ; y = x′ sinθ + y′ cosθ tion (x, y) → (2x + y, 3x – y) is
and x′ = x cosθ + y sinθ;  y′ = – x sinθ + y cosθ
(A) (–2, 17) (B)  (2, 17)
The above relation between (x, y) and (x′, y′) can be (C) (–2, –17) (D)  (2, –17)
easily obtained with the help of following table Solution: (A)
Let (x1, y1) be the image of the point (x, y) under the
x↓ y↓
given transformation.
x′→ cosθ sinθ Then, x1 = 2x + y = 2 (3) – 8 = –2
y′→ – sinθ cosθ and y1 = 3x – y = 3 (3) – (–8) = 17
Hence, the image is (–2, 17).
Change of Origin and Rotation of Axes If origin is
changed to O′ (h, k) and axes are rotated about the new 28. Without changing the direction of coordinates axes,
origin O′ by angle θ in the anticlockwise sense such that origin is transferred to (a, b) so that the linear terms in
the new co-ordinates of P(x, y) become (x′, y′), then the the equation x2 + y2 + 2x – 4y + 6 = 0 are eliminated.
equations of transformation will be The point (a, b) is
Coordinates and Straight Lines  10.11

(A) (–1, 2) (B)  (1, –2) (C) 3 x + y − 5 3 = 0


(C) (1, 2) (D)  (–1, –2)
(D) 3 x − y − 5 3 = 0
Solution: (A)
The given equation is Solution: (C)
x2 + y2 + 2x – 4y + 6 = 0 (1) The refracted ray passes through the point (5, 0) and
Putting x = x′ + a, y = y′ + b in (1), we get makes an angle 120º with positive direction of x-axis
x′2 + y′2 + x′(2a + 2) + y′(2b – 4) + (a 2 + b 2 + 2a – 4b
+ 6) = 0
To eliminate linear terms, we should have 2a + 2 = 0
and 2b – 4 = 0
⇒  a = –1 and b = 2
\ (a, b) ≡ (–1, 2)
29. Let P be the image of the point (–3, 2) with respect
to x-axis. Keeping the origin as same, the coordinate
axes are rotated through an angle 60° in the clockwise \ The equation of the refracted ray is (y – 0) = tan
sense. The coordinates of point P with respect to the 120º (x – 5)
new axes are ⇒ y = − 3 ( x − 5) or 3 x + y − 5 3 = 0
 2 3 − 3 (3 3 + 2) 
(A)  ,−  31. A line L has intercepts a and b on the coordinate axes.
 2 2
  When the axes are rotated through an angle, keeping
 2 3 −3 3 3 + 2 the origin fixed, the same line L has intercepts p and q.
(B)  ,  Then,
 2 2 
 1 1 1 1
(A) 2 + 2 = 2 + 2
 ( 2 3 − 3) 3 3 + 2  a b p q
(C)  − , 
 2 2  1 1 1 1
 (B) − 2 = 2− 2
2
(D) none of these a b p q
Solution: (A) 1 1  1 1 
(C) + 2 = 2 2 + 2 
Since P is the image of the point (–3, 2) with respect a 2
b p q 
to x-axis, therefore, the coordinates of P are (–3, –2). (D) none of these
Let (x′, y′ ) be the coordinates of P with respect to
new axes. Then, Solution: (A)
x′ = x cos a + y sin a = –3 cos (–60º) –2 sin (–60º) Since the line L has intercepts a and b on the coordi-
3 2 3 −3 nate axes, therefore its equation is
=− + 3= . x y
2 2 + = 1 (1)
y′ = – x sin a + y cos a = 3 sin (–60º) – 2 cos (–60º) a b
3 3+2 When the axes are rotated, its equation with respect to
= −
 2  the new axes and same origin will become
 
x y
 2 3 − 3  3 3 + 2  + = 1 (2)
\ Coordinates of P are  ,− p q
 2  
.
 2    In both the cases, the length of the perpendicular from
30. A ray of light travelling along the line x + 3 y = 5 the origin to the line will be same.
is incident on the x-axis and after refraction it enters 1 1
π ∴ =
the other side of the x-axis by turning away from 1 1 1 1
+ +

6
the x-axis. The equation of the line along which the a2 b2 p2 q2
refracted ray travels is
1 1 1 1
(A) x + 3 y − 5 3 = 0 or 2
+ 2 = 2+ 2
a b p q
(B) x − 3 y − 5 3 = 0 which is the required relation.
10.12  Chapter 10

GENERAL EQUATION OF A STRAIGHT LINE


An equation of the form: ax + by + c = 0, where a, b, c are
any real numbers not all zero, always represents a straight
line.
Equation of a straight line is always of first degree in
x and y.

SLOPE OF A LINE
π \ Q = (5, 3)

If a line makes an angle θ  θ ≠  with the positive direc- Since the image of the point Q(5, 3) w.r.t. the line
 2
y = 0 is (a, b).
tion of x-axis, the slope or gradient of that line is usually
\ a = 5 and b = –3
denoted by m, i.e., tan θ = m.
\ (a, b) = (5, –3)

QUICK TIPS
INTERCEPT OF A LINE ON THE AXES
 The slope of a line parallel to x-axis = 0 and perpendicular
to x-axis is undefined. 1. Intercept of a line on x-axis If a line cuts x-axis at a
 If three points A, B, C are collinear, then slope of AB = point (a, 0), then a is called the intercept of the line on
slope of BC = slope of AC. x-axis. | a | is called the length of the intercept of the line
on x-axis. Intercept of a line on x-axis may be positive or
 If a line is equally inclined to the axes, then it will make an
negative.
angle of 45º or 135º with x-axis (i.e., positive direction
2. Intercept of a line on y-axis If a line cuts y-axis at a point
of x-axis) and hence its slope will be tan 45º or tan 135º
(0, b), then b is called the intercept of the line on y-axis
= ± 1.
and | b | is called the length of the intercept of the line on
 Slope of the line joining two points (x1, y1) and (x2, y2) is y-axis. Intercept of a line on y-axis may be positive or
given as
negative.
y1 − y2 y2 − y1 Difference of ordinates Equations of Lines Parallel to Axes
m= = = .
x1 − x2 x2 − x1 Difference of ab
bscissae
Equation of x-axis The equation of x-axis is y = 0.
a Equation of y-axis The equation of y-axis is x = 0.
 Slope of the line ax + by + c = 0, b ≠ 0 is − , i.e.,
b Equation of a line parallel to y-axis The equation of
the straight line parallel to y-axis at a distance ‘a’ from it
(Coefficient of x )
. on the positive side of x-axis is x = a.
(Coefficient of y ) If a line is parallel to y-axis, at a distance a from it and
is on the negative side of x-axis, then its equation is x = – a.
Equation of a line parallel to x-axis The equation of
the straight line parallel to x-axis at a distance b from it on
SOLVED EXAMPLES the positive side of y-axis is y = b.
If a line is parallel to x-axis, at a distance b from it and
is on the negative side of y-axis, then its equation is y = –b.
32. The image of the point P(3, 5) with respect to the line
y = x is the point Q and the image of Q with respect to
the line y = 0 is the point R(a, b), then (a, b)
SOLVED EXAMPLES
(A) (5, 3)
(B)  (5, –3)
33. A square is constructed on the portion of the line x +
(C) (–5, 3)
y = 5 which is intercepted between the axes, on the
(D)  (–5, –3)
side of the line away from origin. The equations to the
Solution: (B) diagonals of the square are
Let (x1, y1) be the image of the point P(3, 5) with (A) x = 5, y = –5
respect to the line y = x. Then, x1 = 5, y1 = 3. (B) x = 5, y = 5
Coordinates and Straight Lines  10.13

(C) x = –5, y = 5
(D) x – y = 5, x – y = –5
Solution: (B)
Clearly, the equations of the two diagonals are x = 5
and y = 5.

FIGURE 10.15

Point-Slope Form
The equation of a straight line passing through the point
(x1, y1) and having slope m is given by
(y – y1) = m(x – x1).

Two-Point Form
34. If a straight line cuts intercepts from the axes of coor- The equation of a straight line passing through two points
dinates the sum of the reciprocals of which is a con- (x1, y1) and (x2, y2) is given by
stant k, then the line passes through the fixed point
y2 − y1
1 1 ( y − y1 ) = ( x − x1 ).
(A) (k, k) (B) k,k  x2 − x1
 
(C) (k, –k) (D) 
(–k, k)
Solution: (B)
Let the equation of the line be SOLVED EXAMPLES
x y
+ = 1 (1) 35. In the above problem, coordinates of the point P such
a b that |PA – PB| is minimum are
Its intercepts on x-axis and y-axis are a and b  3 12 
(A)  − , − 
9 12
respectively. (B)   , 
 20 5  20 5 
1 1
 9 12 
+ =k (C)  , − 
Given: 9 12
a b − , 
(D) 
 20 5  20 5 
1 1 1/k 1/k
⇒ + = 1 or + = 1 (2) Solution: (A)
ak bk a b
The minimum value of |PA – PB| is zero which is
From (2) it follows that the line (1) passes through the attained if PA = PB, i.e., P must lie on the perpendicu-
1 1 lar bisector of AB.
fixed point  ,  .
k k The equation of perpendicular bisector of AB is

1 3
EQUATION OF A STRAIGHT LINE IN y− = 2( x − 1) or y = 2x −
2 2
VARIOUS FORMS
 9 12 
Slope-Intercept Form Solving it with the given line, we get P ≡  − , − 
 20 5
The equation of a straight line whose slope is m and which
cuts an intercept c on the y-axis is given by 36. Given the system of straight line a(2x + y – 3) + b(3x
+ 2y – 5) = 0, the line of the system farthest form the
y = mx + c. point (4, –3) has the equation
If the line passes through the origin, then c = 0 and (A) 3x – 4y + 1 = 0 (B)  4x + 3y – 5 = 0
hence the equation of the line will become y = mx. (C) 7x – y + 4 = 0 (D)  none of these
10.14  Chapter 10

Solution: (A)
The given system of lines pass through (1, 1)
So, the required line is the through (1, 1) and per-
pendicular to the join of (1, 1) and (4, –3).
y −1 3
\ The equation of line is = , i.e., 3 x − 4 y + 1 = 0
x −1 4
37. The image of the point (–8, 12) with respect to the line
mirror 4x + 7y + 13 = 0 is FIGURE 10.16
(A) (16, –2) (B)  (–16, 2)
(C) (16, 2) (D)  (–16, –2)
Solution: (D) SOLVED EXAMPLES
Equation of the given line is
4x + 7y + 13 = 0 (1) 38. Through the point (1, 1), a straight line is drawn so as
Let Q (a, b) be the image of the point P(–8, 12) w.r.t. to form with coordinate axes a triangle of area S. The
line (1). intercepts made by the line on the coordinate axes are
Then, PQ ^ line (1) and PC = CQ. the roots of the equation
Equation of the line PC is (A) x2 – |S| x + 2 |S| = 0
7 (B) x2 + |S| x + 2 |S| = 0
( y − 12) = ( x + 8)
4 (C) x2 – 2 |S| x + 2 |S| = 0
[PC is ^ to the line (1) and passes through (–8, 12)] (D) none of these
or 7x – 4y + 104 = 0 (2) Solution: (C)
Solving Eq. (1) and (2), we get If a, b are the intercepts made by the line, then the
x y
x = –12 and y = 5. \ C ≡ (–12, 5) equation of the line is + = 1.
a b
1 1
Since it passes through (1, 1), ∴ + =1
a b
a+b
⇒ = 1 (1)
ab
Also, area of the triangle made by the straight line
on the coordinate axes is S
1
∴ ab = |S | i.e., ab = 2 |S | (2)
2
So, by (1), a + b = 2 |S|(3)
From (2) and (3), the intercepts a and b are the
roots of the equation x2 – 2 |S| x + 2 |S| = 0.
Since C is mid-point of PQ, 39. A line passing through the point P(4, 2), meets the
α −8 β + 12 x-axis and y-axis at A and B respectively. If O is the
∴ − 12 = and 5 =
2 2 origin, then locus of the centre of the circum circle of
⇒ a = –16 and b = –2 DOAB is
\ Q ≡ (–16, –2). (A) x–1 + y–1 = 2 (B)  2x–1 + y–1 = 1
(C) x + 2y = 1
–1 –1
(D)  2x–1 + 2y–1 = 1
Intercept Form Solution: (B)
Let the coordinates of A and B be (a, 0) and (0, b)
The equation of a straight line which cuts off intercepts a respectively.
and b on x-axis and y-axis respectively is given by Then, equation of line AB is
x y x y
+ =1 + =1
a b a b
Coordinates and Straight Lines  10.15

Since, it passes through the point P(4, 2) Normal Form (or Perpendicular Form)
The equation of a straight line upon which the length of the
perpendicular from the origin is p and the perpendicular
makes an angle α with the positive direction of x-axis is
given by

4 2
∴ + = 1. (1)
a b
a b
Now, centre of the circumcircle of ∆OAB =  ,  . FIGURE 10.17
2 2
x cos α + y sin α = p.
2 1
So, Eq. (1) can be written in the form + =1 In normal form of equation of a straight line p is always
a/2 b/2
taken as positive and α is measured from positive direction
2 1 of x-axis in anti-clockwise direction between 0 and 2π.
\ locus of circumcentre is + = 1 or 2 x −1 + y −1 = 1
x y
40. If the equal sides AB and AC (each equal to a) of a Parametric Form (or Symmetric Form)
right angled isosceles triangle ABC be produced to P The equation of a straight line passing through the point
and Q so that BP × CQ = AB2, then the line PQ always (x1, y1) and making an angle θ with the positive direction
passes through the fixed point of x-axis is given by
(A) (a, 0) (B)  (0, a)
x − x1 y − y1
(C) (a, a) (D)  none of these = =r
cos θ sin θ
Solution: (C)
We take A as the origin and AB and AC as x-axis and where r is the distance of the point (x, y) from the point (x1, y1).
y-axis respectively.
Let AP = h, AQ = k.
Equation of the line PQ is

FIGURE 10.18

QUICK TIPS
The coordinates (x, y) of any point P on the line at a distance
r from the point A(x1, y1) can be taken as
(x1 + r cosθ, y1 + r sinθ)
or (x1 – r cosθ, y1 – r sinθ)
x y
+ = 1 (1) where the line is inclined at an angle θ with x-axis
h k
Given, BP × CQ = AB2
⇒ (h – a) (k – a) = a2 SOLVED EXAMPLES
⇒ hk – ak – ah + a2 = a2 or ak + ha = hk
a a 41. A line joining two points A(2, 0) and B(3, 1) is rotated
or + = 1 (2)
h k about A in anticlockwise direction through an angle
From (2), it follows that line (1) i.e., PQ passes 15°. If B goes to C in the new position, then the coor-
through the fixed point (a, a). dinates of C are
10.16  Chapter 10

  3 Clearly y ≥ 2.
3
(A)  2,   2, −
(B)  
 2   2 

 1 3
(C)  2 + ,  (D)  none of these
 2 2 

Solution: (C)

Also, y-axis is the bisector of the angle between


the two lines. P1, P2 are two points on these lines, at a
distance 5 units from A. Q is the foot of the ^ from P1
and P2 on the bisector (y-axis).
Then, the coordinates of Q are (0, 2 + 5 cos 30º)
Slope of line
0 −1  5 3  1 
AB = = 1 = tan 45° =  0, 2 +  =  0, ( 4 + 5 3 ) 
 2   2 
2−3 
\ ∠BAX = 45°.
Given ∠CAB = 15°. 43. If the straight line drawn through the point P( 3 , 2)
\ ∠CAX = 60°. π
and making an angle ⋅ with the x-axis meets the line
\ Slope of line AC = tan 60º = 3 . 6
Now, line AC makes an angle of 60° with positive 3 x − 4 y + 8 = 0 at Q, then the length of PQ is
direction of x-axis and (a) 4 (b)  5
(c) 6 (d)  none of these
AC = AB = (3 − 2) 2 + (1 − 0) 2 = 2
Solution: (C)
\ Coordinates of C are ( 2 + 2 cos 60°, 0 + 2 sin 60°)
The given line is
 1 3
i.e.,  2 + , 
 2 2 
42. P is a point on either of the two lines y − 3 | x| = 2 at
a distance of 5 units from their point of intersection.
The coordinates of the foot of the perpendicular from
P on the bisector of the angle between them are 3 x − 4 y + 8 = 0 (1)
(A) 0, ( 4 + 5 3 )  or 0, ( 4 − 5 3 )  depend-
1 1
Let PQ = r
 2   2 
ing on which line the point P is taken Then, the coordinates of Q are
 1   π π  3 r
(B) 0, ( 4 + 5 3 )   3 + r cos 6 , 2 + r sin 6  or  3 + 2 r , 2 + 2 
 2     
 1 
(C) 0, ( 4 − 5 3 )  Since the point Q lies on the given line,
 2 
5 5 3   3   r
(D)  ,  ∴ 3 3 + r  − 42 +  + 8 = 0
 2   2 
 2 2   
Solution: (B) ⇒ 6 + 3r – 16 – 4r + 16 = 0 or r = 6
Hence, PQ = 6
Equation, of two lines are
y = 3 x + 2, if x≥0 44. A line is drawn from the point P(a, b), making an
angle q with the positive direction of x-axis, to meet
and y = − 3 x + 2, if x≥0 the line ax + by + c = 0 at Q. The length of PQ is
Coordinates and Straight Lines  10.17

aα + bβ + c aα + bβ + c QUICK TIPS
(A) − (B) 
a cos θ + b sin θ a2 + b2 Intercept of a straight line on x-axis can be found by putting
aα + bβ + c y = 0 in the equation of the line and then finding the value
(C) (D)  none of these of x. Similarly intercept on y-axis can be found by putting x =
a cos θ + b sin θ 0 in the equation of the line and then finding the value of y.
Solution: (A)
Equation of a straight line passing through the point
P(a, b) and making an angle q with positive direction
Normal Form
of x-axis is To reduce the equation Ax + By + C = 0 to the form x cos
x −α y − β α + y sin α = p, first express it as
= = r (say)
cos θ sin θ Ax + By = –C(1)
Coordinates of any point on this line are CASE 1. If C < 0 or –C > 0, then divide both sides of Eq. (1)

(a + r cosq, b + r sinq) by A2 + B 2 , we get


If it lies on the line ax + by + c = 0, then A B C
a(a + r cosq) + b(b + r sinq) + c = 0 x+ y=−
A +B
2 2
A +B2 2
A + B2
2
aα + bβ + c
⇒ r=−
a cos θ + b sin θ which is of the form x cos α + y sin α = p,
aα + bβ + c A B
Thus, PQ = r = − where, cos α = , sin α =
a cos θ + b sin θ A +B 2 2
A + B2
2

REDUCTION OF THE GENERAL EQUATION TO C


and  p = −
DIFFERENT STANDARD FORMS A + B2
2

Slope-Intercept Form: The general form, If C > 0 or –C < 0, then divide Eq. (1) by
CASE 2.
Ax + By + C = 0, of the straight line can be reduced to the − A + B 2 , we get
2

form y = mx + c by expressing y as
A C −A B C
y=− x − = mx + c x− y=
B B A +B 2 2
A +B 2 2
A + B2
2

A C
where m = − and c = − which is of the form x cos α + y sin α = p,
B B
A B
Thus, slope of the line Ax + By + C = 0 is where cos α = − , sin α = −
cofficient of x A A +B2 2
A + B2
2

m= =−
cofficient of y B C
and  p =
A + B2
2

Intercept Form
The equation Ax + By + C = 0 can be reduced to the form ANGLE BETWEEN TWO INTERSECTING LINES
x y The angle θ between two lines y = m1x + c1 and y = m2x +
+ = 1 by expressing it as
a b c2 is given by
Ax + By = –C
m1 − m2
A B tan θ = ± ,
or − x − y = 1, where C ≠ 0 1 + m1m2
C C
x y provided no line is perpendicular to x-axis and the acute
x y
or + = −1, which is of the form + = 1, angle θ is given by
C C a b
− −
A B m1 − m2
C C tan θ =
where a = − and b = − are intercepts on x-axis and 1 + m1m2
A B
y-axis, respectively.
10.18  Chapter 10

QUICK TIPS EQUATION OF A LINE PARALLEL TO A GIVEN


 If both the lines are perpendicular to x-axis, then the LINE
angle between them is 0º. The equation of a line parallel to a given line ax + by + c =
 If slope of one line is not defined (one of the lines is 0 is ax + by + k = 0, where k is a constant.
perpendicular to x-axis and other makes an angle θ with
the positive direction of x-axis), then angle between them WORKING RULE
= π – θ.
 Keep the terms containing x and y unaltered.
 The two lines are parallel if and only if m = m .
1 2  Change the constant.
 The two lines are perpendicular if and only if m × m = –1.
1 2  The constant k is determined from an additional condition

given in the problem.


SOLVED EXAMPLE Thus, the equation of any line parallel to 2x – 3y + 5 = 0
is 2x – 3y + k = 0.
45. Number of equilateral triangles with y = 3 ( x − 1) + 2
and y = 3 x as two of its sides, is
(A)
0 (B)  1 SOLVED EXAMPLES
(C) 2 (D)  none of these
46. The vertices of a DOBC are O (0, 0), B(–3, –1) and
Solution: (D)
C(–1, –3). The equation of a line parallel to BC and
The sides are, intersecting sides OB and OC whose distance from the
y = 3 ( x − 1) + 2 and y = − 3 x 1
origin is , is
2
1 1
(A) x + y + = 0 (B) x + y − =0
2 2
1
(C) x + y − 1 = 0 (D)  x+ y+ =0
2 2
Solution: (A)
The equation of line BC is
x+y+4=0
\ Equation of a line parallel to BC is
x+y+k=0
1
The two lines are at an angle of 60° to each other. This is at a distance from the origin.
2
Now any line parallel to obtuse angle bisector will make
k 1
equilateral triangle with these lines as its two sides. ∴ =
2 2
CONDITION FOR TWO LINES TO BE 1
∴ k=±
COINCIDENT, PARALLEL, PERPENDICULAR OR 2
INTERSECTING Since BC and the required line are on the same
side of the origin
Two lines a1x + b1 y + c1 = 0 and a2x + b2 y + c2 = 0 are
1
a b1 c1 ∴ k=
1. Coincident, if =
1
= ; 2
a2 b2 c2 1
a1 b1 c1 Hence, the required line is x + y + = 0.
2. Parallel, if = ≠ ; 2
a2 b2 c2
47. The distance of the point (2, 3) from the line 3x + 2y =
3. Perpendicular, if a1a2 + b1b2 = 0;
17, measured parallel to the line x – y = 4 is
a1 b1
4. Intersecting, if ≠
⋅ i.e., if they are neither coinci- (A) 4 2 (B)  5 2
a2 b2
dent nor parallel. (C) 2 (D)  none of these
Coordinates and Straight Lines  10.19

Solution: (C) WORKING RULE TO PROVE CONCURRENCY


Coordinates of any point on the line through (2, 3) and Following three methods can be used to prove that the three
parallel to the line x – y = 4, at a distance r, are lines are concurrent:
(2 + r cosq, 3 + r sinq), where tan q = 1.  Find the point of intersection of any two lines by solving

them simultaneously. If this point satisfies the third equa-


tion also, then the given lines are concurrent.
 The three lines

P ≡ a1x + b1 y + c1 = 0, Q ≡ a2x + b2 y + c2 = 0,


R ≡ a3x + b3 y + c3 = 0 are concurrent if
a1 b1 c1
a2 b2 c2 = 0
a3 c3 c3

 The three lines P = 0, Q = 0 and R = 0 are concurrent if


there exist constants l, m and n, not all zero at the same
If this point lies on the line 3x + 2y = 17, time, such that
then  3(2 + r cosq) + 2(3 + r sinq) = 17
lP + mQ + nR = 0.
1 1 5 This method is particularly useful in theoretical results.
⇒ 6 + 3r ⋅ + 6 + 2r ⋅ = 17 ⇒ r =5
2 2 2
⇒ r= 2 SOLVED EXAMPLE
1 1 1
EQUATION OF A LINE PERPENDICULAR TO A 48. If , , are in A.P., then the straight line
a b c
GIVEN LINE x y 1
+ + = 0 always passes through a fixed point,
The equation of a line perpendicular to a given line a b c
ax + by + c = 0 is bx – ay + k = 0, where k is a constant. that point is
(A) (–1, –2) (B)  (–1, 2)
WORKING RULE  1
(C) (1, –2) (D)  1, − 2 
 Interchange the coefficients of x and y and change the  
Solution: (C)
sign of one of them.
1 1 1
 Change the constant. Since , , are in A.P.
 The value of k can be determined from an additional con-
a b c
dition given in the problem. 1 1 2 (1)
⇒ + =
a c b
x y 1
The given line is + + = 0
POINT OF INTERSECTION OF TWO a b c
GIVEN LINES x y 2 1
⇒ + + − = 0 [Using (1)]
a b  b a 
Let the two given lines be
1 1
a1x + b1 y + c1 = 0 and a2x + b2 y + c2 = 0. ⇒ ( x − 1) + ( y + 2) = 0
a a
Solving these two equations, the point of intersection ⇒  The given line passes through the point of inter-
of the given two lines is given by section of x – 1 = 0 and y + 2 = 0 i.e., (1, –2) which is
 b1c2 − b2 c1 c1a2 − c2 a1  a fixed point.
 , 
 a1b2 − a2 b1 a1b2 − a2 b1  POSITION OF TWO POINTS RELATIVE TO A
LINE
CONCURRENT LINES
Two points (x1, y1) and (x2, y2) are on the same side or on
The three given lines are concurrent if they meet in a point. opposite sides of the line ax + by + c = 0 according as the
10.20  Chapter 10

expressions: ax1 + by1 + c and ax2 + by2 + c have same sign WORKING RULE
or opposite signs.
 Make the R.H.S. of the equation of the line zero by trans-
posing every term to L.H.S.
SOLVED EXAMPLES  On the L.H.S., replace x by x and y by y .
1 1

49. The point (1, b) lies on or inside the triangle formed by  Divide by (coeff. of x )2 + (coeff. of y )2 .
the lines y = x, x-axis and x + y = 8, if  Take the modulus of the expression thus obtained. This

(A) 0 < b < 1 (B)  0 ≤ b ≤ 1 will give the length of the perpendicular.
(C) 0 < b < 8 (D)  none of these
Solution: (B)
The point (1, b) lies on the line x = 1, for all real b. SOLVED EXAMPLES
Clearly, from the figure, it will lie on or inside the tri-
angle formed by the given lines if 0 ≤ b ≤ 1. 51. If P and Q are two points on the line 4x + 3y + 30 = 0
such that OP = OQ = 10, where O is the origin, then
the area of the DOPQ is
(A) 48 (B)  16
(C) 32 (D)  none of these
Solution: (A)
Let OR ^ PQ.
| 4(0) + 3(0) + 30 | 30
Then, OR = = = 6.
16 + 9 5

50. Let P(2, 0) and Q(0, 2) be two points and O be the


origin. If A(x, y) is a point such that xy > 0 and x + y <
2, then
(A) A cannot be inside the DOPQ
(B) A lies outside the DOPQ
(C) A lies either inside DOPQ or in the third quadrant
(D) none of these
Solution: (C) ∴ PR = OP 2 − OR 2 = 100 − 36 and
Since xy > 0, therefore the point A lies either in the PQ = 2PR = 16
first quadrant or in the third quadrant. Since x + y < 2,
therefore the point A lies either inside the DOPQ or in 1
∴ Area of ∆OPQ = × PQ × OR
the third quadrant. 2
1
= × 16 × 6 = 48
LENGTH OF PERPENDICULAR FROM A POINT 2
ON A LINE 52. On the portion of the straight line x + y = 2 which is
The length of the perpendicular from the point (x1, y1) to intercepted between the axes, a square is constructed
the line ax + by + c = 0 is given by away from the origin, with this portion as one of its
side. If p denotes the perpendicular distance of a side
| ax1 + by1 + c |
p= . of this square from the origin, then the maximum
a2 + b2 value of p is
Corollary The length of the perpendicular from the origin (A) 2 (B)  2 2
(0, 0) on the line ax + by + c = 0 is
(C) 3 2 (D)  4 2
| a×0 + b×0 + c | |c| Solution: (C)
=
a2 + b2 a2 + b2 p = ON = OM + MN = ^ distance from
Coordinates and Straight Lines  10.21

In the case of a rhombus, p1 = p2. Thus,


p12
 Area of rhombus = .
sinθ
1
Also, area of rhombus = d1d2
2
where d1 and d2 are the lengths of two perpendicular diag-
onals of a rhombus.

O to the line AB + AD SOLVED EXAMPLE


2 53. The diagonals of the parallelogram whose sides are lx
= +2 2 = 2+2 2 =3 2
2 + my + n = 0, lx + my + n′ = 0, mx + ly + n = 0, mx + ly
+ n′ = 0 include an angle
π π
DISTANCE BETWEEN TWO PARALLEL LINES (A) (B)  ⋅

2

3
The distance between two parallel lines ax + by + c1 = 0 and
ax + by + c2 = 0 is given by  l 2 − m2   2lm 
(C) tan −1  2  (D) tan −1  2 2 
l +m
2
 l +m 
| c1 − c2 |
d=
Solution: (B)
a2 + b2
Since the distance between the parallel lines lx + my
+ n = 0 and lx + my + n′ = 0 is same as the distance
Info Box! between the parallel lines mx + ly + n = 0 and mx + ly
The distance between two parallel lines can also be + n′ = 0. Therefore, the parallelogram is a rhombus.
obtained by taking a suitable point (take y = 0 and find Since the diagonals of a rhombus are at right angles,
x or take x = 0 and find y) on one straight line and then π
therefore the required angle is .
finding the length of the perpendicular from this point

2
to the second line.

EQUATIONS OF STRAIGHT LINES


QUICK TIPS
PASSING THROUGH A GIVEN
POINT AND MAKING A GIVEN
 Area of a parallelogram or a rhombus, equations of whose
sides are given, can be obtained by using the following ANGLE WITH A GIVEN LINE
formula The equations of the striaght lines which pass through a
p1p2 given point (x1, y1) and make a given angle α with the given
Area = , straight line y = mx + c are
sinθ
where p1 = DL = distance between lines AB and CD, m ± tan α
p2 = BM = distance between lines AD and BC, y − y1 = ( x − x1 )
1 ∓ m tan α
θ = angle between adjacent sides AB and AD.

REFLECTION ON THE SURFACE


Here, IP = Incident Ray
PN = Normal to the surface
PR = Reflected Ray
Then, ∠IPN = ∠NPR
Angle of incidence
FIGURE 10.19 = Angle of reflection
10.22  Chapter 10

IMAGE OF A POINT WITH


RESPECT TO A LINE
1. The image of a point with respect to the line mirror.
The image of A(x1, y1) with respect to the line mirror ax +
by + c = 0 be B(h, k) given by,
h − x1 k − y1 −2( ax1 + by1 + c)
= =
a b a2 + b2
FIGURE 10.20

SOLVED EXAMPLE

54. A ray of light is sent along the line which passes


through the point (2, 3). The ray is reflected from the
point P on x-axis. If the reflected ray passes through
the point (6, 4), then the coordinates of P are
FIGURE 10.21
 26   26 
(A)  , 0  (B)   0, 7  2. The image of a point with respect to x-axis: Let P(x, y)
 7   
be any point and P′(x′, y′) its image after reflection in the
 26 
(C)  − , 0  (D)  none of these x-axis, then
 7  x′ = x and y′ = – y, (  O′ is the mid point of PP′)
\

Solution: (A)
Let P ≡ (a, 0).
Let the reflected ray makes an angle q with +ve direc-
tion of x-axis, then the incident ray makes angle
(p – q) with positive direction of x-axis.
The slope of the incident ray is

FIGURE 10.22

3. The image of a point with respect to y-axis: P(x, y) be


any point and P′(x′, y′) its image after reflection in the
0−3 3 y-axis, then
= = tan(π − θ ) i.e., tan θ = (1)
α −2 α −2 x′ = – x and y′ = y (  O′ is the mid point of PP′)
\

The slope of the reflected ray is


4−0 4 (2)
= = tan θ i.e., tan θ =
6 −α 6 −α
From Eq. (1) and (2), we get
3 4
= ⇒ 18 − 3α = 4α − 8
α − 2 6 −α
26
⇒ 7α = 26 or α =
7
 26 
\ The coordiantes of A are  , 0  .
 7  FIGURE 10.23
Coordinates and Straight Lines  10.23

4. The image of a point with respect to the origin: Let a1 x + b1 y + c1 a2 x + b2 y + c2


P(x, y) be any point and P′(x′, y′) be its image after reflec- =±
tion through the origin, then a +b
2
1 1
2
a22 + b22
x′ = –x and y′ = –y (  O is the mid-point of PP′)
\

Info Box!
Any point on a bisector is equidistant from the given lines.

Equation of the Bisector of the Acute and Obtuse Angle


between Two Lines Let the equations of the two lines be
a1x + b1 y + c1 = 0 (1)
and a2x + b2 y + c2 = 0 (2)
FIGURE 10.24 where c1 > 0 and c2 > 0.
5. The image of a point with respect to the line y = x: Let Then the equation
P(x, y) be any point and P′(x′, y′) be its image after reflec-
tion in the line y = x, then, a1 x + b1 y + c1 a2 x + b2 y + c2
=+
x′ = y and y′ = x (  O′ is the mid-point of PP′) a +b a22 + b22
\ 2 2
1 1

is the bisector of the acute or obtuse angle between the


lines (1) and (2) according as a1a2 + b1b2 < 0 or > 0.
Similarly, the equation
a1 x + b1 y + c1 a2 x + b2 y + c2
=−
a +b
2
1 1
2
a22 + b22
is the bisector of the acute or obtuse angle between the
lines (1) and (2) according as a1a2 + b1b2 > 0 or < 0.
FIGURE 10.25
6. The image of a point with respect to the line y = x tanθ
SOLVED EXAMPLE
: Let P(x, y) be any point and P′(x′, y′) be its image after
reflection in the line y = x tanθ, then, 55. The point (3, 2) is reflected in the y-axis and then
moved a distance 5 units towards the negative side of
y-axis. The coordinates of the point thus obtained are
(A) (3, –3) (B)  (–3, 3)
(C) (3, 3) (D)  (–3, –3)
Solution: (D)
Reflection in the y-axis gives the new position as (–3, 2).

FIGURE 10.26

x′ = x cos 2θ + y sin 2θ
y′ = x sin 2θ – y cos 2θ,
(  O′ is the mid-point of PP′)
\

EQUATIONS OF THE BISECTORS OF THE


ANGLES BETWEEN TWO LINES
When it moves towards the negative side of y-axis
The equations of the bisectors of the angles between the through 5 units, then the new position is (–3, 2 – 5)
lines a1x + b1 y + c1 = 0 and a2x + b2 y + c2 = 0 are given by i.e., (–3, –3).
10.24  Chapter 10

QUICK TIPS for different values of p and q.


Solving (1) and (2), we get the coordinates of the point
If a1a2 + b1b2 > 0, then the origin lies in obtuse angle and if
 2 3
a1a2 + b1b2 < 0, then the origin lies in acute angle. of intersection as  ,  .
5 5
58. If a + b + g = 0, the line 3a x + b y + 2g = 0 passes
EQUATIONS OF LINES PASSING THROUGH
through the fixed point
THE POINT OF INTERSECTION OF TWO
 2 2 
GIVEN LINES (A)  2,  (B)   3 ,2
 3   
The equation of any line passing through the point of  2
intersection of the lines a1x + b1 y + c1 = 0 and a2x + b2 y (C)  −2,  (D)  none of these
 3
+ c2 = 0 is
Solution: (B)
(a1x + b1 y + c1) + k (a2x + b2 y + c2) = 0,
The given line is 3a x + b y + 2g = 0
where k is a parameter. The value of k can be obtained by ⇒ 3a x + b y + 2 (–a –b) = 0 ( a + b + g = 0)

using one more condition which the required line satisfies. ⇒  a (3x – 2) + b (y – 2) = 0
⇒  the given line passes through the point of inter-
SOLVED EXAMPLES 2 
section of the lines 3x – 2 = 0 and y – 2 = 0 i.e.,  , 2  ,
for all values of a and b. 3 
56. If a, b, c are three terms of an A.P., then the line ax +
by + c = 0 59. The number of integer values of m, for which the x-co-
ordinate of the point of intersection of the lines 3x + 4y
(A) has a fixed direction
= 9 and y = mx + 1 is also an integer, is
(B) always passes through a fixed point
(C) always cuts intercepts on the axes such that their (A) 2 (B)  0
sum is zero (C) 4 (D)  1
(D)  forms a triangle with the axes whose area is Solution: (A)
constant. The given lines are
Solution: (B) 3x + 4y = 9 (1)
Let a, b, c be pth, qth and rth terms of an A.P. whose and y = mx + 1 (2)
first term is A and common difference is d. The given Solving (1) and (2), we get the x-coordinate of the
line is 5
point of intersection as x = .
ax + by + c = 0 4m + 3
⇒ [A + (p – 1) d] x + [A + (q – 1) d] y + [A + (r – 1) Since x-coordinate is an integer,
d] = 0 \ 4m + 3 = ± 5  or  4m + 3 = ± 1.
⇒  A(x + y + 1) + d((p – 1) x + (q – 1) y + r – 1) = 0 Solving these, only integer values of m are –1 and –2.
⇒  The given line passes through the point of inter- \ m = –1, –2
section of lines x + y + 1 = 0 and ( p – 1) x + (q – 1)y +
r – 1 = 0, which is a fixed point. STANDARD POINTS OF A TRIANGLE
57. The line (p + 2q) x + (p – 3q)y = p – q for different
values of p and q passes through the fixed point Centroid of a Triangle
2 2
(A)  ,  (B) 
3 5 The point of intersection of the medians of the triangle is
5,5 called the centroid of the triangle. The centroid divides the
 2 2   
 2 3 medians in the ratio 2 : 1 (2 from the vertex and 1 from the
3 3
(C)  ,  (D)  5,5 opposite side).
5 5  
Solution: (D)
The equation of the given line can be re-written as
p(x + y – 1) + q(2x – 3y + 1) = 0
which, clearly, passes through the point of intersection
of the lines
x + y – 1 = 0 (1)
and  2x – 3y + 1 = 0 (2) FIGURE 10.27
Coordinates and Straight Lines  10.25

The coordinates of the centroid of a triangle with vertices Ex-centres of a Triangle  A circle touches one side outside
(x1, y1), (x2, y2) and (x3, y3) are the triangle and the other two extended sides then circle is
known as excircle.
 x1 + x2 + x3 y1 + y2 + y3 
 ,  Let ABC be a triangle then there are three excircles,
 3 3  with three excentres I1, I2, I3 opposite to vertices A, B and C
respectively. If the vertices of triangle are A (x1, y1), B (x2,
SOLVED EXAMPLE y2) and C (x3, y3) then
 − ax1 + bx2 + cx3 − ay1 + by2 + cy3 
60. If the vertices P, Q, R of a DPQR are rational points, I1 =  , 
which of the following points of the DPQR is (are)  −a+b+c −a +b +c 
always rational point(s)?  ax − bx2 + cx3 ay1 − by2 + cy3 
I2 =  1 ,
(A) centroid (B)  incentre
 a−b+c a − b + c 
(C) circumcentre (D)  orthocentre
(A rational point is a point both of whose coordinates  ax + bx2 − cx3 ay1 + by2 − cy3 
I3 =  1 ,
are rational numbers)  a+b−c a + b − c 
Solution: (A) Circumcentre The circumcentre of a triangle is the point
Let P ≡ (x1, y1), Q ≡ (x2, y2); R ≡ (x3, y3), where xi, yi (i of intersection of the perpendicular bisectors of the sides of
= 1, 2, 3) are rational numbers. a triangle. It is the centre of the circle which passes through
Now, the centroid of DPQR is the vertices of the triangle and so its distance from the ver-
 x1 + x2 + x3 y1 + y2 + y3  tices of the triangle is same and this distance is known as
 ,  the circum-radius of the triangle.
 3 3 
which is rational point. Incentre, circumcentre and
orthocentre depend on sides of the triangle which may
not be rational even if vertices are so. For example, for
P(0, 1) and Q(1, 0); PQ = 2.

Incentre of a Triangle
The point of intersection of the internal bisectors of the
angles of a triangle is called the incentre of the triangle.
The coordinates of the incentre of a triangle with verti- FIGURE 10.29
ces (x1, y1), (x2, y2) and (x3, y3) are
QUICK TIPS
 The circumcentre of a right angled triangle is the mid
point of its hypotenuse.
 The circumcentre of the triangle formed by (0, 0), (x , y )
1 1
and (x2, y2) is
 y2( x12 + y12 ) − y1( x22 + y22 ) x2( x12 + y12 ) − x1( x22 + y22 ) 
 , 
 2( x1y2 − x2 y1) 2( x2 y1 − x1y2 ) 
FIGURE 10.28

 ax1 + bx2 + cx3 ay1 + by2 + cy3  WORKING RULE TO PROVE CONCURRENCY
 a+b+c ,
 a + b + c   Let A(x1, y1), B(x2, y2) and C(x3, y3) be the vertices of the
∆ABC and let circumcentre be P(x, y). Then (x, y) can be
QUICK TIPS found by solving
The incentre of the triangle formed by (0, 0), (a, 0) and (OA)2 = (OB)2 = (OC)2
(0, b) is or (x – x1)2 + ( y – y1)2 = (x – x2)2 + ( y – y2)2
 ab ab  = (x – x3)2 + ( y – y3)2
 ,   Let D, E and F be the mid-points of the sides BC, CA and
 a + b + a 2
+ b2
a + b + a 2
+ b2

AB of the ∆ABC respectively.
10.26  Chapter 10

Then, OD ⊥ BC, OE ⊥ AC, OF ⊥ AB. Solving any two of these, we can get coordinates of O.
 (a) Write down the equations of any two sides of the
triangle.
(b) Find the equations of the lines perpendicular to these
two sides and passing through the opposite vertices.
(c) Solve these equations to get the coordinates of the
orthocentre.
 If angles A, B and C and vertices A(x , y ), B(x , y ) and
1 1 2 2
C(x3, y3) of a ∆ABC are given, then orthocentre of ∆ABC
is given by
FIGURE 10.30
slope of OD × slope of BC = –1  x1 tan A + x2 tan B + x3 tan C
 ,
slope of OE × slope of AC = –1  tan A + tan B + tan C
slope of OF × slope of AB = –1 y1 tan A + y2 tan B + y3 tan C 
Solving any two of the above equations, we get the cir- 
tan A + tan B + tan C 
cumcentre (x, y).
 (a) If the equations of the three sides of the triangle are
given, first of all find the coordinates of the vertices QUICK TIPS
of the triangle by solving the equations of the sides
of the triangle taken two at a time.  If any two lines out of three lines, i.e., AB, BC and CA are
perpendicular, then orthocentre is the point of intersec-
(b)  Find the coordinates of the middle points of two
tion of two perpendicular lines.
sides of the triangle.
 The orthocentre of the triangle with vertices (0, 0), (x ,
(c) Find the equations of the perpendicular bisectors of 1
y1) and (x2, y2) is
these two sides and solve them. This will give the
coordinates of the circumcentre of the triangle.   x1 x2 − y1 y2   x1 x2 + y1 y2  
 If angles A, B, C and vertices A(x , y ), B(x , y ) and C(x , ( y1 − y2 )   , ( x1 − x2 )  .
 x2 y1 − x1 y2   x1 y2 − x2 y1  
1 1 2 2 3
y3) of a ∆ABC are given, then its circumcentre is given by 
 x1 sin 2 A + x2 sin 2B + x3 sin 2C  The orthocentre (O), centroid (G) and circum centre (C)
 ,
 sin 2 A + sin 2B + sin 2C of any triangle lie in a straight line and G divides the join
of O and C in the ratio 2 : 1.
y1 sin 2 A + y2 sin 2B + y3 sin 2C 
  In an equilateral triangle, orthocentre, centroid, circumcen-
sin 2 A + sin 2B + sin 2C 
tre and incentre coincide.

ORTHOCENTRE
The orthocentre of a triangle is the point of intersection of
altitudes.
QUICK TIPS
 Let O be the orthocentre. Since AD ⊥ BC, BE ⊥ CA and CF
⊥ AB, then
OA ⊥ BC
OB ⊥ CA
and OC ⊥ AB
FIGURE 10.32

COORDINATES OF NINE POINT CIRCLE


If a circle passes through the feet of perpendiculars (i.e.,
D, E, F), midpoints of sides BC, CA, AB respectively (i.e.,
H, I, J) and the midpoints of the line joining the ortho-
centre O to the angular points A, B, C (i.e., K, L, M), thus
FIGURE 10.31 the nine points D, E, F, H, I, J, K, L, M, all lie on a circle.
Coordinates and Straight Lines  10.27

This circle is known as nine point circle and its centre is (A) orthocentre (B)  incentre
called the nine-point centre. (C) centroid (D)  none of these
Solution: (A)
I M P O R TA N T P O I N T S The lines y = x and 3 y + x = 0, are inclined at 45°
and 150°, respectively, with the positive direction of
The orthocentre (O). Nine point Centre (N), Centroid (G)

x-axis. So, the angle between the two lines is an obtuse
and Circumcentre (C) all lie in the same striaght line.
angle. Therefore, orthocentre lies outside the given
 The Nine point centre bisects the join of Orthocentre (O)
triangle, whereas incentre and centroid lie within the
and Circumcentre (C)
triangle (In any triangle, the centroid and the incentre
 The radius of Nine Point Circle is half the radius of
lie within the triangle).
Circumcirle.

SOLVED EXAMPLE

61. If the equations of the sides of a triangle are x + y = 2,


y = x and 3 y + x = 0, then which of the following is
an exterior point of the triangle?
10.28  Chapter 10

NCERT EXEMPLARS
1. A line cutting off intercept – 3 from the Y-axis and the 3 from the origin, are
3 2
tangent at angle to the X-axis is , its equation is (A)  3x + y − 3 = 0, 3x − y − 3 = 0
5
(B)  3x + y + 3 = 0, 3x − y + 3 = 0
(A) 5y – 3x + 15 = 0 (B)  3y – 5x + 15 = 0
(C) 5y – 3x – 15 = 0 (D)  None of these (C) x + 3 y − 3 = 0, x − 3 y − 3 = 0
(D)  None of these
2. Slope of a line which cuts off intercepts of equal 10. The distance between the lines y = mx + c1 and y = mx
lengths on the axes is + c2 is
(A) – 1 (B) 0 −
(A)  c1 − c2 c1 c2
(B) 
(C) 2 (D)  3 m + 1
2
1 + m2
(C)  c2 − c1 (D) 
0
3. The equation of the straight line passing through the point
1 + m2
(3, 2) and perpendicular to the line y = x is
11. The coordinates of the foot of perpendiculars from the
(A) x – y = 5 (B)  x + y = 5 point (2, 3) on the line y = 3x + 4 is given by
(C) x + y = 1 (D)  x – y = 1
(A)  37 , −1  (B)   1 37 
 − 10 , 10 
4. The equation of the line passing through the point (1,  10 10   
2) and perpendicular to the line x + y + 1 = 0 is (C)  10 , −10  (D)   2 , − 1 
(A) y – x + 1 = 0 (B)  y – x – 1 = 0  37   3 3
(C) y – x + 2 = 0 (D)  y – x – 2 = 0 .12. If the coordinates of the middle point of the portion of
5. The tangent of angle between the lines whose inter- a line intercepted between the coordinate axes is (3, 2),
cepts on the axes are a, – b and b, – a respectively, is then the equation of the line will be
(A) a − b b −a (A) 2x + 3y = 12 (B)  3x + 2y = 12
2 2 2 2
(B) 
ab 2 (C) 4x – 3y = 6 (D)  5x – 2y = 10
(C)  − a
2 2
b (D)  None of these 13. Equation of the line passing through (1, 2) and parallel
2ab to the line y = 3x – 1 is
6. If the line x + y = 1 passes through the points (2, – 3) and (A) y + 2 = x + 1 (B)  y + 2 = 3(x + 1)
a b (C) y – 2 = 3 (x – 1) (D)  y – 2 = x – 1
(4, – 5), then (a, b) is
14. Equations of diagonals of the square formed by the lines
(A)  (1, 1) (B)  (– 1, 1) x = 0, y = 0, x = 1 and y = 1 are
(C)  (1, – 1) (D)  (– 1, – 1)
NCERT EXEMPLARS

(A) y = x, y + x = 1 (B)  y = x, x + y = 2
(C) 2y = x, y + x = (D) 1 y = 2x, y + 2x = 1
7. The distance of the point of intersection of the lines
2x – 3y + 5 = 0 and 3x + 4y = 0 from the line 5x – 2y = 3
0 is 15. For specifying a straight line, how many geometrical
130 13 parameters should be known ?
(A)  (B) 
17 29 7 29 (A) 1 (B) 2
130 (C) 4 (D) 3
(C)  (D)  None of these
7
16. The point (4, 1) undergoes the following two succes-
8. The equation of the lines which pass through the point sive transformations
(3, – 2) and are inclined at 60° to the line 3x + y = 1 is (i) Reflection about the line y = x
(A) y + 2 = 0, 3x − y − 2 − 3 3 = 0 (ii) Translation through a distance 2 units along the
(B) x − 2 = 0, 3x − y + 2 + 3 3 = 0 positive X-axis.
(C)  3x − y − 2 − 3 3 = 0 Then, the final coordinates of the point are
(D)  None of the above (A)  (4, 3) (B)  (3, 4)
9. The equations of the lines passing through the point 7 7

(C)   2,2
(1, 4) (D) 
(1, 0) and at a distance  
Coordinates and Straight Lines  10.29

17. A point equidistant from the lines 4x + 3y + 10 = 0, 19. The ratio in which the line 3x + 4y + 2 = 0 divides the
5x – 12y + 26 = 0 and 7x + 24y – 50 = 0 is distance between the lines 3x + 4y + 5 = 0 and 3x + 4y
(A)  (1, – 1) (B)  (1, 1) – 5 = 0 is
(C)  (0, 0) (D)  (0, 1) (A)  1 : 2 (B)  3 : 7
(C)  2 : 3 (D)  2 : 5
18. A line passes through (2, 2) and is perpendicular to the
line 3x + y = 3. Its y-intercept is 20. One vertex of the equilateral triangle with centroid at
(A) 1 (B)  2 the origin and one side as x + y – 2 = 0 is
3 3 (A)  (– 1, – 1) (B)  (2, 2)
(C)  1 (D)  4 (C)  (– 2, – 2) (D)  (2, – 2)
3

AN SW ER K EYS
1. (A) 2. (A) 3. (B) 4. (B) 5. (C) 6. (D) 7. (A) 8. (A) 9. (A) 10. (B)
11. (B) 12. (A) 13. (C) 14. (A) 15. (B) 16. (B) 17. (C) 18. (D) 19. (B) 20. (C)

HINTS AND EXPLANATIONS

1. Given that, C = – 3 and m = 3 5. Since, intercepts on the axes are a, – b then equation of the
5
∴ Equation of the line is y = mx + c
line is x − y = 1.
a b
   y = 3 x − 3

5 y x
⇒ = −1

⇒ 5 y = 3 x − 15 b a

HINTS AND EXPLANATIONS



⇒ 5 y − 3 x + 15 = 0 bx
⇒ y= −b
a
2. Let equation of line be x + y = 1 So, the lope of this line i.e., m1 = b .
a a a
⇒ x+y=a Also, for intercepts on the axes as b and – a, then equation of
⇒    y = – x + a the line is
∴ Required slope = – 1 x y
− =1
3. Since, line passes through the point (3, 2) and perpendicular b a
to the line y = x. y x a
⇒ = −1 ⇒ y = x − a
∵ Slope  (m) = – 1 [since, line is perpendicular to the line a b b
y = x]
and slope of this line ie., m2 = a
∴ Equation of line which passes through (3, 2) is b
y – 2 = – 1 (x – 3) b a b2 − a2

y–2=–x+3 b2 − a2
∴ tan θ = a b = ab =
x+y=5 ab 2 2ab
1+ •
4. Given point is (1, 2) and slope of the required line is 1. ba
∵ x + y + 1 = 0 ⇒ − x − 1 ⇒ m1 = −11 6. Given, line is x + y = 1 (i)
−1 a b
∴ slope of the line = =1 Since, the points (2, – 3) and (4, – 5) lies on this line.
−1
∴ Equation of required line is
2 3

∴ − = 1  (ii)

y – 2 = 1 (x – 1) a b

⇒ y − 2 = x −1 and 4 − 5 = 1 
(iii)

⇒ y − x −1 = 0 a b
10.30  Chapter 10

On multiplying by 2 in Eq. (ii) and then subtracting Eq. (ii)


⇒ 3 − 3m2 = − 3 − m2
from Eq. (ii), we get

⇒ 2 3 = 2m2
6 5
− + =1
b b
⇒ m2 = 3
−1 \ Equation of line passing through (3, – 2) is

⇒ =1
b
y + 2 = 3 ( x − 3)
∴ b = −1
On putting b = – 1 in Eq. (ii), we get y − 3 x + 2 + 3 3 = 0
2
+ 3 =1 ⇒ 3 x − y − 2 − 3 3 = 0  (i)
a
2
[taking negative sign from Eq. (i)]
⇒ = −2 ⇒ a = −1
a ⇒ 3 − 3m2 = 3 + m2
∴ ( a, b ) = ( −1, −1)

⇒ m2 = 0
7. Given equation of lines ∴ The equation of line is y + 2 = 0 ( x − 3)


2x – 3y + 5 = 0  (i)
⇒ y + 2 = 0 (ii)

and 3x + 4y = 0  (ii)
So, the required equation of lines are 3 x − y − 2 − 3 3 = 0
−4 y and y + 2 = 0.
From Eq. (ii), put the value of x =
in Eq. (i), we get
3

 −4 y  9. Let slope of the line be m.
2  − 3 y + 5 = 0
 3  ∵ Equation of line passing through (1, 0) is

⇒ − 8 y − 9 y + 15 = 0 y – 0 = m(x – 1)
15

⇒ y= ⇒ y − mx + m = 0
17
Since, the distance from origin is 3 .
From Eq. (ii), 3 x + 4·15 = 0
2
17 3 0−0+m
− 60 − 20 Then, =

⇒ x= = 2 1 + m2
17.3 17
So, the point of intersection is  −20 , 15  .


3
=
m
 17 17  2 1 + m2
∴ Required distance from the line 5x – 2y = 0 is,

HINTS AND EXPLANATIONS

3 m2
20  15  −100 30 ⇒ =
−5 × − 2   − 4 1 + m2
17  17  17 17 130
d= = = ⇒ 3 + 3m 2 = 4 m 2

25 + 4 9 17 29

⇒ m2 = 3

∵ distance of a point p ( x1 , y1 ) from the line
⇒ m=± 3

and the second equation of line is
ax + by1 + c 
ax + by + c = 0 is d = 1 
y = 3 ( x − 1)
 a2 + b2 

⇒ 3x + y − 3 = 0
8. So, the given point A is (3, – 2).
So, the equation of line 3 x + y = 1. 10. Given, equation of the lines are
⇒ y = − 3x + 1
y = mx + c1  (i)

∴ Slope, m1 = − 3
and y = mx + c2  (ii)

Let slope of the required line be m2. ∴ Distance between them is given by

c −c
− 3 − m2  m1 − m2 
d= 1 2
∴ tan θ =
∵ tan θ =  1 + m2
1 − 3m2  1 + m1m2 
11. Given, equation of the line is
 − 3 − m2 

⇒ tan 60° = ±    (i) y = 3x + 4  (i)
 1 − 3m2  ∴ Slope of this line, m1 = 3
 − 3 − m2 

⇒ 3 =    [taking positive sign] 1
 1 − 3m2 
So, the slope of line OP is − . 

3
[∵ OP ⊥ AB ]
Coordinates and Straight Lines  10.31

⇒ y=x


And equation of AC is
1− 0

y−0 = ( x − 1)

y=–x+1 0 −1

x+y–1=0

\ Equation of line OP is
1
y − 3 = − ( x − 2)
3
⇒ 3y − 9 = −x + 2
⇒ x + 3 y − 11 = 0  (ii)
Using the value of y from Eq. (i) in Eq. (ii), we get
x + 3(3x + 4) – 11 = 0 15. Equation of straight line are
⇒ x + 9 x + 12 − 11 = 0 y = mx + c, parameter = 2  (i)
1 x y
⇒ 10 x + 1 = 0 ⇒ x = − + = 1, parameter = 2  (ii)
10 a b
y – y1 = m (x – x1), parameter = 2  (iii)
−1 in Eq. (i), we get
Put x = and x cos w + y sin w = p, parameter = 2  (iv)
10
−3 −3 + 40 37 It is clear that from Eqs. (i), (ii), (iii) and (iv), for specifying
y= +4= = a straight line clearly two parameters should be known.
10 10 10
16. Let the reflection of A(4, 1) in y = x is B (h, k).
So, the foot of perpendicular is  − 1 , 37 
 10 10  Now, mid-point of AB is  4 + h , 1 + k  which lies on y = x.

 2 2 
12. Since, the coordinates of the middle point are P(3, 2). i.e., 4 + h = 1 + k ⇒ h − k = −3
(i)
1.0 + 1.a 2 2

∴ 3=
So, the slope of line y = x is 1.
1+1
h−4
a ∴ Slope of AB =
⇒ 3= ⇒a=6 k −1
2

HINTS AND EXPLANATIONS



Similarly, b=4  h − 4 
⇒ 1·  = −1
x y  k −1 

Equation of the line is + = 1 ⇒ h–4=1–k

6 4

2x + 3y = 12 ⇒ h + k = 5
(ii)

and h – k = – 3
2h = 2  ⇒ h = 1

On putting h = 1 in Eq. (ii), we get

k=4

So, the point is (1, 4).

Hence, after translation the point is (1 + 2, 4) or (3, 4).
17. The given equation of lines are
4x + 3y + 10 = 0  (i)
13. Since, the line passes through (1, 2) and parallel to the line ⇒ 5x – 12y + 26 = 0  (ii)
y = 3x – 1. ⇒ 7x + 24y – 50 = 0  (iii)

So, slope of the required line m = 3. Let the point (h, k) which is equidistant from these lines.
[∵ slope of y = 3x − 1 is 3] Distance from line (i) =
4 h + 3k + 10

Hence, the equation of line is 16 + 9

y – 2 = 3 (x – 1) 5h + 12k + 26
Distance from line (ii) =
14. Equation of OB is 25 + 44
1− 0 7h + 24 k − 50
y−0 = ( x − 0) Distance from the line (iii) =
1− 0 72 + 24 2
10.32  Chapter 10


So, the point (h, k) is equidistant from lines (i), (ii) and (iii). 3 x2 + 4 y2 + 2 +5 − 2
3

= =
4 h + 3k + 10 5h − 12k + 26 7h + 24 k − 50 9 + 16 9 + 16 5
∴ = =
16 + 9 25 + 144 49 + 576 3 7

Hence, the required ratio is : i.e., 3 : 7.
4 h + 3k + 10 5h − 12k + 26 7h + 24 k − 50 5 5

⇒ = =
5 13 25 20. Let ABC be the equilateral triangle with vertex A(h, k)
10 26 50
Clearly, if h = 0, k = 0, then = = =2 Let the coordinates of D are (α, β).
5 13 25
Hence, the required point is (0, 0).
18. Given line is y = 3 – 3x.
Then, slope of the required line = 1 .
3
∵  Equation of the required line is
1
y − 2 = ( x − 2)
3
⇒ 3y − 6 = x − 2
⇒ x − 3y + 4 = 0
For y-intercept, put x = 0,

We know that, 2 : 1 from the vertex A.
0 – 3y + 4 = 0
4 ∴ 0 = 2α + h and 0 = 2 β + k
⇒ y= 3 3
3
⇒ 2α = − h (i)
19. Let point A(x1, y1) lies on the line 3x + 4y + 5 = 0, then 3x1 +
and 2 β = −k
4y1 + 5 = 0 Also, D(α, β) lies on the line x + y – 2 = 0.


∴ α + β − 2 = 0 (ii)

  AD ⊥ BC

Since, the slope of line BC i.e., mBC = – 1

And slope of the line AG i.e., mAG = k − 0 = k



h−0 h
k 
⇒ ( −1)· = −1

h 
HINTS AND EXPLANATIONS


Now, perpendicular distance from A to the line ⇒ h = k 
(iii)

3x + 4y + 2 = 0
From Eqs. (i) and (iii)
3 x1 + 4 y1 + 2 −5 − 2 −7 2 α = – h and 2β = – h


⇒ = =
9 + 16 9 + 16 5
∴ α =β

Let point B(x2, y2) lies on the line 3x + 4y – 5 = 0 i.e., 3x2 + 4y2 From Eq. (ii), 2α – 2 = 0 ⇒ α = 1

– 5 = 0. if α = 1, then β = 1


Now, perpendicular distance from B to the line 3x + 4y + 2 = 0,
So, the vertex A is (– 2, – 2).
Coordinates and Straight Lines  10.33

PRACTICE EXERCISES

Single Option Correct Type


3
1. If one of the diagonals of a square is along the line x = (A) ar (DPQT) (B)  ⋅ ar ( ∆PQT )
y and one of its vertices is (3, 0), then its side through 2
this vertex nearer to the origin is given by the equation. (C) 2 ⋅ ar (DPQT) (D)  none of these
(A) y – 3x + 9 = 0 6. The condition to be imposed on β so that (0, β) lies
(B) 3y + x – 3 = 0 on or inside the triangle having sides y + 3x + 2 = 0,
(C) x – 3y – 3 = 0 3y – 2x – 5 = 0 and 4y + x – 14 = 0 is
(D) 3x + y – 9 = 0 5 7
(A) 0 < β < (B)  0<β <
3 2
2. Through the point P(α, β), where aβ > 0 the straight
x y 5 7
line + = 1 is drawn so as to form with coordinate (C) ≤ β ≤ (D)  none of these
a b 3 2
axes a triangle of area S. If ab > 0, then the least value 7. The point (1, β) lies on or inside the triangle formed by
of S is the lines y = x, x-axis and x + y = 8, if
(A) aβ (B)  2aβ (A) 0 < β < 1 (B)  0 ≤ β ≤ 1
(C) 4aβ (D)  none of these (C) 0 < β < 8 (D)  none of these
8. A ray of light travelling along the line x + 3 y = 5
3. A line joining two points A (2, 0) and B (3, 1) is rotated
is incident on the x-axis and after refraction it enters
about A in anti-clockwise direction through an angle π
the other side of the x-axis by turning away from
15°. If B goes to C in the new position, then the coor- ⋅

6
dinates of C are the x-axis. The equation of the line along which the
refracted ray travels is
 3  3
(A)  2,
  2, −
 (B)   (A) x + 3 y − 5 3 = 0
 2  2 
(B)  x − 3 y − 5 3 = 0
 1 3 (C) 3 x + y − 5 3 = 0
(C)  2 + ,  (D)  none of these
 2 2 
 (D)  3 x − y − 5 3 = 0
4. P is a point on either of the two lines y − 3 | x| = 2 at a 9. A ray of light is sent along the line which passes

PRACTICE EXERCISES
distance of 5 units from their point of intersection. The through the point (2, 3). The ray is reflected from the
coordinates of the foot of the perpendicular from P on point P on x-axis. If the reflected ray passes through
the bisector of the angle between them are the point (6, 4), then the coordinates of P are
 26 
(A)  , 0  (B) 
 1   1  26
(A) 0, ( 4 + 5 3 )  or 0, ( 4 − 5 3 )  depend-  0, 
 2   2   7   7 
ing on which line the point P is taken
(C)  − , 0 
26
 1  (D)  none of these
(B) 0, ( 4 + 5 3 )   7 
 2 
10. A line passing through the point P (4, 2), meets the
(C) 0, ( 4 − 5 3 ) 
1
x-axis and y-axis at A and B, respectively. If O is the
 2 
origin, then locus of the centre of the circum circle of
5 5 3  DOAB is
(D)  , 
 2 2  (A) x–1 + y–1 = 2 (B)  2x–1 + y–1 = 1
(C) x + 2y = 1
–1 –1
(D)  2x–1 + 2y–1 = 1
5. A string of length 12 units is bent first into a square
PQRS and then into a right-angled DPQT by keeping 11. If the point (2 cosθ, 2 sinθ) does not fall in that angle
the side PQ of the square fixed and other is one more between the lines y = |x – 2| in which the origin lies
than its side. Then, the area of PQRS equals then θ belongs to
10.34  Chapter 10

(A) (x + 2y)2 + y2 = 1 (B)  (x – 2y)2 + y2 = 1


 π 3π   π π
(A)  ,  (B)  − 2 , 2  (C) (x + 2y)2 – y2 = 1 (D)  none of these
2 2   
20. A straight line through the origin O meets the parallel
(C) (0, π) (D)  none of these
lines 4x + 2y = 9 and 2x + y + 6 = 0 at points P and Q,
12. If the equations of the sides of a triangle are x + y = 2, respectively. The point O divides the segment PQ in
y = x and 3 y + x = 0, then which of the following is the ratio
an exterior point of the triangle? (A) 1 : 2 (B)  3 : 4
(A) orthocentre (B)  incentre (C) 2 : 1 (D)  4 : 3
(C) centroid (D)  none of these
21. Let O be the origin and let A(2, 0), B(0, 2) be two
13. A line is drawn from the point P(α, β), making an points. If P(x, y) is a point such that xy > 0 and x + y <
angle θ with the positive direction of x-axis, to meet 2, then
the line ax + by + c = 0 at Q. The length of PQ is
(A) P lies either inside the triangle OAB or in the third
aα + bβ + c aα + bβ + c quadrant
(A) − (B) 
a cos θ + b sin θ (B) P cannot be inside the triangle OAB
a2 + b2
aα + bβ + c (C) P lies inside the triangle OAB
(C) (D)  none of these (D) none of these
a cos θ + b sin θ
14. If the equal sides AB and AC (each equal to a) of a 22. Consider the equation y – y1 = m(x – x1). In this equa-
right-angled isosceles triangle ABC be produced to P tion, if m and x1 are fixed and different lines are drawn
and Q so that BP ⋅ CQ = AB2, then the line PQ always for different values of y1, then,
passes through the fixed point (A) the lines will pass through a single point
(A) (a, 0) (B)  (0, a) (B) there will be one possible line only
(C) (a, a) (D)  none of these (C) there will be a set of parallel lines
(D) none of these
15. If x1, x2, x3 as well as y1, y2, y3 are in G. P. with the same
common ratio, then the points (x1, y1), (x2, y2) and (x3, y3) 23. D is a point on AC of the triangle with vertices A(2,
(A) lie on a straight line 3), B(1, –3), C(–4, –7) and BD divides ABC into two
(B) lie on an ellipse triangles of equal area. The equation of the line drawn
(C) lie on a circle through B at right angles to BD is
(D) are vertices of a triangle (A) y – 2x + 5 = 0 (B)  2y – x + 5 = 0
16. Number of equilateral triangles with y = 3 ( x − 1) + 2 (C) y + 2x – 5 = 0 (D)  2y + x – 5 = 0
and y = − 3 x as two of its sides, is
PRACTICE EXERCISES

24. If two points A(a, 0) and B(–a, 0) are stationary and if


(A) 0 (B)  1 ∠A – ∠B = θ in ∆ABC, the locus of C is
(C) 2 (D)  none of these (A) x2 + y2 + 2xy tan θ = a2
17. If the distance of any point P(x, y) from the origin is (B) x2 – y2 + 2xy tan θ = a2
defined as d(x, y) = Max.{|x|, |y|} and d(x, y) = k (non- (C) x2 + y2 + 2xy cot θ = a2
zero constant), then the locus of the point P is (D) x2 – y2 + 2xy cot θ = a2
(A) a straight line (B)  a circle 25. The straight line y = x – 2 rotates about a point where
(C) a parabola (D)  none of these it cuts the x-axis and becomes perpendicular to the
18. If a, b, c form an A. P. with common difference d (≠ 0) straight line ax + by + c = 0. Then, its equation is
and x, y, z form a G. P. with common ratio r(≠ 1), then (A) ax + by + 2a = 0
the area of the triangle with vertices (a, x), (b, y) and (B) ax – by – 2a = 0
(c, z) is independent of (C) by + ay – 2b = 0
(A) b (B)  r (D) ay – bx + 2b = 0
(C) d (D)  x
26. If the point P(a2, a) lies in the region corresponding to
19. A line of fixed length 2 units moves so that its ends are the acute angle between the lines 2y = x and 4y = x, then
on the positive x-axis and that part of the line x + y = (A) a ∈ (2, 6) (B)  a ∈ (4, 6)
0 which lies in the second quadrant. The locus of the (C) a ∈ (2, 4) (D)  none of these
mid-point of the line has the equation
Coordinates and Straight Lines  10.35

27. The point (4, 1) undergoes the following three succes- 33. A line through A(–5, –4) meets the lines x + 3y + 2 = 0,
sive transformations 2x + y + 4 = 0 and x – y – 5 = 0 at the point B, C and D,
(A) Reflection about the line y = x – 1 2 2 2
respectively. If 
15   10   6 
(B) Translation through a distance 1 unit along the  +  =  , the equa-
 AB   AC   AD 
positive x-axis tion of the line is
π
(C) Rotation through an angle ⋅ about the origin in (A) 2x + 3y + 22 = 0 (B)  2x – 3y + 22 = 0
4 (C) 3x + 2y + 22 = 0 (D)  3x – 2y + 22 = 0
the anti-clockwise direction.
Then, the coordinates of the final point are 34. A(0, 0), B(2, 1) and C(3, 0) are the vertices of a ∆ABC
7 7 and BD is its altitude. If the line through D parallel
(A) (4, 3) (B)  2, 2 to the side AB intersects the side BC at a point K,
 
(C) (0, 3 2 ) (D)  (3, 4) then the product of the areas of the triangles ABC and
BDK is
28. A light ray emerging from the point source placed at 1
(A) 1 (B) 
P(2, 3) is reflected at point ‘θ’ on the y-axis and then 1 2
passes through the point R(5, 10). Coordinates of ‘Q’ (C) (D)  none of these
4
are 35. A line cuts the x-axis at A (7, 0) and y-axis at B (0, – 5).
(A) (0, 3) (B)  (0, 2) A variable line PQ is drawn ⊥ to AB cutting the x-axis
(C) (0, 5) (D)  none of these in P and the y-axis in Q. If AQ and BP intersect at R,
then the locus of R is
29. The distance between two parallel lines is unity. A
point P lies between the lines at a distance a from one (A) x(x – 7) + y(y + 5) = 0
of them. The length of a side of an equilateral trian- (B) x(x – 7) – y(y + 5) = 0
gle PQR, vertex Q of which lies on one of the parallel (C) x(x + 7) + y(y – 5) = 0
lines and vertex R lies on the other line, is (D) none of these

2 2 36. The point (2, 3) undergoes the following three trans-


(A) ⋅ a 2 + a + 1 (B)  a2 − a + 1 formations successively
3 3
(i)  reflection about the line y = x
1 1 (ii) translation through a distance 2 units along the
(C) a 2 + a + 1 (D)  a2 − a + 1
3 3 positive direction of y-axis
(iii) rotation through an angle of 45º about the origin
30. Two points A and B are given. P is a moving point on in the anti-clockwise direction.
one side of the line AB such that ∠PAB – ∠PBA is a The final coordinates of the point are

PRACTICE EXERCISES
positive constant 2θ. The locus of the point P is
 1 7   1 7 
(A) x2 + y2 + 2xy cot 2θ = a2 (A)  , − , 
 (B) 
(B) x2 + y2 – 2xy cot 2θ = a2  2 2  2 2
(C) x2 + y2 + 2xy tan 2θ = a2
 1 7 
(D) x2 – y2 + 2xy cot 2θ = a2. (C)  ,−  (D)  none of these
 2 2
31. The four points A( p, 0), B(q, 0), C(r, 0) and D(s, 0) are
such that p, q are the roots of the equation ax2 + 2hx + 37. Lines L1 = ax + by + c = 0 and L2 = lx + my + n = 0
b = 0 and r, s are those of equation a′x2 + 2h′x + b′ = 0. intersect at the point P and make an angle θ with each
If the sum of the ratios in which C and D divide AB is other. The equation of line L different from L2 which
zero, then passes through P and makes the same angle θ with L1
is
(A) ab′ + a′ b = 2hh′ (B) ab′ + a′b = hh′
(C) ab′ – a′b = 2hh′ (D)  none of these (A) 2 (al + bm) (ax + by + c) – (a2 + b2) (lx + my + n)
=0
32. The coordinates of a point P on the line 3x + 2y + 10 (B) 2 (al + bm) (ax + by + c) + (a2 + b2) (lx + my + n)
= 0 such that | PA – PB | is maximum where A is (4, 2) =0
and B is (2, 4), are (C) 2 (a2 + b2) (ax + by + c) – (al + bm) (lx + my + n)
(A) (22, 28) (B)  (22, –28) =0
(C) (–22, 28) (D)  (–22, –28) (D) none of these
10.36  Chapter 10

38. The equations of the perpendicular bisector of the cos α + cos β + cos γ
sides AB and AC of a ∆ ABC are x – y + 5 = 0 and x + (A)
cos α ⋅ cos β ⋅ cos γ
2y = 0, respectively. If the point A is (1, –2) then the
equation of the line BC is sin α + sin β + sin γ
(B)
(A) 14x + 23y = 40 (B)  14x – 23y = 40 sin α ⋅ sin β ⋅ sin γ
(C) 23x + 14y = 40 (D)  23x – 14y = 40 tan α + tan β + tan γ
(C)
39. The equation of a family of lines is given by (2 + 3t) tan α ⋅ tan β ⋅ tan γ
x + (1 – 2t) y + 4 = 0, where t is the parameter. The cos α + cos β + cos γ
(D)
equation of a straight line, belonging to this family, at sin α + sin β + sin γ
the maximum distance from the point (2, 3) is 44. OX and OY are two coordinate axes. On OY is taken a
(A) 21x + 14y = 0 (B)  21x – 14y = 0 fixed point P and on OX any point Q. On PQ an equi-
(C) 14x – 21y = 0 (D)  none of these lateral triangle is described, its vertex R being on the
side of PQ away from O, then the locus of R will be
40. ABCD is a square whose vertices A, B, C and D are
(0, 0), (2, 0), (2, 2) and (0, 2), respectively. This square (A) straight line (B)  circle
is rotated in the X-Y plane with an angle of 30º in (C) ellipse (D)  parabola
anti-clockwise direction about an axis passing through 45. If the vertices of a variable triangle are (3, 4), (5 cos
the vertex A. The equation of the diagonal BD of this θ, 5 sinθ) and (5 sinθ, –5 cosθ), then the locus of its
rotated square is orthocentre is
(A) 3 x + (1 − 3 ) y = 3 (A) (x + y – 1)2 + (x – y – 7)2 = 100
(B) (x + y – 7)2 + (x – y + 1)2 = 100
(B) (1 + 3 ) x − (1 − 2 ) = 2 (C) (x + y – 7)2 + (x – y – 1)2 = 100
(C) ( 2 − 3 ) x + y = 2( 3 − 1) (D) (x + y + 7)2 + (x + y – 1)2 = 100
46. If a right-angled isosceles triangle right-angled at ori-
(D) none of these gin has 3x + 4y = 6 as its base, then the area of the
41. The equations of the straight lines passing through triangle is
(– 2, –7) and cutting an intercept of length three units 11
(A) 7 (B) 
between the straight lines 4x + 3y = 12 and 4x + 3y = 25
3 are 36 12
7 (C) (D) 
(A) x + 2 = 0, y + 7 = ( x + 2) 25 25
24
47. The line x + y = 1 meets x-axis at A and y-axis at B . P is
7 the mid-point of AB . P1 is the foot of the perpendicular
(B) x − 2 = 0, y + 7 = − ( x + 2)
PRACTICE EXERCISES

24 from P to OA; M1 is that from P1 to OP; P2 is that from


(C) x + 2 = 0, y + 7 = − 7 ( x + 2) M1 to OA and so on. If Pn denotes the foot of the nth
24 perpendicular on OA from Mn–1, then OPn is equal to
7
(D) x + 2 = 0, y + 7 = − ( x + 2) 1 1
12 (A) n (B) 
2 2n−1
42. The coordinates of the point which is at unit distance
from the lines L1 ≡ 3x – 4y + 1 = 0 and L2 ≡ 8x + 6y + 1
(C) (D)  none of these
1 = 0 and lies below L1 and above L2 are 2n− 2
6 1 48. The line x + y = a meets x-axis at A. A triangle AMN is
(A)  ,  (B) 
6 1
 ,−  inscribed in the triangle OAB, O being the origin with
 5 10   5 10 
right angle at N; M and N lie respectively on OB and
(C)  ,  (D)  6 1
6 1
 ,−  3
AB. If area of ∆AMN is of the area of triangle OAB,
5 5 5 5 8
AN
43. The vertices of a triangle are A(x1, x1 tan α), B(x2, x2 then is equal to
BN
tan β) and C(x3, x3 tan γ). If the circumcentre of trian- (A) 3 (B) 
1
gle ABC coincides with the origin and H(a, b) be its 3
a
orthocentre then = 2
b
(C)
2 (D) 
3
Coordinates and Straight Lines  10.37

Previous Year's Questions

49. A triangle with vertices (4, 0), (–1, –1), (3, 5) is:[2002] 55. Locus of centroid of the triangle whose vertices are (a
cos t, a sin t), (b sin t, -b cos t) and (1, 0), where t is a
(A) isosceles and right angled
parameter, is[2003]
(B) isosceles but not right angled
(A) (3x – 1)2 + (3y)2 = a2 – b2
(C) right angled but not isosceles (B) (3x – 1)2 + (3y)2 = a2 + b2
(D) neither right angled nor isosceles (C) (3x + 1)2 + (3y)2 = a2 + b2
(D) (3x + 1)2 + (3y)2 = a2 – b2
50. The equation of the directrix of the parabola y2 + 4y +
56. Let A(2, –3) and B(–2, 1) be vertices of a triangle ABC.
4x + 2 = 0 is:[2002]
If the centroid of this triangle moves on the line 2x +
(A) x = -1 (B) 
x=1 3y = 1, then the locus of the vertex C is the line[2004]
3 3 (A) 2x + 3y = 9 (B)  2x – 3y = 7
(C) x =− (D) x=
2 2 (C) 3x + 2y = 5 (D)  3x – 2y = 3
57. The equation of the straight line passing through the
51. The incentre of the triangle with vertices (1, 3 ), (0, point (4, 3) and making intercepts on the co-ordinate
0) and (2, 0) is:[2002] axes whose sum is –1 is[2004]
2 1  x y x y
(A) + = −1 and + = −1
(A) 1, 3  (B) 
3,  2 3 −2 1
 2   3
x y x y
(B) − = −1 and + = −1
 1 
(C)  2 , 3  (D) 
1, 
2 3 −2 1
3 2   3 x y x y
(C) + = 1 and + = 1
2 3 2 1
52. Three straight lines 2x + 11y – 5 = 0, 24x + 7y – 20 =
0 and 4x – 3y – 2 = 0:[2002] x y x y
(D) − = 1 and + =1
(A) form a triangle 2 3 −2 1
(B) are only concurrent
(C) are concurrent with one line bisecting the angle 58. If the sum of the slopes of the lines given by x2 –
between the other two 2cxy – 7y2 = 0 is four times their product, then c has the
(D) none of the above value[2004]
(A) 1 (B)  –1

PRACTICE EXERCISES
53. A straight line through the point (2, 2) intersects the (C) 2 (D)  –2
lines 3 x + y = 0 and 3 x − y = 0 at the points A and
59. If one of the lines given by 6x2 – xy + 4cy2 = 0 is 3x +
B. The equation to the line AB so that the triangle OAB
4y = 0, then c equals[2004]
is equilateral, is:[2002]
(A) 1 (B)  –1
(A) x – 2 = 0 (B)  y–2=0
(C) 3 (D)  –3
(C) x + y – 4 = 0 (D)  none of these
60. Let P be the point (1, 0) and Q a point on the locus y2
54. If the equation of the locus of a point equidistant from = 8x. The locus of mid-point of PQ is[2005]
the points (a1, b1) and (a2, b2) is (a1 – a2)x + (b1 – b2)y (A) y2 – 4x + 2 = 0 (B)  y2 + 4x + 2 = 0
+ c = 0, then the value of ‘c’ is[2003] (C) x + 4y + 2 = 0
2
(D)  x2 – 4y + 2 = 0
(A) 1 ( a22 + b22 − a12 − b12 )
2 61. The line parallel to the x-axis and passing through the
intersection of the lines ax + 2by + 3b = 0 and bx - 2ay
(B) a12 + a22 − b12 − b22 - 3a = 0, where (a, b) ≠ (0, 0) is[2005]
(C) 1 ( a12 + a22 − b12 − b22 ) 3
(A) below the x-axis at a distance of from it
2 2
(D) a12 + b12 − a22 − b22 2
(B) below the x-axis at a distance of from it
3
10.38  Chapter 10

3 68. If one of the lines of my2 + (1 – m2)xy – mx2 = 0 is a


(C) above the x-axis at a distance of from it bisector of the angle between the lines x = 0 and y = 0,
2
then m is [2007]
2 1
(D) above the x-axis at a distance of from it (A) − (B)  –2
3 2
62. If a vertex of a triangle is (1, 1) and the mid-points of (C) 1 (D)  2
two sides through this vertex are (–1, 2) and (3, 2) then
69. The perpendicular bisector of the line segment joining
the centroid of the triangle is [2005]
P(1, 4) and Q(k, 3) has y-intercept –4. Then a possible
7  −1 7 
(A)  −1,  (B)  value of k is [2008]
 , 
 3  3 3 (A) 1 (B)  2
(C) –2 (D)  –4
(C) 1,  (D) 
1 7
7
 , 
 3 3 3
x y
63. A straight line through the point A(3, 4) is such that its 70. The line L given by + = 1 passes through the point
5 b
intercept between the axes is bisected at A. Its equation (13, 32) and the line K which is parallel to L has the
is  [2006] x y
(A) x + y = 7 (B)  3x - 4y + 7 = 0 equation + = 1. Then, the distance between L and
K is c 3 [2010]
(C) 4x + 3y = 24 (D)  3x + 4y = 25
64. The locus of the vertices of the family of parabolas 17
(A) 17 (B) 
a3 x 2 a 2 x 15
y= + − 2a is [2006]
3 2 23 23
(C) (D) 
105 3 17 15
(A) xy = (B)  xy =
64 4
35 64 71. The lines L1: y – x = 0 and L2: 2x + y = 0 intersect the
(C) xy = xy =
(D)  line L3: y + 2 = 0 at two respective points P and Q. The
16 105
bisector of the acute angle between L1 and L2 intersect
x
65. If (a, a2) falls inside the angle made by the lines y = , L3 at R. [2011]
2
x > 0 and y = 3x, x > 0, then a belongs to [2006] Statement - 1 : The ratio PR : RQ equals 2 2 : 5 .
Statement - 2 : In any triangle, bisector of an angle
(A)  0,  (B) 
1
(3,∞)
 2 divides the triangle into two similar triangles.
PRACTICE EXERCISES

(A) Statement - 1 is true, Statement-2 is true; Statement



(C)  , 3  (D) 
1 1
 −3, −  - 2 is not a correct explanation for Statement -1
2   2
(B) Statement - 1 is true, Statement- 2 is false.
66. Let A(h, k), B(1, 1) and C(2, 1) be the vertices of a (C) Statement - 1 is false, Statement- 2 is true.
right angled triangle with AC as its hypotenuse. If the (D) Statement - 1 is true, Statement - 2 is true; Statement
area of the triangle is 1, then the set of values which ‘k’ - 2 is a correct explanation for Statement -1
can take is given by [2007]
(A) {1, 3} (B)  {0, 2} 72. Equation of the ellipse which passes through the point
(C) {–1, 3} (D)  {–3, –2} (– 3, 1), whose axes are the coordinate axes and has
2
eccentricity is [2011]
67. Let P = (–1, 0), Q = (0, 0) and R = (3, 3 3 ) be three 5
points. The equation of the bisector of the angle PQR
(A) 5x2 + 3y2 – 48 = 0 (B)  3x2 + 5y2 – 15 = 0
 [2007]
(C) 5x2 + 3y2 – 32 = 0 (D)  3x2 + 5y2 – 32 = 0
3
(A) 3 x + y = 0 x+
(B)  y=0
2 73. If the line 2x + y = k passes through the point which
3 divides the line segment joining the points (1, 1) and
(C) x + y = 0 x + 3y = 0
(D)  (2, 4) in the ratio 3 : 2, then k equals [2012]
2
Coordinates and Straight Lines  10.39

29 80. Locus of the image of the point (2, 3) in the line (2x –
(A) (B) 
5 3y + 4) + k (x – 2y + 3) = 0, k ∈ R, is a: [2015]
5
11 (A) straight line parallel to y-axis.
(C) 6 (D) 
5 (B) circle of radius 2.
(C) circle of radius 3.
74. A line is drawn through the point (1, 2) to meet the (D) straight line parallel to x-axis.
coordinate axes at points P and Q respectively such
that it forms a triangle OPQ, where O is the origin. If
the area of the triangle OPQ is least, then the slope of 81. Two sides of a rhombus are along the lines, x – y + 1
the line PQ is [2012] = 0 and 7x – y – 5 = 0. If its diagonals intersect at (–1,
–2), then which one of the following is a vertex of this
1
(A) − (B)  –4 rhombus? [2016]
4 1
(C) –2 (D)  −
(A)  − , − 
10 7
2 (B)  (–3, –9)
 3 3
75. A ray of light along x + 3 y = 3 gets reflected upon 1 8
reaching x-axis, the equation of the reflected ray is

(C)  ,− 
(–3, –8) (D) 
3 3
 [2013]
82. Let k be an integer such that the triangle with vertices
(A) 3 y = x − 3 (B) y = 3x − 3 (k, – 3k), (5, k) and (– k, 2) has area 28 sq. units. Then
(C) 3 y = x − 1 y = x+ 3
(D)  the orthocentere of this triangle is at the point
[2017]
⎛ 3⎞ ⎛ 3⎞
76. The abscissa of the incentre of the triangle that has the (A)  ⎜1, ⎟ (B)  ⎜ 1, − ⎟
⎝ 4⎠ ⎝ 4⎠
coordinates of mid points of its sides as (0, 1) (1, 1)
and (1, 0) is [2013] ⎛ 1⎞ ⎛ 1⎞
⎜⎝ 2, − ⎟⎠
(C)  ⎜ 2, ⎟ (D) 
⎝ 2⎠ 2
(A) 2 − 2 (B) 
1+ 2

(C) 1 − 2 (D) 
2+ 2
83. A straight line through a fixed point (2, 3) intersects
77. Let a, b, c and d be non-zero numbers. If the point of the coordinate axes at distinct points P and Q. If O is
intersection of the line 4ax + 2ay + c = 0 with the line the origin and the rectangle OPRQ is completed, then
5bx + 2by + d = 0 lies in the fourth quadrant and is the locus of R is [2018]
equidistant from the two axes then [2014] (A) 3x + 2y = 6 (B)  2x + 3y = xy

PRACTICE EXERCISES
(A) 2bc – 3ad = 0 (B)  2bc + 3ad = 0 (C) 3x + 2y = xy (D) 3x + 2y = 6xy
(C) 3bc – 2ad = 0 (D)  3bc + 2ad = 0 84. Two vertices of a triangle are (0, 2) and (4, 3). If its
orthocenter is at the origin, then its third vertex lies in
78. Let PS be the median of the triangle with vertices P which quadrant? [2019]
(2, 2), Q(6, –1) and R(7, 3). The equation of the line (A) first (B)  second
passing through (1, –1) and parallel to PS is [2014] (C) fourth (D) third
(A) 4x – 7y – 11 = 0 (B)  2x + 9y + 7 = 0
(C) 4x + 7y + 3 = 0 (D)  2x – 9y – 11 = 0 85. If the area of an equilateral triangle inscribed in the
circle, x2 + y2 + 10x + 12y + c = 0 is 27 3 sq. units
79. The number of points, having both co-ordinates as then c is equal to: [2019]
integers, which lie in the interior of the triangle with (A) 13 (B) 20
vertices (0, 0), (0, 41) and (41, 0), is: [2015]
(C) 25 (D) –25
(A) 861 (B)  820
(C) 780 (D)  901
10.40  Chapter 10

ANSWER K EYS
Single Option Correct Type
1. (B) 2. (B) 3. (C) 4. (B) 5. (B) 6. (C) 7. (B) 8. (C) 9. (A) 10. (B)
11. (B) 12. (A) 13. (A) 14. (C) 15. (A) 16. (D) 17. (A) 18. (A) 19. (A) 20. (B)
21. (A) 22. (B) 23. (A) 24. (D) 25. (D) 26. (C) 27.  (C) 28. (C) 29. (B) 30. (D)
31. (A) 32. (C) 33. (A) 34. (B) 35. (A) 36. (B) 37. (A) 38. (A) 39. (D) 40. (C)
41. (C) 42. (B) 43. (D) 44. (A) 45. (B) 46. (C) 47. (A) 48. (A)

Previous Years’ Questions


49. (A) 50. (D) 51. (D) 52. (C) 53. (B) 54. (A) 55. (B) 56. (A) 57. (D) 58. (C)
59. (D) 60. (A) 61. (A) 62. (C) 63. (C) 64. (A) 65. (C) 66. (C) 67. (A) 68. (C)
69. (D) 70. (C) 71. (B) 72. (D) 73. (C) 74. (C) 75. (A) 76. (A) 77. (C) 78. (B)
79.  (C) 80. (B) 81. (D) 82. (C) 83. (C) 84. (B) 85. (C)
PRACTICE EXERCISES
Coordinates and Straight Lines  10.41

HINTS AND EXPLANATIONS

Single Option Correct Type


1. The point (3, 0) does not lie on the diagonal x = 2y. Let the
equation of a side through the vertex (3, 0) be
y – 0 = m(x – 3)
Since the angle between a side and a diagonal of a square is
π
⋅ , we have
4
π m − 1/ 2 2m − 1
± tan = =
4 1 + m(1/ 2) 2 + m
⇒ m = 3, – 1/3
∴ Slope of line AC = tan 60° = 3.
Thus, the equation of a side through (3, 0) is y = 3(x – 3) or

Now, line AC makes an angle of 60º with positive direction
 1 of x-axis and
y =  −  ( x − 3) and the one nearer to the origin is
 3 AC = AB = (3 − 2) 2 + (1 − 0) 2 = 2
3y + x – 3 = 0

∴ Coordinates of C are ( 2 + 2 cos 60°, 0 + 2 sin 60°)

2. The equation of the given line is
 1 3
x y i.e.,  2 +
, 
+ = 1 (1)  2 2 
a b 

4. Equation of two lines are



y = 3 x + 2, if x ≥ 0


and, y = − 3 x + 2, if x ≤ 0

Clearly, y ≥ 2.

HINTS AND EXPLANATIONS



This line cuts x-axis and y-axis at A (a, 0) and B  (0, b)
respectively.
Since area of DOAB = S
(Given)

1

ab = S or ab = 2S (∵ ab > 0) (2)
2
Since the line (1) passes through the point P (α, β)

α β α aβ
Also, y-axis is the bisector of the angle between the two

+ = 1 or + = 1   [Using (2)] lines. P1, P2 are two points on these lines, at a distance 5
a b a 2S
units from A. Q is the foot of the ⊥ from P1 and P2 on the
or, a2β – 2aS + 2aS = 0.
bisector (y-axis).
Since a is real, ∴ 4S2 – 8abS ≥ 0

Then, the coordinates of Q are (0, 2 + 5 cos 30º)
 1   5 3  1 
or, 4 S 2 ≥ 8aβ S or S ≥ 2aβ ∵ S = ab > 0 as ab > 0 
=  0, 2 +
 2 
  =  0, ( 4 + 5 3 ) 
 2   2 
Hence, the least value of S = 2aβ.
3. Slope of line 5. Side of square = 3 unit
⇒  ar (PQRS) = (3)2 = 9 square unit
0 −1
AB = = 1 = tan 45° One side of DPQT is the side PQ of the square i.e., 3 units
2−3
The other is one more than its side, i.e., (3 + 1) = 4 units

 ∠BAX = 45º.
1
Given, ∠CAB = 15º.
⇒ ar ( ∆PQT ) = (3)( 4) = 6 square unit

∠CAX = 60º. 2
10.42  Chapter 10

3 9. Let P ≡ (α, 0).


∴ ar ( PQRS ) = {ar( ∆PQT )}
Let the reflected ray makes an angle θ with positive direction
2
of x-axis, then the incident ray makes angle (π – θ) with pos-
6. Clearly, point (0, β) lies on y-axis. itive direction of x-axis.
Drawing the graph of the three straight lines, we see that The slope of the incident ray is

 7  5
Q ≡  0,  and P ≡  0,  .
 2  3
Therefore, the point (0, β) lies on or inside ∆ABC, when

5 7
≤ β ≤
3 2

0−3 3
= = tan(π − θ ), i.e., tan θ = (1)
α −2 α −2

The slope of the reflected ray is
4−0 4
= = tan θ , i.e., tan θ = (2)
6 −α 6 −α

From (1) and (2), we get
3 4
= ⇒ 18 − 3α = 4α − 8
α − 2 6 −α
26
⇒ 7α = 26 or α =
7
7. The point (1, β) lies on the line x = 1, for all real β. Clearly,
∴ The coordiantes of P are  , 0  .
from the figure, it will lie on or inside the triangle formed by 26

the given lines if 0 ≤ β ≤ 1.  7 
10. Let the coordinates of A and B be (a, 0) and (0, b),
HINTS AND EXPLANATIONS

respectively.
Then, equation of line AB is
x y
+ = 1.
a b

8. The refracted ray passes through the point (5, 0) and makes
an angle 120º with positive direction of x-axis


Since, it passes through the point P (4, 2)
4 2

+ = 1. (1)
a b
a b
Now, centre of the circumcircle of ∆OAB =  ,  .

2 2

So, equation (1) can be written in the form
∴ The equation of the refracted ray is

2 1
(y – 0) = tan 120º (x – 5)
+ =1
a/2 b/2
⇒ y = − 3 ( x − 5) or 3x + y − 5 3 = 0
∴ locus of circumcentre is

Coordinates and Straight Lines  10.43

2 1
Equation of the line PQ is
+ = 1 or 2 x −1 + y −1 = 1.
x y x y

  + = 1 (1)
h k
11. Clearly, the point (2 cosθ, 2 sinθ) lie on the circle

Given, BP ⋅ CQ = AB2
⇒ (h – a) (k – a) = a2

⇒ hk – ak – ah + a2 = a2

or, ak + ha = hk

a a
or,
+ = 1 (2)
h k

From (2), it follows that line (1), i.e., PQ passes through the
fixed point (a, a).
x2 x3 y2 y3
15. Let = = r and = = r
x1 x2 y1 y2
⇒ x2 = x1r, x3 = x1r2, y2 = y1r and y3 = y1r2.
x2 + y2 = 4.
We have,
The two lines represented by the equation y = |x – 2| are y =

x1 y1 1 x1 y1 1 x1 y1 1
x – 2 and y = 2 – x.
π π
∆ = x2 y2 1 = x1r y1r 1 = 0 0 1− r
From the figure, θ can be vary from − to .
x3 y3 1 x1r 2 y1r 2 1 0 0 1− r
2 2
12. The lines y = x and 3 y + x = 0 are inclined at 45º and 150º, [Applying R3 → R3 – rR2 and R2 → R2 – rR1]

respectively, with the positive direction of x-axis. So, the
= 0 ( R2 and R3 are identical)

angle between the two lines is an obtuse angle. Therefore,
orthocentre lies outside the given triangle, whereas incentre
Thus, (x1, y1), (x2, y2), (x3, y3) lie on a straight line.
and centroid lie within the triangle (In any triangle, the cen- 16. The sides are,
troid and the incentre lie within the triangle).
y = 3 ( x − 1) + 2 and y = − 3x
13. Equation of a straight line passing through the point P (α, β)
and making an angle θ with positive direction of x-axis is
x −α y − β
= = r (say)
cosθ sin θ

HINTS AND EXPLANATIONS


Coordinates of any point on this line are
(α + r cosθ, β + r sinθ)
If it lies on the line ax + by + c = 0, then
a (α + r cosθ) + b (β + r sinθ) + c = 0
aα + bβ + c
⇒ r=−
a cosθ + b sin θ
aα + bβ + c
Thus, PQ = r = −

a cosθ + b sin θ
14. We take A as the origin and AB and AC as x-axis and y-axis,
The two lines are at an angle of 60° to each other. Now, any
respectively. line parallel to obtuse angle bisector will make equilateral
Let AP = h, AQ = k. triangle with these lines as its two sides.
17. We have, d(x,y) = Max. {|x|, |y|} = k
If |x| > |y|, then k = |x| ⇒ x = ±k

If |y| > |x|, then k = |y| ⇒ y = ±k

Hence, the locus represents a straight line.

18. Area of the triangle
a x 1 a x 1
1 1
= b y 1 = b−a y−x 0
2 2
c z 1 c−b z−y 0
10.44  Chapter 10

[Applying R2 → R2 – R1 and R3 → R3 – R2]



1
= [(b − a)( z − y ) − (c − b)( y − x )]
2
1 ∴
= d ( z − 2 y + x ) [ b – a = c – b = d]
2
1
= d ( xr 2 − 2 xr + x ) [∴ y = xr and z = xr2]
2
1
= dx( r − 1) 2 , which is independent of b.
2

19. Let ∠BAO = α.


22. Since, y – y1 = m(x – x1), where m and x1 are fixed.
Then, BC = 2 sin  α = CO and CA = 2 cosα. Therefore, the
Therefore, the system represents a set of parallel lines of
coordinates of A and B are slope m.
Since all these parallel lines have fixed x coordinate
(abscissa)
∴ there will only be one possible line.
23. Since the line BD divides the triangle into two parts of
equal area, BD is a median and D is (–1, –2). Slope of
1
BD = −
2

A ≡ (2 cos α – 2 sin α, 0)
and, B ≡ (–2 sin α, 2 sinα).
HINTS AND EXPLANATIONS

If P(x, y) is the mid point of AB, then


2x = 2 cos α – 4 sin α and 2y = 2 sin α
⇒ x = cos α – 2 sin α and y = sin α
⇒ cos α = x + 2y and sin α = y
Squaring and adding, we get (x + 2y)2 + y2 = 1, which is the
required locus. So, the required line is y + 3 = 2(x – 1)
⇒  y – 2x + 5 = 0
20. It is clear that the lines lie on opposite sides of the
k −0
origin O. Let the equation of any line through O be 24. Slope of line AC = mAC = = tan α
h−a
x y
= . If OP = r1 and OQ = r2 then the coordi- k −0
cosθ sin θ Slope of line BC = mBC =
= tan β
h+a
nates of P are (r1 cosθ, r1 sinθ) and that of Q are (–r2 cosθ,
Since, ∠A = π – α
{from figure}
– r2sinθ).
k
Since P lies on 4x + 2y = 9, 2r1 (2 cosθ + sinθ) = 9 and
∴ tan A = tan (π – α) = – tan α =
(1)
h−a
Q lies on 2x + y + 6 = 0, – r2(2 cosθ + sinθ) = –6 so that
r1 9 3
= = and the required ratio is thus 3 : 4.
r2 12 4

21. Since, xy > 0, therefore P lies either in the first quadrant or


in the third quadrant. The inequality x + y < 2 represents all
the points below the line x + y = 2. Therefore, xy > 0 and x
+ y < 2 imply that P lies either inside DOAB or in the third
quadrant.
Coordinates and Straight Lines  10.45

Similarly, ∠B = β

k
∴ tan B = tan β =
(2)
h+a
Also, A – B = θ
{given}
∴ tan (A – B) = tan θ

tan A − tan B

= tan θ
1 + tan A tan B

 −k   k 
 − 
⇒  h − a   h + a  = tan θ {using (1) and (2)}

 −k   k 
After translation through a distance 1 unit along the positive
1+   
 h − a  h + a  x-axis of the point.

The coordinates of the final point are (0, 3 2 ) .
− k ( h + a) − k ( h − a) −2hk
⇒ tan θ =
= 2
( h2 − a 2 ) + ( −k 2 ) h − k 2 − a2 28. If P1 be the reflection of P in y-axis then P1 ≡ (– 2, 3)
⇒ h2 – k2 + 2hk cot θ = a2

Equation of line P1R is
∴ locus is x2 – y2 + 2xy cot θ = a2

10 − 3
( y − 3) = ( x + 2)
25. Slope of the line in the new position is b , since it is ⊥ to the 5+2
a
line ax + by + c = 0 and it cuts the x-axis at (2, 0). Hence,
b
the required line passes through (2, 0) and its slope is .
a
Therefore, its equation is

⇒  y = x + 5. It meets y-axis at (0, 5) ⇒ Q ≡ (0, 5)


29. Let PQ = QR = RP = r

HINTS AND EXPLANATIONS


and, ∠ PQX = θ, then
π
b
∠RQX = +θ .
y − 0 = ( x − 2) 3
a
⇒ ay = bx – 2b Given, PL = a, RN = 1.

⇒ ay – bx + 2b = 0
Now, a = PL = r sin θ
a2 a2
26. We have, a − > 0 and a − < 0
4 2

π 
⇒ 0 < a < 4, a ∈ (– ∞, 0) ∪ (2, ∞) and, 1 = RN = r sin  + θ 

3 
⇒ a ∈ (2, 4)
π π
27. If (α, β) be the image of (4, 1) w.r.t. y = x – 1 then ⇒ 1 = r  sin cosθ + cos sin θ 

(α, β) = (2, 3), say point Q.  3 3 
10.46  Chapter 10

 3 1  2h b
⇒ r 
cosθ + sin θ  = 1 ∴
p+q=− and pq = (1)
 2 2  a a
 

Also, r, s are the roots of the equation
 3 a2 1 a   a a′ x2 + 2h′ x + b′ = 0

⇒ r 
1 − 2 + ⋅  = 1 ∵sin θ = 
 2 r 2 r   r 2h′ b′
  ∴ r + s = −
and rs = (2)
a′ a′

3 2 a Let C divides AB in the ratio α : 1.

r − a2 = 1 −
2 2 αq + p p−r
Then, r =
⇒ α=
α +1 r−q
( 2 − a) 2 4 + a 2 − 4 a

or r 2 − a2 = = Let D divides AB in the ratio β : 1.

3 3
βq + p p−s
4 + a2 − 4a 4 + 4a2 − 4a Then, s =
⇒ β=
⇒ r 2 =
+ a2 = β +1 s−q
3 3
Given, α + β = 0

2 p−r p−s
∴ r = a − a + 1.
2

+ = 0 ⇒ (p + q) (r + s) – 2pq – 2rs = 0
3 r−q s−q

30. Let P (x, y) be the moving point whose locus is required. Let  2h   2h′  2b 2b′
⇒  −   −
− − = 0 (Using (1) and (2))
A ≡ (a, 0) and B (– a, 0).  a   a′  a a′
⇒ ab′ + a′b = 2hh′.

32. Let P ≡ (x1, y1) and ∠APB = θ

(PA) 2 + ( PB ) 2 − ( AB ) 2
Then, cosθ =

HINTS AND EXPLANATIONS

Let ∠PAB = α and ∠PBA = β.


2 PA ⋅ PB
PC y Since cos θ ≤ 1

∴ In DPCA, tan α =
=
CA a − x (PA) 2 + ( PB ) 2 − ( AB ) 2

≤1
2 PA ⋅ PB
PC y
and in DPBC, tan β =
= ⇒ (PA)2 + (PB)2 – (AB)2 ≤ 2 PA ⋅ PB

BC a + x
⇒ (PA – PB)2 ≤ (AB)2

Given, ∠PAB – ∠PBA = 2θ (constant)

⇒ | PA – PB | ≤ | AB | = ( 4 − 2) + ( 2 − 4)
2 2

⇒ α – β = 2θ ⇒ tan (α – β) = tan 2θ

⇒ | PA – PB | ≤ 2 2 .
tan α − tan β

= tan 2θ ⇒ Maximum value of | PA – PB | is 2 2 when θ = 0
1 + tan α tan β
i.e., P lies on the line AB as well as on the given line.
y y  equation of AB is


a − x a + x = tan 2θ 4−2
y − 2 = (x – 4) or x + y = 6  (1)
y y 2−4
1+ ⋅
a−x a+x and given line is 3x + 2y + 10 = 0 (2)
2 xy Solving (1) and (2), we get P ≡ (–22, 28).

= tan 2θ 33. Suppose, the required line has slope tanθ. Its equation is
a2 − x 2 + y 2
x+5 y+4
⇒ x2 – y2 + 2xy cot 2θ = a2, = = r (1)
cosθ sin θ
which is the required locus. Any point on this line has coordinates
31. Since p, q are the roots of the equation (–5 + r cos θ, –4 + r sin θ)
ax2 + 2hx + b = 0 Its distance from (–5, –4) is r.
Coordinates and Straight Lines  10.47


Coordinates of points B, C and D are
1  1  8  2  1
B ≡ (–5 + AB cosθ, –4 + AB sinθ)
= 2  0 −  − 1 2 −  + 1 − 0  = .
2  3  3  3  3
C ≡ (–5 + AC cosθ, –4 + AC sinθ)

3 1 1
D ≡ (–5 + AD cosθ, –4 + AD sinθ)
Hence, area of ∆ABC × area of ∆BDK =
× = .
2 3 2

Points B, C, D lie on the lines 35. Equation of the line AB is
x + 3y + 2 = 0
(2) x y
+ =1
2x + y + 4 = 0
(3) 7 −5
x – y – 5 = 0
(4) ⇒ 5x – 7y – 35 = 0.
The equation of the variable line PQ perpendicular to AB is

respectively.
7x + 5y + k = 0.
∴ – 5 + AB cosθ + 3 (– 4 + AB sinθ) + 2 = 0
 k 
⇒ Coordinates of P are  − , 0  and that of Q are  0, −  .
k
15

= cosθ + 3 sin θ  7   5
AB
x y
and, 2(– 5 + AC cosθ) + (– 4 + AC sinθ) + 4 = 0

Now, equation of the line AQ is + = 1 (1)
10 7 −k

= 2 cosθ + sin θ 5
AC
x y
and, (– 5 + AD cosθ) – (– 4 + AD sinθ) – 5 = 0

and, equation of the line BP is + = 1 (2)
k −5

⇒ 6 = cosθ − sin θ
7
AD

Now, locus of R, the point of intersection of AQ and BP can
2 2 2

Since 
15   10   6  be obtained from (1) and (2) by eliminating k from their
 +  =  , we get equations. Equation (1) can be rewritten as
 AB   AC   AD 
(cosθ + 3 sinθ)2 + (2 cosθ + sinθ)2 y x 7− x
=1− =
= (cosθ – sinθ)2 k 7 7

⇒ 4 cos2θ + 9 sin2θ + 12 sinθ cosθ = 0 5
⇒ (2 cosθ + 3 sinθ)2 = 0 ⇒ 2 cosθ = – 3 sinθ −k 7y  7 y  (3)

= ⇒ k = −5  
2 5 7− x 7− x
⇒ tan θ = − .
3 x y 5+ y

and, equation (2) can be rewritten as =1+ =
∴ From (1), the equation of line is k 5 5

2 7

HINTS AND EXPLANATIONS


y + 4 = − ( x + 5) k 5x 35 x
3 ⇒ − =
⇒ k =− (4)
or 2x + 3y + 22 = 0. 7 5+ y 5+ y
34. Area of ∆ABC = Area of ∆ABD + Area of ∆BDC 35 y 35 x
From (3) and (4), we get −
=−
1 1 3 7− x 5+ y
= ( 2) (1) + (1) (1) = .
2 2 2 ⇒ y(5 + y) = x (7 – x)
1 ⇒ x(x – 7) + y (y + 5) = 0 is the locus of R.
 slope of AB = slope of DK = ,

2 36. Let Q be the position of the point P (2, 3) after first trans-
1 formation. Then, Q ≡ (3, 2). Let R be the position of Q after
∴ equation of DK is y – 0 =
(x – 2) or 2y = x – 2 (1)
2 second transformation. Then, R ≡ (3, 4).
Equation of BC is y – 0 = – 1 (x – 3) or y = – x + 3
(2) Let S be the new position of R after third transformation. Let
S ≡ (x1, y1).
If OR makes an angle α with x-axis, then
4
tan α = .
3

Solving (1) and (2), we get coordinates of k as  ,  .


8 1

2 1 1  3 3
1
∴ Area of ∆BDK = | 2 0 1 |

2
8 1
1
3 3
10.48  Chapter 10

β +2
∴ the slope of AB =
(1)
α −1

(ii) (iii)
The equation of the perpendicular bisector of AB is
x – y + 5 = 0
(2)
4 3  β +2
⇒ sin α =
and cos α = . Also, OR = 5 = OS.
5 5
From (1) and (2), we have   (1) = −1
 α −1 
5 ⇒ α + β + 1 = 0 
(3)

Now, x1 = OS1 = 5  cos  (α + 45º) = (cos  α
2
Also, the mid point of AB lies on (2),
– sin α)  α +1  β − 2 
∴ 
− +5=0
5 3 4 1  2   2 
=  − =−
25 5 2 ⇒ α – β + 13 = 0
(4)
and, y1 = SS1 = 5 sin (α + 45º)
Solving (3) and (4), we get α = –7 and β = 6.

5
So, the coordinates of B are (–7, 6).
= (sin α + cos α )
Similarly, the coordinates of C are  ,  .
11 2
2
 5 5
5  4 3 7 ∴ The equation of the line BC is

=  + = .
2  5 5 2 2
−6
 1 7  y − 6 = 5 ( x + 7)
Hence, the coordinates of S are  −
, .
 2 2 11
+7
5
37. The equation of the line L passing through the intersection of
28
L1 and L2 is ⇒ y – 6 = –
(x + 7) ⇒ 23(y – 6) + 14(x + 7) = 0
46
(ax + by + c) + λ (lx + my + n) = 0
HINTS AND EXPLANATIONS

(1)
⇒ 14x + 23y = 40.

By the given condition, L1 is the bisector of the angle
between L and L2. Let Q (α, β) be any point on L1. Hence, the equation of the line BC is 14x + 23y = 40.

∴ Length of perpendicular from Q (α, β) on L2 and L must 39. We have,
be equal, thus (2 + 3t) x + (1 – 2t) y + 4 = 0
lα + mβ + n ( aα + bβ + c) + λ (lα + mβ + n) ⇒ (2x + y + 4) + t (3x – 2y) = 0
2 =± (2)
l + m2 ( a + λ l ) 2 + ( b + λ m) 2 ⇒  Every member of the given family of lines passes
Since Q (α, β) lies on L1 ∴ aα + bβ + c = 0
(3) through the point of intersection of the lines

From (2) and (3), we get
1 λ
2 =±
l +m 2
( a + λ l ) + ( b + λ m) 2
2

⇒ (a + ll)2 + (b + λm)2 = λ2 (l2 + m2)



⇒ a2 + b2 + 2λ al + 2λ bm = 0

a2 + b2
⇒ λ = −
(4)
2( al + bm)
Substituting the value of λ from (4) in (1), we get
2x + y + 4 = 0
(1)
a2 + b2 and, 3x – 2y = 0
(2)
( ax + by + c) − (lx + my + n) = 0
2 ( al + bm)
Solving (1) and (2), we get
⇒ 2 (al + bm) (ax + by + c) – (a2 + b2) (lx + my + n) = 0, 8 12
x = − and y = − .
which is the required equation of the line L. 7 7
38. Let the coordinates of B be (α, β). Since coordinates of A are  8 12 
So, the point of intersection is  − , −  .

(1, –2),  7 7
Coordinates and Straight Lines  10.49

If θ be the angle that the required line makes with the nor-

The required line passes through the point  − , −  and
8 12
mal, then
 7 7
3 4
 8 12  cosθ − ∴ tan θ = .
is ⊥ to the line joining (2, 3) and  − , −  . 5 3
 7 7 Let θ be the angle that the required line makes with the nor-

3
Slope of the line joining (2, 3) and  − , −  is
8 12 mal whose slope is . Then, we have
4
 7 7
12
3+ 3
7 = 33 = 3 . m−
= 4 4 −7
8 22 2 ± tan θ = ± = ⇒ m = ∞,
2+ 3 1 + 3m 24
7
2 4
∴ Slope of required line = − .

Therefore, the equations of the required lines are
3
∴ Equation of the required line is
−7
x + 2 = 0 and y + 7 =
(x + 2).
24
 12  2 8
 y + 7  = −3 x + 7 42. If (x, y) be the coordinates of the required point, then we
   
have
or, 14x + 21y + 52 = 0.

| 3x − 4 y + 1 |
= 1 (1)
40. Coordinates of vertices B, C and D are 5
(2 cos 30°, 2 sin 30°),
| 8x + 6 y + 1 |
( 8 cos75°, 8 sin 75°)

and, = 1 (2)

10

and, (– 2 cos 60°, 2 sin 60°), respectively.
Since (x, y) lies below L1, therefore
3x − 4 y + 1
< 0, i.e., 3x – 4y + 1 > 0
−4

and since it lies above L2, therefore
8x + 6 y + 1
> 0, i.e., 8x + 6y + 1 > 0
6

Removing the mod sign from equations (1) and (2), we have
+ (3x – 4y + 1) = 5

HINTS AND EXPLANATIONS


and, + (8x + 6y + 1) = 10

i.e., B ≡ ( 3 ,1), C ≡ ( 3 − 1, 3 + 1) and D ≡ ( −1, 3 ).



Solving, we get the coordinates of the required point as
 6 −1 
3 −1 ( 3 − 1) 2  , .
Slope of BD =
= = 3 − 2.  5 10 
−1 − 3 − ( 3 + 1) ( 3 − 1) 43. Let R be the radius of the circumcircle and O be the origin,
∴ Equation of diagonal BD is then AO = x12 + x2 2 tan 2 α
y − 1 = ( 3 − 2) ( x − 3 ) ⇒ R = x1 sec α ⇒ x1 = R cos α. Similarly, x2 = R cos β and
x3 = R cos γ
⇒ ( 2 − 3 ) x + y = 2( 3 − 1).

9 So, the coordinates of vertices are A(R cos α, R sin α), B(R
41. The normal to the given parallel lines cuts an intercept = cos β, R sin β), C(R cos γ, R sin γ). Hence, the
5
∑ R cos α ∑ R sin α 
coordinates of centroid G are  , .
 3 3 
Since the orthocentre H(a, b), circumcentre C(0, 0) and the
centroid G are collinear, therefore
Slope of OH = Slope of OG
b R(sin α + sin β + sin γ )
⇒ = .
a R(cos α + cos β + cos γ )
44. Let P be (0, c), c is a constant
From the figure, h = OL = OQ + QL
= c cot θ + QR cos (180 – 60 – θ)
= c cot θ + PQ {cos 120 cosθ + sin 120 sinq}
10.50  Chapter 10

6 6
OM = = .
3 +4
2 2 5
π 6
Since, ∠OAM =
= OM
, therefore   AM = .
PQ 3 4 5
= c cot θ − cosθ + PQ sin θ
2 2 36
Hence, area of ∆OAB is   ∆ = OM × AM =
.
25
1 3  c 
= c cot θ − c cosec θ cosθ + c cosecθ sinθ ∵ sin θ =
2 2  PQ  1
47.  P1 is mid-point of OA, therefore OP1 =
2
c
= (cotθ + 3 ) (1) 1 1 1
2 P2 is mid-point of OP1, therefore OP2 = × = 2

2 2 2
Again, k = RL = RQ sin (180 – 60 – θ)

 1 3 
= PQ sin θ . + cosθ . 
 2 2 

 1 3 
= c cosec θ  sin θ + cosθ 
 2 2 
c
= (1 + 3 cot θ ) (2)
2
Eliminating cot θ from (1) and (2), we get k = 3 h − c

HINTS AND EXPLANATIONS

∴ The required locus is y = 3 x − c a straight line.


45. Circurmcentre of the triangle is (0, 0) and


Proceeding like this, we have
1
 3 + 5 cosθ + 5 sin θ 4 + 5 sin θ − 5 cosθ  OPn = n .
centroid ≡  ,  2
 3 3 

Centroid divides the join of orthocentre and circumcentre AN
in the ratio, 2 : 1 48. Let =λ
BN
∴ h = 3 + 5 cos θ + 5 sin θ and

k = 4 + 5 sin θ – 5 cos θ, where (h, k) represents the

orthocentre
h+k −7 h − k +1
⇒ sin θ =
and cosθ =
10 10
⇒ (h + k – 7)2 + (h – k + 1)2 = 100

∴ Locus of orthocentre is (x + y – 7)2 + (x – y + 1)2

= 100.

46. Let OAB be the right-angled isosceles triangle. Then,


π
∠OAB = ∠OBA =
4
The perpendicular distance of the line 3x + 4y = 6 from the

origin is
Coordinates and Straight Lines  10.51

 a aλ  1 2aλ 2a a 2λ
Then, N ≡  ,  = × =
1+ λ 1+ λ  2 1+ λ 1+ λ (1 + λ ) 2
Slope of AB = – 1
1 1
∴ slope of MN = + 1 Area of ∆ABC =
× OA × OB = a 2
2 2
Equation of MN is given by
a 2λ 3 1
aλ a
Given, = × a2
y − =x− (1) (1 + λ ) 2 8 2
1+ λ 1+ λ
⇒ 16λ = 3(1 + λ)2

 a (λ − 1)  1
Hence, M ≡  0,
 ⇒ 3l2 – 10λ + 3 = 0  ⇒  λ = , 3

 λ +1  3
[putting x = 0 in equation (1)]
1
1 For λ = , M lies outside segment OB, hence the only

Area of ∆AMN = × AN × MN
3
2 acceptable value for λ = 3.

Previous Year's Questions

49. Let A(4, 0), B(–1, –1) and C(3, 5) be the vertices of a ∆ABC. b = AC ( 2 − 1) 2 + (0 − 3 ) 2 = 2
Then AB = ( −1 − 4) 2 + ( −1 − 0) 2

and c = AB (0 − 1) 2 + (0 − 3 ) 2 = 2
= 25 + 1 = 26
b = AC = -J (2 - 1f + (0 - V3)2 = 2

BC = (3 + 1) 2 + (5 + 1) 2 = 4 2 + 6 2 and c = AB = V(0 - 1)2 + (0 - 41) = 2

= 16 + 36 = 52 ∴ The triangle is an equilateral triangle.

and CA = ( 4 − 3) 2 + (0 − 5) 2
∴ Incentre is same as centroid of the triangle.

⇒ Co-ordinates of incentre are

= 1 + 25 = 26
1+ 0 + 2 3 + 0 + 0   1 
( ) +( )
2 2
CA + AB 2 =
2  ,  i.e., 1,
26 26 
 3 3   3
∴ = 26 + 26 = 52
52. Key Idea: Equations of angle bisectors of lines

HINTS AND EXPLANATIONS


= BC2
⇒ CA2 + AB2 = BC2 a1x + b1y + c1 = 0, and
Thus, the triangle is isosceles and right angled triangle. a2x + b2y + c2 = 0
50. The equation of parabola is a1 x + b1 y + c1 a2 x + b2 y + c2

are =±
y2 + 4y + 4x + 2 = 0 a12 + b12 a22 + b22
⇒ y2 + 4y + 4 = –4x – 2 + 4 For the two lines 24x + 7y - 20 = 0 and 4x - 3y - 2 = 0, the

⇒ ( y + 2) 2 = − 4  x − 
1 angle bisectors are given by

 2 24 x + 7 y − 20 4x − 3y − 2
1 =±
Transformation y + 2 = Y and x − = X gives 25 5
2
Y 2 = – 4X
Taking positive sign, we get
Here a = 1 2x + 11y - 5 = 0

∴ Equation of directrix is X = 1
Therefore, the given three lines are concurrent with one line
1 bisecting the angle between the other two.
⇒ x − = 1
2
3 53. 3 x + y = 0 makes an angle 120° with OX and 3 x + y = 0
⇒ x = makes an angle of 60° with OX. So, the required equation of
2
line is y - 2 = 0.
51. Key Idea: If the triangle is equilateral, then the incentre coin- 54. Let p (x, y) be the point equidistant to the given points, then
cides with the centroid of the triangle. (x - a1)2 + (y - b1)2 = (x - a2)2 + (y - b2)2
Let A(1, 3 ), B(0, 0), C(2, 0) be the vertices of a triangle 1
ABC. ⇒ (a1 - a2) x + (b1 - b2) y + (b22 − b12 + a22 − a12 ) = 0
2
∴ a = BC ( 2 − 0) 2 + (0 − 0) 2 = 2
Hence, (A) is the correct answer.
10.52  Chapter 10

55. Given parametric equations


∴ centroid is 
1− 3 + 5 1+ 3 + 3   7 
a cos t + b sin t + 1 a sin t − b cos t + 1 ,  = 1, 
x= , y=  3 3   3
3 3
 1
2
a2 + b2
This implies that  x −  + y 2 = .
 3  9
Hence, (B) is the correct answer.
 h ( k − 2) 
56. If C be (h, k) then centroid is  ,  it lies on 2x + 3y = 1.
3 3 
Therefore the locus is 2x + 3y = 9.
x y 4 3
57. Point (4, 3) lies on + = 1 with a + b = -1 then + = 1
a b a b 63. The required line intersects the axes at points (0, 8) and (6, 0)
⇒ a = 2, b = -3 or a = -2, b = 1. Hence, the equation of the line is
x y x y
Hence − = 1 and + = 1. −4
2 3 −2 1 y= ( x − 6)
3
2c 1
58. m1 + m2 = − and, m1m2 = − ⇒ 4x + 3y = 24
7 7
⇒ m1 + m2 = 4m1m2  (given)
a2 x 2 a2 x
64. Given parabola: y = + − 2a
⇒ c = 2.
3 2
1 6 3 Vertex: (α, β) implies

59. m1 + m2 = , m1m2 = and m1 = − .
4c 4c 4  a4 
−a 2 a3 1 8
Hence c = -3. −  + 4 ⋅ ⋅ 2a  −  +  a4
3
60. P = (1, 0) α = 23 = − , β =  4 3  = −  4 3
2a 4a a3 4 3
Let Q = (h, k) then k2 = 8h 4 a
3 3 3
Let (α, β) be the midpoint of PQ, then
h +1 k +0 35 a 35
α= ,β = =− ×3 = − a
2 2 12 4 16
⇒ 2α – 1 = h, 2β = k. 3  35  105
⇒ (2β)2 = 8(2a – 1)  ⇒  b 2 = 4α – 2
∴ αβ = −  −  a =
4 a  16  64
HINTS AND EXPLANATIONS

⇒ y2 – 4x + 2 = 0.
65. We must have
61. Required equation is of the from
a
ax + 2by + 3b + λ(bx – 2ay – 3a) = 0 a2 - 3a < 0 and a 2 −
> 0
2
⇒ (a + bλ)x + (2b – 2aλ)y + 3b – 3la = 0
a 1
a + bλ = 0 ⇒ λ = − ⇒ < a< 3
b 2
a 1
⇒ ax + 2by + 3b − (bx – 2ay – 3a) = 0
b 66. We have ( k − 1) = ±1
2
2a 2 3a 2
⇒ ax + 2by + 3b – ax + y+ =0 ⇒ k - 1 = ±2

b b
⇒ k = 3 or k = -1

 2a 2  3a 2
⇒ y  2b +  + 3b + =0
 b  b
 2b 2 + 2 a 2   3b 2 + 3a 2 
⇒ y   = − 
 b   b 
−3( a + b ) −3
2 2
⇒ y = =
2(b 2 + a 2 ) 2
3 3
⇒ y = − so it is units below x-axis.
2 2 2π
62. Vertex of triangle is (1, 1) and midpoint of sides through this =− 3
67. Slope of the line QM is tan
vertex is (–1, 2) and (3, 2) ⇒ vertices B and C come out to 3
be (–3, 3) and (5, 3) Hence, the equation of line QM is y = − 3 x.

Coordinates and Straight Lines  10.53

32
b2 =
5
∴ Required equation of ellipse 3x2 + 5y2 - 32 = 0.

 6 + 2 12 + 2 
73. Point p =  , 
 5 5 
 8 14 
p =  , 
5 5 
 8 14  lies on 2x + y = k
p =  , 
5 5 
68. Equation of bisectors of line x = 0 and y = 0 are y = ±x. 16 14
⇒ + =k
Put y = ±x in my2 + (1 - m2)xy - mx2 = 0, we get (1 - m2)x2 5 5
=0 30
⇒ =k = 6
⇒ m = ±1. 5
69. Slope of the bisector = k - 1 74. Equation of line passing through (1, 2) with slope m is y - 2
 k +1 7  = m(x - 1)
Mid-point of PQ = 
,  ( m − 2) 2
 2 2 Area of ∆ OPQ =


Equation of bisector is 2|m|
7  ( k + 1)  m2 + 4 − 4m
y − = ( k − 1)  x −  ∆=
2  2  2m
Put x = 0 and y = -4.
m 2
⇒ k = ±4.
∆= + −2
2 m
b
70. Slope of line L = − m 2
5 ∆ is least if =
2 m
3
Slope of line K = −
⇒ m2 = 4

c
⇒ m = ±2


Link L is parallel to link k.

HINTS AND EXPLANATIONS


⇒ m = – 2

b 3

= ⇒  bc = 15 1
5 c 75. Slope of the incident ray = − .
3 1

(13, 32) is a point on L
So, the slope of the reflected ray must be .
13 32 32 8 3

+ =1 ⇒ =− Now, the point of incidence is ( 3 , 0) And so, the equation.

5 b b 5
1
⇒ b = -20  ⇒  c = −

3 of reflected ray is y = (x − 3 ).
4 3
Equation of K: y - 4x = 3

| 52 − 32 + 3 | 23 ax1 + bx2 + cx3
Distance between L and K =
= 76. Abscissa =
a+b+c
17 17
71. P(-2, -2); Q = (1, -2) 2× 2 + 2 2 × 0 + 2× 0
Equation of angular bisector OR is
=
2+2+2 2
( 5 + 2 2)x = ( 5 − 2) y
4 2
∴ PR : RQ = 2 2 : 5
= = = 2 − 2.
4+2 2 2+ 2
 2
2
3 3a
72. b 2 = a 2 (1 − e 2 ) = a 2 1 −  = a 2 =
Alternate Solution:
 5 5 5 A
2 2 Abscissa = r = (s - a) tan

x y 9 5 2
2 + 2 = 1 ⇒ + =1
a b a 2 3a 2
4+2 2  π
32 = − 2 2  tan = 2 − 2
a2 =  2  4
3
10.54  Chapter 10

77. Let point of intersection is (h, - h) x y


 4 ah − 2ah + c = 0 2m − 3 + 3 − 2m = 1
⇒ 
Q
R
m
5bh − 2bh + d = 0
c d 2m − 3
P= ,0

So, =− m P
2a 3b
Q = (0, 3 –2m)

3bc - 2ad = 0

 13  ⎛ 2m − 3 ⎞
78. S  R=⎜ , 3 − 2m ⎟
,1 , P ( 2, 2) ⎝ m ⎠
2 
2 2m − 3
x= y = 3 – 2m
Slope = −
m
9
y +1 2 2m = 3 – y


Equation will be =− Substituting the value of m in x

x −1 9
9y + 9 + 2x - 2 = 0
3− y −3
x=
2x + 9y + 7 = 0
(3 − y )
2
79. x + y < 41, x > 0, y > 0 is bounded region.
−2 y
Now, number of positive integral solutions of the equation x ⇒ x =

+ y + k = 41 will be number of integral co-ordinates in the 3− y
bounded region. ⇒ 3x – xy = –2y

=41-1C3-1 = 40C2 = 780. or 3x + 2y = xy

80. Let M be mid-point of BB′ and AM is ⊥ bisector of BB′
k −3k 1
(where A is the point of intersection of the given lines) 1
83. Area = 5 k 1 = 28
(x - 2) (x - 1) + (y - 2) (y - 3) = 0 2
−k 2 1
h+2 h+2  k +3 k +3 
⇒  − 2  − 1 +  − 2  − 3 = 0
 2  2   2  2  k − 5 −4 k 0
⇒ (h - 2)(h) + (k - 1)(k - 3) = 0 5 + k k − 2 0 = ± 56
−k 2 1
HINTS AND EXPLANATIONS

⇒ x2 - 2x + y2 - 4y + 3 = 0
⇒ (x - 1)2 + (y - 2)2 = 2. (k2 – 7k + 10) + 4k2 + 20k = ±56

81. Equation of angle bisector of the lines x – y + 1 = 0 and 7x – 5k2 + 13k + 10 = ±56

y – 5 = 0 is given by
5k 2 + 13k − 46 = 0 5 K 2 + 13K + 66 = 0
x − y +1 7x − y − 5
=± 5k 2 + 13k − 46 = 0
2 5 2
⇒ 5(x – y + 1) = 7x – y – 5
−13 ± 169 + 920
k=

10

and
= 2, –4.6 (reject)


5(x – y + 1) = –7x + y + 5
∴ 2 
x + 4y – 10 = 0  ⇒  x + 2y – 5 = 0 and For k = 2

12 x – 6y = 0  ⇒ 2 
x–y=0
A (2, –6)

Now equation of diagonals are
(x + 1) + 2(y + 2) = 0  ⇒  x + 2y + 5 = 0
(1)
8
=


and E
m

m = –2
2(x + 1) – ( y + 2) = 0  ⇒ 2x – y = 0
(2)
(5, 2) B
Cleary  ,  lies on (1)
1 8 D C (–2, 2)

 3 3 m=0
y −3
82. Any line passing through (2, 3) with slope in m =
x−2
mx – 2m = y – 3
Equation of AD,
mx – y = 2m – 3 x = 2
(i)
Coordinates and Straight Lines  10.55


Also equation BE,
AC = 4x + 3y = 6, BC º y = 3

y–2=
1  3 
(x – 5) \ C = −
, 3
2  4 
2y – 4 = x – 5

x – 2y – 1 = 0
(ii) 85. 3 2 \a= 6 3
.a = 27 3
Solving (i) and (ii), 2y = 1
4
1 3 2
y=
r=a´
× \r=6
2 2 3
⎛ 1⎞ g2 + f2 – c = 36 \ c = 25


Orthocentre is ⎜ 2, ⎟
⎝ 2⎠

Hence, the correct option is (C)
84. As C is the orthocenter of triangle A H B

A (0, 2)


H (0, 0)

B (4, 3) C (h, k)

HINTS AND EXPLANATIONS


This page is intentionally left blank.
CHAPTER
Circles
11
LEARNING OBJECTIVES
After reading this chapter, you will be able to:
 Learn the definition of a circle and standard equation of  
Know how to find the equation of a circle passing
a circle through three non-collinear points, intersection of a line
 Be familiar with the conditions and some special cases and a circle, contact of two circles, equation of the tan-
for an equation to represent a circle gent in slope form
 Understand parametric equations of a circle along with
position of a point with respect to a circle

CIRCLE CONDITIONS FOR AN EQUATION TO


A circle is the locus of a point which moves in a plane such REPRESENT A CIRCLE
that its distance from a fixed point always remains constant. A general equation of second degree
The fixed point is called the centre and the constant dis-
tance is called the radius of the circle. ax2 + 2hxy + by2 + 2gx + 2 fy + c = 0
in x, y represents a circle if
STANDARD EQUATION OF A CIRCLE 1. coefficient of x2 = coefficient of y2, i.e., a = b,
2. coefficient of xy is zero, i.e., h = 0.
The equation of a circle with the centre at (h, k) and
radius a, is To Find the Centre and Radius of a Circle whose
(x - h) + (y - k) = a
2 2 2 Equation is GIven

If the centre of the circle is at the origin and radius is a, SHORT-CUT METHOD
then the equation of circle is x2 + y2 = a2.
 Make the coefficients of x2 and y2 equal to 1 and right
hand side equal to zero.
GENERAL EQUATION OF A CIRCLE  The coordinates of centre will be (h, k), where

The general equation of a circle is of the form 1


h = − (coefficient of x )
2
x2 + y2 + 2gx + 2 fy + c = 0, (1)
and 1
k = − (coefficient of y )
where g, f and c are constants. 2
The coordinates of its centre are (-g, -f  ) and radius is
g2 + f 2 - c.  Radius = h2 + k 2 − constant term
11.2  Chapter 11

Nature of the Circle


1. If g2 + f  2 - c > 0, then the general eqn. (1) represents real
circle with centre (-g, -f ).
2. If g2 + f  2 - c = 0, then the general eqn. (1) represents
a circle whose centre is (-g, -f ) and radius is zero i.e.,
the circle coincides with the centre represented by a point
(-g, -f ). It is, therefore called a point circle.
3. If g2 + f 2 - c < 0, the radius of the circle is imaginary but
the centre is real. Such a circle is called a virtual circle or
imaginary circle as it is not possible to draw such a circle.
FIGURE 11.3
5. If the circle touches x-axis at origin then its equation is x2
EQUATION OF A CIRCLE IN SOME SPECIAL
+ (y ± k)2 = k2 ⇒ x2 + y2 ± 2ky = 0. (Two cases)
CASES
1. If the centre of the circle is (h, k) and it passes through
origin then its equation is (x - h)2 + (y - k)2 = h2 + k2
⇒  x2 + y2 - 2hx - 2ky = 0

FIGURE 11.4
6. If the circle touches y-axis at origin, the equation of circle
is (x ± h)2 + y2 = h2 ⇒ x2 + y2 ⇒ 2xh = 0. (Two cases)

FIGURE 11.1

2. If the circle touches x-axis then its equation is


(x ± h)2 + (y ± k)2 = k2. (Four cases)
3. If the circle touches y-axis then its equation is (x ± h)2 +
(y ± k)2 = h2. (Four cases)

FIGURE 11.5
7. If the circle passes through origin and cuts intercepts a
and b on the axes, the equation of circle is x2 + y2 - ax - by
= 0 and centre is C(a/2, b/2). (Four cases)

FIGURE 11.2
4. If the circle touches both the axes then its equation is
(x ± r)2 + (y ± r)2 = r2. (Four cases) FIGURE 11.6
Circles  11.3

SOLVED EXAMPLES
1. The tangent to the circle x2 + y2 = 9, which is parallel
to y-axis and does not lie in third quadrant, touches the
circle at the point
(A) (−3, 0) (B)  (3, 0)
(C) (0, 3) (D)  (0, −3)
Solution: (B)
Any line parallel to y-axis is x = k. | 12a + 5a − 60 | | 17a − 60 |
∴ =a or = a.
If it touches the circle x2 + y2 = 9, then ^ distance 12 + 5
2 2 13
from the centre (0, 0) of the circle to the line x = k,
or 17a − 60 = ±13a or a = 15, 2
must be equal to radius 3.
It is clear from the figure that a ≠ 15.
|0-k |
i.e., = 3 Þ k = ±3 \ a=2
1
\ k=3 \ The equation of the incircle is
(∵ line does not lie in the IIIrd quadrant) (x − 2)2 + (y − 2)2 = 22
\ The equation of the tangent line is x = 3. or x2 + y2 − 4x − 4y + 4 = 0
This meets the circle when 9 + y2 = 9  ⇒  y = 0. 4. The area of an equilateral triangle inscribed in the
\ Point of contact is (3, 0). ­circle x2 + y2 + 2gx + 2fy + c = 0 is
2. The equation of circle with origin as centre and pass- 3 3 2 3 3 2
(A) ( g + f 2 - c) (B)  ( g + f 2 - c)
ing through the vertices of an equilateral triangle 2 4
whose median is of length 3a is 3 3 2
(C) ( g + f 2 + c) (D)  none of these
(A) x2 + y2 = 9a2 (B)  x2 + y2 = 16a2 4
(C) x + y = 4a (D) 
2 2 2
x2 + y2 = a2 Solution: (B)
Solution: (C) Given Circle is
The centroid of an equilateral triangle is the centre x2 + y2 + 2gx + 2fy + c = 0 (1)
of its circum centre and the radius of the circle is the
­distance of any vertex from the centroid i.e., radius of Let C be its centre and PQR be an equilateral tri-
the circle angle inscribed in the circle, then C ≡ (−g, −f ) and
= distance of centroid from any vertex radius of the circle CQ = g 2 + f 2 - c .
2 2
= ( Median ) = (3a) = 2a
3 3
Hence, equation of circle whose centre is (0, 0)
and radius 2a is
(x − 0)2 + ( y − 0)2 = (2a)2 or x2 + y2 = 4a2
3 The equation of the circle inscribed in the triangle,
formed by the coordinate axes and the line 12x + 5y =
60, is given by
(A) x2 + y2 + 4x + 4y + 4 = 0
(B) x2 + y2 − 4x − 4y + 4 = 0
3
(C) x2 + y2 − 4x − 4y − 4 = 0 From ∆QLC , QL = CQ sin 60° = g2 + f 2 − c
(D) none of these 2

Solution: (B)
\ QR = 2QL = 3 ⋅ g 2 + f 2 − c
Let the radius of the circle be a. 3 3
Now, area of DPQR = × QR 2 = × 3( g 2 + f 2 - c)
Then the centre is C ≡ (a, a). 4 4
Also, the distance of C (a, a) from the line 12x + 3 3 2
= ( g + f 2 - c)
5y = 60 is a. 4
11.4  Chapter 11

5. A point moves so that the sum of the squares of its \ Circumradius = 2 ⋅ 3 = 6.


distances from the four sides of a square is constant. \ The equation of the circumcircle is
The locus of the point is
(x + 2)2 + ( y − 3)2 = (6)2
(A) a circle (B)  an ellipse
or x2 + y2 + 4x − 6y − 23 = 0
(C) a hyperbola (D)  none of these
Solution: (A) 7. The DPQR is inscribed in the circle x2 + y2 = 25. If Q
Take the centre of the square as origin and axes p­ arallel and R have coordinates (3, 4) and (− 4, 3) respectively,
to its sides. then ∠QPR is equal to
Let side of square be 2a. (A) p /2 (B)  p /3
(C) p /4 (D)  p /6
Solution: (C)
4 −3
Let m1 = slope of OQ = and m2 = slope of OR = .
3 4
p
As m1m2 = −1, ∠QOR = .
2
p
Thus, ÐQPR =
4

The equations of sides are


AD : x = a, BC : x = −a
AB : y = a, CD : y = −a
Let P (x, y) be any point on locus.
Distances of P from the sides of square are
x − a, x + a, y − a and y + a
By the given condition,
(x − a)2 + (x + a)2 + ( y − a)2 + ( y + a)2 = constant (∵ angle subtended at the centre of a circle is dou-
= 4c2 (say) ble the angle subtended in the alternate segment).
\ 2x2 + 2y2 = 4c2 − 4a2
or x2 + y2 = 2(c2 − a2), which is a circle. EQUATION OF A CIRCLE IN DIAMETER FORM
6. If the equation of the incircle of an equilateral triangle The equation of the circle drawn on the line segment join-
is x2 + y2 + 4x − 6y + 4 = 0, then the equation of the ing two given points A(x1, y1) and B(x2, y2) as diameter is
circumcircle of the triangle is
(A) x2 + y2 + 4x + 6y − 23 = 0
(B) x2 + y2 + 4x − 6y − 23 = 0
(C) x2 + y2 − 4x − 6y − 23 = 0
(D) none of these
Solution: (B)
Given equation of the incircle is FIGURE 11.7
x2 + y2 + 4x − 6y + 4 = 0
(x - x1) (x - x2) + (y - y1) (y - y2) = 0.
Its incentre is (−2, 3) and inradius = 4 + 9 - 4 = 3.
Since in an equilateral triangle, the incentre and æ x + x y + y2 ö
Its Centre = ç 1 2 , 1 and
the circumcentre coincide, è 2 2 ÷ø
\ Circumcentre ≡ (−2, 3) 2 2
Also, in an equilateral triangle, circumradius = 2 æ x - x ö æ y - y2 ö
Radius = ç 1 2 ÷ + ç 1 ÷
(inradius) è 2 ø è 2 ø
Circles  11.5

SOLVED EXAMPLE SOLVED EXAMPLES


8. Extremities of a diagonal of a rectangle are (0, 0) and 9. A variable circle passes through the point P(1, 2) and
(4, 3). The equations of the tangents to the circumcircle touches the x-axis. The locus of the other end of the
of the rectangle which are parallel to this diagonal are diameter through P is
(A) 16x + 8y ± 25 = 0 (B)  6x − 8y ± 25 = 0 (A) (x − 1)2 = 8y (B) (x + 1)2 = 8y
(C) 8x + 6y ± 25 = 0 (D)  none of these (C) ( y − 1)2 = 8x (D)  none of these
Solution: (B) Solution: (A)
Extremities of the diagonal OA of the rectangle are O The equation of any circle touching x-axis is of the form
(0, 0) and A(4, 3). Then OA is the diameter of the cir-
cumcircle, so equation of the circumcircle is (x − h)2 + ( y − k)2 = k2
x (x − 4) + y ( y − 3) = 0 i.e., x2 + y2 − 4x − 3y = 0 Let the coordinates of the other end of the diame-
æ 3ö æ5ö
2 2
ter through P be (a, b )
i.e., ( x - 2) 2 + ç y - ÷ = ç ÷ (1)
è 2ø è2ø
a +1 b +2
m = slope of OA = 3/4 (2) Then, = h and =k
\ Tangents parallel to the diagonal OA are 2 2
3 3 5 9 i.e., a = 2h − 1 and b = 2k − 2 (1)
y− = ( x − 2) ± 1+
2 4 2 16 Also, (h − 1) + (k − 2) = (radius) = k
2 2 2 2

That is 6x − 8y ± 25 = 0
2 2 2
æ a +1 ö æ b + 2 ö æ b +2ö
INTERCEPTS MADE BY A CIRCLE Þ ç - 1÷ + ç - 2÷ = ç ÷
è 2 ø è 2 ø è 2 ø
ON THE AXES
⇒ (a − 1)2 + (b − 2)2 = (b + 2)2
1. The length of the intercept made by the circle
x2 + y2 + 2gx + 2fc + c = 0 on ⇒ (a − 1)2 = 8b
x-axis = AB = 2 g - c 2
\  Locus of (a, b ) is (x − 1)2 = 8y

y-axis = CD = 2 f - c
2
10. A square is inscribed in the circle x2 + y2 − 2x + 4y + 3
2. Intercepts are always positive. = 0. Its sides are parallel to the coordinate axes. Then
3. If the circle touches x-axis then | AB | = 0 Thus, c = g2 one vertex of the square is
4. If the circle touches y-axis, then | CD | = 0 Thus, c = f 2
5. If the circle touches both the axes, then | AB | = 0 = | CD | (A) (1 + 2 , -2) (B) 
(1 - 2 , -2)
Thus, c = g2 = f 2
(C) (1, -2 + 2 ) (D)  none of these
Solution: (D)
The centre of the given circle is (1, −2). Since the sides
of the square inscribed in the circle are parallel to the
coordinate axes, so the x-coordinate of any vertex can-
not be equal to 1 and its y-coordinate cannot be equal
to −2. Hence none of the points given in (A), (B) and
(C) can be the vertex of the square.
FIGURE 11.8

QUICK TIPS
 The circle x + y + 2gx + 2fy + c = 0 cuts the x-axis in
2 2 PARAMETRIC EQUATIONS OF A CIRCLE
real and distinct points, touches or does not meet in real
(a) The parametric equations of a circle x2 + y2 = a2 are x =
points according as g2 > 0 = or < c.
a cosθ, y = a sinθ, 0 ≤ θ < 2π. θ is called parameter and
 Similarly, the circle x + y + 2gx + 2fy + c = 0 cuts the
2 2
the point P(a cosθ, a sinθ ) is called the point ‘θ ’ on the
y-axis in real and distinct points, touches or does not meet circle x2 + y2 = a2. Thus, the coordinates of any point on
in real points according as f2 >, = or < c.
the circle x2 + y2 = a2 may be taken as (a cosθ, a sin θ ).
11.6  Chapter 11

QUICK TIPS
Let S be a circle and P(x1, y1) and Q(x2, y2) be two points in
the plane of S, then they lie

 on the same side of S iff S1 and S2 have same sign and


 on the opposite sides of S iff S and S have opposite
1 2
signs

SOLVED EXAMPLES
FIGURE 11.9
11. If (a, b ) is a point on the chord PQ of the circle x2 + y2
(b) The parametric equations of a circle (x − h)2 + ( y − k)2 = a2 = 19, where the coordinates of P and Q are (3, - 4) and
are x = h + a cosθ, y = k + a sinθ, 0 ≤ θ < 2π. is called the (4, 3) respectively, then
point ‘θ’ on this circle. Thus the coordinates of any point (A) a ∈ [3, 4], b ∈ [- 4, 3]
on this circle may be taken as (h + a cosθ, k + a sinθ). (B) a ∈ [-4, 3], b ∈ [3, 4]
(C) a ∈ [3, 3], b ∈ [-4, 4]
(D) none of these
Solution: (A)
Clearly, the point (α, β ) is either an internal point or
one of the end points of the line segment joining P(3,
-4) and Q (4, 3).
∴ 3 ≤ α ≤ 4 and - 4 ≤ β ≤ 3
12. If the point (2, k) lies outside the circles
FIGURE 11.10 x2 + y2 + x - 2y - 14 = 0 and x2 + y2 = 13, then
(A) k∈ (-3, -2) ∪ (3, 4)
(c) The parametric coordinates of any point on the circle x2 + (B) k∈ (-3, 4)
y2 + 2gx + 2fy + c = 0 are (C) k∈ (-∞, -3) ∪ (4, ∞)
x = - g + ( g 2 + f 2 - c) cos q and (D) k∈ (-∞, -2) ∪ (3, ∞)
Solution: (C)
y = - f + ( g 2 + f 2 - c) sin q (0 £ q < 2p )
Since the point (2, k) lies outside the circle
x2 + y2 + x - 2y - 14 = 0
∴ 4 + k2 + 2 - 2k - 14 > 0  or  k2 - 2k - 8 > 0
POSITION OF A POINT WITH RESPECT
or (k + 2) (k - 4) > 0
TO A CIRCLE
or  k∈ (-∞, -2) ∪ (4, ∞)(1)
Let S ≡ x2 + y2 + 2gx + 2 f y + c = 0, be a circle and P(x1, y1) Also, the point (2, k) lies outside the circle
be a point in the plane of S, then S1 ≡ x12 + y12 + 2 gx1 + 2 fy1
x2 + y2 = 13.
+ c. The point P(x1, y1) lies outside, on or inside the circle
S according as S1 > , = or < 0, respectively. ∴ 4 + k2 - 13 > 0 or k2 - 9 > 0
or (k - 3) (k + 3) > 0
or k∈ (- ∞, - 3) ∪ (3, ∞)(2)
The common solution of (1) and (2) is given by,
k∈ (-∞, -3) ∪ (4, ∞)
13. If the point (k + 1, k) lies inside the region bounded by
the curve and y-axis, then k belongs to the interval

x = 25 - y .
2

(A) (-1, 3) (B)  (-4, 3)


FIGURE 11.11 (C) (-∞, -4) ∪ (3, ∞) (D)  none of these
Circles  11.7

Solution: (A)  From given three points taking any two as extremities of
Since the point (k + 1, k) lies inside the region bounded diameter of a circle S = 0 and equation of straight line
passing through these two points is L = 0. then required
by x = 25 - y 2 and y-axis,
equation of circle is S + λL = 0, where λ is a parame-
∴ (k + 1)2 + k2 - 25 < 0 ter, which can be found out by putting third point in the
and k+1>0 equation.
 If the two lines a x + b y + c = 0 and a x + b y + c =
1 1 1 2 2 2
0 meet the coordinate axes in four distinct points, then
those points are concyclic if a1a2 = b1b2.
Also, the equation of the circle passing through those
concyclic points is (a1x + b1y + c1) (a2x + b2y + c2)
- (a1b2 + a2b1) xy = 0.
 The equation of the circumcircle of the triangle formed by

the line ax + by + c = 0 with the coordinate axes is ab (x2


+ y2) + c (bx + ay) = 0.

⇒ 2k2 + 2k - 24 < 0 and k > -1


⇒  k2 + k - 12 < 0 and k > -1
⇒ (k + 4) (k - 3) < 0 and k > -1 SOLVED EXAMPLES
⇒  -4 < k < 3 and k > -1
⇒  -1 < k < 3 14. If a circle passes through the points of intersection of
the coordinate axes with the lines lx - y + 1 = 0 and
x - 2y + 3 = 0, then the value of λ is
CIRCLE THROUGH THREE POINTS
(A) 2 (B)  1
We can find a unique circle through three non-collinear (C) -1 (D)  -2
points. To find the unique equation of circle, we can follow
Solution: (A)
the following method.
Let the lines cuts the x-axis at A and B, then
Step I: Assume the general equation of the circle as
S : x2 + y2 + 2gx + 2fy + c = 0 1
OA = - and OB = -3 ×
l
Step II: The coordinates of three points P(x1, y1), Q(x2, y2)
and R(x3, y3) (if they lie on the circle) will satisfy Also, if the lines cut the y-axis at C and D, then
the equation of the circle and thus we shall get
3
three simultaneous equations in g, f and c such that OC = 1 and OB =
2
S2 : x12 + y12 + 2 gx1 + 2 fy1 + c = 0 Now if the circle passes through A, B, C and D then
S2 : x + y + 2 gx2 + 2 fy2 + c = 0
2 2
2 2 æ 1ö 3
OA ´ OB = OC ´ OD Þ ç - ÷ ( -3) = 1´
S3 : x32 + y32 + 2 gx3 + 2 fy3 + c = 0 è l ø 2
Step III: Solve the above three simultaneous equations in ⇒  λ = 2
three variables to obtain the values of g, f and c.
Step IV: Substitute the values of g, f and c obtained from 15. If the lines a1x + b1 y + c1 = 0 and a2x + b2y + c2 = 0 cut
step III in the equation assumed in step I to get the the coordinate axes in concyclic points, then
desired equation of the circle. (A) a1a2 = b1b2
(B) a1b1 = a2b2
QUICK TIPS (C) a1b2 = a2b1
The equation of the circle through three non-collinear
 (D) none of these
points A(x1, y1), B(x2, y2) and C(x3, y3) is Solution: (A)

x2 + y2 x y 1
The line a1x + b1y + c1 = 0 cuts the coordinate axes at
A(-c1/a1, 0) and B (0, -c1/b1) and the line a2x + b2y +
x12 + y12 x1 y1 1
=0 c2 = 0 cuts the axes at C (-c2/a2, 0) and D (0, -c2/b2).
x22 + y22 x2 y2 1
So, AC and BD are chords along x-axis and y-axis
x32 + y32 x3 y3 1
respectively, intersecting at origin O.
11.8  Chapter 11

Since A, B, C, D are concyclic, therefore 1. l intersects S in two distinct points iff d < a.
OA.OC = OB.OD 2. l intersects S in one and only point iff d = a, i.e., the line l
touches the circle if perpendicular distance from the cen-
 −c1   − c2   − c1   − c2  tre to the line l must be equal to radius of the circle.
⇒  a  ⋅  a  =  b  ⋅  b 
1 2 1 2 3. l does not intersect S iff d > a.
or a1a2 = b1b2
LENGTH OF INTERCEPT MADE BY A CIRCLE
16. Two distinct chords drawn from the point (p, q) on the ON A LINE
circle x2 + y2 = px + q y, where pq ≠ 0, are bisected by
the x-axis. Then If a line l meets a circle S, with centre C and radius a, in
two distinct points and if d is the perpendicular distance
(A) | p | = | q | (B)  p2 = 8q2
of centre C from the line l, then the length of the intercept
(C) p < 8q (D) 
2 2
p2 > 8q2
made by the circle on the line = | AB | = 2 a 2 − d 2 .
Solution: (D)
Given circle is x2 + y2 = px + qy.
 p q
Since the centre of the circle is,  ,  , so (p, q) and
 2 2
(0, 0) are the end points of a diameter. As the two chords
are bisected by x-axis, the chords will cut the circle at
the points (x1, -q) and (x2, -q), where x1, x2 are real.
FIGURE 11.13

QUICK TIPS
 Note: If the points of intersection of a line l and a circle S
are known, then the distance between these points is the
required length of intercept and there is no need of using
the above formula.
 The length of the intercept cut off from the line y = mx + c
a2(1 + m2 ) - c 2
by the circle x2 + y2 = a2 is 2 .
1 + m2
The equation of the line joining these points is y = -q.  If a2(1 + m2) - c2 > 0, line will meet the circle at two real
Solving y = -q and x2 + y2 = px + qy, we get and different points.
 If c = a (1 + m ), line will touch the circle.
2 2 2
x2 - px + 2q2 = 0
 If a (1 + m ) - c < 0, line will meet the circle at two
2 2 2

The roots of this equation are x1 and x2. Since the imaginary points.
roots are real and distinct, ∴ discriminent > 0
i.e., p2 - 8q2 > 0  or  p2 > 8q2 SOLVED EXAMPLES
17. If a chord of the circle x2 + y2 = 32 makes equal inter-
INTERSECTION OF A LINE AND A CIRCLE cepts of length l on the coordinate axes, then
(A) | l | < 8 (B)  | l | < 16
(C) | l | > 8 (D)  none of these
Solution: (A)
Since the chord makes equal intercepts of length l on
the coordinate axes, so its equation can be written in
the form x ± y = ±l.
Since the chord intersects the given circle at two
distinct points, therefore, the length of the ⊥ from the
FIGURE 11.12
centre (0, 0) of the given circle to the chord must be
Let S be a circle with centre C and radius a. Let l be any less than the radius
line in the plane of the circle and d be the perpendicular ±l
distance from C to the line l, then i.e., < 32 Þ l 2 < 64 Þ | l |< 8
2
Circles  11.9

18. The equation of the circle whose centre is (3, -1) and
which cuts off an intercept of length 6 from the line
2x - 5y + 18 = 0, is
(A) x2 + y2 - 6x + 2y + 28 = 0
(B) x2 + y2 + 6x + 2y - 28 = 0
(C) x2 + y2 - 6x - 2y + 28 = 0
(D) x2 + y2 - 6x + 2y - 28 = 0
Solution: (D)
Let C be the centre of the circle, then C ≡ (3, -1). ∴  Equation of such circles is
Equation of line AB is 2x - 5y + 18 = 0 and AB = 6 2
∴  AL = 3 æ 9ö
ç x - 2 ÷ + ( y - k)
2

è ø
2
æ9 ö 25
= ç - 2 ÷ + ( k - 0) 2 = + k2
è2 ø 4
or  x2 + y2 - 9x - 2ky + 14 = 0
20. The line y = mx + c intersects the circle x2 + y2 = r2 at
the two real distinct points if
(A) -r 1 + m 2 < c < r 1 + m 2
(B) -c 1 - m < r < c 1 + m
2 2

(C) -r 1 - m 2 < c < r 1 + m 2


(D) none of these
Solution: (A)
CL = length of the ⊥ from C on AB Given line is y = mx + c(1)
and the given circle is x2 + y2 = r2 (2)
| 2 ´ 3 - 5( -1) + 18 | Solving (1) and (2), we get
= = 29
( 2) 2 + ( -5) 2 (1 + m2)x2 + 2mcx + c2 - r2 = 0 (3)

For two real distinct points of intersection, both
the roots of (3) must be real and distinct.
∴  radius of the circle AC = AL2 + CL2 ∴ 4m2c2 - 4(1 + m2) (c2 - r2) > 0
= 32 + 19 = 38 ⇒  c2 < r2 (1 + m2)

⇒  − r 1 + m 2 < c < r 1 + m 2
Thus, equation of the required circle is
(x - 3)2 + (y + 1)2 = 38 THE LEAST AND GREATEST DISTANCE OF A
or x2 + y2 - 6x + 2y - 28 = 0 POINT FROM A CIRCLE
Let S = 0 be a circle and A(x1, y1) be a point. If the diameter
19. Circles are drawn through the point (2, 0) to cut inter-
of the circle through A is passing through the circle at P and
cept of length 5 units on the x-axis. If their centres lie
Q, then AP = |AC - r| = least distance; AQ = AC + r = greatest
in the first quadrant, then their equation is
distance where ‘r’ is the radius and C is the centre of the circle.
(A) x2 + y2 - 9x + 2ky + 14 = 0
(B) 3x2 + 3y2 + 27x - 2ky + 42 = 0
(C) x2 + y2 - 9x - 2ky + 14 = 0
(D) x2 + y2 - 2kx - 9y + 14 = 0

Solution: (C)
It is clear from the figure that the coordinates of centre
9 
of such circles are  , k  .
2  FIGURE 11.14
11.10  Chapter 11

CONTACT OF TWO CIRCLES 5. One circle is contained in the other if AB < |r1 - r2|.
The two circles having centres at A(x1, y1) and B(x2, y2) and
radii r1 and r2 respectively will
1. Intersect in two real distinct points if and only if | r1 - r2 |
< AB < r1 + r2

FIGURE 11.19

SOLVED EXAMPLES
21. The number of common tangents to the circles x2 + y2
FIGURE 11.15
= 4 and x2 + y2 - 8x + 12 = 0 is
2. Touch each other externally AB = r1 + r2 and their point of (A) 1 (B)  2
contact C is given by, (C) 3 (D)  4
Solution: (C)
The equations of the circles are
x2 + y 2 = 4 (1)
and x + y - 8x + 12 = 0
2 2
(2)
Centre of (1) is C1 ≡ (0, 0) and radius r1 = 2
FIGURE 11.16
Centre of (2) is C2 ≡ (4, 0) and radius r2 = 2
d = distance between centres = C1C2 = 4.
ærx +r x ry +r y ö Since C1C2 = r1 + r2,  ∴  the two circles touch each
C ºç 1 2 2 1, 1 2 2 1÷ other externally. Hence 3 common tangents can be
è r1 + r2 r1 + r2 ø
drawn to the two circles.
3. Touch each other internally if AB = | r1 - r2  |, and their 22. The equation of a circle of radius 2 touching the cir-
point of contact C is given by,
cles x2 + y2 - 4 | x | = 0 is
(A) x 2 + y 2 + 2 3 y + 2 = 0
(B) x 2 + y 2 + 4 3 y + 8 = 0
(C) x 2 + y 2 - 4 3 y + 8 = 0
(D) none of these
Solution: (B, C)
The given circles are

FIGURE 11.17
x2 + y2 - 4x = 0, x > 0
i.e., (x - 2)2 + y2 = 22, x > 0
ærx -r x r y -r y ö and x2 + y2 + 4x = 0, x < 0
C ºç 1 2 2 1, 1 2 2 1÷
è r1 - r2 r1 - r2 ø i.e., (x + 2)2 + y2 = 22, x < 0
Clearly, from the figure, the centres of the required cir-
4. One circle lies outside the other if AB > r1 + r2. cles are at (0, 12 ) and (0, 12 ).
∴  Equations of the required circles are
( x - 0) 2 + ( y ∓ 12 ) 2 = 22

i.e., x 2 + y 2 + 2 12 y + 8 = 0

and x 2 + y 2 - 2 12 y + 8 = 0
FIGURE 11.18
Circles  11.11

EQUATION OF THE TANGENT IN SLOPE FORM


The equation of a tangent of slope m to the circle x2 + y2 =
a2 is y = mx ± a 1 + m 2 .
The coordinates of the point of contact are
æ am a ö
çç ± ,∓ ÷÷
è 1+ m 1 + m2 ø
2

CONDITION OF TANGENCY
The straight line y = mx + c will be a tangent to the circle

x2 + y2 = a2 if c = ± a 1 + m 2
23. The coordinates of the point at which the circles x +2

y2 - 4x - 2y - 4 = 0 and x2 + y2 - 12x - 8y -36 = 0 touch QUICK TIPS


each other, are  A line will touch a circle if and only if the length of the
(A) (3, -2) (B)  (-2, 3) perpendicular from the centre of the circle to the line is
(C) (3, 2) (D)  none of these equal to the radius of the circle.
 The condition that the line lx + my + n = 0 touches the
Solution: (D)
circle x2 + y2 + 2gx + 2fy + c = 0 is
Given circles are
(lg + mf - n)2 = (l2 + m2)(g2 + f  2 - c)
x2 + y2 - 4x - 2y - 4 = 0 (1)
and x2 + y2 - 12x - 8y - 36 = 0 (2)  Equation of tangent to the circle x2 + y 2 + 2gx
+ 2fy + c = 0 in terms of slope is y = mx + mg - f
Centre of circle (1) is C1 ≡ (2, 1) ± ( g2 + f 2 − c ) (1 + m2 ).
and radius = r1 = 4 + 1 + 4 = 3  If the line lx + my + n = 0 is a tangent to the circle
Centre of circle (2) is C2 ≡ (6, 4) (x - h)2 + (y - k)2 = a2, then (hl + km + n)2 = a2(l2 + m2).
and radius = r2 = 36 + 16 + 36 = 88
Also, d = distance between C1 and C2
= C1C2 = 16 + 9 = 5 TANGENTS FROM A POINT OUTSIDE THE

Since d ≠ r1 ± r2 CIRCLE
∴  the two circles do not touch each other.
WORKING RULE
 Let the point be (x , y ).
1 1
TANGENT TO A CIRCLE AT A GIVEN POINT Write the equation of a straight line passing through the
1. Equation of the tangent to the circle x2 + y2 = a2 at the point (x1, y1) and having slope m i.e.,
point (x1, y1) on it is xx1 + yy1 = a2. ( y - y1) = m (x - x1)(1)
2. Equation of the tangent to the circle  Find the length of the perpendicular from the centre of
the circle to the line (1) and equate it to the radius of the
x + y + 2gx + 2fy + c = 0
2 2
circle. Call this equation as (2).

at the point (x1, y1) on it is  Obtain the value of m from the Eq. (2).

 Substitute this value of m in Eq. (1) to obtain the required


xx1 + yy1 + g (x + x1) + f (y + y1) + c = 0
equation of tangent.
3. Equation of the tangent to the circle x2 + y2 = a2 at the
point (a cosθ, a sinθ) on it is
x cosθ + y sinθ = a SOLVED EXAMPLES

(Parametric form of equation of tangent)
24. The equation of the circle which has a tangent 2x - y
Notation: The equation of the tangent at the point - 1 = 0 at (3, 5) on it and with the centre on x + y = 5,
(x1, y1) on the circle S = 0 is T = 0. is
11.12  Chapter 11

(A) x2 + y2 + 6x - 16y + 28 = 0
| T | = PT = x12 + y12 + 2 gx, + 2 fy1 + c = S1
(B) x2 + y2 - 6x + 16y - 28 = 0
(C) x2 + y2 + 6x + 6y - 28 = 0
(D) x2 + y2 - 6x - 6y - 28 = 0 QUICK TIPS
Solution: (A)  While calculating the length of tangent using the above
formula, it must be noted that the coefficients of x2 and y2
Clearly, the centre of the circle lies on the line through
must be unity.
the point (3, 5) and ⊥ to the tangent 2x - y - 1 = 0.
 If the point lies inside the circle, then S < 0 and |T | is
The equation of such line is 1
imaginary therefore we cannot have any tangent from a
-1
( y - 5) = ( x - 3) i.e., x + 2 y = 13 (1) point inside the circle.
2  If the point lies on the circle, then S = 0 and hence |T |
1
Also, it is given that centre lies on the line = 0. So, if the point lies on the circle, then we have only
x + y = 5 (2) one tangent having zero length.
 If the point lies outside the circle, then S > 0 and |T | is
Solving (1) and (2), we obtain the coordinates of the 1
finite.
centre of the circle as C ≡ (-3, 8).
Also, radius of the circle = 36 + 9 = 45.
∴  Equation of the circle is ERROR CHECK
( x + 3) + ( y - 8) = ( 45 )
2 2 2
No tangent can be drawn to a circle from a point lying within
That is, x + y2 + 6x - 16y + 28 = 0
2 the circle.

25. The tangent to the circle x2 + y2 = 5 at the point (1, -2)


also touches the circle x2 + y2 - 8x + 6y + 20 = 0. Then
its point of contact is SOLVED EXAMPLE
(A) (3, -1) (b)  (-3, 0)
(C) (-1, -1) (d)  (-2, 1) 26. If the distances from the origin of the centres of the
three circles x2 + y2 + 2aix = a2(i = 1, 2, 3) are in G.P.,
Solution: (A) then the lengths of the tangents drawn to them from
Equation of tangent to the circle any point on the circles x2 + y2 = a2 are in
x2 + y2 = 5 at (1, - 2) is x - 2y - 5 = 0 (1) (A) A.P. (B)  G.P.
Let this line touches the circle (C) H.P. (D)  none of these
x2 + y2 - 8x + 6y + 20 = 0 at (x1, y1) Solution: (B)
∴  Equation of tangent at (x1, y1) is The centres of the three given circles are (-a1, 0),
xx1 + yy1 - 4(x + x1) + 3( y + y1) + 20 = 0 (-a2, 0) and (-a3, 0).
The distances of the three points from the origin are
or x(x1 - 4) + y( y1 + 3) - 4x1 + 3y1 + 20 = 0 (2)
a1, a2 and a3.
Now (1) and (2) represent the same line Given: a1, a2 and a3 are in G.P.
x - 4 y1 + 3 -4 x1 + 3 y1 + 20 i.e., a 22 = a1a 3 (1)
\ 1 = =
1 -2 -5 Now, coordinates of any point on the circle x2 + y2 = a2
⇒  -2x1 + 8 = y1 + 3 or 2x1 + y1 - 5 = 0 are (a cosθ, a sinθ).
Only the point (3, -1) satisfies it. Hence, the point of ∴  The lengths of the tangents drawn from the point
contact is (3, -1). (a cosθ, a sinθ) to the three given circles are
2a1a cos q , 2a 2 a cos q and 2a 3 a cos q
LENGTH OF THE TANGENT FROM A POINT TO
which, in view of (1), are in G.P.
A CIRCLE
The length of the tangent that can be drawn from the point NORMAL TO THE CIRCLE AT A GIVEN POINT
P(x1, y1) to the circle S = 0 is S1 , where the coefficients
of x2 and y2 in the equation of the circle are unity. Thus, the The normal of a circle at any point is a striaght line per-
length of the tangent from the point P(x1, y1) to circle x2 + pendicular to the tangent at the point and always passes
y2 + 2gx + 2f y + c = 0 is given by through the centre of the circle.
Circles  11.13

1. Equation of normal: COMMON TANGENTS TO TWO CIRCLES


•  The equation of normal to the circle x2 + y2 + 2gx + 2fy
+ c = 0 at any point (x1, y1) is y - y1 Direct Common Tangents
The direct common tangents to the two circles meet at a
y +f x - x1 y - y1 point (say P) which lies on the line joining the centres C1
= 1 ( x - x1 ) or = .
x1 + g x1 + g y1 + f and C2 of the two circles and divide C1C2 externally in the
ratio of their radii say (r1 and r2)
•  The equation of normal to the circle x2 + y2 = a2 at any

x y
point (x1, y1) is xy1 - x1y = 0 or = .
x1 y1

FIGURE 11.22

SHORT-CUT METHOD
FIGURE 11.20
 Find the coordinates of centres C1, C2 and radii r1, r2 of
2. Parametric form:  Since parametric coordinates of any two given circles.
point on the circle x2 + y2 = a2 is (a cosθ, a sinθ).  Find the coordinates of the point P dividing C C in the
1 2
∴  equation of normal at (a cosθ, a sinθ) is ratio r1 : r2 externally. Let P ≡ (h, k).
x y  Write the equation of any line through the point P(h, k), i.e.,
= = or y = x tanθ or y = mx,
cos q sin q ( y - k) = m (x - h)(1)
where m = tanθ, which is slope form of normal.
 Find the two values of m, using the fact that the length of

perpendicular on line (1) from the centre C1 of one circle


is equal to its radius r1.
PAIR OF TANGENTS  Substituting these values of m in equation (1), the equa-

tions of two direct common tangents are obtained.


The equation of the pair of tangents drawn from the point
P(x1, y1) to the circle
Transverse Common Tangents
The transverse common tangents to the two circles inter-
sect at a point (say P) which lies on the line joining the cen-
tres C1 and C2 of the two circles and divide C1C2 internally
in the ratio of their radii r1 and r2.

SHORT-CUT METHOD
FIGURE 11.21
 Find the coordinates of centres C1, C2 and radii r1, r2 of
S = 0 is SS1 = T2, where two given circles.
 Find the coordinates of the point P dividing C C , in the
S : x2 + y2 + 2gx + 2fy + c, 1 2
ratio r1 : r2 internally. Let P ≡ (h, k).
S1 = x12 + y12 + 2 gx1 + 2 f y1 + c  Write the equation of any line through the point P (h, k)

( y - k) = m (x - h)(1)
and T : xx1 + yy1 + g(x + x1) + f (y + y1) + c.
 Find the two values of m, using the fact that the length of
perpendicular on (1) from the centre C1 of one circle is
QUICK TIPS equal to its radius r1.
The pair of tangents from (0, 0) to the circle x2 + y2 + 2gx +  Substituting these values of m in eqn. (1), the equations

2fy + c = 0 are at right angles if g2 + f  2 = 2c. of two transverse common tangents are obtained.
11.14  Chapter 11

QUICK TIPS
Two or More Circles
in a plane

Direct common 0 1 2 3 2
Tangents
Transverse 0 0 0 0 2
­common Tangents

Solution: (B)
SOLVED EXAMPLES Clearly, from the figure, the radius of the smallest cir-
26. If the two circles x + y = 4 and x + y - 24x - 10y +
2 2 2 2 cle touching the given circles is
a2 = 0, a ∈Ι, have exactly two common tangents, then
the number of possible values of a is
(A) 11 (B)  13
(C) 0 (D)  2
Solution: (B)
The equations of the circles are
x2 + y2 = 4 (1)
and x2 + y2 - 24x - 10y + a2 = 0 (2)
Centre of (1) is C1 ≡ (0, 0) and radius r1 = 2
Centre of (2) is C2 ≡ (12, 5) and radius r2 = 169 - a 2
d = distance between centres = C1C2
= 4 2 + 4 2 - 4 i.e., 4 2 - 4
= 144 + 25 = 13
If the two circles have exactly two common tangents, 28. Locus of the centre of a circle of radius 4 which
then touches the circle x2 + y2 - 4x + 2y - 4 = 0 externally is
169 - a2 > 0 and r1 + r2 > d (A) x2 + y2 - 4x + 2y - 44 = 0
(B) x2 + y2 + 4x + 2y - 44 = 0
⇒ ( a − 13) ( a + 13) < 0 and 2 + 169 − a 2 > 13 (C) x2 + y2 - 4x - 2y - 44 = 0
⇒ -13 < a < 13 and 169 - a2 > 121 (D) none of these
⇒ -13 < a < 13 and a2 - 48 < 0 Solution: (A)
⇒ −13 < a < 13 and − 48 < a < 48 Let the centre of the circle S1 be C1(x1, y1)
Its radius = r1 = 4.
⇒ − 48 < a < 48
Given circle is S2 ≡ x2 + y2 - 4x + 2y - 4 = 0
Since a is an integer, Its centre is C2(2, -1) and radius
∴  a = -6, -5, -4, ... , 4, 5, 6
∴  The number of possible values of a is 13. = r2 = 4 + 1 + 4 = 3
Also, d = distance between the centres.
27. If the equations of four circles are (x ± 4)2 + ( y ± 4)2
= 42, then the radius of the smallest circle touching all = ( x1 - 2) 2 + ( y1 + 1) 2
the four circles is Since the two circles touch each other externally,
(A) 4( 2 + 1) (B)  4( 2 - 1) ∴ d = r1 + r2
(C) 2( 2 - 1) (D)  none of these ⇒ ( x1 − 2) 2 + ( y1 + 1) 2 = 4 + 3
Circles  11.15

⇒ x12 + y12 − 4 x1 + 2 y1 + 5 = 49 That is for a number of secants


PA . PB = PA1 . PB1 = PA2 . PB2 = ... = PT 2 = S1
∴ locus of (x1, y1) is
x2 + y2 - 4x + 2y - 44 = 0 where S1 = x12 + y12 + 2 gx1 + 2 f y1 + c

29. The number of common tangents to the circles x2 + y2


= 4 and x2 + y2 - 6x - 8y - 24 = 0 is QUICK TIPS
(A) 0 (B)  1  If the two lines a1x + b1x + c1 = 0 and a2x + b2x + c2 =
(C) 3 (D)  4 0 meet the coordinate axes in four distinct points, then
these points are concyclic if a1a2 = b1b2.
Solution: (B)
Also, the equation of the circle passing through these
Given circles are x2 + y2 - 4 = 0 (1) concyclic points is (a1x + b1y + c1)(a2x + b2y + c2) - (a1b2
and x + y - 6x - 8y - 24 = 0
2 2
(2) + a2b1)xy = 0.
 The equation of the circumcircle of the triangle formed by
Centre of circle (1) is C1 ≡ (0, 0) and radius r1 = 2
the line ax + by + c = 0 with the coordinate axes is ab(x2
Centre of circle (2) is C2 ≡ (3, 4) and radius r2 = 7
+ y2) + c(bx + ay) = 0
Also d = distance between the centres = C1C2 = 5
Since d = r2 - r1, therefore the given circles touch
internally, as such they can have just one common tan- DIRECTOR CIRCLE
gent at the point of contact.
The locus of the point of intersection of two perpendicular
30. The number of tangents to the circle tangents to a circle is called the Director circle.
x2 + y2 - 8x - 6y + 9 = 0 Let the circle be x2 + y2 = a2, then equation of director
which pass through the point (3, -2) is circle is x 2 + y 2 = 2a 2 = ( 2a ) 2 .
Clearly, director circle is a concentric circle whose
(A) 2 (B)  1
radius is 2 times the radius of the given circle.
(C) 0 (D)  none of these
Solution: (A)
Let S ≡ x2 + y2 - 8x - 6y + 9 = 0.
Now S for (3, -2) = 9 + 4 - 24 + 12 + 9 > 0,
∴  the point (3, -2) lies outside the circle.
∴  Two tangents can be drawn to the circle from the
point (3, -2).

POWER OF A POINT WITH RESPECT


TO A CIRCLE FIGURE 11.24

If from a point P(x1, y1), inside or outside the circle a secant REMEMBER
be drawn intersecting the circle in two points A and B then Director circle of circle x2 + y2 + 2gx + 2fy + c = 0 is x2 + y2
PA . PB = constant. The product PA . PB is called power of + 2gx + 2fy + 2c - g2 - f  2 = 0.
the point P(x1, y1) w.r.t. the circle

S ≡ x2 + y2 + 2gx + 2fy + c = 0 SOLVED EXAMPLE


31. The coordinates of a point on the line y = 2 from which
the tangents drawn to the circle x2 + y2 = 25 are perpen-
dicular, are
(A) ( 46 , 2) (B)  ( - 46 , 2)
( - 37 , 2)
(C) ( 37 , 2) (D) 
Solution: (A, B)
Let the point on the given line be (x1, 2).
FIGURE 11.23 Since the tangents drawn from (x1, 2) to the given cir-
11.16  Chapter 11

cle are at right angles, so the point (x1, 2) must also lie Centre of circle is C (-2, 1).
on the director circle whose equation is Draw CM ⊥ PQ, then M is the mid point of PQ.
x2 + y2 = 2.25  i.e., x2 + y2 = 50 Equation of any line ⊥ to PQ is x + y + k = 0
∴ x 1 + 4 = 50  ⇒  x1 = ± 46
2 If it passes through C(-2, 1) then
-2 + 1 + k = 0  or  k = 1
So, the points are ( 46 , 2) and ( − 46 , 2). ∴  Equation of CM is x + y + 1 = 0. (2)
3 1
Solving (1) and (2), we obtain x = − and y = .
2 2
EQUATION OF CHORD OF CONTACT æ -3 1 ö
∴  Coordinates of M are ç , ÷ .
The chord joining the points of contact of the two tangents è 2 2ø
to a conic drawn from a given point outside it, is called the
chord of contact of tangents EQUATION OF CHORD IF ITS MID POINT
IS KNOWN
1. Equation of chord of contact
The equation of the chord of contact of tangents drawn from The equation of the chord of the circle S ≡ x2 + y2 + 2gx +
the point (x1 y1) to the circle x2 + y2 = a2 is xx1 + yy1 = a2. 2fy + c = 0 bisected at the point (x1, y1) is given by T = S1.
2. The equation of chord of contact of tangent drawn from the That is xx1 + yy1 + g(x + x1) + f (y + y1) + c
point (x1, y1) to the circle x2 + y2 + 2gx + 2fy - c = 0 is T = 0. = x12 + y12 + 2 gx1 + 2 fy1 + c
⇒ xx1 + yy1 + g(x + x1) + f (y + y1) + c = 0

QUICK TIPS SOLVED EXAMPLES


 It is clear from above that the equation to the chord of
33. The locus of the mid point of the chord of the circle x2
contact coincides with the equation of the tangent, if the
point (x1, y1) lies on the circle.
+ y2 - 2x - 2y - 2 = 0 which makes an angle of 120º at
the centre is
 The length of chord of contact = 2 r − p , p being
2 2

length of perpendicular from centre to the chord. (A) x2 + y2 - 2x - 2y + 1 = 0


a( x12 + y12 − a2 )3/2 (B) x2 + y2 + x + y - 1 = 0
 Area of ∆APQ is given by .
x12 + y12 (C) x2 + y2 - 2x - 2y - 1 = 0
(D) none of these
Solution: (A)
Given equation of circle is
SOLVED EXAMPLE
x2 + y2 - 2x - 2y - 2 = 0
32. The coordinates of the middle point of the chord which Let mid point of chord AB be (h, k)
the circle x2 + y2 + 4x - 2y - 3 = 0 cuts off on the line y Its centre is (1, 1) and radius = 1 + 1 + 2 = 2. = OB.
= x + 2, are
æ -3 1 ö æ3 1ö
ç2,2÷
(A) ç , ÷ (B) 
è 2 2ø è ø
æ -3 -1 ö æ 3 -1 ö
(C) ç , ÷ (D) 
ç2, 2 ÷
è 2 2 ø è ø
Solution: (A)
Equation of chord PQ is y = x + 2
In DOPB, ∠OBP = 30º.
∴ sin 30º = OP/2 or OP = 1
Since, OP = 1  ⇒ (h - 1)2 + (k - 1)2 = 1
or h2 + k2 - 2h - 2k + 1 = 0
∴  Locus of mid point of chord is
x2 + y2 - 2x - 2y + 1 = 0
34. The locus of the centres of circles passing through the
origin and cutting the circle x2 + y2 + 6x - 4y + 2 = 0
or x - y + 2 = 0 (1) orthogonally is
Circles  11.17

(A) 2x - 3y + 1 = 0 (B)  2x + 3y +1 = 0
(C) 3x - 2y + 1 = 0 (D)  none of these
SOLVED EXAMPLE
Solution: (C) 35. If the circle x2 + y2 + 6x + 8y + a = 0 bisects the circum-
Let the equation of one of the circles be ference of the circle x2 + y2 + 2x - 6y - b = 0, then a +
b is equal to
x2 + y2 + 2gx + 2f y + c = 0
(A) 38 (B)  -38
Since it passes through origin,
(C) 42 (D)  none of these
∴  c = 0.
So, the equation becomes Solution: (B)

x2 + y2 + 2gx + 2fy = 0 Given circles are


Since it cuts the circle x2 + y2 + 6x - 4y + 2 = 0 S1: x2 + y2 + 6x + 8y + a = 0 (1)
orthogonally, and S2 : x + y + 2x - 6y - b = 0
2 2
(2)
∴ 2g(3) + 2f(-2) = 0 + 2 The equation of common chord of the two circles is
⇒ -6 (-g) + 4(-f ) = 2 S1 - S2 = 0 i.e., 4x + 14y + (a + b) = 0 (3)
Since the circle S1 bisects the circumference of circle
Thus, the locus of the centre (-g, -f ) is S2, therefore, (1) passes through the centre of circle S2,
-6x + 4y = 2 or 3x - 2y + 1 = 0 i.e., (-1, 3)
∴  4 (-1) + 14 (3) + a + b = 0  ⇒  a + b = - 38
COMMON CHORD OF TWO CIRCLES
DIAMETER OF A CIRCLE
The chord joining the points of intersection of two given
circles is called their common chord. The locus of the middle points of a system of parallel
1. Equation of common chord:  The equation of the chords of a circle is called a diameter of the circle.
common chord of two circles

FIGURE 11.26
FIGURE 11.25
The equation of the diameter bisecting parallel chords y =
S1 ≡ x2 + y2 + 2g1x + 2f1y + c1 = 0 (1)
mx + c (c is a parameter) of the circle x2 + y2 = a2 is x +
and S2 ≡ x2 + y2 + 2g2x + 2f2y + c2 = 0 (2) my = 0.
is 2x(g1 - g2) + 2y(f1 - f2) + c1 - c2 = 0
i.e., S1 - S2 = 0 REMEMBER
2. Length of the common chord The diameter corresponding to a system of parallel chords
of a circle always passes through the centre of the circle and
PQ = 2( PM ) = 2 C1 P 2 - C1 M 2 is perpendicular to the parallel chords.
where C1P = radius of the circle S1 = 0 and C1M = length

of the perpendicular from the centre C1 to the common QUICK TIPS
chord PQ.
 The length of the tangent drawn from any point on the
circle x2 + y2 + 2gx + 2fy + c1 = 0 to the circle
QUICK TIPS
x2 + y2 + 2gx + 2fy + c = 0 is c − c1
 The coefficients of x2 and y2 in both the equations
 If two tangents drawn from the origin to the circle
S1 = 0 and S2 = 0 must be unity.
x2 + y2 + 2gx + 2fy + c = 0 are perpendicular to each
 If the circles S = 0 and S = 0 touch, then the common
1 2 other, then g2 + f   2 = 2c.
chord S1 - S2 = 0 becomes the tangent to both of the
 The angle between the tangents from (α, β) to the circle
circles and hence perpendicular from the centre of the
æ a ö
either circle to it should be equal to the corresponding x2 + y2 = a2 is 2 tan-1 ç ÷.
radius. ç a 2 + b 2 - a2 ÷
è ø
11.18  Chapter 11

 If OA and OB are the tangents from the origin to the cir- (a) α/2 (b)  α
cle x2 + y2 + 2gx + 2fy + c = 0 and C is the centre (c) 2α (d)  none of these
of the circle, then the area of the quadrilateral OACB is Solution: (C)
c( g2 + f 2 − c ).
Let the angle between the tangents be 2θ.
 The length of the common chord of the circles x + y
2 2
From the figure,
+ ax + by + c = 0 and x2 + y2 + bx + ay + c = 0 is
1
(a + b)2 − 4c .
2
 If O is the origin and OP, OQ are tangents to the circle x
2

+ y2 + 2gx + 2fy + c = 0, then the circumcentre of the


− g −f 
triangle OPQ is  , .
 2 2 
 The length of the chord intercepted by the circle x2 + y2 =
x y æ r 2(a2 + b2 ) - a2b2 ö
r2 on the line + = 1 is 2 ç ÷
a b è a2 + b2 ø

a sin a
sin q = = sin a
ANGLE OF INTERSECTION OF TWO CIRCLES a
The angle between the two circles is the angle between ⇒ θ=α
their tangents at their point of intersection. Thus, the required angle
= 2θ = 2α

ORTHOGONAL INTERSECTION OF TWO


CIRCLES
Two circles are said to intersect orthogonally when they
FIGURE 11.27
intersect at right angles.
The condition for the circles x2 + y2 + 2g1x + 2f1 y + c1 =
The angle of intersection θ of two circles 0 and x2 + y2 + 2g2x + 2f2y + c2 = 0 to intersect orthogonally
S ≡ x2 + y2 + 2g1x + 2f1 y + c1 = 0 is given by,
and S′ ≡ x2 + y2 + 2g2x + 2f2 y + c2 = 0 2g1g2 + 2 f1 f2 = c1 + c2
2 g1 g 2 + 2 f1 f 2 - c1 - c2
is given by cos q = ±
2 g12 + f12 - c1 . g 22 + f 22 - c2
r12 + r22 - d 2
or cos q = , where r1 and r2 are radii of the two
2r1r2
circles and d is the distance between their centres.
QUICK TIPS
 cos θ ∈ (-∞, -1) ∪ (1, ∞) ⇒ Circles do not intersect
 cos θ ∈ (-1, 1) ⇒ Circles intersect each other
 cos θ = 0 ⇒ Circles intersect each other orthogonally FIGURE 11.28

 cos θ ∈ {-1, 1} ⇒ Circles touch each other internally or


externally. SOLVED EXAMPLE

37. The locus of the centres of the circles which cut the
SOLVED EXAMPLE
circles x2 + y2 + 4x - 6y + 9 = 0 and x2 + y2 - 4x + 6y +
36. From any point on the circle x2 + y2 = a2 tangents 4 = 0 orthogonally is
are drawn to the circle x2 + y2 = a2 sin2 α. The angle (A) 8x - 12y + 5 = 0 (B)  8x + 12y - 5 = 0
between them is (C) 12x - 8y + 5 = 0 (D)  none of these
Circles  11.19

Solution: (A)
Let the equation of one of the circles be
x2 + y2 + 2gx + 2fy + c = 0
Since it cuts the given circles orthogonally,
∴ 2g (2) + 2f (-3) = c + 9
and 2g (-2) + 2f (3) = c + 4 FIGURE 11.32
i.e., 4g - 6f = c + 9  and  -4g + 6f = c + 4
On subtracting, we get, 8g - 12f = 5 x y 1
i.e., -8 (-g) + 12 (-f ) = 5 ( x - x1 ) ( x - x2 ) + ( y - y1 ) ( y - y2 ) + l x1 y1 1 = 0,
So the locus of (-g, -f ) is - 8x + 12y = 5. x2 y2 1

FAMILY OF CIRCLES (where λ is a parameter)

1. The equation of the family of circles passing through the


point of intersection of two given circles S = 0 and S ′ = 0 SOLVED EXAMPLES
is given as S + lS′ = 0, (where λ is a parameter, λ ≠ -1)
38. The intercept on the line y = x by the circle x2 + y2 - 2x =
0 is AB. Equation of the circle with AB as a diameter is
(A) x2 + y2 + x + y = 0 (B)  x2 + y2 - x - y = 0
(C) x + y + x - y = 0
2 2
(D)  none of these
Solution: (B)
Equation of any circle passing through the point of
FIGURE 11.29
intersection of x2 + y2 - 2x = 0 and y = x is
2. The equation of the family of circles passing through the x2 + y2 - 2x + λ(y - x) = 0
point of intersection of circle S = 0 and a line L = 0 is
or x2 + y2 - (2 + λ) x + ly = 0
given as
æ 2 + l -l ö
Its centre is ç , .
è 2 2 ÷ø
For AB to be the diameter of the required circle, the
centre must lie on AB, i.e.,

2 + l -l
= Þ l = -1
2 2
FIGURE 11.30 Thus, equation of required circle is
S + λL = 0, (where λ is a parameter) x2 + y2 - x - y = 0
3. The equation of the family of circles touching the circle S 39. The distance from the centre of the circle x2 + y2 = 2x
= 0 and the line L = 0 at their point of contact P is to straight line passing though the points of intersec-
tion of the two circles x2 + y2 + 5x - 8y + 1 = 0 and x2
+ y2 - 3x - 7y - 25 = 0 is
1
(A) (B)  2
3
(C) 3 (D)  1
Solution: (B)
The equation of the straight line passing through the
FIGURE 11.31
points of intersection of given circles is
S + λL = 0, (where λ is a parameter) (x2 + y2 + 5x - 8y + 1) - (x2 + y2 - 3x + 7y - 25) = 0
4. The equation of the family of circles passing through two
given points P(x1, y1) and Q(x2, y2) can be written in the i.e., 8x - 15y + 26 = 0 (1)
form Also, centre of the circle x2 + y2 - 2x = 0 is (1, 0).
11.20  Chapter 11

∴  Distance of the point (1, 0) from the straight line The radius of the image circle remains unchanged but
(1) is centre changes. Let the centre of image circle be (x1, y1).
Then,
8(1) - 15(0) + 26 34 Slope of C1C2 × Slope of the line L = -1(1)
= = =2
64 + 225 17 and mid point of C1 (- g, - f ) and C2 (x1, y1) lies on the
line
lx + my + n = 0
IMAGE OF THE CIRCLE BY THE LINE MIRROR æx -gö æ y1 - f ö
Let the circle be
i.e., lç 1
2 ÷ + m ç 2 ÷ + n = 0 (2)
è ø è ø
S ≡ x2 + y2 + 2gx + 2fy + c = 0
Solving eqns. (1) and (2), to get value of (x1, y1). Then
and the line be L ≡ lx + my + n = 0
the required image circle is
(x - x1)2 + (y - y1)2 = r2
where r = g2 + f 2 - c

FIGURE 11.33
Circles  11.21

PRACTICE EXERCISES

Single Option Correct Type

1. An isosceles ∆ABC is inscribed in a circle x2 + y2 = a2 8. The locus of the mid-point of the chord of the circle x2
with the vertex A at (a, 0) and the base angles B and C + y2 - 2x - 2y - 2 = 0 which makes an angle of 120º at
each equal to 75º then length of the base BC is the centre is
a (A) x2 + y2 - 2x - 2y + 1 = 0
(A) (B) 
a (B) x2 + y2 + x + y - 1 = 0
2
(C) x2 + y2 - 2x - 2y - 1 = 0
2a 3a (D) none of these
(C) (D) 
3 2 9. A square is inscribed in the circle x2 + y2 - 2x + 4y + 3
= 0. Its sides are parallel to the coordinate axes. Then,
2. Let an, n = 1, 2, 3, 4 represent four distinct positive
one vertex of the square is
real numbers other than unity such that each pair of the
logarithm of an and the reciprocal of logarithm denotes (A) (1 + 2 , - 2) (1 - 2 , - 2)
(B) 
a point on a circle, whose centre lies on y-axis. The (C) (1, - 2 + 2 ) (D)  none of these
product of these four numbers is
(A) 0 (B)  1 10. If the lines a1x + b1 y + c1 = 0 and a2x + b2y + c2 = 0 cut
(C) 2 (D)  13 the coordinate axes in concyclic points, then
(A) a1a2 = b1b2 (B)  a1b1 = a2b2
3. If the tangents PA and PB are drawn from the point (C) a1b2 = a2b1 (D)  none of these
P(-1, 2) to the circle x2 + y2 + x - 2y - 3 = 0 and C is
11. The circle x2 + y2 = 4 cuts the line joining the points
the centre of the circle, then the area of the quadrilat-
BP
eral PACB is A(1, 0) and B(3, 4) in two points P and Q. Let =a
PA
(A) 4 (B)  16 BQ
(C) does not exist (D)  none of these and = b . Then, α and β are roots of the quadratic
QA
equation
4. If the line (y - 2) = m(x + 1) intersects the circle x2 + (A) 3x2 + 2x - 21 = 0 (B)  3x2 + 2x + 21 = 0
y2 + 2x - 4y - 3 = 0 at two real distinct points, then the (C) 2x + 3x - 21 = 0
2
(D)  none of these
number of possible values of m is
(A) 2 (B)  1 12. If the equation of the incircle of an equilateral triangle

PRACTICE EXERCISES
(C) any real value of m (D)  none of these is x2 + y2 + 4x - 6y + 4 = 0, then the equation of the
circumcircle of the triangle is
5. The number of points on the circle x2 + y2 - 4x - 10y + (A) x2 + y2 + 4x + 6y - 23 = 0
13 = 0 which are at a distance 1 from the point (-3, 2) (B) x2 + y2 + 4x - 6y - 23 = 0
is (C) x2 + y2 - 4x - 6y - 23 = 0
(A) 1 (B)  2 (D) none of these
(C) 3 (D)  none of these
13. Two distinct chords drawn from the point (p, q) on the
6. If the equations of four circles are (x ± 4) + ( y ± 4)
2 2 circle x2 + y2 = px + qy, where pq ≠ 0, are bisected by
= 42, then the radius of the smallest circle touching all the x-axis. Then,
the four circles is (A) | p | = | q | (B)  p2 = 8q2
(A) 4( 2 + 1) (B) 
4( 2 - 1) (C) p < 8q (D) 
2 2
p2 > 8q2

(C) 2( 2 - 1) (D)  none of these 14. For the two circles x2 + y2 = 16 and x2 + y2 - 2y = 0,
there is/are
7. The intercept on the line y = x by the circle x2 + y2 - 2x = (A) one pair of common tangents
0 is AB. Equation of the circle with AB as a diameter is (B) two pairs of common tangents
(A) x2 + y2 + x + y = 0 (B)  x2 + y2 - x - y = 0 (C) three common tangents
(C) x + y + x - y = 0
2 2
(D)  none of these (D) no common tangent
11.22  Chapter 11

15. Let AB be a chord of the circle x2 + y2 = r2 subtending 22. A variable circle passes through the fixed point A(p, q)
a right angle at the centre. Then, the locus of the cen- and touches x-axis. The locus of the other end of the
troid of the DPAB as P moves on the circle is diameter through A is
(A) a parabola (A) (y - q)2 = 4px (B) (x - q)2 = 4py
(B) a circle (C) (y - p) = 4qx
2
(D) (x - p)2 = 4qy
(C) an ellipse 23. The point (1, 4) lies inside the circle x2 + y2 - 6x - 10y
(D) a pair of straight lines + p = 0 which does not touch or intersect the coordi-
16. The equation of the smallest circle passing through the nate axes, then
intersection of the line x + y = 1 and the circle x2 + y2 = (A) 0 < p < 29 (B)  25 < p < 29
9 is (C) 9 < p < 25 (D)  9 < p < 29
(A) x2 + y2 + x + y - 8 = 0 24. A circle C1 of radius 2 touches both x-axis and
(B) x2 + y2 - x - y - 8 = 0 y-axis. Another circle C2 whose radius is greater than
(C) x2 + y2 - x - y + 8 = 0 2 touches circle C1 and both the axes. Then, the radius
(D) none of these of circle C2 is
17. If the circle x2 + y2 + 2gx + 2fy + c = 0 bisects the cir- (A) 6 - 4 2 (B) 
6+4 2
cumference of the circles x2 + y2 + 2g′x + 2f′y + c′ = 0, (C) 6 - 4 3 (D) 
6+4 3
then
25. The equation to the sides AB, BC, CA of a ∆ABC are
(A) 2g′(g - g′) + 2f ′(f - f ′) = c - c′
x + y = 1, 4x - y + 4 = 0 and 2x + 3y = 6. Circles are
(B) g′(g - g′) + f′(f - f ′) + c - c′ = 0
drawn on AB, BC, CA as diameters. The point of con-
(C) 2g(g - g′) + 2f(f - f ′) = c - c′
currence of the common chords is
(D) none of these
(A) centroid of the triangle
18. If (a, b) is a point on the circle whose centre is on the (B) orthocentre
x-axis and which touches the line x + y = 0 at (2, -2), (C) circumcentre
then the greatest value of a is (D) incentre
(A) 4 + 2 2 (B)  2+2 2
26. The coordinates of the point on the circle x2 + y2 - 2x
(C) 4 + 2 (D)  none of these - 4y - 11 = 0 farthest from the origin are
19. The equation (x + y - 6) (xy - 3x - y + 3) = 0 represents æ 8 4 ö
(A) ç 2 + , 1+ ÷
the sides of a triangle then the equation of the circum- è 5 5ø
circle of the triangle is
(A) x2 + y2 - 5x - 9y + 20 = 0 æ 4 8 ö
(B) ç1 + , 2+ ÷
PRACTICE EXERCISES

(B) x2 + y2 - 4x - 8y + 18 = 0 è 5 5ø
(C) x2 + y2 - 3x - 5y + 8 = 0 æ 8 4 ö
(D) x2 + y2 + 2x - 3y - 1 = 0 (C) ç1 + , 2+ ÷
è 5 5ø
20. If a > 2b > 0 then the positive value of m for which (D) none of these
y = mx - b 1 + m 2 is a common tangent to x2 + y2 = 27. If the line 3x + ay - 20 = 0 cuts the circle x2 + y2 = 25
b2 and (x - a)2 + y2 = b2 is at real, distinct or coincident points, then a belongs to
2b a 2 - 4b 2 the interval
(A) (B) 
a 2 - 4b 2 2b (A) [- 7 , 7 ]

2b b (B) ( - 7 , 7 )
(C) (D) 
a - 2b a - 2b (C) ( - ¥ - 7 ] È [ 7 , ¥)
(D) none of these
21. If the locus of a point which moves so that the line
joining the points of contact of the tangents drawn 28. The locus of centre of the circle which touches the cir-
from it to the circle x2 + y2 = b2 touches the circle x2 + cle x2 + (y - 1)2 = 1 externally and also touches x-axis
y2 = a2, is the circle x2 + y2 = c2, then a, b, c are in is
(A) A. P. (B)  G. P. (A) {(x, y): x2 + (y - 1)2 = 4} ∪ {(x, y): y < 0}
(C) H. P. (D)  none of these (B) {(x, y): x2 = 4y} ∪ {(0, y): y < 0}
Circles  11.23

(C) {(x, y): x2 = y} ∪ {(0, y): y < 0} 35. The line Ax + By + C = 0 cuts the circle x2 + y2 + ax +
(D) {(x, y): x2 = 4y} ∪ {(x, y): y < 0} by + c = 0 in P and Q. The line A′x + B′y + C′ = 0 cuts
the circle x2 + y2 + a′x + b′y + c′ = 0 in R and S. If P, Q,
29. Let PQ and RS be tangents at the extremeties of the
R, S are concyclic points, then
diameter PR of a circle of radius r. If PS and RQ inter-
sect at a point X on the circumference of the circle, then a + a′ b + b′ c + c ′
2r equals (A) A B C =0
PQ + RS A′ B′ C′
(A) PQ × RS (B) 
2
a − a′ b − b′ c − c ′
2PQ × RS PQ 2 + RS 2
(C) (D)  (B) A B C =0
PQ + RS 2
A′ B′ C′
30. Circles are drawn through the point (-5, 0) to cut the
x-axis on the positive side and making an intercept of A(a + a′ ) B(b + b′ ) C (c + c′ )
10 units on the x-axis. The equation of the locus of the (C) A B C =0
centre of these circles is A′ B′ C′
(A) x + y = 0 (B)  x-y=0
(D) none of these
(C) x = 0 (D)  y=0
36. If q1, q2 be the inclinations of tangents drawn from the
31. The circle x2 + y2 - 4x - 8y + 16 = 0 rolls up the tangent
point P to the circle x2 + y2 = a2 and cotq1 + cotq2 = k,
to it at ( 2 + 3, 3) by 2 units, assuming the x-axis as then the locus of P is
horizontal, the equation of the circle in the new posi-
(A) k(y2 + a2) = 2xy (B)  k(y2 - a2) = 2xy
tion is
(C) k(y - a ) = xy
2 2
(D)  none of these
(A) x 2 + y 2 - 6 x - 2( 4 + 3 ) y + 24 + 8 3 = 0
37. A line meets the coordinate axes in A and B. A circle
(B) x 2 + y 2 + 6 x - 2( 4 + 3 ) y + 24 + 8 3 = 0
is circumscribed about the ∆AOB. If m, n are the dis-
(C) x 2 + y 2 - 6 x + 2( 4 + 3 ) y + 24 + 8 3 = 0 tances of the tangent to the circle at the origin from the
(D) none of these points A and B, respectively, the diameter of the circle
32. The equation of the circle, passing through the point is
(2, 8), touching the lines 4x - 3y - 24 = 0 and 4x + 3y (A) m(m + n) (B)  m+n
- 42 = 0 and having x coordinate of the centre of the (C) n(m + n) (D)  none of these
circle numerically less then or equal to 8, is 38. If the chord of contact of tangents from a point on the
(A) x2 + y2 + 4x - 6y - 12 = 0 circle x2 + y2 = a2 to the circle x2 + y2 = b2 touches the

PRACTICE EXERCISES
(B) x2 + y2 - 4x + 6y - 12 = 0 circle x2 + y2 = c2, then a, b, c are in
(C) x2 + y2 - 4x - 6y - 12 = 0 (A) A. P. (B)  G. P.
(D) none of these (C) H. P. (D)  none of these
x y
33. If the line + = 1 moves in such a way that
a b 39. To which of the following circles, the line y - x + 3 =
1 1 1 æ 3 3 ö
+ 2 = 2 , where c is a constant, then the locus 0 is normal at the point ç 3 + , ÷?
a 2
b c è 2 2ø
of the foot of the perpendicular from the origin on the 2 2
straight line describes the circle æ 3 ö æ 3 ö
(A) ç x - 3 - ÷ +ç y- ÷ =9
(A) x2 + y2 = 4c2 (B)  x2 + y2 = 2c2 è 2 ø è 2ø
(C) x + y = c
2 2 2
(D)  none of these 2 2
æ 3 ö æ 3 ö
(B) ç x - ÷ +ç y- ÷ =9
34. A circle touches both the x-axis and the line 4x - 3y + è 2 ø è 2ø
4 = 0. If its centre is in the third quadrant and lies on
the line x - y - 1 = 0, then the equation of the circle is (C) x2 + (y - 3)2 = 9
(D) (x - 3)2 + y2 = 9
(A) 9(x2 + y2) + 6x + 24y - 1 = 0
(B) 9(x2 + y2) + 6x - 24y + 1 = 0 40. If a circle passes through the points where the lines
(C) 9(x2 + y2) + 6x + 24y + 1 = 0 3kx - 2y - 1 = 0 and 4x -3y + 2 = 0 meet the coordinate
(D) none of these axes then k =
11.24  Chapter 11

(A) 1 (B)  -1 48. If the circles x2 + y2 = 1 and x2 + y2 - 4x - 6y + 12 = 0


1 -1 cut off equal intercepts on a line which passes through
(C) (D) 
2 2 the point (1, 1), then the slope of the line is
41. Let C be any circle with centre (0, 2 ). Then, on the (A) 1 (B)  -1
circle C, there can be 3 3
(C) (D)  -
(A) at the most one rational point 2 2
(B) at the most two rational points
(C) at the most three rational points 49. Consider a curve ax2 + 2hxy + by2 = 1 and a point P not
(D) none of these on the curve. A line drawn from the point P intersects
the curve at points Q and R. If the product PQ.PR is
42. If the tangents PQ and PR are drawn to the circle x2 + independent of the slope of the line, then the curve is
y2 = a2 from the point P(x1, y1), then the equation of the
(A) an ellipse (B)  a hyperbola
circumcircle of DPQR is
(C) a circle (D)  none of these
(A) x2 + y2 - xx1 - yy1 = 0
(B) x2 + y2 + xx1 + yy1 = 0 50. Let L1 be a straight line passing through the origin and
(C) x2 + y2 - 2xx1 - 2yy1 = 0 L2 be the straight line x + y = 1. If the interecpts made
(D) none of these by the circle x2 + y2 - x + 3y = 0 on L1 and L2 are equal,
then which of the following equations can represent
43. A ray of light, incident at the point (-2, -1), gets L1?
reflected from the tangent at (0, -1) to the circle x2 + y2
(A) x + y = 0 (B)  x-y=0
= 1. The reflected ray touches the circle. The equation
(C) 7y + 2x = 0 (D)  x - 7y = 0
of the line along which the incident ray moved is
(A) 4x - 3y + 11 = 0 (B)  4x + 3y + 11 = 0 51. A triangle has two of its sides along the axes. If the
(C) 3x + 4y + 11 = 0 (D)  none of these third side touches the circle x2 + y2 - 2ax - 2ay + a2 =
0, then the equation of the locus of the circumcentre of
44. The coordinates of a point P on the circle x2 + y2 - 4x
the triangle is
- 6y + 9 = 0 such that ∠POX is minimum, where O is
the origin and OX is the x-axis, are (A) 2a(x + y) = 2xy + a2
(B) 2a(x - y) = 2xy + a2
æ 36 15 ö æ -36 15 ö
(A) ç , ÷ (B)  ç 13 , 13 ÷ (C) 2a(x + y) = 2xy - a2
è 13 13 ø è ø (D) none of these
æ 14 12 ö
(C) ç , ÷ (D)  none of these 52. A point moves such that the sum of the squares of its
è 27 27 ø distances from the sides of a square of side unity is
45. The locus of the centre of a circle which passes through equal to 9. The locus of the point is a circle such that
the point (0, 0) and cuts off a length 2b from the line x
PRACTICE EXERCISES

(A) centre of the circle coincides with that of square


= c, is
æ1 1ö
(A) y2 + 2cx = b2 + c2 (B) x2 + cx = b2 + c2 (B) centre of the circle is ç , ÷
è2 2ø
(C) y + 2cy = b + c
2 2 2
(D)  none of these (C) radius of the circle is 2
46. If P, Q is a pair of conjugate points with respect to a (D) all the above are true
circle S, then the circle on PQ as diameter
53. The range of values of a for which the line y + x = 0 bi­
(A) touches the circle S æ 1 + 2a 1 - 2a ö
p
(B) cuts the circle S at an angle sects two chords drawn from a point çç , ÷
è 2 2 ÷ø
×

4
(C) cuts the circle S orthogonally to the circle 2x2 + 2y2 - (1 + 2a) x -(1 - 2a) y = 0 is
(D) none of these
(A) (-∞, -2) ∪ (2, ∞) (B)  (-2, 2)
47. The common chord of the circle x2 + y2 + 8x + 4y - 5 = (C) (2, ∞) (D)  none of these
0 and a circle passing through the origin and touching
the line y = x, passes through the fixed point 54. The locus of the centres of the circles which touch the
æ 5 -5 ö two circles x2 + y2 = a2 and x2 + y2 = 4ax externally is
(A) æç , ÷ö (B) 
5 5
ç 12 , 12 ÷ (A) 12x2 - 4y2 - 24ax + 9a2 = 0
è 12 12 ø è ø
(B) 12x2 + 4y2 - 24ax + 9a2 = 0
æ -5 5 ö (C) 12x2 - 4y2 + 24ax + 9a2 = 0
(C) ç , ÷ (D)  none of these
è 12 12 ø (D) none of these
Circles  11.25

55. If a circle passes through the points of intersection of 62. The base AB of a triangle is fixed and its vertex C
the coordinate axes with the lines lx - y + 1 = 0 and moves such that sin A = k sin B(k ≠ 1). If a is the length
x - 2y + 3 = 0, then the value of λ is of the base AB, then the locus of C is a circle whose
(A) 2 (B)  1 radius is equal to
(C) -1 (D)  -2 ak ak
(A) (B) 
(2 - k )
2 (1 - k2)
56. A circle touches the line y = x at a point P such that
OP = 4 2, where O is the origin. The circle contains (C) 2ak (D)  none of these
the point (-10, 2) in its interior and the length of its 1- k 2
chord on the line x + y = 0 is 6 2. The equation of the 63. The equation of the image of the circle x2 + y2 + 16x
circle is - 24y + 183 = 0 by the line mirror 4x + 7y + 13 = 0 is
(A) x2 + y2 + 18x - 2y + 32 = 0 (A) x2 + y2 + 32x + 4y + 235 = 0
(B) x2 + y2 - 18x - 2y + 32 = 0 (B) x2 + y2 - 32x + 4y + 235= 0
(C) x2 + y2 + 18x + 2y + 32 = 0 (C) x2 + y2 + 32x + 4y - 235 = 0
(D) none of these (D) none of these
57. If S ≡ x2 + y2 + 2gx + 2f y + c = 0 is a given circle, then
the locus of the foot of the perpendicular drawn from 64. The locus of the centre of a circle touching the circle
origin upon any chord of S which subtends a right x 2 + y 2 - 4 y - 2 x = 2 3 - 1 internally and tangents
angle at the origin, is on which from (1, 2) is making a 60º angle with each
(A) 2(x2 + y2) + 2gx + 2fy + c = 0 other, is
(B) 2(x2 + y2) + 2gx + 2fy - c = 0 (A) (x - 1)2 + ( y - 2)2 = 3
(C) x2 + y2 + gx + fy + c = 0 (B) ( x - 2) 2 + ( y - 1) 2 = 1 + 2 3
(D) none of these (C) x2 + y2 = 1
58. The equation of the circle, having the lines x2 + 2xy + (D) none of these
3x + 6y = 0 as its normals and having size just suffi-
65. The equation of locus of the point of intersection of
cient to contain the circle x(x - 4) + y( y - 3) = 0, is
tangents to the circle x2 + y2 = 1 at the points whose
(A) x2 + y2 + 6x + 3y - 45 = 0 parametric angles differ by 60º is
(B) x2 + y2 + 6x - 3y - 45 = 0
(A) 3x2 + 3y2 = 1 (B)  x2 + y2 = 3
(C) x2 + y2 + 6x - 3y + 45 = 0
(C) 3x + 3y = 4
2 2
(D)  none of these
(D) none of these
66. If a square is inscribed in the circle x2 + y2 - 2x + 4y +
59. The equation of the system of coaxal circles that are
3 = 0 and its sides are parallel to the coordinate axes,
tangent at ( 2 , 4) to the locus of the point of intersec-
then one vertex of the square is

PRACTICE EXERCISES
tion of mutually ⊥ tangents to the circle x2 + y2 = 9, is
(A) (1 + 2 , - 2) (B)  (1 - 2 , - 2)
(A) ( x 2 + y 2 - 18) + l ( 2 x + 4 y - 18) = 0
(C) (1, -2 + 2 ) (D)  none of these
(B) ( x 2 + y 2 - 18) + l ( 4 x + 2 y - 18) = 0
67. The equation of the chord of the circle x2 + y2 = a2 passing
(C) ( x 2 + y 2 - 16) + l ( 2 x + 4 y - 16) = 0 through the point (2, 3) and farthest from the centre is
(D) none of these
(A) 2x + 3y = 13 (B)  3x + 2y = 13
60. The point on the straight line y = 2x + 11 which is (C) 2x - 3y = 13 (D)  none of these
nearest to the circle 16(x2 + y2) + 32x - 8y - 50 = 0 is
68. The range of values of p such that the angle θ between
æ 9 ö
(A) æç , 2 ö÷ (B) 
9
ç - 2 , 2÷ the pair of tangents drawn from the point (p, 0) to the
è2 ø è ø æp ö
circle x2 + y2 = 1 lies in ç , p ÷ is
æ 9 ö è 3 ø
(C) ç , - 2 ÷ (D)  none of these
è2 ø (A) (-2, -1) ∪ (1, 2) (B)  (-3, -2) ∪ (2, 3)
(C) (0, 2) (D)  none of these
61. Extremities of a diagonal of a rectangle are (0, 0) and
(4, 3). The equations of the tangents to the circumcircle 69. A circle whose centre coincides with the origin having
of the rectangle which are parallel to this diagonal are radius ‘a’ cuts x-axis at A and B. If P and Q are two points
(A) 16x + 8y ± 25 = 0 (B)  6x - 8y ± 25 = 0 on the circle whose parametric angles differ by 2θ, then
(C) 8x + 6y ± 25 = 0 (D)  none of these the locus of the intersection point of AP and BQ is
11.26  Chapter 11

(A) x2 + y2 + 2ay tanθ = a2 72. Let S1 and S2 be two circles with S2 lying inside S1.
(B) x2 + y2 - 2ay tanθ = a2 A circle S lying inside S1 touches S1 internally and S2
(C) x2 + y2 + 2ay cotθ = a2 externally. The locus of the centre of S is a/an
(D) none of these (A) parabola (B)  ellipse
70. If a chord AB subtends a right angle at the centre of a (C) hyperbola (D)  circle
given circle, then the locus of the centroid of the trian-
73. S(x, y) = 0 represents a circle. The equation S(x, 2) =
gle PAB as P moves on the circle is a/an
0 gives two identical solutions x = 1 and the equation
(A) parabola (B)  ellipse S(1, y) = 0 gives two distinct solutions y = 0, 2. The
(C) hyperbola (D)  circle equation of the circle is
71. If -3l2 - 6l -1 + 6m2 = 0 then the equation of the circle (A) x2 + y2 + 2x + 2y + 1 = 0
for which lx + my + 1 = 0 is a tangent is (B) x2 + y2 + 2x + 2y - 1 = 0
(A) (x + 3)2 + y2 = 6 (B)  (x - 3)2 + y2 = 6 (C) x2 + y2 - 2x - 2y + 1 = 0
(C) x + (y - 3) = 6
2 2
(D)  x2 + (y + 3)2 = 6 (D) none of these

Previous Year's Questions

74. The greatest distance of the point P(10, 7) from the (A) (x − p)2 = 4qy (B) (x − q)2 = 4py
circle x2 + y2 − 4x − 2y − 20 = 0 is [2002] (C) (y − p)2 = 4qx (D) (y − q)2 = 4px
(A) 10 unit (B)  15 unit 80. If the lines 2x + 3y + 1 = 0 and 3x − y − 4 = 0 lie along
(C) 5 unit (D)  none of these diameters of a circle of circumference 10π, then the
75. The equation of the tangent to the circle x2 + y2 + 4x − equation of the circle is [2004]
4y + 4 = 0 which make equal intercepts on the positive (A) x2 + y2 − 2x + 2y − 23 = 0
co-ordinate axes, is [2002] (B) x2 + y2 − 2x − 2y − 23 = 0
(A) x + y = 2 (B)  x+y= 2 2 (C) x2 + y2 + 2x + 2y − 23 = 0
(C) x + y = 4 (D)  x+y=8 (D) x2 + y2 + 2x − 2y − 23 = 0
76. If the two circles (x − 1)2 + (y − 3)2 = r2 and x2 + y2 81. The intercept on the line y = x by the circle x2 + y2 − 2x
− 8x + 2y + 8 = 0 intersect in two distinct points, then = 0 is AB. Equation of the circle on AB as a diameter is
 [2003]  [2004]
(A) 2 < r < 8 (B)  r<2 (A) x2 + y2 − x − y = 0 (B)  x2 + y2 − x + y = 0
(C) r = 2 (D)  r>2 (C) x2 + y2 + x + y = 0 (D)  x2 + y2 + x − y = 0
PRACTICE EXERCISES

77. The lines 2x − 3y = 5 and 3x − 4y = 7 are diameters of 82. If the circles x2 + y2 + 2ax + cy + a = 0 and x2 + y2 − 3ax
a circle having area as 154 sq units. Then the equation + dy − 1 = 0 intersect in two distinct points P and Q
of the circle is [2003] then the line 5x + by − a = 0 passes through P and Q
(A) x + y + 2x − 2y = 62
2 2 for [2005]
(B) x2 + y2 + 2x − 2y = 47 (A) exactly one value of a
(C) x2 + y2 − 2x + 2y = 47 (B) no value of a
(D) x2 + y2 − 2x + 2y = 62 (C) infinitely many values of a
78. If a circle passes through the point (a, b) and cuts the (D) exactly two values of a
circle x2 + y2 = 4 orthogonally, then the locus of its 83. A circle touches the x-axis and also touches the circle
centre is [2004] with centre at (0, 3) and radius 2. The locus of the cen-
(A) 2ax + 2by + (a2 + b2 + 4) = 0 tre of the circle is [2005]
(B) 2ax + 2by − (a2 + b2 + 4) = 0 (A) an ellipse (B)  a circle
(C) 2ax − 2by + (a2 + b2 + 4) = 0 (C) a hyperbola (D)  a parabola
(D) 2ax − 2by − (a2 + b2 + 4) = 0
79. A variable circle passes through the fixed point A (p, 84. If a circle passes through the point (a, b) and cuts the
q) and touches x-axis. The locus of the other end of the circle x2 + y2 = p2 orthogonally, then the equation of the
diameter through A is [2004] locus of its centre is [2005]
Circles  11.27

(A) x2 + y2 − 3ax − 4by + (a2 + b2 − p2) = 0 æ5 ö


(B) 2ax + 2by − (a2 − β 2 + p2) = 0 (A) (0, 0) (B)  ç 4 , 0÷
è ø
(C) x2 + y2 − 2ax − 3by − (a2 − β 2 − p2) = 0
(D) 2ax + 2by − (a2 + b2 + p2) = 0 æ5 ö æ 5 ö
(C) ç , 0 ÷ (D) 
ç 3, 0÷
è2 ø è ø
85. If the lines 3x − 4y − 7 = 0 and 2x − 3y − 5 = 0 are
two diameters of a circle of area 49π square units, the 91. The circle x2 + y2 = 4x + 8y + 5 intersects the line 3x
equation of the circle is [2006] − 4y = m at two distinct points then m satisfies [2010]
(A) x2 + y2 + 2x − 2y − 47 = 0 (A) −35 < m < 15 (B)  15 < m < 65
(B) x2 + y2 + 2x − 2y − 62 = 0 (C) 35 < m < 85 (D)  −85 < m < −35
(C) x2 + y2 − 2x + 2y − 62 = 0
(D) x2 + y2 − 2x + 2y − 47 = 0 92. The two circles x2 + y2 = ax and x2 + y2 = c2 (c > 0)
touch each other if [2011]
86. Let C be the circle with centre (0, 0) and radius 3 (A) |a| = c (B)  a = 2c
units. The equation of the locus of the mid points of (C) |a| = 2c (D)  2|a| = c
2p
the chords of the circle C that subtend an angle of
93. The length of the diameter of the circle which touches
×

3
at its centre is [2006] the x-axis at the point (l, 0) and passes through the
3 point (2, 3) is [2012]
(A) x 2 + y 2 = x2 + y2 = 1
(B) 
2 10 3
(A) (B) 
27 9 3 5
(C) x 2 + y 2 = x2 + y2 =
(D) 
4 4 6 5
(C) (D) 
87. Consider a family of circles which are passing through 5 3
the point (−1, 1) and are tangent to x-axis. If (h, k) are
94. The circle passing through (1, −2) and touching the
the co-ordinates of the centre of the circles, then the
x-axis at (3, 0) also passes through the point [2013]
set of values of k is given by the interval [2007]
1 (A) (2, −5) (B)  (5, −2)
1
(A) 0 < k < (B)  k³
2
(C) (−2, 5) (D)  (−5, 2)
2
1 1 1 95. Let C be the circle with centre at (l, 1) and with radius
(C) − ≤ k ≤ (D)  k≤ ⋅

2 2 2 1. If T is the circle centered at (0, y), passing through


88. The point diametrically opposite to the point P(1, 0) origin and touching the circle C externally, then the
on the circle x2 + y2 + 2x + 4y − 3 = 0 is [2008] radius of T is equal to [2014]

PRACTICE EXERCISES
(A) (3, −4) (B)  (−3, 4) 3
(A) 3 (B) 
(C) (−3, −4) (D)  (3, 4) 2 2
89. If P and Q are the points of intersection of the circles 1 1
x2 + y2 + 3x + 7y + 2p − 5 = 0 and x2 + y2 + 2x + 2y − p2 (C) (D) 
2 4
= 0, then there is a circle passing through P, Q and (1,
1) for [2009] 96. The number of common tangents to the circles x2 + y2
(A) all values of p − 4x − 6y − 12 = 0 and x2 + y2 + 6x + 18y + 26 = 0, is
(B) all except one value of p  [2015]
(C) all except two values of p (A) 2 (B)  3
(D) exactly one value of p (C) 4 (D)  1
90. Three distinct points A, B and C are given in the
2-dimensional coordinate plane such that the ratio of 97. If one of the diameters of the circle, given by the
the distance of anyone of them from the point (1, 0) to ­equation. x2 + y2 – 4x + 6y – 12 = 0, is a chord of a
1 circle S, whose centre is at (–3, 2), then the radius of S
the distance from the point (−1, 0) is equal to Then is [2016]
3
the circumcentre of the triangle ABC is at the point (A) 10 (B)  5 2
 [2009] (C) 5 3 (D)  5
11.28  Chapter 11

98. Let the orthocentre and centroid of a triangle be 99. If the tangent at (1, 7) to the curve x2 = y – 6 touches the
A(–3,  5) and B(3, 3), respectively. If C is the cir- circle x2 + y2 + 16x + 12y + c = 0 then the value of c is
cumcentre of this triangle, then the radius of the circle  [2018]
having line segment AC as diameter, is [2018] (A) 195 (B) 185
3 5 (C) 85 (D) 95
(A)  (B)  2 10
2
5
(C)  3 (D)  10
2

ANSWER K EYS
Single Option Correct Type
1. (B) 2. (B) 3. (C) 4. (C) 5. (D) 6. (B) 7. (B) 8. (A) 9. (D) 10. (A)
11. (A) 12. (B) 13. (D) 14. (D) 15. (B) 16. (B) 17. (A) 18. (A) 19. (B) 20. (A)
21. (B) 22. (D) 23. (B) 24. (B) 25. (B) 26. (B) 27.  (C) 28.  (B) 29. (A) 30. (C)
31. (A) 32. (C) 33. (C) 34. (C) 35. (B) 36. (B) 37. (B) 38. (B) 39. (D) 40. (C)
41. (B) 42. (A) 43. (B) 44. (A) 45. (A) 46. (C) 47. (A) 48. (C) 49. (C) 50. (B)
51. (A) 52. (D) 53. (A) 54. (A) 55. (A) 56. (A) 57. (A) 58. (B) 59. (A) 60. (B)
61. (B) 62. (B) 63. (A) 64. (A) 65. (C) 66. (D) 67. (A) 68. (A) 69. (B) 70. (D)
71. (B) 72. (B) 73. (C)

Previous Years’ Questions


7 4. (B) 75. (B) 76. (A) 77. (C) 78. (B) 79. (A) 80. (A) 81. (A) 82. (B) 83. (D)
84. (D) 85. (D) 86. (D) 87. (B) 88. (C) 89. (A) 90. (B) 91. (A) 92. (A) 93. (A)
94. (B) 95. (D) 96. (B) 97. (C) 98. (C) 99. (D)
PRACTICE EXERCISES
Circles  11.29

HINTS AND EXPLANATIONS

Single Option Correct Type


1. ∠B = ∠C = 75º ∴ (-1, 2) is an interior point of the circle. Thus, m can have
⇒ ∠BAC = 30º any real value.
⇒ ∠BOC = 60º 5. Given circle is x2 + y2 - 4x - 10y + 13 = 0
Its centre is C ≡ (2, 5) and radius = 4
Also, AC = ( 2 - 5) 2 + ( -3 - 2) 2

= 9 + 25 = 34 = 5.83.
∴ AB = AC - BC = 5.83 - 4 = 1.83 > 1.
∴ There is no point on the circle at a distance 1 from the
point (-3, 2).
6. Clearly, from the figure, the radius of the smallest circle
touching the given circles is

⇒ BOC is an equilateral triangle


⇒ BC = OB = the radius of the cirlce
⇒ BC = a.
2. Let (0, a) be the centre and r be the radius of the given circle,
then its equation is

HINTS AND EXPLANATIONS


(x - 0)2 + (y - a)2 = r2 = 4 2 + 4 2 - 4   i.e.,  4 2 - 4.
⇒ x2 + y2 - 2ay + a2 - r2 = 0 (1) 7. Equation of any circle passing through the point of intersec-
tion of x2 + y2 - 2x = 0 and y = x is
æ 1 ö

Since the point ç log an , ÷ ; n = 1, 2, 3, 4 lie on the above
x2 + y2 - 2x + λ(y - x) = 0
è log an ø

or, x2 + y2 - (2 + λ) x + ly = 0.
1 2a
circle, therefore (log an ) 2 + - + a 2 - r 2 = 0, n
(log a ) 2
log a æ 2 + l -l ö
= 1, 2, 3, 4
Its centre is ç , .
2 ÷ø
n n
è 2
⇒ loga1, loga2, loga3, loga4 are the roots of the equation
For AB to be the diameter of the required circle, the centre
λ4 + (a2 - r2)l2 - 2aλ + 1 = 0. must lie on AB, i.e.,

∴  Sum of the roots = 0 2 + l -l


= Þ l = -1.
⇒ loga1 + loga2 + loga3 + loga4 = 0 2 2
⇒ log(a1a2a3a4) = 0  or  a1a2a3a4 = 1.
Thus, equation of required circle is
3. The given circle is S : x2 + y2 + x - 2y - 3 = 0. x2 + y2 - x - y = 0.
Since S]P  (-1, 2) = 1 + 4 - 1 - 4 - 3 = -3 < 0, the point 8. Given equation of circle is
P(-1, 2) lies inside the circle. Consequently, the tangents
x2 + y2 - 2x - 2y - 2 = 0.
from the point P(-1, 2) to the circle do not exist. Thus, the
quadrilateral PACB cannot be formed.
Let mid-point of chord AB be (h, k)
4. The given line passes through the point (-1, 2). Its centre is (1, 1) and radius =
1 + 1 + 2 = 2 = OB
Given circle is S ≡ x2 + y2 + 2x - 4y - 3 = 0. In DOPB, ∠OBP = 30º.

Since S](-1,2) = 1 + 4 - 2 - 8 - 3 < 0, ∴ sin 30º = OP/2 or OP = 1.

11.30  Chapter 11

12. Given equation of the incircle is


x2 + y2 + 4x - 6y + 4 = 0.
Its incentre is (-2, 3) and inradius = 4 + 9 - 4 = 3.


Since in an equilateral triangle, the incentre and the cir-
cumcentre coincide,
∴ Circumcentre ≡ (-2, 3).

Also, in an equilateral triangle, circumradius = 2 (inradius)

∴ Circumradius = 2 ⋅ 3 = 6.

Since, OP = 1  ⇒ (h - 1)2 + (k - 1)2 = 1 ∴ The equation of the circumcircle is

or, h2 + k2 - 2h - 2k + 1 = 0
(x + 2)2 + ( y - 3)2 = (6)2

∴  Locus of mid point of chord is

or, x2 + y2 + 4x - 6y - 23 = 0.
x2 + y2 - 2x - 2y + 1 = 0.
9. The centre of the given circle is (1, -2). Since the sides of the
square inscribed in the circle are parallel to the coordinate QUICK TIPS
axes, so the x-coordinate of any vertex cannot be equal to 1 In an equilateral triangle
and its y-coordinate cannot be equal to -2. Hence, none of  the incentre and the circumcentre coincide
the points given in (a), (b) and (c) can be the vertex of the  circum radius = 2 (in radius)
square.
10. The line a1x + b1y + c1 = 0 cuts the coordinate axes at A(-c1/
a1, 0) and B(0, - c1/b1) and the line a2x + b2y + c2 = 0 cuts the 13. Given circle is x2 + y2 = px + qy.
axes at C(-c2/a2, 0) and D(0, -c2/b2). æ p qö

Since the centre of the circle is ç , ÷ , so (p, q) and (0,
So, AC and BD are chords along x-axis and y-axis, respec- è 2 2ø
tively, intersecting at origin O. 0) are the end points of a diameter. As the two chords are
Since A, B, C, D are concyclic, therefore bisected by x-axis, the chords will cut the circle at the points
(x1, -q) and (x2, -q), where x1 , x2 are real.
OA.OC = OB.OD
æ -c1 ö æ -c2 ö æ -c1 ö æ -c2 ö

ç ÷. ç ÷=ç ÷. ç ÷
è a1 ø è a2 ø è b1 ø è b2 ø
or, a1a2 = b1b2.
HINTS AND EXPLANATIONS

11. The equation of the line joining A(1, 0) and B(3, 4) is y = 2x


 8 6
-2. This cuts the circle x2 + y2 = 4 at Q(0, -2) and P  ,  .
 5 5

The equation of the line joining these points is y = -q.


Solving y = -q and x2 + y2 = px + qy, we get
x2 - px + 2q2 = 0.
The roots of this equation are x1 and x2. Since the roots are
real and distinct, ∴  discriminent > 0
i.e., p2 - 8q2 > 0  or  p2 > 8q2.
7 3 14. Given circles are
We have, BQ = 3 5, QA = 5, BP =
and PA =
5 5
S1: x2 + y2 - 16 = 0 (1)

and, S2: x + y - 2y = 0
2 2
(2)
BP 7 / 5 7 BQ 3 5
∴ a =
= = and b = = = -3
PA 3/ 5 3 QA - 5 Centre of S1 is C1: (0, 0) and radius r1 = 4

Centre of S2 is C2: (0, 1) and radius r2 = 1

∴ α, β are roots of the equation x2 - x(α + β) + aβ = 0

æ7 ö 7 ∴
C1C2 = 0 + 1 = 1

i.e., x 2 - x ç - 3 ÷ + ( -3) = 0
è3 ø 3 Since | C1C2 | < | r1 - r2 |,  ∴  S2 is completely within S1 and

or, 3x2 + 2x - 21 = 0.
hence there are no common tangents to the two circles.
Circles  11.31

15. Let the centroid ≡ (α, β). Then, 18. Since the slope of the given line is -1,
∴ ∠COP = 45º
∴ OP = 2 2 = CP


OC = ( 2 2 ) 2 + ( 2 2 ) 2 = 4

r + r cosq r + r sin q
a= ,b =
3 3
The point on the circle with the greatest x-coordinate is A.
2 2
æ rö æ rö r2 ∴  a = OA = OC + CA = 4 + 2 2.

or, ça - 3 ÷ + ç b - 3 ÷ = 9
è ø è ø 19. We have, x + y = 6
2 2 2 and, xy - y - 3x + 3 = 0
 r  r  r
∴  The locus is  x −  +  y −  =   , which is a
⇒  y(x - 1) - 3(x - 1) = 0
 3   3   3
circle. ⇒ (x - 1)(y - 3) = 0
Equations of the sides of the triangle are
16. The equation of a circle passing through the intersection of
the given line and the circle is x + y = 6 (1)
y = 3 (2)
(x2 + y2 - 9) + k(x + y - 1) = 0
x = 1 (3)
æ k kö
Its centre is ç - , - ÷ .

è 2 2ø
æ k kö
The circle is the smallest if the centre ç - , - ÷ lies on the

chord x + y = 1 è 2 2ø

HINTS AND EXPLANATIONS


k k
∴  - - = 1 Þ k = -1

2 2
Thus, the equation of the smallest circle is
(x2 + y2 - 9) - 1(x + y - 1) = 0
i.e., x2 + y2 - x - y - 8 = 0
17. The given circles are
S1: x2 + y2 + 2gx + 2fy + c = 0 (1)
Shaded triangle is right angled at (1, 3). ∴  the circumcircle
and, S2: x2 + y2 + 2g′x + 2f′y + c′ = 0 (2)
is the circle on (3, 3) and (1, 5) as ends of a diameter and its
The equation of common chord of (1) and (2) is equation is (x - 3)(x - 1) + (y - 3) (y - 5) = 0, i.e., x2 + y2 - 4x
S1 - S2 = 0 - 8y + 18 = 0.
i.e., 2(g - g′)x + 2(f - f ′ )y + (c - c′) = 0 (3) 20. Clearly, the line y = mx - b 1 + m 2 will pass from the point

Since (1) bisects the circumference of (2), therefore com- (a/2, 0) (mid-point of the centres of the circles)
mon chord will be the diameter of circle (2) 2b
∴ Centre (-g′, - f ′) of circle (2) lies on (3).
Þ m=
a – 4b 2
2

∴ -2(g - g′)g′ - 2(f - f ′)f ′ + c - c′ = 0



or, 2g′(g - g′ ) + 2f ′(f - f  ′ ) = c - c′.

QUICK TIPS
If a circle bisects the circumference of another circle, then
their common chord is the diameter of second circle
11.32  Chapter 11

21. Let P(h, k) be any point on the locus. Equation of the chord
25. Since ADB = ADC = 90°, circles on AB and AC as diame-
of contact of P with respect to the circle x2 + y2 = b2 is hx +
ters pass through D and therefore the altitude AD is the com-
ky = b2. If it touches the circle x2 + y2 = a2, then
mon chord. Similarly, the other two common chords are the
−b 2 other two altitudes and hence they concur at the ortho-centre.
= a ⇒ a 2 ( h2 + k 2 ) = b 4
h2 + k 2

So that the locus of P(h, k) is x2 + y2 = (b2/a)2
2
æ b2 ö

∴ c 2 = ç ÷ Þ ac = b 2
è aø

⇒ a, b, c are in G. P.
22. Let the variable circle be 26. The equation of the given circle can be written as
x2 + y2 + 2gx + 2fy + c = 0 (1) (x - 1)2 + (y - 2)2 = 16 = 42
∴ p2 + q2 + 2gp + 2fq + c = 0 (2)
So, the coordinates of any point P on the circle are
Circle (1) touches x-axis, (1 + 4 cosθ, 2 + 4 sinθ) whose distance from the origin is
∴ g2 - c = 0 ⇒ c = g2. From (2)
d = (1 + 4 cosq ) 2 + ( 2 + 4 sin q ) 2
p2 + q2 + 2gp + 2fq + g2 = 0 (3)
Let the other end of diameter through (p, q) be (h, k), then = 21 + 8 cosq + 16 sin q


h+ p k+q
= - g and =-f ⇒ d = 21 + 8(cosq + 2 sin q )

2 2
Put in (3)
2 = 21 + 8r cos(q - a )
æ h+ pö æ k +qö æh+ pö

p2 + q2 + 2 p ç - ÷ + 2q ç - ÷ +ç ÷ =0
è 2 ø è 2 ø è 2 ø where r cos α = 1, r sin α = 2


⇒ h2 + p2 - 2hp - 4kq = 0
= 21 + 8 5 cos(q - a )
∴  Locus of (h, k) is which is maximum when cos (θ - α) = 1,

x2 + p2 - 2xp - 4yq = 0  ⇒  (x - p)2 = 4qy i.e., θ = α = tan-1 2.

23. Since the circle does not touch or intersect the coordinates 2 1
axes, the absolute values of x and y coordinates of the centre ⇒ tanθ = 2 so sinq =
= and cosq =
5 5
HINTS AND EXPLANATIONS

are greater than the radius of the circle. Coordinates of the



and, the coordinates of the required point are
centre of the circle are (3, 5) and the radius is 9 + 25 - p
so that 3 > 9 + 25 - p
Þ p > 25 (1) æ 4 8 ö
ç1 + , 2+ ÷.
è 5 5ø
5 > 9 + 25 - p Þ p > 9 (2)
and the point (1, 4) lies inside the circle 27. The length of the ⊥ from the centre (0, 0) of the given circle
⇒ 1 + 16 - 6 - 10 × 4 + p < 0  ⇒  p < 29 (3) to the line 3x + ay - 20 = 0 is
From (1), (2), (3) we get | 3(0) + a(0) - 20 | 20
= .
25 < p < 29. 9 + a2 9 + a2
24. First circle touches both axes and radius is 2 unit.
Radius of the given circle = 5

Hence, centre of circle is (2, 2).

Since the line cuts the circle at real, distinct or coincident
Let radius of other circle be a and this circle also touches points
both the axes.
20
Hence, centre of circle is (a, a). ∴ 
£ 5   ⇒  a2 + 9 ≥ 16  ⇒  a2 - 7 ≥ 0
This circle touches first circle 9 + a2
Hence, ( a - 2) 2 + ( a - 2) 2 = a + 2 ⇒ ( a + 7 )( a - 7 ) ³ 0

On squaring both the sides, we get
⇒ a Î ( - ¥, - 7 ] È [ 7 , ¥)

a2 - 12a + 4 = 0
28. According to given condition
12 ± (12) 2 − 4 × 4 × 1 12 ± 128
⇒ a= = =6±4 2 ( h - 0) 2 + ( k - 1) 2 = 1+ | k |
2 2

But a > 2, therefore a = 6 - 4 2 is neglected.
⇒ h2 + (k - 1)2 = (1 + |k |)2

Hence, a = 6 + 4 2
⇒ h2 = 2k + 2|k |

Circles  11.33

Equation of tangent to the circle (1) at P( 2 + 3 , 3) is



( 2 + 3 ) x + 3 y - 2( x + 2 + 3 ) - 4( y + 3) + 16 = 0

or,  3 x - y - 2 3 = 0 (2)
If line (2) makes an angle θ with the positive direction of

x-axis, then tan q = 3,   ∴  θ = 60º.

Hence locus is x2 = 2y + 2| y |

Clearly, for y > 0, x2 = 4y

and for y < 0, x2 = 0  ⇒  x = 0.

PQ PQ
29. tan q = =
PR 2r
æp ö RS
Also, tan ç - q ÷ =

è2 ø 2r
RS
i.e.,  cot q =

Let A and B be the centres of the circles in old and new posi-
2r
tions, respectively, then
PQ × RS
A ≡ (2, 4) and B ≡ (2 + 2 cos 60º, 4 + 2 sin 60º)( AB = 2)
\
∴ tanq cot q =

4r 2
Thus, B º (3, 4 + 3 ).

⇒ 4r2 = PQ ⋅ RS

Radius of the circle = 22 + 4 2 - 16 = 2.

∴  Equation of the circle in the new position is

( x - 3) 2 + ( y - 4 - 3 ) 2 = 22

or, x 2 + y 2 - 6 x - 2( 4 + 3 ) y + 24 + 8 3 = 0
32. Let C (α, β) be the centre of the circle.
Since the circle passes through the point (2, 8),
∴  radius of the circle = (a - 2) 2 + ( b - 8) 2 .

Since the circle touches the lines 4x - 3y - 24 = 0

HINTS AND EXPLANATIONS


and, 4x - 3y - 42 = 0,

| 4a - 3b - 24 | | 4a + 3b - 42 |
= = (a - 2) + ( b - 8)
2 2
∴ 

5 5
⇒ 2r = ( PQ )( RS ).
(i) (ii) (iii)
30. Since the circles pass through (-5, 0) and makes an intercept
From (i) and (ii), we get
of 10 units on the positive side of the x-axis, it also passes 4α - 3β - 24 = ± (4α + 3β - 42)

through (5, 0).
ì6 b = 18 Þ b = 3 ( taking positive sign )
∴ if C(x, y) is the centre of such a circle, (x + 5)2 + y2 = ï
(x - 5)2 + y2 ∴  í
33
ïî8a = 66 Þ a = 4 ( taking negative sign).
(   CA = CB = radius)
\
⇒  x = 0. 33
Given, | α | ≤ 8 or -8 ≤ α ≤ 8,  ∴  a ¹
. ×

4
Putting β = 3 in equations (i) and (iii) and equating, we get

(4α - 33)2 = 25 [(α - 2)2 + 25]

or, 16a2 - 264α + 1089 = 25a2 + 725 - 100α

or, 9a2 + 164α - 364 = 0

-164 ± (164) 2 + 36 ´ 364
∴  a =

18
31. Given circle is -164 ± 200 -182
= = 2, .
x2 + y2 - 4x - 8y + 16 = 0 (1) 18 9
Let P = ( 2 + 3 , 3). But -8 ≤ α ≤ 8,  ∴  α = 2.

11.34  Chapter 11

Now, (radius)2 = (α - 2)2 + (3 - 8)2 1 4 1


= (2 - 2)2 + (3 - 8)2 = 25. Thus, for the required circle g = , f = , c = .

3 3 9
Hence, the equation of the required circle is ∴  Equation of the required circle is

(x - 2)2 + (y - 3)2 = 25 2 8 1
x2 + y2 + x+ y+ =0
or, x + y - 4x - 6y - 12 = 0.
2 2
3 3 9
x y
33. The equation of the line is + = 1 (1) or, 9(x2 + y2) + 6x + 24y + 1 = 0.
a b
1 1 1 35. Equation of any circle through the points of intersection P
where, 2 + 2 = 2 (2)
a b c and Q of line
Here, a, b are parameters while c is a constant. Ax + By + C = 0 (1)
x y and circle x2 + y2 + ax + by + c = 0 (2)
Any line ⊥ r to (1) is
- + k = 0.
b a is x2 + y2 + ax + by + c + λ (Ax + By + C) = 0
If it passes through the origin then k = 0.
or, x2 + y2 + (a + λA) x + (b + λB) y + c + λC = 0 (3)
∴  Equation of the line through the origin and ^r to (1) is
Again, equation of any circle through the points of intersec-
x y tion R and S of line

- = 0 (3)
b a A′x + B′y + C′ = 0 (4)
The locus of the foot of the ^r from origin on (1), i.e., locus
and circle x2 + y2 + a′ x + b′ y + c′ = 0 (5)
of the point of intersection of (1) and (3) is obtained by elim- is x + y + a′ x + b′ y + c′ + µ(A′ x + B′ y + C′) = 0
2 2

inating the parameters a and b between them.


or, x2 + y2 + (a′ + µA′) x + (b′ + µB′) y + c′ + µC′ = 0 (6)

Squaring (1) and (3) and adding, we get
If circles (3) and (6) are same, then points P, Q, R, S will lie
æ 1 1ö 2 æ 1 1 ö 2 on the same circle, i.e., points P, Q, R, S will be concyclic.
ç a2 + b2 ÷ x + ç b2 + a2 ÷ y = 1

è ø è ø Comparing the coefficients in (3) and (6), we get
1 2 1 2 1 1 a + lA b + lB c + lC

or, x + 2 y =1 (Using (2) = = = =
c2 c 1 1 a¢ + m A¢ b¢ + m B¢ b¢ + mC ¢
or x2 + y2 = c2, which is clearly a circle with centre at origin
(i) (ii)  (iii)  (iv)  (v)
and radius c. From (i) and (iii), we get a - a′ + λA - µA′ = 0
(7)
34. Let the equation of the required circle be From (i) and (iv), we get b - b′ + λB - µB′ = 0
(8)
x2 + y2 + 2gx + 2f y + c = 0 (1) From (i) and (v), we get c - c′ + λC - µC′ = 0 
(9)
HINTS AND EXPLANATIONS

Its centre is C(-g, -f ) and radius is g 2 + f 2 - c . Eliminating λ and -µ from equations (7), (8) and (9)
and
Since circle (1) touches the x-axis writing the result in determinant form, we get
∴  g2 - c = 0 or c = g2(2) a − a′ A A′
Again, since circle (1) touches the line b − b′ B B = 0
4x - 3y + 4 = 0 (3)
| - 4 g + 3 f + 4| c − c′ C C ′
∴  = g + f - c = f = |f |
2 2 2
a − a′ b − b′ c − c ′
5
 [from (2)] or,  A B C = 0.
or, - 4g + 3f + 4 = ± 5f A′ B′ C′
∴ 4g + 2f = 4 or 2g + f = 2 (4) 36. Equation of the circle is
- 4g + 8f = - 4  or  g - 2f = 1 (5) x2 + y2 = a2(1)
Again, since centre C(-g,- f ) lies on the line Let P be the point (x1, y1).
x-y-1=0 Equation of any tangent to (1) is y = mx + a 1 + m 2

∴  -g + f = 1 (6)
If it passes through P(x1, y1), then
1 4 y1 = mx1 + a 1 + m 2
Solving (4) and (6), we get g = , f = .
3 3
æ -1 -4 ö or, y1 - mx1 = a 1 + m 2 .

Thus, C = ç , ÷ which lies in the third quadrant. Squaring + 2mx1 y1 + m2 = a2 (1 + m2)

è 3 3ø
1 or, (x21 - a2) m2 - 2x1 y1m + (y21 - a2) = 0
(2)
Also, from (2), c = g 2 = .
9
This is a quadratic in m. If m1 and m2 are its roots, then these
Solving (5) and (6), we get f = -2, g = -3 are the slopes of the tangents from P.
∴  C ≡ (3, 2) which lies in the first quadrant. Since inclinations of tangents are given to be q1 and q2,

Circles  11.35

∴ let m1 = tan q1 and m2 = tan q2.


∴  the length of ⊥ from centre (0, 0) on (2) is = radius c.

1 1
Here, cotq1 + cotq2 = k i.e., 
+ =k b2
tan q1 tan q 2 ⇒
=c
1 1 x + y12
2
or,
+ = k  or m1 + m2 = km1m2. 1

m1 m2 b2
or,
=c [Using (1)]
2 x1 y1 y -a
2 2
a2
∴ 
=k× 1
 or 2x1 y1 = k ( y12 - a2)
x12 - a 2 x - a2
2
1 or, b2 = ac. Hence, a, b, c are in G.P.

∴  Locus of P is k (y2 - a2) = 2xy.  3 3 
37. Let A ≡ (a, 0) and B ≡ (0, b). 39. The point  3 + ,  does not satisfy circles given in
 2 2
Since ∠AOB = 90º,  ∴  AB is the diameter. (a) and (c).
æa bö 1 2 ∴ (a) and (c) cannot be the correct choices. The centre of
∴  Centre of the circle is ç , ÷ and radius = a + b2 .
è 2 2 ø 2  3 3 
circle given in (b) is  , which does not lie on the
 2 2 
line y - x + 3 = 0.
∴  The circle given in (b) cannot be the correct choice. The
centre (3, 0) of circle given in (d) lie on the line
y - x + 3 = 0.
Thus, the line is normal at the given point on the circle given
in (d).
 1 
40. The line 3kx - 2y -1 = 0 meets x-axis and y-axis at A  , 0
 3k 
 1
and B  0, −  respectively and the line 4x - 3y + 2 = 0 cuts
 2
 1   2
x-axis and y-axis at C  − , 0 and D  0,  respectively.
∴  Equation of the circle is
 2   3
2 2
 a  b 1 2
 x −  +  y −  = ( a + b )
2


2 2 4
or,  x2 + y2 - ax - by = 0.


Equation of tangent to the circle at O(0, 0) is

HINTS AND EXPLANATIONS



ax + by = 0 (1)
a2
∴ m = length of ⊥ from A(a, 0) on (1) =

a 2 + b2
b2
and, n = length of ⊥ from (0, b) on (1) =
.
a 2 + b2
∴ Diameter = a 2 + b 2 = m + n.
Since the four points are concyclic, therefore
38. Let (x1, y1) be any point on the circle x2 + y2 = a2.
OA  ⋅ OC = OB ⋅ OD

then, x12 + y12 = a 2 (1) 1 1 1 2 1
⇒ 
⋅ = ⋅ ⇒ | k |= .
Equation of chord of contact of tangents from (x1, y1) to 3| k | 2 2 3 2
the circle x2 + y2 = b2 is xx1 + yy1 = b2 (2)
According to the given geometrical position (see figure), k
must be positive,
1
∴  k = .
2
41. Let the equation of the circle be
x 2 + ( y − 2 )2 = a2
⇒  x 2 + y 2 − 2 2 y = c,

where, c = a2 - 2 → Rational number.




Let (x1, y1), (x2, y2), (x3, y3) be three distinct rational points on
the circle, then
  (2) touches the circle x2 + y2 = c2,
\
x12 + y12 − 2 2 y1 = c (1)
11.36  Chapter 11


x22 + y22 − 2 2 y2 = c (2) 4
∴  Slope of the incident ray = − .

3

x32 + y32 − 2 2 y3 = c (3)

Hence, equation of the incident ray is

Comparing the irrational parts of the equations, we get
−4

y1 = y2 = y3(4)
( y + 1) = ( x + 2) i.e., 3(y + 1) = -4(x + 2)
3

Comparing the rational parts of the equations, we get or, 4x + 3y + 11 = 0.

x12 + y12 = x22 + y22 = x32 + y32 44. Given circle is x2 + y2 - 4x - 6y + 9 = 0 (1)
  y1 = y2 = y3,
\
Its centre is C(2, 3) and radius is 2.

x12 = x22 = x32 . Let OP and ON be the two tangents from 0 to circle (1), then
∴  The only possible values of x are ± x1, ± x2, ± x3. ∠POX will be minimum when OP is tangent to the circle at
P. Let ∠POX = θ, then ∠LCP = θ.
∴  There can be at the most two rational points on
the circle C. Now, CP = 2, OC = 22 + 32 = 13.

42. Given circle is x2 + y2 - a2 = 0 (1)
∴ OP = OC 2 − CP 2 = 13 − 4 = 3.

Since PQ and PR are tangents to the circle (1), therefore QR \
is chord of contact of point P(x1, y1) and hence equation of C ≡ (2, 3),   ∴  OL = 2.
QR is From the figure, OM = OL + LM = OL + HP

xx1 + yy1 - a2 = 0 (2)
Now, equation of any circle through the point of intersection
Q and R of circle (1) and line (2) is
x2 + y2 - a2 + k(xx1 + yy1 - a2) = 0 (3)
Circle (3) will be circumcircle of DPQR if it passes through
the point P(x1, y1).
i.e., if x12 + y12 − a 2 + k ( x12 + y12 − a 2 ) = 0   ⇒  k = -1.

Hence, from (3), equation of required circle is
x2 + y2 - a2 - (xx1 + yy1 - a2) = 0
or, x2 + y2 - xx1 - yy1 = 0. ∴ OP cosθ = 2 + 2 sinθ or 3 cosθ = 2 + 2 sinθ

43. The equation of the reflected ray is ( y + 1) = m(x + 2) or, 3 = 2 secθ + 2 tanθ or 3 - 2 tanθ = 2 secθ

or, mx - y + 2m - 1 = 0 (1)
HINTS AND EXPLANATIONS

or, 9 + 4 tan2θ - 12 tanθ = 4 (1 + tan2θ)



5
or, 5 = 12 tanθ  ∴  tan q =

12
12 5
∴ cosq =
and sin q = .
13 13
 36 15 
∴ P ≡ (OP cosθ, OP sinθ) i.e., P ≡  ,  .

Since it touches the circles x + y = 1.
2 2  13 13 
∴ length of ⊥ from (0, 0) on (1) = radius 1
45. Let the centre of the circle be C(x1, y1).
| m( 0 ) − 0 + 2 m − 1 | 2m − 1 As it passes through (0, 0), its radius = OC = x12 + y12 .


=1 ⇒ = ±1
1+ m 2
1 + m2 Let AB be the line x = c meeting

4 the circle in A and B. Draw CM ⊥ AB.

⇒ (2m - 1)2 = (1 + m2)  ⇒ 3m2 - 4m = 0  ⇒  m = 0, .

3
Join CB.
∴  Equation of the reflected ray is

4
( y + 1) =
( x + 2)   or 4x - 3y + 5 = 0.
3
Let α be the angle between the reflected ray and the line


y = -1.
4
-0
3 4
Then, tan a =
=± .
4 3
1+ ×0
3
Circles  11.37

CB = radius = x12 + y12 .

CM = length of ⊥ from C on AB = x1 - c.

Now, AB = 2b (given).

∴ 2BM = 2b or  CB 2 − CM 2 = b

or, CB2 - CM2 = b2 or  x1 + y1 − ( x1 − c) = b


2 2 2 2


or, y12 + 2cx1 − c 2 = b 2 . | 0 − 0 − m + 1| |1 − m |
= = .
∴  Locus of (x1, y1) is y2 + 2cx = b2 + c2. m +1
2
m2 + 1
46. Let the coordinates of P and Q be (x1, y1) and (x2, y2),
So, the length of the intercept
respectively.
(1 − m) 2 2m

Let the given circle be = 2 12 − 2 =2 .
m + 1 m 2
+1

S ≡ x2 + y2 - a2 = 0 (1)
Polar of P w.r.t. (1) is xx1 + yy1 = a2.
Also, the length of ⊥ from the centre (2, 3) of second circle

| 2m − 3 − m + 1 | | m − 2 |
  P, Q are conjugate points, to the line (1) = =
\
m2 + 1 m2 + 1
∴ polar of P passes through Q

⇒ x1x2 + y1 y2 = a2(2)


Equation of circle on PQ as diameter is
(x - x1) (x - x2) + (y - y1) ( y - y2) = 0

or, x2 + y2 - (x1 + x2) x - ( y1 + y2) y + (x1x2 + y1 y2) = 0
x2 + y2 - (x1 + x2) x - ( y1 + y2) y + a2 = 0 (3)
 [Using (2)]
Clearly, circles (1) and (3) cut each other orthogonally. ∴ the length of intercept in this case

47. Given circle is x2 + y2 + 8x + 4y - 5 = 0 (1)
( m − 2) 2 4m − 3

Let the equation of the second circle be = 2 1− 2 =2
m +1 m2 + 1

x2 + y2 + 2g x + 2f y + c = 0.

HINTS AND EXPLANATIONS


Since it passes through origin,  ∴  c = 0.
2m 4m − 3

Given, 2 =2

So, the equation becomes m +1
2
m2 + 1


x2 + y2 + 2g x + 2f y + c = 0 (2) 2m 4m − 3 3

=   ⇒ 2m = 3  or  m = .

The equation of common chord of (1) and (2) is m2 + 1 m2 + 1 2
49. Let the coordinates of point P be (x1, y1). Equation of any
2(g - 4) x + 2( f - 2) y + 5 = 0
(3)
line through P can be written as
Since the line y = x touches the circle (2)

x - x1 y - y1
∴ x2 + x2 + 2g x + 2f x = 0 has equal roots

= = r (1)
cosq sin q
i.e., f + g = 0.

⇒ x = x1 + r cosθ, y = y1 + r sinθ.

∴ From (3), the equation of common chord is


Coordinates of any point an (1) is of the form

2 (g - 4) x + 2 (- g - 2) y + 5 = 0
(x1 + r cosθ, y1 + r sinθ). This point will lie on

or, (-8x - 4y + 5) + g (2x - 2y) = 0,
ax2 + 2hxy + by2 = 1 if


which passes through the point of intersection of a (x1 + r cosθ)2 + 2h (x1 + r cosθ) ( y1 + r sinθ)

 5 5
+ b (y1 + r sinθ)2 - 1 = 0
8x + 4y - 5 = 0 and x = y, i.e., the point  ,  .

 12 12  ⇒ r (a cos θ + 2h cosθ sinθ + b sin2θ)
2 2

48. Let the equation of line be y = mx + c. Since it passes through


+ 2r [x1 (a cosθ + h sinθ) + y1 (h cosθ + b sinθ)]
(1, 1),

+ ax12 + 2hx1 y1 + by12 - 1 = 0 (2)
∴ 1 = m + c, i.e., c = 1 - m.
Let PQ = r1 and PR = r2. Then r1, r2 are the roots of (2).

So, the line becomes y = mx + 1 - m
i.e., mx - y - m + 1 = 0 (1) ax12 + 2hx1 y1 + by12 - 1

PQ × PR = r1r2 = .
Length of ⊥ from the centre (0, 0) of first circle to the line (1) a cos q + 2h cosq sin q + b sin 2 q
2
11.38  Chapter 11


We know rewrite the denominator. We have 2
æ 1 1 ö æ a a ö
D = a cos2θ + 2h cosθ sinθ + b sin2θ.
∴ From (1), a 2 ç 2 + 2 ÷ = ç
+ - 1÷
è 4r 4d ø è 2g 2d ø
1
= [( a + b) + ( a - b)cos 2q ] + h sin 2q ⇒ 2a(γ + δ ) – a2 = 2gδ
2
So, the locus of (γ, δ) is 2a(x + y) = 2xy + a2.
a+b 1
= + ( a - b)cos 2q + h sin 2q 52. Let the sides of the square be y = 0,
2 2
y = 1, x = 0 and x = 1.
1
Put ( a − b) = k  sinα, h = k cosα.
Let the moving point be (x, y).
2
Then, y2 + (y - 1)2 + x2 + (x - 1)2 = 9 is the equation of the
2
 a − b a−b locus.
⇒ k = 
+ h2 and tan  a = .
 2  2h ⇒ 2x2 + 2y2 - 2x - 2y - 7 = 0,
 1 1
2 which represents a circle having centre  ,  (which is

1 æa-bö  2 2
D = ( a + b) + ç ÷ + h sin( 2q + a )
2

2 è 2 ø also the centre of the square) and radius 2.


ax12 + 2hx1 y1 + by 2 - 1 53. Equation of the given circle can be written as
Thus, PQ × PR =
1

1 æa-bö
2  1 + 2a   1 − 2a 
( a + b) + ç ÷ + h sin( 2q + a )
2
x2 + y2 −   x−  y = 0.
2 è 2 ø  2   2 
For this to be independent of θ, we must have

Since y + x = 0 bisects two chords of this circle, mid-points
2 of the chords must be of the form (α, -α).
 a − b

 + h = 0   ⇒  a = b and h = 0. Equation of the chord having (α, -α) as mid-point is
2

2 
But this is the condition for the given curve to represent a æ 1 + 2a ö
T = S1 i.e., ax + ( -a ) y - çç
÷÷ ( x + a )
circle. è 4 ø
50. Let the equation of line L1 be y = mx. Intercpts made by L1 æ 1 - 2a ö
-ç (y +a)
and L2 on the circle will be equal if L1 and L2 are at the same ç 4 ÷÷
distance from the centre of the circle. Centre of the given è ø
æ 1 -3 ö æ 1 + 2a ö æ 1 - 2a ö
circle is ç , ÷ . Therefore, = a 2 + a2 - ç a - çç ÷÷ ( -a )
ç 2 ÷÷

è2 2 ø
è ø è 2 ø
HINTS AND EXPLANATIONS

1 3 m 3
- -1 +
2 2 2 2 4 m+3
= Þ =
1+1 1 + m2 2 1 + m2

⇒ 7m2 – 6m – 1 = 0

⇒ (7m + 1) (m – 1) = 0
-1

⇒ m = 1, .
7

Thus, two chords are y = x and 7y + x = 0.
x y
51. Let the third side be+ = 1.
a b
For the circle, centre ≡ (a, a) and radius = a.


Since the third side touches the circle, ⇒ 4 ax − 4 ay − (1 + 2a ) x − (1 − 2a ) y

a a = 8a 2 − (1 + 2a )a + (1 − 2a )a .
+ -1
a b
∴ a=
(1)  1 + 2a 1 − 2a 
1 1
This chord will pass through the point  , if
+
a2 b2  2 2 
Vertices of the triangle are (0, 0), (α, 0) and (0, β),
æ 1 + 2a ö æ 1 - 2a ö (1 + 2a) (1 + 2a)
4a ç - 4a ÷÷ -
∴  if the circumcentre is (γ, δ) then
ç 2 ÷÷ çç
è ø è 2 ø 2
a b (1 - 2a) (1 - 2a)
g = and d = . -
2 2 2
Circles  11.39


= 8a 2 - 2 2aa
1
⇒ 2a éë1 + 2a - 1 + 2a ùû - 2 éë(1 + 2a) + (1 - 2a) ùû
2 2

= 8a 2 - 2 2aa

1
⇒ 4 2aa - é 2(1) 2 + 2( 2a) 2 ù = 8a 2 - 2 2aa

2ë û

[ ∵ (a + b)2 + (a - b)2 = 2a2 + 2b2]



⇒ 8a 2 − 6 2a a + 1 + 2a 2 = 0.

2

This quadratic equation will have two distinct roots if æa + b ö 2  (1)
r 2 = AC 2 = CL2 + LA2 = ç ÷ + (3 2 )
(6 2a) 2 − 4(8)(1 + 2a 2 ) > 0. è 2 ø

Since OP = 4 2,   ∴  OC = CP + PO
2 2 2
⇒ 72a2 - 32 (1 + 2a2) > 0 ⇒ 9a2 - 4 - 8a2 > 0
⇒ a2 - 4 > 0 ⇒ (a - 2) (a + 2) > 0
⇒ a 2 + b 2 = r 2 + ( 4 2 ) 2 (2)
⇒ a∈ (-∞, -2) ∪ (2, ∞).
2
54. Let x2 + y2 + 2g x + 2f y + c = 0 be the variable circle. æa + b ö
From (1), and (2), we get a 2 + b 2 = ç
÷ + 18 + 32
Since it touches the given circles externally è 2 ø


( − g − 0) 2 + ( − f − 0) 2 = g 2 + f 2 − c + a  (1) (a + b )
2

⇒ (a + b ) - = 50 ⇒ (α - β)2 = 100
2 2

2

and, ( − g − 2a) 2 + ( − f − 0) 2 = g 2 + f 2 − c + 2a  (2)
⇒ α - β = ±10
(3)

Subtracting (1) from (2), we get
a -b (a - b ) 2

Also, CP = r ⇒ r = ⇒ r =
2
(4)
( g + 2a) + f = g + f + a.
2 2 2 2 2 2


Squaring both sides, we get From (3) and (4), we get r 2 = (5 2 ) 2
⇒ r = (5 2 ) 2 .

( g + 2a) 2 + f 2 = a 2 + g 2 + f 2 + 2a g 2 + f 2 Substituting r = 5 2 in (1), we get

HINTS AND EXPLANATIONS


2
æa + b ö
⇒ 4 ag + 4 a 2 = a 2 + 2a g 2 + f 2

(5 2 ) 2 = ç ÷ + 18
è 2 ø
⇒ (4g + 3a)2 = 4 (g2 + f 2) 2

or, (-4 (-g) + 3a)2 = 4 [(- g)2 + (- f  )2]. ⇒ 32 = (a + b ) ⇒ (α + β)2 = 64



2
∴ Locus of centre (-g, -f ) is (- 4x + 3a)2 = 4 (x2 + y2)
⇒ α + β = ± 8
(5)
or, 12x2 - 4y2 - 24ax + 9a2 = 0.

Form (3) and (5), we get
55. Let the lines cuts the x-axis at A and B, then

α = 9, β = -1 or α = -9, β = 1
1 or α = 1, β = -9 or α = -1, β = 9.


OA = - and OB = -3.
l
Since Q(-10, 2) lies in the interior of the circle,

Also, if the lines cut the y-axis at C and D, then ∴ CQ must be less than 5 2.

3
Thus, centre of the circle must be (-9, 1).

OC = 1 and OB = . ∴ The equation of the required circle is

2

Now, if the circle passes through A, B, C and D then
( x + 9) 2 + ( y − 1) 2 = (5 2 ) 2
⇒ x2 + y2 + 18x - 2y + 32 = 0.

æ 1ö 3
OA × OB = OC × OD Þ ç - ÷ ( -3) = 1 ´ ⇒ λ = 2.

è lø 2 57. Let P be the foot of ⊥ from origin O on any chord of the
circle S whose coordinates are (α, β). Then, the slope of OP
56. Let C(α, β) be the centre of the circle touching OP at P and
b a
is and thus the slope of chord is - and its equation
making intercept AB = 6 2 on the line x + y = 0 as shown a b
in the figure. If r is the radius of the circle then passing through (α, β) is
11.40  Chapter 11

a 9 225
y - b = - ( x - a)   ⇒  βy - β2 = - ax + a2 ⇒ x 2 + y 2 + 6 x − 3 y + 9 +
− =0
b 4 4
⇒ ax + βy = a2 + β 2 

(1) ⇒ x2 + y2 + 6x - 3y - 45 = 0.

Now, homogenizing the equation of the given circle 59. Centre of the circle x2 + y2 = 9 is (0, 0) and any tangent to the

x2 + y2 + 2g x + 2f y + c = 0 circle is

with the help of (1), we get x cosα + y sinα = 3 (1)
2 Its distance from centre (0, 0) is equal to radius 3.
æax + b y ö æax + b y ö
x 2 + y 2 ( 2 gx + 2 fy ) ç 2 2 ÷
+ cç 2 2 ÷
= 0. (2) Any tangent to x2 + y2 = 9 but ⊥ to (1) is obtained by replac-
èa +b ø èa +b ø ing α by (α - 90º) and its equation is
Now, equation (2) represents a pair of straight lines passing x cos(α - 90º) + y sin(α - 90º) = 3
through origin. These lines will be at right angle if sum of the or, x cos(90º - α) - y sin(90º - α) = 3
coefficients of x2 and y2 is zero.
or, x sinα - y cos α = 3 (2)
i.e., (a2 + b2)2 + (a2 + b2)2 + 2gα (a2 + b2)
Squaring and adding (1) and (2) we get x2 + y2 = 18 which is
+ 2β f (a2 + b2) + c (a2 + b2) a circle concentric with the given circle.
⇒ 2 (a2 + b2) + 2gα + 2f β + c = 0 (3) ∴ Locus is S ≡ x2 + y2 - 18 = 0 (3)
From equation (3), the locus of P(α, β) is
Equation of tangent to (3) at ( 2 , 4) is

2(x2 + y2) + 2gx + 2f y + c = 0
which is the required locus. p ≡ 2 x + 4 y − 18 = 0.
58. Given equation of line is
∴ System of coaxal circles is S + lP = 0.
x2 + 2xy + 3x + 6y = 0 (1)
60. Let P be the point on the line
⇒ x(x + 2y) + 3(x + 2y) = 0 ⇒ (x + 2y) (x + 3) = 0
2x - y + 11 = 0 (1)
So, equations of normals are x + 3 = 0 (2)
which is nearest to the circle
and, x + 2y = 0 (3)
3
Solving (2) and (3), we get x = -3, y = .

2

∴ Coordinates of centre of the circle are  −3,  .


3

 2
HINTS AND EXPLANATIONS

Given equation of circle is x(x - 4) + y(y - 3) = 0



⇒ x2 + y2 - 4x - 3y = 0.
(4)
1 25
2 x2 + y2 + 2x − y− = 0 (2)
 −3  5 2 8
Radius of the circle = ( −2) +   = and coordinates
2

 2 2
 1
with centre C  −1,  .

 3  4
of centre are  2,  .
 2
Then, CP is ⊥ to the line (1) and CP > radius.


Since the required circle is just sufficient to contain the circle

[Note that if CP ≤ r, the line intersects or touches the cir-
(4), therefore the distance between the centres
cle and then the point of intersection or point of contact are
 3  3 required points]

 3,  and  2,  = the difference of their radius.
2  2 1
−2−
+ 11

Let the radius of the required circle be a. 4 35 67
Here, CP =
= > (radius).
2
5 4 5 4
 3 3 5 5


( −3 − 2) 2 +  −  = a − ⇒ 5=a− Now, equation of CP [⊥ to line (1)] is
 2 2 2 2
1 1
5 15
x + 2y = λ, where -1 + = l = λ or l = - .
⇒ a = 5+ = . 2 2
2 2
∴ Equation of CP is 2x + 4y + 1 = 0
(3)

Therefore, equation of required circle is
Solving (1) and (3), we get y = 2, x = -9/2.

2 2

 3  15   –9 
( x + 3) 2  y −  =  
Hence, the required point is  , 2 .
 2  2  2 
Circles  11.41

61. Extremities of the diagonal OA of the rectangle are O(0, 0)


and A (4, 3). Then, OA is the diameter of the circumcircle, so
equation of the circumcircle is
x(x - 4) + y(  y - 3) = 0 i.e., x2 + y2 - 4x - 3y = 0
2 2

i.e., ( x − 2) 2 +  y −  =   (1)
3 5

 2  2

3
m = slope of OA =
(2)
4
∴ Tangents parallel to the diagonal OA are

y1 − 12 4

× − = −1 ⇒ 7x - 4y + 104 = 0 (3)
3 3 5 9 x1 + 8 7 1 1

y− = ( x − 2) ± 1+
2 4 2 16  x − 8 y1 + 12 
and mid point of C1C2 i.e.,  1
,  lie on (2)
i.e., 6x - 8y ± 25 = 0.  2 2 
62. Let the coordinates of C be (x1, y1) and the coordinates of A
and B be (0, 0) and (a, 0), respectively. i.e., 4  x1 − 8  + 7  y1 + 12  + 13 = 0

 2   2 
sin A BC
Given, k =
= or, 4x1+ 7y1 + 78 = 0 (4)
sin B AC
Solving (3) and (4), we get (x1, y1) ≡ (-16, -2).
∴ Equation of the imaged circle is
(x + 16)2 + (y + 2)2 = 52

or, x2 + y2 + 32x + 4y + 235 = 0.
64. Let r and R be radius of required and given circle, respec-
tively and let centre is (h, k), By given condition

HINTS AND EXPLANATIONS


⇒ BC2 = k2 AC2

⇒ (x1 - a)2 + y12 = k2 (x12 + y12)

⇒ (1 − k 2 ) x12 + (1 − k 2 ) y12 − 2ax1 + a 2 = 0


2ax1 a2
⇒ x12 + y12 −
+ = 0, [ k ≠ 1]
\
x
1− k2 1− k2

Hence, locus of C is ( h − 1) 2 + ( k − 2) 2 = R − r

2a a2 r
x2 + y2 − + = 0,
Now, = tan 30°
1− k 2
1− k2 AB
1
 a  r = AB tan 30° = ( R − r ) (AB = R - r)

which is a circle whose centre is  ,0
 1 − k 2  3

R  3 
a2 a2 ak ⇒
( h − 1) 2 + ( k − 2) 2 = R − = R

and radius = − = . 
(1 − k )
2 2
(1 − k 2 ) 1 − k 2 1+ 3  1+ 3 

63. The given circle and line are Now, R = 1 + 3



x2 + y2 + 16x - 24y + 183 = 0 (1)

( h − 1) 2 + ( k − 2) 2 = 3
and, 4x + 7y + 13 = 0 (2)
Centre and radius of circle (1) are C1(-8, 12) and 5 respec- ∴ Locus is (x - 1)2 + (y - 2)2 = 3
tively. Let the centre of the imaged circle be C2 (x1, y1). 65. The point of intersection of the tangents at the points P(q1)
Then, slope of C1C2 × slope of given line and Q(q2) on the circle x2 + y2 = 1 is given by
11.42  Chapter 11

æq +q ö æq +q ö ∴1<a<2

cos ç 1 2 ÷ cos ç 1 2 ÷
There can be symmetrical points on the negative x-axis too.
è 2 ø= è 2 ø
x= Hence, we have a ∈ (-2, -1) ∪ (1, 2).
æ q1 - q 2 ö æ 60° ö
cos ç ÷ cos ç ÷
è 2 ø è 2 ø 69. Let P ≡ (a cos α, a sin α) and Q ≡ (a cos β, a sin β), where
β - α = 2θ
æq +q ö æq +q ö
a sin ç 1 2 ÷ sin ç 1 2 ÷ Also, A ≡ (a, 0) and B ≡ (-a, 0)
è 2 ø è 2 ø
y = = If R(h, k) be the intersection point of AP and BQ, the slope of
æ q1 - q 2 ö æ 60° ö
cos ç ÷ cos ç ÷ AR = slope of AP
\
[ R is lies on AP]
è 2 ø è 2 ø
⇒ (x cos 30º) + (y cos 30º) = 1
2 2 k sin a æa ö a - h

= Þ tan ç ÷ = (1)
3 4 h - a cos a - 1 è2ø h
⇒ ( x 2 + y 2 )
= 1 ⇒ x2 + y2 =
4 3 k sin b æb ö k

= Þ tan ç ÷ = (2)
⇒ 3x2 + 3y2 = 4
h + a cos b + 1 è 2 ø h+a
66. Centre is (1, -2). Radius = 1 + 4 − 3 = 2 . Since the sides b a
Since, β - α = 2θ, we have
- =q
are parallel to coordinate axes, vertices do not lie on horizon- 2 2
tal and vertical lines through. (1, 2). ∴ the given points are
not vertices. æb ö æa ö
tan ç ÷ - tan ç ÷
è2ø è 2 ø = tan q


æ b ö æa ö
1 + tan ç ÷ tan ç ÷
è2ø è2ø
k a-h
-

h + a k = tan q
æ k öæ a - h ö
1+ ç ÷ ç ÷
è h + a øè k ø
⇒ h2 + k2 - 2ak tanθ = a2
67. Let (2, 3) be given point, M be the middle point of a chord
Hence, the locus of R is x2 + y2 -2ay tan q = a2.
of the circle x2 + y2 = a2 through P. Then, the distance of the
centre O of the circle from the chord is OM. and (OM)2 = 70. We choose the centre O of the circle as the origin and the
(OP)2 - (PM)2 which is maximum when PM is minimum, lines OA, OB as the x-axis and the y-axis, respectively. If a
i.e., P coincides with M, the middle point of the chord. be a radius of the given circle, then
HINTS AND EXPLANATIONS

A ≡ (a, 0) and B ≡ (0, a)


and a variable point on the circle
P ≡ (a cosθ, a sinθ)


Hence, the equation of the chord is T = S1.
i.e., 2x + 3y - a2 = (2)2 + (3)2 -a2 ⇒ 2x + 3y = 13.

p
68. We have <q <p
3


If (h, k) be the coordinates of the centroid of triangle PAB,
then we have
3h = a (1 + cosθ)(1)

and, 3k = a (1 + sinθ)(2)

Eliminating θ from equations (1) and (2), we have

p q p (3h - a)2 + (3k - a)2 = a2.

1 æq ö
< < Þ < sin ç ÷ < 1
Putting (x, y) in place of (h, k) gives the equation of the
6 2 2 2 è2ø
required locus as
é æq ö 1ù
⇒ 1 < 1 < 1
ê∵ sin ç ÷ = ú (3x - a)2 + (3y - a)2 = a2, which is a circle.

2 a ë è 2 ø aû
Circles  11.43

71. The given expression can be written as


Eliminating r from equations (1) and (2), we have
6 (l2 + m2) = 9l2 + 6l + 1
OP + AP = b + c
3l + 1 i.e., sum of distances of P from two fixed points O and A =


= 6 constant. Hence, P lies on an ellipse having foci at O and A.
l 2 + m2
⇒ the perpendicular distance of the point (3, 0) from the line
73. We have,
lx + my + 1 = 0 is 6. S(x, 2) = 0 gives two identical solutions x = 1
Therefore, the given line is a tangent to the circle ⇒ line y = 2 is a tangent to the circle S(x, y) = 0 at the point
(x - 3)2 + y2 = 6. (1, 2)
72. We choose centre of S1 as the origin and the line joining the and, S(1, y) = 0 gives two distinct solutions y = 0, 2
centres of S1, S2 as the x-axis. Let the centre of S2 be A(a, 0) and ⇒ line x = 1 cuts the circle S(x,y) = 0 at the points, (1, 0) and
b, c(b > c) be the radii of S1, S2, respectively. If P(h, k) be the (1.2)
centre of the variable circle S and r be its radius then we have Clearly, from the fig., the points A(1, 2)
OP + r = b and B(1, 0) are diametrically opposite points.
i.e., h2 + k 2 = b − r 
(1) Thus, equation of the circle, is

and, AP = r + c (x -1)2 + y(y -2) = 0

i.e.,
( h − a) + k = r + c (2)
2 2

i.e.,  x2 + y2 - 2x -2y + 1 = 0.

Previous Year's Questions

HINTS AND EXPLANATIONS


74. Equation of circle is Again when r = 8, they intersect at only one point and for r >
x2 + y2 - 4x - 2y - 20 = 0. 8, no intersection point can be found.
whose centre is C(2, 1) and radius is 5 units. 77. Intersection of diameters is the point (1, -1)
Since S1 = 102 + 72 - 4 × 10 - 2 × 7 - 20 > 0 Also,  πs2 = 154
So P lies outside the circle. ⇒ s2 = 49
∴  Equation becomes (x - 1)2 + (y + 1)2 = 49
Now, PC = ( 2 - 10) 2 + (1 - 7) 2

78. Let the equation of the circle be x2 + y2 + 2gx + 2fy + c = 0

= 82 + 6 2 = 10 2 = 10 then c = 4 and it passes through (a, b)  ⇒  a2 + b2 + 2ga +
2fb + 4 = 0.
∴  Greatest distance between circle and the point P = 10 +
5 = 15 unit. Hence locus of the centre is 2ax + 2by - (a2 + b2 + 4) = 0.
75. Equation of circle is x2 + y2 + 4x - 4y + 4 = 0 with center at 79. Let the other end of diameter is (h, k) then equation of circle
(-2, 2). and radias 2. The equation of tangent is x + y = 2 2. will be (x - h) (x - p) + (y - k)(y - q) = 0 Put y = 0 (since the
76. (x - 1)2 + (y - 3)2 = r2 circle touches the x-axis)
(x - 4)2 + (y + 1)2 - 16 - 1 + 8 = 0 ⇒  x2 - (h + p)x + (hp + kq) = 0  ⇒  (h + p)2 = 4(hp + kq)
(D = 0)
(x - 4)2 + (y + 2)2 = 9.
⇒ (x - p)2 = 4qy.
Distance between their centers = 5
If the circle with center at (1, 3) touches the second circle 80. The intersection of the given lines is the centre of the circle
i.e., (1, -1)
then the radius of the first circle = 2.
And circumference = 10π  ⇒ radius r = 5
But it is given that they touch at two distinct points so r > 2
as long as r < 8. ⇒  Equation of circle is x2 + y2 - 2x + 2y - 23 = 0.

11.44  Chapter 11

81. Points of intersection of line y = x with x2 + y2 - 2x = 0 are (0, 86. The locus (h, k) is given by
0) and (1, 1).
p h2 + k 2 9
Hence equation of circle having end points of diameter (0, 0) cos = Þ h2 + k 2 =
and (1, 1) is x2 + y2 - x - y = 0. 3 3 4

82. Let S1 : x2 + y2 + 2ax + cy + a = 0 87. According to question


The equation of circle is (x − h)2 + (y − k)2 = k2
S2 : x2 + y2 − 3ax + dy − 1 = 0

Since, it passes through point (−1, 1) we have
Equation of radical axis of S1 and S2 is S1 − S2= 0

(−1 −h)2 + (1 − k)2 = k2
⇒ 5ax + (c − d)y + a + 1 = 0

⇒  h2 + 2h − 2k + 2 = 0
Given that 5x + by − a = 0 passes through P and Q

Now, D ≥ 0
a c - d a +1 ⇒  2k − 1 ≥ 0
Þ = =
1 b -a 1
⇒ a + 1 = −a2
⇒ k ³
2
⇒ a2 + a + 1 = 0
88. Centre (−1, −2)

No real value of a. Let (α, β) be the required point, then
83. Equation of circle with centre (0, 3) and radius 2 is x2 + (y a +1 b +0
= -1 and = -2
− 3)2 = 4. 2 2
Let locus of the variable circle be (α, β) 89. Let the given circles be
∵   It touches x-axis. S: x2 + y2 + 3x + 7y + 2p − 5 = 0
∴  Its equation is (x − α)2 + (y − β)2 = b2 S′: x2 + y2 + 2x + 2y − p2 = 0
Circles touch externally Now the equation of the required circle is S + λS ′ = 0
\ a 2 + ( b - 3) 2 = 2 + b - ( 7 + 2 p)
As it passes through (1, 1), the value of l =
(6 - p 2 )
∴  Locus is x 2 = 10 æç y - ö÷ which is a parabola.
1 If 7 + 2p = 0, it becomes the first circle
è 2ø ∴  it is true for all values of p
90. Given points P(1, 0) and Q(−1, 0).
Let A = (x, y), then
HINTS AND EXPLANATIONS

AP BP CP 1
= = = (1)
AQ BQ CQ 3
⇒  3AP = AQ  ⇒ 9AP2 = AQ2  ⇒  9(x − 1)2 + 9y2 = (x + 1)2
+ y2
⇒  9x2 − 18x + 9 + 9y2 = x2 + 2x + 1 + y2  ⇒ 8x2 − 20x + 8y2
+8 = 0
5
Þ x 2 + y 2 - x + 1 = 0 (2)
2
∴  A lies on the circle
84. Let the centre be (α, β)
Similarly B, C are also lies on the same circle
∵   It cuts the circle x2 + y2 = p2 orthogonally
∴  Circum centre of ABC = Centre of Circle (1) = æç , 0 ÷ö
5
We write 2(−α) × 0 + 2 (−β) × 0 = c1 − p2
è4 ø
⇒  c1 = p2 91. Circle x2 + y2 − 4x − 8y − 5 = 0
Let equation of circle be x2 + y2 − 2αx − 2βy + p2 = 0
Centre = (2, 4), Radius = 4 + 16 + 5 = 5
It passes through (a, b) therefore a + b − 2αa − 2βb + p = 0
2 2 2 If circle is intersecting line 3x − 4y = m at two distinct points.
∴  The locus is 2ax + 2by − (a2 + b2 + p2) = 0. ⇒  length of perpendicular from centre < radius
| 6 - 16 - m |
85. The point of intersection of 3x − 4y − 7 = 0 and 2x − 3y − 5 Þ <5
= 0 is (1, −1), which is the centre of the circle and radius of 5
the circle = 7. ⇒ |10 + m|< 25
∴  Equation is (x − 1) + (y + 1) = 49 ⇒ x + y − 2x + 2y
2 2 2 2 ⇒ −25 < m + 10 < 25
− 47 = 0. ⇒ −35 < m < 15.
Circles  11.45

æa ö Note: Centroid divides the orthocenter and circumcentre in


92. c1 = ç , 0 ÷ ; c2 = (0, 0) the ratio 2 : 1
è2 ø
a C
r1 = ; r2 = c
2 A 2 B 1 (x, 6y)
a a (–3, 5) (3, 3)
c1c2 = r1 - r2 Þ =c- Þ c=a
2 2
2× x − 3
93. Let (h, k) be centre.
3= (i)
2 +1
(h − 1)2 + (k − 0)2 = k2  ⇒  h = 1

 [calculating by ratio method]
5
(h − 1)2 + (k − 3)2 = k2  ⇒  k = 2× y + 5
3
3= (ii)
10
∴  diameter is 2k = 3
3
∴ x = 6

94. Assume that the equation of circle be

y=2
(x − 3)2 + y2 + λy = 0
C = (6, 2)
The circle passes through (1, −2)
⇒  4 + 4 − 2λ = 0  ⇒  λ = 4 ∴
AC = ( −3 − 6) 2 + (5 − 2) 2 = 81 + 9 = 3 10
(x − 3)2 + y2 + 4y = 0  ⇒  Clearly (5, −2) satisfies.
AC 3
95. According to the figure Radius =
= 10
2 2
(1 + y)2 = (1 − y)2 + 1 (y > 0)
1 99. Equation of tangent at (1, 7) to the curve x2 = y – 6
Þ y = 1
4 = x1 ⋅ x = ( y + y1 ) − 6 where x1 = 1 and y1 = 7
96. c1(2, 3); 2

r = 4 + 9 + 12 = 5 1
1 ⇒ x = ( y + 7) − 6

2
And c2 (-3, -9);
⇒ 2x = y + 7 – 12

r = 9 + 81 - 26 = 8 ⇒ 2x – y + 5 = 0 → Tangent equation

2
∴  Distance c1c2 = 25 + 144 = 13
Consider the equation of circle
∴  c1c2 = r1 + r2 touching externally.
x2 + y2 + 16x + 12y + c = 0

HINTS AND EXPLANATIONS


⇒  3 common tangents.
x2 + 16x + 64 + y2 + 12y + 36 = 100 – c
97.
⇒ ( x + 8) 2 + ( y + 6) 2 = ( 100 − c ) 2

(2, −3) 5
Radius is 100 − c and centre (–8, –6)

5√ 2 5√ 3 We can calculate the radius of tangent 2x – y + 5 = 0 (Normal

Distance of Tangent From Centre)
(−3, 2)
2 × ( −8) − ( −6) + 5 −16 + 11
r= = = 5
2
2 +1 5
S
98. Ortho Centre, A = (–3, 5) Given, 100 − c = 5

Centroid, B = (3, 3) 100 – c = 5

Circumcentre, C = (x, y) c = 95
(Proved)
Diameter, AC = ?
This page is intentionally left blank.
CHAPTER Conic Sections (Parabola,
12 Ellipse and Hyperbola)
LEARNING OBJECTIVES

After reading this chapter, you will be able to:


 
Be familiar with conic section and its types, general  
Learn about parabola, standard equation of parabola,
equation of conic equation of a chord, point of intersection of tangents

CONIC SECTION 6. Double Ordinate A chord of the conic which is per-


pendicular to the axis is called the double ordinate of the
A conic section or conic is the locus of a point which conic.
moves in a plane in such a way that its distance from a
fixed point bears a constant ratio to its distance from a fixed REMARK
straight line. The curves defined above are called conic sections, because
The fixed point is called the focus and the fixed line is these are obtained when a right circular cone is cut by a
called the directrix of the conic. The constant ratio is called plane in various ways.
the eccentricity of the conic and is denoted by e. If
 e = 1, the conic is called Parabola.
 e < 1, the conic is called Ellipse.
 e > 1, the conic is called Hyperbola.
 e = 0, the conic is called Circle.
 e = ∞, the conic is called pair of straight lines.

IMPORTANT TERMS
1. Axis The straight line passing through the focus and per-
pendicular to the directrix of the conic is known as its axis.
2. Vertex A point of intersection of a conic with its axis is
FIGURE 12.1
known as a vertex of the conic.
3. Centre The point which bisects every chord of the conic
SECTION OF A RIGHT CIRCULAR CONE BY
passing through it, is called the centre of the conic.
4. Focal Chord A chord passing through the focus is DIFFERENT PLANES
known as focal chord of the conic. 1. Section of a right circular cone by a plane passing through
5. Latus Rectum  The focal chord which is perpendicular its vertex is a pair of straight lines passing through the
to the axis is known as latus rectum of the conic. vertex as shown in the figure.
12.2  Chapter 12

EQUATION OF CONIC
Let S(h, k) be the focus and QN be the directrix whose
equation is Ax + By + C = 0

FIGURE 12.2
2. Section of a right circular cone by a plane parallel to its
base is a circle as shown in the figure.

FIGURE 12.7
Let P(x, y) be any point on the conic. From P, draw PN ⊥
QN. If e is the eccentricity of the conic, then by definition
PS
= e Þ PS 2 = e 2 PN 2
PN
FIGURE 12.3
2
3. Section of a right circular cone by a plane parallel to a æ Ax + By + C ö
or (a - a ) + ( y - b ) = e ç
2 2 2
÷
generator of the cone is a parabola as shown in the figure. è A +B ø
2 2

This is the cartesian equation of the conic which, on sim-


plification, takes the form
ax2 + 2hxy + by2 + 2gx + 2fy + c = 0
where a, b, c, f, g and h are constants.
Thus, the equation of a conic is an equation of second
degree in x and y.

FIGURE 12.4 GENERAL EQUATION


4. Section of a right circular cone by a plane not parallel to The equation of conics is represented by the general equa-
any generator of the cone and not perpendicular or par- tion of second degree ax2 + 2hxy + by2 + 2gx + 2fy + c = 0
allel to the axis of the cone is an ellipse as shown in the (1) and discriminant of above equation is represented by Δ
figure.
where Δ = abc + 2fgh – af  2 – bg2 – ch2
Case I: When Δ = 0
In this case equation (1) represents the degenerate conic
whose nature is given in the following table

Table 12.1
FIGURE 12.5
S. No. Condition Nature of conic
5. Section of a right circular cone by a plane parallel to the
1. Δ = 0 and ab – h 2
A pair of coincident straight
axis of the cone is a hyperbola as shown in the figure.
=0 lines
2. Δ = 0 and ab – h2 A pair of intersecting straight
<0 lines
3. Δ = 0 and ab – h2 A point
>0

Case II: When Δ ≠ 0


In this case equation (1) represents the non-degenerate
FIGURE 12.6 conic whose nature is given in the following table:
Conic Sections (Parabola, Ellipse and Hyperbola)  12.3

Table 12.2
S. No. Condition Nature of conic
1. Δ ≠ 0, h = 0, a = b, e = 0 A circle
2. Δ ≠ 0, ab – h2 = 0, e = 1 A parabola
3. Δ ≠ 0, ab – h > 0, e < 1
2
An ellipse
4. Δ ≠ 0, ab – h < 0, e > 1
2
A hyperbola
5. Δ ≠ 0, ab – h2 < 0, a + b = 0, A rectangular
e = √ 
•2 hyperbola FIGURE 12.9

Let S be the focus, QM be the directrix and P be any point


CENTRE OF CONIC on the parabola. Then, by definition, PS = PM where PM is
Centre of the conic is the point which bisect every chord the length of the perpendicular from P on the directrix QM.
of the conic passing through it. If ax2 + 2hxy + by2 + 2gx +
2fy + c = 0 is the equation of the conic section and if C is
SOME TERMS RELATED TO PARABOLA
its centre, then the centre of the conic is
Let y2 = 4ax be the standard parabola, having focus S at
æ hf - bg gh - af ö (a, 0) and directrix represented by the line MZ having the
Cç , 2 ÷
è ab - h ab - h ø
2
equation x + a = 0 (see figure). Then for such a parabola,
the following important terms can be defined.
SHORT-CUT METHOD
Let S ≡ ax + 2hxy + by2 + 2gx + 2fy + c = 0 be the given
2

conic.

FIGURE 12.8
 Differentiate S partially with respect to x treating y as con-
stant, we get
¶S FIGURE 12.10
= 2ax + 2hy + 2g.
¶x
 Differentiate S partially with respect to y treating x as con- AxIs Axis of the conic section is the line passing
stant, we get through the focus and perpendicular to the
directrix.
¶S
= 2hx + 2by + 2f . For the Parabola, y2 = 4ax, the line y = 0 (i.e.
¶y
x-axis) is the AXIS.
 Equating these two equations to zero and solving for x
VERTEX The point A(0, 0) is the point of intersection
and y, we get the coordinates of centre as
of the parabola and its axis, hence, A is called
æ hf - bg gh - af ö the VERTEX of the parabola.
( x, y ) = ç , 2 ÷
è ab - h ab - h ø
2
Double Through the point Q (a , 2 aa ) on the
Ordinate parabola, a perpendicular drawn to the axis of
the parabola such that it meets the other end
of the curve at Q¢(a , - 2 aa ). Then the line
PARABOLA QQ′ is called Double Ordinate.
A parabola is the locus of a point which moves in a plane such  Now, length of Double Ordinate
that its distance from a fixed point (called the focus) is equal = QQ¢ = 2(2 aa ) = Double (Ordinate part)
to its distance from a fixed straight line (called the directrix). Latus The double ordinate passing through the
12.4  Chapter 12

Rectum focus is called Latus Rectum. Length of Latus


Rectum = LL′ = 2(2a) = 4a Info Box!
Focal Since, the chord of the parabola passing  The vertex of the parabola bisects the join of focus
Chord through the focus is called the focal chord of and the point of intersection of directrix with the axis.
the parabola. In this case, the line RR′ is the  Two parabolas are said to be equal when their latus
focal chord of the parabola. recta are equal.
Focal Distance of any point P on the parabola from æ length of perpendicular ö
 Length of latus rectum = 2
ç ÷
Distance the focus is called focal distance è from focus to directrix ø
 ∴  Focal Distance = |SP| = |PM| i.e., Distance
of P from Directrix
⇒ |SP| = x − a

FOUR STANDARD FORMS OF THE PARABOLA


Standard y2 = 4ax (a > 0) y2 = – 4ax (a > 0) x2 = 4ax (a > 0) x2 = – 4ax (a > 0)
Equation
Shape of the
parabola

Vertex A(0, 0) A(0, 0) A(0, 0) A(0, 0)


Focus S(a, 0) S(– a, 0) S(0, a) S(0, – a)
Equation of x = –a x=a y = –a y=a
directrix
Equation of axis y=0 y=0 x=0 x=0
Length of latus 4a 4a 4a 4a
rectum
Extermities of
latus rectum (a, ±2a) (–a, ±2a) (±2a, a) (±2a, –a)
Equation of
latus rectum x=a x = –a y=a y=–a
Equation of
tangents at x=0 x=0 y=0 y=0
vertex
Focal distance of
a point P(x, y) x+a x–a y+a y–a
Parametric (at , 2at)
2
(–at , 2at)
2
(2at, at )
2
(2at, –at2)
coordinates
Parametric x = at2 x = –at2 x = 2at x = 2at
Equations y = 2at y = –2at y = at2 y = –at2
Eccentricity (e) 1 1 1 1
Conic Sections (Parabola, Ellipse and Hyperbola)  12.5

QUICK TIPS Condition for tangency and point of contact  The line y
a
 y2 = 4a(x + a) is the equation of the parabola whose = mx + c touches the parabola y2 = 4ax if c = and the
m
focus is the origin and the axis is x-axis.
coordinates of the point of contact are æç 2 ,
a 2a ö
 y = 4a(x – a) is the equation of parabola whose axis is
÷.
2

x-axis and y-axis is directrix. èm m ø


 x = 4a(y + a) is the equation of parabola whose focus
2

is the origin and the axis is y-axis.


POINT OF INTERSECTION OF TANGENTS
 x = 4a(y – a) is the equation of parabola whose axis is The point of intersection of tangents drawn at two differ-
2

y-axis and the x-axis is directrix. ent points of contact P ( at12 , 2at1 ) and Q( at 22 , 2at 2 ) on the
 The equation of the parabola whose vertex and focus parabola y2 = 4ax is
are on x-axis at a distance a and a′ respectively from the
origin is y2 = 4(a′ – a)(x – a).
 The equation of the parabola whose axis is parallel to

x-axis is x = Ay2 + By + C and y = Ax2 + Bx + C is a parab-


ola with its axis parallel to y-axis.

INTERSECTION OF A LINE AND A PARABOLA


The line y = mx + c does not intersect, touches or intersect
a
a parabola y2 = 4ax according as c ³ =, < .
m FIGURE 12.11

EQUATION OF A CHORD R ≡ [at1t2, a(t1 + t2)].


1. The equation of chord joining the points (x1, y1) and (x2,
y2) on the parabola y2 = 4ax is
QUICK TIPS
y(y1 + y2) = 4ax + y1y2
Angle between tangents at two points P(at12 , 2at1) and
2. The equation of chord joining the points ( at12 , 2at1 ) and 

Q(at22 , 2at2 ) on the parabola y2 = 4ax is:


( at 22 , 2at 2 ) is
y(t1 + t2) = 2(x + at1t2) t2 - t1
q = tan-1
3. Length of the chord y = mx + c to the parabola 1 + t1t2
4 The G. M. of the x-coordinates of P and Q
y2 = 4ax is given by 1 + m2 a( a - mc) 

m2 (i.e., at12 ´ at22 = at1t2 ) is the x-coordinate of the point


of intersection of tangents at P and Q on the parabola.
Condition for the chord to be a focal chord  The chord  The A.M. of the y-coordinates of P and Q
joining the points ( at12 , 2at1 ) and ( at 22 , 2at 2 ) passes é 2at1 + 2at2 ù
êëi.e., = a(t1 + t2 )ú is the y-coordinate of
through focus provided t1t2 = –1. 2 û
Length of focal chord  The length of a focal chord joining the point of intersection of tangents at P and Q on the
the points ( at12 , 2at 2 ) and ( at 22 , 2at 2 ) is  (t2 – t1)2. parabola.
 The orthocentre of the triangle formed by three tangents

to the parabola lies on the directrix.


QUICK TIPS
 The locus of the point of intersection of tangents to the
 The length of the focal chord through the point ‘t’ on the parabola y2 = 4ax which meet at an angle α is
parabola y2 = 4ax is a(t + 1/t)2. (x + α)2 tan2α = y2 – 4ax
 The extremitics of focal chord may be given by (at , 2at)
2
 The tangents to the parabola y = 4ax at P(at1 , 2at1) and
2 2

æa 2a ö 2
Q(at2 , 2at2 ) intersect at R. Then the area of triangle PQR
and ç 2 , - ÷
èt t ø
1 2
 The slope of the chord joining the points ‘t ’ and ‘t ’ is is a (t1 - t2 )3 .
1 2 2
2
 If the straight line lx + my + n = 0 touches the parabola y
2
t1 + t2
= 4ax, then ln = am . 2
12.6  Chapter 12

2. Parametric Form The equation of the normal to the


x y parabola y2 = 4ax at the point (at2, 2at) is
 If the line + = 1 touches the parabola y2 = 4a(x + b)
l m y + tx = 2at + at3
then m2(l + b) + al2 = 0.
 If the two parabolas y2 = 4x and x2 = 4y intersect at point 3. Slope Form  The equation of normal to the parabola y2 =
P, whose abscissa is not zero, then the tangent to each 4ax in terms of slope ‘m’ is
curve at P, make complementary angle with the x-axis. y = mx – 2am – am3
 If the line x cos α + y sin a = p touches the parabola y2 =
4ax, then p cos α + a sin2α = 0 and the point of contact
is (a tan2α, –2a tan α).
Info Box!
 Tangents at the extremities of any focul chord of a parab- The coordinates of the point of contact are (am2, –2am).
ola meet at right angle on the directrix.
 Area of the triangle formed by three points on a parabola
is twice the area of the triangle formed by the tangents at Condition for Normality  The line y = mx + c is a normal
these points. to the parabola
If the tangents at the points P and Q on a parabola meet
y2 = 4ax if c = – 2am – am3

in T, then ST is the geometric mean between SP and SQ,


i.e., ST2 = SP ⋅ SQ. POINT OF INTERSECTION OF NORMALS
The point of intersection of normals drawn at two different
POSITION OF A POINT WITH RESPECT TO A
points of contact P ( at12 , 2at1 ) and Q( at 22 , 2at 2 ) on the
PARABOLA
parabola y2 = 4ax is
The point (x1, y1) lies outside, on or inside the parabola y2 =
4ax according as y12 - 4 ax1 >, = or < 0, , respectively.

NUMBER OF TANGENTS DRAWN FROM A


POINT TO A PARABOLA
Two tangents can be drawn from a point to a parabola. The
two tangents are real and distinct or coincident or imagi-
nary according as the given point lies outside, on or inside FIGURE 12.13
the parabola.
R º [2a + a(t + t 22 + t1t 2 ), -at1t 2 (t1 + t 2 )].
2
1

EQUATION OF THE PAIR OF TANGENTS


QUICK TIPS
The equation of the pair of tangents drawn from a point
P(x1, y1) to the parabola y2 = 4ax is SS1 = T 2, If the normal at the point P (at12 , 2at2 ) meets the
1. 
where S º y 2 - 4 ax, S1 º y12 - 4 ax1 parabola y2 = 4ax again at Q (at22 , 2at2 ), then
and T ≡ yy1 – 2a(x + x1) 2
t 2 = -t1 -
t1
Note that PQ is normal to the parabola at P and not at Q.

FIGURE 12.12

EQUATIONS OF NORMAL IN DIFFERENT


FORMS
1. Point Form  The equation of the normal to the parabola
y2 = 4ax at a point (x1, y1) is FIGURE 12.14

y1 If the normals at the points (at12 , 2at1) and (at22 , 2at2 )


2. 
y - y1 = - ( x - x1 ) meet on the parabola y2 = 4ax, then t1t2 = 2.
2a
Conic Sections (Parabola, Ellipse and Hyperbola)  12.7

CO-NORMAL POINTS SOLVED EXAMPLES


Any three points on a parabola normals at which pass
through a common point are called co-normal points 1. If the tangent to the parabola y2 = 4ax meets the axis
in T and tangent at the vertex A in Y and the rectangle
QUICK TIPS TAYG is completed, then the locus of G is
If three normals are drawn through a point (h, k), then their (A) y2 + 2ax = 0 (B)  y2 + ax = 0
slopes are the roots of the cubic: (C) x + ay = 0
2
(D)  none of these
k = mh – 2am – am3 Solution: (B)
 (i) The sum of the slopes of the normals at co-normal Let P(at2, 2at) be any point on the parabola y2 = 4ax.
points is zero, i.e., m1 + m2 + m3 = 0.
The equation of tangent at P is ty = x + at2.
(ii) The sum of the ordinates of the co-normal points is Since the tangent meets the axis of parabola in T
zero (i.e. – 2am1 – 2am2 – 2am3 = –2a(m1 + m2 +
and tangent at the vertex A in Y,
m3) = 0).
∴ coordinates of T and Y are (–at2, 0) and (0, at)
(iii) The centroid of the triangle formed by the co-normal
respectively.
points lies on the axis of the parabola [the vertices
of the triangle formed by the co-normal points are
(am12 , - 2am1), (am22 , - 2am2 ) and (am32 , - 2am3 ).
Thus, y-coordinate of the centroid becomes
-2a (m1 + m2 + m3 ) -2a
= ´0 = 0
3 3
Hence, the centroid lies on the x-axis, i.e. axis of the
parabola.]
(iv) If three normals drawn to any parabola y2 = 4ax from Let the coordinates of G be (x1, y1).
a given point (h, k) be real, then h > 2a. Since TAYG is a rectangle,
∴ midpoint of diagonals TY and GA is same
CHORD OF CONTACT x + 0 -at 2 + 0 y + 0 0 + at
Þ 1 = and 1 =
2 2 2 2
The equation of chord of contact of tangents drawn from a ⇒ x1 = – at2(1)
point P(x1, y1) to the parabola y2 = 4ax is T = 0 where T ≡ and y1 = at(2)
yy1 – 2a(x + x1). Eliminating t from (1) and (2), we get
2
æy ö
x1 = -a ç 1 ÷ Þ y12 + ax1 = 0
èaø
∴ The locus of G(x1, y1) is y2 + ax = 0.
2. Equation of the parabola whose vertex is (–3, –2), axis
FIGURE 12.15
is horizontal and which passes through the point (1, 2) is
(A) y2 + 4y + 4x – 8 = 0 (B)  y2 + 4y – 4x + 8 = 0
CHORD WITH A GIVEN MID POINT (C) y + 4y – 4x – 8 = 0
2
(D)  none of these
Solution: (C)
The equation of the chord of the parabola y2 = 4ax with
P(x1, y1) as its middle point is given by Since the axis is horizontal and vertex is (–3, –2),
∴ the equation of the parabola must be of the form
T = S1
( y + 2)2 = 4a (x + 3)
where T ≡ yy1 – 2a(x + x1) and S1 º y12 - 4 ax.
It passes through (1, 2), so 16 = 16a i.e. a = 1.
Hence, the equation of the required parabola is
( y + 2)2 = 4(x + 3) or y2 + 4y − 4x − 8 = 0
3. Two tangents are drawn from the point (–2, –1) to the
parabola y2 = 4x. If α is the angle between them, then
FIGURE 12.16
tan α =
12.8  Chapter 12

1 5. The length of the latus rectum of the parabola 25[(x–



(A)
3 (B)  2)2 + (y – 4)2] = (4x – 3y + 12)2 is
3
16 8
1 (A) (B) 

(C)
2 (D)  5 5
2
Solution: (A) 12
(C) (D)  none of these
Given parabola is y2 = 4x. 5
Here 4a = 4, ∴ a = 1 Solution: (B)
Equation of pair of tangents drawn from the point (–2, The given equation of the parabola can be written as
–1) to the parabola is SS1 = T 2 2
That is, ( y2 – 4x) [(–1)2 – 4(–2)] = [y(–1) –2 ⋅ 1(x – 2)]2 æ 4 x - 3 y + 12 ö
⇒ 9(y2 – 4x) = (–2x – y + 4)2 ( x - 2) + ( y - 4 ) = ç
2 2
÷
ç ( 4) 2 + ( -3) 2 ÷
⇒ 4x2 – 8y2 + 4xy + 20x – 8y + 16 = 0 è ø
Since α is the angle between the tangents, ∴ The coordinates of focus are (2, 4) and the equation
of directrix is 4x – 3y + 12 = 0.
2 h2 - ab 2 4 - 4( -8) The distance of the focus from the directrix
∴ tan a = = = |- 3| = 3
a+b 4 -8
4( 2) - 3( 4) + 12 8
4. The portion of a tangent to a parabola y2 = 4ax cut off = =
4 + ( -3)
2 2 5
between the directrix and the curve subtends an angle
θ at the focus, where θ = 8 16
∴ The length of latus rectum = 2 ´ = .
p
(A) p (B)  ×
5 5
3
×

4
p 6. If the parabola x2 = ay makes an intercept of length
(C)
×
(D)  none of these 40 on the line y – 2x = 1, then a is equal to
2
(A) 1 (B)  –2
Solution: (C)
(C) –1 (D)  2
The equation of the tangent at P(at2, 2at) to
y2 = 4ax is ty = x + at2(1) Solution: (A, B)
It meets the directrix x = –a. Solving the two equation x2 = ay and y – 2x = 1, we get
a(t 2 - 1) x2 = a(2x + 1) or x2 – 2ax – a = 0
\ ty = -a + at 2 Þ y =
t ∴ x1 + x2 = 2a and x1x2 = –a
æ a(t 2 - 1) ö So, the given line cuts the parabola at two points
Thus, (1) meets the directrix at Q ç -a, ÷. (x1, y1) and (x2, y2).
è t ø
Now, ( 40 ) 2 = ( x1 - x2 ) 2 + ( y1 - y2 ) 2 [Given]
2
æ x2 x2 ö
Þ 40 = ( x1 - x2 ) 2 + ç 1 - 2 ÷
è a a ø
é ( x + x )2 ù
  = ( x1 - x2 ) 2 ê1 + 1 2 2 ú
ë a û
æ 4a2 ö
 = [( x1 + x2 ) 2 - 4 x1 x2 ] ç 2 + 1÷ = 5( 4 a 2 + 4 a)
è a ø
∴ a + a – 2 = 0 or (a + 2) (a – 1) = 0
2

2at - 0 2t ∴ a = 1, –2
Now, slope of PS is m1 = = and slope of
at 2 - a t 2 - 1 7. A ray of light is coming along the line which is parallel
a(t 2 - 1) to y-axis and strikes a concave mirror whose intersec-
(t 2 - 1)
QS is m2 = t - 0 = - . tion with the xy-plane is a parabola (x – 4)2 = 4(y + 2).
-a - a 2t After reflection, the ray must pass through the point
Since m1m2 = – 1, therefore PQ subtends a right angle (A) (4, –1) (B)  (0, 1)
at the focus. (C) (–4, 1) (D)  none of these
Conic Sections (Parabola, Ellipse and Hyperbola)  12.9

Solution: (A) SP × SQ 6 × SQ
The equation of axis of the parabola is x – 4 = 0 which Þ 4=2 Þ 4=2 Þ SQ = 3
SP + SQ 6 + SQ
is parallel to y-axis. So the ray of light is parallel to the
axis of the parabola. We know that any ray parallel to 12. The circle on focal radii of a parabola as diameter
the axis of a parabola passes through the focus after touches the
reflection. (A) axis (B)  directrix
∴ The ray must pass through the point (4, –1). (C) tangent at the vertex (D)  none of these
8. With respect to the parabola y2 = 2x, the points P(4, 2) Solution: (C)
and Q(1, 4) are such that Let the parabola be y2 = 4ax.
(A) P and Q both lie inside the parabola Let P(at2, 2at) be any point on the parabola.
(B) P lies inside whereas Q lies outside the parabola Then SP is focal radii of the parabola.
(C) P lies outside whereas Q lies inside the parabola The equation of a circle with SP as diameter is
(D) P and Q both lie outside the parabola
(x – a) (x – at2) + ( y – 0) ( y – 2at) = 0
Solution: (B)
It meets y-axis at x = 0
Let S ≡ y2 – 2x. ∴ y2 – 2aty + a2t2 = 0 i.e., (y – at)2 = 0
Then S ]P(4, 2)= (2)2 – 2(4) = 4 – 8 < 0 ⇒ y-axis meets the circle only at one point.
∴  Point P lies inside the parabola. Therefore, the circle touches the tangent at the vertex.
Also, S]Q (1, 4) = (4)2 – 2(1) = 16 – 2 = 14 > 0
13. If the two parabolas y2 = 4a(x – 2) and x2 = 4a(y – 3)
∴  Point Q lies outside the parabola.
touch each other, then their point of contact lies on a
9. If y + 3 = m1(x + 2) and y + 3 = m2(x + 2) are two tan- (A) circle (B)  parabola
gents to the parabola y2 = 8x, then (C) ellipse (D)  hyperbola
(A) m1 + m2 = 0 (B)  m1m2 = – 1
(C) m1m2 = 1 (D)  none of these Solution: (D)
Let P(x1, y1) be the point of contact of the two given
Solution: (B)
parabolas
Clearly the two tangents, having slopes m1 and m2, y2 = 4a(x – 2) (1)
meet on the line x = –2, which is the directrix of the and x2 = 4a (y – 3) (2)
parabola y2 = 8x, therefore the two tangents must be at Equation of tangent at P to (1) is
right angles, i.e. m1m2 = –1.
yy1 = 2a(x + x1) – 8a
10. The parametric representation (3 + t2, 3t – 2) represents or 2ax – y1 y + (2ax1 – 8a) = 0 (3)
a parabola with Equation of tangent at P to (2) is
(A) focus at (–3, –2) (B)  vertex at (3, –2) xx1 = 2a( y + y1) – 12a
(B) directrix x = –5 (D)  all of these or x1x – 2ay – (2ay1 – 12a) = 0 (4)
Solution: (B) Since (3) and (4) represent the same line,
We have, x = 3 + t2 and y = 3t – 2 2a - y1
\ = Þ x1 y1 = 4 a 2
⇒ x – 3 = t2 and y + 2 = 3t x1 -2a
⇒ ( y + 2)2 = 9(x – 3)
∴ point of contact (x1, y1) lies on the curve xy = 4a2.
which is a parabola with vertex at (3, –2), focus at
3 which is a hyperbola.
æ 21 ö
ç 4 , -2 ÷ and directrix x = 4 . 14. If ASB is a focal chord of a parabola such that AS = 2
è ø
and SB = 4, then the latus rectum of the parabola is
11. If PSQ is the focal chord of the parabola y = 8x such
2

that SP = 6, then the length SQ is 8 16


(A) (B) 
(A) 6 (B)  4 3 3
(C) 3 (D)  none of these
(C) 25 (D)  none of these
Solution: (C) 3
Since the semi latus rectum of a parabola is the har- Solution: (B)
monic mean between the segment of any focal chord We know that semi-latus rectum of a parabola is
of a parabola, therefore SP, 4, SQ are in H.P. the harmonic mean of segments of a focal chord.
12.10  Chapter 12

AS × SB 16 which meets the tangent at the vertex, x = 0, at the


∴ Semi latus rectum = 2 = . ∴ Latus rec- point
32 16 AS + SB 6
tum = i.e., .
y=
4
at
6 3
3
15. If the line x – 1 = 0 is the directrix of the parabola y2 –
kx + 8 = 0, then one of the values of k is 17. The difference of the squares of the perpendiculars
1 drawn from the points (a ± k, 0) on any tangent to a
(A) (B)  8
8 1 parabola y2 = 4ax is
(C) 4 (D)  (A) 4 (B)  4a
4
Solution: (C) (C) 4k (D)  4ak
The given parabola is Solution: (B)
æ 8ö
y = kx - 8 = k ç x - ÷
2
Equation of any tangent to the parabola
è kø
y2 = 4ax is
Shifting the origin to (8/k, 0), the parabola becomes
ty = x + at2(1)
Y 2 = kX where X = x – 8/k and Y = y.
-k The difference of the squares of the perpendiculars
Directrix of this parabola is X = from (a ± k, 0) to (1) is
4
8 -k ( a + k + at 2 ) 2 ( a - k + at 2 ) 2 4 a(1 + t 2 )k
or x- = = = 4 ak
k 4 1+ t2 1+ t2
8 k
This will coincide with x = 1 if - = 1
k 4 18. The locus of a point whose sum of the distances from
⇒ 32 – k2 = 4k  ⇒  k2 + 4k – 32 = 0 the origin and the line x = 2 is 4 units, is
or (k + 8) (k – 4) = 0  ⇒  k = –8, 4 (A) y2 = –12(x – 3) (B)  y2 = 12(x – 3)
Thus, k = 4 (C) x = 12(y – 3)
2
(D)  x2 = –12(y – 3)
16. A line bisecting the ordinate PN of a point P(at2, 2at), Solution: (A)
t > 0, on the parabola y2 = 4ax is drawn parallel to the Let the coordinates of the point be (h, k).
axis to meet the curve at Q. If NQ meets the tangent at Distance of the point from origin
the vertex at the point T, then the coordinates of T are
= ( h - 0) 2 + ( k - 0) 2 = h 2 + k 2 .
(A) æç 0, at ö÷
4
(B)  (0, 2at)
è 3 ø Distance of the point from the line x – 2 = 0 is h – 2
According to the given condition,
(C) æç at 2 , at ö÷
1
(D) (0, at)
è4 ø h2 + k 2 + h - 2 = 4 Þ h2 + k 2 = 6 - h
Solution: (A)
Squaring both sides, we have,
Equation of the line parallel to the axis and bisecting
h2 + k2 = 36 + h2 – 12h or k2 = –12 (h – 3)
the ordinate PN of the point P(at2, 2at) is y = at which
∴ The path of the point is y2 = –12(x – 3).
æ1 ö
meets the parabola y2 = 4ax at the point Q ç at 2 , at ÷ .
è 4 ø 19. If three points E, F, G are taken on the parabola y2 =
Coordinates of N are (at2, 0). 4ax so that their ordinates are in G.P. , then the tan-
0 - at gents at E and G intersect on the
Equation of NQ is y = ( x - at 2 ),
1 2 (A) directrix (B)  axis
at - at
2

4 (C) ordinate of F (D)  none of these


Solution: (B)
Given parabola is y2 = 4ax(1)
Let the coordinates of E, F and G be respectively
( at12 , 2at1 ),( at 22 , 2at 2 ) and ( at 23 , 2at3 )
Since ordinates of E, F and G are in G.P.
\ ( 2at 2 ) 2 = ( 2at1 )( 2at3 ) or t 22 = t1t3 (2)
The tangents at E and G are
Conic Sections (Parabola, Ellipse and Hyperbola)  12.11

Solution: (C)
t1 y = x + at12 (3)
The given conic is ax + by = 1
and t3 y = x + at32 (4) Squaring both sides,
Solving (3) and (4), we get x = at1t3 = at 22 [from (2)]. ax + by + 2 abxy = 1
Since the x-coordinate of the point of intersection or ax + by - 1 = -2 abxy
is at 22 , the point lies on the line x = at 22 i.e., on the
ordinate of F ( at 22 , 2at 2 ). Squaring again, (ax + by – 1)2 = 4abxy
or a2x2 – 2abxy + b2y2 – 2ax – 2by + 1 = 0 (1)
20. If the two parabolas y2 = 4a(x – k1) and x2 = 4a(y – Comparing the equation (1) with the equation
k2) always touch each other, k1 and k2 being variable Ax2 + 2Hxy + By2 + 2Gx + 2Fy + C = 0
parameters, then their point of contact lies on the curve ∴ A = a2, H = –ab, B = b2, G = –a, F = –b, C = 1
(A) xy = a2 (B)  xy = 2a2 Then, Δ = ABC + 2FGH – AF2 – BG2 – CH2
(C) xy = 4a 2
(D)  none of these = a2b2 – 2a2b2 – a2b2 – a2b2 – a2b2
= –4a2b2 ≠ 0
Solution: (C)
and H  = a2b2 = AB
2
Given parabolas are y2 = 4a(x – k1)(1) So we have Δ ≠ 0 and H 2 – AB = 0. Hence the given
and x2 = 4a( y – k2)(2) equation represents a parabola.
Let (α, β) be their point of contact.
Equation of tangent to (1) at (α, β) is by = 2a(x – k1 23. If the tangents to the parabola y2 = 4ax at (x1, y1) and
+ α) (x2, y2) intersect at (x3, y3), then
or 2ax – by = 2a(k1 – α)(3) (A) x1, x2, x3 are in G. P (B)  x1, x2, x3 are in A. P
Equation of tangent to (2) at (α, β) is (C) y1, y2, y3 are in G.P (D)  y1, y2, y3 are in A. P
a x = 2a( y – k2 + β) Solution: (A, B)
or a x – 2ay = 2a(β – k2)(4)
Let ( x1 , y1 ) º ( at12 , 2at1 )
Since (3) and (4) are identical, comparing coefficients
of x and y in (3) and (4), we get and ( x2 , y2 ) º ( at 22 , 2at 2 ).
2a b Then, (x3, y3) = [at1t2, a(t1 + t2)]
=
a 2a   \ x1 x2 = at12 × at 22 = ( at1t 2 ) 2 = x32
⇒ aβ = 4a2. i.e., the point of contact (α, β) lies on the 1
curve xy = 4a2. and y3 = a(t1 + t 2 ) =
( y1 + y2 )
2
21. Coordinates of any point on the parabola, whose focus ∴ x1, x2, x3 are in G. P and y1, y2, y3 are in A.P.
æ -3 ö
is ç , -3 ÷ and the directrix is 2x + 5 = 0 is given by 24. A circle has its centre at the vertex of the parabola x2
è 2 ø
(A) (2t2 + 2, 2t – 3) (B)  (2t2 – 2, 2t – 3) = 4y and the circle cuts the parabola at the ends of its
(C) (2t – 2, 2t + 3)
2
(D)  none of these latus rectum. The equation of the circle is
(A) x2 + y2 = 5 (B)  x2 + y2 = 4
Solution: (B) (C) x + y = 1
2 2
(D)  none of these
The equation of the parabola is
Solution: (A)
2 2
æ 3ö æ 2x + 5 ö Coordinates of the vertex of the parabola x2 = 4y are
ç x + 2 ÷ + ( y + 3) = ç 2 ÷
2

è ø è ø (0, 0) and the ends of latus rectum are (2, 1) and (–2,
1).
é 9 ù
Þ 4 ê x 2 + + 3 x ú + 4[ y 2 + 9 + 6 y ] = ( 4 x 2 + 25 + 20 x ) ∴  Centre of the circle is (0, 0) and radius of the circle
ë 4 û is
⇒ ( y + 6y + 9) – 2(x + 2) = 0
2
= ( 2) 2 + (1) 2 = 5
or ( y + 3)2 = 2(x + 2)
Clearly, x = 2t2 – 2 and y = 2t – 3 satisfy it for all t. ∴ Equation of circle is
22. The conic represented by the equation ax + by = 1 x2 + y2 = 5
is 25. If b and c are the lengths of the segments of any focal
(A) ellipse (B)  hyperbola chord of a parabola y2 = 4ax, then the length of the
(C) parabola (D)  none of these semi-latus rectum is
12.12  Chapter 12

bc Let R(α, β) be the middle point of PQ, then


(A) (B) bc
b+c t12 + t 22
a=   (3) and β = t1 + t2(4)
2bc 2
(C) b + c (D) 
2 b+c From (4), b 2 = t12 + t 22 + 2t1t 2 = 2a - 8
Solution: (D) [From (2) and (3)]
Hence locus of R(α, β ) is y2 = 2x – 8.
Since the semi latus rectum of a parabola is the har-
monic mean between the segments of any focal chord 28. If the parabolas y2 = 4a(x – c1) and x2 = 4a(y – c2) touch
of the parabola. each other, then the locus of their point of contact is
∴ l is the harmonic mean between b and c. (A) xy = 4a2 (B)  xy = 2a2
2bc (C) xy = a 2
(D)  none of these
Hence, l =
b+c Solution: (A)
Let P(x, y) be the point of contact.
26. The mirror image of the directrix of the parabola y2 =
4(x + 1) in the line mirror x + 2y = 3 is dy dy
\ 2y = 4 a and 2 x = 4 a
(A) x = –2 (B)  4y – 3x = 16 dx dx
(C) 3x – 4y + 16 = 0 (D)  none of these 4a 2 x
Þ = Þ xy = 4 a 2 ,
2 y 4a
Solution: (C)
which is the required locus.
Directirx of y2 = 4(x + 1) is x = –2
Any point on x = –2 is (–2, k) 29. Maximum number of common normals of y2 = 4ax and
Now, mirror image (x, y) of (–2, k) in the line x2 = 4by may be equal to
x + 2y = 3 is given by (A) 3 (B)  5
x+2 y-k æ -2 + 2k - 3 ö (C) 4 (D)  none of these
= = -2 ç ÷
1 2 è 5 ø Solution: (B)
10 - 4 k 4k Equations of normals to y2 = 4ax and x2 = 4by are given
Þ x= -2Þ x = - (1)
5 5 by
20 - 8k b
Also, y = (2) y = mx - 2am - am3 and y = mx + 2b +
5 m2
From (1) and (2) b
For common normals, 2b + + 2am + am3 = 0
m2
3 æ 5x ö
y = 4+ ç ÷ ⇒ am5 + 2am3 + 2bm2 + b = 0
5è 4 ø So, a maximum of 5 normals are possible.
or 4y = 16 + 3x is the equation of the mirror image of
30. If the segment intercepted by the parabola y2 = 4ax on
the directrix.
the line ax + by + c = 0 subtends a right angle at the
27. Through the vertex O of a parabola y2 = 4x, chords OP vertex, then
and OQ are drawn at right angles to one another. The
(A) 4aα + c = 0 (B)  4bα + c = 0
locus of the middle point of PQ is
(C) 4aα + b = 0 (D)  none of these
(A) y2 = 2x + 8 (B)  y2 = x + 8
(C) y = 2x – 8
2
(D)  none of these Solution: (A)
Making the equation of parabola y2 = 4ax homoge-
Solution: (C)
neous using the equation of line ax + by + c = 0, we get
Given parabola is y2 = 4x(1)
Here 4a = 4, ∴ a = 1
Let P º (t12 , 2t1 ) and Q º (t 22 , 2t 2 )
2t1 2 2
Slope of OP = 2
= and slope of OQ = .
t1 t1 t2
4
Since OP ^ OQ, \ = -1 or t1t 2 = -4 (2)
t1t 2
Conic Sections (Parabola, Ellipse and Hyperbola)  12.13

æ ax + by ö Coefficient of t 2
y 2 = 4a x ç ÷ ∴ Sum of the roots = t1 + t 2 + t3 = - =0
è -c ø Coefficient of t 3
⇒ 4aax2 + 4baxy + cy2 = 0, ∴  Centroid of the triangle formed by the feet of the
which represents the combined equation of OP and normals
OQ. æ a 2a ö
Since ∠POQ = 90°, coefficient of x2 + coefficient of = ç - (t12 + t 22 + t32 ), (t1 + t 2 + t3 ) ÷
è 3 3 ø
y2 = 0
⇒ 4aα + c = 0 æ a ö
= ç - (t12 + t 22 + t32 ), 0 ÷
è 3 ø
31. The tangents at two points P and Q on the parabola y2
which, clearly, lies on the x-axis.
= 4x intersect at T. If SP, ST and SQ are equal to a, b
an c respectively, where S is the focus, then the roots 33. Given the two ends of the latus rectum, the maximum
of the equation ax2 + 2bx + c = 0 are number of parabolas that can be drawn is
(A) real and equal (B)  real and unequal (A) 1 (B)  2
(C) complex numbers (D)  irrational (C) 3 (D)  none of these
Solution: (A) Solution: (B)
2 2
The tangents at the points P (t , 2t1 ) and Q(t , 2t 2 )
1 2
L and L′ are the ends of latus rectum. S bisects
intersect at the point T(t1t2, t1 + t2). LL′. As A′ is perpendicular bisector of LL′, where
1
AS = LL¢ = A¢S .
4

a = SP = 1 + t12 and c = SQ = 1 + t 22
Now,  
∴ b2 = ST 2 = (t1t2 – 1)2 + (t1 + t2)2
  = t12 + t 22 + 1 + t12 t 22
 = (1 + t12 )(1 + t 22 ) = ac Clearly, two parabolas are possible.
∴    Roots of the equation ax + 2bx + c = 0 are real
2
34. A line L passing through the focus of the parabola y2 =
and equal. 4(x – 1) intersects the parabola in two distinct points.
32. The centroid of the triangle formed by the feet of the If m be the slope of the line L, then
normals from the point (h, k) to the parabola y2 + 4ax (A) m ∈ R – {0} (B)  –1 < m < 1
= 0, (a > 0) lies on (C) m < –1 or m > 1 (D)  none of these
(A) x-axis (B)  y-axis Solution: (A)
(C) x = h (B)  y=k The focus of the parabola y2 = 4(x – 1) is (2, 0). Any
Solution: (A) line through the focus is
Co-ordinates of any point on the parabola y2 = – 4ax (y – 0) = m(x – 2), i.e., y = m(x – 2)
are (–at2, 2at). It will meet the given parabola if
Equation of the normal at (–at2, 2at) is m2(x – 2)2 = 4(x – 1)
y – xt = 2at + at3 or m2x2 – 4(m2 + 1)x + 4(m2 + 1) = 0
If the normal passes through the point (h, k), then If m ≠ 0, discriminant = 16(m2 + 1)2 – 16m2(m2 + 1) = 0
k – th = 2at + at3 = 16(m2 + 1) > 0 for all m
or at3 + (2a + h)t – k = 0, But if m = 0, then x does not have two real distinct
which is a cubic equation whose three roots t1, t2, t3 are values
the parameters of the feet of the three normals. ∴ m ∈ R – {0}
12.14  Chapter 12

35. If the length of a focal chord of the parabola y2 = 4ax (A) 2y2 1 = 9x1 (B)  2x2 1 = 9y1
at a distance b from the vertex is c, then (C) 4y1 = 9x1
2 
(D)  none of these
(A) a2c = 4b3 (B)  b2c = 4a3 Solution: (A)
(C) c b = 4a
2 3
(D)  none of these The equation of any tangent to the parabola y2 = 4x is
Solution: (B)
1
Let the ends of the focal chord be ( at12 , 2at1 ) and y = mx +
m
( at 22 , 2at 2 ). Then t1t2 = –1.
If it passes through the point (x1, y1), then
Equation of the focal chord is
(t1 + t2)y = 2x + 2at1t2 1
y1 = mx1 + or x1m 2 - y1m + 1 = 0
2at1t 2 -2a m
Given: b= = Its roots are given to be m1 and 2m1
(t1 + t 2 ) + 4
2
2 + t12 + t 22
y1 y1
\ m1 + 2m1 = Þ 3m1 =
Also, c 2 = a 2 (t12 - t 22 ) 2 + 4 a 2 (t1 - t 2 ) 2 x1 x1
= a 2 (t1 - t 2 ) 2 [(t1 + t 2 ) 2 + 4] 1 1
and m1 × 2m1 = Þ 2m12 =
= a ( t + t + 2)
2 2 2 2
(∵ t1t 2 = -1) x1 x1
1 2
2
\ c = a(t12 + t 22 + 2) æ y ö 1
\ 2ç 1 ÷ = or 2 y12 = 9 x1
è 3 x1 ø x1
4a2 4 a 2 4 a3
Now, b2 = = =
t + t2 + 2 c / a
2
1
2
c 38. If the focus of the parabola (y – β)2 = 4(x – α) always
∴ b2c = 4a3 lies between the lines x + y = 1 and x + y = 3, then
(A) 1 < α + β < 2 (B)  0 < α + β < 1
36. If from a point, the two tangents drawn to the parabola (C) 0 < α + β < 2 (D)  none of these
y2 = 4ax are normals to the parabola x2 = 4by, then
Solution: (C)
(A) a2 > 8b2 (B)  b2 > 8a2
(C) a < 8b
2 2
(D)  none of these The coordinates of the focus of the given parabola are
(α + 1, β).
Solution: (A)
The coordinates of any point on the parabola x2 = 4by
are (2bt, bt2).
dy x
For the parabola x 2 = 4by, = .
dx 2b
2b 1
Slope of the normal at ( 2bt , bt 2 ) = - =-
2bt t
1
∴ Equation of normal is y - bt 2 = - ( x - 2bt )
t
x
or y = - + 2b + bt 2 Clearly, focus must lie to the opposite side of the ori-
t
gin w.r.t. the line x + y – 1 = 0 and same side as origin
It will touch the parabola y2 = 4ax if with respect to the line x + y – 3 = 0. Hence, α + β > 0
a æ aö and α + β < 2.
2b + bt 2 = ç∵ c = m ÷
-1/ t è ø
⇒  bt2 + at + 2b = 0 ELLIPSE
For distinct real roots, discriminant > 0 An ellipse is the locus of a point which moves in a plane
⇒ a2 – 8b2 = 0  or  a2 > 8b2 so that the ratio of its distance from a fixed point (called
37. If two tangents drawn from the point (x1, y1) to the focus) and a fixed line (called directrix) is a constant which
parabola y2 = 4x be such that the slope of one tangent is less than one. This ratio is called eccentricity and is
is double of the other, then denoted by e. For an ellipse, e < 1.
Conic Sections (Parabola, Ellipse and Hyperbola)  12.15

(b) On replacing x by –x, the above equation remains


unchanged. So, the curve is symmetrical about y-axis
2. Foci  If S and S′ are the two foci of the ellipse and their
coordinates are (ae, o) and (–ae, 0) respectively, then dis-
tance between foci is given by
SS′ = 2ae
3. Directrices If ZM and Z′M′ are the two directrices of
a a
the ellipse and their equations are x = and x = -
e e
respectively, then the distance between directrices is
FIGURE 12.17 given by
Let S be the focus, QN be the directrix and P be any point 2a
PS ZZ ¢ = -
on the ellipse. Then, by definition, = e or PS = e PN, e
PN
e < 1, where PN is the length of the perpendicular from P 4. Axes The lines AA′ and BB′ are called the major axis and
on the directrix QN. minor axis respectively of the ellipse.
An alternate definition An ellipse is the locus of a point The length of major axis = AA′ = 2a
that moves in such a way that the sum of its distances from The length of minor axis = BB′ = 2b
two fixed points (called foci) is constant. 5. Centre The point of intersection C of the axes of the
ellipse is called the centre of the ellipse. All chords, pass-
Equation of An Ellipse in Standard Form ing through C are bisected at C.
6. Vertices  The end points A and A′ of the major axis are
The standard form of the equation of an ellipse is: known as the vertices of the ellipse
x2 y2 A ≡ (a, 0) and A′ ≡ (– a, 0)
+ = 1( a > b),
a2 b2 Remember: The vertex divides the join of focus and the
where a and b are constants. point of interesection of directrix with axis internally and
enternally the ratio e : 1.
7. Focal chord  A chord of the ellipse passing through its
focus is called a focal chord.
8. Ordinate and double ordinate Let P be a point on the
ellipse. From P, draw PN ⊥ AA′ (major axis of the ellipse)
and produce PN to meet the ellipse at P′. Then PN is
called an ordinate and PNP′ is called the double ordinate
of the point P.
9. Latus rectum If LL′ and NN′ are the latus rectum of the
ellipse, then these lines are ⊥ to the major axis AA′, pass-
ing through the foci S and S′ respectively.

æ b2 ö æ -b 2 ö
L º ç ae, ÷ , L¢ º ç ae, ÷,
è a ø è a ø
æ b2 ö æ -b 2 ö
N º ç - ae, ÷ , N ¢ º ç - ae, ÷
FIGURE 12.18 a ø a ø
è è
2b 2
Some Terms and Properties Related to an Ellipse Length of latus rectum = LL¢ =
= NN ¢.
a
A sketch of the locus of a moving point satisfying the
10. By definition, SP = ePM = e æ a - x ö = a - ex
x2 y2 çe ÷
equation 2 + 2 = 1( a > b), has been shown in the figure è ø
a b
given above. æa ö

and S ¢P = e ç + x ÷ = a + ex
èe ø
1. Symmetry
This implies that distances of any point P(x, y) lying on
(a) On replacing y by –y, the above equation remains the ellipse from foci are: (a – ex) and (a + ex). In other
unchanged. So, the curve is symmetrical about x-axis words
12.16  Chapter 12

SP + S′P = 2a for all real values of θ. Thus, x = acosθ, y = bsinθ are the
i.e., sum of distances of any point P(x, y) lying on the x2 y2
parametric equations of the ellipse 2 + 2 = 1, where the
ellipse from foci is constant. a b
parameter 0 ≤ θ < 2π.
11. Eccentricity of the ellipse Since, SP = ePM, therefore,
Hence the coordinates of any point on the ellipse
SP2 = e2PM2
2 x2 y2

or æa ö + =1
( x - ae) 2 + ( y - 0) 2 = e 2 ç - x ÷ a2 b2
è e ø

(x – ae)2 + y2 = (a – ex)2 may be taken as (acosθ, bsinθ). This point is also called

x + a e – 2aex + y = a – 2aex + e x
2 2 2 2 2 2 2 the point ‘θ’.
The angle θ is called the eccentric angle of the point

x2(1 – e2) + y2 = a2(1 – e2)
(acosθ, bsinθ) on the ellipse.
x2 y2 14. Equation of Chord The equation of the chord joining
+ =1
a 2 a 2 (1 - e 2 ) the points P ≡ (acosq1, bsinq1) and Q ≡ (acosq2 , bsinq2) is

x2 y2 x æq +q ö y æ q1 + q 2 ö æ q1 - q 2 ö

On comparing with + = 1, we get cos ç 1 2 ÷ + b sin ç 2 ÷ = cos ç 2 ÷
a2 b2 a è 2 ø è ø è ø

b2
b 2 = a 2 (1 - e 2 ) or e = 1 -
a2 Info Box!
12. Auxiliary circle  The circle drawn on major axis AA′ as If the centre of the ellipse lies at (h, k) and the axes are
diameter is known as the Auxiliary circle. parallel to the coordintate axes, then the equation of the
x2 y2 ellipse is

Let the equation of the ellipse be + = 1. Then ( x - h)2 ( y - k )2
a2 b2 + =1
the equation of its auxiliary circle is a2 b2

POSITION OF A POINT WITH RESPECT TO AN


ELLIPSE
The point P(x1, y1) lies outside, on or inside the ellipse
x2 y2 x2 y2
2
+ 2 = 1 according as 12 + 12 - 1 > 0, = 0 or 0.
a b a b
Intersection of line and an Ellipse
FIGURE 12.19 x2 y2
The line y = mx + c intersects the ellipse 2 + 2 = 1
x2 + y2 = a2 a b
Let Q be a point on auxiliary circle so that QM, per- in two distinct points if a m + b > c , in one point if c2 =
2 2 2 2

pendicular to major axis meets the ellipse at P. The points a2m2 + b2 and does not intersect if a2m2 + b2 < c2.
P and Q are called as corresponding points on the ellipse
and auxiliary circle respectively. Condition for Tangency and Points of Contact
The angle θ is known as eccentric angle of the point P
on the ellipse. The condition for the line y = mx + c to be a tangent to the
It may be noted that the CQ and not CP is inclined at θ x2 y2
ellipse 2 + 2 = 1 is that c2 = a2m2 + b2 and the coordi-
with x-axis. a b
13. Parametric equation of the ellipse The coordinates nates of the points of contact are
x = acosθ and y = bsinθ satisfy the equation
æ a2 m b2 ö
x2 y2 çç ± 2 2 ,∓ ÷÷
+ =1
a2 b2 è a m + b2 a2 m2 + b2 ø
Conic Sections (Parabola, Ellipse and Hyperbola)  12.17

Two standard forms of the ellipse are shown below along with their properties:

TWO STANDARD FORMS OF THE ELLIPSE

x2 y 2 x2 y 2
+ = 1(a > b) + = 1(a, b)
Standard Equation a2 b2 b2 a2
(Horizontal Form of an Ellipse) (Vertical Form of an Ellipse

Shape of the Ellipse

Centre (0, 0) (0, 0)


Equation of major axis y=0 x=0
Equation of minor axis x=0 y=0
Length of major axis 2a 2a
Length of minor axis 2b 2b
Foci (±ae, 0) (0, ±ae)
Vertices (±a, 0) (0, ±a)
a a
Equation of directrices x=± y=±
e e

a2 - b2 a2 - b2
Eccentricity e= e=
a2 a2

2b2 2b2
Length of latus rectum
a a

æ b2 ö æ b2 ö
Ends of latra-recta ç ± ae, ± ÷ ç ± , ± ae ÷
è a ø è a ø
Parametric coordinates (a cosq, b sinq) (acosq, bsinq)
Focal radii SP = a – ex1 and S′P = a + ex1 SP = a – ey1 and S′P = a + ey1
Sum of focal radii SP + S′P = 2a 2a
Distance between foci 2ae 2ae
2a 2a
Distance between directrices
e e
Tangents at the vertices x = ±a y = ±a
12.18  Chapter 12

Equation of Tangent in Different Forms


x2 y 2
1. Point form The equation of the tangent to the ellipse + = 1 is x2 + y2 = a2 + b2.
a2 b2
x2 y2
+ = 1.  The product of perpendiculars from the foci on any tan-
a2 b2 x2 y 2
xx yy gent to the ellipse 2 + 2 = 1 is equal to b2.
at the point (x1, y1) is 21 + 21 = 1 a b
a b

QUICK TIPS EQUATION OF NORMAL IN DIFFERENT


The equation of tangent at (x1, y1) can also be obtained by FORMS
x + x1 y + y1
replacing x2 by xx1, y2 by yy1, x by , y by , and 1. Point form The equation of the normal to the ellipse
xy1 + x1 y 2 2 x2 y2
xy by . This method is used only when the equa- + = 1 at the point (x1, y1) is
2 a2 b2
tion of ellipse is a polynomial of second degree in x and y.
a 2 x b2 y
- = a 2 - b2
2. Parametric form The equation of the tangent to the x1 y1
x2 y2
ellipse + = 1 at the point (a cosθ, b sinθ) is 2. Parametric form The equation of the normal to the
a2 b2
x2 y2
x y + = 1 at the point (a cosθ, b sinθ) is
ellipse
cos q + sin q = 1 a2 b2
a b
ax secθ – by cosecθ = a2 – b2
3. Slope form The equation of tangent to the ellipse
x2 y 2 ax by
+ = 1 in terms of slope ‘m’ is

or - = a 2 - b2
a 2 b2 cos q sin q
3. Slope form The equation of normal to the ellipse
y = mx ± a 2 m 2 + b 2
x2 y 2
The coordinates of the points of contact are + = 1 in terms of slope ‘m’ is
a 2 b2
æ a2 m b2 ö
çç ± 2 2 ,∓ ÷÷ m (a 2 - b 2 )
y = mx ±
è a m + b2 a2 m2 + b2 ø a 2 + b2 m2
QUICK TIPS QUICK TIPS
 Number of tangents drawn from a point Two tangents  The coordinates of the points of contact are
can be drawn from a point to an ellipse. The two tangents
æ a2 mb2 ö
are real and distinct or coincident or imaginary according ç± 2 ,± ÷
as the given point lies outside, on or inside the ellipse. è a + b2m2 a +b m ø
2 2 2

 Director circle It is the locus of points from which per-


 Condition for normality The line y = mx + c is a normal
pendicular tangents are drawn to the ellipse.
to the ellipse
The equation of Director Circle of the ellipse
x2 y 2 m2 (a2 - b2 )2
+ 2 = 1if c 2 = 2 .
a 2
b (a + b2m2 )

EQUATION OF THE PAIR OF TANGENTS


The equation of the pair of tangents drawn from a point

FIGURE 12.20
FIGURE 12.21
Conic Sections (Parabola, Ellipse and Hyperbola)  12.19

2 2
P(x1, y1) to the ellipse x + y = 1 is æ b2 ö
Equation of normal at ç ae, ÷ is
2 2
a b a ø
è
SS1 = T 2 2
a x b y 2
æ a x b2 y
2
ö
x2 y2 x12 y12 - 2 = a2 - b2 ç - = a2 - b2 ÷
where Sº + - 1, S º + -1 ae b / a
a 2 b2
1
a 2 b2 è x1 y1 ø
xx yy a æ a2 - b2 ö
and T º 21 + 21 - 1 or x - ay = a 2 e 2 ç∵ e 2
= ÷
a b e è a2 ø
or x – ey – ae3 = 0
CHORD WITH A GIVEN MID POINT
40. If the normal at the end of a latus rectum of an ellipse
x2 y 2 passes through one extremity of the minor axis, then
The equation of the chord of the ellipse 2 + 2 = 1 with
a b (A) e4 + e2 – 1 = 0 (B)  e4 – e2 + 1 = 0
P(x1, y1) as its middle point is given by (C) e – e – 1 = 0
4 2
(D)  none of these
T = S1 Solution: (A)
xx1 yy1 x 2
y 2 Let the equation of the ellipse be
where Tº + 2 - 1 and S1 º +1
-1
1

a 2
b a 2
b 2 x2 y2
+ =1
a2 b2
Chord of Contact Let the normal at the extremity L of the latus rectum
passes through the extremity B′ of the minor axis.
The equation of chord of contact of tangents drawn from
x2 y 2 æ b2 ö
a point P(x1, y1) to the ellipse 2 + 2 = 1 is T = 0, where Coordinates of L are ç ae, ÷ and coordinates of
a b B′ are (0, –b). è a ø
Equation of the normal at L is

FIGURE 12.22

xx1 yy1 a2 × x b2 × y æ a2 x b2 y ö
Tº + -1 - 2 = a2 - b2 ç - = a2 - b2 ÷
a 2 b2 ae b /a è x1 y1 ø
ax
or - ay = a 2 - b 2
e
If it passes through B′(0, –b), then 0 + ab = a2 – b2
⇒ a2b2 = (a2 – b2)2
But b2 = a2(1 – e2).
∴ a2 × a2(1 – e2) = [a2 – a2 (1 – e2)]2
FIGURE 12.23 ⇒ a4 (1 – e2) = a4 (1 – 1 + e2)2
⇒ 1 – e2 = e4 or e4 + e2 – 1 = 0
SOLVED EXAMPLES
41. Eccentric angle of a point on the ellipse x2 + 3y2 = 6 at
2 2
x y a distance 2 units from the centre of the ellipse is
39. The equation of the normal to the ellipse + =1 p
a2 b2 (A) p (B) 
at the end of the latus rectum in the first quadrant, is
×
×

4 3
(A) x + ey – ae3 = 0 (B)  x – ey + ae3 = 0 3p 2p
(C) (D) 
(C) x – ey – ae = 0
×
3
(D)  none of these ×

4 3
Solution: (C) Solution: (A, C)
The end of the latus rectum in the first quadrant is The equation of ellipse can be written in the form
æ b2 ö x2 y2
ç ae, ÷ . + =1
è a ø ( 6 )2 ( 2 )2
12.20  Chapter 12

Let the eccentric angle of the point be θ, then its coor- 1


dinates are ( 6 cos q , 2 sin q ). = × 2ae × b sin q = abe sin q .
2
Since the distance of the point from the centre is 2 Clearly, A is maximum when sin q = 1.
units ∴  Maximum value of A = abe.
\ ( 6 cos q - 0) 2 + ( 2 sin q - 0) 2 = 4
44. The number of real tangents that can be drawn to the
⇒ 6cos2θ + 2(1 – cos2θ) = 4  ⇒ 4cos2θ = 2 ellipse 3x2 + 5y2 = 32 and 25x2 + 9y2 = 450 passing
1 p 3p through (3, 5) is
Þ cos q = ± . \ q = or
2 4 4 (A) 0 (B)  2
(C) 3 (D)  4
42. An ellipse has CB as a semi minor axis, F, F ′ are its
foci and the angle FBF ′ is a right angle. Then the Solution: (C)
eccentricity of the ellipse is Since 3 × 32 + 5 × 52 – 32 > 0, the point (3, 5) lies out-
1 1 side the ellipes 3x2 + 5y2 = 32.
(A) (B) 
2 2 Also, 25 × 32 + 9 × 52 – 450 = 0, ∴ the point (3,
5) lies on the ellipse 25x2 + 9y2 = 450. So the required
(C) 3 (D)  none of these number of tangents is 3.
2
45. The locus of mid-points of focal chords of the ellipse
Solution: (A)
x2 y2
p + = 1 is
Since ÐFBF ¢ = a2 b2
2
x 2 y 2 ex x 2 y 2 ex
(A) 2 + 2 = (B) 2 - 2 =
a b a a b a
(C) x2 + y2 = a2 + b2 (D)  none of these
Solution: (A)
Let (h, k) be the mid point of a focal chord. Then its
p
\ ÐFBC = ÐF ¢BC = equation is T = S1
4
xh ky h2 k 2
∴ CB = CF  ⇒  b = ae i.e., + - 1 = + -1
⇒  b2 = a2e2  ⇒  a2(1 – e2) = a2e2 a2 b2 a2 b2
1 Since it passes through (ae, 0)
 Þ 2e 2 = 1 Þ e = hae h2 k 2
2 \ = 2+ 2
x2 y2 a2 a b
43. Let P be a variable point on the ellipse 2 + 2 = 1 x 2 y 2 xe
a b ∴ Locus of (h, k) is 2 + 2 =
with foci F1 and F2. If A is the area of the triangle a b a
PF1F2, then the maximum value of A is
2 2
(A) 2abe (B)  abe x y x y
46. If + = 2 touches the ellipse 2 + 2 = 1, then
1 a b a b
(C) abe (D)  none of these
2 its eccentric angle θ is equal to
Solution: (B) (A) 0 (B)  90°
Let P ≡ (a cosθ, b sinθ) (C) 45° (D)  60°
Then, A = area of ΔPF1F2 Solution: (C)
Equation of any tangent to the ellipse
x2 y2
+ = 1 is
a2 b2
x y
cos q + sin q = 1 (1)
a cos q b sin q 1 a b
1
= ae 0 1 x y
2 Also, + = 2 touches the given ellipse.
-ae 0 1 a b
Conic Sections (Parabola, Ellipse and Hyperbola)  12.21

Comparing coefficients in (1) and (2), we get x y


cosq /a sinq /b 1 - = 1 on the axis of y and whose axes lie along
= = 3 5
1/a 1/b 2 the axes of coordinates is
3 2
1 (A) 2 6 (B) 
Þ cosq = = sinq 7 7
2
∴ θ = 45° (C) 6 (D)  none of these
7
47. If the angle between the straight lines joining foci and
Solution: (A)
x2 y2
the ends of minor axis of the ellipse 2 + 2 = 1 is x2 y2
a b Let the equation of the ellipse be + = 1.
90°, then the eccentricity is a2 b2
1 3 It is given that it passes through (7, 0) and (0, – 5)
(A) (B)  ∴ a2 = 49 and b2 = 25
2 2
Since b2 = a2(1 – e2), ∴ 25 = 49(1 – e2)
1
(C) (D)  none of these 25 25 24 2 6
2 Þ 1 - e2 = Þ e2 = 1 - = Þ e=
49 49 49 7
Solution: (C)
b-0 b x2 y2
Slope of BS is m1 = =- 50. A point on the ellipse+ = 1 at a distance equal
0 - ae ae 16 9
to the mean of the lengths of the semi-major axis and
and slope of BS ′ is
semi-minor axis from the centre is
b-0 b æ 2 91 3 105 ö
m2 = = (A) ç
0 + ae ae ç 7
, ÷
è 14 ÷ø
æ 2 91 -3 105 ö
(B)  çç , ÷
è 7 14 ÷ø

æ 2 105 -3 91 ö
(C) ç - , ÷
Since ∠SBS′ = 90°, ç 7 14 ÷ø
è
-b b
\ m1m2 = -1 Þ ´ = -1 Þ b 2 = a 2 e 2 æ 2 105 3 91 ö
ae ae (D)  çç - , ÷
⇒ a2(1 – e2) = a2e2  ⇒  1 – e2 = e2  ⇒ 2e2 = 1 è 7 14 ÷ø
1
\ e= Solution: (A)
2 x2 y2 x2 y2
Given ellipse is + = 1 i.e., 2 + 2 = 1
x2 y2 16 9 4 3
48. The line y = 2t2 meets the ellipse + = 1 in real
points if 9 4 ∴ Lengths of semi-major and semi-minor axes are 4
and 3 respectively. So, the mean of these lengths is 7/2.
(A) | t | ≤ 1 (B)  | 
t | > 1
Let P(4cosθ, 3sinθ) be a point on the ellipse at a
(C) | t | < 3 (D)  none of these
distance 7/2 from the centre (0, 0).
Solution: (A)
49
Putting y = 2t2 in the equation of the given ellipse \ 16 cos 2q + 9 sin 2q =
4
x2 y2
+ = 1, we get 49
9 4 16 cos 2q + 9(1 - cos 2q ) = Þ 28 cos 2q = 13
4
x 2 4t 4
+ = 1 Þ x 2 = 9(1 - t 4 ) = 9(1 - t 2 )(1 + t 2 ) 13 91 105
9 4 Þ cos q = ± =± and sin q = ±
This will give real values of x if 1 – t2 ≥ 0 i.e., | t | ≤ 1. 28 14 14
So, the coordinates of the required point are
49. The eccentricity of the ellipse which meets the straight
x y æ 4 91 3 105 ö æ 2 91 3 105 ö
line + = 1 on the axis of x and the straight line çç ± , ÷÷ i.e., çç ± , ÷
7 2 è 14 14 ø è 7 14 ÷ø
12.22  Chapter 12

51. If the extremities of a line segment of length l move in


\ 6 cos 2q + 2 sin 2q = 2 Þ 6 cos 2q + 2 sin 2q = 4
two fixed perpendicular straight lines, then the locus of
that point which divides this line segment in the ratio 1 ⇒ 6cos2θ + 2(1 – cos2θ) = 4 or 4cos2θ = 2
: 2, is 1
Þ cos q = ±
(A) a parabola (B)  an ellipse 2
(C) a hyperbola (D)  none of these
p 3p 5p 7p
Solution: (B) \ q= , , , (∵ 0 £ q < 2p )
4 4 4 4
Let the two fixed ⊥ straight lines be the coordinates
axes. 53. If the focal distance of an end of the minor axis of
Let P(h, k) be the point whose locus is required. any ellipse (referred to its axes as the axes of x and y
respectively) is k and the distance between the foci is
2h, then its equation is
2 2
x2 y2
(A) x 2 + 2 y 2 = 1 (B)  + 2 =1
k k +h k 2
h - k2
2 2
x2 y2
(C) x 2 + 2 y 2 = 1 (D)  + =1
k k -h k 2 h2
Solution: (C)
x2 y2
Let the equation of the ellipse be + = 1.
a2 b2
Let PA : PB = 1 : 2 Let e be the eccentricity of the ellipse.
l 2l Since distance between foci = 2h
Then PA = and PB =
3 3 ∴ 2ae = 2h  ⇒  ae = h(1)
l Focal distance of one end of minor axis say (0, b) is k.
k = sin q or 3k = l sin q (1)
3 ∴ a + e(0) = k  ⇒  a = k(2)
2l 3h From (1) and (2)
and h = cos q or = l cos q (2) b2 = a2(1 – e2) = k2 – h2
3 2
∴  The equation of the ellipse is
Squaring and adding (1) and (2), we get
x2 y2
9h2 + 2 =1
9k 2 + = l 2 or 9h2 + 36 k 2 = 4l 2 k 2
k - h2
4
x2 y2
∴    locus of P(h, k) is 9x2 + 36y2 = 4l2, which is an 54. Let E be the ellipse + = 1 and C be the circle
9 4
ellipse.
x2 + y2 = 9. Let P and Q be the points (1, 2) and (2, 1)
x2 y2 respectively. Then
52. If the distance of a point on the ellipse + =1
6 2 (A) Q lies inside C but outside E
from the centre is 2, then the eccentric angle is (B) Q lies outside both C and E
p p (C) P lies inside both C and E
(A) (B) 
× ×

3 4 (D) P lies inside C but outside E


p p
(C) (D) 
× × Solution: (D)
6 2
Since 12 + 22 = 5 < 9 and 22 + 12 = 5 < 9,
Solution: (B) ∴ both P and Q lie inside C.
x2 y2 12 22 1
Equation of the ellipse is + = 1. Also, + = +1 > 1
6 2 9 4 9
Here a2 = 6 and b 2 = 2 Þ a = 6 and b = 2 . 22 1 16 + 9 25
and + = = <1
Let ‘θ’ be the eccentric angle of the point so that 9 4 36 36
the coordinates of the point are ( 6 cos q , 2 sin q ). ∴  P lies outside E and Q lies inside E.
Since distance of this point from the centre C(0, 0) is 2. Thus, P lies inside C but outside E.
Conic Sections (Parabola, Ellipse and Hyperbola)  12.23

55. S and T are the foci of an ellipse and B is an end of the Solution: (B)
minor axis. If STB is an equilateral triangle, the eccen- The equation of any tangent to the given ellipse is
tricity of the ellipse is x y
(A) 1/4 (B)  1/3 cos q + sin q = 1.
4 3
(C) 1/2 (D)  2/3
æ 4 ö
Solution: (C) The tangent meets x-axis at A ç , 0 ÷ and y-axis
è cos q ø
at æç 0,
We have, S ≡ (ae, 0), T ≡ (– ae, 0) and B ≡ (0, b). 3 ö
÷ .
è sin q ø
4 3
Given : =l=
cos q sin q
4 3
Þ cos q = and sin q =
Since STB is an equilateral triangle l l
∴ ST 2 = TB2  ⇒ 4a2e2 = a2e2 + b2 16 9
Þ cos 2q + sin 2q = 2 + 2 Þ l 2 = 25. \ l = 5
⇒ 3a2e2 = b2 = a2 (1 – e2) l l
1
Þ 3e 2 = 1 - e 2 Þ 4e 2 = 1 Þ e = x2 y2
2 58. If P(a cosθ, b sinθ) is a point on an ellipse 2 + 2 = 1,
then ‘θ ’ is a b
56. An ellipse slides between two lines at right angles to
one another. The locus of its centre is (A) angle of OP line from positive direction of x-axis
(A) a parabola (B)  an ellipse (O is origin)
(C) a circle (D)  None of these (B) angle of OQ line from positive direction of x-axis
[when Q is (a cosθ, a sinθ)]
Solution: (C) (C) it depends on the pont P
Let the two given lines be taken as the coordinate axes. (D) none of the above
Let C(α, β) be the centre of the ellipse in any
Solution: (B)
position. Here the position of centre C changes as the
ellipse slides. P(a cosθ, b sinθ), then θ is angle of a corresponding
Let a and b be the semi major and semiminor axes pont on auxilliary circle x2 + y2 = a2 i.e., (a cosθ, a
of the ellipse. sinθ).
Equation of the director circle of the ellipse is 59. If (5, 12) and (24, 7) are the foci of an ellipse passing
(x – α)2 + (y – β)2 = a2 + b2(1) through the origin, then the eccentricity of the conic is
386
(A) 386 (B) 
12 13
(C) 386 (D)  386
25 38
Solution: (D)
Let S(5, 12), S′(24, 7) be the two foci.
P(0, 0) is a point on the conic.
SP = 25 + 144 = 13
Since OX and OY are mutually perpendicular tan- S ¢P = 576 + 46 = 625 = 25
gents to sliding ellipse for all its positions, therefore, SS ¢ = ( 24 - 5) 2 + (7 - 12) 2 = 192 + 52 = 386
O(0, 0) will lie on circle (1)
If the conic is an ellipse,
∴ a2 + β 2 = a2 + b2 then SP + S′P = 2a and SS ′ = 2ae
Hence, locus of C(α, β) is x2 + y2 = a2 + b2.
SS ¢ 386 386
x2 y2 \ e= = =
57. If any tangent to the ellipse + = 1 intercepts SP + S ¢P 13 + 25 38
16 9
equal lengths l on the axes, then l = 60. A tangent to the ellipse x2 + 4y2 = 4 meets the ellipse x2
(A) 3 (B)  5 + 2y2 = 6 at P and Q. The angle between the tangents at
(C) 5 (D)  none of these P and Q of the ellipse x2 + 2y2 = 6 is
12.24  Chapter 12

p ⇒  18 cos2θ – 9 cosθ – 14 = 0
(A) p (B)  ×
2
3 Þ cos q = -
×

6
p 3
p
(C) (D) 
× ×

4 2 x2 y2
Solution: (D) +
62. If the tangent line to an ellipse= 1 cuts inter-
a2 b2
Given ellipses are x2 + 4y2 = 4 a 2
b 2
cepts h and k from axes, then 2 + 2 =
i.e., x2 y2 h k
+ = 1 (1)
22 12
(A) 0 (B)  1

(C) –1 (D)  2
x2 y2
and x2 + 2y2 = 6  i.e.,  +
= 1 (2) Solution: (B)
( 6 ) ( 3 )2
2

Let the equation of tangent at the point (x1, y1) be


Let R(α, β) be the point of intersection of the tan- xx1 yy1 x y
gents to ellipse (2) at P and Q. then PQ will be chord 2
+ 2 =1 Þ 2
+ 2 =1
of contact of R. a b a b
∴ its equation is x1 y1
ax b y æ a2 ö æ b2 ö
+ =1 which meets axes at ç , 0 ÷ and ç 0, ÷
6 3
è x1 ø è y1 ø
i.e., a x + 2yβ = 6
a 3 a2 b2
or y=- x + (3) but = h and = k (given)
2b b x1 y1
Since (3) touches (1) a2 b2 x12 y12
Þ x1 = , y1 = Þ + =1
æ3ö
2
a2 h k a2 b2
\ ç ÷ = 22 × 2 + 12 (c 2 = a 2 m 2 + b 2 )
èb ø 4b a2 b2
\ + =1
9 a 2
a2 + b2 h2 k 2
Þ = 2 +1 =
b 2
b b2 63. S and T are the foci of an ellipse and B is an end of the
⇒ a + b  = 9
2 2 minor axis. if DSTB is equilateral, then e is
∴ locus of (α, β) is (A) 1/4 (B)  1/3
x 2 + y 2 = 9 = ( 6 )2 + ( 3 )2 (C) 1/2 (D)  none of these
i.e., director circle. Solution: (C)
∴ tangent at P, Q meet at right angles. OB
tan 60° =
x2 y2 OS
61. If the normal at the point P(θ) to the ellipse + =1
14 5
intersects it again at the point Q(2θ), then cosθ is equal
to:
(A) 2/3 (B)  –2/3
1 1
(C) (B)  -
3 3
Solution: (B)
Equation of normal at (a cos2θ, b sin2θ) is
ax secθ – by cosecθ = a2 – b2
Þ 14 x sec q - 5 y cosecq = 9 b b
Þ 3=- Þ = e 3 (1)
It passes through (a cos2θ, b sin2θ) ae a
Þ 14 14 cos 2q × sec q - 5 sin 2q cosecq = 9 b2
Now e 2 = 1 - Þ e 2 = 1 - 3e 2
cos 2q sin 2q a2
Þ 14 -5 =9
cos q sin q 1
Þ 4e 2 = 1 Þ e =
⇒  14(2 cos2θ – 1) – 10 cos2θ = 9(0)θ 2
Conic Sections (Parabola, Ellipse and Hyperbola)  12.25

66. If y = x and 3y + 2x = 0 are the equations of a pair of


64. The centre of the ellipse ( x - y - 2) + ( x - y ) = 1 is
2 2

16 9 conjugate diameters of an ellipse, then the eccentricity


(A) (0, 0) (B)  (1, 1) of the ellipse is
1
(C) (1, 0) (D)  (0, 1) (A) 2 (B) 
Solution: (B) 3 3
1 2
The given equation can be written as (C) (C) 
9(x + y – 2)2 + 16(x – y)2 = 144 2 5
Let t = 9(x + y – 2)2 + 16(x – y)2 – 144 Solution: (B)
∴ Centre of the conic (ellipse in this case) is the Let the equation of the ellipse be x2/a2 + y2/b2 = 1
point of intersection of lines Slope of the given diameter are m1 = 1, m2 = –2/3.
¶f ¶f ⇒ m1m2 = –2/3 = –b2/a2
= 0 and =0 (using the condition of conjugacy of two diameters)
¶x ¶y
⇒ 3b2 = 2a2  ⇒ 3a2(1 – e2) = 2a2
¶f
Now, =0 ⇒  1 – e2 = 2/3  ⇒  e2 = 1/3  ⇒  e =
¶x
⇒ 18(x + y – 2) + 16 . 2(x – y) = 0 67. The product of length of perpendiculars drawn from
⇒ 18(x + y – 2) + 32(x – y) = 0 the two foci to the tangent at any point on the ellipse
⇒ 50x – 14y – 36 = 0 (1) 25x2 + 4y2 = 100 is
¶f (A) 30 5 (B)  25 2
Also, =0
¶y (C) 152 (D)  none of these
⇒ 18(x + y – 2) + 16 . 2(x – y)(–1) = 0 Solution: (D)
⇒ 9(x + y – 2) – 16(x – y) = 0 x2 y2
⇒ –7x + 25y – 18 = 0 Ellipse + = 1 has length of semi-minor axis as
4 25
⇒ 7x – 25y + 18 = 0 (2) 2, hence b2 = 4. Product of length of perpendiculars
Solving (1) and (2), we have centre of conic is C(1, 1). = b2 = 4.
65. On the ellipse 4x2 + 9y2 = 1, the points at which the x2 y2
68. Equation + = 1 represents an ellipse if:
tangents are parallel to the line 8x = 9y are: k ( k - 1)
æ 2 1ö (A) 0 < k < 1 (B)  k > 1
(A) æç , ö÷ (B) 
2 1
ç- 5,5÷ (C) k < 0 or k > 1 (D)  none of these
è 5 5 ø è ø
Solution: (B)
æ 2 1ö
(C) ç - , - ÷ (D)  none of these k > 0, k – 1 > 0  ⇒  k > 1.
è 5 5ø
Solution: (B)
OPTICAL PROPERTY OF PARABOLA
We have PF1 + PF2 = 2a = 10 for every point P on the
ellipse. Differentiating w.r.t. x, we get (a) A ray parallel to the axis of the parabola after reflection
from its internal surface passes through the focus.
dy
8 x + 18 y =0 (b) If a point is at a minimum distance from a parabola, then
dx this point must lie on a normal to the parabola through
dy 8x 4x this point.
Þ =- =-
dx 18 y 9y
Hyperbola
The tangent at point (x, y) will be parallel to 8x = 9y if
-4 x 8 A hyperbola is the locus of a point which moves in a plane
= Þ x = -2 y so that the ratio of its distances from a fixed point (called
9y 9
focus) and a fixed line (called directrix) is a constant which
Substituting x = –2y in 4x2 + 9y2 = 1, we get is greater then one. This ratio is called eccentricity and is
1 denoted by e. For a hyperbola, e > 1.
4( -2 y ) 2 + 9 y 2 = 1 or 25 y 2 = 1 Þ
y=± .
5 Let S be the focus, QN be the directrix and P be any
Thus, the points where the tangents are parallel to 8x = point on the hyperbola. Then, by definition,
æ 2 1ö æ2 1ö PS
9y are ç - , ÷ and ç , - ÷ . = e or PS = ePN , e >1,
è 5 5 ø è 5 5ø PN
12.26  Chapter 12

where PN is the length of the perpendicular from P on the 3. Directrices  ZM and Z′ M′ are the two directrices of the
directrix QN. a a
hyperbola and their equations are x = and x = -
e e
respectively, then the distance between directrices is
2a
given by ZZ ¢ = .
e
4. Axes The lines AA′ and BB′ are called the transverse axis
and conjugate axis respectively of the hyperbola.
The length of transverse axis = AA′ = 2a
The length of conjugate axis = BB′ = 2b
5. Centre The point of intersection C of the axes of the
hyperbola is called the centre of the hyperbola. All
FIGURE 12.24 chords, passing through C, are bisected at C.
6. Vertices  The points A ≡ (a, 0) and A′ ≡ (–a, 0), where the
An alternate definition A hyperbola is the locus of a point curve meets the line joining the foci S and S′, are called
which moves in such a way that the difference of its dis- the vertices of the hyperbola.
tances from two fixed points (called foci) is constant.
Info Box!
EQUATION OF A HYPERBOLA IN STANDARD The vertex divides the join of focuss and the point of
FORM intersection of directrix with axis internally and externally
in the ratio e : 1.
The general form of standard hyperbola is
x2 y 2 7. Focal chord  A chord of the hyperbola passing through
- = 1,
a 2 b2 its focus is called a focal chord.
where a and b are constants. 8. Focal distances of a point The difference of the focal
distances of any point on the hyperbola is constant and
equal to the length of the transverse axis of the hyperbola.
If P is any point on the hyperbola, then
S′P – SP = 2a = Transverse axis.
9. Latus rectum If LL′ and NN′ are the latus rectum of
the hyperbola then these lines are perpendicular to the
transverse axis AA′, passing through the foci S and S′
respectively.
æ b2 ö æ - b2 ö
L º ç ae, ÷ , L¢ º ç ae, ÷,
è a ø è a ø
æ b2 ö æ -b 2 ö
N º ç -ae, ÷ , N ¢ º ç -ae, ÷
è a ø è a ø
FIGURE 12.25 2b 2
Length of latus rectum = LL¢ =
= NN ¢.
a
10. Eccentricity of the hyperbola We know that
SOME TERMS AND PROPERTIES RELATED TO A
SP = e PM  or  SP2 = e2 PM 2
HYPERBOLA 2
æ aö

or ( x - ae) 2 + ( y - 0) 2 = e 2 ç x - ÷
A sketch of the locus of a moving point satisfying the è eø
x2 y 2 (x – ae)2 + y2 = (ex – a)2
equation 2 - 2 = 1, has been shown in the figure above.
a b x2 + a2e2 – 2aex + y2 = e2x2 – 2aex + a2
1. Symmetry Since only even powers of x and y occur in x2(e2 – 1) – y2 = a2(e2 – 1)
the above equation, so the curve is symmetrical about x2 y2
both the axes.
2
- 2 2 = 1.
2. Foci If S and S’ are the two foci of the hyperbola and
a a (e - 1)
2 2
their coordinates are (ae, 0) and (–ae, 0) respectively, then On comparing with x - y = 1, we get

distance between foci is given by SS’ = 2ae. 2 2
a b
Conic Sections (Parabola, Ellipse and Hyperbola)  12.27

hyperbola is called the conjugate hyperbola of the given


2 2 2 b2
b = a (e - 1) or e = 1 + 2 hyperbola.
a The conjugate hyperbola of the hyperbola
11. Parametric equations of the hyperbola  Since the coor-
x2 y 2
dinates x = a secθ and y = b tanθ satisfy the equation - =1
a 2 b2
x2 y 2
- =1
a 2 b2 x2 y 2 æ x2 y2 ö
is - + 2 = 1 ç i.e., 2 - 2 = -1÷
for all real values of θ, therefore, x = a secθ, y = b tanθ are
a 2
b a b
è ø
x2 y 2
the parametric equations of the hyperbola 2 - 2 = 1,
where the parameter 0 ≤ θ < 2π. a b

Hence, the coordinates of any point on the hyperbola
x2 y 2
- = 1 may be taken as (a secθ, b tanθ). This point
a 2 b2
is also called the point ‘θ ’.

The angle θ is called the eccentric angle of the point
(a secθ, b tanθ) on the hyperbola.
12. Equation of chord  The equation of the chord joining the
points
P ≡ (a secq1, b tanq1) and Q ≡ (a secq2, b tanq2) is
x æ q -q ö y æq +q ö æq +q ö
cos ç 1 2 ÷ - sin ç 1 2 ÷ = cos ç 1 2 ÷ .
a è 2 ø b è 2 ø è 2 ø

or
x y 1
a sec q1 b tan q1 1 = 0
a sec q 2 b tan q 2 1

CONJUGATE HYPERBOLA
FIGURE 12.26
The hyperbola whose transverse and conjugate axes are Properties of hyperbola and conjugate are given below in
respectively the conjugate and transverse axes of a given the table:

PROPERTIES OF HYPERBOLA AND ITS CONJUGATE


Hyperbola Conjugate Hyperbola

x2 y 2 - x2 y 2 x2 y 2
Standard equation - =1 + 2 = 1 or 2 - 2 = -1
a2 b2 a 2
b a b
centre (0, 0) (0, 0)
Equation of transverse axis y=0 x=0
Equation of conjugate axis x=0 y=0
Length of transverse axis 2a 2b
Length of conjugate axis 2b 2a
Foci (±ae, 0) (0, ±be)
a b
Equation of directrices x=± y=±
e e
Vertices (±a, 0) (0, ±b)
12.28  Chapter 12

a2 + b2 a2 + b2
Eccentricity e= e=
a2 b2

2b2 2a2
Length of latus rectum
a b
Parametrc coordinates (a secθ, btanθ) (bsecθ, atanθ)
Focal radii SP = ex1 – a and S′P = ex1 + a SP = ey1 – b and S′P = ey1 + b
Difference of focal radii (S’P – SP) 2a 2b
Tangents at the vertices x = ±a y = ±b

xx1 yy1
- =1
Info Box! a 2 b2
If the centre of the hyperbola lies at a point (h, k) and the QUICK TIPS
axes are paralllel to the coordinate axes, then the equa- The equation of tangent at (x1, y1) can also be obtained
tion of the hyperbola is
by replacing x2 by xx1, y2 by yy1, x by x + x1 , y by y + y1
( x - a)2 ( y - k )2 2 2
a2
-
b2
=1 and xy by xy1 + x1y . This method is used only when the
2
equation of hyperbola is a polynomial of second degree in
x and y.

POSITION OF A POINT WITH RESPECT TO A 2. Parametric form The equation of the tangent to the
HYPERBOLA 2 2
hyperbola x - y = 1 at the point (a secθ, b tanθ) is
2 2
The point P(x1, y1) lies outside, on or inside the hyperbola a b
x2 y 2 x2 y 2 x y
2
- 2 = 1 according as 12 - 12 - 1 > 0, = 0 or < 0. sec q - tan q = 1
a b a b a b
3. Slope form The equation of tangent to the hyperbola
Intersection of a line and a Hyperbola x2 y 2
- = 1 in terms of slope ‘m’ is
The straight line y = mx + c will cut the hyperbola a 2 b2
x2 y 2
- = 1 in two points which may be real, concident or y = mx ± a 2 m 2 - b 2
a 2 b2
imaginary according as C 2 > 1 = 1 < a2m2 – b2.
The coordinates of the points of contact are
æ a2m b2 ö
çç ± 2 2 ,± ÷÷
Condition for Tangency and Points of Contact è a m - b2 a 2 m2 - b2 ø
The condition for the line y = mx + c to be a tangent to the
x2 y 2 QUICK TIPS
hyperbola 2 - 2 = 1 is that c2 = a2m2 – b2 and the coor-
a b  Number of tangents from a point Two tangents can
dinates of the points of contact are be drawn from a point to a hyperbola. The two tangents
æ ö are real and distinct or coincident or imaginary according
a2m b2
çç ± 2 2 ,± ÷÷ as the given point lies outside, on or inside the hyperbola.
è a m - b2 a 2 m2 - b2 ø  Director circle It is the locus of points from which ⊥

tangents are drawn to the hyperbola. The equation of


Equation of Tangent in Different Forms director circle of the hyperbola
1. Point form  The equation of the tangent to the hyperbola x2 y 2
2 2 - = 1is x 2 + y 2 = a2 - b2
x y a2 b2
2
- 2 = 1 at the point (x1, y1) is
a b
Conic Sections (Parabola, Ellipse and Hyperbola)  12.29

Equations of Normal in Different Forms SOLVED EXAMPLES


1. Point form  The equation of the normal to the hyperbola
x2 y 2 69. The locus of the centre of a circle which touches two
- = 1 at the point (x1, y1) is given circles externally is
a 2 b2
(A) an ellipse (B)  a parabola
a 2 x b2 y (C) a hyperbola (C)  none of these
+ = a 2 + b2
x1 y1
Solution: (C)
2. Parametric form The equation of the normal to the Let S and S′ be the centres and a and b be the radii of
2 2
hyperbola x - y = 1 at the point (a secθ, b tanθ) is the given circles.
2 2
a b Let P be the centre and r be the radius of the circle
ax by which touches the given circles externally.
+ = a 2 + b2 Then S′P = r + a and SP = r + b
sec q tan q
3. Slope form The equation of normal to the hyperbola
x2 y 2
- = 1 in terms of slope ‘m’ is
a 2 b2
m (a 2 + b 2 )
y = mx ±
a 2 - b2 m2

QUICK TIPS ∴ S′P – SP = (r + a) – (r + b) = a – b = constant.


 The coordinates of the points of contact are Hence the locus of P is a hyperbola whose foci are S
æ a2 mb2 ö and S′.
ç± 2 ,∓ ÷
è a - b2m2 a -b m ø
2 2 2
70. If a circle makes intercepts of length 5 and 3 on two
perpendicular lines, then the locus of the centre of the
circle is
EQUATION OF THE PAIR OF TANGENTS (A) a parabola (B)  an ellipse
The equation of the pair of tangents drawn from a point (C) a hyperbola (D)  none of these
x2 y 2 Solution: (C)
P(x1, y1) to the hyperbola 2 - 2 = 1 is
a b Let the two given ⊥ lines be the coordinate axes
SS1 = T  2 and let the equation of variable circle be
x2 y2 x2 y 2 x2 + y2 + 2gx + 2f y + c = 0 (1)
where S º 2 - 2 - 1, S1 º 12 - 12 - 1
a b a b Then, 5 = 2 g 2 - c and 3 = 2 f 2 - c
xx1 yy1 Squaring and subtracting these, we get
and T º 2 - 2 -1
a b 4(g2 – c) – 4( f  2 – c) = 25 – 9
⇒  g – f   = 4 or (– g)2 – (–f )2 = 4
2 2

CHORD WITH A GIVEN MID POINT Hence, locus of the centre (–g, –f  ) of circle is
x2 y 2 x2 – y2 = 4,
The equation of the chord of the hyperbola - =1
a 2 b2 which is a rectangular hyperbola.
with P(x1, y1) as its middle point is given by T = S1 where
71. If the line ax + by + c = 0 is a normal to the hyperbola
xx1 yy1 x12 y12 xy = 1, then
Tº - - 1 and S1 º - - 1.
a 2 b2 a 2 b2 (A) a > 0, b < 0 (B)  a > 0, b > 0
CHORD OF CONTACT (C) a < 0, b < 0 (D) a < 0, b > 0
Solution: (A, D)
The equation of chord of contact of tangents drawn from a
Equation of normal at the point çæ t , ÷ö to the hyperbola
x2 y 2 1
point P(x1, y1) to the hyperbola 2 - 2 = 1 is T = 0, where
xx yy a b xy = 1 is è tø
T º 21 - 21 - 1.
a b xt3 – yt – t 4 + 1 = 0
12.30  Chapter 12

-a

(C) (x2 + y2)2 = b2x2 – a2y2
Its slope = t = 2

(D) none of these
b
-a a Solution: (B)
\ >0 Þ <0
b b Equation of any tangent to
∴ a > 0, b < 0 or a < 0, , b > 0 x2 y2
- = 1 (1)
72. The equation 2x2 + 3y2 – 8x – 18y + 35 = k represents a2 b2
(A) no locus if k > 0 in the slope form is y = mx + a 2 m 2 - b 2 (2)
(B)  an ellipse if k < 0 slope of tangent = m.
(C) a point if k = 0 1
∴ slope of any line ⊥ to it = -
(D)  a hyperbola if k > 0 m
Solution: (C) Equation of ⊥ from centre (0, 0) of (1) on (2) is
We have 2x2 + 3y2 – 8x – 18y + 35 = k 1 x
y - 0 = - ( x - 0) or m = - (3)
⇒ 2(x2 – 4x) + 3( y2 – 6y) + 35 = k m y
⇒ 2[(x – 2)2 – 4] + 3[( y – 3)2 – 9] + 35 = k The required locus is obtained by eliminating the
⇒ 2(x – 2)2 + 3( y – 3)2 = k parameter m between (2) and (3). Substituting for m
For k = 0, we get 2(x – 2)2 + 3(y – 3)2 = 0 which from (3) in (2), we get
represents the point (2, 3).
73. The eccentricity of the hyperbola x2 x2
y=- + a2 × 2 - b2
y y
1999 2
( x - y 2 ) = 1 is or
3 x 2 + y 2 = a2 x 2 - b2 y 2
(A) 2 (B)  2 or (x2 + y2)2 = a2x2 – b2y2
(C) 2 2 (D) 
3 76. The equation of the hyperbola, referred to its axes as
axes of coordinates, given that the distances of one of
Solution: (A)
its vertices from the foci are 9 and 1 units, is
Equation of hyperbola is
x2 y2 x2 y2
x2 y2 (A) - = 1 (B) - =1
- =1 16 9 9 16
3 3 2 2

1999 1999 (C) x - y = -1 (D)  none of these


16 9
3 Solution: (A)
Here a 2 = b 2 =
1999 Let the equation of hyperbola be
x2 y2
b2 - = 1 (1)
∴ Eccentricity is e = 1 + = 1+1 = 2 a2 b2
a2 Its vertices are A(a, 0) and A′(–a, 0) and foci are S(ae,
0) and S′(– ae, 0).
74. If e, e′ be the eccentricities of two conics S = 0 and S′
Given : S′A = 9 and SA = 1
= 0 and if e2 + e′2 = 3, then both S and S′ can be
⇒ a + ae = 9 and ae – a = 1
(A) hyperbolas (B)  ellipses ⇒ a(1 + e) = 9 and a(e – 1) = 1
(C) parabolas (D)  none of these a(1 + e) 9 5
Þ = Þ 1 + e = 9e - 9 Þ e =
Solution: (A) a(e - 1) 1 4
For ellipse or parabola e2 + e′2 ≤ 2 ≠ 3. æ 5ö
For hyperbola, it is possible ( e > 1, e′ > 1). ∵ a (1 + e) = 9, \ a ç1 + ÷ = 9 Þ a = 4
è 4ø
Also, b 2 = a 2 (e 2 - 1) = 16 æç - 1÷ö = 9
75. The locus of the foot of the perpendicular from the 25
x2 y2 è 16 ø
centre of the hyperbola 2 - 2 = 1 on a variable tan-
gent is a b Thus, from (1), equation of hyperbola is
(A) (x2 + y2)2 = a2x2 + b2y2 x2 y2
- =1

(B) (x2 + y2)2 = a2x2 – b2y2 16 9
Conic Sections (Parabola, Ellipse and Hyperbola)  12.31

77. The eccentricity of the hyperbola 9x2 – 16y2 + 72x – Since it passes through origin (0, 0)
32y – 16 = 0 is 4 4 x1
-1 - =
(A) 5/4 (B)  4/5 x1 + 5 ( x1 + 5) 2
(C) 9/16 (D)  16/9 ⇒ (x1 + 5)2 + 4(x1 + 5) + 4x1 = 0
Solution: (A) Þ x12 + 18 x1 + 45 = 0
The given hyperbola can be written in the form ⇒ (x1 + 15)(x1 + 3) = 0
( x + 4) 2 ( y + 1) 2 ⇒ x1 = –15  or  x1 = –3
- =1
16 9 So equation of tangent is
Here a2 = 16 and b2 = 9. 4 4
y -1- =- ( x + 15)
b2 9 25 5 ( -15 + 5) ( -15 + 5) 2
\ e2 = 1 + 2 = 1 + = Þ e= .
a 16 16 4 2 1
Þ y -1+ = - ( x + 15)
x2 y2 5 25
78. The number of tangents to the hyperbola - =1 3 x 3
through (4, 1) is 4 3 Þ y- =- -
5 25 5
(A) 1 (B)  2
(C) 0 (D)  3 ⇒ x + 25y = 0
4 4
Solution: (B) or y -1- =- ( x + 3)
( -3 + 5) ( -3 + 5) 2
x2 y2 ù 16 1
Since - - 1ú = - -1 > 0 ⇒ y – 1 – 2 = –(x + 3)  or  x + y = 0
4 3 û ( 4 ,1) 4 3
81. If the circle x2 + y2 = a2 intersects the hyperbola xy = c2
∴ the point (4, 1) lies outside the hyperbola, hence
in four points P(x1, y1), Q(x2, y2), R(x3, y3), and S(x4, y4),
the number of tangents through (4, 1) is two.
then
79. The equation of common tangents to the parabola (A) x1 + x2 + x3 + x4 = 0
y2 = 8x and hyperbola 3x2 – y2 = 3, is (B) y1 + y2 + y3 + y4 = 2
(A) 2x ± y + 1 = 0 (B)  2x ± y – 1 = 0 (C) x1x2x3x4 = 2c4
(C) x ± 2y + 1 = 0 (D)  x ± 2y – 1 = 0
(D) y1y2y3y4 = 2c4
Solution: (A) Solution: (A)
2 c2
The equation of tangent to y2 = 8x is y = mx + Since y = and x2 + y2 = a2
m x
x2 y2 c4
Also, the equation of tangent to - =1 Þ x 2 + 2 = a2
1 3 x
Þ y = mx ± m 2 - 3 ⇒  x4 – a2x2 + c4 = 0
On comparing, we get This has four roots say x1, x2, x3, x4
m = ±2 or tangent as 2x ± y + 1 = 0 ∴ x1 + x2 + x3 + x4 = 0
x+9 x2 y2
80. A tangent to the hyperbola y = passing through 82. If PQ is a double ordinate of hyperbola - =1
x+5 a2 b2
the origin is such that OPQ is an equilateral triangle, O being the
(A) x + 25y = 0 (B)  5x + y = 0 centre of the hyperbola. Then the eccentricity e of the
(C) 5x – y = 0 (D)  x – 25y = 0 hyperbola satisfies
Solution: (C) 2 2
(A) 1 < e < (B)  e=
x+9 4 dy 4 3 3
y= = 1+ ; at ( x1 , y1 ) = - 2
x+5 x + 5 dx ( x1 + 5) 2 (C) e = 3 (D)  e>
2 3
4
Equation of tangent is y - y1 = - ( x - x1 ) Solution: (D)
( x1 + 5) 2
Let P(a secθ, b tanθ); Q(a secθ, –b tanθ) be end points
4 4
y -1- =- ( x - x1 ) of double ordinate and C(0, 0), is the centre of the
x1 + 5 ( x1 + 5) 2 hyperbola. Now PQ = 2b tanθ
12.32  Chapter 12

Solution: (C)
CQ = CP = a 2 sec 2 q + b 2 tan 2 q
Any tangent at P(a secθ, b tanθ) to the hyperbola x2 –
since CQ = CP = PQ y2 = a2 is
x secθ – y tanθ = a(1)
Given lines are x – y = 0 (2)
and x + y = 0 (3)
Solve (1) and (2), (2) and (3), (3) and (1), we get ver-
tices of the triangle as
æ a a ö
ç sec q - tan q , sec q - tan q ÷ ,
è ø
∴ 4b2 tan2θ = a2 sec2θ + b2 tan2θ æ a -a ö
⇒ 3b2 tan2θ = a2 sec2θ ç sec q + tan q , sec q + tan q ÷ and (0, 0)
è ø
⇒ 3b2 sin2θ = a2 1
⇒ 3a2(e2 – 1) sin2θ = a2 ∴ Area of the triangle = | x1 y2 - x2 y1 |
2
⇒ 3(e2 – 1)sin2θ = 1
a2 æ -1 1 ö
1 = ç 2 -
Þ = sin 2 q < 1 (∵ sin 2 q < 1) 2 è sec q - tan q sec q - tan q ÷ø
2 2 2
3(e - 1)
2

a2
1 1  = ( -2) = a 2 (in magnitude).
Þ < 3 Þ e2 - 1 > 2
e -1
2
3
86. If x = 9 is the chord of contact of the hyperbola x2 – y2
83. Centre of the hyperbola x2 + 4y2 + 6xy + 8x – 2y + 7 = = 9, then the equation of the corresponding pair of tan-
0 is gents is
(A) (1, 1) (B)  (0, 2) (A) 9x2 – 8y2 + 18x – 9 = 0
(C) (2, 0) (D)  none of these (B) 9x2 – 8y2 – 18x + 9 = 0
Solution: (D) (C) 9x2 – 8y2 – 18x – 9 = 0
(D) 9x2 – 8y2 + 18x + 9 = 0
Let centre be (h, k)
⇒ h + 3k + 4 = 0, 3h + 4k – 1 = 0 Solution: (D)
æ 19 13 ö The equation of chord of contact at ponit (h, k) is xh
Þ ( h, k ) º ç ,- ÷ – yk = 9
è 5 5ø
Comparing with x = 9, we have h = 1, k = 0
84. Equation of the straight line, passing through the point Hence equation of pair of tangents at point (1, 0) is
(3, 4) and farthest from the circle x2 + y2 + 8x + 6y + 16 SS1 = T 2
= 0, is ⇒ (x2 – y2 – 9)(12 – 02 – 9) = (x – 9)2
(A) x – y + 1 = 0 (B)  3x + 4y = 25 ⇒ –8x2 + 8y2 + 72 = x2 – 18x + 81
(C) x + y – 7 = 0 (D)  none of these ⇒ 9x2 – 8y2 – 18x + 9 = 0
Solution: (C) 87. The equation of the common tangent to the curves y2 =
8x and xy = –1 is
Let P ≡ (3, 4) and C being the centre of circle, where C
≡ (–4, –3). The line which is farthest from the circle is (A) 3y = 9x + 2 (B)  y = 2x + 1
the line perpendicular to CP. (C) 2y = x + 8 (D)  y = x + 2
∴ Required equation is Solution: (D)
2
æ 3+ 4 ö Any tangent to the parabola y2 = 8x is y = mx + .
y - 4 = -ç ÷ ( x - 3) m
è 4+3ø Clearly (a), (b) and (d) satisfy the equation (by m =
⇒ x+y=7 3, 2, 1).
For xy = –1
85. The area of the triangle formed by the lines x – y = 0, x
Equation of tangent with slope m, to xy = –1 are
+ y = 0 and any tangent to the hyperbola x2 – y2 = a2 is
(A) 2a2 (B)  4a2 æ 1 ö
y - m = mç x + ÷
(C) a 2
(D)  none of these. è mø
Conic Sections (Parabola, Ellipse and Hyperbola)  12.33

æ dy 1 1 ö Solution: (B)
ç∵ = 2 =m \ x=± ,y = ∓ m÷ Clearly tangents at (2, –1) or (–2, 1) are parallel to
è dx x m ø
2x + y + 8 = 0
Clearly (d) satisfies these equations. Hence common
2 2
choice is (d). 90. If P and Q are two points on the hyperbola x - y = 1
1 a2 b2
88. An ellipse has eccentricity and one focus at the whose centre is C such that CP is perpendicular to CQ,
æ1 ö 2 1 1
point P ç ,1÷ . Its one directrix is the common tan- where a < b, then + is
è2 ø CA2 CQ 2
gent at the point P, to the circle x2 + y2 = 1 and the
1 1 1 1
hyperbola x2 – y2 = 1. The equation of the ellipse in (A) 2 - 2 (B)  2
+ 2
standard form is a b a b
2 (C) a2 – b2 (D)  a2 + b2
(A) 9 æç x - 1 ö÷ + ( y - 1) 2 = 1
è 3ø Solution: (A)
2 Since the line CP passes through the origin i.e. cen-
(B) 9 æç x - 1 ö÷ + 12( y - 1) 2 = 1 tre, let its equation be y = mx. The line CP meets the
è 3ø
hyperbola
2
æ 1ö x2 y2
(C) çè x - 3 ÷ø ( y - 1) 2 - = 1 in P whose abscissa is given by
+ =1 a2 b2
4 3
x 2 m2 x 2 a2b2
(D) none of these - = 1 or x 2
=
a2 b2 b2 - a2 m2
Solution: (B)
a2 b2 m2
Clearly, the common tangent to the circle x2 + y2 = 1 \ y 2 = m2 x 2 =
and hyperbola x2 – y2 = 1 is x = 1 [which is nearer to b2 - a2 m2
P(1/2, 1)]. ∴ CP2 = x2 + y2
æ1 ö a2 b2 + a2 b2 m2
Given one focus at P ç ,1÷ . =
è2 ø b2 - a2 m2
= a b (1 + m2)
2 2

Since CQ ⊥ CP
1
Replace m by - , we get
m
æ 1 ö
a 2 b 2 ç1 + 2 ÷
è m ø = a b ( m + 1)
2 2 2
CQ 2 =
c 2
b m - a2
2 2
b2 - 2
m
1 1 b2 - a2 m2 + b2 m2 - a2
\ + =
CP 2
CQ 2
a 2 b 2 (1 + m 2 )
∴ equation of the directrix is x = 1.
∴ ellipse is b2 - a2 1 1
= = 2- 2.
2 a2b2 a b
æ 1ö 1
ç x - 2 ÷ + ( y - 1) = 2 ( x - 1)
2

è ø 91. Let P(a secθ, b tanθ) and Q(a secf, b tanf), where
p x2 y2
On simplification, it becomes q + f = , be two points on the hyperbola 2 - 2 = 1.
2 2 a b
æ 1ö If (h, k) is the point of intersection of the normals at P
9 ç x - ÷ + 12( y - 1) 2 = 1
è 3ø and Q, then k is equal to
 a2 + b2 
(A) a + b
2 2
89. The tangents to the hyperbola x2 – y2 = 3 are parallel to (B) − 
the st. line 2x + y + 8 = 0 at the following points a  a 
æ a2 + b2 ö
(C) a + b
(A) (2, 1) or (1, 2) (B)  (2, –1) or (–2, 1) 2 2
(D) -ç ÷
(C) (–1, –2) (D)  (–2, –1) b è b ø
12.34  Chapter 12

Solution: (D) On multiplying (1) by sinf and (2) by sinf, we get


Given P(a secθ, b tanθ) and Q(a secf, b tanf). a sinθ sinf x + b sin f y = (a2 + b2) tanθ sinf
The equation of tangent at point P is a sinf sinθ x + b sinθ y = (a2 + b2) tanf sinθ
x sec q y tan q On subtraction we get
- =1 by (sinf – sinθ) = (a2 + b2)(tanθ sinf – tanf sinθ)
a b
a 2 + b 2 tan q sin f - tan f sin q
b sec q b 1 \ y=k = ×
Slope of tangent = ´ = × b sin f - sin q
tan q a a sin q
p p
Hence, the equation of perpendicular at P is ∵ q +f = Þ f = -q
2 2
a sin q ⇒ sinf = cosθ and tanf = cotθ
y - b tan q = - ( x - a sec q )
b a 2 + b 2 tan q cos q - cos q sin q
or by –b2 tanθ = –a sinθ x + a2 tanθ \ y=k= ×
b cos q - sin q
or a sinθ x + by = (a2 + b2) tanθ(1)
Similarly the equation of perpendicular at Q is a + b æ sin q - cos q ö
2 2
(a2 + b2 )
= ç ÷ =- .
a sinf x + by = (a2 + b2) tanf(2) b è cos q - sin q ø b
Conic Sections (Parabola, Ellipse and Hyperbola)  12.35

NCERT EXEMPLARS
1. The area of the circle centred at (1, 2) and passing 8. If equation of the ellipse whose focus is (1, – 1), then
through the point (4, 6) is 1
(A) 5 π (B  10 π directrix the line x – y – 3 = 0 and eccentricity is
2
(C) 25 π (D)  None of these
(A) 7 x 2 + 2 xy + 7 y 2 - 10 x + 10 y + 7 = 0
2. Equation of a circle which passes through (3, 6) and (B)  7 x 2 + 2 xy + 7 y 2 + 7 = 0
touches the axes is
(C)  7 x 2 + 2 xy + 7 y 2 + 10 x - 10 y - 7 = 0
(A)  x 2 + y 2 + 6 x + 6 y + 3 = 0
(D)  None of the above
(B)  x 2 + y 2 - 6 x - 6 y - 9 = 0
9. The length of the latus rectum of the ellipse 3x2 + y2 =
(C)  x + y - 6 x - 6 y + 9 = 0
2 2
12 is
(D)  None of these
(A) 4 (B) 3
3. Equation of the circle with centre on the Y-axis and 4
passing through the origin and the point (2, 3) is (C)  8 (D)  3
(A)  x + y + 13 y = 0
2 2

x2 y2
(B)  3 x 2 + 3 y 2 + 13 x + 3 = 0 10. If e is eccentricity of the ellipse + 2 = 1 (where,
2 3 b
a < b), then
(C) 6 x 2 + 6 y 2 - 13 y = 0
(D)  x 2 + y 2 + 13 x + 3 = 0 (
(A) b 2 = a 2 1 - e 2 ) (
(B)  a 2 = b 2 1 - e 2 )

4. The equation of a circle with origin as centre and (C) a = b 2 2
(e 2
- 1) (D)  b 2
= a (e
2 2
- 1)
passing through the vertices of an equilateral triangle
whose median is of length 3a is 11. The eccentricity of the hyperbola whose latus rectum
is 8 and conjugate axis is equal to half of the distance
(A)  x 2 + y 2 = 9a 2 (B)  x 2 + y 2 = 16 a 2
between the foci is
(C)  x + y = 4 a
2 2 2
(D)  x 2 + y 2 = a 2 4
4
(A)  (B) 
5. If the focus of a parabola is (0, – 3) and its directrix is y = 3, 3 3
then its equation is
2
(A) x2 = – 12y (B) 
x2 = 12y (C)  (D)  None of these
3
(C) y = – 12x (D) 
2
y2 = 12x
6. If the parabola y2 = 4ax passes through the point (3, 2), 12. The distance between the foci of a hyperbola is 16 and
its eccentricity is 2. Its equation is

NCERT EXEMPLARS
then the length of its latus rectum is
2 4 x2 y2
(A)  x - y = 32 - =1
2 2
(A)  (B)  (B) 
3 3 4 9
1 (C) 2 x - 3 y 2 = 7 (D)  None of these
(C)  (D) 4
3 3
13. Equation of the hyperbola with eccentricity and
foci at (± 2, 0) is 2
7. If the vertex of the parabola is the point (– 3, 0) and the
directrix is the line x + 5 = 0, then its equation is x2 y2 4 x2 y2 4
(A)  - = (B)  - =
(A) y2 = 8 (x + 3) (B)  x2 = 8 (y + 3) 4 5 9 9 9 9
(C) y2 = – 8 (x + 3) (D)  y2 = 8 (x + 5) x2 y2
(C)  - =1 (D)  None of these
4 9

ANSWER K EYS
1. (C) 2.  (C) 3. (C) 4. (C) 5. (A) 6. (B) 7. (A) 8. (A) 9. (D) 10. (B)
11. (C) 12. (A) 13. (A)
12.36  Chapter 12

HINTS AND EXPLANATIONS

1. Given that, centre of the circle is (1, 2). Þ f = -13 / 6



So, the equation of circle is

   13 y
x2 + y2 - =0
6
Þ 6 x 2 + 6 y 2 - 13 y = 0

4. Given that, length of the median AD = 3a


∵ CP = 9 + 15 = 5 = Radius of the circle ∵ Radius of the circle = 3 ´ Length of median

\ Required area = p r 2 = 25p 2
2. Let centre of the circle be (a, a), then equation of the circle is 2
(x – a)2 + (y – a)2 = a2.
= ´ 3a = 2a
3


So, the equation of the circle is x + y = 4 a .
2 2 2


Since, the point (3, 6) lies on this circle, then
  ( 3 - a )2 + ( 6 - a )2 = a 2
5. Given that, focus of parabola at F(Q, – 3) and equation of

Þ a 2 + 9 - 6 a + 36 - 12a + a 2 = a 2 directrix is y = 3.

Þ a 2 - 18a + 45 = 0 Let any point on the parabola is P(x, y).

Þ a 2 - 15a - 3a + 45 = 0 Then, PF = |y – 3|
Þ a ( a - 15 ) - 3 ( a - 15 ) = 0

Þ ( x - 0 ) + ( y + 3) = y - 3
2 2
HINTS AND EXPLANATIONS

Þ ( a - 3) ( a - 15 ) = 0

Þ x2 + y2 + 6 y + 9 = y2 - 6 y + 9

Þ a = 3, a = 15
Þ x 2 + 12 y = 0

So, the equation of circle is

Þ x 2 = -12 y
( x - 3) + ( y - 3) = 9
2 2

Þ x2 - 6x + 9 + y2 - 6 y + 9 = 9
6. Given that, parabola is
Þ x2 + y2 - 6x - 6 y + 9 = 0
y2 = 4ax  (i)
3. Let general equation of the circle is x 2 + y 2 + 2 gh + 2 fy + c = 0 ∴ Length of latus rectum = 4a
Since, the parabola passes through the point (3, 2).
Then, 4 = 4a (3)
⇒ a = 1/3
∴ 4a = 4/3
7. Here, vertex = (– 3, 0)
∴ a = – 3 and directrix, x + 5 = 0


Since the point (0, 0) and (2, 3) lie on it c = 0.
∴ 4 + 9 + 4 g + 6 f = 0
Þ 2 g + 3 f = -13 / 2
Since the centre lie on Y-axis, then g = 0.
\ 3 f = -13 / 2

Conic Sections (Parabola, Ellipse and Hyperbola)  12.37


Since, axis of the parabola is a line perpendicular to directrix 1
and A is the mid-point of AS. \
( 2ae ) = 2b
2

Then, x -5
Þ ae = 2b  (ii)
-3 = 1
2

Þ - 6 = x1 - 5 Þ x1 = -1,
Þ b2 = a2 e2 - 1  ( ) .(iii)

0 + y1
From Eqs. (i) and (ii),
0= Þ y1 = 0
2 a 2e 2
4a =
\ S = ( -1, 0 )
4

∵ PM = PS
Þ 16 a = a 2e 2
16
( x + 1)
2

Þ x+5 = + y2
Þ 16 a = a 2 Þ a = 2
e

Þ x 2 + 2 x + 1 + y 2 = x 2 + 10 x + 25


Þ 4a = a2 e 2 - 1 ( )
Þ y 2 = +8 x + 24 4

Þ = e2 -1

Þ y 2 = +8 ( x + 3) a
8. Given that, focus of the ellipse is (1, – 1) and the equation of 4e 2
Þ = e2 -1
directrix is x – y – 3 = 0 and e = 1 16
2
æ 4ö
Let P(x, y) and F(1, – 1). Þ e 2 ç1 - ÷ = 1
PF 1 è 16 ø
\ =
Dis tan ce of P from ( x - y - 3) = 0 2 æ 12 ö æ 16 ö
Þ e2 ç ÷ = 1 Þ e2 = ç ÷
è 16 ø è 12 ø
( x - 1) + ( y + 1)
2 2

1 4 2
Þ =
Þ e = Þe=
2
x - y -3 2 3 3
2
12. Given that, distance between the foci of hyperbola
2 x - 2 x + 1 + y 2 + 2 y + 1ùû 1
é 2

Þ ë
= i.e., 2ae = 16 Þ ae = 8
( x - y - 3)
2
4 and e = 2

Þ 8 x 2 - 16 x + 16 + 8 y 2 + 16 y = x 2 + y 2 + 9 - 2 xy + 6 y - 6 x Now, 2a = 8

Þ 7 x 2 + 7 y 2 + 2 xy - 10 x + 10 y + 7 = 0
Þ a=4 2
9. Given equation of ellipse is (
∵ b2 = a2 e2 - 1 )

HINTS AND EXPLANATIONS


3x2 + y2 = 12 Þ b 2 = 32 ( 2 - 1)

x2 y2
Þ + =1
Þ b 2 = 32
4 12
\ a2 = 4 Þ a = 2 x2 y2
\ - =1
and b 2 = 12 Þ b = 2 3 32 32

Þ x 2 - y 2 = 32
∴ Length of latus rectum = 2 ´ 4 = 4 = 2a
2

2 3 3 b 13. Given that, eccentricity of the hyperbola, e = 3/2
2 2
10. Given that, x 2 + y2 = 1, a < b and foci (± 2, 0), (± ae, 0)
a b ∵ ae = 2
We know that, a2
e = 1 - 2 Þ e2 =
b2 - a2( ) Þ a´3/ 2 = 2 Þ a = 4 / 3
b b2 (
∵ b2 = a2 e2 - 1 )

Þ b e =b -a2 2 2 2
16 æ 9 ö 16 æ 5 ö
Þ b 2 = ç - 1÷ Þ b 2 = ç ÷

Þ a2 = b2 1 - e2 ( ) 9 è4 ø 9 è4ø
20
11. Length of latus rectum of the hyperbola i.e., Þ b2 =
9
2b 2
8= Þ b 2 = 4 a (i)
So, the equation of the hyperbola is
a
∴ Distance between the foci = 2ae x2 y2 x2 y2 4
- =1Þ - =
Since, transverse axis be a and conjugate axis be b 16 20 / 9 4 5 9
9
12.38  Chapter 12

PRACTICE EXERCISES

Single Option Correct Type

1. Let y = f(x) be a parabola, having its axis parallel to 8. If from a point, the two tangents drawn to the parabola
y-axis, which is touched by the line y = x at x = 1, then y2 = 4ax are normals to the parabola x2 = 4by, then
(A) f ′(0) = f ′(1) (A) a2 > 8b2 (B)  b2 > 8a2
(B) 2f(0) = 1 – f ′(0) (C) a < 8b
2 2
(D)  none of these
(C) f ′(1) = 1
(D) f(0) + f ′(0) + f ″(0) = 1 9. If the focus of the parabola (y – β)2 = 4(x – α) always
lies between the lines x + y = 1 and x + y = 3, then
2. A ray of light is coming along the line which is parallel (A) 1 < α + β < 2 (B)  0 < α + β < 1
to y-axis and strikes a concave mirror whose intersec- (C) 0 < α + β < 2 (D)  none of these
tion with the xy-plane is a parabola (x – 4)2 = 4(y + 2).
10. If the focal distance of an end of the minor axis of
After reflection, the ray must pass through the point
any ellipse (referred to its axes as the axes of x and y
(A) (4, –1) (B)  (0, 1) respectively) is k and the distance between the foci is
(C) (– 4, 1) (D)  none of these 2h, then its equation is
3. If y + 3 = m1(x + 2) and y + 3 = m2(x + 2) are two tan- x2 y2 x2 y2
(A) 2 + 2 = 1 (B)  + =1
gents to the parabola y2 = 8x, then k k + h2 k 2 h2 - k 2
(A) m1 + m2 = 0 (B)  m1m2 = –1 x2 y2 x2 y2
(C) 2 + 2 = 1 (D)  + =1
(C) m1m2 = 1 (D)  none of these k k - h2 k 2 h2
11. An ellipse slides between two lines at right angles to
4. A line bisecting the ordinate PN of a point P(at2, 2at),
one another. The locus of its centre is
t > 0, on the parabola y2 = 4ax is drawn parallel to the
axis to meet the curve at Q. If NQ meets the tangent at (A) a parabola (B)  an ellipse
the vertex at the point T, then the coordinates of T are (C) a circle (D)  None of these
x2 y2
12. If P(a cosθ, b sinθ) is a point on an ellipse 2 + 2 = 1,
(A) æç 0, at ö÷
4 a b
(B)  (0, 2at) then ‘θ’ is
è 3 ø (A) angle of OP line from positive direction of x-axis
(O is origin)
(C) æç at 2 , at ö÷
1
PRACTICE EXERCISES

(D) (0, at) (B) angle of OQ line from positive direction of x-axis


è4 ø
[when Q is (a cosθ, a sinθ)]
5. Coordinates of any point on the parabola, whose focus (C) it depends on the point P
æ -3 ö (D) none of the above
is ç , - 3 ÷ and the directrix is 2x + 5 = 0 is given by
è 2 ø 13. If (5, 12) and (24, 7) are the foci of an ellipse passing
(A) (2t2 + 2, 2t – 3) (B)  (2t2 – 2, 2t – 3) through the origin, then the eccentricity of the conic is
(C) (2t – 2, 2t + 3)
2
(D)  none of these 386
(A) 386 (B) 
6. The mirror image of the directrix of the parabola y2 = 12 13
4(x + 1) in the line mirror x + 2y = 3 is (C) 386 (D)  386
(A) x = –2 (B)  4y – 3x = 16 25 38
(C) 3x + 4y + 16 = 0 (D)  none of these 14. A tangent to the ellipse x2 + 4y2 = 4 meets the ellipse x2
+ 2y2 = 6 at P and Q. The angle between the tangents at
7. The centroid of the triangle formed by the feet of the P and Q of the ellipse x2 + 2y2 = 6 is
normals from the point (h, k) to the parabola y2 + 4ax = p
(A) p (B) 
0, (a > 0) lies on
×

3
×

6
(A) x-axis (B)  y-axis p p
(C) (D) 
(C) x = h (B)  y=k
×

2
×

4
Conic Sections (Parabola, Ellipse and Hyperbola)  12.39

15. The locus of the centre of a circle which touches two 1 1


1 1 + 2
given circles externally is
(A) - 2 (B)  2
a 2
b a b
(A) an ellipse (B)  a parabola
(C) a2 – b2 (D)  a2 + b2
(C) a hyperbola (D)  none of these
22. Let P(a secθ, b tanθ) and Q(a secφ, b tanφ),
16. If a circle makes intercepts of length 5 and 3 on two p
where q + f = , be two points on the hyperbola
perpendicular lines, then the locus of the centre of the 2
circle is x2 y2
- = 1. If (h, k) is the point of intersection of the
(A) a parabola (B)  an ellipse a2 b2
(C) a hyperbola (D)  none of these normals at P and Q, then k is equal to
æ a2 + b2 ö
(A) a + b (B) 
2 2

17. The equation 2x2 + 3y2 – 8x – 18y + 35 = k represents -ç ÷


a è a ø
(A) no locus if k > 0 æ a2 + b2 ö
(C) a + b (D) 
2 2
(B) an ellipse if k < 0 -ç ÷
(C) a point if k = 0 b è b ø
(D) a hyperbola if k > 0 23. If b, k are intercepts of a focal chord of the parabola
y2 = 4ax then k is equal to
x2 y2
18. If PQ is a double ordinate of hyperbola - =1 ab b
a2 b2 (A) (B) 
such that OPQ is an equilateral triangle, O being the b-a b-a
centre of the hyperbola. Then, the eccentricity e of the a ab
(C) (D) 
hyperbola satisfies b-a a-b
(A) 1 < e < 2 / 3 (B)  e = 2 / 3 24. The point (2a, a) lies inside the region bounded by the
(C) e = 3 / 2 (D)  e > 2/ 3 parabola x2 = 4y and its latus rectum. Then,
(A) 0 ≤ a ≤ 1 (B)  0 < a < 1
19. The equation of the diameter which bisects the chord (C) 0 < a ≤ 1 (D)  none of these
x2 y2
7x + y – 2 = 0 of the hyperbola - = 1 is 25. The point P on the parabola y2 = 4ax for which |PR –
3 7
(A) x + 2y = 0 (B)  x – 2y = 0 PQ| is maximum, where R(–a, 0), Q(0, a) is
(C) x – 3y = 0 (D)  x + 3y = 0 (A) (a, 2a) (B)  (a, –2a)
(C) (4a, 4a) (D)  (4a, –4a)
1
20. An ellipse has eccentricity and one focus at the 26. The shortest distance between the parabola y2 = 4x and
2 the circle x2 + y2 + 6x – 12y + 20 = 0 is
æ1 ö
point P ç , 1÷ . Its one directrix is the common tan-

PRACTICE EXERCISES
è2 ø (A) 4 2 - 5 (B)  0
gent at the point P, to the circle x2 + y2 = 1 and the (C) 3 2 + 5 (D) 
1
hyperbola x2 – y2 = 1. The equation of the ellipse in
standard form is 27. The tangent and normal at the point P(at2, 2at) to the
2 parabola y2 = 4ax meet the x-axis in T and G, respec-
æ 1ö
(A) 9 ç x - ÷ + ( y - 1) 2 = 1 tively, then angle at which the tangent at P to the
è 3ø parabola is inclined to the tangent at P to the circle
æ 1ö
2
through P, T, G is
(B) 9 ç x - ÷ + 12( y - 1) 2 = 1
è 3ø (A) tan–1(t2) (B)  cot–1(t2)
2 (C) tan (t) (B) 
–1
cot–1(t)
æ 1ö
ç x - 3 ÷ ( y - 1) 2 28. If normals are drawn from a point P(h, k) to the parab-
è ø + =1 ola y2 = 4ax, then the sum of the intercepts which the
(C)
4 3 normals cut off from the axis of the parabola is
(D) none of these (A) (h + a) (B)  3(h + a)
x2 y2 (C) 2(h + a) (D)  none of these
21. If P and Q are two points on the hyperbola - =1
a2 b2 29. If the normal drawn from the point on the axis of the
whose centre is C such that CP is perpendicular to CQ,
1 1 parabola y2 = 8ax whose distance from the focus is 8a
where a < b, then + is and which is not parallel to either axis, makes an angle
CP 2 CQ 2
θ with the axis of x, then θ is equal to
12.40  Chapter 12

p p 1 1 1 1
(A) (B)  (A) + = 1 (B)  2 + =1
2 y2
×
2
2 y2
×

6 4 x 4x
p 2p 1 1 1 1
(C) (D)  (D)  2 + 2 = 1
(C) + = 1
× ×

3 3 2x 2
4 y2 2x y
30. Ordinates of three points A, B, C on the parabola y2 =
4ax are in G. P. Tangents at A and C intersect on x2 y2
35. If the ellipse + = 1 is rotated about centre in its
(A) line through B parallel to x-axis a2 b2
(B) line through B parallel to y-axis own plane by 90º in clockwise direction then the point
(C) line through B and vertex of parabola (a cosθ, b sinθ) becomes
(D) line through B and focus of parabola (A) (a cosθ, –b sinθ) (B) (b sinθ, –a cosθ)
31. The condition that the parabolas y2 = 4ax and y2 = 4c(x (C) (b sinθ, a cosθ) (D)  none of these
– b) have a common normal other than x-axis (a, b, c 36. If two points are taken on minor axis of an ellipse
being distinct positive real numbers) is x2 y2
b b + = 1 at the same distance from the centre as
(A) < 2 (B) > 2 a2 b2
a-c a-c the foci, the sum of the squares of the perpendicu-
b b lars from these points on any tangent to the ellipse, if
(C) < 1 (D)  >1
a-c a-c a < b is
32. The shortest distance between the parabolas y2 = 4x (A) a2 (B)  b2
and y2 = 2x – 6 is (C) 2a (D) 
2
2b2
(A) 2 (B)  5 37. The area of the rectangle formed by the perpendicu-
(C) 3 (D)  none of these
x2 y2
1 lars from the centre of the ellipse 2 + 2 = 1 to the
33. An ellipse has eccentricity and one focus at the a b
2 tangent and normal at a point whose eccentric angle is
æ1 ö
point P ç , 1÷ . Its one directrix is the common tan- p
è2 ø × is
gent nearer to the point P, to the circle x2 + y2 = 1 and 4
the hyperbola x2 – y2 = 1. The equation of the ellipse in ( a 2 - b 2 ) ab ( a 2 + b 2 ) ab
the standard form is (A) 2 (B) 
a + b2 a2 - b2
æ 1ö
2
a -b
2 2
a2 + b2
ç x - ÷ ( y - 1)2 (C) (D) 
(A) è 3ø + ab( a 2 + b 2 ) ab( a 2 - b 2 )
=1
1 1 38. The points of intersection of the two ellipses
PRACTICE EXERCISES

9 12 x2 + 2y2 – 6x – 12y + 23 = 0 and


2
æ 1ö 4x2 + 2y2 – 20x – 12y + 35 = 0
ç x - ÷ ( y + 1)2 æ8 ö
(B) è 3ø + =1 (A) lie on a circle centred at ç , 3 ÷ and of radius
1 1 è3 ø
1 47
9 12
3 2
æ 1ö
2 æ 8 ö
(B) lie on a circle centred at ç - , - 3 ÷ and of radius
ç x - ÷ ( y - 1)2 è 3 ø
(C) è 3ø - =1 1 47
1 1
3 2
9 12 (C) lie on a circle centred at (8, 9), and of radius
2
æ 1ö 1 47
ç x - ÷ ( y + 1)2
(D) è 3ø - =1
3 3
1 1 (D) are not concyclic
9 12 39. If the eccentric angles of the ends of a focal chord of
34. The locus of the middle point of the intercept of the x2 y2
the ellipse 2 + 2 = 1 (a > b) are q1 and q2, then value
tangents drawn from an external point to the ellipse x2 a b
of tanq1 tanq2 equals
+ 2y2 = 2 between the coordinate axes is
Conic Sections (Parabola, Ellipse and Hyperbola)  12.41

e -1 e -1 46. If the sum of the slopes of the normal from a point ‘P’
(A) (B)  to the hyperbola xy = c2 is equal to λ (λ ∈ R+), then
e +1 e2 + 1
locus of point ‘P’ is
e +1 e2 + 1
(C) (D)  (A) x2 = lc2 (B)  y2 = lc2
e -1 e -1
(C) xy = lc 2
(D)  none of these
40. If the eccentric angle of a point lying in the first quad-
x2 y2 47. If a ray of light incident along the line
rant on the ellipse 2 + 2 = 1 be α and the line join-
a b 3 x + (5 - 4 2 ) y = 15, gets reflected from the hyper-
ing the centre to the point makes an angle β with x-axis x2 y2
then α – β will be maximum when α = bola - = 1 then its reflected ray goes along the
16 9
a line
(A) 0 (B)  cot- 1
b (A) x 2 - y + 5 = 0 (B)  2 y - x + 5 = 0
-1 a
(C) tan (D)  π/4 (C) 2 y - x - 5 = 0 (D)  none of these
b
41. If a variable line x cosα + y sinα = p which is a chord 48. If a hyperbola passing through the origin has 3x – 4y
x2 y2 – 1 = 0 and 4x – 3y – 6 = 0 as its asymptotes, then the
of the hyperbola 2 - 2 = 1 (b > a) subtends a right equations of its transverse and conjugate axis are
a b
angle at the centre of the hyperbola, then it always (A) x + y – 5 = 0, x + y – 1 = 0
touches a fixed circle whose radius is (B) x – y + 5 = 0, x – y – 1 = 0
ab ab (C) x + y – 5 = 0, x – y – 1 = 0
(A) 2 (B)  (D) none of these
a +b 2
b - a2
2

ab 49. All the chords of the hyperbola 3x2 – y2 – 2x + 4y = 0,


(C) (D)  none of these subtending a right angle at the origin pass through the
a - b2
2
fixed point
x2 y2
42. The tangent at a point P on the hyperbola 2 - 2 = 1 (A) (1, –2) (B)  (–1, 2)
a b
meets one of the directrix in F. If PF subtends an angle (C) (1, 2) (D)  none of these
θ at the corresponding focus, then θ equals x2 y2
p 50. The point on the hyperbola - = 1 which is near-
(A) p (B)  24 18
est to the line 3x + 2y + 1 = 0 is
× ×

4 2
3p (A) (–6, 3) (B)  (6, –3)
(C) ×
(D)  π (C) (6, 3) (D)  none of these
4

PRACTICE EXERCISES
43. The number of point(s) outside the hyperbola 51. The locus of point of intersection of tangents at the
x2 y2 end of normal chord of hyperbola x2 – y2 = a2 is
- = 1 from where two perpendicular tangents
25 36 (A) a2(y2 – x2) = 4x2y2 (B)  a2(y2 + x2) = 4x2y2
can be drawn to the hyperbola is/are (C) y + x = 4a x
2 2 2 2
(C)  none of these
(A) none (B)  1
(C) 2 (D)  infinte 52. The minimum distance between the curves y2 = 4x and
x2 + y2 – 12x + 31 = 0 is
44. The slopes of common tangents to the hyperbolas
(A) 7 (B)  5
x2 y2 2 2
- = 1 and y - x = 1 are (C) 2 5 (D)  none of these
9 16 9 16
53. The mirror image of the parabola y2 = 4x in the tangent
(A) ±2 (B)  ±1
to the parabola at the point (1, 2) is
(C) ± 2 (D)  none of these
(A) (x + 1)2 = 4(y – 1) (B)  (x – 1)2 = 4(y – 1)
45. The equation of a line passing through the centre of a (C) (x + 1) = 4(y + 1)
2
(D)  none of these
rectangular hyperbola is x – y = 1. If one of the asymp-
54. A ray of light is coming along the line y = b from the
totes is 3x – 4y – 6 = 0, the equation of other asymptote
positive direction of x-axis and strikes a concave mir-
is
ror whose intersection with the xy-plane is a parabola
(A) 4x – 3y + 17 = 0 (B)  – 4x – 3y + 17 = 0 y2 = 4ax. If a and b are positive, then the equation of
(C) – 4x + 3y + 1 = 0 (D)  4x + 3y + 17 = 0 the reflected ray is
12.42  Chapter 12

2t 1 1
(A) y - 2at = ( x - at 2 ) (A) (B) 
t +1
2
1 + sin a
2
1 + cos 2 a
2t
(B) y - 2at = 2 ( x - at 2 ) (C) 1 + sin 2 a (D)  none of these
t -1
-2t 61. If a chord joining two points whose eccentric angles
(C) y - 2at = 2 ( x - at 2 ) x2 y2
t -1 are α, β cut the major axis of the ellipse 2 + 2 = 1,
a b
(D) none of these a b
at a distance d from the centre, then tan × tan =
55. Form a point A, common tangents are drawn to the cir- 2 2
a2 d+a d -a
cle x 2 + y 2 = and parabola y2 = 4ax. The area of (A) (B) 
2 d -a d+a
the quadrilateral, formed by the common tangents, the
chord of contact of the circle and the chord of contact a-d
(C) (D)  none of these
of the parabola is a+d
9 15 2 62. The orbit of the earth is an ellipse with eccentricity
(A) a 2 (B)  a 1
4 4 with the sum at one focus, the major axis being
60
21 approximately 186 × 106 miles in length. The shortest
(C) a 2 (D)  none of these
4 and longest distance of the earth from the sun is
56. Three normals are drawn from the point (14, 7) to the (A) 9145 × 104 miles, 9455 × 104 miles
parabola y2 – 16x – 8y = 0. The coordinates of the feet (B) 9147 × 104 miles, 9457 × 104 miles
of the normals are (C) 9145 × 106 miles, 9455 × 106 miles
(A) (0, 0), (8, –16), (3, –4) (B)  (0, 0), (8, 16), (3, –4) (D) none of these
(C) (0, 0), (–8, 16), (3, –4) (D)  none of these 63. PN is the ordinate of any point P on the hyperbola
57. Consider a curve ax2 + 2hxy + by2 = 1 and a point P not x2 y2
- = 1 and AA′ is its transverse axis. If Q divides
on the curve. A line drawn from the point P intersects a2 b2
the curve at points Q and R. If the product PQ. PR is AP in the ratio a2 : b2, then NQ is
independent of the slope of the line, then the curve is a (A) ⊥ to A′ P (B)  parallel to A′P
(A) parabola (B)  circle (C) ⊥ to OP (D)  none of these
(C) ellipse (D)  none of these 1
64. An ellipse has eccentricity and one focus at the
58. A tangent to the ellipse x + 4y = 4 meets the ellipse x
2 2 2
æ1 ö
2
+ 2y2 = 6 at P and Q. The angle between the tangents at point P ç , 1÷ . Its one directrix is the common tan
PRACTICE EXERCISES

P and Q of the ellipse x2 + 2y2 = 6 is è2 ø


gent, nearer to the point P, to the circle x2 + y2 = 1 and
p p
(A) (B) × × the hyperbola x2 – y2 = 1. The equation of the ellipse in
2 3 the standard form is
p p 2
(C) (D)  æ 1ö
ç x - 3 ÷ ( y - 1) 2
× ×

4 6
è ø + =1
59. The maximum area of an isosceles triangle inscribed (A)
1 1
x2 y2
in the ellipse 2 + 2 = 1 with its vertex at one end of 9 12
a b 2
the major axis is æ 1ö
3 3 ç x - 3 ÷ ( y - 1) 2
(A) 3ab (B) 
4
ab (B) è ø + =1
1 1
5 3
(C) ab (D)  none of these 12 9
4 2
æ 1ö
ç y -
3 ÷ø
x2 y2
(C) ( x - 1) + è
2
60. The tangent at the point ‘α’ on the ellipse + =1
a2 b2 =1
1 1
meets the auxiliary circle in two points which sub-
tend a right angle at the centre. The eccentricity of the 9 12
ellipse is (D) none of these
Conic Sections (Parabola, Ellipse and Hyperbola)  12.43

65. A variable straight line of slope 4 intersects the hyper- (A) 16x2 + 10xy + y2 = 2
bola xy = 1 at two points. The locus of the point which (B) 16x2 – 10xy + y2 = 2
divides the line segment between these two points in (C) 16x2 + 10xy + y2 = 4
the ratio 1 : 2 is (D) none of these
Previous Year's Questions
66. The radius of the circle passing through the foci of the 72. If a ≠ 0 and the line 2bx + 3cy + 4d = 0 passes through
x2 y2 the points of intersection of the parabolas y2 = 4ax and
ellipse + = 1 and having its centre at (0, 3), is:
16 9 x2 = 4ay, then [2004]
 [2002]
(A) 4 unit (B)  3 unit (A) d + (2b + 3c) = 0
2 2
(B)  d + (3b + 2c)2 = 0
2

7 (C) d + (2b – 3c) = 0


2 2
(D)  d2 + (3b– 2c)2 = 0
(C) 12 unit (D)  unit
2
73. The eccentricity of an ellipse, with its centre at the ori-
67. The equation of the ellipse whose foci are (±2, 0) and 1
1 gin, is . If one of the directrices is x = 4, then the 2
eccentricity is is: [2002] 2
2 equation of the ellipse is [2004]
2 2
x2 y2 x y (A) 3x2 + 4y2 = 1 (B)  3x2 + 4y2 = 12
(A) + = 1 (B) + =1
12 16 16 12 (C) 4x2 + 3y2 = 12 (D)  4x2 + 3y2 = 1
x2 y2
(C) + = 1 (D)  none of these
16 8 74. Area of the greatest rectangle that can be inscribed in
x2 y2
68. The equation of the chord joining two points (x1, y1) the ellipse 2 + 2 = 1 is [2005]
a b
and (x2, y2) on the rectangular hyperbola xy = c2 is : (A) 2ab (B)  ab
 [2002] a
x y (C) ab (D) 
(A) + =1 b
x1 + x2 y1 + y2
75. The locus of a point P(α, β) moving under the condi-
x y tion that the line y = α x + β is a tangent to the hyper-
(B) + =1
x1 - x2 y1 - y2 x2 y2
bola 2 - 2 = 1 is [2005]
x y a b
(C) + =1 (A) an ellipse (B)  a circle
y1 + y2 x1 + x2
(C) a parabola (D)  a hyperbola
x y
(D) + =1 76. An ellipse has OB as semi minor axis, F and F′ its
y1 - y2 x1 - x2 focii and the angle FBF′ is a right angle. Then the

PRACTICE EXERCISES
69. If x1, x2, x3 and y1, y2, y3 are both in G.P. with the same ­eccentricity of the ellipse is [2005]
common ratio, then the points (x1, y1) (x2, y2) and (x3, y3) 1 1
 [2003] (A) (B) 
2 2
(A) lie on a straight line
1
(B) lie on an ellipse (C) 1 (D) 
(C) lie on a circle 4 3
(D) are vertices of a triangle 77. In an ellipse, the distance between its foci is 6 and
x2 y2 minor axis is 8. Then its eccentricity is [2006]
70. The foci of the ellipse + = 1 and the hyper-
x2 y2 1 16 b 2 3 1
bola - = coincide. Then the value of b2 is (A) (B) 
144 81 25 5 2
 [2003]
4 1
(A) 1 (B)  5 (C) (D) 
(C) 7 (D)  9 5 5
71. A point on the parabola y2 = 18x at which the ordinate
increases at twice the rate of the abscissa is [2004] x2 y2
78. For the hyperbola - = 1, which of the
(A) (2, 4) (B)  (2, −4) cos 2 a sin 2 a
following remains constant when a. varies? [2007]
æ -9 9 ö æ9 9ö (A) eccentricity (B)  directrix
(C) ç , ÷ (D)  ç , ÷
è 8 2 ø è8 2ø (C) abscissae of vertices (D)  abscissae of foci
12.44  Chapter 12

79. A focus of an ellipse is at the origin. The directrix 85. Given: A circle, 2x2 + 2y2 = 5 and a parabola,
1
is the line x = 4 and the eccentricity is . Then the y 2 = 4 5 x . [2013]
2 Statement - I: An equation of a common tangent to
length of the semi-major axis is [2008]
these curves is y = x + 5.
2
(A) 8 (B)  5
3 3 Statement - II: If the line, y = mx + ( m ¹ 0) is
m
4 5
(C) (D)  their common tangent, then m satisfies m4 – 3m2 + 2 = 0.
3 3
(A)  Statement - I is True; Statement -II is true;
80. A parabola has the origin as its focus and the line x = Statement-II is not a correct explanation for
2 as the directrix. Then the vertex of the parabola is at Statement-I
 [2008] (B) Statement -I is True; Statement -II is False.
(A) (0, 2) (B)  (1, 0) (C) Statement -I is False; Statement -II is True
(C) (0, 1) (D)  (2, 0) (D)  Statement -I is True; Statement -II is True;
Statement-II is a correct explanation for
81. The ellipse x2 + 4y2 = 4 is inscribed in a rectangle Statement-I
aligned with the coordinate axes, which in turn in 86. The circle passing through the foci of the ellipse
inscribed in another ellipse that passes through the x2 y2
+ = 1 with center at (0, 3) has equation  [2013]
point (4,0). Then the equation of the ellipse is [2009] 16 4
(A) x2 + 16y2 = 16 (B)  x2 + 12y2 = 16 (A) x2 + y2 – 6y + 7 = 0 (B)  x2 + y2 – 6y – 5 = 0
(C) 4x + 48y = 48
2 2
(D)  4x2 + 64y2 = 48 (C) x + y – 6y + 5 = 0
2 2
(D)  x2 + y2 – 6y – 7 = 0
82. If two tangents drawn from a point P to the parabola y2 87. The locus of the foot of the perpendicular drawn from
= 4x are at right angles, then the locus of the point P is the centre of the ellipse x2 + 3y2 = 6 on any tangent to
 [2010] it is [2014]
(A) 2x + 1 = 0 (B) x = –1
(A) (x2 – y2)2 = 6x2 + 2y2
(C) 2x – 1 = 0 (D) x = 1
(B) (x2 – y2)2 = 6x2 – 2y2
83. Statement 1: An equation of a common tangent to the (C) (x2 + y2)2 = 6x2 + 2y2
parabola y 2 = 16 3 x and the ellipse [2012] (D) (x2 + y2)2 = 6x2 – 2y2

2x2 + y2 = 4y = 2x + 2 3
88. The slope of the line touching both the parabolas y2 =
4x and x2= –32 y is [2014]
4 3
PRACTICE EXERCISES

Statement 2: If the line y = mx + , ( m ¹ 0) is a 1 3 1 2


m (A)  (B)  (C)  (D) 
2 2 8 3
common tangent to the parabola y 2 = 16 3 x and the
89. Let O be the vertex and Q be any point on the parabola,
ellipse 2x2 + y2 = 4, then m satisfies m4 + 2m2 = 24. x2 = 8y. If the point P divides the line segment OQ inter-
(A) Statement 1 is false, statement 2 is true nally in the ratio 1 : 3, then the locus of P is: [2015]
(B) Statement 1 is true, statement 2 is true; statement (A) y2 = x (B)  y2 = 2x
2 is a correct explanation for statement 1 (C) x = 2y
2
(D)  x2 = y
(C) Statement 1 is true, statement 2 is true; statement
2 is not a correct explanation for statement 1 90. The area (in sq. units) of the quadrilateral formed by
(D) Statement 1 is true, statement 2 is false the tangents at the end points of the latus rectum to the
x2 y2
84. An ellipse is drawn by considering a diameter of the ellipse + = 1, is: [2015]
circle (x − 1)2 + y2 = 1 as its semi-minor axis and a 9 5
27
diameter of the circle x2 + (y − 2)2 = 4 as its semi-major (A) 18 (B)  27 (C) 27 (D) 
axis. If the centre of the ellipse is the origin and its 2 4
axes are the coordinate axes, then the equation of the 91. The eccentricity of the hyperbola whose length of the
ellipse is [2012] latus rectum is equal to 8 and the length of its conju-
(A) 4x + y = 4
2 2
(B)  x + 4y = 8
2 2
gate axis is equal to half of the distance between its
(C) 4x2 + y2 = 8 (D)  x2 + 4y2 = 16 foci, is [2016]
Conic Sections (Parabola, Ellipse and Hyperbola)  12.45

4
(A) 3 (B)  (A) ( 2 2 , 3 3 ) (B) 
( 3, 2 )
3
(C) ( − 2 , − 3 ) (D)  (3 2 , 2 3 )
4 2
(C) (D)  96. Tangent and normal are drawn at P(16, 16) on the
3 3
parabola y2 = 16x, which intersect the axis of the parab-
92. Let P be the point on the parabola, y2 = 8x which is at ola at A and B, respectively. If C is the centre of the
a minimum distance from the cente C of the circle, x2 circle through the points P, A and B and ∠CPB = θ,
+ (y + 6)2 = 1. Then the equation of the circle, passing then a value of tan θ is
through C and having its centre at P is [2016] 1 4
(A) x + y – 4x + 9y + 18 = 0
2 2 (A)  (B) 2 (C) 3 (D) 
2 3
(B) x2 + y2 – 4x + 8y + 12 = 0
(C) x2 + y2 – x + 4y + 12 = 0 97. Tangents are drawn to the hyperbola 4x2 – y2 = 36 at the
x points P and Q. If these tangents intersect at the point
(D) x 2 + y 2 - + 2 y - 24 = 0
4 T(0, 3) then the area (in sq. units) of ΔPTQ is
93. The centres of those circles which touch the circle, x2 (A) 45 5 (B) 54 3 (C) 60 3 (D) 36 5
+ y2 – 8x – 8y – 4 = 0, externally and also touch the
x-axis, lie on [2016] ì y2 x2 ü
98. Let S = í( x, y ) Î R 2 : - = 1ý where r ≠ ± 1.
(A) A parabola î 1 + r 1 - r þ
(B) A circle
Then S represents: [2019]
(C) An ellipse which is not a circle 2
(A) an ellipse whose eccentricity is , when r > 1.
(D) A hyperbola r +1
94. The eccentricity of an ellipse whose centre is at the
1 2
(B) a hyperbola whose eccentricity is , when 0 < r < 1.
origin is . If one of its directrices is x = –4, then the r +1
2
⎛ 3⎞
equation of the normal to it at ⎜1, ⎟ is [2017] 1
⎝ 2⎠ (C) an ellipse whose eccentricity is , when r > 1.
r +1
(A) 4x – 2y = 1 (B)  4x + 2y = 7
2
(C) x + 2y = 4 (D)  2y – x = 2 (D) a hyperbola whose eccentricity is , when 0 < r < 1.
1− r
95. A hyperbola passes through the point P( 2 , 3 ) and
has foci at (± 2, 0). Then the tangent to this hyperbola 99. The length of the chord of the parabola x2 = 4y having
at P also passes through the point [2017] equation x – 2 y + 4 2 0 is: [2019]

(A)  2 11 (B) 8 2 (C) 


3 2 (D) 
6 3

PRACTICE EXERCISES
ANSWER K EYS
Single Option Correct Type
1. (B) 2. (A) 3. (B) 4. (A) 5. (B) 6. (B) 7. (A) 8. (A) 9. (C) 10. (C)
11. (C) 12. (B) 13. (D) 14. (D) 15. (C) 16. (C) 17. (C) 18. (D) 19. (D) 20. (B)
21. (A) 22.  (D) 23.  (A) 24. (B) 25. (A) 26. (A) 27. (C) 28. (C) 29. (C) 30. (B)
31. (B) 32. (B) 33. (A) 34. (C) 35.  (B) 36. (C) 37. (A) 38. (A) 39. (A) 40. (C)
41. (B) 42. (B) 43. (A) 44. (B) 45. (D) 46. (A) 47. (C) 48. (C) 49. (A) 50. (B)
51. (A) 52. (B) 53. (B) 54. (B) 55. (B) 56. (B) 57. (B) 58. (A) 59. (B) 60. (A)
61. (B) 62. (A) 63. (A) 64. (A) 65. (A)
Previous Years’ Questions
6 6. (A) 67. (B) 68. (A) 69. (A) 70. (C) 71. (D) 72. (A) 73. (B) 74. (A) 75. (D)
76. (A) 77. (A) 78. (A) 79. (B) 80. (B) 81. (B) 82. (B) 83. (D) 84. (A) 85. (D)
86. (C) 87. (A) 88. (C) 89. (C) 90. (B) 91. (D) 92. (B) 93. (A) 94. (A) 95. (A)
96. (B) 97. (A) 98. (A) 99. (D)
12.46  Chapter 12

HINTS AND EXPLANATIONS


Single Option Correct Type
1. The general equation of a parabola having its axis parallel to ⇒ ( y2 + 6y + 9) - 2(x + 2) = 0
y-axis is or, ( y + 3)2 = 2(x + 2).
y = ax2 + bx + c(1) Clearly, x = 2t2 - 2 and y = 2t - 3 satisfy it for all t.
Since the line y = x touches the parabola at x = 1, therefore
slope of the tangent at (1, 1) = 1 6. Directirx of y2 = 4(x + 1) is x = -2
æ dy ö Any point on x = -2 is (-2, k)
i.e., ç ÷ = 1   or  2a + b = 1.
è dx ø(1, 1) Now, mirror image (x, y) of (-2, k) in the line
Also, x = ax2 + bx + c must have equal roots x + 2y = 3 is given by
⇒ (b - 1)2 = 4ac.
Since, (1, 1) lies on (1), x+2 y-k æ -2 + 2k - 3 ö
= = -2 ç ÷
\ a + b + c = 1. 1 2 è 5 ø
Now, 2a + b = 1 and a + b + c = 1 ⇒ a - c = 0 or a = c. 10 - 4 k 4k
\ a + b + c = 1 ⇒ 2c + b = 1 Þ x= -2 Þ x =- (1)
\ 5 5
⇒ 2f(0) + f ′(0) = 1 ( f(0) = c and f ′(0) = b)
20 - 8k
2. The equation of axis of the parabola is x - 4 = 0 which is
Also, y= (2)
5
parallel to y-axis. So, the ray of light is parallel to the axis of
the parabola. We know that any ray parallel to the axis of a
From (1) and (2)
parabola passes through the focus after reflection. 3 æ 5x ö
y = 4 + ç ÷
\ The ray must pass through the point (4, -1). 5è 4 ø
3. Clearly, the two tangents, having slopes m1 and m2, meet on or 4y = 16 + 3x is the equation of the mirror image of
the line x = -2, which is the directrix of the parabola y2 = 8x, the directrix.
therefore the two tangents must be at right angles, i.e., m1m2 7. Coordinates of any point on the parabola
= -1. y2 = -4ax are (-at2, 2at).
4. Equation of the line parallel to the axis and bisecting the Equation of the normal at (-at2, 2at) is
ordinate PN of the point P(at2, 2at) is y = at which meets the y - xt = 2at + at3
æ1 ö If the normal passes through the point (h, k), then
HINTS AND EXPLANATIONS

parabola y2 = 4ax at the point Qç at 2 , at ÷ .


è4 ø k - th = 2at + at3
Coordinates of N are (at2, 0). or, at3 + (2a + h)t - k = 0,
0 - at which is a cubic equation whose three roots t1, t2, t3 are
Equation of NQ is y = ( x - at 2 ),
1 2 the parameters of the feet of the three normals.
at - at
2

4 2
\ Sum of the roots = t1 + t 2 + t3 = - Coefficient of t = 0
Coefficient of t 3
\ Centroid of the triangle formed by the feet of the
normals
æ a 2a ö
= ç - (t12 + t 22 + t32 ), (t1 + t 2 + t3 )÷
è 3 3 ø
æ a ö
= ç - (t12 + t 22 + t32 ), 0 ÷
è 3 ø

which, clearly, lies on the x-axis.

which meets the tangent at the vertex, x = 0, at the point 8. The coordinates of any point on the parabola x2 = 4by are
(2bt, bt2).
4 dy x
y = at . For the parabola x2 = 4by, = .
3 dx 2b
5. The equation of the parabola is
æ 2b 1
3ö æ 2x + 5 ö
2 2
Slope of the normal at (2bt, bt2) = -
=-
ç + ÷ + ( y + 3) = ç
2
x ÷ 2bt t
è 2ø è 2 ø
1
é 9 ù \ Equation of normal is y - bt2 = - ( x - 2bt )
Þ 4 ê x 2 + + 3 x ú + 4[ y 2 + 9 + 6 y ] = ( 4 x 2 + 25 + 20 x ) t
ë 4 û
Conic Sections (Parabola, Ellipse and Hyperbola)  12.47

x Since OX and OY are mutually perpendicular tangents to


or, y = -
+ 2b + bt 2 sliding ellipse for all its positions, therefore, O(0, 0) will lie
t
It will touch the parabola y2 = 4ax if on circle (1)
a æ aö \ a2 + b 2 = a2 + b2.
2b + bt 2 =   ç ∵ c = ÷
-1 / t è mø Hence, locus of C(a, b) is x2 + y2 = a2 + b2.
⇒ bt2 + at + 2b = 0 12. P(a cosq, bsinq), then q is angle of a corresponding pont on
For distinct real roots, discriminant > 0 auxilliary circle x2 + y2 = a2 i.e., (a cosq, a sinq).
⇒ a2 - 8b2 = 0  or  a2 > 8b2 13. Let S(5, 12), S′(24, 7) be the two foci.
9. The coordinates of the focus of the given parabola are (a + P(0, 0) is a point on the conic.
1, b). SP = 25 + 144 = 13
S′P = 576 + 46 = 625 = 25

SS′ =
( 24 - 5) 2+ (7 - 12) 2
= 192 + 52 = 386

If the conic is an ellipse,
then, SP + S′P = 2a and SS′ = 2ae

SS¢ 386 386
\ e= = =
SP + S¢P 13 + 25 38
14. Given ellipses are x2 + 4y2 = 4
Clearly, focus must lie to the opposite side of the origin w.r.t. x2 y2
i.e., 2 + 2 = 1 (1)
the line x + y - 1 = 0 and same side as origin with respect to 2 1
the line x + y - 3 = 0. Hence, a + b > 0 and a + b < 2. x2 y2
x2 y2 and, x2 + 2y2 = 6  i.e.,  
2
+ = 1 (2)
10. Let the equation of the ellipse be 2 + 2 = 1 . ( 6) ( 3 )2
a b
Let R(a, b) be the point of intersection of the tangents to
Let e be the eccentricity of the ellipse.
ellipse (2) at P and Q. then PQ will be chord of contact of R.
Since distance between foci = 2h
\ its equation is
\ 2ae = 2h ⇒ ae = h(1)
Focal distance of one end of minor axis say (0, b) is k. ax b y
+ =1

HINTS AND EXPLANATIONS


\ a + e(0) = k ⇒ a = k(2) 6 3
From (1) and (2) i.e., ax + 2yb = 6
b2 = a2(1 - e2) = k2 - h2. a 3
\ The equation of the ellipse is or, y = -
x + (3)
2b b
x2 y2
Since (3) touches (1)
2 + 2 = 1.
k k - h2
æ 3ö
2
a2
11. Let the two given lines be taken as the coordinate axes. \ ç ÷ = 22 × + 12 [c2 = a2m2 + b2]
è bø 4 b2
Let C(a, b) be the centre of the ellipse in any position.
Here the position of centre C changes as the ellipse slides. 9 a2 a2 + b2
Let a and b be the semi-major and semi-minor axes of the Þ = 2 + 1=
b 2
b b2
ellipse.
⇒ a 2 + b 2 = 9
Equation of the director circle of the ellipse is
\ locus of (a , b ) is
(x - a)2 + (y - b)2 = a2 + b2 (1)
x2 + y2 = 9 = ( 6 ) + ( 3 )
2 2

i.e., director circle.


\ tangent at P, Q meet at right angles.
15. Let S and S′ be the centres and a and b be the radii of the
given circles.
Let P be the centre and r be the radius of the circle
which touches the given circles externally.
Then, S′P = r + a and SP = r + b
\ S′P - SP = (r + a) - (r + b) = a - b = constant.
12.48  Chapter 12

Let P(a, b) be the mid point of any of the parallel


chords whose slope is -7.
\ equation of the chord in terms of mid-point P(a, b) is
T = S1
xa y b a2 b2

i.e., - -1 = - -1
3 7 3 7
ax b y a2 b2
Hence, the locus of P is a hyperbola whose foci are S and S′.
i.e., - = -
3 7 3 7
16. Let the two given ^ lines be the coordinate axes and let the
7a
equation of variable circle be Its slope =
= -7   ⇒  a + 3b = 0
3b
x2 + y2 + 2gx + 2f y + c = 0 (1)
\  locus of (a, b) is x + 3y = 0.
Then, 5 = 2 g 2 - c and 3 = 2 f 2 - c .

20. Clearly, the common tangent to the circle x2 + y2 = 1 and
Squaring and subtracting these, we get hyperbola x2 - y2 = 1 is x =1 [which is nearer to P(1/2, 1)].
4(g2 - c) - 4( f  2 - c) = 25 - 9 æ1 ö
⇒ g2 - f 2 = 4 or (-g)2 - (- f )2 = 4. Given, one focus at P ç , 1÷ .
è2 ø
Hence, locus of the centre (-g, -f ) of circle is \  equation of the directrix is x = 1.
x2 - y2 = 4, \  ellipse is
which is a rectangular hyperbola.
æ 1ö
2
17. We have, 2x2 + 3y2 - 8x - 18y + 35 = k 1
ç x - ÷ + ( y - 1) 2 = ( x - 1)
è 2ø 2
⇒ 2(x2 - 4x) + 3( y2 - 6y) + 35 = k
⇒ 2[(x - 2)2 - 4] + 3[( y - 3)2 - 9] + 35 = k
⇒ 2(x - 2)2 + 3( y - 3)2 = k
For k = 0, we get 2(x - 2)2 + 3(y - 3)2 = 0 which represents
the point (2, 3).
18. Let P(asecq, btanq); Q(a secq, -btanq) be end points of dou-
ble ordinate and C(0, 0), is the centre of the hyperbola. Now,
PQ = 2b tanq
CQ = CP = a 2 sec 2q + b 2 tan 2q
HINTS AND EXPLANATIONS

since CQ = CP = PQ
\ 4b2 tan2q = a2 sec2q + b2 tan2q
⇒ 3b2 tan2q = a2 sec2q
On simplification, it becomes
2
æ 1ö
9 ç x – ÷ + 12( y – 1) 2 = 1.
è 3ø
21. Since the line CP passes through the origin, i.e., centre, let
its equation be y = mx. The line CP meets the hyperbola
x2 y2
2 - 2 = 1 in P whose abscissa is given by
a b
x 2 m2 x 2 a2b2
⇒ 3b2 sin2q = a2 2 - = 1 or x 2 = 2
a b 2
b - a2 m2
⇒ 3a2(e2 - 1) sin2q = a2
⇒ 3(e2 - 1)sin2q = 1 a 2b 2 m 2
\ y2 = m2x2 =
1 b2 - a2m2
Þ = sin 2 q < 1 (∴   sin2q < 1)
3(e - 1)
2
\ CP2 = x2 + y2
1 1 a 2b 2 + a 2b 2 m 2
Þ < 3    Þ e 2 - 1 > =
2
e -1 3 b2 - a2m2
x2 y2 = a b (1 + m2)
2 2

19. Given hyperbola: - = 1 (1)


3 7 Since CQ ^ CP

Given chord: 7x + y - 2 = 0 (2) 1
Replace m by -
, we get
Its slope = -7 m
Conic Sections (Parabola, Ellipse and Hyperbola)  12.49

æ 1 ö b
a 2b 2 ç1 + 2 ÷ Dividing (1) by (2), we get t 2 =
.
m ø a b ( m + 1)
2 2 2
CQ = 2 è = 2 2
k

c 2
b m - a2 b b ab
b2 - 2
Putting in (1), + 1= Þ k= .
m k a b- a
1 1 b2 - a2m2 + b2m2 - a2
\ + = 24. Let S ≡ x2 - 4y
CP 2 CQ 2 a 2b 2 (1 + m 2 )
Since the point (2a, a) lies inside the parabola,
b2 - a2 1 1
= = 2– 2 \ S](2a, a) = 4a2 - 4a < 0
a 2b 2 a b
i.e., 4a(a - 1) < 0
22. Given, P(a secq, b tanq) and Q(a secf, b tanf).
or, a(a - 1) < 0 (1)
The equation of tangent at point P is
Also, the vertex A(0, 0) and the point (2a, a) are on the same
x sec q y tan q
- =1 side of the line y = 1 (the equation of latus-rectum)
a b
b sec q b 1
Slope of tangent = ´ = ×
tan q a a sin q
Hence, the equation of perpendicular at P is
a sin q
y - b tan q = - ( x - a secq )
b
or, by -b2 tanq = -a sinq x + a2 tanq
or, a sinq x + by = (a2 + b2) tanq(1)
Similarly, the equation of perpendicular at Q is
a sinf x + by = (a2 + b2) tanf(2) So, a - 1 < 0 i.e., a < 1 (2)
On multiplying (1) by sinf and (2) by sinq, we get From (1) and (2), we have a (a - 1) < 0
a sinq sinf x + b sinf y = (a2 + b2) tanq sinf or, 0 < a < 1.
a sinf sinq x + b sinq y = (a2 + b2) tanf sinq 25. We know any side of the triangle is more than the difference
On subtraction we get of remaining two sides, such that |PR - PQ| ≤ RQ.
by (sinf - sinq) = (a2 + b2)(tanq sinf - tanf sinq) ⇒ The required piont P will be the point of intersection of
a 2 + b 2 tan q sin f - tan f sin q the line RQ with parabola which is (a, 2a) as RQ is a tangent
\ y=k = ×
b sin f - sin q to the parabola.
26. Normal at a point (m2, - 2m) on the parabola y2 = 4x is given

HINTS AND EXPLANATIONS


p p
∵ q +f = Þ f = -q by y = mx - 2m - m3. If this is normal to the circle also, then
2 2
⇒ sinf = cosq and tanf = cotq it will pass through centre of the circle so
a 2 + b 2 tan q cosq - cosq sin q 6 = -3m - 2m - m3 ⇒ m = -1
\ y=k = ×
b cosq - sin q Since shortest distance between parabola and circle will
a 2 + b 2 æ sin q - cosq ö (a2 + b2 ) occur along common normal, shortest distance is 4 2 – 5 .
= ç ÷ =-
b è cosq - sin q ø b 27. Tangent at P is ty = x + at2, which meets axis at
23. If coordinates of one end of focal chord are P(at2, 2at) then T(-at2, 0). Normal at P is tx + y = 2at + at3, which meets axis
æ a - 2a ö at G(2a + at2, 0)
the coordinates of other end will be Qç 2 , ÷. p
èt t ø \ Ð TPG = , so TG is diameter of the circle.
2
\ SP = ( at 2 - a) 2 + ( 2at - 0) 2
Equation of the circle is
= a(t2 + 1) = b   (given) (1)
(x + at2)(x - 2a - atI2) + (y - 0)(y - 0) = 0
æ a ö æ - 2a ö
2 2

and, SQ =
ç 2 - a÷ + ç - 0÷ ⇒ x2 + y2 - 2ax - at2(2a + at2) = 0
èt ø è t ø
Tangent to above circle at P(at2, 2at) is
æ1 ö xat2 + y . 2at - a(x + at2) - a2t2(1 + t2) = 0
= a ç 2 + 1÷ = k (given) (2)
1- t 2
è t ø or, (t2 - 1)x + 2ty - a(2 + t2) = 0. It has slope =
2t
\ Angle q between two tangents is given by
1 1- t 2
-
2t = t (1+ t )   ⇒  q = tan- 1 (t)
2
tan q = t
1 1- t 2
t +1
2
1+ ×
t 2t
12.50  Chapter 12

28. Equation of normal is y = mx - 2am - am3 1 3


y + t(x - 3) - t - t =0
Put y = 0, we get 2
Both are same if -2m - m3 = -4m - 1/2m3
x1 = 2a + am12
⇒ m = 0, ±2
x2 = 2a + am22 So, points will be (4, 4) and (5, 2)
and, x3 = 2a + am33
Hence, shortest distance will be = 1+ 4 = 5 .
where x1, x2, x3 are intercepts on the axis of the parabola. The 33. These are two common tangents to the circle x2 + y2 = 1 and
normal passes through (h, k). the hyperbola x2 - y2 = 1. These are x = 1 and x = -1
\ am3 + (2a - h)m + k = 0
Now, m1 + m2 + m3 = 0
2a - h
m1m2 + m2m3 + m3m1 =
a
⇒ m12 + m22 + m32 = (m1 + m2 + m3)2 - 2(m1m2 + m2m3 +
m3m1)
( 2a - h)
=-2
a
⇒ x1 + x2 + x3 = 6a - 2(2a - h) = 2(h + a)
29. The focus of the parabola y2 = 8ax is (2a, 0). Out of these, x = 1 is nearer to the point P(1/2, 1). Thus, a

So, the coordinates of the point on the axis of the parabola directrix of the required ellipse is x = 1.
at a distance 8a from the focus is (10a, 0). Equation of a
normal to the parabola y2 = 8ax is
If Q(x, y) is any pont on the ellipse, then its distance from the
y = mx - 4am - 2am3
æ 1ö
2

Since it passes through (10a, 0), focus is QP = ç x - ÷ + ( y - 1) and its distance from
2

\ 0 = 10am - 4am - 2am3 è 2ø


∴ the directrix x = 1 is |x - 1|
⇒ 2am(3 - m2) = 0 ⇒ m2 = 3 (  m ≠ 0)
æ pö By definition of ellipse, QP = e|x - 1|

Þ m = ± 3 = tanç ± ÷
è 3ø æ 1ö
2
1
30. Let the three points A, B and C on the parabola are A(x1, y1), Þ ç x - ÷ + ( y - 1) = | x - 1 |
2

è 2ø 2
B(x2, y2) and C(x3, y3), respectively.
⇒ 3x2 - 2x + 4y2 - 8y + 4 = 0
Also, y2 = y1 y3 {given}
2

HINTS AND EXPLANATIONS

æ 1ö
2
If T be the point of intersection of tangents at y1 and y3 then
or, çè x – 3 ÷ø ( y – 1) 2
T is + = 1.
æ y y y + y3 ö æ y 2 y + y3 ö 1/ 9 1 / 12
Tç 1 3, 1 ÷ =Tç 2 , 1 ÷ 34. Let the point of contact be
è 4a 2 ø è 4a 2 ø
R º ( 2 cosq , sin q )
æ y + y3 ö

or, T ç x2 , 1  (∵ y22 = 4 ax2 )
Equation of tangent AB is
è 2 ÷ø
This point lies on x = x2 which is a line through B(x2, y2)

parallel to y-axis.
31. y2 = 4ax, Normal: y = mx - 2am - am3 (1)
y2 = 4c(x - b), Normal: y = m(x - b) - 2cm - cm3 (2)
Since the two parabolas have a common normal, therefore
(1) and (2) must be identical
After comparing the coefficients we get
2( a - c) - b b b
m=±  \ -2- >0 Þ >2
( c - a) c-a a–c
x
QUICK TIPS cosq + y sin q = 1
2
Shortest distance between two curves occurred along the
common normal
Þ A º ( 2 secq , 0 ) ; B º (0, cosecq )
Let the middle point Q of AB be (h, k)
32. Normal to y2 = 4x at (m2, 2m) is y + mx - 2m - m3 = 0 secq cosecq
Þ h= ,k =
Normal to y2 = 2(x - 3) at (1/2t2 + 3, t) is 2 2
Conic Sections (Parabola, Ellipse and Hyperbola)  12.51

1 1 p
Þ cosq = , sin q = 37. Equation of the tangent at × is
h 2 2k 4
Þ 1 + 1 =1 æ 1 ö æ 1 ö
2h 2 4 k 2 xç ÷ yç ÷
è

+ è 2 ø = 1 i.e. x + y - 2 = 0 (1)
1 1
\ Required locus is 2
+ = 1. a b a b
2x 4 y2 p
Trick: The locus of mid-points of the portion of tangents to Equation of the normal at is ×

4
x2 y2 x y a b
the ellipse 2 + 2 = 1 intercepted between axes is a2y2 +
a b - = - (2)
b a b 2 a 2
b2x2 = 4x2y2
p1 = length of the perpendicular from the centre to the
a2 b2 1 1
i.e., 2
+ = 1  or  2 + = 1. - 2
4x 4 y2 2x 4 y2 2ab
35. Point P goes to Q. Its direction with respect to x-axis is q in tangent =
1 1 =
+ a + b2 2
original position. In new position y-axis will play the role of a2 b2
major axis so its inclination with negative direction of y-axis p2 = length of the perpendicular from the centre to the
will be same. So, new coordinates will be (b sinq, -a cosq ).
a b
-
b 2 a 2= a2 - b2
normal = .
1 1 2 a2 + b2
+
a2 b2
ab( a 2 - b 2 )
Area of the rectangle = p1 p2 =
.
a2 + b2
38. If S1 = 0 and S2 = 0 are the equations,
then, lS1 + S2 = 0 is a second degree curve passing through
the points of intersection of S1 = 0 and S2 = 0.
⇒ (l + 4)x2 + 2(l + 1)y2 - 2(3l + 10)x
- 12(l + 1)y + (23l + 35) = 0 (1)
For it to be a circle, choose l such that the coefficients of x2
x2 y2 and y2 are equal:
36. Given, + =1 ⇒ l + 4 = 2l + 2

HINTS AND EXPLANATIONS


a2 b2
b2 \ l = 2
\ OS = ae = a 1- = a2 - b2 This gives the equation of the circle as
a2

So, two points on the minor axis are, 6(x2 + y2) - 32x - 36y + 81 = 0 {(using (1))}
S1 ( 0, a 2 - b 2 ) , S 2 ( 0, - a2 - b2 )
16 27
Þ x2 + y2 - x - 6 y + = 0.
3 2
æ8 ö
Its centre is C ç , 3 ÷ and radius is
è3 ø
64 27 1 47
r= +9– = .
9 2 3 2
39. Equation of the chord joining the points (acosq1, bsinq1) and
(acosq2, b sinq2) is given by
x q +q y q +q q -q
cos 1 2 + sin 1 2 = cos 1 2
a 2 b 2 2
which passes through (ae, 0), we have
Let tangent to the ellipse by y = mx + c = mx + a 2 m 2 + b 2
æq +q ö q -q
e cos ç 1 2 ÷ = cos 1 2
where m is parameter. Now, sum of the squares of ^’s on this è 2 ø 2
tangent from the points S1 and S′ 1 is
é q1 q2 q1 q 2 ù
æ a2 - b2 - a2 m2 + b2 ö æ Þ e êcos cos - sin sin ú
a 2 m 2 + b 2 ö÷
2 2
a2 - b2 - ë 2 2 2 2û
ç ÷ +ç-
ç ÷ ç ÷ q1 q2 q1 q 2
è 1+ m 2 ø è 1+ m 2 ø = cos cos + sin sin
2 2 2 2
æ a 2 - b 2 + a 2 m 2 + b 2 ö 2a 2 (1 + m 2 ) é q1 q2 ù q q
= 2ç ÷= = 2a 2 Þ e ê1 - tan tan ú = 1 + tan 2 tan 2
è 1 + m2 ø 1 + m2 ë 2 2 û 2 2
12.52  Chapter 12

q1 q i.e., x2 + y2 = a2 - b2
Þ (e - 1) = (e + 1) tan tan 2
2 2 director circle of given hyperbola is
q1 q2 e - 1 x2 + y2 = -11,
Þ tan × tan = which is (a)
2 2 e+ 1 which is not possible.
40. The coordinates of the given point P are (a cosa, b sina). 44. Given two hyperbolas are
b sin a - 0 b x2 y2
\ tan b = = tan a - = 1 (1)
a cos a - 0 a 9 16
tan a - tan b y2 x2
\ tan(a - b ) =
and, - = 1 (2)
1 + tan a × tan b 9 16
tan a ( a - b)
Equation of tangent to (1), having slope m is
= = l (say)
a + b tan 2 a y = mx ± 9m 2 - 16 (3)
dl ( a + b tan 2 a )( a - b)sec 2 a - ( a - b) tan a × 2b tan a sec 2 a Eliminating y using equation (2) and (3), we get
=
16 ( mx ± 9m 2 - 16 ) - 9 x 2 = 144
2
da ( a + b tan 2 a ) 2
( a - b)sec 2 a ( a - b tan 2 a ) Þ (16 m 2 - 9) x 2 ± 32m 9m 2 - 16 x + (144 m 2 - 400) = 0 (4)
=
( a + b tan 2 a ) 2
For it to be a tangent, we must have D = 0

dl
\ ( 32m 9m 2 - 16 ) = 4(16m2 - 9)(144m2 - 400)
2
For extremum, = 0  ⇒  a - b tan2a = 0
da
⇒ m2 = 1 ⇒ m = ±1
a
Þ tan a = . 45. Since the asymptotes of rectangular hyperbola are mutually
b perpendicular, the other asymptote should be 4x + 3y + l =
41. Since x cosa + y sina = p subtends a right angle at the centre 0. Also, intersection point of asymptotes is also the centre of
(0, 0), therefore the hyperbola. Thus, intersection point of 4x + 3y + l = 0 and
x2 y2 3x - 4y - 6 = 0
making equation of hyperbola 2 - 2 = 1 homogeneous
a b æ 18 - 4l -12l - 96 ö
with the help of xcosa + ysina = p
i.e., ç , ÷ should lie on the line x - y - 1 = 0.
è 25 100 ø
x 2 y 2 æ x cos a + y sin a ö
2
18 - 4l 12l - 96
we get 2 - 2 = ç ÷ \ - -1 = 0
a b è p ø 25 100
⇒ l = 17.
æ 1 cos 2 a ö 2æ 1 sin 2 a ö -2 sin a cos a xy
i.e., x 2 ç 2 - ÷ + y ç- 2 - ÷+ =0 Hence, the equation of other asymptote is 4x + 3y + 17 = 0
HINTS AND EXPLANATIONS

2

èa p ø è b p2 ø p2
46. Equation of normal at any point (ct, c/t) is ct4 - xt3 + ty - c = 0
Coefficient of x2 + coefficient of y2 = 0
⇒ Slope of normal = t2
1 cos 2 a 1 sin 2 a
Þ 2- - 2- = 0 Let P(h, k) be the point through which the normal is passing.
a p2 b p2
Then, ct4 - ht3 + tk - c = 0
1 1 1 ab ⇒ Sti = h/c and Stitj = 0
Þ 2
- 2 = 2 Þ p= .
a b p b - a2
2
Hence, sum of the slopes of the normal
Since p is also the length of the perpendicular from
åt (å t )
2
(0, 0) to the line x cos a + y sin a = p = i
2
= i = h2 = c 2 l
ab Therefore, required locus is x2 = lc 2

\ Radius of the circle = p = .
b - a2
2
5
47. We have, for the given hyperbola 9 = 16(e2 - 1) Þ e =
42. Let the directrix be x = a/e and focus be S(ae, 0). Let P(a 4
secq, btanq) be any point on the curve. Equation of tangent
Since (5, 0) satisfies the equation of the line
x secq y tan q 3 x + ( 5 - 4 2 ) y = 15, so the reflected ray must pass through
at P is - = 1 . Let F be the intersection point
a b (-5, 0) and P = ( 4 2 , 3)
æ a b(secq - e) ö
of tangent and the directrix, then F = ç ,
èe e tan q ÷ø
b(secq - e) b tan q
Þ mSF = , mPS =
- e tan q ( a 2 - 1) a(secq - e)
⇒ mSF × mPS = -1.
43. Director circle is the locus of point of intersection of perpen-
dicular tangents drawn to a curve:
Conic Sections (Parabola, Ellipse and Hyperbola)  12.53

\ equation of S′P is 52. Centre and radius of the given circle is P(6, 0) and,
2 y = x + 5. respectively.
Equation of normal for y2 = 4x at (t2, 2t) is y = -tx + 2t + t3.
QUICK TIPS The normal must pass though (6, 0) in order that it gives
The transverse axis is the bisector of the angle between the minimum distance between the two curves.
aymptotes containing the origin and the conjugate axis is
the other bisector.

48. The equations of the bisectors of the angles between the


asymptotes are
3x - 4 y - 1 4x - 3y - 6

5 5
So, the equations of transverse and conjugate axis are x +
y - 5 = 0 and x - y - 1 = 0.
49. Let ax + by = 1 be the chord (1)
\ 0 = t3 - 4t ⇒ t = 0 or t = ±2
Making the equation of hyperbola homogeneous using (1),
\ A (4, 4) and C (4, -4)
we get
3x2 - y2 + (-2x + 4y)(ax + by) = 0 PA = PC = 20 = 2 5
or, (3 - 2a)x2 + (-1 + 4b)y2 + (-2b + 4a)xy = 0 \ required minimum distance = 2 5 - 5 = 5 .
Since the angle subtended at the origin is a right angle, so, 53. Any point on the given parabola is (t2, 2t). The equation of
coefficient of x2 + coefficient of y2 = 0 the tangent at (1, 2) is x - y + 1 = 0
⇒ (3 - 2a) + (-1 + 4b) = 0 ⇒ a = 2b + 1
The image (h, k) of the point (t2, 2t) in x - y + 1 = 0 is given
\ The chords are (2b + 1)x + by - 1 = 0
h - t 2 k - 2t 2(t 2 - 2t + 1)
or, b(2 + y) + (x - 1) = 0, by = = =-
which, clearly, pass through the fixed point (1, -2). 1 -1 1+ 1
50. The coordinates of any point on the hyperbola are \ h = t2 - t2 + 2t -1 = 2t -1
( 24 secq , 18 tanq ) . and, k = 2t + t2 -2t + 1 = t2 + 1
Equation of tangent at this point is Eliminating t from h = 2t -1 and k = t2 + 1.
x sec q y tan g q we get (h + 1)2 = 4 (k - 1)
- = 1 (1)
24 18 The required equation of reflection is (x + 1)2 = 4(y -1).
The point is nearest to the line 54. Given parabola is y2 = 4ax(1)

HINTS AND EXPLANATIONS


3x + 2y + 1 = 0 (2) Let QP and PR be the incident and reflected rays, respec-
If (1) and (2) are parallel tively. Let PT be the tangent to the parabola at P and PN be
the normal to the parabola at P.
secq 18 3 1
Þ × =- Þ sin q = -
24 tan q 2 3
Thus, the point is (6, -3).
51. Let the point be (x1, y1),
Equation of chord of contact of tangents drawn from the
point (x1, y1) to the hyperbola x2 - y2 = a2 is
xx1 - yy1 = a2(1)
Equation of normal chord is
x y
+ = 2a (2)
secq tan q
Since (1) and (2) are identical, comparing coefficients in (1)
and (2), we get Let ∠QPN = q, then ∠RPN = q.
x - y1 a2 Let S be the focus of the parabola. Then, S ≡ (a, 0).
1 = =
1 / secq 1 / tan q 2a Let P ≡ (at2, 2at).
a -a Equation of tangent PT is
Þ sec q = and tan q =
2 x1 2 y1 1
yt = x + at2. Its slope = .
sec2q - tan2q = 1 t
a2 a2 \ Slope of normal PN = -t.
Þ - =1
Slope of PQ = 0. Let slope of PR = m.
4 x12 4 y12
\ The required locus is a2( y2 - x2) = 4x2y2. Equating the two values of tanq, we get
12.54  Chapter 12

0+t -t - m -t - m 56. Given parabola is y2 - 16x - 8y = 0 (1)


= Þ t=
1 + 0( -t ) 1 - m 1- m Let the coordinates of the feet of the normal from (14, 7) be
2t P(a, b).
⇒ t - t2m = - t - m ⇒ m = 2 .
t -1 Now, equation of the tangent at P(a, b) to parabola (1) is yb
2t - 8(x + a) - 4(y + b) = 0
\ Equation of PR is y - 2at = 2 ( x – at 2 ).
t –1 or, (b - 4)y = 8x + 8a + 4b(2)
a2 8
55. Given circle is x 2 + y 2 = (1) Its slope = .
2 b -4
and given parabola is y2 = 4ax(2) Equation of normal to parabola (1) at (a, b) is
4-b
y - b = ( x - a ).
8
It passes through (14, 7),
4-b 6b
\ 7-b = (14 - a ) or a = (3)
8 b -4
Also, (a, b) lies on parabola (1), \  b 2 - 16a - 8b = 0
(4)
Let PQ be a common tangent to the circle and the parabola. Putting the value of a from (3) in (4), we get
Let P ≡ (at2, 2at). 96 b
b2 - - 8b = 0   ⇒  b(b 2 - 12b - 64) = 0
Now, equation of PQ which is a tangent to the parabola at P b -4
is ⇒ b (b - 16) (b + 4) = 0.
ty = x + at2  or  x - ty + at2 = 0 (3) \ b = 0, 16, - 4.
Since PQ is also a tangent to circle (1) From (3), when b = 0, a = 0,
0 - 0 × t + at 2 a when b = 16, a = 8 and when b = -4, a = 3.
\ =
Hence, the feet of the normals are (0, 0) , (8, 16) and (3, - 4).
1+ t 2
2
-1 57. Given curve is ax2 + 2hxy + by2 = 1 (1)
⇒ 2t4 - t2 - 1 = 0 ⇒ t2 = 1, . Let P ≡ (a, b).
2
\ t2 = 1 ⇒ t = ±1. Let line PS make an angle q with the positive direction of
x-axis. Coordinates of any point on line PS may be taken as
Hence, there will be two points P′(a, 2a) and P (a, -2a) on
(a + rcosq, b + rsinq).
the parabola, the tangents at which will also be tangents to
HINTS AND EXPLANATIONS

the circle.
Now, equations of tangents to the parabola at P′(a, 2a) and
P(a, -2a) will be
y = x + a(4)
and, -y = x + a(5)
Solving (4) and (5), we get x = -a, y = 0
\ A ≡ (-a, 0).
a2
\ Equation of QQ′ will be - ax + y × 0 =
2
or, 2x = -a(6)
Equation of PP′ is y × 0 = 2a (x - a) or x = a(7) If point (a + rcosq, b + rsinq) lies on curve (1), then
æ -a a ö a(a + rcosq)2 + 2h(a + rcosq) (b + rsinq) + b(b + rsinq)2 = 1
Solving (4) and (6), we get Q′ ≡ ç , ÷ .
è 2 2ø ⇒ (acos2q + 2hcosq sinq + bsin2q)r2
æ a - aö + 2(aacosq + hbcosq + hasinq + bbsinq)r
Solving (5) and (6), we get Q ≡ ç - , ÷
è 2 2 ø + aa2 + 2hab + bb2 - 1 = 0 (2)
Solving (4) and (7), we get P′ ≡ (a, 2a). Equation (2) will give two real values of r say r1 and r2 and
Solving (5) and (6), we get P ≡ (a, -2a). corresponding to these two values of r we will get two points
Clearly, QPP’ Q’ is a trapezium, therefore its area Q and R on curve (1).
1 1 æa ö Also, PQ = | r1 | and PR = | r2 |
= ( QQ¢ + PP ¢ ) × LM = ( a + 4 a) ç + a ÷
2 2 è2 ø Now, PQ × PR = | r1 | | r2 | = | r1r2 |
1 a 15 2 aa 2 + 2hab + bb 2 - 1
= × 5a × 3 = a. =
2 2 4 a cos 2 q + b sin 2 q + h sin 2q
Conic Sections (Parabola, Ellipse and Hyperbola)  12.55

From (3), it is clear that PQ × PR will be independent of


x2 y2
slope of line PS, i.e., independent of q, if a = b and h = 0. 59. The given ellipse is + = 1.
a2 b2
Thus, equation of curve will become a(x2 + y2) = 1,


which is a circle.
58. We can write the ellipse x2 + 4y2 = 4 as
x2
+ y 2 = 1 (1)
4

Equation of any tangent to the ellipse (1) can be written as
x
cosq + y sin q = 1 (2)
2

Let A ≡ (acosq, -bsinq)



Then, C ≡ (acosq, -bsinq)

1
D = Area of ΔABC = ´ AC ´ BD = AD × BD

2
= bsinq(a - acosq)

1
= ab( 2 sin q - sin 2q )
2

Equation of the second ellipse can be written as dD 1

Now, = ab( 2 cosq - 2 cos 2q ) = 0 (1)
x2 y2 dq 2
+ = 1 (3)
6 3 ⇒ cos2q = cosq ⇒ 2cos2q - cosq - 1 = 0
Suppose, the tangents at P and Q meet in A(h, k). Equation ⇒ (2cosq + 1) (cosq - 1) = 0
of the chord of contact of the tangents through A(h, k) is
-1
hx ky ⇒ cosq = or cosq = 1
+ = 1 (4) 2
6 3
If q = 0, D = 0, which is not possible.
Since (4) and (2) represent the same line
\ q = 2p /3.
h/6 k /3 1
= =
\ cosq sin q 1 3 3
\ D max = ab [substitiuting the value of q in (1)]

HINTS AND EXPLANATIONS


2 4
⇒ h = 3 cosq and k = 3sinq.
Thus, coordinates of A are (3cosq, 3sinq). x2 y2
60. Given ellipse is + = 1 (1)
The joint equation of the tangents at A is given by T2 = SS1 a2 b2
2
æ x2 y2 ö æ h2 k 2 ö
i.e., æç + - 1ö÷ = ç +
hx ky
- 1÷ ç + - 1÷  (5)
è 6 3 ø è 6 3 øè 6 3 ø
Let a = coefficient of x in (5)
2

h2 1 æ h2 k 2 ö -k 2 1
= - ç + - 1÷ = +
36 6 è 6 3 ø 18 6
and, b = coefficient of y2 in (5)

- k 2 1 æ h2 k 2 ö -h2 1
= - ç + - 1÷ = + .
9 3è 6 3 ø 18 3
-1 1 1
We have,  a + b = ( h2 + k 2 ) + +

18 6 3 Its auxiliary circle is x2 + y2 = a2(2)

-1 1 Let P ≡ (acosa, bsina)

= (9 cos 2 q + 9 sin 2 q) + Equation of tangent to the ellipse at P(acosa, bsina) is

18 2
-1 1 x cos a y sin a
= (9) + = 0. + = 1 (3)
18 2 a b

Thus, (5) represents two lines which are at right angles to
Making equation (2) homogeneous with the help of (3), we
each other. get
12.56  Chapter 12

62. Let the orbit of the earth be the ellipse


æ x cos a y sin a ö
2

x 2 + y 2 - a 2ç + ÷ = 0 x2 y2
è a b ø 2 + 2 = 1 (1)
a b
æ a2 ö Length of major axis = 2a = 186 × 106 miles (given)
Þ (1- cos 2 a) x 2 + ç 1- 2 sin 2 a÷ y 2 (4) ⇒ a = 93 × 106 miles.
è b ø
1
a Also, eccentricity e = (given).
- 2 cos a sin a xy = 0 60
b
Let the sun be at the focus S(ae, 0). Then, the earth will be

(4) is the joint equation of OL and OM. at shortest and longest distance from the sun when the earth
Since ∠ LOM = 90º,
\  coefficient of x2 + coefficient is at the extremities of the major axis which are respectively
of y2 = 0 nearest and farthest from this focus S.
\ Shortest distance of the earth from the sun
a2 2
Þ 1 - cos 2 a + 1 - sin a = 0 = SA, where S is (ae, 0) and A is (a, 0)
b2 æ 1ö
= a - ae = (93 × 106) ç 1- ÷ = 9145 × 104 miles and lon-
æ a2 ö è 60 ø
Þ sin 2 a ç 2 - 1÷ = 1 gest distance of the earth from the sun
èb ø
= SA′, where S is (ae, 0) and A′ is (-a, 0)
æ 1 ö æ 1ö
or, sin a ç - 1÷ = 1 [ = a + ae = (93 × 106) ç 1+
2
b2 = a2 (1 - e2)] ÷ = 9455 × 104 miles.


è1- e
2
ø è 60 ø
⇒ e2 sin2 a = 1 - e2   or  e2 (1 + sin2 a) = 1 63. Let P ≡ (asecq, btanq)
1
\ e=
1+ sin 2 a

61. Let P ≡ (acosa, bsina)


and Q ≡ (acosb, bsinb)
HINTS AND EXPLANATIONS

Then, N ≡ (asecq, 0).



The equation of the chord PQ is Since Q divides AP in the ratio a2 : b2,
x æa + b ö y æa + b ö æa - b ö \ coordinates of Q are
cos ç ÷ + sin ç ÷ = cos ç ÷ (2)
a è 2 ø b è 2 ø è 2 ø æ ab 2 + a 2 secq a 2b tan q ö
=ç , 2 ÷.

Since it cuts the major axis of the ellipse at a distance d from è a +b
2 2
a + b2 ø
the centre, \  it must pass through the point (d, 0), i.e.,
b tan q
d æa + b ö æa - b ö
Slope of A′ P =
cos ç ÷ = cos ç ÷ a(secq + 1)
a è 2 ø è 2 ø
d æa + b ö æa - b ö a 2b tan q
= cos ç
Slope of QN =
cos ç ÷ ÷ ab 2 + a3 secq - a3 secq - ab 2 secq
a è 2 ø è 2 ø
a 2b tan q .
æa - b ö æa +b ö
=
d -a
cos ç ÷ - cos ç ÷ ab 2 (1- sec q)
Þ = è 2 ø è 2 ø
d+a æa - b ö æa +b ö a 2b 2 tan 2 q
\ Slope of A′ P × slope of QN = = -1.
cos ç ÷ + cos ç ÷ - a 2b 2 tan 2 q
è 2 ø è 2 ø
\ QN is ^ to A′ P.

 [By componendo and dividendo]
64. There are two common tangents to the circle x2 + y2 = 1 and
2 sin a / 2 sin b / 2 the hyperbola x2 - y2 = -1. These are x = 1 and x = - 1. Out
= = tan a/2 × tan b/2.
2 cos a / 2 cos b / 2 of these two, x = 1 is nearer to the point
d- a æ1 ö
\ tan a / 2× tan b / 2 = P ç , 1÷ . Thus, a directrix of the required ellipse is x = 1.
d+ a è2 ø
Conic Sections (Parabola, Ellipse and Hyperbola)  12.57


If Q(x, y) is any point on the ellipse, then its distance from

æ 1ö
2
y - k = 4(x - h)(1)
the focus is QP = ç x - ÷ + ( y - 1) and its distance
2

è 2ø Suppose, this line meets xy = 1 (2)


from the directrix x = 1 is | x - 1 |. By definition of ellipse, in A(x1, y1) and B(x2, y2).
QP = e | x - 1 | Eliminating y from (1) and (2), we get
1
- k = 4(x - h)
æ 1ö
2
1 x
Þ ç x - ÷ + ( y - 1) = | x - 1 |
2

è 2ø 2 ⇒ 4x2 - (4h - k)x - 1 = 0 (3)


Since x1 and x2 are the roots of (3)
1 1 4h - k
⇒ x2 - x + + y2 - 2y + 1 = ( x 2 - 2 x + 1) \ x1 + x2 = (4)
4 4 4
⇒ 3x2 - 2x + 4y2 - 8y + 4 = 0 -1
and, x1 x2 = (5)
4
æ 2 ö
Þ 3 ç x 2 - x ÷ + 4( y - 1) 2 = 0 8h + k ö æ 2 h + k ö
Also, çæ
1
è 3 ø ÷ç - ÷ = - = h or 2x1 + x2 = 3h(6)
è 2 øè 2 ø 4

From (4) and (6), we get
éæ 1ö 1ù
2

Þ 3 êç x - ÷ - ú + 4( y - 1) = 0
2 ( 4 h - k ) 8h + k
3 ø 9 úû x1 = 3h - =
êëè 4 4
(8h + k ) - ( 2h + k )
and, x2 = 3h - =
2
æ 1ö 1

HINTS AND EXPLANATIONS


Þ 3 ç x - ÷ + 4( y - 1) 2 = 2 2
è 3ø 3
Putting these values in (5), we get
2 æ 8h + k ö æ 2 h + k ö 1
æ 1ö ç ÷ç - ÷=-
ç x - 3 ÷ ( y - 1) 2 è 2 øè 2 ø 4
Þ è ø + = 1. ⇒ (8h + k) (2h + k) = 2 or 16h2 + 10hK + k2 = 2.
1/ 9 1 / 12
Thus, equation of required locus is
65. Let P(h, k) be any point on the locus. Equation of the line 16x2 + 10xy + y2 = 2.
through P and having slope 4 is

Previous Year's Questions


x2 y2
66. The equation of an ellipse is 67. Key Idea :The foci of an ellipse 2 + 2 = 1 is given by
(±ae, 0) a b
x2 y2 1
+ =1 Since, e = , ae = 2
16 9 2
Here a = 4, b = 3 ⇒  a = 4
∴  Foci of the above ellipse are ( ± 7 , 0) b2 = a2(1 – e2)
∴  Radius of required circle æ 1ö
∴ 
= 16 ç1 - ÷ = 12
= ( 7 - 0) + (0 - 3)
2 2
è 4ø
x2 y2

Thus, the equation of an ellipse is + = 1.
= ( 7 + 9 = 16 = 4 unit 16 12

12.58  Chapter 12

æ x + x2 y1 + y2 ö Given that y = αx + β is the tangent of hyperbola


68. The mid-point of the chord is ç 1 , .
è 2 2 ÷ø ⇒  m = α and a2m2 − b2 = β2
The equation of the chord in terms of its mid-point is T = S1 ∴  a2α 2 − b2 = β2
Locus is a2x2 − y2 = b2which is hyperbola.
æ y + y2 ö æ x + x2 ö æ x1 + x2 ö æ y1 + y2 ö

i.e., x ç 1 ÷+ yç 1 ÷=2ç 2 ÷ç 2 ÷ 76. ∵  ∠FBF' = 90°
è 2 ø è 2 ø è øè ø
( ) +( )
2 2

⇒  x (y1 + y2) + y (x1 + x2) = (x1 + x2)(y1 + y2) \ a 2e 2 + b 2 a2 e 2 + b 2 = 2( ae) 2



x y
⇒  + =1 ⇒ 2(a2e2 + b2) = 4a2e2
x1 + x2 y1 + y2
69. is correct answer. b2

Þ e2 =
x2 y2 a2
70. - =1
æ 12 ö
2
æ9ö
2 b2
Also, e 2 = 1 -
= 1 - e2
ç 5÷ ç5÷ a2
è ø è ø
5 1
Eccentricity of hyperbola , e1 =
⇒  2e2 = 1  ⇒ e = .
4 2

b2 77. We have a2 = cos2α and b2 = sin2α
Now, ae2 = 1 -
´4 =3
16 So, the coordinates of foci are (±ae, 0)
⇒  b2 = 7 ∴  b2 = a2 (e2 − 1)  ⇒  e = secα
Hence, abscissae of foci remain constant when α varies.
Hence, (C) is the correct answer.
æ9 ö 78. Major axis of length 2a is along x-axis.0
71. Any point on the given parabola is of the form ç t 2 , 9t ÷
è 2 ø a
Differentiating y = 18 x w.r.t. x
2
Now, - ae = 4
dy 9 1 1 e
= = = 2 (given ) Þ t = æ 1ö
dx y t 2 Þ aç 2 - ÷ = 4

è
æ9 9ö
⇒  Point is ç , ÷ 8
è8 2ø Þ a=
72. Points of intersection of given parabolas are (0, 0) and (4a, 3
4a). 79. We have
HINTS AND EXPLANATIONS

And the equation of line passing through these points is y = x 2ae = 6  ⇒  ae = 3


On comparing this line with the given line 2bx + 3cy + 4d = 2b = 8  ⇒  b = 4
0, we get d = 0 and 2b + 3c = 0  ⇒  (2b + 3c)2 + d2 = 0. ∴  b2 = a2(1 − e2)
a ⇒ 16 = a2 − a2e2
73. The equation of directrix is x = = 4. So, a = 2b2 = a2 (1 ⇒  a2 = 16 + 9 = 25
- e2) ⇒ b2 = 3 e
⇒  a = 5
Hence the equation of ellipse is 3x2 + 4y2 = 12. 3 3
\ e= =
74. Area of rectangle ABCD = (2a cosθ) (2b sinθ) = 2ab sin2θ a 5
⇒  Area of greatest rectangle is equal to 2ab
80. Vertex is (1, 0)
When sin2θ = 1

81.

x2 y2
75. Tangent to the hyperbola - = 1 is
a2 b2
y = mx ± a 2 m 2 - b 2
Conic Sections (Parabola, Ellipse and Hyperbola)  12.59

x2 y2 87. Let the foot of perpendicular be P(h, k)


Given ellipse x2 + 4y2 = 4  ⇒ 
+ = 1   ⇒  a = 2, b
4 1 Equation of tangent with slope m passing P(h, k) is
= 1  ⇒  p = (2, 1)
h

Required Ellipse has equation y = mx ± 6 m 2 + 2 where m = -
k
x2 y2 x2 y2
2 + 2 = 1 Þ + =1 6 h2 h2 + k 2
a b 42 b2 Þ +2 =
2
Now, (2, 1) lies on it k k
4 1 1 1 3 4 6h + 2k = (h + k )
2 2 2 2 2
Þ + =1 Þ =1- = Þ b2 =
16 b 2 b2 4 4 3 So required locus is 6x2 + 2y2 = (x2 + y2)2.
88. Equation of tangent at A(t2, 2t)
x2 y2 x2 3y2
\ + =1 Þ + =1 yt = x + t2 is tangent to x2 + 32y = 0 at B
16 æ 4 ö 16 4
ç3÷ Þ x 2 + 32 æç + t ö÷ = 0
x
è ø
èt ø
Þ x 2 + 12 y 2 = 16
32
82. The locus of perpendicular tangents is directrix Þ x + x + 32t = 0
2

t
i.e, x = −a; x = −1
2
83. y 2 = 16 3 x æ 32 ö
Þ ç ÷ - 4(32t ) = 0
è t ø
x2 y2
+ =1 æ 32 ö
2 4 Þ 32 ç 2 - 4t ÷ = 0
è t ø
4 3
y = mx + is tangent to parabola ⇒  t3=8  ⇒  t = 2.
m
1 1
Which is tangent to ellipse ⇒ c2 = a2m2 + b2 ⇒  Slope of tangent is = .
t 2
48
Þ = 2m 2 + 4 4t
m2 89. h = =t
4
⇒  m4 + 2m2 = 24
⇒  m4 = 4 2t 2 t 2
And, k = =
84. Semi minor axis b = 2 4 2
⇒ x2 = 2y
Semi major axis a = 4 2
x y2 90. a = 3, b = 5

HINTS AND EXPLANATIONS


Equation of ellipse = 2 + 2 = 1
a b 5 2
e = 1- =
x 2
y 2
9 3
Þ + =1 Foci = (±2, 0)

16 4
⇒  x2 + 4y2 = 16. 2x 5 y
Tangent at P Þ
+ =1
5 9 3.5
85. Let the tangent to the parabola be y = mx + ( m ¹ 0)
m 2x y
Now, its distance from the center of the circle must be equal ⇒  + =1
9 3
to the radius of the circle. ⇒ 2x + 3y = 9
Therefore, ∴  Area of quadrilateral
5 5 = 4 × (area of triangle QCR)
= 1 + m2 Þ (1 + m 2 )m 2 = 2
m 2 æ1 9 ö
= ç ´ ´ 3 ÷ ´ 4 = 27
Þ m4 + m2 - 2 = 0 è 2 2 ø
⇒ (m − 1) (m + 2) = 0 ⇒ m = ± 1
2 2
91. Given
And so, the common tangents are 2b 2
= 8 (1)
y = x + 5 and y = - x - 5 . a
86. Foci ≡ (±ae, 0) ⇒ 2b = ae(2)
We have, a2e2 = a2 − b2 = 7 We know
Now, the equation of circle is b2 = a2(e2 − 1) (3)

( )
2
( x - 0) 2 + ( y - 3) 2 = 7 - 0 + (0 - 3) 2 b e

Putting = from (2) in (3), we get
a 2
⇒  x2 +y2 − 6y − 7 = 0.
12.60  Chapter 12

e2
= e 2 - 1 x2 y2
+ =1
4 4 3
⇒  4 = 3e2

Equation of normal is
2
Þ e=
3 3
x −1 y − 2
92. Circle and parabola are as shown: = ⇒ 4x – 2y – 1 = 0
1 3
y2 = 8x 4 2×3
a=2

Hence, the correct option is (A)
O x2 y2
95. 2
=1−
2,
P(2t 4t) a b2
(0, − 6)C a2 + b2 = 4

2 3

and − =1
Minimum distance occurs along common normal. a2 b2
Let the equation of normal to parabola 2 3
− =1
y + tx = 2.2.t + 2t3 4 − b2 b2
Since it passes through (0, –6) ⇒ b2 = 3

∴  −6 = 4t + 2t3 ⇒ t3 + 2t + 3 = 0
∴ a2 = 1

⇒  t = −1 (only real value)
∴  coordinates of point P are (2, −4). y2
∴ x2 =
=1
3

\ CP = 4 + 4 = 2 2 y
∴ Tangent at P( 2 , 3 ) is
2x − =1
∴  equation of circle is
3
( x - 2) 2 + ( y + 4) 2 = ( 2 2 ) 2
Clearly it passes through ( 2 2 , 3 3 )
⇒  x2 + y2 – 4x + 8y + 12 = 0
Hence, the correct option is (A)
93. Consider line L at a distance of 6 units below x axis
96. Let the parabola be y2 = 16x; y2 = 4ax a = 4
⇒  PC = PQ
Equation of tangent
⇒  P lies on a parabola, for which C is focus and L is
a
HINTS AND EXPLANATIONS

directrix y = mx +
m
C
4
6 r ⇒ y = mx +

P m
r
It passes through (16, 16)
4
L P (16, 16) ⇒ 16 ± 16 m +

Q m
94. x = –4
⇒ 16 m = 16 m 2 + 4

1
A(–16, 0) B ⇒ m 2 − m +
=0
(24, 0) 4
1
Solving, we get m =

2
Equation of tangent is
1 x
e=
y = +8
2 2
−a x – 2y + 16 = 0 (i)
= –4
e Equation of normal
–a = –4 × e
2x + y = k
a=2
x = 16
Now, b2 = a2(1 – e2) = 3
y = 16

Equation to ellipse k = 48
Conic Sections (Parabola, Ellipse and Hyperbola)  12.61


Normal equation
2x + y = 48
(ii)
T2 : y = 5 x + 3
Axis of Parabola is y = 0

Using T1

x – 2y + 16 = 0 − 5x + y = 3



y=0
x y

x = –16 ⇒
+ =1
3 3

A = (–16, 0) −
5

2x + y = 48 x2 y2

y=0
Equation of tangent to hyperbola 2

= 1 at point
a b2

x = 24 ⎡ xx yy ⎤

B = (24, 0) (x1, y1) on Hyperbola  ⎢ 21 − 21 = 1⎥
⎣a b ⎦
P (16, 16)
(x1, y1) are point of contact
∴ First point of contact is x1 = −3 5 and y1 = –12

θ
Taking equation T2



y = − 5x + 3

5x + y = 3
A (–16, 0) C (4, 0) B (24, 0)
5x y ⎡x x y y ⎤
+ = 1    ⎢ 22 − 22 = 1⎥
AB is diameter as slope of AP × slope of PB = –1
3 3 ⎣a b ⎦

C = (x, y) ∴ We get (x2, y2) on comparison as x2 = 3 5 and y2 = –12

−16 + 24 T (0, 3)

x= =4
2

y=0
16 − 0 16 4
Slope of PC =
= =
16 − 4 12 3

Referring to figure
4
− ( −2) P Q

HINTS AND EXPLANATIONS


4+6 10

tan θ = 3 = = =2 (–3√5, –12) (3√5, –12)
4 3−8 −5
1 + × −2
3 ∴ PQ = 2 × 3 5 = 6 5

Altitude = 3 – (–12) = 15

97. 4x2 – y2 = 36 is a hyperbola
Area of triangle
x2 y2 ⎡ x2 y2 ⎤ 1

− = 1    ⎢ 2 − 2 = 1 form ⎥
PQT = ´ 6 5 ´ 15
9 36 2
⎢⎣ a b ⎥⎦

Equation of tangent = 45 5 unit

y = mx + a 2 m 2 − h2 98. If r Î (0, 1) it’s a hyperbola


1 0, 3
1− r 2
y = mx + 9m 2 − 36 Whose eccentricity is 1+ =
1+ r r +1

It passes through (0, 3)
While if r > 1, then curve is an ellipse having eccentricity
P Q
Hence, 3 = 0 ± 9m 2 − 36

r -1 2
⇒ 9m2 – 36 = 9
1- =
r =1 r +1

m2 = 5

m=± 5 99. Taking minor image of whole
Scenario w.r.t. y = x then solving chord and parabola we have

Equation of Tangents are x2 – 10x + 16 = 0

T1 : y = 5 x + 3 As length of chord is |x1 – x2| 1 + m 2 = 6 1 + 2 = 6 3
This page is intentionally left blank.
CHAPTER
Vector Algebra
13
LEARNING OBJECTRIVES
After reading this chapter, you will be able to:
  Understand the concepts of scalar and vector quantities  Grasp the knowledge on coplanarity of four points, some
  Learn how to represent vectors and what are its types results on linearly dependent and independent vectors,
 Know how to calculate angle between two vectors, product of the two vectors and scalar product of two
addition of two vector, and subtraction of two vectors, product, vector product of two vectors, moment of a
volume of tetrahedron and vector triple product force about a point, scalar triple product
 Be familiar with position vector, component of a vector,
linear combination, collinearity of three points

SCALARS AND VECTORS from a scalar). Sometimes, however, we write the vector a

as a or a .
Scalar Quantity A quantity which has only magnitude and
no direction is called a scalar quantity or simply a scalar.
Examples of scalars are mass, temperature, volume,
I M P O R TA N T P O I N T S
work and so on. To specify a scalar, two things are needed. 
Every vector ( AB) has the following
 three characteristics:
1. a unit in terms of which it is measured 
Length:  The length of AB is denoted by | AB | or
(a) 
2. a real number (+ve, -ve or zero) simply AB.
Vector Quantity  A quantity which has magnitude as well Support:  The line of unlimited length of which
(b)  AB
as direction is called a vector quantity or simply a vector. 
is segment is called thesupport
 of vector AB .
Examples of vectors are displacement, velocity, Sense:  The
(c)  sense of AB is from A to B and that of
acceleration, force and so on. To specify a vector, three 
BA will be from B to A. The sense of directed line
things are needed. segment is from its initial point to the terminal point.
1. a unit in terms of which it is measured
2. a real number (+ve, -ve or zero) Modulus (or Magnitude) of a Vector 
3. a particular direction
The positive real number which is the measure of the length
REPRESENTATION OF VECTORS of the vector, is called the modulus, length, magnitude,
The best way to represent a vector is absolute value or norm of the vector.
with the help of a directed line segment. The modulus
 of a vector a or OA is usually denoted
Suppose A and B are two points, then by by | a | or | OA | or by the corresponding letter ‘a’ (not in
the vector AB. , we mean a quantity whose bold-faced type), i.e.,

magnitude is the length AB and whose | OA | = OA and | a | = a
direction is from A to B. Multiplication of a Vector by a Scalar
Aand B are called the end points of the
vector AB. In particular A is called the ini- FIGURE 13.1 The product of a scalar m and a vector a, is defined as a
tial point and B is called the terminal point. vector ma or am whose magnitude is the product of the

Sometimes a vector AB. is expressed by a single let- magnitudes of m and a and whose direction is that of a or
ter a (which is always written in bold type, to distinguish it opposite to a accordingly as m is positive or negative.
13.2  Chapter 13

TYPES OF VECTORS Co-initial Vectors  The vectors which have the same initial
point are called co-initial vectors.
Equal Vectors  Two vectors a and b are equal when they
Negative of a Vector  A vector having the same modulus as
have (1) the same magnitude and (2) the same direction.
that of a given vector a and the direction opposite to that of
Symbolically such vectors are written as: a = b.
a, is called the negative of a and is denoted by -a. Clearly,
Unit Vectors  A vector whose magnitude is unity is called
if OA = a, then
a unit vector. The unit vector having the same direction as
that of given vector a is usually denoted by the symbol â AO = -a, and therefore, OA = -AO
(read as ‘a cap’), i.e.,
A vector = Modulus of vector × Unit vector in its
direction EQUAL VECTORS
or a = | a | â Two vectors a and b are said to be equal when they have
Also, Unit vector in a direction equal magnitudes and same direction. Geometrically, if
head of one vector coincides with, the head of other and
vector in that direction so do the tails coincide then the vectors are said to be
=
modulus of vector equal.
a
aˆ =
|a| ERROR CHECK

ERROR CHECK If a = b, then a = b, always.


But if, a = b doesn’t always imply a = b
No units are to be attached with a unit vector, i.e., unit
vector is dimensionless physical quantity
FIXED VECTORS
Zero or Null Vector  A vector whose magnitude is zero, is Fixed vector is that vector whose initial point or tail is fixed.
called a zero vector. For such a vector, initial and terminal It is also called localised vector. For example, position vec-
points are coincident so that its direction is indeterminate. tor and displacement vector are fixed vectors.
A zero vector is denoted by the bold-faced symbol O or O.
Collinear (or Parallel Vectors)  The vectors which are par-
FREE VECTORS
allel to the same straight line are called collinear vectors.
Vectors which are not parallel to the same line are Free vector is that whose initial point or tail is not fixed. It is
called non-collinear vectors. also known as non localised vector, For example, velocity
Like and Unlike Vectors    Collinear vectors having the vector of a particular moving particle along a straight line
same direction are called like vectors and those having the is a free vector.
opposite directions are called unlike vectors.
Remark:  If two vectors a and b are collinear, then there
exists a scalar m such that b = ma, m being positive or nega- d
tive according as a and b are like or unlike vectors.
Conversely, if b = ma be given, then a and b must be c
collinear (or parallel) vectors such that | b | = | m | | a |.
Reciprocal Vector  Let | a | be the modulus of the given b
O a
vector a. Then a vector whose direction is that of a but
modulus is 1/| a | (reciprocal of the modulus of a) is called FIGURE 13.2
the reciprocal of a and is written as a-1. Thus,

1 |a| a ANGLE BETWEEN TWO VECTORS


a −1 = aˆ = 2 aˆ = 2
|a| |a| |a|
The angle between two vectors a and b represented by OA
Coplanar and Non-coplanar Vectors  Three or more vec- and OB, is defined as the angle AOB which does not exceed
tors are said to be coplanar when they are parallel to the same π. This is also known as the inclination of given vectors a
plane. Otherwise they are said to be non-coplanar vectors. and b. If the angle AOB be θ, then 0 ≤ θ ≤ π.
Vector Algebra  13.3

OB = OA + AB = a + b (1)
This method of addition of vectors is known as the
triangle law of addition.
Completing the parallelogram OABC. Since
AB = OC = b
OB = OA + AB = OA + OC(2)
FIGURE 13.3
That is, the sum of two co-initial vectors is the vector
p represented by the diagonal of the parallelogram formed
If q = , then vectors are said to be perpendicular
2 with the component vectors as adjacent sides.
or orthogonal and if θ = 0 or π, then vectors are said to be This method of addition of vectors is known as the
parallel or coincident. parallelogram law of addition.
Remark  From Equation (1), -BO = OA + AB or OA + AB
I M P O R TA N T P O I N T S + BO = O, showing that the sum of vectors determined by
the sides of a triangle, taken in order, is zero.
Whenever finding angle between two vectors, make
sure that either their heads coincide or their tails Properties of Vector Addition
coincide.
1. Vector addition is commutative  For any two vec-
tors a and b, we have
a+b=b+a
2. Vector addition is associative  For any three vectors
a, b and c, we have
FIGURE 13.4 (a + b) + c = a + (b + c)
3. Existence of additive identity  For every vector a,
i.e., if heads coincide or tails coincide then internal
we have
angle is the angle between two vectors (whether
acute or obtuse) as in (1), (2), (3) and (4). a+O=a=O+a
  If heads coincide with tall then external angle is
where O is the null vector.
the angle between the two vectors as in (5) and (6). 4. Existence of additive inverse  For a given vector a,
there exists a vector -a such that
a + (-a) = (-a) + a = 0
The vector -a is called the additive inverse of a.

Properties of Multiplication of Vector


FIGURE 13.5
by a Scalar
If m = o, then ma = o
1.
ADDITION (SUM OR RESULTANT) 2.
If m and n be two scalars, then
OF TWO VECTORS m(na) = mna = n(ma)
3.
If m and n be two scalars, then
(m + n)a = ma + na
4.
If a, b are any two vectors and m be any scalar, then
m(a + b) = ma + mb

FIGURE 13.6 Subtraction (or Difference) of Two Vectors


Let a, b be two vectors. Take any point O and draw the Subtraction of vectors    If a and b are two vectors, then
vectors OA = a and AB = b such that the terminal point of their subtration a - b is defined as a - b = a + (-b) where
the vector a is the initial point of vector b. Join OB. Then the -b is the negative of b having magnitude equal to that of b
vector OB is defined as the sum of a and b and is written as and direction opposite to b.
13.4  Chapter 13

Then a - b = (a1- b1) i + (a2 - b2) j + (a3 - b3)k mb + na m n


 Since OP = r = = b+ a
m+n m+n m+n
B
= la + mb

a+b b n m
where l = and m =
m+n m+n
O A
a Thus, p.v. of any point P on AB can always be taken as r = la
−b
+ mb, where λ + µ = 1.
a + (−b) = a − b
mb + na
 Since OP = r =
B m+n
FIGURE 13.7 ⇒ (m + n) r = mb + na
⇒  n ⋅ OA + m ⋅ OB = (n + m) OP,
where P is a point on AB dividing it in the ratio m : n.
Properties of Vector Subraction
In particular, if P is the mid-point of AB, then
(i) a - b ≠ b - a (ii)  (a - b) - c ≠ a - (b - c) OA + OB = 2OP
P
(iii) Since any one side of a trian- r
gle is less than the sum and That is, twice the position vector of the middle point is equal
to the sum of the vectors of the ends.
greater than the difference of
the other two sides, so for any
two vectors a and b, we have
(A) | a + b | ≤ | a | + | b | O(Origin) SOLVED EXAMPLES
(B) | a + b | ≥ | a | - | b | FIGURE 13.8
(C) | a - b | ≤ | a | + | b | 1. In a regular hexagon ABCDEF,
(D) | a - b | ≥ | a | - | b | AB + AC + AD + AE + AF = kAD, where k is equal to
(A) 1 (B)  2
POSITION VECTOR (C) 3 (D)  none of these
If a point O is fixed as the origin in space (or plane) and P Solution (C)
is any point, the OP is called the position vector of P with AB + AC + AD + AE + AF
respect to O.
= ED + AC + AD + AE + CD ( \ AB = ED, AF = CD)
If we say that P is the point r, then we mean that the
position vector of P is r with respect to some origin O.
E D
1. AB in terms of the position vectors of points A and B:
If a and b are position vectors of points A and B respec-
tively. Then OA = a, OB = b F C

∴  AB = (Position vector of B) - (Position vector of A)


= OB - OA = b - a A B
2. Position vector of a dividing point  The position vectors
of the points dividing the line AB in the ratio m : n inter- = (AC + CD) + (AE + ED) + AD
mb + na
nally or externally are = AD + AD + AD = 3AD
m+n
mb − na 2. If M and N are the mid points of the diagonals AC and
or
m−n BD respectively of a quadrilateral ABCD, then
AB + AD + CB + CD =
QUICK TIPS (A) 4NM (B)  4MN
 If P is the mid-point of AB, then it divides AB in the ratio (C) 2MN (D)  none of these
1 : 1. Thus position vector of P is given by
Solution (B)
a+b
OP = In ∆ABD, N is the mid-point of BD,
2
∴ AB + AD = 2AN(1)
Vector Algebra  13.5

In ∆CBD, N is the mid point of BD,


3
∴ CB + CD = 2CN(2) Adding, AL + AM = ( AB + AD )
2
Adding (1) and (2), we have
3 3
AB + AD + CB + CD = 2(AN + CN)(3) = ( AB + BC ) = AC
2 2
In ∆ANC, M is the mid-point of AC
5. ABCD is a quadrilateral and E the point of intersection
∴ AN + CN = 2MN of the lines joining the middle points of opposite sides.
From (3), we get If O is any point, then the resultant of OA, OB, OC and
OD is equal to
AB + AD + CB + CD = 2(2MN) = 4MN (A) 2OE (B)  OE
3. Five forces AB, AC, AD, AE, AF act at the vertex A of (C) 4OE (D)  none of these
a regular hexagon ABCDEF. If O is the centroid of the Solution (C)
hexagon, then their resultant is a force given by
Let P, Q, R, S be the mid-points of sides BC, CD, DA,
(A) 4AO (B)  5AO AB respectively of a quadrilateral ABCD. By geome-
(C) 6AO (D)  none of these try the figure formed by joining the mid-points P, Q,
Solution (C) R, S will be a parallelogram. Hence, its diagonals will
If R is the resultant of given forces, then bisect each other, say at E.

Now, P is the mid-point of BC


R = AB + AC + AD + AE + AF ∴ OB + OC = 2OP(1)
= ED + AC + AD + AE + CD And R is the mid-point of AD
( AB = ED and AF = CD) \
∴ OA + OD = 2OR(2)
= (AC + CD) + (AE + ED) + AD Adding (1) and (2), we have
= AD + AD + AD = 3AD = 6AO. OA + OB + OC + OD = 2(OP + OR) = 2 ⋅ 2OE = 4OE
( E is mid-point of PR, ∴ OP + OR = 2OE)
\
4. ABCD is parallelogram. If L and M are the middle
points of BC and CD, then AL + AM = 6. Two forces act at the vertex A of a quadrilateral ABCD
1 3 represented by AB, AD and two at C represented by CB
(A) AC (B)  AC and CD. If E and F are the middle points of AC and BD
2 2
respectively, then their resultant is represented by
(C) AC (D)  none of these (A) EF (B)  2EF
Solution (B) 3
(C) EF (D)  4EF
1 1 2
AL = AB + BL = AB +BC = AB + AD
2 2 Solution (D)
1 1 We have,
AM = AD + DM = AD + DC = AD + AB
2 2 AB + AD = 2AF, where F is mid-point of BD
13.6  Chapter 13

But | c | = 5 6 (given)
l
\ ± 54 = 5 6 Þ l = ± 15.
9
15 5
Also, CB + CD = 2CF; Hence, c = ± (i - 7 j + 2 k ) = ± (i - 7 j + 2 k )
9 3
∴  AB + AD + CB + CD 9. If the points P, Q, R, S have position vectors p, q, r, s
= 2(AF + CF) = -2(FA + FC) = -2 [2FE], such that p - q = 2(s - r), then
where E is the mid-point of AC = - 4FE = 4EF. (A) PQ and RS bisect each other
7. Let OA = i + 3j - 2k and OB = 3i + j - 2k. The vector (B) PQ and PR bisect each other
OC bisecting the angle AOB where C is a point on the (C) PQ and RS trisect each other
line AB is (D) QS and PR trisect each other
(A) 2(i + j - k) (B) 4(i + j - k) Solution (D)
(C) i + j - k (D)  none of these We have, p - q = 2(s - r)
Solution (A) p + 2r q + 2 s
⇒ p + 2r = q + 2 s Þ =
Taking O as origin, the position vectors of A and B are 1+ 2 1+ 2
a = i + 3j - 2k and b = 3i + j - 2k respectively. We have ∴  Point dividing PR in the ratio 2 : 1 is same as the
| a | = | b | = 14 point dividing QS in the ratio 2 : 1
Hence QS and PR trisect each other.
10. If a and b are position vectors of A and B respectively,
then the position vector of a point C in AB produced
such that AC = 3AB is
(A) 3a - b (B)  3b - a
(C) 3a - 2b (D)  3b - 2a
So, the bisector OC of ∠AOB meets AB at its mid- Solution (D)
point C.
AC = 3AB  ⇒  c - a = 3(b - a)  ⇒  c = 3b - 2a.
1
\ OC = (OA + OB) = 2(i + j - k ) 11. A vector a has components 2p and 1 w.r.t a rectangu-
2
lar cartesian system. This system is rotated through a
8. The vector c, directed along the internal bisector of the certain angle about the origin in the counter-clockwise
angle between the vectors a = 7i - 4j - 4k and b = - 2i sense. If w.r.t the new system, a has components p + 1
- j + 2k with | c | = 5 6 is and 1, then
1
5 5 (A) p = 0 (B)  p = 1 or p = -
(A) (5i + 5 j + 2k ) (B)  (i + 7 j + 2k ) 3
3 3 1
(C) p = -1 or p = (D)  p = 1 or p = -1
5 5 3
(C) ( -5i + 5 j + 2k ) (D)  (i - 7 j + 2k )
3 3 Solution (B)
Solution (D) Let i, j be unit vectors along the co-ordinate axes
The required vector c is given by ∴ a = 2pi + 1 ⋅ j(1)
æ a b ö On rotation, let b be the vector having components
c = l ( a + b) = l ç + ÷ p + 1 and 1.
è |a| |b| ø ∴ b = (p + 1) i + 1⋅j(2)
ì1 1 ü where i, j are unit vectors along the new co-ordinate
= l í (7i - 4 j - 4 k ) + ( -2i - j + 2k ) ý
î 9 3 þ axes.
l ˆ But on rotation | b | = | a |  ⇒ | b |2 = | a |2
or, c= (i − 7 j + 2 k )
9 ⇒ ( p + 1)2 + 1 = (2p)2 + 1  ⇒ 3p2 - 2p - 1 = 0
l l 1
⇒ |c| = ± 1 + 49 + 4 = ± 54 . ⇒ (3 p + 1)( p - 1) = 0 Þ p = 1 or -
9 9 3
Vector Algebra  13.7

12. Let ABCDEF be a regular hexagon. If AD = xBC and 1 1


CF = yAB, then xy = (A) (B) 
4 2
(A) 4 (B)  - 4 1 1
(C) 2 (D)  -2 (C) (D) 
6 8
Solution (B) Solution (A)
Since ABCDEF is a regular hexagon, from plane OM1 = OA + OB + OC + OD (given)
geometry, we have = OM + MA + OM + MB + OM + MC
+ OM + MD
= 4OM + (MA + MC) + (MB + MD)
= 4OM ( MA = - MC, MB = - MD)
\
1 1
\ OM = OM1 \ l =
4 4
15. In a trapezoid the vector BC = λAD and p = µAD and
p = AC + BD is collinear with AD. Then
AD = 2 BC and FC = 2AB (A) µ = λ + 1 (B)  λ = µ + 1
∴ AD = 2BC and FC = 2AB(1) (C) λ + µ = 1 (D)  µ = 2 + λ
Given that AD = xBC. Solution (A)
∴ 2BC = xBC, by (1) We have,
⇒ x = 2 (2) p = AC + BD = AC + BC + CD = AC + λAD + CD
Again, given that CF = yAB or -FC = yAB. = (AC + CD) + λAD = AD + λAD = (1 + λ)AD
∴ -2AB = yAB, using (2) Since p = µAD  ∴  µ = 1 + λ
⇒ y = -2(3) 16. AB = 3i + j - k and AC = i - j + 3k . If the point P on the
From (2) and (3), xy = 2 (-2) = - 4. line segment BC is equidistant from AB and AC, then
15 AP is
13. A vector a is collinear with vector 6i - 8 j - k of
2 (A) 2i - k (B)  i - 2k
magnitude 50 making an obtuse angle with z-axis is (C) 2i + k (D)  none of these
(A) 24i - 32j - 30k (B)  -24i + 32j + 30k
Solution (C)
(C) 24i + 32j - 30k (D)  none of these
A point equidistant from AB and AC is on the bisector
Solution (A) of the angle BAC.
15 A vector along the internal bisector of the angle BAC
Let b = 6i − 8 j − k.
2 AB AC
2 15 = +
A unit vector along b is ± (6i − 8 j − k ). | AB | | AC |
25 2
3i + j - k i - j + 3k 1
∴  a = a vector of length 50 along b = + =
( 4i + 2 k )
9 +1+1 1+1+ 9 11
15 ∴ AP = t (2i + k)
= ±4 (6i − 8 j − k ).
2 ∴ BP = AP - AB = t (2i + k) - (3i + j - k)
Since a makes obtuse angle with z-axis, so we must = (2t - 3) i - j + (t + 1) k
have Also BC = AC - AB = (i - j + 3k) - (3i + j - k)
a ⋅ k < 0 = - 2i - 2j + 4k.
But BP = s BC.
Thus, a = 24i - 32j - 30k ∴ (2t - 3)i - j + (t + 1) k = s(- 2i - 2j + 4k)
14. Given a cube ABCDA1 B1 C1 D1 with lower base ABCD, ∴ 2t - 3 = -2s, -1 = -2s, t + 1 = 4s
upper base A1 B1 C1 D1 and the lateral edges AA1, BB1, 1
∴ s= and t = 1 ∴ AP = 2i + k .
CC1, and DD1; M and M1 are the centres of the faces 2
ABCD and A1B1C1D1 respectively. O is a point on line 17. The position vectors a, b, c and d of four distinct points
MM1, such that A, B, C and D lie on a plane are such that | a - d | =
OA + OB + OC + OD = OM1, then OM = λ. OM1 if λ =
| b - d | = | c - d | then the point D is the
13.8  Chapter 13

(A) centroid of ∆ABC 20 10


(B) orthocentre of ∆ABC = (6 j - 2 k ) - ( -5i + 3 j - k ) = ( 2i + 3 j - k ).
35 7
(C) circumcentre of ∆ABC
(D) none of these 10
\ BM = - ( 2i + 3 j - k ).
Solution (C)
7
We have, a - d = -AD, b - d = -BD and c - d = -CD.
According to the given condition | AD | = | BD | = | CD |. LINEAR COMBINATION
Thus, D is the circumcentre of ∆ABC.
A vector r is said to be a linear combination of the given
vectors a, b, c ..., and so on if there exist a system of scalars
COMPONENT OF A VECTOR x, y, z, ..., and so on such that

The component of a vector PQ on a line l is RS, where R r = xa + yb + zc + ...


and S are the feet of perpendiculars from P and Q on the
line l. LINEARLY DEPENDENT AND INDEPENDENT
The vector component of PQ on l will be denoted SYSTEM OF VECTORS
by RS.
The system of n vectors a1, a2, ..., an is said to be linearly
1. If θ is the angle between PQ and RS, then the component
of PQ on l = PQ cos θ = | PQ | cos θ and the vector com- dependent, if there exist scalars x1, x2, ..., xn not all zero
ponent of PQ on l = PQ cos θ. such that
2. If r is the position vector of a point P, having coordinates x1a1 + x2a2 + ... + xnan = 0
(x, y, z) then r = xi + yj + zk, where i, j, k are unit vectors
along x, y and z axes respectively. The same system of vectors is said to be linearly
3. xi, yj, zk are the vector components of r on x, y and z axes independent, if all scalers are zero, i.e. x1 = x2 = ... = xn = 0.
respectively.
4. If a point P in space has coordinates (x, y, z), then its p.v. REMARK
r is xi + yj + zk and | r | = | r |= x +y +z .
2 2 2
When the system of n vectors a1, a2, ... , an is linearly depen-
dent (or independent), then n vectors a1, a2, ..., an are said
to be linearly dependent (or independent).
SOLVED EXAMPLE
18. The triangle ABC is defined by the vertices A(1, -2, COLLINEARITY OF THREE POINTS
2), B(1, 4, 0) and C(-4, 1, 1). Let M be the foot of the
altitude drawn from the vertex B to side AC. Then BM The necessary and sufficient condition for three points with
= position vectors a, b and c to be collinear is that there exist
three scalars x, y, z, not all zero, such that
(A) (-20/7, -30/7, 10/7) (B)  (-20, -30, 10)
(C) (2, 3, -1) (D)  none of these xa + yb + zc = 0, where x + y + z = 0
Solution (A) Test of Collinearity of Two Vectors  To prove that two vec-
Since MB is the component of AB ⊥ to AC tors a and b are collinear, find a scalar m such that one of
the vectors is m times the other. In case no such scalar m
exists, then the two vectors will be non-collinear vectors.

Test of Collinearity of Three Points


Method 1:  To prove that three points A, B, C are collinear,
find the vectors AB and AC and show that there exists a
MB = AB - AM scalar m such that AB = mAC.
( AB × AC ) AC If no such scalar m exists, then the points are not
= AB - collinear.
( AC ) 2
Method 2:  To prove that three points A, B, C with position
{(6 j - 2k ) × ( -5i + 3 j - k )}( -5i + 3 j - k ) vectors a, b, c respectively are collinear, find three scalars
= (6 j - 2 k ) -
( 25 + 9 + 1) x, y, z (not all zero) such that
Vector Algebra  13.9

xa + yb + zc = 0, where x + y + z = 0 1
Þ 6 - 9l - 1 - l = 0 Þ 5 - 10l = 0, \ l =
If no such scalars x, y, z exist, then the points are not 2
collinear. 1 3 1
\ b1 = la = (3i - i ) = i - j
2 2 2
SOLVED EXAMPLES æ 3ö æ 1ö 1 3
b 2 = ç 2 - ÷ i + ç1 + ÷ j - 3k = i + j - 3k
19. If the vectors a and b are non-collinear, then the value è 2 ø è 2 ø 2 2
of x, for which, the vectors c = (x - 2) a + b and d = (2x ∴ 2i + j - 3k = lα + b2
+ 1) a - b are collinear is Hence, β = b1 + b2
4 2
(A) (B)  COPLANARITY OF FOUR POINTS
3 3
1 The necessary and sufficient condition for four points with
(C) (D)  none of these
3 position vectors a, b, c and d to be coplanar is that there
exist scalars x, y, z and w, not all zero, such that
Solution (C)
xa + yb + zc + wd = 0
The vector c is non-zero since the coefficient in b is
different from zero, and so the vectors c and d are col- where x + y + z + w = 0.
linear if for some number y we have Test of Coplanarity of Three Vectors  To prove that three
d = yc that is vectors a, b and c are coplanar, express one of these vectors
(2x + 1)a - b = y (x - 2)a + yb as the linear combination of the other two such as c = xa + yb.
or ( yx - 2y - 2x - 1)a + (y + 1)b = 0 Now, compare the coefficients from the two sides and
Since a, b are non-collinear, we must have find the values of x and y. If real values of scalars x and y
yx - 2y - 2x - 1 = 0 and y + 1 = 0. exist, then the vectors are coplanar otherwise non-coplanar.
Solving these equations, we get y = -1 and x = 1/3 Test of Coplanarity of Four  Points
20. With reference to a right handed system of mutually Method 1:  To prove that four points A, B, C and D are
perpendicular unit vectors i, j, k α = 3i - j and β = 2i + coplanar, find the vectors AB, AC and AD and show that
j - 3k
these three vectors are coplanar.
If β = b1 + b2, where b1 is parallel to α and b2 is per-
Method 2: To prove that four points A, B, C and D with
pendicular to α, then
position vectors a, b, c and d respectively are coplanar, find
3 1 four scalars x, y, z, w (not all zero) such that
(A) b1 = i+ j
2 2 xa + yb + zc + wd = 0 where x + y + z + w = 0
3 1 If no such scalars x, y, z, w exist, then the points are
(B) b1 = i - j
2 2 non-coplanar.
1 3
(C) b 2 = i + j - 3k
2 2 SOME RESULTS ON LINEARLY DEPENDENT
1 3 AND INDEPENDENT VECTORS
(D) b 2 = i - j - 3k
2 2 1.
If a, b, c are non-coplanar vectors, then these are lin-
Solution (B, C) early independent and conversely if a, b, c are lin-
Since b1 is parallel to α, early independent, then they are non-coplanar.
let b1 = lα, where λ is a scalar. If a = a1i + a2   j + a3k, b = b1i + b2   j + b3k and c = c1i
2.
+ c2   j + c3k are three linearly dependent vectors, then
β = b1 + b2  (Given)
∴ b2 = β - b1 a1 b1 c1
= 2i + j - 3k - lα = 2i + j - 3k - λ(3i - j) a2 b2 c2 = 0
= (2 - 3λ)i + (1 + λ)i - 3k a3 b3 c3
Since b2 is perpendicular to α
∴ b2 ⋅ a = 0 3.
Let a, b, c be three non-coplanar vectors. Then, vec-
⇒ [(2 - 3λ)i + (1 + λ)i - 3i] ⋅ (3i - i) = 0 tors x1a + y1b + z1c, x2a + y2b + z2c and x3a + y3b + z3c
⇒ 3(2 - 3λ) - (1 + λ) = 0 will be coplanar if
13.10  Chapter 13

x1 x2 x3 a ⋅ a = | a |2 = a2
y1 y2 y3 = 0 5.
Scalar product of two perpendicular vectors is zero,
i.e., if a and b are two perpendicular vectors, then
z1 z2 z3
a ⋅ b = 0.
4.
Two non-zero, non-collinear vectors are linearly However, if a ⋅ b = 0 ⇒ Either a = 0 or b = 0 or a ⊥ b.
independent. 6.
Scalar product of mutually orthogonal unit vectors i,
5.
Any two collinear vectors are linearly dependent. j, k:
6.
Any three non-coplanar vectors are linearly i ⋅ i = 1 = j ⋅ j = k ⋅ k
independent. and i ⋅ j = j ⋅ k = k ⋅ i = 0
7.
Any three coplanar vectors are linearly dependent. 7.
Scalar porduct of two vectors in terms of compo-
8.
Any four vectors in 3-dimensional space are linearly nents: If
dependent. a = a1i + a2   j + a3k and b = b1i + b2   j + b3k, then a ⋅ b =
a1b1 + a2b2 + a3b3
Thus, the scalar product of two vectors is equal to
PRODUCT OF TWO VECTORS the sum of the products of their corresponding
Between two vectors, two distinct kinds of products are components.
defined. One being a pure number is called the scalar prod- 8.
Angle between two vectors in terms of the compo-
uct while the other being a vector quantity is called the vec- nents of the given vectors.
tor product. If θ is the angle between two vectors
a = a1i + a2   j + a3k and b = b1i + b2   j + b3k, then
a×b a1b1 + a2 b2 + a3 b3
SCALAR PRODUCT OF TWO VECTORS cos q = =
| a|| b| a + a2 + a2 b2 + b2 + b2
2
1 2 3 1 2 3
The scalar product or dot product of two vectors a and b is
defined as: | a | | b | cos θ, where θ is the angle between them
such that 0 ≤ θ ≤ π. It is denoted by placing a dot between
I M P O R TA N T P O I N T S
the vectors a and b. Thus, If q is acute, a.b is positive and if q is obtuse, a.b is negative
a ⋅ b = | a | | b | cos θ
9.
Components of a vector b along and perpendicular to
If either a or b is O, we define a ⋅ b = O. vector a
 a⋅b 
B Component of b along a =  2  a
| a| 

b  a⋅b 
Component of b perpendicular to a = b −  a
 | a | 2
10. Any vector r can be expressed as:
O M a A
FIGURE 13.9 r = (r ⋅ i)i + (r ⋅ j) j + (r ⋅ k)k

Key Results on Scalar Product SOME USEFUL IDENTITIES


1.
Scalar product is commutative. For any two vectors a Since scalar product satisfies commutative and distributive
and b we have laws, we have
a ⋅ b = b ⋅ a. 1. (a + b)2 = a2 + b2 + 2a ⋅ b
2. (a - b)2 = a2 + b2 - 2a ⋅ b
2.
If m is any scalar and a, b be any two vectors, then
3. (a + b) ⋅ (a - b) = a2 - b2
(ma) ⋅ b = m(a ⋅ b) = a ⋅ (mb)
3.
Scalar product is distributive with respect to vector
addition, i.e., for any three vectors a, b and c, we have WORK DONE BY A FORCE
a ⋅ (b + c) = a ⋅ b + a ⋅ c. Work done by a force F in displacing a particle from A to
4.
Magnitude of a vector as a scalar product: For any B is defined by
vector a  W = F ⋅ AB
Vector Algebra  13.11

15
I M P O R TA N T P O I N T S ⇒ 2a ´ b = 49 - 9 - 25 = 15 Þ a ´ b =
2
 (a + b + c)2 = |a|2 + |b|2 + |c|2 + 2|(a . b + b . c + c . ∴ From (1), we get
a) 15 1
j + kˆ |= 3
 | iˆ + ˆ
cos q = = = cos 60° Þ q = 60°
2 ´ 15 2
 Cauchy - Schwarz Inequality 

(a . b) ≤ |a|2|b|2 23. a and c are unit vectors and | b | = 4. The angle
 If a number of forces are acting on a particle, then   æ1ö   
between a and c is cos -1 ç ÷ . If b - 2c = l a , then
the sum of the works done by the separate forces è4ø
λ is equal to
is equal to the work done by the resultant force
(A) 3, 4 (B)  -3, 4
1 3
(C) 3, -4 (D) ,
SOLVED EXAMPLES 4 4
  Solution (C)
21. If a and b are unit vectors, then the greatest value of   
    Given: | a | = 1, | c | = 1 and | b | = 4.
| a + b | + | a - b | is
  1 1
(A) 2 2 (B) 2 Þ | a × c | = 1 ×1 × =
4 4
(C) 2 (D)  4 2       
Now, b - 2c = l a Þ a × b - 2a × c = l a 2
Solution (A)
    1
Let θ be the angle between a and b . Þ a ×b - 2× = l
  4
Then, a . b = cos q   1
      Þ a ×b = l +
Now, | a + b | = | a |2 + | b |2 + 2a . b = 2 + 2 cos q 2
        
q Again, b - 2c = l a Þ b × b - 2b × c = l b × a
= 4 cos 2
2   æ 1ö
 Þ 16 - 2b × c = l ç l + ÷
 q è 2ø
Þ | a + b | = 2 cos .
2   l 2
l
  q Þ b ×c = 8- -
Similarly, | a - b | = 2 sin 2 4
2         
  Also, b - 2c = l a Þ b × c - 2c × c = l b × a
  æ q qö
\ | a + b | + | a - b | = 2 ç cos + sin ÷ £ 2 2 l2 l
è 2 2ø æ1ö
Þ 8- - - 2(1) = l ç ÷
2 4 è4ø
22. If a + b + c = 0 and | a | = 3, | b | = 5, | c | = 7, then the
⇒  l + λ - 12 = 0
2
angle between a and b is
∴  λ = - 4, 3
p p
(A) (B)  × ×
24. The vectors a = xi - 3j - k and b = 2xi + xj - k include
2 3
p an acute angle and b and positive y-axis include an
p
(C) (D)  × ×
obtuse angle. Then values of x may be
4 6
(A) -2 (B)  -3
Solution (B)
(C) all x < 0 (D)  all x > 0
Let θ be the angle between a and b
∴ a ⋅ b = |a| |b| cosθ Solution (A, B, C)
a×b a×b a×b According to the question
∴ cos q = = = (1) a ⋅ b > 0 and b ⋅ j < 0 ⇒ 2x2 - 3x + 1 > 0 and x < 0
| a || b | (3)(5) 15
⇒ (2x - 1) (x - 1) > 0 and x < 0
Now a + b + c = 0
∴ a + b = -c  1 
i.e.,  x < or x > 1 and x < 0 ⇒ x < 0
⇒ |a + b| = |-c| = |c| ⇒ |a + b|2 = |c|2  2 
⇒ (a + b) ⋅ (a + b) = (7)2 ⇒ |a|2 + 2a ⋅ b + |b|2 = 49 25. If the unit vectors a and b are inclined at angle 2θ (0 ≤
⇒ (3)2 + 2a ⋅ b + (5)2 = 49 θ ≤ π) and |a - b| < 1, then θ lies in the interval
13.12  Chapter 13

é pö ép p ù ⇒ | a + b | = 112 = 4 7
(A) ê0, ÷ (B)  ê6 , 2ú
ë 6ø ë û Similarly, | a - b |= 4 3
é p 5p ù
(C) ê , ú (D)  none of these Hence the lengths of the diagonals are 4 3 and 4 3.
ë2 6 û
Solution (A) 28. Let a = 2i - j + k, b = i + 2j - k and c = i + j - 2k be
three vectors. A vector in the plane of b and c whose
a ⋅ b = | a | | b | cos2θ
2
⇒ a ⋅ b = (1) (1) cos 2θ = cos 2θ. projection on a is of magnitude is
| a - b | < 1 3
⇒ a2 + b2 - 2a ⋅ b < 1 ⇒ 1 + 1 - 2cosθ < 1 (A) 2i + 3j - 3k (B) 2i + 3j + 3k
⇒ 2(1 - cos 2θ) < 1 ⇒ 2(2sin2θ) < 1 (C) - 2i - j + 5k (D) 2i + j + 5k
1 é pö Solution (A, C)
Þ sin 2 q < Þ q lies in ê0, ÷
4 ë 6ø Any vector in the plane of b and c is
r = b + lc = (1 + 2j + k) + λ(i + j - 2k)
26. If a, b, c are three vectors such that | a | = 3, | b | = 4,
= (1 + λ)i + (2 + λ)j + (-1 - 2λ)k
| c | = 5 and a, b, c are perpendicular to b + c, c + a, a
+ b respectively, then | a + b + c | = r×a
Projection of r on a is =
(A) 6 2 (B) 
4 2 |a|
2(1 + l ) - ( 2 + l ) - (1 + 2l ) -l - 1
(C) 3 2 (D) 
5 2 = =
4 +1+1 6
Solution (D)
\ a ⊥ (b + c) -l - 1 2
\ =± Þ - l - 1 = ±2; l = -3 or 1
∴ a ⋅ (b + c) = 0  ⇒  a ⋅ b + a ⋅ c = 0  (1) 6 3
Similarly b ⊥ (c + a)  ⇒  b ⋅ c + b ⋅ a = 0  (2) Hence, r = - 2i - j + 5k or r = 2i + 3j - 3k
and c ⊥ (a + b) = 0  ⇒  c ⋅ a + c ⋅ b = 0  (3)
Adding (1), (2), (3), we get 29. If the moduli of vectors a, b, c are 3, 4, 5 are respec-
tively and a and b + c, b and c + a, c and a + b are
2(a ⋅ b + b ⋅ c + c ⋅ a) = 0 mutually perpendicular then the modulus of a + b + c
Now, (a + b + c)2 = a2 + b2 + c2 + 2(a ⋅ b + b ⋅ c + c ⋅ a) is
= | a |2 + | b |2 + | c |2 + 0 (A) 12 (B)  12
= (3)2 + (4)2 + (c)2 = 50 (C) 5 2 (D)  50
Hence, | a + b + c |= 5 2 Solution (C)
27. A parallelogram is constructed on the vector a = 3p - q According to the given condition, we have
and b = p + 3q, given that |p| = |q| = 2 and the angle a ⋅ (b + c) = 0  (1)
p b ⋅ (c + a) = 0  (2)
between p and q is . The length of a diagonal is
×

3 c ⋅ (a + b) = 0  (3)
(A) 4 5 (B) 
4 3 Now adding (1), (2) and (3), we get

(C) 4 7 (D)  none of these 2 (a ⋅ b + b ⋅ c + c ⋅ a) = 0 (∵ a × b = b × a etc.)


Solution (B) Hence, | a + b + c |2
= a2 + b2 + c2 + 2(a ⋅ b + b ⋅ c + c ⋅ a)
The diagonals of the parallelogram are represented by
the vectors. = 32 + 42 + 52 = 9 + 16 + 25 = 50
a + b = (3p - q) + (p + 3q) = 4p + 2q Þ | a + b + c | = 50 = 5 2
and a - b = (3p - q) - (p + 3q) = 2p - 4q
30. If p = i - 2j + 3k and q = 3i + j + 2k, than a vector r
Now, | a + b |2 = | 4q + 2q |2 = 16 | p |2 + 4 | q |2 + 16p ⋅ q
which is linear combination of p and q and also per-
p pendicular to q is
= 16( 2) 2 + 4( 2) 2 + 16( 2)( 2) cos
3 (A) i + 5j - 4k (B)  i - 5j + 4k
æ p 1ö -1
= 64 + 16 + 12 = 112 ç∵ cos = ÷ (C) (i + 5 j - 4 k ) (D)  none of these
è 3 2ø 2
Vector Algebra  13.13

Solution (C) 33. Let a, b, c be the three vectors such that a ⋅ (b + c) = b ⋅
We have, r = p + lq (c + a)  = 0 and | a | = 1, | b | = 4, | c | = 8, then | a + b
⇒ r ⋅ q = p ⋅ q + lq ⋅ q + c | =
∴ 0 = 7 + 14λ (A) 13 (B)  81
(   p ⋅ q = 3 - 2 + 6 = 7 and q ⋅ q = 9 + 1 + 4 = 14)
\
(C) 9 (D)  5
7 1 1 Solution (C)
Þ l = - = - . \ r = - (i + 5 j - 4 k )
14 2 2 Adding a ⋅ (b + c) = b ⋅ (c + a) = c ⋅ (a + b) = 0,
31. A unit vector in XY plane that makes an angle of 45º we get 2(a ⋅ b + b ⋅ c + c ⋅ a) = 0
with the vector i + j and an angle of 60º with the vector ⇒ |  a + b + c |2 = | a |2 + | b |2 + | c |2 + 2 (a ⋅ b + b ⋅ c +
3i - 4j is c ⋅ a)
i+ j = 1 + 16 + 64 = 81
(A) i (B) 
2 Hence, | a + b + c | = 9
i- j
(C) (D)  none of these 34. If e1, e2 are two unit vectors and θ is the angle between
2 q
them, then cos is
Solution (D) 2
Let the required unit vector in the x-y plane be 1 1
(A) |e1 + e2 | (B)  |e1 - e2 |
r = xi + yi, \ | r | = ( x 2 + y 2 ) = 1 2 2
|e e | |e1 + e2 |
or x2 + y2 = 1  (1) (C) 1 2 (D) 
2 2 | e1 | | e2 |
Now the angle between r and vector 3i - 4j is 60º.
( xi + yj ) × (3i - 4 j ) Solution (A)
cos 60° = Þ x + y = 1 (2)
xi + yj 3i - 4 j (e1 + e2)2 = e1 ⋅ e1 + 2e1 ⋅ e2 + e2 ⋅ e2
5 ⇒ | e1 + e2 | = | e |2 + 2 | e1 | | e2 | cosθ + | e2 |2
and 3 x − 4 y = (3) ⇒ | e1 + e2|2 = 1 + 2 · 1 · 1 cosθ + 1
2
There exists no real values of x and y, satisfying equa- (∵ | e1 | = | e2 | = 1)

tions (1), (2) and (3).
æ qö
Þ | e1 + e2 |2 = 2(1 + cos q ) = 2 ç 2 cos 2 ÷
32. If ABCDEF is a regular hexagon, then AB ⋅ AF is è 2ø
equal to
q q 1
1 1 Þ | e1 + e2 |2 = 4 cos 2 Þ cos = | e1 + e2 |
(A) BC 2 (B)  - BC 2 2 2 2
2 2
1 -1 35. The points O, A, B, C, D are such that OA = a, OB = b,
(C) AC 2
(D)  AC 2
2 2 OC = 2a + 3b and OD = a - 2b . If |a| = 3|b|, then the
Solution (B) angle between BD and AC is
p
Let a be the length of each side of the hexagon (A) π (B)  ×

ABCDEF so that AB = AF = BC = a. Also from plane 2


p
geometry, ∠BAF = 120°. (C) × (D)  none of these
3
Solution (B)
BD ⋅ AC = |BD| |AC| cosθ where θ is the angle between
BD and AC.
⇒ (OD - OB) ⋅ (OC - OA)
= |OD - OB| |OC - OA| cosθ
⇒ (a - 2b - b) ⋅ (2a + 3b - a) = |a - 3b | | a + 3b | cosθ
Hence, we have ⇒ a2 - 9b2 = | a - 3b | | a + 3b| cosθ
AB ⋅ AF = | AB | | AF | cos 120º ⇒ 0 = | a - 3b | | a + 3b | cosθ ( | a | = 3| b |, ∴ a2 =
\
= (AB) (AF) (-1/2) 9b2)
1 p
= −( a 2 /2) = − BC 2 . Þ cos q = 0, \ q =
2 2

13.14  Chapter 13

36. A, B, C, D are four points on a plane with position ⇒ (2x - 1) (x - 1) > 0 and x < 0
vectors a, b, c, d respectively such that (a - d) ⋅ (b - c) ⇒ (x < 1/2, or x > 1) and x < 0
= (b - d) ⋅ (c - a) = 0. For ∆ABC, D is the Hence, x < 0 is the required solution.
(A) Incentre (B)  orthocentre 39. A unit vector in xy plane that makes an angle of 45º
(C) centroid (D)  none of these with the vector i + j and an angle of 60º with the vector
Solution (B) 3i - 4j is
Since (a - d) ⋅ (b - c), = 0  ∴  DA ⋅ CB = 0 (A) i (B)  (i + j ) / 2
∴ AD ⊥ BC (C) (i - j ) / 2 (D)  none of these
Since (b - d) ⋅ (c - a) = 0,  ∴  DB ⋅ AC = 0 Solution (D)
∴ BD ⊥ CA
Let the required unit vector in the x-y plane be
Then D is the intersection of the altitudes through A
and B. Therefore, D is the orthocentre of the triangle r = xi + yj , \ | r | = ( x 2 + y 2 ) = 1
ABC.
   or x2 + y2 = 1 (1)
37. Let a = iˆ + 2 ˆj + kˆ, b = iˆ - ˆj + kˆ, i = iˆ + ˆj - kˆ. A vector Since angle between r and vector i + j is 45º and the
  
coplanar to a and b has a projection along c of mag- angle between r and vector 3i - 4j is 60º.
1 ( xi + yj ) × (i + j )
nitude , then the vector is: \ cos 45° =
3 xi + yj i + j
(A) 2iˆ + ˆj + 2kˆ 4iˆ - ˆj + 4 kˆ
(B)  ( xi + yj ) × (3i - 4 j )
and cos 60° =
(C) 2iˆ - ˆj + 4 kˆ (D)  none of these xi + yj 3i - 4 j
Solution (A, B) ⇒ x + y = 1 (2)
  
Let vector r be coplanar to a and b 3x - 4y = 5/2 (3)
   No real values of x and y exist satisfying equations (1),
\ r = a + tb (2), and (3).
⇒ r = (iˆ + 2 ˆj + kˆ ) + t (iˆ − ˆj + kˆ )
= iˆ(1 + t ) + ˆj ( 2 − t ) + kˆ(1 + t ) VECTOR PRODUCT OF TWO VECTORS
  1 The vector product or cross product of two vectors a and b
Then projection of r on c =
3 is defined as
 
r ⋅c 1 a×b
⇒  =
|c | 3
B
|1⋅ (1 + t ) + 1.( 2 − t ) − 1.(1 + t )| 1
⇒ = n b
3 3
⇒ | 2 - t | = ±1  ⇒  t = 1 or 3
 O a A
When t = 1, we have r = 2iˆ + ˆj + 2kˆ
 FIGURE 13.10
When t = 3, we have r = 4iˆ − ˆj + 4 kˆ
a × b = | a | | b | sin θ n
38. The values of x for which the angle between the vec-
tors a = xi - 3j - k and b = 2xi + xj - k is acute and the 1. | a | | b | sin  θ is the modulus of a × b, θ being the angle
angle between the vector b and the y-axis lies between between the directions of a and b and 0 ≤ θ ≤ π;
p 2. direction of a  ×  b is that of the unit vector n which is
×and π are perpendicular to both a and b such that a, b and n form a
2
right handed system.
1 1
(A) 1, (B)  0< x<
2 2 REMEMBER
(C) all x < 0 (D)  x < 0 or x > 1  By right handed system we mean that as the first vector a
Solution (C) is turned towards the second vector b through an angle θ,
n will point in the direction in which a right handed screw
a ⋅ b > 0 and b ⋅ j < 0.
would advance if turned in a similar manner.
⇒ 2x2 - 3x + 1 > 0 and x < 0
Vector Algebra  13.15

 If either a or b is O, we have a ⋅ b = O. 2.


The area of a triangle with adjacent sides a and b is
 |  a × b | = | a | | b | sin θ 1
given by | a × b | .
 A unit vector perpendicular to the plane of two given vec- 2
a´b 3.
The area of a triangle ABC is
tors a and b is given as n = .
|a ´ b | 1 1 1
 a × b is perpendicular to the plane of a and b.
| AB × AC | or | BC × BA | or | CB × CA |
2 2 2
4.
The area of a parallelogram with diagonals a and b is
Key Results on Vector Product 1
given by | a × b | .
1.
Vector product is not commutative. For any two vec- 2
5.
The area of a quadrilateral ABCD is given by
tors a and b 1
a×b≠b×a | AC × BD | , where AC and BD are its diagonals.
In fact, a × b = - b × a 2
2.
Vector product is associative with respect to a scalar. 6. Vector area of a ∆ABC, when a, b, c are the position vec-
If m and n be any scalars and a, b any vectors, then tors of A, B, C respectively is given by

m(a × b) = (ma) × b = a × (mb) = (a × b)m; 1


∆ ABC = ( a × b + b × c + c × a)
(ma) × (nb) = (na) × (mb) = (mna) × b 2
= a × (mnb) = mn(a × b)
3.
Vector product is distributive with respect to addi- MOMENT OF A FORCE ABOUT A POINT
tion. For any three vectors a, b and c
The vector moment or torque M of a force F acting at a
a × (b + c) = a × b + a × c point A about the point O is given by
If two vectors a and b are parallel, then a × b = 0. In
4. M = r × F = OA × F
particular, a × a = 0.
5.
Vector product of mutually orthogonal unit vectors i, where r = OA is the position vector of the point A with
j, k: respect to the point O.
F
i×i=j×j=k×k=O
and  i × j = k = - j × i,
O

j × k = i = - k × j, k × i = j = - i × k.
r
6.
Vector product in terms of components. Let A
a = a1i + a2   j + a3k FIGURE 13.11

and  b = b1i + b2   j + b3k, then



Info Box!
a × b = (a2b3 - a3b2)i + (a3b1 - a1b3) j
The algebraic sum of the moments of a system of
+ (a1b2 - a2b1) k
forces about any point is equal to the moment of
their resultant about the same point.
i j k
= a1 a2 a3
b1 b2 b3 QUICK TIPS
Lagrange’s Identity
Angle between two vectors: If θ is the angle between
7. If a and b are any two vectors, then
| a´b |
two vectors a and b, then sin q = . |a × b|2 + (a . b)2 = |a|2 |b|2
| a || b |
Geometrical Interpretation
of Cross Product SOLVED EXAMPLES
1. The area of a parallelogram with adjacent sides a and b is    
40. If a , b , c be three non-coplanar vectors and r be any
given by | a × b | . arbitrary vector, then
13.16  Chapter 13

            n1 = AB × AC = - i + 2j - k
( a × b ) × ( r × c ) + (b × c ) × ( r × a ) + (c × a ) × ( r × b )
n2 = AB × AD = i + j - 2k
is equal to Let θ be the acute angle between the planes, then θ is
 
(A) 0 (B)  [a b c ]r the acute angle between their normals n1 and n2
   
(C) 2[a b c ]r (D) 
3[a b c ]r -1+ 2 + 2 3 1 p
\ cos q = = = = cos
Solution (C) 6× 6 2 2 3
   
We have, ( a × b ) × ( r × c ) p
        Þ =
= (( a × b ) ⋅ c ) r − (( a × b ) ⋅ r ) c 3

   
= [a b c ]r − [a b r ]c 43. Given A = ai + bj + ck, B = di + 3j + 4k and C = 3i +
     
  j - 2k. If the vectors A , B and C form a triangle such
Similarly, (b × c ) × ( r × a ) = [b c a ]r − [b c r ]c
         that A = B + C, then
and, (c × a ) × ( r × b ) = [c a b ]r − [c a r ]b (A) a = -8, b = -4, c = 2, d = -11
            (B) a = -8, b = 4, c = -2, d = -11
\  ( a ´ b ) ´ ( r ´ c ) + (b ´ c ) ´ ( r ´ a ) + (c ´ a ) ´ ( r ´ b )
        (C) a = -8, b = 4, c = 2, d = -11
= 3[a b c ]r − ([b c r ]a + [c a r ]b + [a b r ]c ) (D) none of these
   
= 3[a b c ]r − [a b c ]r Solution (C)
  Here A, B, C are the vectors which represent the sides
= 2[a b c ]r
of the triangle ABC where
41. If A = 2i + k, B = i + j + k and C = 4i - 3j + 7k, A = ai + bj + ck
then a vector R which satisfies R × B = C × B and B = di + 3j + 4k
R ⋅ A = O, is C = 3i + j - 2k
(A) - i - 8j + 2k (B)  i - 8j + 2k Given that, A = B + C
(C) i + 8j + 2k (D)  none of these ∴ ai + bj + ck = (d + 3)i + 4j + 2k
⇒ a = d + 3, b = 4, c = 2.
Solution (A) i j k
Let R = xi + yj + zk B ´C = d 3 4
∴ R ⋅ A = 0  ⇒ 2x + z = 0 (1)
3 1 -2
i j k i j k
R´ B = C ´ B Þ x y z = 4 -3 7 = -10i + ( 2d + 12) j + ( d - 9)k
1 1 1 1 1 1 1
∴ Area of the DABC = B ´C
⇒ (y - z)i + (z - x)j + (x - y)k = -10i + 3j + 7k 2
⇒ y - z = -10  (2) 1
z - x = 3 (3) = [100 + ( 2d + 12) 2 + ( d − 9) 2 ]
2
and x-y=7
Solving (1) and (2), we get x = -1, z = 2 = 5 6 (Given )
∴  From (2), y = -8.
⇒ (5d 2 + 30 d + 325) = 10 6
Hence R = - i - 8j + 2k
⇒ 5d2 + 30d + 325 = 600 ⇒ 5d2 + 30d - 275 = 0
42. Let A(0, 0, 0), B(1, 1, 1), C(3, 2, 1) and D(3, 1, 2) be ⇒ d2 + 6d - 55 = 0 ⇒ (d + 11) (d - 5) = 0
four points. The angle between the planes through the ⇒ d = 5 or -11
points A, B, C and through the points A, B, D is When d = 5, a = 8, b = 4, c = 2
p p and when d = -11, a = -8, b = 4, c = 2.
(A) (B) × ×

2 6
p p 44. If x + y = a, x × y = b and x ⋅ a = 1, then
(C) (D) 
a + a´b ( a 2 - 1)a - a ´ b
×
×

4 3 (A) x = (B)  y =
Solution (D) a2 a2
Let n1 and n2 be the vectors normal to the planes ABC b + a´b (b - 1)b - a ´ b
2

(C) x = 2 (D)  y=
and ABD respectively. a a2
Vector Algebra  13.17

Solution (A, B)
= 2 [a 2 b 2 sin 2 q ] = 2 ( a 2 b 2 - a 2 b 2 cos 2 q )
Given x+y=a
⇒ y = a - x(1) = 2 [16 - ( a × b) 2 ] (∵ | a | = | b | = 2)

x × y = b(2)
x ⋅ a = 1 (3) 47. If a ⋅ i = 4, then (a ⋅ j) × (2j - 3k) =
From (1) and (2), we get (A) 12 (B)  2
x × (a - x) = b (C) 0 (D)  -12
⇒ x × a - x × x = b ⇒ x × a = b
Solution (D)
⇒ a × (x × a) = a × b ⇒ (a ⋅ a) x - (a ⋅ x) a = a × b
We have, (a · j) × (2j - 3k)
⇒ | a |2 x - 1 ⋅ a = a × b [From (3)]
= a × ( j ´ ( 2 j - 3k )) = a × ( -3( j ´ k )

( a + a ´ b) = -3( a × i ) (∵ j ´ k = i)
Þ x=
a2
= -3(4) = -12.
( a 2 - 1)a - a ´ b
and y = a - x = 48. If a ⋅ b = β and a × b = c, then b is equal to
a2
(A) (ba - a × c)/a2 (B) (ba + a × c)/a2
45. If a × (b × c) + (a ⋅ b) b = (4 - 2β - sin α) b + (b 2 - 1) c (C) (bc - a × c)/a 2
(D) (bc + a × c)/a2
and (c ⋅ c) a = c, while b and c are non-collinear, then
Solution (A)
p p Here a and c = a × b are non-collinear vectors.
(A) a = , b = -1 (B) a = , b = 1
2 2 ∴ Let b = xa + y(a × c)(1)
p p ∴ β = a ⋅ b = a ⋅ [xa + y(a × c)]
(C) a = , b = -1 (D) a = , b = -1 = x | a |2 + ya ⋅ (a × c) = xa2 ⇒ x = β/a2
3 3
And c = a × b = a × [xa + y (a × c)]
Solution (B)
= xa × a + ya × (a × c)
We have, a × (b × c) + (a · b) b = 0 + y (a ⋅ c) a - y(a ⋅ a)c
= (4 - 2β - sinα) b + (b2 - 1)c(1) = y [a ⋅ (a × b)]a - ya2c = -ya2c
and (c ⋅ c) a = c(2) ⇒ y = -1/a2
where b and c are non-collinear vectors and α, β are ∴ from (1), b = (ba - a × c)/a2
scalars
From (2), (c ⋅ c) a ⋅ c = c ⋅ c 49. Let OA = a, OB = 10a + 2b and OC = b where O, A, C
∴ a ⋅ c = 1 (3) are non-collinear. Let p denote the area of the quadri-
From (1), we get lateral OABC and q denote the area of the parallelo-
(a ⋅ c)b - (a ⋅ b)c + (a ⋅ b)b p
gram with OA and OC as adjacent sides. Then is
= (4 - 2β - sinα) b + (b 2 - 1)c equal to q
or [1 + (a ⋅ b)] b - (a ⋅ b) c (A) 4 (B)  6
= (4 - 2β - sinα) b + (b 2 - 1) c 1 a-b
(C) (D)  none of these
⇒ 1 + (a · b) = 4 - 2β - sin α(4) 2 | a|
and a ⋅ b = -(b2 - 1) (5) Solution (B)
∴ sin α = 1 + (1 - β)2 ⇒ β = 1, sin α = 1 Given, | a × b | = q
p 1 1
i.e., a = + 2np , n Î I . and | b ´ (10 a + 2b)| + | a ´ (10 a + 2b)| = p
2 2 2
46. If u = a - b, v = a + b and | a | = | b | = 2, then | u × v | ∴ 5 | b × a | + | a × b | = p
=
p
(A) 2 [16 - ( a × b) 2 ] (B)  [16 - ( a × b) 2 ] Þ 6 | a ´ b | = p Þ 6q = p Þ = 6.
q
(C) 2 [4 - ( a × b) 2 ] (D)  [4 - ( a × b) 2 ] 50. Two planes are perpendicular to one another. One of
them contains a and b and the other contains vector
Solution (A) c and d, then (a × b) ⋅ (c × d) is equal to
u × v = (a - b) × (a + b) = 2a × b (A) 1 (B)  0
∴ | u × v| = 2| a × b | (C) [a b c] d (D)  [b c d] a
13.18  Chapter 13

Solution (B) SCALAR TRIPLE PRODUCT


a × b is a vector perpendicular to the plane of a and b
Scalar triple product of three vectros  If a, b, c are three
and c × d is a vector perpendicular to the plane of c
vectors, then their scalar triple product is defined as the dot
and d.
product of two vectors a and b × c. It is generally denoted
Since, these planes are ⊥ to one another
by a ⋅ (b × c) or [a b c].
∴ (a × b) ⋅ (c × d) = 0
51. If the vectors c, a = xi + yj + zk and b = j are such that Properties of Scalar Triple Product
a, c and b form a right handed system then c is
(A) zi - xk (B)  0 (i) If a, b, c are cyclicallly permuted, the value of sca-
(C) yj (D)  - zi - xk lar triple product remains the same, i.e., (a × b). c
= (b × c) ⋅ a = (c × a). b or [a b c] = [b c a] =[c a b]
Solution (A) (ii) The change of cyclic order of vectors in scalar triple
Since a, c, b form a right-handed system, so product changes the sign of the scalar triple product
c = λ(b × a), λ > 0 but not the magnitude i.e., [a b c] = -[b a c] = -[c b a]
i.e., c = l i ´ ( xi + yj + zk ) = l ( zi - xk ) = -[a c b]
Taking λ = 1, c = zi - xk (iii) In scalar triple product the positions of dot and cross
can be interchanged provided that the cyclic order of
52. ABCD is a quadrilateral with AB = a, AD = b and AC the vectors remains same i.e., (a × b) . c = a . (b × c)
= 2a + 3b. If its area is α times the area of the parallel- (iv) The scalar triple product of three vectors is zero if
ogram with AB, AD as adjacent sides, then α is equal any two of them are equal.
to (v) For any three vectors a, b, c and scalar λ, [l a b c] = λ
5
(A) 5 (B)  [a b c]
2
(vi) The scalar triple product of three vectors is zero if
1
(C) 1 (D)  any two of them are parallel or collinear.
2 (vii) If a, b, c, d are four vectors, then [(a + b) c d ] = [a c
Solution (B) d] + [b c d]
Area of quadrilateral ABCD (viii) The necessary and sufficient condition for three non-
= Area of ∆ABC + Area of ∆ACD zero non-collinear vectors a, b, c to be coplanar is
1 1 that [a b c] = 0.
= [a ´ ( 2a + 3b)] + [( 2a + 3b) ´ b]
2 2 (ix) Four points with position vectors a, b, c and d will be
1 5 coplanar, if [a b c] + [d c a] + [d a b] = [a b c].
= [3a ´ b + 2a ´ b] = a ´ b (x) Volume of parallelopiped whose coterminous edges
2 2
are a, b, c is [a b c] or a ⋅ (b × c).
5 5
= Area of ||gm ABCD. \ a =
2 2
Scalar Triple Product in Terms
53. Consider a tetrahedron with faces F1, F2, F3, F4. Let of Components
V1, V2, V3, V4 be the vectors whose magnitudes are
respectively equal to areas of F1, F2, F3, F4 and whose (i) If a = a1 i + a2 j + a3k, b = b1i + b2   j + b3k and c = c1i +
directions are perpendicular to their faces in outward c2   j + c2   j + c3k be three vectros
direction. Then | V1 + V2 + V3 + V4 | equals
(A) 1 (B)  4 a1 b1 c1
(C) 0 (D)  none of these then, [a b c] = a2
b2 c2
Solution (C) a3 b3 c3
We have,
(ii) If a = a1l + a2m + a3n, b = b1l + b2m + b3n and c = c1l
1 1 + c2m + c3n, then
v1 = ( a ´ b), v2 = (b ´ c)
2 2
1 1 a1 a2 a3
v3 = (c ´ a) and v4 = [(c - a) ´ (b - a)]
3 2 [a b c] = b1 b2 b3 [l m n]
∴ v1 + v2 + v3 + v4 = 0 c1 c2 c3
∴ | v1 + v2 + v3 + v4 | = 0
Vector Algebra  13.19

(iii) For any three vectors a, b and c


(A) [a + b b + c c + a] = 2[ a b c] SOLVED EXAMPLES
(B) [a - b b - c c - a] = 0 54. If i, j, k are the unit vectors and mutually perpendicu-
(C) [a × b b × c c × a] = [ a b c]2 lar, then [i - j j - k k - i] =
Tetrahedron  A tetrahendron is a three-dimensional figure (A) 0 (B)  1
formed by four triangles. OABC is a tetrahedron with (C) -1 (D)  none of these
DABC as the base. OA, OB, OC, AB, BC and CA are known Solution (A)
as edges of the tetrahedron. OA, BC; OB, CA and OC, AB (i - j) · [(j - k) × (k - i)]
are known as the pairs of oppposite edges. A tetrahedron in = (i - j) ⋅ (j × k - j × i - k × k + k × i)
which all edges are equal, is called a regular tetrahedron. = (i - j) ⋅ (i + k - 0 + j)
Any two edges of regular tetrahedron are perpendicular to =i·i+i⋅k+i⋅j-j⋅i-j⋅k-j⋅j
each other. = i2 + 0 + 0 - 0 - 0 - j2 = i2 - j2 = 1 - 1 = 0.
A (a)
55. If e1 ′, e2 ′, e3 ′ are vectors reciprocal to the non-copla-
nar vectors e1 , e2 , e3  then [e1 ′, e2 ′, e3 ′ ] [e1  e2 e3 ] =
a −1
(A) (B)  1
2
(C) 0 (D)  4
b c
Solution (B)
B (b) C (c)
1
FIGURE 13.12
Since [e1¢e2¢ e3¢ ] =
[e1e2 e3 ]
∴ [e′1 e′2 e′3] [e1 e2 e3] = 1
Volume of Tetrahedron 1
56. If r = λ (a × b) + µ (b × c) + v (c × a) and [a b c] = , then
(i) The volume of a tetrahendron λ + µ + v is equal to 8
1 (A) r ⋅ (a + b + c) (B) 8r ⋅ (a + b + c)
= (area of the base)(corresponding altitude)
3 (C) 4r ⋅ (a + b + c) (D)  none of these
1 Solution (B)
= [ AB BC AD ]
6 1
Clearly r × c = l[a b c] = l
(ii) If a, b, c are position vectors of vertices A, B and 8
C with respect to O, then volume of tetrahedron 1
r × a = m [a b c] = m
1 8
OABC = [a b c]
6 1
(iii) If a, b, c, d are position vectors of vertices A, B, C, D r ´ b = v [a b c] = v
8
of a tetrahedron ABCD, then its volume 1
\ r × ( a + b + c) = (l + m + n )
1 8
= [b − a c − a d − a ] ∴ λ + µ + v = 8r ⋅ (a + b + c)
6
Reciprocal system of vectors  Let a, b, c be three non-co- Vector triple product
b×c c×a a×b
planar vectors, and let a′ = , b′ = , c′ = . Let a, b, c be any three vectors, then the vectors a × (b × c)
[a b c] [a b c] [a b c]
a′, b′, c′ are said to form a reciprocal system of vectors for and (a × b) × c are called vector triple product of a, b, c.
the vectors a, b, c. Thus, a × (b × c) = (a.c) b - (a.b) c
If a, b, c and a′, b′, c′ form a reciprocal system of
vectors then Properties of Vector Triple Product
(i)  a, a′ = b b′ = c.c′ = 1

(i) The vector triple product a × (b × c) is a lin-
(ii)  a. b′ = a. c′ = 0; b.c′ = b.a′ = 0; c. a′ = c.b′ = 0
ear combination of those two vectors which are
1 within brackets.
(iii)  [a′ b′ c′ ] =

[a b c] (ii) The vector r = a × (b × c) is perpendicular to a


(iv)  a, b, c are non-coplanar iff so are a′, b′, c′. and lies in the plane of b and c.
13.20  Chapter 13

(iii) The formula a × (b × c) = (a .c) b - (a.b) c is


p p
true only when the vector outside the bracket is (A) (B)  ×

3 4
on the left most side. If it is not, we first shift on
left by using the properties of cross product and p
(C) × (D)  none of these
then apply the same formula. Thus, (b × c) × a 2
= - {a × (b × c)} = - {(a ⋅ c) b - (a ⋅ b) c} = (a . Solution (C)
b) c - (a ⋅ c) b 1
We have, a × (b × c) = b

(iv)  Vector triple product is a vector quantity. 2

(v)  a × (b × c) ≠ (a × b) × c ⇒ (a ⋅ c) b - (a ⋅ b) c = 1/2b
⇒ (a ⋅ c - 1/2) b = (a ⋅ b) c
Rotation of a Vector About an Axis But since b and c are non-parallel, so the only possibil-
Let a = (a1, a2, a3). If system is rotated about ity is a ⋅ c = 1/2 and a ⋅ b = 0 Hence the angle between
(i) x-axis through an angle α, then the new compo- a and b is π/2.
nents of a are (a1, a2 cos α + a3 sin α, - a2 sinα +
58. If a and b are two unit vectors, then the vector (a + b )
a3 cosα).
× (a × b) is parallel to the vector
(ii) y-axis through an angle α, then the new compo-
nents of a are (-a3 sinα + a1 cosα, a2, a3 cosα + (A) a - b (B)  a+b
a1 sinα). (C) 2a - b (D)  2a + b
(iii) z-axis through an angle α, then the new compo- Solution (A)
nents of a are (a1 cosα + a2 sin α , -a1 sin α + a2 We have, (a + b) × (a × b)
cosα, a3). = a × (a × b) + b × (a × b)
= (a ⋅ b) a - (a ⋅ a) b + (b ⋅ b) a - (b ⋅ a) b
SOLVED EXAMPLES = (a ⋅ b) (a - b) + a - b (b ⋅ b = b2 = 1, a ⋅ a = a2 = 1
as a, b are unit vectors)
57. Given three unit vectors a, b, c no two of which are col- = (a ⋅ b + 1) (a - b)
1
linear satisfying a ´ (b ´ c) = b. The angle between a = x (a - b) where x = a ⋅ b + 1 is a scalar.
and b is 2 ∴ The given vector is parallel to a - b.
Vector Algebra  13.21

NCERT EXEMPLARS

1. The vector in the direction of the vector i - 2 j + 2k 8. For any vector a, the value of
that has magnitude 9 is   
  
( ) ( ) ( )
2 2 2

(A) i - 2 j + 2k (B)  i - 2 j + 2k a ´ i + a ´ j + a ´ k is


3

( 
(C) 3 i - 2 j + 2k 
)

(
(D)  9 i - 2 j + 2k

) 2
(A) a (B)  3a
2

2 2
2. The position (C) 4 a (D)  2a
 vector
 of the point which divides the join
of points 2a - 3b and a + b in the ratio 3 : 1, is     
    9. If a = 10, b = 2 and a·b = 12, then the value of a ´ b
(A) 3a - 2b (B)  7a - 8b is
 2 4 (A) 5 (B) 10
(C) 3a (D)  5a (C) 14 (D) 16
4 4

3. The vector having initial and terminal points as (2, 5, 10. The vectors l i + j + 2k , i + l j - k and c = 2i - j + l k
0) and (– 3, 7, 4), respectively is       
(A) -i + 12 j + 4 k (B)  5i + 2 j - 4 k 11. If a, b and c and unit vectors such that a + b + c = 0,
  
(C) -5i + 2 j + 4 k (D)  i + j + k then the value of a·b + b·c + c·a is
  (A) 1 (B) 3
4. The angle between two vectors a and b with magni- 3
 (C) - (D)  None of these
tudes 3 and 4, respectively and a.b = 2 3 is 2
 
(A) p (B)  p 12. The projection vector of a on b is
6 3 æ   ö 
p (A) ç  ÷ b a ·b (B)  ·b
a
(C)  (D)  5p
2 2 ç b ÷ b
è ø

5. Find the value of l such that the vectors a = 2i + l j + k  æö
 (C) a·b (D)  ç a·b2 ÷ b
and b = i + 2 j + 3k are orthogonal. a ç ÷
ça ÷
(A) 0 (B) 1 è ø
      
(C) 3 (D)  -5 13. If a, b and c are three vectors such that a + b + c = 0
2 2   

NCERT EXEMPLARS
and a = 2, b = 3 and c = 5, then the value of
6. The value of λ for which the vectors 3i - 6 j + k and   
a·b + b·c + c·a is
2i - 4 j + l k are parallel, is

(A) 2 (B)  3 (A) 0 (B) 1


3 2 (C) – 19 (D) 38
5  
(C)  (D)  2 14. If a = 4 and -3 £ l £ 2, then the range of l a is
2 5
(A)  [0, 8] (B)  [–12, 8]
7. The vectors from origin to the points A and B are (C)  [0, 12] (D)  [8, 12]
 
a = 2i - 3 j + 2k and b = 2i + 3 j + k respectively, then
15. The number of vectors of unit length perpendicular to the
the area of ΔOAB is equal to  
vectors a = 2i + j + 2k and b = j + k is
(A)  340 (B)  25
1 (A) one (B) two
(C)  229 (D)  229
2 (C) three (D) infinite
13.22  Chapter 13

ANSWER K EYS
1. (C) 2.  (D) 3. (C) 4.  (B) 5. (D) 6.  (A) 7.  (D) 8.  (D) 9.  (D) 10.  (A)
11.  (C) 12.  (A) 13.  (C) 14.  (C) 15.  (B)

HINTS AND EXPLANATIONS



( 2i + l j + k )·(i + 2 j + 3k ) = 0
   
1. Let a = i - 2 j + 2k  a

\

Any vector in the direction of a vector a is given by a . ⇒ 2 + 2l + 3 = 0
i + 2 j + 2k i - 2 j + 2k -5
\ l =

= =
12 + 22 + 22 3 6. Since, two2 vectors are parallel i.e., angle between them is
 zero.
\ Vector in the direction of a with magnitude

i - 2 j + 2k
( )( )
∴ 3i - 6 j + k · 2i - 4 j + l k = 3i - 6 j + k · 2i - 4 j + l k
 9 = 9.    
3  é∵ a·b = a b cos 0° Þ a·b = a b ù
ë û
 (
= 3 i - 2 j + 2k
 
)
⇒ 6 + 24 + l = 9 + 36 + 1 4 + 16 + l
2

2. Let the position
 vector
  of the point R divides the join of
⇒ 30 + l = 46 20 + l
2
points 2a - 3b and a + b.
   
\ Position vector R =
( ) (
3 a + b + 1 2a - 3b ) ⇒

900 + l 2 + 60l = 46 20 + l 2 ( )
[on squaring both sides]

3 +1 ⇒ l + 60l - 46l = 920 - 900
2 2

Since, the position vector of a point R dividing the line seg- ⇒ -45l + 60l - 20 = 0
2

⇒ -45l + 30l + 30l - 20 = 0
2
ment joining the points P and Q, whose position vectors are
   
p and q in the ratio m : n internally, is given by mq + n p . ⇒ -15l ( 3l - 2 ) + 10 ( 3l - 2 ) = 0

m+n
 ⇒
(10 - 15l ) ( 3l - 2 ) = 0
HINTS AND EXPLANATIONS

\ 5a
R= 2 2
4 \ l = ,
3. Required vector = - ( -3 - 2 ) i + ( 7 - 5 ) i + ( 4 - 0 ) k 3 3

Alternate Method
 

= -5i + 2 j + 4 k
Let a = 3i - 6 j + k and b = 2i - 4 j + l k

Similarly, we can say that for having initial and terminal  

Since, a b
points as
(i) (4, 1, 1) and (3, 13, 5), respectively. 3 -6 1 2
⇒ = = Þl =
(ii) (1, 1, 9) and (6, 3, 5), respectively. 2 -4 l 3
(iii) (1, 2, 3) and (2, 3, 4), respectively, we shall get (a),  
7. ∴ Area of DAOB = 1 O A ´ OB
(b) and (d) as its correct options. 2

4. Here,
  
a = 3 , b = 4 and a.b = 2 3[given]
=
1  
2
(
2i - 3 j + 2k ´ 2i + 3 j + k)( )
   i j k
We know that, a·a = a b cosq
1
= 2 -3 2
Þ
2 3 = 3.4.cosq 2
2 3 1
2 3 1
cosq = = 1
Þ 4 3 2
= éi ( -3 - 6 ) - j ( 2 - 4 ) + k ( 6 + 6 ) ù
2 ë û
p
\ q= 1
3


= -9i + 2 j + 12k
 2
5. (D) Since, two non-zero vectors a and b are orthogonal

i.e., a·b = 0. ∴ Area of DOAB = 1
(81 + 4 + 144 ) =
1
229
2 2
Vector Algebra  13.23


8. Let a = xi + y j + zk
2
(
Þ ( l + 2 ) l 2 - 2l - 2 = 0
)
∴ a = x2 + y2 + z2 2 ± 12
Þ l = -2 or l =
i j k 2
 2 ± 2 3

a ´ i = x y z Þ l = -2 or l = = 1± 3
2
1 0 0    2 2 2
11. We have, a + b + c = 0 and a = 1.b = 1, c = 1

= i [0 ] - j [ - z ] + k [ - y ]      
∵ ( )(
a+b+c a+b+c =0 )

= z j - yk  2        2       2

Þ a + a·b + a·c + b·a + b + b·c + c·a + c·b + c = 0
( ) ( )( )
2

a ´ i = z j - yk z j - yk  2  2 2   

= y +z2 2

Þ (
a + b + c + 2 a·b + b·c + c·a = 0 )
  2      

Similarly, ( )
a ´ j = x2 + z2

Þ é∵ a·b = b·c, b·c = c·b and c·a = a·c ù
ë û
  2   

and ( )
a ´ k = x2 + z2
Þ ( )
1 + 1 + 1 + 2 a·b + b·c + c·a = 0
  
( a ´ i ) + ( a ´ j ) + ( a ´ k )   
2 2 2
∴ = y2 + z2 + x2 + z2 + x2 + y2 3

a·b + b·c + c·a = -
2 2

 = 2 x 2 + y 2 + z 2 = 2a ( )  
   b  æ b ö
   12. Projection vector of a on b is given by = a·  b = ç a·  ÷·b
9. Here, a = 10, b = 2 and a•b = 12  [given] b ç b ÷
è ø
   

a·b = a b cosq 14. We have, a = 4 and - 3 £ l £ 2

12 = 10 × 2 cos θ  
\ la = l a = l 4
12 3
⇒ cosq = =
20 5 

Þ λ a = −3 4 = 12, at λ = 3
9 

sin q = 1 - cos 2 q = 1 -
l a = 0 4 = 0, at l = 0
25

HINTS AND EXPLANATIONS


4 

sin q = ±
and l a = 2 4 = 8, at l = 2
5
    

a ´ b = a b sin q So, the range of l a is = [0, 12].

4
Alternate Method

= 10 ´ 2 ´
5
Since, –3 £ l £ 2
= 16
0£ l £3
   
10. a = l i + j + 2k , b = i + l j - k and c = 2i - j + l k
   0 £ 4 l £ 12


For a, b and c to be coplanar, 

l a Î [0, 12]
l 1 2
15. The number of vectors of unit length perpendicicular to the

1 l -1 = 0      
2 -1 l (
vectors a and b is c (say) i.e., c = ± a ´ b . )
( 2
)
Þ l l - 1 - 1( l + 2 ) + 2 ( -1 - 2l ) = 0
So, therewill be two vectors of unit length perpendicular to the
vectors a and b.
Þ l 3 - l - l - 2 - 2 - 4l = 0

Þ l 3 - 6l - 4 = 0
13.24  Chapter 13

PRACTICE EXERCISES

Single Option Correct Type

1. a and b are mutually perpendicular unit vectors. If r is 7. If AB = 3i + j - k and AC = i - j + 3k . If the point P


a vector satisfying r ⋅ a = 0, r ⋅ b = 1 and [r a b] = 1, on the line segment BC is equidistant from AB and AC,
then r is then AP is
(A) a × b + b (B)  a + (a × b) (A) 2i - k (B)  i - 2k
(C) b + (a × b) (D)  a×b+a (C) 2i + k (D)  none of these
8. A, B, C, D are four points on a plane with position
2. a, b, c are three vectors of magnitude, 3 , 1, 2 such
vectors a, b, c, d, respectively, such that (a - d)⋅
that a × (a × c) + 3b = O. If θ is the angle between a
(b - c) = (b - d) ⋅ (c - a) = 0. For ∆ ABC, D is the
and c, then cos2θ is equal to
1 3 (A) incentre (B)  orthocentre
(A) (B)  (C) centroid (D)  none of these
4 4
1 9. If a and b are two unit vectors, then the vector (a + b )
(C) (D)  none of these × (a × b) is parallel to the vector
2
(A) a - b (B)  a+b
3. A vector a has components 2p and 1 w.r.t. a rectangu-
(C) 2a - b (D) 
2a + b
lar cartesian system. This system is rotated through a
certain angle about the origin in the counter-clockwise 1
sense. If w.r.t. the new system, a has components p + 1 10. If r = λ (a × b) + µ (b × c) + ν (c × a) and [a b c] = ,
8
and 1, then then λ + µ + ν is equal to
1 (A) r ⋅ (a + b + c) (B) 8r ⋅ (a + b + c)
(A) p = 0 (B)  p = 1 or p = -
3 (C) 4r ⋅ (a + b + c) (D)  none of these
1
(C) p = -1 or p = (D) p = 1 or p = -1 11. In a parallelogram ABCD, | AB | = a, | AD | = b and | AC
3
| = c. Then, DB ⋅ AB has the value
4. If a ⋅ b = β and a × b = c, then b is equal to 3a 2 + b 2 - c 2 a 2 + 3b 2 - c 2
(A) (B) 
(A) (ba - a × c)/a2 (B) (ba + a × c)/a2 2 2
PRACTICE EXERCISES

(C) (bc - a × c)/a


2
(D) (bc + a × c)/a2 a 2 - b 2 + 3c 2 a 2 + 3b 2 + c 2
(C) (D) 
2 2
5. Let ABCDEF be a regular hexagon. If AD = x BC and
CF = y AB, then xy = 12. Let a = a1i + a2   j + a3k, b = b1i + b2   j + b3k, and
(A) 4 (B)  -4 c = c1i + c2   j + c3k be three non-zero vectors such that c
(C) 2 (D)  -2 is a unit vector perpendicular to both vectors a and b.
If the angle between vectors a and b is π/6, then
6. Given a cube ABCDA1 B1 C1 D1 with lower base ABCD, 2
upper base A1B1C1D1 and the lateral edges AA1, BB1, a1 a2 a3
CC1 and DD1; M and M1 are the centres of the faces b1 b2 b3 is equal to
ABCD and A1B1C1D1 respectively. O is a point on line c1 c2 c3
MM1, such that
OA + OB + OC + OD = OM1, then OM = λ OM1 (A) 0
if λ = (B) 1
1
1 1 (C) ( a12 + a22 + a32 ) (b12 + b22 + b32 )
(A) (B)  4
4 2
3
1 1 (D) ( a12 + a22 + a32 ) (b12 + b22 + b32 ) (c12 + c22 + c32 )
(C) (D)  4
6 8 (E) none of these
Vector Algebra  13.25

13. If a, b, c are non-coplanar unit vectors such that 20. Forces P, Q act at O and have a resultant R. If any
b+c transversal cuts their lines of action at A, B, C, respec-
a × (b × c) = , then the angle between a and b is
2 tively, then
P Q R
3p p (A) + + =0
(A) (B)  × ×
OA OB OC
4 4
P Q R
p (B) + + =1
(C) (D)  × π OA OB OC
2
P Q R
14. If the vectors a and b are perpendicular to each other, (C) + - =0
then a vector v in terms of a and b satisfying the equa- OA OB OC
tions v ⋅ a = 0, v ⋅ b = 1 and [v a b] = 1 is P Q R
(D) + - = 1.
1 1 OA OB OC
(A) b 2 + a × b
b 2
a×b
21. A vector A has components A1, A2, A3 in a right-handed
b a×b rectangular cartesian coordinate system Ox, Oy, Oz.
(B) | b | + | a × b |2 The coordinate system is rotated about the z-axis
p
b a×b through an angle . The components of A in the new
(C) | b |2 + | a × b |
×

2
coordinate system are
(D) none of these (A) A1, -A2, A3 (B)  A2, A1, A3
(C) A1, A2, -A3 (D)  A2, -A1, A3.
15. Let the unit vectors a and b be perpendicular to each
other and the unit vector c be inclined at an angle θ to 22. In a ∆ OAB, E is the mid-point of OB and D is a point
both a and b. If c = xa + yb + z(a × b), then on AB such that AD : DB = 2 : 1. If OD and AE inter-
(A) x = cosθ, y = sinθ, z = cos2θ sect at P, then the ratio OP : PD is
(B) x = sinθ, y = cosθ, z = -cos2θ (A) 1 : 2 (B)  2 : 1
(C) x = y = cosθ, z2 = cos2θ (C) 3 : 2 (D)  2 : 3.
(D) x = y = cosθ, z2 = -cos2θ
23. If a, b, c are three non-parallel unit vectors such that
16. If S is the circumcentre, O is the orthocentre of ∆ABC, 1
then SA + SB + SC = a × (b × c) = b, then the angles which a makes with
2
(A) SO (B) 2SO b and c are
(C) OS (D) 2OS (A) 90º, 60º (B)  45º, 60º

PRACTICE EXERCISES
17. If a, c, d are non-coplanar vectors and d ⋅ {a × [b × (C) 30º, 60º (D)  none of these
(c × d) ]} is equal to 24. If a = i + j - k, b = i - j + k and c is a unit vector per-
(A) (b ⋅ d) [a c d] (B) (a ⋅ d) [a c d] pendicular to the vector a and coplanar with a and b,
(C) (c ⋅ d) [a c d] (D)  none of these then a unit vector d perpendicular to both a and c is
18. If 4a+ 5b + 9c = 0, then 1 1
(A) ( 2i − j + k ) (B)  (i + j )
(a × b) × [(b × c) × (c × a)] is equal to 6 2
(A) A vector perpendicular to the plane of a, b and c 1 1
(B) A scalar quantity (C) ( j + k ) (D) (i + k )
2 2
(C) 0
(D) none of these 25. If the pth, qth and rth terms of a G. P. are positive num-
n
bers a, b and c, respectively, then the angle between
19. If ∑ a = 0 where
i =1
i | ai | = 1 V i, then the value of is
the vectors ilna + jlnb + klnc and i(q - r) + j(r - p) + k
(p - q) is
∑ ∑a ⋅a
1≤ i < j ≤ n
i j
p p
(A) (B) 
n
× ×

3 6
-n
(A) - (B) 
2 p
n (C) × (D)  none of these
(C) (D) n 2
2
13.26  Chapter 13

 1 1 1
25. A vector a is collinear with vector b =  6, − 8, − 7  (A) £ £
(B) 
 2 3
× ×

2
and make an acute angle with the positive direction of 1 1
z-axis. If | a | = 50, then a = (C) > (D) ³
3 2
(A) (24, 32, 30) (B)  (24, -32, 30)
(C) (-24, 32, 30) (D)  none of these 34. If b and c are any two non-collinear unit vectors and a is
a ⋅ (b + c)
27. The perpendicular distance of a corner of a unit cube any vector, then ( a ⋅ b)b + ( a ⋅ c)c + (b × c) =
form a diagonal not passing through it is | b + c |2
(A) a (B)  b
6 (C) c (D)  none of these
(A) 6 (B) 
3
3
(C) (D)  none of these 35. If the vector - i + j - k bisects the angle between
6
3i + 4j and vector c, then the unit vector along c is
28. The vectors a, b and c are equal in length and taken −11i − 10 j − 2k −11i + 10 j + 2k
pairwise, they make equal angles. If a = i + j, b = j + (A) (B) 
15 15
k, and c makes an obtuse angle with the base vector i,
then c is equal to −11i + 10 j − 2k
(C) (D)  none of these
(A) i + k (B)  - i + 4j - k 15
−1 4 1 1 −4 1 36. If a, b and c are three unit vectors such that a + b + c is
(C) i + j − k (D)  i + j + k.
3 3 3 3 3 3 also a unit vector and q1, q2 and q3 are angles between
the vectors a, b; b, c and c, a, respectively, then among
29. If the four points a, b, c, d are coplanar, then q1, q2 and q3.
[b c d ] + [c a d ] + [a b d ] = (A) all are acute angles
(A) 0 (B)  1 (B) all are right angles
(C) -1 (D)  [a b c] (C) at least one is obtuse angle
30. A tetrahedron has vertices at O(0, 0, 0), A(1, 2, 1), (D) none of these
B(2, 1, 3) and C(-1, 1, 2). Then, the angle between the
37. If x and y are two non-collinear vectors and ABC is a
faces OAB and ABC will be
triangle with side lengths a, b, c satisfying

-1 æ 19 ö æ 71 ö ( 20 a - 15b) x + (15b - 12c) y + (12c - 20 a) ( x ´ y ) = 0,
(A) cos ç ÷ (B)  cos -1 ç ÷
PRACTICE EXERCISES

è 35 ø è 31 ø then ∆ABC is
(C) 30° (D)  90° (A) an acute-angled triangle
31. If a quadrilateral ABCD is such that AB = b, AD = d (B) an obtuse-angled triangle
and AC = mb + pd (m + p ≥ 1), then the area of the (C) a right-angled triangle
quadrilateral is k(p + m) | b × d |, where k is equal to (D) an isosceles triangle
1 1
(A) (B)  38. If a ( a ´ b) + b (b ´ c) + g (c ´ a) = 0, then
4 8
(A) a, b, c are coplanar if all of α, β, γ ≠ 0
1 (B) a, b, c are coplanar if any one of α, β, γ ≠ 0
(C) (D)  none of these
2 (C) a, b, c are non-coplanar for any α, β, γ
32. Let a be a unit vector and b be a non-zero vector not (D) none of these
parallel to a. If two sides of the triangle are represented
by the vectors 3 ( a ´ b) and b - (a · b) a, then the 39. If p × q = r and p . q = c, then q =
angles of the triangle are cp − p × r cp + p × r
(A) 30°, 90°, 60° (B) 45°, 45°, 90° (A) | p |2 (B)  | p |2
(C) 60°, 60°, 60° (D)  none of these
cr − p × r cr + p × r
33. Let u and v be unit vectors. If w is a vector such that w (C) | p|2 (D)  | p |2
+ (w × u) = v, then | (u × v) · w |
Vector Algebra  13.27

40. Let a = i + j and b = 2i - k. The point of intersection of 41. The sides of a parallelogram are 2i + 4j - 5k and i + 2j
the lines r × a = b × a and r × b = a × b is + 3k. The unit vector parallel to one of the diagonals
size is
(A) - i + j + k (B) 3i - j + k 1 1
(C) 3i + j - k (D)  i-j-k (A) (3i + 6 j − 2k ) (B)  (3i − 6 j − 2k )
7 7
1 1
(C) ( −3i + 6 j − 2k ) (D)  (3i + 6 j + 2k )
7 7
Previous Year’s Questions

a a2 1 + a3
42. Given two vectors are iˆ - ˆj and iˆ + 2 ˆj the unit vec- 47. If b b 2 1 + b3 = 0 and vectors (1, a, a2) (1, b, b2)
tor coplanar with the two vectors and perpendicular to c c2 1 + c3
first is: [2002]
and (1, c, c2) are non-coplanar, then the product abc
1 ˆ ˆ 1 ˆ ˆ equals [2003]
(A) (i + j ) (B)  (2i + j )
2 5 (A) 2 (B)  –1
(C) 1 (D)  0
1 ˆ ˆ
(C) ± (i + k ) (D)  none of these   
2 48. Let a , b and c be three non-zero vectors such that no

  If the vector a + 2b is col-
two of these are collinear.

linear with c and b + 3c is collinear  with a (λ being
43. The vector iˆ + xjˆ + 3kˆ is rotated through an angle  
some non-zero scalar) then a + 2b + 6c equals
θ and doubled in magnitude, then it becomes
4iˆ + ( 4 x - 2) ˆj + 2kˆ. The value of x are: [2004]
[2002]  
(A) l a (B)  lb
ì 2 ü ì1 ü 
(A) í- , 2 ý (B)  í , 2ý (C) l c (D)  0
î 3 þ î3 þ 49. A particle is acted upon by constant forces 4iˆ + ˆj - 3kˆ
ì2 ü and 3iˆ + ˆj - kˆ which displace it from a point
(C) í , 0 ý (D) {2, 7}
î3 þ iˆ + 2 ˆj + 3kˆ to the point 5iˆ + 4 ˆj + kˆ. The work done in
   standard units by the forces is given by [2004]
44. If the vectors a , b and c from the sides BC, CA and (A) 40 (B)  30
AB respectively of a triangle ABC, then: [2002] (C) 25 (D)  15

PRACTICE EXERCISES
     
(A) a × b = b × c = c × b = 0 50. If a , b , c are non-coplanar vectors and λ is a real
      number, then the vectors a + 2b + 3c , l b + 4c and
(B) a ´ b = b ´ c = c ´ a
( 2l - 1) c are non-coplanar for [2004]
     
(C) a × b = b × c = c × a = 0 (A) all values of λ
     
(D) a ´ a + a ´ c + c ´ a = 0 (B) all except one value of λ
(C) all except two values of λ
   (D) no value or λ
45. If the vectors a - xiˆ + yjˆ + zkˆ and such that a , c and
 51. Let u , v , w be such that | u | = 1, | v | = 2, |w | = 3. If
b form a right handed system, then c is: [2002]
 the projection v along u is equal to that of w along
(A) ziˆ - xkˆ (B)  0 u and v , w are perpendicular to each other then
(C) yĵ (D)  - ziˆ + xkˆ | u - v + w | equals [2004]
(A) 2 (B)  7
46. The centre of the circle given by r ⋅ (iˆ + 2 ˆj + 2kˆ ) = 15 (C) 14 (D) 
14
and | r ⋅ ( ˆj + 2kˆ ) | = 4 is: [2002] 52. Let a , b and c be non-zero vectors such that
(A) (0, 1, 2) (B)  (1, 3, 4) ( a ´ b ) ´ c = - | b | c | a . If θ is the acute angle
(C) (–1, 3, 4) (D)  none of these be­tween the vectors b and c then sinθ equals[2004]
13.28  Chapter 13

(A) only y
1 2
(A) (B)  (B) only x
3 3 (C) both x and y
2 2 2 (D) neither x nor y
(C) (D) 
3 3         
60. If ( a ´ b ) ´ c = a ´ (b ´ c ), where a , b and c are any
53. If C is the mid point of AB and P is any point outside     
three vectors such that a × b ¹ 0, b × c ¹ 0, then a and
AB, then [2005] 
   c are [2006]
(A) PA + PB = 2 PC p
   (A) inclined at an angle of between them
(B) PA + PB = PC
×

3
  
(C) PA + PB + 2 PC = 0 p
(B) inclined at an angle of between them
   6
×

(D) PA + PB + PC = 0 (C) perpendicular


 (D) parallel
54. The distance between the line r = 2iˆ - 2 ˆj + 3kˆ
61. The values of a, for which the points A, B, C with posi-
+l (iˆ + ˆj + 4 kˆ ) and the plane r ⋅ (iˆ + 5 ˆj + kˆ ) = 5 is
tion vectors 2iˆ - ˆj + kˆ, iˆ - 3 ˆj - 5kˆ and aiˆ - 3 ˆj + kˆ
 [2005]
10 respectively are the vertices of a right-angled triangle
10 p
(A) (B)  with C = are [2006]
9 3 3
2
3 10
(C) (D)  (A) 2 and 1 (B)  -2 and -1
10 3 (C) -2 and 1 (D)  2 and -1

55. For any vector a, the value of ( a × iˆ) 2 + ( a × ˆj ) 2 + ( a × kˆ ) 2 62. If uˆ and vˆ are unit vectors and θ is the acute angle
is equal to [2005] between them, then 2uˆ ´ 3vˆ is a unit vector for[2007]
2 2
(A) 3a (B)  a (A) exactly two values of θ
 
(C) 2a 2 (D)  4a 2 (B) more than two values of θ
(C) no value of θ
56. If non-zero numbers a, b, c are in H.P., then the straight
x y 1 (D) exactly one value of θ
line + + = 0 always passes through a fixed point.
a b c
That point is [2005] 63. Let a = iˆ + ˆj + kˆ, b = iˆ - ˆj + 2kˆ and c = xiˆ + ( x - 2) ˆj - kˆ.
(A) (–1, 2) (B)  (–1, –2) If the vector c lies in the plane of a and b , then x
æ 1ö equals [2007]
PRACTICE EXERCISES

(C) (1, –2) (D)  ç1, - 2 ÷ (A) 0 (B)  1


è ø
(C) −4 (D)  −2
57. Let a, b and c be distinct non-negative numbers. If
the vectors aiˆ + ajˆ + ckˆ, iˆ + kˆ and ciˆ + cjˆ + bkˆ lie in 
64. The vector a = a iˆ + 2 ˆj + b kˆ lies in the plane of the
a plane, then c is [2005]  
vectors b = iˆ + ˆj and c = ˆj + kˆ and bisects the angle
(A) the Geometric Mean of a and b  
(B) the Arithmetic Mean of a and b between b and c . Then which one of the following
(C) equal to zero gives possible values of α and β ? [2008]
(D) the Harmonic Mean of a and b (A) α = 2, β = 2 (B)  α = 1, β = 2
   (C) α = 2, β = 1 (D)  α = l, β = 1
58. If a , b , c are non-coplanar vectors and λ, is a real
           
number then [l ( a + b ) l 2 b l c ] = [ab + cb ] for [2005] 65. The non-zero vectors a , b and c are related by a = 8b
  
(A) exactly one value of λ and c = 7b . Then the angle between a and c is[2008]
(B) no value of λ (A) 0 (B)  π /4
(C) exactly three values of λ (C) π /2 (D) 
π
(D) exactly two values of λ
    
 66. If u , v , w are non-coplanar vectors and p, q are real
59. Let a = iˆ - kˆ, b = xiˆ + ˆj + (1 - x )kˆ and c = yiˆ + xjˆ      
   numbers, then the equality [3u pv pw ] - [ pv w qu ]
+(1 + x − y )kˆ. Then [a , b , c ] depends on [2005]   
- [2w qv qu ] = 0 holds for [2009]
Vector Algebra  13.29

(A) exactly one value of (p, q)    


73. Let ABCD be a parallelogram such that AB = q, AD = p
(B) exactly two values of (p, q) 
and BAD be an acute angle. If r is the vector which
(C) more than two but not all values of (p , q) coincides with the altitude directed from the vertex B
(D) all values of (p, q) 
to the side AD, then r is given by [2012]
 
67. The projections of a vector on the three coordinate   3( p × q ) 
axis are 6, −3, 2 respectively. The direction cosines of (A) r = 3q -   p
( p × p)
the vector are [2009]  
6 3 2   æ p×q ö 
(A) 6, −3, 2 (B)  , - , (B) r = - q + ç   ÷ p
5 5 5 è p× p ø
 
6 3 2 6 3 2   æ p×q ö 
- ,- ,
(C) , - , (D)  (C) r = q - ç   ÷ p
7 7 7 7 7 7 è p× p ø
 
  3( p × q ) 
        (D) r = - 3q +   p
68. Let a = j - k and c = i - j - k . Then, the vector b ( p × p)
    
satisfying a ´ b + c = 0 and a × b = 3 is [2010]
 
(A) 2iˆ - ˆj + 2kˆ iˆ - ˆj - 2kˆ
(B)  74. If the vectors AB = 3iˆ + 4 kˆ and AC = 5iˆ + 2 ˆj + 4 kˆ
represent the sides of a triangle ABC, then the length
-iˆ + ˆj - 2kˆ
(C) iˆ + ˆj - 2kˆ (D)  of the median through A is[2013]

  (A) 72 (B) 
33
69. If the vectors a = iˆ - ˆj + 2kˆ, bˆ = 2iˆ + 4 ˆj + kˆ and c = liˆ + ˆj
(C) 45 (D) 
18
+ m k̂ are mutually orthogonal, then the tuple (λ, µ) =
 [2010]
     
(A) (2, -3) (B)  (-2, 3) 75. If [a ´ bb ´ cc ´ a ] = l[abc ]2 , then the value of λ is
(C) (3, -2) (D)  (-3, 2) equal to [2014]
(A) 2 (B)  3
1 1 (C) 0 (D)  1
70. If a = (3iˆ + kˆ ) and b = (2iˆ + 3 ˆj − 6 kˆ ), then the
10 7
  
value of the expression( 2 z - b).[( a ´ b) ´ ( a + 2b)] is 76. Let a , b and c be three non-zero vectors such that no
   1   
 [2011] two of them are collinear and ( a ´ b ) ´ c = | b || c | a.
(A) −3 (B)  5   3
(C) 3 (D)  −5 If θ is the angle between vectors b and c , then a value

PRACTICE EXERCISES
of sinθ [2015]
 
71. The vectors a and b are not perpendicular and
      - 2 2
c and d are two vectors satisfying: b ´ c = b ´ d and (A) (B) 
   3 3
a × d = 0. Then, the vector d is equal to [2011]
-2 3 2 2
æ a.c ö æ b.c ö (C) (D) 
(A) c + ç
÷ b (B)  b + ç ÷c 3 3
è a.b ø è a.b ø
æ a.c ö æ b.c ö   
(C) c - ç ÷ b b-ç
(D)  ÷c 77. Let a , b and c be three unit vectors such that
è a.b ø è a.b ø
   3    
72. Let aˆ and bˆ be two unit vectors. If the vectors a ´ (b ´ c ) = (b + c ). if b is not parallel to c, then
 2  
c = aˆ + 2bˆ and d = 5aˆ - 4bˆ are perpendicular to each the angle between a and b is [2016]
other, then the angle between aˆ and bˆ is  [2012]
p p 5p 3p

(A) (B)  (A) (B) 
× ×

× ×

6 4
6 2
p p p 2p
(C) (D)  × × (C) (D) 
3
× ×

4 2 3
13.30  Chapter 13

   r r r 1 r 1
78. Let a = 2iˆ + ˆj − 2kˆ and b = iˆ + ˆj . Let c be a vector 80. Let a = (l – 2) a + b and b = (4l – 2) a + 3 b be two
    
such that | c − a |= 3, | ( a × b ) × c |= 3 and the angle 1 1
given vectors where vectors a and b are non-collinear.
    
between c and a × b be 30º. Then a . c is equal to 1 1
The value of l for which vector a and b are collinear,
 [2017] is: [2019]
(A) 2 (B) 5
(A) 4 (B) 3
1 25 (C)  – 3 (D)  – 4
(C)  (D) 
8 8 81. Two sides of a parallelogram are along the lines, x + =
 3 and x – y + 3 = 0. If its diagonals intersect at (2, 4)
79. Let u be a vector co-planar with the vectors
   then one of its vertex is: [2019]
a = 2iˆ + 3 ˆj − kˆ and b = ˆj + kˆ. If u is perpendicular (A)  (2, 1) (B)  (3, 6)
   2 (C)  (2, 6) (D)  (3, 5)
to a and u ⋅ b = 24, then u is equal to [2018]
(A) 336 (B) 315 (C) 256 (D) 84

ANSWER K EYS
Single Option Correct Type
1. (A) 2. (B) 3. (B) 4. (A) 5. (B) 6. (A) 7. (C) 8. (B) 9. (A) 10. (B)
11. (A) 12. (C) 13. (A) 14. (A) 15. (D) 16. (A) 17. (A) 18. (C) 19. (A) 20. (C)
21. (D) 22. (C) 23. (A) 24. (C) 25. (C) 26. (C) 27. (B) 28. (C) 29. (D) 30. (A)
31. (C) 32. (A) 33. (B) 34. (A) 35. (A) 36. (C) 37. (C) 38. (B) 39. (A) 40. (C)
41. (A)

Previous Years’ Questions


42. (A) 43. (A) 44. (B) 45. (A) 46. (B) 47. (B) 48. (D) 49. (A) 50. (C) 51. (C)
52. (A) 53. (A) 54. (B) 55. (C) 56. (C) 57. (C) 58. (B) 59. (D) 60. (D) 61. (A)
62. (D) 63. (D) 64. (D) 65. (D) 66. (A) 67. (C) 68. (B) 69. (D) 70. (D) 71. (C)
PRACTICE EXERCISES

  72. (C) 73. (B) 74. (B) 75. (D) 76. (D) 77. (A) 78. (A) 79. (A) 80. (D) 81. (B)
Vector Algebra  13.31

HINTS AND EXPLANATIONS

Single Option Correct Type


1. Let r = x1a + x2b + x3 (a × b)
Then, r ⋅ a = x1 |a|2, r ⋅ b = x2 |b|2
and, r ⋅ (a × b) = x3 |a × b|2
⇒ x1 = 0, x2 = 1, x3 = 1 ∴  r = a × b + b

2. We have, a × (a × c) + 3b = 0
⇒ (a ⋅ c)a - (a ⋅ a)c + 3b = 0
Þ ( 2 3 cosq ) a - 3c + 3b = 0 AD = 2BC and FC = 2AB

Þ ( 2 cosq ) a - 3 c + 3 b = 0 ∴ AD = 2BC and FC = 2AB.(1)

Given that AD = xBC.

Þ |( 2 cosq ) a - 3 c |2 = | - 3 b |2
∴ 2BC = xBC, by (i)

Þ 4 cos 2 q | a |2 +3 | c |2 - 4 3 cosq ( a × c) = 3 | b |2 ⇒ x = 2.
(2)
Þ 12 cos 2 q + 12 - 4 3 cosq ´ 3 ´ 2 cosq = 3 Again, given that CF = yAB or -FC = yAB.

⇒ 12 cos2θ + 9 - 24cos2θ = 0
∴ -2AB = yAB, using (ii)

⇒ y = -2.(3)

9 3
Þ 12 cos 2 q = 9 Þ cos 2 q = = . From (ii) and (iii), xy = 2 (-2) = -4.

12 4
6. OM1 = OA + OB + OC + OD (given)
3. Let i, j be unit vectors along the coordinate axes
= OM + MA + OM + MB + OM + MC + OM + MD
∴ a = 2pi + 1 · j(1)
= 4OM + (MA + MC) + (MB + MD)

On rotation, let b be the vector having components = 4OM( MA = -MC, MB = -MD) \
p + 1 and 1. 1 1

HINTS AND EXPLANATIONS


\ OM = OM1 , \ l = .
∴ b = (p + 1) i + 1 · j(2)
4 4

where i, j are unit vectors along the new coordinate axes.
7. A point equidistant from AB and AC is on the bisector of the
But on rotation | b | = | a | ⇒ | b |2 = | a |2
angle BAC.
⇒ ( p + 1)2 + 1 = (2p)2 + 1 ⇒ 3p2 - 2p - 1 = 0
A vector along the internal bisector of the angle BAC
AB AC
1 = +
Þ (3 p + 1)( p - 1) = 0 Þ p = 1 or - . | AB | | AC |
3
3i + j − k i − j + 3k 1
= + = ( 4i + 2 k )
4. Here, a and c = a × b are non-collinear vectors. 9 +1+1 1+1+ 9 11
∴ Let b = xa + y (a × c)(1)
∴ AP = t(2i + k)

∴ β = a ⋅ b = a ⋅ [xa + y(a × c)]
∴ BP = AP - AB = t(2i + k) - (3i + j - k)


= x | a |2 + ya ⋅ (a × c) = xa2 ⇒ x = β /a2. = (2t - 3) i - j + (t + 1) k

And, c = a × b = a × [xa + y(a × c)]
Also, BC = AC − AB = (i − j + 3k ) − (3i + j − k )

= xa × a + ya × (a × c)
= - 2i - 2 j + 4k.

= 0 + y(a ⋅ c) a - y (a · a) c
But BP = s BC.

∴ (2t - 3) i - j + (t + 1) k = s (- 2i - 2 j + 4k)

= y {a ⋅ (a × b)} a - ya2c = -ya2c

∴ 2t - 3 = -2s, -1 = -2s, t + 1 = 4s

⇒ y = -1/a2

1
∴ from (1), b = (ba - a × c)/a2.
∴ s = and t = 1 ∴ AP = 2i + k .

2
5. Since ABCDEF is a regular hexagon, from plane geometry, 8. Since (a - d) ⋅ (b - c) = 0 ∴ DA · CB = 0
we have
∴ AD ⊥ BC.
13.32  Chapter 13

Since (b - d) ⋅ (c - a) = 0, ∴ DB ⋅ AC = 0
1
13. ( b + c ) = a × ( b × c ) = ( a ⋅ c ) b − ( a ⋅ b) c
∴ BD ⊥ CA.
2

Then, D is the intersection of the altitudes through A and B.  1   1 
Therefore, D is the orthocentre of the triangle ABC. ⇒ a⋅c −  b −  a ⋅ b +  c=0
 2 2
9. We have, (a + b) × (a × b)
Since a, b, c are non-coplanar so a, b, c are linearly indepen-
= a × (a × b) + b × (a × b)
dent. Hence,
1
= (a ⋅ b) a - (a ⋅ a) b + (b ⋅ b) a - (b ⋅ a) b
a⋅b = − .
2
= (a ⋅ b) (a - b) + a - b (b ⋅ b = b2 = 1, a ⋅ a = a2 = 1

a×b 1

as a, b are unit vectors) \ cosq =
= a×b = -
| a| |b| 2
= (a ⋅ b + 1) (a - b)

(θ is the angle between a and b)
= x(a - b), where x = a ⋅ b + 1 is a scalar.
⇒ θ = 3π /4.
∴ The given vector is parallel to a - b.
14. We have a ⊥ b ⇒ a, b, a × b are linearly independent
⇒ v can be expressed uniquely in terms of a, b and a × b.
1
10. Clearly, r × c = l [a b c] = l Let v = xa + yb + za × b(1)
8
Given that, : a · b = 0, v · a = 0, v · b = 1, [v, a, b] = 1
1
r × a = m [a b c] = m ∴ v · a = xa2 or xa2 = 0 or x = 0 (2)
8
v · b = xv · b + yb + zb · (a × b)
2
1
r × b = n [abc] = n 1
8 or, yb 2 = 1 or y = 2 (3)
b
1 v · a × b = x ⋅ 0 + y ⋅ 0 + z |a × b|2
\ r × ( a + b + c) = (l + m + n )
8
or, z |a × b|2 = 1
∴ λ + µ + ν = 8r ⋅ (a + b + c). 1
11.   DB = DA + AB
\ or, z =
(4)
| a ´ b |2
or, DA = DB - AB
From (1), (2), (3) and (4), we get
D C 1 1
v = 2 b+ a × b.
|b| | a × b |2
HINTS AND EXPLANATIONS

c
b 15. We have, c = xa + yb + z (a × b)
⇒ c ⋅ a = x and c ⋅ b = y ⇒ x = y = cosθ
A a
B Now, c ⋅ c = | c |2
⇒ [xa + yb + z(a × b)] ⋅ [xa + yb + z(a × b)] = | c |2
∴ (DA)2 = (DB)2 + (AB)2 - 2DB ⋅ AB(1)
⇒ 2x2 + z2 | a × b |2 = 1
In parallelogram 2(a2 + b2) = c2 + DB2
⇒ 2x2 + z2 [ | a |2 | b |2 - (a ⋅ b)2] = 1
∴ (DB)2 = 2a2 + 2b2 - c2
⇒ 2 x 2 + z 2 [1 − 0] = 1 [∵ a ⊥ b ∴ a ⋅ b = 0]
∴ From (1), b2 = 2a2 + 2b2 - c2 + a2 - 2AB ⋅ DB
⇒ 2x + z = 1 ⇒ z = 1 - 2x2 = 1 - 2cos2 θ = -cos2θ.
2 2 2

16. From geometry


3a 2 + b 2 − c 2
∴ AB ⋅ DB =
. 2 SD = AO, where D is the mid-point of BC.
2
∴ SA + SB + SC
12. We have,
= SA + SD + DB + SD + DC
a1 a2 a3
2

= SA + 2 SD[ DB + DC = 0]
b1 b2 b3 = [a b c]2 = [( a × b) ⋅ c]2 = SA + AO = SO
c1 c2 c3 ∴ SA + SB + SC = SO.

= [| a | | b | sin π /6 c ⋅ c]2

= | a |2 | b |2 sin2 π /6 · 1 [ |c| = 1]
= ( a + a + a )(b + b + b ) ´ 1 / 4
2
1
2
2
2
3 1
2 2
2
2
3

= 1 / 4 ( a12 + a22 + a32 )(b12 + b22 + b32 ).


Vector Algebra  13.33

17. We have, d ⋅ {a × [b × (c × d)]} Now let OD and AE intersect at P such that OP/OD = λ and

= d ⋅ (a × ((b ⋅ d)c - (b ⋅ c)d )) AP/AE = µ.
= d ⋅ ((b ⋅ d) (a × c) - (b ⋅ c) (a × d)) ∴ OP = lOD and AP = µAE.

= (b ⋅ d) d ⋅ (a × c) - (b ⋅ c) d ⋅ (a × d) 1
or, OP = lOD = ( 2b + a) l .

= (b ⋅ d) [a c d][∴ d ⋅ (a × d) = 0]. 3
18. 4a + 5b + 9c = 0 ⇒ Vectors a, b and c are collinear æ1 ö
and, AP = m AE = m (OE - OA) = m ç b - a ÷ .

⇒ (b × c) × (c × a) = 0. è2 ø
In ∆ OAP, OA + AP = OP

 n   n  n
19.  ∑ ai  ⋅  ∑ ai  = ∑ | ai |2 + 2 ∑ ∑ ai ⋅ a j æ1 ö 1
 i= 1   i = 1  i = 1 1≤ i < j≤ n Þ a + m ç b - a ÷ = ( 2b + a ) l
è2 ø 3
n
⇒ 0 = n+2 ∑ ∑a ⋅a i j ⇒ ∑ ∑a ⋅a i j =−
2 æ 1 ö æ1 2 ö
Þ ç1 - m - l ÷ a + ç m - l ÷ b = 0
1≤ i < j≤ n 1≤ i < j≤ n
è 3 ø è2 3 ø
20. Let O be taken as the origin of reference. Let a, b, c be the
position vectors of A, B, C respectively so that 1 1 2
Þ 1 - m - l = 0 and m - l = 0,
OA = a, OB = b, OC = c. 3 2 3

a and b are non-collinear.
4 3
Solving these equations, we get m = , l = .

5 5
Now, OP = lOD, PD = OD - OP = (1 - λ) OD

OP l 3/5 3
\
= = =
PD 1 - l 1 - 3 / 5 2
3 3
⇒ OP = PD ⇒ OP = PD ⇒ OP : PD = 3 : 2.
Then, the vector representing the force P along OA 2 2
OA P
= P (unit vector along OA) = P = a. 1
OA OA 23. We have, a × (b × c) = b
Similarly, the vector representing the force Q along OB 2
Q 1
= b and the vector representing the force R along OC ∴ ( a ⋅ c ) b − ( a ⋅ b) c = b

OB 2

HINTS AND EXPLANATIONS


R æ 1ö
= c. Þ ç a × c - ÷ b - ( a × b) c = 0 (1)
OC è 2ø
Since R is the resultant of P and Q, we have
Since b and c are non-parallel, (Given)
(P/OA) a + (Q/OB) b = (R/OC) c ∴ (i) exists if coefficients of b and c vanish separately

or, (P/OA) a + (Q/OB) b - (R/OC) a = 0.(1) 1
i.e., a ⋅ c −
= 0 and a ⋅ b = 0.
Since, a, b, c are the position vectors of three collinear 2
points, therefore, we have
1
the algebraic sum of the coefficients of a, b, c in (i) = 0 ⇒ a ⋅ c = and a ⋅ b = 0.
2
i.e., (P/OA) + (Q/OB) - (R/OC) = 0 Let q1 and q2 be the angles which a makes with b and c,
P Q R respectively. Also a, b, c are unit vectors,
Hence, + = .
OA OB OC 1
p \ cosq 2 = and cosq1 = 0
21. When rotated through , the new x-axis is along old
× 2
2
y-axis and new y-axis is along the old negative x-axis; i.e., q2 = 60º and q1 = 90º
z-axis remains same as before. Hence, the components of A Hence, q1 = 90º and q2 = 60º.
in the new system are A2, -A1, A3. 24. Let c = xa + yb
22. Let a and b be the position vectors of A and B respectively = x(i + j - k) + y(i - j + k)
w.r.t. O as the origin. = (x + y)i + (x - y)  j - (x - y)k
Then p.v. of E = OE Since c is ⊥ to a
1 1 ∴ x + y + x - y + x - y = 0 ⇒ 3x = y
= OB = b
2 2 Since | c | = 1,
1 ∴ (x + y)2 + (x - y)2 + (x - y)2 = 1
and, p.v. of D = OD = ( 2b + a), ∵ AD : DB = 2 :1 ∴ (4x)2 + (-2x)2 + (-2x)2 = 1.
3
13.34  Chapter 13

1
Þ 24 x 2 = 1. \ x =
2 6
2 1 1
∴ c = 4 xi + 2 xj + 2 xk =
i− j+ k
6 6 6
1
i.e., c =
( 2i − j − k )
6
Let d = d1i + d2   j + d3k

Since d is ⊥ to a and c

∴ d · a = 0 ⇒ d1 + d2 - d3 = 0
i+ j+k i+ j+k
e= =
and, d ⋅ c = 0 ⇒ 2d1 - d2 + d3 = 0
|i + j + k | (1 + 1 + 1)

⇒ 3d1 = 0, ∴ d1 = 0, ∴ d2 = d3 1
= (i + j + k ).
Since | d | = 1,
3
\ d12 + d22 + d32 = 1
∴ ON = projection of OC on OE
Þ 0 + d22 + d22 = 1 Þ 2d22 = 1 1 1
= k ⋅e = k ⋅ (i + j + k ) = .
1 1 1 3 3
Þ d = , or d2 =
2
, \ d2 = d3 = .
2
2
2 2 ∴ CN2 = OC2 - ON2
[In right triangle ∆ OCN]
2
1 æ 1 ö 1 2
Hence, d =
( j + k ). = 12 - ç ÷ =1- = .
2 è 3ø 3 3
25. Let x be the first term and y the common ratio of G.P.
2 2 3 6
Then, a = pth term = xy p - 1 \ CN =
= ´ =
3 3 3 3
b = qth term = xy q - 1
c = rth term = xy r - 1
28. The length of a is | a | = 12 + 12 = 2 . Similarly, | b | = 2 .
Now, (ilna + jlnb + klnc) ⋅ [i(q - r) + j(r - p) + k (p - q)]
Since the three vectors have equal length, ∴ | c | = 2 . Let
= (q - r) lna + (r - p) lnb + (p - q) lnc
c = c1i + c2   j + c3k. Then, since c makes an obtuse angle with
= (q - r) ln (xyp - 1) + (r - p) ln (xyq - 1) + (p - q)ln (xyr - 1) i, we must have c · i = c1 < 0. We are also given that the angles
= (q - r) {lnx - (p - 1) lny} + ... between the vectors are equal, i.e.,
HINTS AND EXPLANATIONS

= {(q - r) + (r - p) + (p - q)} lnx + {(q - r) (p - 1) a⋅b a⋅c b⋅c


+ (r - p) (q - 1) + (p - q) (r - 1)} lny = 0.
cos −1 = cos −1 = cos −1
| a| |b| | a| |c| |b| |c|
Hence, the vectors are perpendicular.
Now, a ⋅ b = 1, a ⋅ c = c1 + c2 and b ⋅ c = c2 + c3, so

26. Since a is collinear with b = 6i - 8j - (15/2) k.
a⋅b 1 c +c
∴ there exists a scalar t, s.t. cos −1 = cos −1 = cos −1 1 2
| a| |b| 2 2
a = tb = 6ti - 8tj - (15/2) tk.
Now, if a makes an angle θ with positve z-axis, then c2 + c3

= cos -1 .
a×k - (15 / 2) t 2
cosq = = . This gives the equations c1 + c2 = 1 and c2 + c3 = 1, from

| a| | k | | a|
which we get c3 = c1 and c2 = 1 - c1. Putting these values in
-15t c12 + c22 + c32 = 2, we get
∵ q < 90° \ cosq =
> 0 Þ t < 0.
2 |a|
c12 + (1 - c1 ) 2 + c12 = 2
Now, | a |= [{36 + 64 + ( 225 / 4)}t 2 ] = 50

Þ 3c12 - 2c1 - 1 = 0
⇒ t2 = 16 ⇒ t = -4,
∴ 1
[Rejecting t = 4, t < 0] Þ (c1 - 1)(3c1 + 1) = 0 Þ c1 = 1, - .
Hence, a = - 24i + 32  j + 30k = (-24, 32, 30). 3
27. Let the unit vectors i, j and k be denoted by the coterminous Since c1 must be less than zero, we rule out the solution c1
4
edges OA, OB and OC, respectively of the unit cube. Let CN = 1, giving c1 = -1/3 = c3 and c2 = 1 - c1 = . Hence, the
be the perpendicular drawn from C on the diagonal OE of 3
required vector is
the cube which does not pass through C. Here, OE = i + j + 1 4 1
k. Let e be the unit vector along OE. Then, c = − i + j − k.
3 3 3
Vector Algebra  13.35

29. Let the given points be A, B, C, D, respectively. Now, if the Hence, the area of the quadrilateral ABCD
points A, B, C, D are coplanar, then AB, AC, AD are coplanar. 1 1
= ( p + m) | b × d | . ∴ k = .
∴ AB ⋅ (AC × AD) = 0 2 2
⇒ (b - a) ⋅ {(c - a) × (d - a)} = 0.
∴a×a=0 32. Let ABC be a triangle in which the given vectors are repre-
⇒ (b - a) ⋅ {c × d - c × a - a × d} = 0 sented by the sides AB and AC.
⇒ b ⋅ (c × d) - b ⋅ (c × a) - b ⋅ (a × d) - a ⋅ (c × d)
i.e., AB = 3 ( a × b)

+ a ⋅ (c × a) + a ⋅ (a × d) = 0
⇒ [b c d] - [b c a] - [b a d] - [a c d] + 0 + 0 = 0 and, AC = b - (a ⋅ b)a

⇒ [b c d] - [a b c] + [a b d] + [c a d] = 0 ∴ AB ⋅ AC = 3( a × b) ⋅ [b − ( a ⋅ b)a]

⇒ [b c d] + [c a d] + [a b d] = [a b c].
= 3[( a × b) ⋅ b − ( a ⋅ b)( a × b) ⋅ a]
30. Vector ⊥ to face OAB
= 3 [ 0 - 0 ] = 0.
i j k Therefore, ∠ BAC = 90°

= OA × OB = 1 2 1 = 5i − j − 3k (1) AB 2 = [ 3 ( a × b)]2 = 3( a × b) 2 (1)
2 1 3 AC 2 = [b − ( a ⋅ b)a]2

Vector ⊥ to the face ABC



= (b) 2 + ( a ⋅ b) 2 a 2 − 2(b ⋅ a)( a ⋅ b)

i j k
= (b) 2 + ( a ⋅ b) 2 − 2( a ⋅ b) 2
= AB × AC = 1 −1 2 = i − 5 j − 3k (2)
= ( b) 2 − ( a ⋅ b) 2
− 2 −1 1
= (b) 2 - | a |2 | b |2 cos 2 q
Since the angle between the faces = angle between their

= (b) 2 [1- | a |2 cos 2 q ] = (b) 2 (1 - cos 2 q )
normals
= (b) 2 sin 2 q = | a |2 | b |2 sin 2 q = ( a ´ b) 2 . (2)
5+5+9 19
Dividing (i) by (ii), we get
\ cosq =
=
35 35 35 AB 2 3 ( a × b) 2

=
æ 19 ö AC 2 ( a × b) 2
\ q = cos -1 ç ÷ .

è 35 ø ⇒ AB 2 = 3 ⋅ AC 2 ⇒ AB = 3 AC .
AB 3 AC

HINTS AND EXPLANATIONS


31. We have, tan C = = = 3 , \ ÐC = 60°
BC = BA + AC = - b + mb + pd AC AC
= (m - 1) b + pd So ∴
∠ A = 180 - 90° - 60° = 30°.

Hence, angles of the triangle are 30°, 90° and 60°.
and, CD = CA + AD = - mb - pd + d
= - mb + (1 - p) d 33. Given equation is w + (w × u) = v(1)
1 Taking cross-product with u, we get
Area of the triangle ABD = | b × d | .
2 u × [w + (w × u)] = u × v
⇒ u × w + u × (w × u) = u × v
⇒ u × w + (u ⋅ u)w - (u · w)u = u × v
⇒ u × w + w - (u ⋅ w)u = u × v(2)
Taking scalar product of (1) with u we get
d u ⋅ w + u ⋅ (w × u) = u ⋅ v

⇒ u ⋅ w = u ⋅ v [   u ⋅ (w × u) = 0] (3)
b Taking scalar product of (1) with v we get
v ⋅ w + v ⋅ (w × u) = v ⋅ v

Area of the triangle BCD
⇒ v ⋅ w + [vwu] = 1
1
= |( m − 1)b + pd ) × ( − mb + (1 − p)d )| ⇒ (u × v) ⋅ w = 1 - v ⋅ w(4)
2
Taking scalar product of (2) with w we get
1
= | − pmd × b + (1 − p)( m − 1)b × d | (u × w) ⋅ w + w ⋅ w - (u ⋅ w) (u ⋅ w) = (u × v) ⋅ w
2
⇒ 0 + | w |2 - (u ⋅ w)2 = (u × v) ⋅ w(5)
1 1
= | m − 1 + p | | b × d | = ( p + m − 1) | b × d | ⇒ (u × v) · w = |w|2 - (u · w)2.
2 2
Taking scalar product of (1) with w we get
13.36  Chapter 13

w ⋅ w + (w × u) ⋅ w = v ⋅ w
But x2 + y2 + z2 = 1

⇒ | w |2 = v ⋅ w
2
æ 3t + 5 ö æ 5 - 4t ö 1
\ ç- ÷ +ç ÷ + 2 =1
⇒ | w |2 = 1 - (u × v) ⋅ w
[using (4)] (6) è 5t ø è 5t ø t

From (5) we get ⇒ 9t2 + 30t + 25 + 25 + 16t2 - 40t + 25 = 25t2
(u × v) ⋅ w = | w |2 - (u ⋅ w)2
75 15
Þ 75 - 10t = 0, \ t = =

= 1 - (u × v) ⋅ w - (u ⋅ w)2 10 2
⇒ 2(u × v) ⋅ w = 1 - (u ⋅ v)2
[Using (3)] 45
+5
- 55 11
1
Thus, |(u × v ) ⋅ w | = |1 − (u ⋅ v ) 2 |
Hence, x = - 2 = =-
75 75 15
2
2
1 5 - 30 - 50 -10

≤ [∴ (u ⋅ v ) 2 ≥ 0]. y= = =
2 75 75 15
2
34. Let I be a unit vector in the 1 2
direction of b, J in the direc- and, z = - =-
15 15
tion of c. Note that b = I and 2
c = J. ∴ unit vector along c
We have b × c = | b | | c | sin α −11i 10 j 2k 11i + 10 j + 2k
k = sin α k, = − − or − .
15 15 15 15
where k is a unit vector per-
pendicular to b and c. 36. Since | a + b + c | = 1 ⇒ ( a + b + c) ⋅ ( a + b + c) = 1
b´c ⇒ 1 + 1 + 1 + 2 ( a ⋅ b + b ⋅ c + c ⋅ a) = 1
Þ | b ´ c | = sin a Þ k=
|b ´ c| ⇒ a ⋅ b + b ⋅ c + c ⋅ a = −1

Any vector a can be written as a linear combination of I, J ⇒ cosq1 + cosq2 + cosq3 = -1
and k. Let So, at least one of cosq1, cosq2 and cosq3 must be negative.
a = a1i + a2 J + a3k 37. Since x and y are linearly independent,
a b c
Now, a ⋅ b = a ⋅ I = a1 , a ⋅ c = a ⋅ J = a2
20 a - 15b = 15b - 12c = 12c - 20 a = 0 Þ = =
3 4 5
and, a ⋅ b × c = a ⋅ k = a3
⇒ c2 = a2 + b2  ⇒  ∆ABC is right-angled.

HINTS AND EXPLANATIONS

|b × c|
38. We have,
a ⋅ (b × c) α (a × b) + β (b × c) + γ (c × a) = 0
Thus, ( a ⋅ b) b + ( a ⋅ c) c +
(b × c)
| b × c |2 Taking dot product with c, we have
b×c a [a b c] + b [b c c] + g [c a c] = 0
= a1b + a2c + a3
|b × c| i.e., a [a b c] + 0 + 0 = 0
= a1I + a2 J + a3k = a. i.e., a [a b c] = 0
Similarly, taking dot product with b and c, we have
35. Let c = xi + yj + zk , where x2 + y2 + z2 = 1 γ [a b c] = 0, b [a b c] = 0
1 Now, even if one of α, β, γ ≠ 0, then we have [a b c] =
Unit vector along 3i + 4 j = (3i + 4 j )

5 0  ⇒  a, b, c are coplanar.
∴ equation of the bisector of these two is
39. Since p × q = r
 ∴ p × ( p × q) = p × r
 3i + 4 j  
r = t ( xi + yj + zk ) + 
  5   ⇒ ( p ⋅ q) p − ( p ⋅ p) q = p × r
But the bisector is - i + j - k.
⇒ cp − ( p) 2 q = p × r
 3  4 ⇒ ( p) 2 q = cp − p × r
∴ t  x +  i + t  y +  j + tz = − i + j − k

 5  5 cp − p × r
⇒ q= .
1 3 3t + 5 | p |2
Þ x=- - =- ,
t 5 5t

1 4 5 - 4t 1 40. r × a = b × a ⇒ ( r − b) × a = 0
y= - = and z = -
t 5 5t t ⇒ r − b is paralllel to a.
Vector Algebra  13.37

\ r - b = l a i.e. r = b + l a (1)
p = a + b and q = b - a
Similarly, r × b = a × b can be written as
i.e., p = 3i + 6 j - 2k

r = a + m b (2) and, q = - i - 2 j + 8k


For point of intersection of the two lines (1) and (2), ∴ unit vectors along the diagonals are

we get b + l a = a + m b Þ l = m = 1
3i + 6 j - 2k - i - 2 j + 8k

and

Hence, the required point of intersection is given by 9 + 36 + 4 1 + 4 + 64
r = a + b = i + j + 2i − k = 3i + j − k . 3i + 6 j - 2k - i - 2 j + 8k

i.e., and
7 69
41. Let a = 2i + 4 j - 5k , b = i + 2 j + 3k ∴ unit vector parallel to one of the diagonals is

∴ diagonals of the parallelogram are 1

(3i + 6 j - 2k )
7

Previous Year’s Questions


   
42. Given two vectors lie in xy-plane. Hence a vector coplanar ⇒ a×c +b ×c =0
with them is    
   ⇒ b ×c =c ×a
a = xi + yj    
Similarly, a × b = b × c

ˆ ˆ
a ⊥ (i − j ) ⇒ a ⋅ (i − j ) = 0
ˆ ˆ      
Hence a × b = b × c = c × a.

⇒ ( xiˆ + yjˆ ) ⋅ (iˆ − ˆj ) = 0 ⇒ x − y = 0
    
⇒ x = y
45. Given that a = xi + yjˆ + zkˆ and b = ˆj are such that a, c and b
  form a right handed system.
∴ a = xiˆ + xjˆ and | a | − x 2 + x 2 = x 2

iˆ ˆj kˆ
a x(iˆ + ˆj )   
Required unit vector =
= ∴ c =b×a= 0 1 0

|a| x 2
1 ˆ ˆ x y z
∴ = (i + j )
2 ˆ − xkˆ
= iz

HINTS AND EXPLANATIONS


46. The equation of a line through the center represented by vec-
43. The vector, iˆ + xjˆ + zkˆ if it is doubled in magnitude becomes tor ˆj + 2kˆ and normal to the given plane is
4iˆ + ( 4 x − 2) ˆj + 2kˆ 
r = ˆj + 2kˆ + l (iˆ + 2 ˆj + 2kˆ ) (1)
∴ 2 | iˆ + xjˆ + 3kˆ | = | 4iˆ + (4 x − 2) ˆj + 2kˆ |
This meets the plane at a point for which we must have
⇒ 2 1 + x 2 + 9 = 16 + ( 4 x − 2) 2 + 4 [( ˆj + 2kˆ ) + l (iˆ + 2 ˆj + 2kˆ )] ⋅ (iˆ + 2 ˆj + 2kˆ ) = 15
⇒ 6 + 9λ = 15

⇒ 40 + 4x2 = 20 + (4x - 2)2

⇒ λ = 1

⇒ 3x2 - 4x - 4 = 0

On putting λ = 1 in Eq. (i), we get

⇒ 3x2 - 6x + 2x - 4 = 0

r = iˆ + 3 ˆj + 4 kˆ
⇒ 3x (x - 2) + 2(x - 2) = 0

∴ Centre of the circle is (1, 3, 4).

⇒ (x - 2) (3x + 2) = 0

2
⇒ x = 2, − a a2 1 1 a a2
3
47. b b 2 1 + 1 b b 2 = 0
   c c2 1 1 c c2
44. Key Idea: If the vectors a , b and c represent the sides of a
  
triangle, then a + b + c = 0 a a2 1
   ⇒ (1 + abc) b b 2 1 = 0
∵ a+b+c =0
   c c2 1
⇒ a + b = −c
⇒ abc = −1.
    
⇒ (a + b ) × c = − c × c
Hence, (8) is the correct answer
13.38  Chapter 13

  
48. ( a + 2b ) = t1c (1)
  
And b + 3c = t 2 a (2)
 
(1) - 2 ´ ( 2) Þ a (1 + 2t 2 ) + c ( -t1 - 6) = 0 Þ 1 + 2t 2 = 0
Þ t 2 = -1 / 2 & t1 = -6.
  

(since a and c are non-collinear) 54. Distance between the line with equation r = 2iˆ - 2 ˆj + 3kˆ
+ l (iˆ − ˆj + 4 kˆ ) and the plane r ⋅ (iˆ + 5 ˆj + kˆ ) = 5 equiva-

Putting
 the value
 of t1 and t2 in (1) and (2), we get
 lently x + 5 y + z = 5 is equal to the perpendicular distance
a + 2b + 6c = 0.
of point (2, –2, 3) from the plane
      2 − 10 + 3 − 5 10
49. Work done by the forces F1 and F2 is ( F1 + F2 ) ⋅ d where d is
i.e., = .
the displacement. 1 + 52 + 1 3 3


Now, according to question

F1 + F2 = (4iˆ + ˆj − 3kˆ ) + (3iˆ + ˆj − kˆ ) = 7iˆ + 2 ˆj − 4 kˆ and 55. Let a = xiˆ + yjˆ + zkˆ, then

a × iˆ = zjˆ − ykˆ
d = (5iˆ + 4 ˆj + kˆ ) − (iˆ + 2 ˆj + 3kˆ ) = 4iˆ + 2 ˆj − 2kˆ. Hence
   ⇒ ( a × iˆ) = y + z

2 2 2
( F1 + F2 ) ⋅ d is 40.
Similarly ( a × ˆj ) 2 = x 2 + z 2

and ( a × kˆ ) = x + y ⇒ ( a × iˆ) = y + z
2 2 2 2 2 2
50. Condition for given three vectors to be coplanar is
1 2 3 Similarly ( a × kˆ ) 2 = x 2 + y 2

0 l 4 = 0 Þ l = 0, 1 / 2. ⇒ ( a × iˆ) 2 + ( a + ˆj ) 2 + ( a × kˆ ) 2 = 2( x 2 + y 2 + z 2 ) = 2a 2 .

0 0 2l - 1
56.
Given that a, b, c are in H.P.

Hence given vectors will be non coplanar for all real values 2 1 1

− − =0
of λ. except for 0 and 1/2. b a c
    x y 1
    v ⋅u w⋅u + + = 0
51. Projection of v along u and w along u is  and  a b c
|u| |u|
HINTS AND EXPLANATIONS

respectively x y 1

= =
Now, according to question −1 2 −1
    ∴ x = 1, y = -2

v ⋅u w⋅u      
 =  ⇒ v ⋅ u = w ⋅ u ⋅ and v ⋅ w = 0 57. Vectors aiˆ + ajˆ + ckˆ, iˆ + kˆ and ciˆ + cjˆ + bkˆ are coplanar
|u| |u|
        a a c
| u - v + w |2 = | u |2 + | v |2 + | w |2 -2u × w = 14
  
Therefore, 1 0 1 = 0 ⇒ c 2 = ab
Þ | u - v + w |= 14
c c b
   1          1   
∴ a, b, c are in G.P.

52. ( a × b ) × c = | b || c | a ⇒ ( a ⋅ c ) b − (b ⋅ c )a = | b || c | a
3 3      
58. [l ( a + b ) l 2b l c ] = [a b + c b ]
53. By triangle law,
   l l 0 1 0 1
PA + AC + CP = 0 ⇒
0 l 0 = 0 1 1
2

   0 0 l 0 1 0


PB + BC + CP = 0

Adding, we get ⇒ λ4 = –1

     Hence no real value of λ.

PA + PB + AC + BC + 2CP = 0
   
Since AC = − BC
59. Given a = iˆ − kˆ, b = xiˆ + ˆj + (1 − x ) kˆ and
  
& CP = − PC c = yiˆ + xjˆ + (1 + x − y ) kˆ.
       
⇒ PA + PB − 2 PC = 0.
[abc ] = a ⋅ (b × c )
Vector Algebra  13.39

   
iˆ ˆj kˆ ∴ a and b are like vectors and b and c are unlike.

   
⇒ b ´ c = x 1
1 - x = iˆ (1 + x - x - y - x 2 ) ⇒ a and c will be unlike
 
y x 1+ x - y Hence, the angle between a and c = p .

- ˆj ( x + x 2 - xy - y + xy ) + kˆ ( x 2 - y )   
  66. (3 p 2 − pq + 2q 2 )[u v w ] = 0
aˆ ⋅ (b × c ) = 1      
But since u , v , w are non-coplanar, [u v w ] ≠ 0 which

Which does not depend on x and y. implies that
          3p2 − pq + 2q2 = 0
( ) ( )
60. Given condition a × b × c = a × b × c , a ⋅ b ≠ 0, b ⋅ c ≠ 0

q
Þ 2 p 2 + p 2 - pq + ( ) 2 +
7q 2
=0
           
( ) ( )
⇒ ( a ⋅ c ) b − b ⋅ c a = ( a ⋅ c ) b − a ⋅ b c
q
2
7
4

      Þ 2 p2 + ( p - )2 + q2 = 0 = 0
( ) ( )
⇒ a ⋅ b c = b ⋅ c a 2 4
  q
⇒ a || c ⇒ p = 0, q = 0, p =
2
61. The vectors This is possible only when p = 0, q = 0 and therefore exactly

 one value of (p, q).
BA = iˆ − 2 ˆj + 6 kˆ
67. Projection of a vector on coordinate axes are x2 − x1, y2 − y1,
CA = (2 − a)iˆ + 2 ˆj
z2 − z1
CB = (1 − a)iˆ − 6 ˆj
So, ( x2 − x1 ) 2 + ( y2 − y1 ) 2 + ( z2 − z1 ) 2 = 36 + 9 + 4 = 7

imply
  6 3 2
CA ⋅ CB = 0 The D.C’s of the vector are , − , respectively.
7 7 7
⇒ ( 2 − a)(1 − a) = 0   
68. c = b × a
⇒ a = 2,1  
⇒ b ⋅c = 0

62. Given that | 2uˆ × 3vˆ | = 1 ⇒ (b1iˆ + b2 ˆj + b3kˆ ) ⋅ (iˆ − ˆj − kˆ ) = 0



⇒ 6 | uˆ || vˆ ||sin q | = 1 ⇒ b1 − b2 − b3 = 0
1  
and a ⋅ b = 3

HINTS AND EXPLANATIONS


Þ sin q =
6 ⇒ b2 − b3 = 3
Hence there is exactly one value of θ for which 2uˆ × 3vˆ is a ⇒ bl = b2 + b3 = 3 + 2b3
unit vector.
   ⇒ b = (3 + 2b3 )iˆ + (3 + b3 ) ˆj + b3kˆ
63. a = iˆ + ˆj + kˆ, b = iˆ − ˆj + 2kˆ and c = xiˆ + ( x − 2) ˆj − kˆ
 
69. a ⋅ b = 0,
x x − 2 −1  

Therefore, 1 1 1 =0 b ⋅ c = 0,
 
1 −1 2 c ⋅a = 0
Þ 2l + 4 + m = 0l - 1 + 2 m = 0
⇒ 3x + 2 − x + 2 = 0
Solving we get:
⇒ 2 x = −4 l = -3, m = 2.

⇒ x = −2.
70. ( 2a - b ).{( a ´ b ) ´ ( a + 2b )}
64. Let for some l , a = l (bˆ + cˆ), then = ( 2a - b ).{2a - b ){[a .( a + 2b )]b - [b .( a + 2b )a ]}
= -5( a )2(b )2 + 5( ab )2 = -5
iˆ + 2 ˆj + kˆ
a iˆ + 2 ˆj + b kˆ = l ( )
2 71. b × c = b × d

Þ l = 2a and l = 2 and l = 2 b ⇒ a × (b × c ) = a × (b × d )
⇒ ( a .c )b − ( a .b )c = ( a .d )b − ( a .b )d
Þ a = 1 and b = 1.
⇒ ( a .c )b − ( a .b )c = −( a .b )d
    a .c
65. Since a = 8b , c = −7b . ∴ d = c −( )b
a .b
13.40  Chapter 13

      
  
72. c ⋅ d = 0 ⇒ 5 | a |2 + 6 a ⋅ b − 8 | b |2 = 0 79. If u is coplanar with a = 2iˆ + 3 ˆj − kˆ and b × ˆj + kˆ, then
    1   p   
Þ 6a × b = 3 Þ a × b = Þ (a × b ) = u ⋅ (a × b ) = 0
2 3
  iˆ ˆj kˆ
73. AE = vector component of q on p
   
( p ⋅ q)  a × b = 2 3 −1
AE =   p
( p ⋅ q) 0 1 1
  
∴ From ∆ABE ; AB + BE = AE
   = iˆ[3 + 1] − ˆj[2 − 0] + kˆ[2 − 0]
  ( p ⋅ q) p

⇒ q+r =   = 4iˆ − 2 ˆj + 2kˆ
( p ⋅ q)
    
  ( p × q) 
u ⋅ ( a × b ) = 4u1 − 2u2 + 2u3 = 0 (i)
Þ r =-q+   p
( p × q) 
Where u = u1iˆ + u2 ˆj + u3kˆ

74. Let M be the median, then  
 
 AB + AC u is perpendicular to a
AM =  
2 Hence, u ⋅ a = 0


AM = 4i = − ˆj + 4 kˆ
ˆ
2u1 + 3u2 – u3 = 0 (ii)
  
∴ | AM | = 16 + 16 + 1 = 33 u ⋅b = 0
     
u2 + u3 = 24 (iii)
75. [a ´ bb ´ cc ´ a ] = l [abc ]2

Equation (ii) + equation (iii)
λ=1
     1   
2u1 + 4u2 = 24 (iv)
76. Since ( a × c )b - (b × c )a = | b || c | a
Equation (i) – 2(equation (iii))
3
 1  
4u1 – 4u2 = –48
⇒ − b ⋅ c = | b || c |
u1 – u2 = –12 (v)
3
  1   Solving equation (iv) and equation (v), we get u2 = 8

Þ - | b || c | cosq = | b || c |
u1 = –4
3
1
u3 = 16
Þ cosq = - 
3
u = −4iˆ + 8 ˆj + 16 kˆ
HINTS AND EXPLANATIONS

2 2
Þ sin q = .
2
3 u = 4 2 + 82 + 16 2 = 336

æ ö æ  ö r r
77. Given expression is ç a × c - 3 ÷ b - ç a × b + 3 ÷ c = 0 80. As a + b are collinear
è 2 ø è 2 ø
  l -2 1
Þ a × b = cosq = - 3 / 2 Þ q = 5p / 6 = \l=–4
4l - 2 3
  
78. | ( a × b ) × c | = 3
81. C (4, 5)
D
  ˆ ˆ ˆ
a × b = 2i − 2 j + k
  
⇒ | a × b || c | sin 30° = 3 m (2, 4)

|a|=3=|a×b|
  •

 y=x-3
⇒ | c | = 2
 
| c − a |= 3 A (0, 3) B (h, 3 - h)
   
⇒ | c |2 + | a |2 −2( a . c ) = 9
3- h-5
Slope of BC = 1 Þ
= 1 \h=1
  9−3− 2 h-4

a .c = =2
2 B º (1, 2) and D = (3, 6)


Hence, the correct option is (A)
CHAPTER Measures of Central
14 Tendency and Dispersion
LEARNING OBJECTRIVES
After reading this chapter, you will be able to:
 Learn about the measures of central tendency and arith-  Understand deciles and percentiles and mode and sym-
metic mean metric distribution
 Know about geometric mean, harmonic mean, median
and quartiles

MEASURES OF CENTRAL TENDENCY Short Cut Method


For a given data, a single value of the variable which For the given data, we suitably choose a term, usually the mid-
describes its characteristics is identified. This single value dle term and call it the assumed mean, to be denoted by A.
is known as the average. An average value generally lies We find the deviation, di = (xi – A) for each term.
in the central part of the distribution and therefore, such Then the arithmetic mean is given by
­values are called the measures of central tendency.
The commonly used measures of central tendency x =A+
∑ fi di
are: ∑ fi
1. Arithmetic Mean Step Deviation Method
2. Geometric Mean When the class intervals in a grouped data are equal, then
3. Harmonic Mean the calculations can be simplified further by taking out the
4. Median common factor from the deviations. This common factor
5. Mode is equal to the width of the class interval. In such cases,
the deviation di = xi – A, of variates xi from the assumed
ARITHMETIC MEAN mean A are divided by the common factor. The A.M. is then
Mean of Unclassified Data obtained by the following formula:

Let x1, x2, ...., xn be n observations, then their arithmetic x =A+


∑ fi di× h; N = ∑ fi
mean is given by, N
x1 + x2 + ..... + xn 1 n where A = assumed mean,
x = = ∑ xi x −A
n n i =1 di = i = the deviation of any variate from A,
h
Mean of Grouped Data h = the width of the class - interval.
Let x1, x2, x3, ...., xn be n observations and let f1,  f2, . . . , fn be Weighted Arithmetic Mean
their corresponding frequencies, then their arithmetic mean
is given by If w1, w2, w3, ...., wn are the weights assigned to the values
n x1, x2, x3, ...., xn respectively, then the weighted average is
f x + f 2 x2 + ... + f n xn
∑ fi xi defined as:
i =1
x = 1 1 = w1 x1 + w2 x2 + ...... + wn xn
f1 + f 2 + .... + f n n
Weighted A.M. =
∑ fi w1 + w2 + ....... + wn
i =1
14.2  Chapter 14

Combined Mean
( )
1/ N
G.M. = x1f ⋅ x2f ....... xnf 1 2 n
,
If we are given the A.M. of two data sets and their sizes,
then the combined A.M. of two data sets can be obtained n
by the formula: where N= ∑ fi
i =1
n x + n2 x2
x12 = 1 1 ⎛ n ⎞

⎜ ∑ fi log xi ⎟
n1 + n2

where, x12 = Combined mean of the two data sets 1 and 2 or G.M. = antilog ⎜
i =1 ⎟
x1 = Mean of the first data ⎜ N ⎟
⎜ ⎟
x2 = Mean of the second data ⎜⎝ ⎟⎠

n1 = Size of the first data
n2 = Size of the second data
I M P O R TA N T P O I N T S
Properties of A. M.
In the case of continuous or grouped frequency distribution,
1. In a statistical data, the sum of the deviations of indi- the values of the variate x are taken to be the values corre-
vidual values from A.M. is always zero, i.e., sponding to the mid-points of the class intervals.
n
∑ fi ( xi − x )
= 0
Some Points About Geometric Mean
i =1

where fi is the frequency of xi (1 ≤ i ≤ n) • It is based on all items of the series.
2. In a statistical data, the sum of squares of the devia- • It is most suitable for constructing index number, aver-
tions of individual values from A.M. is least, i.e., age ratios, percentages etc.
• G.M. cannot be calculated if the size of any of the
n
items is zero or negative.
∑ fi ( xi − x )2 is least.
i =1

3. If each of the n given observations is doubled, then HARMONIC MEAN
their mean is doubled.
The harmonic mean of n observations x1, x2, ...., xn is
4. If x is the mean of x1, x2, ..... , xn, then the mean of
defined as:
ax1, ax2, ....., axn where a is any number different from
zero, is a x . n
H.M. =
1 1 1
Some Points About Arithmetic Mean + + ...... +
x1 x2 xn
• Of all types of averages, the arithmetic mean is most If x1, x2, x3, ...., xn are n observations which occur with fre-
commonly used average. quencies f1, f2, ..., fn respectively, then, their H.M. is given
• It is based upon all observations. by
• If the number of observations is very large, it is more
n
accurate and more reliable basis for comparison.
∑ fi
i =1
H.M. =
GEOMETRIC MEAN n
⎛ f ⎞
∑ ⎜⎝ xi ⎟⎠
If x1, x2, x3, ..., xn are n observations, none of them being i =1 i
zero, then their geometric mean is defined as Some Points About H.M.
1
G.M. = ( x1 ⋅ x2 ⋅ x3 ... xn )n • It is based on all item of the series.
• This is useful in problems related with rates, ratios,
⎛ log x1 + log x2 + ....... + log xn ⎞ time etc.
G.M. = antilog ⎜ ⎟⎠
⎝ n • A.M. ≥ G.M. ≥ H.M. and also (G.M.)2 = (A.M.)
(H.M.).
In the case of a grouped data, geometric mean of n • A.M. gives more weightage to larger values whereas
­observations x1, x2, ......., xn, is given by G.M. and H.M. give more weightage to smaller values.
Measures of Central Tendency and Dispersion  14.3

Relation Between AM, GM and HM Solution: (A)


Here x = 6, x1 = 8, n1 = 40, n2 = 60. Assuming that
The arithmetic mean (AM), geometric mean (GM) and
the total output is 100, we are required to find out x2,
harmonic mean (HM) for a given set of observations are
we know that
related as under:
n x + n2 x2 40 × 8 + 60 × x2
AM ≥ GM ≥ HM x = 1 1 =
n1 + n2 40 + 60
Equality sign holds only when all the observations are 320 + 60 x2
equal. ⇒ 6 =
100
600 − 320 280 14
⇒ x2 = = = = 4.66.
SOLVED EXAMPLES 60 60 3
Thus, the average profit in womens shirt is 4.66 % of
1. Mean of 25 observations was found to be 78.4. But sales or Re 0.0466 per sale rupee.
later on it was found that 96 was misread as 69. The
correct mean is 4. The weighted mean of the first n natural numbers if
their weights are the same as the numbers, is
(A) 79.48 (B) 76.54
(C) 81.32 (D) 78.4 n +1 2n + 1
(A)  (B) 
Solution: (A) 3 3
We know that the mean is given by 2n − 1
(C)  (D)  None of these
3
∑x
x =
n
or ∑ x = nx Solution: (B)
Here the numbers are 1, 2, 3, ....... , n and their weights
Here x = 78.4, n = 25 also are respectively 1, 2, 3, ....... , n.
\ Sx = 25 × 78.4 = 1960 So weighted

But this Sx is incorrect as 96 was misread as 69. Aw =


∑ wx =
1.1 + 2.2 + 3.3 + ...... + n.n
Correct Sx = 1960 – 69 + 96 = 1987 ∑w 1 + 2 + 3 + ...... + n
1987
\ Correct mean = = 79.48. 12 + 22 + 32 + ..... + n2
25 =
1 + 2 + 3 + ..... + n
2. The mean height of 15 students is 154 cm. It is discov-
ered later on that while calculating the mean the read- n( n + 1) ( 2n + 1) 2n + 1
= =
ing 175 cm was wrongly read as 145 cm. The correct n( n + 1) 3
6.
mean height is 2
(A)  145 cm (B)  170 cm 5. If the frequencies of first four numbers out of 1, 2, 4,
(C)  156 cm (D)  None of these 6, 8 are 2, 3, 3, 2 respectively, then the frequency of 8
Solution: (C) if their A.M. is 5, is
Total height of 15 students = Sx = 154 × 15 = 2310 cm. (A) 4 (B) 5
It was found that 175 cm was wrongly read as 145 (C)  6 (D)  None of these
Correct sum = 2310 – 145 + 175 = 2340 cm. Solution: (C)
2340 Here mean A = 5.
Correct mean = = 156 cm.
15 Let the frequency of 8 be x. Then by the formula
3. A firm of readymade garments make both men’s and
A =
∑ xf
women’s shirts. Its profit average is 6% of sales. Its
profits in men’s shirts average 8% of sales and women’s
∑f
shirts comprise 60% of output. The average profit per 1.2 + 2.3 + 4.3 + 6.2 + 8. x 32 + 8 x
sales rupee in women’s shirts is 5 = =
2+3+3+ 2+ x 10 + x
(A) 0.0466 (B) 0.0166
(C)  0.0666 (D)  None of these or 18 = 3x; \ x = 6.
14.4  Chapter 14

6. The mean weight of 120 students in the second year 8. In a family, there are 8 men, 7 women and 5 children
class of a college is 56 kg. If the mean weights of the whose mean ages separately are respectively 24, 20
boys and that of the girls in the class are 60 kg and and 6 years. The mean age of the family is
50 kg respectively, then the number of boys and girls (A)  17.1 years (B)  18.1 years
separately in the class are (C)  19.1 years (D)  None of these
(A)  72, 64 (B)  38, 64
Solution: (B)
(C)  72, 48 (D)  None of these
Here we have three collections for which A1 = 24, n1 = 8;
Solution: (C) A2 = 20, n2 = 7 and A3 = 6, n3 = 5. Their combined
We know that the combined mean mean is the required mean.
n A + n2 A2 By the formula
A = 1 1 (1)
n1 + n2
n1 A1 + n2 A2 + n3 A3
Here A1 = mean weight of boys = 60 kg. A=
n1 + n2 + n3
A2 = mean weight of girls = 50 kg.
8 × 24 + 7 × 20 + 5 × 6
A = combined mean = 56 kg. \ A=
8+7+3
and n1 + n2 = 120 (2)
192 + 140 + 30 362
So, from (1) and (2), = = = 18.1
20 20
n1 ⋅ 60 + n2 ⋅ 50 \ The mean age of the family = 18.1 years.
56 = ;
120
9. The mean of 100 items is 50 and their S.D. is 4. The
\ 56 × 120 = n1 ⋅ 60 + (120 – n1) 50;
sum of all the items and also the sum of the squares of
\ 120 (56 – 50) = 10n1; the items is
\ n1 = 72, n2 = 48. (A)  5000, 251600 (B)  4000, 251600
Thus, the number of boys = 72 and the number of girls (C)  5000, 261600 (D)  None of these
= 48. Solution: (A)
7. The mean of 10 numbers is 12.5; the mean of the first Here n = 100, A = 50, σ = 4.
six is 15 and the last five is 10. The sixth number is ∑ x ; \ x = nA = 100 × 50 = 5,000.
(A) 15 (B) 12 Now, A =
n

(C)  18 (D)  None of these Again, from the formula,
Solution: (A)
∑ x 2 , we get
Let the mean of the last four be A2. Then by the for- σ 2 + A2 =
n
∑ x2 = n(σ 2 + A2)
mula for combined mean,
6 × 15 + 4 × A2
12.5 = ; \ ∑ x2 = n(σ 2 + A2) = 100(16 + 2500) = 2,51,600
6+4
or 125 = 90 + 4 A2; 10. If the mean of the set of number x1, x2, ........, xn is x ,
then the mean of the numbers xi + 2i, 1 ≤ i ≤ n is
35
\ A2 =
4 (A)  x + 2n (B) 
x +n+1
Let the sixth number = x ; then taking the sixth number (C)  x + 2 (D)  x + n.
as a collection, the combined mean of this collection
and the collection of the last four is 10, by question. Solution: (B)
\ By definition of combined mean n

35 ∑ xi
1× x + 4 × x =
i =1

10 = 4 ; n
1+ 4
n
\ 50 = x + 35; \ x = 15. ⇒ ∑ xi = nx
i =1
\ Sixth number = 15.
Measures of Central Tendency and Dispersion  14.5

n n New Sum = 4900 – 110 + 210 = 5000


∑ ( xi + 2i) ∑ xi + 2 (1 + 2 + .... + n) \ New mean =
5000
= 50
i =1 i =1
\ = 100
n n
n ( n + 1) 14. The mean of n items is X . If the first item is increased
nx + 2 by 1, second by 2 and so on, then the new mean is
= 2 = x + (n + 1).
n n
(A)  X + n (B)  X +
11. The A.M. of n observations is M. If the sum of n – 4 2
n +1
observations is a, then the mean of remaining 4 obser- (C)  X + (D)  None of these
2
vations is
Solution: (C)
nM – a nM – a
(A)  (B)  Let x1, x2, ...xn be n items.
4 2
1
nM – a Then X = Σxi .
(C)  (D)  nM + a n
4
Let y1 = x1 + 1, y2 = x2 + 2
Solution: (A)
Let the mean of the remaining 4 observations be X1 . y3 = x3 + 3, ..... , yn = xn + n
a + 4 X1 nM − a Then the mean of the new series is
Then, M = ⇒ X1 = .
( n − 4) + 4 4 1 1 1 1
Σy = Σ( xi + i ) = Σxi + (1 + 2 + 3 + ..... + n)
12. The weighted mean of first n natural numbers whose n i n n n
weights are equal to the squares of corresponding 1 n( n + 1) n +1
numbers is = X+ ⋅ = X+ .
n 2 2
n +1 3n ( n + 1)
(A)  (B)  15. The number of observations in a group is 40. If the
2 2 ( 2n + 1) average of first 10 is 4.5 and that of the remaining 30
( n + 1) ( 2n + 1) n ( n + 1) is 3.5, then the average of the whole group is
(C)  (D) 
6 2 15 1
(A)  (B) 
4 5
Solution: (B) (C) 8 (D) 4
1.12 + 2.22 + ...... + n.n2
Weighted Mean = Solution: (A)
12 + 22 + ....... + n2 x1 + x2 + ..... + x10
n ( n + 1) n ( n + 1) = 4.5
10
Σn3 2 2
= = ⇒ x1 + x2 + ..... + x10 = 45
Σn 2 n ( n + 1) ( 2 n + 1)
6 x11 + x12 + ..... + x40
Also, = 3.5
3n ( n + 1) 30
= .
2 ( 2n + 1) ⇒ x11 + x12 + ..... + x40 = 105
13. Mean of 100 items is 49. It was discovered that three
items which should have been 60, 70, 80 were wrongly \ x1 + x2 + ..... + x40 = 150
read as 40, 20, 50 respectively. The correct mean is x1 + x2 + ..... + x40 150 15
1 \ = =
(A) 48 (B) 82 40 40 4
2
16. A person purchases one kg of tomatoes from each of
(C) 50 (D) 80
the 4 places at the rate of 1 kg, 2 kg, 3 kg, 4 kg per
Solution: (C) rupee respectively. On the average he has purchased
Sum of 100 items = 49 × 100 = 4900 x kg of tomatoes per rupee, then the value of x is
Sum of items added = 60 + 70 + 80 = 210 (A) 2 (B) 2.5
Sum of items replaced = 40 + 20 + 50 = 110 (C)  1.92 (D)  None of these
14.6  Chapter 14

Solution: (C) 19. The mean of the values 0, 1, 2, 3, ... , n with the
Since we are given rate per rupee, harmonic mean will ­corresponding weights nC0, nC1,..., nCn respectively is
give the correct answer 2n 2n +1
4 4 × 12 48 (A)  (B) 
H.M. = = = ( n + 1) n( n + 1)
1 1 1 1 12 + 6 + 4 + 3 25
+ + + n +1 n
1 2 3 4 (C)  (D) 
= 1.92 kg per rupee. 2 2
Solution: (D)
17. The A.M. of 2nC0, 2nC2, 2nC4,..., 2nC2n is n
C1 + 2 ⋅ nC2 + 3 nC3 + ... + n nCn
n 2n
2 2 n
(A)  (B)  C0 + nC1 + nC2 + ... + nCn
( n + 1) ( n + 1)
n ⋅ 2n−1 n
22 n −1 2n −1 = n
=
(C)  (D)  2 2
( n + 1) ( n + 1)
20. In a factory, workers work in three shifts say shift
Solution: (C) 1, shift 2 and shift 3 and they get wages in the ratio
(1 + x)2n = 2nC0 + 2nC1x + 2nC2x2 + 2nC3x3 +, ..., 4 : 5 : 6 depending on the shift 1, 2 and 3 respectively.
+ 2nC2nx2n Number of workers in the shifts are in the ratio 3 : 2
: 1. If total number of workers working is 1500 and
Put x = – 1 wages per worker in Ist shift is Rs 400. Then mean
2n
C0 – 2nC1 + 2nC2 – 2nC3 +, ..., + 2nC2n = 0 (1) wage of a worker is
(A)  Rs. 467 (B)  Rs. 500
Now put x = 1
(C)  Rs. 600 (D)  Rs. 400
2n
C0 + 2nC1 + 2nC2 + 2nC3 +, ..., + 2nC2n = 22n(2)
Solution: (A)
Adding (1) and (2), we get Workers in Ist shift = 750
2n
C0 + 2nC2 +, ..., + 2nC2n = 22n–1 Wages in Ist shift = Rs. 400
Workers is IInd shift = 500
22 n −1 Wages in IInd shift = Rs. 500
A.M. of 2nC0, 2nC2 +, ..., + 2nC2n =
( n + 1) Workers in IIIrd shift = 250
Wages in IIIrd shift = Rs. 600
18. The A.M. of 2n+1C0, 2n+1C1, 2n+1C2, ... , 2n+1Cn is
750 × 400 + 500 × 500 + 250 × 600
\ Mean =
2n 2n 1500
(A)  (B) 
n n +1 Rs. 467 per worker
22n 22n 21. If a variable takes values 1, 2, 3, ... , n with frequencies
(C)  (D)  12, 22 , ... , n2, then the mean is
n ( n + 1)
Σn3
Solution: (D) (A) Sn (B) 
2n+1
Σn 2
C0 + 2n+1C1 + 2n+1C2 +, ..., Σn3
(C)  (D)  None of these
+ 2n+1C2n + 2n+1C2n+1 = 22n+1 Σn
Solution: (B)
2n+1
Now C0 = 2n+1C2n+1, 2n+1C1 = 2n+1C2n,... 1.12 + 2 . 22 + ... + n . n2 Σn3
2n+1 =
Cr = 2n+1C2n–r+1 12 + 22 + ... + n2 Σn 2
So sum of first (n + 1) terms = Sun of last (n + 1) terms 22. Mean of n items is x. If these n items are increased by
or 2n+1
C0 + 2n+1
C1 + 2n+1
C2 +, ..., + 2n+1
Cn = 2 2n 12, 22, 32, ..., n2 successively, then mean gets increased
2 n +1 2 n +1 2 n +1 2 n +1
by
C0 + C1 + C2 +, ..., + Cn ( n + 1) ( 2n + 1) n( n + 1) ( 2n + 1)
or (A)  (B) 
n +1
6 6
2
22n n
 = (C)  (D)  Remains same
( n + 1) 2
Measures of Central Tendency and Dispersion  14.7

Solution: (A) 3. The median value is given by the formula


x1 + x2 + x3 + ... + ... + xn ⎛ ⎛ n⎞ ⎞
x =
n ⎜ ⎜⎝ 2 ⎟⎠ − c f ⎟
Median = l + ⎜
⎟ × h, where
x1 + 1 + x2 + 22 + x3 + 32 + ... + xn + n2
2
⎜ f ⎟
 x′ = ⎜⎝ ⎟⎠
n
x1 + x2 + ... + xn 12 + 22 + ... + n2
⇒ x′ = + l = lower limit of the median class
n n n = total frequency
( n + 1) ( 2n + 1) f = frequency of the median class
⇒ x′ = x +
6 h = width of the median class
cf = cumulative frequency of the class preceding the
MEDIAN median class

Median is the middle most or the central value of the variate Some Points About Median
in a set of observations, when the observations are arranged
• It is an appropriate average in dealing with qualitative
either in ascending or in descending order of their magni-
data, like intelligence, wealth etc.
tudes. It divides the arranged series in two equal parts.
• The sum of the deviations of the items from median,
Calculation of Median ignoring algebraic signs, is less than the sum from any
other point.
Median of an Individual Series
Let n be the number of observations. QUARTILES, DECILES AND PERCENTILES
1. Arrange the data in ascending or descending order.
2. (A) If n is odd, then Quartile
1 Just as the median divides a set of observations (when
Median = value of the (n + 1)th observation
2 arranged in ascending or descending order of magnitudes),
(B) If n is even, then into two equal parts, similarly Quartile divides the obser-
⎛ n⎞ ⎛n ⎞
Median = mean of the ⎜ ⎟ th and ⎜ + 1⎟ th vations into four equal parts. The value of the item midway,
⎝ 2⎠ ⎝2 ⎠
observation. between the first item and the median is known as first or
lower quartile and is denoted be Q1. The value of the item
midway between the last item and the median is known
Median of a Discrete Series Third or Upper Quartile and is denoted Q3. The median is
1. Arrange the values of the variate in ascending or known as the Second Quartile and is denoted by Q2. The
descending order. methods for finding the values of Q1 and Q3 are similar to
2. Prepare a cumulative frequency table. that of the median. In the case of ungrouped data, when
3. (A) If n is odd, then arranged in ascending or descending order of magnitudes
Q1, Q3 can be obtained as follows:
⎛ n + 1⎞
Median = size of the ⎜
th term.
⎝ 2 ⎟⎠ Q1 =
n +1
th item, Q3 =
3( n + 1)
th item.
4 4

(B) If n is even, then
For a frequency distribution, Q1 and Q3 are given by,
⎛ ⎛ n⎞ ⎛ n ⎞ ⎞
⎜ ⎜⎝ 2 ⎟⎠ + ⎜⎝ 2 + 1⎟⎠ ⎟ [( n / 4) − C f ]
Median = size of the ⎜
⎟ th term. Q1 = l + × h,
⎜ 2 ⎟ f
⎜⎝ ⎟⎠
[(3n / 4) − C f ]
Q3 = l + × h,
f
Median of a Continuous Series
where l = lower limit of the class in which a particular
1. Prepare the cumulative frequency table.
⎛ n⎞ quartile lies,
2. Find the median class, i.e., the class in which the ⎜ ⎟ f =  frequency of the class-interval in which a
⎝ 2⎠
th observation lies. ­particular quartile lies,
14.8  Chapter 14

i = class-interval of the class in which a particular Solution: (A)


quartile lies, 7 5 1
Arrange the data as a – , a – 3, a – , a – 2, a – ,
cf = cumulatively frequency of the class preceding 2 2 2
1
the class in which the particular quartile lies. a + , a + 4, a + 5.
2
[( nh / 4) − c f ] 1 5
In general, Qi = l + × h, i = 1, 2, 3, 4 α −2+α − 2α −
f Median = 2 = 2 = a – 5.
2 2 4
Decile
24. Median of 2nC0, 2nC1, 2nC2, 2nC3,..., 2nCn (when n is
The value of the variable which divides the series, when even) is
arranged in ascending or descending order, into 10 equal (A) 2nCn/2 (B)  2n
C(n+1)/2
parts is called decile. There are 9 deciles denoted by D1, D2 2n
(C)  C(n–1)/2 (D)  None of these
... D9. When the series is ungrouped the deciles are calcu-
lated as follows: Solution: (A)
2n
n×h C0, 2nC1, 2nC2, ... , 2nCn is odd number of binomial
Di = , i = 1, 2, ..., 9 coefficients (when n is even) and middle binomial
10
coefficient is 2nCn/2.
When the data is classified or grouped,
25. Median of 2nC0, 2nC1, 2nC2, 2nCn (when n is odd) is
[( nh / 10) − c f ]
Di = l + ×h
f (A) 
1 2n
2
(
C( n −1)/ 2 + 2n
C( n +1)/ 2 )
where symbols have their usual meaning.
(B) 2nCn/2
Percentile (C) 2nCn
(D)  None of these
The value of the variable which divides the series, when
arranged in ascending or descending order, into 100 equal Solution: (A)
2n
parts is called percentile. There are 99 percentiles denoted C0, 2nC1, 2nC2, ... , 2nCn is even number of bino-
by P1, P2, P3, P4, ..., P99 respectively. When the series is mial coefficients (when n is odd), and then mid-
ungrouped the percentiles are calculated by the following dle terms are 2nCn–1/2 and 2nCn+1/2. So median is
2n 2n
formula: Cn −1/ 2 + Cn +1/ 2
.
n×h 2
Pi = , h = 1, 2, ..., 99. 5
100 26. If a variable takes the discrete values a + 4, α −
When the data is classified or grouped, the percentiles are 2
7 1 1
calculated by the formula α − , a – 3, a – 2, α + , α − , a + 5 (a > 0),
2 2 2
[( nh / 100) − c f ] then the median is
Pi = l + × h, i = 1, 2, ...., 99.
f 1 5
(A)  α − (B)  α−
where symbols have their usual meanings. 2 4
5
(C) a – 2 (D)  α +
4
SOLVED EXAMPLES Solution: (B)
7 1⎛ 1⎞
23. If a variable takes the discrete values a + 4, a – , Median = ⎜ α − 2 + α − ⎟
2 2⎝ 2⎠
5 1 1
a ­ – ←, a – 3, a – 2, a + , a – , a +5 (a > 0), 5
2 2 2 =α−
4
then the median is
5 1 MODE
(A) a – (B)  a–
4 2
Mode is that value in a series which occurs most frequently.
5 In a frequency distribution, mode is that variate
(C) a – 2 (D)  a+
4 which has the maximum frequency.
Measures of Central Tendency and Dispersion  14.9

Computation of Mode The commonly used measures of dispersion are:


Mode of Individual Series 1. Range
2. Quartile Deviation or Semi-interquartile range
In the case of individual series, the value which is repeated
3. Mean Deviation
maximum number of times is the mode of the series.
4. Standard Deviation
Mode of Discrete Series
Range
In the case of discrete frequency distribution, mode is
the value of the variate corresponding to the maximum It is the difference between the greatest and the smallest
frequency. observations of the distribution.
If L is the largest and S is the smallest observation in
Mode of Continuous Series a distribution, then its Range = L – S. Also,
1. Find the modal class, i.e., the class which has maxi- L−S
mum frequency. The modal class can be determined Coefficient of range = .
L+S
either by inspection or with the help of grouping table.
2. The mode is given by the formula Quartile deviation
f m − f m −1 Quartile deviation or semi-interquartile range is given by
Mode = l +
× h, 1
2 f m − f m −1 − f m + 1 Q.D. = (Q3 – Q1)
2
where l = the lower limit of the modal class
h = the width of the modal class Q − Q1
Coefficient of Q.D. = 3
fm – 1 = 
the frequency of the class preceding Q3 + Q1

modal class
fm = the frequency of the modal class
Mean deviation
fm +1 = the frequency of the class succeeding For a frequency distribution, the mean deviation from an
modal class average (median, or arithmetic mean) is given by,
n
In case, the modal value lies in a class other than the one
containing maximum frequency, we take the help of the fol- ∑ fi xi − x
i =1
lowing formula; M.D. = n
f m +1
Mode = l =
f m −1 + f m + 1
×h , ∑ fi
i =1

where symbols have usual meaning. Mean deviation
Coefficient of M.D. =
Some Points about Mode Corresponding average

• It is not based on all items of the series


Standard deviation
• It is not necessary that a distribution has unique mode. The standard deviation of a statistical data is defined as the
• As compared to other averages mode is affected to a positive square root of the squared deviations of observa-
large extent by fluctuations of sampling. tions from the A.M. of the series under consideration.
• It is not suitable in a case where the relative impor-
1. Standard deviation (also denoted by σ ) for ungrouped
tance of items have to be considered.
set of observations is given by
SYMMETRIC DISTRIBUTION n

A distribution is a symmetric distribution if the values of


∑ (xi − x )2
i =1
mean, mode and median coincide. In a symmetric distri- σ=
n
bution frequencies are symmetrically distributed on both
sides of the centre point of the frequency curve. 2. Standard deviation for frequency distribution is given
by,
Measures of Dispersion n

The degree to which numerical values in the set of ­values


∑ fi (xi − x )2
i =1
tend to spread about an average value is called the d­ ispersion S.D. =

N
or variation.
14.10  Chapter 14

where fi is the frequency of xi (1 ≤ i ≤ n).


σ
C.V. = × 100
When the values of the variable are given in the form of x
classes, then their respective mid-points are taken as the
The coefficient of variation is also represented as percentage.
values of the variable.
The square of S.D. is called the variance of the distri-
Standard Deviation of n Natural Numbers bution and is denoted by σ 2.

1/ 2 Computation of Standard Deviation


⎛1 ⎞
σ = ⎜ ( n2 − 1)⎟
⎝ 12 ⎠ Direct Method
Standard deviation shows the limits of variability by which
2
the individual observation in a distribution will vary from
σ=
∑ x2 ⎛ ∑ x⎞
−⎜
the mean. For a symmetrical distribution with mean x , the ⎟
n ⎝ n ⎠
following area relationship holds good:
x ± σ covers 68.27 % observations. Short-cut Method
x ± 2σ covers 95.45 % observations.
2
x ± 3σ covers 99.73 % observations. σ=
∑ d2 ⎛ ∑d⎞
−⎜ ⎟ , for ungrouped data
n ⎝ n ⎠
These limits are illustrated by the following curve known
as Normal Curve. where A is assumed mean and d = x – A.
2

σ=
∑ fd 2 ⎛ ∑ fd ⎞
−⎜ ⎟ , for grouped data
N ⎝ N ⎠
68.27%
where N = ∑f.
95.44%
99.73%
x – 3σ x – 2σ x – 1σ x x + 1σ x + 2σ x + 3σ Step-deviation Method
FIG. 14.1 2

σ =h
∑ fd ′ 2 ⎛ ∑ fd ′ ⎞
−⎜
x−A
⎟ ; d′ =
Empirical relationships N ⎝ N ⎠ h
If the data is moderately non-symmetrical, then the follow-
ing empirical relationships hold: Combined Standard Deviation
4 Let A1 and A2 be two series having n1 and n2 observations
Mean deviation = σ
5 respectively. Let their A.M. be x1 and x2 , and standard
2
Semi-Inter-quartile range = σ deviations be σ1 and σ2. Then the combined standard devi-
3 2 ation σ or σ12 of A1 and A2 is given by
Probable error of standard deviation = σ
3
= Semi-inter-quartile range. n1σ12 + n2σ 22 + n1d12 + n2 d22
5 σ12 or σ =
Quartile deviation = M.D. n1 + n2
6
From these relationships, we have
n1 (σ12 + d12 ) + n2 (σ 22 + d22 )
4 S.D. = 5 M.D. = 6 Q.D =
n1 + n2

Coefficient of S.D. (C.V.) where d1 = ( x1 − x12 ), d2 = ( x2 − x12 ),
For comparing two or more series for variability, the ­relative n1 x1 + n2 x2
measure, called coefficient of variation (C.V.) is used. This and x12 = is the combined mean.
measure is defined as n1 + n2
Measures of Central Tendency and Dispersion  14.11

29. The mean deviation from the mean for the set of
I M P O R TA N T P O I N T S ­observations – 1, 0, 4 is
(A)  less than 3 (B)  less than 4
■ Quartile deviation is less affected by extreme values of
the series.
(C)  greater than 2.5 (D)  greater than 4.9
■ Mean deviation is based on all the items of series. It is Solution: (A and B)
therefore more representative than the range or quartile −1 + 0 + 4
deviation. x = = 1.
3
■ Mean deviation from the median is less than that mea-
1
sured from any other mean. \ Mean Deviation = (| – 1 – 1| + | 0 – 1 | + | 4 – 1 | )
2
■ Standard deviation ≤ Range i.e., variance ≤ (Range) .
3
=2
n2 − 1
■ S.D. of first n natural numbers is .
12 30. If the S.D of a set of observations is 4 and if each
observation is divided by 4, the S.D of the new set of
observations will be
(A) 4 (B) 3
SOLVED EXAMPLES (C) 2 (D) 1
Solution: (D)
27. The coefficient of variation of two series are 58% and
We know that if y = x/h when σ y = σx /| h |.
69%. If their standard deviations are 21.2 and 15.6,
\ The S.D. of new set of observations will be 4/4 = 1.
then their A.Ms are
(A)  36.6, 22.6 (B)  34.8, 22.6 31. A sample of 35 observations has the mean 80 and S.D.
(C)  36.6, 24.4 (D)  None of these as 4. A second sample of 65 observations from the
same population has mean 70 and S.D. 3. The S.D. of
Solution: (A) the combined sample is
We know that (A) 5.85 (B) 5.58
σ × 100 (C)  3.42 (D)  None of these
C.V. =
x Solution: (A)
σ
or x = × 100
C.V. Here n1 = 35, x1 = 80, σ1 = 4,
21.2 × 100
\ Mean of first series = = 36.6 n2 = 65, x2 = 70, σ 2 = 3.
58
35 × 80 + 65 × 70
15.6 × 100 \ x12 = = 73.5.
Mean of second series = = 22.6 35 + 65
69
σ12 = ⎡ 35(16 + 42 × 25) + 65(9 + 12 × 25) ⎤
28. Mean deviation of the series a, a + d, a + 2d, a + 2nd ⎢ ⎥
from its mean is ⎣ 100 ⎦
( n + 1) d nd = 34.21 = 5.85
(A)  (B) 
( 2n + 1) 2n + 1
32. If μ is the mean of a distribution, then
n ( n + 1) d ( 2n + 1) d ∑ fi ( yi − μ ) is equal to
(C)  (D) 
( 2n + 1) n ( n + 1)
(A) M.D. (B) S.D.
Solution: (C) (C)  0 (D)  None of these
2n + 1
.( a + a + 2nd ) Solution: (C)
x = 2 = a + nd Σfi yi
( 2n + 1) We have, μ=
Σf i

Σ x − x = 2d (1 + 2 + ......+ n) = n (n + 1) d
⇒ Σfi yi – S fi μ = 0
n ( n + 1)d
\ M.D. =
2n + 1 ⇒ S fi (yi – μ) = 0
14.12  Chapter 14

33. The means of five observations is 4 and their variance 34. If 25 % of the items are less than 20 and 25 % are more
is 5.2. If three of these observations are 1, 2, and 6, then 40, the quartile deviation is
then the other two are (A) 20 (B) 30
(A)  2 and 9 (B)  3 and 8 (C) 40 (D) 10
(C)  4 and 7 (D)  5 and 6
Solution: (D)
Solution: (C) 40 − 20
Q.D. = = 10.
x1 = 4, N = 5 2

35. The sum of squares of deviations for 10 observations
Σ(x − x ) 2
and = 5.2 taken from mean 50 is 250. The coefficient of variation
N is
⇒ Σ(x - x ) 2 = (5.2) 5 (A) 10 % (B) 40 %
(C) 50 % (D)  None of these
\ Σ(x - x ) 2 = 26
Solution: (A)
2 2 2 Co-efficient of variation
\ (1 – 4) + (2 – 4) + (6 – 4) + 
(a – 4)2 + (b – 4)2 = 26 σ σ
= × 100 = × 100 ( x = 50) = 2σ
x 50
where a, b are the other two observatios.
\ 9 + 4 + 4 + (a – 4)2 + (b – 4)2 = 26 Σ ( xi − 50) 2 250
Also, σ = = = 25 = 5
n 10
\ (a – 4)2 + (b – 4)4 = 9 \ Co.efficient of variation = 2 × 5 = 10%.
1+ 2 + 6 + α + β
Also, = 4
5
\ a + b = 20 – 9 = 11
Clearly 4, 7 only satisfy the above equation in a, b.
Hence reqd. numbers are 4, 7.
Measures of Central Tendency and Dispersion  14.13

PRACTICE EXERCISES

Single Option Correct Type

1. The average of n numbers x1, x2, x3,..., xn is M. If xn is 9. If the standard deviation of n observations x1, x2,...,
replaced by x′, then new average is xn is 4 and another set of n observations y1, y2,..., yn
nM − x + x ′ is 3. The standard deviation of n observations x1 – y1,
(A) M – xn + x′ (B)  n x2 – y2,..., xn – yn is
n
( n − 1) M + x ′ M − xn + x ′ 2
(C)  (D)  (A) 1 (B) 
m n 3
2. The standard deviation of 25 numbers is 40. If each of (C) 5 (D) Data insufficient
the numbers is increased by 5, then the new standard
1 n
deviation will be 10. Let r be the range and S 2 = ∑ ( x − x)2 be the
n − 1i =1 i
(A) 40 (B) 45
21 S.D. of a set of observations x1, x2,..., xn, then
(C) 40 + (D)  None of these
25 n n
3. If M.D. is 12, the value of S.D. will be (A) S ≤ r S= r
(B) 
n −1 n −1

(A) 15 (B) 12
(C)  24 (D)  None of these n
(C) S ≥ r (D)  None of these
n −1
4. The mean weight of 9 items is 51. If one more item is
added to the series the mean becomes 16. The value of 11. The A.M. of n numbers of a series is x . If the sum of
the 10th item is the first (n – 1) term is k, them the nth number is
(A) 35 (B) 30 (A)  x − k (B)  nx − k
(C) 25 (D) 20 (B)  x − nk (D)  nx − nk
5. The mean and S.D. of the marks of 200 candidates
12. If a variable takes values 0, 1, 2,..., n with frequencies
were found to be 40 and 15 respectively. Later, it was
n n( n − 1) n − 2 2
discovered that a score of 40 was wrongly read as 50. qn, q n −1 p , q p ,..., pn, where p + q = 1,
The correct mean and S.D. respectively are 1 1. 2
then the mean is
(A)  14.98, 39.95 (B)  39.95, 14.98

PRACTICE EXERCISES
(C)  39. 95, 224.5 (D)  None of these (A) np (B)  nq
(C) n(p + q) (D)  None of these
6. If Q.D. is 16, the most likely value of S.D. will be
13. The S.D. of a variate x is σ. The S.D. of the variate
(A) 24 (B) 42
ax + b
(C)  10 (D)  None of these where a, b, c are constants, is
c
7. If a variable x takes values 0, 1, 2, ..., n with fre­ ⎛ a⎞ a
quencies proportional to the binomial coefficients nC0, (A)  ⎜ ⎟ σ (B)  σ
n ⎝ c⎠ c
C1, nC2,..., nCn, then the Var (x) is
⎛ a2 ⎞
n2 − 1 n (C)  ⎜ 2 ⎟ σ (D)  None of these
(A)  (B)  ⎝c ⎠
12 2
n 14. Consider any set of observations x1, x2, x3,..., x101; it
(C)  (D)  None of these being given that x1 < x2 < x3 < ... < x100 < x101; then
4
the mean deviation of this set of observations about a
8. The sum of squares of deviations for 10 observations
point k is minimum when k equals
taken from mean 50 is 250. The coefficient of variation
is (A) x1 (B)  x51
(A) 50% (B)  10% x + x2 + ... + x101
(C)  1 (D)  x50
(C) 40% (D)  None of these 101
14.14  Chapter 14

15. The mean of the numbers 23. The mean weight per student in a group of seven stu-
50
C0 C250
C4 50
C50 50 dents is 55 kg If the individual weights of 6 students
, , ..., equals are 52, 58, 55, 53, 56 and 54; then weight of the sev-
1 3 5 51
enth student is
250 249
(A)  (B)  (A)  55 kg (B)  60 kg
51 51 (C)  57 kg (D)  50 kg
49
2 24. If the mean of a set of observations x1, x2, x3,..., xn is
(C)  (D)  None of these
39 × 17 x , then mean of observations xi + 3i ∀ i = 1, 2, 3,... n
equals
16. The standard deviation of a distribution is 30 and each
3( n + 1)
item is raised by 3, then new S.D. is (A)  x + 3(n + 1) (B)  x +
2
(A) 32 (B) 28
(C)  27 (D)  None of these n +1
(C)  x + (D)  None of these
2n
17. For three numbers a, b, c product of the average of the
25. The weighted mean of the square of 1st n natural
1 1 1
numbers a2, b2, c2 and 2 , 2 , 2 cannot be less than numbers whose weights are corresponding numbers,
a b c equals
(A) 1 (B) 3 ( n + 1) ( 2n + 1) n( n + 1)
(C)  9 (D)  None of these (A)  (B) 
2 2
18. The variance of a, b and g is 9, then variance of 5a, 5b n +1
and 5g is (C)  (D)  None of these
2
(A) 45 (B) 9/5 (C) 5/9 (D) 225
26. If the variate of a distribution takes the values 1, 2,
19. Mean of the numbers 1, 2, 3,..., n with respective 3, ...n with frequencies n, n – 1, n – 2, ... 3, 2, 1, then
weights 12 + 1, 22 + 2, 32 + 3,..., n2 + n is mean value of the distribution is
3n( n + 1) 2n + 1 n( n + 2) n( n + 1) ( n + 2)
(A)  (B)  (A)  (B) 
2( 2n + 1) 3 3 6
3n + 1 3n + 1
(C)  (D)  n+2 ( n + 1) ) ( n + 2)
4 2 (C)  (D) 
3 6
20. The G.M. of the number 3, 32, 33,..., 33n is 27. The means of five observations is 4 and their variance
n 3n
PRACTICE EXERCISES

is 5.2. If three of these observations are 1, 2 and 6, then


(A) 3 2 (B) 
32 the other two are
3n+1 n+1
2 (A)  2 and 9 (B)  3 and 8
(C) 3 32
(D) 
(C)  4 and 7 (D)  5 and 6
21. The reciprocal of the weighted mean of first n natural 28. If the variate takes the values 0, 2, 4, 8, ... 2n with fre-
numbers whose weights are equal to the squares of the 91 × 8
quencies nC0, nC1, nCn and if the mean is , then
corresponding numbers is 2n
n equals
2( 2n + 1) 3n( n + 1) (A) 4 (B) 6
(A)  (B) 
3n( n + 1) n( 2n + 1) (C)  5 (D)  None of these
3n( n + 1) 29. The mean of n items is x . If each item is successively
(C)  (D)  None of these
2n + 1 increased by 3, 32, 33,... 3n, then new mean equals

22. The A.M. of a set of 50 numbers is 38. If two numbers


3n +1 (3n − 1)
of the set, namely 55 and 45 are discarded, the A.M. of (A)  x + x +3
(B) 
the remaining set of numbers is n 2n
(A) 38.5 (B) 37.5 3n (3n − 1)
(C)  x + x +3
(D) 
(C) 36.5 (D) 36 n 2n
Measures of Central Tendency and Dispersion  14.15

30. A sequence of odd positive integers is written as 35. If the mean deviation about the median of the numbers
1 a, 2a, ..., 50a is 50, then | a | equals
3  5  7  9 (A) 5 (B) 2 (C) 3 (D) 4
11, 13, 15, 17, 19, 21, 23, 25, 27 36. Let x1, x2, ..., xn be n observations, and let x be their
arithmetic mean and σ2 be the variance
The mean of the nth row is
Statement-1: Variance of 2x1, 2x2, ..., 2xn is 4σ2.
n3 ( 2n2 + 1) n3 ( 4 n2 + 2) Statement-2: Arithmetic mean 2x1, 2x2, ..., 2xn is 4x.
(A)  (B) 
3 6 (A)  Statement-1 is false, Statement-2 is true.
n( n − 1) ( 2n − 1) n( 2n2 + 1) (B)  Statement-1 is true, statement-2 is true; state-
(C)  (D)  ment-2 is a correct explanation for Statement-1.
6 3
(C)  Statement-1 is true, statement-2 is true;
31. The arithmetic mean of a set of observation is . If each statement-2 is not a correct explanation for
observation is divided by a and then is increased by Statement-1.
10, the means of the new series is (D)  Statement-1 is true, statement-2 is false.
x x + 10 37. If the mean of a set of observations x1, x2, ..., x10 is 20
(A)  (B) 
α
α then the mean of x1 + 4, x2 + 8, x3 + 12, ..., x10 + 40 is
x + 10α (A) 34 (B) 42 (C) 38 (D) 40
(C)  (D)  α x + 10 38. The mean of the numbers a, b, 8, 5, 10 is 6 and the
α
variance is 6.80. Then which one of the following
32. The average salary of male employees in a firm was gives possible values of a and b?
Rs. 520 and that of females was Rs. 420. The mean (A) a = 0, b = 7 (B)  a = 5, b = 2
salary of all the employees was Rs. 500. The percent- (C) a = 1, b = 6 (D)  a = 3, b = 4
age of male employees is 39. If the mean deviation of number 1, 1 + d, 1 + 2d, ..., 1
(A) 80 (B) 60 (B) 40 (D) 20 + 100d from their mean is 255, then the d is equal to
(A) 10.0 (B) 20.0
33. The average weight of students in a class of 35 stu- (C) 10.1 (D) 20.2
dents is 40 kg. If the weight of the teacher be included,
1 40. Statement-1: The variance of first n even natural
the average rises by kg; the weight of the teacher is n2 − 1
2 ­numbers is
(A)  40.5 kg (B)  50 kg 4
(C)  41 kg (D)  58 kg Statement-2: The sum of first n natural numbers is
n( n + 1)

PRACTICE EXERCISES
34. An automobile driver travels from plane to a hill station and the sum of squares of first n natural
120 km distant at an average speed of 30 km per hour. 2
He then makes the return trip at an average speed of n( n + 1) ( 2n + 1)
­numbers is
25 km per hour. He covers another 120 km distance on 6
plane at an average speed of 50 km per hour. His aver- (A)  Statement-1 is true, Statement-2 is true;
age speed over the entire distance of 360 km will be Statement-2 is a correct explanation for
1 Statement-1.
30 + 25 + 50
(A)  km/h (B)  (30 ⋅ 25 ⋅ 50) 3 (B)  Statement-1 is true, Statement-2 is true;
3
Statement-2 is not a correct explanation for
3
(C)  km/h (D)  None of these Statement-1.
1 1 1 (C)  Statement-1 is true, Statement-2 is false.
+ +
30 25 50 (D)  Statement-1 is false, Statement-2 is true.

Previous Year’s Questions

41. The median of a set of 9 distinct observations is (A)  is increased by 2


20.5. If each of the largest 4 observations of the set is (B)  is decreased by 2
increased by 2, then the median of the new set (C)  is two times the original median
[2003] (D)  remains the same as that of the original set
14.16  Chapter 14

42. Let two numbers have arithmetic mean 9 and geomet- 49. The mean of the numbers a, b, 8, 5, 10 is 6 and the
ric mean 4. Then these numbers are the roots of the variance is 6.80. Then which one of the following
quadratic equation [2004] gives possible values of a and b? [2008]
(A) x2 + 18x + 16 = 0 (A) a = 0, b = 7
(B) x2 - 18x - 16 = 0 (B) a = 5, b = 2
(C) x2 + 18x - 16 = 0 (C) a = 1, b = 6
(D) x2 - 18x + 16 = 0 (D) a = 3, b = 4
50. If the mean deviation of number 1, 1 + d, 1 + 2d, ….. ,
43. Consider the following statements [2004]
1 + 100d from their mean is 255, then the d is equal to
(A)  Mode can be computed from histogram  [2009]
(B)  Median is not independent of change of scale
(A) 10.0 (B) 20.0
(C) Variance is independent of change of origin and
(C) 10.1 (D) 20.2
scale.
Which of these is/are correct? 51. For two data sets, each with size 5, the variances are
(A)  only (A) (B)  only (B) given to be 4 and 5 and the corresponding means are
(C)  only (A) and (B) (D)  (A), (B) and (C) given to be 2 and 4, respectively. The variance of the
combined data set is [2010]
44. In a series of 2n observations, half of them equal a and 11
remaining half equal -a. If the standard deviation of (A)  (B)  6
2
the observations is 2, then |a| equals [2004]
13 5
1 (C)  (D) 
(A)  (B) 
2 2 2
n 52. If the mean deviation about the median of the numbers
2
(C) 2 (D)  a, 2a . . . 50a is 50, then |a| equals [2011]
n
(A) 3 (B) 4
45. If in a frequency distribution, the mean and median are (C) 5 (D) 2
21 and 22 respectively, then its mode is approximately
 [2005] 53. Let x1, x2 . . . xn be n observations, and let x be their
(A) 22.0 (B) 20.5 arithmetic mean and σ 2 be their variance.
(C) 25.5 (D) 24.0 Statement-1: Variance of 2x1, 2x2 . . . 2xn is 4 σ 2 .
Statement-2: Arithmetic mean of 2x1, 2x2 . . . 2xn is 4
46. Let x1, x2, …, xn be n observations such that x . [2012]
∑ xi2 = 400 and ∑ xi = 80. Then a possible value of (A)  Statement-1 is false, statement-2 is true
PRACTICE EXERCISES

n among the following is [2005] (B) Statement-1 is true, statement-2 is true; statement
(A) 15 (B) 18 2 is a correct explanation for statement 1
(C) 9 (D) 12 (C) Statement-1 is true, statement-2 is true; statement
2 is not a correct explanation for statement 1
47. Suppose a population A has 100 observations 101, (D)  Statement-1 is true, statement-2 is false
102, … , 200, and another population B has 100 obser- 54. All the students of a class performed poorly in
vations 151, 152, … , 250. If VA and VB represent the Mathematics. The teacher decided to give grace marks
variances of the two populations, respectively, then of 10 to entire class. Which of the following statistical
VA measures will not change even after the grace marks
is [2006]
VB were given? [2013]
(A) 1 (B) 9/4
(C) 4/9 (D) 2/3 (A) median (B) mode
(C) variance (D) mean
48. The average marks of boys in a class is 52 and that of 55. The variance of the first 50 even natural numbers is
girls is 42. The average marks of boys and girls com-  [2014]
bined is 50. The percentage of boys in the class is 833
 [2007] (A)  (B)  833
4
(A) 40 (B) 20 437
(C) 437 (D) 
(C) 80 (D) 60 4
Measures of Central Tendency and Dispersion  14.17

56. The mean of the data set comprising of 16 observations (A) 3a 2 − 23a + 44 = 0    
(B) 3a 2 − 26 a + 55 = 0
is 16. If one of the observation valued 16 is deleted and (C) 3a 2 − 32a + 84 = 0    (D) 3a 2 − 34 a + 91 = 0
three new observations valued 3, 4 and 5 are added to
the data, then the mean of the resultant data, is 9 9
 [2015] 58. If ∑ ( xi − 5) = 9 and ∑ ( xi − 5)2 = 45, then the stan-
(A) 16.0 (B) 15.8 i =1 i =1
(C) 14.0 (D) 16.8 dard deviation of the 9 items x1, x2, …, x9 is [2018]
57. If the standard deviation of the numbers 2, 3, a and 11 (A) 9 (B) 4 (C) 2 (D) 3
is 3.5, then which of the following is true?
 [2016]

ANSWER K EYS
Single Option Correct Type
1. (B) 2. (A) 3.  (A) 4. (C) 5.  (B) 6. (A) 7.  (C) 8. (B) 9. (D) 10. (A)
11.  (B) 12.  (A) 13. (B) 14.  (B) 15. (C) 16.  (D) 17. (A) 18.  (D) 19. (C) 20.  (C)
21. (A) 22.  (B) 23. (C) 24. (B) 25. (B) 26.  (C) 27. (C) 28.  (B) 29. (B) 30.  (D)
31. (C) 32. (A) 33. (D) 34.  (C) 35.  (D) 36.  (D) 37.  (B) 38.  (D) 39.  (C) 40.  (D)

Previous Years’ Questions


41. (C) 42. (D) 43. (C) 44. (C) 45. (D) 46. (B) 47. (A) 48. (C) 49. (D) 40. (C)
51. (A) 52. (B) 53. (D) 54. (C) 55. (B) 56. (C) 57. (C) 58. (C)

PRACTICE EXERCISES
14.18  Chapter 14

HINTS AND EXPLANATIONS

Single Option Correct Type


x1 + x2 + x3 ... xn
1. M = 7. We have,
n
0 ⋅ nC0 + 1⋅ nC1 + 2 ⋅ nC2 + ... + n ⋅ nCn
nM = x1 + x2 + x3 + ... xn–1 + xn0 x = n
C0 + nC1 + nC2 + ... + nCn
i.e., nM – xn = x1 + x2 + x3 + ... xn – 1
n
nM − xn + x ′
= 1
x + x2 + x3 + ... xn −1 + x ′ ∑ r ⋅ nCr
1 n n
n ∑
n n r=0 n−1
⇒ x = n ⇒ x = r⋅ Cr −1
2 r=0 r
\ New average =
nM − xn + x ′ ∑ r⋅ n
Cr
r=0
n
The correct option is (B) n
n n n
2. If each item of a data is increased or decreased by the
⇒ x =
2 n ∑ n −1Cr −1 =
2n
2n −1 =
2
r =1
same constant, the standard deviation of the data remains
unchanged. ⎡ n ⎤
⎢∵ ∑ n −1Cr −1 = 2n −1 ⎥
The correct option is (A) ⎢⎣ r = 1 ⎥⎦

3. We know that n
5 5 ∑ r 2nCr
Q.D. = × M.D. = × 12 = 10 1 ∑ fi xi2
6 6
and ∑ fi xi2 = = r=0
3 3 n ∑ fi n
\ S.D. =
2
× Q.D. =
2
× 10 ⇒ S.D. = 15 ∑ n Cr
r=0
The correct option is (A)
n
n
4. Let the values of 9 items be x1, x2, ..., x9
1
∑ r2 r
n −1
Cr −1
Therefore, mean of x1, x2, ..., x9 is
∑ fi xi2 =
r=0

x + x2 + ... + x9 n 2n
15 = 1
9 1 n n
∑ fi xi2 = n ∑ ( r − 1 + 1)
n −1
HINTS AND EXPLANATIONS

⇒ x1 + x2 + ... + x9 = 15 × 9 = 135 ⇒ Cr −1
n 2 r=0
Let x10 be the 10th item. The mean of x1, x2, ... x9, x10 is 16.
x + x2 + ... + x9 + x10 1 n ⎡ n n ⎤
⇒ 1
10
= 16 ⇒
n
∑ fi xi2 = n ⎢ ∑ ( r − 1) n −1Cr −1 + ∑ n −1Cr −1 ⎥
2 ⎢⎣ r = 0 r=0 ⎥⎦
⇒ x1 + x2 + ... + x9 + x10 = 160 ⇒ 135 + x10 = 160
\ x10 = 160 – 135 = 25 n ⎡ n ( n − 1) ⎤
n ∑
n−2
= ⎢ ( r − 1) Cr − 2 + 2n −1 ⎥
The correct option is (C) 2 ⎢⎣ r = 0 ( r − 1) ⎥⎦
5. Corrected Sx = 40 × 200 – 50 + 40 = 7990
1 n
\ Corrected x = 7990 / 200 = 39.95 ⇒
n
∑ fi xi2 = 2n [(n – 1)2n–2 + 2n–1]
2
Incorrect Sx2 = n[σ 2 + x ] = 200[152 + 402] = 365000

n n
Correct Sx2 = 365000 – 2500 + 1600 = 364100
= [(n – 1 + 2)2n–2] = (n + 1)
2n 2n
364100
\ Corrected σ = − (39.95) 2 1
200 Now, Var (x) =

n
∑ fi xi2 − x 2
= (1820.5 − 1596)
n( n + 1) n2 n
= 224.5 = 14.98 \ Var (x) = − =
4 4 4
The correct option is (B)
The correct option is (C)
3
6. We know that, S.D. = Q.D. 250
2 8. S.D. (σ) = = 25 = 5
3 10
\ S.D. = × 16 = 24 σ
2 Hence, coefficient of variation =
× 100
The correct option is (A) mean
Measures of Central Tendency and Dispersion  14.19

5 + ... + n . p n
= × 100 = 10%. ⇒ x =
50 n n −1 n( n − 1) n − 2 2
qn + q p+ q p + ... + p n
The correct option is (B) 1 2!
9. S.D = σ x2− y 0 ⋅ nC0 q n p0 + 1⋅ nC1q n−1 p + ... + n ⋅ nCn q0 p n
⇒ x =
1 n
n
C0 q n p0 + nC1 q n−1 p1 + ... + nCn q n−n p n
= ∑ ( xi − yi − x + y)2
n i =1 n

1 n
1 n ∑ r ⋅ nCr qn−r pr
= ∑ ( xi − x)2 + n ∑ ( yi − y)
n i =1
⇒ x =
r=0
n
i =1
n
∑ n Cr q n− r p r
2 r=0
− ∑ ( xi − x) ( yi − y)
n i =1 n
n
 ∑ r⋅ r n−1
Cr −1 q n−r p ⋅ p r −1
= σ x2 + σ 2y − 2 cov ( x, y ) =
r=0
n

As cov (x, y) is not known, therefore we cannot ∑ n Cr q n− r p r
r=0
find σ x2− y or σ x − y . Hence data is insufficient.


The correct option is (D) ⎛ n ⎞
np ⎜ ∑ n −1Cr −1 p r −1q( n −1) − ( r −1) ⎟
10. We have r = max | xi – xj | ⎝r =1 ⎠
i≠ j
⇒ x =
n

1 n ∑ n Cr q n − r p r

and 2
S = ∑ ( xi − x)2
n − 1i =1
r=0

np( q + p) n −1
⎛ x + x2 + ... + xn ⎞
2 ⇒ x =
Now, ( xi − x ) = ⎜ xi − 1
2
⎟⎠ ( q + p) n
⎝ n
\ x = np ( q + p = 1)
1
= [(xi ­– x1) + (xi – x2) + ... + (xi – xi – 1) The correct option is (A)
n2

 + (xi – xi + 1) + ... + (xi – xn)] ax + b a b
13. Let y = i.e., y = x +
1 c c c
≤ [( n − 1)r ]2 ( | xi – xj | ≤ r) b
n2 a
i.e., y = Ax + B, where A = , B =

HINTS AND EXPLANATIONS



n c c
⇒ ( xi − x ) ≤ r2 ⇒ ∑ ( xi − x)2 ≤ nr 2 \ y = Ax + B
i =1
\ y − y = A( x − x ) ⇒ ( y − y ) 2 = A2 ( x − x ) 2
1 n nr 2 nr 2
⇒ ∑
n − 1i =1
( xi − x ) 2 ≤
( n − 1)
⇒ S2 ≤
( n − 1) ⇒ Σ( y − y ) 2 = A2 Σ( x − x ) 2

n ⇒ n ⋅σ y2 = A2 ⋅ nσ x2
⇒ S≤r
n −1 ⇒ σ 2y = A2σ x2
The correct option is (A) a
⇒ σ y = | A |σx ⇒ σ y = σx
11. Let the numbers be x1, x2,..., xn. Then, c
1 n a
x = ∑ xi
n i =1
Thus, new S.D. =

c
σ.

x1 + x2 + ... + xn −1 + xn The correct option is (B)


⇒ x = 14. Mean deviation is minimum when it is considered about the
n
item, equidistant from the beginning and the end i.e., the
k + xn
⇒ x = [ x1 + x2 + ... + xn–1 = k] 101 + 1
n median. In this case median is th i.e., 51st item i.e.,
2
\ xn = nx − k x51.
The correct option is (B) The correct option is (B)
12. The required mean is
15. Consider (1 + x)50 = 50C0 + 50C1x1 + ...
n n( n − 1) n−2 2
0 ⋅ q n + 1⋅ q n−1 p + 2 ⋅ q p + 50C50x50(1)
1 2!
14.20  Chapter 14

and (1 – x)50 = 50C0 – 50C1x1 + ... + 50C50x50(2)


n n

Adding (1) and (2), we get on integrating with limits 0 to 1, ∑ i3 + ∑ i 2n2 ( n + 1) 2 n( n + 1) ( 2n + 1)
+
i =1 i =1 4 6
1 = =
50
50
C2 3 50
C4 5 50
C50 x 51 ⎤ n
n( n + 1) ( 2n + 1) n( n + 1)
n
C0 x + x + x + ... +
3 5 51 ⎥⎦
⎥ ∑i + ∑i
2
6
+
2
0 i =1 i =1

51 ⎤1
1 ⎡ (1 + x )51 (1 − x ) n( n + 1) ⎡ n( n + 1) 2n + 1⎤
= ⎢ − ⎥ +
⎢ 2 3 ⎥⎦
2 ⎢⎣ 51 51 ⎥⎦
0 =
2 ⎣
n( n + 1) ⎡ 2n + 1 ⎤
50
C50
C2 5150
C4
2 50 50
⎢ 3 + 1⎥
\ 50
C0 + = 12 =
+ + ... + 2 ⎣ ⎦
513 2 51 5
51
3n2 + 7n + 2 (3n + 1) ( n + 2) 3n + 1
50 50
Now number of terms from C0 to C50 are 26 (items) = = =
2( 2n + 4) 4 ( n + 2) 4
\ Required mean
50
C0 50
C2 50
C4 50
C50 The correct option is (C)
+ + + ... + 20. G.M. of 31, 32, 33,..., 33n
1 3 5 51
1
26 1
= (3.32 ⋅ 33...33n ) 3n =
50 (31+ 2 + 3 + ...3n ) 3n
2 1 249
=× = 3n( 3n +1)
51 26 39 × 17 = 3 3n ⋅ 2
=
3n+1


The correct option is (C) 3 2
The correct option is (C)
16. S.D. of a series is unaltered if each item is raised (reduced)
by the same scalar quantity, S.D. is independent of change of 21. Given x 1 2 3 4 ...n
origin. fw 1 22 32 42 ...n2
Hence S.D. will be same as it was already. w1x1 + w2 x2 + ... + wi xi Σwi xi
Weighted mean =
=
\ S.D. = 30 w1 + w2 + ... + wi Σwi
The correct option is (D)
w ( x ) = weighted mean
2 2 2 1.1 + 2.22 + 3.32 + ... + n.n2
17. a + b + c ≥ (a2 b2 c2)1/3 ( A.M ≥ G.M.) =
3 12 + 22 + 32 + ... + n2
13 + 23 + 33 + ... + n3
HINTS AND EXPLANATIONS

1 1 1
2
+ 2 + 2 1/ 3 =
and a b c ≥ ⎛⎜ 1 1 1 ⎞⎟ 12 + 22 + 32 + ... + n2
3 ⎝ a2 b2 c2 ⎠
n2 ( n + 1) 2 6 3n( n + 1)

On multiplying, we get = × =
4 n( n + 1) ( 2n + 1) 2( 2n + 1)
⎛ 1 1 1⎞ 2( 2n + 1)
⎛ a2 + b2 + c2 ⎞ ⎜ a2 + b2 + c2 ⎟ \ the reciprocal of the weighted mean is
⎜ ⎟ .⎜ ⎟ ≥1 3n( n + 1)
⎝ 3 ⎠ ⎜ 3 ⎟
The correct option is (C)
⎝ ⎠

1 1 1 Σxi
\ Product of the averages of a2, b2, c2 and 2 , 2 , 2 22. Given, = 38, \ Sxi = 1900
a b c 50
cannot be less than 1.
The correct option is (A) New value of Sxi = 1900 – 55 – 45 = 1800, n = 48

1800
18. When each item of a data is multiplied by l, variance is mul- \ New mean = = 37.5
48
tiplied by l2.
The correct option is (B)
Hence, new variance = 52 × 9 = 225.
23. Total weight of 7 students is = 55 × 7 = 385 kg
The correct option is (D)
Sum of weights of 6 students
19. Here, for each xi = i, weight wi = i2 + i = 52 + 58 + 55 + 53 + 56 + 54 = 328 kg
n
\ Weight of seventh student = 385 – 328 = 57 kg.
Σwi xi
∑ i (i 2 + i )
Hence, the required mean =
= i =1 The correct option is (C)
Σwi n
∑ (i 2 + i ) 24. Given: n x = x1 + x2 + x3 + ... + xn
i =1 Now new observation are x1 + 3, x2 + 3.2,... xn + 3.n
Measures of Central Tendency and Dispersion  14.21

\ new mean x = 4, y = 7 or x = 7, y = 4
( x + 3) + ( x + 3. 2) + ( x + 3. 3) + ... + ( x + 3. n) The correct option is (C)
= 1 2 3 n
n 28. Given mean = 91 × 23 – n
xi fi fixi
x1 + x2 + x3 + ... + xn 3(1 + 2 + 3 + ... + n)
= + 0 nC0 0
n n
21 nC1 2 ⋅ nC1
3( n + 1)
= x+ 22 nC2 22 ⋅ nC2
2
23 nC3 23 ⋅ nC3
The correct option is (B)
n
25. x 1 2 3 4 ...n Cn 2n nCn
2n n
x2 1 4 9 16 ...n2 Σf i = 2 Σfi xi = 3n − 1
fi 1 2 3 4 ...n Σfi xi
Now mean =

Σfi xi2 2 2
1.1 + 2.2 + 3.3 + ... + n.n 2 Σf i
x = Σf i
=
1 + 2 + 3 + ... + n 3 n
2 2
⇒ 91 × 2 = 3 − 1
n ( n + 1) 2 n( n + 1) 2n 2n
= × =⋅
4 n( n + 1) 2 \ 3 = 3 n 6

The correct option is (B) or n = 6


26. xi : 1 2 3 4 ... n – 1 n The correct option is (B)
fi : n n – 1 n – 2 n – 3 ... 2 1 29. Let n items be denoted by x1, x2, x3,... xn
n \ new items are x1 + 3, x2 + 32, x3 + 33,... xn + 3n
\ ∑ xi fi = 1⋅ n + 2 (n – 1) \ new mean
i =1

 + 3(n – 2) +, ..., + (n – 2) 2 + n.1 ( x1 + 3) + ( x2 + 32 ) + ( x3 + 33 ) + ... + ( xn + 3n )
=
n n
= ∑ (n + 1)r − r 2 ( x1 + x2 ... + xn ) 31 + 32 + ... + 3n
r =1 = +
n n n n
= ( n + 1) ∑ r − ∑ r2 n
r =1 r =1 = x + 3(3 − 1)
2n

HINTS AND EXPLANATIONS


( n + 1) ( n) ( n + 1) n( n + 1) ( 2n + 1)
= − The correct option is (B)
2 6
30. The number of numbers in the nth row = n2
n( n + 1) ( n + 2) Sequence of first terms in different row is
=
6 1, 3, 11, 29, 61, ...
1
n( n + 1) \ Tn of 1, 3, 11, 29, 61,... = (2n3 – 3n2 + n + 3) = first

Also, Sfi = 1 + 2 +, ..., + n = th 3
2 element of n row.
Σfi xi n( n + 1) ( n + 2) 2 n+2 Similarly, sequence of last terms of each row = 1, 9, 27, 59,...

Mean == × =
Σf i 6 n( n + 1) 3 1
\ tn = [2n3 + 3n2 + n – 3]
The correct option is (C) 3
= last element of the nth row.
27. Let the two unknown items be x and y, then Mean = 4
Hence, in the nth row elements can be written as
1+ 2 + 6 + x + y
⇒ =4 1 1
5 (2n3 – 3n2 + n + 3), ... (2n3 + 3n2 + n – 3)
⇒ x + y = 11 (1) 3 3
and variance = 5.2 (Note: adding 2 in the preceding number to get the succeed-
2 2 2 2 2 ing number)
⇒ 1 + 2 + 6 + x + y – (mean)2 = 5.2 \ sum of the elements of nth row (using sum of n terms of
5
A.P.)
⇒ 41 + x2 + y2 = 5[5.2 + (4)2]
N n2 ⎛ 4 n3 + 2n ⎞ 3
⇒ 41 + x2 + y2 = 106 = ( A + L) = ⎜ ⎟ = n ( 2n2 + 1)
or x2 + y2 = 65 (2) 2 2 ⎝ 3 ⎠ 3
Solving (1) and (2) for x and y, we get \ mean of the numbers in the nth row
14.22  Chapter 14

⇒ | a | = 4
n3 ( 2n2 + 1) 2
= = n( 2n + 1) The correct option is (D)
3 × n2 3 2 x + 2 x2 + ... + 2 xn
36. A.M. of 2x1, 2x2, ..., 2xn is 1
⎧ N = n2 , A = Tn , L = t n ⎫ n
⎪ ⎪ ⎛ x1 + x2 + ... + xn ⎞

Here ⎨ = 2⎜ ⎟⎠ = 2x
4 n + 2n ⎬
3
⎝ n
⎪∴ A + L = Tn + t n = ⎪
⎩ 3 ⎭ So statement-2 is false. Variance (2xi) = 22 variance (xi)
= 4σ2 so statement-1 is true.
The correct option is (D)
The correct option is (D)
31. Let x1, x2, ... xn be n observations.
1 ( x1 + 4) + ( x2 + 8) + ( x3 + 12) + ... + ( x10 + 40)
Then, x = Σxi 37. Mean =
n 10
xi
+ 10 x1 + x2 + ... + x10 4(1 + 2 + 3 + ... + 10)
Let yi = α = +
10 10
1 n 1 ⎛1 n ⎞ 1 = 20 + 22 = 42
Then, y =

n i =1
yi = ⎜ ∑ xi ⎟ + (10 n)
α ⎝ n i =1 ⎠ n The correct option is (B)
38. Mean of a, b, 8, 5, 10 is 6
1 x + 10α
\ y= x + 10 = a + b + 8 + 5 + 10
α α ⇒ =6
5
The correct option is (B)
⇒ a + b = 7 (1)
32. x1 = 520, x2 = 420 and x = 500
Given that Variance is 6.8
n1x1 + n2 x2
Also, we know x =
Σ( X i − A) 2
n1 + n2 \ Variance =
n
⇒ 500 (n1 + n2) = 520n1 + 420n2 ⇒ 20n1 = 80n2 2 2
⇒ n1 : n2 = 4 : 1 = ( a − 6) + (b − 6) + 4 + 1 + 16 = 6.8
5
Hence, the percentage of male employees in the firm
⇒ a2 + b2 = 25
⎛ 4 ⎞ a2 + (7 – a)2 = 25 [Using (1)]
= ⎜ × 100 = 80%
⎝ 4 + 1⎟⎠ ⇒ a2 – 7a + 12 = 0
HINTS AND EXPLANATIONS

The correct option is (A) \ a = 4, 3 and b = 3, 4


33. Let the weight of the teacher be w kg, then The correct option is (D)
1 30 × 40 + w sum of quantities
40 + = 39. Mean ( x ) =
2 35 + 1 n
1 n
⇒ 36 × 40 + 36 × = 35 × 40 + w ⇒ w = 58 ( a + 1)
1
2 = 2 = [1 + 1 + 100 d ] = 1 + 50d
\ Weight of the teacher = 58 kg. n 2
The correct option is (D) 1
M.D. =

n
∑ xi − x
120 + 120 + 120
34. Average speed = 120 120 120 1
+ + ⇒ 255 = (50 d + 49d + 48d + ... + d + 0 + d + ... + 50 d )
30 25 50 101
3 2d ⎛ 50 × 51⎞
= ⎜ ⎟
= 1 1 1 km/h. 101 ⎝ 2 ⎠
+ +
30 25 50 255 × 101
⇒ d = = 10.1

The correct option is (C) 50 × 51

The correct option is (C)
25 + 26
35. From the given data, median = a = 25.5a
2 40. Statement-2 is true

Required mean deviation about median Statement-1: Sum of n even natural numbers = n (n + 1)
2 | 0.5 + 1.5 + 2.5 + ... + 24.5 | n( n + 1)
= | a | = 50 Mean ( x ) =
=n+1
50 n
Measures of Central Tendency and Dispersion  14.23

⎡1 ⎤ ( n + 1)[2( 2n + 1) − 3( n + 1)]
Variance = ⎢ ∑ ( xi ) 2 ⎥ − ( x ) 2
=
⎣n ⎦ 3
1 2 ( n + 1)[4 n + 2 − 3n − 3]
= [2 + 4 2 + ... + ( 2n) 2 ] − ( n + 1) 2 =
n 3
2
=
1 2 2
2 (1 + 22 + ... + n2 ) − ( n + 1) 2 = ( n + 1)( n − 1) = n − 1
n 3 3
4 n( n + 1)( 2n + 1) \ Statement 1 is false.
= − ( n + 1) 2
n 6 The correct option is (D)

Previous Year’s Questions


3x + 1 1 − x 1 − 2 x
41. 0 ≤ + + ≤1 Since A and B both has 100 consecutive integers, therefore
3 4 2 both have same standard deviation and hence the variance.
⇒ 12x + 4 + 3 - 3x + 6 - 12x ≤ 1 V
∴ A = 1 (As ∑ di2 is same in both the cases) .
⇒ 0 ≤ 13 - 3x ≤ 12 VB

⇒ 3x ≤ 13 The correct option is (A)
1
⇒x≥ .
13 48. According to question
The correct option is (C) 52x + 42y = 50 (x + y)
42. Let the numbers be a and b then a + b = 18 and ab = 4 ⇒ ⇒ 2x = 8y
ab = 16 x 4 x 4
⇒ = and =
So, a and b are roots of the equation y 1 x+ y 5

x2 - 18x + 16 = 0. \ % of boys = 80.
The correct option is (D) The correct option is (C)
43. Mode can be computed from histogram and median is depen-
dent on the scale. Hence statements (a) and (b) are correct. 49. Mean of a, b, 8, 5, 10 is 6
The correct option is (C)

HINTS AND EXPLANATIONS


a + b + 8 + 5 + 10
⇒ =6
44. xi = a for i = 1, 2,...., n and xi = -a for i = n+1,...., 2n 5
2n ⇒ a + b = 7 (1)
1
S.D = ∑ ( xi − x )2
2n i =1
Given that Variance is 6.8

\ Variance =
∑ ( X i − A)2
1 2n 2 ⎛ 2n ⎞
⇒2= ∑
2n i =1
xi ⎜ Since ∑ xi = 0⎟
⎝ ⎠
n
i =1

( a − 6) 2 + (b − 6) 2 + 4 + 4 + 16
1 = = 6.8
⇒2= ⋅ 2na 2 ⇒ | a |= 2 5
2n
 ⇒ a2 + b2 = 25
The correct option is (C)
45. We have Mode + 2 Mean = 3 Median   a2 + (7 - a)2 = 25 (Using (1))
⇒ Mode = 3 × 22 - 2 × 21 = 66 - 42 = 24 ⇒ a2 - 7a + 12 = 0
The correct option is (D) \ a = 4, 3 and b = 3, 4.
2
The correct option is (D)
46. We have that
∑ xi2 ⎛ ∑ xi ⎞
≥⎜ ⎟
n ⎝ n ⎠ n/
⇒ n ≥ 16 . sum of quantities 2 ( a + l )
50. Mean ( x ) = =
The correct option is (B) n n/
Σdi2 1
47. The variance is given by σ x2 = (Here deviations are = [1 + 1 + 100 d ] = 1 + 50 d
n 2
taken from the mean) Now, since M.D. about mean is 255, we have
14.24  Chapter 14

1
Statement-1 is true.
M.D. =
n
∑ xi − x 2 x1 + 2 x2 + ... + 2 xn
A.M. of 2 x1, 2 x2 ,....., 2 xn =

1 n
⇒ 255 = [50 d + 49d + 48d + ... + d + 0 + d + ..... + 50 d ]
101 ⎛ x1 + x2 + .... + xn ⎞
= 2⎜ ⎟⎠ = 2 x
2 d ⎡ 50 × 51 ⎤ ⎝ n
=
101 ⎢⎣ 2 ⎥⎦ Statement-2 is false.

The correct option is (D)
255 × 101
⇒ d= = 10.1 54. Variance is not changed by the change of origin.
50 × 51

The correct option is (C)
∑ x−x
2

51. σx2 = 4 σ=
n
σy2 = 5
Therefore, y = x + 10 ⇒ y + x + 10

x=2
y=4
∑ y + 10 − y − 10 ∑ y−y
2 2

∑ xi = 2 σ1 = = =σ .
5
∑ xi = 10; ∑ yi = 20 n n
The correct option is (C)
⎛1
⎝ 2


1
5
(
σ x2 = ⎜ ∑ xi2 ⎟ − ( x ) 2 = ∑ yi2 − 16 ) ⎛ ∑ xi2 ⎞
55. σ 2 = ⎜ 2
⎟−x
⎝ n ⎠
∑ i
2
x = 40
50

∑ i ∑ 2r
y 2 = 105
r =1
2 x= = 51
σ z2 =
1
10
(
∑ i ∑ i ⎜⎝ 2 ⎟⎠
x 2
+ y 2
− )
⎛ x + y⎞
50
50
1
= ( 40 + 105) − 9 =
145 − 90 ∑ 4r 2
− (51) = 833
r −1 2
10 10 σ2 =
50
55 11
= = The correct option is (B)
10 2
HINTS AND EXPLANATIONS

The correct option is (A)


56. New sum ∑ yi = (16 × 16 − 16) + (3 + 4 + 5) = 252
1 Number of observation = 18
52. ∑ xi − A
n
⇒ New mean
25a + 26 a 252
A = Median =
= 25 − 5a y= = 14 .
2 18
1 The correct option is (C)
Mean deviation = { a − 25.5a + 2a − 25.5a }

50
2
2 Σxi2 ⎛ Σxi ⎞
= {( 24.5a + 23.5a ) + ... (0.5a )} 57. S.D. = −⎜
50 n ⎝ n ⎟⎠
2
= {312.5a} = 50 Given 49 4 + 9 + a 2 + 121 ⎛ 16 + a ⎞
2
50 ∴ = −⎜
4 4 ⎝ 4 ⎟⎠
⇒ 625a = 2500 ⇒ a = 4
The correct option is (B) ⇒ 3a 2 − 32a + 84 = 0

2

The correct option is (C)
xi 2 ⎛ xi ⎞
53. σ 2 = ∑ ∑ n ⎟⎠
n ⎜⎝ n
( xi − x ) 2
58. Formula of variance, σ 2 = ∑
n
(2 xi )2 − ⎛ 2 xi ⎞
2 i =1
Variance of 2 x1, 2 x2 ,...., 2 xn = ∑
⎜⎝ ∑
n n ⎟⎠ n

⎡ x 2 ⎛ x ⎞ 2⎤
∑ ( xi − x )2
i =1
= 4 ⎢ ∑ i − ⎜ ∑ i ⎟ ⎥ = 4σ 2 Formula of Standard deviation, σ =

⎢⎣ n ⎝ n ⎠ ⎥⎦ n

n=9
Measures of Central Tendency and Dispersion  14.25

9 9 9 9 9 9 9

∑ xi − 5∑1 = 9 ⇒
∑ xi2 − 12∑ xi + 36∑1 = 45 − 2∑ xi + 11∑1
i =1 i =1 i =1 i =1 i =1 i =1 i =1

9 9

∑ xi − 45 = 9 ⇒
∑ ( xi − 6)2 = 45 − 2 × 54 + 99
i =1 i =1

9 9

∑ xi = 54 ⇒
∑ ( xi − 6)2 = 45 − 108 + 99 = 36
i=1 i =1

54 9
( xi − 6)
x= = 6 [ x = Mean ]


9 ⇒
∑ 9
= 4 = σ 2   [VARIANCE]
i =1
9
∑ ( xi − 5)
2

Given = 45 Hence, standard deviation, σ = 4 = 2

i =1

9 9 9

∑ xi2 − 10∑ xi + 25∑1 = 45
i =1 i =1 i =1

HINTS AND EXPLANATIONS


This page is intentionally left blank.
CHAPTER
Trigonometric
15 Ratios and Identities
LEARNING OBJECTIVES
After reading this chapter, you will be able to:
  Learn about the angles and their measurements  Understand fundamental trigonometric identities and
 Grasp various relations between different systems of signs of trigonometric ratios in different quadrants
measurement of angles and sides and interior angles of  Know domain and range of trigonometric ratios and learn
a regular polygon various transformation formulae

ANGLE
Let a revolving line starting from OX revolves about its end
point O on a plane in the direction of arrow and occupy the
position OP. It is said to trace out an angle XOP. Here OX
is called the initial position and OP, the terminal position.
The fixed point O is called the vertex.

FIGURE 15.2

There are three systems for measurement of an angle:


(a) (b)
FIGURE 15.1
An angle is considered as the figure traced by rotating
1. Sexagesimal System or English System
a given ray about its end point.
In this system an angle is measured in degrees, min-
utes and seconds. A complete rotation describes
360°.
MEASUREMENT OF ANGLES
1 right angle = 90°, (read as 90 degrees)
1° = 60′ (read as 60 minutes)
I M P O R TA N T P O I N T S 1′ = 60′′ (read as 60 seconds)
If the rotation is in anticlockwise sense, the angle meas- 2. Centesimal or French System
ured is positive and if the rotation is in clockwise sense, In this system an angle is measured in grades, min-
the angle measured is negative. utes and seconds.
1 right angle = 100g (read as 100 grades)
15.2  Chapter 15

1g = 100′ (read as 100 minutes) D G θ


1′ = 100′′ (read as 100 seconds) = =
180 200 π

ERROR CHECK
1′ of centesimal system ≠ 1′ of sexagesimal system RELATION BETWEEN SIDES AND INTERIOR
1″ of centesimal system ≠ 1″ of sexagesimal system ANGLES OF A REGULAR POLYGON
1.  Sum of interior angles of polygon of n sides
3. Radian or Circular Measure = (2n – 4) × 90°
A radian is a constant angle subtended at the cen-
tre of a circle by an arc whose length is equal to the 2.
Each interior angle of a regular polygon of n sides
radius of the circle and is denoted by 1c.
2n − 4
= × 90°
n

QUICK TIPS
 The angle between two consecutive digits in a clock is
30°.
 The hour hand rotates through an angle of 30° in one

 1 °
hour or   in one minute
FIGURE 15.3 2
 The minute hand rotates through an angle of 6° in one
∠AOB = 1 radian.
minute.
This angle does not depend upon the radius of the
circle from which it is derived.
FUNDAMENTAL TRIGONOMETRIC
ERROR CHECK IDENTITIES
Radian is a unit to measure angle and it should not be inter- sin2θ + cos2θ  = 1 or cos2θ  = 1 – sin2θ
1.
preted that π stands for 180°, π is a real number whereas
p c stands for 180°.
or sin2θ = 1 – cos2θ
1 + tan2θ = sec2θ or sec2θ – tan2θ = 1
2.
1 + cot2θ = cosec2θ or cosec2θ – cot2θ = 1
3.
REMEMBER
π radians = 180° = 200g. QUICK TIPS
Since sin θ + cos θ = 1, | sinθ | ≤ | and | cosθ | ≤ |
2 2

⇒ –1 ≤ sinθ ≤ 1 and –1 ≤ cosθ ≤ 1;


RELATION BETWEEN DIFFERENT SYSTEMS OF 0 ≤ sin2θ ≤ 1, 0 ≤ cos2θ ≤ 1.
MEASUREMENT OF ANGLES Since cosecθ = 1/sinθ, cosecθ ≥ 1 or cosecθ ≤ –1
Also, since secθ = 1/cosθ, sec θ ≥ 1 or secθ ≤ –1
10 9
1°= grades; 1g = degrees ⋅

9 10
π SIGNS OF TRIGONOMETRIC RATIOS IN
1°= radians = 0.0172 radians;
180 DIFFERENT QUADRANTS
180 The following table describes the signs of various t-ratios
1 radian = degrees = 57° 17′ 45′′
π in different quadrants. Also, refer to, the figure given below
the table.
π 200
1g = radians; 1 radian = grades
200 π Quadrant I II III IV

MP = y +ve +ve -ve -ve
Thus if the measure of an angle in degrees, grades and
OM = x +ve -ve -ve +ve
radians be D, G and θ respectively, then
Trigonometric Ratios and Identities  15.3

Quadrant I II III IV Quadrant: → I II III IV


y + ve + ve − ve − ve t-ratios All sinθ tanθ cosθ
sinθ = = + ve = + ve = − ve = − ve
r + ve + ve + ve + ve which are + ve cosecθ cotθ secθ
x + ve − ve − ve + ve
cosθ = = + ve = − ve = − ve = + ve
r + ve + ve + ve + ve REMEMBER
y + ve + ve − ve − ve  In the first quadrant, all are +ve. 
tanθ = , = + ve = − ve = + ve = − ve
x + ve −ve − ve + ve  In the second quadrant, sin and cosec are +ve
x≠0  In the third quadrant, tangent and cotangent are +ve
 In the fourth quadrant, cosine and secant are +ve.

FIGURE 15.5
Simple rule to remember:
FIGURE 15.4
add – sugar – to – coffee
The signs of other t-ratios can be found by using recip- or
rocal relations, i.e. after – school – to – college
1 1 1
cosecθ = , sec θ = and cot θ = . So, we have In the above, ‘a’ stands for ‘all’, ‘s’ stands for ‘sine’, ‘t’ stands
sin θ cos θ tan θ for ‘tan’ and ‘c’ stands for ‘cos’. The reciprocals of these
ratios are also positive in the respective quadrants.

INCREASE AND DECREASE OF TRIGONOMETRIC FUNCTIONS


We can discuss the way of increase and decrease of trigonometric functions as described in the following tables:

I quadrant II quadrant III quadrant IV quadrant


sin θ increases from 0 to 1 decreases from 1 to 0 decreases from 0 to -1 increases from -1 to 0
cos θ decreases from 1 to 0 decreases from 0 to -1 increases from -1 to 0 increases from 0 to 1
tan θ increases from 0 to ∞ decreases from ∞ to 0 increases from 0 to ∞ increases from -∞ to 0
cot θ decreases from ∞ to 0 increases from 0 to ∞ decreases from ∞ to 0 decreases from 0 to -∞
sec θ increases from 1 to ∞ increases from -∞ to -1 decreases from -1 to -∞ decreases from ∞ to 1
cosec θ decreases from ∞ to 1 increases from 1 to ∞ increases from -∞ to -1 decreases from -1 to -∞

DOMAIN AND RANGE OF TRIGONOMETRIC


RATIOS Functions Domain Range
 π  (-∞, -1] ∪ [1,
Functions Domain Range sec x ( − ∞, ∞) − ( 2n + 1) n ∈ I 
 2  ∞)
sin x, cos x (-∞, ∞) [-1, 1]
(-∞, -1] ∪ [1,
 π  cosec x {
( − ∞, ∞ ) − nπ n ∈ I } ∞)
tan x ( −∞, ∞) − ( 2n + 1) n ∈ I  (-∞, ∞)
 2 
cot x (-∞, ∞) - {nπ | n ∈ I} (-∞, ∞)
15.4  Chapter 15

TRIGONOMETRIC RATIOS OF STANDARD Angles 0° 30° 45° 60° 90°


ANGLES
T-Ratios
Angles 0° 30° 45° 60° 90°
2
cosec x Undefined 2 2 1
T-Ratios 3

1 1 3 2
sin x 0 1 sec x 1 2 Undefined
2 2 2 2
3
3 1 1
cos x 1 0 1
2 2 2 cot x Undefined 3 1 0
3
1
tan x 0 1 3 Undefined
3

TRIGONOMETRIC RATIOS FOR SOME SPECIAL ANGLES


Angle sin θ cos θ tan θ cot θ sec θ coses θ
0°/0 0 1 0 ∞ 1 ∞
15° 3−1 3+1 3−1 3+1 2 2 2 2
π /2 2 2 2 2 3+1 3−1 3+1 3−1

18° 5 −1 5 −1 10 + 2 5 4
10 + 2 5
5+1
π /10 4 4 10 + 2 5 5 −1 10 + 2 5

22.5° 2− 2 2+ 2
2 −1 2 +1 4− 2 2 4+ 2 2
π /8 2 2
30° 1 3 1 2
3 2
π /6 2 2 3 3

36° 5+1 5+1 10 − 2 5 4


10 − 2 5
5 −1
π /5 4 4 10 − 2 5 5+1 10 − 2 5

45° 1 1
1 1 2 2
π /4 2 2

54° 5+1 5+1 4


10 − 2 5 10 − 2 5
5 −1
3π /10 4 4 10 − 2 5 5+1 10 − 2 5

60° 3 1 1 2
3 2
π /3 2 2 3 3

67.5° 2+ 2 2− 2
2 +1 2 −1 4+ 2 2 4− 2 2
3π /8 2 2

72° 10 + 2 5 5 −1 10 + 2 5 5 −1 4
5+1
2π/ 5 4 4 5 −1 10 + 2 5 10 + 2 5

72° 3 +1 3 −1 3+1 3−1 2 2 2 2


5π/12 2 2 2 2 3−1 3+1 3−1 3+1

90°
1 0 ∞ 0 ∞ 1
π /2
Trigonometric Ratios and Identities  15.5

TRIGONOMETRIC RATIOS OF ALLIED ANGLES i.e. the angles –θ, 90° ± θ, 180° ± θ, 270° ± θ and 360°
± θ are called allied angles.
Two angles are said to be allied when their sum or differ-
ence is either zero or a multiple of 90°.

FIGURE 15.6

TABLE FOR TRIGONOMETRIC RATIOS OF ALLIED ANGLES


-θ 90° - θ 90° + θ 180° - θ 180° + θ 270° - θ 270° + θ 360° - θ 360° + θ

sinθ -sinθ cosθ cosθ sinθ -sinθ -cosθ -cosθ -sinθ sinθ

cosθ cosθ sinθ -sinθ -cosθ -cosθ -sinθ sinθ cosθ cosθ

tanθ -tanθ cotθ -cotθ -tanθ tanθ cotθ -cotθ -tanθ tanθ

WORKING RULE TO FIND ALLIED ANGLES  The trigonometric function is replaced by its cofunction i.e.
Case I: When the angle is nπ ± θ, where n ∈ I and θ is acute. sin changes to cos, tan changes to cot and sec changes to
 There is no change in trigonometric function i.e., sin
cosec and vice-versa.
remains sin, cos remains cos and tan remains tan. Angle Angle associated becomes θ.
associated becomes θ.  The sign is affixed according to the quadrant in which the

 The sign is affixed according to the quadrant in which the angle lies.
angle lies. Note that the sign is always decided on the basis of the
nπ operating function.
Case II: When the angle is ± θ , where n is an odd inte-
ger and θ is acute. 2
15.6  Chapter 15

I M P O R TA N T P O I N T S
 n +1

 nπ  (−1) sin θ , if n is odd,


2
 sin nπ = 0, cos nπ = (–1)n. cos  +θ  = 
 sin (nπ + θ) = (–1) sinθ, cos (nπ + θ) = (–1) cosθ
n n  2   n

(−1)
2
cos θ , if n is eveen.
 For odd integer n,


nπ n −1
nπ To find ratios for nπ – θ and , replace θ by –θ in all of

sin = (−1) 2 , cos =0 the above 2


2 2

 n −1

 nπ  (−1) 2 cos θ , if n is odd,


 sin
 +θ  = 
 2   n

(−1)
2
sin θ , if n is eveen.

TRIGONOMETRIC RATIOS IN TERMS OF EACH OTHER

sin θ cos θ tan θ cost θ sec θ cosec θ


tanθ 1 sec2θ − 1 1
sinθ sinθ 1 − cos2θ
1 + tan2θ 1+ cot2θ secθ cosecθ

1 cotθ 1 cosec2θ − 1
cosθ 1 − sin2θ cosθ secθ
1 + tan2θ 1 + cot2θ cosecθ
sinθ 1 1
tanθ 1 − cos2θ tanθ sec2θ − 1
1 − sin2θ cosθ cotθ cosec2θ − 1

1− sin2θ cosθ 1 1
cotθ tanθ
cotθ cosec2θ − 1
sinθ 1 − cos2θ sec2θ − 1

1 1 1 + cot2θ cosecθ
secθ cosθ
1 + tan2θ secθ
1 − sin2θ cot cosec2θ − 1

1 1 secθ
cosecθ 1+ sin2θ 1 + cot2θ cosecθ
sinθ 1 − cos2θ tanθ sec2θ − 1

ADDITION AND SUBTRACTION FORMULAE 9. sin(A + B) sin(A – B) = sin2A – sin2B


10. cos(A + B) cos(A – B) = cos2A – sin2B
1. sin(A + B) = sinA cosB + cosA sinB
11. sin(A + B + C) = sin A cos B cos C + cos A sin B cos
2. cos(A + B) = cosA cosB – sinA sinB
C + cos A cos B sin C – sin A sin B sin C
tan A + tan B (Σ (one sine and two cos) – three sines)
3. tan( A + B) =
1 − tan A tan B or
4. sin(A – B) = sinA cosB – cosA sinB = cosA cosBcosC(tanA + tanB + tanC – tanA tanB
5. cos(A – B) = cosA cosB + sinA sinB tanC)
tan A − tan B 12. cos (A + B + C) = cos A cos B cos C – sin A sin B cos
6. tan( A − B) =
1 + tan A tan B C
cot A cot B − 1 – sin A cos B sin C – cos A sin B sin C (three cos –
7. cot( A + B ) = Σ(one cos and two sine))
cot A + cot B
or
cot A cot B + 1 = cos A cosB cosC(1 – tanA tanB – tanB tanC – tanC
8. cot( A − B ) =
cot B − cot A tanA)
Trigonometric Ratios and Identities  15.7

tan A + tan B + tan C − tan A tan B tan C TRANSFORMATION  FORMULAE


13. tan( A + B + C ) =
1 − tan A tan B − tan B tan C − tan C tan A PRODUCT INTO SUM OR DIFFERENCE
14. tanA + tanB + tanC – tanA tanB tanC
2 sinA cosB = sin(A + B) + sin(A – B), A > B
1.
sin ( A + B + C ) 2 cosA sinB = sin(A + B) – sin(A – B), A > B
2.
=
cos A cos B cos C 2 cosA cosB = cos(A + B) + cos(A – B)
3.
15. 1 – tanA tanB – tanB tanC – tanC tanA 2 sinA sinB = cos(A – B) – cos(A + B)
4.

cos ( A + B + C )
=
cos A cos B cos C SUM AND DIFFERENCE INTO PRODUCT
π  1 + tan A C + D  C −D 
16. tan  + A  = 1. sin C + sin D = 2 sin   cos  
4  1 − tan A  2   2 
π  1 − tan A C + D  C −D 
17. tan  − A  = 2. sin C − sin D = 2 cos 
 4  1 + tan A  sin  
 2   2 
18. sin(A1 + A2 + ... + An) = cosA1 cosA2 ... cosAn (S1 – S3
+ S5 – ...) C+D  C −D 
3. cos C + cos D = 2 cos   cos  
19. cos(A1 + A2 + ... + An) = cosA1 cosA2 ... cosAn (1 – S2  2   2 
+ S4 – S6 + ...)
C + D  C −D 
S1 − S3 + S5 −  4. cos C − cos D = −2 sin   sin  
20. tan( A1 + A2 +  + An ) =  2   2 
1 − S2 + S4 − S6 + 
where S1 = Σ tanA1, S2 = Σ tanA1 tanA2, sin (C + D )
5. tan C + tan D =
S3 = Σ tanA1 tanA2 tanA3 and so on. cos C cos D
21. sin α + sin (α + β) + sin (α + 2β) + ... + sin (α + (n –
1) β) sin (C − D )
6. tan C − tan D =
cos C cos D
 β
sin  α + ( n − 1) 
 2   nβ  sin (C + D )
= sin   7. cot C + cot D =
β  2  sin C sin D
sin
2
sin ( D − C )
If β = α, then
8. cot C − cot D =
sin C sin D
 n +1 nα
sin   × sin
 2  2 TRIGONOMETRIC RATIOS OF MULTIPLE
sin α + sin 2+, … , + sin nα =
α ANGLES
sin
2 (An Angle of the form nθ, n ∈ I)
22. cos α + cos(α + β) + cos(α + 2β) + ,..., + cos(α + 2 tan θ
(n – 1)β ) 1. sin 2θ = 2 sin θ cos θ =
1 + tan 2 θ
 β cos 2θ = cos2θ – sin2θ = 2 cos2θ – 1
2.
cos  α + ( n − 1) 
 2  nβ 
= sin   1 − tan 2 θ
β  2  = 1 − 2 sin 2 θ =
sin
2
1 + tan 2 θ

If β = α, then
2 tan θ
3. tan 2θ =
α nα 1 − tan 2 θ

cos  ( n + 1)  × sin
 2 2 cot 2 θ − 1
cos α + cos 2α +, … , + cos nα = 4. cot 2θ =
α 2 cot θ
sin
2
15.8  Chapter 15

1 1 − cos θ θ
5. 1 + cos 2θ = 2 cos 2 θ , cos 2 θ = (1 + cos 2θ ) 9. = tan
2 sin θ 2
1 1 + cos θ θ
6. 1 − cos 2θ = 2 sin 2 θ , sin 2 θ = (1 − cos 2θ ) 10. = cot
2 sin θ 2
1 θ θ π   π
sin 3θ = 3sin θ – 4 sin3θ, sin θ =
3
7. (3 sinθ – sin3θ) 11. sin ± cos = 2 sin  ± θ  = 2 cos  θ ∓ 
4 2 2  4   4
= 4 sin(60° – θ)sinθ sin(60° + θ)
1 QUICK TIPS
cos 3θ = 4 cos3θ – 3 cosθ, cos3 θ = (cos 3θ + 3 cosθ)
8.
4
= 4 cos(60° – θ) cosθ cos(60° + θ) sin A + cos A = 1 + sin A

2 2
= 4 cos(120° – θ) cosθ cos (120° + θ)
A A
or sin + cos = ± 1 + sin A
3 tan θ − tan 3 θ 2 2
9. tan 3θ =
1 − 3 tan 2 θ +, If 2nπ − π /4 ≤ A/2 ≤ 2nπ + 3π /4

= tan (60° – θ) tanθ tan (60° + θ) i.e., 
−, other wise
cot 3 θ − 3 cot θ A A
10. cot 3θ = sin − cos = 1 − sin A
3 cot 2 θ − 1

2 2
sin 2n A
11. cos A cos 2A cos 22A ... cos 2n – 1 A =  A A
2n sin A or  sin − cos  = ± 1 − sin A
 2 2 
TRIGONOMETRIC RATIOS OF +, If 2nπ − π /4 ≤ A/2 ≤ 2nπ + 5π /4
i.e., 
SUBMULTIPLE ANGLES −, other wise
θ A ± tan A + 1 − 1
2
(An Angle of the form , n ∈ I) (i) tan =
n 
2 tan A
θ θ 2 tan θ /2 1 − cos A
1. sin θ = 2 sin cos = =±
2 2 1 + tan 2 θ /2 1 + cos A

θ θ θ θ 1 − cos A
=
2. cos θ = cos 2 − sin 2 = 2 cos 2 − 1 = 1 − 2 sin 2 sin A
2 2 2 2
1 − tan 2θ where  A ≠ (2n + 1)π
= 2
1 + cos A
1 + tan 2 θ (ii) cot
A

2 2 1 − cos A
2 tan θ =
1 + cos A
3. tan θ = 2
sin A
1 − tan 2 θ
2
θ
−1
cot 2
2
4. cot θ =
2 cot θ
2
2 θ 1 + cos θ
5. cos =
2 2
θ 1 − cos θ
6. sin 2 =
2 2
θ 1 − cos θ FIGURE 15.7
7. tan 2 =
2 1 + cos θ
where A ≠ 2nπ
θ 1 + cos θ The ambiguities of signs are removed by locating the quad-
8. cot =2
A
2 1 − cos θ rant in which lies or you can follow the adjoining figure.
2
Trigonometric Ratios and Identities  15.9

3. tan(C + A) = tan B, cot A = –cot(B + C)


A+ B C C A+ B
4. cos = sin , cos = sin
2 2 2 2
C+A B A B+C
5. sin = cos , sin = cos
2 2 2 2
B +C A B C+A
6. tan = cot , tan = cot
2 2 2 2

REMEMBER
If A + B + C = π, then
 sin2A + sin2B + sin2C = 4sinA sinB sinC

 cos2A + cos2B + cos2C = – 1 – 4 cos A cosB cosC

 tan A + tanB + tanC = tanA tanB tanC

 cot A cotB + cotB cotC + cotC cotA = 1

A B C A B C
 cot + cot + cot = cot cot cot
2 2 2 2 2 2
FIGURE 15.8 A B B C C A
 tan tan + tan tan + tan tan = 1
2 2 2 2 2 2
A B C
GREATEST AND LEAST VALUES OF THE  sin A + sin B + sin C = 4 cos cos cos
2 2 2
EXPRESSION A B C
 cos A + cos B + cos C = 1 + 4 sin sin sin
a sinθ + b cosθ 2 2 2
Let a = r cosα, b = r sin α, then
a2 + b2 = r2 or r = a 2 + b 2
GRAPHS OF TRIGONOMETRIC FUNCTIONS
Then a sinθ + b cosθ = r(sinθ cosα + cosθ sinα)
= r sin(θ +α) Graph of y = sin x
1.
But –1 ≤ sin(θ + α) ≤ 1, so
–r ≤ r sin(θ + α) ≤ r
or − a 2 + b 2 ≤ a sin θ + b cos θ ≤ a 2 + b 2
Thus, the greatest and least values of a sinθ + b cosθ are
a 2 + b 2 and –  a 2 + b 2 respectively.

QUICK TIPS
sin x + cosec x ≥ 2, for every real x
2 2
FIGURE 15.9
cos2x + sec2x ≥ 2, for every real x Graph of y = cos x
2.
tan2x + cot2x ≥ 2, for every real x

CONDITIONAL IDENTITIES
When the angles A, B and C satisfy a given relation, many
interesting identities can be established connecting the
trigonometric functions of these angles. In providing these
identities, we require the properties of complementary and
supplementary angles. For example, if A + B + C = π, then FIGURE 15.10
1. sin(B + C) = sinA, cosB = –cos(C + A)
Graph of y = tanx
3.
2. cos(A + B) = –cosC, sin C = sin(A + B)
15.10  Chapter 15

FIGURE 15.14
FIGURE 15.11

Graph of y = cotx
4. Graph of y = 3 sin 2x
6.

FIGURE 15.12
Graph of y = secx and y = cosecx
5. FIGURE 15.15
Since the period of sinx is 2π. Therefore, the period of


sin 2x is = π . Also, –3 ≤ 3 sin2x ≤ 3. The graph
2
of y = 3 sin2x is drawn for a period 0 ≤ x ≤ π. The
complete graph is simply the repetition of the portion.

In a similar way the graphs of other trigonometric
functions can be drawn.

FIGURE 15.13
Trigonometric Ratios and Identities  15.11

PRACTICE EXERCISES

Single Option Correct Type

1. The value of cos a cos 2a cos 3a ....cos 999 a, where 8. If e–π/2 < θ < π/2, then
2π (A) cos logθ < log cosθ
a= , is
1999 (B)  cos logθ > log cosθ
1 1 (C) cos logθ ≤ log cosθ
(A) 99 (B)  (D) none of these
2 2999
1 x y
1 1 9. sin θ =  +  necessarily implies
(C) (D) 1999 2  y x 
29999 2
π π (A) x > y
tan cot
 π 8  π 8 (B) x < y
2. Let a1 =  tan  , a2 =  tan  ,
 8  8 (C) x = y
π π
(D) both x and y are purely imaginary
tan cot
 π 8  π 8 x+ y
a3 =  cot  , a4 =  cot  Then, 10. If xy + yz + zx = 1, then ∑ =
 8  8 1 − xy
(A) a4 > a3 > a2 > a1 (B) a3 > a4 > a2 > a1 4 1
(A) (B) 
(C) a4 > a3 > a1 > a2 (D) a3 > a1 > a2 > a4 xyz xyz
3. If x cos23θ + y cos4θ = 16 cos6θ + 9 cos2θ be an iden- (C) xyz (D)  none of these
tity, then 11. cos 12° cos 24° cos 36° cos 48° cos 72° cos 96° equals
(A) x = –1, y = 24 (B)  x = 1,  y = 24 1 1
(C) x = 24, y = 1 (D)  none of these (A) − 6 (B) 
2 28
4. |tanθ + secθ | = |tan θ | + |secθ |, 0 ≤ θ ≤ 2π is possible 1 1
only if (C) 7 (D)  − 7
2 2
π 
(A) θ ∈ [0, π ] −   (B)  θ ∈ [0, p]  π sin (α + β + γ )
2 12. If α , β , γ ∈  0,  , then is
 2 sin α + sin β + sin γ

PRACTICE EXERCISES
 π
(C) θ ∈ 0,  (D)  none of these (A) < 1 (B)  > 1
 2
(C) = 1 (D)  none of these
5. If sinθ, sinφ and cosθ are in G.P., then the roots of the
equation x2 + 2x cot φ + 1 = 0 are always {
13. If cos θ sin θ + sin 2θ + sin 2α } ≤ k , then the value
(A) real (B)  imaginary of k is
(C) equal (D)  greater than 1
(A) 1 + cos 2 α (B) 1 + sin 2α
6. If cos 25° + sin 25° = k, then cos 50° is equal to
(C) 2 + sin 2α (D) 2 + cos 2α
− 2−k
(A) k 2 − k (B) 2 2

14. The maximum value of (cos a1) (cos a2) . . .(cos an)
π
−k 2 − k2
(C) 2 − k 2 (D)  under the restrictions 0 ≤ a1, a2, . . . , α n ≤ and (cot
a1) (cot a2) . . . (cot an) = 1 is 2
2 sin α 1 − cos α + sin α
7. If = x then =… 1 1
1 + cos α + sin α 1 + sin α (A) n (B) 
2 2 2n
1
(A) (B)  x 1
x (C) (D) 
1
(C) 1 – x (D) 
1+x 2n
15.12  Chapter 15

 1 
1−  7 θ 
15. The inequality 2sin θ + 2cos θ ≥ 2 2  holds for 23. If sin θ + cos θ = and 0 < θ < π/6, then tan  
2 2
(A) 0 ≤ θ < π (B)  π ≤ θ < 2π equals
(C) for all real θ (D)  none of these 1
(A) 7 − 2 (B)  ( 7 − 2)
16. The expression 2sinθ + 2–cosθ is minimum when θ is 3
equal to 1
(C) 2 − 7 (D)  (2 − 7 )
π 7π 3
(A) 2nπ + , n ∈ I (B) 2nπ + , n∈ I
4 4
π  π
(C) nπ ± , n ∈ I (D)  none of these 24. If sin (θ + α) = a and sin (θ + β = b  0 < α , β , θ < 
4  2
then cos (α – β) – 4ab cos (α – β) is equal to
2

1 + xn  1 − x02 
17. If xn +1 = , then cos   ( −1 < x0 < 1) (A) 1 – 2a2 – 2b2 (B)  1+ 2 a2 + 2b2
2  x1 x2 x3 ....to ∞ 
is equal to   (C) 1– a2 – b2 (D)  none of these
(A) x0 (B)  1/x0 25. If sin x + cosec x + tan y + cot y = 4, where x and
(C) 1 (D)  –1
 π y
18. If 0 < θ < π, then y ∈ 0,  , then tan is a root of the equation
 2  2
θ θ
(A) 1 + cot θ ≤ cot (B)  1 + cot θ ≥ cot (A) a2 2+ 2α + 1 = 0 (B)  a2 + 2α – 1 = 0
2 2
(C) 2a – 2α – 1 = 0 (D)  none of these
θ θ
(C) 1 + cot ≥ cot θ (D) 1 + cot ≤ cot θ 26. The value of 2 sin2θ + 4 cos (θ + α) sin α sin θ + cos 2
2 2
(α + θ) is
19. If cos (θ – α) = a and sin (θ – β) = b (0 < θ – α, θ –β <
π/2), then cos2 (α – β) + 2ab sin (α –β) is equal to (A) cosθ + cosα (B)  independent of θ
(C) independent of α (D)  none of these
(A) a2 – b2 (B)  a2 + b2
(C) 2a b (D) 
2 2
a2 b2 27. The value of cos θ ⋅ cos 2θ ⋅ cos 22θ ... cos 2n – 1θ for
A B C π
20. If in the triangle ABC , tan
, tan and tan are in θ= n is
2 2 2 2 +1
B 1
harmonic progression, then the least value of cot is (A)
1 (B) 
2 2n
PRACTICE EXERCISES

(A) 2 (B)  3 (C) 2n (D)  none of these


(C) 2 (D)  none of these
21. If x sin a + y sin 2a + z sin 3a = sin 4a x sin b + 28. The sum of the series sinθ ⋅ sec3θ + sin 3θ ⋅ sec32θ +
y sin 2b + z sin 3b = sin 4b x sin c + y sin 2c + z sin 32θ sec33θ + ... up to n terms is
sin 3c = sin 4c, then the roots of the equation 1
z y+2 z−x (A) (tan 3n θ − tan θ ) (B) (tan 3nθ – tanθ)
t3 − t2 − + = 0; a, b, c ≠ nπ; are 2
2 4 8 (C) tan3nθ – tan3n – 1θ (D)  none of these
(A) sin a, sin b, sin c
(B) cos a, cos b, cos c 29. If x1, x2, x3, ..., xn are in A.P. whose common difference
(C) sin 2a, sin 2b, sin 2c is α, then the value of sin α [sec x1 sec x2 + sec x2 sec
(D) cos 2a, cos 2b, cos 2c x3 + … + sec xn–1 sec xn] is equal to
22. If a sin x + b cos (x + θ) + b cos (x – θ) = d, then the sin nα sin ( n −1) α
minimum value of | cosθ | is (A) (B) 
cos x1 cos xn cos x1 cos xn
1 1
(A) d 2 − a 2 (B)  d 2 − a2 sin ( n +1) α
2 |a| 2 |b| (C) (D)  none of these
cos x1 cos xn
1
(C) a 2 − d 2 (D)  none of these
2 |b|
Trigonometric Ratios and Identities  15.13

Previous Year's Questions


A− B+C 24 24
30. In a triangle ABC, 2ca sin is equal to: (A) −
(B) 
2 25 25
 [2002] 13 13
(A) a2 + b2 − c2 (B)  c2 + a2 − b2 (C) (D)  −
18 18
(C) b2 − c2 − a2 (D)  c2 − a2 − b2 C  A  3b
40. If in a triangle ABC a cos 2   + c cos 2   = then
4 xy  2  2  2,
31. sin θ = ( x + y ) 2 is true if and only if:
2
[2002] the sides a, b and c [2003]
(A) x + y ≠ 0 (B)  x = y, x ≠ 0, y ≠ 0 (A) are in A.P. (B)  are in G.P.
(C) x = y (D)  x ≠ 0, y ≠ 0 (C) are in H.P. (D)  satisfy a + b = c
1 − tan 2 15° 21
32. The value of is: [2002] 41. Let α, β be such that π < α − β < 3π. If sin α + sin β = −
1 + tan 2 15° 65
27 α −β

(A) 3
1 (B)  and cosα + cos β = − , then the value of cos is
65 2
3
(C) (D) 
2  [2004]
2
3 3
4 (A) − (B) 
33. If tanθ = − , then sinθ is: [2002] 130 130
3
6 6
4 4 4 4 (C) −
(D) 
(A) − but not − or
(B)  65 65
5 5 5 5
4 4
(C) but not − (D)  none of these 42. If u = a 2 cos 2 θ + b 2 sin 2 θ + a 2 sin 2 θ + b 2 cos 2 θ , then
5 5
the difference between the maximum and minimum
1 values of u2 is given by [2004]
34. If sin (α + β ) = 1, sin (α − β ) = , then tan(α + 2β) tan
2
(A) 2 ( a 2 + b 2 ) (B) 
2 a2 + b2
(2α + β) is equal to: [2002]
(C) (a + b)2 (D)  (a − b)2
(A) 1 (B)  −1
(C) zero (D)  none of these 43. The sides of a triangle are sinα, cosα and 1 + sin α cos α
35. If y = sin θ + cosec θ, θ ≠ 0 then:
2 2
[2002] π
for some 0 < α < . Then the greatest angle of the tri-
(A) y = 0 (B)  y≤2 2
angle is [2004]
(C) y ≥ −2 (D)  y≥2

(A) 60° (B)  90°

PRACTICE EXERCISES
36. In a triangle ABC, a = 4, b = 3, ∠A = 60°, then c is the
(C) 120° (D)  150°
root of the equation: [2002]
π  P  Q
(A) c2 − 3c − 7 = 0 (B)  c2 + 3c + 7 = 0 44. In a triangle PQR, ∠ R = . If tan  and tan  are
2  2  2
(C) c2 − 3c + 7 = 0 (D)  c2 + 3c − 7 = 0
the roots of ax2 + bx + c = 0, a ≠ 0 then [2005]
A 5 C 2
37. In a ∆ABC , tan = , tan = , then: [2002] (A) a = b + c (B)  c=a+b
2 6 2 5 (C) b = c (D)  b=a+c
(A) a, c, b are in AP (B)  a, b, c are in AP
(C) b, a, c are in AP (D)  a, b, c are in GP π
45. In a triangle ABC, ∠ C = If r is the inradius and R is
2
38. The equation a sin x + b cos x = c where | c | > a 2 + b 2 the circumradius of the the triangle ABC, then 2 (r +
has: [2002] R) equals [2005]
(A) a unique solution (A) b + c (B)  a+b
(B) infinite number of solutions (C) a + b + c (D)  c+a
(C) no solution 46. If the roots of the quadratic equation x2 + px + q = 0 are
(D) none of the above tan 30° and tan l5°, respectively then the value of 2 +
π q − p is [2006]
39. If α is a root of 25 cos 2 θ + 5 cosθ − 12 = 0 < α < π then (A) 2 (B)  3
2
sin 2α is equal to : [2002] (C) 0 (D)  1
15.14  Chapter 15

47. The number of values of x in the interval [0, 3π] satis- 53. If A = sin2x + cos4x then, for all real values of x,
fying the equation 2 sin2x + 5sinx − 3 = 0 is [2006]  [2011]
13
(A) 4 (B)  6 (A) ″ A ″ 1 (B) 
1 ″ A″ 2
16
(C) 1 (D)  2
3 13 3
48. A triangular park is enclosed on two sides by a fence (C) ″ A ″ (D)  ″ A″ 1
4 16 4
and on the third side by a straight river bank. The two
sides having fence are of same length x. The maximum 54. In a ΔPQR, if 3 sin P + 4 cos Q = 6 and 4 sin Q + 3 cos
area enclosed by the park is [2006] P = 1, then the angle R is equal to [2012]
3 x 3
2
(A) x (B)  5π π
2 8 (A) ⋅ (B)  ⋅

6 6
1 2
π x2
(C) 2 x (D)  π 3π
(C) (D) 
⋅ ⋅

4 4
1
49. If 0 < x < and cos x + sin x = , then tan x is [2006]
2 55. ABCD is a trapezium such that AB and CD are parallel
(1 − 7 ) (4 − 7 ) and BC ⊥ CD . If angle ADB = B , BC = P and CD = q,
(A) (B)  then AB is equal to [2013]
4 3
p 2 + q 2 cosθ p2 + q2
(4 + 7 ) (1 + 7 ) (A) (B) p 2 cosθ + q 2 sin θ
(C) − (D)  p cosθ + q sin θ
3 4
50. Let A and B denote the statements [2009] ( p 2 + q 2 )sin θ ( p 2 + q 2 )sin θ
(C) (D) 
( p cosθ + q sin θ ) 2 p cosθ + q sin θ
A: cosα + cosβ + cosλ = 0
B: sinα + sinβ + sinλ = 0
tan A cot A
3 56. The expression + can be written as
If cos( β − λ ) + cos( β − α ) + cos(α − β ) = − , then
1 − cot A 1 − tan A
2  [2013]
(A) A is true and B is false (A) secA cosecA + 1 (B)  tanA + cotA
(B) A is false and B is true (C) secA + cosecA (D) sinA cosA + 1
(C) both A and B are true
(D) both A and B are false 1
57. Let f k ( x ) = (sin k x + cos k x ) where x ∈ R and k ≥ 1 then,
51. Let k
4 5 π the value of f 4 ( x ) − f 6 ( x ) equals [2014]
cos(α + β ) = and sin (α − β ) = , where 0 ≤ α , β ≤ ,
PRACTICE EXERCISES

5 13 4
1 1
then tan 2α = [2010] (A) (B) 
6 3
56 19
(A) (B)  1 1
33 12 (C) (D) 
4 12
20 25
(C) (D) 
7 16 58. If 5(tan2x – cos2x) = 2 cos 2x + 9, then the value of cos
52. For a regular polygon, let r and R be the respective 4x is [2017]
radii of the inscribed and the circumscribed circles. A 1 2 7 3
(A)  (B)  (C) – (D) –
false statement among the following is [2010] 3 9 9 5
r 1
(A) There is a regular polygon with =
R 2
r 2
(B) There is a regular polygon with =
R 3

r 3
(C) There is a regular polygon with =
R 2
r 1
(D) There is a regular polygon with =
R 2
Trigonometric Ratios and Identities  15.15

ANSWER K EYS
Single Option Correct Type
1. (B) 2. (C) 3. (B) 4. (A) 5. (A) 6. (A) 7. (B) 8. (B) 9. (C) 10. (B)
11. (A) 12. (A) 13. (B) 14. (A) 15. (C) 16. (B) 17. (A) 18. (A) 19. (B) 20. (B)
21. (B) 22. (B) 23. (B) 24. (A) 25. (B) 26. (B) 27. (B) 28. (A) 29. (B)

Previous Years’ Questions


30. (B) 31. (B) 32. (C) 33. (B) 34. (A) 35. (D) 36. (A) 37. (A) 38. (C) 39. (B)
40. (A) 41. (A) 42. (D) 43. (C) 44. (B) 45. (B) 46. (B) 47. (A) 48. (C) 49. (C)
50. (C) 51. (A) 52. (B) 53. (D) 54. (B) 55. (D) 56. (A) 57. (D) 58. (C)

PRACTICE EXERCISES
15.16  Chapter 15

HINTS AND EXPLANATIONS

Single Option Correct Type


1. Let P = cos a cos 2a cos 3a ... cos (999a) ⇒ sinθ ≥ 0 and cosθ ≠ 0

Let Q = sin a sin 2a sin 3a ... sin (999 a) π 
⇒ θ ∈ [ 0, π ] −  
Thus, 2999 PQ = 2 sin a cos a) (2 sin 2a cos 2a) 2
(2 sin 3a cos 3a)... 5. We have, sinθ, sinφ and cosθ are in G.P.
(2 sin (999 a) cos (999 a)
1 − cos 2φ sin 2θ
= sin 2a sin 4a sin 6a ... sin (1998 a) ⇒ sin 2 φ = sin θ cosθ ⇒ =
2 2
= (sin 2a sin 4a sin 6a ... sin 998 a)
⇒  cos 2φ = 1 – sin 2θ ≥ 0.
(i)
[–sin (2π – 1000 a)] [–sin (2π – 1002a)]....

Now, the discriminant of the given equation is
[–sin (2π –1998a)]
= sin 2a sin 4a ... sin 998 a sin 999 a  cos 2 φ − sin 2 φ 
4 cot 2 φ − 4 = 4  
sin 997 a ... sin a  sin 2 φ 
= 4 cos 2φ × cosec2φ ≥ 0

= Q [ ∴ 2π = 1999a] (   of (i))
Clearly, Q ≠ 0 ∴  Roots of the given equation are always real.
1 6. We have, cos 50° = cos225° – sin225°

∴ 2999 PQ = Q ⇒ P = 999
2 = (cos 25° + sin 25°) (cos25° – sin 25°)
2. For α >1, the function y = ax is an increasing function = k(cos25° – sin 25°).
But (cos 25° + sin 25°)2 + (cos25° – sin 25°)2 = 2
π π
Since 0 <
<
8 4 ⇒ cos 25° − sin 25° = 2 − k 2

π π π
(As cos 25° – sin 25° is positive)
∴ cot > cot > tan
8 4 8
∴ cos 50° = k 2 − k 2 .
⇒ a4 > a3(1)

For α < 1, the function y = ax is a decreasing function.
2 sin α
7. Given, x =
1 + cos α + sin α
HINTS AND EXPLANATIONS

∴  a2 < a1(2)

⇒  x(1 + cosα + sinα) = 2 sinα

π π
Again, cot
> 1 > tan > 0 ⇒  x(1 + cosα) = (2 – x) sinα

8 8
1 − cos α + sin α x 2 sin α (1 + sin α )
∴  a1 < 1 and a3 > 1  ∴  a1 < a3
(3) Let y =
. Now =
1 + sin α y (1 + sin α ) 2 − cos 2 α

From (1), (2) and (3), we get
a2 < a1 < a3 < a4
x 2 sin α (1 + sin α )
⇒ = =1
3. We have, x cos2 3θ + y cos4θ  = 16 cos6θ + 9 cos2θ y 1 + sin 2 α + 2 sin α − 1 + sin 2 α
⇒  x(4 cos3θ – 3 cosθ)2 + y cos4θ ⇒  x = y

= 9 cos2θ  + 16 cos6θ 8. We have, e–π/2 < θ < π/2
⇒  x(16 cos6θ + 9 cos2θ – 24 cos4θ) + y cos4θ π π
⇒ − log e < log θ < log
= 9 cos2θ  + 16 cos6θ 2 2
⇒ 9x cos2θ + (y – 24x) cos4θ + 16x cos6θ π π
Now, log e e = 1 and
< e, ∴ log < log e = 1
= 9 cos2θ + 16 cos6θ 2 2
On comparing, we get
π π
9x = 9 and y – 24x = 0
∴ − < log θ < 1 <
2 2
∴  x = 1 and y = 24.
⇒ log θ lies in 1st and 4th quadrant

4. | tan θ + sec θ | = | tan θ | + | sec θ | only if sec θ and tanθ both
∴  cos (logθ) is positive
(1)
have same sign
Now, 0 < cosθ < 1  ⇒  log cosθ < log 1 = 0

sin θ
i.e., secθ ⋅ tanθ ≥ 0 or
≥0 ⇒  log cosθ is negative
(2)
cos 2 θ
From (1) and (2), we conclude cos logθ > log cosθ.

Trigonometric Ratios and Identities  15.17

9. Put t =
x
y
>0 {
13. Let u = cosθ sin θ + sin 2 θ + sin 2α }
⇒ (u – sinθ cosθ)2 = cos2θ (sin2θ + sin2α)

1  1
2
⇒  u2tan2θ – 2u tanθ + u2 – sin2α = 0

t+ = t −  +2≥2
t  t Since tan θ is real, therefore

4u2 – 4u2 (u2 – sin2α) ≥ 0

equality holding iff t = 1

⇒  u2 – (1 + sin2α) ≤ 0

1
Also, t + = 2 sin θ ≤ 2, so that t should necessarily be 1. ⇒ | u | ≤ 1 + sin 2 α
t
i.e.,  x = y. 14. cos α1 cos α2 ... cos αn
10. Put x = tan A, y = tan B, z = tan C = sin α1 sin α2 ... sin αn (given)
∴ tan A tan B + tan B tan C + tan C tan A = 1
2 (sin α1 cos α1 )… 2 (sin α n cos α n )
⇒ tan C[tan A + tan B] = 1 – tan A tan B ⇒ cos 2 α1 …cos 2 α n =
2n
x+ y tan A + tan B
Now, ∑
=∑ sin 2α1 …sin 2α n
1 − xy 1 − tan A tan B ⇒ cos 2 α1 …cos 2 α n =
2n
tan A + tan B tan B + tan C tan C + tan A (sin 2α1 …sin 2α n )1/2
= + + ⇒ cos α1 …cos α n =
1 − tan A tan B 1 − tan B tan C 1 − tan C tan A 21/2
Since, maximum value of sin α = 1

1 1 1
= + + 1
tan C tan A tan B So, maximum value of cos α1 …cos α n =

2n / 2
tan A tan B + tan B tan C + tan C tan A 15. We have,
=
tan A tan B tan C 1 sin θ
[2 + 2cos θ ] ≥ 2sin θ ⋅ 2cos θ
1 1 2
= =
[ A.M. ≥ G.M.]
tan A tan B tan C xyz
⇒ 2sin θ + 2cos θ ≥ 2. 2(sinθ + cosθ)/2(1)

π 2π 3π 4π 6π 8π  π 
11. cos cos cos cos cos cos = 12°   π
15 15 15 15 15 15  15  Now, (sin θ + cosθ ) = 2 sin  θ + 

 4
 π 2π 4π 8π  3π  3π  ≥ − 2 for all real θ.
=  cos cos cos cos  cos cos 2  
 15 15 15 15  15  15  ∴ 2sinθ + 2cosθ ≥ 2 ⋅ 2(sinθ + cosθ)/2

HINTS AND EXPLANATIONS


π   3π  > 2 ⋅ 2− 2 /2
= 21− (1 / 2)
sin 24   sin 22  
 15  1  15 
= × 2 Thus, 2sin θ + 2cos θ ≥ 21− (1 /
2)
, for all real θ.
4 π 2  3π 
2 sin sin   16. Since A.M. ≥ G.M.
15  15 
2sin θ + 2− cos θ
16π  12π  ∴
≥ 2sin θ ⋅ 2− cos θ
sin sin   2
15 1  15 
= × 2
1
1+ (sin θ − cos θ )
π 2 3π ⇒ 2sin θ + 2− cos θ ≥ 2 2
16 sin sin
15 15 π
1+ 2 sin θ − 4 
1

=2
 π   3π 
sin  π +  sin  π − Thus, 2sinθ + 2– cosθ is minimum

1  15   15  1
= × =− 6  π π 3π
64 π 3π 2 when sin  θ −  = −1 i.e., θ − = 2nπ +

sin sin 4 4 2
15 15 

12. We have, sin α + sin β + sin γ – sin (α + β + γ) or, θ = 2nπ +
, n∈ I.
4
= sin α + sin β + sin γ – sin α cos β cos γ
– cos α sin β cos γ – cos α cos β sin γ + sin α sin β sin γ 1
17. Let x0 = cosθ, then x1 = (1 + cosθ )
= sin α[1 – cos β cos γ] + sin β [1 – cos α cos γ] 2
+ sin γ [1 – cos α cos β] + sin α sin β sin γ > 0 = cos θ/2, x2 = cos (θ/22), x3 = cos (θ/23),...and so on.

∴ sin α + sin β + sin γ > sin (α + β + γ)
 1 − x02 

∴  
sin (α + β + γ )
⇒ <1  x1 x2 x3 …to ∞ 
sin α + sin β + sin γ
15.18  Chapter 15

sin θ = a 2 (1 − b 2 ) + b 2 (1 − a 2 ) + 2ab (1 − a 2 ) (1 − b 2 )
=
θ θ θ
cos cos 2 … cos n … ∞
2 2 2 + 2a 2b 2 − 2ab (1 − a 2 ) (1 − b 2 ) = a 2 + b 2 .
θ θ 20. Given, A + B + C = π
2 sin cos
= 2 2 A B C A B C
θ θ θ ⇒ cot cot cot = cot + cot + cot (i)
cos cos 2 … cos n … ∞ 2 2 2 2 2 2
2 2 2
A B C A B C
But tan , tan , tan are in H. P. ⇒ cot , cot , cot

θ θ 2 2 2 2 2 2
22 sin
2
cos 2
= 2 2
θ θ θ A C B
cos 2 cos 3 … cos n … ∞ are in A. P. ⇒ cot
+ cot = 2 cot (ii)
2 2 2 2 2 2
A B C B
θ From (i) and (ii), we get cot ⋅ cot ⋅ cot = 3 cot

2n sin 2 2 2 2
2n
= lim θ
n→∞ A C
cos n+1 ∴ cot
⋅ cot = 3 (iii)
2 2 2
 θ  A C
cot + cot
 sin 2n  1
Now, 2 2 ≥ cot A cot C
= lim θ   =θ
 θ  cos θ 2 2 2
n→∞
n +1
 2n  2 B
2 cot
 1− x  2 ⇒ 2 ≥ 3 [From (ii) an (iii)]
∴ cos 
 = cosθ = x0 .
0 2
 x1 x2 … ∞  B
∴ cot ≥ 3
2
θ 21. The first equation can be written as x sin a + y × 2 sin a cos
cot 2 − 1
θ 2 θ a + z sin a(3– 4 sin2 a)
18. 1 + cot θ − cot = 1 + − cot
2 θ 2 = 2 × 2 sin a cos a cos 2a
2 cot
2 ⇒  x + 2y cos a + z (3 + 4 cos2 a – 4)
θ θ
2
θ  cot θ − 1 = 4 cos a (2 cos2 a –1) as sin a ≠ 0
+ cot 2 − 1 − 2 cot 2  2 
2 cot ⇒  8 cos3 a – 4z cos2 a – (2y + 4) cos a + (z – x) = 0
2 2 2 =
HINTS AND EXPLANATIONS

= ≤0
θ θ z y+2 z−x
2 cot 2 cot ⇒ cos3 a − cos 2 a − cos a + =0
2 2 2 4 8
 θ π
which shows that cos a is root of the equation

∵ 0 < 2 < 2 
  z y+2 z−x
t3 − t2 − t+ =0
θ 2 4 8
⇒ 1 + cot θ ≤ cot
2 Similarly, from second and third equations we can verify that
cos b and cos c are the roots of the above equation
19. We have,
22. a sin x + b cos(x + θ) + b cos(x – θ) = d
sin(α – β) = sin(α – θ + θ – β)
⇒  a sin x + 2b. cos x. cos θ = d
= sin[ (θ – β) – (θ – α)]
= sin(θ – β) cos(θ – α) – cos(θ – β) sin (θ – α) ⇒ | d | ≤ a 2 + 4b 2 ⋅ cos 2 θ d 2 − a2
⇒ ≤ cos 2 θ
4b 2
= ba − 1 − b 2 1 − a 2
d 2 − a2
and cos(α – β) = cos[θ – β) – (θ – α)]
⇒ |cosθ | ≥
2 |b|
= cos(θ – β) cos(θ – α) + sin(θ – β)

23. From the given condition, we have
sin(θ – α)

2 tan (α / 2) 1 − tan 2 (α / 2) 7
= a 1 − b2 + b 1 − a2
+ =
1 + tan (α / 2) 1 + tan 2 (α / 2)
2
2

Substituting these values in the given expression, we get  α α  α
⇒ 2  2 tan + 1 − tan 2  = 7 1 + tan 2 
cos2(α – β) + 2ab sin(α – β)
 2 2   2
= ( a 1 − b 2 + b 1 − a 2 ) 2 + 2ab [ab − (1 − a 2 ) (1 − b 2 )] α α
⇒ ( 7 + 2) tan 2 − 4 tan + ( 7 − 2) = 0.
2 2
Trigonometric Ratios and Identities  15.19


This quadratic has the roots = 2sin θ (sin 2θ ⋅ cos 2θ ⋅ cos22θ ... cos2n – 1θ)

α 4 ± 16 − 4 ( 7 + 2)( 7 − 2) 4±2 1
tan = = = ( 2 sin 2θ ⋅ cos 2θ ⋅ cos 22θ … cos 2n− 1θ )
2 2( 7 + 2) 2 ( 7 + 2) 22 sin θ
1
That is, tan (α / 2) = 3 /( 7 + 2) or 1/( 7 + 2). The given
= 2 (sin 22θ cos 22θ … cos 2n− 1θ )
2 sin θ
condition on α implies 0 < α/2 < π/12. Therefore, we get

..........................................
α π α
..........................................
0 < tan < tan ⇒ 0 < tan < 2 − 3.
2 12 2
1

Only the second root satisfies this condition, because = sin 2n − 1θ ⋅ cos 2n − 1θ
2n−1 sin θ
3
= 7 −2>2− 3 1 1
7+2 = ( 2 sin 2n− 1θ cos n−1 θ ) = n sin 2nθ
2n sin θ 2 sin θ
1 1

and, = ( 7 − 2) < 2 − 3. 1  π 
7+2 3 = [sin (π − θ )] ∵ θ = n , ∴ 2nθ + θ = π 
2n sin θ  2 +1 
1
Hence, the required value of tan (α/2) is
( 7 − 2). 1 1
3 = n ⋅ sin θ = n .
2 sin θ 2
24. Using,
cos(α – β) = cos[(α + θ) – (θ + β)] θ
2 tan
28. We have, tan θ = 2
= cos(θ + α) cos(θ + β) + sin(θ + α) sin(θ + β)

1 − tan
= 1 − a 2 1 − b 2 + ab, 2


the given expression can be written as 2
θ θ
2 cos2 (α – β) – 1 – 4ab cos(α – β)
cot 2 cot
1 2 2
⇒ = =
= 2( 1 − a 2 1 − b 2 + ab) 2 cot θ 1 − 1 2θ
cot − 1
θ 2
−1 − 4 ab ( 1 − a 2 1 − b 2 + ab) cot 2
2
= 2 [ (1 – a2) (1 – b2) – a2b2] – 1 = 1 − 2a2 − 2b2. θ
25. Given that sin x + cosec x + tan y + cot y = 4 −1 cot 2
⇒ cot θ = 2
π π θ

HINTS AND EXPLANATIONS


⇒ x= and y = 2 cot
2 4 2
⇒ tan y = 1
θ
cot 2 − 1
θ θ 2
2 tan y / 2 y y Now, cot − 1 − cot θ = cot − 1 −

⇒ = 1 ⇒ tan 2 + + 2 tan − 1 = 0. 2 2 θ
1 − tan 2 y / 2 2 2 2 cot
2
26. We have, θ θ θ
2 cot 2 − 2 cot − cot 2 + 1
2sin2θ + 4 cos(θ + α) sinα sinθ + cos 2(α + θ) = 2 2 2
= 2sin2θ + 2cos(θ + α) 2sinα sinθ + cos 2(α + θ) θ
2 cot
= 2sin2θ + 2cos(θ + α) [cos(θ – α) – cos(θ + α)] 2
2
+ cos 2(α + θ) θ θ  θ 
cot 2 − 2 cot + 1  cot − 1
= 2sin2θ + 2cos(θ + α) cos(θ – α) 2 2  2  .
= =
– 2cos2(θ + α) + cos2(α + θ) θ θ
2 cot 2 cot
= 2sin2θ + 2(cos2θ – sin2α) – 2cos2(θ + α) 2 2
+ [2 cos2(α + θ) – 1] θ π θ
Now, since 0 < θ < π, ∴ 0 < <
⇒ cot cot is
= 2sin2θ + 2cos2θ – 2sin2α – 1 positive. 2 2 2
= 2(sin2θ + cos2θ) – 2sin2α – 1
Also, square of a real number ≥ 0
= 2 – 2sin2α – 1 = 1 – 2sin2α = cos2α,
θ θ
which is independent of θ. ∴ cot
− 1 − cot θ ≥ 0 ⇒ cot ≥ 1 + cot θ .
2 2
27. We have, cos θ ⋅ cos 2θ ⋅ cos 22θ ... cos 2n – 1θ
sin α sin ( x2 − x1 )
1 29. We have, =
= ( 2 sin θ ⋅ cosθ ⋅ cos 2θ ⋅ cos 22θ … cos 2n − 1θ ) cos x1 cos x2 cos x1 cos x2
2 sin θ
15.20  Chapter 15

... + (tan x – tan x ]



sin x2 cos x1 − cos x2 sin x1 n n –1
= = tan xn – tan x1

cos x1 cos x2
= tan x2 – tan x1.
sin xn cos x1 − cos xn sin x1
=
∴ sin α [sec x1 sec x2 + sec x2 sec x3 + ... to n terms]
cos xn cos x1
= (tan x2 – tan x1) + (tan x3 – tan x2) +
sin ( xn − x1 ) sin ( n − 1) α
= =
cos x1 cos xn cos x1 cos xn

Previous Year's Questions

30. We know that: ∴ h=


p 2 + b 2 = 16 + 9 = 25
The sum of angles A + B + C = π ⇒h=5

⇒A + C = π − B p 4
A− B +C π ∴ sin θ =
=
⇒ = −B h 5
2 2
But tanθ is negative which is possible only if θ lies in second

 A− B +C  π  or fourth quadrant.
∴ 2ca sin 
 = 2ca sin  2 − B 
 2    4 4
∴ sin θ may be or = − .

5 5
 a2 + c2 − b2 
= 2ac cos B = 2ac  
 2ac 34. ∵ sin(α + β ) = 1
=a +c −b
2 2 2
π
31. ∵ sin θ < 1, ⇒ sin(α + β ) = sin
2
sin 2θ < 1 π
⇒ α +β = (i)
4 xy 2
⇒ ≤1
( x + y)2 1
and sin (α − β ) =

⇒ 0 ≤ ( x + y ) − 4 xy
2 2
π
⇒ x 2 + y 2 + 2 xy − 4 xy ≥ 0 ⇒ α −β = (ii)
6
⇒ ( x − y)2 ≥ 0
Solving eqs. (i) and (ii), we find
HINTS AND EXPLANATIONS

which is true for all real x and y provided x + y ≠ 0, otherwise



4 xy ∴ tan(α + 2 β ) tan( 2α + β )

will be meaningless.
( x + y)2  2π   5π 
= tan   tan  6 
 3   
Info Box!  π  π
=  − cot   − cot 
4 xy  6  3
sec2θ = is possible only, if x = y.
( x + y )2 2
= 3× =1
3

35. ∵ y = sin θ + cos ec θ


2 2
32. Key Idea: Cosine in terms of tangent
1 − tan 2 θ = (sin θ − cos ecθ ) 2 + 2
cos 2θ =
1 + tan 2 θ
⇒ y≥2
1 − tan 2 15°
= cos 30° But at θ = 0, y becomes meaningless

1 + tan 2 15°
∴y≥2

3 b2 + c2 − a2
∴ =
36. Using cos A =
2 2bc
33. Key Idea: tanθ is negative in second and fourth quadrants. We write for a = 4, b = 3 and ∠A = 60°

4
∴ tan θ = − c 2 + 9 − 16
3 ∴ cos60° =

2×3×c
Here p = 4 and b = 3
Trigonometric Ratios and Identities  15.21

1 c2 − 7 −21 −27
⇒ = 41. Given that sin α + sin β =
and cos α + cos β = .
2 2 × 3c 65 65

Squaring and adding, we get
⇒ c2 − 7 = 3c

⇒ c2 − 3c − 7 = 0
1170
2 + 2 cos(α − β ) =

Thus, c is the root of above equation. (65) 2

A ( s − b)( s − c) α − β  9 α − β 
= ⇒ cos 2   = 130 ⇒ cos  2 
37. Key Idea: tan  2   
2 s( s − a )
−3  π α − β 3π 
A C 5 2 = ∵ 2 < 2 < 2  .

Now tan tan = × 130  
2 2 6 5
( s − b)( s − c) ( s − a)( s − b) 1 42. We have u = a 2 cos 2 θ + b 2 sin 2 θ + a 2 sin 2 θ + b 2 cos 2 θ
⇒ ⋅ =
s( s − a ) s( s − c ) 3
s−b 1 a2 + b2 a2 − b2 a2 + b2 b2 − a2
⇒ =
= + cos 2θ + + cos 2θ
s 3 2 2 2 2
⇒ 3s − 3b = s 2 2
 a2 + b2   a2 − b2 
⇒ 2s = 3b ⇒ u 2 = a2 + b2 + 2   −  cos 2θ
2

⇒ a + b + c = 3b  2   2 
⇒ a + c = 2b Now, min value of u2 = a2 + b2 + 2ab

a, b, c are in AP. And the max. value of u2 = 2(a2 + b2)

38. Using − a 2 + b 2 ≤ a sin x + b cos x ≤ a 2 + b 2 . ⇒ umax


2
− umin
2
= ( a − b) 2 .

Given | c | > a 2 + b 2 which inplies


43. Greatest side is 1 + sin α cos α , by applying cosine rule we
get the greatest angle = 120°.
c = a sin x + b cos x > a + b , or
2 2

c = a sin x + b cos x < − a 2 + b 2  P  Q


44. tan   , tan   are the roots of ax2 + bx + c = 0
which are reverse inequalities. Then no solution exists for  2  2
a sin x + b cos x = c.  P  Q b
tan   + tan   = −
39. ∵ θ = α satisfies the equation 25 cos2θ + 5 cosθ − 12 = 0  2  2 a

HINTS AND EXPLANATIONS


∴ 25cos2α + 5cosα − 12 = 0
 P  Q c
⇒ 25cos2α + 20cosα 15cosα − 12 = 0 tan   tan   =
 2  2 a
⇒ 5cosα(5cosα + 4) − 3(5cosα + 4) = 0
P Q
4 3 tan   + tan  
⇒ cos α = − , 2
   2  = tan  P + Q  = tan  π − R  = 1
3 5 2 2  2 
  Q
P    
π 1 − tan   tan  

But <α <π 2 2
2
b
4 −
∴ cos α = − (∵ cos α < 0)
⇒ a =1 ⇒ − b = a − c ⇒ −b = a−c
5
c a a a
3 1−
⇒ sin α = b
5 ⇒ c = a + b
3 4 45. We have
sin 2α = 2 sin α cos α = −2 × ×
5 5 2ab ( a + b ) 2 + c( a + b )
24 2r + 2 R = c + =
∴ =−
( a + b + c) ( a + b + c)
25 = a + b (since c 2 = a 2 + b 2 )
 1 + cos C   1 + cos A  3b
40. a 
  + c   = 46. x2 + px + q = 0
2 2 2
⇒ a + c + b = 3b Sum and product of the roots respectively are
⇒ a + c = 2b tan 30° + tan 15° = −p
Hence, (A) is the correct answer tan 30° .tan 15° = q
15.22  Chapter 15

tan 30° + tan15° −p a π


∴ tan 45 ° =
= =1 R= cos ec
1 − tan 30° tan15° 1 − q 2 n
⇒−p=1−q
π
⇒ q − p = 1 ∴ 2 + q − p = 3. cot
r n = cos π
47. 2 sin2x + 5 sin x − 3 = 0 =
R π n
cos ec
⇒ (sin x + 3) (2 sin x − 1) = 0 n
1 π 2
⇒ sin x = cos ≠ for any n ∈ N.
2. n 3
Now, in the interval [0, 3π] x has 4 values

7 + cos 4 x 3
1 2 53. A = sin 2 x + cos 4 x = ⇒ ≤ A≤1
48. Area = x sin θ 8 4
2
1 2 π 54. 3sinP + 4cosQ = 6 (1)
Now, Amax =
x at sin θ = 1, θ = 
2  2 4sinQ + 3cosP = 1 (2)
From (1) and (2) P is obtuse.
(3sin P + 4cos Q)2 + (4sin Q + 3cosP)2 = 37
⇒ 9 + 16 + 24 (sinP cosQ + cosP sinQ) = 37
49. ⇒ 24sin (P + Q) = 12
1 1 3
cos x + sin x = ⇒ 1 + sin 2 x = ⇒ sin 2 x = − , 1
2 4 4 ⇒ sin( P + Q ) =
so x is obtuse. 2

2 tan x 3 ⇒ P+Q =

Now, =− ⇒ 3 tan 2 x + 8 tan x + 3 = 0 6
1 + tan 2 x 4
π
−8 ± 64 − 36 −4 ± 7 ⇒ R=
∴ tan x =
= 6
6 3
55. Applying sine rule in triangle ABD,
−4 − 7
∵ tan < 0 ∴ tan x =
AB BD
3 =
sin θ sin(θ + α )
3
HINTS AND EXPLANATIONS

50. Given that cos( β − γ ) + cos(γ − α ) + cos(α − β ) = − which p 2 + q 2 sin θ p 2 + q 2 sin θ


2 ⇒ AB = =
is equivalent to sin θ cos α + cosθ sin α sin θ ⋅ q cosθ ⋅ p
+
2[cos( β − γ ) + cos(γ − α ) + cos(α − β )] + 3 = 0 p +q
2 2
p2 + q2

( p 2 + q 2 )sin θ
⇒ 2[cos( β − γ ) + cos(γ − α ) + cos(α − β )] + sin 2 α ⇒ AB = .
( p cosθ + q sin θ )
+ cos 2 α + sin 2 β + cos 2 β + sin 2 γ + cos 2 γ = 0
tan A cot A
⇒ (sin α + sin β + sin γ ) 2 + (cos α + cos β + cos γ ) 2 = 0 56. +
1 − cot A 1 − tan A
4
51. cos(α + β ) = 1 cot 2 A 1− cot 3 A
5 = − =
cot A(1− cot A) (1− cot A) cot A(1− cot A)
3 cos ec 2 A + cot A
⇒ tan(α + β ) = = = 1+ sec A cos ecA
4 cot A
5 1 1
sin(α − β ) = 57. (sin 4 x + cos 4 x ) − (sin 6 x + cos6 x )
13 4 6
5 3(sin 4 x + cos 4 x ) − 2(sin 6 x + cos6 x )
⇒ tan(α − β ) = =
12 12
a π 3(1 − 2 sin 2 x + cos 2 x ) − 2(1 − 3 sin 2 x + cos 2 x )
52. r =
cot =
2 n 12

‘a’ is side of polygon.
1
= .
12
Trigonometric Ratios and Identities  15.23

58. 5 tan2x = 9 cos2 x + 7


5 sec2 x – 5 = 9 cos2x + 7
1 1
cos2x =
, cos 2x = 2 cos2x – 1 = −
3 3
Let cos2 x = t
2 7
cos 4x = 2 cos 2x – 1 = − 1 = –
2
9t2 + 12t – 5 = 0
9 9
1 5
t=
as t ≠ –
Hence, the correct option is (C)
3 3

HINTS AND EXPLANATIONS


This page is intentionally left blank.
CHAPTER Trigonometric
16 Equations

LEARNING OBJECTIVES
After reading this chapter, you will be able to:
  Know about trigonometric equations  Calculate solution of an equation of the form
 Learn various solutions of trigonometric equations a cosθ + b sinθ
and the method for finding principal value

TRIGONOMETRIC EQUATION (i) Particular solution: A specific value of unknown


angle satisfying the equation.
An equation involving one or more trigonometric ratios of (ii) Principal solution: Smallest numerical value of
unknown angles is called a trigonometric equation. the unknown angle satisfying equation (numerically
1 smallest particular solution).
For example, 2 cos q + 3 cos 2q = 0, cos 2q + sin q = ,
3 (iii) General solution: Complete set of values of the
etc. are trigonometric equations in unknown angle θ. unknown angle satisfying the equation. It contains
all particular solutions as well as principal solutions.
ERROR CHECK
METHOD FOR FINDING PRINCIPAL VALUE
A trigonometric equation is different from a trigonometri- (SOLUTION)
cal identity. An identity is satisfied for every value of the
unknown angle e.g., cos2x = 1 – sin2x is true ∀x ∈ R, while a (i) First draw a trigonometrical circle and mark the
trigonometric equation is satisfied for some particular values quadrant in which the angle may lie.
of the unknown angle. (ii) Select anticlockwise direction for 1st and 2nd quad-
rants and clockwise direction for 3rd and 4th quadrants.
(iii) Find the angle in the first rotation.
(iv) Select the numerically least angle. The angle thus
SOLUTION OR ROOT OF A TRIGONOMETRIC
found will be principle value.
EQUATION (v) In case two angles one with positive sign and the
A value of the unknown angle which satisfies the given other with negative sign qualify for the numerically
equation is called a solution of the equation. For example, least angle, then it is the convention to select the
p angle with positive sign as principal value.
consider the equation 2 sinθ = 1. Clearly, q = and
4 1
For example, if sinq = - , θ lies in 3rd or 4th quad-
3p p 3p 2
q= satisfy this equation. Therefore, and
× are
×
rant. For 3rd and 4th quadrant, we select clockwise direc-
4 4 4
solutions of the equation 2 sinθ = 1. p 5p
tion. So we get two values - and - . Between these
Since all trigonometrical ratios are periodic in nature, p 6 6
generally a trigonometric equation has more than one solu- two, - has the least numerical value. Hence principal
6
tion or an infinite number of solutions. There are basically
three types of solutions. p
value is - .
6
16.2  Chapter 16

Solution (C)
Given equation is
sec2(a + 2)x + a2 – 1 = 0
⇒ tan2(a + 2)x + a2 = 0
⇒ tan2(a + 2)x = 0 and a = 0
p p
Þ tan 2 2 x = 0 Þ x = 0, ,-
2 2
∴ (0, 0), (0, π/2), (0, –π/2) are ordered pairs satisfying
FIGURE 16.1 the equation.

sin 3q - cos3q cosq


General Solutions of Trigonometric Equations
3. If - - 2 tanq cotq = -1,
sinq - cosq 1 + cot 2q
Trigonometric Equation General Solution 0 ∈ [0, 2p], then

sinθ = 0 θ = nπ
p p
cosθ = 0 θ = nπ + π/2 (A) q Î æç 0, ö÷ - ìí üý
tanθ = 0 θ = nπ è 2 ø î4þ
sinθ = 1 θ = 2nπ + π/2 æ p ö ì 3p ü
cosθ = 1 θ = 2nπ (B) q Î ç , p ÷ - í ý
è2 ø î 4 þ
tanθ = 1 θ = nπ + π/4
3p ö ì 5p ü
sinθ = –1 θ = 2nπ – π/2 (C) q Î æç p , ÷-í 4 ý
cosθ = –1 θ = (2n + 1) π è 2 ø î þ
tanθ = –1 θ = nπ – π/4
ìp p ü
sinθ = k, –1 ≤ k ≤ 1 θ = nπ + (–1)n sin–1k (D) q Î (0, p ) - í , ý
cosθ = k, –1 ≤ k ≤ 1 θ = 2nπ ± cos–1k î4 2þ
tanθ = k, –∞ < k < ∞ θ = nπ + tan–1k Solution (D)
sinθ = sinα θ = nπ + (–1)nα Since sinθ – cosθ ≠ 0
cosθ = cosα θ = 2nπ ± α p 5p
tanθ = tanα θ = nπ + α \ tan q ¹ 1 \ q ¹ ,
4 4
sin2θ = sin2α θ = nπ ± α
Now sin2θ + cos2θ + sinθ cosθ
tan2θ = tan2α θ = nπ ± α
cos2θ = cos2α θ = nπ ± α – | sinθ | cosθ – 2tanθ cotθ = –1
⇒ 1 + cosθ (sinθ – | sinθ |) – 2 = –1
⇒ cosθ (sinθ – | sinθ |) = 0
SOLVED EXAMPLES ìp p ü
\ q Î (0, p ) - í , ý
é 7p ù î4 2þ
1. If sinθ = k for exactly one value of θ, θ ∈ ê0, ú,
then the value of k is ë 3 û 4. General solution of the equation
(A) 1 (B)  –1 ( 3 - 1) sin q + ( 3 + 1) cos q = 2 is
(C) 1/ 2 (D)  0
p p p p
Solution (A, B) (A) 2np ± np + ( -1) n +
+ (B) 
4 12 4 12
Clearly –1 ≤ k ≤ 1. For any value of k other than 1 and
–1, sinθ has two values (in quadrant, I II or III, IV). p p p p
(C) 2np ± np + ( -1) n -
- (D) 
∴ k = 1 or –1 4 12 4 12
2. Number of ordered pairs (a, x) satisfying the equation Solution (A)
sec2 (a + 2) x + a2 – 1 = 0; –π < x < π is Let 3 + 1 = r cos a , and 3 - 1 = r sin a
(A) 2 (B)  1
(C) 3 (D)  infinite \ r 2 = ( 3 + 1) 2 + ( 3 - 1) 2 = 8,
Trigonometric Equations  16.3

1 Solution (D)
1- We have, a1 + a2 cos 2x + a3 sin2x = 0
and tan a =
3 -1
= 3 = tan æ p - p ö
ç4 6÷ a3
3 +1 1+ 1 è ø Þ a1 + a2 cos 2 x + (1 - cos 2 x ) = 0
3 2
æp ö p which is zero for all values of x,
= tan ç ÷ , i.e., a =
è 12 ø 12 a
if a1 = - 3 = -a2
From the given equation, we get rcos (θ – α) = 2 2
k k
æ p ö 1 æp ö or a1 = - , a2 = , a3 = k
Þ cos ç q - ÷ = = cos ç ÷ 2 2
è 12 ø 2 è4ø
for any k ∈ R.
∴ θ – π/12 = 2nπ ± π/4 Hence, the required number of triplets is infinite.
or θ = 2nπ ± π/4 + π/12
8. The least difference between the roots, in the first
5. For m ≠ n, if tan mθ = tan nθ, then the different values æ pö
quadrant ç 0 £ x £ ÷ , of the equation 4cosx (2 –
of θ are in è 2ø
(A) A.P. 3sin2x) + (cos 2x + 1) = 0, is
(B) H.P. p p
(C) G.P. (A) (B) 
× ×

6 4
(D) no particular sequence
p p
(C) (D) 
Solution (A)
× ×

3 2
We have, tan mθ = tan nθ ⇒ mθ = nθ + kπ Solution (A)
kp We have,
\ q= , where k ∈ Z.
m-n 4cosx (2 – 3 sin2x) + (cos 2x + 1) = 0
p ⇒ 4cosx (3cos2x – 1) + 2 cos2x = 0
These values of θ are in A.P. with first term
p m -n ⇒ 2cosx (6cos2x + cosx – 2) = 0
and common difference . ⇒ 2cosx (3cosx + 2) (2cosx – 1) = 0
m-n
⇒ either cosx = 0 which gives x = π/2
6. The number of the solutions of the equation or cosx = –2/3,
( )
cos p x - 4 cos (p x ) = 1 is which gives no value of x for which 0 ≤ x ≤ π/2
or cosx = 1/2, which gives x = π/3

(A)
>2 (B) 
2 So, the required difference = π/2 – π/3 = π/6.

(C)
1 (D) 0
9. Solution of the equation 4cot 2θ = cot2θ – tan2θ is
Solution (C)
p p
Clearly, x ≥ 4 (Since x - 4 is real) so that x is (A) q = np ± (B)  q = np ±
also real. 2 3
Again, if cos (p x ) < 1 then cos(p x - 4 ) > 1 and (C) q = np ± p (D)  none of these
4
if cos (p x ) > 1 then cos(p x - 4 ) < 1 (since their
Solution (C)
product = 1).
But both of these are not possible (since cosθ can- We have, 4cot 2θ = cot2θ – tan2θ
not be greater than 1). 4 4
Þ = - tan 2q
\ cos(p x - 4 ) = 1 and cos(p x ) = 1 tan 2q tan q 2

2 tan q
∴  x – 4 = 0 and x = 0 Put tan 2q =
But x = 0 is not possible, ∴ x = 4 is the only solution. 1 - tan 2q

7. The number of all possible triplets (a1, a2, a3) such that 4(1 - tan 2q ) 1 - tan 4q
\ =
a1 + a2 cos2x + a3 sin2x = 0 for all x is 2 tanq tan 2q
(A) 0 (B)  1 ⇒ (1 – tan2θ) [2tanθ – (1 + tan2θ)] = 0
(C) 3 (D)  infinite ⇒ (1 – tan2θ) (tan2θ – 2tanθ  + 1) = 0
16.4  Chapter 16

⇒ (1 – tan2θ) (tanθ – 1)2 = 0 p +1 where p is an even


(D) irrational of the form ,
p 4
Þ tan q = 1, -1. \ q = np ± integer
4
Solution (C)
10. Let α, β be any two positive values of x for which We have, sinπ (x2 + x) = sinpx2
2cosx, | cosx | and 1 – 3 cos2x are in G.P. .The minimum ⇒ π (x2 + x) = nπ + (–1)npx2
value of | α – β | is ∴ Either x2 + x = 2m + x2  ⇒  x = 2m ∈ Z.
p p or x2 + x = k – x2, where k is an odd integer
(A) (B) 
× ×

3 4 -1 ± 1 + 8k
Þ 2x2 + x - k = 0 Þ x =
p 4
(C) × (D)  none of these
2 For least positive non-integral solution,
Solution (D) -1 + 1 + 8k p - 1 where p is an odd integer.
x= = ,
Since 4 4
2cosx, | cosx | and 1 – 3cos2x are in G.P. p p
∴ cos x = 2cosx(1 – 3cos2x)
2 13. In the interval êé - , úù , the equation logsinθ(cos2θ)
⇒ 6cos3x + cos2x – 2cosx = 0 = 2 has ë 2 2û
⇒ cosx(2cosx – 1) (3cosx + 2) = 0 (A) no solution
1 2 p p æ 2ö (B) a unique solution
Þ cos x = 0, , - \ x = , , cos -1 ç - ÷ (C) two solutions
2 3 2 3 è 3ø
(D) infinitely many solutions
p p
∴ If a = ,b = , ( ∴ α, β are are positive) Solution (B)
2 3
p p
p
We have, - £q £
then |a - b | = 2 2
6 ∴ –1 ≤ sinθ ≤ 1, here 0 < sinθ < 1
11. The most general values of θ for which sinθ – cosθ = Now, logsinθcos2θ = 2
a ∈ R (1, a2 – 6a + 10) are given by ⇒ cos2θ = sin2θ
⇒ 1 – 2sin2θ = sin2θ
p p p p 1
(A) np + ( -1) n - (B)  np + ( -1) n +
4 4 4 4 Þ 3 sin 2q = 1 Þ sin 2q =
3
(C) 2np + p (D)  none of these 1
4 Þ sinq = (∵ 0 < sin q < 1)
3
Solution (B) The given equation has a unique solution.
We have,
min 14. The number of solutions of the equation sinx = cos 3x
sinθ – cosθ =  a Î R {1, a2 – 6a + 11}. in [0, p] is
Since a2 − 6a + 11 = (a – 3)2 + 2 > 2 for all a (A) 1 (B)  2
æ pö 1 p (C) 3 (D)  4
\ sin q - cos q = 1 Þ sin ç q - ÷ = = sin
è 4 ø 2 4 Solution (C)
p p The given equation can be written as
Þ q - = np + ( -1) n sinx = 4cos3x – 3cosx
4 4
i.e., sec2x tanx + 3sec2x – 4 = 0
p p
\ q = np + ( -1)n + , where n ∈ Z. In terms of tanx, this leads to the equation
4 4 tan3x + 3tan2x + tanx – 1 = 0
12. The least positive non-integral solution of the equation ⇒ (tan x + 1)(tan2x + 2tanx – 1) = 0
sinπ(x2 + x) = sin px2 is ⇒ tanx = –1 or tan2x = 1
(A) rational 3p p 5p
i.e., x= , ,
(B) irrational of the form p 4 8 8

(C) irrational of the form p -1 where p is an odd 15. The number of values of x in [0, 2p] satisfying the
, equation | cos x - sin x | ³ 2 is,
integer 4
Trigonometric Equations  16.5


(A)
0 (B) 
1 æ p+qö æ p-qö

(C)
2 (D) 
3 Þ cos ç ÷ x cos ç 2 ÷ x = 0
è 2 ø è ø
Solution (C) æ p+qö ( 2n + 1)p
cos ç ÷ x = 0 Þ x = p + q , n = 0, ± 1, ± 2, …
Given equation is | cos x - sin x | ³ 2 è 2 ø
2p
Since | cos x - sin x | £ 1 + 1 = 2 which forms an A.P. with common difference =
p+q
∴ We must have | cos x - sin x | = 2 ( 2n + 1)p
æ p-qö
æ pö æ pö
cos ç
2 ÷ x = 0 Þ x = p - q , n = 0, ± 1, ± 2, …
Þ cos ç x + ÷ = 1 Þ cos ç x + ÷ = 1, - 1 è ø
è 4 ø è 4ø 2p
which forms an A.P. with common difference =
p p-q
\ x + = 0, 2p , 4p , 6p , . . .  or  π, 3π, 5π. . .
4
3p 7p ERROR CHECK
\ x= ,
4 4
In each of the above cases, n ∈ Z, where Z is the set of
3 1
sin 2 x - sin x + integers
16. If |cos x | 2 2
= 1, then possible values of x are:
p
(A) np or np + ( -1) n ,nÎ I
6 SOLUTION OF AN EQUATION OF THE FORM
p p acosθ + bsinθ = c
(B) np or 2np + or np + ( -1) n , n Î I
2 6
The given equation is acosθ + bsinθ = c.(1)
p
(C) np + ( -1) n , n Î I
6 Divide throughout by a2 + b2
(D) nπ, n ∈ I i.e., by (coeff. of cosq ) 2 + (coeff. of sinq ) 2 , we get
Solution (C, D) a b c
cosq + sinq = (2)
The equation holds if | cos x | = 1 a +b
2 2
a +b
2 2
a + b2
2

i.e., if x = nπ , n ∈ I
Let α be the least +ve angle such that
3 1
If | cosx | ≠ 1, then sin 2 x - sin x + = 0 a b
2 2 = cos a and = sin a
1 a +b
2 2
a + b2
2

Þ sin x = 1 or ∴  (2) becomes


2
sin x ≠ 1, as in that case cos x = 0 c
cos a cos q + sin a sin q =
1 p a + b2
2
\ sin x = Þ x = np + ( -1) n
2 6 ⇒ cos(θ – α) = cosβ (say),
17. A set of values of x satisfying the equation c
where cos b =
æ1 ö æ1 ö a + b2
2
cos 2 ç px ÷ + cos 2 ç qx ÷ = 1
è 2 ø è2 ø ∴ θ – α = 2nπ ± β
form an arithmetic progression with common ⇒ θ = 2nπ ± β + α, where n ∈ Z.
difference
2 2 WORKING RULE TO SOLVE AN EQUATION
(A) (B) 
p+q p -q OF THE FORM acosq + bsinθ = c
p 1.
Divide throughout by a2 + b2
(C) (D)  none of these
p+q i.e., by (coeff. of cosq ) 2 + (coeff. of sinq ) 2
2.
Write L.H.S. as a single cosine.
Solution (D)
3.
Use the formula
The given equation can be written as
1 + cospx + 1 + cosqx = 2 θ  = 2nπ ± α, n ∈ Z  for cosθ  = cosα
16.6  Chapter 16

dividing an equation by a common factor, take this


ERROR CHECK factor out as a common factor from all terms of the
equation.
Check that |c| £ a2 + b2 . If it is not satisfied, no real solu-  Make sure that the answer should not contain any value

tion exits. of unknown ‘θ ’ which makes any of the terms undefined.
 If tanθ or secθ is involved in the equation, θ should not be

QUICK TIPS an odd multiple of π/2.


 If cotθ or cosecθ is involved in the equation, θ should not
 Squaring should be avoided as far as possible. If squaring
is done, then check for extra solutions. be a multiple of π or 0.
For example, consider the equation sinθ + cosθ = 1.  The value of f ( q) is always positive. For example,
On squaring, we get cos2 q = |cos q | and not ± cos q .
1 + sin2θ  = 1 or sin 2θ  = 0
np  All the solutions should satisfy the given equation and lie
⇒  q = , n = 0, ±1, ±2, ... in the domain of the variable of the given equation.
2
 Check the validity of the given equation, e.g., 2cosθ +
3p
The values of the angle, θ = π and q = do not satisfy sinθ = 3 can never be true for any θ as the value (2cosθ
2
the given equation. So, we get extra solutions. Thus, if + sinθ) can never exceed 22 + 12 = 5. So there is no
solution for this equation.
squaring is must, verify each of the solutions.
 Never cancel a common factor containing ‘θ ’ from the two
sides of an equation.
ERROR CHECK
For example, consider the equation tan q = 2 sin q . If
1
we divide both sides by sinθ, we get cos q = , which x
2 If the argument in inequalities is not x but x + a or ax or
a
is clearly not equivalent to the given equation as the then always write general solution first in terms of the argu-
solutions obtained by sinx = 0 are lost. Thus, instead of ment and then find x.

SOLUTIONS OF BASIC TRIGONOMETRIC INEQUALITIES


Inequality Solution (n ∈ Z)
sinx > α, where |α| < 1 sin–1α + 2nπ < x < π – sin–1α + 2nπ
sinx < α, where |α| < 1 –π – sin–1α + 2nπ < x < sin–1α + 2nπ
cosx > α, where |α| < 1 –cos–1α + 2nπ < x < cos–1α + 2nπ
cosx < α, where |α| < 1 cos–1α + 2nπ < x < 2π – cos–1α + 2nπ
π
tanx > α, where –∞ < α < ∞ tan−1α + nπ < x < + nπ
2
π −1
tanx < α, where –∞ < α < ∞ − + nπ < x < tan α + nπ
2
cotx > α, where –∞ < α < ∞ nπ < x < cot–1α + nπ
cotx < α, where –∞ < α < ∞ cot–1α + nπ < x < π + nπ

Solution (C)
SOLVED EXAMPLES We have, | cosx | ≤ sinx  ⇒ sinx ≥ 0
(∴ | cosx | ≥ 0)
18. If 0 ≤ x ≤ 2π and | cosx | ≤ sinx, then
⇒ x ∉ (π, 2π)
é pù ép p ù
(A) x Î ê0, ú (B) 
xÎê , ú If x = 2π, | cos2π | ≤ sin2π which is not possible,
ë 4û ë4 2û ∴ x ∈ [0, p]
é p p ù é p 3p ù é pù
(C) x Î ê , ú È ê , ú (D)  none of these If x Î ê0, ú , then | cos x |£ sin x Þ cos x £ sin x
ë4 2û ë2 4 û ë 2û
Trigonometric Equations  16.7

ép p ù ⇒ sin2x = k + 3 ( sin2x = –1 is not possible)



Þ xÎê , ú Since 0 ≤ sin2x ≤ 1, ∴ 0 ≤ k + 3 ≤ 1 or –3 ≤ k ≤ –2.
ë4 2û
æp ö 22. The equation cos2x + asinx = 2a – 7 has a solution for
If x Î ç , p ÷ , then
è2 ø (A) all a (B)  a>6
| cosx | ≤ sinx  ⇒  – cosx ≤ sinx ⇒ tanx ≤ –1 (C) a < 2 (D)  a ∈ [2, 6]
( cosx < 0)

Solution (D)
æ p 3p ù é p p ù æ p 3p ù Given equation is cos2x + asinx = 2a – 7
Þ x Îç , ú . \ x Î ê , ú È ç , ú ⇒ 1 – 2sin2x + asinx = 2a – 7
è 2 4 û ë4 2û è 2 4 û
⇒ 2sin2x – asinx + 2a – 8 = 0
19. The equation (cosp – 1) x2 + (cosp) x + sinp = 0, in the
variable x has a real root. then p can take any value in a ± a 2 - 16( a - 4) a ± ( a - 8)
Þ sin x = =
the interval 4 4
æ -p p ö a-4
(A) (0, π) (B)  ç 2 .2÷ Þ sin x = or sinx = 2 (not possible)
è ø 2
(B) (–π, 0) (C)  (0, 2π) a-4
\ sinx =
Solution (A)
2
Discriminant = cos2p – 4 (cos p – 1)sin  p a-4
∵ -1 £ sin x £ 1, \ - 1 £ £1 Þ 2 £ a £ 6
For equation to have real roots, 2
cos2p – 4(cosp – 1) sinp ≥ 0 23. The set of all x in (–π, π) satisfying | 4 sin x - 1 | < 5
⇒ cos2p ≥ 4(cosp – 1) sinp is given by
p
⇒ cos2p ≥ –8 sin2 sinp æ p 3p ö æ p ö
2 (A) ç - , ÷ ç - 10 , p ÷
(B) 
3p p è 10 10 ø è ø
For p = or - , R.H.S. > 1 but L.H.S. < 1,
2 2 æ 3p ö
∴ the choices (B), (C) and (D) are ruled out. The cor- (C) (–π, π) (D) 
ç -p , 10 ÷
è ø
rect alternative is (A)
Solution (A)
20. The equation sin4x + cos4x = a has a solution for
We have, |4 sin x - 1| < 5
(A) all of values of a (B) a = 1
1 1 Þ - 5 < 4 sin x - 1 < 5
(C) a = (D)  < a <1
2 2 æ 5 -1 ö 5 +1
Þ -ç < sin x <
Solution (B, C,D) ç 4 ÷÷ 4
The given equation can be written as è ø
1 – 2 sin2x cos2x = a p p
Þ - sin < sin x < cos
⇒ sin22x = 2(1 – a) ⇒ 2(1 – a) ≤ 1 10 5
and 2(1 – a) ≥ 0 ⇒ 1/2 ≤ a ≤ 1 æ -p ö æp p ö
Þ sin ç ÷ < sin x < sin ç 2 - 5 ÷
21. The equation sin4x – (k + 2) sin2x – (k + 3) = 0 pos- è 10 ø è ø
sesses a solution if æ -p ö æ 3p ö
Þ sin ç ÷ < sin x < sin ç 10 ÷
(A) k > –3 è 10 ø è ø
(B) k < –2 æ p 3p ö
(C) –3 ≤ k ≤ –2 Þ x Î ç - , ÷ [∵ x Î ( -p , p )]
(D) k is any positive integer è 10 10 ø

Solution (C) 24. The number of values of x in [0, 2p] satisfying the
equation |cos x - sin x| ³ 2 , is
We have, sin4x – (k + 2) sin2x – (k + 3) = 0
(A) 0 (B)  1
( k + 2) ± ( k + 2) 2 + 4( k + 3) (C) 2 (D)  3
Þ sin 2 x =
2
( k + 2) ± ( k + 4) Solution (C)
=
2 Given equation is | cos x - sin x | ³ 2 .
16.8  Chapter 16

Solution (A)
Since | cos x - sin x | £ 1 + 1 = 2 ,
∴  we must have |cos x - sin x| = 2 Since | 4 sin x - 1 | < 5

æ pö æ pö Þ - 5 < 4 sin x - 1 < 5


Þ cos ç x + ÷ = 1 Þ cos ç x + ÷ = 1, -1.
è 4ø è 4ø æ 5 -1 ö 5 +1
Þ -ç < sin x <
p ç 4 ÷÷ 4
\ x+ = 0, 2p , 4p , 6p , ..., π, 3π, 5π, .... è ø
4
p p
3p 7p Þ - sin < sin x < cos
\ x= , 10 5
4 4
æ p ö æp p ö
25. The set of all x in (–π, π) satisfying |4 sin x - 1| < 5 is Þ sin ç - ÷ < sin x < sin ç - ÷
è 10 ø è2 5ø
given by
æ p 3p ö æ p ö 3p
æ p 3p ö Þ sin ç - ÷ < sin x < sin
(A) ç - , ÷ (B)  ç 10 , 10 ÷ è 10 ø 10
è 10 10 ø è ø
p -3p ö æ p 3p ö
(C) çæ , ÷ (D)  none of these \ x Îç- , ÷
è 10 10 ø è 10 10 ø
Trigonometric Equations  16.9

NCERT EXEMPLARS
1. If sinθ + cosecθ = 2, then sin2θ + cosec2θ is equal to 10. Which of the following is correct?
(A) 1 (B) 4 (A)  sin 1° > sin 1 (B)  sin 1° < sin 1
(C)  2 (D)  None of these p
(C) sin 1° = sin 1 (D)  sin 1° = sin 1
2. If ¦ ( x ) = cos x + sec x, then
2 2 18°

(A) f(x) < 1 (B)  f(x) = 1 m 1


11. If tan a = and tan b = , then α + β is
(C)  2 < f (x) < 1 (D)  f (x) ≥ 2 m +1 2m + 1
equal to
1 1 p p
3. If tan q = and tan f = , then the value of θ + ϕ is
2 3 (A)  (B) 
2 3
p p p
(A)  π
(B)  (C)  (D) 
6 6 4
p
(C)  0 (D) 
4 12. The minimum value of 3 cos x + 4 sin x + 8 is
4. Which of the following is not correct ? (A) 5 (B) 9
(C) 7 (D) 3
1
(A) sin θ = − cos q = 1
(B)  13. The value of tan 3A – tan 2A – tan A is
5
1 (A)  tan 3A tan 2A tan A
(C)  secq = - tan q = 20
(D)  (B)  – tan 3A tan 2A tan A
2
(C)  tan A tan 2A – tan 2A tan 3A – tan 3A tan A
5. The value of tan°tan2°tan3°……tan89° is (D)  None of the above
(A) 0 (B) 1 14. The value of sin (45° + θ) – cos (45° – θ) is
1
(C)  (D)  Not defined (A)  2 cos θ (B) 
2 sin θ
2

(C) 1 (D) 0
1 - tan 2 15°
6. The value of is
t + tan 2 15° æp ö æp ö
15. The value of cot ç + q ÷ cot ç - q ÷ is

(A) 
1 (B)  3 è4 ø è4 ø
3 (A) – 1 (B) 0
(C)  (D) 
2 (C)  1 (D)  None of these
2

NCERT EXEMPLARS
16. cos 2q cos 2f + sin (q - f ) - sin ( +f ) to
2 2
7. The value of cos1° cos2° cos3° ….. cos179° is
(A)  sin 2 (q + f ) (B) 
cos 2 (q + f )
1
(A) (B)  0
2
cos 2 (q - f )
(C) sin 2 (q - f ) (D) 
(C) 1 (D) – 1
8. If tan θ = 3 and θ lies in third quadrant, then the value of 17. The value of cos 12° + cos 84° + cos 156° + cos 132° is
sin θ is
1
1 1 (A)  (B) 
1
(A)  (B)  - 2
10 10 1 1
-3 3 (C)  - 2 (D) 
(C)  (D)  8
10 10
1 1
9. The value of tan 75° – cot 75° is 18. If tan A =
and tan B = , then tan(2A + B) is equal to
2 3
2+ 3
(A) 2 3 (B) 
(A) 1 (B) 2
(C) 2 - 3 (D) 
1 (C) 3 (D) 4
16.10  Chapter 16

p 13p 7p 4p
19. The value of sin sin is (A) sin + sin (B) 1
10 10 18 9
1 1 p 3p p p
(A)  (B) - (C) cos + cos (D)  cos + sin
2 2 6 7 9 9
1
(C) - (D) 
1 26. If A lies in the second quadrant and 3 tan A + 4 = 0,
4
then the value of 2 cot A – 5 cos A + sin A is
20. The value of sin 50° – sin 70° + sin 10° is -53 23
(A)  (B) 
(A) 1 (B) 0 10 10
1
(C)  (D)  2 37 7
2 (C)  (D) 
10 10
21. If sinθ + cos θ = 1, then the value of sin2θ is 27. The value of cos2 48º – sin2 12º is
1
(A)  1 (B) 
2 5 +1 5 -1
(A)  (B) 
(C) 0 (D) –1 8 8
p 5 +1 5 +1
22. If a + b = , then the value of (1 + tanθ) (1 + tanβ) is (C)  (D) 
4 5 2 2
(A) 1 (B) 2 1 1
(C)  – 2 (D)  Not defined 28. If tan a =and tan b = , then cos 2 α is equal to
7 3
-4 (A)  sin 2β (B)  sin 4β
23. If sin q = and θ lies in third quadrant, then the
5 (C)  sin 3β (D)  cos 2β
p
value of cos is a
2 29. If tan q = , then b cos 2θ + a sin 2θ is equal to
1 b
1
(A)  (B)  (A) a (B)  b
5 10 a
(C) (D)  None of these
1 1 b
(C) - (D) 
5 10 1
30. If for real values of x, cos q = x + , then
x
24. The number of solutions of equation tan x + sec x = 2
(A) θ is an acute angle
cos x lying in the interval [0, 2 π] is
(B) θ is right angle
NCERT EXEMPLARS

(A) 0 (B) 1 (C)  θ is an obtuse angle


(C) 2 (D) 3 (D)  No value of θ is possible
p p 2p 5p
25. The value of sin + sin + sin + sin is
18 9 9 18

ANSWER K EYS
  1. (C) 2. (D) 3. (D) 4.  (C) 5. (B) 6.  (C) 7. (B) 8. (C) 9. (A)  10.  (B)
  11. (D) 12.  (D) 13. (A) 14.  (D) 15. (C) 16.  (B) 17. (C) 18.  (C) 19. (C)  20.  (B)
  21. (C) 22.  (B) 23. (C) 24. (C) 25.  (A) 26. (B) 27.  (A) 28. (B) 29.  (B)  
30. (D)
Trigonometric Equations  16.11

HINTS AND EXPLANATIONS

1. Given that, sin θ + cosec θ = 4 ⇒


sec2 θ = 1 + tan2 θ
⇒ sin2θ + cosec2θ + 2sinθ . cosecθ = 4 ⇒
secq = 1 + 9 = ± 10
⇒ sin2θ + cosec2θ = 4 – 2 secq = - 10


⇒ sin2θ + cosec2θ = 2 1
secq = -
2. Given that, f (x) = cos2 x + sec2x 10


We know that, AM ≥ GM 1 9 3
cos 2 x + sec 2 x ⇒
sin q = ± 1 - =± =±
³ cos 2 x • sec 2 x 10 10 10
2
 [since, θ lies in third quadrant]
⇒ cos x + sec x ³ 2 
2 2
[∵ cos x • sec x = 1] ∴

3
sin q = -
f ( x) ³ 2 10
⇒ 9. Given expression, tan 75° – cot 75°
1 1 sin 75° cos 75°
3. Given that, tan q = and tan f = = -
2 3 cos 75° sin 75°
tan q + tan f sin 2 75° - cos 2 75°
Now, tan (q + f ) =
=
1 - tan q • tan f sin 75° • cos 75°
1 1 3+ 2 2 cos15°

+
=
5
tan (q + f ) = 2 3 Þ tan (q + f ) = 6 = = 1 sin150°
1 1 6 -1 5
1- • -2 cos ( 90° + 60° )
2 3 6
=
sin ( 90° + 60° )
p

tan (q + f ) = tan +2 sin 60°
4
=
p cos 60°

q +f =
4 3
2.

= 2 =2 3
4. We know that, the range of secθ is R – (– 1, 1). 1
Hence, secθ cannot be equal to 1 . 2

HINTS AND EXPLANATIONS


2 10. We know that, if θ is increasing, then sin θ is also increasing.
5. Given expression, tan 1° tan2° tan3° …..tan 89° ∴ sin 1° < sin 1  [∵ 1 rad = 57°30 ’]
= tan 1° tan2° ….. tan 45°·tan(90° – 44°) tan (90° – 43°)
m 1
 ….. tan(90° – 1°) 11. Given that, tan q = and tan b =
m +1 2m + 1
= tan 1°·cot 1°· tan 2°·cot 2°·cot 2° …. Tan 89°·cot 89°
tan a + tan b
= 1·1 ….. 1·1 = 1
Now, tan (a + b ) =
1 - tan a •tan b
6. Given expression, 1 - tan 2 15°
2
m 1
1 + tan 15° +
⇒ tan (a + b ) =
m + 1 2 m +1
Let q = 15° æ m öæ 1 ö
1 - tan 2 q 1- ç ÷ç ÷
We know that, cos 2q = è m + 1 ø è 2m + 1 ø
1 + tan 2 q
m ( 2m + 1) + m + 1
1 - tan 2 15° ⇒ tan (a + b ) =

∴ cos 30° = ( m + 1) ( 2m + 1) - m
1 + tan 2 15°
2m 2 + m + m + 1
1 - tan 2 15° 3 é 3ù ⇒ tan (a + b ) =


=  ê∵ cos30° = ú 2m 2 + 2m + m + 1 - m
1 + tan 2 15° 2 êë 2 úû
2m 2 + 2m + 1
⇒ tan (a + b ) = Þ tan (a + b ) = 1
7. Given expression, cos1° cos2° cos3° …. cos 179° 2m 2 + 2m + 1
p
= cos 1° cos2° ….. cos90° ….. cos 179° [∵ cos 90° = 0] ⇒ a + b =
4
=0 12. Given expression, 3 cos x + 4 sin x + 8
8. Given that, tan θ = 3 Let y = 3 cos x + 4 sin x + 8
⇒ y – 8 = 3 cos x + 4 sin x
16.12  Chapter 16

∴ Minimum value of y - 8 = - 9 + 16 æ 5 -1 ö 1
= -4 çç
÷÷·cos 36°·
⇒ y – 8 = – 5   ⇒  y = – 5 + 8 è 4 ø 2
⇒ y=3 æ 5 +1ö 1
Hence, the minimum value of 3 cos x + 4 sin x + 8 is 3.
(
= - 5 - 1 çç
4
) 2
æ 5 - 1 ö -4 -1
÷÷· = - ç
è 8 ø 8
÷= =
2
è ø
13. Let 3A = A + 2A
1 1
tan 3A = tan (A + 2A) 18. Given that, tan A =and tan B =
2 3
tan A + tan 2 A
⇒ tan 3 A = tan 2 A + tan B
1 - tan A•tan 2 A
Now, tan ( 2 A + B ) =
1 - tan 2 A•tan B
⇒ tan A + tan 2A = tan 3A – tan 3A·tan 2A · tan A 1
⇒ tan 3A – tan 2A – tan A = tan 3A · tan 2A · tan A 2.
2 tan A 4

Also, tan 2 A = = 2 =
14. Given expression, 1 - tan 2 A 1 - 1 3
sin(45° + θ) – cos(45° – θ) 4
4 1 4 1 5
= sin 45° · cos θ + cos 45° · sin θ – cos 45° · cos θ · – + +
sin45° . sin θ From Eq. (i), tan ( 2 A + B ) =
3 3 = 3 3 = 3 =3
4 1 9-4 5
1- •
1 1 1 1 3 3 9 9
= .cosq + • sin q - • cosq - sin q
2 2 2 2
=0 19. Given expression, sin p sin 13p = sin p sin æç p + 3p ö÷
æp ö æp ö 10 10 10 è 10 ø
15. Given expression,
cot ç + q ÷ - cot ç - q ÷
è4 ø è4 ø p 3p
= - sin sin = - sin18° • sin 54°
æ p ö æ p ö 10 10
ç cot 4 cot q - 1 ÷ ç cot 4 cot q + 1 ÷ = - sin18° • cos 36°
=ç ÷•ç ÷
ç cot p + cot q ÷ ç cot q - cot p ÷ æ 5 -1 ö æ 5 +1 ö
è 4 ø è 4 ø
= - çç ÷÷ çç ÷÷  [since, put this value here]
æ cot q - 1 ö æ cot q + 1 ö è 4 øè 4 ø
=ç ÷•ç ÷
è cot + 1 ø è cot q - 1 ø æ 5 -1 ö 1
= -ç ÷=-4
=1 è 16 ø
20. Given expression, sin 50° – sin 70° + sin 10°
16. Given expression, cos 2q cos 2f + sin 2 (q - f ) - sin 2 ( +f )
æ 50° + 70° ö æ 50° - 70° ö
HINTS AND EXPLANATIONS


= cos 2q • cos 2f + sin q - f + q + f • sin q - f - q - f
= 2 cos ç ÷ • sin ç ÷ + sin10°
( ) ( ) è 2 ø è 2 ø
= – 2cos 60° sin10° + sin10°
= cos 2q • cos 2f - sin 2q • sin 2f 1
= -2. sin10° + sin10° = 0
= cos ( 2q + 2f ) = cos 2 (q + f ) 2


21. Given that, sinθ + cos θ = 1
17. Given expression, cos 12° + cos 84° + cos 150° + cos 132°
On squaring both sides, we get
= cos 12° + cos 150° + cos 84° + cos 132°
sin2θ + cos2θ + 2sinθ · cosθ = 1
æ 12° + 150° ö æ 12° - 150° ö ⇒ 1 + sin2θ = 1
= 2 cos ç ÷ •cos ç ÷
è 2 ø è 2 ø ∴ sin 2θ = 0
æ 84 ° + 132 ° ö æ 84 ° – 132 ° ö
+ 2 cos ç ÷·cos ç
è 2 ø è 2 ÷
ø 22. Given that, a + b = p
4

= 2 cos 84° cos 72° + 2 cos 108° · cos 24º Now, (1+ tan α) (1 + tanβ) = 1 + tanα + tanβ + tanα
= 2 cos 84º cos (90° – 18°) + 2 cos (90° + 18°) · cos 24°
tanβ

= 2 cos 84° sin 18° – 2 sin 18° · cos 24°

= 2 sin 18° (cos 84° – cos 24°) We know that, tan (a + b ) = tan a + tan b

1 - tan a •tan b
æ 84° + 24° ö æ 84° - 24° ö tan a + tan b
= 2 sin18° • 2 sin ç ÷ • sin ç ÷ ⇒ 1 =

è 2 ø è 2 ø 1 - tan a • tan b

= 4 sin 18° · sin 54° sin 30° ⇒ tan a + tan b = 1 - tan a tan b

æ 5 -1 ö 1
From Eq. (i),
= -4 çç
÷÷·cos 36°·
è 4 ø 2
(1 + tanα) (1 + tanβ) = 1 + 1 tan α · tanβ + tan α · tanβ

= – 4 sin 18° · sin 54° sin 30°
=2
Trigonometric Equations  16.13

-4 -4
23. Given that, sin q = ⇒ tan A =

5 3
16 25 - 16 3 -3

cosq = 1 - = =± ⇒ cot A =

25 25 5 4

-3 [since, θ lies in third quadrant]
cosq = 16 25 5
5 ⇒ sec A = 1 +
= =±
9 9 3
2q -3
⇒ 2 cos - 1 =
-5
2 5 ⇒ sec A =
[ since, A lies is second quadrant]
3
q 3 -3
⇒ 2 cos = 1 -
2
⇒ cos A =

2 5 5
⇒ 2 cos 2 q =
2

sin A = 1 -
9
=
25 - 9

4
2 5 25 25 5
q 1 4
∴ cos = ±

sin A = [since, A lies in second quadrant]
2 5 5
q 1
⇒ cos = -
[since, θ lies in third quadrant] æ -3 ö æ -3 ö 4
2 5 ∴ 2 cot A - 5 cos A + sin A = 2 ç ÷ - 5 ç ÷ +
è 2 ø è 5 ø 5
24. Given equation, tan + sec x = 2 cos x
- 6 4
sin x 1 = +3+

+ = 2 cos x 4 5
cos x cos x - 30 + 60 + 16 46
= =
⇒ 1 + sin x = 2 cos2 x
20 20
⇒ 1 + sin x = 2 (1 – sin2 x)
23
=
⇒ 1 + sin x = 2 – 2sin2 x
10
⇒ 2 sin2 x + sin x – 1 = 0
27. Given expression, cos2 48° – sin2 12°
⇒ 2 sin2 x + 2 sin x – sin x – 1= 0
= cos ( 48° + 12° ) - cos ( 48° - 12° )
⇒ 2 sin2 x (sin x + 1) – 1 (sin x + 1) = 0
= cos 60° • cos 36°
⇒ (sin x + 1) (2 sin x – 1) = 0
1 5 +1
⇒ sin x + 1 = 0 or (2 sin x – 1) = 0
= ·
2 4

HINTS AND EXPLANATIONS


⇒ sin x = - x,sin x = 1

5 +1
2 =
8
3p p
∴ x =
,x = 1 1
2 6 28. Given that, tan = and tan b =

Hence, only two solutions possible. 7 3
p p 2p 5p 1 48
25. Given expression, sin + sin + sin + sin 1-
18 9 9 18
cos 2a = 49 = 49
1 50
= sin10° + sin 20° + sin 40° + sin 50° 1+
49 49
= sin 50° + sin10° + sin 40° + sin 20° 48 24

= =
= sin130° + sin10° + sin140° + sin 20° 50 25
= 2 sin 70° cos 60° + 2 sin 80° • cos 60° 24
⇒ cos 2a =
 (i)
25
é x+ y x- yù
 ê∵ sin x + sin y = 2 sin 2 • cos 2 ú We know that, sin 4 b = 2 tan 22 b 
(ii)
ë û
1 + tan 2 b
1 1 é 1 ù
= 2· sin 70° + 2· sin 80°  ê∵ cos60° = 2 ú 2´
1
2 2 ë û 2 tan b 3

and tan 2 b = =
1 1 1 - tan 2 b 1 - 1
= 2. sin 70° + 2. sin 80°
2 2 9
7p 4p 2
= sin 70° + sin 80° = sin + sin
2´9 3
26. Given equation,
18
3tan A + 4 = 0
9
=3= =
8 3´ 8 4
⇒ 3tan A = – 4 9

From Eq. (ii),
16.14  Chapter 16

3 6 æ b 2 - a 2 ö 2a 2b

= bç 2 + 2
4 6 ´ 16 2 ÷
= 4 = èb +a ø a +b
2

sin 4 b =
9 25 4 ´ 25
1+
16 16
= 2
b
é b 2
- a 2
+ 2 a 2 (
ù =
)
a2 + b2 b

24 a + b2 ë û
( )
a2 + b2
⇒ sin 4 b =

45
=b
⇒ sin 4 b = cos 2a 
[from Eq. (i)] 30. Here, cosq = x +
1
⇒ cos 2a = sin 4 b
x
x2 +1
29. Given that, tan q = a ⇒
cosq =
b x
æ 1 - tan 2 q ö æ 2 tan q ö
x2 – x cos θ + 1 = 0
∴ b cos 2 + a sin 2q = b ç
÷ + aç ÷ For real value of x, (– cos θ)2 – 4 × 1 × 1 = 0

è 1 + tan q ø è 1 + tan q ø
2 2

cos2 θ = 4

æ a2 ö æ 2a ö
ç 1- 2 ÷ ç ÷ cos θ = ± 2


= b ç b2 ÷ + aç b 2 ÷
ç1+ a ÷ ç1+ a ÷
which is not possible.  [∵ - 1 £ cosq £ 1]
ç ÷ ç ÷
which is not possible.
è b2 ø è b2 ø
HINTS AND EXPLANATIONS
Trigonometric Equations  16.15

PRACTICE EXERCISES

Single Option Correct Type

 x 1 p p −1
1. The equation 2cos2   . sin2x = x2 + 2 0 ≤ x ≤ (C) irrational of the form , where p is an odd
 2
×

x 2 4
has integer
(A) one real solution p +1
(B) no solution (D) irrational of the form , where p is an even
4
(C) more than one real solution integer
(D) none of these
7. If sin2x - 2sinx - 1 = 0 has exactly four different solu-
2. The general solution of the equation sin50 x – cos50x = 1 tions in x ∈ [0, np], then minimum value of n can be
is (n ∈ N)
p p (A) 4 (B)  3
(A) 2nπ + (B) 
× 2nπ + ×

2 3 (C) 2 (D)  1

p p 8. A set of values of x satisfying the equation


(C) nπ + nπ +
(D) 
1  1 
× ×

2 3 cos 2  px  + cos 2  qx  = 1
2  2 
3. General solution of the equation form an arithmetic progression with common
difference
( 3 - 1) sin q + ( 3 + 1) cos q = 2 is
2 2
p p (A) (B) 
(A) 2nπ ± + p+q p-q
4 12
p
p p (C) (D)  none of these
(B) nπ + (-1) + n p+q
4 12
p p 9. If 0 ≤ x ≤ 2π and | cos x | ≤ sin x, then
(C) 2nπ ± -
4 12 é pù ép p ù
(A) x Î ê0, ú xÎê , ú
(B) 
p p ë 4û ë4 2û
(D) nπ + (- 1)n -

PRACTICE EXERCISES
4 12 é p 3p ù
(C) x Î ê , ú (D)  none of these
ë4 4 û
4. The number of all possible triplets (a1, a2, a3) such that
a1 + a2 cos 2x + a3 sin2 x = 0 for all x is 10. The general solution of the equation
(A) 0 (B)  1 1 - sin x +  + ( -1) n sin n x +  1 - cos 2 x
(C) 3 (D)  infinite =
1 + sin x +  + sin n x +  1 + cos 2 x
5. The equation sin4 x - (k + 2) sin2 x - (k + 3) = 0 possesses p
x ≠ (2n + 1)
, n ∈ Z is
a solution if
×

2
(A) k > -3
æp ö æp ö
(B) k < -2 (A) ( -1) n ç ÷ + np (B) ( -1) n ç ÷ + np
(C) - 3 ≤ k ≤ -2 è3ø è6ø
(D) k is any positive integer
æp ö æp ö
(C) ( -1) n +1 ç ÷ + np (D)  ( -1) n -1 ç ÷ + np
6. The least positive non-integral solution of the equation è6ø è3ø
sin π (x2 + x) = sin p x2 is 11. The general solution of the equation
(A) rational n
1
(B) irrational of the form p å cos r 2q sin rq =
r =1 2
is
16.16  Chapter 16

4k - 1 p 2k + 1 p p (B)  p
(A) , k Î Z (B)  , k Î Z (A) 2np ± np ±
n( n + 1) 2 n( n + 1) 2 4 4
4k + 1 p np p
(C) ± (D)  none of these
(C) , k ÎZ (D)  none of these 2 4
n( n + 1) 2
18. If [sin x] + [ 2 cos x ] = -3, x ∈ [0, 2p] ([ . ] denotes
17
12. The solution of sin8x + cos8x = is the greatest integer function) then x belongs to
32
é 5p ù æ 5p ö
np p p (A) ê , 2p ú (B) 
ç 4 , 2p ÷
np ±
(A) ± (B)  ë 4 û è ø
2 8 4
p æ 5p ö é 5p ù
(C) np ± (D)  no solution (C) ç p , ÷ êp , 4 ú
(D) 
8 è 4 ø ë û
13. The general solution of the equation 19. The number of solutions of the equation
2cos2θ + 1 = 3.2– sin2θ is æ px ö
÷÷ = x - 2 3 x + 4
2
sin çç
p è2 3ø
(A) 2np ± , np , n Î Z
2 (A) forms an empty set
p (B) is only one
(B) np ± , 2np , n Î Z (C) is only two
2
(D) is more than 2
p
(C) np ± , np , n Î Z 20. The number of solutions of the equation | cos x | = 2[x],
2
where [ . ] is the greatest integer, is
(D) none of these
(A) one (B)  two
14. The solution of the inequality (C) infinite (D)  nil
log1/2 sinθ > log1/2 cosθ in [0, 2p] is 21. The general solution of sin x - 3 sin 2x + sin 3x = cos
æ pö æp p ö x - 3cos 2x + cos 3x is
ç4, 2÷
(A) ç 0, ÷ (B)  p np p
è 2ø è ø (A) np + (B)  +
8 2 8
æ pö np p 3
(C) ç 0, ÷ (D)  none of these (C) ( -1) n + (D)  2np + cos -1
è 4ø 2 8 2
22. sin x + 2 sin 2x = 3 + sin 3x, 0 ≤ x ≤ 2π has
æ 7p ö
PRACTICE EXERCISES

15. If cos 3 x + sin ç 2 x - = -2 then x = (A) 2 solutions in I quadrant


è 6 ÷ø
(B) one solution in II quadrant
p p (C) no solution in any quadrant
(A) (6 k + 1) (B)  (6 k - 1) (D) one solution in each quadrant
3 3
23. The solution of the equation 1 + sin2ax = cos x, where
p
(C) ( 2k + 1) (D)  none of these a is irrational, is
3 np
(A) x = 0 (B)  x=
where k ∈ Z. a
2x (C) x = 2nπ (D)  none of these
16. If tan2[π (x + y)] + cot2 [π (x + y)] = 1 + , where
1+ x2 24. The values of α for which the equation
x, y ∈ R, then least positive value of y is sin4x + cos4x + sin 2x + α = 0 may be valid, are
5 1 3 1
(A) (B)  (A) - ≤ α ≤ 1 (B)  0 ≤ α ≤
4 4 2 2
3 3 1
(C) (D)  2 (C) - ≤ α ≤ (D)  none of these
4 2 2
17. The general value of y satisfying the equation 1 - 2x 25. If α and β be two distinct values of θ lying between 0
- x2 = tan2(x + y) + cot2(x + y) is and 2π, satisfying the equation 3 cos θ + 4 sin θ = 2,
then the value of sin (α + β) is
Trigonometric Equations  16.17

12 24 28. A solution of the equation (1 – tanθ) (1 + tanθ) sec2 θ


(A) (B)  æ p pö
25 25 + 2 tan2θ = 0, where θ lies in the interval ç - , ÷ is
13 given by è 2 2ø
(C) (D)  none of these p p
25 (A) θ = 0 (B)  q = or -
26. | tanx + secx | = | tanx | + | secx |, x ∈ [0, 2p], if and only if 3 3
p p
x belongs to the interval (C) q = (D)  q =-
(A) (π, 2p] 6 6
(B) [0, p] 29. If p cosx – 2sinx = 2 + 2 − p has a solution, then
é p ö æp ù p∈
(C) ê0, ÷ È ç , p ú
ë 2ø è2 û (A)  5 + 1, 4   5 − 1, 2
(B) 
é 3p ö æ 3p ù (C)  3 + 1, 3 (D)  none of these
(D)  êp , ÷ È ç , 2p ú
ë 2 ø è 2 û
30. The value of ‘b’ such that the equation
27. | cosx | = cosx - 2sinx if
b cos x b + sin x
(A) x = nπ =
p 2 cos 2 x - 1 (cos x - 3 sin 2 x ) tan x
2

(B) x = 2nπ or (2n + 1) π +


possess solutions, belongs to the set
×

4
p æ 1ö 1 
(C) x = n π + ×
(A) ç -¥, ÷ (B)   , ∞
4 è 2ø 2
p
(D) x = n π or nπ + ×

 1
4 (C) ( −∞, ∞) (D) 
 −∞,  ∪ (1, ∞)
2

Previous Year's Questions


2 13
31. If 0 ≤ x < 2π, then the number of real values of x, (A)  (B) 
which satisfy the equation cos x + cos 2x + cos 3x + 3 9
cos 4x = 0, is: [2016] 8 20
(C)  (D) 
(A) 9 (B)  3 9 9
(C) 5 (D)  7
32. If sum of all the solutions of the equation

PRACTICE EXERCISES
⎛ ⎛π ⎞ ⎛π ⎞ 1⎞
8 cos x ⋅ ⎜ cos ⎜ + x ⎟ ⋅ cos ⎜ − x ⎟ − ⎟ = 1 in [0, π]
⎝ ⎝6 ⎠ ⎝6 ⎠ 2⎠
is kπ, then k is equal to [2018]

ANSWER K EYS
Single Option Correct Type
1. (B) 2. (C) 3. (A) 4. (D) 5. (C) 6. (C) 7. (B) 8. (D) 9. (C) 10. (B)
11. (C) 12. (A) 13. (C) 14. (C) 15. (A) 16. (B) 17. (D) 18. (C) 19. (B) 20. (D)
21. (B) 22. (C) 23. (A) 24. (C) 25. (B) 26. (C) 27. (B) 28. (B) 29. (B) 30. (A)

Previous Years’ Questions


31. (D) 32. (D)
16.18  Chapter 16

HINTS AND EXPLANATIONS

Single Option Correct Type


1. Since x2 + x−2 = (x − x− 1)2 + 2 ≥ 2 ∴  Either x2 + x = 2m + x2 ⇒ x = 2m ∈ Z.

x or, x2 + x = k − x2, where k is an odd integer

and, 2cos2 sin2x ≤ 2,
2 - 1 ± 1 + 8k
∴ the given equation is valid only if ⇒ 2x2 + x − k = 0 ⇒ x = .
4
x
2cos2 sin2x = 2
For least positive non-integral solution,
2
x - 1 + 1 + 8k p -1
⇔ cos = cosecx = 1, which cannot be true. x= = , where p is an odd integer.
2 4 4
7. We have, sin2x − 2sin x − 1 = 0
2. We have, sin50x − cos50x = 1
⇒ (sin x − 1)2 = 2 ⇒ sin x − 1 = ± 2
⇒ sin50x = 1 + cos50x
⇒ sinx = 1 - 2 as sinx >/ 1.
Since sin50x ≤ 1 and 1 + cos50x ≥ 1, therefore, the two sides are
equal only if There are 2 solutions in [0, 2p] and two more in [2π, 4p].
sin50x = 1 = 1 + cos50x i.e., sin50x = 1 and cos50x = 0 Thus, minimum value of n is 4.
p 8. The given equation can be written as
∴ x = nπ + , n ∈ Z.
×

2 1 + cospx + 1 + cosqx = 2
3. Let 3 + 1 = r cosα, and 3 - 1 = r sinα æ p+qö æ p-qö
Þ cos ç ÷ x cos ç 2 ÷ x = 0
è 2 ø è ø
∴ r 2 = ( 3 + 1) 2 + ( 3 − 1) 2 = 8,
( 2n + 1)p
æ p+qö
cos ç
2 ÷ x = 0 Þ x = p + q , n = 0, ± 1, ± 2 … which
3 - 1 1 - 1/ 3 æp p ö è ø
and, tan a =
= = tan ç - ÷
3 + 1 1 + 1/ 3 è4 6ø 2p
forms an A. P. with common difference =
= tan(π/12), i.e., α = π/12.
p+q
From the given equation, we get r cos(θ − α) = 2
æ p-qö ( 2n + 1)p
cos ç x=0 Þ x= , n = 0, ± 1, ± 2,…
⇒ cos(θ − π/12) = 1/ 2 = cos(π/4) è 2 ÷ø p-q
HINTS AND EXPLANATIONS

∴ θ − π/12 = 2nπ ± π/4 2p


or θ = 2nπ ± π/4 + π/12. which forms an A. P. with common difference =
p-q
4. We have, a1 + a2cos2x + a3 sin2x = 0 9. We have, | cos x | ≤ sin x
⇒ sin x ≥ 0 [ | cos x | ≥ 0 |

a
⇒ a1 + a2cos2x + 3 (1 − cos2x) = 0
2 ⇒ x ∉ (π, 2π)
which is zero for all values of x, If x = 2π, | cos 2π | ≤ sin 2π, which is not possible
a ∴ x ∈ [0, p]
if a1 = - 3 = −a2
2
é pù
k k If x ∈ ê0, ú , then | cos x | ≤ sin x
or a1 = - , a2 = , , a3 = k ë 2û
2 2
for any k ∈ R. ép p ù
⇒ cos x ≤ sin x ⇒ x ∈ ê , ú
Hence, the required number of triplets is infinite. ë4 2û
If x ∈ é p ù then | cos x | ≤ sin x
5. We have, sin4x − (k + 2) sin2x − (k + 3) = 0 ê 2 , p ú,
ë û
( k + 2) ± ( k + 2) 2 + 4 ( k + 3) ⇒ –cos x ≤ sin x ⇒ tan x ≤ –1 ( cos x < 0)

Þ sin 2 x =
2 æ p 3p ù
⇒ x ∈ ç , ú
( k + 2) ± ( k + 4 ) è2 4 û
=
2 Combining two results, we get
⇒ sin2x = k + 3 ( sin2x = –1 is not possible)
∴ é p p ù æ p 3p ù
xÎê , ú Èç , ú
Since 0 ≤ sin2x ≤ 1, ∴ 0 ≤ k + 3 ≤ 1 or −3 ≤ k ≤ −2. ë4 2û è 2 4 û
6. We have, sinπ (x2 + x) = sinpx2 é p 3p ù
or, x Î ê ,

⇒ π (x2 + x) = nπ + (–1)npx2 ë 4 4 úû
Trigonometric Equations  16.19

1 - sinx +  + ( -1) n sin n x +  1 - cos 2 x If z =1, then 2−sin2θ = 20 ⇒ sin2θ = 0 ⇒ sinθ = 0


10. = ⇒ θ = nπ
1 + sinx +  + sin n x +  1 + cos 2 x
p
1 1- sin x 2 sin 2 x Thus, q = np , np ±
Þ ´ = as –1 < sin x < 1 2
1+ sin x 1 2 cos 2 x 14. sin θ > 0 ⇒ θ ∈ (0, π)(1)
sin 2 x (1+ sin x ) æ p ö æ 3p ö
⇒ 1 − sin x = cosθ > 0 ⇒ q Î ç 0, ÷ È ç , 2p ÷ (2)

1- sin 2 x è 2ø è 2 ø
⇒ (1 − sin x) = sin2x ⇒ 1 − 2 sin x = 0
2
p
From (1) and (2), q Î çæ 0, ÷ö .

1 p p è 2ø
⇒ sin x = = sin ⇒ x = np + ( -1) n
2 6 6 Also, log1/2sinθ > log1/2 cosθ

n
1
11. We have, å cos r q sin r q = 2
r =1
2
æ pö æ pö
⇒ sinθ < cosθ in ç 0, ÷ Þ q Î ç 0, ÷ .
è 2 ø è 4ø
n
7p ö
Þ å 2 cos r q sin r q = 1
2 æ
15. We have cos 3x + sin ç 2 x - = –2
r =1
è 6 ÷ø
n
7p ö
Þ å [sin r( r + 1)q - sin r( r - 1)q ] = 1 æ
⇒ 1 + cos3x + 1 + sin ç 2 x - =0
r =1
è 6 ÷ø
4k + 1 p æ 2p ö
⇒  sin n(n + 1)θ =1 ⇒ q =
× ,k Î Z ⇒ (1 + cos3x) + 1 − cos ç 2 x - ÷ =0
n( n + 1) 2 è 3 ø
17 3x æ pö
12. Given, sin8x + cos8x = Þ 2 cos 2 + 2 sin 2 ç x - ÷ = 0
32 2 è 3ø
17
⇒ (sin4x + cos4x)2 − 2sin4x cos4x =
3x æ pö
32 ⇒ cos = 0 and sin ç x - ÷ = 0
2 è 3ø
⇒ (1 − 2sin2x cos2x)2 − 2sin4x cos4x = 17
3 x p 3p p
32 Þ = , ,… and x - = 0, π, 2π…
2 2 2 3
17
⇒ 1 − 4 sin2x cos2x + 2sin4x cos4x =
p
32 Þ x=

HINTS AND EXPLANATIONS


3
1 17
⇒ 1 − (2sinx cosx)2 +
( 2 sin x cos x ) 4 =
Therefore, the general solution of
8 32
3x æ pö p
1 4 17 cos = 0 and sin ç x - ÷ = 0 is x = 2kp +
⇒ 1 − sin22x +
sin 2 x - = 0 2 è 3ø 3
8 32
⇒ 4sin42x − 32sin22x + 15 = 0
p

= (6k + 1), where k ∈ Z
3
é 15 ù
⇒ 2sin22x − 1 = 0 ê∵ sin 2 2 x ¹ ú

ë 2û 16. Minimum value of L.H.S. is

1 p æ 1 ö 2
⇒ sin22x =
⇒ sin22x = sin 2
× 2 ç x + form where x > 0 ÷ and 1 + £2
2 4 è x ø 1
x+
x
p
⇒ 2x = np ± , n∈I
4 Equality is possible only when both sides are 2 at x = 1

p
np p i.e., tan2 [π (x + y)] = 1 ⇒ π (x + y) = nπ ± ×

⇒ x = ± , n∈I 4
2 8 5p
⇒ π (x + y) = × (for least value of y)
13. We have, 2cos2θ + 1 = 3.2− sin2θ 4

⇒ 21−2sin2θ + 1 = 3.2−sin2θ ∴ y =
1
1 4
⇒ 2z2–3z + 1 = 0, where z = 2−sin2θ Þ z = ,1.

2 17. The given equation can be written as
1 p 3 − 2x − x2 = 1 + tan2 (x + y) + 1 + cot2 (x + y)
If z = , then 2−sin2θ = 2−1 ⇒ sin2θ =1 ⇒ θ = np ±

2 2 ⇒ 4 − (x + 1)2 = sec2(x + y) + cosec2 (x + y)
16.20  Chapter 16

⇒ cos2(x + y) sin2(x + y) [4 − (x + 1)2] = 1 22. From the given equation we have


⇒ sin2(2x + 2y) [4 − (x + 1)2] = 4 (1) sinx − sin3x + 2sin 2x = 3
Since sin2(2x + 2y) ≤ 1 and 4 − (x + 1)2 ≤ 4 ⇒ 2sinx cos2x − 2sin2x + 3 = 0
∴ (1) holds only if sin2(2x + 2y) = 1 (2) ⇒ (sinx + cos2x)2 + (sin2x − 1)2 + 3
= sin2x + cos22x + sin22x + 1
and, 4 − (x + 1)2 = 4 (3)
⇒ (sinx + cos2x)2 + (sin2x − 1)2 + cos2x = 0
From (3), we get x = –1
which is possible only if
Putting in (2), we get sin (2y − 2) = ±1
sinx + cos2x = 0, sin2x = 1 and cosx = 0
p which is not possible for any value of x.
⇒ 2y − 2 = np ± , n∈Z
2 23. The given equation is
p 1 + sin2ax = cosx(1)
⇒ y = 1 + ( 2n ± 1) , n ∈ Z
4 We have, 1 + sin2ax ≥ 1 and cosx ≤ 1
p ∴ The equation (1) has a solution only if cosx = 1
∴ x = –1, y =1 + ( 2n ± 1) , n ∈ Z
4 and, 1 + sin2ax = 1 i.e., sin2ax = 0 ⇒ sin ax = 0
Now, cos x = 1 gives x = 2nπ, n ∈ I
18. We have, [sinx] + [ 2 cos x ] = –3 mp
and, sinax = 0 gives ax = mπ Þ x = , m∈I
a
⇒ [sin x] = –1 and [ 2 cos x ] = –2 The equation (1) has a solution only if for some values of m
⇒ –1 ≤ sin x < 0  or  x ≤ (π, 2π) and n, we have
and, -2 £ 2 cos x < -1
mπ/ a = 2nπ or 2na = m. (2)
But a is irrational and m and n are integers.
-1 æ 3p 5 p ö
or, cos x <
or x Î ç , ÷ ∴ (2) is possible only if m = 0 = n. But then we have x = 0.
2 è 4 4 ø
Hence, x = 0 is the only solution satisfying the given
æ 5p ö equation.
So x Î ç p ,

è 4 ÷ø 24. Given equation is
æ px ö sin4x + cos4x + sin 2x + α = 0
÷÷ = x - 2 3 x + 4
2
19. sin çç
è2 3ø ⇒ (sin2x)2 + (cos2x)2 + 2sin2x cos 2x − 2sin2x cos2x
+ sin2x + α = 0
= ( x - 3 )2 + 1
1
R.H.S > 1. So, the solution exists if and only if ⇒ (sin2x + cos2x)2 − (4sin2x cos2x) + sin 2x + α = 0

HINTS AND EXPLANATIONS

2
x - 3 = 0 Þ x = 3 and then equation is obviously 1
satisfied. ⇒ 1 − (sin2x)2 + sin2x + α = 0
2
20. From the graph, | cos x | and 2 [x] do not cut each other for ⇒ 2 − sin22x + 2sin 2x + 2α = 0
any real value of x. Hence, number of solutions is nil. ⇒ sin22x − 2sin2x − 2 − 2α = 0
2 ± [( 2) 2 - 4 ´ 1( -2 - 2a )]
⇒ sin2x =
2 ´1
2 ± ( 4 + 8 + 8a ) 2 ± (12 + 8a )
= =
2 2
2{1± (3 + 2 a)}
= = 1± (3 + 2 a)
2
If sin 2 x = 1 + (3 + 2a ) > 1 which is not possible

If sin 2 x = 1 - (3 + 2a ) (i)

21. sin x − 3sin 2x + sin 3x = cos x − 3cos 2x + cos 3x
⇒ −1 ≤ 1 − (3 + 2 a) ≤ 1
⇒ 2sin2xcosx − 3sin2x − 2cos2xcosx
+ 3cos2x = 0 ⇒ −2 ≤ − (3 + 2 a) ≤ 0 ⇒ 0 ≤ (3 + 2α) ≤ 4
⇒ sin2x(2cosx − 3) − cos2x (2cosx − 3) = 0 3 1
⇒ −3 ≤ 2α ≤ 1 Þ - £ a £ .
⇒ (sin2x − cos2x) (2cosx − 3) = 0 2 2
⇒ sin2x = cos2x ( cosx ≠ 3/2)

25.  Since α, β satisfy the given equation,
æp ö np p
Þ 2 x = 2np ± ç - 2 x ÷ i.e., x = x = + ∴ 3cosα + 4sinα = 2 and 3cosβ + 4sinβ = 2
è2 ø 2 8
Trigonometric Equations  16.21

On subtracting, we get 3(cosα − cosβ) + 4 (sinα − sinβ) = 0


29. The given equation has a solution if p ≥ 0, 2 − p ≥ 0
a +b b -a a +b a -b
and, p cos x - 2 sin x £ p+4
⇒ 3 × 2 sin sin + 4 × 2 cos sin =0
2 2 2 2 ⇒ p ≥ 0, 2 − p ≥ 0
æ b -a öé a +b a +bù
2 + 2- p £ p + 4 ⇒ p ≥ 0, 2 − p ≥ 0
Þ 2 sin ç ÷ ê3 sin 2 - 4 cos 2 ú = 0
and,
è 2 øë û
( )
2

b -a
and, 2 + 2- p ≤ p + 4 ⇒ p ≥ 0, p ≤ 2

Here, sin ≠ 0 because β ≠ α and
2
and, 2 + 2 − p + 2
4 - 2p ≤ p+4
æ b -a ö æ b -a ö
ç 2 ÷ < π and ç 2 ÷ > −π ⇒ 0 ≤ p ≤ 2 and 2 4 - 2 p ≤ 2p
è ø max è ø min
⇒ p2 + 2p − 4 ≥ 0
a +b a +b
\ 3 sin
- 4 cos =0 ⇒ 0 ≤ p ≤ 2 and (p + 1)2 ≤ 5
2 2
a +b 4 ⇒ 0 ≤ p ≤ 2 and p ∈ (– ∞, − 5 − 1] ∪ [ 5 −1, ∞)
or, tan
=
2 3 ⇒ p ∈ [ 5 − 1, 2]
a +b 4
2 tan 2´ 30. For the domain of definition of the given equation, we have,
\ sin(a + b ) =
2 = 3 = 24 . p
2a + b 16 25 (i) 2 cos2x − 1 ≠ 0 ⇒ x ≠ nπ ± ×

1 + tan 1+ 6
2 9 np p
(ii) tan ≠ 0 ⇒ x ≠ ± [For odd multiples of , tan x is
×

not defined] 2 2
26. | tan x + sec x | = | tan x | + | sec x | iff sec x and tan x both have
p
sin x (iii) cos2x − 3sin2x ≠ 0 ⇒ x ≠ n π ± ,
same sign. ⇒ sec x. tan x ≥ 0 ⇒ ³ 0, but cos x ≠ 0 ⇒ 6
cos 2 x Also, 2cos2x − 1 = 2(cos2x − sin2x) − (cos2x + sin2x) = cos2 x
é p ö æp ù
x ∈ ê0, ÷ È ç , p ú − 3sin2x
ë 2ø è2 û
27. | cos x | = cos x − 2 sin x Now, the given equation reduces to
⇒ cos x = cos x − 2sin x if cos x ≥ 0 bsin x = b + sinx
⇒ sin x = 0 ⇒ x = 2nπ (as cos x ≥ 0).n ∈ I b
Þ sin x = ,
Also, |cos x| = cos x –2sin x b -1
⇒ −cos x = cos x –2 sin x if cos x < 0 b
–1 ≤ sin x ≤ 1 \ - 1 £ £1

HINTS AND EXPLANATIONS


⇒ cos x − sin x = 0 ⇒ tan x = 1 b -1
5p
Now, cos x < 0 and tan x = 1 ⇒ tan x = tan ×
b b
4 Þ + 1 £ 0 and -1 £ 0
æ 5p ö p b -1 b -1
⇒ x = 2nπ + ç ÷ = (2n + 1)π + ×

è 4 ø 4 2b - 1 1
Þ ³ 0 and £0
b -1 b -1
28. We have, (1– tanθ) (1 + tanθ) sec2θ + 2 tan2θ = 0
⇒ (1 − tan4θ) + 2 tan2θ = 0 1 1
Þ b£ or b > 1 and b < 1 Þ b £
Put tan2θ = x, ∴ (1 − x2) + 2x = 0 2 2
⇒ 2x = x2 − 1 = y (say) 1
When b = , sin x = 1, which is not possible

∴ y = 2x and y = x2 –1 2
By inspection, x = 3, ∴ tan2θ = 3 1
p \ b<

⇒ tan θ = ± 3 ⇒ θ = ± 2
3

Previous Year's Questions


31. 2cos 2x cos x + 2 cos 3x cos x = 0 ∴  7 Solutions

⇒ 2cos  x (cos 2x + cos 3x) = 0
⎡ π π 1⎤
⇒ 2cos  x 2cos 5x/2 cos x/2 = 0 32. 8 cos x ⋅ ⎢cos ⎛⎜ + x ⎞⎟ ⋅ cos ⎛⎜ − x ⎞⎟ − ⎥ = 1
p 3p p 3p 7p 9p ⎣ ⎝6 ⎠ ⎝6 ⎠ 2⎦
\ x= , , p, , , ,
2 2 5 5 5 5 ⎛π ⎞
Let cos A = cos ⎜ + x ⎟

⎝6 ⎠
16.22  Chapter 16

⎛π ⎞ ⇒ cos3 x =
1
cos B = cos ⎜ − x ⎟
⎝ 6 ⎠ 2
4 cos x[2 cos A cos B − 1] = 1 π
3 x = 2nπ ±
3
2cos A cos B = cos( A + B ) + cos( A − B)
2nπ π
x= ±
⎡ π ⎤ 3 9
⇒ 4 cos x ⎢cos + cos 2 x − 1⎥ = 1

⎣ 3 ⎦ All roots lie in the region [0, π]

cos 2 x = 2 cos 2 x − 1 π
n = 0 one root

9
⎡1 ⎤
⇒ 4 cos x ⎢ + 2 cos 2 x − 1 − 1⎥ = 1

⎣ 2 ⎦ 2π π
n = 1 two roots
±
3 9
⎡ 1⎤
⇒ 4 cos x ⎢ 2 cos 2 x − 2 + ⎥ = 1

Summing the three roots
⎣ 2⎦
π ⎛ 2π π ⎞ ⎛ 2π π ⎞
⇒ 4 cos x ⎡⎣ 4 cos 2 x − 3⎤⎦ = 2 + ⎜ + ⎟+⎜ −
9 ⎝ 3 9 ⎠ ⎝ 3 9 ⎟⎠
1 π 2π 2π 13π
⇒ 4 cos3 x − 3 cos x =
+ + =
2 9 3 3 9
HINTS AND EXPLANATIONS
CHAPTER

17 Heights and Distances

LEARNING OBJECTIVES
After reading this chapter, you will be able to:
  Understand inverse trigonometric functions and to know functions and properties of inverse trigonometric
domain and range of inverse trigonometric functions functions
  Be acquainted with the graphs of inverse trigonometric   Know solutions of basic inverse trigonometric inequalities

functions, principal values for inverse trigonometric

One of the applications of trigonometry is to estimate


heights of objects and distance between points, without
actually measuring these heights and distances.

SOME TERMINOLOGY RELATED TO HEIGHT


AND DISTANCE

Angle of elevation and angle of depression


If a horizontal line is drawn from the eye of the observer
(O) and an object P is above this line OX, then ∠POX is
called angle of elevation. FIGURE 17.2

NORTH EAST
North-east means equal-
ly inclined to north and east,
south-east means equally
inclined to south and east. ENE
means equally inclined to east
and northeast.

BEARINGS OF A POINT
FIGURE 17.1 Let EW be a line in the east-west direction and NS be a line
perpendicular to it in the north-south direction. Let the two
If an object Q is below the horizontal line OX, then ∠QOX lines intersect at O.
is called angle of depression.
17.2  Chapter 17

Let P be any point. The acute angle which OP makes 3.


In an isosceles triangle, the median is perpendicular
with NS is called the bearing of the point P from O. The to the base i.e., AD ⊥ BC
bearing of a point is briefly indicated by giving the size of
the acute angle and specifying whether it is measured from
ON or OS and whether to the east or west.
In the Fig.

FIGURE 17.7
4.

FIGURE 17.4

1.
OA is in the direction 60º east of north and the bear-
ing of A is written as N 60º E.
2.
OB is in the direction 30º west of north and the bear- FIGURE 17.8
ing of B is written as N 30º W. BC
3.
OC is in the direction 40º west of south and the bear- In a triangle ABC, if ED || AB, then AB =
=
ED DC
ing of C is written as S 40º W. AC
.
4.
OD is in the direction 75º east of south and the bear- EC
ing of D is written as S 75º E.
5.
In a triangle ABC, if AD is the angle bisector of
∠BAC, then
SOME PROPERTIES RELATED TO TRIANGLE
BD AB c
= =
1.
In any triangle ABC DC AC b
a b c
= =
sin A sin B sin C

FIGURE 17.9
FIGURE 17.5
6.
In a triangle ABC, if Ad is the external angle bisector
2. In any triangle ABC, if BD : DC = m : n, ∠BAD = α, of ∠A, then
∠CAD = β and ∠ADC = θ, then BD AB c
= =
(a) (m + n) cot θ = m cot α – n cot β CD AC b
(b) (m + n) cot θ = n cot B – m cot C

FIGURE 17.6 FIGURE 17.10


Heights and Distances  17.3

7.
In a triangle ABC, the exterior angle is equal to the (A) 50 (3 – 3 ) m (B) 200 (3 – 3 ) m
sum of interior opposite angles.
(C) 100 (3 – 3 ) m (D)  none of these
Solution (C)
Let AB be the cliff with AB = 300 m
Let DE be the tower with height h m.
Let BE = x m.
In right angled ∆ABE,
   AB = tan 45º
FIGURE 17.11 BE
θ=α+β
300
⇒  = 1 ⇒ x = 300 m
x
In right angled ∆ACD
SOLVED EXAMPLES
AC 300 - h 1
= tan 30º ⇒ =
1. A tower subtends an angle α at a point on the same CD x 3
level as the root of the tower and at a second point, b 300 - h 1
metres above the first, the angle of depression of the ⇒ = ⇒ 300 3 – 3h = 300
300 3
foot of the tower is β. The height of the tower is
(A) b cot α tan β (B)  b tan α  tan β 300 ( 3 -1)
⇒ h = ⇒ h = 100 3 ( 3 – 1) m
(C) b tan α cot β (D)  none of these 3
Solution (C) ⇒ h = 100 (3 – 3 ) m
From the figure, in right ∠d ∆ABD 3. At the foot of the mountain the elevation of its summit
is 45º; after ascending 1000m towards the mountain up
a slope of 30º inclination, the elevation is found to be
60º. The height of the mountain is

(A) 3 + 1  m (B)  3 -1  m
2 2
3 +1
(C)  m (D)  none of these
2 3
AB = tan α,
Solution (A)
BD
Let P be the summit of the mountain and Q be the foot.
h Let A be the first position and B the second position of
∴ = tan α ⇒ h = x tan α(1)
x observation. BN and BM are ⊥s from B to PQ and AQ
BE respectively.
In right ∠d ∆BCE, = tan β
EC
b b
⇒ = tan β, ∴ x = (2)
x tan b
From (1) and (2), we get
b
∴ h = × tan α = b tan α cot β
tan b
2. From the top of a cliff 300 metres high, the top of
a tower was observed at an angle of depression 30º
and from the foot of the tower the top of the cliff was
observed at an angle of elevation 45º. The height of the Then AB = 1000 m = 1km,
tower is ∠MAB = 30º,
17.4  Chapter 17

∠MAP = 45º, ∠NBP = 60º Solution (B)


Now, ∠BAP = ∠MAP – ∠MAB = 45º – 30º = 15º Let ABC be an equilateral triangle with each side = a,
∠APB = ∠APN – ∠BPN = 45º – 30º = 15º AP be the pillar of height, making angle of 45º at C,
∴ ∆ABP is isosceles and ∴ AB = BP then AC = h tan 45º = h ⇒ a = h.
But AB = 1 kilometre, ∴ BP = 1 kilometre. If the elevation of the pillar at D is θ. (see figure)
Now, PQ = PN + NQ = PN + BM h
= BP sin 60º + AB sin 30º Then, tan θ = h =
AD 3 a/2
= 1 ⋅  3 + 1 ⋅  1 = 3 + 1  m
2h 2
2 2 2 = =
3a 3
4. A balloon moving in a straight line passes vertically
above two points A and B on a horizontal plane 1000  2 
⇒ θ = tan  
−1
ft apart; when above A it has an altitude of 60º as seen  3
from B. When above B it has an attitude of 45º as seen
from A. The distance of B from the point C where it 6. A pole of length h stands inside
will touch the plane is: a triangular plot ABC and sub-
tends equal angles α at its vertices, then
(A) 500( 3 + 1) ft (B) 1500 ft
(A) 2h cos α sin A = a sin α
(B) 500 (3 + 3 ) ft (D)  none of these (B) 2h cos α sin C = c sin α
(C) 2h cos α sin B = b sin α
Solution (A) (D) 2h cot α = R
In ∆ABE,
Solution (A)
From the figure (if PQ is the pole of height h),
BQ = h cot α = CQ = AQ
∴ Q is the circumcentre of ∆ABC.
a
Hence, = 2R = 2h cot α
sin A
⇒ a sin α = 2h cos α sin A

h
tan 45º = ⇒ h = 1000 ft ...(1)
1000
and in ∆ABD,
H
tan 60º = ⇒ H = 1000 3 ft ...(2)
1000
H h
Also, = (Similar triangles)
1000 BC
1000 3 1000
⇒ =  [using Eq. (1) and (2)]
1000 + x x SOME PROPERTIES RELATED TO CIRCLE
⇒ 3x = 1000 + x 1.
The angle subtended by
1000 any chord at the centre
⇒ x = = 500( 3 + 1) ft. is twice the angle sub-
3 -1
tended by the same at any
5. ABC is a triangular park with all sides equal. If a pillar point on the circumfer-
at A subtends an angle of 45° at C, the angle of eleva- ence of the circle.
tion of the pillar at D, the middle point of BC is 2.
If two secants AB and
-1
tan-1 ( 2 / 3 )
(A) tan ( 3 / 2) (B)  PQ of a circle meet at
point ‘O’, then OA.OB =
(C) cot -1 3 (D)  FIGURE 17.12
tan-1 3 OP.OQ.
Heights and Distances  17.5

FIGURE 17.13

3.
If the line joining two points A and B subtends the
greatest angle α, at a point P on a given line XX' then
the circle, through A, B and P will touch the straight
line XX' at the point P. FIGURE 17.17

SOLVED EXAMPLE

7. Two poles of height a and b stand at the centres of two


circular plots which touch each other externally at a
point and the two poles subtend angles of 30º and 60º
respectively at this point, then distance between the
centres of these plots is
FIGURE 17.14
(A) a + b (3a + b) 3
(B) 
4. Angles in the same segment of a circle are equal i.e,
∠APB = ∠AQB = ∠ARB. a 3+b
(C) ( a + 3b) / 3 (D) 

Solution (B)
Let A and B be the centres of the two circles where
the poles of height a and b respectively stand making
angles 30º and 60º respectively at the point O where
these circles touch each other externally.

FIGURE 17.15

5.
Angles in the alternate segments of a circle are equal.

Sum of the radii of the two circles


= a cot 30º + b cot 60º

FIGURE 17.16 b 3a + b
= a 3 + =
3 3
6.
PAB is a secant of a circle and PT is tangent then
= distance between the centres of the two circles.
PA ⋅ PB = PT2
17.6  Chapter 17

QUICK TIPS ∴ BP = AB sin 30º = 10 × 1 = 5 cm.


(a) If a line l is perpendicular to a plane π then the line l is
2
perpendicular to any line m in the plane π [See fig. (i)] From right ∠d ∆BPC, we have BP2 + PC2 = BC2
(b) If a line l is perpendicular to two intersecting lines m ⇒ 52 + PC2 = 62
and n then the line l is perpendicular to the plane π ⇒ PC2 = 62 – 52 = (6 – 5) (6 + 5) = 11
passing through the lines m and n [see fig. (ii)] ∴ PC = 11 cm. (∴ longer diagonal)
AC = AP + PC = 5 3 + 11
9. The top of two poles of height 20 m and 14 m are con-
nected by a wire. If the wire makes an angle 30º with
the horizontal, then the length of the wire is
(A) 12 m (B)  8 m
(C) 10 m (D)  6 m
Solution (A)
  (i)  (ii) Let AB = 20 and CD = 14 be the two poles.
FIGURE 17.18 AC = l is the length of the wire.
(c) 
PQ is perpendicular to the horizontal plane and the CE, the horizontal through    C . ACE = 30º
AE
lines QA, QB, QC, ... are in the plane. Then from geom- = sin 30º = 1
etry, we know that PQ ⊥ QA, PQ ⊥ QB, PQ ⊥ QC, ... l 2
Hence the triangles PQA, PQB, PQC, ... are all right
∴ l = 2AE = 12m
angled triangles where ∠PQA = 90º, ∠PQB = 90º,
∠PQC = 90º, ...

SOLVED EXAMPLES

8. The longer side of a parallelogram is 10 cm and the


shorter is 6 cm. If the longer diagonal makes an angle
30º with the longer side, the length of the longer diag-
onal is
(A) 5 3 + 11 (B) 4 3 + 11
(C) 5 3 + 13 (D)  none of these 10. A vertical tower stands on a declivity which is inclined
Solution (A) at 15º to the horizontal. From the foot of the tower a
Draw BP ⊥ AC, the longer diagonal. From right ∠d man ascends the declivity for 80 feet and then finds
∆APB, we have that the tower subtends an angle of 30º. The height of
the tower is
AP = cos 30º
AB (A) 20 ( 6 - 2 ) (B) 40 ( 6 - 2 )

(C) 40 ( 6 + 2 ) (D)  none of these


Solution (B)
Let BC be the declivity and BA be the tower.
BC AB
=
sin 75° sin 30°
⇒ AP = AB cos 30º
80 sin 30°
⇒ AB =
∴ AP = 10 × 3 = 5 3 cm. sin 75°
2
Also, BP = sin 30º AB =
40 × 2 2
= 40( 6 − 2 )
AB 3 +1
Heights and Distances  17.7

H sin (β − α ) h cot α
5. h = or H =
cos α sin β cot α − cot β

SOME IMPORTANT RESULTS


hsin (β − α )
1. a = h(cot α – cot β) =
sin α sin β
∴  h = a sin α sin β cosec(β – α) and
FIGURE 17.23
∴  d = h cot β = a sin α ⋅ cos β ⋅ cosec(β – α)

a sin (α + β)
6. H =
sin (β − α )

FIGURE 17.19

2. H = x cot α tan(α + β)

FIGURE 17.24

 α + β 
FIGURE 17.20 7. AB = CD. Then, x = y tan 
 2 
3. a = h(cot α + cot β)
h = a sin α ⋅ sin β cosec(α + β)
d = h cot β = a sin α ⋅ cos β ⋅ cosec(α + β)

FIGURE 17.21

h cot β FIGURE 17.25


4. H =
cot α
d
8. h =
cot β + cot 2α
2
17.8  Chapter 17

FIGURE 17.29

SOLVED EXAMPLES
FIGURE 17.26
11. The angle of elevation of a tower from a point A due
AB south of it is x and from a point B due east of A is y. If
9. h =
cot 2 β − cot 2α AB = l, then the height h of the tower is given by
l l
(A) 2 2   (B) 
cot y - cot x tan y - tan 2 x
2

2l
(C)   (D)  none of these
cot y - cot 2 x
2

Solution (A)
Let OP be the tower of height h.
In right ∠d ∆OAP,
∠OAP = x
OA
= cot x
h
⇒ OA = h cot x ...(1)
FIGURE 17.27

10. h = AP sin α
  = a sin α ⋅ sinγ ⋅ cosec(β – γ) and if AQ = d, then
d = AP cos α = a cos α sin γ ⋅ cosec(β – γ)

In right ∠d ∆OBP, ∠OBP = y


OB
∴ = cot y ⇒ OB = h cot y ...(2)
h
In right ∠d ∆OAB, AB2 + OA2 = OB2
∴ l2 + h2 cot2 x = h2 cot2 y
⇒ h2 (cot2 y – cot2 x) = l2
l
∴ h =
FIGURE 17.28 cot y - cot 2 x
2

11. AP = a sin γ ⋅ cosec(α – γ), 12. PQ is a post of given height a, and AB is a tower at


some distance; α and β are the angles of elevation of
AQ = a sin δ ⋅ cosec(β – δ),
B, the top of the tower, at P and Q respectively. The
and apply, PQ2 = AP2 + AQ2 – 2AP ⋅ AQ cos θ
height of the tower and its distance from the post are
Heights and Distances  17.9

a sin α cos β a cos α cos β (c + c) cot (θ – 30º) = c cot 15º – c cot 30º


(A) (B) 
sin (α − β ) sin (α − β ) 1 sin (30 ”-15”)
or cot (θ – 30º) =
a sin α sin β 2 sin 15”sin 30 ”
(C) (D)  none of these
sin (α − β ) 1 1
or cot (θ – 30º) =
2 sin 30 ”
Solution (A)
= 1 = cot 45º
Let AB = h be the height of tower at a distance x from ⇒  θ – 30º = 45º  ∴  θ = 75º
the post of PQ = a.
14. A pole stands vertically on the centre of a sqaure.
AB subtends angles α and β at P and Q respectively.
When α is the elevation of the sun, its shadow just
We have to determine h and x in terms of known quan-
reaches the side of the square and is at a distance x and
tities α, β and a.
y from the ends of that side. The height of the pole is
x2 + y2 x2 + y2
(A)  tan α (b)   cot α
2 2
x2 - y2
(C)  tan α (d)  none of these
2
Solution (A)
Let O be the centre of the square, OP the pole. Shadow
AB = x tan α; BR = x tan β
of the pole OP is OQ. From question, BQ = y and CQ
∴ PQ = AR = AB – BR = x (tan α – tan β)
= x. Then, BC = x + y
PQ a Let OR ⊥ BC.
∴ x = =
tan α − tan β tan α − tan β
∴ OR = x + y
a cos α cos β 2
=
sin (α − β ) and BR = y
∴ h = AB = x tan α ∴ RQ = x + y – y = x - y
a cos α cos β sin α 2 2
= ⋅ Let h be the height of the pole.
sin (α − β ) cos α
From right angled triangle POQ,
a sin α cos β
= h
sin (α − β ) tan α =
OQ
13. A man observes that when he moves up a distance c
∴ OQ = h cot α
metres on a slope, the angle of depression of a point on
Now, from right angled ∆ORQ, OQ2 = OR2 + RQ2
the horizontal plane from the base of the slope is 30º,
2 2
and when he moves up further a distance c metres the  x + y   x − y 
⇒ h2 cot2α =  +
angle of depression of that point is 45º. The angle of  2   2 
inclination of the slope with the horizontal is 2 2
(A) 60º (B)  45º ⇒ h2 cot2 α = 2 ( x + y )
(C) 75º (D)  30º 4
Solution (C) x2 + y2
∴ h = tan α
Applying m – n theorem of trigonometry, we get 2
15. A tower leans towards west making an angle α with
the vertical. The angular elevation of B, the top most
point of the tower, is β as observed from a point C due
east of A at a distance d from A. If the angular eleva-
tion of B from a point due east of C at a distance 2d
from C is γ, then 2 tan α can be written as
(A) 3 cot β – 2cot γ (B)  3 cot γ – 2cot β
(C) 3 cot β – cot γ (D) cot β – 3cot γ
17.10  Chapter 17

Solution (C) 17. A bird is perched on the top of a tree 20m high and
By (m – n) theorem, we have its elevation from a point on the ground is 45º. It flies
off horizontally straight away from the observer and in
one second the elevation of the bird is reduced to 30º.
The speed of the bird is
(A) 14.64 m/s (B)  17.71 m/s
(C) 12 m/s (d)  none of these
Solution (A)
Let the bird be perched at B, the top of the tree BD,
and O be the observer. Then, ∠BOD = 45º and BD =
20 metres.
Now, the bird flying horizontally reaches M in 1
π  second.
(d + 2d)cotβ = d cot γ – 2d cot  + α 
 2 
⇒ 3 cotβ = cotγ + 2tanα
∴ 2 tan α = 3 cot β – cot γ
16. If from the top of a tower, 60 metre high, the angles
of depression of the top and floor of a house are α
and β respectively and if the height of the house is
60 sin(β − α )
, then x =
x Then, ∠ΜΟΝ = 30º, where MN ⊥ ON.
(A) sin α sin β (B) cos α cos β Now, BD = MN = 20 metres
(C) sin α cos β (D) cos α sin β From triangle BOD,
Solution (D) BD 20
tan 45º = = ⇒ OD = 20 metres
Let d be the distance between the tower and house OD OD
∴ d = 60 cot β Again from ∆MON,
⇒ d = (60 – h) cot α MN 20
⇒  60 cotβ = (60 – h) cot α tan 30º = =
ON 20 + DN
60 (cot α − cot β) 1 20
⇒ h = ⇒  = or 20 + DN = 20 3
cot α 3 20 + DN
60 sin(β − α ) ∴    DN = 20( 3 - 1) = 20 × 0.732 = 14.64 meters.
⇒ h = (given)
x Distance
∴  Speed = = BM = DN = 14.64 m/s
time 1 1
18. Due south of a tower which is leaning towards north
there are two stations at distances x and y respectively
from its foot. If α, β respectively be the angles of ele-
vation of the top of the tower at these stations, then the
inclination θ of the tower to the horizontal is given by
cot θ =
y cot α − x cot β y cot α + x cot β
(A) (B) 
y−x y−x
y cot α − x cot β y tan α − x sin β
60[sin(β − α )] (C) (D) 
⇒ h = × sin α y+x y−x
sin α sin β cos α
Solution (A)
∴  x = cos α sin β By (m – n) theorem, we have
Heights and Distances  17.11

Solution (A)
Let AB = a. Then OC = 2a

(x + y – x) cotα = x cot β – (y – x) cot(π – θ)


y cot α − x cot β
cot θ =
∴  
y−x

19. The elevation of C from A and B are α and β respec- ∠CAO = α, ∠CBL = β
tively. B is vertically above A and AB is equal to half OA = 2a cot α = a cot β
the height of C above the level of A. Then, cot α tan β ∴ cot α tan β = 1
is equal to 2
1
(A) 1 (B) 
2 3
1
(C) 1 (D) 
4 5
17.12  Chapter 17

PRACTICE EXERCISES

Single Option Correct Type

1. The angle of elevation of the top of a vertical pole 5. An observer finds that the angular elevation of a tower
when observed from each vertex of a regular hexagon is A. On advancing 3 m towards the tower the elevation
p is 45º and on advancing 2m nearer, the elevation is 90º
is . If the area of the circle circumscribing the hexa-
3 – A. The height of the tower is
gon be A sq. metre, then the area of the hexagon is (A) 2 m (B)  4 m
(C) 6 m (D)  8 m
(A) 3 3 A sq. metres
2 6. A lamp post standing at a point A on a circular path
of radius r subtends an angle α at some point B on
(B) 3 3 A sq. metres the path, and AB subtends an angle of 45º at any other
p point on the path, then height of the lampost is
(C) 3 3 A sq. metres (A) 2 r cot a ( r / 2 ) tan a
(B) 
2 p
(D) none of these (C) 2 r tan a ( r / 2 ) cot a
(D) 

7. PQ is a vertical tower, P is the foot, Q the top of the


2. Two straight roads OA and OB intersect at O. A tower tower, A, B, C are three points in the horizontal plane
is situated within the angle formed by them and sub- through P. The angles of elevation of Q from A, B, C
tends angles of 45º and 30º at the points A and B where are equal and each is equal to θ. The sides of the trian-
the roads are nearest to it. If OA = a and OB = b, then gle ABC are a, b, c and the area of the triangle ABC is
the height of the tower is ∆. The height of the tower is
(A) (abc) tan θ /4∆ (B) (abc) cot θ /4∆
a2 + b2 (C) (abc) tan θ /4∆ (D)  none of these
(A) (B) 
2 a2 + b2
8. The angle of elevation of a cloud from a point h
a2 - b2 metres above the surface of a lake is θ and the angles
(C) (D) 
2 a2 - b2 of depression of its reflection is φ. Then the height of
the cloud is
PRACTICE EXERCISES

3. A and B are two points in the horizontal plane


hsin(φ + θ) h
through O, the foot of pillar OP of height h, such that (A) (B) 
∆AOB = θ. If the elevation of the top of the pilar from sin(φ − θ) sin(φ − θ)
A and B are also equal to θ, then AB is equal to hsin(φ − θ)
(C) h tan (φ – θ) (D) 
q sin(φ + θ)
(A) hcotθ (B) h cos q sec
2 9. A person standing at the foot of a tower walks a dis-
(C) h cot q sin q (D) 
h cos q cosec
q tance 3a away from the tower and observes that the
2 2 angle of elevation of the top of the tower is α. He then
walks a distance 4a perpendicular to the previous
4. A flag is mounted on the semicircular dome of radius
direction and observes the angle of elevation to be β.
r. The elevation of the top of the flag at any point
Then height of the tower is
on the ground is 30º. Moving d distance towards the
dome, when the flag is just visible the angle of eleva- (A) 3a tan α or 5a tan β
tion is 45º. The relation between r and d is (B) 5a tan α or 3a tan β
d 2 2 (C) 4a tan β`
(A) r = (B) r = d (D) 7a tan α
2 ( 3 - 1) 3 +1
10. The angle of elevation of the top of a tree at point B
d 2 2 due south of it is 60º and at a point C due north of it
(C) r = (D) r = d
2 ( 3 + 1) 3 -1 is 30º. D is a point due north of C where the angle of
Heights and Distances  17.13

8 16. OAB is a triangle in the horizontal plane through the


elevation is 15º. If
3 = 1 and BC × CD = 2 × 3
3 2
foot P of the tower at the middle point of the side OB
11
× 19 × 11, the height of the tree is of he triangle. If OA = 2m, OB = 6m, AB = 5m and
∠AOB is equal to the angle subtended by the tower at
(A) 33 (B)  38
A then the height of the tower is
(C) 57 (D)  88
11´ 39 11´ 39
11. A stationary balloon is observed from three points A, (A) (B) 
25 ´ 3 25 ´ 2
B and C on the plane ground and is found that its angle
of elevation from each point is α. If ∠ABC = β and 11´ 25
AC = b, the height of the balloon is (C) (D)  none of these
39 ´ 2
(A) b tan α cosec β (B)  b tan α sin β 17. The angle of elevation of a stationary cloud from a
2 2 point 2,500m above a lake is 15° and the angle of
(C) b cot α cosec β (D)  b cot α sin β depression of its reflection in the lake is 45°. The
2 2 height of the cloud above the lake level is
(A) 2500 3 m (B) 2500m
12. The angular elevation of tower CD at a point A due
south of it is 60º and at a point B due west of A, the (C) 500 3 m (D)  none of these
elevation is 30º. If AB = 3 km, the height of the tower is 18. From the top of a tower 100 m height, the angles of
depression of two objects 200 m apart on the horizon-
(A) 2 3 km (B)  2 6 km tal plane and in a line passing through the foot of the
tower and on the same side of the tower are 45º – A and
(C) 3 3 km (D) 
3 6 km
2 4 45º + A. The angle A is equal to
(A) 15º (B)  35º
13. An isosceles triangle of wood of base 2a and height
h is placed with its base on the ground and vertex (C) 22 1 ° (D) 45º
directly above. The triangle faces the sun whose alti- 2
tude is 30º. Then the tangent of the angle at the apex of 19. ABC is triangular park with AB = AC = 100m. A clock
the shadow is tower is situated at the mid-point of BC. The angles of
2ha 2ha 3 elevation of the top of the tower at A and B are cot–1 3.2
(A) (B)  2 and cosec–12.6 respectively. The height of the tower is
3 3h - a 2
(A) 50m (B) 25m
a 2 + h2 2ah 3 (C) 40m (D)  none of these

PRACTICE EXERCISES
(C) (D) 
2 3 3h2 + a 2 20. Each side of a square subtends an angle of 60º at the
14. The length of the shadow of a rod inclined at 10º to top of a tower h metre high standing in the centre of the
the vertical towards the sun is 2.05 metre when the square. If a is the length of each side of the square, then
elevation of the sun is 38º. The length of the rod is (A) 2a2 = h2 (B) 2h2 = a2
(C) 3a = 2h
2 2
(D) 2h2 = 3a2
2.05 sin 38” 2.05 cos 38”
(A) (B)  21. A tower PQ subtends an angle α at a point A on the
sin 42” sin 42”
same level as the foot Q of the tower. It also subtends
2.05 sin 42” 2.05 cos 42” the same angle α at a point at a point B where AB sub-
(C) (D) 
sin 38” sin 38” tends the angle α with AP then
(A) AB = BQ (B)  BQ = 2AQ
15. ABCD is a rectangular field. A vertical lamp post of
height 12m stands at the corner A. If the angle of ele- AB AB 1
(C) = 1 sin a (D)  cosec a =
vation of its top from B is 60º and from C is 45º, then BQ 2 BQ 2
the area of the field is
22. A flag staff 5m high is placed on a building 25 m high.
(A) 48 2sq.m (B) 
48 3sq.m If flag and building both subtend equal angles on the
observer at a height 30 m, the distance between the
(C) 48sq.m (D) 48 3sq.m observer and the top of the flag is
17.14  Chapter 17

3 (A) 81 2 sq.m (B) 


9 2 sq.m
(A) 5 3 (B) 
5
2 2 (C) 9 sq. m (D)  none of these
2 5 2 28. ABC is an equilateral triangular plot. An electric pole
(C) 5 (D) 
3 3 stands at the vertex and makes an angle of 60º at either
23. The angle of elevation of the top of a vertical pole of the other two vertices. If the height of the triangle is
when observed from each vertex of a regular hexagon 100m, the height of the pole is
p (A) 200 m (B)  100 m
is . If the area of the circle circumscribing the hexa- (C) 150 m (D)  none of these
3
gon be A metre2 then the height of the tower is 29. A ladder rests against a wall at an angle α to the hor-
2A A izontal. If the foot is pulled away through a distance
(A) metre (B)  metre a, it slides a distance b down the wall, finally making
3p 3p
 α + β 
an angle β with the horizontal. Then, tan 
 2 
A A is
(C) 2 metre (D)  metre
3p 3p equal to
a b
24. Two vertical poles AL and BM of height 20m and 80m (A) (B) 
respectively stand apart on a horizontal plane. If A, B b a
be the feet of the poles and AM and BL intersect at P, (C) ab (D)  none of these
then the height of P is equal to 30. If two vertical poles of height h1 and h2 standing at the
(A) 50 m (B)  18 m middle point of the side BC and at the centroid of the
(C) 16 m (D)  15 m triangle ABC respectively, make the same angle at the
25. From a point on a hill-side of constant inclination, the h
vertex A, then 1 is equal to
angle of elevation of the top of a flagstaff on its summit h2
is observed to be α and a metre nearer the top of the 2 3
(A) (B) 
hill, it is β. If h is the height of the flagstaff, the incli- 3 2
1
nation of the hill to the horizontal is (C) (D)  none of these
  3
−1 a sin α sin β 
(A) sin  
 h sin(β − α )  31. A tower AB leans towards west making an angle α
with the vertical. The angular elevation of B, the top

−1 a sin α sin β 

(B) cos   most point of the tower is β, as observed from a point
 h sin(β − α )  C due east of A at a distnace d from A. If the angular
PRACTICE EXERCISES

  elevation of B from a point due east of C at a distance


−1 a sin α sin β 
(C) tan   2d from C is γ, then
 h sin(β − α ) 
(A) 2tanα = 2 cotβ – cotγ
(D) none of these (B) 2tanα = 3 cotβ – cotγ
(C) tanα = cotβ – cotγ
26. A flagstaff stands verticality on a pillar, the height of (D)  none of these
the flagstaff being double the height of the pillar. A
man on the ground at a distance finds that both the 32. AB is a vertical pole with B at the ground level and A at
pillar and the flagstaff subtend equal angles at his eyes. the top. A man finds that the angle of elevation of the
The ratio of the height of the pillar and the distance of point A from a certain point C on the ground is 60°. He
the man from the pillar is moves away from the pole along the line BC to a point
D such that CD = 7 m. From D the angle of elevation
(A) 3 : 1 (B)  1: 3
of the point A is 45°. Then the height of the pole is
(C) 2 : 3 (D)  none of these
7 3 1
(A) ⋅ m (B) 7 3 ⋅ ( 3 + 1) m
27. A vertical lamp-post of height 9 metres stands at the 2 3 −1 2
corner of a rectangular field. The angle of elevation
7 3 1
of its top from the farthest corner is 30º, while from (C) 7 3 ⋅ ( 3 − 1) m (D)  ⋅
another corner it is 45º. The area of the field is 2 2 3 +1
Heights and Distances  17.15

33. At the foot of the mountain, the elevation of its summit y cot α − x cot β y cot α + x cot β
is 45º; after ascending 1000 m towards the mountain (A) (B) 
y−x y−x
up a slope of 30º inclination, the elevation is found to
be 60º. The height of the mountain is y cot α − x cot β y tan α − x sin β
(C) (D) 
y+x y−x
(A) 3 + 1  m (B)  3 -1  m
2 2 39. Two straight roads OA and OB intersect at O. A tower
is situated within the angle formed by them and sub-
3 +1 tends angles of 45º and 30º at the points A and B where
(C)  m (D)  none of these
2 3 the roads are nearest to it. If OA = a and OB = b, then
the height of the tower is
34. The longer side of a parallelogram is 10 cm and the
shorter is 6 cm. If the longer diagonal makes an angle a2 + b2
(A) a2 + b2
(B) 
30º with the longer side, the length of the longer diag- 2
onal (in cm) is
a2 - b2
(A) 5 3 + 11 (B) 4 3 + (C) a2 - b2
(D) 
11 2
(C) 5 3 + 13 (D)  none of these 40. A and B are two points in the horizontal plane through
O, the foot of pillar OP of height h, such that ∠AOB =
35. The angle of elevation of a tower from a point A due θ. If the elevation of the top of the pillar from A and B
south of it is x and from a point B due east of A is y. If are also equal to θ, then AB is equal to
AB = l, then the height h of the tower is given by
(A) hcotθ (B)  q
l l h cos q sec
(A) (B)  2
cot 2 y - cot 2 x tan 2 y - tan 2 x
(C) h cot q sin q (D)  h cos q cosec
q
2l 2 2
(C) (D)  none of these
cot y - cot 2 x
2
41. A flag is mounted on the semicircular dome of radius r.
The elevation of the top of the flag at any point on the
36. A pole stands vertically on the centre of a square. ground is 30º. Moving d distance towards the dome,
When α is the elevation of the sun, its shadow just when the flag is just visible, the angle of elevation is
reaches the side of the square and is at a distance x and 45º. The relation between r and d is
y from the ends of that side. The height of the pole is d 2 2
(A) r = (B)  r = d
2 2
x2 + y2 2 ( 3 - 1) 3 +1
(A) x + y  tan α

PRACTICE EXERCISES
(B)   cot α
2 2 d 2 2
(C) r = (D)  r = d
x2 - y2 2 ( 3 + 1) 3 -1
(C)  tan α (D)  none of these
2 42. A lamp post standing at a point A on a circular path
37. If from the top of a tower, 60 metre high, the angles of radius r subtends an angle α at some point B on the
of depression of the top and floor of a house are α path, and AB subtends an angle of 45º at any other
and β respectively and if the height of the house is point on the path, then height of the lampost is
60 sin(β − α ) (A) 2 r cot a ( r / 2 ) tan a
(B) 
, then x =
x (C) 2 r tan a ( r / 2 ) cot a
(D) 
(A) sin α sin β (B) cos α cos β
43. PQ is a vertical tower, P is the foot, Q the top of the
(C) sin α cos β (D) cos α sin β
tower, A, B, C are three points in the horizontal plane
38. Due south of a tower which is leaning towards north through P. The angles of elevation of Q from A, B, C
there are two stations at distances x and y respectively are equal and each is equal to θ. The sides of the trian-
from its foot. If α, β respectively be the angles of ele- gle ABC are a, b, c and the area of the triangle ABC is
vation of the top of the tower at these stations, then the ∆. The height of the tower is
inclination θ of the tower to the horizontal is given by (A) (abc) tan θ /4∆ (B)  (abc) cot θ /4∆
cot θ = (C) (abc) tan θ /4∆ (D)  none of these
17.16  Chapter 17

44. The angle of elevation of a cloud from a point h 49. A tower PQ subtends an angle α at a point A on the
metres above the surface of a lake is θ and the angles same level as the foot Q of the tower. It also subtends
of depression of its reflection is φ. Then the height of the same angle α at a point at a point B where AB sub-
the cloud is tends the angle α with AP then
hsin(φ + θ) h (A) AB = BQ (B)  BQ = 2AQ
(A) (B) 
sin(φ − θ) sin(φ − θ) AB AB
(C) = 1 sin a (D)  = 1 cosec a
hsin(φ − θ) BQ 2 BQ 2
(C) h tan (φ – θ) (D) 
sin(φ + θ)
50. The angle of elevation of the top of a vertical pole
45. A person standing at the foot of a tower walks a dis- when observed from each vertex of a regular hexagon
tance 3a away from the tower and observes that the p
is . If the area of the circle circumscribing the hexa-
angle of elevation of the top of the tower is α. He then 3
walks a distance 4a perpendicular to the previous gon be A m2 then the height of the tower is
direction and observes the angle of elevation to be β.
Then height of the tower is 2A A
(A) m (B)  m
(A) 3a tan α or 5a tan β 3p 3p
(B)  5a tan α or 3a tan β A A
(C) 4a tan β` (C) 2 m (D)  m
3p 3p
(D)  7a tan α
46. A stationary balloon is observed from three points A, 51. From a point on a hill-side of constant inclination, the
B and C on the plane ground and is found that its angle of elevation of the top of a flagstaff on its summit
angle of elevation from each point is α. If ∠ABC = β is observed to be α and a metre nearer the top of the
and AC = b, the height of the balloon is hill, it is β. If h is the height of the flagstaff, the incli-
nation of the hill to the horizontal is
(A) b tan α cosec β (B)  b tan α sin β 
−1 a sin α sin β 

(A) sin  
2 2  h sin(β − α ) 
(C) b cot α cosec β (D)  b cot α sin β 
−1 a sin α sin β 

2 2 (B)  cos  
 h sin(β − α ) 
47. An isosceles triangle of wood of base 2a and height  
−1 a sin α sin β 
h is placed with its base on the ground and vertex (C) tan  
 h sin(β − α ) 
directly above. The triangle faces the sun whose alti-
PRACTICE EXERCISES

tude is 30º. Then the tangent of the angle at the apex


(D)  none of these
of the shadow is
52. ABC is an equilateral triangular plot. An electric pole
2ha 2ha 3
(A) (B)  stands at the vertex and makes an angle of 60º at either
3 3h2 - a 2 of the other two vertices. If the height of the triangle is
a 2 + h2 2ah 3 100m, the height of the pole is
(C) (D)  (A) 200 m (B)  100 m
2 3 3h2 + a 2
(C) 150 m (D)  none of these
48. OAB is a triangle in the horizontal plane through the
53. A tower AB leans towards west making an angle α
foot P of the tower at the middle point of the side OB
with the vertical. The angular elevation of B, the top
of he triangle. If OA = 2 m, OB = 6 m, AB = 5 m and
most point of the tower is β, as observed from a point
∠AOB is equal to the angle subtended by the tower at
C due east of A at a distnace d from A. If the angular
A then the height of the tower is
elevation of B from a point due east of C at a distance
11´ 39 11´ 39 2d from C is γ, then
(A) (B) 
25 ´ 3 25 ´ 2 (A) 2tanα = 2 cotβ – cotγ
(B) 2tanα = 3 cotβ – cotγ
11´ 25 (C) tanα = cotβ – cotγ
(C) (D)  none of these
39 ´ 2 (D)  none of these
Heights and Distances  17.17

54. Two flagstaffs stand on a horizontal plane. A and B are 56. Two objects P and Q subtend an angle of 30º at A.
two points on the line joining their feet and between Length of 20 m and 10 m are measured from A at right
them. The angles of elevation of the tops of the flag- angles to AP and AQ respectively to points R and S at
staffs as seen from A are 30° and 60º and as seen from each of which PQ subtends angles of 30º, the length of
B are 60° and 45°. If AB is 30 m, then the distance PQ is
between the flagstaffs in metres is (A) 300 - 200 3 (B)  500 - 200 3
(A) 30 + 15 3 (B)  45 + 15 3
(C) 60 – 15 3 (D)  60 + 15 3 (C) 500 3 - 200 (D)  300

55. In a cubical hall abcdpqrs with each side 10 m, G is 57. At each end of a horizontal base of length 2a, the
the centre of the wall bcrq and T is the mid point of the angular height of a certain peak is 15º and that at the
side AB. The angle of elevation of G at the point T is mid point of the base is 45º, the height of the peak is
1 1 ( 3 -1)a 3 ( 3 -1)a
(A) sin–1 (B)  cos–1 (A) (B) 
3 3 2 3 21/ 3
1 1 3 -1 3 / 4 3 -1
(C) tan–1 (D) 
cot–1 (C) .3 a (D)  a
3 3 6 6

Previous Year’s Questions

58. A person standing on the bank of a river observes 61. A bird is sitting on the top of a vertical pole 20 m high
that the angle of elevation of the top of a tree on the which makes an angle of elevation 45° from a point
opposite bank of the river is 60° and when he retires O on the ground. It flies off horizontally straight away
40 meter away from the tree the angle of elevation from the point O. After one second, the elevation of the
becomes 30°. The breadth of the river is [2004] bird from O is reduced to 30°. Then the speed (in m/s)
(A) 20 m (B)  30 m of the bird is  [2014]
(C) 40 m (D)  60 m (A) 40 ( ) 40 3 − 2
2 −1 (B)  ( )
59. A tower stands at the centre of a circular park. A and
B are two points on the boundary of the park such that
AB ( = a) subtends an angle of 60° at the foot of the
(C) 20 2 (D) 20 ( 3 −1 )
tower, and the angle of elevation of the top of the tower
from A or B is 30°. The height of the tower is [2007] 62. If the angles of elevation of the top of a tower from

PRACTICE EXERCISES
three collinear points A, B and C, on a line leading to
(A) 2a (B) 
2a 3 the foot of the tower, are 30°, 45° and 60° respectively,
3 then the ratio, AB : BC, is: [2015]
(C) a (D)  a 3 (A) 3 : 2 (B)  1 : 3
3
60. AB is a vertical pole with B at the ground level and (C) 2 : 3 (D)  3 :1
A at the top. A man finds that the angle of elevation
of the point A from a certain point C on the ground is 63. A man is walking towards a vertical pillar in a straight
60°. He moves away from the pole along the line BC path, at a uniform speed. At a certain point A on the
to a point D such that CD = 7 m. From D the angle of path, he observes that the angle of elevation of the
elevation of the point A is 45°. Then the height of the top of the pillar is 3013. After walking for 10 minutes
pole is [2008] from A in the same direction, at a point B, he observes
that the angle of elevation of the top of the pillar is 60°.
(A) 7 3 ⋅ 1 m (B)  7 3 ⋅ ( 3 + 1)m
2 Then the time taken (in minutes) by him, form B to
2 3 −1 reach the pillar, is: [2016]
(A) 5 (B)  6
(C)  7 3 ⋅ ( 3 −1)m (D)  7 3 ⋅ 1
2 2 3 +1 (C)10 (D) 20
17.18  Chapter 17

64. Let a vertical tower AB have its end A on the level 65. PQR is a triangular park with PQ = PR = 200 m. A T.V.
ground. Let C be the mid-point of AB and P be a point tower stands at the midpoint of QR. If the angles of
on the ground such that AP = 2AB. If ∠BPC = β then elevation of the top of the tower at P, Q and R are
tan β is [2017] respectively 45°, 30° and 30°, then the height of the
1 2 4 6 tower (in m) is [2018]
(A)  (B)  (C)  (D) 
4 9 9 7 (A) 100 (B) 50
(C) 100 3 (D)  50 2

ANSWER K EYS
Single Option Correct Type
1. (C) 2.  (C) 3. (B) 4. (A) 5.  (C) 6.  (C) 7. (A) 8. (A) 9.  (A) 10.  (C)
11.  (A) 12. 
(D) 13. 
(B) 14. 
(A) 15.  (A) 16.  (B) 17. 
(A) 18. 
(C) 19.  (B) 20.  (B)
21.  (D) 22. 
(B) 23. 
(D) 24. 
(C) 25.  (B) 26.  (B) 27. 
(A) 28. 
(A) 29.  (A) 30. (B)
31.  (B) 32. 
(B) 33. 
(A) 34. 
(A) 35.  (A) 36.  (A) 37. 
(D) 38. 
(A) 39.  (C) 40.  (B)
41.  (A) 42. 
(C) 43. 
(A) 44. 
(A) 45.  (A) 46.  (A) 47.  (B) 49.  (D) 50. 
(B) 48.  (D)
51.  (B) 52. 
(A) 53. 
(B) 54. 
(D) 55.  (A) 56.  (B) 57. 
(C)

Previous Years’ Questions


58. (A) 59. (C) 60. (B) 61.  (D) 62. (D) 63. (A) 64. (B) 65. (A)
PRACTICE EXERCISES
Heights and Distances  17.19

HINTS AND EXPLANATIONS

Single Option Correct Type


1. If the height of the pole is h metres, then
h
= tan 60º
circumradius
∴ circumradius
h

= metres
3
2
 h 
∴  A = π  
 3  2
⇒  AP = AC = r + h
3  h  3 3 A Also, AM = MP = MC = r
∴  Area of the hexagon = 6   =
4  3  2 p
sq. metres. ∴  PC = 2r
The correct option is (C) Now, from ∆ACP, we have
2.  Let PQ be the tower of height h. Let PA, PB be perpendicu- PC2 = PA2 + AC2
lars from P upon OA and OB respectively. ⇒ 4r2 = (r + h)2 + (r + h)2 ⇒ r + h = r 2
Then, ∠PAQ = 45º ⇒ h = r( 2 -1)
∠PBQ = 30º 1
OA = a, OB = b r + h = AQ tan 30º ⇒ r + h = ( r + h + d )

3
PA
= cot 45º =1  ∴  PA = h ⇒  3( r + h) = (r + h) + d ⇒ d = ( r + h)( 3 − 1)
h
PB ⇒  d = ( 3 - 1)r 2
= cot 30º = 3   ∴  PB = 3h
h d
Now, OP2 = PA2 + OA2 = PB2 + OB2 ∴  r =

HINTS AND EXPLANATIONS


2 ( 3 − 1)
⇒  h2 + a2 = 3h2 + b2  ⇒ 2h2 = a2 – b2
The correct option is (A)
a 2 - b2 5. Let PQ be the tower of height h
⇒  h =
2 ∠PAQ = A, ∠PBQ = 45º and ∠PCQ = 90º – A
The correct option is (C) AB = 3  and  BC = 2
3. (b).  OA = OB = h cot θ

Now, AB = 2AN = 2 AO sin q



2
cos q q
= 2h
.sin = h cos q sec q
sin q 2 2 ⇒  3 = AB = PA – PB = h cot A – h
and 2 = BC = PB – CP = h – h tan A

The correct option is (B)
h+3 h-2
4. In ∆ACP, we have tan45º = AC ∴  = cot A, = tan A
AP h h
17.20  Chapter 17

h+3 h−2
∴  . = 1 ⇒ h2 + h – 6 = h2 ⇒  h = 6m.
h h

The correct option is (C)

6. Let AP be the lampost of height h at a point A on a circular


path of radius r and centre C. Let B be the point on this path
such that ∠PBA = α, AB = h cot α.
Since AB subtends an angle 45º at another point of the path, it
subtends an angle of 90º at the centre C so that ∠BCA = 90º

The correct option is (A)


9. Let O be the foot of the tower

Also, CA = CB = r ⇒ AB =
2r
and then h cot α =
2r
OA = 3a, AB = 4a  ∴  OB = 5a
⇒  h = 2 r tan a.
∴ In ∆AOP,

The correct option is (C)
h
tan α =
∴ h = 3a tan α
7. AQ = BQ = CQ = h cot θ 3a
HINTS AND EXPLANATIONS

h
and in ∆OPB, tanβ =
,  ∴  h = 5α tanβ.
5a
Thus, h = 3a tan α or 5 a tan β.


The correct option is (A)

10. Let A be the top of the tree OA = h B, C, D be the three


points of observation such then ∠ABO = 60º, ∠ACO =30º
and ∠ADO = 15º

∴  Q is circumcentre of ∆ABC

∴  AQ = BQ = CQ = R = abc
4D
∴  h = abc tan θ
4∆

The correct option is (A)  1 
Then, BC = BO + OC = h(cot 60º + cot30) = h  3 +

8. Let C be the cloud and D its reflection, PB is the surface of  4 
 3 
lake = 
 h
 3 
PC = PD = H, QA = (H – h) cot θ = (H + h) cot φ
CD = h cot 15º – h cot 30º = h( 2 + 3 − 3 ) = 2h
 cot θ + cot φ 
 = [h sin(θ + φ)]/sin(φ – θ)
H = h  
4
´ 2h 2
 cot θ − cot φ 
So that  23 × 32 × 19 × 11 =
3
Heights and Distances  17.21

4 × 2 ×11 2  19 

= h ∵ 3= 
19  11 
⇒ h2 = 32 × 192  ⇒  h = 57
The correct option is (C)
11. Let PQ be vertical and at its end Q there is a balloon P is on
the ground
PA = PB = PC = h cot α
∴  P is circumcentre of ∆ABC and its
circumradius (R) = h cot α a
2
h 3 2 3 ah

= =
a2 3h2 - a 2
1- 2
3h

The correct option is (B)

14. By Sine Rule

b b
⇒ 2R = ⇒ 2h cotα =
sin b sin b

∴  h = b tan α cosecβ
2

The correct option is (A)
h
12. From ∆CDA, x = h cot 30º =
3  2.05
=
From ∆CDB, y = h cot 30º =
3h sin 38” sin ÐBSA

HINTS AND EXPLANATIONS


From ∆ABC, by Pythagarous theorem,
sin 38”× 2.05
∴   =
sin 42”

The correct option is (A)

15. Let AE be a vertical lamp-post. Given AE = 12m.

x2 + 32 = y2
h  
⇒    + 32 = ( 3h) 2 ⇒ h = 3 6 km.
 3  4 AE

tan 45º =
The correct option is (D) AC
13. Height of the triangular shadow = DE = h cot 30º = h 3 ⇒ AC = AE = 12m
a AE 4 3
∴ tan α = ; tan 60º = AE ⇒ AB =
=
h 3 AB 3
2 tan α

tan (2α) = BC = AC 2 - AB 2 = 144 - 48 = 96 = 4 6
1 − tan 2 α
17.22  Chapter 17


Area = AB × BC = 4 3 ´ 4 6 = 48 2sq.m 18. The distance between the objects
= 100 [cot (45º – A) – cot (45º + A)]
The correct option is (A)
1 + tan A 1 − tan A 
16. Let PQ be the tower of height h at the middle point P of the = 100  
side OB of the triangle OAB.

1 − tan A − 1 + tan A 

 2 2

100  (1 + tan A) − (1 − tan A) 



=  
 1 − tan 2 A 

4 tan A
= 100.
= 200 tan2A
1- tan 2 A
∴  200 = 200 tan 2A

OA = 2, OB = 6, AB = 5 and ∠AOB = ∠PAQ = α, then


AP = h cot α, OP = 3
2 2 + 6 2 − 52 5

From triangle OAB, cosα = =
2× 2×6 8
and from ∆ OAP

22 + 32 − h2 cot 2 α ⇒  tan 2A = 1 = tan45º
cos α =

2× 2×3 °
∴ 2A = 45º ⇒ A = 22 1
25 2
5 13 − h2 × 11´ 39
The correct option is (C)
⇒  = 39 ⇒ h2 =
8 12 25 ´ 2
19. DP is a clock tower standing at the middle point D of BC.
11´ 39 ∠PAD = α = cot–13.2 ⇒ cot α = 3.2
⇒  h2 =
25 ´ 2 and ∠PBD = β = cosec–12.6 ⇒ cosec β = 2.6
HINTS AND EXPLANATIONS


The correct option is (B) ∴ cot β = cosec 2 β − 1 = 5.76 = 2.4
17. From ∆ABC and ∆ABD, we have
(H – h) cot 15º = (H + h) cot45º

In ∆PAD, AD = h cot α = 3.2 h



h(cot 15” + 1) In ∆PBD, BD = h cot β = 2.4 h


or H =
(cot 15” − 1) In the right angled ∆ABD, AB2 = AD2 + BD2


Since h = 2500 ⇒ (100)2 = (3.2) + ( 2.4)  h = 16 h2
2 2 2

and  cot 15º = 2 + 3 , we get, H = 2500 3 .
⇒  h = 25 m

The correct option is (B) The correct option is (A)
Heights and Distances  17.23

20. Let ABCD be a square of each side of length a. It is given 5


that ∠BPC = 60º.
tanα = ...(1)
x
Let M be the midpoint of BC. 30

Also, tan 2α = ...(2)
Then ∠BPM = ∠CPM = 30º x
In ∆BMP, right angled at M,
Dividing (1) & (2), we have
we have
2 tan α
tan(∠BPM) = BM tan 2α = 30 tan a ⇒
= 6 tan α
PM 5 1 − tan 2 α
2 2
⇒ 3a = a + h2 ⇒ tan2α =
2
⇒ tan α =
2
4 4 3 3
∴  a2 = 2h2
The correct option is (B) 3
∴  x = 5 cot α = 5.
21. Since ∠PAQ = ∠PBQ = α(angles in the same segment are 2
equal) PQAB is a cyclic quadrilateral
The correct option is (B)
Also, since ∠PAB = α (given), ∠PQB = α(angles in the same
23. Let a be the side of the regular hexagon
segment.)
Now, for the equilateral triangle FOA, we have
OF = OA = AF = a
Hence, Area of Circle = A = πa2
Α
⇒  a = ...(1)
π

Now ∠PBQ = α = ∠PQB

HINTS AND EXPLANATIONS


⇒  ∠ABQ = ∠AQB = 90º – α
⇒  ABQ is an isosceles triangle.
AB BQ
In right angled triangle AOT, right angled at O, we have
⇒  AB = AQ, and =
sin(90 ” − α ) sin 2a
OA
⇒ 2AB sin α = BQ.
 π  OA π 1 A
AB 1 tan   = ⇒ OT = tan   =
 3  OT  3 
⇒  = = 1 cosec α. 3 p
BQ 2 sin a 2

The correct option is (D) ∴  Height of Tower = A

22. In ∆OBC, we have

The correct option is (D)

20 z
24. =
x+y y
z( x + y)
⇒  y =
20
80 z

Also, =
x+y x
z( x + y)
⇒  x =
80
17.24  Chapter 17

 z z h
∴  x + y = (x + y)  +    ⇒  z = 16 26. We have, = tan α
 20 80  x

= a(say)
3h

tan 2α = = 3a
x
2 tan α
⇒  = 3a.
1 − tan 2 α
2a 1
⇒  = 3a ⇒ 2 = 3 – 3a2 ⇒ 3a2 = 1 ⇒ a =
1 - a2 3
∴  h : x = 1 : 3

The correct option is (C)
25. Let OP be the flagstaff of height h and θ be the inclination of
the hill.


The correct option is (B)

27. AB = 9 cot 45º = 9m.


Diagonal, AD = 9 cot 30º
Then, ∠ORP = α,  ∠OQP = β
= 9 3m.

and RQ = a
∴  ∠RPQ = β – α ∴  BD = (9 3 ) 2 - 92
HINTS AND EXPLANATIONS

From ∆PQR, we have


PQ PR RQ = 9 2 m.

= =
sin a sin(180 ” − β) sin(β − α ) Thus, area of the field = 9 ´ 9 2 = 81 2 sq.m

a sin β
∴ PR = ...(1)
sin (β − α )

Also, ∠PQR = π + θ
∴ from ∆POR,2
PR
PO
=  π 
sin a sin  + θ
 2 
h cos θ
∴  PR = ...(2)
sin α
a sin β h cos θ
From (1) and (2), =
The correct option is (A)
sin (β − α ) sin α
28. Let AP be the electric pole of height h.
a sin α sin β
∴ cosθ = h
h sin(β − α )
Then, AB = h cot 60º =
  3
−1 a sin α sin β
⇒  θ = cos   h
 h sin(β − α )  ∴  AB = BC = AC =
 

The correct option is (B) 3
Heights and Distances  17.25

2
 1 h 
2
 h 
  =   − (100)
2
∴  BD = AB – AD   ⇒   .
2 2 2
 2 3   3 

h2 h2 2
⇒  - = (100)2  ⇒  h = (100)2
Thus, AD = h1 cot α and AG = h2 cot α,
3 12 4 2

Since AG = AD
∴  h = 200m. 3
The correct option is (A) h1
2
∴  h2cot α = h1 cot a ⇒ = 3
29. Let the length of the ladder = l 3 h2 2
The correct option is (B)
31. By mn-theorem at C

HINTS AND EXPLANATIONS


∴  AD = EC = l
Now, a = BE – BD
= l cosβ – l cosα (d + 2d) cotβ = d cot γ – 2d cot (90° + α)
= l(cosβ–cosα) ⇒ 3d cotβ = d cotγ + 2d tanα
⇒  3 cotβ = cotγ + 2 tanα
= 2l cos α + β sin α − β
∴  2 tanα = 3 cotβ – cotγ
2 2
32. BD = AB = 7 + x

Also, b = BA – BC A

= l sinα – lsinβ = l(sinα – sinβ)

= 2l cos α + β sin α − β

2 2
a
∴  = tan α + β
b 2 45° 60°

The correct option is (A) D 7 C x B
Also AB = x tan 60°= x 3
30. Let ABC be the given triangle with D as the mid point of BC. ∴  x 3 = 7 + x
let G be the centoid of the triangle ABC.
The poles of height h1 at D and h2 at G subtend the same 7

x=
angle α at A. 3 -1
17.26  Chapter 17

⇒  PC2 = 62 – 52 = (6 – 5) (6 + 5) = 11
AB = 7 3 ( 3 + 1)

2 ∴  PC = 11 cm

The correct option is (B) ∴  longer diagonal
33. Let P be the summit of the mountain and Q be the foot. Let A
AC = AP + PC = 5 3 + 11 cm.
be the first position and B the second position of observation.
The correct option is (A)
BN and BM are ⊥s from B to PQ and AQ respectively.
Then AB = 1000 m = 1km, 35. Let OP be the tower of height h.
∠MAB = 30º, In right ∠d ∆OAP,
∠MAP = 45º, ∠NBP = 60º ∠OAP = x
Now, ∠BAP = ∠MAP – ∠MAB = 45º – 30º = 15º OA
= cot x
∠APB = ∠APN – ∠BPN = 45º – 30º = 15º h
∴ ∆ABP is isosceles and ∴ AB = BP ⇒  OA = h cot x   ...(1)
In right ∠d ∆OBP, ∠OBP = y
OB
∴  = cot y ⇒ OB = h cot y ...(2)
h
In right ∠d ∆OAB, AB2 + OA2 = OB2
∴  l2 + h2 cot2 x = h2 cot2 y
⇒  h2 (cot2 y – cot2 x) = l2
l
∴ h = .
cot y - cot 2 x
2


The correct option is (A)
But AB = 1 kilometre, ∴ BP = 1 kilometre.

36. Let O be the centre of the square, OP the pole. Shadow of the

Now, PQ = PN + NQ = PN + BM
pole OP is OQ. From question, BQ = y and CQ = x. Then, BC

= BP sin 60º + AB sin 30º =x+y

= 1⋅ 3 + 1⋅ 1 = 3 + 1  m. Let OR ⊥ BC.
HINTS AND EXPLANATIONS

2 2 2 ∴  OR = x + y
The correct option is (A) 2
34. Draw BP ⊥ AC, the longer diagonal. From right ∠d ∆APB,
and BQ = y
we have
AP ∴  RQ = x + y – y = x - y .
= cos 30º 2 2
AB

Let h be the height of the pole.

From right angled triangle POQ,
h

tan α =
OQ
∴  OQ = h cot α.
Now, from right angled ∆ORQ, OQ2 = OR2 + RQ2
⇒  AP = AB cos 30º 2 2
 
x + y  x− y  
3 =5 ⇒  h2 cot2α =   +  
∴  AP = 10 × 3 cm.   2   2 
2
2 2
Also BP = sin 30º.
⇒  h2 cot2 α = 2 ( x + y )
AB 4
∴  BP =AB sin 30º = 10 × 1 = 5 cm.
2 ∴ h =
x 2 + y 2 tan α.
From right ∠d ∆BPC, we have BP2 + PC2 = BC2 2
⇒ 52 + PC2 = 62
The correct option is (A)
Heights and Distances  17.27

37. Let d be the distance between the tower and house ⇒  h2 + a2 = 3h2 + b2  ⇒ 2h2 = a2 – b2

a 2 - b2
⇒  h =
2

The correct option is (C)

40. OA = OB = h cot θ

∴  d = 60 cot β
⇒  d = (60 – h) cot α
⇒  60 cotβ = (60 – h) cot α
60 (cot α − cot β)
⇒  h =
cot α
60 sin(β − α ) q
⇒  h = (given) Now, AB = 2AN = 2 AO sin

x 2
cos q q q
60[sin(β − α )] = 2h
.sin = h cos q sec
⇒  h = × sin α sin q 2 2
sin α sin β cos α

The correct option is (B)
∴  x =cosα sinβ
The correct option is (D) 41. In ∆ACP, we have tan45º = AC
38. By (m – n) theorem, we have AP

HINTS AND EXPLANATIONS


⇒  AP = AC = r + h
Also, AM = MP = MC = r
(x + y – x) cotα = x cot β – (y – x) cot(π – θ)
∴  PC = 2r
y cot α − x cot β Now, from ∆ACP, we have
∴ cot θ

y−x PC2 = PA2 + AC2
The correct option is (A) ⇒ 4r2 = (r + h)2 + (r + h)2 ⇒ r + h = r 2
39. Let PQ be the tower of height h. Let PA, PB be perpendicu- ⇒  h = r 2 – 1
lars from P upon OA and OB respectively. 1
r + h = AQ tan 30º ⇒ r + h = ( r + h + d )

Then, ∠PAQ = 45º 3
∠PBQ = 30º
OA = a, OB = b. ⇒  3( r + h) = (r + h) + d ⇒ d = ( r + h)( 3 − 1)

PA = cot 45º =1  ∴  PA = h ⇒  d = ( 3 - 1)r 2


h
d
∴  r =
PB = cot 30º = 3   ∴  PB = 3h 2 ( 3 − 1)
h

Now, OP2 = PA2 + OA2 = PB2 + OB2
The correct option is (A)
17.28  Chapter 17

42. Let AP be the lamp post of height h at a point A on a circular


path of radius r and centre C. Let B be the point on this path
such that ∠PBA = α, AB = h cot α.
Since AB subtends an angle 45º at another point of the path, it
subtends an angle of 90º at the centre C so that ∠BCA = 90º
Also, CA = CB = r ⇒ AB =
2r
and then h cot α =
2r
⇒  h = 2 r tan a.

The correct option is (A)


45. Let O be the foot of the tower


The correct option is (C)

43. AQ = BQ = CQ = h cot θ
∴  Q is circumcentre of ∆ABC

∴  AQ = BQ = CQ = R = abc OA = 3a, AB = 4a  ∴  OB = 5a


4D
∴ In ∆AOP,
h
HINTS AND EXPLANATIONS

tan α =
  ∴  h = 3a tan α
3a
h
and in ∆OPB, tanβ =
,  ∴  h = 5α tanβ.
5a
Thus, h = 3a tan α or 5a tan β.
The correct option is (A)
46. Let PQ be vertical and at its end Q there is a balloon. P is on
the ground
PA = PB = PC = h cot α
∴  h = abc tan θ ∴  P is circumcentre of ∆ABC and its
4∆ circumradius (R) = h cot α

The correct option is (A) b b
⇒ 2R = ⇒ 2h cotα =
44. Let C be the cloud and D its reflection, PB is the surface of lake sin b sin b
PC = PD = H, QA
= (H – h) cot θ
= (H + h) cot φ
 cot θ + cot φ 

H = h  
 cot θ − cot φ 
= (h sin(θ + φ))
sin(φ – θ)

Heights and Distances  17.29

49 Since ∠PAQ = ∠PBQ = α(angles in the same segment are


∴  h = b tan α cosecβ equal) PQAB is a cyclic quadrilateral
2
Also, since ∠PAB = α (given), ∠PQB = α(angles in the same

The correct option is (A) segment.)
47 Height of the triangular shadow = DE = h cot 30º = h 3
a
∴ tan α = ;
h 3
2 tan α

tan (2α) =
1 − tan 2 α
a
2

= h 3 = 2 3 ah
a2 3h2 - a 2
1- 2
3h
Now ∠PBQ = α = ∠PQB
⇒  ∠ABQ = ∠AQB = 90º – α
⇒  ABQ is an isosceles triangle.
AB BQ
⇒  AB = AQ and =
sin(90 ” − α ) sin 2a
⇒ 2AB sin α = BQ.
AB 1
⇒  = = 1 cosec α.
BQ 2 sin a 2

The correct option is (D)

The correct option is (B)
50. Let a be the side of the regular hexagon
48. Let PQ be the tower of height h at the middle point P of the
side OB of the triangle OAB. Now, for the equilateral triangle FOA, we have
OF = OA = AF = a

HINTS AND EXPLANATIONS


Hence, area of circle = A = πa2

A
⇒  a = ...(1)
p

OA = 2, OB = 6, AB = 5 and ∠AOB = ∠PAQ = α, then


AP = h cot α, OP = 3
2 2 + 6 2 − 52 5
From triangle OAB, cosα = =
2× 2×6 8
and from ∆ OAP
22 + 32 − h2 cot 2 α
cos α =

In right angled triangle AOT, right angled at O, we have
2× 2×3
 π  OA OA
25 1 A
13 − h2 × 11´ 39 tan   = ⇒ OT =  π  =
5 39  3  OT 
tan   3 p
⇒  = ⇒ h2 =  3 
8 12 25 ´ 2
11 ´ 39 A
⇒  h2 = . ∴  Height of tower =
25 ´ 2 3p

The correct option is (B)
The correct option is (D)
17.30  Chapter 17

51 Let OP be the flagstaff of height h and θ be the inclination of ∴  BD2 = AB2 – AD2
the hill. 2 2
 1 h   h 
Then, ∠ORP = α,  ∠OQP = β ⇒   .  =   − (100) 2
2 3   3 
and RQ = a
∴  ∠RPQ = β – α. h2 h2 2
⇒  - = (100)2  ⇒  h = (100)2
3 12 4
∴  h = 200 m.
The correct option is (A)

53. By mn-theorem at C

From ∆PQR, we have



PQ PR RQ
= =
sin a sin(180° − β) sin(β − α )
a sin β
∴  PR = ...(1)
sin (β − α ) (d + 2d) cotβ = d cot γ – 2d cot (90° + α)
π ⇒ 3d cotβ = d cotγ + 2d tanα
Also, ∠PQR =

2 ⇒  3 cotβ = cotγ + 2 tanα
∴ from ∆POR, ∴  2 tanα = 3 cotβ – cotγ.
PR The correct option is (B)
PO h cos θ
= π  ⇒ PR = ...(2) 54. Let x and y be the heights of the flagstaffs at P and Q
sin a 
sin  + θ sin α
 2  respectively
x 3

Then, AP = x cos 60º = , AQ = y cot 30º = y
a sin β h cos θ 3
HINTS AND EXPLANATIONS


From (1) and (2), =
sin (β − α ) sin α y
BP = x cot 45º = x, BQ = y cot 60º =
a sin α sin β 3
∴ cosθ = x
⇒  AB = BP – AP = x – [ ∴ AB = 30 m]
h sin(β − α ) 3
 a sin α sin β  ⇒  30 3 = ( 3 –1) x ⇒ x = 15 (3 + 3 ) m
⇒  θ = cos  
−1

 h sin(β − α )  .
 

The correct option is (B)
52. Let AP be the electric pole of height h.

h  1 

Then, AB = h cot 60º = . Similarly, 30 = y  3 −
 ⇒ y = 15 3
3  3 
h y
∴  AB = BC = AC = .
so that PQ = BP + BQ = x + = 15 (3 + 3 ) + 15
3 3
Heights and Distances  17.31


= (60 + 15 3 ) m. Then pq and rs are chords of the same circle making an angle
of 30º at points on the circumference of the circle and hence

The correct option is (D)
are equal in length.
55. Let H be the mid point of BC since Now, from ∆ sar,
∠ tbh = 90º, TH2 = BT2 + BH2 = 52 + 52 = 50 RS2 = 202 + 102 – 2 × 20 × 10cos30º
Also, since ∠thg = 90º, TG2 = TH2 + GH2 = 50 + 25 = 75 ⇒  PQ2 = 500 – 200 3 ⇒ PQ = 500 - 200 3

The correct option is (B)


57. Let OP be the peak of height h.
AB be the horizontal base of height 2a with C as its middle
point then AC = CB = a. Also ∠ PAO = ∠ pbo = 15º and
∠ pco = 45º, then oa = ob = h cot 15º, oc = h cot 45º = h

Let θ be the required angle of elevation of G at T.



GH 5 1

Then, sin θ = = =
TG 5 3 3
1
⇒  sin–1
3
The correct option is (A)
56. Since PQ subtends the same angle of 30º at each of the
points A, R and S, the points P, Q, A, R and S lie on a circle
and AR ⊥ AQ Since ∆oab is isosceles, oc is perpendicular on AB and from
right angled triangle ocb, ob2 = oc2 + bc2
⇒ h2 cot2 15º = h2 + a2
⇒ h2 [cot 15º – 1] = a2

a2
2 a 2 ( 3 -1) 2
⇒   h2 =  3 + 1 =

HINTS AND EXPLANATIONS


  −1 2( 2 3 )
 3 −1 

a 3 −1 3 / 4
⇒  h = ( 3 −1) = ×3 a
2 × 31/ 2 6
∠ RAS = 90º – ∠ raq = ∠ paq = 30º
Also, it is given
The correct option is (C)
AR = 20 m, AS = 10 m.

Previous Year’s Questions


h ⇒ b = 20 m
58. From fig. tan30° =
40 + b
59. From the figure, ∆OAB is equilateral
⇒ 3h = 40 + b …..(1)
∴ OA = OB = AB = a
Also, tan60° = h/b ⇒ h = 3b ….(2)

h

Now, tan30° =
a

a
∴h=
3
17.32  Chapter 17

63.

60. According to fig. h



∆QPA : = tan30°⇒ 3h = x + y  (i)

BD = AB = 7 + x x+y

Also, AB = x tan 60° = x 3
∆QPB :
h
= tan60°⇒ h = 3  (ii)
y
∴x 3 =7+x x
From (i) and (ii), we have, 3y = x + y ⇒ y =

7 2
x=
since the speed is uniform,

3 −1
Distance x in 10 mins
x
7 3 ⇒ Distance
in 5 mins
AB = ( 3 + 1) 2
2
A 1
64. tan θ =
4
B
x
C
b x
45° 60°
u
D 7 C x B P A
4x
1
tan (θ + β ) =
HINTS AND EXPLANATIONS

61. tan 30° = 20 1


= 2
20 + x 3
1
+ tan β 1

20 + x = 20 3 \
4 =
1
1 − tan β 2

x = 20 ( 3 −1 ) 4
2
⇒ Speed is
20 ( 3 −1) m/sec. Solving tan β =
9


Hence, the correct option is (B)
62. AB = 3 x − x 65. Refer to figure: R
Let height = h 30°
x L
BC = x − QM = h cot 30 200 m


3 PM = h cot 45 h
Using Pythagoras theorem
AB 3x − x 3 45°
m
= = PQ2 = PM 2 + QM 2 P

BC x 1 .
x−
200 2 = h2 + ( h 3 ) 2 30°
3 200 m
⇒ 4h = 200 × 200
2

h2 = 100 × 100

Q
h = 100m

CHAPTER
Mathematical Reasoning
18
LEARNING OBJECTIVES
After reading this chapter, you will be able to:
 Understand the meaning of mathematical reasoning and  Learn how to connect two statements with the words
statement ‘and’, ‘or’, and phrases ‘there exists’ and ‘for all’
 Know about negation of a statement and compound  Be acquainted with implications
statement

MATHEMATICAL REASONING This may also be written as


~p: It if false that diagonals of a rectangle are equal.
In mathematical language, there are two kinds of
reasoning-inductive and deductive. We have already
­ We may also write it as
­discussed the induc­tive reasoning in the context of math- ~p: There is at least one rectangle whose diagonals
ematical induction. In this chapter, we shall discuss some are not equal.
fundamentals of deductive rea­soning. Compound Statement
STATEMENT A compound statement is a statement which is made up of
two or more statements. Each statement is called a com-
A sentence is called mathematically acceptable statement if
pound statement.
it is either true or false but not both.
For example, consider the following sentences: The Connecting Word ‘AND’
1. Eight is less than six We can connect the two statement by the word ‘AND’. For
This sentence is false because eight is greater than six. example,
Hence, it is a statement. P: All rational numbers are real
2. The sun is a star q: All real numbers are complex
It is a scientifically established fact that sun is a star
p and q: All rational numbers are real and all real
and therefore, this sentence is always true. Hence, it is
numbers are complex.
a statement.
3. Mathematics is fun Truth Value of p and q
This sentence is subjective in the sense that for those The compound statement with ‘and’ is true if all its com-
who like mathematics, it may be fun but for others, it ponents of statements are true otherwise it is false, i.e., it
may not be. This means that this sentence is not always is false when
true. Hence, it is not a statement. 1. p is true and q is false
Negation of a Statement 2. p is false and q is true
3. p is false and q is false
The denial of a statement is called negation of the state-
ment. If p is a statement, then the negation of p is also For example, consider the following compound statements:
a statement and is denoted by ~p and read and not p. 1. p and q: A line is straight and extends indefinitely in
For example, If both directions.
p: Diagonals of a rectangle are equal, Then The components of statements are
~p: Diagonals of a rectangle are not equal. p: A line is straight
18.2  Chapter 18

q: A line extends indefinitely in both directions. Inclusive ‘OR’


Both these statements are true, therefore, the­ In a statement p or q, if at least one of the two ­alternatives
compound statement is true. occurs, the connecting of word “OR” is inclusive. For
2. P and q: 0 is less than every positive integer and every example, consider the statement:
negative integer.
p or q: A student who has taken physics or mathemat-
The components of statements are
ics can apply for M.Tech programme.
p: 0 is less than every positive integer.
This means that the students who have taken physics or
q: 0 is less than every negative integer.
mathematics or both can apply for the M.Tech programme.
The second statement is false. Therefore, the com- In this case, we are using inclusive ‘OR’
pound statement is false.
3. p and q: All living things have two legs and two eyes. Truth Value of p or q
The components of statements are
When p and q statements both are false, then p or q is also
p: All living things have two legs.
false, otherwise it is true. Thus, p or q is true when
q: All living things have two eyes. 1. p is true, q is false.
Both these statements are false. Therefore, the com- 2. p is false, q is true.
pound statements is false. 3. p and q both are true.

ERROR CHECK QUANTIFIERS


A Statement with ‘And’ is not always a compound statement. Quantifiers are phrases like, “There exists” and “For all”.
For example, consider the statement:
•  A mixture of alcohol and water can be separated by “There Exists”
chemical methods.
There exists, is used of at least one. For example, consider
This statement cannot be considered as a compound state-
the statement
ment with ‘And’. Here, the word ‘And’ refers to two things—
alcohol and water. p: There exists, a quadrilateral whose all sides are
equal.
The statements is equivalent to
THE CONNECTING WORD ‘OR’ There is atleast one quadrilateral whose all sides are
equal.
The statement p, q may be connected by the connecting
word ‘OR’, i.e., p or q. For example,
“For All”
p: Cold drink is available at dinner.
The quantifier “for all” or “for every” can be interpreted
q: Coffee is available at dinner. as saying that all the members of the given set S where the
p or q: Cold drink or coffee is available at dinner. property applies must satisfy that property. For example,
consider the statement
Exclusive ‘OR’ p: For every prime number p, p is an irrational
In a statement p or q, if exactly one of the two alterna- number.
tives occurs, the connecting of word ‘OR’ is exclusive. For This means that if S denotes the set of all prime numbers,
example, consider the statement then for all the members p of the set S, p is an irrational
q or q: An ice cream or pepsi is available with a thali number.
in a restaurant.
This means that a person who does not want ice cream can
have a pepsi along with thali or one does not want pepsi can Info Box!
have an ice cream along with thali. A person cannot have The words ‘And’ and ‘Or’ are called connectives and
both ice cream and pepsi. This is called exclusive ‘OR’. ‘There exists’ and ‘For all’ are called quantifiers.
Mathematical Reasoning  18.3

IMPLICATIONS Converse Statement


The statements with “If–then”, “only if ” and “if and only If p and q are two statements, then the converse of the
if ” are known as implications. implication “if p then q” is “if q then p”. For example,
“If p then q” If a number x is odd, then x2 is also odd.
The statement “if p then q” says that in the event if p is true Its converse is : If x2 is odd then x is also odd.
then q must be true. For example, consider the following
statement: Contrapositive statement
If a number is a multiple of 4 then it is a multiple of 2. If p and q are two statements, then the contrapositive of the
Here, p: A number is a multiple of 4. implication “if p then q” is “if ~q then ~p”. For example,
q: The number is a multiple of 2. If a number is divisible by 4, then it is divisible by 2.
When p is true i.e., a number is a multiple of 4, then q is Its contrapositive statement is
true i.e., the number is a multiple of 2. If a number is not divisible by 2, then it is not divis-
ible by 4.
Info Box!
“If p then q” is the same as the following: “If And Only If” Implication
■ p implies q: It is denoted by p ⇒ q. The symbol ⇒ If p and q are two statements, then the compound statement
stands for implies. p ⇒ q and q ⇒ p is called “if and only if ” implication and
This says that a number is a multiple of 4 implies that is denoted by p ⇔ q.
it is a multiple of 2. For example, consider the following two statements:
■ p is a sufficient condition for q.
p: If a rectangle is a square, then all its four sides are
This says that knowing that a number is a multiple of equal.
4 is sufficient to conclude that it is a multiple of 2.
q: If all the four sides of a rectangle are equal, then
■ p only if q.
the rectangle is a square.
This says that a number is a multiple of 4 only if it is
a multiple of 2. Then, the compound statement using “if and only if ” is
■ q is a necessary condition for p. A rectangle is a square if and only if all its four sides
This says that when a number is a multiple of 4, it is are equal.
necessary a multiple of 2.
■ ~q implies ~p.
Truth Value of “If and Only If”
This says that if a number is not a multiple of 2, then The statement with “if and only if ” is true when
it is not a multiple of 4.
1. p is true, q is true
2. p is false, q is false
Truth Value of “If p then q” The statement with “if and only if ” is false when
Truth value of the statement “if p then q” is false when p 1. p is true, q is false
is true and q is false otherwise it is true i.e., it is true when 2. p is false, q is true
1. p is true, q is true
2. p is false, q is true
3. p is false, q is false SOLVED EXAMPLES

Info Box! 1. Consider the following statements


There methods are adopted to test the truth value of this P: Suman is brilliant.
statement: Q: Suman is rich.
■ Assuming that p is true, prove that q must be true. R: Suman is honest.
■ Assuming that q is false, prove that p must be false. The negation of the statement “Suman is brilliant
■ Assuming that p is true and q is false, obtain a and dishonest if any only if Suman is rich” can be
contradiction. expressed as
18.4  Chapter 18

(A) ~(P ∧ ~ R) ↔ Q (B) ~ P ∧ (Q ↔ ~ R) 4. Let p be the statement “x is an irrational number”, q be


(C) ~(Q ↔ (P ∧ ~ R)) (D) ~Q ↔ ~P ∧ R the statement “y is a transcendental number”, and r be
the statement “x is a rational number if y is a transcen-
Solution: (C)
dental number”.
 Suman is brilliant and dishonest is P ∧ ~R
Suman is brilliant and dishonest iff suman is rich is Statement-1: r is equivalent to either q or p
Statement-2: r is equivalent to ~ (p ↔ ~ q).
Q ↔ (P ∧ ~ R)
(A) Statement-1 is false, Statement-2 is true
Negative of statement is expressed as (B) Statement-1 is true, Statement-2 is true, Statement-2
~[Q ↔ (P ∧ ~ R)] is a correct explanation for Statement-1
(C) Statement-1 is true, Statement-2 is true; Statement-2
2. The negation of the statement “If I become a teacher, is not a correct explanation for Statement-1.
then I will open a school”, is: (D) Statement-1 is true, Statement-2 is false.
(A) I will become a teacher and I will not open a Solution: (D)
school  Given statement r = ~ p ↔ q
(B) Either I will not become a teacher or I will not Statement-1: r1 = (p ∧ ~ q) ∨ (~ p ∧ q)
open a school.
(C) Neither I will become a teacher nor I will open a Statement-2: r2 = ~ (p ↔ –q) = (p ∧ q) ∨ (~q ∧ ~p)
school. From the truth table of r, r1 and r2, r = r1.
(D) I will not become a teacher or I will open a school. Hence Statement-1 is true and Statement-2 is false.
Solution: (A) 5. The statement p → (q → p) is equivalent to
 Let p : I will become a teacher (A) p → (p → q) (B) p → (p ∨ q)
      q : I will open a school (C) p → (p ∧ q) (D)  p → (p ↔ q)
Negation of p → q is ~ (p → q) = p ∧ ~ q Solution: (B)
i.e., I will become a teacher and I will not open a
  p → (q → p) = ~ p ∨ (q → p)
school.
= ~ p ∨ (~ q ∨ p)
3. Statement-1: Consider the statements since p ∨ ~p is always true
p: Delhi is in India = ~p ∨ p ∨ q = p → (p ∨ q)
q: Mumbai is not in Italy. 6. Statement-1: ~ (p ↔ ~ q) is equivalent to p ↔ q.
Then the negation of statement p ∨ q, is ‘Delhi is not in Statement-2: ~ (p ↔ ~ q) is a tautology.
India and Mumbai is in Italy’. (A) Statement-1 is true, Statement-2 is true; Statement-2
Statement-2: For any two statements p and q is a correct explanation for Statement-1
(B) Statement-1 is true, Statement-2 is true; Statement-2
~(p ∨ q) = ~p ∨ ~q is not a correct explanation for Statement-1
(A) Statement-1 is true, Statement-2 is true, Statement-2 (C) Statement-1 is true, Statement-2 is false
is a correct explanation for Statement-1. (D) Statement-1 is false, Statement-2 is true
(B) Statement-1 is true, Statement-2 is true, Statement-2 Solution: (C)
is not a correct explanation for Statement-1.
p q p↔q ~q p↔~q ~ (p ↔ ~ q)
(C) Statement-1 is true, Statement-2 is false.
(D) Statement-1 is false, Statement-2 is true. T T T F F T
T F F T T F
Solution: (C) F T F F T F
~(p ∨ q) = ~p ∧ ~q = Delhi is not in India and Mumbai F F T T F T
is in Italy
So statement-1 is correct and statement-2 is false.
Mathematical Reasoning  18.5

NCERT EXEMPLARS
1. Which of the following is a statement? (C) Rajesh did not live in Bangaluru and Rajni did not
(A) x is a real number (B)  Switch off the fan live in Bangaluru
(C)  6 is a natural number (D)  Let me go (D) Rajesh did not live in Bangaluru or Rajni did not
live in Bangaluru
2. Which of the following is not a statement.
(A)  Smoking is injurious to health 10. The negation of the statement “101 is not a multiple of
(B)  2 + 2 = 4 3” is
(C)  2 is the only even prime number (A)  101 is a multiple of 3
(D)  Come here (B)  101 is a multiple of 2
(C)  101 is an odd number
3. The connective in the statement ‘2 + 7 > 9 or 2 + 7 < (D)  101 is an even number
9’ is
(A) and (B) or 11. The contrapositive of the statement
(C) > (D) < “If 7 is greater than 5, then 8 is greater than 6” is
(A)  If 8 is greater than 6, then 7 is greater than 5
4. The connective in the statement “Earth revolves sound (B)  If 8 is not greater than 6, then 7 is greater than 5
the Sun and Moon is a satellite of earth” is (C)  If 8 is not greater than 6, then 7 is not greater than 5
(A) or (B) Earth (D)  If 8 is greater than 6, then 7 is not greater than 5
(C) Sun (D) and
12. The converse of the statement “If x > y, then x + a > y
5. The negation of the statement “A circle is an ellipse” is + a” is
(A) An ellipse is a circle (A) If x < y, then x + a < y + a
(B) An ellipse is not a circle (B) If x + a > y + a, then x > y
(C) A circle is not an ellipse (C) If x < y, then x + a < y + a
(D) A circle is an ellipse (D) If x > y, then x + a < y + a
6. The negation of the statement “7 is greater than 8” is 13. The converse of the statement “If sun is not shining,
(A)  7 is equal to 8 then sky is filled with clouds” is
(B)  7 is not greater than 8 (A) If sky is filled with clouds, then the Sun is not
(C)  8 is less than 7 shining
(D)  None of these (B)  If Sun is shining, then sky is filled with clouds
7. The negation of the statement “72 is divisible by 2 and (C)  If sky is clear, then Sun is shining
3” is (D) If Sun is not shining, then sky is not filled with
clouds

NCERT EXEMPLARS
(A)  72 is not divisible by 2 or 72 is not divisible by 3
(B)  72 is not divisible by 2 and 72 is not divisible by 3 14. The contrapositive of the statement “If p, then q”, is
(C)  72 is divisible by 2 and 72 is not divisible by 3 (A) if q, then p (B) if p, then – q
(D)  72 is not divisible by 2 and 72 is divisible by 3 (C)  if ~ q, then ~p (D)  if ~ p, then ~ q
8. The negation of the statement “Plants take in CO2 and 15. The statement “If x2 is not even, then x is not even” is
give out O2” is converse of the statement
(A)  Plants do not take in CO2 and do not given out O2 (A) If x2 is odd, then x is even
(B)  Plants do not take in CO2 or do not give out O2 (B) If x is not even, then x2 is not even
(C)  Plants take is CO2 and do not give out O2 (C) If x is even, then x2 is even
(D)  Plants take in CO2 or do not give out O2 (D) If x is odd, then x2 is even
9. The negative of the statement “Rajesh or Rajni lived in 16. The contrapositive of statement ‘If Chandigarh is cap-
Bangaluru” is ital of Punjab, then Chandigarh is in India’ is
(A) Rajesh did not live in Bangaluru or Rajni lives in (A) if Chandigarh is not in India, then Chandigarh is
bangaluru not the capital of Punjab
(B) Rajesh lives in Bangaluru and Rajni did not live in (B)  if Chandigarh is in India, then Chandigarh is
Bangaluru Capital of Punjab
18.6  Chapter 18

(C) 
if Chandigarh is not capital of Punjab, then (A)  A natural number is not greater than zero
Chandigarh is not capital of India (B) It is false that a natural number is greater than
(D) 
if Chandigarh is capital of Punjab, then zero
Chandigarh is not is India. (C) It is false that a natural number is not greater than
zero
17. Which of the following is the conditional p → q?
(D)  None of the above
(A) q is sufficient for p
(B) p is necessary for q 20. Which of the following statement is a conjunction?
(C) p only if q (A) Ram and Shyam are friends
(D) if q then p (B) Both Ram and Shyam are tall
(C) Both Ram and Shyam are enemies
18. The negation of the statement “The product of 3 and 4 (D) None of the above
is 9” is 21. State whether the following sentences are statements
(A)  it is false that the product of 3 and 4 is 9 or not.
(B)  the product of 3 and 4 is 12 (i) The angles opposite to equal sides of a triangle are
(C)  the product of 3 and 4 is not 12 equal.
(D)  it is false that the product of 3 and 4 is not 9 (ii) The moon is a satellites of Earth.
19. Which of the following is not a negation of ‘A nature (iii) May God bless you.
number is greater than zero’? (iv) Asia is a continent.
(v) How are you?

ANSWER K EYS
1. (C) 2.  (D) 3. (B) 4.  (D) 5. (C) 6.  (B) 7.  (B) 8.  (B) 9.  (C) 10.  (A)
11.  (C) 12.  (B) 13.  (A) 14.  (C) 15.  (B) 16.  (A) 17.  (C) 18.  (A) 19.  (C) 20.  (D)
21. (Refer Hints and Explanations)
NCERT EXEMPLARS
Mathematical Reasoning  18.7

HINTS AND EXPLANATIONS

1. As we know a statement is a sentence which is either true or ~ q : 8 is not greater than 6.


false. (~ q) → (~ pi)i.e., If 8 is not greater than 6, then 7 is not
So, 6 is a natural number, which is true greater than 5.
Hence, it is a statement. 12. Let p : x > y
2. ‘Come here’ is not a statement. Since, no sentence can be q:x+a>y+a
called a statement, if it is an order. p→p
3. In ‘2 + 7 > 9 or 2 + 7 < 9’, or is the connective. Converse of the above statement is
4. Connective world is ‘and’. q→p
5. Let p : A circle is an ellipse. i.e., If x + a > y + a, then x > y.
~ p : A circle is not an ellipse. 13. Let p : Sun is not shining.
6. Let p : 7 is greater than 8. and q : Sky is filled with clouds.
~ p : 7 is not greater than 8. Converse of the above statement p → q is q → p.
7. Let p : 72 is divisible by 2 and 3. If sky is filled withc clouds, then the Sun is not shining.
Let q : 72 is divisible by 2. 14. p → q
r : 72 is divisible by 3. If p, then q
~q : 72 is not divisible by 2. Contrapositive of the statement p ® q is (~ q) ® (~ p)
~r : 72 is not divisible by 3. If ~ q, then ~ p.
~ (q ^ r) : q ~ r
^
15. Let p : x2 is not even.
⇒ 72 is not divisible by 2 or 72 is not divisible by 3. and q : x is not even.
8. Let p : Plants take in CO2 and give out O2. Converse of the statement p → q is q → p.
Let q : Plants take in CO2. i.e., If x is not even, then x2 is not even.
r : Plants give out O2. 16. Let p : Chandigarh is capital of Punjab.
~ q : Plants do not take in CO2. and q : Chandigarh is in India.
~ r : Plants do not give out O2. ~ p : Chandigarh is not capital of Punjab.
~ ( q ^ r ) : Plants do not take in CO2 or do not give out O2. ~ q : Chandigarh is not in India.

HINTS AND EXPLANATIONS


9. Let p : Rajesh or Rajni lived in Bengaluru. Contrapositive of the statement p → q is
and p : Rajesh lived in Bengaluru. If (~ q), then (~ p).
r : Rajni lived in Bengaluru. If Chandigarh is not in India, then Chandigarh is not the cap-
~ q : Rajesh did not live in Bengaluru. ital of Punjab.
~ r : Rajni did not live in Bengaluru. 17. ‘p → q’ is same as ‘p only if q’.
^
~ (q r) : Rajesh did not live in Bengaluru and Rajni did not 19. The false negation of the given statement is “It is false that a
live in Bangaluru. natural number is not greater than zero”.
10. Let p : 101 is not a multiple of 3. 21. (i) It is a statement.
~ p : 101 is a multiple of 3. (ii) It is a statement.
11. Let p : 7 is greater than 5. (iii) It is not a statement, since it is an exclamations.
and q : 8 is greater than 6. (iv) It is a statement.
∴p→q (v) It is not a statement, since it is a questions.
~ p : 7 is not greater than 5.
18.8  Chapter 18

PRACTICE EXERCISES

Single Option Correct Type

1. Which of the following is not a statement? (D) None of these


(A) Give me a glass of water. 8. Negation of “2 + 3 = 5 and 8 < 10” is
(B) Asia is a continent.
(A) 2 + 3 ≠ 5 and 8 < 10
(C) The earth revolves round the sun.
(B) 2 + 3 = 5 and 8 v 10
(D) The number 6 has two prime factors 2, 3.
(C) 2 + 3 ≠ 5 or 8 v 10
2. Which of the following is not a statement? (D) None of these
(A) Every set is a finite set. 9. Negation of the conditional: “If it rains, I shall go to
(B) 8 is less then 6. school” is
(C) Where are you going?
(A) It rains and I shall go to school.
(D) The sum of interior angles of a triangle is 180
(B) It rain and I shall not go to school.
degrees.
(C) It does not rain and I shall go to school.
3. Which of the following is not a statement? (D) None of these
(A) Roses are red. 10.
Negation of “Manu is in class X or Anu is in class XII” is
(B) New Delhi is in India.
(A) Manu is not in class X but Anu is in class XII.
(C) Every square is a rectangle.
(B) Manu is not is class X but Anu is not in class XII.
(D) Alas! I have failed.
(C) Either Manu is not in class X or Anu is not is class
4. Which of the following is a statement? XII.
(A) Open the door. (D) None of these
(B)  Do your homework. 11. Truth value of the statement “if p then q” is false when
(C) Switch on the fan.
(A) p is true, q is true
(D) Two plus two is four.
(B) p is true, q is false
5. Which of the following is a statement? (C) p is false, q is true
(A) May you live long! (D) p is false, q is false
(B) May God bless you! 12. Truth value of the statement “p or q” is false, when
(C) The sun is a star.
PRACTICE EXERCISES

(A) p is true, q is false


(D) Hurrah! We have won the match.
(B) p is false, q is true
6. Which of the following is not a statement? (C) p and q both are true
(A) Please do me a favour. (D) p and q both are false
(B) 2 is an even integer. 13. Truth value of the statement with “if and only if ” is
(C) 2 + 1 = 3. false, when
(D) The number 17 is prime.
(A) p is true, q is true
7. Negation of “Paris is in France and London is in (B) p is false, q is false
England” is (C) p is true, q is false
(A) Paris is in England and London is in France. (D) p is false, q is true
(B) Paris is not in France or London is not in England.
(C) Paris is in England or London is in France.
Mathematical Reasoning  18.9

Previous Year’s Questions

14. The Boolean Expression ( p ∧ ~ q) ∨ q ∨ (~ p ∧ q) is (B)  Equivalent to p → ~ q


equivalent to [2016] (C)  A fallacy
(A)  p∨ ~ q (B)  ~ p ∧ q (D)  A tautology
(C)  p ∧ q (D) p ∨ q 16. The Boolean expression: ∼ (p ∨ q) ∨ (∼ p ∧ q) is equiv-
15. The following statement (p → q) → [( ~ p → q) → q] is alent to [2018]
 [2017] (A) p (B)  p (C)  q (D)  ∼p
(A)  Equivalent to ~ p → q

ANSWER K EYS
Single Option Correct Type
1. (A) 2.  (C) 3. (D) 4.  (D) 5. (C) 6.  (A) 7.  (B) 8.  (C) 9.  (B) 10.  (D)
11.  (B) 12.  (D) 13.  (D)

Previous Years’ Questions


14.  (D) 15.  (D) 16.  (D)

PRACTICE EXERCISES
18.10  Chapter 18

HINTS AND EXPLANATIONS

Previous Year’s Questions


14. Given Boolean expression is
Truth table: 2
( p ∧ ~ q) ∨ q ∨ (~ p ∧ q) p ∼p q ∼p ∧ q

( p ∧ ~ q) ∨ q = ( p ∨ q) ∧ (~ q ∨ q) = ( p ∨ q) ∧ t = ( p ∨ q)
1 T F T F
Now,
2 T F F F
( p ∨ q) ∨ (~ p ∧ q) = p ∨ q
The correct option is (D) 3 F T T T
15. 4 F T F F

p q p→ (–p → q) (–p → q) (p → q) →
Truth table: 3
q →q [[–p → p] → q ∼(p ∨ q) ∼p ∧ q ∼(p ∨ q) ∨ (∼p ∧ q)
T T T T T T 1 F F F
T F F F F T 2 F F F
F T T T T T 3 F T T
F F T F T T 4 T F T

(a tautology)
The result is equal to ∼p refer to truth table 2

Hence, the correct option is (D) Alternate solution:
16. Truth table: 1 ∼(p ∨ q) = ∼p ∧ ∼q (Theorem)
∼(p ∨ q) ∨ (∼p ∧ q) ⇒ (∼p ∧ ∼q) ∨ (∼p ∧ q)
p q p∨q ∼(p ∨ q)
⇒ (∼p ∨ ∼p) ∧ (∼q ∨ q) = ∼p
1 T T T F
2 T F T F
3 F T T F
HINTS AND EXPLANATIONS

4 F F F T

You might also like